You are on page 1of 426

RECTO, GAYLE ANGELI M.

2011-0008 | AUSL
Personal Notes on Remedial Law 2 Review (based on the syllabus of Prof. Henedino M. Brondial)

I. PROVISIONAL REMEDIES [Rules 57-61] Section 13. Discharge of attachment on other grounds. — The party whose property
has been ordered attached may file a motion with the court in which he action is pending,
before or after levy or even after the release of the attached property, for an order to set
A. Preliminary Attachment (Rule 57)
aside or discharge the attachment on the ground that the same was improperly or irregularly
issued or enforced, or that the bond is insufficient. If the attachment is excessive, the
1. Grounds discharge shall be limited to the excess. If the motion be made on affidavits on the part of the
movant but not otherwise, the attaching party may oppose the motion by counter-affidavits or
Section 1. Grounds upon which attachment may issue. — At the commencement of other evidence in addition to that on which the attachment was made. After due notice and
the action or at any time before entry of judgment, a plaintiff or any proper party may have hearing, the court shall order the setting aside or the corresponding discharge of the
the property of the adverse party attached as security for the satisfaction of any judgment attachment if it appears that it was improperly or irregularly issued or enforced, or that the
that may be recovered in the following cases: bond is insufficient, or that the attachment is excessive, and the defect is not cured forthwith.

(a) In an action for the recovery of a specified amount of money or damages, other
than moral and exemplary, on a cause of action arising from law, contract, quasi-contract,
5. Third Party Claim
delict or quasi-delict against a party who is about to depart from the Philippines with intent to
Section 14. Proceedings where property claimed by third person. — If the property
defraud his creditors;
attached is claimed by any person other than the party against whom attachment had been
issued or his agent, and such person makes an affidavit of his title thereto, or right to the
(b) In an action for money or property embezzled or fraudulently misapplied or
possession thereof, stating the grounds of such right or title, and serves such affidavit upon
converted to his own use by a public officer, or an officer of a corporation, or an attorney,
the sheriff while the latter has possession of the attached property, and a copy thereof upon
factor, broker, agent, or clerk, in the course of his employment as such, or by any other
the attaching party, the sheriff shall not be bound to keep the property under attachment,
person in a fiduciary capacity, or for a willful violation of duty;
unless the attaching party or his agent, on demand of the sheriff, shall file a bond approved by
the court to indemnify the third-party claimant in a sum not less than the value of the property
(c) In an action to recover the possession of property unjustly or fraudulently taken,
levied upon. In case of disagreement as to such value, the same shall be decided by the court
detained or converted, when the property, or any part thereof, has been concealed, removed,
issuing the writ of attachment. No claim for damages for the taking or keeping of the property
or disposed of to prevent its being found or taken by the applicant or an authorized person;
may be enforced against the bond unless the action therefor is filed within one hundred
twenty (120) days from the date of the filing of the bond.
(d) In an action against a party who has been guilty of a fraud in contracting the
debt or incurring the obligation upon which the action is brought, or in the performance
The sheriff shall not be liable for damages for the taking or keeping of such property to any
thereof;
such third-party claimant, if such bond shall be filed. Nothing herein contained shall prevent
such claimant or any third person from vindicating his claim to the property, or prevent the
(e) In an action against a party who has removed or disposed of his property, or is
attaching party from claiming damages against a third-party claimant who filed a frivolous or
about to do so, with intent to defraud his creditors; or
plainly spurious claim, in the same or a separate action.
(f) In an action against a party who does not reside and is not found in the
When the writ of attachment is issued in favor of the Republic of the Philippines, or any officer
Philippines, or on whom summons may be served by publication.
duly representing it, the filing of such bond shall not be required, and in case the sheriff is
sued for damages as a result of the attachment, he shall be represented by the Solicitor
2. Requirements General, and if held liable therefor, the actual damages adjudged by the court shall be paid by
the National Treasurer out of the funds to be appropriated for the purpose.
Section 3. Affidavit and bond required. — An order of attachment shall be granted only
when it appears by the affidavit of the applicant, or of some other person who personally
6. Claim for Damages
knows the facts, that a sufficient cause of action exists, that the case is one of those mentioned
in section 1 hereof, that there is no other sufficient security for the claim sought to be enforced
by the action, and that the amount due to the applicant, or the value of the property the Section 20. Claim for damages on account of improper, irregular or excessive
possession of which he is entitled to recover, is as much as the sum for which the order is attachment. — An application for damages on account of improper, irregular or excessive
granted above all legal counterclaims. The affidavit, and the bond required by the next attachment must be filed before the trial or before appeal is perfected or before the judgment
succeeding section, must be duly filed with the court before the order issues. becomes executory, with due notice to the attaching party and his surety or sureties setting
forth the facts showing his right to damages and the amount thereof. Such damages may be
awarded only after proper hearing and shall be included in the judgment on the main case.
3. Manner of Attaching
If the judgment of the appellate court be favorable to the party against whom the attachment
Section 5. Manner of attaching property. — The sheriff enforcing the writ shall without was issued he must claim damages sustained during the pendency of the appeal by filing an
delay and with all reasonable diligence attach, to await judgment and execution in the action, application in the appellate court, with notice to the party in whose favor the attachment was
only so much of the property in the Philippines of the party against whom the writ is issued, issued or his surety or sureties, before the judgment of the appellate court becomes
not exempt from execution, as may be sufficient to satisfy the applicant's demand, unless the executory. The appellate court may allow the application to be heard and decided by the trial
former makes a deposit with the court from which the writ is issued, or gives a counter-bond court.
executed to the applicant, in an amount equal to the bond fixed by the court in the order of
attachment or to the value of the property to be attached, exclusive of costs. No levy on Nothing herein contained shall prevent the party against whom the attachment was issued
attachment pursuant to the writ issued under section 2 hereof shall be enforced unless it is from recovering in the same action the damages awarded to him from any property of the
preceded, or contemporaneously accompanied, by service of summons, together with a copy attaching party not exempt from execution should the bond or deposit given by the latter be
of the complaint, the application for attachment the applicant's affidavit and bond, and the insufficient or fail to fully satisfy the award.
order and writ of attachment, on the defendant within the Philippines.
ANITA MANGILA, petitioner, vs. COURT OF APPEALS and LORETA GUINA, respondents.
The requirement of prior or contemporaneous service of summons shall not apply where the G.R. No. 125027 August 12, 2002
summons could not be served personally or by substituted service despite diligent efforts, or the THIRD DIVISION
defendant is a resident of the Philippines temporarily absent therefrom, or the defendant is a CARPIO, J.
non-resident of the Philippines, or the action is one in rem or quasi in rem.
FACTS:
4. Discharge of Attachment x Mangila, an exporter of sea foods and doing business under the name and style
of Seafoods Products, entered into a contract with PR Guina, the President and
See Section 5 above General Manager of Air Swift International, a single registered proprietorship
engaged in the freight forwarding business, for shipment of the sea foods to
Section 12. Discharge of attachment upon giving counter-bond. — After a writ of Guam
attachment has been enforced, the party whose property has been attached, or the person x Mangila requested for 7 days within which to pay Guina for the 1st shipment
appearing on his behalf, may move for the discharge of the attachment wholly or in part on x However, on the next 3 shipments, Mangila failed to pay
the security given. The court shall, after due notice and hearing, order the discharge of the x No payment was made despite demands
attachment if the movant makes a cash deposit, or files a counter-bond executed to the x Guina Æ filed an action for sum of money before RTC Pasay
attaching party with the clerk of the court where the application is made, in an amount equal x In the Sheriff’s Return, however, it was shown that the summons was not served
to that fixed by the court in the order of attachment, exclusive of costs. But if the attachment upon Mangila
is sought to be discharged with respect to a particular property, the counter-bond shall be o A woman found at petitioner’s house informed the sheriff that
equal to the value of that property as determined by the court. In either case, the cash deposit petitioner transferred her residence to Sto. Niño, Guagua,
or the counter-bond shall secure the payment of any judgment that the attaching party may Pampanga. The sheriff found out further that petitioner had left the
recover in the action. A notice of the deposit shall forthwith be served on the attaching party. Philippines for Guam.
Upon the discharge of an attachment in accordance with the provisions of this section, the x Guina Æ filed a Motion for Preliminary Attachment
property attached, or the proceeds of any sale thereof, shall be delivered to the party making x RTC Æ granted; issued an Order of Preliminary Attachment
the deposit or giving the counter-bond, or to the person appearing on his behalf, the deposit o The trial court granted the request of its sheriff for assistance from
or counter-bond aforesaid standing in place of the property so released. Should such counter- their counterparts in RTC, Pampanga. Thus, on October 28, 1988,
bond for any reason be found to be or become insufficient, and the party furnishing the same Sheriff Alfredo San Miguel of RTC Pampanga served on petitioner’s
fail to file an additional counter-bond, the attaching party may apply for a new order of household help in San Fernando, Pampanga, the Notice of Levy
attachment. (12a) with the Order, Affidavit and Bond

1
RECTO, GAYLE ANGELI M.
2011-0008 | AUSL
Personal Notes on Remedial Law 2 Review (based on the syllabus of Prof. Henedino M. Brondial)

x Mangila Æ filed an Urgent Motion to Discharge Attachment without submitting but also the summons addressed to said defendant as well as a
herself to the jurisdiction of the trial court. copy of the complaint xxx." (Emphasis supplied)
o She pointed out that up to then, she had not been served a copy of x Furthermore, we have held that the grant of the provisional remedy of
the Complaint and the summons. Hence, petitioner claimed the attachment involves three stages: first, the court issues the order
court had not acquired jurisdiction over her person granting the application; second, the writ of attachment issues
x Guinan’s counsel did not appear during the setting, hence, the Motion was pursuant to the order granting the writ; and third, the writ is
deemed submitted for resolution implemented. For the initial two stages, it is not necessary that
x RTC Æ granted Mangila’s Motion to Discharge Attachment jurisdiction over the person of the defendant be first obtained.
o BUT did not rule on the issue of jurisdiction However, once the implementation of the writ commences, the court
x Guinan Æ applied for alias summons must have acquired jurisdiction over the defendant for without such
o RTC Æ granted jurisdiction, the court has no power and authority to act in any manner
x Summons was then properly served against the defendant. Any order issuing from the Court will not bind the
x Mangila Æ filed an MD on the ground of improper venue defendant.23
o The contact stipulates Æ "if court litigation becomes necessary to x In the instant case, the Writ of Preliminary Attachment was issued on September
enforce collection xxx the agreed venue for such action is Makati, 27, 1988 and implemented on October 28, 1988. However, the alias summons
Metro Manila." was served only on January 26, 1989 or almost three months after the
x Guinan Æ filed an Opposition implementation of the writ of attachment.
o asserting that although "Makati" appears as the stipulated venue, x The trial court had the authority to issue the Writ of Attachment on September
the same was merely an inadvertence by the printing press whose 27 since a motion for its issuance can be filed "at the commencement of the
general manager executed an affidavit14 admitting such action." However, on the day the writ was implemented, the trial court should
inadvertence. Moreover, private respondent claimed that petitioner have, previously or simultaneously with the implementation of the writ, acquired
knew that private respondent was holding office in Pasay City and jurisdiction over the petitioner. Yet, as was shown in the records of the case, the
not in Makati. summons was actually served on petitioner several months after the writ had
x RTC Æ denied .the MD been implemented.
x Mangila Æ filed her Answer x Private respondent, nevertheless, claims that the prior or contemporaneous
o With affirmative defense of improper venue service of summons contemplated in Section 5 of Rule 57 provides for
x During the setting for pre-trial, only the counsel for Guinan was present exceptions. Among such exceptions are "where the summons could not be
o RTC Æ issued an Order allowing Guinan to present her evidence ex served personally or by substituted service despite diligent efforts or where the
parte defendant is a resident temporarily absent therefrom x x x." Private respondent
x Mangila Æ filed an MR asserts that when she commenced this action, she tried to serve summons on
o RTC Æ denied petitioner but the latter could not be located at her customary address in
x Mangila Æ filed an Omnibus Motion opposing the Order Kamuning, Quezon City or at her new address in Guagua, Pampanga.24
o Her counsel arrived 5 minutes late during the 2nd call, as evidenced Furthermore, respondent claims that petitioner was not even in Pampanga;
by the TSN rather, she was in Guam purportedly on a business trip.
o the presentation of evidence ex-parte should be suspended x Private respondent never showed that she effected substituted service on
because there was no declaration of petitioner as in default petitioner after her personal service failed. Likewise, if it were true that private
x RTC Æ denied the Omnibus Motion respondent could not ascertain the whereabouts of petitioner after a diligent
x Mangila Æ appealed to CA inquiry, still she had some other recourse under the Rules of Civil Procedure.
x CA Æ affirmed RTC x The rules provide for certain remedies in cases where personal service could not
o The Court of Appeals upheld the validity of the issuance of the writ be effected on a party. Section 14, Rule 14 of the Rules of Court provides that
of attachment and sustained the filing of the action in the RTC of whenever the defendant’s "whereabouts are unknown and cannot be ascertained
Pasay. The Court of Appeals also affirmed the declaration of default by diligent inquiry, service may, by leave of court, be effected upon him by
on petitioner and concluded that the trial court did not commit any publication in a newspaper of general circulation x x x." Thus, if petitioner’s
reversible error. whereabouts could not be ascertained after the sheriff had served the summons
x Mangila Æ filed an MR at her given address, then respondent could have immediately asked the court for
o CA Æ denied MR service of summons by publication on petitioner.25
x Mangila Æ filed a Rule 45 before the SC x Moreover, as private respondent also claims that petitioner was abroad at the
time of the service of summons, this made petitioner a resident who is
ISSUE # 1: Whether the issuance and service of the writ of attachment were proper. temporarily out of the country. This is the exact situation contemplated in
Section 16,26 Rule 14 of the Rules of Civil Procedure, providing for service of
HELD # 1: NO. summons by publication.
x Petitioner ascribes several errors to the issuance and implementation of the writ x In conclusion, we hold that the alias summons belatedly served on petitioner
of attachment. Among petitioner’s arguments are: first, there was no ground for cannot be deemed to have cured the fatal defect in the enforcement of the writ.
the issuance of the writ since the intent to defraud her creditors had not been The trial court cannot enforce such a coercive process on petitioner without first
established; second, the value of the properties levied exceeded the value of obtaining jurisdiction over her person. The preliminary writ of attachment must be
private respondent’s claim. However, the crux of petitioner’s arguments rests on served after or simultaneous with the service of summons on the defendant
the question of the validity of the writ of attachment. Because of failure to serve whether by personal service, substituted service or by publication as warranted by
summons on her before or simultaneously with the writ’s implementation, the circumstances of the case.27 The subsequent service of summons does not
petitioner claims that the trial court had not acquired jurisdiction over her person confer a retroactive acquisition of jurisdiction over her person because the law
and thus the service of the writ is void. does not allow for retroactivity of a belated service.
x As a preliminary note, a distinction should be made between issuance and
implementation of the writ of attachment. It is necessary to distinguish ISSUE # 2: Whether the venue was properly laid in Pasay City.
between the two to determine when jurisdiction over the person of the
defendant should be acquired to validly implement the writ. This HELD # 2: NO. (NOT because of the STIPULATION in the contract BUT because the business
distinction is crucial in resolving whether there is merit in petitioner’s argument. of Guina is a sole proprietorship, hence, does not have a separate and distinct personality of
x This Court has long settled the issue of when jurisdiction over the its own. Thus, the venue should have been the residence of Guina)
person of the defendant should be acquired in cases where a party x Petitioner assails the filing of this case in the RTC of Pasay and points to a
resorts to provisional remedies. A party to a suit may, at any time after filing provision in private respondent’s invoice which contains the following:
the complaint, avail of the provisional remedies under the Rules of Court. o "3. If court litigation becomes necessary to enforce collection, an
Specifically, Rule 57 on preliminary attachment speaks of the grant of additional equivalent (sic) to 25% of the principal amount will be
the remedy "at the commencement of the action or at any time charged. The agreed venue for such action is Makati, Metro
thereafter."21 This phrase refers to the date of filing of the complaint Manila, Philippines."28
which is the moment that marks "the commencement of the action." The x Based on this provision, petitioner contends that the action should have been
reference plainly is to a time before summons is served on the instituted in the RTC of Makati and to do otherwise would be a ground for the
defendant, or even before summons issues. dismissal of the case.
x In Davao Light & Power Co., Inc. v. Court of Appeals,22 this Court clarified the x We resolve to dismiss the case on the ground of improper venue but not for the
actual time when jurisdiction should be had: reason stated by petitioner.
o "It goes without saying that whatever be the acts done by the x The Rules of Court provide that parties to an action may agree in writing on the
Court prior to the acquisition of jurisdiction over the person of venue on which an action should be brought.29 However, a mere stipulation on
defendant - issuance of summons, order of attachment and writ of the venue of an action is not enough to preclude parties from bringing a case in
attachment - these do not and cannot bind and affect the other venues.30 The parties must be able to show that such stipulation
defendant until and unless jurisdiction over his person is is exclusive. Thus, absent words that show the parties’ intention to
eventually obtained by the court, either by service on him restrict the filing of a suit in a particular place, courts will allow the
of summons or other coercive process or his voluntary filing of a case in any venue, as long as jurisdictional requirements are
submission to the court’s authority. Hence, when the sheriff or followed. Venue stipulations in a contract, while considered valid and
other proper officer commences implementation of the writ of enforceable, do not as a rule supersede the general rule set forth in
attachment, it is essential that he serve on the defendant not only Rule 4 of the Revised Rules of Court.31 In the absence of qualifying or
a copy of the applicant’s affidavit and attachment bond, and of the restrictive words, they should be considered merely as an agreement on
order of attachment, as explicitly required by Section 5 of Rule 57, additional forum, not as limiting venue to the specified place.32

2
RECTO, GAYLE ANGELI M.
2011-0008 | AUSL
Personal Notes on Remedial Law 2 Review (based on the syllabus of Prof. Henedino M. Brondial)

x In the instant case, the stipulation does not limit the venue exclusively to
Makati. There are no qualifying or restrictive words in the invoice that would FACTS:
evince the intention of the parties that Makati is the "only or exclusive" venue x Chuidian, alleged to be a dummy of then President Marcos, allegedly used false
where the action could be instituted. We therefore agree with private pretenses to induce the officers of the Philippine Export and Foreign Loan
respondent that Makati is not the only venue where this case could be filed. Guarantee Corporation (PHILGUARANTEE), the Board of Investments (BOI) and
x Nevertheless, we hold that Pasay is not the proper venue for this case. the Central Bank, to facilitate the procurement and issuance of a loan guarantee
x Under the 1997 Rules of Civil Procedure, the general rule is venue in personal in favor of the Asian Reliability Company, Incorporated (ARCI), which was
actions is "where the defendant or any of the defendants resides or may be granted a loan of $25M
found, or where the plaintiff or any of the plaintiffs resides, at the election of x Chuidian reneged on the approved business plan and instead invested the
the plaintiff."33 The exception to this rule is when the parties agree on an proceeds of the loan in corporations operating in the United States, more
exclusive venue other than the places mentioned in the rules. But, as we have particularly Dynetics, Incorporated and Interlek, Incorporated
discussed, this exception is not applicable in this case. Hence, following the x ARCI then defaulted in the payment of the loan
general rule, the instant case may be brought in the place of residence x Govt (through PCGG)Æ filed before the Sandiganbayan Civil Case No. 0027
of the plaintiff or defendant, at the election of the plaintiff (private against the Marcos spouses, several government officials who served under the
respondent herein). Marcos administration, and a number of individuals known to be cronies of the
x In the instant case, the residence of private respondent (plaintiff in the lower Marcoses, including Chuidian
court) was not alleged in the complaint. Rather, what was alleged was the o Seeking the reconveyance, reversion, accounting and restitution of
postal address of her sole proprietorship, Air Swift International. It was only all forms of wealth allegedly procured illegally and stashed away by
when private respondent testified in court, after petitioner was declared in the defendants
default, that she mentioned her residence to be in Better Living Subdivision, o charged that Chuidian, by himself and/or in conspiracy with the
Parañaque City. Marcos spouses, engaged in "devices, schemes and stratagems"
x In the earlier case of Sy v. Tyson Enterprises, Inc.,34 the reverse happened. by: (1) forming corporations for the purpose of hiding and avoiding
The plaintiff in that case was Tyson Enterprises, Inc., a corporation owned and discovery of illegally obtained assets; (2) pillaging the coffers of
managed by Dominador Ti. The complaint, however, did not allege the office or government financial institutions such as the Philguarantee; and (3)
place of business of the corporation, which was in Binondo, Manila. What was executing the court settlement between Philguarantee and Chuidian
alleged was the residence of Dominador Ti, who lived in San Juan, Rizal. The which was grossly disadvantageous to the government and the
case was filed in the Court of First Instance of Rizal, Pasig. The Court there held Filipino people
that the evident purpose of alleging the address of the corporation’s president x While the case was pending, PCGG filed a motion for issuance of writ of
and manager was to justify the filing of the suit in Rizal, Pasig instead of in attachment on the ff grounds:
Manila. Thus, the Court ruled that there was no question that venue was o (1) Chuidian embezzled or fraudulently misapplied the funds of
improperly laid in that case and held that the place of business of Tyson ARCI acting in a fiduciary capacity, justifying issuance of the writ
Enterpises, Inc. is considered as its residence for purposes of venue. under Section 1(b), Rule 57 of the Rules of Court;
Furthermore, the Court held that the residence of its president is not the o (2) The writ is justified under Section 1(d) of the same rule as
residence of the corporation because a corporation has a personality separate Chuidian is guilty of fraud in contracting the debt or incurring the
and distinct from that of its officers and stockholders. obligation upon which the action was brought, or that he concealed
x In the instant case, it was established in the lower court that petitioner resides or disposed of the property that is the subject of the action;
in San Fernando, Pampanga35 while private respondent resides in Parañaque o (3) Chuidian has removed or disposed of his property with the
City.36 However, this case was brought in Pasay City, where the business of intent of defrauding the plaintiff as justified under Section 1(c) of
private respondent is found. This would have been permissible had Rule 57; and
private respondent’s business been a corporation, just like the case in o (4) Chuidian is residing out of the country or one on whom
Sy v. Tyson Enterprises, Inc. However, as admitted by private summons may be served by publication, which justifies the writ of
respondent in her Complaint37 in the lower court, her business is a attachment prayed for under Section 1(e) of the same rule.
sole proprietorship, and as such, does not have a separate juridical x Chiudian Æ filed an Opposition
personality that could enable it to file a suit in court.38 In fact, there o (1) The plaintiff's affidavit appended to the motion was in form and
is no law authorizing sole proprietorships to file a suit in court.39 substance fatally defective;
x A sole proprietorship does not possess a juridical personality separate and o (2) Section 1(b) of Rule 57 does not apply since there was no
distinct from the personality of the owner of the enterprise.40 The law merely fiduciary relationship between the plaintiff and Chuidian;
recognizes the existence of a sole proprietorship as a form of business o (3) While Chuidian does not admit fraud on his part, if ever there
organization conducted for profit by a single individual and requires its was breach of contract, such fraud must be present at the time the
proprietor or owner to secure licenses and permits, register its business name, contract is entered into;
and pay taxes to the national government.41 The law does not vest a separate o (4) Chuidian has not removed or disposed of his property in the
legal personality on the sole proprietorship or empower it to file or defend an absence of any intent to defraud plaintiff;
action in court.42 o (5) Chuidian's absence from the country does not necessarily make
x Thus, not being vested with legal personality to file this case, the sole him a non-resident; and
proprietorship is not the plaintiff in this case but rather Loreta Guina in her o (6) Service of summons by publication cannot be used to justify the
personal capacity. In fact, the complaint in the lower court acknowledges in its issuance of the writ since Chuidian had already submitted to the
caption that the plaintiff and defendant are Loreta Guina and Anita Mangila, jurisdiction of the Court by way of a motion to lift the freeze order
respectively. The title of the petition before us does not state, and rightly so, filed through his counsel.
Anita Mangila v. Air Swift International, but rather Anita Mangila v. Loreta x Sandiganbayan Æ issued a Resolution ordering the issuance of a writ of
Guina. Logically then, it is the residence of private respondent Guina, the attachment against L/C No. SSD-005-85 as security for the satisfaction of
proprietor with the juridical personality, which should be considered as one of judgment.
the proper venues for this case. o On the first issue, the Sandiganbayan found that although no
x All these considered, private respondent should have filed this case separate affidavit was attached to the motion, the motion itself
either in San Fernando, Pampanga (petitioner’s residence) or contained all the requisites of an affidavit, and the verification
Parañaque (private respondent’s residence). Since private respondent thereof is deemed a substantial compliance of Rule 57, Section 3 of
(complainant below) filed this case in Pasay, we hold that the case the Rules of Court.
should be dismissed on the ground of improper venue. o Anent the second contention, the Sandiganbayan ruled that there
x Although petitioner filed an Urgent Motion to Discharge Attachment in the lower was no fiduciary relationship existing between Chuidian and the
court, petitioner expressly stated that she was filing the motion without Republic, but only between Chuidian and ARCI. Since the Republic
submitting to the jurisdiction of the court. At that time, petitioner had not been is not privy to the fiduciary relationship between Chuidian and
served the summons and a copy of the complaint.43 Thereafter, petitioner ARCI, it cannot invoke Section 1(b) of Rule 57.
timely filed a Motion to Dismiss44 on the ground of improper venue. Rule 16, o On the third issue of fraud on the part of Chuidian in contracting
Section 1 of the Rules of Court provides that a motion to dismiss may be filed the loan, or in concealing or disposing of the subject property, the
"[W]ithin the time for but before filing the answer to the complaint or pleading Sandiganbayan held that there was a prima facie case of fraud
asserting a claim." Petitioner even raised the issue of improper venue in his committed by Chuidian, justifying the issuance of the writ of
Answer45 as a special and affirmative defense. Petitioner also continued to raise attachment. The Sandiganbayan also adopted the Republic's
the issue of improper venue in her Petition for Review46 before this Court. We position that since it was compelled to pay, through Philguarantee,
thus hold that the dismissal of this case on the ground of improper venue is the bank loans taken out by Chuidian, the proceeds of which were
warranted. fraudulently diverted, it is entitled to the issuance of the writ of
x The rules on venue, like other procedural rules, are designed to insure a just attachment to protect its rights as creditor.
and orderly administration of justice or the impartial and evenhanded o Assuming that there is truth to the government's allegation that
determination of every action and proceeding. Obviously, this objective will not Chuidian has removed or disposed of his property with the intent to
be attained if the plaintiff is given unrestricted freedom to choose where to file defraud, the Sandiganbayan held that the writ of attachment is
the complaint or petition warranted, applying Section 1(e) of Rule 57. Besides, the Rules
provide for sufficient security should the owner of the property
attached suffer damage or prejudice caused by the attachment.
VICENTE B. CHUIDIAN, petitioner, vs. SANDIGANBAYAN (Fifth Division) and the o Chuidian's absence from the country was considered by the
REPUBLIC OF THE PHILIPPINES, respondents. Sandiganbayan to be "the most potent insofar as the relief being
G.R. No. 139941 January 19, 2001 sought is concerned."
FIRST DIVISION
3
RECTO, GAYLE ANGELI M.
2011-0008 | AUSL
Personal Notes on Remedial Law 2 Review (based on the syllabus of Prof. Henedino M. Brondial)

o Sandiganbayan ruled that even if Chuidian is one who ordinarily found to be, or become, insufficient, and the party furnishing the
resides in the Philippines, but is temporarily living outside, he is still same fail to file an additional counter-bond, the attaching creditor
subject to the provisional remedy of attachment. may apply for a new order of attachment.
x Chuidian after 4 years) Æ filed a motion to lift the attachment x Second. To quash the attachment on the ground that it was irregularly
o First, he had returned to the Philippines; hence, the or improvidently issued, as provided for in Section 13 of the same Rule:
Sandiganbayan's "most potent ground" for the issuance of the writ o SEC. 13. Discharge of attachment for improper or irregular
of preliminary attachment no longer existed. Since his absence in issuance. - The party whose property has been attached may also,
the past was the very foundation of the Sandiganbayan's writ of at any time either before or after the release of the attached
preliminary attachment, his presence in the country warrants the property, or before any attachment shall have been actually levied,
immediate lifting thereof. upon reasonable notice to the attaching creditor, apply to the judge
o Second, there was no evidence at all of initial fraud or subsequent who granted the order, or to the judge of the court in which the
concealment except for the affidavit submitted by the PCGG action is pending, for an order to discharge the attachment on the
Chairman citing mere "belief and information" and "not on ground that the same was improperly or irregularly issued. If the
knowledge of the facts." Moreover, this statement is hearsay since motion be made on affidavits on the part of the party whose
the PCGG Chairman was not a witness to the litigated incidents, property has been attached, but not otherwise, the attaching
was never presented as a witness by the Republic and thus was not creditor may oppose the same by counter-affidavits or other
subject to cross-examination. evidence in addition to that on which the attachment was made.
o Third, Chuidian denies that he ever disposed of his assets to After hearing, the judge shall order the discharge of the attachment if
defraud the Republic, and there is nothing in the records that it appears that it was improperly or irregularly issued and the defect
support the Sandiganbayan's erroneous conclusion on the matter. is not cured forthwith.
o Fourth, Chuidian belied the allegation that he was also a defendant x It would appear that petitioner chose the latter because the grounds he raised
in "other related criminal action," for in fact, he had "never been a assail the propriety of the issuance of the writ of attachment. By his own
defendant in any prosecution of any sort in the Philippines."21 admission, however, he repeatedly acknowledged that his justifications to
Moreover, he could not have personally appeared in any other warrant the lifting of the attachment are facts or events that came to light or
action because he had been deprived of his right to a travel took place after the writ of attachment had already been implemented.
document by the government. x More particularly, petitioner emphasized that four (4) years after the writ was
o Fifth, the preliminary attachment was, in the first place, issued, he had returned to the Philippines. Yet while he noted that he would
unwarranted because he was not "guilty of fraud in contracting the have returned earlier but for the cancellation of his passport by the PCGG, he
debt or incurring the obligation". In fact, the L/C was not a product was not barred from returning to the Philippines. Then he informed the
of fraudulent transactions, but was the result of a US Court- Sandiganbayan that while the case against him was pending, but after the
approved settlement. Although he was accused of employing attachment had already been executed, the government lost two (2) cases for
blackmail tactics to procure the settlement, the California Supreme fraud lodged against him before the U.S. Courts, thus invoking res judicata.
Court ruled otherwise. And in relation thereto, he cites as a sixth Next, he also pointed out that the government is estopped from pursuing the
ground the fact that all these allegations of fraud and wrongdoing case against him for failing to prosecute for the number of years that it had been
had already been dealt with in actions before the State and Federal pending litigation.
Courts of California. While it cannot technically be considered as x It is clear that these grounds have nothing to do with the issuance of the writ of
forum shopping, it is nevertheless a "form of suit multiplicity over attachment. Much less do they attack the issuance of the writ at that time as
the same issues, parties and subject matter." 22 These foreign improper or irregular. And yet, the rule contemplates that the defect must be in
judgments constitute res judicata which warrant the dismissal of the very issuance of the attachment writ. For instance, the attachment may be
the case itself. discharged under Section 13 of Rule 57 when it is proven that the allegations of
x SB Æ denied Chuidian's motion to lift attachment the complaint were deceptively framed,30 or when the complaint fails to state a
x Chuidian Æ filed an MR cause of action.31 Supervening events which may or may not justify the
o First, Chuidian was out of the country in 1993, but is now presently discharge of the writ are not within the purview of this particular rule.
residing in the country. x In the instant case, there is no showing that the issuance of the writ of
o Second, the Sandiganbayan could not have known then that his attachment was attended by impropriety or irregularity. Apart from seeking a
absence was due to the non-renewal of his passport at the instance reconsideration of the resolution granting the application for the writ, petitioner
of the PCGG. Neither was it revealed that the Republic had already no longer questioned the writ itself. For four (4) long years he kept silent and did
disposed of Chuidian's assets ceded to the Republic in exchange for not exercise any of the remedies available to a defendant whose property or
the L/C. The foreign judgment was not an issue then because at asset has been attached. It is rather too late in the day for petitioner to question
that time, said judgment had not yet been issued and much less the propriety of the issuance of the writ.
final. Furthermore, the authority of the PCGG Commissioner to x Petitioner also makes capital of the two foreign judgments which he claims
subscribe as a knowledgeable witness relative to the issuance of warrant the application of the principle of res judicata. The first judgment, in Civil
the writ of preliminary attachment was raised for the first time in Case Nos. 575867 and 577697 brought by Philguarantee before the Santa Clara
the motion to lift the attachment. Finally, the issue of laches could Country Superior Court, denied Philguarantee's prayer to set aside the stipulated
not have been raised then because it was the Republic's judgment wherein Philguarantee and Chuidian agreed on the subject attached
subsequent neglect or failure to prosecute despite the passing of L/C. On March 14, 1990, the Court of Appeal of the State of California affirmed
the years that gave rise to laches. the Superior Court's judgment. The said judgment became the subject of a
x SB Æ denied MR petition for review by the California Supreme Court. There is no showing,
x Chuidian Æ filed a Rule 65 before the SC however, of any final judgment by the California Supreme Court. The records,
including petitioner's pleadings, are bereft of any evidence to show that there is a
ISSUE # 1: What are Chuidian’s remedies against the attachment of the L/C? final foreign judgment which the Philippine courts must defer to. Hence, res
judicata finds no application in this instance because it is a requisite that the
HELD # 1: There are two courses of action available to the petitioner former judgment or order must be final.
x The Rules of Court specifically provide for the remedies of a defendant whose
property or asset has been attached. As has been consistently ruled by this ISSUE # 2: Whether Chuidian may raise the issue of fraud in his motion to lift the attachment
Court, the determination of the existence of grounds to discharge a writ of and in the ensuing motion for reconsideration.
attachment rests in the sound discretion of the lower courts
x First. To file a counterbond in accordance with Rule 57, Section 12, HELD # 2: NO.
which provides: x Petitioner may argue, albeit belatedly, that he also raised the issue that there
o SEC. 12. Discharge of attachment upon giving counterbond. - At was no evidence of fraud on record other than the affidavit of PCGG Chairman
anytime after an order of attachment has been granted, the party Gunigundo. This issue of fraud, however, touches on the very merits of the main
whose property has been attached, or the person appearing on his case which accuses petitioner of committing fraudulent acts in his dealings with
behalf, may, upon reasonable notice to the applicant, apply to the the government. Moreover, this alleged fraud was one of the grounds for the
judge who granted the order, or to the judge of the court in which application of the writ, and the Sandiganbayan granted said application after it
the action is pending, for an order discharging the attachment found a prima facie case of fraud committed by petitioner.
wholly or in part on the security given. The judge shall, after x In fine, fraud was not only one of the grounds for the issuance of the preliminary
hearing, order the discharge of the attachment if a cash deposit is attachment, it was at the same time the government's cause of action in the
made, or a counterbond executed to the attaching creditor is filed, main case.
on behalf of the adverse party, with the clerk or judge of the court x We have uniformly held that:
where the application is made, in an amount equal to the value of o x x x when the preliminary attachment is issued upon a ground
the property attached as determined by the judge, to secure the which is at the same time the applicant's cause of action; e.g., "an
payment of any judgment that the attaching creditor may recover action for money or property embezzled or fraudulently misapplied
in the action. Upon the filing of such counter-bond, copy thereof or converted to his own use by a public officer, or an officer of a
shall forthwith be served on the attaching creditor or his lawyer. corporation, or an attorney, factor, broker, agent, or clerk, in the
Upon the discharge of an attachment in accordance with the course of his employment as such, or by any other person in a
provisions of this section the property attached, or the proceeds of fiduciary capacity, or for a willful violation of duty," or "an action
any sale thereof, shall be delivered to the party making the deposit against a party who has been guilty of fraud in contracting the debt
or giving the counter-bond, or the person appearing on his behalf, or incurring the obligation upon which the action is brought," the
the deposit or counter-bond aforesaid standing in place of the defendant is not allowed to file a motion to dissolve the attachment
property so released. Should such counterbond for any reason be under Section 13 of Rule 57 by offering to show the falsity of the
4
RECTO, GAYLE ANGELI M.
2011-0008 | AUSL
Personal Notes on Remedial Law 2 Review (based on the syllabus of Prof. Henedino M. Brondial)

factual averments in the plaintiff's application and affidavits on x RTC Æ issued an Order9 directing the petitioners to post a bond in the amount
which the writ was based - and consequently that the writ based of P7,000,000.00 before the court issues the writ of attachment
thereon had been improperly or irregularly issued - the reason x Torreses filed the said bond
being that the hearing on such a motion for dissolution of the writ x RTC Æ issued the writ
would be tantamount to a trial of the merits of the action. In other o directing the sheriff to attach the estate, real or personal, of the
words, the merits of the action would be ventilated at a mere respondents
hearing of a motion, instead of at the regular trial.34 (Underscoring x Summons was then served upon the Satsatins
ours) x Satsatins Æ filed their answer and a Motion to Discharge Writ of Attachment
x Thus, this Court has time and again ruled that the merits of the action in which a o the bond was issued before the issuance of the writ of attachment;
writ of preliminary attachment has been issued are not triable on a motion for the writ of attachment was issued before the summons was
dissolution of the attachment, otherwise an applicant for the lifting of the writ received by the respondents; the sheriff did not serve copies of the
could force a trial of the merits of the case on a mere motion.35 application for attachment, order of attachment, plaintiffs’ affidavit,
x It is not the Republic's fault that the litigation has been protracted. There is as and attachment bond, to the respondents; the sheriff did not
yet no evidence of fraud on the part of petitioner. Petitioner is only one of the submit a sheriff’s return in violation of the Rules; and the grounds
twenty-three (23) defendants in the main action. As such, the litigation would cited for the issuance of the writ are baseless and devoid of merit.
take longer than most cases. Petitioner cannot invoke this delay in the In the alternative, respondents offered to post a counter-bond for
proceedings as an excuse for not seeking the proper recourse in having the writ the lifting of the writ of attachment.
of attachment lifted in due time. If ever laches set in, it was petitioner, not the x RTC Æ denied the motion; directed Satsatins to file counterbond of 7M
government, who failed to take action within a reasonable time period. x Satsatins Æ filed an Urgent Motion to Lift/Set Aside Order
Challenging the issuance of the writ of attachment four (4) years after its o RTC Æ denied; denied also the ensuing Urgent MR
implementation showed petitioner's apparent indifference towards the x Satsatins Æ filed a Petition for Certiorari, Mandamus and Prohibition with
proceedings before the Sandiganbayan. Preliminary Injunction and Temporary Restraining Order before the CA
x In sum, petitioner has failed to convince this Court that the Sandiganbayan o Respondents argued that the subject writ was improper and
gravely abused its discretion in a whimsical, capricious and arbitrary manner. irregular having been issued and enforced without the lower court
There are no compelling reasons to warrant the immediate lifting of the acquiring jurisdiction over the persons of the respondents. They
attachment even as the main case is still pending. On the other hand, allowing maintained that the writ of attachment was implemented without
the discharge of the attachment at this stage of the proceedings would put in serving upon them the summons together with the complaint. They
jeopardy the right of the attaching party to realize upon the relief sought and also argued that the bond issued in favor of the petitioners was
expected to be granted in the main or principal action. It would have the effect defective, because the bonding company failed to obtain the proper
of prejudging the main case. clearance that it can transact business with the RTC of Dasmariñas,
x The attachment is a mere provisional remedy to ensure the safety and Cavite. They added that the various clearances which were issued
preservation of the thing attached until the plaintiff can, by appropriate in favor of the bonding company were applicable only in the courts
proceedings, obtain a judgment and have such property applied to its of the cities of Pasay, Pasig, Manila, and Makati, but not in the
satisfaction.36 To discharge the attachment at this stage of the proceedings RTC, Imus, Cavite.
would render inutile any favorable judgment should the government prevail in x CA Æ ruled in favor of Satsatins; ordered the lifting of the writ of attachment;
the principal action against petitioner. Thus, the Sandiganbayan, in issuing the denied ensuing MR of the Torreses
questioned resolutions, which are interlocutory in nature, committed no grave x Torreses Æ filed a Rule 45 before the SC
abuse of discretion amounting to lack or excess of jurisdiction. As long as the
Sandiganbayan acted within its jurisdiction, any alleged errors committed in the ISSUE: Whether the CA was correct in lifting the subject writ of attachment.
exercise of its jurisdiction will amount to nothing more than errors of judgment
which are reviewable by timely appeal and not by special civil action of HELD: YES.
certiorari.37 x A writ of preliminary attachment is defined as a provisional remedy
x Moreover, we have held that when the writ of attachment is issued upon a issued upon order of the court where an action is pending to be levied
ground which is at the same time the applicant's cause of action, the only other upon the property or properties of the defendant therein, the same to
way the writ can be lifted or dissolved is by a counterbond, in accordance with be held thereafter by the sheriff as security for the satisfaction of
Section 12 of the same rule.38 This recourse, however, was not availed of by whatever judgment that might be secured in the said action by the
petitioner, as noted by the Solicitor General in his comment.39 attaching creditor against the defendant.36
x To reiterate, there are only two ways of quashing a writ of attachment: (a) by x In the case at bar, the CA correctly found that there was grave abuse of
filing a counterbond immediately; or (b) by moving to quash on the ground of discretion amounting to lack of or in excess of jurisdiction on the part of the trial
improper and irregular issuance.40 These grounds for the dissolution of an court in approving the bond posted by petitioners despite the fact that not all the
attachment are fixed in Rule 57 of the Rules of Court and the power of the Court requisites for its approval were complied with. In accepting a surety bond, it is
to dissolve an attachment is circumscribed by the grounds specified therein.41 necessary that all the requisites for its approval are met; otherwise, the bond
Petitioner's motion to lift attachment failed to demonstrate any infirmity or defect should be rejected.37
in the issuance of the writ of attachment; neither did he file a counterbond. x Every bond should be accompanied by a clearance from the Supreme Court
showing that the company concerned is qualified to transact business which is
valid only for thirty (30) days from the date of its issuance.38 However, it is
SOFIA TORRES, FRUCTOSA TORRES, HEIRS OF MARIO TORRES and SOLAR RESOURCES, apparent that the Certification39 issued by the Office of the Court Administrator
INC., Petitioners, vs. NICANOR SATSATIN, EMILINDA AUSTRIA SATSATIN, NIKKI NORMEL (OCA) at the time the bond was issued would clearly show that the bonds
SATSATIN and NIKKI NORLIN SATSATIN, Respondents. offered by Western Guaranty Corporation may be accepted only in the RTCs of
G.R. No. 166759 November 25, 2009 the cities of Makati, Pasay, and Pasig. Therefore, the surety bond issued by the
THIRD DIVISION bonding company should not have been accepted by the RTC of Dasmariñas,
PERALTA, J. Branch 90, since the certification secured by the bonding company from the OCA
at the time of the issuance of the bond certified that it may only be accepted in
FACTS: the above-mentioned cities. Thus, the trial court acted with grave abuse of
x The siblings Sofia Torres (Sofia), Fructosa Torres (Fructosa), and Mario Torres discretion amounting to lack of or in excess of jurisdiction when it issued the writ
(Mario) each own adjacent 20,000 square meters track of land situated at Barrio of attachment founded on the said bond.
Lankaan, Dasmariñas, Cavite x Moreover, in provisional remedies, particularly that of preliminary
x Agripina (their mother) Æ agreed to sell the properties to Nicanor Satsatin attachment, the distinction between the issuance and the
(Nicanor) after consultation with the said siblings implementation of the writ of attachment is of utmost importance to
x Nicanor offered to sell the properties to Solar Resources, Inc. (Solar), which the the validity of the writ. The distinction is indispensably necessary to
latter agreed (10,000 sqm for for P35,000,000.00) determine when jurisdiction over the person of the defendant should
x Torreses Æ alleged that Nicanor was supposed to remit to them the total be acquired in order to validly implement the writ of attachment upon
amount of P28,000,000.00 or P9,333,333.00 each to Sofia, Fructosa, and the his person.
heirs of Mario x This Court has long put to rest the issue of when jurisdiction over the person of
o Nicanor only remitted the total amount of P9,000,000.00, leaving the defendant should be acquired in cases where a party resorts to provisional
an unremitted balance of P19,000,000.00 remedies. A party to a suit may, at any time after filing the complaint, avail of
x Torreses Æ filed with the RTC Dasma Cavite Br 90 a Complaint7 for sum of the provisional remedies under the Rules of Court. Specifically, Rule 57 on
money and damages against Nicanor, Ermilinda Satsatin, Nikki Normel Satsatin, preliminary attachment speaks of the grant of the remedy "at the
and Nikki Norlin Satsatin commencement of the action or at any time before entry of judgment."40 This
o filed an Ex-Parte Motion for the Issuance of a Writ of Attachment,8 phrase refers to the date of the filing of the complaint, which is the moment that
alleging among other things: that respondents are about to depart marks "the commencement of the action." The reference plainly is to a time
the Philippines; that they have properties, real and personal in before summons is served on the defendant, or even before summons issues.41
Metro Manila and in the nearby provinces; that the amount due x In Davao Light & Power Co., Inc. v. Court of Appeals,42 this Court clarified the
them is P19,000,000.00 above all other claims; that there is no actual time when jurisdiction should be had:
other sufficient security for the claim sought to be enforced; and o It goes without saying that whatever be the acts done by the Court
that they are willing to post a bond fixed by the court to answer for prior to the acquisition of jurisdiction over the person of defendant x
all costs which may be adjudged to the respondents and all x x issuance of summons, order of attachment and writ of
damages which respondents may sustain by reason of the attachment x x x these do not and cannot bind and affect the
attachment prayed for, if it shall be finally adjudged that petitioners defendant until and unless jurisdiction over his person is eventually
are not entitled thereto obtained by the court, either by service on him of summons or
5
RECTO, GAYLE ANGELI M.
2011-0008 | AUSL
Personal Notes on Remedial Law 2 Review (based on the syllabus of Prof. Henedino M. Brondial)

other coercive process or his voluntary submission to the court’s x Wee Æ learned some news on Wincorp's financial condition
authority. Hence, when the sheriff or other proper officer o He then discovered that the company extended a loan equal to his
commences implementation of the writ of attachment, it is essential total money placement to a corporation [Power Merge] with a
that he serve on the defendant not only a copy of the applicant’s subscribed capital of only P37.5M. This credit facility originated
affidavit and attachment bond, and of the order of attachment, as from another loan of about P1.5B extended by Wincorp to another
explicitly required by Section 5 of Rule 57, but also the summons corporation [Hottick Holdings]. When the latter defaulted in its
addressed to said defendant as well as a copy of the complaint x x obligation, Wincorp instituted a case against it and its surety.
x. (Emphasis supplied.) Settlement was, however, reached in which Hottick's president,
x In Cuartero v. Court of Appeals,43 this Court held that the grant of the Luis Juan L. Virata (Virata), assumed the obligation of the surety
provisional remedy of attachment involves three stages: first, the court issues o petitioner's money placements were transferred without his
the order granting the application; second, the writ of attachment knowledge and consent to the loan account of Power Merge
issues pursuant to the order granting the writ; and third, the writ is through an agreement that virtually freed the latter of any liability
implemented. For the initial two stages, it is not necessary that x Wee Æ filed an action for damages before RTC Manila
jurisdiction over the person of the defendant be first obtained. o the trial court ordered the issuance of a writ of preliminary
However, once the implementation of the writ commences, the court attachment against the properties not exempt from execution of all
must have acquired jurisdiction over the defendant, for without such the defendants in the civil case subject, among others, to
jurisdiction, the court has no power and authority to act in any manner petitioner's filing of a P50M-bond.
against the defendant. Any order issuing from the Court will not bind x RTC Æ ordered the issuance of a writ of preliminary attachment against the
the defendant.44 properties not exempt from execution of all the defendants in the civil case
x Thus, it is indispensable not only for the acquisition of jurisdiction over the subject, among others, to petitioner's filing of a P50M-bond.
person of the defendant, but also upon consideration of fairness, to apprise the x Tankiansee Æ moved for the discharge of the attachment
defendant of the complaint against him and the issuance of a writ of preliminary o RTC Æ denied
attachment and the grounds therefor that prior or contemporaneously to the x co-defendants, Virata and UEM-MARA Philippines Corporation (UEM-MARA) Æ
serving of the writ of attachment, service of summons, together with a copy of filed a Rule 65 before the CA
the complaint, the application for attachment, the applicant’s affidavit and bond, o CA Æ denied
and the order must be served upon him. x Tankiansee Æ filed before the trial court another Motion to Discharge
x In the instant case, assuming arguendo that the trial court validly issued the writ Attachment,20 re-pleading the grounds he raised in his first motion but raising
of attachment on November 15, 2002, which was implemented on November 19, the following additional grounds: (1) that he was not present in Wincorp's board
2002, it is to be noted that the summons, together with a copy of the complaint, meetings approving the questionable transactions;21 and (2) that he could not
was served only on November 21, 2002. have connived with Wincorp and the other defendants because he and Pearlbank
x At the time the trial court issued the writ of attachment on November 15, 2002, Securities, Inc., in which he is a major stockholder, filed cases against the
it can validly to do so since the motion for its issuance can be filed "at the company as they were also victimized by its fraudulent schemes
commencement of the action or at any time before entry of judgment." o RTC Æ denied (since the same has been passed upon by the CA);
However, at the time the writ was implemented, the trial court has not acquired denied ensuing MR
jurisdiction over the persons of the respondent since no summons was yet x Tankiansee Æ filed a Rule 65 before the CA
served upon them. The proper officer should have previously or simultaneously o CA Æ ruled in favor of Tankiansee and reversed RTC
with the implementation of the writ of attachment, served a copy of the x Wee Æ filed a Rule 45 before the SC
summons upon the respondents in order for the trial court to have acquired
jurisdiction upon them and for the writ to have binding effect. Consequently, ISSUE: Whether there is basis for issuing a writ of attachment in favor of Wee.
even if the writ of attachment was validly issued, it was improperly or irregularly
enforced and, therefore, cannot bind and affect the respondents. HELD: NO.
x Moreover, although there is truth in the petitioners’ contention that an x In the case at bench, the basis of petitioner's application for the issuance of the
attachment may not be dissolved by a showing of its irregular or improper writ of preliminary attachment against the properties of respondent is Section
issuance if it is upon a ground which is at the same time the applicant’s cause of 1(d) of Rule 57 of the Rules of Court which pertinently reads:
action in the main case, since an anomalous situation would result if the issues o Section 1. Grounds upon which attachment may issue.-At the
of the main case would be ventilated and resolved in a mere hearing of a commencement of the action or at any time before entry of
motion. However, the same is not applicable in the case bar. It is clear from the judgment, a plaintiff or any proper party may have the property of
respondents’ pleadings that the grounds on which they base the lifting of the the adverse party attached as security for the satisfaction of any
writ of attachment are the irregularities in its issuance and in the service of the judgment that may be recovered in the following cases:
writ; not petitioners’ cause of action. ƒ (d) In an action against a party who has been guilty
x Further, petitioners’ contention that respondents are barred by estoppel, laches, of a fraud in contracting the debt or incurring the
and prescription from questioning the orders of the RTC issuing the writ of obligation upon which the action is brought, or in the
attachment and that the issue has become moot and academic by the renewal of performance thereof.
the attachment bond covering after its expiration, is devoid of merit. As correctly x For a writ of attachment to issue under this rule, the applicant must
held by the CA: sufficiently show the factual circumstances of the alleged fraud
o There are two ways of discharging the attachment. First, to file a because fraudulent intent cannot be inferred from the debtor's mere
counter-bond in accordance with Section 12 of Rule 57. Second[,] non-payment of the debt or failure to comply with his obligation.30 The
[t]o quash the attachment on the ground that it was irregularly or applicant must then be able to demonstrate that the debtor has
improvidently issued, as provided for in Section 13 of the same intended to defraud the creditor.31 In Liberty Insurance Corporation v. Court
rule. Whether the attachment was discharged by either of the two of Appeals,32 we explained as follows:
ways indicated in the law, the attachment debtor cannot be o To sustain an attachment on this ground, it must be shown that the
deemed to have waived any defect in the issuance of the debtor in contracting the debt or incurring the obligation intended
attachment writ by simply availing himself of one way of to defraud the creditor. The fraud must relate to the execution of
discharging the attachment writ, instead of the other. The filing of the agreement and must have been the reason which induced the
a counter-bond is merely a speedier way of discharging the other party into giving consent which he would not have otherwise
attachment writ instead of the other way.45 given. To constitute a ground for attachment in Section 1 (d), Rule
x Moreover, again assuming arguendo that the writ of attachment was validly 57 of the Rules of Court, fraud should be committed upon
issued, although the trial court later acquired jurisdiction over the respondents contracting the obligation sued upon. A debt is fraudulently
by service of the summons upon them, such belated service of summons on contracted if at the time of contracting it the debtor has a
respondents cannot be deemed to have cured the fatal defect in the preconceived plan or intention not to pay, as it is in this case.
enforcement of the writ. The trial court cannot enforce such a coercive process Fraud is a state of mind and need not be proved by direct evidence
on respondents without first obtaining jurisdiction over their person. The but may be inferred from the circumstances attendant in each
preliminary writ of attachment must be served after or simultaneous with the case.33
service of summons on the defendant whether by personal service, substituted x In the instant case, petitioner's October 12, 2000 Affidavit34 is bereft of any
service or by publication as warranted by the circumstances of the case. The factual statement that respondent committed a fraud. The affidavit narrated only
subsequent service of summons does not confer a retroactive acquisition of the alleged fraudulent transaction between Wincorp and Virata and/or Power
jurisdiction over her person because the law does not allow for retroactivity of a Merge, which, by the way, explains why this Court, in G.R. No. 162928, affirmed
belated service. the writ of attachment issued against the latter. As to the participation of
respondent in the said transaction, the affidavit merely states that respondent,
ALEJANDRO NG WEE, petitioner, vs. MANUEL TANKIANSEE, respondent. an officer and director of Wincorp, connived with the other defendants in the civil
G.R. No. 171124 February 13, 2008 case to defraud petitioner of his money placements. No other factual averment
THIRD DIVISION or circumstance details how respondent committed a fraud or how he connived
NACHURA, J.: with the other defendants to commit a fraud in the transaction sued upon. In
other words, petitioner has not shown any specific act or deed to support the
FACTS: allegation that respondent is guilty of fraud.
x Petitioner Alejandro Ng Wee, a valued client of Westmont Bank (now United x The affidavit, being the foundation of the writ,35 must contain such particulars
Overseas Bank), made several money placements totaling P210,595,991.62 with as to how the fraud imputed to respondent was committed for the court to
the bank's affiliate, Westmont Investment Corporation (Wincorp), a domestic decide whether or not to issue the writ.36 Absent any statement of other factual
entity engaged in the business of an investment house with the authority and circumstances to show that respondent, at the time of contracting the obligation,
license to extend credit. had a preconceived plan or intention not to pay, or without any showing of how
6
RECTO, GAYLE ANGELI M.
2011-0008 | AUSL
Personal Notes on Remedial Law 2 Review (based on the syllabus of Prof. Henedino M. Brondial)

respondent committed the alleged fraud, the general averment in the affidavit damages which he may sustain by reason of the injunction or temporary restraining order if
that respondent is an officer and director of Wincorp who allegedly connived with the court should finally decide that the applicant was not entitled thereto. Upon approval of
the other defendants to commit a fraud, is insufficient to support the issuance of the requisite bond, a writ of preliminary injunction shall be issued. (4a)
a writ of preliminary attachment.37 In the application for the writ under the said
ground, compelling is the need to give a hint about what constituted the fraud (c) When an application for a writ of preliminary injunction or a temporary
and how it was perpetrated38 because established is the rule that fraud is never restraining order is included in a complaint or any initiatory pleading, the case, if filed in a
presumed.39 Verily, the mere fact that respondent is an officer and director of multiple-sala court, shall be raffled only after notice to and in the presence of the adverse
the company does not necessarily give rise to the inference that he committed a party or the person to be enjoined. In any event, such notice shall be preceded, or
fraud or that he connived with the other defendants to commit a fraud. While contemporaneously accompanied, by service of summons, together with a copy of the
under certain circumstances, courts may treat a corporation as a mere complaint or initiatory pleading and the applicant's affidavit and bond, upon the adverse party
aggroupment of persons, to whom liability will directly attach, this is only done in the Philippines.
when the wrongdoing has been clearly and convincingly established.40
x Let it be stressed that the provisional remedy of preliminary attachment is harsh However, where the summons could not be served personally or by substituted service despite
and rigorous for it exposes the debtor to humiliation and annoyance.41 The rules diligent efforts, or the adverse party is a resident of the Philippines temporarily absent
governing its issuance are, therefore, strictly construed against the applicant,42 therefrom or is a nonresident thereof, the requirement of prior or contemporaneous service of
such that if the requisites for its grant are not shown to be all present, the court summons shall not apply.
shall refrain from issuing it, for, otherwise, the court which issues it acts in
excess of its jurisdiction.43 Likewise, the writ should not be abused to cause (d) The application for a temporary restraining order shall thereafter be acted upon
unnecessary prejudice. If it is wrongfully issued on the basis of false or only after all parties are heard in a summary hearing which shall be conducted within twenty-
insufficient allegations, it should at once be corrected.44 four (24) hours after the sheriff's return of service and/or the records are received by the
x Considering, therefore, that, in this case, petitioner has not fully satisfied the branch selected by raffle and to which the records shall be transmitted immediately.
legal obligation to show the specific acts constitutive of the alleged fraud
committed by respondent, the trial court acted in excess of its jurisdiction when 4. Damages
it issued the writ of preliminary attachment against the properties of respondent.
x We are not unmindful of the rule enunciated in G.B. Inc., etc. v. Sanchez, et
Section 8. Judgment to include damages against party and sureties. — At the trial,
al.,45 that
the amount of damages to be awarded to either party, upon the bond of the adverse party,
o [t]he merits of the main action are not triable in a motion to
shall be claimed, ascertained, and awarded under the same procedure prescribed in section 20
discharge an attachment otherwise an applicant for the dissolution
of Rule 57.
could force a trial of the merits of the case on his motion.46
x However, the principle finds no application here because petitioner has
not yet fulfilled the requirements set by the Rules of Court for the
issuance of the writ against the properties of respondent.47 The evil
sought to be prevented by the said ruling will not arise, because the TERESITA V. IDOLOR, petitioner, vs. HON. COURT OF APPEALS, SPS. GUMERSINDO DE
propriety or impropriety of the issuance of the writ in this case can be GUZMAN and ILUMINADA DE GUZMAN and HON. PRUDENCIO A. CASTILLO, JR., Presiding
determined by simply reading the complaint and the affidavit in Judge, Regional Trial Court, National Capital Judicial Region, Branch 220, Quezon City,
support of the application. respondents.
x Furthermore, our ruling in G.R. No. 162928, to the effect that the writ of G.R. No. 141853 February 7, 2001
attachment is properly issued insofar as it concerns the properties of Virata and THIRD DIVISION
UEM-MARA, does not affect respondent herein, for, as correctly ruled by the CA, GONZAGA-REYES, J.
respondent is "never a party thereto."48 Also, he is not in the same situation as
Virata and UEM-MARA since, as aforesaid, while petitioner's affidavit detailed the FACTS:
alleged fraudulent scheme perpetrated by Virata and/or Power Merge, only a x On 21 March 1994, to secure a loan of P520,000.00, Teresita Idolor executed in
general allegation of fraud was made against respondent. favor of Gumersindo De Guzman a Deed of Real Estate Mortgage with right of
x We state, in closing, that our ruling herein deals only with the writ of preliminary extra-judicial foreclosure upon failure to redeem the mortgage on or before 20
attachment issued against the properties of respondent-it does not concern the September 1994.
other parties in the civil case, nor affect the trial court's resolution on the merits x On 21 September 1996, Iluminada de Guzman, wife of Gumersindo de Guzman,
of the aforesaid civil case. filed a complaint against dolor before the Office of the Barangay Captain of
Barangay Ramon Magsaysay, Quezon City, which resulted in a "Kasunduang Pag-
aayos."
B. Preliminary Injunction (Rule 58) x Idolor failed to comply with her undertaking; thus Gumersindo filed a motion for
execution before the Office of the Barangay captain who subsequently issued a
certification to file action.
1. Definition, Classes
x On 21 March 1997, Gumersindo De Guzman filed an extra judicial foreclosure of
the real estate mortgage pursuant to the parties agreement set forth in the real
Section 1. Preliminary injunction defined; classes. — A preliminary injunction is an estate mortgage.On 23 May 1997, the mortgaged property was sold in a public
order granted at any stage of an action or proceeding prior to the judgment or final order, auction to respondent Gumersindo, as the highest bidder and consequently, the
requiring a party or a court, agency or a person to refrain from a particular act or acts. It may Sheriff's Certificate of Sale was registered with the Registry of Deeds of Quezon
also require the performance of a particular act or acts, in which case it shall be known as a City on 23 June 1997.
preliminary mandatory injunction. x On 25 June 1998, Idolor filed with the Regional Trial Court of Quezon City,
Branch 220, a complaint for annulment of Sheriff's Certificate of Sale with prayer
2. Grounds; TRO for the issuance of a temporary restraining order (TRO) and a writ of preliminary
injunction against the De Guzman spouses, Deputy Sheriffs Marino Cachero and
Section 3. Grounds for issuance of preliminary injunction. — A preliminary injunction Rodolfo Lescano and the Registry of Deeds of Quezon City.
may be granted when it is established: x In the meantime, a temporary restraining order was issued by the trial court. On
28 July 1998, the trial court issued a writ of preliminary injunction enjoining the
(a) That the applicant is entitled to the relief demanded, and the whole or part of Sheriff and the Registry of Deeds from causing the issuance of a final deed of
such relief consists in restraining the commission or continuance of the act or acts complained sale and consolidation of ownership of the subject property in favor of the De
of, or in requiring the performance of an act or acts either for a limited period or perpetually; Guzman spouses.
x The trial court denied the motion for reconsideration filed by the de Guzman
(b) That the commission, continuance or non-performance of the act or acts spouses.
complained of during the litigation would probably work injustice to the applicant; or x Spouses de Guzman filed with the Court of Appeals a petition for certiorari
seeking annulment of the trial court's order.
(c) That a party, court, agency or a person is doing, threatening, or is attempting to x On 28 September 1999, the appellate court granted the petition and annulled the
do, or is procuring or suffering to be done some act or acts probably in violation of the rights assailed writ of preliminary injunction.
of the applicant respecting the subject of the action or proceeding, and tending to render the x Teresita Idolor filed her motion for reconsideration which was denied in a
judgment ineffectual. resolution dated 4 February 2000.
x Teresita Idolor filed the petition for review on certiorari with the Supreme Court.
3. Requirements
ISSUE: Whether Idolor has proprietary right over the foreclosed property to entitle her to the
issuance of a writ of injunction.
Section 4. Verified application and bond for preliminary injunction or temporary
restraining order. — A preliminary injunction or temporary restraining order may be granted HELD: NO.
only when: x Injunction is a preservative remedy aimed at protecting substantive rights and
interests. Before an injunction can be issued, it is essential that the following
(a) The application in the action or proceeding is verified, and shows facts entitling requisites be present: 1) there must be aright in esse or the existence of a right
the applicant to the relief demanded; and to be protected; 2) the act against which the injunction is to be directed is a
violation of such right. Hence the existence of a right violated, is a prerequisite to
(b) Unless exempted by the court the applicant files with the court where the action the granting of an injunction. Injunction is not designed to protect contingent or
or proceeding is pending, a bond executed to the party or person enjoined, in an amount to future rights. Failure to establish either the existence of a clear and positive right
be fixed by the court, to the effect that the applicant will pay to such party or person all which should be judicially protected through the writ of injunction or that the
7
RECTO, GAYLE ANGELI M.
2011-0008 | AUSL
Personal Notes on Remedial Law 2 Review (based on the syllabus of Prof. Henedino M. Brondial)

defendant has committed or has attempted to commit any act which has Further, there must be a showing that the invasion of the right is material and
endangered or tends to endanger the existence of said right, is a sufficient substantial and that there is an urgent and paramount necessity for the writ to
ground for denying the injunction. The controlling reason for the existence of prevent a serious damage. In this case, complainant had been duly proclaimed
the judicial power to issue the writ is that the court may thereby prevent a as the winning candidate for punong barangay. He had taken his oath of office.
threatened or continuous irremediable injury to some of the parties before their Unless his election was annulled, he was entitled to all the rights of said office.
claims can be thoroughly investigated and advisedly adjudicated. It is to be We do not see how the complainant's exercise of such rights would cause an
resorted to only when there is a pressing necessity to avoid injurious irreparable injury or violate the right of the losing candidate so as to justify the
consequences which cannot be remedied under any standard of compensation. issuance of a temporary restraining order "to maintain the status quo."
It is always a ground for denying injunction that the party seeking it has
insufficient title or interest to sustain it, and no claim to the ultimate relief sought
- in other words, that she shows no equity. The possibility of irreparable damage MICHAEL J. LAGROSAS, Petitioner, vs. BRISTOL-MYERS SQUIBB (PHIL.), INC./MEAD
without proof of actual existing right is not aground for an injunction. JOHNSON PHIL., RICHARD SMYTH as General Manager and FERDIE SARFATI, as Medical
x In the instant case, we agree with the respondent Court that petitioner has no Sales Director, Respondents.
more proprietary right to speak of over the foreclosed property to entitle her to G.R. No. 168637
the issuance of a writ of injunction. It appears that the mortgaged property was BRISTOL-MYERS SQUIBB (PHIL.), INC./MEAD JOHNSON PHIL.,Petitioner, vs. COURT OF
sold in a public auction to private respondent Gumersindo on May 23, 1997 and APPEALS and MICHAEL J. LAGROSAS, Respondents.
the sheriff's certificate of sale was registered with the Registry of Deeds of G.R. No. 170684, 12 September 2008,
Quezon City on June 23, 1997. Petitioner had one year from the registration of the Second Division
sheriff's sale to redeem the property but she failed to exercise her right on or Quisumbing [J]
before June 23, 1998, thus spouses de Guzman are now entitled to a
conveyance and possession of the foreclosed property. When petitioner filed her FACTS:
complaint for annulment of sheriff's sale against private respondents with prayer x Michael J. Lagrosas was employed by Bristol-Myers Squibb (Phil.), Inc./Mead
for the issuance of a writ of preliminary injunction on June 25, 1998, she failed Johnson Phil. from 6 January 1997 until 23 March 2000 as Territory Manager in
to show sufficient interest or title in the property sought to be protected as her its Medical Sales Force Division.
right of redemption had already expired on June 23, 1998, i.e. two (2) days x On 4 February 2000, Ma. Dulcinea S. Lim, also a Territory Manager and Lagrosas’
before the filing of the complaint. It is always a ground for denying injunction former girlfriend, attended a district meeting of territory managers at McDonald’s
that the party seeking it has insufficient title or interest to sustain it, and no Alabang Town Center.
claim to the ultimate relief sought - in other words, that she shows no equity.11 x After the meeting, she dined out with her friends and rode with Cesar R.
The possibility of irreparable damage without proof of actual existing right is not Menquito, Jr. Lim told Menquito not to stop his car but Lagrosas followed them
aground for an injunction and slammed Menquito’s car thrice.
x Menquito and Lim alighted from the car. Lagrosas approached them and hit
Menquito with a metal steering wheel lock. When Lim tried to intervene,
Lagrosas accidentally hit her head.
x Upon learning of the incident, Bristol-Myers required Lagrosas to explain in
writing why he should not be dismissed for assaulting a co-employee outside of
RIMEO S. GUSTILO, complainant, vs. HON. RICARDO S. REAL, SR., Presiding Judge, 2nd business hours.
Municipal Circuit Trial Court of Victorias-Manapla, Negros Occidental, respondent. x After administrative proceedings, on 23 March 2000, Bristol-Myers dismissed
AM MTJ-00-1250, 28 February 2001, Lagrosas effective immediately. Lagrosas then filed a complaint for illegal
Second Division dismissal, among others.
Quisumbing [J] x On 28 February 2002, the Labor Arbiter rendered a Decision, declaring the
dismissal illegal.
FACTS: x On appeal, the National Labor Relations Commission (NLRC) set aside the
x Rimeo Gustilo was a candidate for punong barangay of Barangay Punta Mesa, Decision of Labor Arbiter in its Decision dated 24 September 2002. Lagrosas
Manapla, Negros Occidental in the 12 May 1997 elections. moved for reconsideration.
x His lone opponent was Weddy C. Libo-on, then the incumbent punong barangay x On 7 May 2003, the NLRC issued a Resolution reversing its earlier ruling, which
and the representative of the Association of Barangay Captains (ABC) to the ratiocinated that the incident was not work-related since it occurred only after
Sangguniang Bayan of Manapla and the Sangguniang Panlalawigan of Negros the district meeting of territory managers.
Occidental. x Bristol-Myers filed a motion for reconsideration which the NLRC denied in an
x Both Gustilo and Libo-on garnered 819 votes during the elections, resulting in a Order dated 4 February 2004. Later, the Labor Arbiter issued a writ of execution.
tie. The breaking of the tie by the Board of Canvassers was in Gustilo's favor and Notices of garnishment were then served upon the Philippine British Assurance
he was proclaimed duly elected punong barangay. Co., Inc. for the supersedeas bond posted by Bristol-Myers and the Bank of the
x On 20 May 1997, Libo-on filed an election protest case before the MCTC of Philippine Islands for the balance of the judgment award.
Victorias-Manapla, Negros Occidental. x Bristol-Myers moved to quash the writ of execution contending that it timely filed
x Libo-on sought the recounting of ballots in two precincts, preliminary prohibitory a petition for certiorari with the Court of Appeals.
injunction, and damages. x The appellate court gave due course to Bristol-Myers’ petition and issued a
x On 29 May 1997, the judge issued a temporary restraining order (TRO), after temporary restraining order (TRO) enjoining the enforcement of the writ of
receiving the Lobo-on's evidence ex parte, and annulled the proclamation of execution and notices of garnishment.
complainant as the duly elected punong barangay of Punta Mesa, Manapla. x Upon the expiration of the TRO, the appellate court issued a writ of preliminary
x On 20 May 1997, Gustilo took his oath of office as punong barangay. That same injunction dated 17 September 2004. Bristol-Myers then moved to discharge and
day, he also filed a petition for certiorari before the Regional Trial Court of Silay release the TRO cash bond.
City, Negros Occidental, Branch 69. x It argued that since it has posted an injunction cash bond, the TRO cash bond
x On 5 June 1997, the RTC lifted the TRO issued by the Judge and declared as null should be legally discharged and released. On 28 January 2005, the appellate
and void the order nullifying Gustilo's proclamation as duly elected punong court rendered a Decision, granting the petition. Lagrosas filed a motion for
barangay. reconsideration which the appellate court denied.
x Believing that the Judge could not decide the case impartially, Gustilo moved for x In the meantime, Bristol-Myers moved to release the TRO cash bond and
his inhibition. injunction cash bond.
x On 11 June 1997, the Judge denied Gustilo's motion for inhibition and after x On 12 August 2005, the appellate court denied the motion as premature since
hearing Libo-on's motion for permanent injunction, issued a second TRO "to the decision is not yet final and executory due to Lagrosas’ appeal to this Court.
maintain the status quo between the contending parties." Bristol-Myers filed a motion for reconsideration.
x In a verified complaint dated 15 June 1997, Gustilo charged Judge Real with x On 28 October 2005, the appellate court resolved to reconsider and set aside its
gross misconduct, gross incompetence, gross ignorance of the law, and violation August 2005 resolution.
of the Anti-Graft and Corrupt Practices.
ISSUE # 1: Whether the conditions of writ of preliminary injunction were satisfied.
ISSUE # 1: Whether a TRO (the first) can be issued without notice and hearing.
HELD #1. YES.
HELD # 1: NO. x [I]t is settled that the purpose of a preliminary injunction is to prevent
x Under Supreme Court Administrative Circular No. 20-95], whenever an threatened or continuous irremediable injury to some of the parties before their
application for a TRO is filed, the court may act on the application only after all claims can be thoroughly studied and adjudicated. Its sole aim is to preserve the
parties have been notified and heard in a summary hearing. In other words, a status quo until the merits of the case can be heard fully.
summary hearing may not be dispensed with. x A preliminary injunction may be granted only when, among other things, the
applicant, not explicitly exempted, files with the court where the action or
ISSUE # 2: Whether a TRO (the second) can be issued in favor of the losing candidate on proceeding is pending, a bond executed to the party or person enjoined, in an
the ground that "extreme urgency" and "grave injustice and irreparable injury will arise." amount to be fixed by the court, to the effect that the applicant will pay such
party or person all damages which he may sustain by reason of the injunction or
HELD # 2: NO. temporary restraining order if the court should finally decide that the applicant
x Before an injunctive writ can be issued, it is essential that the following requisites was not entitled thereto. Upon approval of the requisite bond, a writ of
be present: (1) there must be aright in esse or the existence of a right to be preliminary injunction shall be issued.
protected; and (2) the act against which injunction to be directed is a violation of x The injunction bond is intended as a security for damages in case it is finally
such right. The onus probandi is on movant to show that there exists a right to decided that the injunction ought not to have been granted. Its principal purpose
be protected, which is directly threatened by the act sought to be enjoined.
8
RECTO, GAYLE ANGELI M.
2011-0008 | AUSL
Personal Notes on Remedial Law 2 Review (based on the syllabus of Prof. Henedino M. Brondial)

is to protect the enjoined party against loss or damage by reason of the x In this case, we rule that the Principal had the authority to order the immediate
injunction, and the bond is usually conditioned accordingly. transfer of petitioner students because of the 28 November 2002 agreement.28
x In this case, the Court of Appeals issued the writ of preliminary injunction to Petitioner parents affixed their signatures to the minutes of the 28 November
enjoin the implementation of the writ of execution and notices of garnishment 2002 meeting and signified their conformity to transfer their children to another
"pending final resolution of this case or unless the [w]rit is sooner lifted by the school. Petitioners Socorro Canto and Nelia Duro even wrote a letter to inform
Court." the University that they would transfer their children to another school and
x By its Decision, the appellate court disposed of the case by granting Bristol- requested for the pertinent papers needed for the transfer.29 In turn, the
Myers’ petition and reinstating the Decision dated 24 September 2002 of the University did not anymore convene the COSD. The University agreed that it
NLRC which dismissed the complaint for dismissal. It also ordered the discharge would no longer conduct disciplinary proceedings and instead issue the transfer
of the TRO cash bond and injunction cash bond. Thus, both conditions of the writ credentials of petitioner students. Then petitioners reneged on their agreement
of preliminary injunction were satisfied. without any justifiable reason. Since petitioners’ present complaint is one for
injunction, and injunction is the strong arm of equity, petitioners must come to
ISSUE # 2: Whether the injunction bond is a security for the judgment award by the labor court with clean hands. As held in University of the Philippines v. Hon. Catungal,
arbiter. Jr.,:
o Since injunction is the strong arm of equity, he who must apply for
HELD # 2: NO. it must come with equity or with clean hands. This is so because
x [T]he appellate court ruled that Lagrosas had no right to the monetary awards among the maxims of equity are (1) he who seeks equity must do
granted by the labor arbiter and the NLRC, and that the implementation of the equity, and (2) he who comes into equity must come with clean
writ of execution and notices of garnishment was properly enjoined. This in hands. The latter is a frequently stated maxim which is also
effect amounted to a finding that Lagrosas did not sustain any damage by expressed in the principle that he who has done inequity shall not
reason of the injunction. To reiterate, the injunction bond is intended to protect have equity. It signifies that a litigant may be denied relief by a
Lagrosas against loss or damage by reason of the injunction only. Contrary to court of equity on the ground that his conduct has been
Lagrosas’ claim, it is not a security for the judgment award by the labor arbiter. inequitable, unfair and dishonest, or fraudulent, or deceitful as to
the controversy in issue.
NELSON JENOSA and his son NIÑO CARLO JENOSA, SOCORRO CANTO and her son x Here, petitioners, having reneged on their agreement without any justifiable
PATRICK CANTO, CYNTHIA APALISOK and her daughter CYNDY APALISOK, EDUARDO reason, come to court with unclean hands. This Court may deny a litigant relief if
VARGAS and his son CLINT EDUARD VARGAS, and NELIA DURO and her son NONELL his conduct has been inequitable, unfair and dishonest as to the controversy in
GREGORY DURO, Petitioners, vs. REV. FR. JOSE RENE C. DELARIARTE, O.S.A., in his issue
capacity as the incumbent Principal of the High School Department of the University of San
Agustin, and the UNIVERSITY OF SAN AGUSTIN, herein represented by its incumbent SOLID BUILDERS, INC. and MEDINA FOODS INDUSTRIES, INC., Petitioners, vs. CHINA
President REV. FR. MANUEL G. VERGARA, O.S.A., Respondents. BANKING CORPORATION, Respondent.
GR 172138, 8 September 2010, GR 179665, 3 April 2013
Second Division First Division
Carpio [J] Leonardo-De Castro [J]

FACTS: FACTS:
x On 22 November 2002, some students of the University were caught engaging in x During the period from 4 September 1992 to 27 March 1996, China Banking
hazing outside the school premises. The hazing incident was entered into the Corporation (CBC) granted several loans to Solid Builders, Inc. (SBI), which
blotter of the Iloilo City Police. amounted to P139,999,234.34, exclusive of interests and other charges.
x Thereafter, dialogues and consultations were conducted among the school x To secure the loans, Medina Foods Industries, Inc. (MFII) executed in CBC’s
authorities, the apprehended students and their parents. During the 28 favor several surety agreements and contracts of real estate mortgage over
November 2002 meeting, the parties agreed that, instead of the possibility of parcels of land in the Loyola Grand Villas in Quezon City and New Cubao Central
being charged and found guilty of hazing, the students who participated in the in Cainta, Rizal.
hazing incident as initiators would just transfer to another school, while those x Subsequently, SBI proposed to CBC a scheme through which SBI would sell the
who participated as neophytes would be suspended for one month. The parents mortgaged properties and share the proceeds with CBC on a 50-50 basis until
of the apprehended students affixed their signatures to the minutes of the such time that the whole obligation would be fully paid.
meeting to signify their conformity. x SBI also proposed that there be partial releases of the certificates of title of the
x In view of the agreement, the University did not anymore convene the mortgaged properties without the burden of updating interests on all loans. In a
Committee on Student Discipline (COSD) to investigate the hazing incident and letter dated 20 March 2000 addressed to CBC, SBI requested the restructuring of
instead issue the transfer credentials of petitioner students. its loans, a reduction of interests and penalties and the implementation of a
x Then petitioners reneged on their agreement without any justifiable reason. dacion en pago of the New Cubao Central property.
x On 3 January 2003, the students filed a complaint for injunction and damages x In response, CBC sent SBI a letter dated 17 April 2000 stating that the loans had
with the Regional Trial Court, Branch 29, Iloilo City. been completely restructured effective 1 March 1999 in the amount of
x On 5 February 2003, the trial court issued a writ of preliminary injunction and P218,540,646.00. On the aspect of interests and charges, CBC suggested the
directed the University to admit the students during the pendency of the case. updating of the obligation to avoid paying interests and charges.
x The University filed a motion for reconsideration and asked for the dissolution of x Subsequently, in a letter dated 18 September 2000, CBC demanded SBI to settle
the writ. its outstanding account within ten days from receipt thereof. On 5 October 2000,
o The trial court denied their motion. claiming that the interests, penalties and charges imposed by CBC were
x They complied but with reservations. iniquitous and unconscionable and to enjoin CBC from initiating foreclosure
x On 25 March 2003, respondents filed a motion to dismiss. proceedings, SBI and MFII filed a Complaint "To Compel Execution of Contract
o On 19 May 2003, the trial court denied the university's motion. and for Performance and Damages, With Prayer for Writ of Preliminary
x They filed a motion for reconsideration. Injunction and Ex-Parte Temporary Restraining Order" in the Regional Trial Court
x On 28 May 2003, the students filed another complaint for mandatory injunction (RTC) of Pasig City.
praying for the release of the students’ report cards and other credentials. x After hearing the parties, the trial court issued an Order dated 14 December
o The trial court consolidated the two cases. 2000 granting the application of SBI and MFII for the issuance of a writ of
x On 17 June 2003, the trial court issued a writ of preliminary injunction and preliminary injunction.
directed the University to release the students’ report cards and other x CBC sought reconsideration but the trial court denied it in an Order dated 10
credentials. December 2001. Aggrieved, CBC filed a Petition for Certiorari in the Court of
x The University filed a motion for reconsideration. Appeals.
o On 14 July 2003, the trial court issued an Order denying both x The Court of Appeals granted the petition of CBC, set aside the Orders and
motions for reconsideration. dissolved the injunctive writ issued by the RTC of Pasig City.
x On 1 September 2003, the University filed a special civil action for certiorari with x SBI and MFII filed a motion for reconsideration but it was denied by the Court of
the Court of Appeals. Appeals in a Resolution dated 18 September 2007.
x The Court of Appeals reversed the order of the trial court, holding that the case x Hence, the petition.
was premature or for failure to exhaust administrative remedies.
x The students filed motion for reconsideration, but was denied. ISSUE # 1: Whether the requisite of “clear right” for the issuance of the writ was complied
with.
x They filed the petition for review with the Supreme Court.
ISSUE: Whether the students can avail of injunction. HELD # 1: NO.
x A preliminary injunction is an order granted at any stage of an action
HELD: NO. prior to judgment of final order, requiring a party, court, agency, or
x Discipline in education is specifically mandated by the 1987 Constitution which person to refrain from a particular act or acts. It is a preservative
provides that all educational institutions shall "teach the rights and duties of remedy to ensure the protection of a party’s substantive rights or
citizenship, strengthen ethical and spiritual values, develop moral character and interests pending the final judgment in the principal action. A plea for
personal discipline."24 Schools and school administrators have the authority to an injunctive writ lies upon the existence of a claimed emergency or
maintain school discipline25 and the right to impose appropriate and reasonable extraordinary situation which should be avoided for otherwise, the
disciplinary measures.26 On the other hand, students have the duty and the outcome of a litigation would be useless as far as the party applying for
responsibility to promote and maintain the peace and tranquility of the school by the writ is concerned.
observing the rules of discipline.27 x At times referred to as the "Strong Arm of Equity," we have consistently ruled
that there is no power the exercise of which is more delicate and which calls for
9
RECTO, GAYLE ANGELI M.
2011-0008 | AUSL
Personal Notes on Remedial Law 2 Review (based on the syllabus of Prof. Henedino M. Brondial)

greater circumspection than the issuance of an injunction. It should only be and disqualified it from availing of the equitable remedy of preliminary
extended in cases of great injury where courts of law cannot afford an adequate injunction.
or commensurate remedy in damages; "in cases of extreme urgency; where the
right is very clear; where considerations of relative inconvenience bear strongly ISSUE # 3: Whether MFII's interest is prejudiced by SBI's conduct and breach.
in complainant’s favor; where there is a willful and unlawful invasion of plaintiff’s
right against his protest and remonstrance, the injury being a continuing one, HELD # 3: YES.
and where the effect of the mandatory injunction is rather to reestablish and x As SBI is not entitled to the issuance of a writ of preliminary injunction, so is
maintain a preexisting continuing relation between the parties, recently and MFII. The accessory follows the principal. The accessory obligation of MFII as
arbitrarily interrupted by the defendant, than to establish a new relation." accommodation mortgagor and surety is tied to SBI’s principal obligation to CBC
x A writ of preliminary injunction is an extraordinary event which must be granted and arises only in the event of SBI’s default. Thus, MFII’s interest in the issuance
only in the face of actual and existing substantial rights. The duty of the court of the writ of preliminary injunction is necessarily prejudiced by SBI’s wrongful
taking cognizance of a prayer for a writ of preliminary injunction is to determine conduct and breach of contract.
whether the requisites necessary for the grant of an injunction are present in the x Thus, MFII’s interest in the issuance of the writ of preliminary injunction is
case before it.25 In this connection, a writ of preliminary injunction is issued to necessarily prejudiced by SBI’s wrongful conduct and breach of contract.
preserve the status quo ante, upon the applicant’s showing of two important x Even Article 1229 of the Civil Code, which SBI and MFII invoke, works against
requisite conditions, namely: (1) the right to be protected exists prima facie, and them. Under that provision, the equitable reduction of the penalty stipulated by
(2) the acts sought to be enjoined are violative of that right. It must be proven the parties in their contract will be based on a finding by the court that such
that the violation sought to be prevented would cause an irreparable injury.26 penalty is iniquitous or unconscionable. Here, the trial court has not yet made a
x Here, SBI and MFII basically claim a right to have their mortgaged properties ruling as to whether the penalty agreed upon by CBC with SBI and MFII is
shielded from foreclosure by CBC on the ground that the interest rate and unconscionable. Such finding will be made by the trial court only after it has
penalty charges imposed by CBC on the loans availed of by SBI are iniquitous heard both parties and weighed their respective evidence in light of all relevant
and unconscionable. In particular, SBI and MFII assert: circumstances. Hence, for SBI and MFII to claim any right or benefit under that
o There is therefore an urgent necessity for the issuance of a writ of provision at this point is premature.
preliminary injunction or at least a status quo [order], otherwise, x As no clear right that warrants the extraordinary protection of an injunctive writ
respondent bank will definitely foreclose petitioners’ properties has been shown by SBI and MFII to exist in their favor, the first requirement for
without awaiting the trial of the main case on the merits, with said the grant of a preliminary injunction has not been satisfied. In the absence of
usurious and confiscatory rates of interest as basis and any requisite, and where facts are shown to be wanting in bringing the matter
o There is therefore no legal justification for the Honorable Court of within the conditions for its issuance, the ancillary writ of injunction must be
Appeals to lift/dissolve the injunction issued by the trial court, struck down for having been rendered in grave abuse of discretion.43 Thus, the
otherwise, respondent bank - on the basis of this illegal imposition Court of Appeals did not err when it granted the petition for certiorari of CBC and
of interest - can already foreclose the properties of petitioners and ordered the dissolution of the writ of preliminary injunction issued by the trial
render the whole case (sans trial on the merits) moot and academic court.
x On this matter, the Order dated December 14, 2000 of the trial court enumerates
as the first argument raised by SBI and MFII in support of their application for ISSUE # 4: Whether there was showing of irreparable injury to merit issuance of the writ.
the issuance of a writ of preliminary injunction:
o 1. Their rights basically are for the protection of their properties put HELD # 4: NO.
up as collateral for the loans extended by defendant bank to them x Neither has there been a showing of irreparable injury. An injury is
x As debtor-mortgagors, however, SBI and MFII do not have a right to considered irreparable if it is of such constant and frequent recurrence
prevent the creditor-mortgagee CBC from foreclosing on the that no fair or reasonable redress can be had therefor in a court of law,
mortgaged properties simply on the basis of alleged "usurious, or where there is no standard by which their amount can be measured
exorbitant and confiscatory rate of interest." First, assuming that the with reasonable accuracy, that is, it is not susceptible of mathematical
interest rate agreed upon by the parties is usurious, the nullity of the computation. The provisional remedy of preliminary injunction may
stipulation of usurious interest does not affect the lender’s right to only be resorted to when there is a pressing necessity to avoid
recover the principal loan, nor affect the other terms thereof. Thus, in a injurious consequences which cannot be remedied under any standard
usurious loan with mortgage, the right to foreclose the mortgage of compensation.
subsists, and this right can be exercised by the creditor upon failure by x In the first place, any injury that SBI and MFII may suffer in case of foreclosure
the debtor to pay the debt due. of the mortgaged properties will be purely monetary and compensable by an
x Second, even the Order dated December 14, 2000 of the trial court, which appropriate judgment in a proper case against CBC. Moreover, where there is a
granted the application for the issuance of a writ of preliminary injunction, valid cause to foreclose on the mortgages, it cannot be correctly claimed that the
recognizes that the parties still have to be heard on the alleged lack of "fairness irreparable damage sought to be prevented by the application for preliminary
of the increase in interests and penalties" during the trial on the merits. Thus, injunction is the loss of the mortgaged properties to auction sale.45 The alleged
the basis of the right claimed by SBI and MFII remains to be controversial or entitlement of SBI and MFII to the "protection of their properties put up as
disputable as there is still a need to determine whether or not, upon collateral for the loans" they procured from CBC is not the kind of irreparable
consideration of the various circumstances surrounding the agreement of the injury contemplated by law. Foreclosure of mortgaged property is not an
parties, the interest rates and penalty charges are unconscionable. Therefore, irreparable damage that will merit for the debtor-mortgagor the extraordinary
such claimed right cannot be considered clear, actual and subsisting. In the provisional remedy of preliminary injunction. As this Court stated in Philippine
absence of a clear legal right, the issuance of the injunctive writ constitutes National Bank v. Castalloy Technology Corporation46:
grave abuse of discretion. o All is not lost for defaulting mortgagors whose properties were
x [The] finding of the trial court that the validity of the increase in the amount of foreclosed by creditors-mortgagees. The respondents will not be
the loan obligation is in issue simply amounted to a finding that the rights of SBI deprived outrightly of their property, given the right of redemption
and MFII vis-à-vis that of CBC are disputed and debatable. In such a case where granted to them under the law. Moreover, in extrajudicial
the complainant-movant’s right is doubtful or disputed, the issuance of an foreclosures, mortgagors have the right to receive any surplus in the
injunctive writ is not proper. selling price. Thus, if the mortgagee is retaining more of the
proceeds of the sale than he is entitled to, this fact alone will not
ISSUE #2: Whether SBI's default of payment negates its resort to motion for the issuance of affect the validity of the sale but will give the mortgagor a cause of
an injunctive writ. action to recover such surplus. (Citation omitted.)

HELD # 2: YES. ISSUE # 5: Whether the reinstatement of writ of preliminary injunction circumvents the
x Even assuming that SBI and MFII are correct in claiming their supposed right, it Court's En Banc Resolution in A.M. No. 99-10-05-0, Re: Procedure in Extrajudicial or Judicial
nonetheless disintegrates in the face of the ten promissory notes in the total Foreclosure of Real Estate Mortgages.
amount of P218,540,648.00, exclusive of interest and penalties, issued by SBI in
favor of CBC on March 1, 1999 which until now remain unpaid despite the HELD # 5 YES.
maturity of the said notes on March 1, 2004 and CBC’s repeated demands for x Issued on February 20, 2007, or some two months before the Court of Appeals
payment.37 Foreclosure is but a necessary consequence of nonpayment of promulgated its decision in this case, the resolution embodies the additional
mortgage indebtedness.38 As this Court held in Equitable PCI Bank, Inc. v. OJ- guidelines intended to aid courts in foreclosure proceedings, specifically limiting
Mark Trading, Inc.39: the instances, and citing the conditions, when a writ against foreclosure of a
o Where the parties stipulated in their credit agreements, mortgage mortgage may be issued, to wit:
contracts and promissory notes that the mortgagee is authorized to o (1) No temporary restraining order or writ of preliminary injunction
foreclose the mortgaged properties in case of default by the against the extrajudicial foreclosure of real estate mortgage shall be
mortgagors, the mortgagee has a clear right to foreclosure in case issued on the allegation that the loan secured by the mortgage has
of default, making the issuance of a Writ of Preliminary Injunction been paid or is not delinquent unless the application is verified and
improper. x x x. supported by evidence of payment.
x In addition, the default of SBI and MFII to pay the mortgage o (2) No temporary restraining order or writ of preliminary injunction
indebtedness disqualifies them from availing of the equitable relief against the extrajudicial foreclosure of real estate mortgage shall
that is the injunctive writ. A debtor’s various and constant requests for be issued on the allegation that the interest on the loan is
deferment of payment and restructuring of loan, without actually unconscionable, unless the debtor pays the mortgagee at least
paying the amount due, are clear indications that said debtor was twelve percent per annum interest on the principal obligation as
unable to settle his obligation. SBI’s default or failure to settle its stated in the application for foreclosure sale, which shall be
obligation is a breach of contractual obligation which tainted its hands updated monthly while the case is pending.

10
RECTO, GAYLE ANGELI M.
2011-0008 | AUSL
Personal Notes on Remedial Law 2 Review (based on the syllabus of Prof. Henedino M. Brondial)

o (3) Where a writ of preliminary injunction has been issued against


a foreclosure of mortgage, the disposition of the case shall be 1. When writ may issue
speedily resolved. To this end, the court concerned shall submit to
the Supreme Court, through the Office of the Court Administrator,
Section 1. Appointment of receiver. — Upon a verified application, one or more
quarterly reports on the progress of the cases involving ten million
receivers of the property subject of the action or proceeding may be appointed by the court
pesos and above.
where the action is pending or by the Court of Appeals or by the Supreme Court, or a member
o (4) All requirements and restrictions prescribed for the issuance of
thereof, in the following cases:
a temporary restraining order/writ of preliminary injunction, such
as the posting of a bond, which shall be equal to the amount of the
(a) When it appears from the verified application, and such other proof as the court
outstanding debt, and the time limitation for its effectivity, shall
may require, that the party applying for the appointment of a receiver has an interest in the
apply as well to a status quo order.
property or fund which is the subject of the action or proceeding, and that such property or
x The guidelines speak of strict exceptions and conditions. To reverse the decision
fund is in danger of being lost, removed, or materially injured unless a receiver be appointed to
of the Court of Appeals and reinstate the writ of preliminary injunction issued by
administer and preserve it;
the trial court will be to allow SBI and MFII to circumvent the guidelines and
conditions provided by the En Banc Resolution in A.M. No. 99-10-05-0 dated
(b) When it appears in an action by the mortgagee for the foreclosure of a mortgage
February 20, 2007 and prevent CBC from foreclosing on the mortgaged
that the property is in danger of being wasted or dissipated or materially injured, and that its
properties based simply on the allegation that the interest on the loan is
value is probably insufficient to discharge the mortgage debt, or that the parties have so
unconscionable. This Court will not permit such a situation. What cannot be done
stipulated in the contract of mortgage;
directly cannot be done indirectly.
(c) After judgment, to preserve the property during the pendency of an appeal, or to
dispose of it according to the judgment, or to aid execution when the execution has been
returned unsatisfied or the judgment obligor refuses to apply his property in satisfaction of the
judgment, or otherwise to carry the judgment into effect;

(d) Whenever in other cases it appears that the appointment of a receiver is the
SPOUSES SILVESTRE O. PLAZA AND ELENA Y. PLAZA, Petitioners, vs. GUILLERMO
most convenient and feasible means of preserving, administering, or disposing of the property
LUSTIVA, ELEODORA VDA. DE MARTINEZ AND VICKY SAYSON GOLOSENO, Respondents.
in litigation.
GR 172909, 5 March 2014, Second Division,
Brion [J]
During the pendency of an appeal, the appellate court may allow an application for the
appointment of a receiver to be filed in and decided by the court of origin and the receiver
FACTS:
appointed to be subject to the control of said court. (1a)
x On 28 August 1997, the CA ruled that among the Plaza siblings (Aureliano,
Emiliana, Vidal, Marciano, and Barbara), Barbara was the owner of the subject
agricultural land. 2. Requirements
x The decision became final and executory and Barbara's successors, Guillermo
Lustiva, Eleodora Vda. de Martinez and Vicky Sayson Goloseno, have continued Section 2. Bond on appointment of receiver. — Before issuing the order appointing a
occupying the property. receiver the court shall require the applicant to file a bond executed to the party against whom
x On 14 September 1999, Vidal’s son and daughter-in-law, the spouses Plaza, filed the application is presented, in an amount to be fixed by the court, to the effect that the
a Complaint for Injunction, Damages, Attorney’s Fees with Prayer for the applicant will pay such party all damages he may sustain by reason of the appointment of such
Issuance of the Writ of Preliminary Injunction and/or Temporary Restraining receiver in case the applicant shall have procured such appointment without sufficient cause;
Order against Lustiva, et al. and the City Government of Butuan. and the court may, in its discretion, at any time after the appointment, require an additional
x They prayed that Lustiva, et al. be enjoined from unlawfully and illegally bond as further security for such damages.
threatening to take possession of the subject property.
x In its 14 December 1999 order, the Regional Trial Court (RTC) of Butuan City, 3. Power of Receiver
Branch 5, reconsidered its earlier order, denied the prayer for a Writ of
Preliminary Injunction, and ordered that the possession and occupation of the
Section 6. General powers of receiver. — Subject to the control of the court in which
land be returned to Lustiva, et al.
the action or proceeding is pending a receiver shall have the power to bring and defend, in
x Through a petition for review on certiorari under Rule 65, the petitioners
such capacity, actions in his own name; to take and keep possession of the property in
challenged the RTC’s order before the CA.
controversy; to receive rents; to collect debts due to himself as receiver or to the fund,
x While the petition for review on certiorari was pending before the CA, the
property, estate, person, or corporation of which he is the receiver; to compound for and
spouses Plaza filed an action for specific performance against the City
compromise the same; to make transfers; to pay outstanding debts; to divide the money and
Government of Butuan, claiming that the latter should issue them a certificate of
other property that shall remain among the persons legally entitled to receive the same; and
sale.
generally to do such acts respecting the property as the court may authorize. However, funds in
x In its 24 October 2005 decision, the CA affirmed the RTC’s ruling, found the
the hands of a receiver may be invested only by order of the court upon the written consent of
petitioners guilty of forum shopping, dismissed the case, and referred the case to
all the parties to the action. (7a)
the Court and to the Integrated Bar of the Philippines for investigation and
institution of the appropriate administrative action.
No action may be filed by or against a receiver without leave of the court which appointed
x On 6 April 2006, the CA rejected the spouses Plaza's motion for reconsideration.
him.
x Hence, the petition for review on certiorari.
ISSUE # 1: Whether spouses Plaza are entitled to the writ. 4. Termination and Compensation

HELD # 1: NO. Section 8. Termination of receivership; compensation of receiver. — Whenever the


x As the lower courts correctly found, Tuazon had no ownership to confer to the court, motu proprio or on motion of either party, shall determine that the necessity for a
petitioners despite the latter’s reimbursement of Tuazon’s purchase expenses. receiver no longer exists, it shall, after due notice to all interested parties and hearing, settle
Because they were never owners of the property, the petitioners failed to the accounts of the receiver, direct the delivery of the funds and other property in his
establish entitlement to the writ of preliminary injunction. "[T]o be entitled to an possession to the person adjudged to be entitled to receive them and order the discharge of
injunctive writ, the right to be protected and the violation against that right must the receiver from further duty as such. The court shall allow the receiver such reasonable
be shown. A writ of preliminary injunction may be issued only upon clear compensation as the circumstances of the case warrant, to be taxed as costs against the
showing of an actual existing right to be protected during the pendency of the defeated party, or apportioned, as justice requires.
principal action. When the complainant’s right or title is doubtful or disputed, he
does not have a clear legal right and, therefore, the issuance of injunctive relief SPS. CESAR A. LARROBIS, JR. and VIRGINIA S. LARROBIS, petitioners, vs.
is not proper."23 PHILIPPINE VETERANS BANK, respondent.
GR 135706, 1 October 2004
ISSUE # 2: Whether the question of issuance of the writ has become moot and academic. Second Division
Austria-Martinez [J]
HELD # 2: YES.
x Likewise, upon the dismissal of the main case by the RTC on August 8, 2013, the FACTS:
question of issuance of the writ of preliminary injunction has become moot and x On 3 March 1980, spouses Larrobis contracted a monetary loan with Philippine
academic. In Arevalo v. Planters Development Bank,24 the Court ruled that a Veterans Bank in the amount of P135,000.00, evidenced by a promissory note,
case becomes moot and academic when there is no more issue between the due and demandable on 27 February 1981, and secured by a Real Estate
parties or object that can be served in deciding the merits of the case. Upon the Mortgage executed on their lot together with the improvements thereon.
dismissal of the main action, the question of the non-issuance of a writ of x On 23 March 1985, the bank went bankrupt and was placed under
preliminary injunction automatically died with it. A writ of preliminary injunction receivership/liquidation by the Central Bank from 25 April 1985 until August
is a provisional remedy; it is auxiliary, an adjunct of, and subject to the 1992.
determination of the main action. It is deemed lifted upon the dismissal of the x On 23 August 1985, the bank, through Francisco Go, sent the spouses a demand
main case, any appeal therefrom notwithstanding. letter for "accounts receivable in the total amount of P6,345.00 as of 15 August
1984," which pertains to the insurance premiums advanced by the bank over the
C. Receivership (Rule 59) mortgaged property of the spouses.

11
RECTO, GAYLE ANGELI M.
2011-0008 | AUSL
Personal Notes on Remedial Law 2 Review (based on the syllabus of Prof. Henedino M. Brondial)

x On 23 August 1995, more than fourteen years from the time the loan became x Further examination of the Central Bank case reveals that the circumstances of
due and demandable, the bank filed a petition for extrajudicial foreclosure of Provident Savings Bank at the time were peculiar because after the Monetary
mortgage of the spouses'’ property. Board issued MB Resolution No. 1766 on September 15, 1972, prohibiting it from
x On 18 October 1995, the property was sold in a public auction with Philippine doing business in the Philippines, the bank’s majority stockholders immediately
Veterans Bank as the lone bidder. went to the Court of First Instance of Manila, which prompted the trial court to
x On 26 April 1996, the spouses filed a complaint with the RTC, Cebu City, to issue its judgment dated February 20, 1974, declaring null and void the
declare the extra-judicial foreclosure and the subsequent sale thereof to the resolution and ordering the Central Bank to desist from liquidating Provident. The
bank null and void. decision was appealed to and affirmed by this Court in 1981. Thus, the
x On 17 April 1998, the RTC rendered its decision dismissing the complaint. Superintendent of Banks, which was instructed to take charge of the assets of
x The spouses filed a motion for reconsideration which the RTC denied on 25 the bank in the name of the Monetary Board, had no power to act as a receiver
August 1998. of the bank and carry out the obligations specified in Sec. 29 of the Central Bank
x Hence, the petition for review, raising pure questions of law. Act.32
x In this case, it is not disputed that Philippine Veterans Bank was placed under
ISSUE: Whether the bank, through a receiver, can foreclose properties during the period of receivership by the Monetary Board of the Central Bank by virtue of Resolution
receivership. No. 364 on April 25, 1985, pursuant to Section 29 of the Central Bank Act on
insolvency of banks.33
HELD # 1: YES. x Unlike Provident Savings Bank, there was no legal prohibition imposed upon
x Respondent’s claims that because of a fortuitous event, it was not able to herein respondent to deter its receiver and liquidator from performing their
exercise its right to foreclose the mortgage on petitioners’ property; and that obligations under the law. Thus, the ruling laid down in the Provident case
since it was banned from pursuing its business and was placed under cannot apply in the case at bar.
receivership from April 25, 1985 until August 1992, it could not foreclose the x There is also no truth to respondent’s claim that it could not continue doing
mortgage on petitioners’ property within such period since foreclosure is business from the period of April 1985 to August 1992, the time it was under
embraced in the phrase "doing business," are without merit. receivership. As correctly pointed out by petitioner, respondent was even able to
x While it is true that foreclosure falls within the broad definition of send petitioners a demand letter, through Francisco Go, on August 23, 1985 for
"doing business," it should not be considered included, however, in the "accounts receivable in the total amount of P6,345.00 as of August 15, 1984" for
acts prohibited whenever banks are "prohibited from doing business" the insurance premiums advanced by respondent bank over the mortgaged
during receivership and liquidation proceedings. property of petitioners. How it could send a demand letter on unpaid insurance
x This we made clear in Banco Filipino Savings & Mortgage Bank vs. premiums and not foreclose the mortgage during the time it was "prohibited
Monetary Board, Central Bank of the Philippines where we explained from doing business" was not adequately explained by respondent.
that x Settled is the principle that a bank is bound by the acts, or failure to act of its
o “Section 29 of the Republic Act No. 265, as amended receiver.34 As we held in Philippine Veterans Bank vs. NLRC,35 a labor case
known as the Central Bank Act, provides that when a bank which also involved respondent bank,
is forbidden to do business in the Philippines and placed o … all the acts of the receiver and liquidator pertain to petitioner,
under receivership, the person designated as receiver shall both having assumed petitioner’s corporate existence. Petitioner
immediately take charge of the bank’s assets and cannot disclaim liability by arguing that the non-payment of
liabilities, as expeditiously as possible, collect and gather MOLINA’s just wages was committed by the liquidators during the
all the assets and administer the same for the benefit of its liquidation period.36
creditors, and represent the bank personally or through x However, the bank may go after the receiver who is liable to it for any culpable
counsel as he may retain in all actions or proceedings for or or negligent failure to collect the assets of such bank and to safeguard its assets.
against the institution, exercising all the powers necessary
for these purposes including, but not limited to, bringing ANA MARIA A. KORUGA, Petitioner, vs. TEODORO O. ARCENAS, JR., ALBERT C.
and foreclosing mortgages in the name of the bank.” AGUIRRE, CESAR S. PAGUIO, FRANCISCO A. RIVERA, and THE HONORABLE COURT OF
x This is consistent with the purpose of receivership proceedings, i.e., to receive APPEALS, THIRD DIVISION, Respondents.
collectibles and preserve the assets of the bank in substitution of its former G.R. No. 168332 June 19, 2009
management, and prevent the dissipation of its assets to the detriment of the TEODORO O. ARCENAS, JR., ALBERT C. AGUIRRE, CESAR S. PAGUIO, and FRANCISCO A.
creditors of the bank. RIVERA, Petitioners, vs. HON. SIXTO MARELLA, JR., Presiding Judge, Branch 138, Regional
x When a bank is declared insolvent and placed under receivership, the Central Trial Court of Makati City, and ANA MARIA A. KORUGA, Respondents.
Bank, through the Monetary Board, determines whether to proceed with the G.R. No. 169053 June 19, 2009
liquidation or reorganization of the financially distressed bank. A receiver, who THIRD DIVISION
concurrently represents the bank, then takes control and possession of its assets NACHURA, J.:
for the benefit of the bank’s creditors. A liquidator meanwhile assumes the role
of the receiver upon the determination by the Monetary Board that the bank can
no longer resume business. His task is to dispose of all the assets of the bank FACTS:
and effect partial payments of the bank’s obligations in accordance with legal x Koruga is a minority stockholder of Banco Filipino Savings and Mortgage Bank.
priority. In both receivership and liquidation proceedings, the bank retains its x On August 20, 2003, she filed a complaint before the Makati RTC, alleging
juridical personality notwithstanding the closure of its business and may even be Violation of Sections 31 to 34 of the Corporation Code ("Code") which prohibit
sued as its corporate existence is assumed by the receiver or liquidator. The self-dealing and conflicts of interest of directors and officers, Right of a
receiver or liquidator meanwhile acts not only for the benefit of the bank, but for stockholder to inspect the records of a corporation (including financial
its creditors as well.27 statements) under Sections 74 and 75 of the Code, Receivership and Creation of a
x In Provident Savings Bank vs. Court of Appeals,28 we further stated that: Management Committee pursuant to: (a) Rule 59 of the 1997 Rules of Civil
o When a bank is prohibited from continuing to do business by the Procedure ("Rules");(b) Section 5.2 of R.A. No. 8799; (c) Rule 1, Section 1(a)(1)
Central Bank and a receiver is appointed for such bank, that bank of the Interim Rules; (d) Rule 1, Section 1(a)(2) of the Interim Rules; (e) Rule 7
would not be able to do new business, i.e., to grant new loans or to of the Interim Rules; (f) Rule 9 of the Interim Rules; and (g) The General
accept new deposits. However, the receiver of the bank is in fact Banking Law of 2000 and the New Central Bank Act.
obliged to collect debts owing to the bank, which debts form part x On 12 September 2003, Arcenas, et al. filed their Answer raising, among others,
of the assets of the bank. The receiver must assemble the assets the trial court’s lack of jurisdiction to take cognizance of the case.
and pay the obligation of the bank under receivership, and take o They also filed a Manifestation and Motion seeking the dismissal of
steps to prevent dissipation of such assets. Accordingly, the the case.
receiver of the bank is obliged to collect pre-existing debts due to x In an Order dated 18 October 2004, the trial court denied the Manifestation and
the bank, and in connection therewith, to foreclose mortgages Motion. Arcenas, et al. moved for reconsideration but, on 18 January 2005, the
securing such debts.29 (Emphasis supplied.) RTC denied the motion.
x It is true that we also held in said case that the period during which the bank x This prompted Arcenas, et al. to file before the CA a Petition for Certiorari and
was placed under receivership was deemed fuerza mayor which validly Prohibition under Rule 65 of the Rules of Court with a prayer for the issuance of a
interrupted the prescriptive period.30 This is being invoked by the respondent writ of preliminary injunction and a temporary retraining order (TRO).
and was used as basis by the trial court in its decision. Contrary to the position x On 9 February 2005, the CA issued a 60-day TRO enjoining Judge Marella from
of the respondent and court a quo however, such ruling does not find application conducting further proceedings in the case.
in the case at bar. x On 22 February 2005, the RTC issued a Notice of Pre-trial9 setting the case for
x A close scrutiny of the Provident case, shows that the Court arrived at said pre-trial on June 2 and 9, 2005. Arcenas, et al. filed a Manifestation and Motion
conclusion, which is an exception to the general rule, due to the peculiar before the CA, reiterating their application for a writ of preliminary injunction.
circumstances of Provident Savings Bank at the time. In said case, we stated x Thus, on 18 April 2005, the CA issued the assailed Resolution, granting the
that: preliminary injunction.
o Having arrived at the conclusion that a foreclosure is part of a x Hence, the petition for certiorari under Rule 65.
bank’s business activity which could not have been pursued by the
receiver then because of the circumstances discussed in the Central ISSUE: Whether the RTC has jurisdiction to hear and decide a suit that seeks to place Banco
Bank case, we are thus convinced that the prescriptive period was Filipino under receivership.
legally interrupted by fuerza mayor in 1972 on account of the
prohibition imposed by the Monetary Board against petitioner from HELD: NO.
transacting business, until the directive of the Board was nullified in x It is not the Interim Rules of Procedure on Intra-Corporate Controversies, or Rule
1981.31 (Emphasis supplied.) 59 of the Rules of Civil Procedure on Receivership, that would apply to this case.
Instead, Sections 29 and 30 of the New Central Bank Act should be followed.
12
RECTO, GAYLE ANGELI M.
2011-0008 | AUSL
Personal Notes on Remedial Law 2 Review (based on the syllabus of Prof. Henedino M. Brondial)

o Section 29. Appointment of Conservator. - Whenever, on the basis with her family, planting coconut seedlings on the land and supervising the
of a report submitted by the appropriate supervising or examining harvest of coconut and palay.
department, the Monetary Board finds that a bank or a quasi-bank x Fidela and Evelina agreed to divide the gross sales of all products from the land
is in a state of continuing inability or unwillingness to maintain a between themselves.
condition of liquidity deemed adequate to protect the interest of x Since Fidela was busy with her law practice, Evelina undertook to hold in trust
depositors and creditors, the Monetary Board may appoint a for Fidela her half of the profits. But Fidela claimed that Evelina had failed to
conservator with such powers as the Monetary Board shall deem remit her share of the profits and, despite demand to turn over the
necessary to take charge of the assets, liabilities, and the administration of the property to Fidela, had refused to do so.
management thereof, reorganize the management, collect all x Consequently, Fidela filed a complaint against Evelina and her daughter, Aida C.
monies and debts due said institution, and exercise all powers Deles, who was assisting her mother, for recovery of possession, rent, and
necessary to restore its viability. The conservator shall report and damages with prayer for the immediate appointment of a receiver before the
be responsible to the Monetary Board and shall have the power to Regional Trial Court (RTC) of Bulan, Sorsogon.
overrule or revoke the actions of the previous management and x In their answer, Evelina and Aida claimed that the RTC did not have jurisdiction
board of directors of the bank or quasi-bank. over the subject matter of the case since it actually involved an agrarian dispute.
ƒ The Monetary Board shall terminate the x After hearing, the RTC dismissed the complaint for lack of jurisdiction.
conservatorship when it is satisfied that the institution Dissatisfied, Fidela appealed to the CA.
can continue to operate on its own and the x She also filed with that court a motion for the appointment of a receiver.
conservatorship is no longer necessary. The x On April 12, 2006 the CA granted the motion and ordained receivership of the
conservatorship shall likewise be terminated should land, noting that there appeared to be a need to preserve the property and its
the Monetary Board, on the basis of the report of the fruits in light of Fidela’s allegation that Evelina and Aida failed to account for her
conservator or of its own findings, determine that the share of such fruits.
continuance in business of the institution would
involve probable loss to its depositors or creditors, in ISSUE # 1: Whether receivership is the proper remedy for the landowner's deprivation of her
which case the provisions of Section 30 shall apply. share of the land's produce.
o Section 30. Proceedings in Receivership and Liquidation. -
Whenever, upon report of the head of the supervising or examining HELD # 1: NO.
department, the Monetary Board finds that a bank or quasi-bank: x [A] petition for receivership under Section 1(b), Rule 59 of the Rules of Civil
ƒ (a) is unable to pay its liabilities as they become due Procedure requires that the property or fund subject of the action is in danger of
in the ordinary course of business: Provided, That this being lost, removed, or materially injured, necessitating its protection or
shall not include inability to pay caused by preservation. Its object is the prevention of imminent danger to the property. If
extraordinary demands induced by financial panic in the action does not require such protection or preservation, the remedy is not
the banking community; receivership. Here Fidela’s main gripe is that Evelina and Aida deprived her of her
ƒ (b) has insufficient realizable assets, as determined by share of the land’s produce. She does not claim that the land or its productive
the Bangko Sentral, to meet its liabilities; or capacity would disappear or be wasted if not entrusted to a receiver. Nor does
ƒ (c) cannot continue in business without involving Fidela claim that the land has been materially injured, necessitating its protection
probable losses to its depositors or creditors; or and preservation. Because receivership is a harsh remedy that can be granted
ƒ (d) has willfully violated a cease and desist order only in extreme situations, Fidela must prove a clear right to its issuance. But she
under Section 37 that has become final, involving acts has not. Indeed, in none of the other cases she filed against Evelina and Aida
or transactions which amount to fraud or a dissipation has that remedy been granted her.
of the assets of the institution; in which cases, the
Monetary Board may summarily and without need for ISSUE # 2: Whether the Court of Appeals should have been more retrospect in ordaining
prior hearing forbid the institution from doing receivership, considering that RTC has no jurisdiction over the case.
business in the Philippines and designate the
Philippine Deposit Insurance Corporation as receiver HELD # 2: YES.
of the banking institution. x Besides, the RTC dismissed Fidela’s action for lack of jurisdiction over the case,
o The actions of the Monetary Board taken under this section or holding that the issues it raised properly belong to the DARAB. The case before
under Section 29 of this Act shall be final and executory, and may the CA is but an offshoot of that RTC case. Given that the RTC has found that it
not be restrained or set aside by the court except on petition for had no jurisdiction over the case, it would seem more prudent for the CA to first
certiorari on the ground that the action taken was in excess of provisionally determine that the RTC had jurisdiction before granting receivership
jurisdiction or with such grave abuse of discretion as to amount to which is but an incident of the main action.
lack or excess of jurisdiction. The petition for certiorari may only be
filed by the stockholders of record representing the majority of the D. Replevin (Rule 60)
capital stock within ten (10) days from receipt by the board of
directors of the institution of the order directing receivership,
liquidation or conservatorship. 1. When writ may issue
o The designation of a conservator under Section 29 of this Act or
the appointment of a receiver under this section shall be vested Section 1. Application. — A party praying for the recovery of possession of personal
exclusively with the Monetary Board. Furthermore, the designation property may, at the commencement of the action or at any time before answer, apply for an
of a conservator is not a precondition to the designation of a order for the delivery of such property to him, in the manner hereinafter provided.
receiver
x On the strength of these provisions, it is the Monetary Board that 2. Requirements
exercises exclusive jurisdiction over proceedings for receivership of
banks.
Section 2. Affidavit and bond. — The applicant must show by his own affidavit or that of
x Crystal clear in Section 30 is the provision that says the "appointment some other person who personally knows the facts:
of a receiver under this section shall be vested exclusively with the
Monetary Board." The term "exclusively" connotes that only the (a) That the applicant is the owner of the property claimed, particularly describing it,
Monetary Board can resolve the issue of whether a bank is to be placed or is entitled to the possession thereof;
under receivership and, upon an affirmative finding, it also has
authority to appoint a receiver. This is further affirmed by the fact that (b) That the property is wrongfully detained by the adverse party, alleging the cause
the law allows the Monetary Board to take action "summarily and of detention thereof according to the best of his knowledge, information, and belief ;
without need for prior hearing." And, as a clincher, the law explicitly
provides that "actions of the Monetary Board taken under this section (c) That the property has not been distrained or taken for a tax assessment or a fine
or under Section 29 of this Act shall be final and executory, and may
pursuant to law, or seized under a writ of execution or preliminary attachment, or otherwise
not be restrained or set aside by the court except on a petition for
placed under custodia legis, or if so seized, that it is exempt from such seizure or custody; and
certiorari on the ground that the action taken was in excess of
jurisdiction or with such grave abuse of discretion as to amount to lack
(d) The actual market value of the property.
or excess of jurisdiction." [T]here is no doubt that the RTC has no
jurisdiction to hear and decide a suit that seeks to place Banco Filipino
under receivership. 3. Third Party Claim

EVELINA G. CHAVEZ and AIDA CHAVEZ-DELES, Petitioners, vs. COURT OF APPEALS Section 7. Proceedings where property claimed by third person. — If the property
and ATTY. FIDELA Y. VARGAS, Respondents. taken is claimed by any person other than the party against whom the writ of replevin had
G.R. No. 174356 January 20, 2010 been issued or his agent, and such person makes an affidavit of his title thereto, or right to
SECOND DIVISION the possession thereof, stating the grounds therefor, and serves such affidavit upon the sheriff
ABAD, J.: while the latter has possession of the property and a copy thereof upon the applicant, the
sheriff shall not be bound to keep the property under replevin or deliver it to the applicant
unless the applicant or his agent, on demand of said sheriff, shall file a bond approved by the
FACTS: court to indemnify the third-party claimant in a sum not less than the value of the property
x Fidela Y. Vargas owned a five-hectare mixed coconut land and rice fields in under replevin as provided in section 2 hereof. In case of disagreement as to such value, the
Sorsogon. Evelina G. Chavez had been staying in a remote portion of the land court shall determine the same. No claim for damages for the taking or keeping, of the
13
RECTO, GAYLE ANGELI M.
2011-0008 | AUSL
Personal Notes on Remedial Law 2 Review (based on the syllabus of Prof. Henedino M. Brondial)

property may be enforced against the bond unless the action therefor is filed within one same. Petitioner alleges that the present appeal involves the same causes of
hundred twenty (120) days from the date of the filing of the bond. action, same parties, same facts and same relief involved in the decision
rendered by the First Division and affirmed by this Court in G.R. No. 84979. 11
The sheriff shall not be liable for damages, for the taking or keeping of such property, to any x Petitioner's argument is untenable. Jurisdiction is simply the power or authority
such third-party claimant if such bond shall be filed. Nothing herein contained shall prevent to hear a case. The appellate jurisdiction of the Court of Appeals to review
such claimant or any third person from vindicating his claim to the property, or prevent the decisions and orders of lower courts is conferred by Batas Pambansa Blg. 129.
applicant from claiming damages against a third-party claimant who filed a frivolous or plainly More importantly, petitioner cannot now assail the Court of Appeals' jurisdiction
spurious claim, in the same or a separate action. after having actively participated in the appeal and after praying for affirmative
relief.
When the writ of replevin is issued in favor of the Republic of the Philippines, or any officer
duly representing it, the filing of such bond shall not be required, and in case the sheriff is ISSUE # 2: Whether res judicata may be invoked.
sued for damages as a result of the replevin, he shall be represented by the Solicitor General,
and if held liable therefor, the actual damages adjudged by the court shall be paid by the HELD # 2: NO.
National Treasurer out of the funds to be appropriated for the purpose. (7a) x he petition docketed as CA-G.R. SP No. 14938 was for certiorari with injunction,
brought by Stronghold Insurance Company, Inc. alleging that there was grave
4. Judgment and Damages abuse of discretion when the trial court adjudged it liable for damages without
due process, in violation of Rule 60, Section 10 in relation to Rule 57, Section 20,
of the Rules of Court. The surety also questioned the propriety of the writ of
Section 9. Judgment. — After trial of the issues the court shall determine who has the
execution issued by the trial court pending appeal. 13
right of possession to and the value of the property and shall render judgment in the
x On the other hand, CA-G.R. CV No. 25929 was filed by petitioner Orosa under
alternative for the delivery thereof to the party entitled to the same, or for its value in case
Rule 45 of the Revised Rules of Court raising alleged errors of law on the part of
delivery cannot be made, and also for such damages as either party may prove, with costs.
the trial court. The subject of the appeal was the main decision, while the
subject of the petition in CA-G.R. SP No. 14938 was the Supplemental Decision.
Section 10. Judgment to include recovery against sureties. — The amount, if any, to
x We agree with the Court of Appeals that: 14
be awarded to any party upon any bond filed in accordance with the provisions of this Rule,
o The decisions of the Court of Appeals in CA-G.R. SP No. 14938 and
shall be claimed, ascertained, and granted under the same procedure as prescribed in section
the Supreme Court in G.R. No. 84979 did not pass on the merits of
20 of Rule 57.
this case. It merely ruled on the issues of whether the surety,
Stronghold Insurance, Co., Inc., can be held jointly and solidarily
liable with plaintiff-appellant and whether execution pending appeal
is proper under the facts and circumstances of this case.
Consequently, this Court is not estopped from reviewing the
conclusions reached by the court a quo.

ISSUE # 3: Whether the CA correctly took cognizance of issues raised for the first time on
JOSE S. OROSA and MARTHA P. OROSA, petitioners, vs. HON. COURT OF APPEALS and
appeal.
FCP CREDIT CORPORATION, respondents.
G.R. No. 111080 April 5, 2000
HELD # 3: YES.
FIRST DIVISION
x Private respondent belatedly pointed out that: 16
YNARES_SANTIAGO, J.:
o 1.1. It is pertinent to note that Defendant-Appellee has waived
prior notice and demand in order to be rendered in default, as in
FACTS:
fact the Promissory Note expressly stipulates that the monthly
x Orosa Æ bought Ford Laser 1.5 Sedan from Fiesta Motor Sales Corporation on
installments shall be paid on the date they fall due, without need of
installment basis
prior notice or demand.
x A PN and a chattel mortgage was executed covering the subject vehicle
o 1.2. Said Promissory Note likewise expressly stipulates that a late
x Fiesta Motor Sales assigned the promissory note and chattel mortgage to private
payment charge of 2% per month shall be added on each unpaid
respondent FCP Credit Corporation
installment from maturity thereof until fully paid.
x Orosa Æ Failed to pay the installments which fell due on July 28, 1984 as well as
o 1.3. Of equal significance is the Acceleration Clause in the
three (3) consecutive installment which fell due on August 28, September 28,
Promissory Note which states that if default be made in the
and October 28, 1984
payment of any of the installments or late payment charges
x FCP Credit Æ demanded payment
thereon when the same became due and payable, the total
x Orosa Æ failed to pay
principle sum then remaining unpaid, together with the agreed late
x FCP Æ filed a complaint with the RTC Manila for replevin and damages
payment charges thereon, shall at once become due and payable.
x Orosa Æ filed a counterclaim
x Private respondent argued that based on the provisions of the Promissory Note
x RTC Æ dismissed the complaint; granted the counterclaim
itself, petitioner incurred in default since, even though there was actual
o The trial court ruled that private respondent FCP had no reason to
payment of the installments which fell due on July 28, 1984, as well as the three
file the present action since petitioner already paid the installments
installments on August 28 to October 28, 1984, the payments were all late and
for the months of July to November 1984, which are the sole bases
irregular. 17 Private respondent also argued that petitioner assigned the subject
of the complaint. The lower court declared that private respondent
car to his daughter without the written consent of the obligee, and hence, violated
was not entitled to the writ of replevin, and was liable to petitioner
the terms of the chattel mortgage. 18 Meritorious as these arguments are, they
for actual damages under the replevin bond it filed. 3
come too late in the day. Basic is the rule that matters not raised in the complaint
o Ruling on petitioner's counterclaim, the trial court stated that there
cannot be raised for the first time on appeal.
was no legal or factual basis for the writ of replevin and that its
x Contrary to petitioner's accusation, the Court of Appeals restricted the
enforcement by the sheriff was "highly irregular, and unlawful,
determination of the case to matters alleged in the complaint and raised during
done, as it was, under shades of extortion, threats and force." 4
trial. 19 Citing jurisprudence, 20 the Court of Appeals held that "it would be
The trial court ordered private respondent to pay the sum of
offensive to the basic rule of fair play, justice and due process" if it considered
P400,000.00 as moral damages; P100,000.00 as exemplary
issues raised for the first time on appeal. 21
damages and P50,000.00 as attorney's fees. Private respondent
x The Court of Appeals' statement that "under the terms and conditions of the
was also ordered to return to petitioner the 1983 Ford Laser 1.5
chattel mortgage, defendant-appellee Jose S. Orosa was already in default," was
Sedan, or its equivalent, in kind or value in cash, as of date of
made only to justify the deletion of the trial court's award of moral, exemplary
judgment and to pay the costs of the suit
damages and attorney's fees, in consonance with its finding that private
x RTC Æ issued a supplemental order
respondent was motivated by a sincere belief that it had sufficient basis and
o ordering private respondent's surety, Stronghold Insurance Co.,
acted in good faith when it filed the claim.
Inc. to jointly and severally [with private respondent] return to
petitioner the 1983 Ford Laser 1.5 Sedan or its, equivalent in kind
ISSUE # 4: Whether the Orosas were entitled to their counterclaim.
or in cash and to pay the damages specified in the main decision to
the extent of the value of the replevin bond in the amount of
HELD # 4: NO.
P210,000.00.
x We now come to the matter of moral damages. Petitioner insists that he
x Stronghold Æ filed a Rule 65 before the CA
suffered untold embarrassment when the complaint was filed against him.
o CA Æ dismissed
According to petitioner, the car subject of this case was being used by his
o SC Æ affirmed CA but deleted the order for the issuance of a writ
daughter, married to Jose Concepcion III, a scion of a prominent family.
of execution pending appeal.
Petitioner laments that he assigned the car to his daughter so that she could
x FCP Æ appealed to CA
"approximate without equaling the status of her in-laws." This being the case,
o CA Æ affirmed RTC ruling
petitioner experienced anguish and unquantifiable humiliation when he had to
face his daughter's wealthy in-laws to explain the "why and the whats of the
ISSUE # 1: Whether CA Eighth Division has jurisdiction to hear the appeal subject case." Petitioner further insists that an award of moral damages is
especially justified since he is no ordinary man, but a businessman of high social
HELD # 1: YES. standing, a graduate of De La Salle University and belongs to a well known
x In its first assignment of error, petitioner alleges that the Eighth Division of the
family of bankers. 23
Court of Appeals had no jurisdiction to review the present case since the First
x We must deny the claim. The law clearly states that one may only recover moral
Division of the Court of Appeals already passed upon the law and the facts of the
damages if they are the proximate result of the, other party's wrongful act or
14
RECTO, GAYLE ANGELI M.
2011-0008 | AUSL
Personal Notes on Remedial Law 2 Review (based on the syllabus of Prof. Henedino M. Brondial)

omission. 24 Two elements are required. First, the act or omission must be the o Astora Æ filed an MD on grounds of (i) lack of jurisdiction; (ii)
proximate result of the physical suffering, mental anguish, fright, serious failure to state a cause of action; (iii) litis pendentia; and (iv)
anxiety, besmirched reputation, wounded feelings, moral shock, social forum-shopping
humiliation and similar injury. Second, the act must be wrongful. ƒ That RTC had no jurisdiction because the benefit
x Petitioner maintains that embarrassment resulted when he had to explain the arose of an employment contract
suit to his daughter's in-laws. However, that could have been avoided had he x LA (pending resolution of the MD in the replevin case) Æ ruled in favor of
not assigned the car to his daughter and had he been faithful and prompt in Astorga
paying the installments required. Petitioner brought the situation upon himself o Declared that the dismissal was illegal
and cannot now complain that private respondent is liable for the mental o the abolition of CSMG done neither in good faith nor for causes
anguish and humiliation he suffered. beyond the control of SMART, but a ploy to terminate Astorga’s
x Furthermore, we agree with the appellate court that when private respondent employment
brought the complaint, it did so only to exercise a legal right, believing that it x RTC Æ denied Astorga’s MD the replevin case; denied ensuing MR
had a meritorious cause of action clearly borne out by a mere perusal of the o As correctly pointed out, this case is to enforce a right of
promissory note and chattel mortgage. To constitute malicious prosecution, possession over a company car assigned to the defendant under a
there must be proof that the prosecution was prompted by a sinister design to car plan privilege arrangement. The car is registered in the name of
vex and humiliate a person, and that it was initiated deliberately, knowing that the plaintiff. Recovery thereof via replevin suit is allowed by Rule
the charges were false and groundless. 25 Such was not the case when the 60 of the 1997 Rules of Civil Procedure, which is undoubtedly
instant complaint was filed. The rule has always been that moral damages within the jurisdiction of the Regional Trial Court.
cannot be recovered from a person who has filed a complaint against another in o In the Complaint, plaintiff claims to be the owner of the company
good faith. 26 The law always presumes good faith such that any person who car and despite demand, defendant refused to return said car. This
seeks to be awarded damages due to acts of another has the burden of proving is clearly sufficient statement of plaintiff’s cause of action.
that the latter acted in bad faith or with ill motive x Astorga Æ filed a Rule 65 before the CA against the order of the RTC denying
x Anent the award of exemplary damages, jurisprudence provides that where a her MD
party is not entitled to actual or moral damages, an award of exemplary o CA Æ granted the petition
damages is likewise baseless. 28 ƒ the CA held that the case is intertwined with Astorga’s
x In the matter of attorney's fees, petitioner avers that to prosecute and defend complaint for illegal dismissal; thus, it is the labor
this case in the lower court and in the appellate court, he incurred expenses tribunal that has rightful jurisdiction over the
amounting to P50,000.00, 29 and as such, attorney's fees should be granted. complaint
We deny the claim. No premium should be placed on the right to litigate and not x Smart Æ elevated the CA decision to the SC
every winning party is entitled to an automatic grant of attorney's fees. 30 The x Smart Æ appealed the LA decision to the NLRC
party must show that he falls under one of the instances enumerated in Article o NLRC Æ reversed LA; partially granted Astorga’s MR insofar as the
2208 of the Civil Code. 31 This, petitioner failed to do. Furthermore, where the payment of backwages is concerned
award of moral and exemplary damages is eliminated, so must the award for ƒ the NLRC declared the abolition of CSMG and the
attorney's fees be deleted. creation of SNMI to do the sales and marketing
services for SMART a valid organizational action. It
ISSUE # 5: Whether the subject vehicle should be returned to the Orosas. overruled the Labor Arbiter’s ruling that SNMI is an in-
house agency, holding that it lacked legal basis. It
HELD # 5: NO. also declared that contracting, subcontracting and
x We also agree with the Court of Appeals that the trial court erred when it streamlining of operations for the purpose of
ordered private respondent to return the subject car or its equivalent considering increasing efficiency are allowed under the law. The
that petitioner had not yet fully paid the purchase price. Verily, to sustain the NLRC further found erroneous the Labor Arbiter’s
trial court's decision would amount to unjust enrichment. The Court of Appeals disquisition that redundancy to be valid must be
was correct when it instead ordered private respondent to return, not the car impelled by economic reasons, and upheld the
itself, but only the amount equivalent to the fourteen installments actually paid redundancy measures undertaken by SMART.
with interest. x Astorga and Smart Æ filed their separate Rule 45 before the SC
o Consolidated
SMART COMMUNICATIONS, INC., petitioner, vs. REGINA M. ASTORGA, respondent.
G.R. No. 148132 January 28, 2008 ISSUE: Whether the RTC has jurisdiction over the replevin case for the recovery of the car
SMART COMMUNICATIONS, INC., petitioner, vs. REGINA M. ASTORGA, respondent. which Astorga acquired as part of her employment case.
G.R. No. 151079 January 28, 2008
REGINA M. ASTORGA, petitioner, vs. SMART COMMUNICATIONS, INC. and ANN MARGARET V. HELD: YES.
SANTIAGO, respondents. x The Court shall first deal with the propriety of dismissing the replevin case filed
G.R. No. 151372 January 28, 2008 with the RTC of Makati City allegedly for lack of jurisdiction, which is the issue
raised in G.R. No. 148132.
NACHURA, J.: x Replevin is an action whereby the owner or person entitled to
repossession of goods or chattels may recover those goods or chattels
FACTS: from one who has wrongfully distrained or taken, or who wrongfully
x Astorga was a District Manager of SMART who enjoyed benefits including a car detains such goods or chattels. It is designed to permit one having
plan right to possession to recover property in specie from one who has
x SMART then launched an organizational realignment to achieve more efficient wrongfully taken or detained the property.30 The term may refer
operations either to the action itself, for the recovery of personalty, or to the
x Thus, SMART entered into a joint venture agreement with NTT of Japan, and provisional remedy traditionally associated with it, by which
formed SMART-NTT Multimedia, Incorporated (SNMI). Since SNMI was formed to possession of the property may be obtained by the plaintiff and
do the sales and marketing work, SMART abolished the CSMG/FSD, Astorga’s retained during the pendency of the action.31
division x That the action commenced by SMART against Astorga in the RTC of Makati City
x SNMI agreed to absorb the CSMG personnel who would be recommended by was one for replevin hardly admits of doubt.
SMART x In reversing the RTC ruling and consequently dismissing the case for lack of
x One of those recommended was Astorga, who was offered a supervisory position jurisdiction, the CA made the following disquisition, viz.:
in the Customer Care Department o [I]t is plain to see that the vehicle was issued to [Astorga] by
x Astorga refused because the position carried lower salary rank and rate [Smart] as part of the employment package. We doubt that
x SMART then issued a memorandum advising Astorga of the termination of her [SMART] would extend [to Astorga] the same car plan privilege
employment on ground of redundancy were it not for her employment as district sales manager of the
x Astorga Æ filed an action for illegal dismissal, non-payment of salaries and other company. Furthermore, there is no civil contract for a loan
benefits with prayer for moral and exemplary damages against SMART and Ann between [Astorga] and [Smart]. Consequently, We find that the
Margaret V. Santiago (Santiago) car plan privilege is a benefit arising out of employer-employee
x SMART Æ filed an Answer relationship. Thus, the claim for such falls squarely within the
o argued that Astorga was dismissed by reason of redundancy, which original and exclusive jurisdiction of the labor arbiters and the
is an authorized cause for termination of employment, and the NLRC.32
dismissal was effected in accordance with the requirements of the x We do not agree. Contrary to the CA’s ratiocination, the RTC rightfully
Labor Code. The redundancy of Astorga’s position was the result of assumed jurisdiction over the suit and acted well within its discretion
the abolition of CSMG and the creation of a specialized and more in denying Astorga’s motion to dismiss. SMART’s demand for payment
technically equipped SNMI, which is a valid and legitimate exercise of the market value of the car or, in the alternative, the surrender of
of management prerogative. the car, is not a labor, but a civil, dispute. It involves the relationship of
x SMART sent a letter to Astorga demanding that she pay the current market value debtor and creditor rather than employee-employer relations.33 As
of the Honda Civic Sedan which was given to her under the company’s car plan such, the dispute falls within the jurisdiction of the regular courts.
program, or to surrender the same to the company for proper disposition x In Basaya, Jr. v. Militante,34 this Court, in upholding the jurisdiction of the RTC
x Astorga failed to comply with the demand over the replevin suit, explained:
x SMART Æ filed an action for replevin with RTC Makati Br 57 o Replevin is a possessory action, the gist of which is the right of
possession in the plaintiff. The primary relief sought therein is the
return of the property in specie wrongfully detained by another
15
RECTO, GAYLE ANGELI M.
2011-0008 | AUSL
Personal Notes on Remedial Law 2 Review (based on the syllabus of Prof. Henedino M. Brondial)

person. It is an ordinary statutory proceeding to adjudicate rights building or enclosure to be broken open and take the property into
to the title or possession of personal property. The question of his possession. After the sheriff has taken possession of the
whether or not a party has the right of possession over the property as herein provided, he must keep it in a secure place and
property involved and if so, whether or not the adverse party has shall be responsible for its delivery to the party entitled thereto
wrongfully taken and detained said property as to require its upon receiving his fees and necessary expenses for taking and
return to plaintiff, is outside the pale of competence of a labor keeping the same. (Emphasis supplied.)
tribunal and beyond the field of specialization of Labor Arbiters. o SEC. 6. Disposition of property by sheriff.-If within five (5) days
o The labor dispute involved is not intertwined with the issue in the after the taking of the property by the sheriff, the adverse party
Replevin Case. The respective issues raised in each forum can be does not object to the sufficiency of the bond, or of the surety or
resolved independently on the other. In fact in 18 November 1986, sureties thereon; or if the adverse party so objects and the court
the NLRC in the case before it had issued an Injunctive Writ affirms its approval of the applicant’s bond or approves a new
enjoining the petitioners from blocking the free ingress and egress bond, or if the adverse party requires the return of the property but
to the Vessel and ordering the petitioners to disembark and his bond is objected to and found insufficient and he does not
vacate. That aspect of the controversy is properly settled under forthwith file an approved bond, the property shall be delivered to
the Labor Code. So also with petitioners’ right to picket. But the the applicant. If for any reason the property is not delivered to the
determination of the question of who has the better right to take applicant, the sheriff must return it to the adverse party. (Emphasis
possession of the Vessel and whether petitioners can deprive the supplied.)
Charterer, as the legal possessor of the Vessel, of that right to x First, the rules provide that property seized under a writ of replevin is
possess in addressed to the competence of Civil Courts. not to be delivered immediately to the plaintiff.22 In accordance with
o In thus ruling, this Court is not sanctioning split jurisdiction but the said rules, Andres should have waited no less than five days in
defining avenues of jurisdiction as laid down by pertinent laws. order to give the complainant an opportunity to object to the
x The CA, therefore, committed reversible error when it overturned the RTC ruling sufficiency of the bond or of the surety or sureties thereon, or require
and ordered the dismissal of the replevin case for lack of jurisdiction. the return of the seized motor vehicles by filing a counter-bond. This,
he failed to do.
KENNETH HAO, complainant, vs. ABE C. ANDRES, Sheriff IV, Regional Trial Court, Branch o Records show that Andres took possession of two of the subject
16, Davao City, respondent. motor vehicles on October 17, 2005, four on October 18, 2005, and
A.M. No. P-07-2384 June 18, 2008 another three on October 19, 2005. Simultaneously, as evidenced
SECOND DIVISION by the depository receipts, on October 18, 2005, Silver received
QUISUMBING, J.: from Andres six of the seized motor vehicles, and three more motor
vehicles on October 19, 2005. Consequently, there is no question
FACTS: that Silver was already in possession of the nine seized vehicles
x Hao is one of the defendants in a civil case for replevin docketed as Civil Case immediately after seizure, or no more than three days after the
No. 31, 127-20052 entitled "Zenaida Silver, doing trade and business under the taking of the vehicles. Thus, Andres committed a clear violation of
name and style ZHS Commercial v. Loreto Hao, Atty. Amado Cantos, Kenneth Section 6, Rule 60 of the Rules of Court with regard to the proper
Hao and John Does," pending before the RTC of Davao City, Branch 16 disposal of the property.
x Judge Fuentes Æ issued an Order of Seizure4 against 22 motor vehicles allegedly x It matters not that Silver was in possession of the seized vehicles merely for
owned by the complainant. safekeeping as stated in the depository receipts. The rule is clear that the
o On the strength of the said order, Andres was able to seize two of property seized should not be immediately delivered to the plaintiff, and the
the subject motor vehicles on October 17, 2005; four on October 18, sheriff must retain custody of the seized property for at least five days.23 Hence,
2005, and another three on October 19, 2005, or a total of nine the act of Andres in delivering the seized vehicles immediately after seizure to
motor vehicles Silver for whatever purpose, without observing the five-day requirement finds no
x Hao Æ filed a complaint against Andres before the OCA legal justification.
o Hao alleged that Andres gave undue advantage to Zenaida Silver in x In Pardo v. Velasco,24 this Court held that
the implementation of the order and that Andres seized the nine o …Respondent as an officer of the Court is charged with certain
motor vehicles in an oppressive manner. Hao also averred that ministerial duties which must be performed faithfully to the letter.
Andres was accompanied by unidentified armed personnel on board a Every provision in the Revised Rules of Court has a specific reason
military vehicle which was excessive since there were no or objective. In this case, the purpose of the five (5) days is to give
resistance from them. Hao also discovered that the compound a chance to the defendant to object to the sufficiency of the bond
where the seized motor vehicles were placed is actually owned by or the surety or sureties thereon or require the return of the
Silver property by filing a counterbond.…25 (Emphasis supplied.)
x Hao then filed a counterbond x In Sebastian v. Valino,26 this Court reiterated that
x Judge Emmanuel C. Carpio Æ then ordered Andres to immediately cease and o Under the Revised Rules of Court, the property seized under a writ
desist from further implementing the order of seizure, and to return the seized of replevin is not to be delivered immediately to the plaintiff. The
motor vehicles including its accessories to their lawful owners. sheriff must retain it in his custody for five days and he shall return it
x The vehicles were however reported to have gone missing to the defendant, if the latter, as in the instant case, requires its
x Andres Æ filed his report return and files a counterbond.…27 (Emphasis supplied.)
o He narrated that on October 21, 2005, PO3 Rodrigo Despe, one of x Likewise, Andres’ claim that he had no knowledge that the compound is owned
the policemen guarding the subject motor vehicles, reported to him by Silver fails to convince us. Regardless of who actually owns the
that a certain "Nonoy" entered the compound and caused the compound, the fact remains that Andres delivered the vehicles to Silver
duplication of the vehicles’ keys prematurely. It violates the rule requiring him to safekeep the vehicles
x Subsequently, Hao reported that three of the carnapped vehicles were recovered in his custody.28 The alleged lack of facility to store the seized vehicles is
by the police.12 He then accused Andres of conspiring and conniving with Atty. unacceptable considering that he should have deposited the same in a bonded
Oswaldo Macadangdang (Silver’s counsel) and the policemen in the carnapping warehouse. If this was not feasible, he should have sought prior authorization
of the motor vehicles. Hao also accused Andres of concealing the depository from the court issuing the writ before delivering the vehicles to Silver.
receipts from them and pointed out that the depository receipts show that Silver x Second, it must be stressed that from the moment an order of delivery
and Atty. Macadangdang were the ones who chose the policemen who will guard in replevin is executed by taking possession of the property specified
the motor vehicles. therein, such property is in custodia legis. As legal custodian, it is
x Andres denied the allegations Andres’ duty to safekeep the seized motor vehicles. Hence, when he
x the case was recommended to EJ Fuentes passed his duty to safeguard the motor vehicles to Silver, he
x EJ Fuentes Æ found Andres guilty of serious negligence in the custody of the committed a clear neglect of duty.
nine motor vehicles. He recommended that Andres be suspended from office x Third, we are appalled that even after PO3 Despe reported the unauthorized
x OCA Æ Andres be held liable only for simple neglect of duty and be suspended duplication of the vehicles’ keys, Andres failed to take extra precautionary
for one (1) month and one (1) day. measures to ensure the safety of the vehicles. It is obvious that the vehicles
were put at risk by the unauthorized duplication of the keys of the vehicles.
ISSUE: Whether Sheriff Andres complied with his duties under the writ of replevin. Neither did he immediately report the incident to the police or to the court. The
loss of the motor vehicles could have been prevented if Andres immediately
HELD: NO. asked the court for an order to transfer the vehicles to another secured place as
x Being an officer of the court, Andres must be aware that there are well-defined soon as he discovered the unauthorized duplication. Under these circumstances,
steps provided in the Rules of Court regarding the proper implementation of a even an ordinary prudent man would have exercised extra diligence. His warning
writ of replevin and/or an order of seizure. The Rules, likewise, is explicit on the to the policemen to closely watch the vehicles was insufficient. Andres cannot
duty of the sheriff in its implementation. To recapitulate what should be common toss back to Silver or to the policemen the responsibility for the loss of the motor
knowledge to sheriffs, the pertinent provisions of Rule 60, of the Rules of Court vehicles since he remains chiefly responsible for their safekeeping as legal
are quoted hereunder: custodian thereof. Indeed, Andres’ failure to take the necessary precaution and
o SEC. 4. Duty of the sheriff.-Upon receiving such order, the sheriff proper monitoring of the vehicles to ensure its safety constitutes plain
must serve a copy thereof on the adverse party, together with a negligence.
copy of the application, affidavit and bond, and must forthwith take x Fourth, despite the cease and desist order, Andres failed to return the motor
the property, if it be in the possession of the adverse party, or his vehicles to their lawful owners. Instead of returning the motor vehicles
agent, and retain it in his custody. If the property or any part immediately as directed, he opted to write Silver and demand that she put up an
thereof be concealed in a building or enclosure, the sheriff must indemnity bond to secure the third-party claims. Consequently, due to his delay,
demand its delivery, and if it be not delivered, he must cause the
16
RECTO, GAYLE ANGELI M.
2011-0008 | AUSL
Personal Notes on Remedial Law 2 Review (based on the syllabus of Prof. Henedino M. Brondial)

the eventual loss of the motor vehicles rendered the order to return the seized x For a writ of replevin to issue, all that the applicant must do is to file an affidavit
vehicles ineffectual to the prejudice of the complaining owners. and bond, pursuant to Section 2, Rule 60 of the Rules, which states:
o It must be stressed that as court custodian, it was Andres’ o Sec. 2. Affidavit and bond.
responsibility to ensure that the motor vehicles were safely kept o The applicant must show by his own affidavit or that of some other
and that the same were readily available upon order of the court or person who personally knows the facts:
demand of the parties concerned. Specifically, sheriffs, being ƒ (a) That the applicant is the owner of the property
ranking officers of the court and agents of the law, must discharge claimed, particularly describing it, or is entitled to the
their duties with great care and diligence. In serving and possession thereof;
implementing court writs, as well as processes and orders of the ƒ (b) That the property is wrongfully detained by the
court, they cannot afford to err without affecting adversely the adverse party, alleging the cause of detention thereof
proper dispensation of justice. Sheriffs play an important role in the according to the best of his knowledge, information,
administration of justice and as agents of the law, high standards and belief;
of performance are expected of them.29 Hence, his failure to ƒ (c) That the property has not been distrained or taken
return the motor vehicles at the time when its return was still for a tax assessment or a fine pursuant to law, or
feasible constitutes another instance of neglect of duty. seized under a writ of execution or preliminary
x Fifth, as found by the OCA, we agree that Andres also disregarded the provisions attachment, or otherwise placed under custodia legis,
of Rule 141 of the Rules of Court with regard to payment of expenses. or if so seized, that it is exempt from such seizure or
x Under Section 9,31 Rule 141 of the Rules of Court, the procedure for the custody; and
execution of writs and other processes are: First, the sheriff must make an ƒ (d) The actual market value of the property.
estimate of the expenses to be incurred by him; Second, he must obtain court o The applicant must also give a bond, executed to the adverse party
approval for such estimated expenses; Third, the approved estimated expenses in double the value of the property as stated in the affidavit
shall be deposited by the interested party with the Clerk of Court and ex officio aforementioned, for the return of the property to the adverse party
sheriff; Fourth, the Clerk of Court shall disburse the amount to the executing if such return be adjudged, and for the payment to the adverse
sheriff; and Fifth, the executing sheriff shall liquidate his expenses within the party of such sum as he may recover from the applicant in the
same period for rendering a return on the writ. action.
x In this case, no estimate of sheriff’s expenses was submitted to the court by x We see nothing in these provisions which requires the applicant to
Andres. Without approval of the court, he also allowed Silver to pay directly to make a prior demand on the possessor of the property before he can
the policemen the expenses for the safeguarding of the motor vehicles including file an action for a writ of replevin. Thus, prior demand is not a
their meals.32 Obviously, this practice departed from the accepted procedure condition precedent to an action for a writ of replevin.
provided in the Rules of Court. x More importantly, Navarro is no longer in the position to claim that a
prior demand is necessary, as he has already admitted in his Answers
that he had received the letters that Karen Go sent him, demanding
that he either pay his unpaid obligations or return the leased motor
vehicles. Navarro’s position that a demand is necessary and has not
been made is therefore totally unmeritorious.

ROGER V. NAVARRO, Petitioner, vs. HON. JOSE L. ESCOBIDO, Presiding Judge, RTC
Branch 37, Cagayan de Oro City, and KAREN T. GO, doing business under the name KARGO
ENTERPRISES, Respondents. SPOUSES DEO AGNER and MARICON AGNER, Petitioners, vs. BPI FAMILY SAVINGS
G.R. No. 153788 November 27, 2009 BANK, INC., Respondent.
SECOND DIVISION G.R. No. 182963 June 3, 2013
BRION, J.: THIRD DIVISION
PERALTA, J.:
FACTS:
x Petitioner allegedly entered into a lease agreement with option to purchase with FACTS:
PR Karen Go, who was married to one Glenn Go and doing business under the x On 15 February 2001, spouses Deo Agner and Maricon Agner executed a
name of Kargo Enterprises which buys and sells motor vehicles, and paid post Promissory Note with Chattel Mortgage in favor of Citimotors, Inc.
dated checks which were later on dishonored. x The contract provides, among others, that: for receiving the amount of Php834,
x As a result, PR filed 2 complaints before RTC for replevin and/or sum of money 768.00, petitioners shall pay Php 17,391.00 every 15th day of each succeeding
with damages month until fully paid; the loan is secured by a 2001 Mitsubishi Adventure Super
x RTC Æ issued writs of replevin on the 2 cases Sport; and an interest of 6% per month shall be imposed for failure to pay each
x As a result, sheriff seized to vehicles and delivered them to PR installment on or before the stated due date.
x Petitioner Æ sought consolidation of the case which was granted x On the same day, Citimotors, Inc. assigned all its rights, title and interests in the
x Petitioner Æ filed an MD on the ground that complaint did not state cause of Promissory Note with Chattel Mortgage to ABN AMRO Savings Bank, Inc. (ABN
action and of lack of legal personality AMRO), which, on 31 May 2002, likewise assigned the same to BPI Family
x RTC Æ granted MD on the ground complaint did not state cause of action Savings Bank, Inc. For failure to pay four successive installments, the bank,
x PR filed MR through counsel, sent to the spouses a demand letter dated 29 August 2002,
x RTC Æ reversed declaring the entire obligation as due and demandable and requiring to pay
o Acting on the presumption that Glenn Go’s leasing business is a Php576,664.04, or surrender the mortgaged vehicle immediately upon receiving
conjugal property, the RTC held that Karen Go had sufficient the letter. As the demand was left unheeded, the bank filed on 4 October 2002
interest in his leasing business to file the action against Navarro. an action for Replevin and Damages before the Manila Regional Trial Court
However, the RTC held that Karen Go should have included her (RTC).
husband, Glenn Go, in the complaint based on Section 4, Rule 3 of x A writ of replevin was issued. Despite this, the subject vehicle was not seized.
the Rules of Court (Rules).12 Thus, the lower court ordered Karen Trial on the merits ensued.
Go to file a motion for the inclusion of Glenn Go as co-plaintiff. x On August 11, 2005, the Manila RTC Br. 33 ruled for the bank and ordered the
x Petitioner’s MR was denied spouses to jointly and severally pay the amount of Php576,664.04 plus interest
x Petitioner elevated to CA until fully paid, and the costs of suit.
o According to Navarro, a complaint which failed to state a cause of x The spouses appealed the decision to the Court of Appeals (CA), but the CA
action could not be converted into one with a cause of action by affirmed the lower court’s decision and, subsequently, denied the motion for
mere amendment or supplemental pleading. reconsideration
x CA Æ affirmed RTC x Hence, this petition.
x Petitioner Navarro Æ elevated the case to SC by way of Rule 45
o Even assuming the complaints stated a cause of action against him, ISSUE: Whether demand is a condition precedent to an action for a writ of replevin.
Navarro maintains that the complaints were premature because no
prior demand was made on him to comply with the provisions of HELD: NO.
the lease agreements before the complaints for replevin were filed. x A provision on waiver of notice or demand has been recognized as legal and
o Lastly, Navarro posits that since the two writs of replevin were valid in Bank of the Philippine Islands v. Court of Appeals,13 wherein We held:
issued based on flawed complaints, the vehicles were illegally o The Civil Code in Article 1169 provides that one incurs in delay or is
seized from his possession and should be returned to him in default from the time the obligor demands the fulfillment of the
immediately obligation from the obligee. However, the law expressly provides that
demand is not necessary under certain circumstances, and one of
ISSUE: Whether prior demand is a condition precedent to the issuance of a writ of replevin. these circumstances is when the parties expressly waive
demand. Hence, since the co-signors expressly waived demand in
HELD: NO. the promissory notes, demand was unnecessary for them to be in
x In arguing that prior demand is required before an action for a writ of replevin is default.14
filed, Navarro apparently likens a replevin action to an unlawful detainer. x Further, the Court even ruled in Navarro v. Escobido15 that prior demand is not
a condition precedent to an action for a writ of replevin, since there is nothing in
17
RECTO, GAYLE ANGELI M.
2011-0008 | AUSL
Personal Notes on Remedial Law 2 Review (based on the syllabus of Prof. Henedino M. Brondial)

Section 2, Rule 60 of the Rules of Court that requires the applicant to make a therefore, rightfully granted the alternative prayer for sum of money, which is
demand on the possessor of the property before an action for a writ of replevin equivalent to the remedy of "exacting fulfillment of the obligation." Certainly,
could be filed. there is no double recovery or unjust enrichment30 to speak of.
x Also, petitioners’ representation that they have not received a demand letter is x All the foregoing notwithstanding, We are of the opinion that the interest of 6%
completely inconsequential as the mere act of sending it would suffice. Again, per month should be equitably reduced to one percent (1%) per month or twelve
We look into the Promissory Note with Chattel Mortgage, which provides: percent (12%) per annum, to be reckoned from May 16, 2002 until full payment
o All correspondence relative to this mortgage, including demand and with the remaining outstanding balance of their car loan as of May 15, 2002 as
letters, summonses, subpoenas, or notifications of any judicial or the base amount.
extrajudicial action shall be sent to the MORTGAGOR at the address
indicated on this promissory note with chattel mortgage or at the F. Support Pendente Lite (Rule 61)
address that may hereafter be given in writing by the MORTGAGOR
to the MORTGAGEE or his/its assignee. The mere act of sending
1. Application
any correspondence by mail or by personal delivery to the said
address shall be valid and effective notice to the mortgagor for all
legal purposes and the fact that any communication is not actually Section 1. Application. — At the commencement of the proper action or proceeding, or at
received by the MORTGAGOR or that it has been returned any time prior to the judgment or final order, a verified application for support pendente lite
unclaimed to the MORTGAGEE or that no person was found at the may be filed by any party stating the grounds for the claim and the financial conditions of both
address given, or that the address is fictitious or cannot be located parties, and accompanied by affidavits, depositions or other authentic documents in support
shall not excuse or relieve the MORTGAGOR from the effects of thereof.
such notice.16 (Emphasis and underscoring supplied)
x The Court cannot yield to petitioners’ denial in receiving respondent’s demand 2. Procedure: Comment, Hearing Order
letter. To note, their postal address evidently remained unchanged from the time
they executed the Promissory Note with Chattel Mortgage up to time the case Section 2. Comment. — A copy of the application and all supporting documents shall be
was filed against them. Thus, the presumption that "a letter duly directed and served upon the adverse party, who shall have five (5) days to comment thereon unless a
mailed was received in the regular course of the mail"17 stands in the absence of different period is fixed by the court upon his motion. The comment shall be verified and shall
satisfactory proof to the contrary. be accompanied by affidavits, depositions or other authentic documents in support thereof.
x Petitioners cannot find succour from Ting v. Court of Appeals18 simply because it (2a, 3a)
pertained to violation of Batas Pambansa Blg. 22 or the Bouncing Checks Law. As
a higher quantum of proof - that is, proof beyond reasonable doubt - is required in Section 3. Hearing. — After the comment is filed, or after the expiration of the period for
view of the criminal nature of the case, We found insufficient the mere its filing, the application shall be set for hearing not more than three (3) days thereafter. The
presentation of a copy of the demand letter allegedly sent through registered facts in issue shall be proved in the same manner as is provided for evidence on motions. (4a)
mail and its corresponding registry receipt as proof of receiving the notice of
dishonor. Section 4. Order. — The court shall determine provisionally the pertinent facts, and shall
x Perusing over the records, what is clear is that petitioners did not take advantage render such orders as justice and equity may require, having the regard to the probable
of all the opportunities to present their evidence in the proceedings before the outcome of the case and such other circumstances as may aid in the proper resolution of the
courts below. They miserably failed to produce the original cash deposit slips question involved. If the application is granted, the court shall fix the amount of money to be
proving payment of the monthly amortizations in question. Not even a photocopy provisionally paid or such other forms of support as should be provided, taking into account
of the alleged proof of payment was appended to their Answer or shown during the necessities of the applicant and the resources or means of the adverse party, and the
the trial. Neither have they demonstrated any written requests to respondent to terms of payment or mode for providing the support. If the application is denied, the principal
furnish them with official receipts or a statement of account. Worse, petitioners case shall be tried and decided as early as possible. (5a)
were not able to make a formal offer of evidence considering that they have not
marked any documentary evidence during the presentation of Deo Agner’s
testimony.19
x Jurisprudence abounds that, in civil cases, one who pleads payment has the 3. Enforcement
burden of proving it; the burden rests on the defendant to prove payment,
rather than on the plaintiff to prove non-payment.20 When the creditor is in Section 5. Enforcement of order. — If the adverse party fails to comply with an order
possession of the document of credit, proof of non-payment is not needed for it granting support pendente lite, the court shall, motu proprio or upon motion; issue an order of
is presumed.21 Respondent's possession of the Promissory Note with Chattel execution against him, without prejudice to his liability for contempt. (6a)
Mortgage strongly buttresses its claim that the obligation has not been
extinguished. As held in Bank of the Philippine Islands v. Spouses Royeca:2 When the person ordered to give support pendente lite refuses or fails to do so, any third
o x x x The creditor's possession of the evidence of debt is proof that person who furnished that support to the applicant may, after due notice and hearing in the
the debt has not been discharged by payment. A promissory note same case obtain a writ of execution to enforce his right of reimbursement against the person
in the hands of the creditor is a proof of indebtedness rather than ordered to provide such support. (h)
proof of payment. In an action for replevin by a mortgagee, it is
prima facie evidence that the promissory note has not been paid. 4. Restitution
Likewise, an uncanceled mortgage in the possession of the
mortgagee gives rise to the presumption that the mortgage debt is Section 7. Restitution. — When the judgment or final order of the court finds that the
unpaid.23 person who has been providing support pendente lite is not liable therefor, it shall order the
x Indeed, when the existence of a debt is fully established by the evidence recipient thereof to return to the former the amounts already paid with legal interest from the
contained in the record, the burden of proving that it has been extinguished by dates of actual payment, without prejudice to the right of the recipient to obtain
payment devolves upon the debtor who offers such defense to the claim of the reimbursement in a separate action from the person legally obliged to give the support. Should
creditor.24 The debtor has the burden of showing with legal certainty that the the recipient fail to reimburse said amounts, the person who provided the same may likewise
obligation has been discharged by payment seek reimbursement thereof in a separate action from the person legally obliged to give such
x The remedies provided for in Art. 1484 are alternative, not cumulative. The support.
exercise of one bars the exercise of the others. This limitation applies to
contracts purporting to be leases of personal property with option to buy by
virtue of Art. 1485. The condition that the lessor has deprived the lessee of
possession or enjoyment of the thing for the purpose of applying Art. 1485 was
fulfilled in this case by the filing by petitioner of the complaint for replevin to
recover possession of movable property. By virtue of the writ of seizure issued by
the trial court, the deputy sheriff seized the vehicle on August 6, 1986 and
thereby deprived private respondents of its use. The car was not returned to
private respondent until April 16, 1989, after two (2) years and eight (8) months,
upon issuance by the Court of Appeals of a writ of execution.
x Petitioner prayed that private respondents be made to pay the sum of
P39,054.86, the amount that they were supposed to pay as of May 1986, plus
interest at the legal rate. At the same time, it prayed for the issuance of a writ of
replevin or the delivery to it of the motor vehicle "complete with accessories and
equipment." In the event the car could not be delivered to petitioner, it was
prayed that private respondent Rolando Lantan be made to pay petitioner the
amount of P60,000.00, the "estimated actual value" of the car, "plus accrued
monthly rentals thereof with interests at the rate of fourteen percent (14%) per
annum until fully paid." This prayer of course cannot be granted, even assuming
that private respondents have defaulted in the payment of their obligation. This
led the trial court to say that petitioner wanted to eat its cake and have it too
x Compared with Elisco, the vehicle subject matter of this case was never
recovered and delivered to respondent despite the issuance of a writ of replevin.
As there was no seizure that transpired, it cannot be said that petitioners were
deprived of the use and enjoyment of the mortgaged vehicle or that respondent
pursued, commenced or concluded its actual foreclosure. The trial court,
18
RECTO, GAYLE ANGELI M.
2011-0008 | AUSL
Personal Notes on Remedial Law 2 Review (based on the syllabus of Prof. Henedino M. Brondial)

ƒ (2) The validity of a marriage or legal


separation;
ƒ (3) Any ground for legal separation
ƒ (4) Future support;
ƒ (5) The jurisdiction of courts;
ƒ (6) Future legitime.
x The raison d' etre behind the proscription against renunciation, transmission
and/or compromise of the right to support is stated, thus:
o The right to support being founded upon the need of the
recipient to maintain his existence, he is not entitled to
renounce or transfer the right for this would mean
sanctioning the voluntary giving up of life itself. The right
to life cannot be renounce; hence, support which is the
means to attain the former, cannot be renounced.
o To allow renunciation or transmission or compensation of the family
right of a person to support is virtually to allow either suicide or the
conversion of the recipient to a public burden. This is contrary to
public policy. 4
x In the case at bar, respondent minor's mother, who was the plaintiff in the first
case, manifested that she was withdrawing the case as it seemed futile to claim
support from petitioner who denied his paternity over the child. Since the right to
claim for support is predicated on the existence of filiation between the minor
child and the putative parent, petitioner would like us to believe that such
manifestation admitting the futility of claiming support from him puts the issue to
rest and bars any and all future complaint for support.
x The manifestation sent in by respondent's mother in the first case, which
acknowledged that it would be useless to pursue its complaint for support,
amounted to renunciation as it severed the vinculum that gives the minor, Glen
Camil, the right to claim support from his putative parent, the petitioner.
Furthermore, the agreement entered into between the petitioner and
respondent's mother for the dismissal of the complaint for maintenance and
support conditioned upon the dismissal of the counterclaim is in the nature of a
compromise which cannot be countenanced. It violates the prohibition against
any compromise of the right to support.
x Thus, the admission made by counsel for the wife of the facts alleged in a
motion of the husband, in which the latter prayed that his obligation to support be
extinguished cannot be considered as an assent to the prayer, and much less, as a
waiver of the right to claim for support. 5
x It is true that in order to claim support, filiation and/or paternity must
first be shown between the claimant and the parent. However,
paternity and filiation or the lack of the same is a relationship that
must be judicially established and it is for the court to declare its
existence or absence. It cannot be left to the will or agreement of the
parties.
MANUEL DE ASIS, petitioner, vs. COURT OF APPEALS, HON. JAIME T. HAMOY, Branch
130, RTC, Kalookan City and GLEN CAMIL ANDRES DE ASIS represented by her x The civil status of a son having been denied, and this civil status, from which the
mother/guardian VIRCEL D. ANDRES, respondents. right to support is derived being in issue, it is apparent that no effect can be
G.R. No. 127578 February 15, 1999 .given to such a claim until an authoritative declaration has been made as to the
THIRD DIVISION existence of the cause. 6
x Although in the case under scrutiny, the admission may be binding upon the
FACTS: respondent, such an admission is at most evidentiary and does not conclusively
x Vircel D. Andres, (the herein private respondent) in her capacity as the legal establish the lack of filiation.
guardian of the minor, Glen Camil Andres de Asis, brought an action for x Neither are we persuaded by petitioner's theory that the dismissal with prejudice
maintenance and support against Manuel de Asis, alleging that the defendant of Civil Case Q-88-935 has the effect of res judicata on the subsequent case for
Manuel de Asis (the petitioner here) is the father of subject minor Glen Camil support. The case of Advincula vs. Advincula 7 comes to the fore. In Advincula,
Andres de Asis, and the former refused and/or failed to provide for the the minor, Manuela Advincula, instituted a case for acknowledgment and support
maintenance of the latter, despite repeated demands. against her putative father, Manuel Advincula. On motion of both parties and for
x Vircel D. Andres, through counsel, sent in a manifestation the pertinent portion the reason that the "plaintiff has lost interest and is no longer interested in
of which, reads; defendant (herein petitioner) has made a judicial continuing the case against the defendant and has no further evidence to
admission/declaration that defendant denies that the said minor child (Glen introduce in support of the complaint", the case was dismissed. Thereafter, a
Camil) is his child he (petitioner) has no obligation to the plaintiff Glen Camil . . . similar case was instituted by Manuela, which the defendant moved to dismiss,
That with the aforesaid judicial admission/declarations by the defendant, it theorizing that the dismissal of the first case precluded the filing of the second
seems futile and a useless exercise to claim support from said defendant.That case.
under the foregoing circumstances it would be more practical that plaintiff x In disposing such case, this Court ruled, thus:
withdraws the complains against the defendant subject to the condition that the o The new Civil Code provides that the allowance for support
defendant should not pursue his counterclaim in the above-entitled case, . . . is provisional because the amount may be increased or
x By virtue of the said manifestation, RTC dismissed with prejudice decreased depending upon the means of the giver and the
x On September 7, 1995, another Complaint for maintenance and support was needs of the recipient (Art. 297); and that the right to
brought against Manuel A. de Asis, this time in the name of Glen Camil Andres receive support cannot be renounced nor can it be
de Asis, represented by her legal guardian/mother, Vircel D. Andres transmitted to a third person neither can it be
x On October 8, 1993, petitioner moved to dismiss the Complaint on the ground of compensated with what the recipient owes the obligator
res judicata (Art .301). Furthermore, the right to support can not be
x RTC ruled that res judicata is inapplicable in an action for support for the reason waived or transferred to third parties and future support
that renunciation or waiver of future support is prohibited by law cannot be the subject of compromise (Art. 2035; Coral v.
x CA affirmed RTC Gallego, 38 O.G. 3135, cited in IV Civil Code by Padilla, p.
648; 1956 Ed.). This being true, it is indisputable that the
ISSUE: Whether an action for support may be barred by res judicata. present action for support can be brought, notwithstanding
the fact the previous case filed against the same defendant
HELD: NO. was dismissed. And it also appearing that the dismissal of
x The right to receive support can neither be renounced nor transmitted to a third Civil Case No. 3553, was not an adjudication upon the
person. Article 301 of the Civil Code, the law in point, reads: merits, as heretofore shown, the right of herein plaintiff-
o Art. 301. The right to receive support cannot be renounced, appellant to reiterate her suit for support and
nor can it be transmitted to a third person. Neither can it be acknowledgment is available, as her needs arise. Once the
compensated with what the recipient owes the obligor ...... needs of plaintiff arise, she has the right to bring an action
o Furthermore, future support cannot be the subject of a for support, for it is only then that her cause for action is
compromise. accrues ......
x Art. 2035, ibid, provides, that: o It appears that the former dismissal was predicated upon
o No compromise upon the following questions shall be valid: compromise. Acknowledgment, affecting as it does the civil status
ƒ (1) The civil status of persons; of a persons and future support, cannot be the subject of
compromise (pars. 1 & 4, Art. 2035, Civil Code). Hence, the first
19
RECTO, GAYLE ANGELI M.
2011-0008 | AUSL
Personal Notes on Remedial Law 2 Review (based on the syllabus of Prof. Henedino M. Brondial)

dismissal cannot have force and effect and can not bar the filing of obligee's lifetime.. Also, while parental authority under Title IX (and the
another action, asking for the same relief against the same correlative parental rights) pertains to parents, passing to ascendants only upon
defendant. (emphasis supplied). its termination or suspension, the obligation to provide legal support passes on
x Conformably, notwithstanding the dismissal of Civil Case Q-88-935 and the lower to ascendants not only upon default of the parents but also for the latter’s
court's pronouncement that such dismissal was with prejudice, the second action inability to provide sufficient support. As we observed in another case raising the
for support may still prosper. ancillary issue of an ascendant’s obligation to give support in light of the father’s
sufficient means:
PEOPLE OF THE PHILIPPINES, plaintiff-appellee, vs. MANUEL MANAHAN, alias Maning, o Professor Pineda is of the view that grandchildren cannot demand
defendant-appellant. support directly from their grandparents if they have parents
G.R. No. 128157. September 29, 1999 (ascendants of nearest degree) who are capable of supporting
EN BANC them. This is so because we have to follow the order of support
under Art. 199. We agree with this view.
FACTS: o There is no showing that private respondent is without means to
x RTC found Manahan guilty of rape and was sentenced to death support his son; neither is there any evidence to prove that
o He was also ordered to indemnify the victim P50,000.00 as moral petitioner, as the paternal grandmother, was willing to voluntarily
damages, pay the costs, and acknowledge and support the provide for her grandson's legal support. x x x18 (Emphasis
offspring of his indiscretion supplied; internal citations omitted)
x Manahan appealed x Here, there is no question that Cheryl is unable to discharge her obligation to
provide sufficient legal support to her children, then all school-bound. It is also
ISSUE: Whether Manahan should be ordered to support the child. undisputed that the amount of support Edward is able to give to respondents,
P6,000 a month, is insufficient to meet respondents’ basic needs. This inability of
HELD: YES. Edward and Cheryl to sufficiently provide for their children shifts a portion of
x On the matter of acknowledgment and support of the child, a correction of the their obligation to the ascendants in the nearest degree, both in the paternal
view of the court a quo is in order. Article 345 of The Revised Penal Code (petitioners) and maternal19 lines, following the ordering in Article 199. To hold
provides that persons guilty of rape shall also be sentenced to "acknowledge the otherwise, and thus subscribe to petitioners’ theory, is to sanction the anomalous
offspring, unless the law should prevent him from doing so," and "in every case scenario of tolerating extreme material deprivation of children because of
to support the offspring." In the case before us, compulsory acknowledgment of parental inability to give adequate support even if ascendants one degree
the child Melanie Tibigar is not proper there being a legal impediment in doing so removed are more than able to fill the void.1avvphi1
as it appears that the accused is a married man. As pronounced by this Court in x However, petitioners’ partial concurrent obligation extends only to their
People v. Guerrero,[16] "the rule is that if the rapist is a married man, he cannot descendants as this word is commonly understood to refer to relatives, by blood
be compelled to recognize the offspring of the crime, should there be any, as his of lower degree. As petitioners’ grandchildren by blood, only respondents Lester
child, whether legitimate or illegitimate." Consequently, that portion of the Edward, Candice Grace and Mariano III belong to this category. Indeed, Cheryl’s
judgment under review is accordingly deleted. In any case, we sustain that part right to receive support from the Lim family extends only to her husband
ordering the accused to support the child as it is in accordance with law. Edward, arising from their marital bond.20 Unfortunately, Cheryl’s share from the
amount of monthly support the trial court awarded cannot be determined from the
records. Thus, we are constrained to remand the case to the trial court for this
SPOUSES PRUDENCIO and FILOMENA LIM, Petitioners, vs. MA. CHERYL S. LIM, for limited purpose
herself and on behalf of her minor children LESTER EDWARD S. LIM, CANDICE GRACE S. LIM, x As an alternative proposition, petitioners wish to avail of the option in Article 204
and MARIANO S. LIM, III, Respondents. of the Civil Code, as amended, and pray that they be allowed to fulfill their
G.R. No. 163209 October 30, 2009 obligation by maintaining respondents at petitioners’ Makati residence. The
THIRD DIVISION option is unavailable to petitioners.
x The application of Article 204 which provides that —
FACTS: o The person obliged to give support shall have the option to fulfill
x In 1979, respondent married Edward Lim , son of petitioners. Cheryl bore the obligation either by paying the allowance fixed, or by receiving
Edward three children, respondents Lester Edward, Candice Grace and Mariano and maintaining in the family dwelling the person who has a right
III. Edward’s family business, which provided him with a monthly salary of to receive support. The latter alternative cannot be availed of in
P6,000, shouldered the family expenses. Cheryl had no steady source of income. case there is a moral or legal obstacle thereto. (Emphasis supplied)
x On 14 October 1990, Cheryl abandoned the Forbes Park residence, bringing the x is subject to its exception clause. Here, the persons entitled to receive support
children with her (then all minors), after a violent confrontation with Edward are petitioners’ grandchildren and daughter-in-law. Granting petitioners the
whom she caught with the in-house midwife of Chua Giak option in Article 204 will secure to the grandchildren a well-provided future;
x Cheryl sued petitioners, Edward, Chua Giak and Mariano for support. however, it will also force Cheryl to return to the house which, for her, is the
x The trial court ordered Edward to provide monthly support of P6,000 pendente scene of her husband’s infidelity. While not rising to the level of a legal obstacle,
lite; then rendered judgment ordering Edward and petitioners to "jointly" provide as indeed, Cheryl’s charge against Edward for concubinage did not prosper for
P40,000 monthly support to respondents, with Edward shouldering P6,000 and insufficient evidence, her steadfast insistence on its occurrence amounts to a
moral impediment bringing the case within the ambit of the exception clause of
clarified that petitioners and Chua Giak were held jointly liable with Edward Article 204, precluding its application.
because of the latter’s inability to give sufficient support
x Petitioners appealed to the Court of Appeals assailing, among others, their
liability to support respondents. Petitioners argued that while Edward’s income is CHARLES GOTARDO, Petitioner, vs. DIVINA BULING, Respondent.
insufficient, the law itself sanctions its effects by providing that legal support G.R. No. 165166 August 15, 2012
should be "in keeping with the financial capacity of the family" under Article 194 SECOND DIVISION
of the Civil Code, as amended by Executive Order No. 209 (The Family Code of
the Philippines).7 FACTS:
x CA affirmed the trial court x Respondent Divina Buling filed a complaint for compulsory recognition and
support pendente lite, claiming that the petitioner is the father of her child
Gliffze.
ISSUE: Whether there is basis to hold petitioners, as Edward’s parents, liable with him to x In his answer, the petitioner denied the imputed paternity of Gliffze.
support respondents. x RTC dismissed the complaint for insufficiency of evidence proving Gliffze’s
filiation.
HELD: YES. x CA reversed RTC’s decision.
x By statutory9 and jurisprudential mandate,10 the liability of ascendants to
provide legal support to their descendants is beyond cavil. Petitioners themselves ISSUE: Whether Divina’s testimony that she had been sexually involved only with one man,
admit as much - they limit their petition to the narrow question of when their herein petitioner Charles, at the time of her conception, is sufficient in establishing a prima
liability is triggered, not if they are liable. Relying on provisions11 found in Title facie case against the latter.
IX of the Civil Code, as amended, on Parental Authority, petitioners theorize that
their liability is activated only upon default of parental authority, conceivably HELD: YES.
either by its termination12 or suspension13 during the children’s minority. x We have recognized that "[f]iliation proceedings are usually filed not just to
Because at the time respondents sued for support, Cheryl and Edward exercised adjudicate paternity but also to secure a legal right associated with paternity,
parental authority over their children,14 petitioners submit that the obligation to such as citizenship, support (as in this case) or inheritance. [In paternity cases,
support the latter’s offspring ends with them. the burden of proof] is on the person who alleges that the putative father is the
x Neither the text of the law nor the teaching of jurisprudence supports this severe biological father of the child."31
constriction of the scope of familial obligation to give support. In the first place, x One can prove filiation, either legitimate or illegitimate, through the
the governing text are the relevant provisions in Title VIII of the Civil Code, as record of birth appearing in the civil register or a final judgment, an
amended, on Support, not the provisions in Title IX on Parental Authority. While admission of filiation in a public document or a private handwritten
both areas share a common ground in that parental authority encompasses the instrument and signed by the parent concerned, or the open and
obligation to provide legal support,15 they differ in other concerns including the continuous possession of the status of a legitimate or illegitimate child,
duration of the obligation and its concurrence among relatives of differing or any other means allowed by the Rules of Court and special laws.32
degrees.16 Thus, although the obligation to provide support arising from We have held that such other proof of one's filiation may be a
parental authority ends upon the emancipation of the child,17 the same "baptismal certificate, a judicial admission, a family bible in which his
obligation arising from spousal and general familial ties ideally lasts during the name has been entered, common reputation respecting [his] pedigree,
20
RECTO, GAYLE ANGELI M.
2011-0008 | AUSL
Personal Notes on Remedial Law 2 Review (based on the syllabus of Prof. Henedino M. Brondial)

admission by silence, the [testimonies] of witnesses, and other kinds of the motion is denied, the movant may file his answer within the remaining period, but which
proof admissible under Rule 130 of the Rules of Court."33 shall not be less than five (5) days in any event, reckoned from notice of denial. (n)
x In Herrera v. Alba,34 we stressed that there are four significant procedural
aspects of a traditional paternity action that parties have to face: a prima facie Section 5. Answer and other pleadings. — Each claimant shall file his answer setting
case, affirmative defenses, presumption of legitimacy, and physical forth his claim within fifteen (15) days from service of the summons upon him, serving a copy
resemblance between the putative father and the child.35 We explained thereof upon each of the other conflicting claimants who may file their reply thereto as
that a prima facie case exists if a woman declares — supported by corroborative provided by these Rules. If any claimant fails to plead within the time herein fixed, the court
proof — that she had sexual relations with the putative father; at this point, the may, on motion, declare him in default and thereafter render judgment barring him from any
burden of evidence shifts to the putative father.36 We explained further that the claim in respect to the subject matter.
two affirmative defenses available to the putative father are: (1) incapability of
sexual relations with the mother due to either physical absence or impotency, or The parties in an interpleader action may file counterclaims, cross-claims, third-party
(2) that the mother had sexual relations with other men at the time of complaints and responsive pleadings thereto, as provided by these Rules. (4a, R63)
conception.37
x In this case, the respondent established a prima facie case that the petitioner is Section 6. Determination. — After the pleadings of the conflicting claimants have been
the putative father of Gliffze through testimony that she had been sexually filed, and pre-trial has been conducted in accordance with the Rules, the court shall proceed to
involved only with one man, the petitioner, at the time of her conception.38 determine their respective rights and adjudicate their several claims. (5a, R63)
Rodulfo corroborated her testimony that the petitioner and the respondent had
intimate relationship.39 Section 7. Docket and other lawful fees, costs and litigation expenses as liens. —
x On the other hand, the petitioner did not deny that he had sexual encounters The docket and other lawful fees paid by the party who filed a complaint under this Rule, as
with the respondent, only that it occurred on a much later date than the well as the costs and litigation expenses, shall constitute a lien or change upon the subject
respondent asserted, such that it was physically impossible for the respondent to matter of the action, unless the court shall order otherwise. (6a, R63)
have been three (3) months pregnant already in September 1994 when he was
informed of the pregnancy.40 However, the petitioner failed to substantiate his
allegations of infidelity and insinuations of promiscuity. His allegations, therefore, WACK WACK GOLF & COUNTRY CLUB, INC., plaintiff-appellant, vs. LEE E. WON alias
cannot be given credence for lack of evidentiary support. The petitioner’s denial RAMON LEE and BIENVENIDO A. TAN, defendants-appellees.
cannot overcome the respondent’s clear and categorical assertions. EN BANC, G.R. No. L-23851 March 26, 1976
x We find that the contradictions are for the most part more apparent than real,
having resulted from the failure of the respondent to comprehend the question FACTS:
posed, but this misunderstanding was later corrected and satisfactorily explained. x Lee E. Won claims ownership of Wack Wack’s membership fee certificate 201, by
Indeed, when confronted for her contradictory statements, the respondent virtue of the decision rendered in civil case 26044 of the CFI of Manila, entitled
explained that that portion of the transcript of stenographic notes was incorrect "Lee E. Won alias Ramon Lee vs. Wack Wack Golf & Country Club, Inc." and also
and she had brought it to the attention of Atty. Josefino Go Cinco (her former by virtue of membership fee certificate 201-serial no. 1478 issued on October 17,
counsel) but the latter took no action on the matter.42 1963 by Ponciano B. Jacinto, deputy clerk of court of the said CFI of Manila, for
x Jurisprudence teaches that in assessing the credibility of a witness, his testimony and in behalf of the president and the secretary of the Corporation and of the
must be considered in its entirety instead of in truncated parts. The technique in People's Bank & Trust Company as transfer agent of the said Corporation,
deciphering a testimony is not to consider only its isolated parts and to anchor a pursuant to the order of September 23, 1963 in the said case
conclusion based on these parts. "In ascertaining the facts established by a
x On the other hand, Bienvenido A. Tan claims to be lawful owner of its aforesaid
witness, everything stated by him on direct, cross and redirect examinations
membership fee certificate 201 by virtue of membership fee certificate 201-serial
must be calibrated and considered."43 Evidently, the totality of the respondent's
no. 1199 issued to him on July 24, 1950 pursuant to an assignment made in his
testimony positively and convincingly shows that no real inconsistency exists.
favor by "Swan, Culbertson and Fritz," the original owner and holder of
The respondent has consistently asserted that she started intimate sexual
membership fee certificate 201
relations with the petitioner sometime in September 1993.44
x Wack Wack Golf & Country Club, Inc. Æ filed an action for interpleader praying
x Since filiation is beyond question, support follows as a matter of obligation; a
for:
parent is obliged to support his child, whether legitimate or illegitimate.45
o (a) an order be issued requiring Lee and Tan to interplead and
Support consists of everything indispensable for sustenance, dwelling, clothing,
litigate their conflicting claims; and
medical attendance, education and transportation, in keeping with the financial
o (b) judgment. be rendered, after hearing, declaring who of the two
capacity of the family.46 Thus, the amount of support is variable and, for this
is the lawful owner of membership fee certificate 201, and ordering
reason, no final judgment on the amount of support is made as the amount shall
the surrender and cancellation of membership fee certificate 201-
be in proportion to the resources or means of the giver and the necessities of the
serial no. 1478 issued in the name of Lee.
recipient.47 It may be reduced or increased proportionately according to the
x Won and Tan Æ filed separate MDs
reduction or increase of the necessities of the recipient and the resources or
o upon the grounds of res judicata, failure of the complaint to state a
means of the person obliged to support.48
cause of action, and bar by prescription.
x In this case, we sustain the award of P 2,000.00 monthly child support, without
x RTC Æ granted MDs; dismissed the complaint
prejudice to the filing of the proper motion in the RTC for the determination of
x Wack-Wack Æ appealed
any support in arrears, considering the needs of the child, Gliffze, during the
o The trial court erred in dismissing the complaint, instead of
pendency of this case.
compelling the appellees to interplead because there actually are
conflicting claims between the latter with respect to the ownership
_____________________________________
of membership fee certificate 201, and, as there is not Identity of
parties, of subject-matter, and of cause of action, between civil
SPECIAL CIVIL ACTIONS case 26044 of the CFI of Manila and the present action, the
complaint should not have been dismissed upon the ground of res
A. Interpleader [RULE 62] judicata.

1. What is an action in Interpleader? ISSUE: Whether the action for interpleader was timely filed.

HELD: NO.
Section 1. When interpleader proper. — Whenever conflicting claims upon the same
x Although res judicata or bar by a prior judgment was the principal ground availed
subject matter are or may be made against a person who claims no interest whatever in the
of by the appellees in moving for the dismissal of the complaint and upon which
subject matter, or an interest which in whole or in part is not disputed by the claimants, he
the trial court actually dismissed the complaint, the determinative issue, as can
may bring an action against the conflicting claimants to compel them to interplead and litigate
be gleaned from the pleadings of the parties, relates to the propriety and
their several claims among themselves.
timeliness of the remedy of interpleader.
x The action of interpleader, under section 120 of the Code of Civil
2. Requisites Procedure, 2 is a remedy whereby a person who has personal property
in his possession, or an obligation to render wholly or partially, without
See Section 1 above claiming any right to either, comes to court and asks that the persons
who claim the said personal property or who consider themselves
3. Procedure entitled to demand compliance with the obligation, be required to
litigate among themselves in order to determine finally who is entitled
to tone or the one thing. The remedy is afforded to protect a person not
Section 2. Order. — Upon the filing of the complaint, the court shall issue an order
against double liability but against double vexation in respect of one
requiring the conflicting claimants to interplead with one another. If the interests of justice so
liability. 3 The procedure under the Rules of Court 4 is the same as that
require, the court may direct in such order that the subject matter be paid or delivered to the
under the Code of Civil Procedure, 5 except that under the former the
court. (2a, R63)
remedy of interpleader is available regardless of the nature of the
subject-matter of the controversy, whereas under the latter an
Section 3. Summons. — Summons shall be served upon the conflicting claimants, together
interpleader suit is proper only if the subject-matter of the controversy
with a copy of the complaint and order. (3, R63)
is personal property or relates to the performance of an obligation.
x There is no question that the subject matter of the present controversy, i.e., the
Section 4. Motion to dismiss. — Within the time for filing an answer, each claimant may
membership fee certificate 201, is proper for an interpleader suit. What is
file a motion to dismiss on the ground of impropriety of the interpleader action or on other
here disputed is the propriety and timeliness of the remedy in the light
appropriate grounds specified in Rule 16. The period to file the answer shall be tolled and if
of the facts and circumstances obtaining.
21
RECTO, GAYLE ANGELI M.
2011-0008 | AUSL
Personal Notes on Remedial Law 2 Review (based on the syllabus of Prof. Henedino M. Brondial)

x A stakeholder 6 should use reasonable diligence to hale the contending claimants other than the assignees of the judgment (the bank and Mrs. Pabb) and no
to court. 7 He need not await actual institution of independent suits against him excuse is shown why he did not implead them in the suit. 18
before filing a bill of interpleader. 8 He should file an action of interpleader within x To now permit the Corporation to bring Lee to court after the latter's successful
a reasonable time after a dispute has arisen without waiting to be sued by either establishment of his rights in civil case 26044 to the membership fee certificate
of the contending claimants. 9 Otherwise, he may be barred by laches 10 or 201, is to increase instead of to diminish the number of suits, which is one of the
undue delay. 11 But where he acts with reasonable diligence in view of the purposes of an action of interpleader, with the possibility that the latter would lose
environmental circumstances, the remedy is not barred. 12 the benefits of the favorable judgment. This cannot be done because having
x Has the Corporation in this case acted with diligence, in view of all the elected to take its chances of success in said civil case 26044, with full
circumstances, such that it may properly invoke the remedy of knowledge of all the fact, the Corporation must submit to the consequences of
interpleader? We do not think so. It was aware of the conflicting claims defeat.
of the appellees with respect to the membership fee certificate 201 x The act providing for the proceeding has nothing to say touching the right of
long before it filed the present interpleader suit. It had been one, after contesting a claim of one of the claimants to final judgment
recognizing Tan as the lawful owner thereof. It was sued by Lee who unsuccessfully, to involve the successful litigant in litigation anew by bringing an
also claimed the same membership fee certificate. Yet it did not interpleader action. The question seems to be one of first impression here, but, in
interplead Tan. It preferred to proceed with the litigation (civil case other jurisdictions, from which the substance of the act was apparently taken, the
26044) and to defend itself therein. As a matter of fact, final judgment rule prevails that the action cannot be resorted to after an unsuccessful trial
was rendered against it and said judgment has already been executed. against one of the claimants.
It is not therefore too late for it to invoke the remedy of interpleader. x It is well settled, both by reasons and authority, that one who asks the
x It has been held that a stakeholder's action of interpleader is too late when filed interposition of a court of equity to compel others, claiming property in his
after judgment has been rendered against him in favor of one of the contending hands, to interplead, must do so before putting them to the test of trials at law.
claimants, 13 especially where he had notice of the conflicting claims prior to the Yarborough v. Thompson, 3 Smedes & M. 291 (41 Am. Dec. 626); Gornish v.
rendition of the judgment and neglected the opportunity to implead the adverse Tanner, 1 You. & Jer. 333; Haseltine v. Brickery, 16 Grat. (Va.) 116. The remedy
claimants in the suit where judgment was entered. This must be so, because by interpleader is afforded to protect the party from the annoyance and hazard
once judgment is obtained against him by one claimant he becomes liable to the of two or more actions touching the same property or demand; but one who,
latter. 14 In once case, 15 it was declared: with knowledge of all the facts, neglects to avail himself of the relief, or elects to
o The record here discloses that long before the rendition of the take the chances for success in the actions at law, ought to submit to the
judgment in favor of relators against the Hanover Fire Insurance consequences of defeat. To permit an unsuccessful defendant to compel the
Company the latter had notice of the adverse claim of South to the successful plaintiffs to interplead, is to increase instead of to diminish the
proceeds of the policy. No reason is shown why the Insurance number of suits; to put upon the shoulders of others the burden which he asks
Company did not implead South in the former suit and have the may be taken from his own ..... '
conflicting claims there determined. The Insurance Company x It is urged, however, that the American Surety Company of New York was not in
elected not to do so and that suit proceeded to a final judgment in position to file an interpleader until it had tested the claim of relatrix to final
favor of relators. The Company thereby became independently judgment, and that, failing to meet with success, it promptly filed the
liable to relators. It was then too late for such company to invoke interpleader. The reason why, it urges, it was not in such position until then is
the remedy of interpleader that had it succeeded before this court in sustaining its construction of the bond
x The Corporation has not shown any justifiable reason why it did not file an and the law governing the bond, it would not have been called upon to file an
application for interpleader in civil case 26044 to compel the appellees herein to interpleader, since there would have been sufficient funds in its hands to have
litigate between themselves their conflicting claims of ownership. It was only satisfied all lawful claimants. It may be observed, however, that the surety
after adverse final judgment was rendered against it that the remedy of company was acquainted with all of the facts, and hence that it simply took its
interpleader was invoked by it. By then it was too late, because to he entitled to chances of meeting with success by its own construction of the bond and the
this remedy the applicant must be able to show that lie has not been made law. Having failed to sustain it, it cannot now force relatrix into litigation anew
independently liable to any of the claimants. And since the Corporation is already with others, involving most likely a repetition of what has been decided, or force
liable to Lee under a final judgment, the present interpleader suit is clearly her to accept a pro rata part of a fund, which is far from benefits of the
improper and unavailing. judgment. 19
o It is the general rule that before a person will be deemed x Besides, a successful litigant cannot later be impleaded by his defeated
to be in a position to ask for an order of intrepleader, he adversary in an interpleader suit and compelled to prove his claim anew against
must be prepared to show, among other prerequisites, that other adverse claimants, as that would in effect be a collateral attack upon the
he has not become independently liable to any of the judgment.
claimants. 25 Tex. Jur. p. 52, Sec. 3; 30 Am. Jur. p. 218, x The jurisprudence of this state and the common law states is well-settled that a
Section 8. claimant who has been put to test of a trial by a surety, and has establish his
o It is also the general rule that a bill of interpleader comes claim, may not be impleaded later by the surety in an interpleader suit, and
too late when it is filed after judgment has been rendered compelled to prove his claim again with other adverse claimants. American
in favor of one of the claimants of the fund, this being Surety Company of New York v. Brim, 175 La. 959, 144 So. 727; American
especially true when the holder of the funds had notice of Surety Company of New York v. Brim (In Re Lyong Lumber Company), 176 La.
the conflicting claims prior to the rendition of the judgment 867, 147 So. 18; Dugas v. N.Y. Casualty Co., 181 La. 322, 159 So. 572; 15
and had an opportunity to implead the adverse claimants in Ruling Case Law, 228; 33 Corpus Juris, 477; 4 Pomeroy's Jurisprudence, 1023;
the suit in which the judgment was rendered. United Royal Neighbors of America v. Lowary (D.C.) 46 F2d 565; Brackett v. Graves, 30
Procedures Pipe Line Co. v. Britton, Tex. Civ. App. 264 S.W. App. Div. 162, 51 N.Y.S. 895; De Zouche v. Garrison, 140 Pa. 430, 21 A. 450,
176; Nash v. McCullum, Tex. Civ. 74 S.W. 2d 1046; 30 Am. 451; Manufacturer's Finance Co. v. W.I. Jones Co. 141 Ga., 519, 81 S.E. 1033;
Jur. p. 223, Sec. 11; 25 Tex. Jur. p. 56, Sec. 5; 108 A.L.R., Hancock Mutual Life Ins. Co. v. Lawder, 22 R.I. 416, 84 A. 383.
note 5, p. 275. 16 x There can be no doubt that relator's claim has been finally and definitely
x Indeed, if a stakeholder defends a suit filed by one of the adverse claimants and established, because that matter was passed upon by three courts in definitive
allows said suit to proceed to final judgment against him, he cannot later on judgments. The only remaining item is the value of the use of the land during
have that part of the litigation repeated in an interpleader suit. In the case at the time that relator occupied it. The case was remanded solely and only for the
hand, the Corporation allowed civil case 26044 to proceed to final judgment. And purpose of determining the amount of that credit. In all other aspects the
it offered no satisfactory explanation for its failure to implead Tan in the same judgment is final. 20
litigation. In this factual situation, it is clear that this interpleader suit cannot
prosper because it was filed much too late. x It is generally held by the cases it is the office of interpleader to protect a party,
x If a stakeholder defends a suit by one claimant and allows it to proceed so far as not against double liability, but against double vexation on account of one
a judgment against him without filing a bill of interpleader, it then becomes too liability. Gonia v. O'Brien, 223 Mass. 177, 111 N.E. 787. And so it is said that it is
late for him to do so. Union Bank v. Kerr, 2 Md. Ch. 460; Home Life Ins. Co. v. too late for the remedy of interpleader if the party seeking this relef has
Gaulk, 86 Md. 385, 390, 38 A. 901; Gonia v. O'Brien, 223 Mass. 177, 111 N.E. contested the claim of one of the parties and suffered judgment to be taken.
787. It is one o the main offices of a bill of interpleader to restrain a separate x In United P.P.I. Co. v. Britton (Tex. Civ. App.) 264 S.W. 576. 578, it was said: 'It
proceeding at law by claimant so as to avoid the resulting partial judgment; and is the general rule that a bill of interpleader comes too late when application
if the stakeholder acquiesces in one claimant's trying out his claim and therefor is delayed until after judgment has been rendered in favor of one of the
establishing it at law, he cannot then have that part of the litigation repeated in an claimants of the fund, and this is especially true where the holder of the fund
interpleader suit. 4 Pomeroy's Eq. Juris. No. 162; Mitfor's Eq. Pleading (Tyler's Ed.) had notice of the conflicting claims prior to the rendition of such judgment and
147 and 236; Langdell's Summary of Eq. Pleading, No. 162' De Zouche v. an opportunity to implead the adverse claimants in the suit in which such
Garrizon, 140 Pa. 430, 21 A/450. 17 judgment was rendered. See notes and cases cited 35 Am. Dec. 703; 91 An. St.
x It is the general rule that a bill of interpleader comes too late when Rep. 598; also 5 Pomeroy's Equity Jurisprudence No. 41.'
application therefore is delayed until after judgment has been rendered x The principle thus stated has been recognized in many cases in other
in favor of one of the claimants of the fund, and that this is especially jurisdictions, among which may be cited American Surety Co. v. O'Brien, 223
true where the holder of the fund had notice of the conflicting claims Mass. 177, 111 N.E. 787; Phillips v. Taylor, 148 Md. 157, 129 A. 18; Moore v.
prior to the rendition of such judgment and an opportunity to implead Hill, 59 Ga. 760, 761; Yearborough v. Thompson, 3 Smedes & M. (11 Miss.) 291,
the adverse claimants in the suit in which such judgment was 41 Am. Dec. 626. See, also, 33 C.J. p. 447, No. 30; Nash v. McCullum, (Tex. Civ.
rendered. (See notes and cases cited 36 Am. Dec. 703, Am. St. Rep. 598, also App.) 74 S.W. 2d 1042, 1047.
5 Pomeroy's Eq. Juris. Sec. 41.) x It would seem that this rule should logically follow since, after the recovery of
x The evidence in the opinion of the majority shows beyond dispute that the judgment, the interpleading of the judgment creditor is in effect a collateral
appellant permitted the Parker county suit to proceed to judgment in favor of attack upon the judgment. 21
Britton with full notice of the adverse claims of the defendants in the present suit
22
RECTO, GAYLE ANGELI M.
2011-0008 | AUSL
Personal Notes on Remedial Law 2 Review (based on the syllabus of Prof. Henedino M. Brondial)

x In fine, the instant interpleader suit cannot prosper because the MISSION under the Land Development Agreement, said amounts deposited to be
Corporation had already been made independently liable in civil case paid to whomever may be found later to be entitled thereto, with costs.
26044 and, therefore, its present application for interpleader would in o IAC Æ dismissed petition BUT reversed itself
effect be a collateral attack upon the final judgment in the said civil ƒ WHEREFORE, the Court reconsiders its decision of
case; the appellee Lee had already established his rights to membership fee February 27, 1986, and sets aside the questioned
certificate 201 in the aforesaid civil case and, therefore, this interpleader suit portions of the respondent Court's orders of February
would compel him to establish his rights anew, and thereby increase instead of 13 and October 26, 1984. The private respondent is
diminish litigations, which is one of the purposes of an interpleader suit, with the hereby ordered to deposit whatever amounts are due
possiblity that the benefits of the final judgment in the said civil case might from it under the Land Development Agreement of
eventually be taken away from him; and because the Corporation allowed itself October 6, 1976 with a reputable bank to be
to be sued to final judgment in the said case, its action of interpleader was filed designated by the respondent court to be the
inexcusably late, for which reason it is barred by laches or unreasonable delay. depository trustee of the said amounts to be paid to
whoever shall be found entitled thereto. No costs
ETERNAL GARDENS MEMORIAL PARKS CORPORATION, petitioner, vs. FIRST x Eternal Æ filed an MR
SPECIAL CASES DIVISION INTERMEDIATE APPELLATE COURT and NORTH x IAC Æ resolved as follows
PHILIPPINE UNION MISSION OF THE SEVENTH-DAY ADVENTISTS, respondents. o In the meantime, to avoid possible wastage of funds, the Court
SECOND DIVISION, G.R. No. 73794 September 19, 1988 RESOLVED to require the private respondent 6 to DEPOSIT its
accruing installments within ten (10) days from notice with a
FACTS: reputable commercial bank in a savings deposit account, in the
x Petitioner Eternal Gardens Memorial Parks Corporation and private respondent name of the Supreme Court of the Philippines, with the details to
North Philippine Union Mission Corporation of the Seventh Day Adventists be reported or manifested to this Court within ten (10) days from
(MISSION for short) executed a Land Development Agreement whereby the the time the deposit/deposits are made, such deposits not to be
former undertook to introduce and construct at its own expense and withdrawn without authority from this Court.
responsibility necessary improvements on the property owned by private
respondent into a memorial park to be subdivided into and sold as memorial plot ISSUE: Whether Mission correctly resorted to an interpleader.
lots, at a stipulated area and price per lot and the latter is entitled to 40% of the
net gross collection from the project to be remitted monthly by petitioner to HELD: YES.
private respondent through a designated depositary trustee bank x There is no question that courts have inherent power to amend their judgments,
x Maysilo Estate asserted its claim of ownership over the parcel of land in question to make them conformable to the law applicable provided that said judgments
x Eternal Æ filed a complaint for interpleader (Rollo, pp. 169-179) against private have not yet attained finality (Villanueva v. Court of First Instance of Oriental
respondent MISSION and Maysilo Estate, docketed as Special Court Case No. C- Mindoro, Pinamalayan Branch II, 119 SCRA 288 [1982]). In fact, motions for
9556 of the then CFI of Rizal, Branch XII, Caloocan, alleging among others, that reconsideration are allowed to convince the courts that their rulings are
in view of the conflicting claims of ownership of the defendants (herein private erroneous and improper Siy v. Court of Appeals, 138 SCRA 543-544 [1985];
respondent and Maysilo Estate) over the properties subject matter of the Guerra Enterprises Co., Inc. v. CFI of Lanao del Sur (32 SCRA 317 [1970]) and in
contracts, over which plaintiff corporation (herein petitioner) has no claim of so doing, said courts are given sufficient opportunity to correct their errors.
ownership except as a purchaser thereof, and to protect the interests of plaintiff x In the case at bar, a careful analysis of the records will show that petitioner
corporation which has no interest in the subject matter of the dispute and is admitted among others in its complaint in Interpleader that it is still obligated to
willing to pay whoever is entitled or declared to be the owners of said properties, pay certain amounts to private respondent; that it claims no interest in such
the defendants should be required to interplead and litigate their several claims amounts due and is willing to pay whoever is declared entitled to said amounts.
between themselves Such admissions in the complaint were reaffirmed in open court before the Court
x Maysilo Æ filed an MD of Appeals as stated in the latter court's resolution dated September 5, 1985 in
o for lack of cause of action but also presented an answer dated A.C. G.R. No. 04869 which states:
November 12, 1981 o The private respondent (MEMORIAL) then reaffirms before the
x RTC Æ denied MD Court its original position that it is a disinterested party with respect
x Mission Æ then filed a motion for the placing on judicial deposit the amounts due to the property now the subject of the interpleader case ...
and unpaid from petitioner o In the light of the willingness, expressly made before the court,
o RTC Æ denied affirming the complaint filed below, that the private respondent
x RTC Æ set aside the order (MEMORIAL) will pay whatever is due on the Land Development
o (a) The order directing the NORTH PHILIPPINE UNION Agreement to the rightful owner/owners, there is no reason why
MISSION CORPORATION OF SEVENTH-DAY ADVENTISTS to the amount due on subject agreement has not been placed in the
deposit the amounts it received under the implementation of the custody of the Court. (Rollo, p. 227).
LAND DEVELOPMENT AGREEMENT which is not questioned by the x Under the circumstances, there appears to be no plausible reason for
plaintiff, Eternal Gardens, is hereby ordered set aside for the petitioner's objections to the deposit of the amounts in litigation after
reason that the titles to ownership, the North Philippine Union having asked for the assistance of the lower court by filing a complaint
Mission Corporation of Seventh Day Adventists on the lots subject for interpleader where the deposit of aforesaid amounts is not only
matter of the aforesaid agreement is not established invalid, and required by the nature of the action but is a contractual obligation of
the alleged titles of intervenors are not proven yet by competent the petitioner under the Land Development Program (Rollo, p. 252).
evidence; x As correctly observed by the Court of Appeals, the essence of an
o (b) The motion to require Eternal Gardens to deposit the interpleader, aside from the disavowal of interest in the property in
balance under the Land Development Agreement is likewise hereby litigation on the part of the petitioner, is the deposit of the property or
ordered denied considering the fact the aforesaid plaintiff had not funds in controversy with the court. it is a rule founded on justice and
denied its obligations under the aforesaid contract; and equity: "that the plaintiff may not continue to benefit from the
o (c) The trial or hearing is hereby ordered as scheduled to property or funds in litigation during the pendency of the suit at the
proceed on November 29, 1984 and on December 6, 1984 at 8:30 expense of whoever will ultimately be decided as entitled thereto."
in the morning per order of this Court dated October 4, 1984 in (Rollo, p. 24).
order to determine the alleged claims of ownership by the x The case at bar was elevated to the Court of Appeals on certiorari with
intervenors and all claims and allegations of each party to the prohibitory and mandatory injunction. Said appellate court found that more than
instant" case will be considered and decided carefully by this court twenty million pesos are involved; so that on interest alone for savings or time
on just and meritorious grounds. (Rollo, p. 39) deposit would be considerable, now accruing in favor of the Eternal Gardens.
x Mission Æ filed an MD on the Interpleader and the claims of the Maysilo Estate Finding that such is violative of the very essence of the complaint for
and the Intervenors and to order the Eternal Gardens to comply with its Land interpleader as it clearly runs against the interest of justice in this case, the Court
Management with MISSION. of Appeals cannot be faulted for finding that the lower court committed a grave
x RTC Æ granted the MD; dismissed the interpleader abuse of discretion which requires correction by the requirement that a deposit
x RTC Æ REVERSED itself of said amounts should be made to a bank approved by the Court. (Rollo, p.-25)
x Inspite of the new order of the RTC reversing itself, Mission filed a motion for x Petitioner would now compound the issue by its obvious turn-about, presently
execution claiming in its memorandum that there is a novation of contract so that the
o RTC Æ denied amounts due under the Land Development Agreement were allegedly
x Mission Æ elevated to CA via Rule 65 extinguished, and the requirement to make a deposit of said amounts in a
o CA Æ dismissed petition depositary bank should be held in abeyance until after the conflicting claims of
x Mission Æ elevated to SC ownership now on trial before Branch CXXII RTC-Caloocan City, has finally been
o SC Æ denied resolved.
x Heirs of the late spouses Vicente Singson Encarnacion and Lucila Conde Æ filed x All these notwithstanding, the need for the deposit in question has been
Civil Case No. C-11836 for quieting of title with Branch CXXII, Regional Trial established, riot only in the lower courts and in the Court of Appeals but also in
Court, Caloocan City, where petitioner and private respondent were named as the Supreme Court where such deposit was required in "the resolution of July 8,
defendants. 1987 to avoid wastage of funds.
x Mission Æ filed a petition for certiorari with the then Intermediate Appellate
Court docketed as AC-G.R. No. 04869 praying that the aforementioned Orders of
February 13, 1984 and October 26, 1984 of the Regional Trial Court be set aside SUBHASH C. PASRICHA and JOSEPHINE A. PASRICHA, Petitioners, vs. DON LUIS DISON
and that an order be issued to deposit in court or in a depositor trustee bank of REALTY, INC., Respondent.
any and all payments, plus interest thereon, due the private respondent Third Division, G.R. No. 136409 March 14, 2008
23
RECTO, GAYLE ANGELI M.
2011-0008 | AUSL
Personal Notes on Remedial Law 2 Review (based on the syllabus of Prof. Henedino M. Brondial)

accompanied by consignation in order that the effects of payment may be


FACTS: produced.59
x Respondent Don Luis Dison Realty, Inc. and petitioners executed two Contracts x Moreover, Section 1, Rule 62 of the Rules of Court provides:
of Lease3 whereby the former, as lessor, agreed to lease to the latter Units 22, o Section 1. When interpleader proper. - Whenever conflicting claims
24, 32, 33, 34, 35, 36, 37 and 38 of the San Luis Building, located at 1006 M.Y. upon the same subject matter are or may be made against a
Orosa cor. T.M. Kalaw Streets, Ermita, Manila. Petitioners, in turn, agreed to pay person who claims no interest whatever in the subject matter, or
monthly rentals an interest which in whole or in part is not disputed by the
x The lease of Rooms 36, 37 and 38 did not materialize leaving only Rooms 22, 24, claimants, he may bring an action against the conflicting claimants
32, 33, 34 and 35 as subjects of the lease contracts to compel them to interplead and litigate their several claims
x Pasricha Æ religiously paid the monthly rentals until May 1992 among themselves.
x Demands followed x Otherwise stated, an action for interpleader is proper when the lessee
x Don Luis (through representative Ms. Bautista) Æ filed a complaint for ejectment does not know to whom payment of rentals should be made due to
before MeTC Mla Branch 12 conflicting claims on the property (or on the right to collect).60 The
x MeTC Æ dismissed the complaint remedy is afforded not to protect a person against double liability but
o It considered petitioners’ non-payment of rentals as unjustified. to protect him against double vexation in respect of one liability.61
The court held that mere willingness to pay the rent did not x Notably, instead of availing of the above remedies, petitioners opted to refrain
amount to payment of the obligation; petitioners should have from making payments.
deposited their payment in the name of respondent company. On x Neither can petitioners validly invoke the non-delivery of Rooms 36, 37 and 38 as
the matter of possession of the subject premises, the court did not a justification for non-payment of rentals. Although the two contracts embraced
give credence to petitioners’ claim that private respondent failed to the lease of nine (9) rooms, the terms of the contracts - with their particular
turn over possession of the premises. The court, however, reference to specific rooms and the monthly rental for each - easily raise the
dismissed the complaint because of Ms. Bautista’s alleged lack of inference that the parties intended the lease of each room separate from that of
authority to sue on behalf of the corporation. the others.lavvphil There is nothing in the contract which would lead to the
x RTC Æ reversed conclusion that the lease of one or more rooms was to be made dependent upon
o The court adopted the MeTC’s finding on petitioners’ unjustified the lease of all the nine (9) rooms. Accordingly, the use of each room by the
refusal to pay the rent, which is a valid ground for ejectment. It, lessee gave rise to the corresponding obligation to pay the monthly rental for the
however, faulted the MeTC in dismissing the case on the ground of same. Notably, respondent demanded payment of rentals only for the rooms
lack of capacity to sue. Instead, it upheld Ms. Bautista’s authority to actually delivered to, and used by, petitioners.
represent respondent notwithstanding the absence of a board x It may also be mentioned that the contract specifically provides that the lease of
resolution to that effect, since her authority was implied from her Rooms 36, 37 and 38 was to take effect only when the tenants thereof would
power as a general manager/treasurer of the company. vacate the premises. Absent a clear showing that the previous tenants had
vacated the premises, respondent had no obligation to deliver possession of the
subject rooms to petitioners. Thus, petitioners cannot use the non-delivery of
ISSUE: Whether Pasricha should have availed of the remedy of interpleader considering their Rooms 36, 37 and 38 as an excuse for their failure to pay the rentals due on the
defense that they did not know to whom payments are due. other rooms they occupied.1avvphil
x In light of the foregoing disquisition, respondent has every right to exercise his
HELD: YES. right to eject the erring lessees. The parties’ contracts of lease contain identical
x Petitioners’ justifications are belied by the evidence on record. As correctly held provisions, to wit:
by the CA, petitioners’ communications to respondent prior to the filing of the o In case of default by the LESSEE in the payment of rental on the
complaint never mentioned their alleged inability to use the rooms.52 What they fifth (5th) day of each month, the amount owing shall as penalty
pointed out in their letters is that they did not know to whom payment should be bear interest at the rate of FOUR percent (4%) per month, to be
made, whether to Ms. Bautista or to Pacheco.53 In their July 26 and October 30, paid, without prejudice to the right of the LESSOR to terminate his
1993 letters, petitioners only questioned the method of computing their electric contract, enter the premises, and/or eject the LESSEE as
billings without, however, raising a complaint about their failure to use the hereinafter set forth;62
rooms.54 Although petitioners stated in their December 30, 1993 letter that x Moreover, Article 1673 of the Civil Code gives the lessor the right to judicially
respondent failed to fulfill its part of the contract,55 nowhere did they specifically eject the lessees in case of non-payment of the monthly rentals. A contract of
refer to their inability to use the leased rooms. Besides, at that time, they were lease is a consensual, bilateral, onerous and commutative contract by which the
already in default on their rentals for more than a year. owner temporarily grants the use of his property to another, who undertakes to
x If it were true that they were allowed to use only one of the nine (9) rooms pay the rent therefor.64 For failure to pay the rent, petitioners have no right to
subject of the contract of lease, and considering that the rooms were intended remain in the leased premises.
for a business purpose, we cannot understand why they did not specifically
assert their right. If we believe petitioners’ contention that they had been
prevented from using the rooms for more than a year before the complaint for BANK OF COMMERCE, Petitioner, vs. PLANTERS DEVELOPMENT BANK and BANGKO
ejectment was filed, they should have demanded specific performance from the SENTRAL NG PILIPINAS, Respondent.
lessor and commenced an action in court. With the execution of the contract, G.R. Nos. 154470-71 September 24, 2012
petitioners were already in a position to exercise their right to the use and
enjoyment of the property according to the terms of the lease contract.56 As BANGKO SENTRAL NG PILIPINAS, Petitioner, vs. PLANTERS DEVELOPMENT BANK,
borne out by the records, the fact is that respondent turned over to petitioners Respondent.
the keys to the leased premises and petitioners, in fact, renovated the rooms. Second Division, G.R. Nos. 154589-90
Thus, they were placed in possession of the premises and they had the right to
the use and enjoyment of the same. They, likewise, had the right to resist any FACTS:
act of intrusion into their peaceful possession of the property, even as against x These are two consolidated petitions for review on certiorari under Rule 45, on
the lessor itself. Yet, they did not lift a finger to protect their right if, indeed, pure questions of law, filed by the petitioners Bank of Commerce (BOC) and the
there was a violation of the contract by the lessor. Bangko Sentral ng Pilipinas (BSP)
x What was, instead, clearly established by the evidence was petitioners’ x Rizal Commercial Banking Corporation (RCBC) was the registered owner of seven
non-payment of rentals because ostensibly they did not know to whom Central Bank (CB) bills with a total face value of P 70 million, issued on January
payment should be made. However, this did not justify their failure to 2, 1994 and would mature on January 2, 1995.2 As evidenced by a "Detached
pay, because if such were the case, they were not without any remedy. Assignment" dated April 8, 1994,3 the RCBC sold these CB bills to the BOC
They should have availed of the provisions of the Civil Code of the x BOC, in turn, sold these CB bills to the PDB
Philippines on the consignation of payment and of the Rules of Court x PDB, in turn, sold to the BOC Treasury Bills worth P 70 million, with maturity
on interpleader. date of June 29, 1994, as evidenced by a Trading Order8 and a Confirmation of
x Article 1256 of the Civil Code provides: Sale
o Article 1256. If the creditor to whom tender of payment has been x However, instead of delivering the Treasury Bills, the PDB delivered the seven CB
made refuses without just cause to accept it, the debtor shall be bills to the BOC, as evidenced by a PDB Security Delivery Receipt, bearing a
released from responsibility by the consignation of the thing or sum "note: ** substitution in lieu of 06-29-94" - referring to the Treasury Bills.10
due. x Nevertheless, the PDB retained possession of the Detached Assignments. It is
o Consignation alone shall produce the same effect in the following basically the nature of this April 15 transaction that the PDB and the BOC cannot
cases: agree on.
ƒ (4) When two or more persons claim the same right x On April 20, 1994, according to the BOC, it "sold back"11 to the PDB three of the
x Consignation shall be made by depositing the things due at the disposal of a seven CB bills. In turn, the PDB transferred these three CB bills to Bancapital
judicial authority, before whom the tender of payment shall be proved in a Development Corporation (Bancap). On April 25, 1994, the BOC bought the three
proper case, and the announcement of the consignation in other cases.57 CB bills from Bancap - so, ultimately, the BOC reacquired these three CB bills
x In the instant case, consignation alone would have produced the effect of x On April 20, 1994, the BOC sold the remaining four (4) CB bills to Capital One
payment of the rentals. The rationale for consignation is to avoid the Equities Corporation13 which transferred them to All-Asia Capital and Trust
performance of an obligation becoming more onerous to the debtor by reason of Corporation (All Asia). On September 30, 1994, All Asia further transferred the
causes not imputable to him.58 Petitioners claim that they made a written tender four CB bills back to the RCBC.
of payment and actually prepared vouchers for their monthly rentals. But that x On November 16, 1994, the RCBC sold back to All Asia one of these 4 CB bills.
was insufficient to constitute a valid tender of payment. Even assuming that it o When the BSP refused to release the amount of this CB bill on
was valid tender, still, it would not constitute payment for want of consignation maturity, the BOC purchased from All Asia this lone CB bill
of the amount. Well-settled is the rule that tender of payment must be
24
RECTO, GAYLE ANGELI M.
2011-0008 | AUSL
Personal Notes on Remedial Law 2 Review (based on the syllabus of Prof. Henedino M. Brondial)

x As the registered owner of the remaining three CB bills, the RCBC sold them to
IVI Capital and Insular Savings Bank. Again, when the BSP refused to release the
amount of this CB bill on maturity, the RCBC paid back its transferees, reacquired ISSUE # 1: Whether the RTC should have taken cognizance of PDB’s petition.
these three CB bills and sold them to the BOC - ultimately, the BOC acquired
these three CB bills. HELD # 1: YES.
x All in all, the BOC acquired the first set of seven CB bills x Based on the unique factual premise of the present case, the RTC acted correctly
x On April 19, 1994, the RCBC, as registered owner, in initially assuming jurisdiction over the PDB’s petition for mandamus,
o (i) sold two CB bills with a total face value of P 20 million to the prohibition and injunction.128 While the RTC agreed (albeit erroneously) with the
PDB and PDB’s view (that the BSP has jurisdiction), it, however, dismissed not only the
o (ii) delivered to the PDB the corresponding Detached Assignment BOC’s/the BSP’s counterclaims but the PDB’s petition itself as well, on the ground
x PDB delivered to Bancap the two CB bills18 (April 19 transaction). In turn, that it lacks jurisdiction.
Bancap sold the CB bills to Al-Amanah Islamic Investment Bank of the o This is plain error.
Philippines, which in turn sold it to the BOC x Not only the parties themselves, but more so the courts, are bound by the rule
x On June 30, 1994, upon learning of the transfers involving the CB bills, the PDB on non-waiver of jurisdiction.129 believes that jurisdiction over the BOC’s
informed20 the Officer-in-Charge of the BSP’s Government Securities counterclaims and the BSP’s counterclaim/crossclaim for interpleader calls for the
Department,21 Lagrimas Nuqui, of the PDB’s claim over these CB bills, based on application of the doctrine of primary jurisdiction, the allowance of the PDB’s
the Detached Assignments in its possession. The PDB requested the BSP22 to petition even becomes imperative because courts may raise the issue of primary
record its claim in the BSP’s books, explaining that its non-possession of the CB jurisdiction sua sponte.130
bills is "on account of imperfect negotiations thereof and/or subsequent setoff or x Of the three possible options available to the RTC, the adoption of either of these
transfer. two would lead the trial court into serious legal error: first, if it granted the PDB’s
o Nuqui Æ denied the request petition, its decision would have to be set aside on appeal because the BSP has
x PDB Æ filed29 with the RTC two separate petitions for Mandamus, Prohibition no jurisdiction as previously discussed; and second when it dismissed the PDB’s
and Injunction with prayer for Preliminary Injunction and Temporary Restraining petitions and the BOC’s counterclaims on the ground that it lacks jurisdiction, the
Order trial court seriously erred because precisely, the resolution of the conflicting
o The PDB essentially claims that in both the April 15 transaction claims over the CB bills falls within its general jurisdiction.
(involving the first set of CB bills) and the April 19 transaction x Without emasculating its jurisdiction, the RTC could have properly dismissed the
(involving the second set of CB bills), there was no intent on its PDB’s petition but on the ground that mandamus does not lie against the BSP;
part to transfer title of the CB bills, as shown by its non-issuance of but even this correct alternative is no longer plausible since the BSP, as a
a detached assignment in favor of the BOC and Bancap, respondent below, already properly brought before the RTC the remaining
respectively. The PDB particularly alleges that it merely conflicting claims over the subject CB bills by way of a counterclaim/crossclaim
"warehoused"31 the first set of CB bills with the BOC, as security for interpleader. Section 1, Rule 62 of the Rules of Court provides when an
collateral. interpleader is proper:
x RTC Æ issued a TRO o SECTION 1. When interpleader proper. - Whenever conflicting
o temporarily enjoined Nuqui and the BSP from paying the face value claims upon the same subject matter are or may be made against a
of the CB bills on maturity person who claims no interest whatever in the subject matter, or an
x PDB Æ filed an Amended Petition, additionally impleading the BOC and All Asia. interest which in whole or in part is not disputed by the
o Cases were then consolidated claimants, he may bring an action against the conflicting claimants
x RTC Ægranted the PDB’s application for a writ of preliminary prohibitory to compel them to interplead and litigate their several claims
injunction among themselves.
x BOC Æ filed their Answer x The remedy of an action of interpleader131 is designed to protect a
o praying for the dismissal of the petition. It argued that the PDB has person against double vexation in respect of a single liability.7 It
no cause of action against it since the PDB is no longer the owner requires, as an indispensable requisite, that conflicting claims upon the
of the CB bills. Contrary to the PDB’s "warehousing theory,"38 the same subject matter are or may be made against the stakeholder (the
BOC asserted that the (i) April 15 transaction and the (ii) April 19 possessor of the subject matter) who claims no interest whatever in the
transaction - covering both sets of CB bills - were valid contracts of subject matter or an interest which in whole or in part is not
sale, followed by a transfer of title (i) to the BOC (in the April 15 disputed by the claimants.132
transaction) upon the PDB’s delivery of the 1st set of CB bills in x Through this remedy, the stakeholder can join all competing claimants in a single
substitution of the Treasury Bills the PDB originally intended to sell, proceeding to determine conflicting claims without exposing the stakeholder to
and (ii) to Bancap (in the April 19 transaction) upon the PDB’s the possibility of having to pay more than once on a single liability.133
delivery of the 2nd set of CB bills to Bancap, likewise by way of x When the court orders that the claimants litigate among themselves, in
substitution. reality a new action arises,134 where the claims of the interpleaders
x BOC and the PDB entered into two separate Escrow Agreements.45 themselves are brought to the fore, the stakeholder as plaintiff is
o The first agreement covered the first set of CB bills, while the relegated merely to the role of initiating the suit. In short, the remedy
second agreement covered the second set of CB bills. The parties of interpleader, when proper, merely provides an avenue for the
agreed to jointly collect from the BSP the maturity proceeds of conflicting claims on the same subject matter to be threshed out in an
these CB bills and to deposit said amount in escrow, "pending final action. Section 2 of Rule 62 provides:
determination by Court judgment, or amicable settlement as to o SEC. 2. Order. - Upon the filing of the complaint, the court shall
who shall be eventually entitled thereto."46 issue an order requiring the conflicting claimants to interplead with
x The BOC and the PDB filed a Joint Motion,47 submitting these Escrow one another. If the interests of justice so require, the court may
Agreements for court approval. direct in such order that the subject matter be paid or delivered to
o RTC Æ gave its approval to the parties’ Joint Motion the court.
x In view of the BOC’s acquisition of all the CB bills, All Asia50 moved to be x This is precisely what the RTC did by granting the BSP’s motion to interplead.
dropped as a respondent (with the PDB’s conformity51), which the RTC The PDB itself "agreed that the various claimants should now interplead." Thus,
granted.52 The RCBC subsequently followed suit. the PDB and the BOC subsequently entered into two separate escrow
o RTC Æ required the parties to manifest their intention regarding agreements, covering the CB bills, and submitted them to the RTC for approval.
the case and to inform the court of any amicable settlement; x In granting the BSP’s motion, the RTC acted on the correct premise that it has
"otherwise, th[e] case shall be dismissed for lack of interest." jurisdiction to resolve the parties’ conflicting claims over the CB bills - consistent
x BOC Æ moved (i) that the case be set for pre-trial and (ii) for further with the rules and the parties’ conduct - and accordingly required the BOC to
proceeding to resolve the remaining issues between the BOC and the PDB, amend its answer and for the PDB to comment thereon. Suddenly, however, the
particularly on "who has a better right over the subject CB bills." PDB made an about-face and questioned the jurisdiction of the RTC. Swayed by
o PDB joined the BOC in its motion the PDB’s argument, the RTC dismissed even the PDB’s petition - which means
x RTC Æ granted the BSP’s motion to interplead and, accordingly, required the that it did not actually compel the BSP to resolve the BOC’s and the PDB’s claims.
BOC to amend its Answer and for the conflicting claimants to comment thereon x Without the motion to interplead and the order granting it, the RTC could only
x BOC Æ filed its Amended Consolidated Answer with Compulsory Counterclaim, dismiss the PDB’s petition since it is the RTC which has jurisdiction to resolve the
reiterating its earlier arguments asserting ownership over the subject CB bills. parties’ conflicting claims - not the BSP. Given that the motion to interplead has
x RTC Æ admitted60 the BOC’s Amended Consolidated Answer with Compulsory been actually filed, the RTC could not have really granted the relief originally
Counterclaims. sought in the PDB’s petition since the RTC’s order granting the BSP’s motion to
x PDB Æmfiled an Omnibus Motion,61 questioning the RTC’s jurisdiction over the interplead - to which the PDB in fact acquiesced into - effectively resulted in the
BOC’s "additional counterclaims." The PDB argues that its petitions pray for the dismissal of the PDB’s petition. This is not altered by the fact that the PDB
BSP (not the RTC) to determine who among the conflicting claimants to the CB additionally prayed in its petition for damages, attorney’s fees and costs of suit
bills stands in the position of the bona fide holder for value. The RTC cannot "against the public respondents" because the grant of the order to interplead
entertain the BOC’s counterclaim, regardless of its nature, because it is the BSP effectively sustained the propriety of the BSP’s resort to this procedural device.
which has jurisdiction to determine who is entitled to receive the proceeds of the x Interpleader
CB bills. o 1. as a special civil action
x RTC Æ dismissed the PDB’s petition, the BOC’s counterclaim and the BSP’s x What is quite unique in this case is that the BSP did not initiate the interpleader
counter-complaint/cross-claim for interpleader, holding that under CB Circular No. suit through an original complaint but through its Answer. This circumstance
28, it has no jurisdiction (i) over the BOC’s "counterclaims" and (ii) to resolve the becomes understandable if it is considered that insofar as the BSP is concerned,
issue of ownership of the CB bills.64 the PDB does not possess any right to have its claim recorded in the BSP’s
x With the denial of their separate motions for Reconsideration,65 the BOC and books; consequently, the PDB cannot properly be considered even as a potential
the BSP separately filed the present petitions for review on certiorari
25
RECTO, GAYLE ANGELI M.
2011-0008 | AUSL
Personal Notes on Remedial Law 2 Review (based on the syllabus of Prof. Henedino M. Brondial)

claimant to the proceeds of the CB bills upon maturity. Thus, the interpleader the applicable prescriptive or reglementary period, especially when the claimant
was only an alternative position, made only in the BSP’s Answer.135 demonstrates a willingness to abide by the rules prescribing such payment.148
x The remedy of interpleader, as a special civil action, is primarily x In the present case, considering the lack of a clear guideline on the payment of
governed by the specific provisions in Rule 62 of the Rules of Court and docket fee by the claimants in an interpleader suit, compounded by the unusual
secondarily by the provisions applicable to ordinary civil actions.136 manner in which the interpleader suit was initiated and the circumstances
Indeed, Rule 62 does not expressly authorize the filing of a complaint-in- surrounding it, we surely cannot deduce from the BOC’s mere failure to specify in
interpleader as part of, although separate and independent from, the answer. its prayer the total amount of the CB bills it lays claim to (or the value of the
Similarly, Section 5, Rule 6, in relation to Section 1, Rule 9 of the Rules of subjects of the sales in the April 15 and April 19 transactions, in its alternative
Court137 does not include a complaint-in-interpleader as a claim,138 a form of prayer) an intention to defraud the government that would warrant the dismissal
defense,139 or as an objection that a defendant may be allowed to put up in his of its claim.149
answer or in a motion to dismiss. This does not mean, however, that the BSP’s x At any rate, regardless of the nature of the BOC’s "counterclaims," for
"counter-complaint/cross-claim for interpleader" runs counter to general purposes of payment of filing fees, both the BOC and the PDB, properly
procedures. as defendants-in-interpleader, must be assessed the payment of the
x Apart from a pleading,140 the rules141 allow a party to seek an affirmative relief correct docket fee arising from their respective claims. The seminal
from the court through the procedural device of a motion. While captioned case of Sun Insurance Office, Ltd. v. Judge Asuncion150 provides us
"Answer with counter complaint/cross-claim for interpleader," the RTC guidance in the payment of docket fees, to wit:
understood this as in the nature of a motion,142 seeking relief which essentially o 1. x x x Where the filing of the initiatory pleading is not
consists in an order for the conflicting claimants to litigate with each other so accompanied by payment of the docket fee, the court may
that "payment is made to the rightful or legitimate owner"143 of the subject CB allow payment of the fee within a reasonable time but in
bills. no case beyond the applicable prescriptive or reglementary
period
x The rules define a "civil action" as "one by which a party sues another for the o 2. The same rule applies to permissive counterclaims,
enforcement or protection of a right, or the prevention or redress of a wrong." third-party claims and similar pleadings, which shall not be
Interpleader may be considered as a stakeholder’s remedy to prevent a wrong, considered filed until and unless the filing fee prescribed
that is, from making payment to one not entitled to it, thereby rendering itself therefor is paid. The court may also allow payment of said
vulnerable to lawsuit/s from those legally entitled to payment. fee within a reasonable time but also in no case beyond its
x Interpleader is a civil action made special by the existence of particular rules to applicable prescriptive or reglementary period.
govern the uniqueness of its application and operation. Under Section 2, Rule 6 [underscoring ours]
of the Rules of Court, governing ordinary civil actions, a party’s claim is asserted x This must be the rule considering that Section 7, Rule 62 of which reads:
"in a complaint, counterclaim, cross-claim, third (fourth, etc.)-party complaint, or o SEC. 7. Docket and other lawful fees, costs and litigation expenses
complaint-in-intervention." In an interpleader suit, however, a claim is not as liens. - The docket and other lawful fees paid by the party who
required to be contained in any of these pleadings but in the answer-(of the filed a complaint under this Rule, as well as the costs and litigation
conflicting claimants)-in-interpleader. This claim is different from the counter- expenses, shall constitute a lien or charge upon the subject matter
claim (or cross-claim, third party-complaint) which is separately allowed under of the action, unless the court shall order otherwise.
Section 5, par. 2 of Rule 62. x only pertain to the docket and lawful fees to be paid by the one who
initiated the interpleader suit, and who, under the Rules, actually
ISSUE # 2: Whether BOC’s failure to pay the appropriate docket fees prevents the RTC from "claims no interest whatever in the subject matter." By constituting a
acquiring jurisdiction over the BOC’s "counterclaims." lien on the subject matter of the action, Section 7 in effect only aims to
actually compensate the complainant-in-interpleader, who happens to
HELD: # 2: NO. be the stakeholder unfortunate enough to get caught in a legal
o 2. the payment of docket fees covering BOC’s counterclaim crossfire between two or more conflicting claimants, for the faultless
x The PDB argues that, even assuming that the RTC has jurisdiction over the issue trouble it found itself into. Since the defendants-in-interpleader are
of ownership of the CB bills, the BOC’s failure to pay the appropriate docket fees actually the ones who make a claim - only that it was extraordinarily
prevents the RTC from acquiring jurisdiction over the BOC’s "counterclaims." done through the procedural device of interpleader - then to them
o We disagree with the PDB. devolves the duty to pay the docket fees prescribed under Rule 141 of
x To reiterate and recall, the order granting the "PDB’s motion to interplead," the Rules of Court, as amended.151
already resulted in the dismissal of the PDB’s petition. The same order x The importance of paying the correct amount of docket fee cannot be
required the BOC to amend its answer and for the conflicting claimants overemphasized:
to comment, presumably to conform to the nature of an answer-in o The matter of payment of docket fees is not a mere triviality. These
interpleader. Perhaps, by reason of the BOC’s denomination of its claim fees are necessary to defray court expenses in the handling of
as a "compulsory counterclaim" and the PDB’s failure to fully cases. Consequently, in order to avoid tremendous losses to the
appreciate the RTC’s order granting the "BSP’s motion for interpleader" judiciary, and to the government as well, the payment of docket
(with the PDB’s conformity), the PDB mistakenly treated the BOC’s fees cannot be made dependent on the outcome of the case,
claim as a "permissive counterclaim" which necessitates the payment except when the claimant is a pauper-litigant
of docket fees.
x As the preceding discussions would show, however, the BOC’s "claim" - i.e., its
assertion of ownership over the CB bills - is in reality just that, a "claim" against B. Declaratory Relief [RULE 63]
the stakeholder and not as a "counterclaim,"144 whether compulsory145 or
permissive. It is only the BOC’s alternative prayer (for the PDB to deliver to the 1. Nature; Kinds
BOC, as the buyer in the April 15 transaction and the ultimate successor-in-
interest of the buyer in the April 19 transaction, either the original subjects of the
sales or the value thereof plus whatever income that may have been earned Section 1. Who may file petition. — Any person interested under a deed, will, contract or
pendente lite) and its prayer for damages that are obviously compulsory other written instrument, or whose rights are affected by a statute, executive order or
counterclaims against the PDB and, therefore, does not require payment of regulation, ordinance, or any other governmental regulation may, before breach or violation
docket fees.146 thereof bring an action in the appropriate Regional Trial Court to determine any question of
construction or validity arising, and for a declaration of his rights or duties, thereunder. (Bar
x The PDB takes a contrary position through its insistence that a compulsory
Matter No. 803, 17 February 1998)
counterclaim should be one where the presence of third parties, of whom the
court cannot acquire jurisdiction, is not required. It reasons out that since the
RCBC and All Asia (the intervening holders of the CB bills) have already been An action for the reformation of an instrument, to quiet title to real property or remove clouds
dropped from the case, then the BOC’s counterclaim must only be permissive in therefrom, or to consolidate ownership under Article 1607 of the Civil Code, may be brought
nature and the BOC should have paid the correct docket fees. under this Rule.
x We see no reason to belabor this claim. Even if we gloss over the PDB’s own
conformity to the dropping of these entities as parties, the BOC correctly argues 2. Parties
that a remedy is provided under the Rules. Section 12, Rule 6 of the Rules of
Court reads: Section 2. Parties. — All persons who have or claim any interest which would be affected
o SEC. 12. Bringing new parties. - When the presence of parties by the declaration shall be made parties; and no declaration shall, except as otherwise
other than those to the original action is required for the granting of provided in these Rules, prejudice the rights of persons not parties to the action.
complete relief in the determination of a counterclaim or cross-
claim, the court shall order them to be brought in as defendants, if 3. Conversion into ordinary action
jurisdiction over them can be obtained.
x Even then, the strict characterization of the BOC’s counterclaim is no longer
material in disposing of the PDB’s argument based on non-payment of docket Section 6. Conversion into ordinary action. — If before the final termination of the
fees. case, a breach or violation of an instrument or a statute, executive order or regulation,
x When an action is filed in court, the complaint must be accompanied by the ordinance, or any other governmental regulation should take place, the action may thereupon
payment of the requisite docket and filing fees by the party seeking affirmative be converted into an ordinary action, and the parties shall be allowed to file such pleadings as
relief from the court. It is the filing of the complaint or appropriate initiatory may be necessary or proper.
pleading, accompanied by the payment of the prescribed docket fee, that vests a
trial court with jurisdiction over the claim or the nature of the action.147 EUFEMIA ALMEDA and ROMEL ALMEDA, petitioners, vs. BATHALA MARKETING
INDUSTRIES, INC., respondent.
However, the non-payment of the docket fee at the time of filing does not
THIRD DIVISION, G.R. No. 150806 January 28, 2008
automatically cause the dismissal of the case, so long as the fee is paid within
26
RECTO, GAYLE ANGELI M.
2011-0008 | AUSL
Personal Notes on Remedial Law 2 Review (based on the syllabus of Prof. Henedino M. Brondial)

suit. There is no showing that respondent committed an act constituting a breach


FACTS: of the subject contract of lease. Thus, respondent is not barred from instituting
x Bathala Marketing Industries, Inc., as lessee, represented by its president Ramon before the trial court the petition for declaratory relief.
H. Garcia, renewed its Contract of Lease4 with Ponciano L. Almeda (Ponciano), x Petitioners claim that the instant petition is not proper because a separate action
as lessor, husband of petitioner Eufemia and father of petitioner Romel Almeda for rescission, ejectment and damages had been commenced before another
o Under the said contract, Ponciano agreed to lease a portion of the court; thus, the construction of the subject contractual provisions should be
Almeda Compound, located at 2208 Pasong Tamo Street, Makati ventilated in the same forum.
City, consisting of 7,348.25 square meters, for a monthly rental of o We are not convinced.
P1,107,348.69, for a term of four (4) years from May 1, 1997 x It is true that in Panganiban v. Pilipinas Shell Petroleum Corporation17 we held
unless sooner terminated as provided in the contract that the petition for declaratory relief should be dismissed in view of the
x During the effectivity of the contract, Ponciano died. pendency of a separate action for unlawful detainer. However, we cannot apply
x Almedas Æ then wrote to Bathala that the former shall assess and collect Value the same ruling to the instant case. In Panganiban, the unlawful detainer case
Added Tax (VAT) on its monthly rentals. had already been resolved by the trial court before the dismissal of the
o Bathala Æ contended that VAT may not be imposed as the rentals declaratory relief case; and it was petitioner in that case who insisted that the
fixed in the contract of lease were supposed to include the VAT action for declaratory relief be preferred over the action for unlawful detainer.
therein, considering that their contract was executed on May 1, Conversely, in the case at bench, the trial court had not yet resolved the
1997 when the VAT law had long been in effect rescission/ejectment case during the pendency of the declaratory relief petition.
x Bathala Æ then received another letter from Almedas informing the former that In fact, the trial court, where the rescission case was on appeal, itself initiated
its monthly rental should be increased by 73% pursuant to condition No. 7 of the the suspension of the proceedings pending the resolution of the action for
contract and Article 1250 of the Civil Code. Respondent opposed petitioners' declaratory relief.
demand and insisted that there was no extraordinary inflation to warrant the x We are not unmindful of the doctrine enunciated in Teodoro, Jr. v. Mirasol18
application of Article 1250 in light of the pronouncement of this Court in various where the declaratory relief action was dismissed because the issue therein could
cases be threshed out in the unlawful detainer suit. Yet, again, in that case, there was
x Bathala Æ refused to pay the VAT and adjusted rentals as demanded by already a breach of contract at the time of the filing of the declaratory relief
petitioners but continued to pay the stipulated amount set forth in their contract. petition. This dissimilar factual milieu proscribes the Court from applying Teodoro
x Bathala Æ instituted an action for declaratory relief (DR) for purposes of to the instant case.
determining the correct interpretation of condition Nos. 6 and 7 of the lease x Given all these attendant circumstances, the Court is disposed to entertain the
contract to prevent damage and prejudice before the RTC Makati instant declaratory relief action instead of dismissing it, notwithstanding the
x Almedas Æ in turn filed an action for ejectment, rescission and damages against pendency of the ejectment/rescission case before the trial court. The resolution of
respondent for failure of the latter to vacate the premises after the demand the present petition would write finis to the parties' dispute, as it would settle
made by the former. once and for all the question of the proper interpretation of the two contractual
o Before Bathala can file its Answer, Almedas filed a Notice of stipulations subject of this controversy.
Dismissal
o Almedas then subsequently refiled the same in MeTC Makati
x Almedas Æ filed an MD in the DR case for being an improper remedy considering REPUBLIC OF THE PHILIPPINES, Petitioner, vs. CIPRIANO ORBECIDO III,
that respondent was already in breach of the obligation and that the case would Respondent.
not end the litigation and settle the rights of the parties. FIRST DIVISION, G.R. No. 154380 October 5, 2005
x RTC Æ denied MD; ruled in favor of Bathala
o denied petitioners their right to pass on to respondent the burden FACTS:
of paying the VAT since it was not a new tax that would call for the x Cipriano Orbecido III married Lady Myros M. Villanueva at the United Church of
application of the sixth clause of the contract. The court, likewise, Christ in the Philippines in Lam-an, Ozamis City.
denied their right to collect the demanded increase in rental, there o Their marriage was blessed with a son and a daughter, Kristoffer
being no extraordinary inflation or devaluation as provided for in Simbortriz V. Orbecido and Lady Kimberly V. Orbecido.
the seventh clause of the contract. Because of the payment made x Cipriano’s wife left for the United States bringing along their son Kristoffer.
by respondent of the rental adjustment demanded by petitioners, x A few years later, Cipriano discovered that his wife had been naturalized as an
the court ordered the restitution by the latter to the former of the American citizen.
amounts paid, notwithstanding the well-established rule that in an x Cipriano learned from his son that his wife had obtained a divorce decree and
action for declaratory relief, other than a declaration of rights and then married a certain Innocent Stanley
obligations, affirmative reliefs are not sought by or awarded to the x Cipriano Æ thereafter filed with the trial court a petition for authority to remarry
parties. invoking Paragraph 2 of Article 26 of the Family Code.
x Almedas Æ appealed to CA x Republic did not file its Opposition
x CA Æ affirmed with modification the RTC decision x RTC Æ granted Cipriano’s petition
x Almedas Æ filed a Rule 45 before the SC x Republic Æ filed an MR
o The appellate court agreed with the conclusions of law and the o RTC Æ denied
application of the decisional rules on the matter made by the RTC. x Republic Æ filed a Rule 45 before the SC
However, it found that the trial court exceeded its jurisdiction in o The OSG contends that Paragraph 2 of Article 26 of the Family
granting affirmative relief to the respondent, particularly the Code is not applicable to the instant case because it only applies to
restitution of its excess payment. a valid mixed marriage; that is, a marriage celebrated between a
Filipino citizen and an alien. The proper remedy, according to the
ISSUE: Whether Bathala correctly resorted to a petition for declaratory relief considering the OSG, is to file a petition for annulment or for legal separation.5
allegation of the Almedas that the former has already breached the contract of lease. Furthermore, the OSG argues there is no law that governs
respondent’s situation. The OSG posits that this is a matter of
HELD: YES. legislation and not of judicial determination
x Declaratory relief is defined as an action by any person interested in a x Cipriano Æ For his part, respondent admits that Article 26 is not directly
deed, will, contract or other written instrument, executive order or applicable to his case but insists that when his naturalized alien wife obtained a
resolution, to determine any question of construction or validity arising divorce decree which capacitated her to remarry, he is likewise capacitated by
from the instrument, executive order or regulation, or statute, and for operation of law pursuant to Section 12, Article II of the Constitution.
a declaration of his rights and duties thereunder. The only issue that
may be raised in such a petition is the question of construction or ISSUE: What is the nature of Cipriano’s petition before the RTC?
validity of provisions in an instrument or statute. Corollary is the
general rule that such an action must be justified, as no other adequate HELD: DECLARATORY RELIEF.
relief or remedy is available under the circumstances. 15 x At the outset, we note that the petition for authority to remarry filed before the
x Decisional law enumerates the requisites of an action for declaratory trial court actually constituted a petition for declaratory relief. In this connection,
relief, as follows: 1) the subject matter of the controversy must be a Section 1, Rule 63 of the Rules of Court provides:
deed, will, contract or other written instrument, statute, executive o Section 1. Who may file petition—Any person interested under a
order or regulation, or ordinance; 2) the terms of said documents and deed, will, contract or other written instrument, or whose rights are
the validity thereof are doubtful and require judicial construction; 3) affected by a statute, executive order or regulation, ordinance, or
there must have been no breach of the documents in question; 4) there other governmental regulation may, before breach or violation
must be an actual justiciable controversy or the "ripening seeds" of one thereof, bring an action in the appropriate Regional Trial Court to
between persons whose interests are adverse; 5) the issue must be determine any question of construction or validity arising, and for a
ripe for judicial determination; and 6) adequate relief is not available declaration of his rights or duties, thereunder.
through other means or other forms of action or proceeding.16 x The requisites of a petition for declaratory relief are: (1) there must be
x It is beyond cavil that the foregoing requisites are present in the instant case, a justiciable controversy; (2) the controversy must be between persons
except that petitioners insist that respondent was already in breach of the whose interests are adverse; (3) that the party seeking the relief has a
contract when the petition was filed. legal interest in the controversy; and (4) that the issue is ripe for
o We do not agree. judicial determination.8
x After petitioners demanded payment of adjusted rentals and in the months that x This case concerns the applicability of Paragraph 2 of Article 26 to a marriage
followed, respondent complied with the terms and conditions set forth in their between two Filipino citizens where one later acquired alien citizenship, obtained
contract of lease by paying the rentals stipulated therein. Respondent religiously a divorce decree, and remarried while in the U.S.A. The interests of the parties
fulfilled its obligations to petitioners even during the pendency of the present are also adverse, as petitioner representing the State asserts its duty to protect
27
RECTO, GAYLE ANGELI M.
2011-0008 | AUSL
Personal Notes on Remedial Law 2 Review (based on the syllabus of Prof. Henedino M. Brondial)

the institution of marriage while respondent, a private citizen, insists on a must be read together with those of the Judiciary Reorganization Act of 1980, as
declaration of his capacity to remarry. Respondent, praying for relief, has legal amended.
interest in the controversy. The issue raised is also ripe for judicial determination x It is important to note that Section 1, Rule 63 of the Rules of Court
inasmuch as when respondent remarries, litigation ensues and puts into question does not categorically require that an action to quiet title be filed
the validity of his second marriage. before the RTC. It repeatedly uses the word "may" - that an action for
quieting of title "may be brought under [the] Rule" on petitions for
declaratory relief, and a person desiring to file a petition for
CARMEN DANAO MALANA, MARIA DANAO ACORDA, EVELYN DANAO, FERMINA DANAO, declaratory relief "may x x x bring an action in the appropriate
LETICIA DANAO and LEONORA DANAO, the last two are represented herein by their Attorney- Regional Trial Court." The use of the word "may" in a statute denotes
in-Fact, MARIA DANAO ACORDA, Petitioners, vs. BENIGNO TAPPA, JERRY REYNA, that the provision is merely permissive and indicates a mere possibility,
SATURNINO CAMBRI and SPOUSES FRANCISCO AND MARIA LIGUTAN, Respondents. an opportunity or an option.23
THIRD DIVISION, G.R. No. 181303 September 17, 2009 x In contrast, the mandatory provision of the Judiciary Reorganization Act of 1980,
as amended, uses the word "shall" and explicitly requires the MTC to exercise
FACTS: exclusive original jurisdiction over all civil actions which involve title to or
x During the lifetime of Anastacio Danao, he had allowed Consuelo Pauig possession of real property where the assessed value does not exceed
(Consuelo), who was married to Joaquin Boncad, to build on and occupy the P20,000.00, thus:
southern portion of the subject property. Anastacio and Consuelo agreed that the o Section 33. Jurisdiction of Metropolitan Trial Courts, Municipal Trial
latter would vacate the said land at any time that Anastacio and his heirs might Courts and Municipal Circuit Trial Courts in Civil Cases.—
need it. Metropolitan Trial Courts, Municipal Trial Courts and Municipal
x Danao died. Circuit Trial Courts shall exercise:
x Malana are the Owners of a parcel of land (TCT T-1279373 situated in ƒ (3) Exclusive original jurisdiction in all civil actions
Tuguegarao City, Cagayan). They inherited the subject property from Danao, which involve title to, possession of, real property, or
who died intestate. any interest therein where the assessed value of the
x MalanaÆ Demand against Consuelo’s family members (Tappa, etc.) to vacate property or interest therein does not exceed Twenty
the same, as they need the land. Tappa refused. thousand pesos (P20,000.00) or, in civil actions in
x MalanaÆ referred land dispute to the Lupong Tagapamayapa of Barangay Metro Manila, where such assessed value does not
Annafunan West for conciliation. Tappa asserted they owned land. exceeds Fifty thousand pesos (P50,000.00) exclusive
x Malana Æ Filed Complaint for Reivindicacion, Quieting of Title, and Damages of interest, damages of whatever kind, attorney’s
with RTC (Branch 3, Tuguegarao City) fees, litigation expenses and costs: x x x (Emphasis
x RTC Æ Dismissed Complaint ours.)
o on the ground of lack of jurisdiction (BP 129, as amended by RA x As found by the RTC, the assessed value of the subject property as stated in Tax
7691), where RTC with jurisdiction over real actions, where the Declaration No. 02-48386 is only P410.00; therefore, petitioners’ Complaint
assessed value of the property involved exceeds P20,000.00. It involving title to and possession of the said property is within the exclusive
found that the subject property had a value of less than original jurisdiction of the MTC, not the RTC.
P20,000.00. x Furthermore, an action for declaratory relief presupposes that there
x Malana Æ Filed a Motion for Reconsideration has been no actual breach of the instruments involved or of rights
o arguing that their principal cause of action was for quieting of title; arising thereunder.24 Since the purpose of an action for declaratory
the accion reivindicacion was included merely to enable them to relief is to secure an authoritative statement of the rights and
seek complete relief. obligations of the parties under a statute, deed, or contract for their
x RTC Æ Denied Motion for Reconsideration. guidance in the enforcement thereof, or compliance therewith, and not
x Malana Æ Filed another pleading, simply designated as Motion, for RTC to set to settle issues arising from an alleged breach thereof, it may be
aside dismissal. entertained only before the breach or violation of the statute, deed, or
o RTC ÆDenied Motion. contract to which it refers. A petition for declaratory relief gives a
x Malana Æ Filed Petition for Certiorari under Rule 65 of the Rules of Court, with practical remedy for ending controversies that have not reached the
the Supreme Court. state where another relief is immediately available; and supplies the
need for a form of action that will set controversies at rest before they
ISSUE: Whether an action to quiet title to real property or remove clouds therefrom Sis the lead to a repudiation of obligations, an invasion of rights, and a
correct remedy considering that the Malana has already demanded but the Tappa et. al. commission of wrongs.25
refused to vacate the property. x Where the law or contract has already been contravened prior to the filing of an
action for declaratory relief, the courts can no longer assume jurisdiction over the
HELD: NO. action. In other words, a court has no more jurisdiction over an action for
x An action for declaratory relief should be filed by a person interested declaratory relief if its subject has already been infringed or transgressed before
under a deed, a will, a contract or other written instrument, and whose the institution of the action.26
rights are affected by a statute, an executive order, a regulation or an x In the present case, petitioners’ Complaint for quieting of title was filed after
ordinance. The relief sought under this remedy includes the petitioners already demanded and respondents refused to vacate the subject
interpretation and determination of the validity of the written property. In fact, said Complaint was filed only subsequent to the latter’s express
instrument and the judicial declaration of the parties’ rights or duties claim of ownership over the subject property before the Lupong Tagapamayapa, in
thereunder.21 direct challenge to petitioners’ title.
x Petitions for declaratory relief are governed by Rule 63 of the Rules of Court. The x Since petitioners averred in the Complaint that they had already been deprived
RTC correctly made a distinction between the first and the second paragraphs of of the possession of their property, the proper remedy for them is the filing of an
Section 1, Rule 63 of the Rules of Court. accion publiciana or an accion reivindicatoria, not a case for declaratory relief. An
x The first paragraph of Section 1, Rule 63 of the Rules of Court, describes the accion publiciana is a suit for the recovery of possession, filed one year after the
general circumstances in which a person may file a petition for declaratory relief, occurrence of the cause of action or from the unlawful withholding of possession
to wit: of the realty. An accion reivindicatoria is a suit that has for its object one’s
o Any person interested under a deed, will, contract or other written recovery of possession over the real property as owner.
instrument, or whose rights are affected by a statute, executive x Petitioners’ Complaint contained sufficient allegations for an accion
order or regulation, ordinance, or any other governmental reivindicatoria. Jurisdiction over such an action would depend on the value of the
regulation may, before breach or violation thereof, bring an action in property involved. Given that the subject property herein is valued only at
the appropriate Regional Trial Court to determine any question of P410.00, then the MTC, not the RTC, has jurisdiction over an action to recover
construction or validity arising, and for a declaration of his rights or the same. The RTC, therefore, did not commit grave abuse of discretion in
duties, thereunder. (Emphasis ours.) dismissing, without prejudice, petitioners’ Complaint in Civil Case No. 6868 for
x As the afore-quoted provision states, a petition for declaratory relief under the lack of jurisdiction.
first paragraph of Section 1, Rule 63 may be brought before the appropriate x As for the RTC dismissing petitioners’ Complaint motu proprio, the following
RTC. pronouncements of the Court in Laresma v. Abellana28 proves instructive:
x Section 1, Rule 63 of the Rules of Court further provides in its second paragraph o It is axiomatic that the nature of an action and the jurisdiction of a
that: tribunal are determined by the material allegations of the complaint
o An action for the reformation of an instrument, to quiet and the law at the time the action was commenced. Jurisdiction of
title to real property or remove clouds therefrom, or to the tribunal over the subject matter or nature of an action is
consolidate ownership under Article 1607 of the Civil Code, conferred only by law and not by the consent or waiver upon a
may be brought under this Rule. (Emphasis ours.) court which, otherwise, would have no jurisdiction over the subject
x The second paragraph of Section 1, Rule 63 of the Rules of Court specifically matter or nature of an action. Lack of jurisdiction of the court over
refers to (1) an action for the reformation of an instrument, recognized under an action or the subject matter of an action cannot be cured by the
Articles 1359 to 1369 of the Civil Code; (2) an action to quiet title, authorized by silence, acquiescence, or even by express consent of the parties. If
Articles 476 to 481 of the Civil Code; and (3) an action to consolidate ownership the court has no jurisdiction over the nature of an action, it may
required by Article 1607 of the Civil Code in a sale with a right to repurchase. dismiss the same ex mero motu or motu proprio. x x x. (Emphasis
These three remedies are considered similar to declaratory relief because they also supplied.
result in the adjudication of the legal rights of the litigants, often without the need x Since the RTC, in dismissing petitioners’ Complaint, acted in complete accord
of execution to carry the judgment into effect.22 with law and jurisprudence, it cannot be said to have done so with grave abuse
x To determine which court has jurisdiction over the actions identified in the of discretion amounting to lack or excess of jurisdiction. An act of a court or
second paragraph of Section 1, Rule 63 of the Rules of Court, said provision tribunal may only be considered to have been committed in grave abuse of
28
RECTO, GAYLE ANGELI M.
2011-0008 | AUSL
Personal Notes on Remedial Law 2 Review (based on the syllabus of Prof. Henedino M. Brondial)

discretion when the same was performed in a capricious or whimsical exercise of x Muertegui Æ Juanito, through his attorney-in-fact Domingo Jr., filed a Civil Case
judgment, which is equivalent to lack of jurisdiction. The abuse of discretion for quieting of title and preliminary injunction, against Atty. Sabitsana and his
must be so patent and gross as to amount to an evasion of a positive duty or to wife, Rosario. (Regional Trial Court [RTC] of Naval, Biliran)
a virtual refusal to perform a duty enjoined by law or to act at all in x Sabitsana Æ Filed Answer with Counterclaim, asserting that the sale to Juanito is
contemplation of law, as where the power is exercised in an arbitrary and null and void and that the Complaint is barred by prescription and laches.
despotic manner by reason of passion or personal hostility.29 No such x RTC Æ Decided in favor of the Muerteguis, declaring that their deed of sale is
circumstances exist herein as to justify the issuance of a writ of certiorari. valid and preferred.
x Sabitsana Æ Filed a Motion for Reconsideration
o RTC Æ Denied the Motion for Reconsideration
FRANCISCO I. CHAVEZ, Petitioner, vs. JUDICIAL AND BAR COUNCIL, SEN. FRANCIS x Sabitsana Æ Appealed to CA
JOSEPH G. ESCUDERO and REP. NIEL C. TUPAS, JR., Respondents. o CA → Denied the appeal and affirmed the RTC decision in toto.
EN BANC, G.R. No. 202242 April 16, 2013 x Sabitsana Æ Filed a Motion for Reconsideration
o CA Æ Denied the Motion for Reconsideration
x Sabitsana Æ Filed Rule 45 before the SC
FACTS:
x Chief Justice Renato C. Corona Æ Departed on 29 May 2012. ISSUE: Whether the Regional Trial Court has jurisdiction over the suit for quieting of title.
x Former Solicitor General Francisco I. Chavez Æ Nominated in the Judicial and
Bar Council as CJ Corona’s potential successor Held: YES.
o Failed to manifest his acceptance of his recommendation to the x On the question of jurisdiction, it is clear under the Rules that an action for
position of Chief Justice. quieting of title may be instituted in the RTCs, regardless of the assessed value
x Chavez Æ Filed petition (certiorari, 65) with the Supreme Court. of the real property in dispute. Under Rule 63 of the Rules of Court,29 an action
o In his initiatory pleading, petitioner asked the Court to determine 1] to quiet title to real property or remove clouds therefrom may be brought in the
whether the first paragraph of Section 8, Article VIII of the 1987 appropriate RTC.
Constitution allows more than one (1) member of Congress to sit in x It must be remembered that the suit for quieting of title was prompted by
the JBC; and 2] if the practice of having two (2) representatives petitioners’ August 24, 1998 letter-opposition to respondent’s application for
from each House of Congress with one (1) vote each is sanctioned registration. Thus, in order to prevent30 a cloud from being cast upon his
by the Constitution. application for a title, respondent filed Civil Case No. B-1097 to obtain a
x JBC Æ Filed comment. declaration of his rights. In this sense, the action is one for declaratory relief,
x Congress Æ Sen. Francis Joseph G. Escudero and Rep. Niel C. Tupas Jr. filed which properly falls within the jurisdiction of the RTC pursuant to Rule 63 of the
comments through Office of the Solicitor General (OSG). Rules.
x SC Æ granted petition
o WHEREFORE, the petition is GRANTED. The current numerical
composition of the Judicial and Bar Council is declared REPUBLIC OF THE PHILIPPINES, represented by THE EXECUTIVE SECRETARY, THE
UNCONSTITUTIONAL. The Judicial and Bar Council is hereby SECRETARY OF JUSTICE, THE SECRETARY OF FOREIGN AFFAIRS, THE SECRETARY OF
enjoined to reconstitute itself so that only one (1) member of NATIONALDEFENSE, THE SECRETARY OF THE INTERIOR AND LOCAL GOVERNMENT THE
Congress will sit as a representative in its proceedings, in SECRETARY OF FINANCE, THE NATIONAL SECURITY ADVISER, THE SECRETARY OF BUDGET
accordance with Section 8(1), Article VIII of the 1987 Constitution. AND MANAGEMENT THE TREASURER OF THE PHILIPPINES, THE CHIEF OF STAFF OF THE
x Respondents Æ filed an MR ARMED FORCES OF THE PHILIPPINES, and THE CHIEFOF THE PHILIPPINE NATIONAL
POLICE, Petitioners, vs. HERMINIO HARRY ROQUE, MORO CHRISTIAN PEOPLE'S
ISSUE # 1: Whether the petition is one for certiorari under Rule 65. ALLIANCE, FR. JOE DIZON, RODINIE SORIANO, STEPHANIE ABIERA, MARIA LOURDES
ALCAIN, VOLTAIRE ALFEREZ, CZARINA MAYALTEZ, SHERYL BALOT, RENIZZA BATACAN,
HELD # 1: NO. EDAN MARRI CAÑETE, LEANA CARAMOAN, ALDWIN CAMANCE, RENE DELORINO, PAULYN
x Pursuant to the rule that the nature of an action is determined by the allegations MAY DUMAN, RODRIGO FAJARDO III, ANNAMARIE GO, ANNA ARMINDA JIMENEZ, MARY ANN
therein and the character of the relief sought, the Court views the petition as LEE,LUISA MANALAYSAY, MIGUEL MUSNGI, MICHAEL OCAMPO, NORMAN ROLAND OCANA
essentially an action for declaratory relief under Rule 63 of the 1997 Rules of III, WILLIAM RAGAMAT, MARICAR RAMOS, CHERRY LOU REYES, MELISSA ANN SICAT,
Civil Procedure. The Constitution as the subject matter, and the validity and CRISTINE MAE TABING, VANESSA TORNO, and HON. JUDGE ELEUTERIO L. BATHAN, as
construction of Section 8 (1), Article VIII as the issue raised, the petition should Presiding Judge of Regional Trial Court, Quezon City, Branch 92, Respondents.
properly be considered as that which would result in the adjudication of rights EN BANC, G.R. No. 204603 September 24, 2013
sans the execution process because the only relief to be granted is the very
declaration of the rights under the document sought to be construed. It being
so, the original jurisdiction over the petition lies with the appropriate Regional FACTS:
Trial Court (RTC). Notwithstanding the fact that only questions of law are raised x Roque Æ Filed a Petition for declaratory relief before the RTC (Regional Trial
in the petition, an action for declaratory relief is not among those within the Court of Quezon City, Branch 92), assailing the constitutionality of the following
original jurisdiction of this Court as provided in Section 5, Article VIII of the sections of RA 9372 ("An Act to Secure the State and Protect our People from
Constitution. Terrorism," otherwise known as the Human Security Act of 2007):
o (a) Section 3, for being void for vagueness;
ISSUE # 2: Whether the Supreme Court correctly took cognizance of the petition considering o (b) Section 7, for violating the right to privacy of communication
that it is for declaratory relief. and due process and the privileged nature of priest-penitent
relationships;
HELD # 2: YES. o (c )Section 18, for violating due process, the prohibition against ex
x At any rate, due to its serious implications, not only to government processes post facto laws or bills of attainder, the Universal Declaration of
involved but also to the sanctity of the Constitution, the Court deems it more Human Rights, and the International Covenant on Civil and Political
prudent to take cognizance of it. After all, the petition is also for prohibition Rights, as well as for contradicting Article 125 of the Revised Penal
under Rule 65 seeking to enjoin Congress from sending two (2) representatives Code, as amended;
with one (1) full vote each to the JBC. o (d) Section 26, for violating the right to travel;15 and
o (e) Section 27, for violating the prohibition against unreasonable
searches and seizures.
SPOUSES CLEMENCIO C. SABITSANA, JR. and MA. ROSARIO M. SABITSANA, Petitioners, x Republic Æ Moved to suspend the proceedings, averring that certain petitions
vs. JUANITO F. MUERTEGUI, represented by his Attorney-in-Fact DOMINGO A. (SC petitions) raising the issue of RA 9372’s constitutionality have been lodged
MUERTEGUI, JR., Respondent. before the Court.
SECOND DIVISION, G.R. No. 181359 August 5, 2013 o RTC Æ Granted motion.
x SC Æ promulgated its Decision in the Southern Hemisphere cases and thereby
FACTS: dismissed the SC petitions.
x Garcia Æ Executed an unnotarized Deed of Sale in favor of Juanito Muertegui x Republic Æ Filed motion to dismiss, contending that Roque, et al. failed to satisfy
(Juanito) over a 7,500-square meter parcel of unregistered land (the lot) located the requisites for declaratory relief. Likewise, they averred that the
in Dalutan Island, Talahid, Almeira, Biliran, Leyte del Norte covered by Tax constitutionality of RA 9372 had already been upheld by the Court in the
Declaration (TD) No. 1996 issued in 1985 in Garcia’s name. Southern Hemisphere cases.
x Muertegui Æ Juanito’s father (Domingo Sr.) and brother (Domingo Jr.) took x Roque Æ Filed Comment/Opposition.
actual possession of the lot and planted thereon coconut and ipil-ipil trees. They x RTC Æ Denied motion to dismiss
also paid the real property taxes on the lot for the years 1980 up to 1998. o finding that the Court did not pass upon the constitutionality of RA
x GarciaÆ Sold the lot to the Muertegui family lawyer, Atty. Clemencio C. 9372 and that Roque’s petition for declaratory relief was properly
Sabitsana, Jr. (Atty. Sabitsana), through a notarized deed of absolute sale. filed.
x Sabitsana Æ Sale was registered with the RD. TD No. 1996 was cancelled and a x Republic Æ Moved for reconsideration
new one, TD No. 5327, was issued in Atty. Sabitsana’s name. Although Domingo x RTC Æ Denied the motion.
Jr. and Sr. paid the real estate taxes, Atty. Sabitsana also paid real property
x Republic Æ Filed the Petition for certiorari (65) with the Supreme Court.
taxes in 1992, 1993, and 1999. In 1996, he introduced concrete improvements
on the property, which shortly thereafter were destroyed by a typhoon. ISSUE: Whether the petition for declaratory relief was sufficient.
x Muertegui Æ Domingo Sr. passed away. His heirs applied for registration and
coverage of the lot under the Public Land Act or Commonwealth Act No. 141. HELD: NO.
x Sabitsana Æ Opposed the application in a letter.
29
RECTO, GAYLE ANGELI M.
2011-0008 | AUSL
Personal Notes on Remedial Law 2 Review (based on the syllabus of Prof. Henedino M. Brondial)

x An act of a court or tribunal can only be considered as with grave abuse of belong to an altogether different genus of constitutional litigation.44 Towards
discretion when such act is done in a capricious or whimsical exercise of this end, compelling State and societal interests in the proscription of harmful
judgment as is equivalent to lack of jurisdiction.28 It is well-settled that the conduct necessitate a closer judicial scrutiny of locus standi,45 as in this case. To
abuse of discretion to be qualified as "grave" must be so patent or gross as to rule otherwise, would be to corrupt the settled doctrine of locus standi, as every
constitute an evasion of a positive duty or a virtual refusal to perform the duty or worthy cause is an interest shared by the general public.46
to act at all in contemplation of law.29 In this relation, case law states that not x As to the fifth requisite for an action for declaratory relief, neither can it be
every error in the proceedings, or every erroneous conclusion of law or fact, inferred that the controversy at hand is ripe for adjudication since the possibility
constitutes grave abuse of discretion.30 The degree of gravity, as above- of abuse, based on the above-discussed allegations in private respondents’
described, must be met. petition, remain highly-speculative and merely theorized.1âwphi1 It is well-
x Applying these principles, the Court observes that while no grave settled that a question is ripe for adjudication when the act being challenged has
abuse of discretion could be ascribed on the part of the RTC when it had a direct adverse effect on the individual challenging it.47 This private
found that the Court did not pass upon the constitutionality of RA 9372 respondents failed to demonstrate in the case at bar.
in the Southern Hemisphere cases, it, however, exceeded its x Finally, as regards the sixth requisite, the Court finds it irrelevant to proceed with
jurisdiction when it ruled that private respondents’ petition had met all a discussion on the availability of adequate reliefs since no impending threat or
the requisites for an action for declaratory relief. Consequently, its injury to the private respondents exists in the first place.
denial of the subject motion to dismiss was altogether improper.
x To elucidate, it is clear that the Court, in Southern Hemisphere, did not x All told, in view of the absence of the fourth and fifth requisites for an action for
make any definitive ruling on the constitutionality of RA 9372. The declaratory relief, as well as the irrelevance of the sixth requisite, private
certiorari petitions in those consolidated cases were dismissed based respondents’ petition for declaratory relief should have been dismissed. Thus, by
solely on procedural grounds, namely: (a) the remedy of certiorari was giving due course to the same, it cannot be gainsaid that the RTC gravely
improper;31 (b) petitioners therein lack locus standi;32 and (c) abused its discretion.
petitioners therein failed to present an actual case or controversy.33
Therefore, there was no grave abuse of discretion.
x The same conclusion cannot, however, be reached with regard to the RTC’s
ruling on the sufficiency of private respondents’ petition for declaratory relief. SOUTHERN HEMISPHERE ENGAGEMENT NETWORK, INC., on behalf of the South-South
x Case law states that the following are the requisites for an action for declaratory Network (SSN) for Non-State Armed Group Engagement, and ATTY. SOLIMAN M. SANTOS,
relief: JR., Petitioners, vs. ANTI-TERRORISM COUNCIL, THE EXECUTIVE SECRETARY, THE
o first , the subject matter of the controversy must be a SECRETARY OF JUSTICE, THE SECRETARY OF FOREIGN AFFAIRS, THE SECRETARY OF
deed, will, contract or other written instrument, statute, NATIONAL DEFENSE, THE SECRETARY OF THE INTERIOR AND LOCAL GOVERNMENT, THE
executive order or regulation, or ordinance; SECRETARY OF FINANCE, THE NATIONAL SECURITY ADVISER, THE CHIEF OF STAFF OF THE
o second , the terms of said documents and the validity ARMED FORCES OF THE PHILIPPINES, AND THE CHIEF OF THE PHILIPPINE NATIONAL
thereof are doubtful and require judicial construction; POLICE, Respondents.
o third , there must have been no breach of the documents in EN BANC, G.R. No. 178552 October 5, 2010
question;
o fourth , there must be an actual justiciable controversy or
the "ripening seeds" of one between persons whose KILUSANG MAYO UNO (KMU), represented by its Chairperson Elmer Labog, NATIONAL
interests are adverse; FEDERATION OF LABOR UNIONS-KILUSANG MAYO UNO (NAFLU-KMU), represented by its
o fifth , the issue must be ripe for judicial determination; and National President Joselito V. Ustarez and Secretary General Antonio C. Pascual, and CENTER
o sixth , adequate relief is not available through other means FOR TRADE UNION AND HUMAN RIGHTS, represented by its Executive Director Daisy Arago,
or other forms of action or proceeding.34 Petitioners, vs. HON. EDUARDO ERMITA, in his capacity as Executive Secretary, NORBERTO
x Based on a judicious review of the records, the Court observes that while the GONZALES, in his capacity as Acting Secretary of National Defense, HON. RAUL GONZALES, in
first,35 second,36 and third37 requirements appear to exist in this case, the his capacity as Secretary of Justice, HON. RONALDO PUNO, in his capacity as Secretary of the
fourth, fifth, and sixth requirements, however, remain wanting. Interior and Local Government, GEN. HERMOGENES ESPERON, in his capacity as AFP Chief of
x As to the fourth requisite, there is serious doubt that an actual justiciable Staff, and DIRECTOR GENERAL OSCAR CALDERON, in his capacity as PNP Chief of Staff,
controversy or the "ripening seeds" of one exists in this case. Respondents.
x Pertinently, a justiciable controversy refers to an existing case or controversy G.R. No. 178554
that is appropriate or ripe for judicial determination, not one that is conjectural
or merely anticipatory.38 Corollary thereto, by "ripening seeds" it is meant, not BAGONG ALYANSANG MAKABAYAN (BAYAN), GENERAL ALLIANCE BINDING WOMEN FOR
that sufficient accrued facts may be dispensed with, but that a dispute may be REFORMS, INTEGRITY, EQUALITY, LEADERSHIP AND ACTION (GABRIELA), KILUSANG
tried at its inception before it has accumulated the asperity, distemper, MAGBUBUKID NG PILIPINAS (KMP), MOVEMENT OF CONCERNED CITIZENS FOR CIVIL
animosity, passion, and violence of a full blown battle that looms ahead. The LIBERTIES (MCCCL), CONFEDERATION FOR UNITY, RECOGNITION AND ADVANCEMENT OF
concept describes a state of facts indicating imminent and inevitable litigation GOVERNMENT EMPLOYEES (COURAGE), KALIPUNAN NG DAMAYANG MAHIHIRAP (KADAMAY),
provided that the issue is not settled and stabilized by tranquilizing SOLIDARITY OF CAVITE WORKERS, LEAGUE OF FILIPINO STUDENTS (LFS), ANAKBAYAN,
declaration.39 PAMBANSANG LAKAS NG KILUSANG MAMAMALAKAYA (PAMALAKAYA), ALLIANCE OF
x A perusal of private respondents’ petition for declaratory relief would show that CONCERNED TEACHERS (ACT), MIGRANTE, HEALTH ALLIANCE FOR DEMOCRACY (HEAD),
they have failed to demonstrate how they are left to sustain or are in immediate AGHAM, TEOFISTO GUINGONA, JR., DR. BIENVENIDO LUMBERA, RENATO CONSTANTINO,
danger to sustain some direct injury as a result of the enforcement of the JR., SISTER MARY JOHN MANANSAN OSB, DEAN CONSUELO PAZ, ATTY. JOSEFINA
assailed provisions of RA 9372. Not far removed from the factual milieu in the LICHAUCO, COL. GERRY CUNANAN (ret.), CARLITOS SIGUION-REYNA, DR. CAROLINA
Southern Hemisphere cases, private respondents only assert general interests as PAGADUAN-ARAULLO, RENATO REYES, DANILO RAMOS, EMERENCIANA DE LESUS, RITA
citizens, and taxpayers and infractions which the government could prospectively BAUA, REY CLARO CASAMBRE, Petitioners, vs. GLORIA MACAPAGAL-ARROYO, in her capacity
commit if the enforcement of the said law would remain untrammeled. As their as President and Commander-in-Chief, EXECUTIVE SECRETARY EDUARDO ERMITA,
petition would disclose, private respondents’ fear of prosecution was solely based DEPARTMENT OF JUSTICE SECRETARY RAUL GONZALES, DEPARTMENT OF FOREIGN
on remarks of certain government officials which were addressed to the general AFFAIRS SECRETARY ALBERTO ROMULO, DEPARTMENT OF NATIONAL DEFENSE ACTING
public.40 They, however, failed to show how these remarks tended towards any SECRETARY NORBERTO GONZALES, DEPARTMENT OF INTERIOR AND LOCAL GOVERNMENT
prosecutorial or governmental action geared towards the implementation of RA SECRETARY RONALDO PUNO. DEPARTMENT OF FINANCE SECRETARY MARGARITO TEVES,
9372 against them. In other words, there was no particular, real or imminent NATIONAL SECURITY ADVISER NORBERTO GONZALES, THE NATIONAL INTELLIGENCE
threat to any of them. As held in Southern Hemisphere: COORDINATING AGENCY (NICA), THE NATIONAL BUREAU OF INVESTIGATION (NBI), THE
x Without any justiciable controversy, the petitions have become pleas for BUREAU OF IMMIGRATION, THE OFFICE OF CIVIL DEFENSE, THE INTELLIGENCE SERVICE OF
declaratory relief, over which the Court has no original jurisdiction. Then again, THE ARMED FORCES OF THE PHILIPPINES (ISAFP), THE ANTI-MONEY LAUNDERING COUNCIL
declaratory actions characterized by "double contingency," where both the (AMLC), THE PHILIPPINE CENTER ON TRANSNATIONAL CRIME, THE CHIEF OF THE
activity the petitioners intend to undertake and the anticipated reaction to it of a PHILIPPINE NATIONAL POLICE GEN. OSCAR CALDERON, THE PNP, including its intelligence
public official are merely theorized, lie beyond judicial review for lack of and investigative elements, AFP CHIEF GEN. HERMOGENES ESPERON, Respondents.
ripeness.1âwphi1 G.R. No. 178581
x The possibility of abuse in the implementation of RA 9372does not avail to take
the present petitions out of the realm of the surreal and merely imagined. Such KARAPATAN, ALLIANCE FOR THE ADVANCEMENT OF PEOPLE'S RIGHTS, represented herein
possibility is not peculiar to RA 9372 since the exercise of any power granted by by Dr. Edelina de la Paz, and representing the following organizations: HUSTISYA, represented
law may be abused. Allegations of abuse must be anchored on real events by Evangeline Hernandez and also on her own behalf; DESAPARECIDOS, represented by Mary
before courts may step in to settle actual controversies involving rights which are Guy Portajada and also on her own behalf, SAMAHAN NG MGA EX-DETAINEES LABAN SA
legally demandable and enforceable.41 (Emphasis supplied; citations omitted) DETENSYON AT PARA SA AMNESTIYA (SELDA), represented by Donato Continente and also
x Thus, in the same light that the Court dismissed the SC petitions in the Southern on his own behalf, ECUMENICAL MOVEMENT FOR JUSTICE AND PEACE (EMJP), represented
Hemisphere cases on the basis of, among others, lack of actual justiciable by Bishop Elmer M. Bolocon, UCCP, and PROMOTION OF CHURCH PEOPLE'S RESPONSE,
controversy (or the ripening seeds of one), the RTC should have dismissed represented by Fr. Gilbert Sabado, OCARM, Petitioners, vs. GLORIA MACAPAGAL-ARROYO, in
private respondents’ petition for declaratory relief all the same. her capacity as President and Commander-in-Chief, EXECUTIVE SECRETARTY EDUARDO
x It is well to note that private respondents also lack the required locus standi to ERMITA, DEPARTMENT OF JUSTICE SECRETARY RAUL GONZALEZ, DEPARTMENT OF
mount their constitutional challenge against the implementation of the above- FOREIGN AFFAIRS SECRETARY ALBERTO ROMULO, DEPARTMENT OF NATIONAL DEFENSE
stated provisions of RA 9372 since they have not shown any direct and personal ACTING SECRETARY NORBERTO GONZALES, DEPARTMENT OF INTERIOR AND LOCAL
interest in the case.42 While it has been previously held that transcendental GOVERNMENT SECRETARY RONALDO PUNO, DEPARTMENT OF FINANCE SECRETARY
public importance dispenses with the requirement that the petitioner has MARGARITO TEVES, NATIONAL SECURITY ADVISER NORBERTO GONZALES, THE NATIONAL
experienced or is in actual danger of suffering direct and personal injury,43 it INTELLIGENCE COORDINATING AGENCY (NICA), THE NATIONAL BUREAU OF
must be stressed that cases involving the constitutionality of penal legislation INVESTIGATION (NBI), THE BUREAU OF IMMIGRATION, THE OFFICE OF CIVIL DEFENSE, THE
30
RECTO, GAYLE ANGELI M.
2011-0008 | AUSL
Personal Notes on Remedial Law 2 Review (based on the syllabus of Prof. Henedino M. Brondial)

INTELLIGENCE SERVICE OF THE ARMED FORCES OF THE PHILIPPINES (ISAFP), THE ANTI- possibility is not peculiar to RA 9372 since the exercise of any power granted by
MONEY LAUNDERING COUNCIL (AMLC), THE PHILIPPINE CENTER ON TRANSNATIONAL law may be abused.45 Allegations of abuse must be anchored on real events
CRIME, THE CHIEF OF THE PHILIPPINE NATIONAL POLICE GEN. OSCAR CALDERON, THE before courts may step in to settle actual controversies involving rights which are
PNP, including its intelligence and investigative elements, AFP CHIEF GEN. HERMOGENES legally demandable and enforceable.
ESPERON, Respondents.
G.R. No. 178890 C. Review of Judgments and Final Orders of the COMELEC and COA
[Rule 64]
THE INTEGRATED BAR OF THE PHILIPPINES (IBP), represented by Atty. Feliciano M. Bautista,
COUNSELS FOR THE DEFENSE OF LIBERTY (CODAL), SEN. MA. ANA CONSUELO A.S. The distinctive nature and procedure of the special civil action
MADRIGAL and FORMER SENATORS SERGIO OSMEÑA III and WIGBERTO E. TAÑADA,
Petitioners, vs. EXECUTIVE SECRETARY EDUARDO ERMITA AND THE MEMBERS OF THE ANTI-
TERRORISM COUNCIL (ATC), Respondents. ALLIANCE FOR NATIONALISM AND DEMOCRACY (ANAD), Petitioner, vs.
G.R. No. 179157 COMMISSION ON ELECTIONS, Respondent.
EN BANC, G.R. No. 206987 September 10, 2013

BAGONG ALYANSANG MAKABAYAN-SOUTHERN TAGALOG (BAYAN-ST), GABRIELA-ST, FACTS:


KATIPUNAN NG MGA SAMAHYANG MAGSASAKA-TIMOG KATAGALUGAN (KASAMA-TK), x COMELEC En Banc Æ promulgated a Resolution canceling Alliance for
MOVEMENT OF CONCERNED CITIZENS FOR CIVIL LIBERTIES (MCCCL), PEOPLES MARTYRS, Nationalism and Democracy (ANAD) Certificate of Registration and/or
ANAKBAYAN-ST, PAMALAKAYA-ST, CONFEDERATION FOR UNITY, RECOGNITION AND Accreditation for 2013 Party-List Elections.
ADVANCEMENT OF GOVERNMENT EMPLOYEES (COURAGE-ST), PAGKAKAISA'T UGNAYAN NG x ANAD Æ Filed the Petition for Certiorari with Urgent Prayer for the Issuance of a
MGA MAGBUBUKID SA LAGUNA (PUMALAG), SAMAHAN NG MGA MAMAMAYAN SA TABING Temporary Restraining Order and Writ of Mandamus with the Supreme Court
RILES (SMTR-ST), LEAGUE OF FILIPINO STUDENTS (LFS), BAYAN MUNA-ST, KONGRESO NG
MGA MAGBUBUKID PARA SA REPORMANG AGRARYO KOMPRA, BIGKIS AT LAKAS NG MGA ISSUE: Whether the issues on (1) grave abuse of discretion on the part of the COMELEC as to
KATUTUBO SA TIMOG KATAGALUGAN (BALATIK), SAMAHAN AT UGNAYAN NG MGA its promulgation of the assailed Resolution in that it was made without the benefit of a
MAGSASAKANG KABABAIHAN SA TIMOG KATAGALUGAN (SUMAMAKA-TK), STARTER, LOSÑOS summary evidentiary hearing mandated by the due process clause and (2) COMELEC’s error in
RURAL POOR ORGANIZATION FOR PROGRESS & EQUALITY, CHRISTIAN NIÑO LAJARA, finding that petitioner submitted only three nominees and that it failed to submit its Statement
TEODORO REYES, FRANCESCA B. TOLENTINO, JANNETTE E. BARRIENTOS, OSCAR T. of Contributions and Expenditures in the 2007 Elections may be raised in a Rule 64 petition.
LAPIDA, JR., DELFIN DE CLARO, SALLY P. ASTRERA, ARNEL SEGUNE BELTRAN, Petitioners,
vs. GLORIA MACAPAGAL-ARROYO, in her capacity as President and Commander-in-Chief, HELD: ONLY GRAVE ABUSE OF DISCRETION.
EXECUTIVE SECRETARY EDUARDO ERMITA, DEPARTMENT OF JUSTICE SECRETARY RAUL x The only question that may be raised in a petition for certiorari under
GONZALEZ, DEPARTMENT OF FOREIGN AFFAIRS SECRETARY ALBERTO ROMULO, Section 2, Rule 64 of the Rules of Court is whether or not the COMELEC
DEPARTMENT OF NATIONAL DEFENSE ACTING SECRETARY NORBERTO GONZALES, acted with grave abuse of discretion amounting to lack or excess of
DEPARTMENT OF INTERIOR AND LOCAL GOVERNMEN T SECRETARY RONALDO PUNO, jurisdiction. For a petition for certiorari to prosper, there must be a
DEPARTMENT OF FINCANCE SECRETARY MARGARITO TEVES, NATIONAL SECURITY ADVISER clear showing of caprice and arbitrariness in the exercise of
NORBERTO GONZALES, THE NATIONAL INTELLIGENCE COORDINATING AGENCY (NICA), THE discretion.5
NATIONAL BUREAU OF INVESTIGATION (NBI), THE BUREAU OF IMMIGRATION, THE OFFICE x "Grave abuse of discretion," under Rule 65, has a specific meaning. It
OF CIVIL DEFENSE, THE INTELLIGENCE SERVICE OF THE ARMED FORCES OF THE is the arbitrary or despotic exercise of power due to passion, prejudice
PHILIPPINES (ISAFP), THE ANTI-MONEY LAUNDERING COUNCIL (AMLC), THE PHILIPPINE or personal hostility; or the whimsical, arbitrary, or capricious exercise
CENTER ON TRANSNATIONAL CRIME, THE CHIEF OF THE PHILIPPINE NATIONAL POLICE of power that amounts to an evasion or a refusal to perform a positive
GEN. OSCAR CALDERON, THE PNP, including its intelligence and investigative elements, AFP duty enjoined by law or to act at all in contemplation of law. For an act
CHIEF GEN. HERMOGENES ESPERON, Respondents. to be struck down as having been done with grave abuse of discretion,
G.R. No. 179461 the abuse of discretion must be patent and gross.6
x ANAD claims that the COMELEC gravely abused its discretion when it
FACTS: promulgated the assailed Resolution without giving ANAD the benefit of a
x RA 9372 ("An Act to Secure the State and Protect our People from Terrorism," summary evidentiary hearing, thus violating its right to due process. It is to be
otherwise known as the Human Security Act of 2007) was signed into law on noted, however, that ANAD was already afforded a summary hearing on23
March 6, 2007. August 2013, during which Mr. Domingo M. Balang, ANAD’s president,
o RA 9372 subsequently became effective. authenticated documents and answered questions from the members of the
x Petitioners Æ Filed petitions for certiorari and prohibition COMELEC pertinent to ANAD’s qualifications.7
x ANAD, nonetheless, insists that the COMELEC should have called for another
ISSUE: Whether declaratory actions characterized by “double contingency” are within the summary hearing after this Court remanded the case to the COMELEC for re-
ambit of judicial review. evaluation in accordance with the parameters laid down in Atong Paglaum, Inc.
v. Comelec . This is a superfluity.
HELD: NO. x ANAD was already given the opportunity to prove its qualifications during the
x The Court is not unaware that a reasonable certainty of the occurrence of a summary hearing of 23 August 2012, during which ANAD submitted documents
perceived threat to any constitutional interest suffices to provide a basis for and other pieces of evidence to establish said qualifications. In re-evaluating
mounting a constitutional challenge. This, however, is qualified by the ANAD’s qualifications in accordance with the parameters laid down in Atong
requirement that there must be sufficient facts to enable the Court to Paglaum, Inc. v. COMELEC , the COMELEC need not have called another
intelligently adjudicate the issues.38 summary hearing. The Comelec could, as in fact it did,8 readily resort to
x Very recently, the US Supreme Court, in Holder v. Humanitarian Law Project,39 documents and other pieces of evidence previously submitted by petitioners in
allowed the pre-enforcement review of a criminal statute, challenged on re-appraising ANAD’s qualifications. After all, it can be presumed that the
vagueness grounds, since plaintiffs faced a "credible threat of prosecution" and qualifications, or lack thereof, which were established during the summary
"should not be required to await and undergo a criminal prosecution as the sole hearing of 23 August2012 continued until election day and even there after.
means of seeking relief."40 The plaintiffs therein filed an action before a federal x As to ANAD’s averment that the COMELEC erred in finding that it violated
court to assail the constitutionality of the material support statute, 18 U.S.C. election laws and regulations, we hold that the COMELEC, being a specialized
§2339B (a) (1),41 proscribing the provision of material support to organizations agency tasked with the supervision of elections all over the country, its factual
declared by the Secretary of State as foreign terrorist organizations. They findings, conclusions, rulings and decisions rendered on matters falling within its
claimed that they intended to provide support for the humanitarian and political competence shall not be interfered with by this Court in the absence of grave
activities of two such organizations. abuse of discretion or any jurisdictional infirmity or error of law.9
x Prevailing American jurisprudence allows an adjudication on the merits when an x As found by the COMELEC, ANAD, for unknown reasons, submitted only three
anticipatory petition clearly shows that the challenged prohibition forbids the nominees instead of five, in violation of Sec. 8 of R.A. No. 7941( An Act Providing
conduct or activity that a petitioner seeks to do, as there would then be a for the Election of Party-List Representatives through the Party-List System, and
justiciable controversy.42 Appropriating Funds Therefor).10 Such factual finding of the COMELEC was
x Unlike the plaintiffs in Holder, however, herein petitioners have failed to show based on the Certificate of Nomination presented and marked by petitioner
that the challenged provisions of RA 9372 forbid constitutionally protected during the 22 and 23 August 2012summary hearings.11
conduct or activity that they seek to do. No demonstrable threat has been x Compliance with Section 8 of R.A. No. 7941 is essential as the said provision is a
established, much less a real and existing one. safeguard against arbitrariness.1âwphi1 Section 8 of R.A. No. 7941rids a party-
x Petitioners’ obscure allegations of sporadic "surveillance" and supposedly being list organization of the prerogative to substitute and replace its nominees, or
tagged as "communist fronts" in no way approximate a credible threat of even to switch the order of the nominees, after submission of the list to the
prosecution. From these allegations, the Court is being lured to render an COMELEC.
advisory opinion, which is not its function.43 x In Lokin, Jr. v. Comelec,12 the Court discussed the importance of Sec.8 of R.A.
x Without any justiciable controversy, the petitions have become pleas No. 7941 in this wise:
for declaratory relief, over which the Court has no original jurisdiction. o The prohibition is not arbitrary or capricious; neither is it
Then again, declaratory actions characterized by "double contingency," without reason on the part of lawmakers. The COMELEC
where both the activity the petitioners intend to undertake and the can rightly presume from the submission of the list that the
anticipated reaction to it of a public official are merely theorized, lie list reflects the true will of the party-list organization. The
beyond judicial review for lack of ripeness.44 COMELEC will not concern itself with whether or not the
x The possibility of abuse in the implementation of RA 9372 does not avail to take list contains the real intended nominees of the party-list
the present petitions out of the realm of the surreal and merely imagined. Such organization, but will only determine whether the
31
RECTO, GAYLE ANGELI M.
2011-0008 | AUSL
Personal Notes on Remedial Law 2 Review (based on the syllabus of Prof. Henedino M. Brondial)

nominees pass all the requirements prescribed by the law


and whether or not the nominees possess all the 1. Certiorari
qualifications and none of the disqualifications. Thereafter,
the names of the nominees will be published in newspapers
of general circulation. Although the people vote for the a. grounds
party-list organization itself in a party-list system of b. requirements
election, not for the individual nominees, they still have the c. procedure; parties and effects
right to know who the nominees of any particular party-list
organization are. The publication of the list of the party-list Section 1. Petition for certiorari. — When any tribunal, board or officer exercising
nominees in newspapers of general circulation serves that judicial or quasi-judicial functions has acted without or in excess its or his jurisdiction, or with
right of the people, enabling the voters to make intelligent grave abuse of discretion amounting to lack or excess of jurisdiction, and there is no appeal,
and informed choices. In contrast, allowing the party-list or any plain, speedy, and adequate remedy in the ordinary course of law, a person aggrieved
organization to change its nominees through withdrawal of thereby may file a verified petition in the proper court, alleging the facts with certainty and
their nominations, or to alter the order of the nominations praying that judgment be rendered annulling or modifying the proceedings of such tribunal,
after the submission of the list of nominees circumvents board or officer, and granting such incidental reliefs as law and justice may require.
the voters’ demand for transparency. The lawmakers’
exclusion of such arbitrary withdrawal has eliminated the The petition shall be accompanied by a certified true copy of the judgment, order or resolution
possibility of such circumvention. subject thereof, copies of all pleadings and documents relevant and pertinent thereto, and a
x Moreover, the COMELEC also noted ANAD’s failure to submit a proper Statement sworn certification of non-forum shopping as provided in the third paragraph of section 3, Rule
of Contributions and Expenditures for the 2007 Elections, in violation of 46.
COMELEC Resolution No. 9476, viz:
o Rule 8, Sec. 3. Form and contents of statements. - The statement
required in next preceding section shall be in writing, subscribed
and sworn to by the candidate or by the treasurer of the party. It
shall set forth in detail the following:
ƒ The amount of contribution, the date of receipt, and
the full name, profession, business, taxpayer
identification number (TIN) and exact home and
business address of the person or entity from whom OSCAR R. AMPIL, Petitioner, vs. THE HON. OFFICE OF THE OMBUDSMAN, POLICARPIO
the contribution was received; (See Schedule of L. ESPENESIN, Registrar, Register of Deeds, Pasig City, FRANCIS SERRANO, YVONNE S.
Contributions Received, Annex "G") YUCHENGCO, and GEMA O. CHENG, Respondents.
ƒ The amount of every expenditure, the date thereof, SECOND DIVISION , G.R. No. 192685 July 31, 2013
the full name and exact address of the person or
entity to whom payment was made, and the purpose OSCAR R. AMPIL, Petitioner, vs. POLICARPIO L. ESPENESIN, Respondent.
of the expenditure; (See Schedule of Expenditures, G.R. No. 199115
Annex "H")
ƒ A Summary Report of Lawful Expenditure categorized
FACTS:
according to the list specified above shall be x SB Realty Corporation (ASB) and Malayan Insurance Company (MICO) entered
submitted by the candidate or party treasurer within into a Joint Project Development Agreement (JPDA) for the construction of a
thirty (30) days after the day of the election. The condominium building to be known as "The Malayan Tower, " wherein MICO
prescribed form for this Summary Report is hereby shall provide the real property located at the heart of the Ortigas Business
attached to these Rules as Annex "H-1." District, Pasig City, while ASB would construct, and shoulder the cost of
ƒ Any unpaid obligation, its nature and amount, the full construction and development of the condominium building.
name and exact home and business address of the x ASB & MICO entered into another contract, with MICO selling to ASB the land it
person or entity to whom said obligation is owing; was contributing under the JPDA. Under the Contract to Sell, ownership of the
and (See Schedule of Unpaid Obligations, Annex "I") land will vest on ASB only upon full payment of the purchase price.
ƒ If the candidate or treasurer of the party has received x As part of the ASB Group of Companies, filed a Petition for Rehabilitation with
no contribution, made no expenditure, or has no Prayer for Suspension of Actions and Proceedings before the Securities and
pending obligation, the statement shall reflect such Exchange Commission (SEC).
fact; x SEC Æ approved the Rehabilitation Plan submitted by the ASB Group of
ƒ And such other information that the Commission may Companies.
require. x ASB was unable to perform its obligations to MICO under the JPDA and the
x The prescribed form for the Statement of Election Contributions and Expenses is
Contract to Sell due to obvious financial difficulties.
attached to these Rules as Annex "F." The Schedules of Contributions and x ASB & MICO executed their Third contract, allowing MICO to assume the entire
Expenditures (Annexes "G" and "H", respectively) should be supported and responsibility for the development and completion of The Malayan Tower. At the
accompanied by certified true copies of official receipts, invoices and other time of the execution of the MOA, ASB had already paid MICO P427,231,952.32
similar documents. out of the P640,847,928.48 purchase price of the realty.
x An incomplete statement, or a statement that does not contain all the required x RD Pasig City (Espensin) Æ issued Condominium Certificates of Title (CCTs) for
information and attachments, or does not conform to the prescribed form, shall 38 units and the allotted parking spaces in the name of ASB, and another set for
be considered as not filed and shall subject the candidate or party treasurer to the same units, with MICO as registered owner thereof.
the penalties prescribed by law. x ASB Æ wrote Espenesin calling his attention to the supposed amendment in the
x As found by the COMELEC, ANAD failed to comply with the above-mentioned CCTs which he had originally issued in ASB’s name.
requirements as the exhibits submitted by ANAD consisted mainly of a list of x RD Æ replied that, believing it was a simple error and on representation of the
total contributions from other persons, a list of official receipts and amounts person we came to know and considered the representative of both parties (Atty.
without corresponding receipts, and a list of expenditures based on order slips Serrano), he erased the name ASB Realty Corporation on those specified titles
and donations without distinction as to whether the amounts listed were and placed instead the name MICO.
advanced subject to reimbursement or donated.13 This factual finding was x Ampil Æ Wrote Yuchengco and Cheng, President and Chief Financial Officer of
neither contested nor rebutted by ANAD. MICO, respectively, introducing himself as an unsecured creditor of ASB
x We herein take the opportunity to reiterate the well-established principle that the
rule that factual findings of administrative bodies will not be disturbed by the Holdings, Inc., one of the corporations forming part of the ASB Group of
Companies. He demanded that Yuchengco and Cheng rectify the resulting error
courts of justice except when there is absolutely no evidence or no substantial in the CCTs, and facilitate the registration of the subject units back to ASB’s
evidence in support of such findings should be applied with greater force when it name.
concerns the COMELEC, as the framers of the Constitution intended to place the x MICO Æ paid no heed to ASB’s and Ampil’s demands.
COMELEC - created and explicitly made independent by the Constitution itself - x Ampil Æ Charged Yuchengco, Cheng, Espenesin, and Serrano with Falsification
on a level higher than statutory administrative organs. The COMELEC has broad of Public Documents under Article 171(6) of the Revised Penal Code and violation
powers to ascertain the true results of the election by means available to it. For of Sections 3(a) and (e) of Republic Act No. 3019 before the Office of the
the attainment of that end, it is not strictly bound by the rules of evidence.14 Ombudsman.
x As empowered by law, the COMELEC may motu proprio cancel, after x Yuchengco, Cheng, Espenesin, and Serrano Æ Filed their counter-affidavits.
due notice and hearing, the registration of any party-list organization if x Ombudsman Æ Dismissed Ampil's complaint
it violates or fails to comply with laws, rules or regulations relating to
o holding that the resolution of whether respondents falsified the
elections.15 Thus, we find no grave abuse of discretion on the part of CCTs must be prefaced by a determination of who, between MICO
the COMELEC when it issued the assailed Resolution dated 11 May
and ASB, is the rightful owner of the subject units. The
2013.
Ombudsman held that it had no authority to interpret the
x In any event, the official tally results of the COMELEC show that ANAD garnered
200,972 votes.16 As such, even if petitioner is declared qualified and the votes provisions of the MOA and, thus, refrained from resolving the
preliminary question of ownership. Ombudsman did not have
cast for it are canvassed, statistics show that it will still fail to qualify for a seat in probable cause to indict respondents for falsification of the CCTs
the House of Representatives. because the last element of the crime, i.e., that the change made
the document speak something false, had not been established.
D. Certiorari, Prohibition, and Mandamus [RULE 65] x Ampil Æ Filed a Motion for Reconsideration.

32
RECTO, GAYLE ANGELI M.
2011-0008 | AUSL
Personal Notes on Remedial Law 2 Review (based on the syllabus of Prof. Henedino M. Brondial)

o Ombudsman Æ Denied Ampil’s motion and affirmed the dismissal FACTS:


of his complaint. x AL Ang Network is allegedly the duly authorized to supply water to and collect
o On the administrative litigation front and as previously narrated, payment therefor from the homeowners of Regent Pearl Subdivision
the Ombudsman found Espenesin liable for Simple Misconduct. x AL Ang Æ filed a complaint for sum of money under the Rule of Procedure for
x Ampil Æ Filed a Motion for Reconsideration. Small Claims Cases before the MTCC (Bacolod City, Branch 4), seeking to collect
x Ombudsman Æ Reconsidered its earlier resolution and recalled the one-month from Mondejar the amount of P23,111.71 which represented her unpaid water
suspension meted on Espenesin. bills.
x Ampil Æ Filed a petition for review under Rule 43 of the Rules of Court before x Mondejar Æ Alleged payment, but that AL Ang Network made unreasonable and
the appellate court. excessive adjusments in the rate, and questioned basis of the claim.
o CA Æ Affirmed the Ombudsman’s resolution absolving Espenesin of x AL Ang Network, in the interim, Disconnected Mondejar's water line.
not just Grave Misconduct and Dishonesty, but also of Simple x MTCC Æ Held that AL Ang Network was issued a Certificate of Public
Misconduct. Convenience (CPC) by the National Water Resources Board (NWRB) only on
x Ampil Æ Filed a Petition for certiorari under Rule 65 of the Rules of August 7, 2003, then, it can only charge Mondejar the agreed flat rate of P75.00
Court, as to dismissal of the Criminal complaint; and filed a petition for per month prior thereto or the sum of P1,050.00 for the period June 1, 2002 to
review on certiorari under Rule 45 of the Rules of Court, as to the August 7, 2003. Thus, given that Mondejar had made total payments equivalent
ruling of the Ombudsman on the administrative liability of Espenesin. to P1,685.99 for the same period, she should be considered to have fully paid AL
x Petitions were consolidated. Ang Network.
x AL Ang Network Æ Filed a petition for certiorari under Rule 65 of the Rules of
ISSUE: Whether the Ombudsman committed grave abuse of discretion in the dismissal of Court before the RTC (Bacolod City, Branch 45)
complaint filed before it. o ascribing grave abuse of discretion on the part of the MTCC in
finding that it (AL Ang Network) failed to establish with certainty
HELD: YES. Mondejar's obligation, and in not ordering the latter to pay the full
x G.R. No. 192685 is partially impressed with merit. Accordingly, we find grave amount sought to be collected.
abuse of discretion in the Ombudsman’s incomplete disposition of x RTC Æ dismissed the petition for certiorari.
Ampil’s complaint. x AL Ang Network Æ Moved for reconsideration.
x That the Ombudsman is a constitutional officer duty bound to o RTC Æ Denied motion for reconsideration.
"investigate on its own, or on complaint by any person, any act or x AL Ang Network Æ Filed a Petition for Review on Certiorari, as a direct recourse,
omission of any public official, employee, office or agency, when such with the Supreme Court.
act or omission appears to be illegal, unjust, improper, or inefficient"17
brooks no objection. The Ombudsman’s conduct of preliminary ISSUE: Whether a petition for certiorari under Rule 65 is a proper remedy to assail the
investigation is both power and duty. Thus, the Ombudsman and his propriety of the MTCC decision in a small claims case
Deputies, are constitutionalized as protectors of the people, who "shall
act promptly on complaints filed in any form or manner against public HELD: YES.
officials or employees of the government x x x, and shall, x x x notify x Section 23 of the Rule of Procedure for Small Claims Cases states that:
the complainants of the action taken and the result thereof."18 o SEC. 23. Decision. — After the hearing, the court shall render its
x The raison d'être for its creation and endowment of broad investigative decision on the same day, based on the facts established by the
authority is to insulate the Office of the Ombudsman from the long evidence (Form 13-SCC). The decision shall immediately be entered
tentacles of officialdom that are able to penetrate judges’ and fiscals’ by the Clerk of Court in the court docket for civil cases and a copy
offices, and others involved in the prosecution of erring public officials, thereof forthwith served on the parties.
and through the execution of official pressure and influence, quash, o The decision shall be final and unappealable.
delay, or dismiss investigations into malfeasances and misfeasances x Considering the final nature of a small claims case decision under the
committed by public officers.19 above-stated rule, the remedy of appeal is not allowed, and the
x Plainly, the Ombudsman has "full discretion," based on the attendant prevailing party may, thus, immediately move for its execution.25
facts and circumstances, to determine the existence of probable cause Nevertheless, the proscription on appeals in small claims cases, similar
or the lack thereof.20 On this score, we have consistently hewed to the to other proceedings where appeal is not an available remedy,26 does
policy of non-interference with the Ombudsman’s exercise of its not preclude the aggrieved party from filing a petition for certiorari
constitutionally mandated powers.21 The Ombudsman’s finding to under Rule 65 of the Rules of Court. This general rule has been enunciated
proceed or desist in the prosecution of a criminal case can only be in the case of Okada v. Security Pacific Assurance Corporation,27 wherein it was
assailed through certiorari proceedings before this Court on the ground held that
that such determination is tainted with grave abuse of discretion which o In a long line of cases, the Court has consistently ruled that "the
contemplates an abuse so grave and so patent equivalent to lack or extraordinary writ of certiorari is always available where there is no
excess of jurisdiction.22 appeal or any other plain, speedy and adequate remedy in the
x However, on several occasions, we have interfered with the Ombudsman’s ordinary course of law." In Jaca v. Davao Lumber Co., the Court
discretion in determining probable cause: ruled:
o (a) To afford protection to the constitutional rights of the accused; ƒ x x x Although Section 1, Rule 65 of the Rules of
o (b) When necessary for the orderly administration of justice or to Court provides that the special civil action of certiorari
avoid oppression or multiplicity of actions; may only be invoked when "there is no appeal, nor any
o (c) When there is a prejudicial question which is sub judice; plain, speedy and adequate remedy in the course of
o (d) When the acts of the officer are without or in excess of law," this rule is not without exception. The
authority; availability of the ordinary course of appeal does not
o (e) Where the prosecution is under an invalid law, ordinance or constitute sufficient ground to prevent a party from
regulation; making use of the extraordinary remedy of certiorari
o (f) When double jeopardy is clearly apparent; where appeal is not an adequate remedy or equally
o (g) Where the court has no jurisdiction over the offense; beneficial, speedy and sufficient. It is the inadequacy
o (h) Where it is a case of persecution rather than prosecution; - not the mere absence - of all other legal remedies
o (i) Where the charges are manifestly false and motivated by the and the danger of failure of justice without the writ
lust for vengeance.23 (Emphasis supplied). that usually determines the propriety of certiorari.
x The fourth circumstance is present in G.R. No. 192685. x This ruling was reiterated in Conti v. Court of Appeals:
x While we agree with the Ombudsman’s disquisition that there is no probable o Truly, an essential requisite for the availability of the
cause to indict respondents for Falsification of Public Documents under Article extraordinary remedies under the Rules is an absence of an
171(6) of the Revised Penal Code, we are puzzled why the Ombudsman appeal nor any "plain, speedy and adequate remedy" in the
completely glossed over Ampil’s charge that respondents committed prohibited ordinary course of law, one which has been so defined as a
acts listed in Sections 3(a) and (e) of Republic Act No. 3019. Nowhere in the "remedy which (would) equally (be) beneficial, speedy and
Resolution or in the Order denying reconsideration thereof did the Ombudsman sufficient not merely a remedy which at some time in the
tackle and resolve the issue of whether respondents violated the particular future will bring about a revival of the judgment x x x
provisions of Republic Act No. 3019. complained of in the certiorari proceeding, but a remedy
x Curiously, the Ombudsman docketed Ampil’s complaint-affidavit as one "for: which will promptly relieve the petitioner from the
Falsification of Public Documents and Violation of Sections 3(a) and (e) of injurious effects of that judgment and the acts of the
Republic Act No. 3019, as amended."24 The Ombudsman even prefaced the inferior court or tribunal" concerned. x x x (Emphasis
Resolution, thus: "this has reference to the complaint filed by Oscar Ampil on 17 supplied)
September 2007 against respondents, for Falsification of Public Documents and x In this relation, it may not be amiss to placate the RTC’s apprehension that
Violation of Sections 3, paragraphs (a) and (e) of Republic Act No. 3019, respondent’s recourse before it (was only filed to circumvent the non-appealable
otherwise known as the Anti-Graft and Corrupt Practices Act, as amended nature of [small claims cases], because it asks [the court] to supplant the
decision of the lower [c]ourt with another decision directing the private
respondent to pay the petitioner a bigger sum than what has been awarded."28
A.L. ANG NETWORK, INC., Petitioner, vs. EMMA MONDEJAR, accompanied by her Verily, a petition for certiorari, unlike an appeal, is an original action29 designed
husband, EFREN MONDEJAR, Respondent. to correct only errors of jurisdiction and not of judgment. Owing to its nature, it
SECOND DIVISION, G.R. No. 200804 January 22, 2014 is therefore incumbent upon petitioner to establish that jurisdictional errors
tainted the MTCC Decision. The RTC, in turn, could either grant or dismiss the
petition based on an evaluation of whether or not the MTCC gravely abused its
33
RECTO, GAYLE ANGELI M.
2011-0008 | AUSL
Personal Notes on Remedial Law 2 Review (based on the syllabus of Prof. Henedino M. Brondial)

discretion by capriciously, whimsically, or arbitrarily disregarding evidence that is as to give the administrative agency concerned every opportunity
material to the controversy.30 to correct its error and dispose of the case.
x In view of the foregoing, the Court thus finds that petitioner correctly availed of x However, the doctrine of exhaustion of administrative remedies is not absolute
the remedy of certiorari to assail the propriety of the MTCC Decision in the as it admits of the following exceptions:
subject small claims case, contrary to the RTC’s ruling. o (1) when there is a violation of due process;
x Likewise, the Court finds that petitioner filed the said petition before the proper o (2) when the issue involved is purely a legal question;
forum (i.e., the RTC).1âwphi1 To be sure, the Court, the Court of Appeals and o (3) when the administrative action is patently illegal amounting to
the Regional Trial Courts have concurrent jurisdiction to issue a writ of lack or excess of jurisdiction;
certiorari.31 Such concurrence of jurisdiction, however, does not give a party o (4) when there is estoppel on the part of the administrative agency
unbridled freedom to choose the venue of his action lest he ran afoul of the concerned;
doctrine of hierarchy of courts. Instead, a becoming regard for judicial hierarchy o (5) when there is irreparable injury;
dictates that petitions for the issuance of writs of certiorari against first level o (6) when the respondent is a department secretary whose acts as
courts should be filed with the Regional Trial Court, and those against the latter, an alter ego of the President bears the implied and assumed
with the Court of Appeals, before resort may be had before the Court.32 This approval of the latter;
procedure is also in consonance with Section 4, Rule 65 of the Rules of Court.33 o (7) when to require exhaustion of administrative remedies would
x Hence, considering that small claims cases are exclusively within the jurisdiction be unreasonable;
of the Metropolitan Trial Courts, Municipal Trial Courts in Cities, Municipal Trial o (8) when it would amount to a nullification of a claim;
Courts, and Municipal Circuit Trial Courts,34 certiorari petitions assailing its o (9) when the subject matter is a private land in land case
dispositions should be filed before their corresponding Regional Trial Courts. This proceedings;
petitioner complied with when it instituted its petition for certiorari before the o (10) when the rule does not provide a plain, speedy and adequate
RTC which, as previously mentioned, has jurisdiction over the same. In fine, the remedy, and
RTC erred in dismissing the said petition on the ground that it was an improper o (11) when there are circumstances indicating the urgency of
remedy, and, as such, RTC Case No. 11-13833 must be reinstated and remanded judicial intervention, and unreasonable delay would greatly
thereto for its proper disposition. prejudice the complainant; (12) where no administrative review is
provided by law; (13) where the rule of qualified political agency
applies and (14) where the issue of non-exhaustion of
MARK JEROME S. MAGLALANG, Petitioner, vs. PHILIPPINE AMUSEMENT AND administrative remedies has been rendered moot.29
GAMING CORPORATION (PAGCOR), as represented by its incumbent Chairman EFRAIM x The case before us falls squarely under exception number 12 since the law per
GENUINO, Respondent. se provides no administrative review for administrative cases whereby an
FIRST DIVISION, G.R. No. 190566 December 11, 2013 employee like petitioner is covered by Civil Service law, rules and regulations and
penalized with a suspension for not more than 30 days.
x Section 37 (a) and (b) of P.D. No. 807, otherwise known as the Civil Service
FACTS: Decree of the Philippines, provides for the unavailability of any appeal:
x Maglalang a teller at the Casino Filipino, Angeles City Branch, Angeles City, which o Section 37. Disciplinary Jurisdiction.
was operated by PAGCOR, erroneously spread the bills, handed by one Cecilia o (a) The Commission shall decide upon appeal all administrative
Nakasato, in four clusters instead of five. For his mistake, Cecilia accused him of disciplinary cases involving the imposition of a penalty of
trying to shortchange her and that petitioner tried to deliberately fool her of her suspension for more than thirty days, or fine in an amount
money. He tried to explain, but Cecilia allegedly continued to berate and curse exceeding thirty days’ salary, demotion in rank or salary or transfer,
him. To ease the tension, Maglalang was asked to take a break, after which he removal or dismissal from Office. A complaint may be filed directly
returned to his booth. However, Cecilia allegedly showed up and continued to with the Commission by a private citizen against a government
berate Maglalang. As a result, the two of them were invited to the casino’s official or employee in which case it may hear and decide the case
Internal Security Office in order to air their respective sides. Thereafter, or it may deputize any department or agency or official or group of
Maglalang was required to file an Incident Report which he submitted on the officials to conduct the investigation. The results of the
same day of the incident. investigation shall be submitted to the Commission with
x Maglalang received a Memorandum issued by the casino’s Branch Manager recommendation as to the penalty to be imposed or other action to
informing him that he was being charged with Discourtesy towards a casino be taken.
customer, in which he submitted a letter-explanation. o (b) The heads of departments, agencies and instrumentalities,
x Maglalang received another Memorandum stating that the Board of Directors of provinces, cities and municipalities shall have jurisdiction to
PAGCOR found him guilty of Discourtesy towards a casino customer and imposed investigate and decide matters involving disciplinary action against
on him a 30-day suspension for this first offense. officers and employees under their jurisdiction. Their decisions shall
x Maglalang Æ Filed a Motion for Reconsideration. be final in case the penalty imposed is suspension for not more
x Maglalang Æ Filed a Motion for Production, praying that he be furnished with than thirty days or fine in an amount not exceeding thirty days’
copies of documents relative to the case including the recommendation of the salary. In case the decision rendered by a bureau or office head is
investigating committee and the Decision/Resolution of the Board supposedly appealable to the Commission, the same may be initially appealed
containing the latter’s factual findings. to the department and finally to the Commission and pending
x Maglalang Æ Received letter-reply of one Atty. Carlos R. Bautista, Jr. denying the appeal, the same shall be executory except when the penalty is
motion. removal, in which case the same shall be executory only after
x PAGCOR Æ Issued two memoranda, the latter one denying Maglalang's confirmation by the department head. (Emphasis supplied.)
reconsideration. x Similar provisions are reiterated in the aforequoted Section 4730 of E.O. No. 292
x Maglalang Æ Filed a petition for certiorari under Rule 65, before the CA. essentially providing that cases of this sort are not appealable to the CSC.
o CA Æ Outrightly dismissed the petition for certiorari for being Correlatively, we are not unaware of the Concurring Opinion of then Chief Justice
premature as Maglalang failed to exhaust administrative remedies Puno in CSC v. Dacoycoy,31 where he opined, to wit:
before seeking recourse from the CA. o In truth, the doctrine barring appeal is not categorically sanctioned
x Maglalang Æ Filed a Rule 45 before the SC by the Civil Service Law. For what the law declares as "final" are
decisions of heads of agencies involving suspension for not more
ISSUE # 1: Whether the doctrine of exhaustion of administrative remedies applies in cases than thirty (30) days or fine in an amount not exceeding thirty (30)
where an employee under the civil service is penalized with a suspension for not more than 30 days salary. But there is a clear policy reason for declaring these
days and is a condition precedent prior to the filing of a Rule 65 before the CA. decisions final. These decisions involve minor offenses. They are
numerous for they are the usual offenses committed by
HELD # 1: NO. government officials and employees. To allow their multiple level
x Our ruling in Public Hearing Committee of the Laguna Lake Development appeal will doubtless overburden the quasijudicial machinery of our
Authority v. SM Prime Holdings, Inc.28 on the doctrine of exhaustion of administrative system and defeat the expectation of fast and
administrative remedies is instructive, to wit: efficient action from these administrative agencies. Nepotism,
o Under the doctrine of exhaustion of administrative remedies, before however, is not a petty offense. Its deleterious effect on
a party is allowed to seek the intervention of the court, he or she government cannot be over-emphasized. And it is a stubborn evil.
should have availed himself or herself of all the means of The objective should be to eliminate nepotic acts, hence, erroneous
administrative processes afforded him or her. Hence, if resort to a decisions allowing nepotism cannot be given immunity from review,
remedy within the administrative machinery can still be made by especially judicial review. It is thus non sequitur to contend that
giving the administrative officer concerned every opportunity to since some decisions exonerating public officials from minor
decide on a matter that comes within his or her jurisdiction, then offenses can not be appealed, ergo, even a decision acquitting a
such remedy should be exhausted first before the court's judicial government official from a major offense like nepotism cannot also
power can be sought. The premature invocation of the intervention be appealed.
of the court is fatal to one’s cause of action. The doctrine of
exhaustion of administrative remedies is based on practical and ISSUE # 2: Whether decisions of administrative agencies which are declared final and
legal reasons. The availment of administrative remedy entails lesser unappealable by law are still subject to judicial review.
expenses and provides for a speedier disposition of controversies.
Furthermore, the courts of justice, for reasons of comity and HELD # 2: YES.
convenience, will shy away from a dispute until the system of x Nevertheless, decisions of administrative agencies which are declared final and
administrative redress has been completed and complied with, so unappealable by law are still subject to judicial review. In Republic of the Phils. v.
Francisco,32 we held:
34
RECTO, GAYLE ANGELI M.
2011-0008 | AUSL
Personal Notes on Remedial Law 2 Review (based on the syllabus of Prof. Henedino M. Brondial)

o Since the decision of the Ombudsman suspending from notice of the judgment, order or resolution sought to be assailed.
respondents for one (1) month is final and unappealable, it The 60-day period is inextendible to avoid any unreasonable delay that
follows that the CA had no appellate jurisdiction to review, would violate the constitutional rights of parties to a speedy
rectify or reverse the same. The Ombudsman was not disposition of their case.13 While there are recognized exceptions14 to
estopped from asserting in this Court that the CA had no such strict observance, there should be an effort on the part of the
appellate jurisdiction to review and reverse the decision of party invoking liberality to advance a reasonable or meritorious
the Ombudsman via petition for review under Rule 43 of the explanation for his/her failure to comply with the rules.15
Rules of Court. This is not to say that decisions of the x In the case at bench, no convincing justification for the belated filing of the
Ombudsman cannot be questioned. Decisions of petition was advanced to warrant the relaxation of the Rules.Notably, the records
administrative or quasi-administrative agencies which are show that the petition was filedonly on August 12, 2013, or almost a month late
declared by law final and unappealable are subject to from the due date which fell on July 16, 2013. To excuse this grave procedural
judicial review if they fail the test of arbitrariness, or upon lapse will not only be unfairto the other party, but it will also sanction a seeming
proof of gross abuse of discretion, fraud or error of law. rudimentary attempt to circumvent standing rules of procedure. Suffice it to say,
When such administrative or quasi-judicial bodies grossly the reasons proffered by the petitioner do not carry even a tinge of merit that
misappreciate evidence of such nature as to compel a would deserve leniency.
contrary conclusion, the Court will not hesitate to reverse x The late filing of the petition was borne out of the petitioner’s failure to monitor
the factual findings. Thus, the decision of the Ombudsman incoming court processes that neededto be addressed by the office. Clearly, this
may be reviewed, modified or reversed via petition for is an admission of inefficiency, if not lack of zeal, on the part of an office tasked
certiorari under Rule 65 of the Rules of Court, on a finding toeffectively curb smuggling activities which rob the government of millions of
that it had no jurisdiction over the complaint, or of grave revenue every year.
abuse of discretion amounting to excess or lack of x The display of patent violations of even the elementary rules leads the Court to
jurisdiction.It bears stressing that the judicial recourse suspectthat the case against Garcia and Vestidas Jr. was doomed by designfrom
petitioner availed of in this case before the CA is a special the start. The failure to present the certified true copies of documentary
civil action for certiorari ascribing grave abuse of evidence; the failure to competently and properly identify the misdeclared goods;
discretion, amounting to lack or excess of jurisdiction on the failure to identify the accused in court; and,worse, the failure to file this
the part of PAGCOR, not an appeal. Suffice it to state that petition on time challenging a judgment of acquittal, are tell-tale signs ofa
an appeal and a special civil action such as certiorari under reluctantand subduedattitude in pursuing the case. This stance taken by the
Rule 65 are entirely distinct and separate from each other. lawyers in government service rouses the Court’s vigilance against inefficiency in
One cannot file petition for certiorari under Rule 65 of the the administration of justice. Verily, the lawyersrepresenting the offices under
Rules where appeal is available, even if the ground availed the executive branchshould be reminded that theystill remain as officers of the
of is grave abuse of discretion. A special civil action for courtfrom whom a high sense of competence and fervor is expected. The
certiorari under Rule 65 lies only when there is no appeal, Courtwill not close its eyes to this sense of apathy in RATS lawyers, lest the
or plain, speedy and adequate remedy in the ordinary government’s goal of revenue enhancement continues to suffer the blows of
course of law. Certiorari cannot be allowed when a party to smuggling and similar activities.
a case fails to appeal a judgment despite the availability of x Even the error committed by the RATS in filing a motion for reconsideration with
that remedy, as the same should not be a substitute for the the CTA displays gross ignorance as to the effects of an acquittal in a criminal
lost remedy of appeal. The remedies of appeal and case and the constitutional proscription on double jeopardy. Had the RATS been
certiorari are mutually exclusive and not alternative or eager and keen in prosecuting the respondents, it would have, in the first place,
successive.33 presented its evidence with the CTA in strict compliance with the Rules.
x In sum, there being no appeal or any plain, speedy, and adequate
remedy in the ordinary course of law in view of petitioner's allegation ISSUE # 2: Whether a judgment of acquittal in the present criminal case may correctly be
that P AGCOR has acted without or in excess of jurisdiction, or with assailed in a petition for certiorari under Rule 65.
grave abuse of discretion amounting to lack or excess of jurisdiction,
the CA's outright dismissal of the petition for certiorari on the basis of HELD # 2: NO.
non-exhaustion of administrative remedies is bereft of any legal x In any case, even if the Court decides to suspend the rules and permit
standing and should therefore be set aside. this recourse, the end result would remain the same. While a judgment
x Finally, as a rule, a petition for certiorari under Rule 65 is valid only of acquittal in a criminal case may be assailed in a petition for
when the question involved is an error of jurisdiction, or when there is certiorari under Rule 65 of the Rules of Court,it must be shown that
grave abuse of discretion amounting to lack or excess of jurisdiction on there was grave abuse of discretion amounting to lack or excess of
the part of the court or tribunals exercising quasi-judicial functions. jurisdiction or a denial of due process. In this case, a perusal of the
Hence, courts exercising certiorari jurisdiction should refrain from challenged resolutions ofthe CTAdoes not disclose any indication of
reviewing factual assessments of the respondent court or agency. grave abuse of discretion on its partor denial of due process.The
Occasionally, however, they are constrained to wade into factual records are replete with indicators that the petitioner actively
matters when the evidence on record does not support those factual participated during the trial and, in fact, presented its offer of evidence
findings; or when too much is concluded, inferred or deduced from the and opposed the demurrer.
bare or incomplete facts appearing on record.34 Considering the x Grave abuse of discretion is defined as capricious or whimsical exercise of
circumstances and since this Court is not a trier of facts, 35 remand of judgment as is equivalent to lack of jurisdiction. The abuse of discretion must be
this case to the CA for its judicious resolution is in order. patent and gross as to amount to an evasion of a positive duty or a virtual
refusal to perform a duty enjoined by law, or to act at all in contemplation of
law, as where the power is exercised in an arbitrary and despotic manner by
PEOPLE OF THE PHILIPPINES, Petitioner, vs. THE HONORABLE JUANITO C. reason of passion and hostility.16 Here, the subject resolutions of the CTA have
CASTANEDA, JR., HONORABLE CAESAR A. CASANOVA, HONORABLE CIELITO N. MINDARO- been issued in accordance with the rules on evidence and existing jurisprudence.
GRULLA, AS ASSOCIATE JUSTICES OF THE SPECIAL SECOND DIVISION, COURT OF TAX
APPEALS; and MYRNA M. GARCIA AND CUSTODIO MENDOZA VESTIDAS, JR., Respondents.
THIRD DIVISION, G.R. No. 208290 December 11, 2013 UNIVERSITY OF THE PHILIPPINES BOARD OF REGENTS and DR. OLIVIA C. CAOILI in
her capacity as Secretary of the Board, petitioners, vs. HON. ELSIE LIGOT-TELAN in her
FACTS: capacity as Presiding Judge of Branch 87, Regional Trial Court of Quezon City and RAMON P.
x Myrna M. Garcia (Garcia) and Custodio Mendoza Vestidas, Jr. (Vestidas Jr.) as NADAL, respondents.
owner/proprietress and broker of Plinth Enterprise respectively, were charged for EN BANC, G.R. No. 110280 October 12, 1993
violation of Section 36023 in relation to Sections 2503 and 2530 (f) (i) and 1, (3)
(4) and (5) of the Tariff and Customs Code of the Philippines, as amended, for
importing 858 cartons of 17,160 pieces of Anti-Virus Software Kaspersky Internet FACTS:
Security Premium 2012, with the CTA. x UP Administration conceptualized and implemented the socialized scheme of
o Pleaded "Not Guilty" tuition fee payments through the Socialized Tuition Fee and Assistance Program
x Prosecution Æ During trial, and subsequent to the presentation of witnesses, (STFAP), popularly known as the "Iskolar ng Bayan" program.
filed its Formal Offer of Evidence. x Congress granted official recognition through allocation of a portion of the
x Garcia and Vestidas Æ Filed their Omnibus Motion to File Demurrer to Evidence National Budget for the implementation of the program.
o CTA Æ granted x All students were entitled to apply for STFAP benefits.
x Garcia and Vestidas Æ Filed the Demurrer to Evidence. x UPÆ Adopted measures to safeguard the integrity of the program, such as the
o CTA Æ granted Demurer; Dismissed the case inclusion as one of the punishable acts under Section 2 (a) of the Rules and
x Prosecution Æ Filed its motion for reconsideration Regulations on Student Conduct and Discipline of the University the deliberate
o CTA Æ Denied motion. falsification or suppression/withholding of any material information required in
x Prosecution → Filed petition for certiorari with the Supreme Court, more than 60 the application form.
days from receipt of the decision x UP Administration Æ Made a random sample, such as Ramon P. Nadal, a
student enrolled in the College of Law.
ISSUE #1: Whether the belated filing should be allowed by the Court. o Nadal's aunt's interview included in home visit report. Discrepancies
found. Matter was presented to the Diliman Committee on
HELD # 1: NO. Scholarships and Financial Assistance.
x Section 4, Rule 65 of the 1997 Rules of Civil Procedure is explicit in x UP Æ Informed Nadal of misdeclarations and his reclassification to bracket 9
stating that certiorari should be instituted within a period of 60 days from bracket 4; required Nadal to pay back school fees; and warned him that
35
RECTO, GAYLE ANGELI M.
2011-0008 | AUSL
Personal Notes on Remedial Law 2 Review (based on the syllabus of Prof. Henedino M. Brondial)

failure to settle account would mean the suspension of his registration privileges behalf of the other members of the "Consuelo Heights Homeowners Association," petitioners-
and the withholding of clearance and transcript of records; and that his case intervenors.
might be referred to the Student Disciplinary Tribunal for further investigation. EN BANC, G.R. No. 70484 January 29, 1988
x Nadal Æ Issued a certification stating, among other things, that his mother
migrated to the United States in 1981 but because her residency status had not
yet been legalized, she had not been able to find a "stable, regular, well-paying FACTS:
employment." He also stated that his mother, jointly with his brother Virgilio, was x Spouses Tuason (Roman C. Tuason and Remedios Tuason) were retired public
shouldering the expenses of the college education of his two younger brothers. school teachers.
x UP Æ Charged Nadal before the Student Disciplinary Tribunal (SDT). x Spouses Tuason → With funds pooled from their retirement benefits and
x UP SDT → exculpated Nadal of charge of deliberately withholding in his STFAP savings, they bought from Carmel Farms, Inc. of a piece of land in the latter's
application form information that he was maintaining a Toyota Corolla car, but subdivision in Barrio Makatipo, Caloocan City.
finding him guilty of "wilfully and deliberately withholding information about the x RD → Carmel's Torrens title (No. 64007) over the lot was cancelled and a new
income of his mother, who is living abroad, in support of the studies of his one (No. 8314) issued in the name of the Tuasons.
brothers Antonio and Federico, which is tantamount to acts of dishonesty in x Spouses Tuason → Took possession of their property.
relation to his studies in violation of paragraph [a], Section 2 of the Rules. As x President Marcos → Issued PD 293, declaring lots in Carmel's subdivision “open
such, the SDT imposed upon Nadal the penalty of expulsion from the University for disposition and sale to the members of the Malacanang Homeowners
and required him to reimburse all STFAP benefits he had received but if he does Association, Inc., the present bona fide occupants thereof."
not voluntarily make reimbursement, it shall be "effected by the University thru x RD → Caused the inscription on the Tuasons' title of “MEMORANDUM. —
outside legal action." The SDT decision was thereafter automatically elevated to Pursuant to Presidential Decree No. 293, this certificate of title is declared invalid
the Executive Committee of U.P. Diliman for review pursuant to Sec. 20 of the and null and void ab initio and considered cancelled as against the Government
U.P. Rules on Student Conduct and Discipline. and the property described herein is declared open for disposition and sale to the
x UP Executive Committee Æ Affirmed the decision of the SDT members of the Malacanang Homeowners Association, Inc.”
x Nadal Æ Appealed to the Board of Regents (BOR). x Spouses Tuason → Filed with the Supreme Court a petition for certiorari.
x UP BOR Æ Affirmed decision of the SDT, but modified penalty "from Expulsion to
One Year- Suspension, effective immediately, plus reimbursement of all benefits ISSUE # 1: Whether a petition for certiorari is the proper remedy to question the acts
received from the STFAP, with legal interest." The BOR also decided against committed by President Marcos, through the issuance of PD 293.
giving Nadal, a certification of good moral character.
x NADAL Æ Filed a motion for reconsideration of the BOR decision, allegedly HELD (1): YES.
against the advice of his counsel, to which UP opposed. x The procedural issue is quite easily disposed of. It is true that the
x UP BOR Æ Modified the decision, imposing on Nadal the penalties of suspension extraodinary writ of certiorari 9 may properly issue to nullify only
for one (1) year effective March 29, 1993, non-issuance of any certificate of good judicial or quasi-judicial acts, unlike the writ of prohibition which may
moral character during the suspension and/or as long as Nadal has not be directed against acts either judicial or ministerial. Section 1, Rule 65
reimbursed the STFAP benefits he had received with 12% interest per annum of the Rules of Court deals with the writ of certiorari in relation to "any
from 30 March 1993 and non-issuance of his transcript of records until he has tribunal, board or officer exercising judicial functions, while Section 2
settled his financial obligations with the university. of the same Rule treats of the writ of prohibition in relation to
x Nadal Æ Filed with the Regional Trial Court of Quezon City a petition for "proceedings of any tribunal, corporation, board, or person ...
mandamus with preliminary injunction and prayer for a temporary restraining exercising functions judicial or ministerial." But the petition will be
order against President Abueva, the BOR, Oscar M. Alfonso, Cesar A. shown upon analysis to be in reality directed against an unlawful
Buenaventura, Armand V. Fabella and Olivia C. Caoili. exercise of judicial power.
x RTC Æ Issued TRO. x The decree reveals that Mr. Marcos exercised an obviously judicial function. He
x RTC Æ Issued writ of preliminary injunction. made a determination of facts, and applied the law to those facts, declaring what
x UP Æ Dispensing with the filing of a motion for reconsideration, filed the petition the legal rights of the parties were in the premises. These acts essentially
for certiorari and prohibition with prayer for the issuance of an injunction or constitute a judicial function, 10 or an exercise of jurisdiction — which
temporary restraining order. is the power and authority to hear or try and decide or determine a
cause. 11 He adjudged it to be an established fact that neither the original
ISSUE: Whether lower court gravely abused its discretion in issuing the writ of preliminary purchasers nor their subsequent transferees have made full payment of all
injunction. installments of the purchase money and interest on the lots claimed by Carmel
Farms, Inc., including those on which the dwellings of the members of ... (the)
HELD: YES. Association (of homeowners) stand." And applying the law to that situation, he
x On the second issue presented for adjudication, the Court finds that the lower made the adjudication that "title to said land has remained with the Government,
court gravely abused its discretion in issuing the writ of preliminary injunction of and the land now occupied by the members of said association has never ceased
May 29, 1993. The issuance of the said writ was based on the lower court's to form part of the property of the Republic of the Philippines," and that 'any and
finding that the implementation of the disciplinary sanction of suspension on all acts affecting said land and purporting to segregate it from the said property
Nadal "would work injustice to the petitioner as it would delay him in finishing his of the Republic ... (were) null and void ab initio as against the law and public
course, and consequently, in getting a decent and good paying job." Sadly, policy.
such a ruling considers only the situation of Nadal without taking into x These acts may thus be properly struck down by the writ of certiorari,
account the circumstances clearly of his own making, which led him because done by an officer in the performance of what in essence is a
into such a predicament. More importantly, it has completely judicial function, if it be shown that the acts were done without or in
disregarded the overriding issue of academic freedom which provides excess of jurisdiction, or with grave abuse of discretion. Since Mr.
more than ample justification for the imposition of a disciplinary Marcos was never vested with judicial power, such power, as everyone
sanction upon an erring student of an institution of higher learning. knows, being vested in the Supreme Court and such inferior courts as
x From the foregoing arguments, it is clear that the lower court should may be established by law 12 — the judicial acts done by him were in
have restrained itself from assuming jurisdiction over the petition filed the circumstances indisputably perpetrated without jurisdiction. The
by Nadal. Mandamus is never issued in doubtful cases, a showing of a acts were completely alien to his office as chief executive, and utterly
clear and certain right on the part of the petitioner being required. 38 It beyond the permissible scope of the legislative power that he had
is of no avail against an official or government agency whose duty assumed as head of the martial law regime.
requires the exercise of discretion or judgment. 39 x Moreover, he had assumed to exercise power — i.e. determined the
x Hence, by issuing the writ of preliminary injunction, the lower court dared to relevant facts and applied the law thereto without a trial at which all
tread upon legally forbidden grounds. For, by virtue of the writ, the University's interested parties were accorded the opportunity to adduce evidence
exercise of academic freedom was peremptorily curtailed. Moreover, the door to furnish the basis for a determination of the facts material to the
was flung wide open for Nadal to do exactly what the decision of the BOR controversy. He made the finding ostensibly on the basis of "the records of the
prohibited him from doing and that is, to violate the suspension order by Bureau of Lands." Prescinding from the fact that there is no indication whatever
enrolling for the first semester of 1993-1994. It must have been with the nature and reliability of these records and that they are in no sense
consternation that the University officials helplessly watching him complete his conclusive, it is undeniable that the petitioner Tuasons (and the petitioners in
academic requirements for taking the Bar. 40 In the event that he be allowed to intervention) were never confronted with those records and afforded a chance to
continue with his studies he would, in effect render moot and academic the dispute their trustworthiness and present countervailing evidence. This is yet
disciplinary sanction of suspension legally imposed upon him by the BOR's final another fatal defect. The adjudication was patently and grossly violative of the
decision of March 29, 1993. What is to prevent other aspirants for STFAP right to due process to which the petitioners are entitled in virtue of the
scholarships from misleading the University authorities by misrepresenting Constitution. Mr. Marcos, in other words, not only arrogated unto himself a
certain facts or as in instant case, withholding vital information and stating power never granted to him by the Constitution or the laws but had in addition
downright falsehoods, in their application forms with impunity? Not only would exercised it unconstitutionally.
this undermine the authority of the U.P. to discipline its students who violated x In any event, this Court has it in its power to treat the petition for
the rules and regulations of the institution but, more importantly, subvert the certiorari as one for prohibition if the averments of the former
very concept and lofty intent to give financial assistance to poor but deserving sufficiently made out a case for the latter. 13 Considered in this wise, it
students through the STFAP which, incidentally, has not ceased refining and will also appear that an executive officer had acted without jurisdiction
modifying it operations. — exercised judicial power not granted to him by the Constitution or
the laws — and had furthermore performed the act in violation of the
ROMAN C. TUASON and REMEDIOS V. TUASON, by attorney-in-fact Trinidad S. constitutional rights of the parties thereby affected. The Court will
Viado, petitioners, vs. REGISTER OF DEEDS, CALOOCAN City, MINISTRY OF JUSTICE, and grant such relief as may be proper and efficacious in the premises even
the NATIONAL TREASURER, respondents. TOMASA BARTOLOME, in her own behalf and in if not specifically sought or set out in the prayer of the appropriate
36
RECTO, GAYLE ANGELI M.
2011-0008 | AUSL
Personal Notes on Remedial Law 2 Review (based on the syllabus of Prof. Henedino M. Brondial)

pleading, the permissible relief being determined after all not by the x MB Æ Issued Resolution No. 27623 placing ECBI under receivership in
prayer but by the basic averments of the parties' pleadings. accordance with the recommendation of the ISD II.
x Vivas Æ Filed petition for prohibition before the Supreme Court.
2. Prohibition
ISSUE: Whether Vivas correctly resorted to a petition for prohibition.
a. grounds
HELD: NO.
b. requirements x To begin with, Vivas availed of the wrong remedy. The MB issued Resolution No.
c. procedure; parties and effects 276, dated March 4, 2010, in the exercise of its power under R.A. No. 7653.
Under Section 30 thereof, any act of the MB placing a bank under
conservatorship, receivership or liquidation may not be restrained or set aside
Section 2. Petition for prohibition. — When the proceedings of any tribunal, corporation, except on a petition for certiorari. Pertinent portions of R.A. 7653 read:
board, officer or person, whether exercising judicial, quasi-judicial or ministerial functions, are o Section 30. -
without or in excess of its or his jurisdiction, or with grave abuse of discretion amounting to ƒ The actions of the Monetary Board taken under this
lack or excess of jurisdiction, and there is no appeal or any other plain, speedy, and adequate section or under Section 29 of this Act shall be final
remedy in the ordinary course of law, a person aggrieved thereby may file a verified petition in and executory, and may not be restrained or set aside
the proper court, alleging the facts with certainty and praying that judgment be rendered by the court except on petition for certiorari on the
commanding the respondent to desist from further proceedings in the action or matter ground that the action taken was in excess of
specified therein, or otherwise granting such incidental reliefs as law and justice may require. jurisdiction or with such grave abuse of discretion as
to amount to lack or excess of jurisdiction. The
The petition shall likewise be accompanied by a certified true copy of the judgment, order or petition for certiorari may only be filed by the
resolution subject thereof, copies of all pleadings and documents relevant and pertinent stockholders of record representing the majority of
thereto, and a sworn certification of non-forum shopping as provided in the third paragraph of the capital stock within ten (10) days from receipt by
section 3, Rule 46. the board of directors of the institution of the order
directing receivership, liquidation or conservatorship.
x Prohibition is already unavailing
ALFEO D. VIVAS, ON HIS BEHALF AND ON BEHALF OF THE SHAREHOLDERS OF x Granting that a petition for prohibition is allowed, it is already an
EUROCREDIT COMMUNITY BANK, PETITIONER, vs. THE MONETARY BOARD OF THE ineffective remedy under the circumstances obtaining. Prohibition or a
BANGKO SENTRAL NG PILIPINAS AND THE PHILIPPINE DEPOSIT INSURANCE "writ of prohibition" is that process by which a superior court prevents
CORPORATION, RESPONDENTS. inferior courts, tribunals, officers, or persons from usurping or
THIRD DIVISION, G.R. No. 191424 August 7, 2013 exercising a jurisdiction with which they have not been vested by law,
and confines them to the exercise of those powers legally conferred.
FACTS: Its office is to restrain subordinate courts, tribunals or persons from
x Rural Bank of Faire, Incorporated (RBFI), is a uly registered rural banking exercising jurisdiction over matters not within its cognizance or
institution with principal office in Centro Sur, Sto. Niño, Cagayan. The corporate exceeding its jurisdiction in matters of which it has cognizance.26 In
life of RBFI expired on May 31, 2005. our jurisdiction, the rule on prohibition is enshrined in Section 2, Rule 65 of the
x Alfeo D. Vivas (Vivas) and his principals Æ Acquired the controlling interest in Rules on Civil Procedure, to wit:
RBFI sometime in January 2006, and tt the initiative of Vivas and the new o Sec. 2. Petition for prohibition - When the proceedings of any
management team, an internal audit was conducted on RBFI, and certain tribunal, corporation, board, officer or person, whether exercising
measures calculated to revitalize the bank were allegedly introduced. judicial, quasi-judicial or ministerial functions, are without or in
x BSP Æ Issued the Certificate of Authority extending the corporate life of RBFI for excess of its or his jurisdiction, or with grave abuse of discretion
another fifty (50) years; and approved the change of its corporate name to amounting to lack or excess of jurisdiction, and there is no appeal
EuroCredit Community Bank, Incorporated, as well as the increase in the number or any other plain, speedy, and adequate remedy in the ordinary
of the members of its BOD, from 5 to 11. course of law, a person aggrieved thereby may file a verified
x Integrated Supervision Department II (ISD II) of the BSP Æ Conducted a general petition in the proper court, alleging the facts with certainty and
examination on ECBI with the cut-off date of December 31, 2007. praying that the judgment be rendered commanding the
x Examiners from the Department of Loans and Credit of the BSP Æ Cancelled the respondent to desist from further proceedings in the action or
rediscounting line of the bank. matter specified therein, or otherwise granting such incidental
x VivasÆ Appealed the cancellation to BSP. reliefs as the law and justice require.
x Monetary Board Æ Issued Resolution No. 1255 (2008), placing ECBI under x Indeed, prohibition is a preventive remedy seeking that a judgment be
Prompt Corrective Action (PCA) framework because of the serious findings and rendered which would direct the defendant to desist from continuing
supervisory concerns noted during the general examination. with the commission of an act perceived to be illegal.27 As a rule, the
x Vivas Æ Moved for a reconsideration of Resolution No. 1255 on the grounds of proper function of a writ of prohibition is to prevent the doing of an act
non-observance of due process and arbitrariness. which is about to be done. It is not intended to provide a remedy for
x BSP Æ Directed ECBI to explain why it transferred the majority shares of RBFI acts already accomplished.28
x Though couched in imprecise terms, this petition for prohibition apparently seeks
without securing the prior approval of the MB in apparent violation of Subsection
X126.2 of the Manual of Regulation for Banks (MORB). to prevent the acts of closing of ECBI and placing it under receivership.
x ISD II Æ Required ECBI to explain why it did not obtain the prior approval of the Resolution No. 276, however, had already been issued by the MB and the closure
BSP anent the establishment and operation of the bank’s sub-offices. of ECBI and its placement under receivership by the PDIC were already
x MB Æ Issued Resolution No. 726 (2009), imposing monetary penalty/fine on accomplished. Apparently, the remedy of prohibition is no longer
ECBI, in view of ECBI’s refusal to comply with the required examination, and appropriate. Settled is the rule that prohibition does not lie to restrain an
referred the matter to the Office of the Special Investigation (OSI) for the filing act that is already a fait accompli
of appropriate legal action. x Even if treated as a petition for certiorari, the petition should have been filed
x BSP Æ Wrote a letter, advising ECBI to comply with MB Resolution No. 771, with the CA. Section 4 of Rule 65 reads:
which essentially required the bank to follow its directives. o Section 4. When and where petition filed. — The petition shall be
x ISD II Æ Reiterated its demand upon the ECBI BOD to allow the BSP examiners filed not later than sixty (60) days from notice of the judgment,
to conduct a general examination. order or resolution. In case a motion for reconsideration or new
x ESBI Æ Asked for another deferment of the examination due to the pendency of trial is timely filed, whether such motion is required or not, the sixty
certain unresolved issues subject of its appeal before the MB, and because Vivas (60) day period shall be counted from notice of the denial of said
was then out of the country. motion.
x ISD II Æ Denied ECBI’s request and ordered the general examination to proceed x The petition shall be filed in the Supreme Court or, if it relates to the acts or
as previously scheduled. omissions of a lower court or of a corporation, board, officer or person, in the
x MB Æ Issued Resolution No. 823 (2009) approving the issuance of a cease and Regional Trial Court exercising jurisdiction over the territorial area as defined by
desist order against ECBI, which enjoined it from pursuing certain acts and the Supreme Court. It may also be filed in the Court of Appeals whether or not
transactions that were considered unsafe or unsound banking practices, and the same is in aid of its appellate jurisdiction, or in the Sandiganbayan if it is in
from doing such other acts or transactions constituting fraud or might result in aid of its appellate jurisdiction. If it involves the acts or omissions of a quasi-
the dissipation of its assets. judicial agency, unless otherwise provided by law or these Rules, the petition
x OSI Æ Filed with the Department of Justice (DOJ) a complaint for Estafa shall be filed in and cognizable only by the Court of Appeals. [Emphases
Through Falsification of Commercial Documents against certain officials and supplied]
x That the MB is a quasi-judicial agency was already settled and reiterated in the
employees of ECBI.
x MB Æ Issued Resolution No. 1164 (2009), denying the appeal of ECBI from case of Bank of Commerce v. Planters Development Bank And Bangko Sentral Ng
Resolution No. 1255 which placed it under PCA framework. Pilipinas.
x ISD II Æ Conducted the general examination of the books and records of ECBI x Even in the absence of such provision, the petition is also dismissible because it
with the cut-off date of September 30, 2009 simply ignored the doctrine of hierarchy of courts. True, the Court, the CA and
x ISD II Æ Reminded ECBI of the non-submission of its financial audit reports for the RTC have original concurrent jurisdiction to issue writs of certiorari,
the years 2007 and 2008 with a warning that failure to submit those reports and prohibition and mandamus. The concurrence of jurisdiction, however, does not
the written explanation for such omission shall result in the imposition of a grant the party seeking any of the extraordinary writs the absolute freedom to
monetary penalty. file a petition in any court of his choice. The petitioner has not advanced any
x ISD II Æ Informed ECBI of MB Resolution No. 1548 which denied its request for special or important reason which would allow a direct resort to this Court. Under
reconsideration of Resolution No. 726. the Rules of Court, a party may directly appeal to this Court only on pure
37
RECTO, GAYLE ANGELI M.
2011-0008 | AUSL
Personal Notes on Remedial Law 2 Review (based on the syllabus of Prof. Henedino M. Brondial)

questions of law.31 In the case at bench, there are certainly factual issues as ISSUE # 1: Whether the authority of a public official to request comment/reply from
Vivas is questioning the findings of the investigating team. petitioners is within the ambit of a writ of prohibition.
x Strict observance of the policy of judicial hierarchy demands that where the
issuance of the extraordinary writs is also within the competence of the CA or the HELD # 1: NO.
RTC, the special action for the obtainment of such writ must be presented to x petitioner Corales was simply required to submit his comment/reply on the
either court. As a rule, the Court will not entertain direct resort to it unless the observations stated in the AOM. As so keenly observed by the Court of Appeals,
redress desired cannot be obtained in the appropriate lower courts; or where any mention in the AOM that petitioner Corales shall reimburse the salaries paid
exceptional and compelling circumstances, such as cases of national interest and to petitioner Dr. Angeles in light of the repeated disapproval or rejection by the
with serious implications, justify the availment of the extraordinary remedy of Sangguniang Bayan of his appointment as Municipal Administrator was merely an
writ of certiorari, prohibition, or mandamus calling for the exercise of its primary initial opinion, not conclusive, as there was no showing that Andal had taken any
jurisdiction.32 The judicial policy must be observed to prevent an imposition on affirmative action thereafter to compel petitioner Corales to make the necessary
the precious time and attention of the Court. reimbursement. Otherwise stated, it has not been shown that Andal carried out
x In any event, no grave abuse of discretion can be attributed to the MB for the or enforced what was stated in the AOM. On the contrary, petitioner Corales was
issuance of the assailed Resolution No. 276. given an opportunity to refute the findings and observations in the AOM by
x Vivas insists that the circumstances of the case warrant the application of Section requesting him to comment/reply thereto, but he never did. More so, even
11 of R.A. No. 7353, which provides: though the AOM already contained a recommendation for the issuance of a
o Sec. 11. The power to supervise the operation of any rural bank by Notice of Disallowance of the payment of salary expenses, the records are bereft
the Monetary Board as herein indicated shall consist in placing of any evidence to show that a Notice of Disallowance has, in fact, been issued.
limits to the maximum credit allowed to any individual borrower; in Concomitantly, the AOM did not contain any recommendation to the effect that
prescribing the interest rate, in determining the loan period and petitioner Corales would be held personally liable for the amount that would be
loan procedures, in indicating the manner in which technical disallowed. It is, therefore, incongruous to conclude that the said AOM is
assistance shall be extended to rural banks, in imposing a uniform tantamount to a directive requiring petitioner Corales to reimburse the salaries
accounting system and manner of keeping the accounts and paid to and received by petitioner Dr. Angeles during the latter’s stint as
records of rural banks; in instituting periodic surveys of loan and Municipal Administrator after his appointment thereto was held invalid for want
lending procedures, audits, test-check of cash and other of conformity from the Sangguniang Bayan.
transactions of the rural banks; in conducting training courses for x In relation thereto, as aptly observed by the OSG, to which the Court of Appeals
personnel of rural banks; and, in general, in supervising the conformed, the issuance of the AOM is just an initiatory step in the investigative
business operations of the rural banks. audit being conducted by Andal as Provincial State Auditor to determine the
o The Central Bank shall have the power to enforce the laws, orders, propriety of the disbursements made by the Municipal Government of Laguna.
instructions, rules and regulations promulgated by the Monetary That the issuance of an AOM can be regarded as just an initiatory step in the
Board, applicable to rural banks; to require rural banks, their investigative audit is evident from COA Memorandum No. 2002-053 dated 26
directors, officers and agents to conduct and manage the affairs of August 2002.24 A perusal of COA Memorandum No. 2002-053, particularly
the rural banks in a lawful and orderly manner; and, upon proof Roman Numeral III, Letter A, paragraphs 1 to 5 and 9, reveals that any finding
that the rural bank or its Board of Directors, or officers are or observation by the Auditor stated in the AOM is not yet conclusive, as the
conducting and managing the affairs of the bank in a manner comment/justification25 of the head of office or his duly authorized
contrary to laws, orders, instructions, rules and regulations representative is still necessary before the Auditor can make any conclusion. The
promulgated by the Monetary Board or in a manner substantially Auditor may give due course or find the comment/justification to be without
prejudicial to the interest of the Government, depositors or merit but in either case, the Auditor shall clearly state the reason for the
creditors, to take over the management of such bank when conclusion reached and recommendation made. Subsequent thereto, the Auditor
specifically authorized to do so by the Monetary Board after due shall transmit the AOM, together with the comment or justification of the Auditee
hearing process until a new board of directors and officers are and the former’s recommendation to the Director, Legal and Adjudication Office
elected and qualified without prejudice to the prosecution of the (DLAO), for the sector concerned in Metro Manila and/or the Regional Legal and
persons responsible for such violations under the provisions of Adjudication Cluster Director (RLACD) in the case of regions. The transmittal
Sections 32, 33 and 34 of Republic Act No. 265, as amended. shall be coursed through the Cluster Director concerned and the Regional Cluster
Director, as the case may be, for their own comment and recommendation. The
ROSENDO R. CORALES, IN HIS OFFICIAL CAPACITY AS MUNICIPAL MAYOR OF DLAO for the sector concerned in the Central Office and the RLACD shall make
NAGCARLAN, LAGUNA, AND DR. RODOLFO R. ANGELES, IN HIS OFFICIAL CAPACITY AS the necessary evaluation of the records transmitted with the AOM. When, on the
MUNICIPAL ADMINISTRATOR OF NAGCARLAN, LAGUNA, PETITIONERS, vs. REPUBLIC OF basis thereof, he finds that the transaction should be suspended or disallowed,
THE PHILIPPINES, REPRESENTED BY THE COMMISSION ON AUDIT, AS REPRESENTED BY he will then issue the corresponding Notice of Suspension (NS), Notice of
PROVINCIAL STATE AUDITOR OF LAGUNA MAXIMO L. ANDAL, RESPONDENT. Disallowance (ND) or Notice of Charge (NC), as the case may be, furnishing a
EN BANC, G.R. No. 186613 August 27, 2013 copy thereof to the Cluster Director. Otherwise, the Director may dispatch a
team to conduct further investigation work to justify the contemplated action. If
after in-depth investigation, the DLAO for each sector in Metro Manila and the
FACTS: RLACD for the regions find that the issuance of the NS, ND, and NC is warranted,
x Corales Æ Elected Municipal Mayor of Nagcarlan, Laguna for three (3) he shall issue the same and transmit such NS, ND or NC, as the case may be, to
consecutive terms, i.e., the 1998, 2001 and 2004 elections. the agency head and other persons found liable therefor.
x Corales Æ Appointed Dr. Angeles to the position of Municipal Administrator, x From the foregoing, it is beyond doubt that the issuance of an AOM is, indeed,
whose appointment was unanimously approved by the Sangguniang Bayan of an initial step in the conduct of an investigative audit considering that after its
Nagcarlan, Laguna (Sangguniang Bayan) . issuance there are still several steps to be conducted before a final conclusion
x Corales Æ Renewed the appointment of Dr. Angeles, but Sangguniang Bayan can be made or before the proper action can be had against the Auditee. There
disapprove appointment on the grround of nepotism, as well as the latter’s is, therefore, no basis for petitioner Corales’ claim that his comment thereon
purported unfitness and unsatisfactory performance. would be a mere formality. Further, even though the AOM issued to petitioner
x Dr. Angeles Æ Continued to discharge the functions and duties of a Municipal Corales already contained a recommendation for the issuance of a Notice of
Administrator for which he received an annual salary of P210,012.00. Disallowance, still, it cannot be argued that his comment/reply to the AOM would
x Provincial State Auditor of Laguna Æ Issued an Audit Observation Memorandum, be a futile act since no Notice of Disallowance was yet issued. Again, the records
which recommends an appropriate Notice of Disallowance be issued for the are bereft of any evidence showing that Andal has already taken any affirmative
payment of salary expenses (against Dr. Angeles) incurred without legal basis. action against petitioner Corales after the issuance of the AOM.
Corales was required to comment or reply. x Viewed in this light, this Court can hardly see any actual case or controversy to
x Corales and Angeles Æ Filed a Petition for Prohibition and Mandamus against warrant the exercise of its power of judicial review. Settled is the rule that for
Andal and the then members of the Sangguniang Bayan before the RTC (San the courts to exercise the power of judicial review, the following must be extant:
Pablo City, Laguna, Branch 32) (1) there must be an actual case calling for the exercise of judicial power; (2) the
x OSG Æ Filed a Motion to Dismiss petitioners’ Petition for Prohibition and question must be ripe for adjudication; and (3) the person challenging must have
Mandamus grounded on lack of cause of action, prematurity and non-exhaustion the "standing." An actual case or controversy involves a conflict of legal rights, an
of administrative remedies. assertion of opposite legal claims, susceptible of judicial resolution as
x RTC Æ Denied Motion to Dismiss on the ground that Andal was merely a nominal distinguished from a mere hypothetical or abstract difference or dispute. There
party. must be a contrariety of legal rights that can be interpreted and enforced on the
x OSG Æ Moved for reconsideration. basis of existing law and jurisprudence. Closely related thereto is that the
o RTC Æ Denied motion for reconsideration question must be ripe for adjudication. A question is considered ripe for
x Republic of the Philippines (represented by COA, represented by Andal) Æ Filed adjudication when the act being challenged has had a direct adverse effect on
a Petition for Certiorari with the Court of Appeals. the individual challenging it. The third requisite is legal standing or locus standi,
x CA Æ Granted Petition for Certiorari which has been defined as a personal or substantial interest in the case such
o thereby annulling and setting aside the RTC Orders and, that the party has sustained or will sustain direct injury as a result of the
accordingly, dismissing the Petition for Prohibition with the trial governmental act that is being challenged, alleging more than a generalized
court. grievance. The gist of the question of standing is whether a party alleges "such
x Corales and Angeles Æ Moved for reconsideration. personal stake in the outcome of the controversy as to assure that concrete
o CA → Denied motion for reconsideration adverseness which sharpens the presentation of issues upon which the court
x Corales and Angeles Æ Filed Petition for Review on Certiorari under Rule 45 of depends for illumination of difficult constitutional questions." Unless a person is
the Rules of Court. injuriously affected in any of his constitutional rights by the operation of statute
or ordinance, he has no standing.26

38
RECTO, GAYLE ANGELI M.
2011-0008 | AUSL
Personal Notes on Remedial Law 2 Review (based on the syllabus of Prof. Henedino M. Brondial)

x The requisites of actual case and ripeness are absent in the present case. To o We have consistently declared that the doctrine of exhaustion of
repeat, the AOM issued by Andal merely requested petitioner Corales to administrative remedies is a cornerstone of our judicial system. The
comment/reply thereto.1awp++i1 Truly, the AOM already contained a thrust of the rule is that courts must allow administrative agencies to
recommendation to issue a Notice of Disallowance; however, no Notice of carry out their functions and discharge their responsibilities
Disallowance was yet issued. More so, there was no evidence to show that Andal within the specialized areas of their respective competence. The
had already enforced against petitioner Corales the contents of the AOM. rationale for this doctrine is obvious. It entails lesser expenses and
Similarly, there was no clear showing that petitioners, particularly petitioner provides for the speedier resolution of controversies. Comity and
Corales, would sustain actual or imminent injury by reason of the issuance of the convenience also impel courts of justice to shy away from a dispute
AOM. The action taken by the petitioners to assail the AOM was, indeed, until the system of administrative redress has been completed.
premature and based entirely on surmises, conjectures and speculations that x The 1987 Constitution created the constitutional commissions as independent
petitioner Corales would eventually be compelled to reimburse petitioner Dr. constitutional bodies, tasked with specific roles in the system of governance that
Angeles’ salaries, should the audit investigation confirm the irregularity of such require expertise in certain fields. For COA, this role involves:
disbursements. Further, as correctly pointed out by respondent Republic o The power, authority, and duty to examine, audit, and settle all
in its Memorandum, what petitioners actually assail is Andal’s authority accounts pertaining to the revenue and receipts of, and
to request them to file the desired comment/reply to the AOM, which is expenditures or uses of funds and property, owned or held in trust
beyond the scope of the action for prohibition, as such request is by, or pertaining to, the Government, or any of its subdivisions,
neither an actionable wrong nor constitutive of an act perceived to be agencies, instrumentalities, including government-owned and
illegal. Andal, being the Provincial State Auditor, is clothed with the controlled corporations with original charter. x x x.
authority to audit petitioners’ disbursements, conduct an investigation x As one of the three (3) independent constitutional commissions, COA has been
thereon and render a final finding and recommendation thereafter. empowered to define the scope of its audit and examination and to establish the
Hence, it is beyond question that in relation to his audit investigation techniques and methods required therefor; and to promulgate accounting and
function, Andal can validly and legally require petitioners to submit auditing rules and regulations, including those for the prevention and
comment/reply to the AOM, which the latter cannot pre-empt by disallowance of irregular, unnecessary, excessive, extravagant or unconscionable
prematurely seeking judicial intervention, like filing an action for expenditures or uses of government funds and properties.
prohibition. x Thus, in the light of this constitutionally delegated task, the courts must exercise
x Moreover, prohibition, being a preventive remedy to seek a judgment caution when intervening with disputes involving these independent bodies, for
ordering the defendant to desist from continuing with the commission the general rule is that before a party may seek the intervention of the court, he
of an act perceived to be illegal, may only be resorted to when there is should first avail of all the means afforded him by administrative processes. The
"no appeal or any other plain, speedy, and adequate remedy in the issues which administrative agencies are authorized to decide should not be
ordinary course of law. summarily taken from them and submitted to a court without first giving such
administrative agency the opportunity to dispose of the same after due
ISSUE # 2: Whether the peititioners exhausted all remedies before resorting to judicial deliberation.33 (Emphasis supplied).
intervention, through the filing of the petition for prohibition. x In their futile attempt to convince this Court to rule in their favor, petitioners
aver that by filing a Motion to Dismiss on the ground of lack of cause of action,
HELD # 2: NO. respondent Republic, in essence, admitted all the material averments and
x In this case, petitioners insist that it is no longer necessary to exhaust narration of facts stated in the Petition for Prohibition and Mandamus. As such,
administrative remedies considering that there is no appeal or any other plain, there is no longer any question of fact to speak of and what remains is a pure
speedy and appropriate remedial measure to assail the imposition under the question of law. The judgment, therefore, of the trial court denying the Motion to
AOM aside from an action for prohibition. Dismiss is no longer subject to any appeal or review by the Court of Appeals.
o This Court finds the said contention plain self-deception. Instead, it is already appealable and reviewable by this Court under Rule 45 of
x As previously stated, petitioners’ action for prohibition was premature. the Rules of Court, where only pure questions of law may be raised and dealt
The audit investigative process was still in its initial phase. There was with. This is in line with the pronouncement in China Road and Bridge
yet no Notice of Disallowance issued. And, even granting that the AOM Corporation v. Court of Appeals34 (China Road Case). The Court of Appeals
issued to petitioner Corales is already equivalent to an order, decision should have dismissed respondent Republic’s Petition for Certiorari under Rule 65
or resolution of the Auditor or that such AOM is already tantamount to of the Rules of Court for being an improper and inappropriate mode of review.
a directive for petitioner Corales to reimburse the salaries paid to o Petitioners’ above argument is misplaced.
petitioner Dr. Angeles, still, the action for prohibition is premature x China Road Case is not at all applicable in the case at bench. Therein, the Motion
since there are still many administrative remedies available to to Dismiss the Complaint was granted. As the order granting the motion to
petitioners to contest the said AOM. Section 1, Rule V of the 1997 dismiss was a final, as distinguished from an interlocutory order, the proper
Revised Rules of Procedure of the COA, provides: "[a]n aggrieved party remedy was an appeal in due course.35 Thus, this Court in China Road Case held
may appeal from an order or decision or ruling rendered by the Auditor that:
embodied in a report, memorandum, letter, notice of disallowances and o x x x Applying the test to the instant case, it is clear that private
charges, Certificate of Settlement and Balances, to the Director who respondent raises pure questions of law which are not proper in an
has jurisdiction over the agency under audit." From the final order or ordinary appeal under Rule 41, but should be raised by way of a
decision of the Director, an aggrieved party may appeal to the petition for review on certiorari under Rule 45.1âwphi1
Commission proper.28 It is the decision or resolution of the x We agree with private respondent that in a motion to dismiss due to failure to
Commission proper which can be appealed to this Court.29 state a cause of action, the trial court can consider all the pleadings filed,
x Clearly, petitioners have all the remedies available to them at the including annexes, motions and the evidence on record. However in so doing,
administrative level but they failed to exhaust the same and instead, the trial court does not rule on the truth or falsity of such documents. It merely
immediately sought judicial intervention. Otherwise stated, the includes such documents in the hypothetical admission. Any review of a finding
auditing process has just begun but the petitioners already thwarted of lack of cause of action based on these documents would not involve a
the same by immediately filing a Petition for Prohibition. In Fua, Jr. v. calibration of the probative value of such pieces of evidence but would only limit
COA,30 citing Sison v. Tablang,31 this Court declared that the general itself to the inquiry of whether the law was properly applied given the facts and
rule is that before a party may seek the intervention of the court, he these supporting documents. Therefore, what would inevitably arise from such a
should first avail himself of all the means afforded him by review are pure questions of law, and not questions of fact.36 (Emphasis
administrative processes. The issues which administrative agencies are supplied).
authorized to decide should not be summarily taken from them and x In the case at bench, however, the Motion to Dismiss was denied. It is well-
submitted to the court without first giving such administrative agency entrenched that an order denying a motion to dismiss is an interlocutory order
the opportunity to dispose of the same after due deliberation. Also, in which neither terminates nor finally disposes of a case as it leaves something to
The Special Audit Team, Commission on Audit v. Court of Appeals and be done by the court before the case is finally decided on the merits.37
Government Service Insurance System,32 this Court has extensively pronounced Therefore, contrary to the claim of petitioners, the denial of a Motion to Dismiss
that: is not appealable, not even via Rule 45 of the Rules of Court. The only remedy
o If resort to a remedy within the administrative machinery can still for the denial of the Motion to Dismiss is a special civil action for certiorari
be made by giving the administrative officer concerned every showing that such denial was made with grave abuse of discretion.38
opportunity to decide on a matter that comes within his or her x Taking into consideration all the foregoing, this Court finds no reversible error on
jurisdiction, then such remedy should be exhausted first before the the part of the Court of Appeals in reversing the Orders of the court a quo and
court’s judicial power can be sought. The premature invocation of consequently dismissing petitioners’ Petition for Prohibition filed thereat
the intervention of the court is fatal to one’s cause of action. The
doctrine of exhaustion of administrative remedies is based on
practical and legal reasons. The availment of administrative remedy ROLANDO TAN, ELENA TAN and LAMBERTO TAN, petitioners, vs. THE HONORABLE
entails lesser expenses and provides for a speedier disposition of COURT OF APPEALS, HON. HERMES B. MONTERO, in his capacity as Assistant Provincial
controversies. Furthermore, the courts of justice, for reasons of Prosecutor, and the PEOPLE OF THE PHILIPPINES, respondents.
comity and convenience, will shy away from a dispute until the THIRD DIVISION, G.R. No. 164966 June 8, 2007
system of administrative redress has been completed and complied
with, so as to give the administrative agency concerned every
opportunity to correct its error and dispose of the case. x x x. FACTS:
x Moreover, courts have accorded respect for the specialized ability of other x James L. King (King) lent money to spouses Roderick Lim Go and Grace Tan-Go
agencies of government to deal with the issues within their respective (spouses Go), who proposed to him a business transaction wherein the spouses
specializations prior to any court intervention. The Court has reasoned thus: Go would borrow cash from King in exchange for which Roderick Go would issue

39
RECTO, GAYLE ANGELI M.
2011-0008 | AUSL
Personal Notes on Remedial Law 2 Review (based on the syllabus of Prof. Henedino M. Brondial)

postdated checks corresponding to the amount borrowed plus interest. Checks Secretary of Justice Simeon A. Datumanong subsequently issued a Resolution19
were honored by the bank. dated December 23, 2003 granting Grace Tan-Go’s petition for review. The
x King lent P40M to the spouses Go and Rolando Tan (father of Grace Tan-Go), aforesaid Resolution was, likewise, favorable to petitioners’ cause.
Elena Tan (mother of Grace Tan-Go), allegedly for the renovation of their movie x When King moved for reconsideration of the above Resolution, petitioners
houses in Butuan City. Go issued several checks to King in the amount of P61.28 participated in the proceedings before the Secretary of Justice by opposing the
Million, inclusive of the interest for three months. Initial checks were honored by same together with Grace Tan-Go.21 In a Resolution22 dated February 11, 2004,
the bank, but subsequent checks were dishonored for having been drawn against then Acting Secretary of Justice Merceditas N. Guitierrez granted King’s motion
insufficient funds. for reconsideration and reinstated public respondent Montero’s Joint Resolution
x King Æ Filed a complaint for violation of BP Blg. 22 and Estafa, after demands dated November 8, 2002. Grace Tan-Go then filed a motion for reconsideration
were unheeded. which was joined by petitioners through their motion for leave to join the motion
x Prosecutor (Montero) Æ Found probable cause, and caused the filing of an for reconsideration.23 However, Acting Secretary Guiterrez denied the same in a
information. Resolution dated August 18, 2004. Thereafter, Grace Tan-Go filed a motion to
x Go, et al. Æ Posted bail, before any warrant of arrest could be issued. resolve the second ground raised in her motion for reconsideration. In a
x Go, et. al. Æ Pleaded not guilty during arraignment. Resolution24 dated December 17, 2004, Secretary of Justice Raul M. Gonzalez
x Go, et. al. Æ Filed a Petition for Prohibition and Injunction with Preliminary reversed and set aside the February 11, 2004 and August 18, 2004 Resolutions
Injunction and Prayer for Temporary Restraining Order before the Court of of Acting Secretary Gutierrez, and reinstated former Secretary Datumanong’s
Appeals, seeking to restrain the trial court from proceeding with the subject Resolution dated December 23, 2003. Consequently, a motion to withdraw
criminal cases against them and prayed that the same be dismissed. informations25 was filed by the prosecution before the trial court.
o CA Æ Dismissed the case for lack of merit. x By participating in the proceedings before the Secretary of Justice, petitioners
x Go, et. al. Æ Moved for reconsideration. have actively litigated the issues regarding the factual and legal bases of the
o CA Æ Denied motion for reconsideration. finding of probable cause against them as well as the authority of public
respondent Montero to file the subject criminal informations. This is clearly borne
x Go, et. al. Æ Filed petition for review on certiorari with the Supreme Court.
by the tenor of the Resolution dated December 17, 2004 issued by the Secretary
ISSUE: Whether resort to the extraordinary relief of prohibition was proper. of Justice. Yet, these issues are exactly the same issues being raised by
petitioners before this Court through the instant petition which is separate and
HELD: NO. distinct from the proceedings before the Secretary of Justice whose aforesaid
x Basic is the rule that the writ of prohibition is an extraordinary remedy Resolution is not the one before us for review. To reiterate, what is before us for
to prevent the unlawful and oppressive exercise of legal authority and review is the Decision of the Court of Appeals which dismissed the petition for
to provide for a fair and orderly administration of justice.9 It is prohibition filed by petitioners to restrain the trial court from proceeding with the
available only when there is no appeal or any plain, speedy and criminal cases against them.
adequate remedy in the ordinary course of law, and when the x In effect, by taking these two distinct courses of actions, petitioners have
proceedings are done without or in excess of jurisdiction or with grave pursued the same or related causes, prayed for the same or substantially the
abuse of discretion. The petitioner must allege in his petition and same reliefs, and, in the process, have created the possibility of conflicting
establish facts to show that any other existing remedy is not speedy or decisions being rendered by the different fora upon the same issues which is
adequate.10 A remedy is plain, speedy and adequate if it will promptly precisely the evil that the rule on forum-shopping seeks to prevent.26 Doubtless,
relieve the petitioner from the injurious effects of that judgment and they have engaged in a form of forum-shopping. Their attempt to trifle with the
the acts of the tribunal or inferior court.11 Further, the writ will not lie courts and abuse their processes must not be countenanced. As a consequence
to correct errors of judgment but only errors of jurisdiction. As long as of petitioners’ violation of the rule against forum-shopping and in order to
the tribunal acts within its jurisdiction, any alleged errors committed in preserve the laudable objectives of the rule against forum-shopping, the
the exercise of its discretion will amount to nothing more than mere dismissal of the petition for prohibition should be upheld.
errors of judgment which are correctible by a timely appeal.12 In
determining whether a tribunal acted in grave abuse of discretion,
mere abuse of discretion is not enough. There must be grave abuse of 3. Mandamus
discretion as where the tribunal exercised its power in an arbitrary or
despotic manner, by reason of passion or personal hostility, and it must a. grounds
be so patent or gross as would amount to an evasion, or virtual refusal
to perform the duty enjoined, or to act in contemplation of law.13
b. requisites
x In the case at bar, petitioners contend that there was no appeal or other plain, c. procedure; parties and effects
speedy or adequate remedy available in the ordinary course of law because they d. damages
were prevented by the trial court from appealing public respondent Montero’s
Joint Resolution dated November 8, 2002 which found, among others, probable Section 3. Petition for mandamus. — When any tribunal, corporation, board, officer or
cause for estafa against them. They claim that the trial court "forced arraigned" person unlawfully neglects the performance of an act which the law specifically enjoins as a
them on November 19, 2002. This was allegedly done in order to prevent them duty resulting from an office, trust, or station, or unlawfully excludes another from the use and
from appealing the Joint Resolution dated November 8, 2002 to the Secretary of enjoyment of a right or office to which such other is entitled, and there is no other plain,
Justice as a consequence of paragraph 2, section 7 of DOJ Circular No. 7014 speedy and adequate remedy in the ordinary course of law, the person aggrieved thereby may
("2000 National Prosecution Service Rule on Appeal") which provides in part that file a verified petition in the proper court, alleging the facts with certainty and praying that
"[i]f an information has been filed in court pursuant to the appealed resolution, judgment be rendered commanding the respondent, immediately or at some other time to be
the petition shall not be given due course if the accused has already been specified by the court, to do the act required to be done to protect the rights of the petitioner,
arraigned x x x." and to pay the damages sustained by the petitioner by reason of the wrongful acts of the
o We are not persuaded. respondent.
x Petitioners admit15 that they received a copy of the Joint Resolution dated
November 8, 2002 as early as November 13, 2002. However, from the time they The petition shall also contain a sworn certification of non-forum shopping as provided in the
received the copy of the aforesaid Resolution to the time they were arraigned on third paragraph of section 3, Rule 46.
November 19, 2002, petitioners did not take steps to move for reconsideration,
or appeal the aforesaid Resolution to the Secretary of Justice. More importantly,
the Court of Appeals observed that there is no evidence on record to support
petitioners’ claim that they were "forced arraigned." In fact, the arraignment of
petitioners proceeded without objections on the part of petitioners or their NILO HIPOS, SR. REPRESENTING DARRYL HIPOS, BENJAMIN CORSIÑO REPRESENTING
counsel.16 Absent proof of force or intimidation, the trial judge enjoys the JAYCEE CORSIÑO, and ERLINDA VILLARUEL REPRESENTING ARTHUR VILLARUEL, Petitioners,
presumption of regularity in the performance of his functions.17 We also note vs. HONORABLE RTC JUDGE TEODORO A. BAY, Presiding Judge, RTC, Hall of Justice,
that petitioners’ other co-accused, Roderick Lim Go and Grace Tan-Go, were able Quezon City, Branch 86, Respondent.
to timely appeal the Joint Resolution dated November 8, 2002 to the Secretary of THIRD DIVISION, G.R. Nos. 174813-15 March 17, 2009
Justice while petitioners failed to appeal the same before their arraignment.
x In fine, the arguments raised in their petition for prohibition ineluctably shows FACTS:
that petitioners are principally questioning the factual and legal bases of the x Hipos, et al. were charged, through two Informations, for the crime of rape and
finding of probable cause against them. This is but a veiled attempt to litigate one Information for the crime of acts of lasciviousness before Branch 86 of the
issues which should have been timely appealed to the Secretary of Justice via a Regional Trial Court of Quezon City, acting as a Family Court, presided by Judge
petition for review. However, petitioners, through their own fault, failed to avail Bay.
themselves of this remedy. Countless times we have ruled that the extraordinary x Complainants Æ Filed a Motion for Reinvestigation asking Judge Bay to order the
remedy of certiorari or prohibition is not a substitute for a lost appeal.18 This City Prosecutor of Quezon City to study if the proper Informations had been filed
case is no different. against Hipos.
x There is another equally important reason why the instant petition x Judge Bay Æ Granted the Motion and ordered a reinvestigation of the cases.
should be denied outright. After the Court of Appeals issued the x Hipos, et al. Æ Filed their Joint Memorandum to Dismiss the Case[s] before the
assailed Decision dated November 24, 2003 which dismissed City Prosecutor.
petitioners’ petition for prohibition, several supervening events took x Prosecutor (Cruz, Arellano) Æ Issued a Resolution on the reinvestigation
place. affirming the Informations filed against Hipos, et al.
x As earlier noted, petitioners failed to appeal from the Joint Resolution dated x Prosecutor (de Vera) Æ Treated the Joint Memorandum to Dismiss the Case as
November 8, 2002 issued by public respondent Montero which found, among an appeal, reversed the Resolution holding that there was lack of probable
others, probable cause against them for estafa. Only co-accused Grace Tan-Go cause.
and Roderick Go separately and timely appealed to the Secretary of Justice. Then
40
RECTO, GAYLE ANGELI M.
2011-0008 | AUSL
Personal Notes on Remedial Law 2 Review (based on the syllabus of Prof. Henedino M. Brondial)

x Prosecutor (de Vera) Æ Filed a Motion to Withdraw Informations before Judge competence, and discretion of the trial court. The trial
Bay. court is the best and sole judge on what to do with the
o Judge Bay Æ Denied the Motion to Withdraw Informations. case before it. A motion to dismiss the case filed by the
x Hipos, et al. Æ Filed the Petition for Mandamus, without moving for a public prosecutor should be addressed to the court who
reconsideration. has the option to grant or deny the same. Contrary to the
contention of the petitioner, the rule applies to a motion to
ISSUE: Whether the Judge may be compelled to dismiss the case, by way of mandamus and, withdraw the Information or to dismiss the case even
on the ground that the resolution of the prosecutor finding no probable cause. before or after arraignment of the accused. The only
qualification is that the action of the court must not impair
HELD: NO. the substantial rights of the accused or the right of the
x Mandamus is an extraordinary writ commanding a tribunal, People or the private complainant to due process of law.
corporation, board, officer or person, immediately or at some other When the trial court grants a motion of the public
specified time, to do the act required to be done, when the respondent prosecutor to dismiss the case, or to quash the
unlawfully neglects the performance of an act which the law Information, or to withdraw the Information in compliance
specifically enjoins as a duty resulting from an office, trust, or station; with the directive of the Secretary of Justice, or to deny the
or when the respondent excludes another from the use and enjoyment said motion, it does so not out of subservience to or
of a right or office to which the latter is entitled, and there is no other defiance of the directive of the Secretary of Justice but in
plain, speedy and adequate remedy in the ordinary course of law.3 sound exercise of its judicial prerogative.
x As an extraordinary writ, the remedy of mandamus lies only to compel x Petitioners also claim that since Judge Bay granted a Motion for Reinvestigation,
an officer to perform a ministerial duty, not a discretionary one; he should have "deferred to the Resolution of Asst. City Prosecutor De Vera
mandamus will not issue to control the exercise of discretion by a withdrawing the case."11 Petitioners cite the following portion of our Decision in
public officer where the law imposes upon him the duty to exercise his People v. Montesa, Jr.12:
judgment in reference to any manner in which he is required to act, o In the instant case, the respondent Judge granted the motion for
because it is his judgment that is to be exercised and not that of the reinvestigation and directed the Office of the Provincial Prosecutor
court.4 of Bulacan to conduct the reinvestigation. The former was,
x In the case at bar, the act which petitioners pray that we compel the trial court therefore, deemed to have deferred to the authority of the
to do is to grant the Office of the City Prosecutor’s Motion for Withdrawal of prosecution arm of the Government to consider the so-called new
Informations against petitioners. In effect, petitioners seek to curb Judge Bay’s relevant and material evidence and determine whether the
exercise of judicial discretion. information it had filed should stand.13
x There is indeed an exception to the rule that matters involving x Like what was done to our ruling in Sanchez, petitioners took specific statements
judgment and discretion are beyond the reach of a writ of mandamus, from our Decision, carefully cutting off the portions which would expose the real
for such writ may be issued to compel action in those matters, when import of our pronouncements. The Petition for Certiorari in Montesa, Jr. was
refused.5 However, mandamus is never available to direct the exercise directed against a judge who, after granting the Petition for Reinvestigation filed
of judgment or discretion in a particular way or the retraction or by the accused, proceeded nonetheless to arraign the accused; and, shortly
reversal of an action already taken in the exercise of either.6 In other thereafter, the judge decided to dismiss the case on the basis of a Resolution of
words, while a judge refusing to act on a Motion to Withdraw the Assistant Provincial Prosecutor recommending the dismissal of the case. The
Informations can be compelled by mandamus to act on the same, he dismissal of the case in Montesa, Jr. was done despite the disapproval of the
cannot be compelled to act in a certain way, i.e., to grant or deny such Assistant Provincial Prosecutor’s Resolution by the Provincial Prosecutor
Motion. In the case at bar, Judge Bay did not refuse to act on the (annotated in the same Resolution), and despite the fact that the reinvestigation
Motion to Withdraw Informations; he had already acted on it by the latter ordered was still ongoing, since the Resolution of the Assistant
denying the same. Accordingly, mandamus is not available anymore. If Provincial Prosecutor had not yet attained finality. We held that the judge should
petitioners believed that Judge Bay committed grave abuse of have waited for the conclusion of the Petition for Reinvestigation he ordered,
discretion in the issuance of such Order denying the Motion to before acting on whether or not the case should be dismissed for lack of
Withdraw Informations, the proper remedy of petitioners should have probable cause, and before proceeding with the arraignment. Thus, the
been to file a Petition for Certiorari against the assailed Order of Judge continuation of the above paragraph of our Decision in Montesa, Jr. reads:
Bay. o Having done so, it behooved the respondent Judge to wait for a
x Petitioners counter that the above conclusion, which has been argued by the final resolution of the incident. In Marcelo vs. Court of Appeals, this
Solicitor General, is contrary to a ruling of this Court, which allegedly states that Court ruled:
the proper remedy in such cases is a Petition for Mandamus and not Certiorari. ƒ Accordingly, we rule that the trial court in a criminal
Petitioners cite the following excerpt from our ruling in Sanchez v. Demetriou7: case which takes cognizance of an accused's motion
o The appreciation of the evidence involves the use of discretion on for review of the resolution of the investigating
the part of the prosecutor, and we do not find in the case at bar a prosecutor or for reinvestigation and defers the
clear showing by the petitioner of a grave abuse of such discretion. arraignment until resolution of the said motion must
o The decision of the prosecutor may be reversed or modified by the act on the resolution reversing the investigating
Secretary of Justice or in special cases by the President of the prosecutor's finding or on a motion to dismiss based
Philippines. But even this Court cannot order the prosecution of a thereon only upon proof that such resolution is
person against whom the prosecutor does not find sufficient already final in that no appeal was taken thereon to
evidence to support at least a prima facie case. The courts try and the Department of Justice.
absolve or convict the accused but as a rule have no part in the ƒ The resolution of Assistant Provincial Prosecutor Rutor
initial decision to prosecute him. recommending the dismissal of the case never
o The possible exception is where there is an unmistakable showing became final, for it was not approved by the
of grave abuse of discretion that will justify a judicial intrusion into Provincial Prosecutor. On the contrary, the latter
the precincts of the executive. But in such a case the proper disapproved it. As a consequence, the final resolution
remedy to call for such exception is a petition for mandamus, not with respect to the reinvestigation is that of the
certiorari or prohibition.8 (Emphases supplied.) Provincial Prosecutor, for under Section 4, Rule 112 of
x Petitioners have taken the above passage way out of its context. In the case of the Rules of Court, no complaint or information may
Sanchez, Calauan Mayor Antonio Sanchez brought a Petition for Certiorari before be filed or dismissed by an investigating fiscal without
this Court, challenging the order of the respondent Judge therein denying his the prior written authority or approval of the
motion to quash the Information filed against him and six other persons for provincial or city fiscal or chief state prosecutor. Also,
alleged rape and homicide. One of the arguments of Mayor Sanchez was that under Section l(d) of R.A. No. 5180, as amended by
there was discrimination against him because of the non-inclusion of two other P.D. No. 77 and P.D. No. 911.14
persons in the Information. We held that even this Court cannot order the x As can be clearly seen, the statement quoted by petitioners from Montesa, Jr. is
prosecution of a person against whom the prosecutor does not find sufficient not meant to establish a doctrine that the judge should just follow the
evidence to support at least a prima facie case. However, if there was an determination by the prosecutor of whether or not there is probable cause. On
unmistakable showing of grave abuse of discretion on the part of the prosecutors the contrary, Montesa, Jr. states:
in that case, Mayor Sanchez should have filed a Petition for Mandamus to compel o The rule is settled that once a criminal complaint or information is
the filing of charges against said two other persons. filed in court, any disposition thereof, such as its dismissal or the
x In the case at bar, the Petition for Mandamus is directed not against the conviction or acquittal of the accused, rests in the sound discretion
prosecution, but against the trial court, seeking to compel the trial court to grant of the court. While the prosecutor retains the discretion and control
the Motion to Withdraw Informations by the City Prosecutor’s Office. The of the prosecution of the case, he cannot impose his opinion on the
prosecution has already filed a case against petitioners. Recently, in Santos v. court. The court is the best and sole judge on what to do with the
Orda, Jr.,9 we reiterated the doctrine we established in the leading case of case. Accordingly, a motion to dismiss the case filed by the
Crespo v. Mogul,10 that once a criminal complaint or an information is filed in prosecutor before or after the arraignment, or after a
court, any disposition or dismissal of the case or acquittal or conviction of the reinvestigation, or upon instructions of the Secretary of Justice who
accused rests within the jurisdiction, competence, and discretion of the trial reviewed the records upon reinvestigation, should be addressed to
court. Thus, we held: the discretion of the court. The action of the court must not,
o In Crespo v. Mogul, the Court held that once a criminal however, impair the substantial rights of the accused or the right of
complaint or information is filed in court, any disposition of the People to due process of law.
the case or dismissal or acquittal or conviction of the x In a seemingly desperate attempt on the part of petitioners’ counsel, he tries to
accused rests within the exclusive jurisdiction, convince us that a judge is allowed to deny a Motion to Withdraw Informations
41
RECTO, GAYLE ANGELI M.
2011-0008 | AUSL
Personal Notes on Remedial Law 2 Review (based on the syllabus of Prof. Henedino M. Brondial)

from the prosecution only when there is grave abuse of discretion on the part of service, subsequently cleared of the charges against them, applied for
the prosecutors moving for such withdrawal; and that, where there is no grave reinstatement but their applications were not acted upon until the integration of
abuse of discretion on the part of the prosecutors, the denial of the Motion to the PC into the PNP in 1990 when R.A. No. 697523 was enacted. Thus, we no
Withdraw Informations is void. longer speak of the reinstatement of the petitioners to the service because the
x The Court finds no need to discuss in detail the alleged actuations of the Philippine Constabulary no longer exists, but of their employment in the PNP
complainants after the alleged rapes and acts of lasciviousness. The alleged which is, as we held in Gloria v. De Guzman,24 technically an issuance of a new
actuations are evidentiary in nature and should be evaluated after full blown trial appointment. The power to appoint is essentially discretionary to be performed
on the merits. This is necessary to avoid a suspicion of prejudgment against the by the officer in which it is vested according to his best lights, the only condition
accused.22 being that the appointee should possess the qualifications required by law.25
o This statement of petitioners’ counsel is utterly misleading. There is Consequently, it cannot be the subject of an application for a writ of
no such statement in our Decision in Ledesma mandamus.26
x As can be seen, the body of the assailed Order not only plainly stated that the x Furthermore, the petitioners do not have a clear legal right over the issuance of
court found probable cause against the petitioners, but likewise provided an the absorption orders.1âwphi1 They cannot claim the right to be issued an
adequate discussion of the reasons for such finding. Indeed, the general rule is appointment based on the NAPOLCOM issuances, specifically Resolution Nos. 98-
that where there is a conflict between the dispositive portion or the fallo and the 037 and 98-105. Suffice it to state that R.A. No. 6975 clearly provides that the
body of the decision, the fallo controls. However, where the inevitable conclusion power to appoint PNP personnel with the rank of "Police Officer I" to "Senior
from the body of the decision is so clear as to show that there was a mistake in Police Officer IV" to which petitioners may be appointed27 is vested in the PNP
the dispositive portion, the body of the decision will prevail.23 regional director or in the Chief of the PNP as the case may be, and not in the
x In sum, petitioners’ resort to a Petition for Mandamus to compel the NAPOLCOM, thus:
trial judge to grant their Motion to Withdraw Informations is improper. o Section 31. Appointment of PNP Officers and Members.—The
While mandamus is available to compel action on matters involving appointment of the officers and members of the PNP shall be
judgment and discretion when refused, it is never available to direct effected in the following manner:
the exercise of judgment or discretion in a particular way or the ƒ (a) Police Officer I to Senior Police Officer IV.—
retraction or reversal of an action already taken in the exercise of Appointed by the PNP regional director for regional
either.24 The trial court, when confronted with a Motion to Withdraw personnel or by the Chief of the PNP for the national
an Information on the ground of lack of probable cause, is not bound headquarters personnel and attested by the Civil
by the resolution of the prosecuting arm of the government, but is Service Commission.
required to make an independent assessment of the merits of such x Even if, for the sake of argument, petitioners can derive a right from
motion, a requirement satisfied by the respondent judge in the case at NAPOLCOM Resolution Nos. 98-037 and 98-105, still their right
bar collapses and their mandamus petition becomes moot with the
issuance by NAPOLCOM of Resolution No. 99-061 recalling the
approval of their absorption. The trial court should then have
EX-C1C JIMMY B. SANCHEZ and EX-C2C SALVADOR A. METEORO, Petitioners, vs. immediately dismissed the mandamus petition when the OSG
ROBERTO T. LASTIMOSO, in his capacity as DIRECTOR GENERAL OF THE PHILIPPINE submitted a copy of Resolution No. 99-061 because well-settled is the
NATIONAL POLICE, Respondent. rule that courts will not resolve a moot question.29
THIRD DIVISION, G.R. No. 161735 September 25, 2007 x Also improper is the trial court’s declaration that NAPOLCOM Resolution No. 99-
061 is void ab initio. In the petition filed below, only the Chief of the PNP is
FACTS: impleaded as the party-defendant.30 NAPOLCOM was never impleaded. As it was
x Sanchez (a constable in the Philippine Constabulary [PC]) was discharged from the latter, a separate entity, which had issued Resolution No. 99-061,
the service for allegedly losing his service firearm. NAPOLCOM was an indispensable party over which the trial court should have
x Meteoro (also a constable) was likewise discharged from the service for being acquired jurisdiction. Since it was not impleaded, NAPOLCOM remains a stranger
absent without leave. to the case, and strangers are not bound by the judgment rendered by the
x Sanchez and Meteoro Æ Appealed, and both were cleared of all charges. court.31 The absence of an indispensable party renders all subsequent actions of
x Sanchez and Meteoro Æ Applied for reinstatement. the court null and void for want of authority to act, not only as to the absent
x PNP Æ No absorption order yet issued, nor application acted upon. parties but even as to those present..
x Sanchez and Meteoro Æ Instituted 1998, a petition for mandamus in the
Regional Trial Court (RTC) of Quezon City.
x PNP Æ Issued Resolution No. 99-061, recalling the earlier Resolution No. 98-105. SOCIAL JUSTICE SOCIETY (SJS), VLADIMIR ALARIQUE T. CABIGAO, and BONIFACIO S.
x RTC Æ Rendered its Decision declaring as void ab initio NAPOLCOM Resolution TUMBOKON, Petitioners, vs. HON. JOSE L. ATIENZA, JR., in his capacity as Mayor of the
No. 99-061 and ruling in favor of the petitioners; issued writ of mandamus City of Manila, Respondent.
x PNP Æ Appealed. FIRST DIVISION, G.R. No. 156052 March 7, 2007
x CA Æ Reversed the ruling of the trial court
o ruled that a writ of mandamus could not be issued because FACTS:
petitioners had not established with distinct clarity their right to be x Sangguniang Panlungsod of Manila Æ Enacted Ordinance No. 8027. Ordinance
absorbed into the PNP. No. 8027 reclassified the area described therein from industrial to commercial and
x Sanchez and Meteoro Æ Moved for reconsideration. directed the owners and operators of businesses disallowed under Section 1 to
o CA Æ Denied MR cease and desist from operating their businesses within six months from the date
x Sanchez and Meteoro Æ Filed petition for review on certiorari. of effectivity of the ordinance. Among the businesses situated in the area are the
so-called "Pandacan Terminals" of the oil companies Caltex (Philippines), Inc.,
ISSUE: Whether the petitioners have a cause of action to compel the PNP Chief to absorb Petron Corporation and Pilipinas Shell Petroleum Corporation.
them into the PNP. x Mayor Atienza Æ Approved the ordinance, which became effective after its
publication.
HELD: NO. x City of Manila and Department of Energy Æ Entered into a memorandum of
x We have repeatedly stressed in our prior decisions that the remedy of understanding (MOU) with the oil companies in which they agreed that "the
mandamus is employed only to compel the performance, when refused, scaling down of the Pandacan Terminals [was] the most viable and practicable
of a ministerial duty, but not to require anyone to fulfill a discretionary option."
one. The issuance of the writ is simply a command to exercise a power x Sangguniang Panlungsod Æ Ratified the MOU in Resolution No. 97. In the same
already possessed and to perform a duty already imposed.20 In Manila resolution, the Sanggunian declared that the MOU was effective only for a period
International Airport Authority v. Rivera Village Lessee Homeowners Association, of six months starting July 25, 2002.
Inc.,21 we emphasized, through the erudite and eloquent ponencia of Justice x Sangguniang Panlungsod Æ Adopted Resolution No. 13 extending the validity of
Dante O. Tinga, that the writ can be issued only when the applicant’s Resolution No. 97 to April 30, 2003 and authorizing Mayor Atienza to issue
legal right to the performance of a particular act sought to be special business permits to the oil companies. Resolution No. 13, s. 2003 also
compelled is clear and complete, one which is indubitably granted by called for a reassessment of the ordinance.
law or is inferable as a matter of law, thus: x Social Justice Society Æ Filed the original action for mandamus before the SC
o In order that a writ of mandamus may aptly issue, it is essential praying that Mayor Atienza be compelled to enforce Ordinance No. 8027 and
that, on the one hand, petitioner has a clear legal right to the claim order the immediate removal of the terminals of the oil companies.
that is sought and that, on the other hand, respondent has an
imperative duty to perform that which is demanded of him. ISSUE: Whether the Mayor may be compelled via mandamus to enforce the Ordinance,
Mandamus will not issue to enforce a right, or to compel notwithstanding the MOU made and resolutions subsequently issued after the promulgation of
compliance with a duty, which is questionable or over which a the Ordinance.
substantial doubt exists. The principal function of the writ of
mandamus is to command and to expedite, not to inquire and to HELD: YES.
adjudicate. Thus, it is neither the office nor the aim of the writ to x Under Rule 65, Section 3 of the Rules of Court, a petition for mandamus may be
secure a legal right but to implement that which is already filed when any tribunal, corporation, board, officer or person unlawfully neglects
established. Unless the right to relief sought is unclouded, the performance of an act which the law specifically enjoins as a duty resulting
mandamus will not issue.22 from an office, trust or station. Mandamus is an extraordinary writ that is
x Viewed in light of the said guideposts, the PNP Chief’s issuance of the orders for employed to compel the performance, when refused, of a ministerial duty that is
the absorption of herein petitioners in the police force is not compellable by a already imposed on the respondent and there is no other plain, speedy and
writ of mandamus precisely because the same does not involve a performance of adequate remedy in the ordinary course of law. The petitioner should have a
a ministerial duty. Let it be noted that petitioners were discharged from the PC well-defined, clear and certain legal right to the performance of the act and it
42
RECTO, GAYLE ANGELI M.
2011-0008 | AUSL
Personal Notes on Remedial Law 2 Review (based on the syllabus of Prof. Henedino M. Brondial)

must be the clear and imperative duty of respondent to do the act required to be "constitutional right to information on matters of public concern." He made the
done.17 request on the belief that the MECO, being under the "operational supervision" of
x Mandamus will not issue to enforce a right, or to compel compliance the Department of Trade and Industry (DTI), is a government owned and
with a duty, which is questionable or over which a substantial doubt controlled corporation (GOCC) and thus subject to the audit jurisdiction of the
exists. The principal function of the writ of mandamus is to command COA.
and to expedite, not to inquire and to adjudicate; thus, it is neither the x COA (Naranjo) Æ Issued a memorandum referring the petitioner’s request to
office nor the aim of the writ to secure a legal right but to implement COA Assistant Commissioner Emma M. Espina for "further disposition." In this
that which is already established. Unless the right to the relief sought is memorandum, however, Assistant Commissioner Naranjo revealed that the
unclouded, mandamus will not issue.18 MECO was "not among the agencies audited by any of the three Clusters of the
x To support the assertion that petitioners have a clear legal right to the Corporate Government Sector."
enforcement of the ordinance, petitioner SJS states that it is a political party x Funa Æ Taking the 25 August 2010 memorandum as an admission that the COA
registered with the Commission on Elections and has its offices in Manila. It had never audited and examined the accounts of the MECO, he filed the petition
claims to have many members who are residents of Manila. The other for mandamus before the SCto compel:
petitioners, Cabigao and Tumbokon, are allegedly residents of Manila. o (1) the Commission on Audit (COA) to audit and examine the funds
x We need not belabor this point. We have ruled in previous cases that when a of the Manila Economic and Cultural Office (MECO), and
mandamus proceeding concerns a public right and its object is to compel a public o (2) the MECO to submit to such audit and examination.
duty, the people who are interested in the execution of the laws are regarded as
the real parties in interest and they need not show any specific interest.19 ISSUE # 1: Whether the mandamus case should be dismissed, considering that the COA has
Besides, as residents of Manila, petitioners have a direct interest in the already obligated itself to audit MECO.
enforcement of the city’s ordinances. Respondent never questioned the
right of petitioners to institute this proceeding. HELD # 1: NO.
x On the other hand, the Local Government Code imposes upon respondent the x The first preliminary issue relates to the alleged mootness of the instant
duty, as city mayor, to "enforce all laws and ordinances relative to the mandamus petition, occasioned by the COA’s issuance of Office Order No. 2011-
governance of the city.">20 One of these is Ordinance No. 8027. As the chief 698. The COA claims that by issuing Office Order No. 2011-698, it had already
executive of the city, he has the duty to enforce Ordinance No. 8027 as long as it conceded its jurisdiction over the accounts of the MECO and so fulfilled the
has not been repealed by the Sanggunian or annulled by the courts.21 He has no objective of the instant petition.58 The COA thus urges that the instant petition
other choice. It is his ministerial duty to do so. In Dimaporo v. Mitra, Jr.,22 we be dismissed for being moot and academic.59
stated the reason for this: o We decline to dismiss the mandamus petition on the ground of
o These officers cannot refuse to perform their duty on the ground of mootness.
an alleged invalidity of the statute imposing the duty. The reason x A case is deemed moot and academic when, by reason of the occurrence of a
for this is obvious. It might seriously hinder the transaction of supervening event, it ceases to present any justiciable controversy.60 Since they
public business if these officers were to be permitted in all cases to lack an actual controversy otherwise cognizable by courts, moot cases are, as a
question the constitutionality of statutes and ordinances imposing rule, dismissible.61
duties upon them and which have not judicially been declared x The rule that requires dismissal of moot cases, however, is not absolute. It is
unconstitutional. Officers of the government from the highest to subject to exceptions. In David v. Macapagal-Arroyo,62 this Court
the lowest are creatures of the law and are bound to obey it.23 comprehensively captured these exceptions scattered throughout our
x The question now is whether the MOU entered into by respondent with the oil jurisprudence:
companies and the subsequent resolutions passed by the Sanggunian have made o The "moot and academic" principle is not a magical formula that
the respondent’s duty to enforce Ordinance No. 8027 doubtful, unclear or can automatically dissuade the courts in resolving a case. Courts
uncertain. This is also connected to the second issue raised by petitioners, that will decide cases, otherwise moot and academic, if: first, there is a
is, whether the MOU and Resolution Nos. 97, s. 2002 and 13, s. 2003 of the grave violation of the Constitution;63 second, the exceptional
Sanggunian can amend or repeal Ordinance No. 8027. character of the situation and the paramount public interest is
x We need not resolve this issue. Assuming that the terms of the MOU were involved;64 third, when constitutional issue raised requires
inconsistent with Ordinance No. 8027, the resolutions which ratified it and made it formulation of controlling principles to guide the bench, the bar,
binding on the City of Manila expressly gave it full force and effect only until April and the public;65 and fourth, the case is capable of repetition yet
30, 2003. Thus, at present, there is nothing that legally hinders respondent from evading review.66
enforcing Ordinance No. 8027.24 x In this case, We find that the issuance by the COA of Office Order No. 2011-698
x Ordinance No. 8027 was enacted right after the Philippines, along with the rest indeed qualifies as a supervening event that effectively renders moot and
of the world, witnessed the horror of the September 11, 2001 attack on the Twin academic the main prayer of the instant mandamus petition. A writ of mandamus
Towers of the World Trade Center in New York City. The objective of the to compel the COA to audit the accounts of the MECO would certainly be a mere
ordinance is to protect the residents of Manila from the catastrophic devastation superfluity, when the former had already obliged itself to do the same.
that will surely occur in case of a terrorist attack25 on the Pandacan Terminals. x Be that as it may, this Court refrains from dismissing outright the petition. We
No reason exists why such a protective measure should be delayed. believe that the mandamus petition was able to craft substantial issues
presupposing the commission of a grave violation of the Constitution and
involving paramount public interest, which need to be resolved nonetheless:
DENNIS A.B. FUNA, Petitioner, vs. MANILA ECONOMIC AND CULTURAL OFFICE and o First. The petition makes a serious allegation that the COA had
the COMMISSION ON AUDIT, Respondents. been remiss in its constitutional or legal duty to audit and examine
EN BANC, G.R. No. 193462 February 4, 2014 the accounts of an otherwise auditable entity in the MECO.
o Second. There is paramount public interest in the resolution of the
FACTS: issue concerning the failure of the COA to audit the accounts of the
x The case stemmed from PH’s adherence to the One China policy of the People’s MECO. The propriety or impropriety of such a refusal is
Republic of China (PROC) determinative of whether the COA was able to faithfully fulfill its
o The Philippines’ commitment to the One China policy of the PROC, constitutional role as the guardian of the public treasury, in which
however, did not preclude the country from keeping unofficial any citizen has an interest.
relations with Taiwan on a "people-to-people" basis.10 Maintaining o Third. There is also paramount public interest in the resolution of
ties with Taiwan that is permissible by the terms of the Joint the issue regarding the legal status of the MECO; a novelty insofar
Communiqué, however, necessarily required the Philippines, and as our jurisprudence is concerned. We find that the status of the
Taiwan, to course any such relations thru offices outside of the MECO—whether it may be considered as a government agency or
official or governmental organs. not—has a direct bearing on the country’s commitment to the One
o Hence, despite ending their diplomatic ties, the people of Taiwan China policy of the PROC.67
and of the Philippines maintained an unofficial relationship facilitated x An allegation as serious as a violation of a constitutional or legal duty, coupled
by the offices of the Taipei Economic and Cultural Office, for the with the pressing public interest in the resolution of all related issues, prompts
former, and the MECO, for the latter. this Court to pursue a definitive ruling thereon, if not for the proper guidance of
x MECO’s purposes are: the government or agency concerned, then for the formulation of controlling
o 1. To establish and develop the commercial and industrial interests principles for the education of the bench, bar and the public in general.68 For
of Filipino nationals here and abroad, and assist on all measures this purpose, the Court invokes its symbolic function.69
designed to promote and maintain the trade relations of the x If the foregoing reasons are not enough to convince, We still add another:
country with the citizens of other foreign countries; o Assuming that the allegations of neglect on the part of the COA
o 2. To receive and accept grants and subsidies that are reasonably were true, Office Order No. 2011-698 does not offer the strongest
necessary in carrying out the corporate purposes provided they are certainty that they would not be replicated in the future. In the first
not subject to conditions defeatist for or incompatible with said place, Office Order No. 2011-698 did not state any legal
purpose; justification as to why, after decades of not auditing the accounts
o 3. To acquire by purchase, lease or by any gratuitous title real and of the MECO, the COA suddenly decided to do so. Neither does it
personal properties as may be necessary for the use and need of state any determination regarding the true status of the MECO. The
the corporation, and to dispose of the same in like manner when justifications provided by the COA, in fact, only appears in the
they are no longer needed or useful; and memorandum70 it submitted to this Court for purposes of this
o 4. To do and perform any and all acts which are deemed case.
reasonably necessary to carry out the purposes. x Thus, the inclusion of the MECO in Office Order No. 2011-698 appears to be
x Funa (a concerned citizen) sent a letter to the COA requesting for a "copy of the entirely dependent upon the judgment of the incumbent chairperson of the COA;
latest financial and audit report" of the MECO invoking, for that purpose, his susceptible of being undone, with or without reason, by her or even her
43
RECTO, GAYLE ANGELI M.
2011-0008 | AUSL
Personal Notes on Remedial Law 2 Review (based on the syllabus of Prof. Henedino M. Brondial)

successor. Hence, the case now before this Court is dangerously capable of order to that effect, copies of which shall be served on all interested parties, and the
being repeated yet evading review. petition shall then be filed within the period ordered by the court. (5a)
o Verily, this Court should not dismiss the mandamus petition on the
ground of mootness. Section 5. When an individual may commence such an action. — A person claiming
to be entitled to a public office or position usurped or unlawfully held or exercised by another
may bring an action therefor in his own name. (6)
ISSUE # 2: Whether Funa correctly filed the present petition
Section 6. Parties and contents of petition against usurpation. — When the action is
HELD # 2: YES. against a person for usurping a public office, position or franchise, the petition shall set forth
x The second preliminary issue is concerned with the standing of the petitioner to the name of the person who claim to be entitled thereto, if any, with an averment of
file the instant mandamus petition. The COA claims that petitioner has none, for his right to the same and that the respondent is unlawfully in possession thereof. All persons
the latter was not able to concretely establish that he had been aggrieved or who claim to be entitled to the public office, position or franchise may be made
prejudiced by its failure to audit the accounts of the MECO.71 parties, and their respective rights to such public office, position or franchise determined, in
x Related to the issue of lack of standing is the MECO’s contention that the same action. (7a)
petitioner has no cause of action to file the instant mandamus petition.
The MECO faults petitioner for not making any demand for it to submit 2. Period
to an audit by the COA or for the COA to perform such an audit, prior to
filing the instant petition.72
Section 8. Period for pleadings and proceedings may be reduced; action given
x We sustain petitioner’s standing, as a concerned citizen, to file the instant
precedence. — The court may reduce the period provided by these Rules for filing pleadings
petition.
and for all other proceedings in the action in order to secure the most expeditious
x The rules regarding legal standing in bringing public suits, or locus standi, are
determination of the matters involved therein consistent with the rights of the parties. Such
already well-defined in our case law. Again, We cite David, which summarizes
action may be given precedence over any other civil matter pending in the court.
jurisprudence on this point:73
(9a)
x By way of summary, the following rules may be culled from the cases decided by
this Court.1a\^/phi1 Taxpayers, voters, concerned citizens, and legislators may
be accorded standing to sue, provided that the following requirements are met: 3. Limitations
o (1) the cases involve constitutional issues;
o (2) for taxpayers, there must be a claim of illegal disbursement of Section 11. Limitations. — Nothing contained in this Rule shall be construed to authorize
public funds or that the tax measure is unconstitutional; an action against a public officer or employee for his ouster from office unless the same be
o (3) for voters, there must be a showing of obvious interest in the commenced within one (1) year after the cause of such ouster, or the right of the petitioner to
validity of the election law in question; hold such office or position, arose, nor to authorize an action for damages in accordance with
o (4) for concerned citizens, there must be a showing that the issues the provisions of the next preceding section unless the same be commenced within one (1)
raised are of transcendental importance which must be settled year after the entry of the judgment establishing the petitioner's right to the office in question.
early; and (16a)
o (5) for legislators, there must be a claim that the official action
complained of infringes upon their prerogatives as legislators. 4. Judgment for Cost
x We rule that the instant petition raises issues of transcendental
importance, involved as they are with the performance of a
Section 12. Judgment for costs. — In an action brought in accordance with the provisions
constitutional duty, allegedly neglected, by the COA. Hence, We hold
of this Rule, the court may render judgment for costs against either the petitioner, the relator,
that the petitioner, as a concerned citizen, has the requisite legal
or the respondent, or the person or persons claiming to be a corporation, or may apportion the
standing to file the instant mandamus petition.
costs, as justice requires. (17a)
x To be sure, petitioner does not need to make any prior demand on the
MECO or the COA in order to maintain the instant petition. The duty of
PEDRO MENDOZA, petitioner, vs. RAY ALLAS and GODOFREDO OLORES, respondents.
the COA sought to be compelled by mandamus, emanates from the
G.R. No. 131977 February 4, 1999
Constitution and law, which explicitly require, or "demand," that it
SECOND DIVISION
perform the said duty. To the mind of this Court, petitioner already
established his cause of action against the COA when he alleged that
FACTS:
the COA had neglected its duty in violation of the Constitution and the
x Mendoza Æ joined Bureau of Customs and held the ff positions:
law.
o Port Security Chief from March 1972 to August 1972
o Deputy Commissioner of Customs from August 1972 to September
_______________________________________________
1975,
o Acting Commissioner of Customs from September 1975 to April
1977 and
QUO WARRANTO [Rule 66] o Customs Operations Chief I from October 1987 to February 1988
x His position of Customs Service Chief was reclassified by the Civil Service as
1. Parties "Director III" in accordance with Republic Act No. 6758 and National
Compensation Circular No. 50. Petitioner's position was thus categorized as
Section 1. Action by Government against individuals. — An action for the usurpation "Director III, CIIS" and he discharged the function and duties of said office.
x Mendoza Æ was then temporarily designated as Acting District Collector,
of a public office, position or franchise may be commenced by a verified petition brought in
the name of the Republic of the Philippines against: Collection District X, Cagayan de Oro City
x Ray Allas Æ was appointed as "Acting Director III" of the CIIS.
(a) A person who usurps, intrudes into, or unlawfully holds or exercises a public x Mendoza Æ then received a letter from Deputy Customs Commissioner Cesar Z.
office, position or franchise; Dario, informing him of his termination from the Bureau of Customs, in view of
respondent Allas' appointment as Director III by President Fidel V. Ramos
(b) A public officer who does or suffers an act which, by the provision of law, x Mendoza Æ wrote to BOC demanding for reinstatement, among others
constitutes a ground for the forfeiture of his office; or o No reply
x Mendoza Æ filed a quo warranto case before RTC
(c) An association which acts as a corporation within the Philippines without being x RTC Æ ruled in favor of Mendoza
legally incorporated or without lawful authority so to act. (1a) o The court found that petitioner was illegally terminated from office
without due process of law and in violation of his security of
Section 2. When Solicitor General or public prosecutor must commence action. — tenure, and that as he was deemed not to have vacated his office,
The Solicitor General or a public prosecutor, when directed by the President of the the appointment of respondent Allas to the same office was void ab
Philippines, or when upon complaint or otherwise he has good reason to believe initio. The court ordered the ouster of respondent Allas from the
that any case specified in the preceding section can be established by proof, must position of Director III, and at the same time directed the
commence such action. (3a) reinstatement of petitioner to the same position with payment of
full back salaries and other benefits appurtenant thereto.
Section 3. When Solicitor General or public prosecutor may commence action with x Allas Æ appealed to CA
permission of court. — The Solicitor General or a public prosecutor may, with the x During pendency of appeal, Allas was promoted by President Ramos to the
permission of the court in which the action is to be commenced, bring such an action at the position of Deputy Commissioner of Customs for Assessment and Operations
request and upon the relation of another person; but in such case the officer bringing it x Mendoza Æ filed an MD on the ground of mootness
may first require an indemnity for the expenses and costs of the action in an amount approved x CA Æ granted MD
by and to be deposited in the court by the person at whose request and upon whose relation x Mendoza Æ filed a Motion for Execution before the RTC
the same is brought. (4a) x RTC Æ denied the motion
o on the ground that the contested position vacated by respondent
Section 4. When hearing had on application for permission to commence action. Allas was now being occupied by respondent Godofredo Olores who
— Upon application for permission to commence such action in accordance with the next was not a party to the quo warranto petition
preceding section, the court shall direct that notice be given to the respondent so that he x Mendoza Æ filed a special civil action for certiorari and mandamus before the CA
may be heard in opposition thereto; and if permission is granted, the court shall issue an x CA Æ dismissed the petition

44
RECTO, GAYLE ANGELI M.
2011-0008 | AUSL
Personal Notes on Remedial Law 2 Review (based on the syllabus of Prof. Henedino M. Brondial)

ISSUE: Whether the RTC decision reinstating Mendoza may bind Olores, the new holder of the constitutionally and legally authorized to perform any act in, or
position, such that the latter may be ousted therefrom, considering that he was not made a exercise any function of the office to which he lays claim. 22 In the
party to the original case. case at bar, the petition for quo warranto was filed by petitioner solely
against respondent Allas. What was threshed out before the trial court
HELD: NO. was the qualification and right of petitioner to the contested position
x The instant petition arose from a special civil action for quo warranto under Rule as against respondent Ray Allas, not against Godofredo Olores. The
66 of the Revised Rules of Court. Quo warranto is a demand made by the state Court of Appeals did not err in denying execution of the trial court's
upon some individual or corporation to show by what right they exercise some decision.
franchise or privilege appertaining to the state which, according to the x Petitioner has apprised this Court that he reached the compulsory retirement age
Constitution and laws of the land, they cannot legally exercise except by virtue of of sixty-five (65) years on November 13, 1997. Reinstatement not being possible,
a grant or authority from the state. 9 In other words, a petition for quo petitioner now prays for the payment of his back salaries and other benefits from
warranto is a proceeding to determine the right of a person to the use the time he was illegally dismissed until finality of the trial court's decision. 23
or exercise of a franchise or office and to oust the holder from its x Respondent Allas cannot be held personally liable for petitioner's back salaries
enjoyment, if his claim is not well-founded, or if he has forfeited his and benefits. He was merely appointed to the subject position by the President
right to enjoy the privilege. 10 The action may be commenced for the of the Philippines in the exercise of his constitutional power as Chief Executive.
Government by the Solicitor General or the fiscal 11 against individuals Neither can the Bureau of Customs be compelled to pay the said back salaries
who usurp a public office, against a public officer whose acts constitute a ground and benefits of petitioner. The Bureau of Customs was not a party to the petition
for the forfeiture of his office, and against an association which acts as a for quo warranto.
corporation without being legally incorporated. 12 The action may also be
instituted by an individual in his own name who claims to be entitled to
the public office or position usurped or unlawfully held or exercised by MA. LUTGARDA P. CALLEJA, JOAQUIN M. CALLEJA, JR., JADELSON PETER P. CALLEJA, MA.
another. 13 JESSICA T. FLORES, MERCIE C. TIPONES and PERFECTO NIXON C. TABORA, Petitioners, vs.
x Where the action is filed by a private person, he must prove that he is JOSE PIERRE A. PANDAY, AUGUSTO R. PANDAY and MA. THELNA P. MALLARI,
entitled to the controverted position, otherwise respondent has a right Respondents.
to the undisturbed possession of the office. 14 If the court finds for the G.R. No. 168696 February 28, 2006
respondent, the judgment should simply state that the respondent is entitled to FIRST DIVISION
the office. 15 If, however, the court finds for the petitioner and declares the
respondent guilty of usurping, intruding into, or unlawfully holding or exercising FACTS:
the office, judgment may be rendered as follows: x Panday and Mallari (respondents) have allegedly been members of the board of
o Sec. 10. Judgment where usurpation found. — When the directors and officers of St. John Hospital, Incorporated
defendant is found guilty of usurping, intruding into, or unlawfully x Calleja et al (also among the incorporators and stockholders of said corporation)
holding or exercising an office, position, right, privilege, or Æ allegedly forcibly and with the aid of armed men usurped the powers which
franchise, judgment shall be rendered that such defendant be supposedly belonged to Respondents.
ousted and altogether excluded therefrom, and that the plaintiff or x Panday et al Æ filed a quo warranto case(with Damages and Prayer for
relator, as the case may be, recover his costs. Such further Mandatory and Prohibitory Injunction, Damages and Issuance of Temporary
judgment may be rendered determining the respective rights in and Restraining Order) before the RTC San Jose, Camarines Sur Br 58
to the office, position, right, privilege, or franchise of all the parties x RTC CamSur Br 58 Æ issued an order transferring the case to RTC Naga
to the action as justice requires. o since the verified petition showed petitioners therein (herein
x If it is found that the respondent or defendant is usurping or intruding into the respondents) to be residents of Naga City, then pursuant to Section
office, or unlawfully holding the same, the court may order: 7, Rule 66 of the 1997 Rules of Civil Procedure, the action for quo
o (1) The ouster and exclusion of the defendant from warranto should be brought in the Regional Trial Court exercising
office; jurisdiction over the territorial area where the respondents or any of
o (2) The recovery of costs by plaintiff or relator; the respondents resides
o (3) The determination of the respective rights in and to x EJ of RTC Naga Æ effused to receive the case
the office, position, right, privilege or franchise of all the parties to o stating that improper venue is not a ground for transferring a quo
the action as justice requires. 16 warranto case to another administrative jurisdiction.
x The character of the judgment to be rendered in quo warranto rests to x RTC Br 58 Æ proceeded with the case
some extent in the discretion of the court and on the relief sought. 17 x RTC Br 58 Æ REMANDED the case to RTC Br 23 Naga City (because the latter
In the case at bar, petitioner prayed for the following relief: has been designated as special court to try and decide intra-corporate
o WHEREFORE, it is respectfully prayed that respondent be ousted controversies under R.A. 8799)
and altogether excluded from the position of Director III, o cause of action involves controversies arising out of intra-corporate
Customs Intelligence and Investigation Service of the Bureau of relations, between and among stockholders, members or associates
Customs, and petitioner be seated to the position as the one of the St. John Hospital Inc. which originally under PD 902-A
legally appointed and entitled thereto. approved on March 11, 1976 is within the original and exclusive
o Other reliefs, just or equitable in the premises, are likewise prayed jurisdiction of the Securities and Exchange Commission to try and
for. 18 decide
x In granting the petition, the trial court ordered that: o A.M. No. 00-11-03-SC which took effect 15 December 2000
o WHEREFORE, viewed in the light of the foregoing, judgment is designated certain branches of the Regional Trial Court to try and
hereby rendered granting this petition for quo warranto by: decide Securities and Exchange Commission Cases arising within
ƒ 1. Ousting and excluding respondent Ray their respective territorial jurisdiction with respect to the National
Allas from the position of Director III, Customs Capital Region and within the respective provinces in the First to
Intelligence and Investigation Service of the Bureau Twelve Judicial Region. Accordingly, in the Province of Camarines
of Customs; and Sur, (Naga City) RTC Branch 23 presided by the Hon. Pablo M.
ƒ 2. Reinstating petitioner Pedro C. Paqueo, Jr. was designated as "special court" (Section 1, A.M. No.
Mendoza, Jr. to the position of Director III, Customs 00-11-03-SC).
Intelligence and Investigation Service of the Bureau of o Administrative Circular No. 8-01 Æ "all SEC cases originally
Customs with full back wages and other monetary assigned or transmitted to the regular Regional Trial Court shall be
benefits appurtenant thereto from the time they were transferred to branches of the Regional Trial Court specially
withheld until reinstated. 19 designated to hear such cases in accordance with A.M. No. 00-11-
x The trial court found that respondent Allas usurped the position of "Director III, 03-SC.
Chief of the Customs Intelligence and Investigation Service." Consequently, the o Under Section 8, of the Interim Rules, [a] Motion to Dismiss is
court ordered that respondent Allas be ousted from the contested position and among the prohibited pleadings. On the otherhand, the Supreme
that petitioner be reinstated in his stead. Although petitioner did not specifically Court under Administrative Order 8-01 has directed the transfer
pray for his back salaries, the court ordered that he be paid his "full back wages from the regular courts to the branches of the Regional Trial Courts
and other monetary benefits" appurtenant to the contested position "from the specially designated to try and decide intra-corporate dispute.
time they were withheld until reinstated." x Calleja et al Æ did not file an MR
x The decision of the trial court had long become final and executory, and
o Filed DIRECTLY a Rule 45 before the SC
petitioner prays for its execution. He alleges that he should have been reinstated
despite respondent Olores' appointment because the subject position was never ISSUE # 1: Whether a Rule 45 is the correct remedy.
vacant to begin with. Petitioner's removal was illegal and he was deemed never
to have vacated his office when respondent Allas was appointed to the same. HELD # 1: NO.
Respondent Allas' appointment was null and void and this nullity allegedly x The Court notes that, indeed, petitioners chose the wrong remedy to assail the
extends to respondent Olores, his successor-in-interest. 20 Order of July 13, 2005. It is hornbook principle that Rule 45 of the 1997 Rules of
x Ordinarily, a judgment against a public officer in regard to a public Civil Procedure governs appeals from judgments or final orders.4 The Order
right binds his successor in office. This rule, however, is not applicable dated July 13, 2005 is basically a denial of herein petitioners’ prayer in their
in quo warranto cases. 21 A judgment in quo warranto does not bind Answer for the dismissal of respondents’ case against them. As a consequence
the respondent's successor in office, even though such successor may of the trial court’s refusal to dismiss the case, it then directed the transfer of the
trace his title to the same source. This follows from the nature of the case to another branch of the Regional Trial Court that had been designated as
writ of quo warranto itself. It is never directed to an officer as such, but a special court to hear cases formerly cognizable by the SEC. Verily, the order
always against the person — to determine whether he is was merely interlocutory as it does not dispose of the case completely, but
45
RECTO, GAYLE ANGELI M.
2011-0008 | AUSL
Personal Notes on Remedial Law 2 Review (based on the syllabus of Prof. Henedino M. Brondial)

leaves something more to be done on its merits. Such being the case, the ƒ (3) Controversies in the election or appointment of
assailed Order cannot ordinarily be reviewed through a petition under Rule 45. directors, trustees, officers, or managers of
As we held in Tolentino v. Natanauan, 5 to wit: corporations, partnerships, or associations;
o In the case of Bangko Silangan Development Bank vs. Court of o SEC. 5. Venue. - All actions covered by these Rules shall be
Appeals, the Court reiterated the well-settled rule that: commenced and tried in the Regional Trial Court which has
ƒ . . . an order denying a motion to dismiss is merely jurisdiction over the principal office of the corporation, partnership,
interlocutory and therefore not appealable, nor can it or association concerned. xxx (Emphasis ours)
be the subject of a petition for review on certiorari. x Pursuant to Section 5.2 of R.A. No. 8799, the Supreme Court promulgated A.M.
Such order may only be reviewed in the ordinary No. 00-11-03-SC (effective December 15, 2000) designating certain branches of
course of law by an appeal from the judgment after the Regional Trial Courts to try and decide cases formerly cognizable by the
trial. The ordinary procedure to be followed in that Securities and Exchange Commission.
event is to file an answer, go to trial, and if the x Subsequently, the Court promulgated A.M. No. 03-03-03-SC, effective July 1,
decision is adverse, reiterate the issue on appeal 2003, which provides that:
from the final judgment.6 o 1. The Regional Courts previously designated as SEC Courts
x It appears, however, that the longer this case remains unresolved, the greater through the: (a) Resolutions of this Court dated 21 November
chance there is for more violence between the parties to erupt. In Philippine 2000, 4 July 2001, 12 November 2002, and 9 July 2002, all issued
Airlines v. Spouses Kurangking,7 the Court proceeded to give due course to a in A.M. No. 00-11-03-SC, (b) Resolution dated 27 August 2001 in
case despite the wrong remedy resorted to by the petitioner therein, stating A.M. No. 01-5-298-RTC; and (c) Resolution dated 8 July 2002 in
thus: A.M. No. 01-12-656-RTC are hereby DESIGNATED and shall be
o While a petition for review on certiorari under Rule 45 would CALLED as Special Commercial Courts to try and decide cases
ordinarily be inappropriate to assail an interlocutory order, in the involving violations of Intellectual Property Rights which fall within
interest, however, of arresting the perpetuation of an apparent their jurisdiction and those cases formerly cognizable by the
error committed below that could only serve to unnecessarily Securities and Exchange Commission;
burden the parties, the Court has resolved to ignore the technical o 4. The Special Commercial Courts shall have jurisdiction over cases
flaw and, also, to treat the petition, there being no other plain, arising within their respective territorial jurisdiction with respect to
speedy and adequate remedy, as a special civil action for the National Capital Judicial Region and within the respective
certiorari. Not much, after all, can be gained if the Court were to provinces with respect to the First to Twelfth Judicial Regions.
refrain from now making a pronouncement on an issue so basic as Thus, cases shall be filed in the Office of the Clerk of Court in the
that submitted by the parties.8 official station of the designated Special Commercial Court;
(Emphasis ours)
ISSUE # 2: Whether Panday et al. correctly resorted to a quo warranto under the Rules of
Court, considering that the subject positions are those of a private corporation. ISSUE # 3: Which court/tribunal may validly take cognizance of the present case?

HELD # 2: NO. HELD # 3: RTC that has jurisdiction over the principal office of the corporation.
x It should be noted that allegations in a complaint for quo warranto that certain x The next question then is, which branch of the Regional Trial Court has
persons usurped the offices, powers and functions of duly elected members of jurisdiction over the present action for quo warrato? Section 5 of the Interim
the board, trustees and/or officers make out a case for an intra-corporate Rules provides that the petition should be commenced and tried in the
controversy.9 Prior to the enactment of R.A. No. 8799, the Court, adopting Regional Trial Court that has jurisdiction over the principal office of the
Justice Jose Y. Feria’s view, declared in Unilongo v. Court of Appeals 10 that corporation. It is undisputed that the principal office of the corporation is
Section 1, Rule 66 of the 1997 Rules of Civil Procedure is "limited to actions of situated at Goa, Camarines Sur. Thus, pursuant to A.M. No. 00-11-03-SC and
quo warranto against persons who usurp a public office, position or franchise; A.M. No. 03-03-03-SC, it is the Regional Trial Court designated as Special
public officers who forfeit their office; and associations which act as corporations Commercial Courts in Camarines Sur which shall have jurisdiction over the
without being legally incorporated," while "[a]ctions of quo warranto against petition for quo warranto filed by herein Respondents.
corporations, or against persons who usurp an office in a corporation, fall under x Evidently, the RTC-Br. 58 in San Jose, Camarines Sur is bereft of jurisdiction over
the jurisdiction of the Securities and Exchange Commission and are governed by respondents’ petition for quo warranto. Based on the allegations in the petition,
its rules. (P.D. No. 902-A as amended)."11 the case was clearly one involving an intra-corporate dispute. The trial court
x However, R.A. No. 8799 was passed and Section 5.2 thereof provides as follows: should have been aware that under R.A. No. 8799 and the aforementioned
o 5.2. The Commission’s jurisdiction over all cases enumerated under administrative issuances of this Court, RTC-Br. 58 was never designated as a
Section 5 of Presidential Decree No. 902-A is hereby transferred to Special Commercial Court; hence, it was never vested with jurisdiction over cases
the Courts of general jurisdiction or the appropriate Regional Trial previously cognizable by the SEC.
Court: Provided, That the Supreme Court in the exercise of its x Such being the case, RTC-Br. 58 did not have the requisite authority or power to
authority may designate the Regional Trial Court branches that order the transfer of the case to another branch of the Regional Trial Court. The
shall exercise jurisdiction over these cases. xxx only action that RTC-Br. 58 could take on the matter was to dismiss the petition
x Therefore, actions of quo warranto against persons who usurp an office for lack of jurisdiction. In HLC Construction and Development Corp. v. Emily
in a corporation, which were formerly cognizable by the Securities and Homes Subdivision Homeowners’ Association,13 the Court held that the trial
Exchange Commission under PD 902-A, have been transferred to the court, having no jurisdiction over the subject matter of the complaint, should
courts of general jurisdiction. But, this does not change the fact that dismiss the same so the issues therein could be expeditiously heard and resolved
Rule 66 of the 1997 Rules of Civil Procedure does not apply to quo by the tribunal which was clothed with jurisdiction.
warranto cases against persons who usurp an office in a private x Note, further, that respondents’ petition for quo warranto was filed as late as
corporation. Presently, Section 1(a) of Rule 66 reads thus: 2005. A.M. No. 03-03-03-SC took effect as early as July 1, 2003 and it was
o Section 1. Action by Government against individuals. - An action clearly provided therein that such petitions shall be filed in the Office of the Clerk
for the usurpation of a public office, position or franchise may be of Court in the official station of the designated Special Commercial Court. Since
commenced by a verified petition brought in the name of the the official station of the designated Special Commercial Court for Camarines Sur
Republic of the Philippines against is the Regional Trial Court in Naga City, respondents should have filed their
ƒ (a) A person who usurps, intrudes into, or unlawfully petition with said court. A.M. No. 00-11-03-SC having been in effect for four
holds or exercises a public office, position or years and A.M. No. 03-03-03-SC having been in effect for almost two years by
franchise; the time respondents filed their petition, there is no cogent reason why
x As explained in the Unilongo12 case, Section 1(a) of Rule 66 of the present Rules respondents were not aware of the appropriate court where their petition should
no longer contains the phrase "or an office in a corporation created by authority be filed.
of law" which was found in the old Rules. Clearly, the present Rule 66 only x The ratiocination of RTC-Br.58 that Administrative Circular No. 08-2001
applies to actions of quo warranto against persons who usurp a public authorized said trial court to order the transfer of respondents’ petition to the
office, position or franchise; public officers who forfeit their office; and Regional Trial Court of Naga City is specious because as of the time of filing of
associations which act as corporations without being legally the petition, A.M. No. 03-03-03-SC, which clearly stated that cases formerly
incorporated despite the passage of R.A. No. 8799. It is, therefore, The cognizable by the SEC should be filed with the Office of the Clerk of Court in the
Interim Rules of Procedure Governing Intra-Corporate Controversies Under R.A. official station of the designated Special Commercial Court, had been in effect for
No. 8799 (hereinafter the Interim Rules) which applies to the petition for quo almost two years. Thus, the filing of the petition with the Regional Trial Court of
warranto filed by respondents before the trial court since what is being San Jose, Camarines Sur, which had no jurisdiction over those kinds of actions,
questioned is the authority of herein petitioners to assume the office and act as was clearly erroneous.
the board of directors and officers of St. John Hospital, Incorporated.
x The Interim Rules provide thus:
o Section 1. (a) Cases covered. - These Rules shall govern the LUIS K. LOKIN, JR., as the second nominee of CITIZENS BATTLE AGAINST CORRUPTION
procedure to be observed in civil cases involving the following: (CIBAC), Petitioner, vs. COMMISSION ON ELECTIONS and the HOUSE OF
ƒ (2) Controversies arising out of intra-corporate, REPRESENTATIVES, Respondents.
partnership, or association relations, between and G.R. Nos. 179431-32 June 22, 2010
among stockholders, members, or associates, and EN BANC
between, any or all of them and the corporation,
partnership, or association of which they are LUIS K. LOKIN, JR., Petitioner, vs. COMMISSION ON ELECTIONS (COMELEC), EMMANUEL
stockholders, members, or associates, respectively; JOEL J. VILLANUEVA, CINCHONA C. GONZALES and ARMI JANE R. BORJE, Respondents.
G.R. No. 180443

46
RECTO, GAYLE ANGELI M.
2011-0008 | AUSL
Personal Notes on Remedial Law 2 Review (based on the syllabus of Prof. Henedino M. Brondial)

FACTS: x Lokin has correctly brought this special civil action for certiorari against the
x Citizens’ Battle Against Corruption (CIBAC) manifested its intent to participate in COMELEC to seek the review of the September 14, 2007 resolution of the
the May 2007 elections COMELEC in accordance with Section 7 of Article IX-A of the 1987 Constitution,
x It then submitted the names of its 5 nominees: notwithstanding the oath and assumption of office by Cruz-Gonzales. The
o (1) Emmanuel Joel J. Villanueva; (2) herein petitioner Luis K. Lokin, constitutional mandate is now implemented by Rule 64 of the 1997 Rules of Civil
Jr.; (3) Cinchona C. Cruz-Gonzales; (4) Sherwin Tugna; and (5) Procedure, which provides for the review of the judgments, final orders or
Emil L. Galang resolutions of the COMELEC and the Commission on Audit. As Rule 64 states, the
x CIBAC then filed a motion to amend the names of its nominees, among others mode of review is by a petition for certiorari in accordance with Rule 65 to be
o withdrew the nominations of Lokin, Tugna and Galang and filed in the Supreme Court within a limited period of 30 days. Undoubtedly, the
substituted Armi Jane R. Borje as one of the nominees Court has original and exclusive jurisdiction over Lokin’s petitions for certiorari
o The amended list of nominees of CIBAC thus included: (1) and for mandamus against the COMELEC.
Villanueva, (2) Cruz-Gonzales, and (3) Borje.
x CIBAC Æ then submitted signed petitions of more than 81% of the CIBAC
members, in order to confirm the withdrawal of the nomination of Lokin, Tugna EFREN RACEL ARATEA, Petitioner, vs. COMMISSiON ON ELECTIONS and ESTELA D.
and Galang and the substitution of Borje ANTlPOLO, Respondents.
o averred that Lokin and Tugna were not among the nominees G.R. No. 195229 October 9, 2012
presented and proclaimed by CIBAC in its proclamation rally held in EN BANC
May 2007; and that Galang had signified his desire to focus on his
family life FACTS:
x CIBAC Æ filed before the COMELEC En Banc a motion seeking the proclamation x Romeo D. Lonzanida (Lonzanida) and Estela D. Antipolo (Antipolo) were
of Lokin as its second nominee candidates for Mayor of San Antonio, Zambales in the May 2010 elections
o The right of CIBAC to a second seat as well as the right of Lokin to x Dra. Sigrid S. Rodolfo (Rodolfo) Æ filed a petition under Section 78 of the
be thus proclaimed were purportedly based on Party-List Canvass Omnibus Election Code to disqualify Lonzanida and to deny due course or to
Report No. 26, which showed CIBAC to have garnered a grand total cancel Lonzanida’s certificate of candidacy
of 744,674 votes. Using all relevant formulas, the motion asserted o on the ground that Lonzanida was elected, and had served, as
that CIBAC was clearly entitled to a second seat and Lokin to a mayor of San Antonio, Zambales for four (4) consecutive terms
proclamation. immediately prior to the term for the May 2010 elections
x Villanueva and Cruz-Gonzales Æ opposed the motion o asserted that Lonzanida made a false material representation in his
x COMELEC Æ failed to act on the matter certificate of candidacy when Lonzanida certified under oath that
x COMELEC En Banc Æ then declared that CIBAC was entitled to an additional seat he was eligible for the office he sought election
o Then resolved E.M. No. 07-054: x COMELEC 2nd division Æ cancelled Lonzanida’s CoC
ƒ WHEREFORE, considering the above discussion, the o Respondent Lonzanida, for holding the office of mayor for more
Commission hereby approves the withdrawal of the than three consecutive terms, went against the three-term limit
nomination of Atty. Luis K. Lokin, Sherwin N. Tugna rule; therefore, he could not be allowed to run anew in the 2010
and Emil Galang as second, third and fourth nominees elections.
respectively and the substitution thereby with Atty. x Lonzanida Æ filed an MR before COMELEC En Banc
Cinchona C. Cruz-Gonzales as second nominee and x Lonzanida and Efren Racel Aratea (Aratea) garnered the highest number of votes
Atty. Armi Jane R. Borje as third nominee for the and were respectively proclaimed Mayor and Vice-Mayor
party list CIBAC. The new order of CIBAC's nominees x Aratea took his oath of office as Acting Mayor before Regional Trial Court (RTC)
therefore shall be: Judge Raymond C. Viray of Branch 75, Olongapo City
x 1. Emmanuel Joel J. Villanueva o Upon his inquiry, he was allowed by DOJ Secretary Jesse M.
x 2. Cinchona C. Cruz-Gonzales Robredo allowed Aratea to take an oath of office as "the permanent
x 3. Armi Jane R. Borje Municipal Mayor of San Antonio, Zambales without prejudice
x THUS, COMELEC en banc proclaimed Cruz-Gonzales as the official second however to the outcome of the cases pending before the
nominee of CIBAC and the latter took oath and assumed office [COMELEC]."
x The COMELEC posits that once the proclamation of the winning party-list x COMELEC EN Banc Æ issued a Reso disqualifying Lonzanida from running for
organization has been done and its nominee has assumed office, any question Mayor in the May 2010 elections
relating to the election, returns and qualifications of the candidates to the House o first, Lonzanida had been elected and had served as Mayor for
of Representatives falls under the jurisdiction of the HRET pursuant to Section more than three consecutive terms without interruption; and
17, Article VI of the 1987 Constitution. Thus, Lokin should raise the question he o second, Lonzanida had been convicted by final judgment of ten
poses herein either in an election protest or in a special civil action for quo (10) counts of falsification under the Revised Penal Code.
warranto in the HRET, not in a special civil action for certiorari in this Court. Lonzanida was sentenced for each count of falsification to
o We do not agree. imprisonment of four (4) years and one (1) day of prisión
x Lokin Æ filed a Rule 65 correccional as minimum, to eight (8) years and one (1) day of
x The COMELEC posits that once the proclamation of the winning party-list prisión mayor as maximum
organization has been done and its nominee has assumed office, any question x COMELEC EN Banc Æ ordered Vice-Mayor Efren Racel Aratea to cease and desist
relating to the election, returns and qualifications of the candidates to the House from discharging the functions of the Office of the Mayor, and to cause a
of Representatives falls under the jurisdiction of the HRET pursuant to Section peaceful turn-over of the said office to Antipolo upon her proclamation
17, Article VI of the 1987 Constitution. Thus, Lokin should raise the question he
poses herein either in an election protest or in a special civil action for quo ISSUE: Whether
warranto in the HRET, not in a special civil action for certiorari in this Court.
HELD:
ISSUE: Whether COMELEC is correct in its position that a quo warranto may be a correct x Lest it be misunderstood, the denial of due course to or the cancellation of the
remedy for Lokin, and not a petition for certiorari. CoC is not based on the lack of qualifications but on a finding that the candidate
made a material representation that is false, which may relate to the
HELD: NO. qualifications required of the public office he/she is running for. It is noted that
x An election protest proposes to oust the winning candidate from office. It is the candidate states in his/her CoC that he/she is eligible for the office he/she
strictly a contest between the defeated and the winning candidates, based on seeks. Section 78 of the OEC, therefore, is to be read in relation to the
the grounds of electoral frauds and irregularities, to determine who between constitutional and statutory provisions on qualifications or eligibility for public
them has actually obtained the majority of the legal votes cast and is entitled to office. If the candidate subsequently states a material representation in the CoC
hold the office. It can only be filed by a candidate who has duly filed a certificate that is false, the COMELEC, following the law, is empowered to deny due course
of candidacy and has been voted for in the preceding elections. to or cancel such certificate. Indeed, the Court has already likened a proceeding
x A special civil action for quo warranto refers to questions of disloyalty under Section 78 to a quo warranto proceeding under Section 253 of the OEC
to the State, or of ineligibility of the winning candidate. The objective since they both deal with the eligibility or qualification of a candidate, with the
of the action is to unseat the ineligible person from the office, but not distinction mainly in the fact that a "Section 78" petition is filed before
to install the petitioner in his place. Any voter may initiate the action, proclamation, while a petition for quo warranto is filed after proclamation of the
which is, strictly speaking, not a contest where the parties strive for winning candidate
supremacy because the petitioner will not be seated even if the x It is obvious from a reading of the laws and jurisprudence that there is an
respondent may be unseated. overlap in the grounds for eligibility and ineligibility vis-à-vis qualifications and
x The controversy involving Lokin is neither an election protest nor an action for disqualifications. For example, a candidate may represent that he is a resident of
quo warranto, for it concerns a very peculiar situation in which Lokin is seeking a particular Philippine locality37 when he is actually a permanent resident of
to be seated as the second nominee of CIBAC. Although an election protest may another country.38 In cases of such overlap, the petitioner should not be
properly be available to one party-list organization seeking to unseat another constrained in his choice of remedy when the Omnibus Election Code explicitly
party-list organization to determine which between the defeated and the winning makes available multiple remedies.39 Section 78 allows the filing of a petition to
party-list organizations actually obtained the majority of the legal votes, Lokin’s deny due course or to cancel a certificate of candidacy before the election, while
case is not one in which a nominee of a particular party-list organization thereby Section 253 allows the filing of a petition for quo warranto after the election.
wants to unseat another nominee of the same party-list organization. Neither Despite the overlap of the grounds, one should not confuse a petition for
does an action for quo warranto lie, considering that the case does not disqualification using grounds enumerated in Section 68 with a petition to deny
involve the ineligibility and disloyalty of Cruz-Gonzales to the Republic due course or to cancel a certificate of candidacy under Section 78.
of the Philippines, or some other cause of disqualification for her. x
47
RECTO, GAYLE ANGELI M.
2011-0008 | AUSL
Personal Notes on Remedial Law 2 Review (based on the syllabus of Prof. Henedino M. Brondial)

x The distinction between a petition under Section 68 and a petition under Section the violation of the three-term limit rule and falsification under the Revised Penal
78 was discussed in Loong v. Commission on Elections40 with respect to the Code, which are obviously not found in the enumeration in Section 68.
applicable prescriptive period. Respondent Nur Hussein Ututalum filed a petition x The dissenting opinions equate Lonzanida’s possession of a disqualifying
under Section 78 to disqualify petitioner Benjamin Loong for the office of condition (violation of the three-term limit rule) with the grounds for
Regional Vice-Governor of the Autonomous Government of Muslim Mindanao for disqualification under Section 68. Section 68 is explicit as to the proper grounds
false representation as to his age. The petition was filed 16 days after the for disqualification: the commission of specific prohibited acts under the Omnibus
election, and clearly beyond the prescribed 25 day period from the last day of Election Code and possession of a permanent residency or immigrant status in a
filing certificates of candidacy. This Court ruled that Ututalum’s petition was one foreign country. Any other false representation regarding a material fact should
based on false representation under Section 78, and not for disqualification be filed under Section 78, specifically under the candidate’s certification of his
under Section 68. Hence, the 25-day prescriptive period provided in Section 78 eligibility. In rejecting a violation of the three-term limit as a condition for
should be strictly applied. We recognized the possible gap in the law: eligibility, the dissenting opinions resort to judicial legislation, ignoring the verba
o It is true that the discovery of false representation as to material legis doctrine and well-established jurisprudence on this very issue.
facts required to be stated in a certificate of candidacy, under x In a certificate of candidacy, the candidate is asked to certify under oath his
Section 74 of the Code, may be made only after the lapse of the eligibility, and thus qualification, to the office he seeks election. Even though the
25-day period prescribed by Section 78 of the Code, through no certificate of candidacy does not specifically ask the candidate for the number of
fault of the person who discovers such misrepresentations and who terms elected and served in an elective position, such fact is material in
would want the disqualification of the candidate committing the determining a candidate’s eligibility, and thus qualification for the office. Election
misrepresentations. It would seem, therefore, that there could to and service of the same local elective position for three consecutive terms
indeed be a gap between the time of the discovery of the renders a candidate ineligible from running for the same position in the
misrepresentation, (when the discovery is made after the 25-day succeeding elections. Lonzanida misrepresented his eligibility because he knew
period under Sec. 78 of the Code has lapsed) and the time when full well that he had been elected, and had served, as mayor of San Antonio,
the proclamation of the results of the election is made. During this Zambales for more than three consecutive terms yet he still certified that he was
so-called "gap" the would-be petitioner (who would seek the eligible to run for mayor for the next succeeding term. Thus, Lonzanida’s
disqualification of the candidate) is left with nothing to do except to representation that he was eligible for the office that he sought election
wait for the proclamation of the results, so that he could avail of a constitutes false material representation as to his qualification or eligibility for the
remedy against the misrepresenting candidate, that is, by filing a office.
petition for quo warranto against him. Respondent Commission x The nature of the eligibility requirements for a local elective office and the
sees this "gap" in what it calls a procedural gap which, according to disqualifications that may apply to candidates necessarily create distinctions on
it, is unnecessary and should be remedied. the remedies available, on the effects of lack of eligibility and on the application
x At the same time, it can not be denied that it is the purpose and intent of the of disqualification. The remedies available are essentially: the cancellation of a
legislative branch of the government to fix a definite time within which petitions CoC, disqualification from candidacy or from holding office, and quo
of protests related to eligibility of candidates for elective offices must be filed, as warranto, which are distinct remedies with varying applicability and effects. For
seen in Sections 78 and 253 of the Code. Respondent Commission may have ease of presentation and understanding, their availability, grounds and effects are
seen the need to remedy this so-called “procedural gap", but it is not for it to topically discussed below.
prescribe what the law does not provide, its function not being legislative. The o As to the grounds:
question of whether the time to file these petitions or protests is too short or x In the denial of due course to or cancellation of a CoC, the ground is essentially
ineffective is one for the Legislature to decide and remedy.41 lack of eligibility under the pertinent constitutional and statutory provisions on
x In Fermin v. Commission on Elections,42 the issue of a candidate’s possession of qualifications or eligibility for public office;20 the governing provisions are
the required one-year residency requirement was raised in a petition for Sections 78 and 69 of the OEC.21
disqualification under Section 68 instead of a petition to deny due course or to x In a disqualification case, as mentioned above, the grounds are traits, conditions,
cancel a certificate of candidacy under Section 78. Despite the question of the characteristics or acts of disqualification,22 individually applicable to a candidate,
one-year residency being a proper ground under Section 78, Dilangalen, the as provided under Sections 68 and 12 of B.P. Blg. 881; Section 40 of LGC 1991;
petitioner before the COMELEC in Fermin, relied on Section 5(C)(1) and and, as discussed below, Section 8, Article X of the Constitution. As previously
5(C)(3)(a)(4) of COMELEC Resolution No. 780043 and filed the petition under discussed, the grounds for disqualification are different from, and have nothing
Section 68. In Fermin, we ruled that "a COMELEC rule or resolution cannot to do with, a candidate’s CoC although they may result in disqualification from
supplant or vary legislative enactments that distinguish the grounds for candidacy whose immediate effect upon finality before the elections is the same
disqualification from those of ineligibility, and the appropriate proceedings to as a cancellation. If they are cited in a petition filed before the elections, they
raise the said grounds."44 A petition for disqualification can only be premised on remain as disqualification grounds and carry effects that are distinctly peculiar to
a ground specified in Section 12 or 68 of the Omnibus Election Code or Section disqualification.
40 of the Local Government Code. Thus, a petition questioning a candidate’s x In a quo warranto petition, the grounds to oust an elected official from
possession of the required one-year residency requirement, as distinguished his office are ineligibility and disloyalty to the Republic of the
from permanent residency or immigrant status in a foreign country, should be Philippines. This is provided under Section 253 of the OEC and
filed under Section 78, and a petition under Section 68 is the wrong remedy. governed by the Rules of Court as to procedures. While quo warranto
x In Munder v. Commission on Elections,45 petitioner Alfais Munder filed a and cancellation share the same ineligibility grounds, they differ as to
certificate of candidacy for Mayor of Bubong, Lanao del Sur on 26 November the time these grounds are cited. A cancellation case is brought before
2009. Respondent Atty. Tago Sarip filed a petition for Munder’s disqualification the elections, while a quo warranto is filed after and may still be filed
on 13 April 2010. Sarip claimed that Munder misrepresented that he was a even if a CoC cancellation case was not filed before elections.
registered voter of Bubong, Lanao del Sur, and that he was eligible to register as x The only difference between the two proceedings is that, under section 78, the
a voter in 2003 even though he was not yet 18 years of age at the time of the qualifications for elective office are misrepresented in the certificate of candidacy
voter’s registration. Moreover, Munder’s certificate of candidacy was not and the proceedings must be initiated before the elections, whereas a petition
accomplished in full as he failed to indicate his precinct and did not affix his for quo warranto under section 253 may be brought on the basis of two grounds
thumb-mark. The COMELEC Second Division dismissed Sarip’s petition and - (1) ineligibility or (2) disloyalty to the Republic of the Philippines, and must be
declared that his grounds are not grounds for disqualification under Section 68 initiated within ten days after the proclamation of the election results. Under
but for denial or cancellation of Munder’s certificate of candidacy under Section section 253, a candidate is ineligible if he is disqualified to be elected to office,
78. Sarip’s petition was filed out of time as he had only 25 days after the filing of and he is disqualified if he lacks any of the qualifications for elective office.23
Munder’s certificate of candidacy, or until 21 December 2009, within which to file x Note that the question of what would constitute acts of disqualification - under
his petition. Sections 68 and 12 of the OEC and Section 40 of LGC 1991 - is best resolved by
x The COMELEC En Banc, however, disqualified Munder. In reversing the COMELEC directly referring to the provisions involved. On the other hand, what constitutes
Second Division, the COMELEC En Banc did not rule on the propriety of Sarip’s a violation of the three-term limit rule under the Constitution has been clarified in
remedy but focused on the question of whether Munder was a registered voter our case law.24 The approach is not as straight forward in a petition to deny due
of Bubong, Lanao del Sur. This Court reinstated the COMELEC Second Division’s course to or cancel a CoC and also to a quo warranto petition, which similarly
resolution. This Court ruled that the ground raised in the petition, lack of covers the ineligibility of a candidate/elected official. In Salcedo II v.
registration as voter in the locality where he was running as a candidate, is COMELEC,25 we ruled that -
inappropriate for a petition for disqualification. We further declared that with our o [I]n order to justify the cancellation of the certificate of candidacy
ruling in Fermin, we had already rejected the claim that lack of substantive under Section 78, it is essential that the false representation
qualifications of a candidate is a ground for a petition for disqualification under mentioned therein pertain to a material matter for the sanction
Section 68. The only substantive qualification the absence of which is a ground for imposed by this provision would affect the substantive rights of a
a petition under Section 68 is the candidate’s permanent residency or candidate — the right to run for the elective post for which he filed
immigrant status in a foreign country. the certificate of candidacy. Although the law does not specify what
x The dissenting opinions place the violation of the three-term limit rule as a would be considered as a "material representation," the Court has
disqualification under Section 68 as the violation allegedly is "a status, interpreted this phrase in a line of decisions applying Section 78 of
circumstance or condition which bars him from running for public office despite the Code.
the possession of all the qualifications under Section 39 of the [Local o Therefore, it may be concluded that the material misrepresentation
Government Code]." In so holding the dissenting opinions write in the law what contemplated by Section 78 of the Code refer to qualifications for
is not found in the law. Section 68 is explicit as to the proper grounds for elective office. This conclusion is strengthened by the fact that the
disqualification under said Section. The grounds for filing a petition for consequences imposed upon a candidate guilty of having made a
disqualification under Section 68 are specifically enumerated in said Section. false representation in his certificate of candidacy are grave — to
However, contrary to the specific enumeration in Section 68 and contrary to prevent the candidate from running or, if elected, from serving, or
prevailing jurisprudence, the dissenting opinions add to the enumerated grounds to prosecute him for violation of the election laws. It could not have
been the intention of the law to deprive a person of such a basic
48
RECTO, GAYLE ANGELI M.
2011-0008 | AUSL
Personal Notes on Remedial Law 2 Review (based on the syllabus of Prof. Henedino M. Brondial)

and substantive political right to be voted for a public office upon x Atty. Francis N. Tolentino, chairperson of the MMDA, issued Office Order No.
just any innocuous mistake. [emphases ours, citation omitted] 106,5 designating Corazon B. Cruz as officer-in-charge (OIC) of the Office of the
x Thus, in addition to the failure to satisfy or comply with the eligibility AGMO
requirements, a material misrepresentation must be present in a cancellation of x De Castro Æ then reassigned to the Legal and Legislative Affairs Office, Office of
CoC situation. The law apparently does not allow material divergence from the the General Manager
listed requirements to qualify for candidacy and enforces its edict by requiring x Carlos Æ was then designated by Atty Tolentino as OIC of the Office of the
positive representation of compliance under oath. Significantly, where AGMO by virtue of Memorandum Order No. 24
disqualification is involved, the mere existence of a ground appears sufficient and x De Castro Æ was stricken off the MMDA payroll, and he was no longer paid his
a material representation assumes no relevance. salary beginning November 2010
o As to the period for filing: o was later offered the position of Director IV of MMDA Public Health
x The period to file a petition to deny due course to or cancel a CoC depends on and Safety Services and/or MMDA consultant. He turned down the
the provision of law invoked. If the petition is filed under Section 78 of the OEC, offer, claiming that it was a demotion in rank.
the petition must be filed within twenty-five (25) days from the filing of the x Pres Noynoy Æ then appointed Carlos as AGMO
CoC.26 However, if the petition is brought under Section 69 of the same law, the x De Castro Æ filed Petition for the issuance of a writ of quo warranto under Rule
petition must be filed within five (5) days from the last day of filing the CoC.27 66 seeking to oust respondent Emerson S. Carlos (respondent) from the position
x On the other hand, the period to file a disqualification case is at any time before ISSUE # 1: Whether De Castro correctly filed the quo warranto case directly to the SC
the proclamation of a winning candidate, as provided in COMELEC Resolution No.
8696.28 The three-term limit disqualification, because of its unique HELD # 1: NO.
characteristics, does not strictly follow this time limitation and is discussed at x As to the procedural issue, petitioner submits that a direct recourse to this Court
length below. At the very least, it should follow the temporal limitations of a quo is warranted by the urgent demands of public interest, particularly the veritable
warranto petition which must be filed within ten (10) days from proclamation.29 need for stability in the civil service and the protection of the rights of civil
The constitutional nature of the violation, however, argues against the servants. Moreover, considering that no other than the President of the Philippines
application of this time requirement; the rationale for the rule and the role of the is the appointing authority, petitioner doubts if a trial court judge or an appellate
Constitution in the country’s legal order dictate that a petition should be allowed court justice, with a prospect of promotion in the judiciary would be willing to go
while a consecutive fourth-termer is in office. against a presidential appointment.
o As to the effects of a successful suit: x Although Section 5(1) of Article VIII of the 1987 Constitution explicitly
x A candidate whose CoC was denied due course or cancelled is not considered a provides that the Supreme Court has original jurisdiction over petitions
candidate at all. Note that the law fixes the period within which a CoC may be for certiorari, prohibition, mandamus, quo warranto, and habeas
filed.30 After this period, generally no other person may join the election contest. corpus, the jurisdiction of this Court is not exclusive but is concurrent
A notable exception to this general rule is the rule on substitution. The with that of the Court of Appeals and regional trial court and does not
application of the exception, however, presupposes a valid CoC. Unavoidably, a give petitioner unrestricted freedom of choice of court forum.16 The
"candidate" whose CoC has been cancelled or denied due course cannot be hierarchy of courts must be strictly observed.
substituted for lack of a CoC, to all intents and purposes.31 Similarly, a x Settled is the rule that "the Supreme Court is a court of last resort and must so
successful quo warranto suit results in the ouster of an already elected official remain if it is to satisfactorily perform the functions assigned to it by the
from office; substitution, for obvious reasons, can no longer apply. fundamental charter and immemorial tradition."17 A disregard of the doctrine of
x On the other hand, a candidate who was simply disqualified is merely prohibited hierarchy of courts warrants, as a rule, the outright dismissal of a petition.18
from continuing as a candidate or from assuming or continuing to assume the x A direct invocation of this Court’s jurisdiction is allowed only when there are
functions of the office; substitution can thus take place under the terms of special and important reasons that are clearly and specifically set forth in a
Section 77 of the OEC.32 However, a three-term candidate with a valid and petition.19 The rationale behind this policy arises from the necessity of
subsisting CoC cannot be substituted if the basis of the substitution is his preventing (1) inordinate demands upon the time and attention of the Court,
disqualification on account of his three-term limitation. Disqualification that is which is better devoted to those matters within its exclusive jurisdiction; and (2)
based on a breach of the three-term limit rule cannot be invoked as this further overcrowding of the Court’s docket.20
disqualification can only take place after election where the three-term official x In this case, petitioner justified his act of directly filing with this Court only when
emerged as winner. As in a quo warranto, any substitution is too late at he filed his Reply and after respondent had already raised the procedural infirmity
this point. that may cause the outright dismissal of the present Petition. Petitioner likewise
o As to the effects of a successful suit on the right of the cites stability in the civil service and protection of the rights of civil servants as
second placer in the elections: rationale for disregarding the hierarchy of courts.
x In any of these three remedies, the doctrine of rejection of the second placer x Petitioner’s excuses are not special and important circumstances that would
applies for the simple reason that - allow a direct recourse to this Court. More so, mere speculation and doubt to the
o To simplistically assume that the second placer would have exercise of judicial discretion of the lower courts are not and cannot be valid
received the other votes would be to substitute our judgment for justifications to hurdle the hierarchy of courts. Thus, the Petition must be
the mind of the voter. The second placer is just that, a second dismissed.
placer. He lost the elections. He was repudiated by either a
majority or plurality of voters. He could not be considered the first ISSUE # 2: Whether quo warranto is the correct remedy.
among qualified candidates because in a field which excludes the
disqualified candidate, the conditions would have substantially HELD # 2: NO.
changed. We are not prepared to extrapolate the results under x "A petition for quo warranto is a proceeding to determine the right of a
such circumstances.33 person to use or exercise a franchise or an office and to oust the holder
x With the disqualification of the winning candidate and the application of the from the enjoyment, thereof, if the claim is not well-founded, or if his
doctrine of rejection of the second placer, the rules on succession under the law right to enjoy the privilege has been forfeited."21 Where the action is
accordingly apply. filed by a private person, in his own name, he must prove that he is
x As an exceptional situation, however, the candidate with the second highest entitled to the controverted position, otherwise, respondent has a right
number of votes (second placer) may be validly proclaimed as the winner in the to the undisturbed possession of the office.22
elections should the winning candidate be disqualified by final judgment before x The controversy arose from the issuance of OP Memorandum Circular Nos. 1 and
the elections, as clearly provided in Section 6 of R.A. No. 6646.34 The same 2, which applies to all non-CESO’s occupying CES positions in all agencies of the
effect obtains when the electorate is fully aware, in fact and in law and within executive branch. Petitioner, being a non-CESO, avers that he is not covered by
the realm of notoriety, of the disqualification, yet they still voted for the these OP memoranda considering that the AGMO of the MMDA is a non-CES
disqualified candidate. In this situation, the electorate that cast the plurality of position.
votes in favor of the notoriously disqualified candidate is simply deemed to have x Therefore, considering that petitioner is an appointee of then President Arroyo
waived their right to vote.35 whose term ended on 30 June 2010, petitioner’s term of office was also deemed
x In a CoC cancellation proceeding, the law is silent on the legal effect of a terminated upon the assumption of President Aquino.
judgment cancelling the CoC and does not also provide any temporal distinction. x Likewise, it is inconsequential that petitioner was allegedly replaced by another
Given, however, the formal initiatory role a CoC plays and the standing it gives to non-CESO eligible. In a quo warranto proceeding, the person suing must
a political aspirant, the cancellation of the CoC based on a finding of its invalidity show that he has a clear right to the office allegedly held unlawfully by
effectively results in a vote for an inexistent "candidate" or for one who is another. Absent a showing of that right, the lack of qualification or
deemed not to be in the ballot. Although legally a misnomer, the "second placer" eligibility of the supposed usurper is immaterial.41
should be proclaimed the winner as the candidate with the highest number of x All the foregoing considered, the petition merits an outright dismissal for
votes for the contested position. This same consequence should result if the disregarding the hierarchy of courts and petitioner’s lack of cause of action
cancellation case becomes final after elections, as the cancellation signifies non- against respondent for failure to sufficiently show that he has undisturbed rights
candidacy from the very start, i.e., from before the elections. to the position of AGMO of the MMDA.

EMMANUEL A. DE CASTRO, Petitioner, vs. EMERSON S. CARLOS, Respondent. EXPROPRIATION [Rule 67]
G.R. No. 194994 April 16, 2013
EN BANC
1. The right of eminent domain
FACTS:
x President GMA appointed De Castro as assistant general manager for operations Section 1. The complaint. — The right of eminent domain shall be exercised by the filing
(AGMO) of the Metropolitan Manila Development Authority (MMDA) of a verified complaint which shall state with certainty the right and purpose of expropriation,
o concurred in by the members of the Metro Manila Council in MMDA describe the real or personal property sought to be expropriated, and join as defendants all
Resolution No. 09-10, Series of 2009 persons owning or claiming to own, or occupying, any part thereof or interest therein,

49
RECTO, GAYLE ANGELI M.
2011-0008 | AUSL
Personal Notes on Remedial Law 2 Review (based on the syllabus of Prof. Henedino M. Brondial)

showing, so far as practicable, the separate interest of each defendant. If the title to any tribunal, board or officer, and granting such incidental reliefs as
property sought to be expropriated appears to be in the Republic of the Philippines, although laws and justice may require.
occupied by private individuals, or if the title is otherwise obscure or doubtful so that the x Respondents' petition before the Court of Appeals alleged that the trial court had
plaintiff cannot with accuracy or certainty specify who are the real owners, averment to that acted without or in excess of its jurisdiction or with grave abuse of discretion
effect shall be made in the complaint. amounting to lack of jurisdiction in issuing the order, dated December 15, 1998,
resolving that Lot 1-C is not exempt from expropriation and ordering the issuance
2. Who may expropriate of the writ of possession in favor of petitioner
x Petitioner faults the Court of Appeals for deciding issues not raised in the trial
court, specifically the question of whether or not there was compliance with §§9
3. Two stages in expropriation and 10 of RA. No. 7279. It argues that the sole defense set up by respondents in
their petition before the Court of Appeals was that their property was exempted
a. Determination of public use from expropriation because it comes within the purview of a "small property" as
b. Just compensation defined by R.A. No. 7279 . Accordingly, the Court of Appeals should not have
applied the doctrine laid down by this Court in the Filstream19 case as such issue
was not raised by respondents in their petition before the Court of Appeals.
CITY OF MANILA, petitioner, vs. OSCAR, FELICITAS, JOSE, BENJAMIN, ESTELITA, x This contention likewise has no merit. In their petition before the Court of
LEONORA AND ADELAIDA, ALL SURNAMED SERRANO, respondents. Appeals, respondents raised the following issues:
G.R. No. 142304 June 20, 2001 o 1. Whether or not the subject Lot 1-C with an area of 343.10
SECOND DIVISION square meters covered by T.C.T. No. 226048 in the name of
petitioners' mother, the late Demetria [De Guia] Serrano, may be
FACTS: lawfully expropriated "for the public purpose of providing landless
x City Council of Manila enacted the Ordinance 7833, authorizing the expropriation occupants thereof homelots of their own under the "land-for-the
of certain properties in Manila 's First District in Tondo, covered by TCT Nos. landless program of respondent City of Manila."
70869, 105201, 105202, and 138273 of the Register of Deeds of Manila, which o 2. Whether or not the expropriation of the said Lot l-C by
are to be sold and distributed to qualified occupants pursuant to the Land Use respondent City of Manila violates the equal protection clause of
Development Program of the City of Manila. the Constitution, since petitioners, with the exemption of petitioner
x One of the properties sought to be expropriated, denominated as Lot 1-C, Oscar G. Serranno, who are likewise landless are actual occupants
consists of 343.10 square meters. It is covered by TCT No. 138272 which was hereof.
o 3. Whether or not Lot 1-C is or may be exempted from
derived from TCT No. 70869 issued in the name of Feliza De Guia. After her expropriation pursuant to R.A. 7279, otherwise known as the Urban
death, the estate of Feliza De Guia was settled among her heirs by virtue of a
compromise agreement, which was duly approved by the Regional Trial Court, Development and Housing Act of 1992.20
Branch 53, Manila in its decision. Subsequently, Alberto De Guia, one of the heirs x It is clear that respondents raised in issue the propriety of the expropriation of
of Feliza De Guia, died, as a result of which his estate, consisting of his share in their property in connection with RA. No. 7279. Although what was discussed at
the properties left by his mother, was partitioned among his heirs. Lot 1-C was length in their petition before the Court of Appeals was whether or not the said
assigned to Edgardo De Guia, one of the heirs of Alberto De Guia. Subsequently, property could be considered a small property within the purview of the
Edgardo De Guia was issued TCT No. 215593, covering Lot 1-C. The said exemption under the said law, the other provisions of the said law concerning
property was transferred to Lee Kuan Hui, in whose name TCT No. 217018 was expropriation proceedings need also be looked into to address the first issue
issued. The property was subsequently sold to Demetria De Guia to whom TCT raised by the respondents and to determine whether or not expropriation of Lot
No. 226048 was issued. 1-C was proper under the circumstances. The Court of Appeals properly
x City of Manila filed an amended complaint for expropriation, with the Regional considered relevant provisions of R A. No.7279 to determine the issues raised by
Trial Court, Branch 16, Manila, against the supposed owners of the lots covered respondents. Whether or not it correctly applied the doctrine laid down in
by TCT Nos. 70869 (including Lot 1-C), 105201, 105202 and 138273, which Filstream in resolving the issues raised by respondents, however, is a different
included the Serranos. On November 12, 1997, respondents filed a consolidated matter altogether, and this brings us to the next point.
answer, praying that judgment be rendered declaring Lot l-C exempt from x Petitioner contends that the Court of Appeals erroneously presumed that Lot 1-C
expropriation and ordering the cancellation of the notice annotated on the back has been ordered condemned in its favor when the fact is that the order of the
of TCT No. 226048, regarding the pendency of the Civil Case for eminent trial court, dated December 15, 1998, merely authorized the issuance of a writ of
domain. possession and petitioner's entry into the property pursuant to Rule 67, §2. At
x Upon motion of the City of Manila, the trial court issued an order, directing the that stage, it was premature to determine whether the requirements of RA. No.
City to deposit the amount of Pl,825,241.00 equivalent to the assessed value of 7279, §§9 - 10 have been complied with since no evidentiary hearing had yet
the properties. After the City had made the deposit, the trial court issued another been conducted by the trial court.21
order, directing the issuance of a writ of possession in favor of the City. x This contention is well taken. Rule 67, §2 provides:
x The Serranos filed a petition for certiorari with the Court of Appeals. o Upon the filing of the complaint or at any time thereafter and after
x The Court of Appeals rendered a decision holding that Lot l-C is not exempt from due notice to the defendant, the plaintiff shall have the right to
expropriation because it undeniably exceeds 300 square meters which is no take or enter upon possession of the real property involved if he
longer considered a small property within the framework of R.A. No. 7279. deposits with the authorized government depository an amount
However, it held that in accordance with the ruling in Filstream International Inc. equivalent to the assessed value of the property for purposes of
v. Court of Appeals, the other modes of acquisition of lands enumerated in §§9- taxation to be held by such bank subject to the orders of the court.
10 of the law must first be tried by the city government before it can resort to Such deposit shall be in money, unless in lieu thereof the court
expropriation. As petitioner failed to show that it had done so, the Court of authorizes the deposit of a certificate of deposit of a government
Appeals gave judgment for respondents and enjoined petitioner from bank of the Republic of the Philippines payable on demand to the
expropriating Lot 1-C. authorized government depositary.
x In its resolution, the Court of Appeals likewise denied two motions for x If personal property is involved, its value shall be provisionally ascertained and
reconsideration filed by the City. the amount to be deposited shall be fixed by the court.
x Hence, the petition for review on certiorari. x After such deposit is made the court shall order the sheriff or other proper officer
to forthwith place the plaintiff in possession of the property involved and
ISSUE: Whether the authorization of the issuance of a writ of possession and petitioner's entry promptly submit a report thereof to the court with service of copies to the
into the property pursuant to Rule 67, §2 by the trial court is tantamount to condemnation of parties.
the subject lot in the expropriator's favor. x Thus, a writ of execution may be issued by a court upon the filing by the
government of a complaint for expropriation sufficient in form and substance and
HELD: NO. upon deposit made by the government of the amount equivalent to the assessed
x Petitioner contends that the respondents' remedy against the order of the trial value of the property subject to expropriation. Upon compliance with these
court granting a writ of possession was not to file a petition for certiorari under requirements, the issuance of the writ of possession becomes ministerial.22 In
Rule 65 but a petition for review under Rule 45 which should have been filed in this case, these requirements were satisfied and, therefore, it became the
the Supreme Court.17 ministerial duty of the court to issue the writ of possession.
o This contention has no merit. x The Court of Appeals, however, ruled that petitioner failed to comply with the
x A petition for review under Rule 45 is a mode of appeal. Accordingly, it could not requirements laid down in §§9 - 10 of RA. No. 7279 and reiterated in Filstream
have been resorted to by the respondents inasmuch as the order of the trial ruling. This is error. The ruling in the Filstream was necessitated because an
court granting a writ of possession was merely interlocutory from which no order of condemnation had already been issued by the trial court in that case.
appeal could be taken. Rule 45, §1 of the 1997 Rules for Civil Procedure applies Thus, the judgment in that case had already become final. In this case, the trial
only to final judgments or orders of the Court of Appeals, the Sandiganbayan, court has not gone beyond the issuance of a writ of possession. Hearing is still to
and the Regional Trial Court. On the other hand, a petition for certiorari is the be held to determine whether or not petitioner indeed complied with the
suitable remedy in view of Rule 65, §1 which provides: requirements provided in RA. No. 7279. It is, therefore, premature at this stage
o When any tribunal, board or officer exercising judicial or quasi- of the proceedings to find that petitioner resorted expropriation without first
judicial functions has acted without or in excess of its or his trying the other modes of acquisition enumerated in § 10 of the law.
jurisdiction, or with grave abuse of discretion amounting to lack or x RA. No 7279 in pertinent parts provide:
o SEC. 9. Priorities in the Acquisition of Land… Lands for socialized
excess of jurisdiction, and there is no appeal, nor any plain, housing shall be acquired in the following order:
speedy, and adequate remedy in the ordinary course of law, a
person aggrieved thereby may file a verified petition in the proper
court, alleging the facts with certainly and praying that judgment ƒ Those owned by the Government or any of its
be rendered annulling or modifying the proceedings of such subdivisions, instrumentalities, or agencies, including
50
RECTO, GAYLE ANGELI M.
2011-0008 | AUSL
Personal Notes on Remedial Law 2 Review (based on the syllabus of Prof. Henedino M. Brondial)

government owned and controlled corporations and x The Intermediate Appellate Court dismissed NPC's petition but directed the lower
their subsidiaries; court to rule on NPC's objections to Pobre's documentary exhibits.
ƒ Alienable lands of the public domain; x The trial court admitted all of Pobre's exhibits and upheld its Order, and
ƒ Unregistered or abandoned and idle lands; considered the case submitted for decision.
ƒ Those within the declares Areas or Priority x The trial court issued its Decision in favor of Pobre.
Development, Zone Improvement Program sites, and x NPC filed its motion for reconsideration of the decision.
Slum Improvement and Resettlement Programs sites x The trial court issued its Order denying NPC's motion for reconsideration.
which have not yet been acquired; x NPC appealed to the Court of Appeals.
ƒ Bagong Lipunan Improvement and Sites and Services x The Court of Appeals upheld the decision of the trial court but deleted the award
or BLISS sites which have not yet been acquired, and; of attorney's fees.
ƒ Privately-owned lands. x The Court of Appeals denied NPC's motion for reconsideration
o Where on-site development is found more practicable and x Hence, the petition for review.
advantageously to the beneficiaries, the priorities mentioned in this
section shall not apply. The local government units shall give ISSUE # 1: Whether Section 1, Rule 17 of the Rules applies squarely in expropriation cases.
budgetary priority on-site development of government lands.
o SEC. 10. Modes of Lands Acquisition. -- The modes of acquiring HELD # 1: NO.
lands for purposes of this Act shall include, amount others, x Section 1, Rule 17 of the 1964 Rules of Court provided the exception to the
community mortgage, land swapping, land assembly or general rule that the dismissal of the complaint is addressed to the sound
consolidation, land banking, donation to the Government, joint- discretion of the court. For as long as all of the elements of Section 1, Rule 17
venture agreement, negotiated purchase, and expropriation: were present the dismissal of the complaint rested exclusively on the plaintiff's
Provided, however; That expropriation shall be resorted to only will. The defending party and even the courts were powerless to prevent the
when other modes of acquisition have been exhausted: Provided, dismissal. The courts could only accept and record the dismissal. A plain reading
further; That were expropriation is resorted to, parcels of land of Section 1, Rule 17 of the 1964 Rules of Court makes it obvious that this rule
owned by small property owners shall be exempted for purposes of was not intended to supplement Rule 67 of the same Rules … While Section 1,
this Act: Provided finally, That abandoned property, as herein Rule 17 spoke of the "service of answer or summary judgment," the Rules then
defined, shall be reverted and escheated to the State in a did not require the filing of an answer or summary judgment in eminent domain
proceeding analogous to the procedure laid down in Rule 91 of the cases. In lieu of an answer, Section 3 of Rule 67 required the defendant to file a
Rules of Court. single motion to dismiss where he should present all of his objections and
o For the purpose of socialized housing, government-owned and defenses to the taking of his property for the purpose specified in the complaint.
foreclosed properties shall be acquired by the local government In short, in expropriation cases under Section 3 of Rule 67, the motion to dismiss
units, or by the National Housing Authority primarily through took the place of the answer. In expropriation cases, there is no such thing as
negotiated purchase: Provided, That qualified beneficiaries who are the plaintiff's matter of right to dismiss the complaint precisely because the
actual occupants of the lands shall be given the right of first landowner may have already suffered damages at the start of the taking. The
refusal. plaintiff's right in expropriation cases to dismiss the complaint has always been
x Whether petitioner has complied with these provisions requires the presentation subject to court approval and to certain conditions. The exceptional right that
of evidence, although in its amended complaint petitioner did allege that it had Section 1, Rule 17 of the 1964 Rules of Court conferred on the plaintiff must be
complied with the requirements.23 The determination of this question must understood to have applied only to other civil actions. The 1997 Rules of Civil
await that hearing on the complaint for expropriation, particularly the hearing for Procedure abrogated this exceptional right.
the condemnation of the properties sought to be expropriated. Expropriation
proceedings consist of two stages: first, condemnation of the property after it is ISSUE # 2: Whether the dismissal of the expropriation case by the expropriator (NPC) includes
determined that its acquisition will be for a public purpose or public use and, the dismissal of the claim for damages by the landowner.
second, the determination of just compensation to be paid for the taking of the
private property to be made by the court with the assistance of not more than HELD # 2: NO.
three commissioners. x There is nothing in Rule 67 of the 1964 Rules of Court that provided for the
dismissal of the defendant's claim for damages, upon the dismissal of the
expropriation case. Case law holds that in the event of dismissal of the
NATIONAL POWER CORPORATION, petitioner, vs. COURT OF APPEALS and ANTONINO expropriation case, the claim for damages may be made either in a separate or
POBRE, respondents. in the same action, for all damages occasioned by the institution of the
G.R. No. 106804 August 12, 2004 expropriation case. The dismissal of the complaint can be made under certain
SECOND DIVISION conditions, such as the reservation of the defendant's right to recover damages
either in the same or in another action.
FACTS:
x Pobre began developing the 68,969 square-meter land located in Barangay Bano, REPUBLIC OF THE PHILIPPINES (Department of Public Works and Highways), petitioner,
Municipality of Tiwi, Albay ("Property") as a resort-subdivision, which he named vs. ISMAEL ANDAYA, respondent.
as "Tiwi Hot Springs Resort Subdivision." G.R. No. 160656 June 15, 2007
x The then Court of First Instance of Albay approved the subdivision plan of the SECOND DIVISION
Property. The Register of Deeds thus cancelled TCT No. 4067 and issued
independent titles for the approved lots.
x Pobre started advertising and selling the lots. FACTS:
x Subsequently, the Commission on Volcanology certified that thermal mineral x Ismael Andaya is the registered owner of two parcels of land in Bading, Butuan
water and steam were present beneath the Property. The Commission on City (TCT RT-10225 and RT-10646)
Volcanology found the thermal mineral water and steam suitable for domestic x These properties are subject to a 60-meter wide perpetual easement for public
use and potentially for commercial or industrial use. highways, irrigation ditches, aqueducts, and other similar works of the
x Pobre leased to National Power Corporation (NPC), for one year, 11 lots from the government or public enterprise, at no cost to the government, except only the
approved subdivision plan. * Subsequently, NPC filed its first expropriation case value of the improvements existing thereon that may be affected.
against Pobre to acquire an 8,311.60 square-meter portion of the Property. x Republic of the Philippines (Republic) negotiated with Andaya to enforce the 60-
x The trial court ordered the expropriation of the lots upon NPC's payment of P25 meter easement of right-of-way. The easement was for concrete levees and
per square meter or a total amount of P207,790. floodwalls for Phase 1, Stage 1 of the Lower Agusan Development Project. The
x NPC began drilling operations and construction of steam wells. parties, however, failed to reach an agreement.
x While this first expropriation case was pending, NPC dumped waste materials x The Republic instituted an action before the Regional Trial Court of Butuan City
beyond the site agreed upon by NPC with Pobre. The dumping of waste to enforce the easement of right-of-way or eminent domain. The trial court
materials altered the topography of some portions of the Property. NPC did not issued a writ of possession. It also constituted a Board of Commissioners (Board)
act on Pobre's complaints and NPC continued with its dumping. to determine the just compensation. Eventually, the trial court issued an Order of
x Subsequently, NPC filed its second expropriation case against Pobre to acquire Expropriation upon payment of just compensation. Later, the Board reported that
an additional 5,554 square meters of the Property, as it needed the lot for the there was a discrepancy in the description of the property sought to be
construction and maintenance of Naglagbong Well Site F-20, pursuant to expropriated.
Proclamation No. 7396 and Republic Act No. 5092. x The Republic thus amended its complaint, reducing the 60-meter easement to 10
x NPC immediately deposited P5,546.36 with the Philippine National Bank. The meters, or an equivalent of 701 square meters.The Board reported that the
deposit represented 10% of the total market value of the lots covered by the project would affect a total of 10,380 square meters of Andaya’s properties,
second expropriation. 4,443 square meters of which will be for the 60-meter easement. The Board also
x NPC entered the 5,554 square-meter lot upon the trial court's issuance of a writ reported that the easement would diminish the value of the remaining 5,937
of possession to NPC. square meters. As a result, it recommended the payment of consequential
x Pobre filed a motion to dismiss the second complaint for expropriation, and damages amounting to P2,820,430 for the remaining area.
prayed for just compensation of all the lots affected by NPC's actions and for the x Andaya objected to the report because although the Republic reduced the
payment of damages. easement to 10 meters or an equivalent of 701 square meters, the Board still
x The trial court ordered the case submitted for decision since NPC failed to appear granted it 4,443 square meters. He contended that the consequential damages
to present its evidence. The trial court denied NPC's motion to reconsider the should be based on the remaining area of 9,679 square meters. Thus, the just
submission of the case for decision. compensation should be P11,373,405. The Republic did not file any comment,
x NPC filed a petition for certiorari with the then Intermediate Appellate Court. opposition, nor objection.

51
RECTO, GAYLE ANGELI M.
2011-0008 | AUSL
Personal Notes on Remedial Law 2 Review (based on the syllabus of Prof. Henedino M. Brondial)

x After considering the Board’s report, the trial court decreed that the plaintiff is building complex constructed on land which the State already owns. There is an
legally entitled to its inherent right of expropriation to lots, it being shown that it inherent illogic in the resort to eminent domain on property already owned by
is for public use and purpose --- free of charge by reason of the statutory lien of the State. At first blush, since the State already owns the property on which
easement of right-of-way imposed on defendant’s titles; That however, the NAIA 3 stands, the proper remedy should be akin to an action for ejectment.
plaintiff is obligated to pay defendant the sum of 2,820,430.00 as fair and However, the reason for the resort by the Government to expropriation
reasonable severance damages; To pay members of the Board of proceedings is understandable in this case. The 2004 Resolution, in requiring the
Commissioners; To pay defendant’s counsel P50,000.00 as Attorney’s fees; and payment of just compensation prior to the takeover by the Government of NAIA
That the Registry of Deeds of Butuan City is also directed to effect the issuance 3, effectively precluded it from acquiring possession or ownership of the NAIA 3
of TCT for the 2 lots in the name of the Republic of the Philippines. through the unilateral exercise of its rights as the owner of the ground on which
x Both parties appealed to the Court of Appeals. the facilities stood. Thus, as things stood after the 2004 Resolution, the right of
x The Court of Appeals modified the trial court’s decision by imposing a 6% the Government to take over the NAIA 3 terminal was preconditioned by lawful
interest on the consequential damages from the date of the writ of possession or order on the payment of just compensation to PIATCO as builder of the
the actual taking, and by deleting the attorney’s fees. structures. Xxx The right of eminent domain extends to personal and real
x Hence, the petition for review. property, and the NAIA 3 structures, adhered as they are to the soil, are
considered as real property. The public purpose for the expropriation is also
ISSUE # 1: Whether the Republic is required to resort to expropriation proceedings in beyond dispute. It should also be noted that Section 1 of Rule 67 (on
enforcing existing easements over a landowner's lot. Expropriation) recognizes the possibility that the property sought to be
expropriated may be titled in the name of the Republic of the Philippines,
HELD # 1: NO. although occupied by private individuals, and in such case an averment to that
x It is undisputed that there is a legal easement of right-of-way in favor of the effect should be made in the complaint. The instant expropriation complaint did
Republic. [The landowner's] transfer certificates of title contained the reservation aver that the NAIA 3 complex "stands on a parcel of land owned by the Bases
that the lands covered thereby are subject to the provisions of the Land Conversion Development Authority, another agency of [the Republic of the
Registration Act and the Public Land Act. Section 112 of the Public Land Act Philippines]." Admittedly, eminent domain is not the sole judicial recourse by
provides that lands granted by patent shall be subject to a right-of-way not which the Government may have acquired the NAIA 3 facilities while satisfying
exceeding 60 meters in width for public highways, irrigation ditches, aqueducts, the requisites in the 2004 Resolution. Eminent domain though may be the most
and other similar works of the government or any public enterprise, free of effective, as well as the speediest means by which such goals may be
charge, except only for the value of the improvements existing thereon that may accomplished. Not only does it enable immediate possession after satisfaction of
be affected. In view of this, the Court of Appeals declared that all the Republic the requisites under the law, it also has a built-in procedure through which just
needs to do is to enforce such right without having to initiate expropriation compensation may be ascertained. Thus, there should be no question as to the
proceedings and without having to pay any just compensation. Hence, the propriety of eminent domain proceedings in this case. Still, in applying the laws
Republic may appropriate the 701 square meters necessary for the construction and rules on expropriation in the case at bar, we are impelled to apply or
of the floodwalls without paying for it. construe these rules in accordance with the Court's prescriptions in the 2004
Resolution to achieve the end effect that the Government may validly take over
ISSUE # 2: Whether the Republic is required to pay full value for just compensation to a the NAIA 3 facilities. Insofar as this case is concerned, the 2004 Resolution is
landowner due to the deprivation of beneficial use of the subject lots owned by the landowner. effective not only as a legal precedent, but as the source of rights and
prescriptions that must be guaranteed, if not enforced, in the resolution of this
HELD # 2: NO. petition. Otherwise, the integrity and efficacy of the rulings of this Court will be
x [J]ust compensation should be paid only for 5,937 square meters of the total severely diminished.”
area of 10,380 square meters. Admittedly, the Republic needs only a 10-meter
easement or an equivalent of 701 square meters. Yet, it is also settled that it is ISSUE # 2: Whether expropriation proceedings should be held in accordance with Republic Act
legally entitled to a 60-meter wide easement or an equivalent of 4,443 square No. 8974.
meters. Clearly, although the Republic will use only 701 square meters, it should
not be liable for the 3,742 square meters, which constitute the difference HELD # 2: YES.
between this area of 701 square meters and the 4,443 square meters to which it x It was further settled in Gingoyon that the expropriation proceedings shall be
is fully entitled to use as easement, free of charge except for damages to held in accordance with Republic Act No. 8974, thus: “Unlike in the case of Rule
affected existing improvements, if any, under Section 112 of the Public Land Act. 67, the application of Rep. Act No. 8974 will not contravene the 2004 Resolution,
In effect, without such damages alleged and proved, the Republic is liable for which requires the payment of just compensation before any takeover of the
just compensation of only the remaining areas consisting of 5,937 square NAIA 3 facilities by the Government. The 2004 Resolution does not particularize
meters, with interest thereon at the legal rate of 6% per annum from the date of the extent such payment must be effected before the takeover, but it
the writ of possession or the actual taking until full payment is made. For the unquestionably requires at least some degree of payment to the private property
purpose of determining the final just compensation, the case is remanded to the owner before a writ of possession may issue. The utilization of Rep. Act No. 8974
trial court. guarantees compliance with this bare minimum requirement, as it assures the
private property owner the payment of, at the very least, the proffered value of
ASIA'S EMERGING DRAGON CORPORATION, Petitioner, vs. DEPARTMENT OF the property to be seized. Such payment of the proffered value to the owner,
TRANSPORTATION AND COMMUNICATION, SECRETARY LEANDRO R. MENDOZA and followed by the issuance of the writ of possession in favor of the Government, is
MANILA INTERNATIONAL AIRPORT AUTHORITY, Respondents. precisely the schematic under Rep. Act No. 8974, one which facially complies
G.R. No. 169914 March 24, 2008 with the prescription laid down in the 2004 Resolution.”
EN BANC
ISSUE # 3: Whether the BIR zonal valuation, under Rule 67, apply for determination of
REPUBLIC OF THE PHILIPPINES, Represented by the DEPARTMENT OF TRANSPORTATION preliminary compensation to acquire possession of the structures built in government land.
AND COMMUNICATIONS and MANILA INTERNATIONAL AIRPORT AUTHORITY, Petitioner, vs.
COURT OF APPEALS (Eighth Division) and SALACNIB BATERINA, Respondents. HELD # 3: NO.
G.R. No. 174166 x [A]s to the determination of the amount due PIATCO, this Court ruled in
Gingoyon that “Under Rep. Act No. 8974, the Government is required to
"immediately pay" the owner of the property the amount equivalent to the sum
of (1) one hundred percent (100%) of the value of the property based on the
FACTS: current relevant zonal valuation of the [BIR]; and (2) the value of the
x Despite the promulgation by this Court of Decisions and Resolutions in two improvements and/or structures as determined under Section 7. As stated above,
cases, Agan, Jr. v. Philippine International Air Terminals Co., Inc. and Republic v. the BIR zonal valuation cannot apply in this case, thus the amount subject to
Gingoyon, which already resolved the more basic and immediate issues arising immediate payment should be limited to "the value of the improvements and/or
from the said award, this Court is still continuously besieged by Petitions arising structures as determined under Section 7," with Section 7 referring to the
from the awarding of the Ninoy Aquino International Airport International "implementing rules and regulations for the equitable valuation of the
Passenger Terminal III (NAIA IPT III) Project to the Philippine International Air improvements and/or structures on the land." Under the present implementing
Terminals Co., Inc. (PIATCO). The sheer magnitude of the project, the rules in place, the valuation of the improvements/structures are to be based
substantial cost of its building, the expected high profits from its operations, and using "the replacement cost method." However, the replacement cost is only one
its remarkable impact on the Philippine economy, consequently raised significant of the factors to be considered in determining the just compensation. In addition
interest in the project from various quarters. to Rep. Act No. 8974, the 2004 Resolution in Agan also mandated that the
x Once more, two new Petitions concerning the NAIA IPT III Project are before payment of just compensation should be in accordance with equity as well. Thus,
this Court. in ascertaining the ultimate amount of just compensation, the duty of the trial
court is to ensure that such amount conforms not only to the law, such as Rep.
ISSUE # 1: Whether it was appropriate for the Republic to pursue expropriation proceedings Act No. 8974, but to principles of equity as well. Admittedly, there is no way, at
over the airport project which was built on the government's land. least for the present, to immediately ascertain the value of the improvements
and structures since such valuation is a matter for factual determination. Yet
HELD # 1: YES. Rep. Act No. 8974 permits an expedited means by which the Government can
x The Court.. , in Gingoyon, directly addressed the issue on the appropriateness of immediately take possession of the property without having to await precise
the Republic's resort to expropriation proceedings: “The Government has chosen determination of the valuation. Section 4(c) of Rep. Act No. 8974 states that "in
to resort to expropriation, a remedy available under the law, which has the case the completion of a government infrastructure project is of utmost urgency
added benefit of an integrated process for the determination of just and importance, and there is no existing valuation of the area concerned, the
compensation and the payment thereof to PIATCO. We appreciate that the case implementing agency shall immediately pay the owner of the property its
at bar is a highly unusual case, whereby the Government seeks to expropriate a proferred value, taking into consideration the standards prescribed in Section 5
52
RECTO, GAYLE ANGELI M.
2011-0008 | AUSL
Personal Notes on Remedial Law 2 Review (based on the syllabus of Prof. Henedino M. Brondial)

[of the law]." The "proffered value" may strike as a highly subjective standard complaint, upon the payment of just compensation to be determined as of the
based solely on the intuition of the government, but Rep. Act No. 8974 does date of the filing of the complaint x x x. The second phase of the eminent
provide relevant standards by which "proffered value" should be based, as well domain action is concerned with the determination by the court of "the just
as the certainty of judicial determination of the propriety of the proffered value. compensation for the property sought to be taken." This is done by the court
In filing the complaint for expropriation, the Government alleged to have with the assistance of not more than three (3) commissioners x x x . It is only
deposited the amount of P3 Billion earmarked for expropriation, representing the upon the completion of these two stages that expropriation is said to have been
assessed value of the property. The making of the deposit, including the completed. The process is not complete until payment of just compensation.
determination of the amount of the deposit, was undertaken under the Accordingly, the issuance of the writ of possession in this case does not write finis
erroneous notion that Rule 67, and not Rep. Act No. 8974, is the applicable law. to the expropriation proceedings. To effectuate the transfer of ownership, it is
Still, as regards the amount, the Court sees no impediment to recognize this sum necessary for the NPC to pay the property owners the final just
of P3 Billion as the proffered value under Section 4(b) of Rep. Act No. 8974. After compensation.” [T]he mere issuance of a writ of possession in the expropriation
all, in the initial determination of the proffered value, the Government is not proceedings did not transfer ownership of the lots in favor of the City. Such
strictly required to adhere to any predetermined standards, although its proffered issuance was only the first stage in expropriation. There is even no evidence that
value may later be subjected to judicial review using the standards enumerated judicial deposit had been made in favor of respondents prior to the City’s
under Section 5 of Rep. Act No. 8974.” possession of the lots, contrary to Section 19 of the LGC.

SPOUSES LETICIA & JOSE ERVIN ABAD, SPS. ROSARIO AND ERWIN COLLANTES, SPS. NATIONAL POWER CORPORATION, Petitioner, vs. YCLA SUGAR DEVELOPMENT
RICARDO AND FELITA ANN, SPS. ELSIE AND ROGER LAS PIÑAS, LINDA LAYDA, RESTITUTO CORPORATION, Respondent.
MARIANO, SPS. ARNOLD AND MIRIAM MERCINES, SPS. LUCITA AND WENCESLAO A. G.R. No. 193936 December 11, 2013
RAPACON, SPS. ROMEO AND EMILYN HULLEZA, LUZ MIPANTAO, SPS. HELEN AND ANTHONY FIRST DIVISION
TEVES, MARLENE TUAZON, SPS. ZALDO AND MIA SALES, SPS. JOSEFINA AND JOEL YBERA,
SPS. LINDA AND JESSIE CABATUAN, SPS. WILMA AND MARIO ANDRADA, SPS. RAYMUNDO FACTS:
AND ARSENIA LELIS, FREDY AND SUSANA PILONEO, Petitioners, vs. FIL-HOMES REALTY x YCLA Sugar Development Corporation (YCLA) is the registered owner of three
and DEVELOPMENT CORPORATION and MAGDIWANG REALTY CORPORATION, parcels of land situated in Puerto Galera, Oriental Mindoro (TCTs T-5209, T-
Respondents. 21280 and T-78583)
G.R. No. 189239 November 24, 2010 x In order to complete its 69 KV Calapan-Mamburao Island Grid Project in Puerto
THIRD DIVISION Galera, Oriental Mindoro, NPC had to construct transmission lines that would
traverse several private properties, including the said parcels of land owned by
YCLA.
FACTS: x NPC filed a Complaint for expropriation with the RTC against YCLA and several
x Fil-Homes Realty and Development Corporation and Magdiwang Realty other individuals. The NPC sought the expropriation of a portion of the parcels of
Corporation, co-owners of two lots situated in Sucat, Parañaque City (TCTs land owned by the said defendants for the acquisition of an easement of right-of-
21712 and 21713), filed a complaint for unlawful detainer against Abad, et al. way over areas that would be affected by the construction of transmission lines.
before the Parañaque Metropolitan Trial Court (MeTC). They alleged that Abad, The portion of YCLA’s properties that would be affected by the construction of
et al., through tolerance, had occupied the subject lots since 1980 but ignored NPC’s transmission lines has an aggregate area of 5,846 square meters.
their repeated demands to vacate them. x The parties moved, inter alia, for the constitution of a Board of Commissioners to
x Abad, et al. countered that there is no possession by tolerance for they have be appointed by the RTC to determine the reasonable amount of just
been in adverse, continuous and uninterrupted possession of the lots for more compensation to be paid by the NPC.
than 30 years; and that the companies’ predecessor-in-interest, Pilipinas x The RTC issued an order terminating the pre-trial conference and directing the
Development Corporation, had no title to the lots. In any event, they contend constitution of a Board of Commissioners, which would submit a report and
that the question of ownership must first be settled before the issue of recommendation as to the reasonable amount of just compensation for the
possession may be resolved. properties sought to be expropriated.
x During the pendency of the case, the City of Parañaque filed expropriation x The RTC, acting on NPC’s urgent ex- parte motion, issued a writ of possession
proceedings covering the lots before the Regional Trial Court of Parañaque with placing NPC in possession of the properties sought to be expropriated.
the intention of establishing a socialized housing project therein for distribution to x The Board of Commissioners submitted its Report, which fixed the amount of
the occupants. A writ of possession was consequently issued and a Certificate of just compensation of the subject properties (at at P900.00 per sq m).
Turn-over given to the City. x YCLA filed a motion asking the RTC to direct the Board of Commissioners to
x Branch 77 of the MeTC rendered judgment in the unlawful detainer case against conduct an ocular inspection over the subject properties and, thereafter,
Abad, et al. amend/revise the Board of Commissioner’s Report.
x On appeal, the Regional Trial Court (RTC), by Decision of September 4, 2008,2 x The RTC rendered a Partial Decision as regards the amount of just compensation
reversed the MeTC decision and dismissed the companies’ complaint. that would be paid by the NPC to the other defendants.
x The companies filed a petition for review with the Court of Appeals. x The Board of Commissioners submitted its second Report, which fixed the just
x The Court of Appeals, , noting that Abad et al. did not present evidence to rebut compensation of the subject properties (at P1,000.00 per sq m).
respondents’ allegation of possession by tolerance, and considering Abad, et al.’s x The RTC rendered a Decision, which adopted the report and recommendation of
admission that they commenced occupation of the property without the the Board of Commissioners.
permission of the previous owner ─ Pilipinas Development Corporation ─ as x NPC appealed the RTC Decision to the Court of Appeals.
indicium of tolerance by the companies’ predecessor-in-interest, ruled in favor of x The Court of Appeals rendered the Decision which affirmed the RTC decision
the companies. with modification only in so far as the value of just compensation for the
x Abad, et al.’s motion for reconsideration was denied. property involved is concerned (at P900.00 per sq.m).
x Hence, the petition for review. x Hence, the petition for review on certiorari.

ISSUE: Whether the occupiers of the lot, owned by the landowners and subject to ISSUE: Whether the Commissioners' report, which is not substantiated by documents, should
expropriation, can maintain themselves therein, considering that a writ of possession has been be disregarded by the Courts.
granted to the City in pursuance of a socialized housing project.
HELD: YES.
HELD: NO. x The Court has consistently ruled that just compensation cannot be arrived at
x Section 1 of Commonwealth Act No. 538 [provides that] “ Section 1. When the arbitrarily; several factors must be considered such as, but not limited to,
Government seeks to acquire, through purchase or expropriation proceedings, acquisition cost, current market value of like properties, tax value of the
lands belonging to any estate or chaplaincy (cappellania), any action for condemned property, its size, shape, and location. But before these factors can
ejectment against the tenants occupying said lands shall be automatically be considered and given weight, the same must be supported by documentary
suspended, for such time as may be required by the expropriation proceedings evidence. The amount of just compensation could only be attained by using
or the necessary negotiations for the purchase of the lands, in which latter case, reliable and actual data as bases for fixing the value of the condemned property. A
the period of suspension shall not exceed one year. To avail himself of the commissioners’ report of land prices which is not based on any documentary
benefits of the suspension, the tenants shall pay to the landowner the current evidence is manifestly hearsay and should be disregarded by the court. A
rents as they become due or deposit the same with the court where the action commissioners’ report of land prices is considered as evidence in the
for ejectment has been instituted.” Petitioners did not comply with any of the determination of the amount of just compensation due the land owner in
acts mentioned in the law to avail of the benefits of the suspension. They expropriation cases. The recommended amount of just compensation contained
nevertheless posit that since the lots are the subject of expropriation in the commissioners’ report of land prices, in turn, is based on various factors
proceedings, respondents can no longer assert a better right of possession; and such as the fair market value of the property, the value of like properties. Thus,
that the City Ordinance authorizing the initiation of expropriation proceedings it becomes imperative that the commissioners’ report of land prices be supported
designated them as beneficiaries of the lots, hence, they are entitled to continue by pertinent documents, which impelled the commissioners to arrive at the
staying there. Petitioners’ position does not lie. The exercise of expropriation by recommended amount for the condemned properties, to aid the court in its
a local government unit is covered by Section 19 of the Local Government Code determination of the amount of just compensation. Otherwise, the
(LGC)[.] Lintag v. National Power Corporation clearly outlines the stages of commissioner’s report becomes hearsay and should thus not be considered by
expropriation, viz: “Expropriation of lands consists of two stages: The first is the court. The trial court, in expropriation cases, may accept or reject, whether
concerned with the determination of the authority of the plaintiff to exercise the in whole or in part, the report submitted by the Board of Commissioners, which
power of eminent domain and the propriety of its exercise in the context of the is merely advisory and recommendatory in character. It may also recommit the
facts involved in the suit. It ends with an order, if not of dismissal of the action, report or set aside the same and appoint new commissioners.
"of condemnation declaring that the plaintiff has a lawful right to take the
property sought to be condemned, for the public use or purpose described in the
53
RECTO, GAYLE ANGELI M.
2011-0008 | AUSL
Personal Notes on Remedial Law 2 Review (based on the syllabus of Prof. Henedino M. Brondial)

FORECLOSURE OF REAL ESTATE MORTGAGE [Rule 68]

1. Complaint

Section 1. Complaint in action for foreclosure. — In an action for the foreclosure of a


mortgage or other encumbrance upon real estate, the complaint shall set forth the date and
due execution of the mortgage; its assignments, if any; the names and residences of the
mortgagor and the mortgagee; a description of the mortgaged property; a statement of the
date of the note or other documentary evidence of the obligation secured by the mortgage,
the amount claimed to be unpaid thereon; and the names and residences of all persons having
or claiming an interest in the property subordinate in right to that of the holder of the
mortgage, all of whom shall be made defendants in the action. (1a)

2. Judgment

Section 2. Judgment on foreclosure for payment or sale. — If upon the trial in such
action the court shall find the facts set forth in the complaint to be true, it shall ascertain the
amount due to the plaintiff upon the mortgage debt or obligation, including interest and other
charges as approved by the court, and costs, and shall render judgment for the sum so found
due and order that the same be paid to the court or to the judgment obligee within a period of
not less than ninety (90) days nor more than one hundred twenty (120) days from the entry of
judgment, and that in default of such payment the property shall be sold at public auction to
satisfy the judgment. (2a)

3. Sale of Foreclosed property

Section 3. Sale of mortgaged property; effect. — When the defendant, after being
directed to do so as provided in the next preceding section, fails to pay the amount of the
judgment within the period specified therein, the court, upon motion, shall order the property
to be sold in the manner and under the provisions of Rule 39 and other regulations governing
sales of real estate under execution. Such sale shall not affect the rights of persons holding
prior encumbrances upon the property or a part thereof, and when confirmed by an order of
the court, also upon motion, it shall operate to divest the rights in the property of all the
parties to the action and to vest their rights in the purchaser, subject to such rights of
redemption as may be allowed by law.

Upon the finality of the order of confirmation or upon the expiration of the period of
redemption when allowed by law, the purchaser at the auction sale or last redemptioner, if
any, shall be entitled to the possession of the property unless a third party is actually holding
the same adversely to the judgment obligor. The said purchaser or last redemptioner may
secure a writ of possession, upon motion, from the court which ordered the foreclosure. (3a)

Equity of redemption

4. Deficiency Judgment

Section 6. Deficiency judgment. — If upon the sale of any real property as provided in
the next preceding section there be a balance due to the plaintiff after applying the proceeds
of the sale, the court, upon motion, shall render judgment against the defendant for any such
balance for which, by the record of the case, he may be personally liable to the plaintiff, upon
which execution may issue immediately if the balance is all due at the time of the rendition of
the judgment; otherwise; the plaintiff shall be entitled to execution at such time as the
balance remaining becomes due under the terms of the original contract, which time shall be
stated in the judgment. (6a)

JOSE T. RAMIREZ, Petitioner, vs. THE MANILA BANKING CORPORATION, Respondent.


G.R. No. 198800 December 11, 2013
FIRST DIVISION

FACTS:
x Ramirez mortgaged 2 parcels of land in Marikina in favor of MBC to secure his
P265,000 loan.
x The real estate mortgage provides that all correspondence relative to the
mortgage including notifications of extrajudicial actions shall be sent to petitioner
Ramirez at his given address
o N) All correspondence relative to this MORTGAGE, including
demand letters, summons, subpoenas or notifications of any
judicial or extrajudicial actions shall be sent to the MORTGAGOR at
the address given above or at the address that may hereafter be
given in writing by the MORTGAGOR to the MORTGAGEE, and the
mere act of sending any correspondence by mail or by personal
delivery to the said address shall be valid and effective notice to
the MORTGAGOR for all legal purposes and the fact that any
communication is not actually received by the MORTGAGOR, or
that it has been returned unclaimed to the MORTGAGEE, or that no
person was found at the address given, or that the address is
fictitious or cannot be located, shall not excuse or relieve the
MORTGAGOR from the effects of such notice
x Ramirez failed to pay

54
RECTO, GAYLE ANGELI M.
2011-0008 | AUSL
Personal Notes on Remedial Law 2 Review (based on the syllabus of Prof. Henedino M. Brondial)

x MBC then filed a request for extrajudicial foreclosure of real estate mortgage9 certificate of title in their names because Gonzales - to whom they
before Atty. Hipolito Sañez have entrusted said task - had deceived them in that they were
x MBC was the only bidder during the auction sale thus a certificate of sale was assured that the said certificate was already being processed when
issued in its favor such was not the case.14
x Ramirez Æ sued MBC for annulment of sale and prayed that the certificate of o Eventually, they found out that the property had already been
sale be annulled on the ground, among others, that paragraph N of the real mortgaged to Sps. Marquez, and that when they tried to contact
estate mortgage was violated for he was not notified of the foreclosure and Gonzales for an explanation, she could no longer be found.
auction sale. o Separately, Sps. Alindog averred that when the mortgage was
x RTC Æ ruled that the extrajudicial foreclosure proceedings were null and void executed in favor of Sps. Marquez, Gutierrez was already dead.
and the certificate of sale is invalid x Sps. Marquez Æ filed their answer
x CA Æ reversed RTC o disputed Sps. Alindog’s ownership over the subject property,
o ruled that absence of personal notice of foreclosure to Ramirez as arguing that the purported sale in the latter’s favor was never
required by paragraph N of the real estate mortgage is not a registered and therefore, not binding upon them. Further, they
ground to set aside the foreclosure sale insisted that their certificate of title, TCT No. T-41939, was already
indefeasible, and cannot be attacked collaterally.
ISSUE: Whether Ramirez was entitled to personal notice of the foreclosure proceedings. x Anita Æ filed an ex-parte petition for the issuance of a writ of possession
o RTC Æ granted
HELD: YES. x Sps. Alindog Æ sought the issuance of a temporary restraining order (TRO)
x We have consistently held that unless the parties stipulate, personal and/or writ of preliminary injunction with prayer for damages,21 in a separate
notice to the mortgagor in extrajudicial foreclosure proceedings is not case docketed as SCA No. TG-05-252122 (injunction case) which was raffled to
necessary because Section 31 of Act No. 31352 only requires the the same court.
posting of the notice of sale in three public places and the publication o RTC Æ issued a writ of preliminary injunction enjoining Sps.
of that notice in a newspaper of general circulation Marquez from taking possession of the subject property until after
x In this case, the parties stipulated in paragraph N of the real estate mortgage the controversy has been fully resolved on the merits
that all correspondence relative to the mortgage including notifications of x Sps Marquez Æ filed an MR
extrajudicial actions shall be sent to mortgagor Ramirez at his given address. o essentially pointing out that, as the confirmed and registered
Respondent had no choice but to comply with this contractual provision it has owners of the subject property, they are entitled to its possession
entered into with Ramirez. The contract is the law between them. Hence, we as a matter of right. They argued that pursuant to Sections 728
cannot agree with the bank that paragraph N of the real estate mortgage does and 829 of Act No. 3135,30 as amended by Act No. 4118,31 the
not impose an additional obligation upon it to provide personal notice of the RTC was legally bound to place them in possession of the subject
extrajudicial foreclosure sale to the mortgagor Ramirez. property pending resolution of the annulment case. Further, it is
x As we explained in Metropolitan Bank v. Wong,22 the bank’s violation of their position that the purpose for the issuance of the injunctive
paragraph N of the real estate mortgage is sufficient to invalidate the writ - i.e., to restrain the implementation of the writ of possession
extrajudicial foreclosure sale: - had already been rendered moot and academic by its actual
o [A] contract is the law between the parties and … absent any enforcement in the interim.
showing that its provisions are wholly or in part contrary to law, x Sps. Alindog Æ filed a Motion for Approval of Cash Bond and to Regain
morals, good customs, public order, or public policy, it shall be Possession
enforced to the letter by the courts. Section 3, Act No. 3135 reads: x RTC Æ denied motion of Sps. Alindog; granted motion of Sps Marquez
ƒ "Sec. 3. Notice shall be given by posting notices of x Sps Alindog Æ filed a Rule 65 before CA
the sale for not less than twenty days in at least three x CA Æ dismissed petition
public places of the municipality or city where the o no grave abuse of discretion on the RTC’s part when it issued the
property is situated, and if such property is worth injunctive writ that enjoined Sps. Marquez from taking possession
more than four hundred pesos, such notice shall also of the subject property. It observed that Sps. Alindog had indeed
be published once a week for at least three "adduced prima facie proof of their right to possess the subject
consecutive weeks in a newspaper of general property"36 while the annulment case was pending, adding that
circulation in the municipality and city." the latter’s "right to remain in possession"37 proceeds from the fact
x The Act only requires (1) the posting of notices of sale in three public places, and of the subject property’s earlier sale to them. Thus, while Sps.
(2) the publication of the same in a newspaper of general circulation. Personal Marquez concededly had a right to possess the subject property on
notice to the mortgagor is not necessary. Nevertheless, the parties to the account of the consolidation of the title in their names, the CA
mortgage contract are not precluded from exacting additional nonetheless found no fault on the part of the RTC for "proceeding
requirements. In this case, petitioner and respondent in entering into a with caution"38 in weighing the conflicting claims of the parties and
contract of real estate mortgage, agreed inter alia: subsequently issuing the writ of preliminary injunction in Sps.
o "all correspondence relative to this mortgage, including demand Alindog’s favor.
letters, summonses, subpoenas, or notifications of any judicial or x Sps Alindog Æ filed a Rule 45 before the SC
extra-judicial action shall be sent to the MORTGAGOR…."
x Precisely, the purpose of the foregoing stipulation is to apprise ISSUE: Whether a writ of preliminary injunction may still enjoin Sps. Alindog from possession
respondent of any action which petitioner might take on the subject of the land, considering the consolidation of title in their names.
property, thus according him the opportunity to safeguard his rights.
When petitioner failed to send the notice of foreclosure sale to HELD: NO.
respondent, he committed a contractual breach sufficient to render the x It is an established rule that the purchaser in an extra-judicial foreclosure sale is
foreclosure sale on November 23, 1981 null and void. entitled to the possession of the property and can demand that he be placed in
x We reiterated the Wong ruling in Global Holiday Ownership Corporation v. possession of the same either during (with bond) or after the expiration (without
Metropolitan Bank and Trust Company23 and recently, in Carlos Lim, et al. v. bond) of the redemption period therefor. To this end, the Court, in China
Development Bank of the Philippines.24 Notably, all these cases involved Banking Corp. v. Sps. Lozada41 (China Banking Corp.), citing several cases on
provisions similar to paragraph N of the real estate mortgage in this case. the matter, explained that a writ of possession duly applied for by said purchaser
should issue as a matter of course, and thus, merely constitutes a ministerial
duty on the part of the court, viz.:42
SPOUSES NICASIO C. MARQUEZ AND ANITA J. MARQUEZ, Petitioners, vs. SPOUSES o The procedure for extrajudicial foreclosure of real estate mortgage
CARLITO ALINDOG AND CARMEN ALINDOG, Respondents. is governed by Act No. 3135, as amended. The purchaser at the
G.R. No. 184045 January 22, 2014 public auction sale of an extrajudicially foreclosed real property
SECOND DIVISION may seek possession thereof in accordance with Section 7 of Act
No. 3135, as amended, which provides:
FACTS: ƒ SEC. 7. In any sale made under the provisions of this
x Benjamin Gutierrez (Gutierrez) executed a Deed of Real Estate Mortgage in favor Act, the purchaser may petition the Court of First
of Anita J. Marquez (Anita) to secure his loan from the latter amounting to Instance of the province or place where the property
P500,000.00 over a parcel of land in Tagaytay or any part thereof is situated, to give him possession
x Gutierrez defaulted in the payment of his loan obligation thereof during the redemption period, furnishing bond
x Anita sought to foreclose the mortage in an amount equivalent to the use of the property for
x Anita became the highest bidder a period of twelve months, to indemnify the debtor in
x Upon Gutierrez’s failure to redeem the same property within the prescribed case it be shown that the sale was made without
period therefor, title was consolidated under TCT No. T-41939 in the name of violating the mortgage or without complying with the
Anita J. Marquez, married to Nicasio C. Marquez requirements of this Act. Such petition shall be made
o ANNOTATION OF ADVERSE CLAIM (copied from the old TCT): in under oath and filed in form or an ex parte motion in
the names of respondents-spouses Carlito and Carmen Alindog the registration or cadastral proceedings if the
(Sps. Alindog) property is registered, or in special proceedings in the
x Sps. Alindog Æ filed a civil case for annulment of real estate mortgage and case of property registered under the Mortgage Law
certificate of sale with prayer for damages against Sps. Marquez and a certain or under section one hundred and ninety-four of the
Agripina Gonzales Administrative Code, or of any other real property
o alleged that they purchased13 the subject property from Gutierrez encumbered with a mortgage duly registered in the
way back in September 1989, but were unable to secure a office of any register of deeds in accordance with any
55
RECTO, GAYLE ANGELI M.
2011-0008 | AUSL
Personal Notes on Remedial Law 2 Review (based on the syllabus of Prof. Henedino M. Brondial)

existing law, and in each case the clerk of court shall, the mortgaged property may be awarded to a purchaser in an extra-judicial
upon the filing of such petition, collect the fees foreclosure unless a third party is actually holding the property by adverse title or
specified in paragraph eleven of section one hundred right. In the recent case of Rural Bank of Sta. Barbara (Iloilo), Inc. v. Centeno,46
and fourteen of Act Numbered Four hundred and citing the case of China Banking Corp., the Court illumined that "the phrase ‘a
ninety six as amended by Act Numbered Twenty-eight third party who is actually holding the property adversely to the judgment
hundred and sixty-six, and the court shall, upon obligor’ contemplates a situation in which a third party holds the property by
approval of the bond, order that a writ of possession adverse title or right, such as that of a co-owner, tenant or usufructuary. The co-
issue addressed to the sheriff of the province in which owner, agricultural tenant, and usufructuary possess the property in their own
the property is situated, who shall execute said order right, and they are not merely the successor or transferee of the right of
immediately. possession of another co-owner or the owner of the property. Notably, the
x The Court expounded on the application of the foregoing provision in De Gracia property should not only be possessed by a third party, but also held by the third
v. San Jose, thus: party adversely to the judgment obligor."47 In other words, as mentioned in
o As may be seen, the law expressly authorizes the purchaser to Villanueva v. Cherdan Lending Investors Corporation,48 the third person must
petition for a writ of possession during the redemption period by therefore claim a right superior to that of the original mortgagor.
filing an ex parte motion under oath for that purpose in the x In this case, it is clear that the issuance of a writ of possession in favor of Sps.
corresponding registration or cadastral proceeding in the case of Marquez, who had already consolidated their title over the extra-judicially
property with Torrens title; and upon the filing of such motion and foreclosed property, is merely ministerial in nature. The general rule as herein
the approval of the corresponding bond, the law also in express stated - and not the exception found under Section 33, Rule 39 of the Rules -
terms directs the court to issue the order for a writ of possession. should apply since Sps. Alindog hinged their claim over the subject property on
Under the legal provisions above copied, the order for a writ of their purported purchase of the same from its previous owner, i.e., Sps.
possession issues as a matter of course upon the filing of the Gutierrez (with Gutierrez being the original mortgagor). Accordingly, it cannot be
proper motion and the approval of the corresponding bond. No seriously doubted that Sps. Alindog are only the latter’s (Sps. Gutierrez)
discretion is left to the court. And any question regarding the successors-in-interest who do not have a right superior to them.
regularity and validity of the sale (and the consequent cancellation x That said, the RTC therefore gravely abused its discretion when it issued the
of the writ) is left to be determined in a subsequent proceeding as injunctive writ which enjoined Sps. Marquez from taking possession of the
outlined in section 8. Such question is not to be raised as a subject property. To be sure, grave abuse of discretion arises when a lower court
justification for opposing the issuance of the writ of possession, or tribunal patently violates the Constitution, the law or existing jurisprudence.49
since, under the Act, the proceeding for this is ex parte. Here, while the RTC had initially issued a writ of possession in favor of Sps.
o Strictly, Section 7 of Act No. 3135, as amended, refers to a Marquez, it defied existing jurisprudence when it effectively rescinded the said
situation wherein the purchaser seeks possession of the foreclosed writ by subsequently granting Sps. Alindog's prayer for injunctive relief. The
property during the 12-month period for redemption. Upon the RTC's finding anent the initial evidence adduced by Sps. Alindog constitutes
purchaser’s filing of the ex parte petition and posting of the improper basis to justify the issuance of the writ of preliminary injunction in their
appropriate bond, the RTC shall, as a matter of course, order the favor since, in the first place, it had no authority to exercise any discretion in this
issuance of the writ of possession in the purchaser’s favor. respect. Jurisprudence is clear on the matter: without the exception under
o In IFC Service Leasing and Acceptance Corporation v. Nera, the Section 33, Rule 39 of the Rules availing, the issuance of a writ of possession in
Court reasoned that if under Section 7 of Act No. 3135, as favor of the purchaser of an extra-judicially foreclosed property - such as Sps
amended, the RTC has the power during the period of redemption Marquez in this case - should come as a matter of course, and, in such regard,
to issue a writ of possession on the ex parte application of the constitutes only a ministerial duty on the part of the court. Besides, it was
purchaser, there is no reason why it should not also have the same improper for the RTC to have issued a writ of preliminary injunction since the act
power after the expiration of the redemption period, especially sought to be enjoined, i.e., the implementation of the writ of possession, had
where a new title has already been issued in the name of the already been accomplished in the interim and thus, rendered the matter moot.
purchaser. Hence, the procedure under Section 7 of Act No. 3135, Case law instructs that injunction would not lie where the acts sought to be
as amended, may be availed of by a purchaser seeking possession enjoined had already become fait accompli (meaning, an accomplished or
of the foreclosed property he bought at the public auction sale after consummated act).50 Hence, since the consummation of the act sought to be
the redemption period has expired without redemption having been restrained had rendered Sps. Alindog's injunction petition moot, the issuance of
o It is thus settled that the buyer in a foreclosure sale the said injunctive writ was altogether improper.
becomes the absolute owner of the property purchased if it x All told, by acting averse to well-settled jurisprudential rules and resultantly
is not redeemed during the period of one year after the depriving Sps. Marquez of their right of possession over the subject property, the
registration of the sale. As such, he is entitled to the Court therefore concludes that the RTC gravely abused its discretion in this case.
possession of the said property and can demand it at any In effect, the CA's contrary ruling thereto is hereby reversed and set aside, which
time following the consolidation of ownership in his name consequentially leads to the nullification of the writ of preliminary injunction
and the issuance to him of a new transfer certificate of issued by the RTC in favor of Sps. Alindog, and the reinstatement of the writ of
title. The buyer can in fact demand possession of the land possession issued by the same court in favor of Sps. Marquez. It must, however,
even during the redemption period except that he has to be noted that these pronouncements are without prejudice to any separate
post a bond in accordance with Section 7 of Act No. 3135, action which Sps. Alindog may file in order to recover ownership of the subject
as amended. No such bond is required after the redemption property.
period if the property is not redeemed. Possession of the
land then becomes an absolute right of the purchaser as RUSTICO A. ARDIENTE and ASUNCION PALOMARDIENTE, petitioners, vs. PROVINCIAL
confirmed owner. Upon proper application and proof of SHERIFF, REGISTER OF DEEDS OF QUEZON and PENINSULA DEVELOPMENT BANK,
title, the issuance of the writ of possession becomes a respondents.
ministerial duty of the court. (Emphases and underscoring G.R. No. 148448 August 17, 2004
supplied; citations and emphases in the original omitted) THIRD DIVISION
x In the case of Spouses Espiridion v. CA,43 the Court expounded on the
ministerial nature of the foregoing issuance as follows:44 FACTS:
o The issuance of a writ of possession to a purchaser in a x spouses Rustico Ardiente and Asuncion Paloma, together with their son Angel P.
public auction is a ministerial act.1âwphi1 After the Ardiente and the latter’s wife Gliceria Ardiente, obtained a loan in the amount of
consolidation of title in the buyer’s name for failure of the P100,000.00 from the Peninsula Development Bank (the bank)
mortgagor to redeem the property, the writ of possession x Ardientes executed in favor of the bank a PN and a Real Estate Mortgage over 1
becomes a matter of right. Its issuance to a purchaser in an parcel of land at Mabutag, Cawa, Buenavista, Quezon and 3 others at Cadlit,
extrajudicial foreclosure sale is merely a ministerial Guinayangan, Quezon
function. The trial court has no discretion on this matter. x Out of the proceeds of the loan, the Ardientes purchased a mini bus costing
Hence, any talk of discretion in connection with such P81,875.00.
issuance is misplaced. x The minu bus, however, met an accident
x A clear line demarcates a discretionary act from a ministerial one. Thus: o rendered the Ardientes unable to meet their obligation to the bank
o The distinction between a ministerial and discretionary act is well x The Ardientes were later on granted by the bank an additional loan of P46,000
delineated. A purely ministerial act or duty is one which an officer for which they executed an October 29, 1981 Promissory Note,3 the Real Estate
or tribunal performs in a given state of facts, in a prescribed Mortgage was amended.
manner, in obedience to the mandate of a legal authority, without x Ardientes failed to pay
regard to or the exercise of his own judgment upon the propriety x The bank thus extra-judicially foreclosed the mortgage and the parcels of land
or impropriety of the act done. If the law imposes a duty upon a covered thereby were sold at public auction to the bank which was the highest
public officer and gives him the right to decide how or when the bidder.
duty shall be performed, such duty is discretionary and not x The bank later notified the Ardientes by letter of February 24, 19844 that they
ministerial. The duty is ministerial only when the discharge of the had one (1) year from November 11, 1983 or up to November 11, 1984 to
same requires neither the exercise of official discretion or redeem the foreclosed mortgage.
judgment. x Ardientes (2 days before the expiration of its period to redeem) Æ filed before
x Clearly, the use of discretion and the performance of a ministerial act are the Regional Trial Court (RTC) of Quezon at Gumaca a complaint, denominated as
mutually exclusive. (Emphases and underscoring supplied; citations omitted) Petition,5 against the bank, the provincial Sheriff of Quezon, and the Register of
x The ministerial issuance of a writ of possession in favor of the purchaser in an deeds of Quezon, for Annulment of Auction Sale with Preliminary Injunction and
extra-judicial foreclosure sale, however, admits of an exception. Section 33,45 Damages, anchored to two grounds:
Rule 39 of the Rules of Court (Rules) pertinently provides that the possession of
56
RECTO, GAYLE ANGELI M.
2011-0008 | AUSL
Personal Notes on Remedial Law 2 Review (based on the syllabus of Prof. Henedino M. Brondial)

o (a) that it was the defendant, not herein petitioners, who had x It is settled that personal notice to the mortgagor in extra-judicial
violated the Real Estate Mortgage and Amended Real Estate foreclosure proceedings is not necessary, hence, not a ground to set
Mortgage, and aside the foreclosure sale.22
o (b) that the requisite of notifying the mortgagors of the intended x With respect to petitioners’ argument that the bank, in paragraph 25 of its
extra-judicial foreclosure sale was not duly complied with ¾ the Answer, in fact put in issue its compliance with the requirements of Act 3135,
FORECLOSURE SALE should be annulled "more specifically with regards to the notices of the public auction sale as well as
ƒ Ardiente spouses capitalized on the alleged lack of the extra-judicial application in accordance with law," to thus call for the
notice to them of the "judicial foreclosure auction presentation of evidence, they citing again Benavides,23 the same fails.
sale. o Benavides bears on the rendition of judgment on the pleadings. It
x Bank Æ filed its answer holds that where the defendant’s answer tenders an issue, as
o petitioners were duly notified of the extra-judicial foreclosure and where it does not only deny the material allegations of the
public auction sale. There was sufficient notice and publication complaint but also sets up certain special and affirmative defenses,
served to all concern[ed] of said public auction sale of the the nature of such answer calls for presentation of evidence,
properties offered as collaterals. hence, it is error to render a judgment on the pleadings thereon
x Ardientes Æ amended complaint without such evidence.
o alleged that, among other things, the purchase price of the
mortgaged parcels of land was so "grossly and greatly inadequate," x No doubt, it is a well-settled rule that statutory provisions governing
hence, the foreclosure sale should be annulled; by reason of the publication of notice of mortgage foreclosure sales must be strictly
unlawful foreclosure of the real estate mortgage, they suffered complied with, and that even slight deviations therefrom will invalidate
damages; and to protect their interests, they filed a formal request the notice and the sale at least voidable.24
with the Register of Deeds to cause a notice of lis pendens. x Despite petitioners’ non-allegation of lack of publication of notice of foreclosure
x RTC Gumaca Br 61 Æ declared the extra-judicial foreclosure and the sale of the in their Complaint, the bank pleaded in its Answer (1) "that petitioners were duly
mortgaged properties null and void notified of the extrajudicial foreclosure and public auction sale" and "[t]here was
o Despite these allegations on record, no documentary exhibits of sufficient notice and publication served to all concern[ed] of said public auction
such publication of notice of public auction sale in a newspaper of sale," and (2) that it and the Office of the provincial Sheriff "fully compl[ied] with
general circulations supported by publisher’s affidavit were ever the requirements of law under Act 3135, more specifically with regard to notices
submitted by respondent Bank. Considering that petitioners are of the public auction as well as the extra-judicial foreclosure in accordance with
clearly attacking the validity of the public auction sale for which law."
respondent Bank was the sole bidder, said documentary exhibits x Yet petitioners never refuted in their Reply and Answer to Counterclaim such
should have been presented in court and not merely alleged to be defense of the bank nor presented evidence before the trial court to disprove the
attached to the record in the Office of the Provincial Sheriff of same.
Quezon at Calauag, Quezon. x In fact, in its Comment on petitioners’ Formal Offer of Evidence before the trial
o Therefore, in the absence of convincing proof that the statutory court, the bank, passing on Exhibit "D" - its letter to petitioners advising them
provisions governing publication of notice of mortgage foreclosure that they had one year from November 11, 1993 to exercise their right of
sales have been strictly complied with, this Court has no other redemption, stated that said exhibit was admitted "with the qualification as to
recourse except to declare as null and void the sale in favor of the purpose to the effect that said extra-judicial foreclosure was filed in
judgment creditor, made by respondent Sheriff on November 12, accordance with law and that all requirements of said law were complied with
1983, awarding the properties in question to respondent Bank, and and that plaintiffs were duly notified of said proceedings."25
for which, the titles in the name of petitioner-spouses were already x Despite the bank’s repeated claim that the statutory requirements governing
cancelled and registered in its name extra-judicial foreclosure had been complied with, the bank’s plea of lack of
x CA Æ reversed RTC publication of notice of foreclosure was not raised by petitioners either in the
o In this case, even perfunctory reading of the Petition and the Amended Complaint or in the Reply and Answer to Counterclaim. It was not also
Amended Petition, readily reveals the absence of any averment raised during the trial as the entire transcripts of the stenographic notes of the
relating to the required posting and publication of the notice of proceedings before the trial court show. Nor even in their memorandum filed
foreclosure sale. Understandably then, the defendant-appellant before the trial court, petitioners having merely assailed the lack of "personal"
Bank saw no need to present the Sheriff’s Certification of Posting notification to them of any "intended" extrajudicial foreclosure and the "grossly
and the newspaper where the notice was published as well as the and greatly inadequate" purchase price of the lands.
publisher’s affidavit. Clearly, the presumption that the Provincial x As the appellate court thus held, the issue of lack of publication of notice cannot
Sheriff of Quezon has discharged his official duty in a regular be raised for the first time on appeal.
manner and that the defendant-appellant Bank complied with the x In the Tambunting case cited by petitioners to support their thesis that failure
requirements under the law will suffice. And while it may be true to strictly comply with statutory requirements governing publication of notice of
that the Supreme Court said, in the case of Tambunting v. Court of mortgage foreclosure sales renders the sale at least voidable, the therein
Appeals and relied upon by the trial court, that the presumption of mortgagors, in their complaint for annulment of mortgage and damages, sought
compliance with official duty is rebutted by the failure to present to enjoin the extra-judicial foreclosure of mortgage. During the pendency of the
proof of posting and publication of the notice of sale, such may be case, the extra-judicial foreclosure pushed thru just the same. The mortgaged
applied only when these omissions are alleged and raised by the property was sold at public auction to the mortgagees, and the property was
party in the complaint. eventually sold to the Tambunting Realty. The mortgagors thereupon filed a
x Ardientes Æ filed a Rule 45 before SC Supplemental Complaint impleading the realty firm, the provincial sheriff as the
officer responsible for holding the foreclosure, and the Register of Deeds for the
ISSUE: Whether the absence of allegations on the lack of (1) sheriff certification of posting, subsequent transfer of the property "despite alleged non-compliance with the
(2) newspaper where the notice was published, and (3) the publisher’s affidavit are requirements of Act 3135, Sec. 3 (as amended by Act 4118) on posting and
indispensable requirements is tantamount to a waiver of such defenses. publication of the notice of foreclosure sale." In other words, the lack of
publication was raised in issue by the mortgagors in their
HELD: YES. Supplemental Complaint.
x The spouses Ardientes (hereinafter referred to as petitioners) argue that x In the case of Go v. Court of Appeals,26 as in the present case, despite the fact
paragraph 15 of their Complaint and paragraph 16 of the Amended Complaint that the mortgagees pleaded as a defense in their Answer the "receipt of the
show that they were "attacking the validity of the extra-judicial sale"; that the ‘notice of the sale which was published in a newspaper of general circulation,’’’
impleading of the sheriff demonstrates that they are "questioning the validity and the issue of lack of publication of the notice of foreclosure was never raised in
legality of his performance of officially duty"; that the bank was sufficiently issue by the mortgagors.
informed of their "cause of action, theory of their case and relief being sought" x In disposing of the issue of lack of publication of the notice of foreclosure of
as shown by the bank’s allegations in paragraphs 15 and 16 of its Answer; and mortgage which was raised for the first time on appeal, this Court in Go held:
that in fact in the bank’s Special and Affirmative Defenses, particularly paragraph o Indeed, as correctly held by the respondent Court, the issue of lack
25 thereof which reads: of publication of the notice of foreclosure of the mortgage was
o 25) That answering respondent as well as the Office of Provincial raised only on appeal. Petitioner does not represent that he directly
Sheriff fully compl[ied] [with] the requirements of law under Act attacked in his complaint in Civil Case No. 8920 the validity of the
3135 as amended, more specifically with regards to notices of the foreclosure because of such lack of notice. His own Statement of
public auction sale as well as the extra-judicial foreclosure the Facts and of the Case in the instant petition makes no
application in accordance with the law.20, reference to such lack o notice as one, or even just as a basis for
x an issue was tendered, the nature of which affirmative defense-answer called for any, of his causes of action in the complaint. He sought the
the presentation of evidence, they citing Benavides v. Alabastro,21 but the bank cancellation of the contract of mortgage because he allegedly never
did not present "proof of proper compliance with Act 3135, "AN ACT TO received the amounts indicated in the promissory notes. Of course,
REGULATE THE SALE OF PROPERTY UNDER SPECIAL POWERS INSERTED IN OR nullity of the mortgage due to absence of consideration is leagues
ANNEXED TO REAL ESTATE MORTAGES,: as to posting and publication of notices apart form the nullity of the foreclosure of a mortgage because of
of public auction sale. non-publication of the notice of foreclosure.
o The Court is not persuaded. x Additionally, petitioner presented no evidence before the trial court to prove the
x With respect to petitioners’ paragraphs 15 and 16 allegations in their Complaint, absence of publication of the notice despite the fact that private respondents, in
clearly, they were questioning the validity of the extra-judicial foreclosure of the their Answer, squarely pleaded as a defense the foreclosure sale and petitioner’s
mortgage on the basis of lack of notice to them as mortgagors. receipt of the "notice of the sale which was published in a newspaper of general
circulation." That the lack of publication of the notice of foreclosure was never
raised in issue by petitioner and that it is not within the issues framed by the
57
RECTO, GAYLE ANGELI M.
2011-0008 | AUSL
Personal Notes on Remedial Law 2 Review (based on the syllabus of Prof. Henedino M. Brondial)

parties in the trial court are then too obvious. (Emphasis and underscoring x No hearing is required prior to the issuance of a writ of possession. This
supplied) is clear from the following disquisitions in Espinoza v. United Overseas Bank
Phils.24 which reiterates the settled rules on writs of possession, to wit:
LZK HOLDINGS and DEVELOPMENT CORPORATION, Petitioner, vs. PLANTERS o The proceeding in a petition for a writ of possession is ex
DEVELOPMENT BANK, Respondent. parte and summary in nature.1âwphi1 It is a judicial
G.R. No. 187973 January 20, 2014 proceeding brought for the benefit of one party only and
FIRST DIVISION without notice by the court to any person adverse of
interest. It is a proceeding wherein relief is granted
FACTS: without giving the person against whom the relief is
x LZK Holdings obtained a P40,000,000.00 loan from Planters Bank and secured sought an opportunity to be heard.
the same with a Real Estate Mortgage over its lot located in La Union o By its very nature, an ex parte petition for issuance of a writ of
x LZk failed to pay possession is a non-litigious proceeding. It is a judicial proceeding
x the lot was sold at a public auction after Planters Bank extrajudicially foreclosed for the enforcement of one's right of possession as purchaser in a
the real estate mortgage and the latter emerged as the highest bidder during the foreclosure sale. It is not an ordinary suit filed in court, by which
auction sale one party sues another for the enforcement of a wrong or
x LZK Holdings Æ filed before the RTC of Makati City, Branch 150, a complaint for protection of a right, or the prevention or redress of a wrong.25
annulment of extra judicial foreclosure, mortgage contract, promissory note and (Citations omitted)
damages x Given the ex-parte nature of the proceedings for a writ of possession, the R TC
o also prayed for the issuance of a temporary restraining order (TRO) did not err in cancelling the previously scheduled hearing and in granting
or writ of preliminary injunction to enjoin the consolidation of title Planters Bank's motion without affording notice to LZK Holdings or allowing it to
over the lot by Planters Bank. participate.
x Planters Bank Æ filed an ex-parte motion for the issuance of a writ of possession x Anent the correct amount of surety bond, it is well to emphasize that our task in
with the RTC-San Fernando. an appeal by petition for review on certiorari is limited, as a jurisdictional matter,
x RTC Makati (3 days before the expiration of LZK’s redemption period) Æ issued a to reviewing errors of law that might have been committed by the CA.26 The
TRO effective for 20 days enjoining Planters Bank from consolidating its title over allegations of incorrect computation of the surety bond involve factual matters
the property. within the competence of the trial court to address as this Court is not a trier of
x RTC-Makati Æ then ordered the issuance of a writ of preliminary injunction for facts. The RTC found the amount of P2,000,000.00 to be sufficiently equivalent
the same purpose3 but the writ was issued only on June 20, 2000 upon LZK to the use of the property for a period of twelve (12) months. We are bound by
Holdings' posting of a P40,000.00 bond. such factual finding especially considering the affirmation accorded it by the CA.
x Planters Bank succeeded in consolidating its ownership over the property x In fine, the decision of the CA is in accordance with the law and jurisprudence on
x RTC Pampanga Æ suspended the proceedings for its ex-parte motion for the the matter. It correctly sustained the Order of the RTC in issuing a writ of
issuance of a writ of possession in view of the TRO and writ of preliminary possession in favor of Planters Bank.
injunction issued by the RTC-Makati
o Planters Æ filed an MR GOLDENWAY MERCHANDISING CORPORATION, Petitioner, vs. EQUITABLE PCI
o RTC Pampanga Æ denied MR BANK, Respondent.
o CA Æ reversed G.R. No. 195540 March 13, 2013
o SC Æ affirmed CA FIRST DIVISION
x LZK Æ moved that the consolidated title of Planters be declared as null and void
o RTC Makati Æ granted motion FACTS:
o CA Æ affirmed x Goldenway Merchandising Corporation (petitioner) executed a Real Estate
x Planters (armed with the SC decision affirming the CA) Æ went to RTC Mortgage in favor of Equitable PCI Bank (respondent) over its real properties
Pampanga and filed a motion to set ex-parte hearing for the issuance of a writ of situated in Valenzuela to secure its Two Million Pesos (P2,000,000.00) loan
possession x Goldenway failed to pay
x RTC Pampanga Æ issued another Order10 declaring the scheduled hearing moot x Equitable foreclosed and the mortgaged properties were sold for P3,500,000.00
and academic and granting Planter Bank's ex-parte motion for the issuance of a to it
writ of possession which was filed as early as December 27, 1999. x Certificate of Sale was then issued, registered and inscribed on the TCTs
o CA Æ affirmed and dismissed LZK Holdings' petition for certiorari x Goldenway’s counsel (March 8, 2001) Æ offered to redeem the foreclosed
docketed; likewise denied LZK Holdings' motion for reconsideration properties by tendering a check in the amount of P3,500,000.00
x LZK Æ moved for a 30 day extension to file Rule 45 o Met with Equitable’s counsel to reiterate its offer BUT was told that
o SC Æ granted BUT ordered that the 30-day extended period shall redemption can no longer be had because the certificate of sale
be counted from the expiration of the original reglementary period. had already been registered
ƒ As such, LZK Holdings had until July 23, 2009 to file x Goldenway Æ verified with RD and confirmed that it ownership has already been
its petition and not August 24, 2009 or the date when consolidated
the petition was actually filed. x Goldenway Æ filed a complaint7 for specific performance and damages against
the respondent, asserting that it is the one-year period of redemption under Act
ISSUE: Whether LZK is still entitled to notice on the scheduled hearing for the issuance of the No. 3135 which should apply and not the shorter redemption period provided in
writ of possession in favor of Planters. Republic Act (R.A.) No. 8791
o argued that applying Section 47 of R.A. 8791 to the real estate
HELD: NO. mortgage executed in 1985 would result in the impairment of
x Under the principle of conclusiveness of judgment, the right of Planter's Bank to obligation of contracts and violation of the equal protection clause
a writ of possession as adjudged in G.R. No. 167998 is binding and conclusive on under the Constitution.
the parties. o Additionally, petitioner faulted the respondent for allegedly failing
x The doctrine of res judicata by conclusiveness of judgment postulates to furnish it and the Office of the Clerk of Court, RTC of Valenzuela
that "when a right or fact has been judicially tried and determined by a City with a Statement of Account as directed in the Certificate of
court of competent jurisdiction, or when an opportunity for such trial Sale, due to which petitioner was not apprised of the assessment
has been given, the judgment of the court, as long as it remains and fees incurred by respondent, thus depriving petitioner of the
unreversed, should be conclusive upon the parties and those in privity opportunity to exercise its right of redemption prior to the
with them."19 registration of the certificate of sale
x All the elements of the doctrine are present in this case. The final judgment in x Equitable Æ filed answer with counterclaim
G.R. No. 167998 was rendered by the Court pursuant to its jurisdiction over the o petitioner cannot claim that it was unaware of the redemption price
review of decisions and rulings of the CA. It was a judgment on the merits of which is clearly provided in Section 47 of R.A. No. 8791, and that
Planters Banks's right to apply for and be issued a writ of possession. Lastly, the petitioner had all the opportune time to redeem the foreclosed
parties in G.R. No. 167998 are the same parties involved in the present case.20 properties from the time it received the letter of demand and the
x Hence, LZK Holdings can no longer question Planter Bank's right to a writ of notice of sale before the registration of the certificate of sale. As to
possession over the subject property because the doctrine of conclusiveness of the check payment tendered by petitioner, respondent said that
judgment bars the relitigation of such particular issue. even assuming arguendo such redemption was timely made, it was
x Moreover, the authority relied upon by LZK Holdings defeats rather than support not for the amount as required by law.
its position. The ruling in PNB21 echoes the very same rationale of the judgment x RTC Æ dismissed the complainant AND counterclaim
in G.R. No. 167998 that is - the purchaser in foreclosure sale may take o the issue of constitutionality of Sec. 47 of R.A. No. 8791 was never
possession of the property even before the expiration of the redemption period raised by the petitioner during the pre-trial and the trial. Aside from
by filing an ex parte motion for such purpose and upon posting of the necessary the fact that petitioner’s attempt to redeem was already late, there
bond.22 was no valid redemption made because Atty. Judy Ann Abat-Vera
x The pronouncement in PNB that right of possession is based on the ownership of who talked to Atty. Joseph E. Mabilog of the Legal Division of
the subject property by the applicant pertains to applications for writ of respondent bank, was not properly authorized by petitioner’s Board
possession after the expiration of the redemption period, a situation not of Directors to transact for and in its behalf; it was only a certain
contemplated within the facts of the present case. Chan Guan Pue, the alleged President of petitioner corporation,
x We cannot also uphold the contentions of LZK Holdings that the RTC, in issuing who gave instruction to Atty. Abat-Vera to redeem the foreclosed
the writ of possession, transgressed Act No. 3135.23 properties.
x CA Æ affirmed RTC

58
RECTO, GAYLE ANGELI M.
2011-0008 | AUSL
Personal Notes on Remedial Law 2 Review (based on the syllabus of Prof. Henedino M. Brondial)

o petitioner failed to justify why Section 47 of R.A. No. 8791 should conditioned that he will pay all the damages which the
be declared unconstitutional. Furthermore, the appellate court bank may suffer by the enjoining or the restraint of the
concluded that a reading of Section 47 plainly reveals the intention foreclosure proceeding.
to shorten the period of redemption for juridical persons and that o Notwithstanding Act 3135, juridical persons whose
the foreclosure of the mortgaged properties in this case when R.A. property is being sold pursuant to an extrajudicial
No. 8791 was already in effect clearly falls within the purview of foreclosure, shall have the right to redeem the property in
the said provision. accordance with this provision until, but not after, the
x Goldenway Æ filed a Rule 45 before the SC registration of the certificate of foreclosure sale with the
applicable Register of Deeds which in no case shall be more
ISSUE: Which, between Act No. 3135 and R.A. 8791, is the law applicable in this case? than three (3) months after foreclosure, whichever is
earlier. Owners of property that has been sold in a
foreclosure sale prior to the effectivity of this Act shall
HELD: Section 47 of R.A. No. 8791 (which amended Act No. 3135).
retain their redemption rights until their expiration.
x In the present petition, it is contended that Section 47 of R.A. No. 8791 is
(Emphasis supplied.)
inapplicable considering that the contracting parties expressly and categorically
x Under the new law, an exception is thus made in the case of juridical persons
agreed that the foreclosure of the real estate mortgage shall be in accordance
which are allowed to exercise the right of redemption only "until, but not after,
with Act No. 3135. Citing Co v. Philippine National Bank11 petitioner contended
the registration of the certificate of foreclosure sale" and in no case more than
that the right of redemption is part and parcel of the Deed of Real Estate
three (3) months after foreclosure, whichever comes first.16
Mortgage itself and attaches thereto upon its execution, a vested right flowing
x May the foregoing amendment be validly applied in this case when the real
out of and made dependent upon the law governing the contract of mortgage
estate mortgage contract was executed in 1985 and the mortgage foreclosed
and not on the mortgagee’s act of extrajudicially foreclosing the mortgaged
when R.A. No. 8791 was already in effect?
properties. This Court thus held in said case that "Under the terms of the
o We answer in the affirmative.
mortgage contract, the terms and conditions under which redemption may be
x When confronted with a constitutional question, it is elementary that every court
exercised are deemed part and parcel thereof whether the same be merely
must approach it with grave care and considerable caution bearing in mind that
conventional or imposed by law."
every statute is presumed valid and every reasonable doubt should be resolved
x Petitioner then argues that applying Section 47 of R.A. No. 8791 to the present
in favor of its constitutionality.17 For a law to be nullified, it must be shown that
case would be a substantial impairment of its vested right of redemption under
there is a clear and unequivocal breach of the Constitution. The ground for
the real estate mortgage contract. Such impairment would be violative of the
nullity must be clear and beyond reasonable doubt.18 Indeed, those who petition
constitutional proscription against impairment of obligations of contract, a patent
this Court to declare a law, or parts thereof, unconstitutional must clearly
derogation of petitioner’s vested right and clearly changes the intention of the
establish the basis therefor. Otherwise, the petition must fail.19
contracting parties. Moreover, citing this Court’s ruling in Rural Bank of Davao
x Petitioner’s contention that Section 47 of R.A. 8791 violates the constitutional
City, Inc. v. Court of Appeals12 where it was held that "Section 119 prevails over
proscription against impairment of the obligation of contract has no basis.
statutes which provide for a shorter period of redemption in extrajudicial
x The purpose of the non-impairment clause of the Constitution20 is to safeguard
foreclosure sales", and in Sulit v. Court of Appeals,13 petitioner stresses that it
the integrity of contracts against unwarranted interference by the State. As a
has always been the policy of this Court to aid rather than defeat the
rule, contracts should not be tampered with by subsequent laws that would
mortgagor’s right to redeem his property.
change or modify the rights and obligations of the parties.21 Impairment is
x Petitioner further argues that since R.A. No. 8791 does not provide for its
anything that diminishes the efficacy of the contract. There is an impairment if a
retroactive application, courts therefore cannot retroactively apply its provisions
subsequent law changes the terms of a contract between the parties, imposes new
to contracts executed and consummated before its effectivity. Also, since R.A.
conditions, dispenses with those agreed upon or withdraws remedies for the
8791 is a general law pertaining to the banking industry while Act No. 3135 is a
enforcement of the rights of the parties.22
special law specifically governing real estate mortgage and foreclosure, under
x Section 47 did not divest juridical persons of the right to redeem their foreclosed
the rules of statutory construction that in case of conflict a special law prevails
properties but only modified the time for the exercise of such right by reducing
over a general law regardless of the dates of enactment of both laws, Act No.
the one-year period originally provided in Act No. 3135. The new redemption
3135 clearly should prevail on the redemption period to be applied in this case.
period commences from the date of foreclosure sale, and expires upon
x The constitutional issue having been squarely raised in the pleadings filed in the
registration of the certificate of sale or three months after foreclosure, whichever
trial and appellate courts, we shall proceed to resolve the same.
is earlier. There is likewise no retroactive application of the new redemption
x The law governing cases of extrajudicial foreclosure of mortgage is Act No.
period because Section 47 exempts from its operation those properties
3135,14 as amended by Act No. 4118. Section 6 thereof provides:
foreclosed prior to its effectivity and whose owners shall retain their redemption
o SEC. 6. In all cases in which an extrajudicial sale is made
rights under Act No. 3135.
under the special power hereinbefore referred to, the
x Petitioner’s claim that Section 47 infringes the equal protection clause as it
debtor, his successors-in-interest or any judicial creditor or
discriminates mortgagors/property owners who are juridical persons is equally
judgment creditor of said debtor, or any person having a
bereft of merit.
lien on the property subsequent to the mortgage or deed of
x The equal protection clause is directed principally against undue favor and
trust under which the property is sold, may redeem the
individual or class privilege.1âwphi1 It is not intended to prohibit legislation
same at any time within the term of one year from and
which is limited to the object to which it is directed or by the territory in which it
after the date of the sale; and such redemption shall be
is to operate. It does not require absolute equality, but merely that all persons
governed by the provisions of sections four hundred and
be treated alike under like conditions both as to privileges conferred and
sixty-four to four hundred and sixty-six, inclusive, of the
liabilities imposed.23 Equal protection permits of reasonable classification.24 We
Code of Civil Procedure,15 in so far as these are not
have ruled that one class may be treated differently from another where the
inconsistent with the provisions of this Act.
groupings are based on reasonable and real distinctions.25 If classification is
x The one-year period of redemption is counted from the date of the registration
germane to the purpose of the law, concerns all members of the class, and
of the certificate of sale. In this case, the parties provided in their real estate
applies equally to present and future conditions, the classification does not
mortgage contract that upon petitioner’s default and the latter’s entire loan
violate the equal protection guarantee.26
obligation becoming due, respondent may immediately foreclose the mortgage
x We agree with the CA that the legislature clearly intended to shorten the period
judicially in accordance with the Rules of Court, or extrajudicially in accordance
of redemption for juridical persons whose properties were foreclosed and sold in
with Act No. 3135, as amended.
accordance with the provisions of Act No. 3135.27
x However, Section 47 of R.A. No. 8791 otherwise known as "The
x The difference in the treatment of juridical persons and natural persons
General Banking Law of 2000" which took effect on June 13, 2000,
was based on the nature of the properties foreclosed - whether these
amended Act No. 3135. Said provision reads:
are used as residence, for which the more liberal one-year redemption
o SECTION 47. Foreclosure of Real Estate Mortgage. — In the
period is retained, or used for industrial or commercial purposes, in
event of foreclosure, whether judicially or extrajudicially,
which case a shorter term is deemed necessary to reduce the period of
of any mortgage on real estate which is security for any
uncertainty in the ownership of property and enable mortgagee-banks
loan or other credit accommodation granted, the
to dispose sooner of these acquired assets. It must be underscored that
mortgagor or debtor whose real property has been sold for
the General Banking Law of 2000, crafted in the aftermath of the 1997 Southeast
the full or partial payment of his obligation shall have the
Asian financial crisis, sought to reform the General Banking Act of 1949 by
right within one year after the sale of the real estate, to
fashioning a legal framework for maintaining a safe and sound banking
redeem the property by paying the amount due under the
system.28 In this context, the amendment introduced by Section 47 embodied
mortgage deed, with interest thereon at the rate specified
one of such safe and sound practices aimed at ensuring the solvency and
in the mortgage, and all the costs and expenses incurred by
liquidity of our banks.1âwphi1 It cannot therefore be disputed that the said
the bank or institution from the sale and custody of said
provision amending the redemption period in Act 3135 was based on a
property less the income derived therefrom. However, the
reasonable classification and germane to the purpose of the law.
purchaser at the auction sale concerned whether in a
x This legitimate public interest pursued by the legislature further enfeebles
judicial or extrajudicial foreclosure shall have the right to
petitioner’s impairment of contract theory.
enter upon and take possession of such property
x The right of redemption being statutory, it must be exercised in the
immediately after the date of the confirmation of the
manner prescribed by the statute,29 and within the prescribed time
auction sale and administer the same in accordance with
limit, to make it effective. Furthermore, as with other individual rights to
law. Any petition in court to enjoin or restrain the conduct
contract and to property, it has to give way to police power exercised for public
of foreclosure proceedings instituted pursuant to this
welfare.30 The concept of police power is well-established in this jurisdiction. It
provision shall be given due course only upon the filing by
has been defined as the "state authority to enact legislation that may interfere
the petitioner of a bond in an amount fixed by the court
with personal liberty or property in order to promote the general welfare." Its
59
RECTO, GAYLE ANGELI M.
2011-0008 | AUSL
Personal Notes on Remedial Law 2 Review (based on the syllabus of Prof. Henedino M. Brondial)

scope, ever-expanding to meet the exigencies of the times, even to anticipate commissioners for further report of facts; or set aside the report and appoint new
the future where it could be done, provides enough room for an efficient and commissioners; or accept the report in part and reject it in part; and may make such order
flexible response to conditions and circumstances thus assuming the greatest and render such judgment as shall effectuate a fair and just partition of the real estate, or of
benefits.31 its value, if assigned or sold as above provided, between the several owners thereof. (7)
x The freedom to contract is not absolute; all contracts and all rights are
subject to the police power of the State and not only may regulations 5. Judgment
which affect them be established by the State, but all such regulations
must be subject to change from time to time, as the general well-being
Section 11. The judgment and its effect; copy to be recorded in registry of deeds. —
of the community may require, or as the circumstances may change, or
If actual partition of property is made, the judgment shall state definitely, by metes and
as experience may demonstrate the necessity.32 Settled is the rule that
bounds and adequate description, the particular portion of the real estate assigned to each
the non-impairment clause of the Constitution must yield to the loftier purposes
party, and the effect of the judgment shall be to vest in each party to the action in severalty
targeted by the Government. The right granted by this provision must submit to
the portion of the real estate assigned to him. If the whole property is assigned to one of the
the demands and necessities of the State’s power of regulation.33 Such authority
parties upon his paying to the others the sum or sums ordered by the court, the judgment
to regulate businesses extends to the banking industry which, as this Court has
shall state the fact of such payment and of the assignment of the real estate to the party
time and again emphasized, is undeniably imbued with public interest.34
making the payment, and the effect of the judgment shall be to vest in the party making the
x Having ruled that the assailed Section 47 of R.A. No. 8791 is constitutional, we
payment the whole of the real estate free from any interest on the part of the other parties to
find no reversible error committed by the CA in holding that petitioner can no
the action. If the property is sold and the sale confirmed by the court, the judgment shall state
longer exercise the right of redemption over its foreclosed properties after the
the name of the purchaser or purchasers and a definite description of the parcels of real estate
certificate of sale in favor of respondent had been registered.
sold to each purchaser, and the effect of the judgment shall be to vest the real estate in the
purchaser or purchasers making the payment or payments, free from the claims of any of the
parties to the action. A certified copy of the judgment shall in either case be recorded in the
____________________________________________
registry of deeds of the place in which the real estate is situated, and the expenses of such
recording shall be taxed as part of the costs of the action.
PARTITON [Rule 69]

1. Complaint CELESTINO BALUS, Petitioner, vs. SATURNINO BALUS and LEONARDA BALUS VDA. DE
CALUNOD, Respondents.
Section 1. Complaint in action for partition of real estate. — A person having the G.R. No. 168970 January 15, 2010
right to compel the partition of real estate may do so as provided in this Rule, setting forth in THIRD DIVISION
his complaint the nature and extent of his title and an adequate description of the real estate
of which partition is demanded and joining as defendants all other persons interested in the FACTS:
property. x Rufo Balus (father of the parties herein) Æ mortgaged a parcel of land, which he
owns, as security for a loan he obtained from the Rural Bank of Maigo, Lanao del
Norte
2. Order
x Rufo failed to pay
x mortgaged property was foreclosed and was subsequently sold to the Bank as
Section 2. Order for partition and partition by agreement thereunder. — If after the the sole bidder at a public auction held for that purpose
trial the court finds that the plaintiff has the right thereto, it shall order the partition of the real x Rufo failed to redeem
estate among all the parties in interest. Thereupon the parties may, if they are able to agree, x Upon death of their parents, Celestino, Saturnino and Leornarda executed an
make the partition among themselves by proper instruments of conveyance, and the court Extrajudicial Settlement of Estate5 adjudicating to each of them a specific one-
shall confirm the partition so agreed upon by all the parties, and such partition, together with third portion of the subject property
the order of the court confirming the same, shall be recorded in the registry of deeds of the o The Extrajudicial Settlement also contained provisions wherein the
place in which the property is situated. (2a) parties admitted knowledge of the fact that their father mortgaged
the subject property to the Bank and that they intended to redeem
A final order decreeing partition and accounting may be appealed by any party aggrieved the same at the soonest possible time.
thereby. x Saturnino and Leonarda (3 years after execution of the Extrajudicial Settlement)
Æ bought the subject property from the Bank
3. Stages of Partition o TCT was issued in their favor
x Celestino Æ maintained possession over the subject property
4. Rule of Commissioners x Saturnino and Leonarda Æ filed a Complaint8 for Recovery of Possession and
Damages against petitioner, contending that they had already informed
petitioner of the fact that they were the new owners of the disputed property,
Section 3. Commissioners to make partition when parties fail to agree. — If the
but the petitioner still refused to surrender possession of the same to them.
parties are unable to agree upon the partition, the court shall appoint not more than three (3)
o Respondents claimed that they had exhausted all remedies for the
competent and disinterested persons as commissioners to make the partition, commanding
amicable settlement of the case, but to no avail.
them to set off to the plaintiff and to each party in interest such part and proportion of the
x RTC Æ ordered Saturnino and Leonarda to execute a Deed of Sale in favor of the
property as the court shall direct. (3a)
Celestino, the one-third share of the property in question, presently possessed
by him, and described in the deed of partition
Section 4. Oath and duties of commissioners. — Before making such partition; the
o The RTC held that the right of petitioner to purchase from the
commissioners shall take and subscribe an oath that they will faithfully perform their duties as
respondents his share in the disputed property was recognized by
commissioners, which oath shall be filed in court with the other proceedings in the case. In
the provisions of the Extrajudicial Settlement of Estate, which the
making the partition, the commissioners shall view and examine the real estate, after due
parties had executed before the respondents bought the subject lot
notice to the parties to attend at such view and examination, and shall hear the parties as to
from the Bank.
their preference in the portion of the property to be set apart to them and the comparative
x Saturnino and Leonarda Æ appealed to CA
value thereof, and shall set apart the same to the parties in lots or parcels as will be most
x CA Æ reversed RTC
advantageous and equitable, having due regard to the improvements, situation and quality of
the different parts thereof. (4a) o ordering petitioner to immediately surrender possession of the
subject property to the respondents
Section 5. Assignment or sale of real estate by commissioners. — When it is made to o when petitioner and respondents did not redeem the subject
appear to the commissioners that the real state, or a portion thereof, cannot be divided property within the redemption period and allowed the
consolidation of ownership and the issuance of a new title in the
without prejudice to the interests of the parties, the court may order it assigned to one of the
parties willing to take the same, provided he pays to the other parties such amount as the name of the Bank, their co-ownership was extinguished.
commissioners deem equitable, unless one of the interested parties asks that the property be
sold instead of being so assigned, in which case the court shall order the commissioners to sell ISSUE: Whether there was co-ownership between and among Celestino, Saturnino and
the real estate at public sale under such conditions and within such time as the court may Leonarda over the property in that Celestino may only be ordered by the court to deliver 2/3
of the subject lot to his two siblings.
determine. (5a)

Section 6. Report of commissioners; proceedings not binding until confirmed. — HELD: NO.
x Petitioner and respondents are arguing on the wrong premise that, at
The commissioners shall make a full and accurate report to the court of all their proceedings
the time of the execution of the Extrajudicial Settlement, the subject
as to the partition, or the assignment of real estate to one of the parties, or the sale of the
property formed part of the estate of their deceased father to which
same. Upon the filing of such report, the clerk of court shall serve copies thereof on all the
they may lay claim as his heirs.
interested parties with notice that they are allowed ten (10) days within which to file
objections to the findings of the report, if they so desire. No proceeding had before or x At the outset, it bears to emphasize that there is no dispute with respect to the
conducted by the commissioners and rendered judgment thereon. (6a) fact that the subject property was exclusively owned by petitioner and
respondents' father, Rufo, at the time that it was mortgaged in 1979. This was
stipulated by the parties during the hearing conducted by the trial court on
Section 7. Action of the court upon commissioners report. — Upon the expiration of
October 28, 1996.12 Evidence shows that a Definite Deed of Sale13 was issued
the period of ten (10) days referred to in the preceding section or even before the expiration of
in favor of the Bank on January 25, 1984, after the period of redemption expired.
such period but after the interested parties have filed their objections to the report or their
There is neither any dispute that a new title was issued in the Bank's name
statement of agreement therewith the court may, upon hearing, accept the report and render
judgment in accordance therewith, or, for cause shown recommit the same to the
60
RECTO, GAYLE ANGELI M.
2011-0008 | AUSL
Personal Notes on Remedial Law 2 Review (based on the syllabus of Prof. Henedino M. Brondial)

before Rufo died on July 6, 1984. Hence, there is no question that the Bank
acquired exclusive ownership of the contested lot during the lifetime of Rufo. FACTS:
x The rights to a person's succession are transmitted from the moment of his x Antonio Feliciano passed away, leaving behind his only property, a parcel of land
death.14 In addition, the inheritance of a person consists of the property and located at Bunga4 Mayor, Bustos, Bulacan
transmissible rights and obligations existing at the time of his death, as well as x Leona, Maria, Pedro and Salina, all surnamed Feliciano, declared themselves to
those which have accrued thereto since the opening of the succession.15 In the be the only surviving heirs of Antonio Feliciano, with the exception of Salina
present case, since Rufo lost ownership of the subject property during o executed an extrajudicial settlement of Antonio Feliciano’s estate6
his lifetime, it only follows that at the time of his death, the disputed and appropriated among themselves the said parcel of land, to the
parcel of land no longer formed part of his estate to which his heirs exclusion of the heirs of Esteban Feliciano and Doroteo
may lay claim. Stated differently, petitioner and respondents never inherited Feliciano, deceased children of Antonio Feliciano
the subject lot from their father. x Leona, Maria, Pedro and Salina Æ executed a deed of absolute sale or Kasulatan
x Petitioner and respondents, therefore, were wrong in assuming that sa Ganap Na Bilihan over the property in favor of the late Jacinto Feliciano
they became co-owners of the subject lot. Thus, any issue arising from (Pedro’s portion), Felisa Feliciano (Salina’s portion) and Pedro Canoza (Leona and
the supposed right of petitioner as co-owner of the contested parcel of Maria’s portions)
land is negated by the fact that, in the eyes of the law, the disputed lot o Jacinto (during lifetime) Æ applied for a free patent over the
did not pass into the hands of petitioner and respondents as portion of land he bought, declaring that the same was a public
compulsory heirs of Rufo at any given point in time. land, first occupied and cultivated by Pedro Feliciano; GRANTED
x The foregoing notwithstanding, the Court finds a necessity for a complete o Pedro Æ also applied for a free patent over the portion of land
determination of the issues raised in the instant case to look into petitioner's which he bought, claiming that the same was public land, first
argument that the Extrajudicial Settlement is an independent contract which occupied and cultivated by Leona and Maria Feliciano; GRANTED
gives him the right to enforce his right to claim a portion of the disputed lot x Eugenio Feliciano and Angelina Feliciano-de Leon, surviving heirs of the late
bought by respondents.1avvphi1 Esteban Feliciano, and Trinidad Feliciano-Valiente and Basilia Feliciano-Trinidad
x It is true that under Article 1315 of the Civil Code of the Philippines, contracts (surviving heirs of DOROTEO) Æ filed complaint against Salina Feliciano, Felisa
are perfected by mere consent; and from that moment, the parties are bound not Feliciano, Pedro Canoza and the heirs of the late Jacinto Feliciano, namely Delia,
only to the fulfillment of what has been expressly stipulated but also to all the Rosauro, Elsa, Nardo and Ponciano, all surnamed Feliciano, for the Declaration of
consequences which, according to their nature, may be in keeping with good faith, Nullity of Documents and Title, Recovery of Real Property and Damages
usage and law. o They alleged that the settlement of the estate and sale were done
x Article 1306 of the same Code also provides that the contracting parties may without their participation and consent as heirs of Esteban and
establish such stipulations, clauses, terms and conditions as they may deem Doroteo. Likewise, they averred that the ancestral home of the
convenient, provided these are not contrary to law, morals, good customs, public Felicianos is erected on the subject property and that they have
order or public policy. occupied the same since birth. Canoza and Jacinto falsely declared
x In the present case, however, there is nothing in the subject Extrajudicial that the property was not occupied, so their titles to the property
Settlement to indicate any express stipulation for petitioner and respondents to should be declared null and void on the ground that they have
continue with their supposed co-ownership of the contested lot. made false statements in their respective applications for free
x On the contrary, a plain reading of the provisions of the Extrajudicial Settlement patent.
would not, in any way, support petitioner's contention that it was his and his x Doroteo’s heirs Æ filed an amended complaint
sibling's intention to buy the subject property from the Bank and continue what o to include the allegation that they sought to recover the shares of
they believed to be co-ownership thereof. It is a cardinal rule in the their fathers, Esteban and Doroteo, which they could have acquired
interpretation of contracts that the intention of the parties shall be accorded as heirs of Antonio Feliciano
primordial consideration.16 It is the duty of the courts to place a practical and x Pedro and spouse Delia Æ filed their answer
realistic construction upon it, giving due consideration to the context in which it o alleged that they were buyers in good faith and for value.
is negotiated and the purpose which it is intended to serve.17 Such intention is o They likewise contended that assuming that there was preterition
determined from the express terms of their agreement, as well as their of legal heirs, they never took part in it.
contemporaneous and subsequent acts.18 Absurd and illogical interpretations o As affirmative defenses, they alleged that the complaint failed to
should also be avoided.19 state a cause of action; the lower court had no jurisdiction as the
x For petitioner to claim that the Extrajudicial Settlement is an agreement between subject of the case were free patents and therefore prior
him and his siblings to continue what they thought was their ownership of the exhaustion of administrative remedies was required; the case was
subject property, even after the same had been bought by the Bank, is prematurely filed; no effort was exerted towards a settlement;
stretching the interpretation of the said Extrajudicial Settlement too far. plaintiffs’ right has prescribed; Eugenio Feliciano was a mere
x In the first place, as earlier discussed, there is no co-ownership to talk squatter who should be ordered to vacate; the deed of sale was
about and no property to partition, as the disputed lot never formed validly, genuinely and duly executed; Eugenio and Angelina were
part of the estate of their deceased father. guilty of misleading the court because there were other heirs who
x Moreover, petitioner's asseveration of his and respondents' intention of were indispensable parties but who were not included; and
continuing with their supposed co-ownership is negated by no less than his Presidential Decree No. 1508 or the Revised Katarungang
assertions in the present petition that on several occasions he had the chance to Pambarangay Law was not resorted to by plaintiffs.
purchase the subject property back, but he refused to do so. In fact, he claims x Rosauro Feliciano, Elsa Feliciano and Ponciano Feliciano Æ likewise filed an
that after the Bank acquired the disputed lot, it offered to re-sell the same to him Answer15
but he ignored such offer. How then can petitioner now claim that it was also his o containing the same allegations and defenses as respondents Pedro
intention to purchase the subject property from the Bank, when he admitted that Canoza and Delia Feliciano.
he refused the Bank's offer to re-sell the subject property to him? x The other defendants, Salina Feliciano, Felisa Feliciano and Nardo Feliciano were
x In addition, it appears from the recitals in the Extrajudicial Settlement that, at declared in default.
the time of the execution thereof, the parties were not yet aware that the x RTC Æ in favor of Doroteo’s heirs
subject property was already exclusively owned by the Bank. Nonetheless, the o The trial court explained that by operation of law, the plaintiffs
lack of knowledge on the part of petitioner and respondents that the mortgage (herein petitioners) have as much right as Leona, Maria, Pedro and
was already foreclosed and title to the property was already transferred to the Salina Feliciano to inherit the property in question, and they cannot
Bank does not give them the right or the authority to unilaterally declare be deprived of their right unless by disinheritance for causes set
themselves as co-owners of the disputed property; otherwise, the disposition of forth in the law. When Leona Feliciano, Pedro Feliciano, Maria
the case would be made to depend on the belief and conviction of the party- Feliciano and Salina Feliciano appropriated the disputed lot solely to
litigants and not on the evidence adduced and the law and jurisprudence themselves through the extrajudicial settlement of estate, they
applicable thereto. committed a fraudulent act. To the extent that Doroteo and
x Furthermore, petitioner's contention that he and his siblings intended to continue Esteban were deprived of their rightful share, the said out-of-court
their supposed co-ownership of the subject property contradicts the provisions of settlement was annullable, said the trial court. The trial court also
the subject Extrajudicial Settlement where they clearly manifested their intention declared that Pedro Canoza was not a buyer in good faith of Leona
of having the subject property divided or partitioned by assigning to each of the and Maria’s shares. Records show that Pedro Canoza’s live-in
petitioner and respondents a specific 1/3 portion of the same. Partition calls for partner, Delia Feliciano, was a relative of the petitioners and the
the segregation and conveyance of a determinate portion of the property owned other defendants; thus, he could be reasonably charged with the
in common. It seeks a severance of the individual interests of each co-owner, knowledge of petitioners’ status vis-à-vis the subject property. The
vesting in each of them a sole estate in a specific property and giving each one a acquisition by Canoza and Jacinto Feliciano of free patent titles
right to enjoy his estate without supervision or interference from the other.20 In over portions of the contested lot also did not legitimize their
other words, the purpose of partition is to put an end to co-ownership,21 an ownership thereof, as they acquired no greater rights over the
objective which negates petitioner's claims in the present case. property than their predecessors-in-interest, having merely stepped
into their shoes
x CA Æ reversed RTC; denied ensuing MR
EUGENIO FELICIANO, substituted by his wife CEFERINA DE PALMA- FELICIANO, ANGELINA o The CA ruled that prescription had set in, citing the case of Pedrosa
DE LEON, representing the heirs of ESTEBAN FELICIANO, TRINIDAD VALIENTE, AND BASILIA v. Court of Appeals,20 which held that the applicable prescriptive
TRINIDAD, represented by her son DOMINADOR T. FELICIANO, Petitioners, vs. PEDRO period to annul a deed of extrajudicial settlement is four (4) years
CANOZA, DELIA FELICIANO, ROSAURO FELICIANO, ELSA FELICIANO AND PONCIANO from the discovery of the fraud. It reasoned that when petitioners
FELICIANO, Respondents. filed the instant complaint for the annulment of the extrajudicial
G.R. No. 161746 September 1, 2010 settlement of Antonio Feliciano’s estate, more than four (4) years
THIRD DIVISION had elapsed from the issuance of the free patents. As regards the
61
RECTO, GAYLE ANGELI M.
2011-0008 | AUSL
Personal Notes on Remedial Law 2 Review (based on the syllabus of Prof. Henedino M. Brondial)

portion claimed by the late Jacinto Feliciano, sixteen (16) years had registration of the Free Patent, they clearly failed to institute the
elapsed from the time the free patent was issued to him before present civil action within the allowable period. The same result
petitioners filed the complaint, while in the case of Canoza, obtains even if their complaint is treated as one (1) essentially for
fourteen (14) years had elapsed from the issuance of the free reconveyance as more than ten (10) years have passed since
patent in Canoza’s favor. Hence, according to the CA, the action for petitioners’ cause of action accrued. The CA committed no error in
the annulment of the documents had prescribed. dismissing their complaint.

ISSUE: Whether the COA of Doroteo’s heirs has prescribed, considering that the action was
filed only 16 years and 14 years after Jacinto and Feliciano were issued free patents CARMELA BROBIO MANGAHAS, Petitioner, vs. EUFROCINA A. BROBIO, Respondent.
respectively. G.R. No. 183852 October 20, 2010
SECOND DIVISION
HELD: YES.
x While respondents have not assigned the defense of prescription in their appeal FACTS:
before the CA, they raised such defense in their December 1, 1993 Answer as x Pacifico S. Brobio (Pacifico) died intestate, leaving three parcels of land.
one (1) of their affirmative defenses.22 In their brief before the CA, respondents o He was survived by:
specifically prayed for the reliefs mentioned in their respective answers before ƒ his wife, respondent Eufrocina A. Brobio, and
the trial court. Thus, by reference, they are deemed to have adopted the defense ƒ four legitimate and
of prescription, and could not properly be said to have waived the defense of ƒ three illegitimate children
prescription. x petitioner Carmela Brobio Mangahas is
x Moreover, Rule 9, Section 1 of the 1997 Rules of Civil Procedure, as amended, one of the illegitimate children.
provides that when it appears from the pleadings or the evidence on record that x Eufrocina (and 4 LC) Æ executed a Deed of Extrajudicial Settlement of Estate of
the action is already barred by the statute of limitations, the court shall dismiss the Late Pacifico Brobio with Waiver
the claim. Thus, in Gicano v. Gegato,23 we held: o in consideration of their love and affection for respondent and the
o We have ruled that trial courts have authority and discretion to sum of P150,000.00, waived and ceded their respective shares over
dismiss an action on the ground of prescription when the parties’ the three parcels of land in favor of respondent
pleadings or other facts on record show it to be indeed time-barred x Carmela Æ claimed that Eufrocina promised to give her an additional amount for
x x x; and it may do so on the basis of a motion to dismiss, or an her share in her father’s estate.
answer which sets up such ground as an affirmative defense; or o Thus, after the signing of the Deed, petitioner demanded from
even if the ground is alleged after judgment on the merits, as in a respondent the promised additional amount, but respondent
motion for reconsideration; or even if the defense has not been refused to pay, claiming that she had no more money
asserted at all, as where no statement thereof is found in the x Eufrocina (after being required to submit an original copy of the Deed by the
pleadings, or where a defendant has been declared in default. BIR) Æ asked her to countersign a copy of the Deed x
What is essential only, to repeat, is that the facts demonstrating Carmela refused
the lapse of the prescriptive period, be otherwise sufficiently and o demanding that respondent first give her the additional amount
satisfactorily apparent on the record: either in the averments of the that she promised. Considering the value of the three parcels of
plaintiffs complaint, or otherwise established by the evidence. land (which she claimed to be worth P20M), petitioner asked for
(Underscoring supplied.) P1M, but respondent begged her to lower the amount.
x But did the CA nonetheless commit error when it held that the applicable o Petitioner agreed to lower it to P600,000.00. Because respondent
prescriptive period is four (4) years? did not have the money at that time and petitioner refused to
o Petitioners argue that the CA erroneously treated the action they countersign the Deed without any assurance that the amount
filed at the trial court as one (1) for annulment of the extrajudicial would be paid, respondent executed a promissory note.
settlement and applied the four (4)-year prescriptive period in o Petitioner agreed to sign the Deed when respondent signed the
dismissing the same. They contend that the action they filed was one promissory note
(1) for Declaration of Nullity of Documents and Titles, Recovery of x Eufrocina Æ failed to pay upon demand
Real Property and Damages, and as such, their action was x Carmela Æ filed a Complaint for Specific Performance with Damages
imprescriptible pursuant to Article 141024 of the Civil Code. x Eufrocina Æ filed answer
o Respondents, for their part, maintain that the CA did not err in o Claimed that she was only forced to sign the PN
holding that the deed of extrajudicial partition executed without x RTC Æ in favor of Carmela
including some of the heirs, who had no knowledge of the partition o The RTC found that the alleged "pressure and confused disposition"
and did not consent thereto, is merely fraudulent and not void. They experienced by respondent and the circumstances that led to the
stress that the action to rescind the partition based on fraud execution of the promissory note do not constitute undue influence
prescribes in four (4) years counted from the date of registration, as would vitiate respondent’s consent thereto.
which is constructive notice to the whole world. o The court maintained that the promissory note was an additional
x We affirm the ruling of the CA. As the records show, the heirs of consideration for the waiver of petitioner’s share in the three
Doroteo and Esteban did not participate in the extrajudicial partition properties in favor of respondent. Its conclusion was bolstered by
executed by Salina with the other compulsory heirs, Leona, Maria and the fact that the promissory note was executed after negotiation
Pedro. Undeniably, the said deed was fraudulently obtained as it and haggling between the parties.
deprived the known heirs of Doroteo and Esteban of their shares in the x CA Æ reversed RTC
estate. A deed of extrajudicial partition executed without including o The CA found that there was a complete absence of consideration
some of the heirs, who had no knowledge of and consent to the same, in the execution of the promissory note, which made it inexistent
is fraudulent and vicious.25 Hence, an action to set it aside on the and without any legal force and effect. The court noted that
ground of fraud could be instituted. Such action for the annulment of the "financial assistance" was not the real reason why respondent
said partition, however, must be brought within four (4) years from the executed the promissory note, but only to secure petitioner’s
discovery of the fraud.1avvphi1 signature. The CA held that the waiver of petitioner’s share in the
x In Gerona v. De Guzman,26 respondents therein executed a deed of extrajudicial three properties, as expressed in the deed of extrajudicial
settlement declaring themselves to be the sole heirs of the late Marcelo de settlement, may not be considered as the consideration of the
Guzman. They secured new transfer certificates of title in their own names, promissory note, considering that petitioner signed the Deed way
thereby excluding the petitioners therein from the estate of the deceased. The back in 2002 and she had already received the consideration of
petitioners brought an action for the annulment of the said deed upon the P150,000.00 for signing the same. The CA went on to hold that if
ground that the same is tainted with fraud. The Court held, petitioner disagreed with the amount she received, then she should
o Inasmuch as petitioners seek to annul the aforementioned deed of have filed an action for partition.
"extra-judicial settlement" upon the ground of fraud in the o CA found that intimidation attended the signing of the promissory
execution thereof, the action therefor may be filed within four (4) note. Respondent needed the Deed countersigned by petitioner in
years from the discovery of the fraud (Mauricio v. Villanueva, L- order to comply with a BIR requirement; and, with petitioner’s
11072, September 24, 1959). Such discovery is deemed to have refusal to sign the said document, respondent was forced to sign
taken place, in the case at bar, on June 25, 1948, when said the promissory note to assure petitioner that the money promised
instrument was filed with the Register of Deeds and new to her would be paid.
certificates of title were issued in the name of respondents x Carmela Æ filed Rule 45
exclusively, for the registration of the deed of extra-judicial
settlement constitute constructive notice to the whole world.27 ISSUE: Whether the CA is correct in ruling that Carmela’s remedy should have been an action
(Emphasis and underscoring supplied.) for partition
x Evidently, the applicable prescriptive period to institute the action to
annul the deed of extrajudicial settlement was four (4) years counted HELD: NO.
from the discovery of fraud as held in the case of Gerona v. De x Contracts are voidable where consent thereto is given through mistake, violence,
Guzman.28 However, the records show that petitioners’ complaint was filed intimidation, undue influence, or fraud. In determining whether consent is
only on October 18, 1993, or almost sixteen (16) years after Jacinto vitiated by any of these circumstances, courts are given a wide latitude in
Feliciano was issued Free Patent No. (IV-4) 012293 on November 28, weighing the facts or circumstances in a given case and in deciding in favor of
1977, and almost fourteen (14) years from the time Pedro Canoza was what they believe actually occurred, considering the age, physical infirmity,
issued OCT No. P-364 on November 28, 1979. As petitioners are intelligence, relationship, and conduct of the parties at the time of the execution
deemed to have obtained constructive notice of the fraud upon the
62
RECTO, GAYLE ANGELI M.
2011-0008 | AUSL
Personal Notes on Remedial Law 2 Review (based on the syllabus of Prof. Henedino M. Brondial)

of the contract and subsequent thereto, irrespective of whether the contract is in Section 4. Pleadings allowed. — The only pleadings allowed to be filed are the complaint,
a public or private writing.14 compulsory counterclaim and cross-claim pleaded in the answer, and the answers thereto. All
x Nowhere is it alleged that mistake, violence, fraud, or intimidation attended the pleadings shall be verified. (3a, RSP)
execution of the promissory note. Still, respondent insists that she was "forced"
into signing the promissory note because petitioner would not sign the document Section 5. Action on complaint. — The court may, from an examination of the allegations
required by the BIR. In one case, the Court - in characterizing a similar in the complaint and such evidence as may be attached thereto, dismiss the case outright on
argument by respondents therein - held that such allegation is tantamount to any of the grounds for the dismissal of a civil action which are apparent therein. If no ground
saying that the other party exerted undue influence upon them. However, the for dismissal is found, it shall forthwith issue summons. (n)
Court said that the fact that respondents were "forced" to sign the documents
does not amount to vitiated consent.15 Section 6. Answers. — Within ten (10) days from service of summons, the defendant shall
x There is undue influence when a person takes improper advantage of his power file his answer to the complaint and serve a copy thereof on the plaintiff. Affirmative and
over the will of another, depriving the latter of a reasonable freedom of choice.16 negative defenses not pleaded therein shall be deemed waived, except lack of jurisdiction over
For undue influence to be present, the influence exerted must have so the subject matter. Cross-claims and compulsory counterclaims not asserted in the answer
overpowered or subjugated the mind of a contracting party as to destroy his free shall be considered barred. The answer to counterclaims or cross-claims shall be served and
agency, making him express the will of another rather than his own.17 filed within ten (10) days from service of the answer in which they are pleaded. (5 RSP)
x Respondent may have desperately needed petitioner’s signature on the Deed,
but there is no showing that she was deprived of free agency when she signed Section 7. Effect of failure to answer. — Should the defendant fail to answer the
the promissory note. Being forced into a situation does not amount to vitiated complaint within the period above provided, the court, motu proprio or on motion of the
consent where it is not shown that the party is deprived of free will and choice. plaintiff, shall render judgment as may be warranted by the facts alleged in the complaint and
Respondent still had a choice: she could have refused to execute the promissory limited to what is prayed for therein. The court may in its discretion reduce the amount of
note and resorted to judicial means to obtain petitioner’s signature. Instead, damages and attorney's fees claimed for being excessive or otherwise unconscionable, without
respondent chose to execute the promissory note to obtain petitioner’s signature, prejudice to the applicability of section 3 (c), Rule 9 if there are two or more defendants.
thereby agreeing to pay the amount demanded by petitioner. (6, RSP)
x The fact that respondent may have felt compelled, under the circumstances, to
execute the promissory note will not negate the voluntariness of the act. As Section 8. Preliminary conference; appearance of parties. — Not later than thirty (30)
rightly observed by the trial court, the execution of the promissory note in the days after the last answer is filed, a preliminary conference shall be held. The provisions of
amount of P600,000.00 was, in fact, the product of a negotiation between the Rule 18 on pre-trial shall be applicable to the preliminary conference unless inconsistent with
parties. the provisions of this Rule.
x Contrary to the CA’s findings, the situation did not amount to intimidation that
vitiated consent.1awphil There is intimidation when one of the contracting parties The failure of the plaintiff to appear in the preliminary conference shall be cause for the
is compelled to give his consent by a reasonable and well-grounded fear of an dismissal of his complaint. The defendant who appears in the absence of the plaintiff shall be
imminent and grave evil upon his person or property, or upon the person or entitled to judgment on his counterclaim in accordance with the next preceding section. All
property of his spouse, descendants, or ascendants.19 Certainly, the payment of cross-claims shall be dismissed. (7, RSP)
penalties for delayed payment of taxes would not qualify as a "reasonable and
well-grounded fear of an imminent and grave evil." If a sole defendant shall fail to appear, the plaintiff shall likewise be entitled to judgment in
x We join the RTC in holding that courts will not set aside contracts merely accordance with the next preceding section. This procedure shall not apply where one of two
because solicitation, importunity, argument, persuasion, or appeal to affection or more defendants sued under a common cause of action defense shall appear at the
was used to obtain the consent of the other party. Influence obtained by preliminary conference.
persuasion or argument or by appeal to affection is not prohibited either in law
or morals and is not obnoxious even in courts of equity.20 No postponement of the preliminary conference shall be granted except for highly meritorious
x On the issue that the promissory note is void for not being supported by a grounds and without prejudice to such sanctions as the court in the exercise of sound
consideration, we likewise disagree with the CA. discretion may impose on the movant. (n)
x A contract is presumed to be supported by cause or consideration.21 The
presumption that a contract has sufficient consideration cannot be overthrown by Section 9. Record of preliminary conference. — Within five (5) days after the
a mere assertion that it has no consideration. To overcome the presumption, the termination of the preliminary conference, the court shall issue an order stating the matters
alleged lack of consideration must be shown by preponderance of evidence.22 taken up therein, including but not limited to:
The burden to prove lack of consideration rests upon whoever alleges it, which,
in the present case, is respondent. 1. Whether the parties have arrived at an amicable settlement, and if so, the terms
x Respondent failed to prove that the promissory note was not supported by any thereof;
consideration. From her testimony and her assertions in the pleadings, it is clear
that the promissory note was issued for a cause or consideration, which, at the 2. The stipulations or admissions entered into by the parties;
very least, was petitioner’s signature on the document.1avvphi1
x It may very well be argued that if such was the consideration, it was inadequate. 3. Whether, on the basis of the pleadings and the stipulations and admission made
Nonetheless, even if the consideration is inadequate, the contract would not be by the parties, judgment may be rendered without the need of further proceedings, in which
invalidated, unless there has been fraud, mistake, or undue influence.23 As event the judgment shall be rendered within thirty (30) days from issuance of the order;
previously stated, none of these grounds had been proven present in this case.
x The foregoing discussion renders the final issue insignificant. Be that as it may, 4. A clear specification of material facts which remain converted; and
we would like to state that the remedy suggested by the CA is not the
proper one under the circumstances. An action for partition implies 5. Such other matters intended to expedite the disposition of the case. (8, RSP)
that the property is still owned in common.24 Considering that the
heirs had already executed a deed of extrajudicial settlement and Section 10. Submission of affidavits and position papers. — Within ten (10) days from
waived their shares in favor of respondent, the properties are no longer receipt of the order mentioned in the next preceding section, the parties shall submit the
under a state of co-ownership; there is nothing more to be partitioned, affidavits of their witnesses and other evidence on the factual issues defined in the order,
as ownership had already been merged in one person. together with their position papers setting forth the law and the facts relied upon by them. (9,
RSP)

FORCIBLE ENTRY AND UNLAWFUL DETAINER [Rule 70] Section 11. Period for rendition of judgment. — Within thirty (30) days after receipt of
the affidavits and position papers, or the expiration of the period for filing the same, the court
shall render judgment.
1. Parties
However, should the court find it necessary to clarify certain material facts, during the said
Section 1. Who may institute proceedings, and when. — Subject to the provisions of period, issue an order specifying the matters to be clarified, and require the parties to submit
the next succeeding section, a person deprived of the possession of any land or building by affidavits or other evidence on the said matters within ten (10) days from receipt of said order.
force, intimidation, threat, strategy, or stealth, or a lessor, vendor, vendee, or other person Judgment shall be rendered within fifteen (15) days after the receipt of the last affidavit or the
against whom the possession of any land or building is unlawfully withheld after the expiration expiration of the period for filing the same.
or termination of the right to hold possession, by virtue of any contract, express or implied, or
the legal representatives or assigns of any such lessor, vendor, vendee, or other person, may, The court shall not resort to the foregoing procedure just to gain time for the rendition of the
at any time within one (1) year after such unlawful deprivation or withholding of possession, judgment. (n)
bring an action in the proper Municipal Trial Court against the person or persons unlawfully
withholding or depriving of possession, or any person or persons claiming under them, for the Section 12. Referral for conciliation. — Cases requiring referral for conciliation, where
restitution of such possession, together with damages and costs. (1a) there is no showing of compliance with such requirement, shall be dismissed without
prejudice, and may be revived only after that requirement shall have been complied with.
2. Procedure: Summary (18a, RSP)

Section 3. Summary procedure. — Except in cases covered by the agricultural tenancy Section 13. Prohibited pleadings and motions. — The following petitions, motions, or
laws or when the law otherwise expressly provides, all actions for forcible entry and unlawful pleadings shall not be allowed:
detainer, irrespective of the amount of damages or unpaid rentals sought to be recovered,
shall be governed by the summary procedure hereunder provided. (n) 1. Motion to dismiss the complaint except on the ground of lack of jurisdiction over
the subject matter, or failure to comply with section 12;

63
RECTO, GAYLE ANGELI M.
2011-0008 | AUSL
Personal Notes on Remedial Law 2 Review (based on the syllabus of Prof. Henedino M. Brondial)

2. Motion for a bill of particulars; Preliminary injunction


3. Motion for new trial, or for reconsideration of a judgment, or for reopening of Section 20. Preliminary mandatory injunction in case of appeal. — Upon motion of the
trial; plaintiff, within ten (10) days from the perfection of the appeal to the Regional Trial Court, the
latter may issue a writ of preliminary mandatory injunction to restore the plaintiff in
4. Petition for relief from judgment; possession if the court is satisfied that the defendant's appeal is frivolous or dilatory or that
the appeal of the plaintiff is prima facie meritorious. (9a)
5. Motion for extension of time to file pleadings, affidavits or any other paper;

6. Memoranda; 5. Appeals

7. Petition for certiorari, mandamus, or prohibition against any interlocutory order CARMENCITA SUAREZ, Petitioner, v. MR. AND MRS. FELIX E. EMBOY, JR. AND
issued by the court; MARILOU P. EMBOY-DELANTAR, Respondents.
G.R. No. 187944, March 12, 2014
8. Motion to declare the defendant in default;
FACTS:
9. Dilatory motions for postponement; x Subject lot is located in Cebu and owned by Carmencita
x Said lot was used to be a part of Lot No. 1907-A,8 which was partitioned in the
10. Reply; following manner among the heirs of Spouses Carlos Padilla (Carlos) and
Asuncion Pacres (Asuncion)
11. Third-party complaints;
Lot No. TCT No. Heirs
12. Interventions. (19a, RSP)
1907-A-1 T-54359 Spouses Rogelio and Praxedes Padilla
Section 14. Affidavits. — The affidavits required to be submitted under this Rule shall state
only facts of direct personal knowledge of the affiants which are admissible in evidence, and
shall show their competence to testify to the matters stated therein. Heirs of Vicente Padilla (Vicente),
namely: (1) Azucena Padilla, married to
A violation of this requirement may subject the party or the counsel who submits the same to Felly Carrera; (2) Remedios Padilla
disciplinary action, and shall be cause to expunge the inadmissible affidavit or portion thereof 1907-A-2 T-54360 (Remedios), married to Oscar Dimay;
(3) Veronica Padilla (Veronica);10 and (4)
from the record. (20, RSP) Moreno Padilla (Moreno), married to
Teresita Curso (Teresita)
Section 15. Preliminary injunction. — The court may grant preliminary injunction, in
accordance with the provisions of Rule 58 hereof, to prevent the defendant from committing
further acts of dispossession against the plaintiff. 1907-A-3 T-54361 Cresencio Padilla

A possessor deprived of his possession through forcible from the filing of the complaint, 1907-A-4 T-54362 Fructousa Baricuatro
present a motion in the action for forcible entry or unlawful detainer for the issuance of a writ
of preliminary mandatory injunction to restore him in his possession. The court shall decide the 1907-A-5 T-54363 Claudia Padilla-Emboy (Claudia)
motion within thirty (30) days from the filing thereof. (3a)

3. Judgment
x Felix and Marilou own the house which stood on the subject lot
Section 17. Judgment. — If after trial court finds that the allegations of the complaint are o They claim that their mother, Claudia, had occupied the subject lot
true, it shall render judgment in favor of the plaintiff for the restitution of the premises, the during her lifetime and it was earmarked to become her share in
sum justly due as arrears of rent or as reasonable compensation for the use and occupation of Lot No. 1907-A. They had thereafter stayed in the subject lot for
the premises, attorney's fees and costs. If a counterclaim is established, the court shall render decades after inheriting the same from Claudia, who had in turn
judgment for the sum found in arrears from either party and award costs as justice requires. succeeded her own parents, Carlos and Asuncion
(6a) x Heirs of Vicente Æ asked their cousins Felix and Marilou to VACATE the subject
lot
4. Immediate Execution o They refused
x They then received a demand letter from the lawyer of Carmencita on February
Section 19. Immediate execution of judgment; how to stay same. — If judgment is 23, 2004
o They were informed that Carmencita had already purchased on
rendered against the defendant, execution shall issue immediately upon motion unless an
appeal has been perfected and the defendant to stay execution files a sufficient supersedeas February 12, 2004 the subject lot from the former’s relatives.
bond, approved by the Municipal Trial Court and executed in favor of the plaintiff to pay the o However, the respondents did not heed the demand.
o Instead, they examined the records pertaining to the subject lot
rents, damages, and costs accruing down to the time of the judgment appealed from, and
unless, during the pendency of the appeal, he deposits with the appellate court the amount of and uncovered possible anomalies, i.e., forged signatures and
rent due from time to time under the contract, if any, as determined by the judgment of the alterations, in the execution of a series of deeds of partition relative
Municipal Trial Court. In the absence of a contract, he shall deposit with the Regional Trial to Lot No. 1907-A
Court the reasonable value of the use and occupation of the premises for the preceding month x Felix and Marilou Æ filed before the RTC of Cebu City a complaint13 for
or period at the rate determined by the judgment of the lower court on or before the tenth nullification of the partition and for the issuance of new TCTs covering the heirs’
day of each succeeding month or period. The supersedeas bond shall be transmitted by the respective portions of Lot No. 1907-A.
Municipal Trial Court, with the papers, to the clerk of the Regional Trial Court to which the x Carmencita Æ filed before the MTCC and against the respondents a complaint for
action is appealed. unlawful detainer
o She alleged that she bought the subject lot from Remedios,
All amounts so paid to the appellate court shall be deposited with said court or authorized Moreno, Veronica and Dionesia,15 the registered owners thereof and
government depositary bank, and shall be held there until the final disposition of the appeal, the persons who allowed the respondents to occupy the same by
unless the court, by agreement of the interested parties, or in the absence of reasonable mere tolerance. As their successor-in-interest, she claimed her
grounds of opposition to a motion to withdraw, or for justifiable reasons, shall decree entitlement to possession of the subject lot and the right to
otherwise. Should the defendant fail to make the payments above prescribed from time to time demand from the respondents to vacate the same
during the pendency of the appeal, the appellate court, upon motion of the plaintiff, and upon x MTCC Æ in favor of CARMENCITA; ordered Felix and Marilou to vacate
proof of such failure, shall order the execution of the judgment appealed from with respect to o RTC Æ affirmed
the restoration of possession, but such execution shall not be a bar to the appeal taking its x Felix and Marilou Æ appealed to CA
course until the final disposition thereof on the merits. x CA Æ reversed
o A close perusal of [Carmencita’s] complaint a quo reveals that the
After the case is decided by the Regional Trial Court, any money paid to the court by the action was neither one of forcible entry nor unlawful detainer but
defendant for purposes of the stay of execution shall be disposed of in accordance with the essentially involved an issue of ownership which must be resolved
provisions of the judgment of the Regional Trial Court. In any case wherein it appears that the in an accion reivindicatoria. It did not characterize [the
defendant has been deprived of the lawful possession of land or building pending the appeal respondents’] alleged entry into the land: whether the same was
by virtue of the execution of the judgment of the Municipal Trial Court, damages for such legal or illegal. It did not state how [the respondents] entered the
deprivation of possession and restoration of possession and restoration of possession may be land and constructed a house thereon. It was also silent on
allowed the defendant in the judgment of the Regional Trial Court disposing of the appeal. whether [the respondents’] possession became legal before
(8a) [Carmencita] demanded from them to vacate the land. The
complaint merely averred that their relatives previously owned the
Section 21. Immediate execution on appeal to Court of Appeals or Supreme Court. lot [the respondents] were occupying and that after [Carmencita]
— The judgment of the Regional Trial Court against the defendant shall be immediately purchased it[,] she, as its new owner, demanded [for the
executory, without prejudice to a further appeal that may be taken therefrom. (10a) respondents] to vacate the land. Moreover, it is undisputed that
[the respondents] and their ancestors have been occupying the
64
RECTO, GAYLE ANGELI M.
2011-0008 | AUSL
Personal Notes on Remedial Law 2 Review (based on the syllabus of Prof. Henedino M. Brondial)

land for several decades already. There was no averment as to x Again, this Court stresses that to give the court jurisdiction to effect the
how or when [Carmencita’s] predecessors tolerated [the ejectment of an occupant or deforciant on the land, it is necessary that the
respondents’] possession of the land. Consequently, there was no complaint must sufficiently show such a statement of facts as to bring the party
contract to speak of, whether express or implied, between [the clearly within the class of cases for which the statutes provide a remedy, without
respondents], on one hand, and [Carmencita] or her predecessors, resort to parol testimony, as these proceedings are summary in nature. In short,
on the other, as would qualify [the respondents’] possession of the the jurisdictional facts must appear on the face of the complaint. When the
land as a case of unlawful detainer. Neither was it alleged that complaint fails to aver facts constitutive of forcible entry or unlawful detainer, as
[the respondents] took possession of the land through force, where it does not state how entry was effected or how and when dispossession
intimidation, threat, strategy or stealth to make out a case of started, the remedy should either be an accion publiciana or accion
forcible entry. In any event, [Carmencita] cannot legally assert reivindicatoria.44
that [the respondents’] possession of the land was by mere x As an exception to the general rule, the respondents’ petition for
tolerance. This is because [Carmencita’s] predecessors-in-interest nullification of the partition of Lot No. 1907-A can abate Carmencita’s
did not yet own the property when [Claudia] took possession suit for unlawful detainer.
thereof. Take note that [Carmencita’s] predecessors-in-interest x In Amagan, the Court is emphatic that:
merely stepped into the shoes of their parents who were also co- o As a general rule, therefore, a pending civil action involving
heirs of [Claudia]. Finally, to categorize a cause of action as one ownership of the same property does not justify the
constitutive of unlawful detainer, plaintiff’s supposed acts of suspension of ejectment proceedings. “The underlying
tolerance must have been present from the start of the possession reasons for the above ruling were that the actions in the
which he later seek[s] to recover. This is clearly wanting in the Regional Trial Court did not involve physical or de facto
case at bar. possession, and, on not a few occasions, that the case in
the Regional Trial Court was merely a ploy to delay
ISSUE: Whether Carmencita’s complaint was correctly nominated as unlawful detainer. disposition of the ejectment proceeding, or that the issues
presented in the former could quite as easily be set up as
HELD: NO. defenses in the ejectment action and there resolved.”
x Carmencita had not amply alleged and proven that all the requisites for x Only in rare instances is suspension allowed to await the outcome of the pending
unlawful detainer are present in the case at bar. civil action. One such exception is Vda. de Legaspi v. Avendaño, wherein the
x “Without a doubt, the registered owner of real property is entitled to its Court declared:
possession. However, the owner cannot simply wrest possession thereof from o “x x x. Where the action, therefore, is one of illegal
whoever is in actual occupation of the property. To recover possession, he must detainer, as distinguished from one of forcible entry, and the
resort to the proper judicial remedy and, once he chooses what action to file, he right of the plaintiff to recover the premises is seriously
is required to satisfy the conditions necessary for such action to prosper.”37 placed in issue in a proper judicial proceeding, it is more
x In Spouses Valdez, Jr.,38 the Court is instructive anent the three kinds of actions equitable and just and less productive of confusion and disturbance
available to recover possession of real property, viz: of physical possession, with all its concomitant inconvenience and
o (a) accion interdictal; expenses. For the Court in which the issue of legal possession,
o (b) accion publiciana; and whether involving ownership or not, is brought to restrain, should a
o (c) accion reivindicatoria. petition for preliminary injunction be filed with it, the
x Accion interdictal comprises two distinct causes of action, namely, effects of any order or decision in the unlawful detainer case in
forcible entry (detentacion) and unlawful detainer (desahuico) [sic]. In order to await the final judgment in the more substantive case
forcible entry, one is deprived of physical possession of real property by involving legal possession or ownership. It is only where there has
means of force, intimidation, strategy, threats, or stealth whereas in been forcible entry that as a matter of public policy the right to
unlawful detainer, one illegally withholds possession after the physical possession should be immediately set at rest in favor of
expiration or termination of his right to hold possession under any the prior possession regardless of the fact that the other party
contract, express or implied. The two are distinguished from each might ultimately be found to have superior claim to the premises
other in that in forcible entry, the possession of the defendant is illegal involved, thereby to discourage any attempt to recover possession
from the beginning, and that the issue is which party has prior de facto thru force, strategy or stealth and without resorting to the courts.”
possession while in unlawful detainer, possession of the defendant is o Indisputably, the execution of the MCTC Decision would have
originally legal but became illegal due to the expiration or termination resulted in the demolition of the house subject of the ejectment
of the right to possess. suit; thus, by parity of reasoning, considerations of equity require
x The jurisdiction of these two actions, which are summary in nature, lies in the the suspension of the ejectment proceedings. We note that, like
proper municipal trial court or metropolitan trial court. Both actions must be Vda. de Legaspi, the respondent’s suit is one of unlawful detainer
brought within one year from the date of actual entry on the land, in case of and not of forcible entry. And most certainly, the ejectment of
forcible entry, and from the date of last demand, in case of unlawful detainer. petitioners would mean a demolition of their house, a matter that is
The issue in said cases is the right to physical possession. likely to create the “confusion, disturbance, inconveniences and
x Accion publiciana is the plenary action to recover the right of expenses” mentioned in the said exceptional case.
possession which should be brought in the proper regional trial court o Necessarily, the affirmance of the MCTC Decision would cause the
when dispossession has lasted for more than one year. It is an respondent to go through the whole gamut of enforcing it by
ordinary civil proceeding to determine the better right of possession of physically removing the petitioners from the premises they claim to
realty independently of title. In other words, if at the time of the filing have been occupying since 1937. (Respondent is claiming
of the complaint more than one year had elapsed since defendant had ownership only of the land, not of the house.) Needlessly, the
turned plaintiff out of possession or defendant’s possession had litigants as well as the courts will be wasting much time and effort
become illegal, the action will be, not one of the forcible entry or illegal by proceeding at a stage wherein the outcome is at best
detainer, but an accion publiciana. temporary, but the result of enforcement is permanent, unjust and
x On the other hand, accion reivindicatoria is an action to recover probably irreparable.
ownership also brought in the proper regional trial court in an ordinary o We should stress that respondent’s claim to physical possession is
civil proceeding.39 (Citations omitted) based not on an expired or a violated contract of lease, but
x In a complaint for unlawful detainer, the following key jurisdictional facts must allegedly on “mere tolerance.” Without in any way prejudging the
be alleged and sufficiently established: proceedings for the quieting of title, we deem it judicious under the
o (1) initially, possession of property by the defendant was by present exceptional circumstances to suspend the ejectment
contract with or by tolerance of the plaintiff; case.45 (Citations omitted)
o (2) eventually, such possession became illegal upon notice by x The Court then quoted with favor the following portion of the Decision dated July
plaintiff to defendant of the termination of the latter’s right of 8, 1997, penned by Associate Justice Artemio G. Tuquero in CA-G.R. No. 43611-
possession; SP, from which the Amagan case sprang:
o (3) thereafter, the defendant remained in possession of the o “ONE. Private respondent Teodorico T. Marayag anchors his action
property and deprived the plaintiff of the enjoyment thereof; and for unlawful detainer on the theory that petitioners’ possession of
o (4) within one year from the last demand on defendant to vacate the property in question was by mere tolerance. However, in
the property, the plaintiff instituted the complaint for ejectment.40 answer to his demand letter dated April 13, 1996 x x x, petitioners
x In the case at bar, the first requisite mentioned above is markedly categorically denied having any agreement with him, verbal or
absent. Carmencita failed to clearly allege and prove how and when the written, asserting that they are ‘owners of the premises we are
respondents entered the subject lot and constructed a house upon it.41 occupying at 108 J.P. Rizal Street, San Vicente, Silang, Cavite.’ In
Carmencita was likewise conspicuously silent about the details on who other words, it is not merely physical possession but ownership as
specifically permitted the respondents to occupy the lot, and how and when such well that is involved in this case.[”]
tolerance came about.42 Instead, Carmencita cavalierly formulated a legal o “TWO. In fact, to protect their rights to the premises in question,
conclusion, sans factual substantiation, that (a) the respondents’ initial petitioners filed an action for reconveyance, quieting of title and
occupation of the subject lot was lawful by virtue of tolerance by the registered damages against private respondents, docketed as Civil Case No.
owners, and (b) the respondents became deforciants unlawfully withholding the TG-1682 of the Regional Trial Court, Branch 18, Tagaytay City.
subject lot’s possession after Carmencita, as purchaser and new registered The issue of ownership is squarely raised in this action.
owner, had demanded for the former to vacate the property.43 It is worth Undoubtedly, the resolution of this issue will be determinative of
noting that the absence of the first requisite assumes even more importance in who is entitled to the possession of the premises in question.[”]
the light of the respondents’ claim that for decades, they have been occupying
the subject lot as owners thereof.
65
RECTO, GAYLE ANGELI M.
2011-0008 | AUSL
Personal Notes on Remedial Law 2 Review (based on the syllabus of Prof. Henedino M. Brondial)

o “THREE. The immediate execution of the judgment in the unlawful x Clearly then under said Sec. 19, Rule 70, a judgment on a forcible entry
detainer case will include the removal of the petitioners’ house and detainer action is made immediately executory to avoid further
[from] the lot in question.[”] injustice to a lawful possessor. The defendant in such a case may have such
o “To the mind of the Court it is injudicious, nay inequitable, to allow judgment stayed only by (a) perfecting an appeal; (b) filing a supersedeas
demolition of petitioners’ house prior to the determination of the bond; and (c) making a periodic deposit of the rental or reasonable
question of ownership [of] the lot on which it stands.”46 (Citation compensation for the use and occupancy of the property during the
omitted) pendency of the appeal.9 The failure of the defendant to comply with any of
x We find the doctrines enunciated in Amagan squarely applicable to the instant these conditions is a ground for the outright execution of the judgment, the duty
petition for reasons discussed hereunder. of the court in this respect being ministerial and imperative. Hence, if the
x Carmencita’s complaint for unlawful detainer is anchored upon the proposition defendant-appellant has perfected the appeal but failed to file a
that the respondents have been in possession of the subject lot by mere supersedeas bond, the immediate execution of the judgment would
tolerance of the owners. The respondents, on the other hand, raise the defense automatically follow. Conversely, the filing of a supersedeas bond will
of ownership of the subject lot and point to the pendency of Civil Case No. not stay the execution of the judgment if the appeal is not perfected.
CEB-30548, a petition for nullification of the partition of Lot No. 1907-A, in Necessarily then, the supersedeas bond should be filed within the
which Carmencita and the Heirs of Vicente were impleaded as parties. Further, period for the perfection of the appeal.10
should Carmencita’s complaint be granted, the respondents’ house, which has x In the case at bar, complainant lost his client’s case and appealed to
been standing in the subject lot for decades, would be subject to demolition. the RTC. His client has also been periodically depositing rental with the court for
The foregoing circumstances, thus, justify the exclusion of the instant petition the use of the property pending appeal. However, as ruled by the RTC, the
from the purview of the general rule. bond filed did not meet the legal requirements because first and
x All told, we find no reversible error committed by the CA in dismissing foremost, the bond posted was a property bond, not cash nor surety.
Carmencita’s complaint for unlawful detainer. As discussed above, the Furthermore, Rafols did not own the property he posted as bond and besides, it
jurisdictional requirement of possession by mere tolerance of the owners had not was also not issued in favour of the plaintiff in the ejectment case. Because of
been amply alleged and proven. Moreover, circumstances exist which justify the the non-compliance with the requirements under the above-quoted rule, the
abatement of the ejectment proceedings. Carmencita can ventilate her execution of the judgment was not effectively stayed. The only exceptions to
ownership claims in an action more suited for the purpose. The respondents, on non-compliance are the existence of fraud, accident, mistake or excusable
other hand, need not be exposed to the risk of having their house demolished negligence which prevented the defendant from posting the supersedeas bond or
pending the resolution of their petition for nullification of the partition of Lot No. making the monthly deposit, or the occurrence of supervening events which
1907-A, where ownership over the subject lot is likewise presented as an issue. brought about a material change in the situation of the parties and which would
make the execution inequitable.11 But whether or not these obtain in the case at
bar is an issue best left to the court that issued the writ of execution.
ATTY. VIRGILIO P. ALCONERA, Complainant, vs. ALFREDO T. PALLANAN, Respondent. x Given the above circumstances, there was no legal impediment preventing
A.M. No. P-12-3069 January 20, 2014 respondent sheriff from performing his responsibility of enforcing the writ of
THIRD DIVISION execution. Since Rafols failed to comply with the requirements under the Rules,
Cua Beng who prevailed in the unlawful detainer case is entitled as a matter of
FACTS: right to the immediate execution of the court’s judgment both as to the
x Atty Alconera was the counsel for Morito Rafols, the defendant in Civil Case No. restoration of possession and the payment of the accrued rentals or
5967-2, an unlawful detainer case entitled Cua Beng a.k.a. Manuel Sy and Ka compensation for the use and occupation of the premises.12
Kieng v. Morita Rafols, et al., filed before MTCC South Cotabato x Well-settled is that the sheriff’s duty in the execution of a writ is purely
o MTCC Æ ruled against Rafols and his co-defendants ministerial; he is to execute the order of the court strictly to the letter.
o Rafols, through complainant Alconera Æ appealed the case to the He has no discretion whether to execute the judgment or not. When
RTC the writ is placed in his hands, it is his duty, in the absence of any
x Cua Beng (Pending appeal before the RTC) Æ filed motion for execution instructions to the contrary, to proceed with reasonable celerity and
o MTCC Æ granted promptness to implement it in accordance with its mandate. It is only
x Rafols (through Alconera) Æ filed an MR by doing so could he ensure that the order is executed without undue
o MTCC Æ denied (the denial of the MR has not reached Alconera yet delay.13 This holds especially true herein where the nature of the case
at the time the writ was implemented) requires immediate execution. Absent a TRO, an order of quashal, or
x Rafols’ daughter, Evelyn, called Alconera and told the latter about the writ of compliance with Sec. 19, Rule 70 of the Rules of Court, respondent
execution being implemented by Sheriff Pallanan sheriff has no alternative but to enforce the writ.
x RTC Æ ruled that there was no pending Motion to Approve Supersedeas Bond x Immediacy of the execution, however, does not mean instant
filed with it. Instead, what was filed not with the RTC but with the MTCC was a execution. The sheriff must comply with the Rules of Court in executing a writ.
"NOTICE OF APPEAL - and - MOTION TO APPROVE PROPERTY SUPERSEDEAS Any act deviating from the procedure laid down in the Rules of Court is a
BOND," which was not granted. misconduct and warrants disciplinary action. In this case, Sec. 10(c), Rule 39 of
x Alconera Æ conforonted Pallana the Rules prescribes the procedure in the implementation of the writ. It provides:
o And altercation ensued x Section 10. Execution of judgments for specific act. —
x Alconera Æ filed a Complaint-Affidavit3 against the respondent sheriff for grave o (c) Delivery or restitution of real property. — The officer shall
misconduct, Discourtesy in the Performance of Official Duties, and Making demand of the person against whom the judgment for the delivery
Untruthful Statements or restitution of real property is rendered and all persons claiming
x Pallanan Æ filed comment rights under him to peaceably vacate the property within three (3)
o he averred that the duty of a court sheriff in enforcing a writ of working days, and restore possession thereof to the judgment
execution is ministerial, and without a TRO enjoining it, a sheriff is obligee, otherwise, the officer shall oust all such persons therefrom
duty bound to implement it. with the assistance, if necessary, of appropriate peace officers, and
x EJ of RTC Æ recommended that respondent Sheriff be ADMONISHED employing such means as may be reasonably necessary to retake
possession, and place the judgment obligee in possession of such
ISSUE: Whether Pallanan was correct in his position that, without a TRO, enforcement of a property. Any costs, damages, rents or profits awarded by the
writ of execution in an ejectment case is his ministerial duty. judgment shall be satisfied in the same manner as a judgment for
money.
HELD: YES x Based on this provision, enforcement in ejectment cases requires the
x It must be borne in mind that the case at bar traces its roots to an unlawful sheriff to give notice of such writ and to demand from defendant to
detainer case wherein the MTCC ruled against Rafols, complainant’s client. In vacate the property within three days. Only after such period can the
ejectment cases, the rulings of the courts are immediately executory and can sheriff enforce the writ by the bodily removal of the defendant in the
only be stayed via compliance with Section 19, Rule 70 of the Rules of Court, to ejectment case and his personal belongings. Even in cases wherein
wit: decisions are immediately executory, the required three-day notice
o Section 19. Immediate execution of judgment; how to stay same. cannot be dispensed with. A sheriff who enforces the writ without the
— If judgment is rendered against the defendant, execution shall required notice or before the expiry of the three-day period is running
issue immediately upon motion, unless an appeal has been afoul with the Rules.15
perfected and the defendant to stay execution files a sufficient x In the present controversy, the Order denying the motion for reconsideration
supersedeas bond, approved by the Municipal Trial Court and was allegedly served, according to the respondent, on the same day the writ was
executed in favor of the plaintiff to pay the rents, damages, and executed on March 17, 2011. Complainant, however, avers that his office was
costs accruing down to the time of the judgment appealed from, only able to receive the denial the day after the execution or on March 18, 2011.
and unless, during the pendency of the appeal, he deposits with At first blush, one might hastily conclude that the three-day notice rule was
the appellate court the amount of rent due from time to time under apparently not observed. This Court, however, is not prepared to make such a
the contract, if any, as determined by the judgment of the finding. We are mindful of the possibility that a demand to vacate has already
Municipal Trial Court. In the absence of a contract, he shall deposit been given when complainant and Rafols were first served the Order granting
with the Regional Trial Court the reasonable value of the use and the issuance of a writ of execution, before the motion for reconsideration was
occupation of the premises for the preceding month or period at filed. More importantly, complainant failed to allege con-compliance with Sec.
the rate determined by the judgment of the lower court on or 10(c) of Rule 39.
before the tenth day of each succeeding month or period. The x Thus far, no deviation from the Rules has been properly ascribed to respondent.
supersedeas bond shall be transmitted by the Municipal Trial Court, As an officer of the court, he is accorded the presumption of regularity in the
with the other papers, to the clerk of the Regional Trial Court to performance of his duties. The burden was on complainant to adduce evidence
which the action is appealed. that would prove the respondent’s culpability, if any. Without evidence of any
66
RECTO, GAYLE ANGELI M.
2011-0008 | AUSL
Personal Notes on Remedial Law 2 Review (based on the syllabus of Prof. Henedino M. Brondial)

departure from well established rules, any unlawful behaviour, or any gross o Art. 484. There is co-ownership whenever the ownership of an
negligence on his part, the presumption remains applicable and respondent undivided thing or right belongs to different persons.
cannot be held administratively liable for the offense of grave misconduct. o Art. 1078. When there are two or more heirs, the whole estate of
the decedent is, before its partition, owned in common by such
TEODORO S. TEODORO (Deceased), Substituted by his heirs/sons NELSON TEODORO and heirs, subject to the payment of debts of the deceased.
ROLANDO TEODORO, Petitioners, vs. DANILO ESPINO, ROSARIO SANTIAGO, JULIANA x Certainly, and as found by the trial courts, the whole of Lot No. 2476
CASTILLO, PAULINA LITAO, RAQUEL RODRIGUEZ, RUFINA DELA CRUZ, and LEONILA CRUZ, including the portion now litigated is, owing to the fact that it has
Respondents. remained registered in the name of Genaro who is the common
G.R. No. 189248 February 5, 2014 ancestor of both parties herein, co-owned property. All, or both
Teodoro Teodoro and respondents are entitled to exercise the right of
FACTS: possession as co-owners.
x Subject land is located in Bulacan and registered in the name of Genaro, long x Neither party can exclude the other from possession. Although the property
deceased ascendant of all the parties remains unpartitioned, the respondents in fact possess specific areas. Teodoro
x The subject property pertains to the vacant lot where the old ancestral house of Teodoro can likewise point to a specific area, which is that which was possessed
Genaro stood until its demolition in June 2004, at the instance of Teodoro by Petra. Teodoro Teodoro cannot be dispossessed of such area, not only by
Teodoro. virtue of Petra's bequeathal in his favor but also because of his own right of
x Genaro had five children: possession that comes from his co-ownership of the property. As the RTC
o Santiago; concluded, petitioners, as heirs substituting Teodoro
o Maria, from whom respondents descended and trace their claim of x Teodoro in this suit, should be restored in the lawful possession of the disputed
ownership and right of possession; area.
o Petra,
o Mariano, Teodoro Teodoro’s father; LOURDES B. FERRER and PROSPERIDAD M. ARANDEZ, Complainants, vs. JUDGE ROMEO
ƒ Respondents’ respective parents are first cousins of A. RABACA, Metropolitan Trial Court, Branch 25, Manila, Respondent.
Teodoro Teodoro. A.M. No. MTJ-05-1580 October 6, 2010
o Ana. Genaro and his children are all deceased [Formerly OCA IPI No. 04-1608-MTJ]
x Petra died THIRD DIVISION
o her purported will, a holographic will, was probated in Special
Proceedings No. 1615-M before RTC, Branch 8, Malolos, Bulacan, FACTS:
which Decision on the will’s extrinsic validity has become final and x Complainants were the President and the Executive Director of the plaintiff in
executory Civil Case No. 176394-CV of the MeTC, an ejectment suit entitled Young Women’s
o Petra, asserting ownership, devised the subject property to Christian Association, Inc. v. Conrado Cano pending before MTC Manila Br 25 with
Teodoro Teodoro Judge Rabaca as PJ
x Teodoro Teodoro Æ effected the demolition of the ancestral house, intending to x Judge Rabaca Æ in favor of YMCA; ordered Cano to vacate, among others
use the subject property for other purposes x YMCA Æ filed a motion for immediate execution, praying that a writ of execution
x Respondents (who resided at portions of Lot No. 2476 that surround the subject be issued "for the immediate execution of the aforesaid Judgment."
property on which the ancestral house previously stood) Æ erected a fence on o The plaintiff cited Section 19, Rule 70 of the Rules of Court as basis
the surrounding portion, barricaded its frontage, and put up a sign thereat, for its motion
effectively dispossessing Teodoro Teodoro of the property bequeathed to him by x Judge Rabaca Æ denied the motion on the ground that a notice of appeal has
Petra been seasonably filed
x Teodoro Teodoro Æ made demand to vacate x Judge Rabaca allegedly advised the counsel for the plaintiff to file an MR
o BUT this was unheeded o BUT he denied the MR subsequently
x Teodoro Teodoro Æ filed complaint for forcible entry against respondents x The BCC was then ordered to transmit the records to the RTC
x Respondents Æ filed answer x Complainants Æ filed an administrative complaint against Judge Rabaca
o asserted their own ownership and possession of the subject o refusal to perform an act mandated by the Rules of Court had
property given undue advantage to the defendant to the plaintiff’s damage
x MTC Æ dismissed the complaint and prejudice.
o ruling on the issue of ownership and ultimately resolving the issue x Judge Rabaca Æ filed comment
of who between Teodoro Teodoro and respondents had a better o He explained that he had honestly thought that his court had lost
right to possess the subject property jurisdiction over the case pursuant to the provision of Section 9,
x RTC Æ adopted MTC’s factual findings BUT REVERSED its ruling Rule 41 of the Rules of Court (which provides that "in appeals by
x CA Æ reversed RTC notice of appeal, the court loses jurisdiction over the case upon the
o In all, the appellate court found that Teodoro Teodoro (substituted perfection of the appeals filed in due time and the expiration of the
by his heirs Nelson and Rolando Teodoro at that juncture) "failed to time to appeal of the other parties") once he had given due course
discharge the burden of proof that he had prior actual physical to the defendant’s notice of appeal. He claimed that he had issued
possession of the subject [property] before it was barricaded by the orders in good faith and with no malice after a fair and
[respondents] to warrant the institution of the forcible entry suit." impartial evaluation of the facts, applicable rules, and
The appellate court disposed of the case jurisprudence; and that if he had thereby committed lapses in the
issuance of the orders, his doing so should be considered as error
ISSUE: Whether exclusive ownership should be proven before a prayer that herein of judgment on his part.
respondents be made to vacate may be sustained. o He lastly insisted that he did not know personally the parties in Civil
Case No. 176394-CV, and had absolutely no reason to give undue
HELD: NO. favor or advantage to the defendant; that the complainants did not
x We grant the petition. We reverse the decision of the Court of Appeals and submit evidence to show that the orders had been issued for a
restore the decision of the RTC on the appeal reversing the MTC. consideration, material or otherwise, or that his issuance of the
x We affirm the finding of fact by the RTC which is decisive of the issue that has orders had been motivated by ill-will or bad faith.
remained unresolved inspite of a summary procedure and two appellate reviews x Complainants Æ filed reply
of the forcible entry case filed by Teodoro Teodoro. The RTC said: o contended that respondent Judge exhibited his ignorance of the
o Analyzing the facts of the case, the lower [court] concluded that law and procedure in relying on Section 9, Rule 41 of the Rules of
the subject parcel is a part of the estate of the late Genaro Teodoro Court which referred to appeals from the Regional Trial Court; that
and in the absence of an approved partition among the heirs, Rule 40, which contained provisions on appeal from the Municipal
remains a community property over which the legal heirs of Genaro Trial Courts to the Regional Trial Courts, and which provided in its
Teodoro have the right to inherit. All therefore are entitled to Section 4 that the perfection of the appeal and the effect of such
exercise the right of dominion including the right of possession.17 perfection should be governed by the provisions of Section 9 of
(Emphasis supplied). Rule 41, concerned appeals by notice of appeal in general; and that
x The RTC’s comment that it "disagrees with the said ruling" only meant that "the instead, the applicable rule should be Section 19, Rule 70 of the
lower court cannot dispose with finality the issue of ownership" since such Rules of Court.
ownership issue is "inutile in an ejectment suit except to throw light on the o The complainants pointed out that respondent Judge apparently
question of possession."18 And so the RTC ruled that Teodoro Teodoro should did not know that appeal in forcible entry and detainer cases was
be restored in the lawful possession of the disputed area of Lot No. 2476 in light not perfected by the mere filing of a notice of appeal (as in
of the finding of the MTC that the subject lot still forms part of the estate of the ordinary actions) but by filing of a notice of appeal and a sufficient
late Genaro Teodoro. It is from this same fact that the MTC reached the contrary supersedeas bond approved by the trial judge executed to the
conclusion that Teodoro Teodoro’s complaint should be dismissed because he plaintiff to pay the rents, damages and costs accruing down to the
has "failed to prove his ownership."19 time of the judgment appealed from. They asserted that
x In the sense that Teodoro Teodoro has not proven exclusive ownership, the MTC respondent Judge’s invocation of good faith and error of judgment
was right.1âwphi1 But exclusive ownership of Lot No. 2476 or a portion did not absolve him of liability, because he had grossly neglected
thereof is not in this case required of Teodoro Teodoro for him to be his duties mandated by law by failing and refusing to act on their
entitled to possession. Co-ownership, the finding of both the MTC at motion for immediate execution and motion for reconsideration and
first instance and by the RTC on appeal, is sufficient. The pertinent by giving due course to the appeal despite no supersedeas bond
provisions of the Civil Code state: having been filed and approved by the trial court.

67
RECTO, GAYLE ANGELI M.
2011-0008 | AUSL
Personal Notes on Remedial Law 2 Review (based on the syllabus of Prof. Henedino M. Brondial)

x OCA Presbi Velasco Æ recommended that the administrative complaint against x Indeed, respondent Judge should have granted the plaintiff’s motion for
respondent Judge be re-docketed as a regular administrative matter; and that immediate execution considering that the defendant did not file the
respondent Judge be fined in the amount of P5,000.00 with warning that a sufficient supersedeas bond despite having appealed. Granting the
repetition of the same or similar act would be dealt with more severely, based on plaintiff’s motion for immediate execution became his ministerial duty
an evaluation of the charges upon the defendant’s failure to file the sufficient supersedeas bond.
Section 19, Rule 70, of the Rules of Court clearly imposes such duty, viz:
ISSUE: Whether Judge Rabaca was correct in refusing to execute his decision in favor of YMCA o Section 19. Immediate execution of judgment; how to stay same.
on the ground that the defendants in the said case were able to seasonably file an appeal. — If judgment is rendered against the defendant, execution shall
issue immediately upon motion, unless an appeal has been
HELD: NO. perfected and the defendant to stay execution files a sufficient
x EVALUATION: We agree with the complainants that respondent erred when he supersedeas bond, approved by the Municipal Trial Court and
did not act on complainants’ motion for immediate execution. executed in favor of the plaintiff to pay the rents, damages, and
x Section 19, Rule 70 of the 1997 Revised Rules on Civil Procedure provides: costs accruing down to the time of the judgment appealed from,
o "SEC. 19. If judgment is rendered against the defendant, execution and unless, during the pendency of the appeal, he deposits with
shall issue immediately upon motion, unless an appeal has been the appellate court the amount of rent due from time to time under
perfected and the defendant to stay execution files a supersedeas the contract, if any, as determined by the judgment of the
bond, approved by the Municipal Trial Court and executed in favor Municipal Trial Court. In the absence of a contract, he shall deposit
of the plaintiff to pay the rents, damages, and costs accruing down with the Regional Trial Court the reasonable value of the use and
to the time of the judgment appealed from, and unless, during the occupation of the premises for the preceding month or period at
pendency of the appeal, he deposits with the appellate court the the rate determined by the judgment of the lower court on or
amount of rent due from time to time under the contract, if any, as before the tenth day of each succeeding month or period. The
determined by the judgment of the Municipal Trial Court. XXXX supersedeas bond shall be transmitted by the Municipal Trial Court,
XXXX XXXX." with the other papers, to the clerk of the Regional Trial Court to
x It is clear from the foregoing that the perfection of an appeal by itself which the action is appealed.
is not sufficient to stay the execution of the judgment in an ejectment x Respondent Judge’s excuse, that he had lost jurisdiction over the case by virtue
case. The losing party should likewise file a supersedeas bond executed of the defendant’s appeal, was unacceptable in light of the clear and explicit text
in favor of the plaintiff to answer for rents, damages and costs, and, if of the aforequoted rule. To begin with, the perfection of the appeal by the
the judgment of the court requires it, he should likewise deposit the defendant did not forbid the favorable action on the plaintiff’s motion
amount of the rent before the appellate court from the time during the for immediate execution. The execution of the decision could not be stayed
pendency of the appeal. Otherwise, execution becomes ministerial and by the mere taking of the appeal. Only the filing of the sufficient supersedeas
imperative. (Philippine Holding Corporation vs. Valenzuela, 104 SCRA 401 as bond and the deposit with the appellate court of the amount of rent due from
cited in Hualam Construction and Development Corporation vs. Court of Appeals, time to time, coupled with the perfection of the appeal, could stay the execution.
214 SCRA 612, 626). Secondly, he could not also credibly justify his omission to act
x In the case at bar, defendant seasonably filed his Notice of Appeal dated 9 July according to the provision by claiming good faith or honest belief, or by
2004 on 13 July 2004; he however failed to file any supersedeas bond. asserting lack of malice or bad faith.1avvphil A rule as clear and explicit as
Prior to the filing of such notice of appeal, more specifically on 12 July Section 19 could not be misread or misapplied, but should be implemented
2004, complainants have already filed their Motion for Execution dated without evasion or hesitation. To us, good faith, or honest belief, or lack of
8 July 2004. Instead of acting on the Motion for Execution, respondent malice, or lack of bad faith justifies a non-compliance only when there is an as-
Judge Rabaca gave due course to the appeal in an Order dated 14 July yet unsettled doubt on the meaning or applicability of a rule or legal provision. It
2004 and directed his Branch Clerk of Court to elevate the records of was not so herein. And, thirdly, given that his court, being vested with
the case to the Regional Trial Court (RTC). The Branch Clerk of Court original exclusive jurisdiction over cases similar to Civil Case No.
however failed to forward the records to the RTC. This fact is clear from 176394-CV, had been assigned many such cases, he was not a trial
Judge Rabaca’s Order dated 28 July 2004 wherein he directed the Branch Clerk judge bereft of the pertinent prior experience to act on the issue of
of Court to forward the records of the case to the Manila Regional Trial Court immediate execution, a fact that further exposed the abject inanity of
immediately. his excuses.
x From the foregoing, it is clear that when the complainant moved for x We agree with the complainants’ insistence, therefore, that respondent Judge’s
the immediate execution of Judge Rabaca’s decision, the latter still had omission to apply Section 19 was inexcusable. He had ignored the urging to
jurisdiction over the case. He therefore clearly erred when he refused to follow the clear and explicit provision of the rule made in the plaintiff’s motion for
act on the Motion for Execution. The relevant question that we immediate execution. Had he any genuine doubt about his authority to grant the
should resolve however is whether such error is an error of judgment motion for immediate execution, as he would have us believe, he could have
or an error amounting to incompetence that calls for administrative easily and correctly resolved the doubt by a resort to the Rules of Court, which
discipline. he well knew was the repository of the guidelines he was seeking for his judicial
x Judge Rabaca claims that he refused to act on the complainant’s Motion for action. Neither was it relevant that he did not know any of the parties, or that he
execution because he honestly thought that when he gave due course to the did not corruptly favor the defendant by his omission. His mere failure to perform
defendant’s appeal which was seasonably filed, and ordered the elevation of the a duty enjoined by the Rules of Court sufficed to render him administratively
records to the appellate court, his court already lost jurisdiction over the case.. In accountable.
making his ruling, respondent asserts he relied on the provisions of Section 9, x This case is an opportune occasion to remind judges of the first level courts to
Rule 41 of the Rules of Court. This provision reads as follows: adhere always to the mandate under Section 19, Rule 70, of the Rules of Court
x In appeals by notice of appeal, the court loses jurisdiction over the to issue writs of execution upon motion of the plaintiffs in actions for forcible
case upon the perfection of the appeals filed in due time and the entry or unlawful detainer when the defendant has appealed but has not filed a
expiration of the time to appeal of the other parties. sufficient supersedeas bond. The summary nature of the special civil action
x He likewise allegedly relied on the ruling of the Court in Administrative Matter under Rule 70 and the purpose underlying the mandate for an immediate
OCA IPI No. 03-1513-MTJ: Susana Joaquin Vda. De Agregado vs. Judge Thelma execution, which is to prevent the plaintiffs from being further deprived of their
Bunyi-Medina, MeTJ wherein the Court said that- rightful possession, should always be borne in mind.
o Respondent Judge is correct in saying that she had lost jurisdiction x The recommended penalty of P5,000.00 with warning that a repetition of the
to entertain the motion for execution after the perfection of the same or similar act would be dealt with more severely is also correct. The Court
appeal and after she issued an order to transmit the records of the Administrator rationalized the recommendation of the penalty thuswise:
case to the appellate court for review. o Under A.M. No. 01-8-10-SC, ‘Gross Ignorance of the Law or
x The facts of the case against Judge Bunyi-Medina are however different from Procedure’ is classified as serious offense for which the imposable
those prevailing in the instant case. In the Medina case, the fifteen (15) day penalty ranges from a fine to dismissal. However, we find
period within which to perfect the appeal had already lapsed before the respondent’s acts not ingrained with malice or bad faith. It is a
complainant therein moved for the execution of the execution judgment. Clearly matter of public policy that in the absence of fraud, dishonesty or
therefore, appeal had already been perfected. In the instant case, although the corrupt motive, the acts of a judge in his judicial capacity are not
defendant had filed his appeal, the period to appeal had not yet lapsed since the subject to disciplinary action even though such acts are erroneous.
plaintiff still had his own period to appeal from the judgment and such period In Domingo vs. Judge Pagayatan, A.M. No. RTJ-03-1751, 10 June
had not yet lapsed. The provision relied upon by judge Rabaca, more specifically, 2003, the penalty of fine in the amount of five thousand pesos was
Section 9, Rule 41 of the Rules of Court, clearly states that, "In appeals by notice deemed sufficient where it was held that respondent’s lack of
of appeal, the court loses jurisdiction over the case upon perfection of the malice or bad faith frees him from administrative liability but not for
appeals filed on due time and the expiration of the time to appeal of the other gross ignorance of the law.
parties." Moreover and more importantly, the herein complainants filed their x We concur with the rationalization of the Court Administrator. Verily, even if
Motion for Execution even before the defendant had filed his Notice of Appeal. respondent Judge’s omission would have easily amounted to gross ignorance of
Such motion was therefore still well within the jurisdiction of the lower court. the law and procedure, a serious offense under Section 8,8 Rule 140, of the
x It is basic rule in ejectment cases that the execution of judgment in Rules of Court, as amended, the fact that the complainants did not establish that
favor of the plaintiff is a matter of right and mandatory. This has been malice or bad faith impelled his omission to act, or that fraud, dishonesty, or a
the consistent ruling of the Court in a number of cases involving the same issue corrupt motive attended his omission to act demands a downgrading of the
posed before the respondent judge. Respondent Judge is expected to know this liability. In the absence of any showing that he had been held guilty of any other
and his justification of erroneous application of the law, although mitigating, administrative offense,9 and without our attention being called to other
could not exculpate him from liability. circumstances that might demonstrate respondent Judge’s dark motives for his
x We agree with and adopt the evaluation of the Court Administrator. inaction, we should find and consider the recommended penalty of P5,000.00

68
RECTO, GAYLE ANGELI M.
2011-0008 | AUSL
Personal Notes on Remedial Law 2 Review (based on the syllabus of Prof. Henedino M. Brondial)

with warning that a repetition of the same or similar act would be dealt with concerned, but also that the claim for damages-moral and
more severely to be commensurate to the offense exemplary in addition to actual and compensatory-constitutes
splitting a single cause of action. Since this runs counter to the rule
against multiplicity of suits, the dismissal of the second action
becomes imperative.
CGR CORPORATION herein represented by its President ALBERTO RAMOS, III, HERMAN M. x The complaint for forcible entry contains the following pertinent allegations -
BENEDICTO and ALBERTO R. BENEDICTO, Petitioners, vs. ERNESTO L. TREYES, JR., o 2.01 On 02 January 1989, plaintiff entered into a contract of lease
Respondent with defendant PDC over a property designated as Ground Floor,
G.R. No. 170916 April 27, 2007 Seafood Market (hereinafter "Subject Premises") situated at the
SECOND DIVISION corner of EDSA corner MacArthur Street, Araneta Center, Cubao,
Quezon City, for a period of ten (10) years from 02 January 1989
FACTS: to 30 April 1998.
x CGR Corporation, Herman M. Benedicto and Alberto R. Benedicto (petitioners) o 2.02 Immediately after having acquired actual physical possession
claimed to have occupied 37.3033 hectares of public land in Barangay Bulanon, of the Subject Premises, plaintiff established and now operates
Sagay City, Negros Occidental thereon the now famous Seafood Market Restaurant. Since then,
x Ernesto L. Treyes, Jr. (respondent) allegedly forcibly and unlawfully entered the plaintiff had been in actual, continuous, and peaceful physical
leased properties and once inside barricaded the entrance to the fishponds, set possession of the Subject Premises until 31 October 1992.
up a barbed wire fence along the road going to petitioners’ fishponds, and o 3.02 Plaintiff, being the lessee of the Subject Premises, is entitled
harvested several tons of milkfish, fry and fingerlings owned by the CGR to the peaceful occupation and enjoyment of the Subject Premises
x CGR Æ promptly filed with the Municipal Trial Court (MTC) in Sagay City to the exclusion of all others, including defendants herein.
separate complaints for Forcible Entry With Temporary Restraining Order And/Or o 3.03 Defendants’ resort to strong arms tactics to forcibly wrest
Preliminary Injunction And Damages against Treyes possession of the Subject Premises from plaintiff and maintain
x CGR Æ then filed with the Bacolod RTC a complaint for damages against Treyes possession thereof through the use of force, threat, strategy and
o Treyes Æ filed an MD on three grounds - litis pendentia, res intimidation by the use of superior number of men and arms
judicata and forum shopping. amounts to the taking of the law into their own hands.
o RTC Æ granted MD; dismissed action o 3.04 Thus, defendants’ act of unlawfully evicting out plaintiff from
x CGR Æ filed a Rule 45 before the SC the Subject Premises it is leasing from defendant PDC and depriving
it of possession thereof through the use of force, threat, strategy and
ISSUE: Whether a complainant in a forcible entry case can file an independent action for intimidation should be condemned and declared illegal for being
damages arising after the act of dispossession had occurred. contrary to public order and policy.
o 3.05 Consequently, defendants should be enjoined from continuing
HELD: YES. with their illegal acts and be ordered to vacate the Subject
x Section 17, Rule 70 of the Rules of Court provides: Premises and restore possession thereof, together with its contents
o SEC. 17. Judgment. - If after trial the court finds that the to plaintiff.
allegations of the complaint are true, it shall render judgment in o 4.07 Considering that defendants’ act of forcibly grabbing
favor of the plaintiff for the restitution of the premises, the sum possession of the Subject Premises from plaintiff is illegal and null
justly due as arrears of rent or as reasonable compensation for the and void, defendant should be adjudged liable to plaintiff for all the
use and occupation of the premises, attorney’s fees and costs. If it aforedescribed damages which plaintiff incurred as a result thereof.
finds that said allegations are not true, it shall render judgment for x The amended complaint for damages filed by private respondent alleges basically
the defendant to recover his costs. If a counterclaim is established, the same factual circumstances and issues as bases for the relief prayed for, to
the court shall render judgment for the sum found in arrears from wit:
either party and award costs as justice requires. (Emphasis o 4. On May 28, 1991, plaintiff and defendant PDC entered into a
supplied) Contract of Lease for a period of ten years or from January 2, 1989
x The recoverable damages in forcible entry and detainer cases thus up to April 30, 1998 over a property designated as Ground Floor,
refer to "rents" or "the reasonable compensation for the use and Seafood Market (hereinafter referred to as Subject Premises)
occupation of the premises" or "fair rental value of the property" and situated at the corner of EDSA corner McArthur Street, Araneta
attorney’s fees and costs.13 Center, Cubao, Quezon City. A copy of the lease contract is
x The 2006 case of Dumo v. Espinas14 reiterates the long-established rule that the attached hereto as Annex "A."
only form of damages that may be recovered in an action for forcible entry is the o 5. Immediately thereafter, plaintiff took over actual physical
fair rental value or the reasonable compensation for the use and possession of Subject Premises, and established thereon the now
occupation of the property: famous "Seafood Market Restaurant."
o Lastly, we agree with the CA and the RTC that there is no basis for o 7. On October 31, 1992 at around 8:30 p.m., defendant PDC,
the MTC to award actual, moral, and exemplary damages in view of without the benefit of any writ of possession or any lawful court
the settled rule that in ejectment cases, the only damage that can be order and with the aid of approximately forty (40) armed security
recovered is the fair rental value or the reasonable guards and policemen under the supervision of defendant Tejam,
compensation for the use and occupation of the property. forcibly entered the subject premises through force, intimidation,
Considering that the only issue raised in ejectment is that of threats and stealth and relying on brute force and in a
rightful possession, damages which could be recovered are those thunderboltish manner and against plaintiff’s will, unceremoniously
which the plaintiff could have sustained as a mere possessor, or drew away all of plaintiffs men out of the subject premises, thereby
those caused by the loss of the use and occupation of the property, depriving herein plaintiff of its actual, physical and natural
and not the damages which he may have suffered but which have no possession of the subject premises. The illegal high-handed manner
direct relation to his loss of material possession. x x x15 of gestapo like take-over by defendants of subject premises is more
(Emphasis, underscoring and italics supplied; citations omitted) particularly described as follows: x x x x
x Other damages must thus be claimed in an ordinary action.16 o 8. To date, defendants continue to illegally possess and hold the
x In asserting the negative of the issue, respondent cites the 1999 case of Subject Premises, including all the multi-million improvements,
Progressive Development Corporation, Inc. v. Court of Appeals.17 In this case, fixtures and equipment therein owned by plaintiff, all to the
Progressive Development Corporation, Inc. (Progressive), as lessor, repossessed damage and prejudice of plaintiff. The actuations of defendants
the leased premises from the lessee allegedly pursuant to their contract of lease constitute an unlawful appropriation, seizure and taking of property
whereby it was authorized to do so if the lessee failed to pay monthly rentals. against the will and consent of plaintiff. Worse, defendants are
The lessee filed a case for forcible entry with damages against Progressive threatening to sell at public auction and without the consent, of
before the Metropolitan Trial Court (MeTC) of Quezon City. During the pendency plaintiff and without lawful authority, the multi-million fixtures and
of the case, the lessee filed an action for damages before the RTC, drawing equipment of plaintiff and at prices way below the market value
Progressive to file a motion to dismiss based on litis pendentia. The RTC denied thereof. Plaintiff hereby attaches as Annex "B" the letter from
the motion. defendants dated August 6, 1993 addressed to plaintiff, informing
x On appeal by Progressive, the Court of Appeals sustained the RTC order denying the latter that the former intends to sell at an auction on August
the motion to dismiss. 19, 1993 at 2:00 p.m. properties of the plaintiff presently in
x Progressive brought the case to this Court. Citing Section 1, Rule 70 of the Rules defendants’ possession.
of Court, this Court reversed the lower courts’ ruling, it holding that "all cases for o 12. Defendant’s unlawful takeover of the premises constitutes a
forcible entry or unlawful detainer shall be filed before the Municipal Trial Court violation of its obligation under Art. 1654 of the New Civil Code
which shall include not only the plea for restoration of possession but also all requiring the lessor to maintain the lessee in peaceful and
claims for damages and costs therefrom." In other words, this Court held that adequate enjoyment of the lease for the entire duration of the
"no claim for damages arising out of forcible entry or unlawful detainer may be contract. Hence, plaintiff has filed the present suit for the recovery
filed separately and independently of the claim for restoration of possession."18 of damages under Art. 1659 of the New Civil Code x x x x19
(Underscoring supplied) (Emphasis in the original; underscoring supplied)
x In thus ruling, this Court in Progressive made a comparative study of the therein x Analyzing the two complaints, this Court, still in Progressive, observed:
two complaints, thus: o Restated in its bare essentials, the forcible entry case has one
o A comparative study of the two (2) complaints filed by private cause of action, namely, the alleged unlawful entry by petitioner
respondent against petitioner before the two (2) trial courts shows into the leased premises out of which three (3) reliefs
that not only are the elements of res adjudicata present, at least (denominated by private respondent as its causes of action) arose:
insofar as the claim for actual and compensatory damages is (a) the restoration by the lessor (petitioner herein) of the
69
RECTO, GAYLE ANGELI M.
2011-0008 | AUSL
Personal Notes on Remedial Law 2 Review (based on the syllabus of Prof. Henedino M. Brondial)

possession of the leased premises to the lessee, (b) the claim for thereon within such period as may be fixed by the court and to be heard by himself or
actual damages due to the losses suffered by private respondent counsel, a person guilty of any of the following acts may be punished for indirect contempt;
such as the deterioration of perishable foodstuffs stored inside the
premises and the deprivation of the use of the premises causing (a) Misbehavior of an officer of a court in the performance of his official duties or in
loss of expected profits; and, (c) the claim for attorney’s fees and his official transactions;
costs of suit.
x On the other hand, the complaint for damages prays for a monetary (b) Disobedience of or resistance to a lawful writ, process, order, or judgment of a
award consisting of (a) moral damages of P500,000.00 and exemplary court, including the act of a person who, after being dispossessed or ejected from any real
damages of another P500,000.00; (b) actual damages of P20,000.00 property by the judgment or process of any court of competent jurisdiction, enters or attempts
and compensatory damages of P1,000,000.00 representing unrealized or induces another to enter into or upon such real property, for the purpose of executing acts
profits; and, (c) P200,000.00 for attorney’s fees and costs, all based on of ownership or possession, or in any manner disturbs the possession given to the person
the alleged forcible takeover of the leased premises by petitioner. adjudged to be entitled thereto;
Since actual and compensatory damages were already prayed for in the
forcible entry case before the MeTC, it is obvious that this cannot be (c) Any abuse of or any unlawful interference with the processes or proceedings of a
relitigated in the damage suit before the RTC by reason of res court not constituting direct contempt under section 1 of this Rule;
adjudicata.
x The other claims for moral and exemplary damages cannot also succeed (d) Any improper conduct tending, directly or indirectly, to impede, obstruct, or
considering that these sprung from the main incident being heard before the degrade the administration of justice;
MeTC. x x x20 (Italics in the original; Emphasis and underscoring supplied)
x It bears noting, however, that as reflected in the earlier-quoted allegations in the (e) Assuming to be an attorney or an officer of a court, and acting as such without
complaint for damages of herein petitioners, their claim for damages have no authority;
direct relation to their loss of possession of the premises. It had to do with
respondent’s alleged harvesting and carting away several tons of milkfish and (f) Failure to obey a subpoena duly served;
other marine products in their fishponds, ransacking and destroying of a chapel
built by petitioner CGR Corporation, and stealing religious icons and even (g) The rescue, or attempted rescue, of a person or property in the custody of an
decapitating the heads of some of them, after the act of dispossession had officer by virtue of an order or process of a court held by him.
occurred.
x Surely, one of the elements of litis pendentia - that the identity between the But nothing in this section shall be so construed as to prevent the court from issuing process
pending actions, with respect to the parties, rights asserted and reliefs prayed to bring the respondent into court, or from holding him in custody pending such proceedings.
for, is such that any judgment rendered on one action will, regardless of which is
successful, amount to res judicata in the action under consideration - is not 2. Procedure
present, hence, it may not be invoked to dismiss petitioners’ complaint for
damages.21 Section 4. How proceedings commenced. — Proceedings for indirect contempt may be
x Res judicata may not apply because the court in a forcible entry case has no initiated motu propio by the court against which the contempt was committed by an order or
jurisdiction over claims for damages other than the use and occupation of the any other formal charge requiring the respondent to show cause why he should not be
premises and attorney’s fees.22 punished for contempt.
x Neither may forum-shopping justify a dismissal of the complaint for damages,
the elements of litis pendentia not being present, or where a final judgment in In all other cases, charges for indirect contempt shall be commenced by a verified petition
the forcible entry case will not amount to res judicata in the former.23 with supporting particulars and certified true copies of documents or papers involved therein,
x Petitioners’ filing of an independent action for damages other than those and upon full compliance with the requirements for filing initiatory pleadings for civil actions in
sustained as a result of their dispossession or those caused by the loss of their the court concerned. If the contempt charges arose out of or are related to a principal action
use and occupation of their properties could not thus be considered as splitting pending in the court, the petition for contempt shall allege that fact but said petition shall be
of a cause of action. docketed, heard and decided separately, unless the court in its discretion orders the
consolidation of the contempt charge and the principal action for joint hearing and decision.
(n)

Section 5. Where charge to be filed. — Where the charge for indirect contempt has been
committed against a Regional Trial Court or a court of equivalent or higher rank, or against an
officer appointed by it, the charge may be filed with such court. Where such contempt has
been committed against a lower court, the charge may be filed with the Regional Trial Court of
the place in which the lower court is sitting; but the proceedings may also be instituted in such
lower court subject to appeal to the Regional Trial Court of such place in the same manner as
provided in section 11 of this Rule. (4a; Bar Matter No. 803, 21 July 1998)

Section 6. Hearing; release on bail. — If the hearing is not ordered to be had forthwith,
the respondent may be released from custody upon filing a bond, in an amount fixed by the
court, for his appearance at the hearing of the charge. On the day set therefor, the court shall
proceed to investigate the charge and consider such comment, testimony or defense as the
respondent may make or offer. (5a)

Section 7. Punishment for indirect contempt. — If the respondent is adjudged guilty of


indirect contempt committed against a Regional Trial Court or a court of equivalent or higher
rank, he may be punished by a fine not exceeding thirty thousand pesos or imprisonment not
exceeding six (6) months, or both. If he is adjudged guilty of contempt committed against a
lower court, he may be punished by a fine not exceeding five thousand pesos or imprisonment
not exceeding one (1) month, or both. If the contempt consists in the violation of a writ of
injunction, temporary restraining order or status quo order, he may also be ordered to make
complete restitution to the party injured by such violation of the property involved or such
CONTEMPT [Rule 71] amount as may be alleged and proved.

1. Kinds The writ of execution, as in ordinary civil actions, shall issue for the enforcement of a
judgment imposing a fine unless the court otherwise provides. (6a)
Direct Section 8. Imprisonment until order obeyed. — When the contempt consists in the
refusal or omission to do an act which is yet in the power of the respondent to perform, he
Section 1. Direct contempt punished summarily. — A person guilty of misbehavior in may be imprisoned by order of the court concerned until he performs it. (7a)
the presence of or so near a court as to obstruct or interrupt the proceedings before the same,
including disrespect toward the court, offensive personalities toward others, or refusal to be Section 9. Proceeding when party released on bail fails to answer. — When a
sworn or to answer as a witness, or to subscribe an affidavit or deposition when lawfully respondent released on bail fails to appear on the day fixed for the hearing, the court may
required to do so, may be summarily adjudged in contempt by such court and punished by a issue another order of arrest or may order the bond for his appearance to be forfeited and
fine not exceeding two thousand pesos or imprisonment not exceeding ten (10) days, or both, confiscated, or both; and, if the bond be proceeded against, the measure of damages shall be
if it be a Regional Trial Court or a court of equivalent or higher rank, or by a fine not the extent of the loss or injury sustained by the aggrieved party by reason of the misconduct
exceeding two hundred pesos or imprisonment not exceeding one (1) day, or both, if it be a for which the contempt charge was prosecuted, with the costs of the proceedings, and such
lower court. recovery shall be for the benefit of the party injured. If there is no aggrieved party, the bond
shall be liable and disposed of as in criminal cases. (8a)
Indirect
3. Judgment and Review
Section 3. Indirect contempt to be punished after charge and hearing. — After a
charge in writing has been filed, and an opportunity given to the respondent to comment

70
RECTO, GAYLE ANGELI M.
2011-0008 | AUSL
Personal Notes on Remedial Law 2 Review (based on the syllabus of Prof. Henedino M. Brondial)

Section 11. Review of judgment or final order; bond for stay. — The judgment or final The Court of Appeals enjoined that order. Consequently, respondents' act in
order of a court in a case of indirect contempt may be appealed to the proper court as in proceeding with the scheduled stock-holders' meeting was not contumacious as
criminal cases. But execution of the judgment or final order shall not be suspended until a bond there was no willful disobedience to an order of the SEC. 22 The disobedience
is filed by the person adjudged in contempt, in an amount fixed by the court from which the which the law punishes as constructive contempt implies willfulness. For, at
appeal is taken, conditioned that if the appeal be decided against him he will abide by and bottom, contempt is a willful disregard or disobedience. 23
perform the judgment or final order. (10a) x The SEC was rather hasty in asserting its power to punish for contempt. The
chairman and commissioners of the SEC must exercise the power of contempt
SECURITIES AND EXCHANGE COMMISSION CHAIRMAN PERFECTO R. YASAY, JR., judiciously and sparingly with utmost self-restraint. 24
ASSOCIATE COMMISSIONERS FE ELOISA C. GLORIA, EDIJER MARTINEZ and ROSALINDA U. x Finally, the penalty imposed exceeded those authorized in the powers of the SEC
CASIGURAN, petitioners, vs. MANUEL D. RECTO, PELAGIO T. RICALDE and CESAR P. 25 in relation to the 1964 Revised Rules of Court as amended. 26 If the
MANALAYSAY, respondents. contempt was committed against a superior court or judge, the accused may be
G.R. No. 129521 September 7, 1999 fined not exceeding thirty thousand pesos (P30,000.00) or imprisoned not more
FIRST DIVISION than six (6) months, or both. The SEC suspended respondent Manalaysay from
the practice of law in the SEC, a power vested exclusively in the Supreme Court
FACTS:
x SEC Chairman Yasay upon request of certain stockholders of Interport Resources SALVADOR SISON, complainant, vs. JUDGE JOSE F. CAOIBES, JR., Presiding Judge, and
Corporation, directed respondent Ricalde to submit to the SEC a list of TEODORO S. ALVAREZ, Sheriff IV, Regional Trial Court, Las Pias City, Branch 253,
stockholders and to set a definite time and place for the validation of proxies and respondents.
nominations for directors of the firm A.M. No. RTJ-03-1771. May 27, 2004
x SEC Æ issued a TRO enjoining the Interport Resources Corporation from holding EN BANC
the July 9, 1996 scheduled annual meeting of the stockholders.
x Iterport nonetheless proceeded with the meeting FACTS:
o Presided by Manalaysay x Judge Æ issued an Order requiring the complainant to appear before him to
x SEC Æ declared the meeting null and void explain a traffic incident involving his son and the Salvador Sison, a Metropolitan
o directed respondents to appear before the SEC on July 15, 1996, at Manila Development Authority (MMDA) traffic enforcer
3:00 p.m., to show cause why no disciplinary action should be x Sison failed to appear
taken against them or why they should not be cited in contempt. x Judge Æ issued another Order for Sison’s arrest and commitment
x Manalyasay Æ questioned the validity of the TRO as well as the CONTEMPT x Sheriff served the same
proceedings x Sison Æ appeared and executed an affidavit[6] admitting to the court that he
x SEC Æ declared Atty. Cesar Manalaysay, Manuel D. Recto and Atty. Pelagio T. made a mistake and that it was all a misunderstanding
Ricalde guilty of contempt x Judge Æ lifted the 2nd order
x Respondents Æ appealed x Sison Æ filed a complaint against Judge
x CA Æ reversed SEC o complainant was greatly surprised when respondent TEODORO
ALVAREZ came and arrested him without any warrant of arrest,
ISSUE: Whether the distinction between civil and criminal contempt is material as to the only on orders of the respondent Judge, and he was ordered to
determination whether appeal lies as a remedy. board a motor vehicle and was brought to the respondent Judge in
Las Pias City who ordered him detained in the Las Pias City Jail.
HELD: NO. o respondent Teodoro Alvarez informed him that there will be a
x We agree with respondents that the charge of contempt partakes of the nature hearing of his indirect contempt charge before the sala of the
of a criminal offense. 9 The exoneration of the contemner from the charge respondent Judge in Las Pias City.
amounts to an acquittal from which an appeal would not lie. x Judge Æ filed comment
x A distinction is made between a civil and criminal contempt. Civil contempt is the o vehemently denied the accusations against him, contending that he
failure to do something ordered by a court to be done for the benefit of a party. was merely preserving the dignity and honor due to the courts of
A criminal contempt is any conduct directed against the authority or dignity of law. The respondent narrated that on September 8, 1999, he
the court. 10 ordered his son, Jose R. Caoibes III, to go to the Pasig City
x Petitioners argue that the contempt committed by respondents was civil in Regional Trial Court to secure certain records. While on his way
nature, as the temporary restraining order the SEC issued was for the benefit of there, he was flagged down by the complainant for an alleged
a party to a case. The contention is untenable. traffic violation. Caoibes III explained to the complainant that he was
x "Civil contempt proceedings are generally held to be remedial and civil on an errand for his father, the respondent judge, to which the
in their nature; that is, they are proceedings for the enforcement of complainant reportedly uttered, Walang Judge, Judge Caoibes sa
some duty, and essentially a remedy for coercing a person to do the akin; kahapon nga, abogado ang hinuli ko.
thing required." 11 "In general, civil contempt proceedings should be x OCA Æ recommended:
instituted by an aggrieved party, or his successor, or someone who has a o Dismissal of charge against Sheriff Alvarez
pecuniary interest in the right to be protected." 12 If the contempt is o Referral of charge against Judge to the CA
initiated by the court or tribunal exercising the power to punish a given ƒ although the complainant never appeared to prove
contempt, it is criminal in nature, and the proceedings are to be the charges against the respondent judge, the facts
conducted in accordance with the principles and rules applicable to averred in the complaint appear to be substantially
criminal cases. The State is the real prosecutor. 13 correct and true. Thus, the respondent judge abused
x "The real character of the proceedings in contempt cases is to be determined by his authority to charge and punish any person for
the relief sought or by the dominant purpose. The proceedings are to be indirect contempt under Rule 71 of the Rules of Civil
regarded as criminal when the purpose is primarily punishment, and civil Procedure
when the purpose is primarily compensatory or remedial." 14
x "But whether the first or the second, contempt is still a criminal ISSUE: Whether Judge Caoibes was correct in citing Sison in indirect contempt, considering
proceeding in which acquittal, for instance, is a bar to a second that the latter never became a party to a case pending before Judge Caoibes’ sala.
prosecution. The distinction is for the purpose only of determining the
character of punishment to be administered." 15 HELD: NO.
x In this case, the contempt is not civil in nature, but criminal, imposed to x Thus, the power to declare a person in contempt of court and in dealing with him
vindicate the dignity and power of the Commission; hence, as in criminal accordingly is an inherent power lodged in courts of justice, to be used as a
proceedings, an appeal would not lie from the order of dismissal of, or an means to protect and preserve the dignity of the court, the solemnity of the
exoneration from, a charge of contempt." 16 proceedings therein, and the administration of justice from callous misbehavior,
x At any rate, the SEC order directing respondents to show cause why they should offensive personalities, and contumacious refusal to comply with court
not be cited in contempt was highly improper. The Court of Appeals issued on orders.[15] Indeed, the power of contempt is power assumed by a court
July 8, 1996, a temporary restraining order against the order of the SEC of June or judge to coerce cooperation and punish disobedience, disrespect or
28, 1996 directing the Interport Resources Corporation to desist from holding the interference with the courts orderly process by exacting summary
stockholders' meeting on July 9, 1996. Contrary to the view of petitioners, the punishment. The contempt power was given to the courts in trust for
effect of the temporary restraining order of the Court of Appeals directing the the public, by tradition and necessity, in as much as respect for the
SEC to desist from enforcing its own TRO was to allow such meeting to proceed courts, which are ordained to administer the laws which are necessary
as scheduled. More, the Court of Appeals in its final decision nullified the SEC's to the good order of society, is as necessary as respect for the laws
order. 17 Hence, there was no willful disobedience to a lawful order of the SEC. themselves.[16] And, as in all other powers of the court, the contempt
Respondents were not guilty of contempt. power, however plenary it may seem, must be exercised judiciously and
x While the SEC is vested with the power to punish for contempt, 18 the salutary sparingly.[17] A judge should never allow himself to be moved by pride,
rule is that the power to punish for contempt must be exercised on the prejudice, passion, or pettiness in the performance of his
preservative, not vindictive principle, 19 and on the corrective and not duties.[18]
retaliatory idea of punishment. 20 The courts and other tribunals vested with the x At first blush, it would seem that the respondent judge was justified in holding
power of contempt must exercise the power to punish for contempt for purposes the complainant for contempt, due to the latters refusal to comply with the
that are impersonal, because that power is intended as a safeguard not for the judges Order of September 15, 1999. However, it is not lost upon this Court that
judges as persons but for the functions that they exercise. 21 the complainant was not a party to any of the cases pending before the
x In this case, the SEC issued the citation for contempt sua sponte. There was no RTC, Branch 253. What triggered the contempt charge was, in fact, the traffic
charge filed by a private party aggrieved by the acts of respondents. Strictly violation incident involving the respondent judges son. Furthermore, the record
speaking, there was no disobedience to the SEC's temporary restraining order. shows that when the complainant filed his reply to the charge as
71
RECTO, GAYLE ANGELI M.
2011-0008 | AUSL
Personal Notes on Remedial Law 2 Review (based on the syllabus of Prof. Henedino M. Brondial)

required by the respondent judge, the same was refused by some staff the service without pay for a period of three months;[29] and even the ultimate
member in the latters sala.[19] penalty of dismissal from the service.[30]
x In Cortes v. Bangalan,[20] we held that a judge may not hold a party in x Furthermore, we take judicial notice that the respondent judge was previously
contempt of court for expressing concern on the judges impartiality through a sanctioned by the Court for violating Canon 2 of the Code of Judicial Conduct,
motion for voluntary inhibition, even if the latter may have felt insulted therein. where he was meted a fine of P20,000.[31] He was found guilty of serious
The Court also declared, thus: impropriety unbecoming a judge, for delivering fistic blows on a complainant
o [W]hile the power to punish in contempt is inherent in all courts so judge. To our mind, the instant case falls under similar conduct, which the Court
as to preserve order in judicial proceedings and to uphold due avowed would be dealt with more severely if repeated, and of which the
administration of justice, judges, however, should exercise their respondent was duly warned. The respondent was, likewise, found guilty of
contempt powers judiciously and sparingly, with utmost restraint, gross ignorance of procedural law and unreasonable delay in the issuance of an
and with the end in view of utilizing their contempt powers for order of execution, where he was meted a fine of P30,000;[32] and delay in
correction and preservation not for retaliation and vindication.[21] resolving a motion to dismiss in a civil case pending before his sala where he
x We agree with the Investigating Justice when he opined that the respondent was, likewise, fined P40,000
judge should have refrained from ordering the arrest and detention of the
complainant, since the incident involved his own son, and the matter JUDGE DOLORES L. ESPAÑOL,* Presiding Judge, Regional Trial Court, Branch 90,
was very personal to him. The fact that the respondent judge insisted that Dasmariñas, Cavite, petitioner, vs. ATTY. BENJAMIN S. FORMOSO and SPOUSES BENITO
the complainant personally file his comment in court gives rise to doubts as to SEE and MARLY SEE, respondents.
the motive behind it; as the Investigating Justice puts it, the requirement of G.R. No. 150949 June 21, 2007
personal filing was deliberately inserted so that the respondent could confront FIRST DIVISION
and harass the complainant.[22]
x We also agree with the following ruminations of Justice Bersamin: FACTS:
o [T]he respondent judge obviously resented the refusal of Sison to x Sharcons Builders Philippines, Inc. (Sharcons) bought from Evanswinda Morales
let off Caoibes III from the traffic violation apprehension. The a piece of land in Cavite
refusal of Sison was apparently aggravated by the sons reporting to x TCT was issued in the name of Sharcons
the father that Sison had supposedly made the remarks of Walang x Sharcons’ worker then tried to fence the land
judge, judge Caoibes sa akin; Kahapon nga, abogado ang hinuli ko. x HOWEVER, the caretaker thereof prevented them, claiming that spouses Joseph
o The respondent Judge was not justified to so consider the act and and Enriqueta Mapua were the true owners of the land
remarks of Sison as thereby displaying arrogance towards and x Sharcons Æ filed with RTC Dasma complaint for quieting of title
deliberate disregard of the usual respect, courtesy and o Impleaded as defendants were spouses Mapua, Evanswinda
accommodation due to a court of law and its representative. First Morales, and the Register of Deeds of Trece Martires City
of all, the refusal of Sison and the supposed remarks should not x Sps Mapua Æ filed Answer
cause resentment on the part of the respondent Judge (whom o Alleged that all documents used by Sharcons were spurious and
Sison most likely did not yet know at the time) because he knew, falsified
as a public official himself, that Sison was only doing his duty of x Judge Espanol (PJ of RTC Dasma Br 90) Æ issued an Order stating that Benito
enforcing evenly the particular traffic regulation against swerving See and Marly See, president and treasurer, respectively, of Sharcons, and its
into a one-way street from the wrong direction, regardless of the counsel, Atty. Benjamin Formoso, respondents, have used a spurious certificate
office or position of the violators father. Secondly, the respondent of title and tax declaration when it (Sharcons) filed with the RTC its complaint for
Judge should have had the circumspection expected of him as a quieting of title.
judge to realize that the remarks of Sison were invited by Caoibes o Consequently, petitioner declared respondents guilty of direct
IIIs attempt to bluff his way out of the apprehension because he contempt of court and ordered their confinement for ten (10) days
was the son of an RTC judge. Hence, the respondent Judge would in the municipal jail of Dasmariñas, Cavite.
have no grounds to cite Sison for contempt of court. And, thirdly, o Petitioner stated that in determining the merits of Sharcons'
the respondent Judge and his son should have challenged the complaint for quieting of title, she "stumbled" upon Civil Case No.
issuance of the traffic violation receipt pursuant to the pertinent 623-92 for cancellation of title and damages filed with the RTC,
rules if they did not agree with the basis of the apprehension and Branch 20, Imus, Cavite, presided by then Judge Lucenito N.
also administratively charged Sison for any unwarranted act Tagle.2
committed. Since neither was done by them, but, on the contrary, o Petitioner then took judicial notice of the judge’s Decision declaring
both ultimately accepted the validity of the apprehension, as borne that Sharcons' TCT and other supporting documents are falsified
out by the retrieval of the drivers license after September 29, 1999 and that respondents are responsible therefor.
by paying the fines corresponding to the traffic violation, then it x Judge Æ then issued a warrant against respondents
follows that the respondent Judge had the consciousness that his x Respondents were then arrested and confined in the municipal jail
son was at fault, instead of Sison. x Respondents Æ filed a petition for a writ of habeas corpus before the CA
o [T]he respondent Judge claimed at the hearing that his son was at x CA Æ nullified and set aside
that time working with (sic) me as my personal driver; and that his o The Court of Appeals ruled that Judge Español erred in taking
errand was to secure some papers from the Regional Trial Court in cognizance of the Decision rendered by then Judge Tagle in Civil
Pasig City involved in a personal case which the respondent Judge Case No. 623-92 since it was not offered in evidence in Civil Case
had filed against a bank for specific performance and damages, and No. 2035-00 for quieting of title. Moreover, as the direct contempt
since I just suffered a mild stroke at that time, specifically on June of court is criminal in nature, petitioner should have conducted a
10, 1999, and the incident took place (sic) September, I could not hearing. Thus, she could have determined whether respondents are
at that time personally go to Pasig to secure the documents I guilty as charged.
needed for the next hearing of the case so I had to send my son.
o The foregoing renders clear that the respondent Judge had ISSUE: Whether herein respondents were correctly cited in direct contempt so as to warrant
no legitimate basis by which to consider Sisons their incarceration.
apprehension of his son as indirect contempt. As indicated
earlier, the act complained against must be any of those specified HELD: NO.
in Sec. 3, Rule 71, 1997 Rules of Civil Procedure; otherwise, there x The early case of In re Jones3 defined contempt of court as "some act or
is no contempt of court, which requires that the person obstructed conduct which tends to interfere with the business of the court, by a
should be performing a duty connected with judicial functions. As refusal to obey some lawful order of the court, or some act of
such, the respondent Judge acted oppressively and vindictively. disrespect to the dignity of the court which in some way tends to
x Parenthetically, it is odd that the respondent Judge would even propose herein interfere with or hamper the orderly proceedings of the court and thus
that Caoibes III, already 25 years at the time of the apprehension, was serving lessens the general efficiency of the same." It has also been described as "a
his father as the latters personal driver, albeit not officially employed in the defiance of the authority, justice or dignity of the court; such conduct as tends to
Judiciary. Most likely, therefore, Caoibes III might not be doing anything for his bring the authority and administration of the law into disrespect or to interfere with
father at the time of his apprehension but was in the place for his own or prejudice parties litigants or their witnesses during litigation."4 Simply put, it is
purposes.[23] despising of the authority, justice, or dignity of the court.5
x The act of a judge in citing a person in contempt of court in a manner x The offense of contempt traces its origin to that time in England when all courts
which smacks of retaliation, as in the case at bar, is appalling and violative of in the realm were but divisions of the Curia Regia, the supreme court of the
Rule 2.01 of the Code of Judicial Conduct which mandates that a judge should so monarch, and to scandalize a court was an affront to the sovereign.6 This
behave at all times to promote public confidence in the integrity and concept was adopted by the Americans and brought to our shores with
impartiality of the judiciary.[24] The very delicate function of administering modifications. In this jurisdiction, it is now recognized that courts have
justice demands that a judge should conduct himself at all times in a manner the inherent power to punish for contempt on the ground that respect
which would reasonably merit the respect and confidence of the people, for he is for the courts guarantees the very stability of the judicial institution.7
the visible representation of the law.[25] The irresponsible or improper conduct of Such stability is essential to the preservation of order in judicial
judges erodes public confidence in the judiciary; as such, a judge must avoid all proceedings, to the enforcement of judgments, orders, and mandates
impropriety and the appearance thereof.[26] of the courts, and, consequently, to the very administration of justice.8
x We do not agree, however, that the respondent judge should be merely x Rule 71 of the 1997 Rules of Civil Procedure, as amended, provides:
reprimanded for his actuations. The Court has not been blind to the improper o SEC. 1. Direct contempt punished summarily. - A person guilty of
use by judges of the erstwhile inherent power of contempt which, in fine, misbehavior in the presence of or so near a court as to obstruct
amounts to grave abuse of authority. The penalty imposed by the Court in such or interrupt the proceedings before the same, including disrespect
cases ranges from a fine of P2,500;[27] one months salary;[28] suspension from toward the court, offensive personalities toward others, or refusal
72
RECTO, GAYLE ANGELI M.
2011-0008 | AUSL
Personal Notes on Remedial Law 2 Review (based on the syllabus of Prof. Henedino M. Brondial)

to be sworn or to answer as a witness, or to subscribe an x In Gener v. De Leon,16 we held that courts are not authorized to take judicial
affidavit or deposition when lawfully required to do so, may notice of the contents of records of other cases even when such cases have been
be summarily adjudged in contempt by such court and punished by tried or pending in the same court. Hence, we reiterate that petitioner took
a fine not exceeding two thousand pesos or imprisonment not judicial notice of the Decision rendered by another RTC branch and on the basis
exceeding ten (10) days, or both, if it be a Regional Trial Court or a thereof, concluded that respondents used falsified documents (such as land title
court of equivalent or higher rank, or by a fine not exceeding two and tax declaration) when Sharcons filed its complaint for quieting. Verily, the
hundred pesos or imprisonment, not exceeding one (1) day, or Court of Appeals did not err in ruling that respondents are not guilty of direct
both, if it be a lower court. contempt of court.
x In Narcida v. Bowen,9 this Court characterized direct contempt as one done x Meanwhile, the instant petition challenging the Decision of the Court of Appeals
"in the presence of or so near the court or judge as to obstruct the granting the writ of habeas corpus in favor of respondents has become moot. We
administration of justice." It is a contumacious act done facie curiae recall that respondents were released after posting the required bail as ordered
and may be punished summarily without hearing.10 In other words, one by the Court of Appeals. A writ of habeas corpus will not lie on behalf of a person
may be summarily adjudged in direct contempt at the very moment or at the who is not actually restrained of his liberty. And a person discharged on bail is
very instance of the commission of the act of contumely. not restrained of his liberty as to be entitled to a writ of habeas corpus
x Section 3, Rule 71 of the same Rules states:
o SEC. 3. Indirect contempt to be punished after charge and hearing. P/SUPT. HANSEL M. MARANTAN, Petitioner, vs. ATTY. JOSE MANUEL DIOKNO and
- After a charge in writing has been filed and an opportunity given MONIQUE CU-UNJIENG LA'O, Respondents.
to the respondent to comment thereon within such period as may G.R. No. 205956 February 12, 2014
be fixed by the court and to be heard by himself or by counsel, a THIRD DIVISION
person guilty of any of the following acts may be punished for
indirect contempt: FACTS:
ƒ (a) Misbehavior of an officer of court in the x This case stemmed from a criminal case entitled "People of the Philippines v.
performance of his official duties or in his official P/SINSP Hansel M. Marantan, et al.," pending before the Regional Trial Court of
transactions; Pasig City, Branch 265
ƒ (b) Disobedience of or resistance to a lawful writ, x Marantan and his co-accused were charged with homicide
process, order, or judgment of a court, including the o The case arose from the deaths of Anton Cu-Unjieng (son of
act of a person who, after being dispossessed or respondent La’O), Francis Xavier Manzano, and Brian Anthony Dulay
ejected from any real property by the judgment or who were allegedly shot and killed by police officers in front of the
process of any court of competent jurisdiction, enters AIC Gold Tower at Ortigas Center, which incident was captured by a
or attempts or induces another to enter into or upon television crew from UNTV 37
such real property, for the purpose of executing acts x In the meantime, another shooting incident transpired which also involved
of ownership or possession, or in any manner disturbs Marantan
the possession given to the person adjudged to be x This allegedly resulted in much negative publicity on the part of Marantan
entitled thereto; x Marantan Æ filed a petition before SC to cite La’o (the private complainant in the
ƒ (c) Any abuse of or any unlawful interference with the homicide charge) and Atty. Diokno (La’o’s counsel therein) in contempt
processes or proceedings of a court not constituting o submits that the respondents violated the sub judice rule, making
direct contempt under Section 1 of this Rule; them liable for indirect contempt under Section 3(d) of Rule 71 of
ƒ (d) Any improper conduct tending, directly or the Rules of Court, for their contemptuous statements and
indirectly, to impede, obstruct, or degrade the improper conduct tending directly or indirectly to impede, obstruct
administration of justice; or degrade the administration of justice. He argues that their
ƒ (e) Assuming to be an attorney or an officer of a pronouncements and malicious comments delved not only on the
court and acting as such without authority; supposed inaction of the Court in resolving the petitions filed, but
ƒ (f) Failure to obey a subpoena duly served; also on the merits of the criminal cases before the RTC and
ƒ (g) The rescue, or attempted rescue, of a person or prematurely concluded that he and his co-accused are guilty of
property in the custody of an officer by virtue of an murder. It is Maranta’s position that the press conference was
order or process of a court held by him. organized by the respondents for the sole purpose of influencing
o But nothing in this section shall be so construed as to prevent the the decision of the Court in the petition filed before it and the
court from issuing process to bring the respondent into court, or outcome of the criminal cases before the RTC by drawing an
from holding him in custody pending such proceedings. ostensible parallelism between the Ortigas incident and the
x Indirect or constructive contempt, in turn, is one perpetrated outside of Atimonan incident.
the sitting of the court and may include misbehavior of an officer of a x La’o and Diokno Æ filed their comment
court in the performance of his official duties or in his official o argue that there was no violation of the sub judice rule as their
transactions, disobedience of or resistance to a lawful writ, process, statements were legitimate expressions of their desires, hopes and
order, judgment, or command of a court, or injunction granted by a opinions which were taken out of context and did not actually
court or a judge, any abuse or any unlawful interference with the impede, obstruct or degrade the administration of justice in a
process or proceedings of a court not constituting direct contempt, or concrete way; that no criminal intent was shown as the utterances
any improper conduct tending directly or indirectly to impede, obstruct were not on their face actionable being a fair comment of a matter
or degrade the administration of justice.11 of public interest and concern; and that this petition is intended to
x We agree with petitioner that the use of falsified and forged documents is a stifle legitimate speech.
contumacious act. However, it constitutes indirect contempt not direct
contempt. Pursuant to the above provision, such act is an improper conduct ISSUE: Whether there was a violation of the sub judice rule on the part of La’o and Atty.
which degrades the administration of justice. In Santos v. Court of First Instance Diokno so as to merit their being cited in contempt.
of Cebu, Branch VI,12 we ruled that the imputed use of a falsified document,
more so where the falsity of the document is not apparent on its face, merely HELD: NO.
constitutes indirect contempt, and as such is subject to such defenses as the x The sub judice rule restricts comments and disclosures pertaining to
accused may raise in the proper proceedings. Thus, following Section 3, Rule 71, the judicial proceedings in order to avoid prejudging the issue,
a contemner may be punished only after a charge in writing has been influencing the court, or obstructing the administration of justice. A
filed, and an opportunity has been given to the accused to be heard by violation of this rule may render one liable for indirect contempt under Sec. 3(d),
himself and counsel.13 Moreover, settled is the rule that a contempt Rule 71 of the Rules of Court,4 which reads:
proceeding is not a civil action, but a separate proceeding of a criminal nature in o Section 3. Indirect contempt to be punished after charge and
which the court exercises limited jurisdiction.14 Thus, the modes of procedure hearing. - x x x a person guilty of any of the following acts may be
and the rules of evidence in contempt proceedings are assimilated as far as punished for indirect contempt:
practicable to those adapted to criminal prosecutions.15 Perforce, petitioner ƒ (d) Any improper conduct tending, directly or
judge erred in declaring summarily that respondents are guilty of indirectly, to impede, obstruct, or degrade the
direct contempt and ordering their incarceration. She should have administration of justice[.]
conducted a hearing with notice to respondents. x The proceedings for punishment of indirect contempt are criminal in
x Petitioner, in convicting respondents for direct contempt of court, took judicial nature.5 This form of contempt is conduct that is directed against the dignity
notice of the Decision in Civil Case No. 623-92, assigned to another RTC branch, and authority of the court or a judge acting judicially; it is an act obstructing the
presided by then Judge Tagle. Section 1, Rule 129 of the Revised Rules of Court administration of justice which tends to bring the court into disrepute or
provides: disrespect. Intent is a necessary element in criminal contempt, and no
o SEC. 1. Judicial notice, when mandatory. - A court shall take one can be punished for a criminal contempt unless the evidence
judicial notice, without the introduction of evidence, of the makes it clear that he intended to commit it.6
existence and territorial extent of states, their political history, x For a comment to be considered as contempt of court "it must really appear"
forms of government, and symbols of nationality, the law of that such does impede, interfere with and embarrass the administration of
nations, the admiralty and maritime courts of the world and their justice.7 What is, thus, sought to be protected is the all-important duty of the
seals, the political constitution and history of the Philippines, the court to administer justice in the decision of a pending case.8 The specific
official acts of the legislative, executive and judicial departments of rationale for the sub judice rule is that courts, in the decision of issues of fact
the Philippines, the laws of nature, the measure of time, and the and law should be immune from every extraneous influence; that facts should be
geographical divisions. decided upon evidence produced in court; and that the determination of such
facts should be uninfluenced by bias, prejudice or sympathies.91âwphi1
73
RECTO, GAYLE ANGELI M.
2011-0008 | AUSL
Personal Notes on Remedial Law 2 Review (based on the syllabus of Prof. Henedino M. Brondial)

x The power of contempt is inherent in all courts in order to allow them to conduct delay to comply with the order of September 10, 2002 until the
their business unhampered by publications and comments which tend to impair defendants shall have fully and completely complied with the said
the impartiality of their decisions or otherwise obstruct the administration of order.
justice. As important as the maintenance of freedom of speech, is the x Capitol Æ filed a Rule 65 before the CA
maintenance of the independence of the Judiciary. The "clear and present x CA Æ dismissed the petition
danger" rule may serve as an aid in determining the proper constitutional o ruled that there is no indication that the RTC committed grave
boundary between these two rights.10 abuse of discretion amounting to lack or excess of jurisdiction.
x The "clear and present danger" rule means that the evil consequence of According to the appellate court, the September 3, 2007 Resolution
the comment must be "extremely serious and the degree of imminence was issued pursuant to Section 3,30 Rule 3 of the Interim Rules,
extremely high" before an utterance can be punished. There must exist a with the suppletory application of Section 1,31 Rule 27 of the
clear and present danger that the utterance will harm the administration of Rules. It noted that, except for the sanctions contained therein, the
justice. Freedom of speech should not be impaired through the exercise of the assailed Resolution merely reiterated the September 10, 2002
power of contempt of court unless there is no doubt that the utterances in Order of Judge Bruselas, which petitioners did not dispute in
question make a serious and imminent threat to the administration of justice. It accordance with Section 2,32 Rule 3 of the Interim Rules or via
must constitute an imminent, not merely a likely, threat.11 petition for certiorari. The CA further held that petitioners were not
x The contemptuous statements made by the respondents allegedly relate to the denied due process as they were able to move for a
merits of the case, particularly the guilt of petitioner, and the conduct of the reconsideration of the September 10, 2002 Order, but not opted to
Court as to its failure to decide G.R. No. 199462. file the same with respect to the September 3, 2007 Resolution.
x As to the merits, the comments seem to be what the respondents claim to be an x Capitol Æ filed a Rule 45 before the SC
expression of their opinion that their loved ones were murdered by Marantan. o contend that the "threatened imminent action" by the RTC to
This is merely a reiteration of their position in G.R. No. 199462, which precisely penalize them sua sponte or without regard to the guideline laid
calls the Court to upgrade the charges from homicide to murder. The Court down by the Court in Engr. Torcende v. Judge Sardido37 is not
detects no malice on the face of the said statements. The mere restatement of proper and calls for the exercise of Our power of supervision over
their argument in their petition cannot actually, or does not even tend to, the lower courts. Likewise, citing Panaligan v. Judge Ibay,38
influence the Court. among others, they claim that the threatened citation for contempt
x As to the conduct of the Court, a review of the respondents' comments reveals is not in line with the policy that there should be wilfullness or that
that they were simply stating that it had not yet resolved their petition. the contumacious act be done deliberately in disregard of the
There was no complaint, express or implied, that an inordinate amount of time authority of the court.
had passed since the petition was filed without any action from the Court. There
appears no attack or insult on the dignity of the Court either. ISSUE: Whether RTC was justified in issuing an order constituting a threat to cite Capitol in
x "A public utterance or publication is not to be denied the constitutional protection contempt.
of freedom of speech and press merely because it concerns a judicial proceeding
still pending in the cou1is, upon the theory that in such a case, it must HELD: YES.
necessarily tend to obstruct the orderly and fair administration of justice."12 By x A person guilty of disobedience of or resistance to a lawful order of a court39 or
no stretch of the imagination could the respondents' comments pose a serious commits any improper conduct tending, directly or indirectly, to impede,
and imminent threat to the administration of justice. No criminal intent to obstruct, or degrade the administration of justice40 may be punished for indirect
impede, obstruct, or degrade the administration of justice can be inferred from contempt. In particular, Section 4, Rule 3 of the Interim Rules states that, in
the comments of the respondents. addition to a possible treatment of a party as non-suited or as in default, the
x Freedom of public comment should, in borderline instances, weigh heavily sanctions prescribed in the Rules for failure to avail of, or refusal to comply with,
against a possible tendency to influence pending cases.13 The power to the modes of discovery shall apply. Under Section 3, Rule 29 of the Rules, if a
punish for contempt, being drastic and extraordinary in its nature, party or an officer or managing agent of a party refuses to obey an order to
should not be resorted to unless necessary in the interest of justice.14 produce any document or other things for inspection, copying, or photographing
In the present case, such necessity is wanting. or to permit it to be done, the court may make such orders as are just. The
enumeration of options given to the court under Section 3, Rule 29 of the Rules
is not exclusive, as shown by the phrase "among others." Thus, in Republic v.
CAPITOL HILLS GOLF & COUNTRY CLUB, INC. and PABLO B. ROMAN, JR., Petitioners, vs. Sandiganbayan,41 We said:
MANUEL O. SANCHEZ, Respondent. o To ensure that availment of the modes of discovery is otherwise
G.R. No. 182738 February 24, 2014 untrammeled and efficacious, the law imposes serious sanctions on
THIRD DIVISION the party who refuses to make discovery, such as dismissing the
action or proceeding or part thereof, or rendering judgment by
FACTS: default against the disobedient party; contempt of court, or arrest
x Manuel O. Sanchez (respondent), a stockholder of petitioner Capitol Hills Golf & of the party or agent of the party; payment of the amount of
Country Club, Inc. (Corporation) filed a petition for the nullification of the annual reasonable expenses incurred in obtaining a court order to compel
meeting of stockholders of May 21, 2002 and the special meeting of stockholders discovery; taking the matters inquired into as established in
of April 23, 2002 accordance with the claim of the party seeking discovery; refusal to
x Sanchez Æ filed a Motion for Production and Inspection of Documents allow the disobedient party support or oppose designated claims or
o RTC Æ granted defenses; striking out pleadings or parts thereof; staying further
x Capitol Æ filed an MR proceedings.42
o RTC Æ denied x If adjudged guilty of indirect contempt, the respondent who committed it against
x Capitol Æ filed a Rule 65 before the CA a Regional Trial Court or a court of equivalent or higher rank may be punished
o CA Æ dismissed with a fine not exceeding thirty thousand pesos, or imprisonment not exceeding
x Capitol Æ filed a Rule 45 before the SC six (6) months, or both.43 In this case, the threatened sanction of possibly
o SC Æ dismissed ordering petitioners to solidarily pay a fine of P10,000.00 for every day of delay
x The supposed inspection was not held in complying with the September 10, 2002 Order is well within the allowable
x Sanchez Æ set the inspection on August 1, 2003 range of penalty.
o BUT Atty. Matias V. Defensor, then Corporate Secretary of the x As far as the proceedings for indirect contempt is concerned, the case of Baculi
Corporation, was alleged to be out of town and petitioner Pablo B. v. Judge Belen44 is instructive:
Roman, Jr. (Roman) purported to have shown no willingness to o x x x Under the Rules of Court, there are two ways of initiating
comply with the directive indirect contempt proceedings: (1) motu proprio by the court; or
x Finally, the inspection was conducted on January 11, 2007 (2) by a verified petition.
o the only document produced by the Acting Corporate Secretary, x In the Matter of the Contempt Orders against Lt. Gen. Jose M. Calimlim and Atty.
Atty. Antonio V. Meriz, and one of the staff, Malou Santos, was the Domingo A. Doctor, Jr. (Calimlim) clarified the procedure prescribed for indirect
Stock and Transfer Book of the Corporation. They alleged that they contempt proceedings. We held in that case:
could not find from the corporate records the copies of the proxies o In contempt proceedings, the prescribed procedure must be
submitted by the stockholders, including the tape recordings taken followed. Sections 3 and 4, Rule 71 of the Rules of Court provide
during the stockholders’ meetings, and that they needed more time the procedure to be followed in case of indirect contempt. First,
to locate and find the list of stockholders as of March 2002, which there must be an order requiring the respondent to show cause
was in the bodega of the Corporation. why he should not be cited for contempt. Second, the respondent
x Sanchez Æ filed a Manifestation with Omnibus Motion praying that an order be must be given the opportunity to comment on the charge against
issued in accordance with Section 3, Paragraphs (a) to (d) of Rule 29 of the him. Third, there must be a hearing and the court must investigate
Rules of Court (Rules), in relation to Section 4, Rule 3 of the Interim Rules of the charge and consider respondent’s answer. Finally, only if found
Procedure Governing Intra-Corporate Controversies under Republic Act No. 8799 guilty will respondent be punished accordingly. (Citations omitted.)
(Interim Rules). x As to the second mode of initiating indirect contempt proceedings, that is,
x RTC Æ gave the parties one last chance to comply with the order dated through a verified petition, the rule is already settled in Regalado v. Go:
September 10, 2002 o In cases where the court did not initiate the contempt charge, the
o This Court orders the defendants to strictly comply with this order. Rules prescribe that a verified petition which has complied with the
Failure of the defendants to comply with all the requirements of the requirements of initiatory pleadings as outlined in the heretofore
order dated September 10, 2002 will result in this court citing all quoted provision of second paragraph, Section 4, Rule 71 of the
the defendants in contempt of court. This Court shall order Rules of Court, must be filed.
defendants solidarily to pay a fine of P10,000.00 for every day of x The Rules itself is explicit on this point:
74
RECTO, GAYLE ANGELI M.
2011-0008 | AUSL
Personal Notes on Remedial Law 2 Review (based on the syllabus of Prof. Henedino M. Brondial)

o In all other cases, charges for indirect contempt shall be


commenced by a verified petition with supporting particulars and III. SPECIAL PROCEEDINGS
certified true copies of documents or papers involved therein, and
upon full compliance with the requirements for filing initiatory
pleadings for civil actions in the court concerned. If the contempt [Rule 72]
charges arose out of or are related to a principal action pending in
the court, the petition for contempt shall allege that fact but said Section 1. Subject matter of special proceedings. — Rules of special proceedings are
petition shall be docketed, heard and decided separately, unless provided for in the following cases:
the court in its discretion orders the consolidation of the contempt
charge and the principal action for joint hearing and decision. (a) Settlement of estate of deceased persons;
(Emphasis added.)
x Thus, where there is a verified petition to cite someone in contempt of court, (b) Escheat;
courts have the duty to ensure that all the requirements for filing initiatory
pleadings have been complied with. It behooves them too to docket the petition, (c) Guardianship and custody of children;
and to hear and decide it separately from the main case, unless the presiding
judge orders the consolidation of the contempt proceedings and the main action. (d) Trustees;
x But in indirect contempt proceedings initiated motu proprio by the
court, the above rules, as clarified in Regalado, do not necessarily (e) Adoption;
apply. First, since the court itself motu proprio initiates the
proceedings, there can be no verified petition to speak of. Instead, the (f) Rescission and revocation of adoption;
court has the duty to inform the respondent in writing, in accordance
with his or her right to due process. This formal charge is done by the (g) Hospitalization of insane persons;
court in the form of an Order requiring the respondent to explain why
he or she should not be cited in contempt of court. (h) Habeas corpus;
x In Calimlim, the Judge issued an Order requiring the petitioners to explain their
failure to bring the accused before the RTC for his scheduled arraignment. We (i) Change of name;
held in that case that such Order was not yet sufficient to initiate the contempt
proceedings because it did not yet amount to a show-cause order directing the (j) Voluntary dissolution of corporations;
petitioners to explain why they should not be cited in contempt. The formal
charge has to be specific enough to inform the person, against whom contempt (k) Judicial approval of voluntary recognition of minor natural children;
proceedings are being conducted, that he or she must explain to the court;
otherwise, he or she will be cited in contempt. The Order must express this in (l) Constitution of family home;
clear and unambiguous language.
x Second, when the court issues motu proprio a show-cause order, the duty of the (m) Declaration of absence and death;
court (1) to docket and (2) to hear and decide the case separately from the main
case does not arise, much less to exercise the discretion to order the (n) Cancellation of correction of entries in the civil registry.
consolidation of the cases. There is no petition from any party to be docketed,
heard and decided separately from the main case precisely because it is the Section 2. Applicability of rules of civil actions. — In the absence of special provisions,
show-cause order that initiated the proceedings. the rules provided for in ordinary actions shall be, as far as practicable, applicable in special
x What remains in any case, whether the proceedings are initiated by a verified proceedings.
petition or by the court motu proprio, is the duty of the court to ensure that the
proceedings are conducted respecting the right to due process of the party being A. Settlement of Estate
cited in contempt. In both modes of initiating indirect contempt proceedings, if
the court deems that the answer to the contempt charge is satisfactory, the
1. Venue vs. Jurisdiction [Rule 73]
proceedings end. The court must conduct a hearing, and the court must consider
the respondent’s answer. Only if found guilty will the respondent be punished
accordingly. Section 1. Where estate of deceased persons settled. — If the decedents is an
x In contempt proceedings, the respondent must be given the right to defend inhabitant of the Philippines at the time of his death, whether a citizen or an alien, his will shall
himself or herself and have a day in court - a basic requirement of due process. be proved, or letters of administration granted, and his estate settled, in the Court of First
This is especially so in indirect contempt proceedings, as the court cannot decide Instance in the province in which he resides at the time of his death, and if he is an inhabitant
them summarily pursuant to the Rules of Court. As We have stated in Calimlim, of a foreign country, the Court of First Instance of any province in which he had estate. The
in indirect contempt proceedings, the respondent must be given the opportunity court first taking cognizance of the settlement of the estate of a decedent, shall exercise
to comment on the charge against him or her, and there must be a hearing, and jurisdiction to the exclusion of all other courts. The jurisdiction assumed by a court, so far as it
the court must investigate the charge and consider the respondent’s answer.45 depends on the place of residence of the decedent, or of the location of his estate, shall not be
contested in a suit or proceeding, except in an appeal from that court, in the original case, or
x In this case, the proceedings for indirect contempt have not been when the want of jurisdiction appears on the record.
initiated.1âwphi1 To the Court’s mind, the September 3, 2007 Resolution could be
treated as a mere reiteration of the September 10, 2002 Order. It is not yet a Section 2. Where estate settled upon dissolution of marriage. — When the marriage is
"judgment or final order of a court in a case of indirect contempt" as dissolved by the death of the husband or wife, the community property shall be inventoried,
contemplated under the Rules. The penalty mentioned therein only serves as a administered, and liquidated, and the debts thereof paid, in the testate or intestate
reminder to caution petitioners of the consequence of possible non-observance proceedings of the deceased spouse. If both spouses have died, the conjugal partnership shall
of the long-overdue order to produce and make available for inspection and be liquidated in the testate or intestate proceedings of either.
photocopying of the requested records/documents. In case of another failure or
refusal to comply with the directive, the court or respondent could formally Section 3. Process. — In the exercise of probate jurisdiction, Courts of First Instance may
initiate the indirect contempt proceedings pursuant to the mandatory issue warrants and process necessary to compel the attendance of witnesses or to carry into
requirements of the Rules and existing jurisprudence. effect theirs orders and judgments, and all other powers granted them by law. If a person
x Even if We are to treat the September 3, 2007 Resolution as a "judgment or final does not perform an order or judgment rendered by a court in the exercise of its probate
order of a court in a case of indirect contempt," this would still not work to jurisdiction, it may issue a warrant for the apprehension and imprisonment of such person
petitioners’ advantage. Section 11, Rule 71 of the Rules of Court lays down the until he performs such order or judgment, or is released.
proper remedy from a judgment in indirect contempt proceedings. It states:
o Sec. 11. Review of judgment or final order; bond for stay.--The Section 4. Presumption of death. — For purposes of settlement of his estate, a person
judgment or final order of a court in a case of indirect contempt shall be presumed dead if absent and unheard from for the periods fixed in the Civil Code. But
may be appealed to the proper court as in criminal cases. But if such person proves to be alive, he shall be entitled to the balance of his estate after
execution of the judgment or final order shall not be suspended payment of all his debts. The balance may be recovered by motion in the same proceeding.
until a bond is filed by the person adjudged in contempt, in an
amount fixed by the court from which the appeal is taken, 2. Kinds of Settlement
conditioned that if the appeal be decided against him he will abide
by and perform the judgment or final order. a. Extrajudicial
x The recourse provided for in the above-mentioned provision is clear enough: the
person adjudged in indirect contempt must file an appeal under Rule 41 (Appeal
from the Regional Trial Courts) and post a bond for its suspension pendente 1. By Agreement
lite.46 Obviously, these were not done in this case. Instead, petitioners filed a 2. By Self-Adjudication
petition for certiorari under Rule 65 of the Rules and did not post the required
bond, effectively making the September 3, 2007 Resolution final and executory. b. Judicial
________________________________________
1. Summary [Rule 74]

Section 1. Extrajudicial settlement by agreement between heirs. — If the decedent


left no will and no debts and the heirs are all of age, or the minors are represented by their

75
RECTO, GAYLE ANGELI M.
2011-0008 | AUSL
Personal Notes on Remedial Law 2 Review (based on the syllabus of Prof. Henedino M. Brondial)

judicial or legal representatives duly authorized for the purpose, the parties may without unless the will has reached the court in any other manner, and shall, within such period,
securing letters of administration, divide the estate among themselves as they see fit by signify to the court in writing his acceptance of the trust or his refusal to accept it.
means of a public instrument filed in the office of the register of deeds, and should they
disagree, they may do so in an ordinary action of partition. If there is only one heir, he may Section 4. Custodian and executor subject to fine for neglect. — A person who
adjudicate to himself the entire estate by means of an affidavit filled in the office of the neglects any of the duties required in the two last preceding sections without excused
register of deeds. The parties to an extrajudicial settlement, whether by public instrument or satisfactory to the court shall be fined not exceeding two thousand pesos.
by stipulation in a pending action for partition, or the sole heir who adjudicates the entire
estate to himself by means of an affidavit shall file, simultaneously with and as a condition Section 5. Person retaining will may be committed. — A person having custody of a will
precedent to the filing of the public instrument, or stipulation in the action for partition, or of after the death of the testator who neglects without reasonable cause to deliver the same,
the affidavit in the office of the register of deeds, a bond with the said register of deeds, in an when ordered so to do, to the court having jurisdiction, may be committed to prison and there
amount equivalent to the value of the personal property involved as certified to under oath by kept until he delivers the will.
the parties concerned and conditioned upon the payment of any just claim that may be filed
under section 4 of this rule. It shall be presumed that the decedent left no debts if no creditor
files a petition for letters of administration within two (2) years after the death of the RULE 76
decedent. Allowance or Disallowance of Will

The fact of the extrajudicial settlement or administration shall be published in a newspaper of Section 1. Who may petition for the allowance of will. — Any executor, devisee, or
general circulation in the manner provided in the nest succeeding section; but no extrajudicial legatee named in a will, or any other person interested in the estate, may, at any time after
settlement shall be binding upon any person who has not participated therein or had no notice the death of the testator, petition the court having jurisdiction to have the will allowed,
thereof. whether the same be in his possession or not, or is lost or destroyed.

Section 2. Summary settlement of estate of small value. — Whenever the gross value The testator himself may, during his lifetime, petition the court for the allowance of his will.
of the estate of a deceased person, whether he died testate or intestate, does not exceed ten
thousand pesos, and that fact is made to appear to the Court of First Instance having Section 2. Contents of petition. — A petition for the allowance of a will must show, so far
jurisdiction of the estate by the petition of an interested person and upon hearing, which shall as known to the petitioner:
be held not less than one (1) month nor more than three (3) months from the date of the last
publication of a notice which shall be published once a week for three (3) consecutive weeks (a) The jurisdictional facts;
in a newspaper of general circulation in the province, and after such other notice to interest
persons as the court may direct, the court may proceed summarily, without the appointment (b) The names, ages, and residences of the heirs, legatees, and devisees of the
of an executor or administrator, and without delay, to grant, if proper, allowance of the will, if testator or decedent;
any there be, to determine who are the persons legally entitled to participate in the estate,
and to apportion and divide it among them after the payment of such debts of the estate as (c) The probable value and character of the property of the estate;
the court shall then find to be due; and such persons, in their own right, if they are of lawful
age and legal capacity, or by their guardians or trustees legally appointed and qualified, if (d) The name of the person for whom letters are prayed;
otherwise, shall thereupon be entitled to receive and enter into the possession of the portions of
the estate so awarded to them respectively. The court shall make such order as may be just (e) If the will has not been delivered to the court, the name of the person having
respecting the costs of the proceedings, and all orders and judgments made or rendered in the custody of it.
course thereof shall be recorded in the office of the clerk, and the order of partition or award, if
it involves real estate, shall be recorded in the proper register's office. But no defect in the petition shall render void the allowance of the will, or the issuance of
letters testamentary or of administration with the will annexed.
Section 3. Bond to be filed by distributees. — The court, before allowing a partition in
accordance with the provisions of the preceding section, my require the distributees, if Section 3. Court to appoint time for proving will. Notice thereof to be published.
property other than real is to be distributed, to file a bond in an amount to be fixed by court, — When a will is delivered to, or a petition for the allowance of a will is filed in, the court having
conditioned for the payment of any just claim which may be filed under the next succeeding jurisdiction, such court shall fix a time and place for proving the will when all concerned may
section. appear to contest the allowance thereof, and shall cause notice of such time and place to be
published three (3) weeks successively, previous to the time appointed, in a newspaper of
Section 4. Liability of distributees and estate. — If it shall appear at any time within general circulation in the province.
two (2) years after the settlement and distribution of an estate in accordance with the
provisions of either of the first two sections of this rule, that an heir or other person has been But no newspaper publication shall be made where the petition for probate has been filed by
unduly deprived of his lawful participation in the estate, such heir or such other person may the testator himself.
compel the settlement of the estate in the courts in the manner hereinafter provided for the
purpose of satisfying such lawful participation. And if within the same time of two (2) years, it Section 4. Heirs, devisees, legatees, and executors to be notified by mail or
shall appear that there are debts outstanding against the estate which have not been paid, or personally. — The court shall also cause copies of the notice of the time and place fixed for
that an heir or other person has been unduly deprived of his lawful participation payable in proving the will to be addressed to the designated or other known heirs, legatees, and
money, the court having jurisdiction of the estate may, by order for that purpose, after devisees of the testator resident in the Philippines at their places of residence, and deposited
hearing, settle the amount of such debts or lawful participation and order how much and in in the post office with the postage thereon prepaid at least twenty (20) days before the
what manner each distributee shall contribute in the payment thereof, and may issue hearing, if such places of residence be known. A copy of the notice must in like manner be
execution, if circumstances require, against the bond provided in the preceding section or mailed to the person named as executor, if he be not the petitioner; also, to any person
against the real estate belonging to the deceased, or both. Such bond and such real estate named as coexecutor not petitioning, if their places of residence be known. Personal service of
shall remain charged with a liability to creditors, heirs, or other persons for the full period of copies of the notice at lest (10) days before the day of hearing shall be equivalent to mailing.
two (2) years after such distribution, notwithstanding any transfers of real estate that may
have been made. If the testator asks for the allowance of his own will, notice shall be sent only to his
compulsory heirs.
Section 5. Period for claim of minor or incapacitated person. — If on the date of the
expiration of the period of two (2) years prescribed in the preceding section the person Section 5. Proof at hearing. What sufficient in absence of contest. — At the hearing
authorized to file a claim is a minor or mentally incapacitated, or is in prison or outside the compliance with the provisions of the last two preceding sections must be shown before the
Philippines, he may present his claim within one (1) year after such disability is removed. introduction of testimony in support of the will. All such testimony shall be taken under oath
and reduced to writing. It no person appears to contest the allowance of the will, the court
2. By Petition [Rule 75-90] may grant allowance thereof on the testimony of one of the subscribing witnesses only, if such
witness testify that the will was executed as is required by law.
a. Intestate
In the case of a holographic will, it shall be necessary that at least one witness who knows the
b. Testate handwriting and signature of the testator explicitly declare that the will and the signature are in
the handwriting of the testator. In the absence of any such competent witness, and if the court
RULE 75 deem it necessary, expert testimony may be resorted to.
Production of Will. Allowance of Will Necessary
Section 6. Proof of lost or destroyed will. Certificate thereupon. — No will shall be
Section 1. Allowance necessary. Conclusive as to execution. — No will shall pass proved as a lost or destroyed will unless the execution and validity of the same be established,
either real or personal estate unless it is proved and allowed in the proper court. Subject to and the will is proved to have been in existence at the time of the death of the testator, or is
the right of appeal, such allowance of the will shall be conclusive as to its due execution. shown to have been fraudulently or accidentally destroyed in the lifetime of the testator
without his knowledge, nor unless its provisions are clearly and distinctly proved by at least two
Section 2. Custodian of will to deliver. — The person who has custody of a will shall, (2) credible witnesses. When a lost will is proved, the provisions thereof must be distinctly
within twenty (20) days after he knows of the death of the testator, deliver the will to the stated and certified by the judge, under the seal of the court, and the certificate must be filed
court having jurisdiction, or to the executor named in the will. and recorded as other wills are filed and recorded.

Section 3. Executor to present will and accept or refuse trust. — A person named as Section 7. Proof when witnesses do not reside in province. — If it appears at the
executor in a will shall, within twenty (20) days after he knows of the death of the testate, or time fixed for the hearing that none of the subscribing witnesses resides in the province, but
within twenty (20) days after he knows that he is named executor if he obtained such that the deposition of one or more of them can be taken elsewhere, the court may, on motion,
knowledge after the death of the testator, present such will to the court having jurisdiction, direct it to be taken, and may authorize a photographic copy of the will to be made and to be
presented to the witness on his examination, who may be asked the same questions with
76
RECTO, GAYLE ANGELI M.
2011-0008 | AUSL
Personal Notes on Remedial Law 2 Review (based on the syllabus of Prof. Henedino M. Brondial)

respect to it, and to the handwriting of the testator and others, as would be pertinent and
competent if the original will were present.
RULE 78
Section 8. Proof when witnesses dead or insane or do not reside in the Letters Testamentary and of Administration, When and to Whom Issued
Philippines. — If the appears at the time fixed for the hearing that the subscribing witnesses
are dead or insane, or that none of them resides in the Philippines, the court may admit the Section 1. Who are incompetent to serve as executors or administrators. — No
testimony of other witnesses to prove the sanity of the testator, and the due execution of the person in competent to serve as executor or administrator who:
will; and as evidence of the execution of the will, it may admit proof of the handwriting of the
testator and of the subscribing witnesses, or of any of them. (a) Is a minor;

Section 9. Grounds for disallowing will. — The will shall be disallowed in any of the (b) Is not a resident of the Philippines; and
following cases:
(c) Is in the opinion of the court unfit to execute the duties of the trust by reason of
(a) If not executed and attested as required by law; drunkenness, improvidence, or want of understanding or integrity, or by reason of conviction of
an offense involving moral turpitude.
(b) If the testator was insane, or otherwise mentally incapable to make a will, at the
time of its execution; Section 2. Executor of executor not to administer estate. — The executor of an
executor shall not, as such, administer the estate of the first testator.
(c) If it was executed under duress, or the influence of fear, or threats;
Section 3. Married women may serve. — A married woman may serve as executrix or
(d) If it was procured by undue and improper pressure and influence, on the part of administratrix, and the marriage of a single woman shall not affect her authority so to serve
the beneficiary, or of some other person for his benefit; under a previous appointment.

(e) If the signature of the testator was procured by fraud or trick, and he did not Section 4. Letters testamentary issued when will allowed. — When a will has been
intend that the instrument should be his will at the time of fixing his signature thereto. proved and allowed, the court shall issue letters testamentary thereon to the person named as
executor therein, if he is competent, accepts the trust, and gives bond as required by these
Section 10. Contestant to file grounds of contest. — Anyone appearing to contest the rules.
will must state in writing his grounds for opposing its allowance, and serve a copy thereof on
the petitioner and other parties interested in the estate. Section 5. Where some coexecutors disqualified others may act. — When all of the
executors named in a will can not act because of incompetency, refusal to accept the trust, or
Section 11. Subscribing witnesses produced or accounted for where will contested. failure to give bond, on the part of one or more of them, letters testamentary may issue to
— If the will is contested, all the subscribing witnesses, and the notary in the case of wills such of them as are competent, accept and give bond, and they may perform the duties and
executed under the Civil Code of the Philippines, if present in the Philippines and not insane, discharge the trust required by the will.
must be produced and examined, and the death, absence, or insanity of any of them must be
satisfactorily shown to the court. If all or some of such witnesses are present in the Philippines Section 6. When and to whom letters of administration granted. — If no executor is
but outside the province where the will has been filed, their deposition must be taken. If any or named in the will, or the executor or executors are incompetent, refuse the trust, or fail to
all of them testify against the due execution of the will, or do not remember having attested to give bond, or a person dies intestate, administration shall be granted:
it, or are otherwise of doubtful credibility, the will may nevertheless, be allowed if the court is
satisfied from the testimony of other witnesses and from all the evidence presented that the will (a) To the surviving husband or wife, as the case may be, or next of kin, or both, in
was executed and attested in the manner required by law. the discretion of the court, or to such person as such surviving husband or wife, or next of kin,
requests to have appointed, if competent and willing to serve;
If a holdgraphic will is contested, the same shall be allowed if at least three (3) witnesses who
know the handwriting of the testator explicitly declare that the will and the signature are in the (b) If such surviving husband or wife, as the case may be, or next of kin, or the
handwriting of the testator; in the absence of any competent witnesses, and if the court deem it person selected by them, be incompetent or unwilling, or if the husband or widow, or next of
necessary, expert testimony may be resorted to. kin, neglects for thirty (30) days after the death of the person to apply for administration or to
request that administration be granted to some other person, it may be granted to one or
Section 12. Proof where testator petitions for allowance of holographic will. — more of the principal creditors, if may be granted to one or more of the principal creditors, if
Where the testator himself petitions for the probate of his holographic will and no contest is competent and willing to serve;
filed, the fact that the affirms that the holographic will and the signature are in his own
handwriting, shall be sufficient evidence of the genuineness and due execution thereof. If the (c) If there is no such creditor competent and willing to serve, it may be granted to
holographic will is contested, the burden of disproving the genuineness and due execution such other person as the court may select.
thereof shall be on the contestant. The testator to rebut the evidence for the contestant.

Section 13. Certificate of allowance attached to prove will. To be recorded in the RULE 79
Office of Register of Deeds. — If the court is satisfied, upon proof taken and filed, that the Opposing Issuance Of Letters Testamentary. Petition And Contest For Letters Of
will was duly executed, and that the testator at the time of its execution was of sound and Administration
disposing mind, and not acting under duress, menace, and undue influence, or fraud, a
certificate of its allowance, signed by the judge, and attested by the seal of the court shall be Section 1. Opposition to issuance of letters testamentary. Simultaneous petition
attached to the will and the will and certificate filed and recorded by the clerk. Attested copies for administration. — Any person interested in a will may state in writing the grounds why
of the will devising real estate and of certificate of allowance thereof, shall be recorded in the letters testamentary should not issue to the persons named therein as executors, or any of
register of deeds of the province in which the lands lie. them, and the court, after hearing upon notice, shall pass upon the sufficiency of such
grounds. A petition may, at the time, be filed for letters of administration with the will
annexed.
RULE 77
Allowance of Will Proved Outside of Philippines and Administration of Estate Section 2. Contents of petition for letters of administration. — A petition for letters
Thereunder of administration must be filed by an interested person and must show, so far as known to the
petitioner:
Section 1. Will proved outside Philippines may be allowed here. — Wills proved and
allowed in a foreign country, according to the laws of such country, may be allowed, filed, and (a) The jurisdictional facts;
recorded by the proper Court of First Instance in the Philippines.
(b) The names, ages, and residences of the heirs, and the names and residences of
Section 2. Notice of hearing for allowance. — When a copy of such will and of the order the creditors, of the decedent;
or decree of the allowance thereof, both duly authenticated, are filed with a petition for
allowance in the Philippines, by the executor or other person interested, in the court having (c) The probable value and character of the property of the estate;
jurisdiction, such court shall fix a time and place for the hearing, and cause notice thereof to
be given as in case of an original will presented for allowance. (d) The name of the person for whom letters of administration are prayed.

Section 3. When will allowed, and effect thereof. — If it appears at the hearing that But no defect in the petition shall render void the issuance of letters of administration.
the will should be allowed in the Philippines, the shall so allow it, and a certificate of its
allowance, signed by the judge, and attested by the seal of the court, to which shall be Section 3. Court to set time for hearing. Notice thereof. — When a petition for letters
attached a copy of the will, shall be filed and recorded by the clerk, and the will shall have the of administration is filed in the court having jurisdiction, such court shall fix a time and place
same effect as if originally proves and allowed in such court. for hearing the petition, and shall cause notice thereof to be given to the known heirs and
creditors of the decedent, and to any other persons believed to have an interest in the estate,
Section 4. Estate, how administered. — When a will is thus allowed, the court shall grant in the manner provided in sections 3 and 4 of Rule 76.
letters testamentary, or letters of administration with the will annexed, and such letters
testamentary or of administration, shall extend to all the estate of the testator in the Section 4. Opposition to petition for administration. — Any interested person may, by
Philippines. Such estate, after the payment of just debts and expenses of administration, shall filing a written opposition, contest the petition on the ground of the incompetency of the
be disposed of according to such will, so far as such will may operate upon it; and the residue, if person for whom letters are prayed therein, or on the ground of the contestant's own right to
any shall be disposed of as is provided by law in cases of estates in the Philippines belonging to the administration, and may pray that letters issue to himself, or to any competent person or
persons who are inhabitants of another state or country. person named in the opposition.
77
RECTO, GAYLE ANGELI M.
2011-0008 | AUSL
Personal Notes on Remedial Law 2 Review (based on the syllabus of Prof. Henedino M. Brondial)

the issuance of letters testamentary or of administration under the will shall be as


Section 5. Hearing and order for letters to issue. — At the hearing of the petition, it hereinbefore provided.
must first be shown that notice has been given as hereinabove required, and thereafter the
court shall hear the proofs of the parties in support of their respective allegations, and if Section 2. Court may be remove or accept resignation of executor or
satisfied that the decedent left no will, or that there is no competent and willing executor, it administrator. Proceeding upon death, resignation, or removal. — If an executor or
shall order the issuance of letters of administration to the party best entitled thereto. administrator neglects to render his account and settle the estate according to law, or to
perform an order or judgment of the court, or a duty expressly provided by these rules, or
Section 6. When letters of administration granted to any applicant. — Letters of absconds, or becomes insane, or otherwise incapable or insuitable to discharge the trust, the
administration may be granted to any qualified applicant, though it appears that there are court may remove him, or in its discretion, may permit him to resign. When an executor or
other competent persons having better right to the administration, if such persons fail to administrator dies, resign, or is removed the remaining executor or administrator may
appear when notified and claim the issuance of letters to themselves. administer the the trust alone, unless the court grants letters to someone to act with him. If
there is no remaining executor or administrator, administration may be to any suitable person.

Section 3. Acts before revocation, resignation, or removal to be valid. — The lawful


RULE 80 acts of an executor or administrator before the revocation of his letters testamentary or of
Special Administrator administration, or before his resignation or removal, shall have the like validity as if there had
been no such revocation, resignation, or removal.
Section 1. Appointment of special administrator. — When there is delay in granting
letters testamentary or of administration by any cause including an appeal from the allowance Section 4. Powers of new executor or administrator. Renewal of license to sell
or disallowance of a will, the court may appoint a special administrator to take possession and real estate. — The person to whom letters testamentary or of administration are granted
charge of the estate of the deceased until the questions causing the delay are decided and after the revocation of former letters, or the death, resignation, or removal of a former
executors or administrators appointed. executor or administrator, shall have the like powers to collect and settle the estate not
administered that the former executor or administrator had, and may prosecute or defend
Section 2. Powers and duties of special adminsitrator. — Such special administrator actions commenced by or against the former executor or administrator, and have execution on
shall take possession and charge of the goods, chattels, rights, credits, and estate of the judgments recovered in the name of such former executor or administrator. An authority
deceased and preserve the same for the executors or administrator afterwards appointed, and granted by the court to the former executor or administrator for the sale or mortgage of real
for that purpose may commence and maintain suits as administrator. He may sell only such estate may be renewed in favor of such person without further notice or hearing.
perishable and other property as the court orders sold. A special administrator shall not be
liable to pay any debts of the deceased unless so ordered by the court.
RULE 83
Section 3. When powers of special administrator cease. Transfer of effects. Inventory and Appraisal. Provision for Support of Family
Pending suits. — When letters testamentary or of administration are granted on the estate
of the deceased, the powers of the special administrator shall cease, and he shall forthwith Section 1. Inventory and appraisal to be returned within three months. — Within
deliver to the executor or administrator the goods, chattels, money, and estate of the three (3) months after his appointment every executor or administrator shall return to the
deceased in his hands. The executor or administrator may prosecute to final judgment suits court a true inventory and appraisal of all real and personal estate of the deceased which has
commenced by such special administrator. come into his possession or knowledge. In the appraisement of such estate, the court may
order one or more of the inheritance tax appraisers to give his or their assistance.

RULE 81 Section 2. Certain article not to be inventoried. — The wearing apparel of the
Bond of Executors and Administrators surviving husband or wife and minor children., the marriage bed and bedding, and such
provisions and other articles as will necessarily be consumed in the substinence of the family
Section 1. Bond to be given issuance of letters. Amount. Conditions. — Before an of the deceased, under the direction of the court, shall not be considered as assets, nor
executor or administrator enters upon the execution of his trust, and letters testamentary or administered as such, and shall not be included in the inventory.
administration issue, he shall give a bond, in such sum as the court directs, conditioned as
follows: Section 3. Allowance to widow and family. — The widow and minor or incapacitated
children of a deceased person, during the settlement of the estate, shall receive therefrom,
(a) To make and return to the court, within three (3) months, a true and complete under the direction of the court, such allowance as are provided by law.
inventory of all goods, chattels, rights, credits, and estate of the deceased which shall come to
his possession or knowledge or to the possession of any other person for him;
RULE 84
(b) To administer according to these rules, and, if an executor, according to the will General Powers and Duties of Executors and Administrators
of the testator, all goods, chattels, rights, credits, and estate which shall at any time come to
his possession or to the possession of any other person for him, and from the proceeds to pay Section 1. Executor or administrator to have access to partnership books and
and discharge all debts, legacies, and charges on the same, or such dividends thereon as shall property. How right enforced. — The executor or administrator of the estate of a deceased
be decreed by the court; partner shall at all times have access to, and may examine and take copies of, books and
papers relating to the partnership business, and make examine and make invoices of the
(c) To render a true and just account of his administration to the court within one property belonging to such partnership; and the surviving partner or partners, on request,
(1) years, and at any other time when required by the court; shall exhibit to him all such books, papers, and property in their hands or control. On the
written application of such executor or administrator, the court having jurisdiction of the estate
(d) To perform all orders of the court by him to be performed. may order any such surviving partner or partners to freely permit the exercise of the rights,
and to exhibit the books, papers, and property, as in this section provided, and may punish
Section 2. Bond of executor where directed in will. When further bond required. any partner failing to do so for contempt.
— If the testator in his will directs that the executors serve without bond, or with only his
individual bond, he may be allowed by the court to give bond in such sum and with such Section 2. Executor or administrator to keep buildings in repair. — An executor or
surety as the court approves conditioned only to pay the debts of the testator; but the court administrator shall maintain in tenanble repair the houses and other structures and fences
may require of the executor a further bond in case of a change in his circumstance, or for belonging to the estate, and deliver the same in such repair to the heirs or devisees when
other sufficient case, with the conditions named in the last preceding section. directed so to do by the court.

Section 3. Bonds of joint executors and administrators. — When two or more persons Section 3. Executor or administrator to retain whole estate to pay debts, and to
are appointed executors or administrators the court may take a separate bond from each, or a administer estate not willed. — An executor or administrator shall have the right to the
joint bond from all. possession and management of the real as well as the personal estate of the deceased so long
as it is necessary for the payment of the debts and the expenses of administration.
Section 4. Bond of special administrator. — A special administrator before entering
upon the duties of his trust shall give a bond, in such sum as the court directs, conditioned
that he will make and return a true inventory of the goods, chattels, rights, credits, and estate RULE 85
of the deceased which come to his possession or knowledge, and that he will truly account for Accountability and Compensation of Executors and Administrators
such as are received by him when required by the court, and will deliver the same to the
person appointed executor or administrator, or to such other person as may be authorized to Section 1. Executor or administrator chargeable with all estate and income. —
receive them. Except as otherwise expressly provided in the following sections, every executor or
administrator is chargeable in his account with the whole of the estate of the deceased which
has come into his possession, at the value of the appraisement contained in the inventory;
RULE 82 with all the interest, profit, and income of such estate; and with the proceeds of so much of
Revocation of Administration, Death, Resignation, and Removal of Executors or the estate as is sold by him, at the price at which it was sold.
Administrators
Section 2. Not to profit by increase or lose by decrease in value. — No executor or
Section 1. Administration revoked if will discovered. Proceedings thereupon. — If administrator shall profit by the increase, or suffer loss by the decrease or destruction, without
after letters of administration have been granted on the estate of a decedent as if he had died his fault, of any part of the estate. He must account for the excess when he sells any part of the
intestate, his will is proved and allowed by the court, the letters of administration shall be estate for more than the appraisement, and if any is sold for the less than the
revoked and all powers thereunder cease, and the administrator shall forthwith surrender the appraisement, he is not responsible for the loss, if the sale has justly made. If he settles any
letters to the court, and render his account with such time as the court directs. Proceeding for
78
RECTO, GAYLE ANGELI M.
2011-0008 | AUSL
Personal Notes on Remedial Law 2 Review (based on the syllabus of Prof. Henedino M. Brondial)

claim against the estate for less than its nominal value, he is entitled to charge in his account
only the amount he actually paid on the settlement. Section 3. Publication of notice to creditors. — Every executor or administrator shall,
immediately after the notice to creditors is issued, cause the same to be published three (3)
Section 3. When not accountable for debts due estate. — No executor or weeks successively in a newspaper of general circulation in the province, and to be posted for
administrator shall be accountable for debts due the deceased which remain uncollected the same period in four public places in the province and in two public places in the
without his fault. municipality where the decedent last resided.

Section 4. Accountable for income from realty used by him. — If the executor or Section 4. Filing of copy of printed notice. — Within ten (10) days after the notice has
administrator uses or occupies any part of the real estate himself, he shall account for it as been published and posted in accordance with the preceding section, the executor or
may be agreed upon between him and the parties interested, or adjusted by the court with administrator shall file or cause to be filed in the court a printed copy of the notice
their assent; and if the parties do not agree upon the sum to be allowed, the same may be accompanied with an affidavit setting forth the dates of the first and last publication thereof
ascertained by the court, whose determination in this respect shall be final. and the name of the newspaper in which the same is printed.

Section 5. Accountable if he neglects or delays to raise or pay money. — When an Section 5. Claims which must be filed under the notice. If not filed, barred;
executor or administrator neglects or unreasonably delays to raise money, by collecting the exceptions. — All claims for money against the decent, arising from contract, express or
debts or selling the real or personal estate of the deceased, or neglects to pay over the money implied, whether the same be due, not due, or contingent, all claims for funeral expenses and
he has in his hands, and the value of the estate is thereby lessened or unnecessary cost or expense for the last sickness of the decedent, and judgment for money against the decent,
interest accrues, or the persons interested suffer loss, the same shall be deemed waste and the must be filed within the time limited in the notice; otherwise they are barred forever, except
damage sustained may be charged and allowed against him in his account, and he shall be that they may be set forth as counterclaims in any action that the executor or administrator
liable therefor on his bond. may bring against the claimants. Where an executor or administrator commences an action, or
prosecutes an action already commenced by the deceased in his lifetime, the debtor may set
Section 6. When allowed money paid as cost. — The amount paid by an executor or forth by answer the claims he has against the decedent, instead of presenting them
administrator for costs awarded against him shall be allowed in his administration account, independently to the court as herein provided, and mutual claims may be set off against each
unless it appears that the action or proceeding in which the costs are taxed was prosecuted or other in such action; and if final judgment is rendered in favor of the defendant, the amount
resisted without just cause, and not in good faith. so determined shall be considered the true balance against the estate, as though the claim
had been presented directly before the court in the administration proceedings. Claims not yet
Section 7. What expenses and fees allowed executor or administrator. Not to due, or contingent, may be approved at their present value.
charge for services as attorney. Compensation provided by will controls unless
renounced. — An executor or administrator shall be allowed the necessary expenses the Section 6. Solidary obligation of decedent. — Where the obligation of the decedent is
care, management, and settlement of the estate, and for his services, four pesos per day for solidary with another debtor, the claim shall be filed against the decedent as if he were the
the time actually and necessarily employed, or a commission upon the value of so much of the only debtor, without prejudice to the right of the estate to recover contribution from the
estate as comes into his possession and is finally disposed of by him in the payment of debts, debtor. In a joint obligation of the decedent, the claim shall be confined to the portion
expenses, legacies, or distributive shares, or by delivery to heirs or devisees, of two per belonging to him.
centum of the first five thousand pesos of such value, one per centum of so much of such
value as exceeds five thousand pesos and does not exceed thirty thousand pesos, one-half per Section 7. Mortgage debt due from estate. — A creditor holding a claim against the
centum of so much of such value as exceed one hundred thousand pesos. But in any special deceased secured by mortgage or other colateral security, may abandon the security and
case, where the estate is large, and the settlement has been attended with great difficulty, prosecute his claim in the manner provided in this rule, and share in the general distribution of
and has required a high degree of capacity on the part of the executor or administrator, a the assets of the estate; or he may foreclose his mortgage or realize upon his security, by
greater sum may be allowed. If objection to the fees allowed be taken, the allowance may be action in court, making the executor or administrator a party defendant, and if there is a
re-examined on appeal. judgment for a deficiency, after the sale of the mortgaged premises, or the property pledged,
in the foreclosure or other proceeding to realize upon the security, he may claim his deficiency
If there are two or more executors or administrators, the compensation shall be apportioned judgment in the manner provided in the preceding section or he may rely upon his mortgage
among them by the court according to the services actually rendered by them respectively. or other security alone, and foreclosure the same at any time within the period of the statute
of limitations, and in that event he shall not be admitted as a creditor, and shall receive no
When the executors or administrator is an attorney, he shall not charge against the estate any share in the distribution of the other assets of estate; but nothing herein contained shall
professional fees for legal services rendered by him. prohibit the executor or administrator from redeeming the property mortgaged or pledged, by
paying the debt for which it is held as security, under the direction of the court, if the court
When the deceased by will makes some other provision for the compensation of his executor, shall adjudge it to be for the best interest of the estate that such redemption shall be made.
that provision shall be a full satisfaction for his services unless by a written instrument filed in
the court he renounces all claim to the compensation provided by the will. Section 8. Claim of executor or administrator against an estate. — If the executor or
administrator has a claim against the estate he represents, he shall give notice thereof, in
Section 8. When executor or administrator to render account. — Every executor or writing, to the court, and the court shall appoint a special administrator, who shall, in the
administrator shall render an account of his administration within one (1) year from the time adjustment of such claim, have the same power and be subject to the same liability as the
of receiving letters testamentary or of administration, unless the court otherwise directs general administrator or executor in the settlement of other claims. The court may order the
because of extensions of time for presenting claims against, or paying the debts of, the estate, executor or administrator to pay to the special administrator necessary funds to defend such
or for disposing of the estate; and he shall render such further accounts as the court may claim.
require until the estate is wholly settled.
Section 9. How to file a claim. Contents thereof. Notice to executor or
Section 9. Examinations on oath with respect to account — The court may examine administrator. — A claim may be filed by delivering the same with the necessary vouchers to
the executor or administrator upon oath with respect to every matter relating to any account the clerk of court and by serving a copy thereof on the executor or administrator. If the claim
rendered by him, and shall so examine him as to the correctness of his account before the be founded on a bond, bill, note, or any other instrument, the original need not be filed, but a
same is allowed, except when no objection is made to the allowance of the account and its copy thereof with all indorsements shall be attached to the claim and filed therewith. On
correctness is satisfactorily established by competent proof. The heirs, legatees, distributees, demand, however, of the executor or administrator, or by order of the court or judge, the
and creditors of the estate shall have the same privilege as the executor or administrator of original shall be exhibited, unless it be list or destroyed, in which case the claimant must
being examined on oath on any matter relating to an administration account. accompany his claim with affidavit or affidavits containing a copy or particular description of
the instrument and stating its loss or destruction. When the claim is due, it must be supported
Section 10. Account to be settled on notice. — Before the account of an executor or by affidavit stating the amount justly due, that no payments have been made thereon which
administrator is allowed, notice shall be given to persons interested of the time and place of are not credited, and that there are no offsets to the same, to the knowledge of the affiant. If
examining and allowing the same; and such notice may be given personally to such persons the claim is not due, or is contingent, when filed, it must also be supported by affidavits
interested or by advertisement in a newspaper or newspapers, or both, as the court directs. stating the particulars thereof. When the affidavit is made by a person other than the
claimant, he must set forth therein the reason why it is not made by the claimant. The claim
Section 11. Surety on bond may be party to accounting. — Upon the settlement of the once filed shall be attached to the record of the case in which the letters testamentary or of
account of an executor or administrator, a person liable as surety in respect to such account administration were issued, although the court, in its discretion, and as a matter of
may, upon application, be admitted as party to such accounting. convenience, may order all the claims to be collected in a separate folder.

Section 10. Answer of executor or administrator. Offsets —Within fifteen (15) days
RULE 86 after service of a copy of the claim on the executor or administrator, he shall file his answer
Claims Against Estate admitting or denying the claim specifically, and setting forth the admission or denial. If he has
no knowledge sufficient to enable him to admit or deny specifically, he shall state such want of
Section 1. Notice to creditors to be issued by court. — Immediately after granting knowledge. The executor or administrator in his answer shall allege in offset any claim which
letters testamentary or of administration, the court shall issue a notice requiring all persons the decedent before death had against the claimant, and his failure to do so shall bar the
having money claims against the decedent to file them in the office of the clerk of said court. claim forever. A copy of the answer shall be served by the executor or administrator on the
claimant. The court in its discretion may extend the time for filing such answer.
Section 2. Time within which claims shall be filed. — In the notice provided in the
preceding section, the court shall estate the time for the filing of claims against the estate, Section 11. Disposition of admitted claim. — Any claim admitted entirely by the executor
which shall not be more than twelve (12) not less than six (6) months after the date of the or administrator shall immediately be submitted by the clerk to the court who may approve the
first publication of the notice. However, at any time before an order of distribution is entered, same without hearing; but the court, in its discretion, before approving the claim, may order
on application of a creditor who has failed to file his claim within the previously limited, the that known heirs, legatees, or devisees be notified and heard. If upon hearing, an heir,
court may, for cause shown and on such terms as are equitable, allow such claim to be filed legatees, or devisee opposes the claim, the court may, in its discretion, allow him fifteen (15)
within a time not exceeding one (1) month. days to file an answer to the claim in the manner prescribed in the preceding section.
79
RECTO, GAYLE ANGELI M.
2011-0008 | AUSL
Personal Notes on Remedial Law 2 Review (based on the syllabus of Prof. Henedino M. Brondial)

shall not be bound to commence the action unless on application of the creditors of the
Section 12. Trial of contested claim. — Upon the filing of an answer to a claim, or upon deceased, not unless the creditors making the application pay such part of the costs and
the expiration of the time for such filing, the clerk of court shall set the claim for trial with expenses, or give security therefor to the executor or administrator, as the court deems
notice to both parties. The court may refer the claim to a commissioner. equitable.

Section 13. Judgment appealable. — The judgment of the court approving or Section 10. When creditor may bring action. Lien for costs. — When there is such a
disapproving a claim, shall be filed with the record of the administration proceedings with notice deficiency of assets, and the deceased in his lifetime had made or attempted such a
to both parties, and is appealable as in ordinary cases. A judgment against the executor or conveyance, as is stated in the last preceding section, and the executor or administrator has
administrator shall be that he pay, in due course of administration, the amount ascertained to not commenced the action therein provided for, any creditor of the estate may, with the
be due, and it shall not create any lien upon the property of the estate, or give to the judgment permission of the court, commence and prosecute to final judgment, in the name of the
creditor any priority of payment. executor or administrator, a like action for the recovery of the subject of the conveyance or
attempted conveyance for the benefit of the creditors. But the action shall not be commenced
Section 14. Costs. — When the executor or administrator, in his answer, admits and offers to until the creditor has filed in a court a bond executed to the executor or administrator, in an
pay part of a claim, and the claimant refuses to accept the amount offered in satisfaction of his amount approved by the judge, conditioned to indemnify the executor or administrator against
claim, if he fails to obtain a more favorable judgment, he cannot recover costs, but must pay to the costs and expenses incurred by reason of such action. Such creditor shall have a lien upon
the executor or administrator costs from the time of the offer. Where an action any judgment recovered by him in the action for such costs and other expenses incurred
commenced against the deceased for money has been discontinued and the claim embraced therein as the court deems equitable. Where the conveyance or attempted conveyance had
therein presented as in this rule provided, the prevailing party shall be allowed the costs of his been made by the deceased in his lifetime in favor of the executor or administrator, the action
action up to the time of its discontinuance. which a credit may bring shall be in the name of all the creditors, and permission of the court
and filing of bond as above prescribed, are not necessary.

RULE 87
Actions By and Against Executors and Administrators
RULE 88
Section 1. Actions which may and which may not be brought against executor or Payment of the Debts of the Estate
administrator. — No action upon a claim for the recovery of money or debt or interest
thereon shall be commenced against the executor or administrator; but to recover real or Section 1. Debts paid in full if estate sufficient. — If, after hearing all the money
personal property, or an interest therein, from the estate, or to enforce a lien thereon, and claims against the estate, and after ascertaining the amount of such claims, it appears that
actions to recover damages for an injury to person or property, real or personal, may be there are sufficient assets to pay the debts, the executor or administrator pay the same within
commenced against him. the time limited for that purpose.

Section 2. Executor or administrator may bring or defend actions which survive. Section 2. Part of estate from which debt paid when provision made by will. — If
— For the recovery or protection of the property or rights of the deceased, an executor or the testator makes provision by his will, or designates the estate to be appropriated for the
administrator may bring or defend, in the right of deceased, actions for causes which survive. payment of his debts, the expenses of administration, or the family expenses, they shall be
paid according to the provisions of the will; but if the provision made by the will or the estate
Section 3. Heir may not sue until shall assigned — When an executor or administrator is appropriated, is not sufficient for that purpose, such part of the estate of the testator, real or
appointed and assumes the trust, no action to recover the title or possession of lands or for personal, as is not disposed of by will, if any shall be appropriated for that purpose.
damages done to such lands shall be maintained against him by an heir or devisee until there
is an order of the court assigning such lands to such heir or devisee or until the time allowed Section 3. Personalty first chargeable for debts, then realty. — The personal estate of
for paying debts has expired. the deceased not disposed of by will shall be first chargeable with the payment of debts and
expenses; and if said personal estate is not sufficient for tat purpose, or its sale would
Section 4. Executor or administrator may compound with debtor. — Within the redound to the detriment of the participants for the estate, the whole of the real estate not
approval of the court, an executor or administrator may compound with the debtor of the dispose of by will, or so much thereof as is necessary, may be sold, mortgaged, or otherwise
deceased for a debt due, and may give a discharge of such debt on receiving a just dividend encumbered for that purpose by the executor or administrator, after obtaining the authority of
of the estate of the debtor. the court therefor. Any deficiency shall be met by contributions in accordance with the
provisions of section 6 of this rule.
Section 5. Mortgage due estate may be foreclosed. — A mortgage belonging to the
estate of a deceased person, as mortgagee or assignee of the right or a mortgage, may be Section 4. Estate to be retained to meet contingent claims. — If the court is satisfied
foreclosed by the executor or administrator. that a contingent claim duly filed is valid, it may order the executor or administrator to retain
in his hands sufficient estate to pay such contingent claim when the same becomes absolute,
Section 6. Proceedings when property concealed, embezzled, or fraudulently or if the estate is insolvent, sufficient to pay a portion equal to the dividend of the other
conveyed. — If an executor or administrator, heir, legatee, creditor or other individual creditors.
interested in the estate of the deceased, complains to the court having jurisdiction of the
estate that a person is suspected of having concealed, embezzled, or conveyed away any of Section 5. How contingent claim becoming absolute in two years allowed and
the money, goods, or chattels of the deceased, or that such person has in his possession or paid. Action against distributees later. — If such contingent claim becomes absolute and
has knowledge of any deed, conveyance, bond, contract, or other writing which contains is presented to the court, or to the executor or administrator, within two (2) years from the
evidence of or tends or discloses the right, title, interest, or claim of the deceased, the court time limited for other creditors to present their claims, it may be allowed by the court if not
may cite such suspected person to appear before it any may examine him on oath on the disputed by the executor or administrator and, if disputed, it may be proved and allowed or
matter of such complaint; and if the person so cited refuses to appear, or to answer on such disallowed by the court as the facts may warrant. If the contingent claim is allowed, the
examination or such interrogatories as are put to him, the court may punish him for contempt, creditor shall receive payment to the same extent as the other creditors if the estate retained
and may commit him to prison until he submits to the order of the court. The interrogatories by the executor or administrator is sufficient. But if the claim is not so presented, after having
put any such person, and his answers thereto, shall be in writing and shall be filed in the become absolute, within said two (2) years, and allowed, the assets retained in the hands of
clerk's office. the executor or administrator, not exhausted in the payment of claims, shall be disturbed by
the order of the court to the persons entitled to the same; but the assets so distributed may
Section 7. Person entrusted with estate compelled to render account. — The court, still be applied to the payment of the claim when established, and the creditor may maintain
on complaint of an executor or administrator, may cite a person entrusted by an executor or an action against the distributees to recover the debt, and such distributees and their estates
administrator with any part of the estate of the deceased to appear before it, and may require shall be liable for the debt in proportion to the estate they have respectively received from the
such person to render a full account, on oath, of the money, goods, chattels, bonds, account, property of the deceased.
or other papers belonging to such estate as came to his possession in trust for such executor
or administrator, and of his proceedings thereon; and if the person so cited refuses to appear Section 6. Court to fix contributive shares where devisees, legalitees, or heirs have
to render such account, the court may punish him for contempt as having disobeyed a lawful been possession. — Where devisees, legalitees, or heirs have entered into possession of
order of the court. portions of the estate before the debts and expenses have been settled and paid, and have
become liable to contribute for the payment of such debts and expenses, the court having
Section 8. Embezzlement before letters issued — If a person, before the granting of jurisdiction of the estate may, by order for that purpose, after hearing, settle the amount of
letters testamentary or of administration on the estate of the deceased, embezzles or alienates their several liabilities, and order how much and in what manner each person shall contribute,
any of the money, goods, chattels, or effects of such deceased, such person shall be liable to and may issue execution as circumstances require.
an action in favor of the executor or administrator of the estate for double the value of the
property sold, embezzled, or alienated, to be recovered for the benefit of such estate. Section 7. Order of payment if estate insolvent — If the assets which can be
appropriated for the payment of debts are not sufficient for that purpose, the executor or
Section 9. Property fraudulently conveyed by deceased may be recovered. When administrator shall pay the debts against the estate, observing the provisions of Articles 1059
executor or administrator must bring action. — When there is a deficiency of assets in and 2239 to 2251 of the Civil Code.
the hands of an executor or administrator for the payment of debts and expenses of
administration, and the deceased in his lifetime had conveyed real or personal property, or a Section 8. Dividends to be paid in proportion to claims. — If there are no assets
right or interest therein, or an debt or credit, with intent to defraud his creditors or to avoid sufficient to pay the credits of any once class of creditors after paying the credits entitled to
any right, debt, or duty; or had so conveyed such property, right, interest, debt or credit that preference over it, each creditor within such class shall be paid a dividend in proportion to his
by law the conveyance would be void as against his creditors, and the subject of the claim. No creditor of any one class shall receive any payment until those of the preceding class
attempted conveyance would be liable to attachment by any of them in his lifetime, the are paid.
executor or administrator may commence and prosecute to final judgment an action for the
recovery of such property, right, interest, debt, or credit for the benefit of the creditors; but he
80
RECTO, GAYLE ANGELI M.
2011-0008 | AUSL
Personal Notes on Remedial Law 2 Review (based on the syllabus of Prof. Henedino M. Brondial)

Section 9. Estate of insolvent non-resident, how disposed of. — In case granted if any person interested in the estate gives a bond, in a sum to be fixed by the court,
administration is taken in the Philippine of the estate of a person who was at the time of his conditioned to pay the debts, expenses of administration, and legacies within such time as the
death an inhabitant of another country, and who died insolvent, hi estate found in the court directs; and such bond shall be for the security of the creditors, as well as of the
Philippines shall, as far as practicable, be so disposed of that his creditors here and elsewhere executor or administrator, and may be prosecuted for the benefit of either.
may receive each an equal share, in proportion to their respective credits.
Section 4. When court may authorize sale of estate as beneficial to interested
Section 10. When and how claim proved outside the Philippines against insolvent persons. Disposal of proceeds. — When it appears that the sale of the whole or a part of
resident's estate paid. — If it appears to the court having jurisdiction that claims have been the real or personal estate, will be beneficial to the heirs, devisees, legatees, and other
duly proven in another country against the estate of an insolvent who was at the time of his interested persons, the court may, upon application of the executor or administrator and on
death an inhabitant of the Philippines, and that the executor or administrator in the Philippines written notice to the heirs, devisees, and legatees who are interested in the estate to be sold,
had knowledge of the presentation of such claims in such country and an opportunity to authorize the executor or administrator to sell the whole or a part of said estate, although not
contest their allowance, the court shall receive a certified list of such claims, when perfected in necessary to pay debts, legacies, or expenses of administration; but such authority shall not be
such country, and add the same to the list of claims proved against the deceased person in granted if inconsistent with the provisions of a will. In case of such sale, the proceeds shall be
the Philippines so that a just distribution of the whole estate may be made equally among all assigned to the persons entitled to the estate in the proper proportions.
its creditors according to their respective claims; but the benefit of this and the preceding
sections shall not be extended to the creditors in another country if the property of such Section 5. When court may authorize sale, mortgage, or other encumbrance of
deceased person there found is not equally apportioned to the creditors residing in the estate to pay debts and legacies in other countries. — When the sale of personal
Philippines and the other creditor, according to their respective claims. estate, or the sale, mortgage, or other encumbrance of real estate is not necessary to pay the
debts, expenses of administration, or legacies in the Philippines, but it appears from records
Section 11. Order for payment of debts. — Before the expiration of the time limited for and proceedings of a probate court in another country that the estate of the deceased in such
the payment of the debts, the court shall order the payment thereof, and the distribution of other country is not sufficient to pay the debts, expenses of administration, and legacies there,
the assets received by the executor or administrator for that purpose among the creditors, as the court here may authorize the executor or administrator to sell the personal estate or to
the circumstances of the estate require and in accordance with the provisions of this rule. sell, mortgage, or otherwise encumber the real estate for the payment of debts or legacies in
the other country, in same manner as for the payment of debts or legacies in the Philippines.
Section 12. Orders relating to payment of debts where appeal is taken. — If an
appeal has been taken from a decision of the court concerning a claim, the court may suspend Section 6. When court may authorize sale, mortgage, or other encumbrance of
the order for the payment of the debts or may order the distributions among the creditors realty acquired on execution or foreclosure. — The court may authorize an executor or
whose claims are definitely allowed, leaving in the hands of the executor or administrator administrator to sell mortgage, or otherwise encumber real estate acquired by him on
sufficient assets to pay the claim disputed and appealed. When a disputed claim is finally execution or foreclosure sale, under the same cicumstances and under the same regulations
settled the court having jurisdiction of the estate shall order the same to be paid out of the as prescribed in this rule for the sale, mortgage, or other encumbrance of other real estate.
assets retained to the same extent and in the same proportion with the claims of other
creditors. Section 7. Regulation for granting authority to sell, mortgage, or otherwise
encumber estate. — The court having jurisdiction of the estate of the deceased may
Section 13. When subsequent distribution of assets ordered. — If the whole of the authorize the executor or administrator to sell personal estate, or to sell, mortgage, or
debts are not paid on the first distribution, and if the whole assets are not distributed, or other otherwise encumber real estate, in cases provided by these rules and when it appears
assets afterwards come to the hands of the executor or administrator, the court may from time necessary or beneficial under the following regulations.
to time make further orders for the distributions of assets.
(a) The executor or administrator shall file a written petition setting forth the debts
Section 14. Creditors to be paid in accordance with terms of order. — When an order is due from the deceased, the expenses of administration, the legacies, the value of the personal
made for the distribution of assets among the creditors, the executor or administration shall, as estate, the situation of the estate to be sold, mortgaged, or otherwise encumbered, and such
soon as the time of payment arrives, pay the creditors the amounts of their claims, or the other facts as show that the sale, mortgage, or other encumbrance is necessary or beneficial.
dividend thereon, in accordance with the terms of such order.
(b) The court shall thereupon fix a time and place for hearing such petition, and
Section 15. Time for paying debts and legacies fixed, or extended after notice, cause notice stating the nature of the petition, the reasons for the same, and the time and
within what periods. — On granting letters testamentary or administration the court shall place of hearing, to be given personally or by mail to the persons interested, and may cause
allow to the executor or administrator a time for disposing of the estate and paying the debts such further notice to be given, by publication or otherwise, as it shall deem proper;
and legacies of the deceased, which shall not, in the first instance, exceed one (1) year; but
the court may, on application of the executor or administrator and after hearing on such notice (c) If the court requires it, the executor or administrator shall give an additional
of the time and place therefor given to all persons interested as it shall direct, extend the time bond, in such sum as the court directs, conditioned that such executor or administrator will
as the circumstances of the estate require not exceeding six (6) months for a single extension account for the proceeds of the sale, mortgage, or other encumbrance;
not so that the whole period allowed to the original executor or administrator shall exceed two
(2) years. (d) If the requirements in the preceding subdivisions of this section have been
complied with, the court, by order stating such compliance, may authorize the executor or
Section 16. Successor of dead executor or administrator may have time extended administrator to sell, mortgage, or otherwise encumber, in proper cases, such part of the
on notice within certain period. — When an executor or administrator dies, and a new estate as is deemed necessary, and in case of sale the court may authorize it to be public or
administrator of the same estate is appointed, the court may extend the time allowed for the private, as would be most beneficial to all parties concerned. The executor or administrator
payment of the debts or legacies beyond the time allowed to the original executor or shall be furnished with a certified copy of such order;
administrator, not exceeding six (6) months at a time and not exceeding six (6) months
beyond the time which the court might have allowed to such original executor or (e) If the estate is to be sold at auction, the mode of giving notice of the time and
administrator; and notice shall be given of the time and place for hearing such application, as place of the sale shall be governed by the provisions concerning notice of execution sale;
required in the last preceding section.
(f) There shall be recorded in the registry of deeds of the province in which the real
estate thus sold, mortgage, or otherwise encumbered is situated, a certified copy of the order
RULE 89 of the court, together with the deed of the executor or administrator for such real estate,
Sales, Mortgages, and Other Encumbrances of Property of Decedent which shall be as valid as if the deed had been executed by the deceased in his lifetime.

Section 1. Order of sale of personalty. — Upon the application of the executor or Section 8. When court may authorize conveyance of realty which deceased
administrator, and on written notice to the heirs and other persons interested, the court may contracted to convey. Notice. Effect of deed. — Where the deceased was in his lifetime
order the whole or a part of the personal estate to be sold, if it appears necessary for the under contract, binding in law, to deed real property, or an interest therein, the court having
purpose of paying debts, expenses of administration, or legacies, or for the preservation of the jurisdiction of the estate may, on application for that purpose, authorize the executor or
property. administrator to convey such property according to such contract, or with such modifications
as are agreed upon by the parties and approved by the court; and if the contract is to convey
Section 2. When court may authorize sale, mortgage, or other encumbrance of real property to the executor or administrator, the clerk of court shall execute the deed. The
realty to pay debts and legacies through personalty not exhausted. — When the deed executed by such executor, administrator, or clerk of court shall be as affectual to
personal estate of the deceased is not sufficient to pay the debts, expenses of administration, convey the property as if executed by the deceased in his lifetime; but no such conveyance
and legacies, or where the sale of such personal estate may injure the business or other shall be authorized until notice of the application for that purpose has been given personally or
interests of those interested in the estate, and where a testator has not otherwise made by mail to all persons interested, and such further notice has been given, by publication or
sufficient provision for the payment of such debts, expenses, and legacies, the court, on the otherwise, as the court deems proper; nor if the assets in the hands of the executor or
application of the executor or administrator and on written notice of the heirs, devisees, and administrator will thereby be reduced so as to prevent a creditor from receiving his full debt or
legatees residing in the Philippines, may authorize the executor or administrator to sell, diminish his dividend.
mortgage, or otherwise encumber so much as may be necessary of the real estate, in lieu of
personal estate, for the purpose of paying such debts, expenses, and legacies, if it clearly Section 9. When court may authorize conveyance of lands which deceased held in
appears that such sale, mortgage, or encumbrance would be beneficial to the persons trust. — Where the deceased in his lifetime held real property in trust for another person, the
interested; and if a part cannot be sold, mortgaged, or otherwise encumbered without injury court may after notice given as required in the last preceding section, authorize the executor
to those interested in the remainder, the authority may be for the sale, mortgage, or other or administrator to deed such property to the person, or his executor or administrator, for
encumbrance of the whole of such real estate, or so much thereof as is necessary or beneficial whose use and benefit it was so held; and the court may order the execution of such trust,
under the circumstances. whether created by deed or by law.

Section 3. Persons interested may prevent such sale, etc., by giving bond. — No
such authority to sell, mortgage, or otherwise encumber real or personal estate shall be RULE 90
81
RECTO, GAYLE ANGELI M.
2011-0008 | AUSL
Personal Notes on Remedial Law 2 Review (based on the syllabus of Prof. Henedino M. Brondial)

Distribution and Partition of the Estate the court concerning questions as to advancements made shall be
binding on the person raising the question and on the heir.
Section 1. When order for distribution of reside made. — When the debts, funeral x Teresita Æ filed an MR to the said Order
charges, and expenses of administration, the allowance to the widow, and inheritance tax, if x RTC Æ issued another Order
any, chargeable to the estate in accordance with law, have been paid, the court, on the o x x x Foremost to be resolved is the issue of collation of the
application of the executor or administrator, or of a person interested in the estate, and after properties of the deceased Rafael Nicolas which were disposed by
hearing upon notice, shall assign the residue of the estate to the persons entitled to the same, the latter long before his death. The oppositor-applicant Ramon
naming them and the proportions, or parts, to which each is entitled, and such persons may Nicolas should prove to the satisfaction of the Court whether the
demand and recover their respective shares from the executor or administrator, or any other properties disposed of by the late Rafael Nicolas before the latters
person having the same in his possession. If there is a controversy before the court as to who death was gratuitous or for valuable consideration. The Court
are the lawful heirs of the deceased person or as the distributive shares to which each person believes that he or she who asserts should prove whether the
is entitled under the law, the controversy shall be heard and decided as in ordinary cases. disposition of the properties was gratuitously made or for valuable
consideration.
No distribution shall be allowed until the payment of the obligations above mentioned has o The Court has already set for hearing on July 21, 1995, at 8:30
been made or provided for, unless the distributees, or any of them, give a bond, in a sum to a.m., the reception and/or presentation of evidence in the issue of
be fixed by the court, conditioned for the payment of said obligations within such time as the collated properties disposed before the death of Rafael Nicolas
court directs. x RTC Æ then removed Teresita as administratix
o on ground of conflict of interest considering her claim that she paid
Section 2. Questions as to advancement to be determined. — Questions as to valuable consideration for the subject properties acquired by her
advancement made, or alleged to have been made, by the deceased to any heir may be heard from their deceased father and therefore the same should not be
and determined by the court having jurisdiction of the estate proceedings; and the final order included in the collation;[13] and, ordered the hearing on the
of the court thereon shall be binding on the person raising the questions and on the heir. collation of properties covered by TCT No. T-V-1211 and T-V-1210
only.
Section 3. By whom expenses of partition paid. — If at the time of distribution the x TeresitaÆ filed an MR
executor or administrator has retained sufficient effects in his hands which may lawfully be o RTC Æ denied
applied for the expenses of partition of the properties distributed, such expenses of partition x Teresita (together with Zenaida Nicolas (the surviving spouse of Antonio Nicolas)
may be paid by such executor or administrator when it appears equitable to the court and not and the Heirs of Antonio Nicolas) Æ filed a Rule 65 before the CA x
inconsistent with the intention of the testator; otherwise, they shall be paid by the parties in CA Æ dismissed the Rule 65 petition
proportion to their respective shares or interest in the premises, and the apportionment shall
be settled and allowed by the court, and, if any person interested in the partition does not pay
his proportion or share, the court may issue an execution in the name of the executor or
administrator against the party not paying the sum assessed.

Section 4. Recording the order of partition of estate. — Certified copies of final orders ISSUE: Whether the subject order is indeed one of collation which is a final order.
and judgments of the court relating to the real estate or the partition thereof shall be recorded
in the registry of deeds of the province where the property is situated. HELD: NO.
x Petitioners claim that: private respondent never presented any document to
3. Partition [Rule 69] prove that the properties transferred by their deceased parents to petitioners are
by gratuitous title; private respondent never notified petitioner of any hearing on
3. The Administrator or Executor said documents to give them opportunity to show cause why their properties
should not be collated; the assailed Order dated November 11, 1994 is arbitrary,
capricious, whimsical, confiscatory, depriving them of due process; the said order
a. Special vs. Regular [Rule 80] is interlocutory in nature and therefore non-appealable; the properties acquired
b. Bonds [Rule 81] by petitioner Teresita N. de Leon and her deceased brother Antonio Nicolas,
c. Powers and Duties [Rule 84] married to petitioner Zenaida C. Nicolas and their children, were sold to them as
d. Accountability [ Rule 85] evidenced by public documents; and, the properties were already titled in their
respective names or sold to third persons.
x Private respondent contends that: due process has been afforded the petitioners
4. Claims against the Estate [Rule 86] when the RTC resolved the issue of collation of the subject properties after
hearing; petitioner deliberately omitted certain material facts in the petition to
5. Actions by and against Executor and Administrator [Rule 87] mislead the Court because petitioners were actually given at least three (3) times
the opportunity to ventilate and oppose the issue of collation; as stated by the
6. Distribution and Partition [Rule 90] appellate court in the Resolution promulgated on February 10, 1997, both parties
affirmed that the RTC had proceeded to conduct hearings on January 21 and 28,
1997 as originally scheduled; presentation of evidence had been terminated and
the twin issues of the appointment of a new administratrix and the collation of
TERESITA N. DE LEON, ZENAIDA C. NICOLAS and the HEIRS OF ANTONIO NICOLAS,
two (2) properties covered by TCT No. T-V-1210 and T-V-1211 were already
petitioners, vs. HON. COURT OF APPEALS, HON. PABLO P. INVENTOR and RAMON
submitted for resolution to the court below;[20] subject properties are collatable
NICOLAS, respondents.
under Articles 1601 and 1071 of the Civil Code and Section 2 of Rule 90 of the
G.R. NO. 128781. August 6, 2002
Rules of Court and the ruling in Guinguing v. Abuton and Abuton, 48 Phil. 144;
FIRST DIVISION
petitioner failed to present evidence that there was valuable consideration for
these properties and failed to rebut the evidence that petitioners do not have the
FACTS:
financial capability to pay for these properties as evidenced by the testimony of
x Deceased spouses Rafael and Salud Nicolas were the parents of petitioner
credible witnesses who are relatives of spouses decedents.
Teresita N. de Leon, Estrellita N. Vizconde, Antonio Nicolas (deceased husband of
x We find the petition partly meritorious.
petitioner Zenaida Nicolas and predecessor of the petitioners Heirs of Antonio
x Contrary to the finding of the Court of Appeals that the Order of November 11,
Nicolas), Ramon Nicolas and Roberto Nicolas.
1994 had become final for failure of petitioners to appeal therefrom in due time,
x Teresita N. de Leon was appointed administratrix of the estate of Rafael C.
we hold that said Order is interlocutory in nature. Our pronouncement in Garcia
Nicolas in a case entitled In the Matter of the Intestate Estate of Rafael C.
v. Garcia supports this ruling:
Nicolas
o The court which acquires jurisdiction over the properties of
x Ramon (as oppositor) Æ filed a Motion for Collation
a deceased person through the filing of the corresponding
o claiming that deceased Rafael Nicolas, during his lifetime, had
proceedings, has supervision and control over the said
given certain real properties to his children by gratuitous title and
properties, and under the said power, it is its inherent duty
that administratrix-petitioner Teresita failed to include the same in
to see that the inventory submitted by the administrator
the inventory of the estate of the decedent
appointed by it contains all the properties, rights and
x RTC Æ directed Ramon to substantiate his claims
credits which the law requires the administrator to set out
x Ramon Æ filed an Amended Motion for Collation specifying the properties to be
in his inventory. In compliance with this duty the court has
collated and attaching to said motion, the documents in support thereof
also inherent power to determine what properties, rights
x RTC Æ issued an Order
and credits of the deceased should be included in or
o the Administratrix is hereby ordered to include the said properties
excluded from the inventory. Should an heir or person
which were received from the decedent for collation in the instant
interested in the properties of a deceased person duly call
probate proceedings.
the courts attention to the fact that certain properties,
x Teresita Æ filed an MR
rights or credits have been left out in the inventory, it is
o alleging that the properties subject of the Order were already titled
likewise the courts duty to hear the observations, with
in their names years ago[8] and that titles may not be collaterally
power to determine if such observations should be
attacked in a motion for collation
attended to or not and if the properties referred to therein
x RTC Æ denied the MR
belong prima facie to the intestate, but no such
o ruling that it is within the jurisdiction of the court to determine
determination is final and ultimate in nature as to the
whether titled properties should be collated,[9] citing Section 2,
ownership of the said properties.[21] (Emphasis supplied)
Rule 90 of the Rules of Court which provides that the final order of

82
RECTO, GAYLE ANGELI M.
2011-0008 | AUSL
Personal Notes on Remedial Law 2 Review (based on the syllabus of Prof. Henedino M. Brondial)

x A probate court, whether in a testate or intestate proceeding,[22] can only pass x In the light of the foregoing, Section 2, Rule 90 should be interpreted in the
upon questions of title provisionally.[23] The rationale therefor and the proper context of Section 1 of the same Rule, to wit:
recourse of the aggrieved party are expounded in Jimenez v. Court of Appeals: o Section 1. When order for distribution of residue made. When the
o The patent reason is the probate courts limited jurisdiction debts, funeral charges, and expenses of administration, the
and the principle that questions of title or ownership, which allowance to the widow, and inheritance tax, if any, chargeable to
result in inclusion or exclusion from the inventory of the the estate in accordance with law, have been paid, the court, on
property, can only be settled in a separate action. the application of the executor or administrator, or of a person
x All that the said court could do as regards said properties is determine whether interested in the estate, and after hearing upon notice, shall assign
they should or should not be included in the inventory or list of properties to be the residue of the estate to the persons entitled to the same,
administered by the administrator. If there is a dispute as to the ownership, then naming them and the proportions, or parts, to which each is
the opposing parties and the administrator have to resort to an ordinary action entitled, and such person may demand and recover their respective
for a final determination of the conflicting claims of title because the probate shares from the executor or administrator, or any other person
court cannot do so.[24] having the same in his possession. If there is a controversy before
x Further, In Sanchez v. Court of Appeals, we held: the court as to who are the lawful heirs of the deceased person or
o [A] probate court or one in charge of proceedings whether testate as to the distributive shares to which each person is entitled under
or intestate cannot adjudicate or determine title to properties the law, the controversy shall be heard and decided as in ordinary
claimed to be a part of the estate and which are claimed to belong cases.
to outside parties. All that the said court could do as regards said o No distribution shall be allowed until the payment of the obligations
properties is to determine whether they should or should not be above mentioned has been made or provided for, unless the
included in the inventory or list of properties to be administered by distributes, or any of them, give a bond, in a sum to be fixed by
the administrator. If there is no dispute, well and good, but if there the court, conditioned for the payment of said obligations within
is, then the parties, the administrator, and the opposing parties such time as the court directs.
have to resort to an ordinary action for a final determination of the x Based thereon, we find that what the parties and the lower courts have
conflicting claims of title because the probate court cannot do perceived to be as an Order of Collation is nothing more than an order of
so.[25] inclusion in the inventory of the estate which, as we have already
x Guided by the above jurisprudence, it is clear that the Court of Appeals discussed, is an interlocutory order. The motion for collation was filed with
committed an error in considering the assailed Order dated November 11, 1994 the probate court at the early stage of the intestate estate proceedings. We have
as final or binding upon the heirs or third persons who dispute the examined the records of the case and we found no indication that the debts of
inclusion of certain properties in the intestate estate of the deceased the decedents spouses have been paid and the net remainder of the conjugal
Rafael Nicolas. Under the foregoing rulings of the Court, any aggrieved estate have already been determined, and the estates of the deceased spouses
party, or a third person for that matter, may bring an ordinary action for at the time filing of the motion for collation were ready for partition and
a final determination of the conflicting claims. distribution. In other words, the issue on collation is still premature.
x Private respondents reliance on Section 2, Rule 90 of the Rules of Court, to wit: x And even if we consider, en arguendo, that said assailed Order is a collation
o SEC. 2. Questions as to advancement to be determined. Questions order and a final order, still, the same would have no force and effect upon the
as to advancement made, or alleged to have been made, by the parties. It is a hornbook doctrine that a final order is appealable. As such, the
deceased to any heir may be heard and determined by the court Order should have expressed therein clearly and distinctly the facts and the laws
having jurisdiction of the estate proceedings; and the final order of on which it is based as mandated by Section 14, Article VIII of the 1987
the court thereon shall be binding on the person raising the Constitution of the Republic of the Philippines, which provides:
question and on the heir. o SEC. 14. No decision shall be rendered by any court without
x in support of his claim that the assailed Order is a final order and therefore expressing therein clearly and distinctly the facts and the law on
appealable and that due to petitioners failure to appeal in due time, they are which it is based.
now bound by said Order, is not feasible. o No petition for review or motion for reconsideration of a decision of
x What seems to be a conflict between the above-quoted Rule and the the court shall be refused due course or denied without stating the
aforediscussed jurisprudence that the Order in question is an legal basis therefore.
interlocutory and not a final order is more apparent than real. This is x An examination of the subject Order as quoted earlier,[28] readily reveals that
because the questioned Order was erroneously referred to as an order of the presiding Judge failed to comply with the said constitutional mandate. The
collation both by the RTC and the appellate court. For all intents and assailed Order did not state the reasons for ordering the collation of
purposes, said Order is a mere order including the subject properties in the properties enumerated therein. The Order simply directed the
the inventory of the estate of the decedent. inclusion of certain real properties in the estate of the deceased. It did
x The Court held in Valero Vda. de Rodriguez v. Court of Appeals[26] that the not declare that the properties enumerated therein were given to the
order of exclusion (or inclusion) is not a final order; that it is children of the deceased gratuitously, despite the title in the childrens
interlocutory in the sense that it did not settle once and for all the title names or deeds of sale in their favor. Moreover, in his Comment, private
to the subject lots; that the prevailing rule is that for the purpose of respondent makes mention of the testimonies of his witnesses but these were
determining whether a certain property should or should not be not even mentioned in the Order of November 11, 1994. Petitioner would have
included in the inventory, the probate court may pass upon the title been deprived of due process as they would be divested of the opportunity of
thereto but such determination is not conclusive and is subject to the being able to point out in a motion for reconsideration or on appeal, any errors
final decision in a separate action regarding ownership which may be of facts and/or law considering that there were no facts or laws cited in support
instituted by the parties. of the assailed Order of collation. As a final Order, it is, on its face patently null
x In the Rodriguez case, the Court distinguished between an order of collation and void. It could have never become final. A void judgment is not entitled to
and an order of exclusion from or inclusion in the estates inventory, thus: the respect accorded to a valid judgment, but may be entirely disregarded or
o We hold further that the dictum of the Court of Appeals and the declared inoperative by any tribunal in which effect is sought to be given to
probate court that the two disputed lots are not subject to collation it.[29] For it to be considered as a valid final order, the RTC must then first rule
was a supererogation and was not necessary to the disposition of and state in its order whether the properties covered by TCT Nos. T-36734, T-
the case which merely involved the issue of inclusion in, or 36989, T-33658, T-36987, T-40333, T-10907 and the 4,009 square meter lot
exclusion from, the inventory of the testators estate. The issue of were acquired by petitioners from the deceased parents of the parties by
collation was not yet justiciable at that early stage of the testate onerous or gratuitous title; and must specifically state in its order the reasons
proceeding. It is not necessary to mention in the order of exclusion why it ordered the subject properties collated. It is only then that the order of
the controversial matter of collation. collation may be the subject of a motion for reconsideration and/or appeal within
o Whether collation may exist with respect to the two lots and the 15-day reglementary period. Until and unless the constitutional mandate is
whether Mrs. Rustias Torrens titles thereto are indefeasible are complied with, any appeal from said Order would have been premature.
matters that may be raised later or may not be raised at all. How x Either way therefore, whether the Order in question is a final or interlocutory
those issues should be resolved, if and when they are raised, need order, it is a reversible error on the part of the appellate court to rule that the
not be touched upon in the adjudication of this appeal. so-called order of collation dated November 11, 1994 had already attained
x The intestate and testate proceedings for the settlement of the estates of the finality.
deceased Valero spouses were consolidated, as ordered by the lower court on x As to the prayer of petitioners that the RTC be ordered to give due course to
November 21, 1974, so that the conjugal estate of the deceased spouses may be their notice of appeal from the Orders dated November 4, 1996 and December
properly liquidated, as contemplated in section 2, Rule 73 of the Rules of Court 23, 1996 removing petitioner Teresita N. de Leon as administratrix of the estate
and Act No. 3176. of private parties deceased parents,[30] to approve their record on appeal[31]
x We have examined the expedientes of the two cases. We found that the and to elevate the records of Special Proceeding No. C-1679 to the Court of
proceedings have not yet reached the stage when the question of collation or Appeals It is not disputed by the parties that said Orders are appealable. In fact,
advancement to an heir may be raised and decided. The numerous debts of the the Court of Appeals had correctly directed the RTC to give due course to
decedents are still being paid. The net remainder (remanente liquido) of their petitioners appeal and this is not assailed by the private respondent.
conjugal estate has not yet been determined. On the other hand, up to this time, x But, the approval or disapproval of the record on appeal is not a proper subject
no separate action has been brought by the appellants to nullify Mrs. Rustias matter of the present petition for review on certiorari as it is not even a subject-
Torrens titles to the disputed lots or to show that the sale was in reality a matter in CA-G.R. SP No. 42958. Whether or not the record on appeal should be
donation. approved is a matter that is subject to the sound discretion of the RTC, provided
x In this appeal, it is not proper to pass upon the question of collation and to that Sections 6 to 9, Rule 41 of the Rules of Court are observed by appellant.
decide whether Mrs. Rustias titles to the disputed lots are questionable. The x Finally, the elevation of the records of Special Proceedings No. C-1679 to the
proceedings below have not reached the stage of partition and distribution when Court of Appeals for the purpose of petitioners appeal from the order removing
the legitimes of the compulsory heirs have to be determined.[27] the administratrix is unnecessary where a record on appeal is allowed under the
83
RECTO, GAYLE ANGELI M.
2011-0008 | AUSL
Personal Notes on Remedial Law 2 Review (based on the syllabus of Prof. Henedino M. Brondial)

Rules of Court. The court a quo loses jurisdiction over the subject of the appeal o SEC. 2. Contents of petition for letters of administration. - A
upon the approval of the record on appeal and the expiration of the time to petition for letters of administration must be filed by an interested
appeal of the other parties; but retains jurisdiction over the remaining subject person and must show, as far as known to the petitioner: x x x.
matter not covered by the appeal x An "interested person" has been defined as one who would be benefited by the
estate, such as an heir, or one who has a claim against the estate, such as a
EDGAR SAN LUIS, Petitioner, vs. FELICIDAD SAN LUIS, Respondent. creditor. The interest must be material and direct, and not merely indirect or
G.R. No. 133743 February 6, 2007 contingent. 75
x In the instant case, respondent would qualify as an interested person who has a
RODOLFO SAN LUIS, Petitioner, vs. FELICIDAD SAGALONGOS alias FELICIDAD SAN LUIS, direct interest in the estate of Felicisimo by virtue of their cohabitation, the
Respondent. existence of which was not denied by petitioners. If she proves the validity of the
G.R. No. 134029 February 6, 2007 divorce and Felicisimo’s capacity to remarry, but fails to prove that her marriage
THIRD DIVISION with him was validly performed under the laws of the U.S.A., then she may be
considered as a co-owner under Article 144 76 of the Civil Code. This provision
FACTS: governs the property relations between parties who live together as husband and
x Settlement of the estate of Felicisimo San Luis (former gov of Laguna) wife without the benefit of marriage, or their marriage is void from the
x During his lifetime, Felicisimo contracted 3 marriages: beginning. It provides that the property acquired by either or both of them
o (1) Virginia Sulit on Mar 17, 1942 but Virginia predeceased through their work or industry or their wages and salaries shall be governed by
Felicisimo; the rules on co-ownership. In a co-ownership, it is not necessary that the
o (2) May 1, 1968 Merry Lee Corwin, but Merry Lee was able to property be acquired through their joint labor, efforts and industry. Any property
secure a divoce decree; and acquired during the union is prima facie presumed to have been obtained
o (3) June 20, 1974 Felicidad San Luis through their joint efforts. Hence, the portions belonging to the co-owners shall
x Respondent Æ filed a petition for letters of administration before the RTC Makati be presumed equal, unless the contrary is proven. 77
o alleged that she is the widow of Felicisimo; that the decedent’s x Meanwhile, if respondent fails to prove the validity of both the divorce and the
surviving heirs are respondent as legal spouse, his 6 children by his marriage, the applicable provision would be Article 148 of the Family Code which
1st marriage, and son by his 2nd marriage has filled the hiatus in Article 144 of the Civil Code by expressly regulating the
x Petitioner Rodolfo, one of the children of Felicisimo by his 1st marriage Æ filed property relations of couples living together as husband and wife but are
MD on the grounds of failure to state COA and improper venue incapacitated to marry. 78 In Saguid v. Court of Appeals, 79 we held that even if
o Rodolfo claimed that the petition for letters of administration should the cohabitation or the acquisition of property occurred before the Family Code
have been filed in the Province of Laguna because this was took effect, Article 148 governs. 80 The Court described the property regime
Felicisimo’s place of residence prior to his death. under this provision as follows:
o He further claimed that respondent has no legal personality to file o The regime of limited co-ownership of property governing the
the petition because she was only a mistress of Felicisimo since the union of parties who are not legally capacitated to marry each
latter, at the time of his death, was still legally married to Merry other, but who nonetheless live together as husband and wife,
Lee. applies to properties acquired during said cohabitation in proportion
x RTC Æ denied MD to their respective contributions. Co-ownership will only be up to the
x Rodolfo et al Æ filed an MR extent of the proven actual contribution of money, property or
o They asserted that paragraph 2, Article 26 of the Family Code industry. Absent proof of the extent thereof, their contributions and
cannot be given retroactive effect to validate respondent’s corresponding shares shall be presumed to be equal.
bigamous marriage with Felicisimo because this would impair x In the cases of Agapay v. Palang, and Tumlos v. Fernandez, which involved the
vested rights in derogation of Article 256 16 of the Family Code. issue of co-ownership of properties acquired by the parties to a bigamous
x RTC Æ denied MR marriage and an adulterous relationship, respectively, we ruled that proof of
o It ruled that respondent, as widow of the decedent, possessed the actual contribution in the acquisition of the property is essential. x x x
legal standing to file the petition and that venue was properly laid. x As in other civil cases, the burden of proof rests upon the party who, as
x Meanwhile, the motion for disqualification was deemed moot and academic 18 determined by the pleadings or the nature of the case, asserts an affirmative
because then Acting Presiding Judge Santos was substituted by Judge Salvador issue. Contentions must be proved by competent evidence and reliance must be
S. Tensuan pending the resolution of said motion. had on the strength of the party’s own evidence and not upon the weakness of
x Mila (daughter of deceased) Æ filed a motion for inhibition the opponent’s defense. x x x 81
x Judge Tensuan Æ granted the motion x In view of the foregoing, we find that respondent’s legal capacity to file the
x Case was re-raffled subject petition for letters of administration may arise from her status as the
x RTC Æ dismissed the petition for letters of administration surviving wife of Felicisimo or as his co-owner under Article 144 of the Civil Code
o It held that, at the time of his death, Felicisimo was the duly or Article 148 of the Family Code.
elected governor and a resident of the Province of Laguna. Hence,
the petition should have been filed in Sta. Cruz, Laguna and not in ISSUE # 2: Whether the petition for letters of administration was filed in the properly filed in
Makati City. It also ruled that respondent was without legal Makati.
capacity to file the petition for letters of administration because her
marriage with Felicisimo was bigamous, thus, void ab initio. It HELD # 2: YES.
found that the decree of absolute divorce dissolving Felicisimo’s x Under Section 1, 39 Rule 73 of the Rules of Court, the petition for letters of
marriage to Merry Lee was not valid in the Philippines and did not administration of the estate of Felicisimo should be filed in the Regional Trial
bind Felicisimo who was a Filipino citizen. It also ruled that Court of the province "in which he resides at the time of his death." In the case
paragraph 2, Article 26 of the Family Code cannot be retroactively of Garcia Fule v. Court of Appeals, 40 we laid down the doctrinal rule for
applied because it would impair the vested rights of Felicisimo’s determining the residence - as contradistinguished from domicile - of the
legitimate children. decedent for purposes of fixing the venue of the settlement of his estate:
x CA Æ reversed RTC o [T]he term "resides" connotes ex vi termini "actual
o ruled that under Section 1, Rule 73 of the Rules of Court, the term residence" as distinguished from "legal residence or
"place of residence" of the decedent, for purposes of fixing the domicile." This term "resides," like the terms "residing"
venue of the settlement of his estate, refers to the personal, actual and "residence," is elastic and should be interpreted in the
or physical habitation, or actual residence or place of abode of a light of the object or purpose of the statute or rule in
person as distinguished from legal residence or domicile. It noted which it is employed. In the application of venue statutes
that although Felicisimo discharged his functions as governor in and rules - Section 1, Rule 73 of the Revised Rules of
Laguna, he actually resided in Alabang, Muntinlupa. Thus, the Court is of such nature - residence rather than domicile is
petition for letters of administration was properly filed in Makati the significant factor. Even where the statute uses the
City. word "domicile" still it is construed as meaning residence
o the marriage between Felicisimo and Merry Lee was validly and not domicile in the technical sense. Some cases make
dissolved by virtue of the decree of absolute divorce; Therefore, a distinction between the terms "residence" and
under Art 130 of the Family Code, the petitioner as the surviving "domicile" but as generally used in statutes fixing venue,
spouse can institute the judicial proceeding for the settlement of the terms are synonymous, and convey the same meaning
the estate of the deceased as the term "inhabitant." In other words, "resides" should
be viewed or understood in its popular sense, meaning,
ISSUE # 1: Whether respondent Felicidad, assuming that she was never legally married to the personal, actual or physical habitation of a person,
deceased, has the legal capacity to file the subject petition for letters of administration. actual residence or place of abode. It signifies physical
presence in a place and actual stay thereat. In this popular
HELD # 1: YES. sense, the term means merely residence, that is, personal
x Even assuming that Felicisimo was not capacitated to marry respondent in 1974, residence, not legal residence or domicile. Residence
nevertheless, we find that the latter has the legal personality to file the subject simply requires bodily presence as an inhabitant in a given
petition for letters of administration, as she may be considered the co-owner of place, while domicile requires bodily presence in that place
Felicisimo as regards the properties that were acquired through their joint efforts and also an intention to make it one’s domicile. No
during their cohabitation. particular length of time of residence is required though;
x Section 6, 74 Rule 78 of the Rules of Court states that letters of administration however, the residence must be more than temporary. 41
may be granted to the surviving spouse of the decedent. However, Section 2, (Emphasis supplied)
Rule 79 thereof also provides in part:
84
RECTO, GAYLE ANGELI M.
2011-0008 | AUSL
Personal Notes on Remedial Law 2 Review (based on the syllabus of Prof. Henedino M. Brondial)

x It is incorrect for petitioners to argue that "residence," for purposes of fixing the x (3) he does not possess the desire to
venue of the settlement of the estate of Felicisimo, is synonymous with earn. They claimed that the best
"domicile." The rulings in Nuval and Romualdez are inapplicable to the instant interests of the estate dictate that
case because they involve election cases. Needless to say, there is a distinction Joseph be appointed as special or
between "residence" for purposes of election laws and "residence" for purposes regular administrator.
of fixing the venue of actions. In election cases, "residence" and "domicile" are x RTC Æ issued a resolution appointing Eduardo as regular administrator of
treated as synonymous terms, that is, the fixed permanent residence to which Joaquin’s estate
when absent, one has the intention of returning. 42 However, for purposes of x Abelardo Dagoro Æ filed an answer in intervention
fixing venue under the Rules of Court, the "residence" of a person is his o alleging that Mercedes is survived not only by her daughter Cecile,
personal, actual or physical habitation, or actual residence or place of abode, but also by him as her husband. He also averred that there is a
which may not necessarily be his legal residence or domicile provided he resides need to appoint a special administrator to the estate, but claimed
therein with continuity and consistency. 43 Hence, it is possible that a person that Eduardo is not the person best qualified for the task.
may have his residence in one place and domicile in another. x RTC Æ issued an Order of Partition
x In the instant case, while petitioners established that Felicisimo was domiciled in o In the light of the filing by the heirs of their respective proposed
Sta. Cruz, Laguna, respondent proved that he also maintained a residence in projects of partition and the payment of inheritance taxes due the
Alabang, Muntinlupa from 1982 up to the time of his death. Respondent estate as early as 1965, and there being no claim in Court against
submitted in evidence the Deed of Absolute Sale 44 dated January 5, 1983 the estate of the deceased, the estate of JOAQUIN AGTARAP is
showing that the deceased purchased the aforesaid property. She also now consequently - ripe - for distribution among the heirs minus
presented billing statements 45 from the Philippine Heart Center and Chinese the surviving spouse Caridad Garcia who died on Aug 25, 1999.
General Hospital for the period August to December 1992 indicating the address o Considering that the bulk of the estate property were acquired
of Felicisimo at "100 San Juanico, Ayala Alabang, Muntinlupa." Respondent also during the existence of the second marriage, the greater part of
presented proof of membership of the deceased in the Ayala Alabang Village the estate is perforce accounted by the second marriage and the
Association 46 and Ayala Country Club, Inc., 47 letter-envelopes 48 from 1988 compulsory heirs thereunder (with a total value of P14,177,500
to 1990 sent by the deceased’s children to him at his Alabang address, and the x Eduardo Æ rendered accounting
deceased’s calling cards 49 stating that his home/city address is at "100 San o The accounting report included the income earned and received for
Juanico, Ayala Alabang Village, Muntinlupa" while his office/provincial address is the period and the expenses incurred in the administration,
in "Provincial Capitol, Sta. Cruz, Laguna." sustenance and allowance of the widow.
x From the foregoing, we find that Felicisimo was a resident of Alabang, x RTC Æ issued an order
Muntinlupa for purposes of fixing the venue of the settlement of his estate. o WHEREFORE, the net assets of the estate of the late JOAQUIN
Consequently, the subject petition for letters of administration was validly filed AGTARAP with a total value of P14,177,500.00, together with
in the Regional Trial Court 50 which has territorial jurisdiction over Alabang, whatever interest from bank deposits and all other incomes or
Muntinlupa. The subject petition was filed on December 17, 1993. At that time, increments thereof accruing after the Accounting Report of
Muntinlupa was still a municipality and the branches of the Regional Trial Court December 31, 1996, after deducting therefrom the compensation of
of the National Capital Judicial Region which had territorial jurisdiction over the administrator and other expenses allowed by the Court, are
Muntinlupa were then seated in Makati City as per Supreme Court Administrative hereby ordered distributed as follows ---
Order No. 3. 51 Thus, the subject petition was validly filed before the Regional x CA Æ affirmed RTC
Trial Court of Makati City. o The two (2) properties, together with their improvements,
embraced by TCT No. 38254 and TCT No. 38255, respectively, are
first to be distributed among the following:
ƒ Lucia Mendietta - ½ of the property. But since she is
deceased, her share shall be inherited by Joaquin,
Jesus, Milagros and Jose in equal shares.
ƒ Joaquin Agtarap - ½ of the property and ¼ of the
other half of the property which pertains to Lucia
Mendietta’s share.
EDUARDO G. AGTARAP, Petitioner, vs. SEBASTIAN AGTARAP, JOSEPH AGTARAP, ƒ Jesus Agtarap - ¼ of Lucia Mendietta’s share. But
TERESA AGTARAP, WALTER DE SANTOS, and ABELARDO DAGORO, Respondents. since he is already deceased (and died without issue),
G.R. No. 177099 June 8, 2011 his inheritance shall, in turn, be acquired by Joaquin
Agtarap.
SEBASTIAN G. AGTARAP, Petitioner, vs. EDUARDO G. AGTARAP, JOSEPH AGTARAP, TERESA ƒ Milagros Agtarap - ¼ of Lucia Mendietta’s share. But
AGTARAP, WALTER DE SANTOS, and ABELARDO DAGORO, Respondents. since she died in 1996 without issue, 5/8 of her
G.R. No. 177192 inheritance shall be inherited by Gloria (represented
SECOND DIVISION by her husband Walter de Santos and her daughter
Samantha), Joseph Agtarap and Teresa Agtarap, (in
FACTS: representation of Milagros’ brother Jose Agtarap) and
x Eduardo Æ filed a verified petition for the judicial settlement of the estate of his 1/8 each shall be inherited by Mercedes (represented
deceased father Joaquin by her husband Abelardo Dagoro and her daughter
o alleged that Joaquin died intestate on Nov 21, 1964; Cecile), Sebastian Eduardo, all surnamed Agtarap.
o During his lifetime, Joaquin contracted two marriages, ƒ Jose Agtarap - ¼ of Lucia Mendietta’s share. But
ƒ first with Lucia Garcia since he died in 1967, his inheritance shall be
x Lucia died on April 24, 1924 (3 children) acquired by his wife Priscilla, and children Gloria
ƒ second with Caridad Garcia (represented by her husband Walter de Santos and
x Joaquin married Caridad on Feb 9, 1926 her daughter Samantha), Joseph Agtarap and Teresa
(3 children) in equal shares.
o At the time of his death, Joaquin left 2 parcels of land with o Then, Joaquin Agtarap’s estate, comprising three-fourths (3/4) of
improvements the subject properties and its improvements, shall be distributed as
o Joseph, a grandson of Joaquin, had been leasing and improving the follows:
said realties and had been appropriating for himself P26K per ƒ Caridad Garcia - 1/6 of the estate. But since she died
month since April 1994 in 1999, her share shall be inherited by her children
o Eduardo further alleged that there was an imperative need to namely Mercedes Agtarap (represented by her
appoint him as special administrator to take possession and charge husband Abelardo Dagoro and her daughter Cecilia),
of the estate assets and their civil fruits, pending the appointment of Sebastian Agtarap and Eduardo Agtarap in their own
a regular administrator. right, dividing the inheritance in equal shares.
x Sebastian Æ filed Comment ƒ Milagros Agtarap - 1/6 of the estate. But since she
o generally admitting the allegations in the petition, and conceding to died in 1996 without issue, 5/8 of her inheritance
the appointment of Eduardo as special administrator. shall be inherited by Gloria (represented by her
x Joseph, Gloria, and Teresa Æ filed their answer/opposition husband Walter de Santos and her daughter
o They alleged that the two subject lots belong to the conjugal Samantha), Joseph Agtarap and Teresa Agtarap, (in
partnership of Joaquin with Lucia, and that, upon Lucia’s death in representation of Milagros’ brother Jose Agtarap) and
April 1924, they became the pro indiviso owners of the subject 1/8 each shall be inherited by Mercedes (represented
properties. They said that their residence was built with the by her husband Abelardo Dagoro and her daughter
exclusive money of their late father Jose, and the expenses of the Cecile), Sebastian and Eduardo, all surnamed
extensions to the house were shouldered by Gloria and Teresa, Agtarap.
while the restaurant (Manong’s Restaurant) was built with the o Jose Agtarap - 1/6 of the estate. But since he died in 1967, his
exclusive money of Joseph and his business partner. inheritance shall be acquired by his wife Priscilla, and children
o They opposed the appointment of Eduardo as administrator on the Gloria (represented by her husband Walter de Santos and her
following grounds: daughter Samantha), Joseph Agtarap and Teresa Agtarap in equal
ƒ (1) he is not physically and mentally fit to do so; shares.
ƒ (2) his interest in the lots is minimal; and

85
RECTO, GAYLE ANGELI M.
2011-0008 | AUSL
Personal Notes on Remedial Law 2 Review (based on the syllabus of Prof. Henedino M. Brondial)

o Mercedes Agtarap - 1/6 of the estate. But since she died in 1984, Mendiet[t]a was crossed out and replaced by en segundas nuptias con Caridad
her inheritance shall be acquired by her husband Abelardo Dagoro Garcia, referring to the second marriage of Joaquin to Caridad. It cannot be
and her daughter Cecile in equal shares. gainsaid, therefore, that prior to the replacement of Caridad’s name in TCT No.
o Sebastian Agtarap - 1/6 of the estate. 32184, Lucia, upon her demise, already left, as her estate, one-half (1/2)
o Eduardo Agtarap - 1/6 of the estate. conjugal share in TCT No. 32184. Lucia’s share in the property covered by the
x Sebastian and Eduardo Æ filed their respective MRs said TCT was carried over to the properties covered by the certificates of title
x CA Æ denied derivative of TCT No. 32184, now TCT Nos. 38254 and 38255. And as found by
both the RTC and the CA, Lucia was survived by her compulsory heirs - Joaquin,
ISSUE # 1: Whether the RTC was correct in determining ownership over the subject real Jesus, Milagros, and Jose.
properties. x Section 2, Rule 73 of the Rules of Court provides that when the
marriage is dissolved by the death of the husband or the wife, the
HELD # 1: YES. community property shall be inventoried, administered, and liquidated,
x As to Sebastian’s and Eduardo’s common issue on the ownership of the subject and the debts thereof paid; in the testate or intestate proceedings of
real properties, we hold that the RTC, as an intestate court, had jurisdiction to the deceased spouse, and if both spouses have died, the conjugal
resolve the same. partnership shall be liquidated in the testate or intestate proceedings
x The general rule is that the jurisdiction of the trial court, either as a probate or of either. Thus, the RTC had jurisdiction to determine whether the properties
an intestate court, relates only to matters having to do with the probate of the are conjugal as it had to liquidate the conjugal partnership to determine the
will and/or settlement of the estate of deceased persons, but does not extend to estate of the decedent. In fact, should Joseph and Teresa institute a settlement
the determination of questions of ownership that arise during the proceedings.15 proceeding for the intestate estate of Lucia, the same should be consolidated
The patent rationale for this rule is that such court merely exercises special and with the settlement proceedings of Joaquin, being Lucia’s spouse.24 Accordingly,
limited jurisdiction.16 As held in several cases,17 a probate court or one in the CA correctly distributed the estate of Lucia, with respect to the properties
charge of estate proceedings, whether testate or intestate, cannot adjudicate or covered by TCT Nos. 38254 and 38255 subject of this case, to her compulsory
determine title to properties claimed to be a part of the estate and which are heirs.
claimed to belong to outside parties, not by virtue of any right of inheritance x Therefore, in light of the foregoing evidence, as correctly found by the RTC and
from the deceased but by title adverse to that of the deceased and his estate. All the CA, the claim of Sebastian and Eduardo that TCT Nos. 38254 and 38255
that the said court could do as regards said properties is to determine whether or conclusively show that the owners of the properties covered therein were
not they should be included in the inventory of properties to be administered by Joaquin and Caridad by virtue of the registration in the name of Joaquin Agtarap
the administrator. If there is no dispute, there poses no problem, but if there is, casado con (married to) Caridad Garcia, deserves scant consideration. This
then the parties, the administrator, and the opposing parties have to resort to an cannot be said to be a collateral attack on the said TCTs. Indeed, simple
ordinary action before a court exercising general jurisdiction for a final possession of a certificate of title is not necessarily conclusive of a holder’s true
determination of the conflicting claims of title. ownership of property.25 A certificate of title under the Torrens system aims to
x However, this general rule is subject to exceptions as justified by protect dominion; it cannot be used as an instrument for the deprivation of
expediency and convenience. ownership.26 Thus, the fact that the properties were registered in the name of
o First, the probate court may provisionally pass upon in an Joaquin Agtarap, married to Caridad Garcia, is not sufficient proof that the
intestate or a testate proceeding the question of inclusion properties were acquired during the spouses’ coverture.27 The phrase "married
in, or exclusion from, the inventory of a piece of property to Caridad Garcia" in the TCTs is merely descriptive of the civil status of Joaquin
without prejudice to the final determination of ownership as the registered owner, and does not necessarily prove that the realties are
in a separate action.18 their conjugal properties.28
o Second, if the interested parties are all heirs to the estate, x Neither can Sebastian’s claim that Joaquin’s estate could have already been
or the question is one of collation or advancement, or the settled in 1965 after the payment of the inheritance tax be upheld. Payment of
parties consent to the assumption of jurisdiction by the the inheritance tax, per se, does not settle the estate of a deceased person. As
probate court and the rights of third parties are not provided in Section 1, Rule 90 of the Rules of Court—
impaired, then the probate court is competent to resolve o SECTION 1. When order for distribution of residue made. -- When
issues on ownership.19 the debts, funeral charges, and expenses of administration, the
ƒ Verily, its jurisdiction extends to matters allowance to the widow, and inheritance tax, if any, chargeable to
incidental or collateral to the settlement and the estate in accordance with law, have been paid, the court, on
distribution of the estate, such as the the application of the executor or administrator, or of a person
determination of the status of each heir and interested in the estate, and after hearing upon notice, shall assign
whether the property in the inventory is the residue of the estate to the persons entitled to the same,
conjugal or exclusive property of the deceased naming them and the proportions, or parts, to which each is
spouse.20 entitled, and such persons may demand and recover their
x We hold that the general rule does not apply to the instant case considering that respective shares from the executor or administrator, or any other
the parties are all heirs of Joaquin and that no rights of third parties will be person having the same in his possession. If there is a controversy
impaired by the resolution of the ownership issue. More importantly, the before the court as to who are the lawful heirs of the deceased
determination of whether the subject properties are conjugal is but collateral to person or as to the distributive share to which each person is
the probate court’s jurisdiction to settle the estate of Joaquin. entitled under the law, the controversy shall be heard and decided
x It should be remembered that when Eduardo filed his verified petition for judicial as in ordinary cases.
settlement of Joaquin’s estate, he alleged that the subject properties were x No distribution shall be allowed until the payment of the obligations above
owned by Joaquin and Caridad since the TCTs state that the lots were registered mentioned has been made or provided for, unless the distributees, or any of
in the name of Joaquin Agtarap, married to Caridad Garcia. He also admitted in them, give a bond, in a sum to be fixed by the court, conditioned for the
his petition that Joaquin, prior to contracting marriage with Caridad, contracted a payment of said obligations within such time as the court directs.
first marriage with Lucia. Oppositors to the petition, Joseph and Teresa, x Thus, an estate is settled and distributed among the heirs only after the payment
however, were able to present proof before the RTC that TCT Nos. 38254 and of the debts of the estate, funeral charges, expenses of administration,
38255 were derived from a mother title, TCT No. 5239, dated March 17, 1920, in allowance to the widow, and inheritance tax. The records of these cases do not
the name of FRANCISCO VICTOR BARNES Y JOAQUIN AGTARAP, el primero show that these were complied with in 1965.
casado con Emilia Muscat, y el Segundo con Lucia Garcia Mendietta (FRANCISCO x As regards the issue raised by Sebastian on the legitimacy of Joseph and Teresa,
VICTOR BARNES y JOAQUIN AGTARAP, the first married to Emilia Muscat, and suffice it to say that both the RTC and the CA found them to be the legitimate
the second married to Lucia Garcia Mendietta).21 When TCT No. 5239 was children of Jose. The RTC found that Sebastian did not present clear and
divided between Francisco Barnes and Joaquin Agtarap, TCT No. 10864, in the convincing evidence to support his averments in his motion to exclude them as
name of Joaquin Agtarap, married to Lucia Garcia Mendietta, was issued for a heirs of Joaquin, aside from his negative allegations. The RTC also noted the fact
parcel of land, identified as Lot No. 745 of the Cadastral Survey of Pasay, of Joseph and Teresa being the children of Jose was never questioned by
Cadastral Case No. 23, G.L.R.O. Cadastral Record No. 1368, consisting of 8,872 Sebastian and Eduardo, and the latter two even admitted this in their petitions,
square meters. This same lot was covered by TCT No. 5577 (32184)22 issued on as well as in the stipulation of facts in the August 21, 1995 hearing.29
April 23, 1937, also in the name of Joaquin Agtarap, married to Lucia Garcia Furthermore, the CA affirmed this finding of fact in its November 21, 2006
Mendietta. Decision.30
x The findings of the RTC and the CA show that Lucia died on April 24, 1924, and x Also, Sebastian’s insistence that Abelardo Dagoro and Walter de Santos are not
subsequently, on February 9, 1926, Joaquin married Caridad. It is worthy to note heirs to the estate of Joaquin cannot be sustained. Per its October 23, 2000
that TCT No. 5577 (32184) contained an annotation, which reads— Order of Partition, the RTC found that Gloria Agtarap de Santos died on May 4,
o Ap-4966 - NOTA: Se ha enmendado el presente certificado de 1995, and was later substituted in the proceedings below by her husband Walter
titulo, tal como aparece, tanchando las palabras "con Lucia Garcia de Santos. Gloria begot a daughter with Walter de Santos, Georgina Samantha
Mendiet[t]a" y poniendo en su lugar, entre lineas y en tinta de Santos. The RTC likewise noted that, on September 16, 1995, Abelardo
encarnada, las palabras "en segundas nupcias con Caridad Garcia", Dagoro filed a motion for leave of court to intervene, alleging that he is the
en complimiento de un orden de fecha 28 de abril de 1937, dictada surviving spouse of Mercedes Agtarap and the father of Cecilia Agtarap Dagoro,
por el Hon. Sixto de la Costa, juez del Juzgado de Primera Instancia and his answer in intervention. The RTC later granted the motion, thereby
de Rizal, en el expediente cadastal No. 23, G.L.R.O. Cad. Record admitting his answer on October 18, 1995.31 The CA also noted that, during the
No. 1368; copia de cual orden has sido presentada con el No. 4966 hearing of the motion to intervene on October 18, 1995, Sebastian and Eduardo
del Libro Diario, Tomo 6.0 y, archivada en el Legajo T-No. 32184. did not interpose any objection when the intervention was submitted to the RTC
o Pasig, Rizal, a 29 abril de 1937.23 for resolution.32
x Thus, per the order dated April 28, 1937 of Hon. Sixto de la Costa, presiding x Indeed, this Court is not a trier of facts, and there appears no compelling reason
judge of the Court of First Instance of Rizal, the phrase con Lucia Garcia to hold that both courts erred in ruling that Joseph, Teresa, Walter de Santos,
86
RECTO, GAYLE ANGELI M.
2011-0008 | AUSL
Personal Notes on Remedial Law 2 Review (based on the syllabus of Prof. Henedino M. Brondial)

and Abelardo Dagoro rightfully participated in the estate of Joaquin. It was wife, or next of kin, requests to have appointed, if
incumbent upon Sebastian to present competent evidence to refute his and competent and willing to serve;
Eduardo’s admissions that Joseph and Teresa were heirs of Jose, and thus ƒ (b) If such surviving husband or wife, as the case
rightful heirs of Joaquin, and to timely object to the participation of Walter de may be, or next of kin, or the person selected by
Santos and Abelardo Dagoro. Unfortunately, Sebastian failed to do so. them, be incompetent or unwilling, or if the husband
Nevertheless, Walter de Santos and Abelardo Dagoro had the right to participate or widow, or next of kin, neglects for thirty (30) days
in the estate in representation of the Joaquin’s compulsory heirs, Gloria and after the death of the person to apply for
Mercedes, respectively.33 administration or to request that administration be
x This Court also differs from Eduardo’s asseveration that the CA erred in settling, granted to some other person, it may be granted to
together with Joaquin’s estate, the respective estates of Lucia, Jesus, Jose, one or more of the principal creditors, if competent
Mercedes, and Gloria. A perusal of the November 21, 2006 CA Decision would and willing to serve;
readily show that the disposition of the properties related only to the settlement ƒ (c) If there is not such creditor competent and willing
of the estate of Joaquin. Pursuant to Section 1, Rule 90 of the Rules of Court, as to serve, it may be granted to such other person as
cited above, the RTC was specifically granted jurisdiction to determine who are the court may select.
the lawful heirs of Joaquin, as well as their respective shares after the payment x Textually, the rule lists a sequence to be observed, an order of
of the obligations of the estate, as enumerated in the said provision. The preference, in the appointment of an administrator. This order of
inclusion of Lucia, Jesus, Jose, Mercedes, and Gloria in the distribution of the preference, which categorically seeks out the surviving spouse, the
shares was merely a necessary consequence of the settlement of Joaquin’s next of kin and the creditors in the appointment of an administrator,
estate, they being his legal heirs. has been reinforced in jurisprudence.8
x The paramount consideration in the appointment of an administrator
ISSUE # 2: Whether RTC acquired jurisdiction over the estate of Milagros Agtarap and was over the estate of a decedent is the prospective administrator’s
correct in distributing the said estate, considering the allegation with respect to the existence interest in the estate.9 This is the same consideration which Section 6, Rule
of her will. 78 takes into account in establishing the order of preference in the appointment
of administrator for the estate. The rationale behind the rule is that those who
will reap the benefit of a wise, speedy and economical administration of the
HELD # 2: NO. DOCTRINE OF PRECEDENCE OF TESTATE PROCEEDINGS OVER INTESTATE estate, or, in the alternative, suffer the consequences of waste, improvidence or
PROCEEDINGS. mismanagement, have the highest interest and most influential motive to
x However, we agree with Eduardo’s position that the CA erred in distributing administer the estate correctly.10 In all, given that the rule speaks of an
Joaquin’s estate pertinent to the share allotted in favor of Milagros. Eduardo was order of preference, the person to be appointed administrator of a
able to show that a separate proceeding was instituted for the probate of the will decedent’s estate must demonstrate not only an interest in the estate,
allegedly executed by Milagros before the RTC, Branch 108, Pasay City.34 While but an interest therein greater than any other candidate.
there has been no showing that the alleged will of Milagros, x To illustrate, the preference bestowed by law to the surviving spouse in the
bequeathing all of her share from Joaquin’s estate in favor of Eduardo, administration of a decedent’s estate presupposes the surviving spouse’s interest in
has already been probated and approved, prudence dictates that this the conjugal partnership or community property forming part of the decedent’s
Court refrain from distributing Milagros’ share in Joaquin’s estate. estate.11 Likewise, a surviving spouse is a compulsory heir of a decedent12
x It is also worthy to mention that Sebastian died on January 15, 2010, per his which evinces as much, if not more, interest in administering the entire estate of a
Certificate of Death.35 He is survived by his wife Teresita B. Agtarap (Teresita) decedent, aside from her share in the conjugal partnership or absolute
and his children Joaquin Julian B. Agtarap (Joaquin Julian) and Ana Ma. Agtarap community property.
Panlilio (Ana Ma.). x It is to this requirement of observation of the order of preference in the
x Henceforth, in light of the foregoing, the assailed November 21, 2006 Decision appointment of administrator of a decedent’s estate, that the appointment of co-
and the March 27, 2007 Resolution of the CA should be affirmed with administrators has been allowed, but as an exception. We again refer to Section
modifications such that the share of Milagros shall not yet be distributed until 6(a) of Rule 78 of the Rules of Court which specifically states that letters of
after the final determination of the probate of her purported will, and that administration may be issued to both the surviving spouse and the next of kin. In
Sebastian shall be represented by his compulsory heirs. addition and impliedly, we can refer to Section 2 of Rule 82 of the Rules of Court
which say that "x x x when an executor or administrator dies, resigns, or is
removed, the remaining executor or administrator may administer the trust
EMILIO A.M. SUNTAY III, Petitioner, vs. ISABEL COJUANGCO-SUNTAY, Respondent alone, x x x."
G.R. No. 183053 October 10, 2012 x In a number of cases, we have sanctioned the appointment of more than one
Special Second Division administrator for the benefit of the estate and those interested therein.13 We
recognized that the appointment of administrator of the estate of a decedent or
FACTS: the determination of a person’s suitability for the office of judicial administrator
x This is a Motion for Reconsideration filed by respondent of SC’s decision in G.R. rests, to a great extent, in the sound judgment of the court exercising the power
No. 183053 dated June 16, 2010, directing the issuance of joint letters of of appointment.14
administration to both petitioner and respondent. x Under certain circumstances and for various reasons well-settled in Philippine
x Respondent ´ pleads for sole administratorship based on her status as a and American jurisprudence, we have upheld the appointment of co-
legitimate grandchild of decedent Cristina administrators: (1) to have the benefits of their judgment and perhaps at all
o Contends that the explicit provisions of Sec. 6, Rule 78 of the Rules times to have different interests represented;15 (2) where justice and equity
of Court on the order of preference for the issuance of letters of demand that opposing parties or factions be represented in the management of
administration cannot be ignored and that Article 992 of the Civil the estate of the deceased; (3) where the estate is large or, from any cause, an
Code must be followed intricate and perplexing one to settle;16 (4) to have all interested persons
o Asserts that Emilio III had demonstrated adverse interests and satisfied and the representatives to work in harmony for the best interests of the
disloyalty to the estate, thus, he does not deserve to become a co- estate;17 and when a person entitled to the administration of an estate desires
administrator thereof to have another competent person associated with him in the office.18
o Bewails that: (1) Emilio III is an illegitimate grandchild and x In the frequently cited Matias v. Gonzales, we dwelt on the appointment of
therefore, not an heir of the decedent; (2) corollary thereto, Emilio special co-administrators during the pendency of the appeal for the probate of
III, not being a “next of kin” of the decedent, has no interest in the the decedent’s will. Pending the probate thereof, we recognized Matias’ special
estate to justify his appointment as administrator thereof interest in the decedent’s estate as universal heir and executrix designated in the
instrument who should not be excluded in the administration thereof. Thus, we
held that justice and equity demands that the two (2) factions among the non-
compulsory heirs of the decedent, consisting of an instituted heir (Matias) and
ISSUE: Whether Emilio III may still qualify as administrator of his grandmother’s estate, intestate heirs (respondents thereat), should be represented in the management
considering that he is an illegitimate child. of the decedent’s estate.19
x Another oft-cited case is Vda. de Dayrit v. Ramolete, where we held that
HELD: NO. "inasmuch as petitioner-wife owns one-half of the conjugal properties and that
x To begin with, the case at bar reached us on the issue of who, as between Emilio
she, too, is a compulsory heir of her husband, to deprive her of any hand in the
III and Isabel, is better qualified to act as administrator of the decedent’s estate. administration of the estate prior to the probate of the will would be unfair to her
We did not choose. Considering merely his demonstrable interest in the subject proprietary interests."20
estate, we ruled that Emilio III should likewise administer the estate of his x Hewing closely to the aforementioned cases is our ruling in Ventura v. Ventura21
illegitimate grandmother, Cristina, as a co-administrator. In the context of this where we allowed the appointment of the surviving spouse and legitimate
case, we have to make a choice and therefore, reconsider our decision of 16 children of the decedent as co-administrators. However, we drew a distinction
June 2010. between the heirs categorized as next of kin, the nearest of kin in the category
x The general rule in the appointment of administrator of the estate of a decedent being preferred, thus:
is laid down in Section 6, Rule 78 of the Rules of Court: o In the case at bar, the surviving spouse of the deceased Gregorio
o SEC. 6. When and to whom letters of administration granted. - If Ventura is Juana Cardona while the next of kin are: Mercedes and
no executor is named in the will, or the executor or executors are Gregoria Ventura and Maria and Miguel Ventura. The "next of kin"
incompetent, refuse the trust, or fail to give bond, or a person dies has been defined as those persons who are entitled under the
intestate, administration shall be granted: statute of distribution to the decedent’s property (citations
ƒ (a) To the surviving husband or wife, as the case may
omitted). It is generally said that "the nearest of kin, whose
be, or next of kin, or both, in the discretion of the interest in the estate is more preponderant, is preferred in the
court, or to such person as such surviving husband or choice of administrator. ‘Among members of a class the strongest
87
RECTO, GAYLE ANGELI M.
2011-0008 | AUSL
Personal Notes on Remedial Law 2 Review (based on the syllabus of Prof. Henedino M. Brondial)

ground for preference is the amount or preponderance of interest. necessarily related to the demand for representation being
As between next of kin, the nearest of kin is to be preferred.’" repeatedly urged by respondents.26 (Emphasis supplied)
(citations omitted) x In Gabriel v. Court of Appeals, we unequivocally declared the mandatory
o As decided by the lower court and sustained by the Supreme Court, character of the rule on the order of preference for the issuance of letters of
Mercedes and Gregoria Ventura are the legitimate children of administration:
Gregorio Ventura and his wife, the late Paulina Simpliciano. o Evidently, the foregoing provision of the Rules prescribes
Therefore, as the nearest of kin of Gregorio Ventura, they are the order of preference in the issuance of letters of
entitled to preference over the illegitimate children of Gregorio administration, it categorically seeks out the surviving
Ventura, namely: Maria and Miguel Ventura. Hence, under the spouse, the next of kin and the creditors, and requires that
aforestated preference provided in Section 6 of Rule 78, the person sequence to be observed in appointing an administrator. It
or persons to be appointed administrator are Juana Cardona, as the would be a grave abuse of discretion for the probate court
surviving spouse, or Mercedes and Gregoria Ventura as nearest of to imperiously set aside and insouciantly ignore that
kin, or Juana Cardona and Mercedes and Gregoria Ventura in the directive without any valid and sufficient reason
discretion of the Court, in order to represent both interests.22 therefor.27
(Emphasis supplied) x Subsequently, in Angeles v. Angeles-Maglaya,28 we expounded on the legal
x In Silverio, Sr. v. Court of Appeals,23 we maintained that the order of contemplation of a "next of kin," thus:
preference in the appointment of an administrator depends on the o Finally, it should be noted that on the matter of appointment of
attendant facts and circumstances. In that case, we affirmed the legitimate administrator of the estate of the deceased, the surviving spouse is
child’s appointment as special administrator, and eventually as regular preferred over the next of kin of the decedent. When the law
administrator, of the decedent’s estate as against the surviving spouse who the speaks of "next of kin," the reference is to those who are
lower court found unsuitable. Reiterating Sioca v. Garcia24 as good law, we entitled, under the statute of distribution, to the decedent's
pointed out that unsuitableness for appointment as administrator may consist in property; one whose relationship is such that he is entitled
adverse interest of some kind or hostility to those immediately interested in the to share in the estate as distributed, or, in short, an heir. In
estate. resolving, therefore, the issue of whether an applicant for
x In Valarao v. Pascual,25 we see another story with a running theme of heirs letters of administration is a next of kin or an heir of the
squabbling over the estate of a decedent. We found no reason to set aside the decedent, the probate court perforce has to determine and
probate court’s refusal to appoint as special co-administrator Diaz, even if he had pass upon the issue of filiation. A separate action will only
a demonstrable interest in the estate of the decedent and represented one of the result in a multiplicity of suits. Upon this consideration, the trial
factions of heirs, because the evidence weighed by the probate court pointed to court acted within bounds when it looked into and passed upon the
Diaz’s being remiss in his previous duty as co-administrator of the estatein the claimed relationship of respondent to the late Francisco Angeles.29
early part of his administration. Surveying the previously discussed cases of x Finally, in Uy v. Court of Appeals,30 we took into consideration the size of, and
Matias, Corona, and Vda. de Dayrit, we clarified, thus: benefits to, the estate should respondent therein be appointed as co-
o Respondents cannot take comfort in the cases of Matias v. administrator. We emphasized that where the estate is large or, from any cause,
Gonzales, Corona v. Court of Appeals, and Vda. de Dayrit v. an intricate and perplexing one to settle, the appointment of co-administrators
Ramolete, cited in the assailed Decision. Contrary to their claim, may be sanctioned by law.
these cases do not establish an absolute right demandable from the x In our Decision under consideration, we zeroed in on Emilio III’s
probate court to appoint special co-administrators who would demonstrable interest in the estate and glossed over the order of
represent the respective interests of squabbling heirs. Rather, the preference set forth in the Rules. We gave weight to Emilio III’s
cases constitute precedents for the authority of the probate court demonstrable interest in Cristina’s estate and without a closer scrutiny
to designate not just one but also two or more special co- of the attendant facts and circumstances, directed co-administration
administrators for a single estate. Now whether the probate court thereof. We are led to a review of such position by the foregoing survey of
exercises such prerogative when the heirs are fighting among cases.
themselves is a matter left entirely to its sound discretion. x The collected teaching is that mere demonstration of interest in the estate to be
o Furthermore, the cases of Matias, Corona and Vda. de Dayrit hinge settled does not ipso facto entitle an interested person to co-administration
upon factual circumstances other than the incompatible interests of thereof. Neither does squabbling among the heirs nor adverse interests
the heirs which are glaringly absent from the instant case. In necessitate the discounting of the order of preference set forth in Section 6, Rule
Matias this Court ordered the appointment of a special co- 78. Indeed, in the appointment of administrator of the estate of a deceased
administrator because of the applicant's status as the universal heir person, the principal consideration reckoned with is the interest in said estate of
and executrix designated in the will, which we considered to be a the one to be appointed as administrator.31 Given Isabel’s unassailable interest
"special interest" deserving protection during the pendency of the in the estate as one of the decedent’s legitimate grandchildren and undoubted
appeal. Quite significantly, since the lower court in Matias had nearest "next of kin," the appointment of Emilio III as co-administrator of the
already deemed it best to appoint more than one special same estate, cannot be a demandable right. It is a matter left entirely to the
administrator, we found grave abuse of discretion in the act of the sound discretion of the Court32 and depends on the facts and the attendant
lower court in ignoring the applicant's distinctive status in the circumstances of the case.33
selection of another special administrator. x Thus, we proceed to scrutinize the attendant facts and circumstances of this case
o In Corona we gave "highest consideration" to the even as we reiterate Isabel’s and her sibling’s apparent greater interest in the
"executrix's choice of Special Administrator, considering estate of Cristina.
her own inability to serve and the wide latitude of x These considerations do not warrant the setting aside of the order of
discretion given her by the testatrix in her will," for this preference mapped out in Section 6, Rule 78 of the Rules of Court.
Court to compel her appointment as special co-administrator. It is They compel that a choice be made of one over the other.
also manifest from the decision in Corona that the presence of o 1. The bitter estrangement and long-standing animosity
conflicting interests among the heirs therein was not per se the key between Isabel, on the one hand, and Emilio III, on the other,
factor in the designation of a second special administrator as this traced back from the time their paternal grandparents were alive,
fact was taken into account only to disregard or, in the words of which can be characterized as adverse interest of some kind by, or
Corona, to "overshadow" the objections to the appointment on hostility of, Emilio III to Isabel who is immediately interested in the
grounds of "impracticality and lack of kinship." estate;
o Finally in Vda. de Dayrit we justified the designation of the wife of o 2. Corollary thereto, the seeming impossibility of Isabel and
the decedent as special co-administrator because it was "our Emilio III working harmoniously as co-administrators may
considered opinion that inasmuch as petitioner-wife owns result in prejudice to the decedent’s estate, ultimately delaying
one-half of the conjugal properties and that she, too, is a settlement thereof; and
compulsory heir of her husband, to deprive her of any hand o 3. Emilio III, for all his claims of knowledge in the
in the administration of the estate prior to the probate of management of Cristina’s estate, has not looked after the
the will would be unfair to her proprietary interests." The estate’s welfare and has acted to the damage and prejudice
special status of a surviving spouse in the special administration of thereof.
an estate was also emphasized in Fule v. Court of Appeals where x Contrary to the assumption made in the Decision that Emilio III’s demonstrable
we held that the widow would have more interest than any other interest in the estate makes him a suitable co-administrator thereof, the
next of kin in the proper administration of the entire estate since evidence reveals that Emilio III has turned out to be an unsuitable administrator of
she possesses not only the right of succession over a portion of the the estate. Respondent Isabel points out that after Emilio III’s appointment as
exclusive property of the decedent but also a share in the conjugal administrator of the subject estate in 2001, he has not looked after the welfare of
partnership for which the good or bad administration of the estate the subject estate and has actually acted to the damage and prejudice thereof as
may affect not just the fruits but more critically the naked evidenced by the following:
ownership thereof. And in Gabriel v. Court of Appeals we o 1. Emilio III, despite several orders from the probate court for a
recognized the distinctive status of a surviving spouse applying as complete inventory, omitted in the partial inventories34 he filed
regular administrator of the deceased spouse's estate when we therewith properties of the estate35 including several parcels of
counseled the probate court that "there must be a very strong case land, cash, bank deposits, jewelry, shares of stock, motor vehicles,
to justify the exclusion of the widow from the administration." and other personal properties, contrary to Section 1,36 paragraph a,
o Clearly, the selection of a special co-administrator in Matias, Corona Rule 81 of the Rules of Court.
and Vda. de Dayrit was based upon the independent proprietary o 2. Emilio III did not take action on both occasions against
interests and moral circumstances of the appointee that were not Federico’s settlement of the decedent’s estate which adjudicated to
himself a number of properties properly belonging to said estate
88
RECTO, GAYLE ANGELI M.
2011-0008 | AUSL
Personal Notes on Remedial Law 2 Review (based on the syllabus of Prof. Henedino M. Brondial)

(whether wholly or partially), and which contained a declaration If an executor or administrator neglects to render his account and
that the decedent did not leave any descendants or heirs, except settle the estate according to law, or to perform an order or
for Federico, entitled to succeed to her estate.37 judgment of the court, or a duty expressly provided by these rules,
x In compliance to our Resolution dated 18 April 2012 requiring Emilio III to or absconds, or becomes insane, or otherwise incapable or
respond to the following imputations of Isabel that: unsuitable to discharge the trust, the court may remove him, or, in
o 1. Emilio III did not file an inventory of the assets until November its discretion, may permit him to resign. When an executor or
14, 2002; administrator dies, resigns, or is removed, the remaining executor
o 2. The inventory Emilio III submitted did not include several or administrator may administer the trust alone, unless the court
properties of the decedent; grants letters to someone to act with him. If there is no remaining
o 3. That properties belonging to the decedent have found their way executor or administrator, administration may be granted to any
to different individuals or persons; several properties to Federico suitable person.
Suntay himself; and x Once again, as we have done in the Decision, we exercise judicial restraint: we
o 4. While some properties have found their way to Emilio III, by uphold that the question of who are the heirs of the decedent Cristina is not yet
reason of falsified documents;38 upon us. Article 992 of the Civil Code or the curtain bar rule is inapplicable in
x Emilio III refutes Isabel’s imputations that he was lackadaisical in assuming and resolving the issue of who is better qualified to administer the estate of the
performing the functions of administrator of Cristina’s estate: decedent.
o 1. From the time of the RTC’s Order appointing Emilio III as x Thus, our disquisition in the assailed Decision:
administrator, Isabel, in her pleadings before the RTC, had o Nonetheless, it must be pointed out that judicial restraint impels us
vigorously opposed Emilio III’s assumption of that office, arguing to refrain from making a final declaration of heirship and
that "the decision of the RTC dated 9 November 2001 is not among distributing the presumptive shares of the parties in the estates of
the judgments authorized by the Rules of Court which may be Cristina and Federico, considering that the question on who will
immediately implemented or executed;" administer the properties of the long deceased couple has yet to be
o 2. The delay in Emilio III’s filing of an inventory was due to Isabel’s settled.
vociferous objections to Emilio III’s attempts to act as administrator x Our holding in Capistrano v. Nadurata on the same issue remains good law:
while the RTC decision was under appeal to the Court of Appeals; o The declaration of heirs made by the lower court is premature,
o 3. The complained partial inventory is only initiatory, inherent in although the evidence sufficiently shows who are entitled to
the nature thereof, and one of the first steps in the lengthy process succeed the deceased. The estate had hardly been judicially
of settlement of a decedent’s estate, such that it cannot constitute a opened, and the proceeding has not as yet reached the stage of
complete and total listing of the decedent’s properties; and distribution of the estate which must come after the inheritance is
o 4. The criminal cases adverted to are trumped-up charges where liquidated.
Isabel, as private complainant, has been unwilling to appear and x Section 1, Rule 90 of the Rules of Court does not depart from the foregoing
testify, leading the Judge of the Regional Trial Court, Branch 44 of admonition:
Mamburao, Occidental Mindoro, to warn the prosecutor of a o Sec. 1. When order for distribution of residue is made. - x x x. If
possible motu propio dismissal of the cases there is a controversy before the court as to who are the lawful
x While we can subscribe to Emilio III’s counsel’s explanation for the blamed delay heirs of the deceased person or as to the distributive shares to
in the filing of an inventory and his exposition on the nature thereof, partial as which each person is entitled under the law, the controversy shall
opposed to complete, in the course of the settlement of a decedent’s estate, we be heard and decided as in ordinary cases.
do not find any clarification on Isabel’s accusation that Emilio III had deliberately x No distribution shall be allowed until the payment of the obligations above
omitted properties in the inventory, which properties of Cristina he knew existed mentioned has been made or provided for, unless the distributees, or any of
and which he claims to be knowledgeable about. them, give a bond, in a sum to be fixed by the court, conditioned for the
x The general denial made by Emilio III does not erase his unsuitability as payment of said obligations within such time as the court directs.45
administrator rooted in his failure to "make and return x x x a true and complete x Lastly, we dispose of a peripheral issue raised in the Supplemental Comment46
inventory" which became proven fact when he actually filed partial inventories of Emilio III questioning the Special Second Division which issued the 18 April
before the probate court and by his inaction on two occasions of Federico’s 2012 Resolution. Emilio III asseverates that "the operation of the Special Second
exclusion of Cristina’s other compulsory heirs, herein Isabel and her siblings, Division in Baguio is unconstitutional and void" as the Second Division in Manila
from the list of heirs. had already promulgated its Decision on 16 June 2010 on the petition filed by
x As administrator, Emilio III enters into the office, posts a bond and executes an him:
oath to faithfully discharge the duties of settling the decedent’s estate with the o 7. The question is: who created the Special Second Division in
end in view of distribution to the heirs, if any. This he failed to do. The foregoing Baguio, acting separately from the Second Division of the Supreme
circumstances of Emilio III’s omission and inaction become even more significant Court in Manila? There will then be two Second Divisions of the
and speak volume of his unsuitability as administrator as it demonstrates his Supreme Court: one acting with the Supreme Court in Manila, and
interest adverse to those immediately interested in the estate of the decedent, another Special Second Division acting independently of the Second
Cristina. Division of the Supreme Court in Manila.47
x In this case, palpable from the evidence on record, the pleadings, and the x For Emilio III’s counsels’ edification, the Special Second Division in Baguio is not
protracted litigation, is the inescapable fact that Emilio III and respondent Isabel a different division created by the Supreme Court.
have a deep aversion for each other.1awp++i1 To our mind, it becomes highly x The Second Division which promulgated its Decision on this case on 16 June
impractical, nay, improbable, for the two to work as co-administrators of their 2010, penned by Justice Antonio Eduardo B. Nachura, now has a different
grandmother’s estate. The allegations of Emilio III, the testimony of Federico composition, with the advent of Justice Nachura’s retirement on 13 June 2011.
and the other witnesses for Federico and Emilio III that Isabel and her siblings Section 7, Rule 2 of the Internal Rules of the Supreme Court provides:
were estranged from their grandparents further drive home the point that Emilio o Sec. 7. Resolutions of motions for reconsideration or clarification of
III bears hostility towards Isabel. More importantly, it appears detrimental to the decisions or signed resolutions and all other motions and incidents
decedent’s estate to appoint a co-administrator (Emilio III) who has shown an subsequently filed; creation of a Special Division. - Motions for
adverse interest of some kind or hostility to those, such as herein respondent reconsideration or clarification of a decision or of a signed
Isabel, immediately interested in the said estate. resolution and all other motions and incidents subsequently filed in
the case shall be acted upon by the ponente and the other
x Bearing in mind that the issuance of letters of administration is simply a Members of the Division who participated in the rendition of the
preliminary order to facilitate the settlement of a decedent’s estate, we here decision or signed resolution.
point out that Emilio III is not without remedies to protect his interests in the o If the ponente has retired, is no longer a Member of the Court, is
estate of the decedent. In Hilado v. Court of Appeals,39 we mapped out as disqualified, or has inhibited himself or herself from acting on the
among the allowable participation of "any interested persons" or "any persons motion for reconsideration or clarification, he or she shall be
interested in the estate" in either testate or intestate proceedings: replaced through raffle by a new ponente who shall be chosen
o 4. Section 640 of Rule 87, which allows an individual interested in among the new Members of the Division who participated in the
the estate of the deceased "to complain to the court of the rendition of the decision or signed resolution and who concurred
concealment, embezzlement, or conveyance of any asset of the therein. If only one Member of the Court who participated and
decedent, or of evidence of the decedent’s title or interest therein;" concurred in the rendition of the decision or signed resolution
o 5. Section 1041 of Rule 85, which requires notice of the time and remains, he or she shall be designated as the new ponente.
place of the examination and allowance of the Administrator’s o If a Member (not the ponente) of the Division which rendered the
account "to persons interested;" decision or signed resolution has retired, is no longer a Member of
o 6. Section 7(b)42 of Rule 89, which requires the court to give the Court, is disqualified, or has inhibited himself or herself from
notice "to the persons interested" before it may hear and grant a acting on the motion for reconsideration or clarification, he or she
petition seeking the disposition or encumbrance of the properties of shall be replaced through raffle by a replacement Member who
the estate; and shall be chosen from the other Divisions until a new Justice is
o 7. Section 1,43 Rule 90, which allows "any person interested in the appointed as replacement for the retired Justice. Upon the
estate" to petition for an order for the distribution of the residue of appointment of a new Justice, he or she shall replace the
the estate of the decedent, after all obligations are either satisfied designated Justice as replacement Member of the Special Division.
or provided for.44 o Any vacancy or vacancies in the Special Division shall be filled by
x In addition to the foregoing, Emilio III may likewise avail of the remedy found in raffle from among the other Members of the Court to constitute a
Section 2, Rule 82 of the Rules of Court, to wit: Special Division of five (5) Members.
o Sec. 2. Court may remove or accept resignation of executor or o If the ponente and all the Members of the Division that rendered
administrator. Proceedings upon death, resignation, or removal. - the Decision or signed Resolution are no longer Members of the
89
RECTO, GAYLE ANGELI M.
2011-0008 | AUSL
Personal Notes on Remedial Law 2 Review (based on the syllabus of Prof. Henedino M. Brondial)

Court, the case shall be raffled to any Member of the Court and the and release of Ma. Divina Ortañez-Enderes as Special
motion shall be acted upon by him or her with the participation of Administratrix, the "Urgent Motion to Declare Void Ab Initio
the other Members of the Division to which he or she belongs. Memorandum of Agreement" dated December 19, 1995. . . is
o If there are pleadings, motions or incidents subsequent to the hereby impliedly partially resolved insofar as the
denial of the motion for reconsideration or clarification, the case transfer/waiver/renunciation of the Philinterlife shares of stock are
shall be acted upon by the ponente on record with the participation concerned, in particular, No. 5, 9(c), 10(b) and 11(d)(ii) of the
of the other Members of the Division to which he or she belongs at Memorandum of Agreement.
the time said pleading, motion or incident is to be taken up by the o WHEREFORE, this Court hereby declares the Memorandum of
Court. (Emphasis supplied) Agreement dated March 4, 1982 executed by Juliana S. Ortañez,
x As regards the operation thereof in Baguio City, such is simply a change in venue Rafael S. Ortañez and Jose S. Ortañez as partially void ab initio
for the Supreme Court's summer session held last April. insofar as the transfer/waiver/renunciation of the Philinterlife
shares of stocks are concerned.
JOSE C. LEE AND ALMA AGGABAO, in their capacities as President and Corporate Secretary, x Jose Æ filed a Rule 65 before the CA
respectively, of Philippines International Life Insurance Company, and FILIPINO LOAN x CA Æ dismissed
ASSISTANCE GROUP, petitioners vs. REGIONAL TRIAL COURT OF QUEZON CITY o ruling that there was no legal justification whatsoever for the
BRANCH 85 presided by JUDGE PEDRO M. AREOLA, BRANCH CLERK OF COURT JANICE Y. extrajudicial partition of the estate by Jose Ortañez, his brother
ANTERO, DEPUTY SHERIFFS ADENAUER G. RIVERA and PEDRO L. BORJA, all of the Regional Rafael Ortañez and mother Juliana Ortañez during the pendency of
Trial Court of Quezon City Branch 85, MA. DIVINA ENDERES claiming to be Special the settlement of the estate of Dr. Ortañez, without the requisite
Administratrix, and other persons/ public officers acting for and in their behalf, respondents. approval of the intestate court, when it was clear that there were
G.R. No. 146006 February 23, 2004 other heirs to the estate who stood to be prejudiced thereby.
THIRD DIVISION Consequently, the sale made by Jose Ortañez and his mother
Juliana Ortañez to FLAG of the shares of stock they invalidly
FACTS: appropriated for themselves, without approval of the intestate
x Dr. Juvencio P. Ortañez incorporated the Philippine International Life Insurance court, was void
Company, Inc. on July 6, 1956. x Jose Æ filed a Rule 45 before the SC
o At the time of the company’s incorporation, Dr. Ortañez owned o SC Æ dismissed
ninety percent (90%) of the subscribed capital stock. x Special Administratrix Enderes and her siblings Æ filed a motion for execution of
x Dr. Ortañez died the Orders of the intestate court dated August 11 and August 29, 1997 because
x Heirs are as follows: the orders of the intestate court nullifying the sale (upheld by the Court of
o wife (Juliana Salgado Ortañez), Appeals and the Supreme Court) had long became final
o three legitimate children (Rafael, Jose and Antonio Ortañez) and o CFI Æ granted
o five illegitimate children by Ligaya Novicio (herein private x Lee and Aggabao Æ filed Rule 65 before the CA
respondent Ma. Divina Ortañez-Enderes and her siblings Jose, o Petitioners alleged that the intestate court gravely abused its
Romeo, Enrico Manuel and Cesar, all surnamed Ortañez) discretion in (1) declaring that the ownership of FLAG over the
x Rafael Ortañez Æ a petition for letters of administration of the intestate estate of Philinterlife shares of stock was null and void; (2) ordering the
Dr. Ortañez before CFI QC Br 85 execution of its order declaring such nullity and (3) depriving the
x Ma. Divina Ortañez-Enderes and her siblings Æ filed an opposition to the petition petitioners of their right to due process.
for letters of administration x CA Æ dismissed outright
o in a subsequent urgent motion, prayed that the intestate court x Lee and Aggabao Æ filed Rule 45 before the SC
appoint a special administrator x Lee and Aggabao Æ filed MR
x Judge Ernani Cruz Paño (then presiding judge of Branch 85) Æ appointed Rafael o SC Æ granted; reinstated their petition
and Jose Ortañez joint special administrators of their father’s estate x Special Administratrix Enderes Æ filed a motion to direct the branch clerk of
x Rafael and Jose Ortañez Æ submitted an inventory of the estate of their father court in lieu of herein petitioners Lee and Aggabao to reinstate the name of Dr.
which included, among other properties, 2,0293 shares of stock in Philippine Ortañez in the stock and transfer book of Philinterlife and issue the
International Life Insurance Company (hereafter Philinterlife), representing corresponding stock certificate pursuant to Section 10, Rule 39 of the Rules of
50.725% of the company’s outstanding capital stock. Court which provides that "the court may direct the act to be done at the cost of
x Juliana S. Ortañez (wife) Æ then claimed that she owned 1,0144 Philinterlife the disobedient party by some other person appointed by the court and the act
shares of stock as her conjugal share in the estate, sold said shares with right to when so done shall have the effect as if done by the party."
repurchase in favor of herein petitioner Filipino Loan Assistance Group (FLAG), o CFI Æ granted
represented by its president, herein petitioner Jose C. Lee ƒ ruling that there was no prohibition for the intestate
o failed to repurchase the shares of stock within the stipulated court to execute its orders inasmuch as the appellate
period, thus ownership thereof was consolidated by petitioner FLAG court did not issue any TRO or writ of preliminary
in its name. injunction.
x Jose Ortañez (acting in his personal capacity) Æ then claimed that he owned the x Lee and Aggabao Æ filed Rule 65 before the CA
remaining 1,0115 Philinterlife shares of stocks as his inheritance share in the o CA Æ denied
estate, sold said shares with right to repurchase also in favor of herein petitioner o Lee and Aggabao Æ filed MR
FLAG, represented by its president, herein petitioner Jose C. Lee o Pending before the CA
x Years before BUT during the pendency of the present case, uliana Ortañez and x Lee and Aggabao Æ filed Rule 45 before the SC
her two children, Special Administrators Rafael and Jose Ortañez, entered into a o Petitioners Jose Lee and Alma Aggabao, representing Philinterlife
memorandum of agreement dated March 4, 1982 for the extrajudicial settlement and FLAG, assail before us not only the validity of the writ of
of the estate of Dr. Juvencio Ortañez, partitioning the estate (including the execution issued by the intestate court dated July 7, 2000 but also
Philinterlife shares of stock) among themselves. the validity of the August 11, 1997 order of the intestate court
o This was the basis of the number of shares separately sold by nullifying the sale of the 2,029 Philinterlife shares of stock made by
Juliana Ortañez on April 15, 1989 (1,014 shares) and by Jose Juliana Ortañez and Jose Ortañez, in their personal capacities and
Ortañez on October 30, 1991 (1,011 shares) in favor of herein without court approval, in favor of petitioner FLAG.
petitioner FLAG.
x Ma. Divina Ortañez-Enderes and her siblings (hereafter referred to as private ISSUE # 1: Whether Lee and Aggabao may still reopen the issue of nullity of the subject sale.
respondents Enderes et al.) Æ filed a motion for appointment of special
administrator of Philinterlife shares of stock HELD # 1: NO.
o CFIÆ granted x We cannot allow petitioners to reopen the issue of nullity of the sale of the
x Special Administratrix Enderes Æ filed the ff motions: Philinterlife shares of stock in their favor because this was already settled a long
o an urgent motion to declare void ab initio the memorandum of time ago by the Court of Appeals in its decision dated June 23, 1998 in CA-G.R.
agreement dated March 4, 1982. SP No. 46342. This decision was effectively upheld by us in our resolution dated
o a motion to declare the partial nullity of the extrajudicial settlement October 9, 1998 in G.R. No. 135177 dismissing the petition for review on a
of the decedent’s estate technicality and thereafter denying the motion for reconsideration on January 13,
o an urgent motion to declare void ab initio the deeds of sale of 1999 on the ground that there was no compelling reason to reconsider said
Philinterlife shares of stock denial.18 Our decision became final on February 23, 1999 and was accordingly
x Jose Ortañez Æ filed an omnibus motion for (1) the approval of the deeds of sale entered in the book of entry of judgments. For all intents and purposes
of the Philinterlife shares of stock and (2) the release of Ma. Divina Ortañez- therefore, the nullity of the sale of the Philinterlife shares of stock made by
Enderes as special administratrix of the Philinterlife shares of stock on the Juliana Ortañez and Jose Ortañez in favor of petitioner FLAG is already a closed
ground that there were no longer any shares of stock for her to administer. case. To reopen said issue would set a bad precedent, opening the door wide
x CFI Æ denied the OMNIBUS motion of Jose open for dissatisfied parties to relitigate unfavorable decisions no end. This is
o Under the Godoy case, supra, it was held in substance that a sale completely inimical to the orderly and efficient administration of justice.
of a property of the estate without an Order of the probate court is x What we have here is a situation where some of the heirs of the decedent
void and passes no title to the purchaser. Since the sales in without securing court approval have appropriated as their own personal
question were entered into by Juliana S. Ortañez and Jose S. property the properties of [the] Estate, to the exclusion and the extreme
Ortañez in their personal capacity without prior approval of the prejudice of the other claimant/heirs. In other words, these heirs, without court
Court, the same is not binding upon the Estate. approval, have distributed the asset of the estate among themselves and
o In consonance with the Order of this Court dated August 11, 1997 proceeded to dispose the same to third parties even in the absence of an order
DENYING the approval of the sale of Philinterlife shares of stocks of distribution by the Estate Court. As admitted by petitioner’s counsel, there was
90
RECTO, GAYLE ANGELI M.
2011-0008 | AUSL
Personal Notes on Remedial Law 2 Review (based on the syllabus of Prof. Henedino M. Brondial)

absolutely no legal justification for this action by the heirs. There being no legal property of the deceased, which is not authorized by the probate
justification, petitioner has no basis for demanding that public respondent [the court is null and void and title does not pass to the purchaser.
intestate court] approve the sale of the Philinterlife shares of the Estate by o There is hardly any doubt that the probate court can declare null
Juliana and Jose Ortañez in favor of the Filipino Loan Assistance Group. and void the disposition of the property under administration, made
x It is an undisputed fact that the parties to the Memorandum of Agreement dated by private respondent, the same having been effected without
March 4, 1982 (see Annex 7 of the Comment). . . are not the only heirs claiming authority from said court. It is the probate court that has the power
an interest in the estate left by Dr. Juvencio P. Ortañez. The records of this case. to authorize and/or approve the sale (Section 4 and 7, Rule 89),
. . clearly show that as early as March 3, 1981 an Opposition to the Application hence, a fortiori, it is said court that can declare it null and void for
for Issuance of Letters of Administration was filed by the acknowledged natural as long as the proceedings had not been closed or terminated. To
children of Dr. Juvencio P. Ortañez with Ligaya Novicio. . . This claim by the uphold petitioner’s contention that the probate court cannot annul
acknowledged natural children of Dr. Juvencio P. Ortañez is admittedly known to the unauthorized sale, would render meaningless the power
the parties to the Memorandum of Agreement before they executed the same. pertaining to the said court. (Bonga vs. Soler, 2 SCRA 755).
This much was admitted by petitioner’s counsel during the oral argument. xxx (emphasis ours)
x Given the foregoing facts, and the applicable jurisprudence, public respondent x Our jurisprudence is therefore clear that (1) any disposition of estate
can never be faulted for not approving. . . the subsequent sale by the petitioner property by an administrator or prospective heir pending final
[Jose Ortañez] and his mother [Juliana Ortañez] of the Philinterlife shares adjudication requires court approval and (2) any unauthorized
belonging to the Estate of Dr. Juvencio P. Ortañez." (pages 3-4 of Private disposition of estate property can be annulled by the probate court,
Respondent’s Memorandum; pages 243-244 of the Rollo) there being no need for a separate action to annul the unauthorized
x Amidst the foregoing, We found no grave abuse of discretion amounting to disposition.
excess or want of jurisdiction committed by respondent judge.
ISSUE # 3: Whether the intestate court may execute its order nullifying the sale.
ISSUE # 2: Whether the intestate court was correct in nullifying the subject sale.
HELD # 3: YES.
HELD # 2: YES. x We see no reason why it cannot. The intestate court has the power to
x From the above decision, it is clear that Juliana Ortañez, and her three sons, execute its order with regard to the nullity of an unauthorized sale of
Jose, Rafael and Antonio, all surnamed Ortañez, invalidly entered into a estate property, otherwise its power to annul the unauthorized or
memorandum of agreement extrajudicially partitioning the intestate estate fraudulent disposition of estate property would be meaningless. In
among themselves, despite their knowledge that there were other heirs or other words, enforcement is a necessary adjunct of the intestate or
claimants to the estate and before final settlement of the estate by the intestate probate court’s power to annul unauthorized or fraudulent transactions
court. Since the appropriation of the estate properties by Juliana Ortañez and her to prevent the dissipation of estate property before final adjudication.
children (Jose, Rafael and Antonio Ortañez) was invalid, the subsequent sale x Moreover, in this case, the order of the intestate court nullifying the sale was
thereof by Juliana and Jose to a third party (FLAG), without court approval, was affirmed by the appellate courts (the Court of Appeals in CA-G.R. SP No. 46342
likewise void. dated June 23, 1998 and subsequently by the Supreme Court in G.R. No. 135177
x An heir can sell his right, interest, or participation in the property under dated October 9, 1998). The finality of the decision of the Supreme Court was
administration under Art. 533 of the Civil Code which provides that possession of entered in the book of entry of judgments on February 23, 1999. Considering the
hereditary property is deemed transmitted to the heir without interruption from finality of the order of the intestate court nullifying the sale, as affirmed by the
the moment of death of the decedent.20 However, an heir can only alienate such appellate courts, it was correct for private respondent-Special Administratrix
portion of the estate that may be allotted to him in the division of the estate by Enderes to thereafter move for a writ of execution and for the intestate court to
the probate or intestate court after final adjudication, that is, after all debtors grant it.
shall have been paid or the devisees or legatees shall have been given their x Petitioners Jose Lee, Alma Aggabao and FLAG, however, contend that the
shares.21 This means that an heir may only sell his ideal or undivided share in probate court could not issue a writ of execution with regard to its order
the estate, not any specific property therein. In the present case, Juliana Ortañez nullifying the sale because said order was merely provisional:
and Jose Ortañez sold specific properties of the estate (1,014 and 1,011 shares o The only authority given by law is for respondent judge to
of stock in Philinterlife) in favor of petitioner FLAG. This they could not lawfully determine provisionally whether said shares are included or
do pending the final adjudication of the estate by the intestate court because of excluded in the inventory… In ordering the execution of the
the undue prejudice it would cause the other claimants to the estate, as what orders, respondent judge acted in excess of his
happened in the present case. jurisdiction and grossly violated settled law and
x Juliana Ortañez and Jose Ortañez sold specific properties of the estate, without jurisprudence, i.e., that the determination by a probate or
court approval. It is well-settled that court approval is necessary for the validity intestate court of whether a property is included or
of any disposition of the decedent’s estate. In the early case of Godoy vs. excluded in the inventory of the estate being provisional in
Orellano,22 we laid down the rule that the sale of the property of the estate by nature, cannot be the subject of execution.24 (emphasis
an administrator without the order of the probate court is void and passes no ours)
title to the purchaser. And in the case of Dillena vs. Court of Appeals,23 we ruled x Petitioners’ argument is misplaced. There is no question, based on the facts of
that: this case, that the Philinterlife shares of stock were part of the estate of Dr.
o [I]t must be emphasized that the questioned properties (fishpond) Juvencio Ortañez from the very start as in fact these shares were included in the
were included in the inventory of properties of the estate submitted inventory of the properties of the estate submitted by Rafael Ortañez after he
by then Administratrix Fausta Carreon Herrera on November 14, and his brother, Jose Ortañez, were appointed special administrators by the
1974. Private respondent was appointed as administratrix of the intestate court.25
estate on March 3, 1976 in lieu of Fausta Carreon Herrera. On x The controversy here actually started when, during the pendency of the
November 1, 1978, the questioned deed of sale of the fishponds settlement of the estate of Dr. Ortañez, his wife Juliana Ortañez sold the 1,014
was executed between petitioner and private respondent without Philinterlife shares of stock in favor petitioner FLAG without the approval of the
notice and approval of the probate court. Even after the sale, intestate court. Her son Jose Ortañez later sold the remaining 1,011 Philinterlife
administratrix Aurora Carreon still included the three fishponds as shares also in favor of FLAG without the approval of the intestate court.
among the real properties of the estate in her inventory submitted x We are not dealing here with the issue of inclusion or exclusion of properties in
on August 13, 1981. In fact, as stated by the Court of Appeals, the inventory of the estate because there is no question that, from the very start,
petitioner, at the time of the sale of the fishponds in question, the Philinterlife shares of stock were owned by the decedent, Dr. Juvencio
knew that the same were part of the estate under administration. Ortañez. Rather, we are concerned here with the effect of the sale made by the
o The subject properties therefore are under the jurisdiction of the decedent’s heirs, Juliana Ortañez and Jose Ortañez, without the required approval
probate court which according to our settled jurisprudence has the of the intestate court. This being so, the contention of petitioners that the
authority to approve any disposition regarding properties under determination of the intestate court was merely provisional and should have been
administration. . . More emphatic is the declaration We made in threshed out in a separate proceeding is incorrect.
Estate of Olave vs. Reyes (123 SCRA 767) where We stated that x The petitioners Jose Lee and Alma Aggabao next contend that the writ of
when the estate of the deceased person is already the subject of a execution should not be executed against them because they were not notified,
testate or intestate proceeding, the administrator cannot enter into nor they were aware, of the proceedings nullifying the sale of the shares of
any transaction involving it without prior approval of the probate stock.
court.
o Only recently, in Manotok Realty, Inc. vs. Court of Appeals (149 x We are not persuaded. The title of the purchaser like herein petitioner FLAG can
SCRA 174), We held that the sale of an immovable property be struck down by the intestate court after a clear showing of the nullity of the
belonging to the estate of a decedent, in a special proceedings, alienation. This is the logical consequence of our ruling in Godoy and in several
needs court approval. . . This pronouncement finds support in the subsequent cases.26 The sale of any property of the estate by an administrator
previous case of Dolores Vda. De Gil vs. Agustin Cancio (14 SCRA or prospective heir without order of the probate or intestate court is void and
797) wherein We emphasized that it is within the jurisdiction of a passes no title to the purchaser. Thus, in Juan Lao et al. vs. Hon. Melencio
probate court to approve the sale of properties of a deceased Geneto, G.R. No. 56451, June 19, 1985, we ordered the probate court to cancel
person by his prospective heirs before final adjudication. x x x the transfer certificate of title issued to the vendees at the instance of the
o It being settled that property under administration needs the administrator after finding that the sale of real property under probate
approval of the probate court before it can be disposed of, any proceedings was made without the prior approval of the court. The dispositive
unauthorized disposition does not bind the estate and is null and portion of our decision read:
void. As early as 1921 in the case of Godoy vs. Orellano (42 Phil o IN VIEW OF THE FOREGOING CONSIDERATIONS, the assailed
347), We laid down the rule that a sale by an administrator of Order dated February 18, 1981 of the respondent Judge approving
the questioned Amicable Settlement is declared NULL and VOID
91
RECTO, GAYLE ANGELI M.
2011-0008 | AUSL
Personal Notes on Remedial Law 2 Review (based on the syllabus of Prof. Henedino M. Brondial)

and hereby SET ASIDE. Consequently, the sale in favor of Sotero x Finally, petitioners filed several criminal cases such as libel (Criminal Case No.
Dioniosio III and by the latter to William Go is likewise declared 97-7179-81), grave coercion (Criminal Case No. 84624) and robbery (Criminal
NULL and VOID. The Transfer Certificate of Title issued to the latter Case No. Q-96-67919) against private respondent’s mother Ligaya Novicio who
is hereby ordered cancelled. was a director of Philinterlife,31 all of which criminal cases were related to the
x It goes without saying that the increase in Philinterlife’s authorized capital stock, questionable sale to petitioners of the Philinterlife shares of stock.
approved on the vote of petitioners’ non-existent shareholdings and obviously x Considering these circumstances, we cannot accept petitioners’ claim of denial of
calculated to make it difficult for Dr. Ortañez’s estate to reassume its controlling due process. The essence of due process is the reasonable opportunity to be
interest in Philinterlife, was likewise void ab initio. heard. Where the opportunity to be heard has been accorded, there is no denial of
x Petitioners next argue that they were denied due process. due process.32 In this case, petitioners knew of the pending instestate
o We do not think so. proceedings for the settlement of Dr. Juvencio Ortañez’s estate but for reasons
x The facts show that petitioners, for reasons known only to them, did not appeal they alone knew, they never intervened. When the court declared the nullity of the
the decision of the intestate court nullifying the sale of shares of stock in their sale, they did not bother to appeal. And when they were notified of the
favor. Only the vendor, Jose Ortañez, appealed the case. A careful review of the motion for execution of the Orders of the intestate court, they ignored the same.
records shows that petitioners had actual knowledge of the estate settlement Clearly, petitioners alone should bear the blame.
proceedings and that they knew private respondent Enderes was questioning x Petitioners next contend that we are bound by our ruling in G.R. No. 128525
therein the sale to them of the Philinterlife shares of stock. entitled Ma. Divina Ortañez-Enderes vs. Court of Appeals, dated December 17,
x It must be noted that private respondent-Special Administratrix Enderes filed 1999, where we allegedly ruled that the intestate court "may not pass upon the
before the intestate court (RTC of Quezon City, Branch 85) a "Motion to Declare title to a certain property for the purpose of determining whether the same
Void Ab Initio Deeds of Sale of Philinterlife Shares of Stock" on March 22, 1996. should or should not be included in the inventory but such determination is not
But as early as 1994, petitioners already knew of the pending settlement conclusive and is subject to final decision in a separate action regarding
proceedings and that the shares they bought were under the administration by ownership which may be constituted by the parties."
the intestate court because private respondent Ma. Divina Ortañez-Enderes and x We are not unaware of our decision in G.R. No. 128525. The issue therein was
her mother Ligaya Novicio had filed a case against them at the Securities and whether the Court of Appeals erred in affirming the resolution of the SEC that
Exchange Commission on November 7, 1994, docketed as SEC No. 11-94-4909, Enderes et al. were not entitled to the issuance of the writ of preliminary
for annulment of transfer of shares of stock, annulment of sale of corporate injunction. We ruled that the Court of Appeals was correct in affirming the
properties, annulment of subscriptions on increased capital stocks, accounting, resolution of the SEC denying the issuance of the writ of preliminary injunction
inspection of corporate books and records and damages with prayer for a writ of because injunction is not designed to protect contingent rights. Said case did not
preliminary injunction and/or temporary restraining order.27 In said case, rule on the issue of the validity of the sale of shares of stock belonging to the
Enderes and her mother questioned the sale of the aforesaid shares of stock to decedent’s estate without court approval nor of the validity of the writ of
petitioners. The SEC hearing officer in fact, in his resolution dated March 24, execution issued by the intestate court. G.R. No. 128525 clearly involved a
1995, deferred to the jurisdiction of the intestate court to rule on the validity of different issue and it does not therefore apply to the present case.
the sale of shares of stock sold to petitioners by Jose Ortañez and Juliana x Petitioners and all parties claiming rights under them are hereby warned not to
Ortañez: further delay the execution of the Orders of the intestate court dated August 11
o Petitioners also averred that. . . the Philinterlife shares of Dr. and August 29, 1997.
Juvencio Ortañez who died, in 1980, are part of his estate which is
presently the subject matter of an intestate proceeding of the RTC THE ESTATE OF HILARIO M. RUIZ, EDMOND RUIZ, Executor, petitioner, vs. THE COURT
of Quezon City, Branch 85. Although, private respondents [Jose Lee OF APPEALS (Former Special Sixth Division), MARIA PILAR RUIZ-MONTES, MARIA CATHRYN
et al.] presented the documents of partition whereby the foregoing RUIZ, CANDICE ALBERTINE RUIZ, MARIA ANGELINE RUIZ and THE PRESIDING JUDGE OF THE
share of stocks were allegedly partitioned and conveyed to Jose S. REGIONAL TRIAL COURT OF PASIG, respondents.
Ortañez who allegedly assigned the same to the other private G.R. No. 118671 January 29, 1996
respondents, approval of the Court was not presented. Thus, the SECOND DIVISION
assignments to the private respondents [Jose Lee et al.] of the
subject shares of stocks are void. FACTS:
o With respect to the alleged extrajudicial partition of the shares of x Hilario M. Ruiz (during his lifetime) Æ executed a holographic will naming as his
stock owned by the late Dr. Juvencio Ortañez, we rule that the heirs
matter properly belongs to the jurisdiction of the regular court o his only son, Edmond Ruiz,
where the intestate proceedings are currently pending.28 o his adopted daughter, private respondent Maria Pilar Ruiz Montes,
x With this resolution of the SEC hearing officer dated as early as March 24, 1995 o his three granddaughters, private respondents Maria Cathryn,
recognizing the jurisdiction of the intestate court to determine the validity of the Candice Albertine and Maria Angeline, all children of Edmond Ruiz
extrajudicial partition of the estate of Dr. Ortañez and the subsequent sale by the ƒ named Edmond Ruiz executor of his estate
heirs of the decedent of the Philinterlife shares of stock to petitioners, how can x Hilario died
petitioners claim that they were not aware of the intestate proceedings? x The estate was distributed BUT Edmond DID NOT take any action for the
x Furthermore, when the resolution of the SEC hearing officer reached the probate of his father's holographic will.
Supreme Court in 1996 (docketed as G.R. 128525), herein petitioners who were x Maria Pilar Ruiz (4 years after the death) Æ filed petition for probate before RTC
respondents therein filed their answer which contained statements showing that Pasig and for the issuance of letters testamentary to Edmond Ruiz
they knew of the pending intestate proceedings: o Edmond Æ opposed
o [T]he subject matter of the complaint is not within the jurisdiction x During the pendency of the action, one of the properties of the estate — the
of the SEC but with the Regional Trial Court; Ligaya Novicio and house and lot at No. 2 Oliva Street, Valle Verde IV, Pasig which the testator
children represented themselves to be the common law wife and bequeathed to Maria Cathryn, Candice Albertine and Maria Angeline4 — was
illegitimate children of the late Ortañez; that on March 4, 1982, the leased out by Edmond Ruiz to third persons.
surviving spouse Juliana Ortañez, on her behalf and for her minor x RTC Æ ordered Edmond to deposit with the Branch Clerk of Court the rental
son Antonio, executed a Memorandum of Agreement with her other deposit and payments totalling P540,000.00 representing the one-year lease of
sons Rafael and Jose, both surnamed Ortañez, dividing the estate the Valle Verde property.
of the deceased composed of his one-half (1/2) share in the o EdmondÆ turned over the amount of P348,583.56, representing
conjugal properties; that in the said Memorandum of Agreement, the balance of the rent after deducting P191,416.14 for repair and
Jose S. Ortañez acquired as his share of the estate the 1,329 maintenance expenses on the estate
shares of stock in Philinterlife; that on March 4, 1982, Juliana and x Edmond Æ withdrew his opposition to the probate of the will
Rafael assigned their respective shares of stock in Philinterlife to x RTC Æ admitted the will to probate and ordered the issuance of letters
Jose; that contrary to the contentions of petitioners, private testamentary to Edmond conditioned upon the filing of a bond in the amount of
respondents Jose Lee, Carlos Lee, Benjamin Lee and Alma Aggabao P50,000.00
became stockholders of Philinterlife on March 23, 1983 when Jose x Letters testamentary were then issued
S. Ortañez, the principal stockholder at that time, executed a deed x Edmond Ruiz (as executor) Æ filed an "Ex-Parte Motion for Release of Funds."
of sale of his shares of stock to private respondents; and that the o Prayed for the release of the rent payments deposited with the
right of petitioners to question the Memorandum of Agreement and Branch Clerk of Court. Respondent Montes opposed the motion and
the acquisition of shares of stock of private respondent is barred by concurrently filed a "Motion for Release of Funds to Certain Heirs"
prescription.29 and "Motion for Issuance of Certificate of Allowance of Probate
x Also, private respondent-Special Administratrix Enderes offered additional proof Will." Montes prayed for the release of the said rent payments to
of actual knowledge of the settlement proceedings by petitioners which Maria Cathryn, Candice Albertine and Maria Angeline and for the
petitioners never denied: (1) that petitioners were represented by Atty. Ricardo distribution of the testator's properties, specifically the Valle Verde
Calimag previously hired by the mother of private respondent Enderes to initiate property and the Blue Ridge apartments, in accordance with the
cases against petitioners Jose Lee and Alma Aggabao for the nullification of the provisions of the holographic will.
sale of the shares of stock but said counsel made a conflicting turn-around and x RTC Æ denied
appeared instead as counsel of petitioners, and (2) that the deeds of sale o It thus ordered the release of the rent payments to the decedent's
executed between petitioners and the heirs of the decedent (vendors Juliana three granddaughters. It further ordered the delivery of the titles to
Ortañez and Jose Ortañez) were acknowledged before Atty. Ramon Carpio who, and possession of the properties bequeathed to the three
during the pendency of the settlement proceedings, filed a motion for the granddaughters and respondent Montes upon the filing of a bond
approval of the sale of Philinterlife shares of stock to the Knights of Columbus of P50,000.00.
Fraternal Association, Inc. (which motion was, however, later abandoned).30 All x Edmond Æ filed an MR
this sufficiently proves that petitioners, through their counsels, knew of the
pending settlement proceedings.
92
RECTO, GAYLE ANGELI M.
2011-0008 | AUSL
Personal Notes on Remedial Law 2 Review (based on the syllabus of Prof. Henedino M. Brondial)

o alleging that he actually filed his opposition to respondent Montes's distributees, or any of them, give a bond, in a sum to be fixed by
motion for release of rent payments which opposition the court the court, conditioned for the payment of said obligations within
failed to consider. such time as the court directs.18
o Petitioner likewise reiterated his previous motion for release of x In settlement of estate proceedings, the distribution of the estate
funds properties can only be made: (1) after all the debts, funeral charges,
o manifested that he was withdrawing his motion for release of funds expenses of administration, allowance to the widow, and estate tax
in view of the fact that the lease contract over the Valle Verde have been paid; or (2) before payment of said obligations only if the
property had been renewed for another year distributees or any of them gives a bond in a sum fixed by the court
x RTC Æ ordered the release of the funds to Edmond but only "such amount as conditioned upon the payment of said obligations within such time as
may be necessary to cover the expenses of administration and allowances for the court directs, or when provision is made to meet those
support" of the testator's three granddaughters subject to collation and obligations.19
deductible from their share in the inheritance. x In the case at bar, the probate court ordered the release of the titles to the Valle
o The court, however, held in abeyance the release of the titles to Verde property and the Blue Ridge apartments to the private respondents after
respondent Montes and the three granddaughters until the lapse of the lapse of six months from the date of first publication of the notice to
six months from the date of first publication of the notice to creditors. The questioned order speaks of "notice" to creditors, not payment of
creditors debts and obligations. Hilario Ruiz allegedly left no debts when he died but the
x Edmond Æ filed Rule 65 before CA taxes on his estate had not hitherto been paid, much less ascertained. The
o CA Æ dismissed estate tax is one of those obligations that must be paid before distribution of the
x Edmond Æ filed Rule 45 estate. If not yet paid, the rule requires that the distributees post a bond or
make such provisions as to meet the said tax obligation in proportion to their
ISSUE : Whether the intestate court after admitting the will to probate but before payment of respective shares in the inheritance.20 Notably, at the time the order was issued
the estate's debts and obligations, has the authority: (1) to grant an allowance from the funds the properties of the estate had not yet been inventoried and appraised.
of the estate for the support of the testator's grandchildren; (2) to order the release of the x It was also too early in the day for the probate court to order the
titles to certain heirs; and (3) to grant possession of all properties of the estate to the executor release of the titles six months after admitting the will to probate. The
of the will. probate of a will is conclusive as to its due execution and extrinsic
validity21 and settles only the question of whether the testator, being
HELD # 1: NO as to the grant of allowance of sound mind, freely executed it in accordance with the formalities
x On the matter of allowance, Section 3 of Rule 83 of the Revised Rules of Court prescribed by law.22 Questions as to the intrinsic validity and efficacy
provides: of the provisions of the will, the legality of any devise or legacy may be
o Sec. 3. Allowance to widow and family. — The widow and raised even after the will has been authenticated.23
minor or incapacitated children of a deceased person, during the x The intrinsic validity of Hilario's holographic will was controverted by petitioner
settlement of the estate, shall receive therefrom under the direction before the probate court in his Reply to Montes' Opposition to his motion for
of the court, such allowance as are provided by law. release of funds24 and his motion for reconsideration of the August 26, 1993
x Petitioner alleges that this provision only gives the widow and the minor or order of the said court.25 Therein, petitioner assailed the distributive shares of
incapacitated children of the deceased the right to receive allowances for support the devisees and legatees inasmuch as his father's will included the estate of his
during the settlement of estate proceedings. He contends that the testator's mother and allegedly impaired his legitime as an intestate heir of his mother.
three granddaughters do not qualify for an allowance because they are not The Rules provide that if there is a controversy as to who are the lawful heirs of
incapacitated and are no longer minors but of legal age, married and gainfully the decedent and their distributive shares in his estate, the probate court shall
employed. In addition, the provision expressly states "children" of the deceased proceed to hear and decide the same as in ordinary cases.26
which excludes the latter's grandchildren.
x It is settled that allowances for support under Section 3 of Rule 83 HELD # 3: NO as to the grant of all properties to the executor.
should not be limited to the "minor or incapacitated" children of the x Still and all, petitioner cannot correctly claim that the assailed order deprived him
deceased. Article 18813 of the Civil Code of the Philippines, the of his right to take possession of all the real and personal properties of the
substantive law in force at the time of the testator's death, provides estate. The right of an executor or administrator to the possession and
that during the liquidation of the conjugal partnership, the deceased's management of the real and personal properties of the deceased is not absolute
legitimate spouse and children, regardless of their age, civil status or and can only be exercised "so long as it is necessary for the payment of the
gainful employment, are entitled to provisional support from the funds debts and expenses of administration,"27 Section 3 of Rule 84 of the Revised
of the estate.14 The law is rooted on the fact that the right and duty to Rules of Court explicitly provides:
support, especially the right to education, subsist even beyond the age o Sec. 3. Executor or administrator to retain whole estate to
of majority.15 pay debts, and to administer estate not willed. — An executor or
x Be that as it may, grandchildren are not entitled to provisional support administrator shall have the right to the possession and
from the funds of the decedent's estate. The law clearly limits the management of the real as well as the personal estate of the
allowance to "widow and children" and does not extend it to the deceased so long as it is necessary for the payment of the debts
deceased's grandchildren, regardless of their minority or incapacity.16 and expenses for administration.28
It was error, therefore, for the appellate court to sustain the probate x When petitioner moved for further release of the funds deposited with the clerk
court's order granting an allowance to the grandchildren of the testator of court, he had been previously granted by the probate court certain amounts
pending settlement of his estate. for repair and maintenance expenses on the properties of the estate, and
payment of the real estate taxes thereon. But petitioner moved again for the
HELD # 2: NO as to the release of titles. release of additional funds for the same reasons he previously cited. It was
x Respondent courts also erred when they ordered the release of the titles of the correct for the probate court to require him to submit an accounting of the
bequeathed properties to private respondents six months after the date of first necessary expenses for administration before releasing any further money in his
publication of notice to creditors. An order releasing titles to properties of the favor.
estate amounts to an advance distribution of the estate which is allowed only x It was relevantly noted by the probate court that petitioner had deposited with it
under the following conditions: only a portion of the one-year rental income from the Valle Verde property.
o Sec. 2. Advance distribution in special proceedings. — Petitioner did not deposit its succeeding rents after renewal of the lease.29
Nothwithstanding a pending controversy or appeal in proceedings Neither did he render an accounting of such funds.
to settle the estate of a decedent, the court may, in its discretion x Petitioner must be reminded that his right of ownership over the properties of his
and upon such terms as it may deem proper and just, permit that father is merely inchoate as long as the estate has not been fully settled and
such part of the estate as may not be affected by the controversy partitioned.30 As executor, he is a mere trustee of his father's estate. The funds
or appeal be distributed among the heirs or legatees, upon of the estate in his hands are trust funds and he is held to the duties and
compliance with the conditions set forth in Rule 90 of these responsibilities of a trustee of the highest order.31 He cannot unilaterally assign
Rules.17 to himself and possess all his parents' properties and the fruits thereof without
x And Rule 90 provides that: first submitting an inventory and appraisal of all real and personal properties of
o Sec. 1. When order for distribution of residue made. — When the deceased, rendering a true account of his administration, the expenses of
the debts, funeral charges, and expenses of administration the administration, the amount of the obligations and estate tax, all of which are
allowance to the widow, and inheritance tax if any, chargeable to subject to a determination by the court as to their veracity, propriety and
the estate in accordance with law, have been paid, the court, on justness
the application of the executor or administrator, or of a person
interested in the estate, and after hearing upon notice shall assign
the residue of the estate to the persons entitled to the same,
naming them and the proportions or parts, to which each is
entitled, and such persons may demand and recover their
respective shares from the executor or administrator, or any other
person having the same in his possession. If there is a controversy
before the court as to who are the lawful heirs of the deceased
person or as to the distributive shares to which each person is UNION BANK OF THE PHILIPPINES, petitioner, vs. EDMUND SANTIBAÑEZ and
entitled under the law, the controversy shall be heard and decided FLORENCE SANTIBAÑEZ ARIOLA, respondents.
as in ordinary cases. G.R. No. 149926 February 23, 2005
o No distribution shall be allowed until the payment of the obligations SECOND DIVISION
above-mentioned has been made or provided for, unless the
93
RECTO, GAYLE ANGELI M.
2011-0008 | AUSL
Personal Notes on Remedial Law 2 Review (based on the syllabus of Prof. Henedino M. Brondial)

FACTS: primarily concerned with the administration, liquidation and distribution of the
x First Countryside Credit Corporation (FCCC) and Efraim M. Santibañez entered 2 estate.21
loan agreements x In our jurisdiction, the rule is that there can be no valid partition among the heirs
o in the amount of P128,000.00 which was intended for the payment until after the will has been probated:
of the purchase price of one (1) unit Ford 6600 Agricultural All- o In testate succession, there can be no valid partition among the
Purpose Diesel Tractor heirs until after the will has been probated. The law enjoins the
ƒ Efraim and his son, Edmund Æ executed a promissory probate of a will and the public requires it, because unless a will is
note in favor of the FCCC, the principal sum payable probated and notice thereof given to the whole world, the right of a
in five equal annual amortizations of P43,745.96 due person to dispose of his property by will may be rendered
on May 31, 1981 and every May 31st thereafter up to nugatory. The authentication of a will decides no other question
May 31, 1985 than such as touch upon the capacity of the testator and the
o in the amount of P123,156.00 intended for the balance of the compliance with those requirements or solemnities which the law
purchase price of another unit of Ford 6600 Agricultural All-Purpose prescribes for the validity of a will.22
Diesel Tractor, with accessories, and one (1) unit Howard x This, of course, presupposes that the properties to be partitioned are the same
Rotamotor Model AR 60K. properties embraced in the will.23 In the present case, the deceased, Efraim
ƒ Again, Efraim and his son, Edmund, executed a Santibañez, left a holographic will24 which contained, inter alia, the provision
promissory note for the said amount in favor of the which reads as follows:
FCCC o (e) All other properties, real or personal, which I own and may be
x Efraim died, leaving a holographic will discovered later after my demise, shall be distributed in the
x testate proceedings commenced before RTC Iloilo Br. 7 proportion indicated in the immediately preceding paragraph in
x Edmund, as one of the heirs, was appointed as the special administrator of the favor of Edmund and Florence, my children.
estate of the decedent x We agree with the appellate court that the above-quoted is an all-encompassing
x During the pendency of the testate proceedings, the surviving heirs, Edmund and provision embracing all the properties left by the decedent which might have
his sister Florence Santibañez Ariola, executed a Joint Agreement8 dated July 22, escaped his mind at that time he was making his will, and other properties he
1981, wherein they agreed to divide between themselves and take possession of may acquire thereafter. Included therein are the three (3) subject tractors. This
the three (3) tractors; that is, two (2) tractors for Edmund and one (1) tractor being so, any partition involving the said tractors among the heirs is not valid.
for Florence. The joint agreement25 executed by Edmund and Florence, partitioning
o Each of them was to assume the indebtedness of their late father the tractors among themselves, is invalid, specially so since at the time
to FCCC, corresponding to the tractor respectively taken by them. of its execution, there was already a pending proceeding for the
x Meanwhile, a Deed of Assignment with Assumption of Liabilities9 was executed probate of their late father’s holographic will covering the said
by and between FCCC and Union Savings and Mortgage Bank, wherein the FCCC tractors.
as the assignor, among others, assigned all its assets and liabilities to Union x It must be stressed that the probate proceeding had already acquired jurisdiction
Savings and Mortgage Bank. over all the properties of the deceased, including the three (3) tractors. To
x UnionBank Æ sent demand letter to Edmund dispose of them in any way without the probate court’s approval is tantamount
o This was unheeded to divesting it with jurisdiction which the Court cannot allow.26 Every act
x UnionBank Æ filed action for sum of money against the heirs of Efraim intended to put an end to indivision among co-heirs and legatees or devisees is
Santibañez, Edmund and Florence before the RTC Makati Br. 150 deemed to be a partition, although it should purport to be a sale, an exchange, a
x Summonses were issued against both, but the one intended for Edmund was not compromise, or any other transaction.27 Thus, in executing any joint agreement
served since he was in the United States and there was no information on his which appears to be in the nature of an extra-judicial partition, as in the case at
address or the date of his return to the Philippines.12 bar, court approval is imperative, and the heirs cannot just divest the court of its
o Accordingly, the complaint was narrowed down to respondent jurisdiction over that part of the estate. Moreover, it is within the jurisdiction of
Florence S. Ariola. the probate court to determine the identity of the heirs of the decedent.28 In the
x Florence Æ filed answer instant case, there is no showing that the signatories in the joint agreement were
o alleged that the loan documents did not bind her since she was not the only heirs of the decedent. When it was executed, the probate of the will
a party thereto was still pending before the court and the latter had yet to determine who the
o Considering that the joint agreement signed by her and her brother heirs of the decedent were. Thus, for Edmund and respondent Florence S. Ariola
Edmund was not approved by the probate court, it was null and to adjudicate unto themselves the three (3) tractors was a premature act, and
void; hence, she was not liable to the petitioner under the joint prejudicial to the other possible heirs and creditors who may have a valid claim
agreement. against the estate of the deceased.
x Case was unloaded and re-raffled to RTC Makati Br 63
x RTC Æ dismissed ISSUE # 2: Whether the heirs’ assumption of the indebtedness of the deceased is valid.
o The trial court found that the claim of the petitioner should have
been filed with the probate court before which the testate estate of HELD # 2: NO.
the late Efraim Santibañez was pending, as the sum of money x The question that now comes to fore is whether the heirs’ assumption of the
being claimed was an obligation incurred by the said decedent. The indebtedness of the decedent is binding. We rule in the negative. Perusing the
trial court also found that the Joint Agreement apparently executed joint agreement, it provides that the heirs as parties thereto "have agreed to
by his heirs, Edmund and Florence, on July 22, 1981, was, in divide between themselves and take possession and use the above-described
effect, a partition of the estate of the decedent. However, the said chattel and each of them to assume the indebtedness corresponding to the
agreement was void, considering that it had not been approved by chattel taken as herein after stated which is in favor of First Countryside Credit
the probate court, and that there can be no valid partition until Corp."29 The assumption of liability was conditioned upon the happening of an
after the will has been probated. The trial court further declared event, that is, that each heir shall take possession and use of their respective
that petitioner failed to prove that it was the now defunct Union share under the agreement. It was made dependent on the validity of the
Savings and Mortgage Bank to which the FCCC had assigned its partition, and that they were to assume the indebtedness corresponding to the
assets and liabilities. The court also agreed to the contention of chattel that they were each to receive. The partition being invalid as earlier
respondent Florence S. Ariola that the list of assets and liabilities of discussed, the heirs in effect did not receive any such tractor. It follows
the FCCC assigned to Union Savings and Mortgage Bank did not then that the assumption of liability cannot be given any force and
clearly refer to the decedent’s account. Ruling that the joint effect.
agreement executed by the heirs was null and void, the trial court x The Court notes that the loan was contracted by the decedent.l^vvphi1.net The
held that the petitioner’s cause of action against respondent petitioner, purportedly a creditor of the late Efraim Santibañez, should have thus
Florence S. Ariola must necessarily fail. filed its money claim with the probate court in accordance with Section 5, Rule
x UnionBank Æ appealed to CA 86 of the Revised Rules of Court, which provides:
x CA Æ affirmed RTC o Section 5. Claims which must be filed under the notice. If not filed
o The appellate court found that the appeal was not meritorious and barred; exceptions. — All claims for money against the decedent,
held that the petitioner should have filed its claim with the probate arising from contract, express or implied, whether the same be
court as provided under Sections 1 and 5, Rule 86 of the Rules of due, not due, or contingent, all claims for funeral expenses for the
Court. It further held that the partition made in the agreement was last sickness of the decedent, and judgment for money against the
null and void, since no valid partition may be had until after the will decedent, must be filed within the time limited in the notice;
has been probated. According to the CA, page 2, paragraph (e) of otherwise they are barred forever, except that they may be set
the holographic will covered the subject properties (tractors) in forth as counterclaims in any action that the executor or
generic terms when the deceased referred to them as "all other administrator may bring against the claimants. Where an executor
properties." Moreover, the active participation of respondent or administrator commences an action, or prosecutes an action
Florence S. Ariola in the case did not amount to a waiver. already commenced by the deceased in his lifetime, the debtor may
set forth by answer the claims he has against the decedent, instead
ISSUE # 1: Whether the partition in the Agreement executed by the heirs is valid. of presenting them independently to the court as herein provided,
and mutual claims may be set off against each other in such action;
HELD # 1: NO. and if final judgment is rendered in favor of the defendant, the
x At the outset, well-settled is the rule that a probate court has the amount so determined shall be considered the true balance against
jurisdiction to determine all the properties of the deceased, to the estate, as though the claim had been presented directly before
determine whether they should or should not be included in the the court in the administration proceedings. Claims not yet due, or
inventory or list of properties to be administered.20 The said court is contingent, may be approved at their present value.
94
RECTO, GAYLE ANGELI M.
2011-0008 | AUSL
Personal Notes on Remedial Law 2 Review (based on the syllabus of Prof. Henedino M. Brondial)

x The filing of a money claim against the decedent’s estate in the probate
court is mandatory.30 As we held in the vintage case of Py Eng Chong v. o Sec. 1. Where estate of deceased persons settled. - If the
Herrera:31 decedent is an inhabitant of the Philippines at the time of his death,
o … This requirement is for the purpose of protecting the whether a citizen or an alien, his will shall be proved, or letters of
estate of the deceased by informing the executor or administration granted, and his estate settled, in the Court of First
administrator of the claims against it, thus enabling him to Instance now Regional Trial Court in the province in which he
examine each claim and to determine whether it is a proper resides at the time of his death, and if he is an inhabitant of a
one which should be allowed. The plain and obvious design foreign country, the Court of First Instance now Regional Trial
of the rule is the speedy settlement of the affairs of the Court of any province in which he had estate. The court first taking
deceased and the early delivery of the property to the cognizance of the settlement of the estate of a decedent, shall
distributees, legatees, or heirs. `The law strictly requires exercise jurisdiction to the exclusion of all other courts. The
the prompt presentation and disposition of the claims jurisdiction assumed by a court, so far as it depends on the place of
against the decedent's estate in order to settle the affairs residence of the decedent, or of the location of his estate, shall not
of the estate as soon as possible, pay off its debts and be contested in a suit or proceeding, except in an appeal from that
distribute the residue.32 court, in the original case, or when the want of jurisdiction appears
x Perusing the records of the case, nothing therein could hold private respondent on the record. (Emphasis supplied).
Florence S. Ariola accountable for any liability incurred by her late father. The x The term "resides" connotes ex vi termini "actual residence" as
documentary evidence presented, particularly the promissory notes and the distinguished from "legal residence or domicile." This term "resides,"
continuing guaranty agreement, were executed and signed only by the late like the terms "residing" and "residence," is elastic and should be
Efraim Santibañez and his son Edmund. As the petitioner failed to file its money interpreted in the light of the object or purpose of the statute or rule in
claim with the probate court, at most, it may only go after Edmund as co-maker which it is employed. In the application of venue statutes and rules -
of the decedent under the said promissory notes and continuing guaranty, of Section 1, Rule 73 of the Revised Rules of Court is of such nature -
course, subject to any defenses Edmund may have as against the petitioner. As residence rather than domicile is the significant factor.13 Even where
the court had not acquired jurisdiction over the person of Edmund, we find it the statute uses word "domicile" still it is construed as meaning
unnecessary to delve into the matter further. residence and not domicile in the technical sense.14 Some cases make a
distinction between the terms "residence" and "domicile" but as generally used in
ISSUE # 3: Whether the petitioner can hold the heirs liable on the obligation of the deceased. statutes fixing venue, the terms are synonymous, and convey the same meaning
as the term "inhabitant."15 In other words, "resides" should be viewed or
HELD # 3: NO. understood in its popular sense, meaning, the personal, actual or physical
x We agree with the finding of the trial court that the petitioner had not sufficiently habitation of a person, actual residence or place of abode.16 It signifies physical
shown that it is the successor-in-interest of the Union Savings and Mortgage presence in a place and actual stay thereat.17 Venue for ordinary civil actions and
Bank to which the FCCC assigned its assets and liabilities.33 The petitioner in its that for special proceedings have one and the same meaning.18 As thus defined,
complaint alleged that "by virtue of the Deed of Assignment dated August 20, "residence," in the context of venue provisions, means nothing more than a
1981 executed by and between First Countryside Credit Corporation and Union person’s actual residence or place of abode, provided he resides therein with
Bank of the Philippines…"34 However, the documentary evidence35 clearly continuity and consistency.19
reflects that the parties in the deed of assignment with assumption of liabilities x Viewed in light of the foregoing principles, the Court of Appeals cannot be
were the FCCC, and the Union Savings and Mortgage Bank, with the conformity faulted for affirming the ruling of the RTC that the venue for the settlement of
of Bancom Philippine Holdings, Inc. Nowhere can the petitioner’s participation the estate of Eliseo was properly laid in Las Piñas City. It is evident from the
therein as a party be found. Furthermore, no documentary or testimonial records that during his lifetime, Eliseo resided at No. 26 Everlasting Road, Phase
evidence was presented during trial to show that Union Savings and Mortgage 5, Pilar Village, Las Piñas City. For this reason, the venue for the settlement of his
Bank is now, in fact, petitioner Union Bank of the Philippines. As the trial court estate may be laid in the said city.
declared in its decision: x In opposing the issuance of letters of administration, the petitioners harp on the
o … [T]he court also finds merit to the contention of defendant that entry in Eliseo’s Death Certificate that he is a resident of Capas, Tarlac where
plaintiff failed to prove or did not present evidence to prove that they insist his estate should be settled. While the recitals in death certificates can
Union Savings and Mortgage Bank is now the Union Bank of the be considered proofs of a decedent’s residence at the time of his death, the
Philippines. Judicial notice does not apply here. "The power to take contents thereof, however, is not binding on the courts. Both the RTC and the
judicial notice is to [be] exercised by the courts with caution; care Court of Appeals found that Eliseo had been living with Lourdes, deporting
must be taken that the requisite notoriety exists; and every themselves as husband and wife, from 1972 up to the time of his death in 1995.
reasonable doubt upon the subject should be promptly resolved in This finding is consistent with the fact that in 1985, Eliseo filed an action for
the negative." (Republic vs. Court of Appeals, 107 SCRA 504).36 judicial partition of properties against Amelia before the RTC of Quezon City,
x This being the case, the petitioner’s personality to file the complaint is wanting. Branch 106, on the ground that their marriage is void for being bigamous.20
Consequently, it failed to establish its cause of action. Thus, the trial court did That Eliseo went to the extent of taking his marital feud with Amelia before the
not err in dismissing the complaint, and the CA in affirming the same. courts of law renders untenable petitioners’ position that Eliseo spent the final
days of his life in Tarlac with Amelia and her children. It disproves rather than
supports petitioners’ submission that the lower courts’ findings arose from an
AMELIA GARCIA-QUIAZON, JENNETH QUIAZON and MARIA JENNIFER QUIAZON, erroneous appreciation of the evidence on record. Factual findings of the trial
Petitioners, vs. MA. LOURDES BELEN, for and in behalf of MARIA LOURDES ELISE court, when affirmed by the appellate court, must be held to be conclusive and
QUIAZON, Respondent. binding upon this Court
G.R. No. 189121 July 31, 2013
SECOND DIVISION ISSUE # 2: Whether the CA correctly ruled on the validity of the marriage between Amelia and
Eliso.
FACTS:
x Eliseo Quiazon died intestate HELD # 2: YES.
x Respondent, claiming to be the natural child of Eliseo and represented by her x Likewise unmeritorious is petitioners’ contention that the Court of Appeals erred
mother, Eliseo’s common law spouse, filed for petition for letters of in declaring Amelia’s marriage to Eliseo as void ab initio. In a void marriage, it
administration was though no marriage has taken place, thus, it cannot be the source of rights.
x Petitioners, previous spouse and children, respectively, of Eliseo, opposed said Any interested party may attack the marriage directly or collaterally. A void
petition marriage can be questioned even beyond the lifetime of the parties to the
x Respondent impugned the validity of Eliseo’s marriage to Amelia by claiming that marriage.22 It must be pointed out that at the time of the celebration of the
it was bigamous for having been contracted during the subsistence of the latter’s marriage of Eliseo and Amelia, the law in effect was the Civil Code, and not the
marriage with one Filipito Sandico (Filipito). Family Code, making the ruling in Niñal v. Bayadog23 applicable four-square to
x RTC Æ issued letters the case at hand. In Niñal, the Court, in no uncertain terms, allowed therein
x CA Æ affirmed petitioners to file a petition for the declaration of nullity of their father’s marriage
o In validating the findings of the RTC, the Court of Appeals held that to therein respondent after the death of their father, by contradistinguishing void
Elise was able to prove that Eliseo and Lourdes lived together as from voidable marriages, to wit:
husband and wife by establishing a common residence at No. 26 o Consequently, void marriages can be questioned even after
Everlasting Road, Phase 5, Pilar Village, Las Piñas City, from 1975 the death of either party but voidable marriages can be
up to the time of Eliseo’s death in 1992. For purposes of fixing the assailed only during the lifetime of the parties and not after
venue of the settlement of Eliseo’s estate, the Court of Appeals death of either, in which case the parties and their
upheld the conclusion reached by the RTC that the decedent was a offspring will be left as if the marriage had been perfectly
resident of Las Piñas City. valid. That is why the action or defense for nullity is
x Petitioners Æ filed a Rule 45 before the SC imprescriptible, unlike voidable marriages where the action
prescribes. Only the parties to a voidable marriage can
ISSUE # 1: Whether the petition was properly laid in Las Piñas City, considering the entry in assail it but any proper interested party may attack a void
Eliseo’s Death Certificate that he was a resident of Capas, Tarlac. marriage.24
x It was emphasized in Niñal that in a void marriage, no marriage has taken place
HELD # 1: YES. and it cannot be the source of rights, such that any interested party may attack
x Under Section 1, Rule 73 of the Rules of Court, the petition for letters of the marriage directly or collaterally without prescription, which may be filed even
administration of the estate of a decedent should be filed in the RTC of the beyond the lifetime of the parties to the marriage.25
province where the decedent resides at the time of his death:
95
RECTO, GAYLE ANGELI M.
2011-0008 | AUSL
Personal Notes on Remedial Law 2 Review (based on the syllabus of Prof. Henedino M. Brondial)

x Relevant to the foregoing, there is no doubt that Elise, whose successional rights
would be prejudiced by her father’s marriage to Amelia, may impugn the
existence of such marriage even after the death of her father. The said marriage
may be questioned directly by filing an action attacking the validity thereof, or
collaterally by raising it as an issue in a proceeding for the settlement of the estate
of the deceased spouse, such as in the case at bar. Ineluctably, Elise, as a ERLINDA PILAPIL and HEIRS OF DONATA ORTIZ BRIONES, namely: ESTELA, ERIBERTO
compulsory heir,26 has a cause of action for the declaration of the absolute nullity AND VIRGILIO SANTOS, ANA SANTOS CULTURA, ELVIRA SANTOS INOCENTES, ERNESTO
of the void marriage of Eliseo and Amelia, and the death of either party to the said MENDOZA, RIZALINA SANTOS, ADOLFO MENDOZA and PACITA MENDOZA, Petitioners, vs.
marriage does not extinguish such cause of action. HEIRS OF MAXIMINO R. BRIONES, namely: SILVERIO S. BRIONES, PETRA BRIONES,
x Having established the right of Elise to impugn Eliseo’s marriage to Amelia, we BONIFACIO CABAHUG, JR., ANITA TRASMONTE, CIRILITA FORTUNA, CRESENCIA BRIONES,
now proceed to determine whether or not the decedent’s marriage to Amelia is FUGURACION MEDALLE and MERCEDES LAGBAS, Respondents.
void for being bigamous. G.R. No. 150175 February 5, 2007
x Contrary to the position taken by the petitioners, the existence of a previous THIRD DIVISION
marriage between Amelia and Filipito was sufficiently established by no less than
the Certificate of Marriage issued by the Diocese of Tarlac and signed by the FACTS:
officiating priest of the Parish of San Nicolas de Tolentino in Capas, Tarlac. The x This is an MR of SC’s March 2006 decision which reads:
said marriage certificate is a competent evidence of marriage and the o IN VIEW OF THE FOREGOING, the assailed Decision of the Court of
certification from the National Archive that no information relative to the said Appeals in CA-GR CV No. 55194, dated 31 August 2001, affirming
marriage exists does not diminish the probative value of the entries therein. We the Decision of the Cebu City RTC in Civil Case No. CEB-5794,
take judicial notice of the fact that the first marriage was celebrated more than dated 28 September 1986, is hereby REVERSED and SET ASIDE;
50 years ago, thus, the possibility that a record of marriage can no longer be and the Complaint for partition, annulment, and recovery of
found in the National Archive, given the interval of time, is not completely possession filed by the heirs of Maximino in Civil Case No. CEB-
remote. Consequently, in the absence of any showing that such marriage had 5794 is hereby DISMISSED.
been dissolved at the time Amelia and Eliseo’s marriage was solemnized, the x Said MR was filed by respondents, heirs of Maximino R. Briones
inescapable conclusion is that the latter marriage is bigamous and, therefore, x Parties to the case are:
void ab initio o Petitioners are the heirs of the late Donata Ortiz-Briones (Donata),
consisting of
ISSUE # 3: Whether Respondent Elise Quiazon has shown any interest in the Petition for ƒ her surviving sister, Rizalina Ortiz-Aguila (Rizalina);
Letters of Administration. ƒ Rizalina’s daughter, Erlinda Pilapil (Erlinda); and
ƒ the other nephews and nieces of Donata, in
HELD # 3: YES. representation of her two other sisters who had also
x Neither are we inclined to lend credence to the petitioners’ contention that Elise passed away.
has not shown any interest in the Petition for Letters of Administration. o Respondents, on the other hand, are the heirs of the late Maximino
x Section 6, Rule 78 of the Revised Rules of Court lays down the preferred persons Briones (Maximino), composed of
who are entitled to the issuance of letters of administration, thus: ƒ his nephews and nieces, and
o Sec. 6. When and to whom letters of administration granted. — If ƒ grandnephews and grandnieces, in representation of
no executor is named in the will, or the executor or executors are the deceased siblings of Maximino
incompetent, refuse the trust, or fail to give bond, or a person dies x Antecedent facts:
intestate, administration shall be granted: o Deceased Maximino was married to Donata BUT they had NO
ƒ (a) To the surviving husband or wife, as the case may children
be, or next of kin, or both, in the discretion of the o Maximino died
court, or to such person as such surviving husband or o Donata instituted intestate proceedings to settle her husband’s
wife, or next of kin, requests to have appointed, if estate before CFI Cebu
competent and willing to serve; o CFI Æ appointed Donata as administratix
ƒ (b) If such surviving husband or wife, as the case o CFI Æ then awarded ownership over the properties to Donata
may be, or next of kin, or the person selected by o Donata died
them, be incompetent or unwilling, or if the husband o Erlinda (Donata’s niece) Æ instituted with the RTC a petition for the
or widow, or next of kin, neglects for thirty (30) days administration of the intestate estate of Donata
after the death of the person to apply for o Erlinda and husband Æ appointed as administrators
administration or to request that administration be o Erlinda Æ then claimed ownership over the properties
granted to some other person, it may be granted to o Silverio Briones (Silverio), a nephew of Maximino Æ filed a Petition
one or more of the principal creditors, if competent with the RTC for Letters of Administration for the intestate estate of
and willing to serve; Maximin
ƒ (c) If there is no such creditor competent and willing ƒ RTC Æinitially granted BUT set aside
to serve, it may be granted to such other person as o Heirs of Maximino Æ filed a Complaint with the RTC against the
the court may select. heirs of Donata for the partition, annulment, and recovery of
x Upon the other hand, Section 2 of Rule 79 provides that a petition for Letters of possession of real property
Administration must be filed by an interested person, thus: ƒ RTC Æ decided in favor of Heirs of Maximino
o Sec. 2. Contents of petition for letters of administration. — A ƒ CA Æ affirmed RTC
petition for letters of administration must be filed by an interested ƒ SC Æ reversed CA (this is the decision subject of the
person and must show, so far as known to the petitioner: present MR)
ƒ (a) The jurisdictional facts;
ƒ (b) The names, ages, and residences of the heirs, and ISSUE # 1: Whether Donata’s misrepresentations constituted fraud.
the names and residences of the creditors, of the
decedent; HELD # 1: NO.
ƒ (c) The probable value and character of the property x As this Court declared in its Decision, the existence of any trust relations
of the estate; between petitioners and respondents shall be examined in the light of Article
ƒ (d) The name of the person for whom letters of 1456 of the New Civil Code, which provides that, "[i]f property is acquired
administration are prayed. through mistake or fraud, the person obtaining it is, by force of law, considered a
o But no defect in the petition shall render void the issuance of trustee of an implied trust for the benefit of the person from whom the property
letters of administration. comes." Hence, the foremost question to be answered is still whether an implied
x An "interested party," in estate proceedings, is one who would be trust under Article 1456 of the New Civil Code had been sufficiently established
benefited in the estate, such as an heir, or one who has a claim against in the present case.
the estate, such as a creditor. Also, in estate proceedings, the phrase x In the Decision, this Court ruled in the negative, since there was insufficient
"next of kin" refers to those whose relationship with the decedent Is evidence to establish that Donata committed fraud. It should be remembered
such that they are entitled to share in the estate as distributees.28 that Donata was able to secure certificates of title to the disputed properties by
x In the instant case, Elise, as a compulsory heir who stands to be virtue of the CFI Order in Special Proceedings No. 928-R (the proceedings she
benefited by the distribution of Eliseo’s estate, is deemed to be an instituted to settle Maximino’s intestate estate), which declared her as
interested party. With the overwhelming evidence on record produced Maximino’s sole heir. In the absence of proof to the contrary, the Court accorded
by Elise to prove her filiation to Eliseo, the petitioners’ pounding on her to Special Proceedings No. 928-R the presumptions of regularity and validity.
lack of interest in the administration of the decedent’s estate, is just a Reproduced below are the relevant portions15 of the Decision -
desperate attempt to sway this Court to reverse the findings of the o At the onset, it should be emphasized that Donata was able to
Court of Appeals. Certainly, the right of Elise to be appointed secure the TCTs covering the real properties belonging to the
administratix of the estate of Eliseo is on good grounds. It is founded estate of Maximino by virtue of a CFI Order, dated 2 October 1952.
on her right as a compulsory heir, who, under the law, is entitled to her It is undisputed that the said CFI Order was issued by the CFI in
legitimate after the debts of the estate are satisfied.29 Having a vested Special Proceedings No. 928-R, instituted by Donata herself, to
right in the distribution of Eliseo’s estate as one of his natural children, settle the intestate estate of Maximino. The petitioners, heirs of
Elise can rightfully be considered as an interested party within the Donata, were unable to present a copy of the CFI Order, but this is
purview of the law. not surprising considering that it was issued 35 years prior to the
filing by the heirs of Maximino of their Complaint in Civil Case No.
96
RECTO, GAYLE ANGELI M.
2011-0008 | AUSL
Personal Notes on Remedial Law 2 Review (based on the syllabus of Prof. Henedino M. Brondial)

CEB-5794 on 3 March 1987. The existence of such CFI Order, Ortiz that she was the nearest surviving relative of the deceased
nonetheless, cannot be denied. It was recorded in the Primary Maximino Suico Briones at the time of the latter’s death, and
Entry Book of the Register of Deeds on 27 June 1960, at 1:10 p.m., pursuant to the pertinent provisions of the new Civil Code of the
as Entry No. 1714. It was annotated on the TCTs covering the real Philippines, the Court hereby declares the aforesaid Donata G. Ortiz
properties as having declared Donata the sole, absolute, and the sole, absolute and exclusive heir of the estate of the deceased
exclusive heir of Maximino. The non-presentation of the actual CFI Maximino Suico Briones, and she is hereby entitled to inherit all the
Order was not fatal to the cause of the heirs of Donata considering residue of this estate after paying all the obligations thereof, which
that its authenticity and contents were never questioned. The properties are those contained in the Inventory, dated October 2,
allegation of fraud by the heirs of Maximino did not pertain to the 1952.1awphi1.net
CFI Order, but to the manner or procedure by which it was issued x From the contents of the afore-quoted Order, this Court is able to deduce that
in favor of Donata. Moreover, the non-presentation of the CFI the CFI Order was in fact issued on 15 January 1960 and not 2 October 1952, as
Order, contrary to the declaration by the RTC, does not amount to earlier stated in the Decision. It was the inventory of properties, submitted by
a willful suppression of evidence that would give rise to the Donata as administratrix of Maximino’s intestate estate, which was dated 2
presumption that it would be adverse to the heirs of Donata if October 1952.18 Other than such observation, this Court finds nothing in the CFI
produced. x x x. Order which could change its original position in the Decision under
o The CFI Order, dated 2 October 1952, issued in Special consideration.
Proceedings No. 928-R, effectively settled the intestate estate of x While it is true that since the CFI was not informed that Maximino still
Maximino by declaring Donata as the sole, absolute, and exclusive had surviving siblings and so the court was not able to order that these
heir of her deceased husband. The issuance by the CFI of the said siblings be given personal notices of the intestate proceedings, it
Order, as well as its conduct of the entire Special Proceedings No. should be borne in mind that the settlement of estate, whether testate
928-R, enjoy the presumption of validity pursuant to the Section or intestate, is a proceeding in rem,19 and that the publication in the
3(m) and (n) of Rule 131 of the Revised Rules of Court, reproduced newspapers of the filing of the application and of the date set for the
below - hearing of the same, in the manner prescribed by law, is a notice to the
ƒ SEC. 3. Disputable presumptions. - The following whole world of the existence of the proceedings and of the hearing on
presumptions are satisfactory if uncontradicted, but the date and time indicated in the publication. The publication
may be contradicted and overcome by other requirement of the notice in newspapers is precisely for the purpose of
evidence: informing all interested parties in the estate of the deceased of the
x (m) That official duty has been regularly existence of the settlement proceedings, most especially those who
performed; were not named as heirs or creditors in the petition, regardless of
x (n) That a court, or judge acting as whether such omission was voluntarily or involuntarily made.
such, whether in the Philippines or x This Court cannot stress enough that the CFI Order was the result of the
elsewhere, was acting in the lawful intestate proceedings instituted by Donata before the trial court. As this Court
exercise of jurisdiction. pointed out in its earlier Decision, the manner by which the CFI judge conducted
o By reason of the foregoing provisions, this Court must presume, in the proceedings enjoys the presumption of regularity, and encompassed in such
the absence of any clear and convincing proof to the contrary, that presumption is the order of publication of the notice of the intestate proceedings.
the CFI in Special Proceedings No. 928-R had jurisdiction of the A review of the records fails to show any allegation or concrete proof that the
subject matter and the parties, and to have rendered a judgment CFI also failed to order the publication in newspapers of the notice of the
valid in every respect; and it could not give credence to the intestate proceedings and to require proof from Donata of compliance therewith.
following statements made by the Court of Appeals in its Decision. Neither can this Court find any reason or explanation as to why Maximino’s
x There was totally no evidentiary basis for the foregoing pronouncements. First of siblings could have missed the published notice of the intestate proceedings of
all, the Petition filed by Donata for Letters of Administration in Special their brother.
Proceedings No. 928-R before the CFI was not even referred to nor presented x In relying on the presumptions of the regular performance of official duty and
during the course of the trial of Civil Case No. CEB-5794 before the RTC. How lawful exercise of jurisdiction by the CFI in rendering the questioned Order,
then could the Court of Appeals make a finding that Donata willfully excluded dated 15 January 1960, this Court is not, as counsel for respondents allege,
from the said Petition the names, ages, and residences of the other heirs of sacrificing the substantive right of respondents to their share in the inheritance in
Maximino? Second, there was also no evidence showing that the CFI actually favor of mere procedural fiats. There is a rationale for the establishment of rules
failed to send notices of Special Proceedings No. 928-R to the heirs of Maximino of procedure, as amply explained by this Court in De Dios v. Court of Appeals20
or that it did not require presentation of proof of service of such notices. It o Procedural rules are designed to insure the orderly and expeditious
should be remembered that there stands a presumption that the CFI Judge had administration of justice by providing for a practical system by
regularly performed his duties in Special Proceedings No. 928-R, which included which the parties to a litigation may be accorded a full and fair
sending out of notices and requiring the presentation of proof of service of such opportunity to present their respective positions and refute each
notices; and, the heirs of Maximino did not propound sufficient evidence to other's submissions under the prescribed requirements, conditions
debunk such presumption. They only made a general denial of knowledge of and limitations. Adjective law is not the counterfoil of substantive
Special Proceedings No. 928-R, at least until 1985. There was no testimony or law. In fact, there is a symbiotic relationship between them. By
document presented in which the heirs of Maximino categorically denied receipt complying faithfully with the Rules of Court, the bench and the bar
of notice from the CFI of the pendency of Special Proceedings No. 928-R. The are better able to discuss, analyze and understand substantive
only evidence on record in reference to the absence of notice of such rights and duties and consequently to more effectively protect and
proceedings was the testimony of Aurelia Briones (Aurelia), one of the heirs of enforce them. The other alternative is judicial anarchy.
Maximino, x x x. o Thus, compliance with the procedural rules is the general rule, and
x Aurelia’s testimony deserves scant credit considering that she was not testifying abandonment thereof should only be done in the most exceptional
on matters within her personal knowledge. The phrase "I don’t think" is a clear circumstances. The presumptions relied upon by this Court in the
indication that she is merely voicing out her opinion on how she believed her instant case are disputable presumptions, which are satisfactory,
uncles and aunts would have acted had they received notice of Special unless contradicted or overcome by evidence. This Court finds that
Proceedings No. 928-R. the evidence presented by respondents failed to overcome the
x It is worth noting that, in its foregoing ratiocination, the Court was proceeding given presumptions.
from an evaluation of the evidence on record, which did not include an actual x Although Donata may have alleged before the CFI that she was her husband’s
copy of the CFI Order in Special Proceedings No. 928-R. Respondents only sole heir, it was not established that she did so knowingly, maliciously and in bad
submitted a certified true copy thereof on 15 June 2006, annexed to their faith, so as for this Court to conclude that she indeed committed fraud. This
Supplemental Reply to petitioners’ opposition to their motion for reconsideration Court again brings to the fore the delay by which respondents filed the present
of this Court’s Decision. Respondents did not offer any explanation as to why case, when the principal actors involved, particularly, Donata and Maximino’s
they belatedly produced a copy of the said Order, but merely claimed to have siblings, have already passed away and their lips forever sealed as to what truly
been "fortunate enough to obtain a copy" thereof from the Register of Deeds of transpired between them. On the other hand, Special Proceedings No. 928-R
Cebu.16 took place when all these principal actors were still alive and each would have
x Respondents should be taken to task for springing new evidence so late into the been capable to act to protect his or her own right to Maximino’s estate. Letters
proceedings of this case. Parties should present all their available evidence at the of Administration of Maximino’s estate were issued in favor of Donata as early as
courts below so as to give the opposing party the opportunity to scrutinize and 8 July 1952, and the CFI Order in question was issued only on 15 January 1960.
challenge such evidence during the course of the trial. However, given that the The intestate proceedings for the settlement of Maximino’s estate were thus
existence of the CFI Order in Special Proceedings No. 928-R was never in issue pending for almost eight years, and it is the burden of the respondents to
and was, in fact, admitted by the petitioners; that the copy submitted is a establish that their parents or grandparents, Maximino’s surviving siblings, had
certified true copy of the said Order; and that the said Order may provide new absolutely no knowledge of the said proceedings all these years. As established
information vital to a just resolution of the present case, this Court is compelled in Ramos v. Ramos,21 the degree of proof to establish fraud in a case where the
to consider the same as part of the evidence on record. principal actors to the transaction have already passed away is proof beyond
x The CFI Order17 in question reads in full as - reasonable doubt, to wit -
o This is with reference to the Motion of the Administratrix, dated o "x x x But length of time necessarily obscures all human evidence;
January 5, 1960, that she be declared the sole heir of her deceased and as it thus removes from the parties all the immediate means to
husband, Maximino Suico Briones, the latter having died without verify the nature of the original transactions, it operates by way of
any legitimate ascendant nor descendant, nor any legitimate presumption, in favor of innocence, and against imputation of
brother or sister, nephews or nieces. fraud. It would be unreasonable, after a great length of time, to
o At the hearing of this incident today, nobody appeared to resist the require exact proof of all the minute circumstances of any
motion, and based on the uncontradicted testimony of Donata G. transaction, or to expect a satisfactory explanation of every
97
RECTO, GAYLE ANGELI M.
2011-0008 | AUSL
Personal Notes on Remedial Law 2 Review (based on the syllabus of Prof. Henedino M. Brondial)

difficulty, real or apparent, with which it may be encumbered. The he does not acquire by prescription the property held in trust.
most that can fairly be expected, in such cases, if the parties are Thus, Section 38 of Act 190 provides that the law of prescription
living, from the frailty of memory, and human infirmity, is, that the does not apply "in the case of a continuing and subsisting trust"
material facts can be given with certainty to a common intent; and, (Diaz vs. Gorricho and Aguado, 103 Phil. 261, 266; Laguna vs.
if the parties are dead, and the cases rest in confidence, and in Levantino, 71 Phil. 566; Sumira vs. Vistan, 74 Phil. 138; Golfeo vs.
parol agreements, the most that we can hope is to arrive at Court of Appeals, 63 O.G. 4895, 12 SCRA 199; Caladiao vs. Santos,
probable conjectures, and to substitute general presumptions of 63 O.G. 1956, 10 SCRA 691).
law, for exact knowledge. Fraud, or breach of trust, ought not o The rule of imprescriptibility of the action to recover property held
lightly to be imputed to the living; for, the legal presumption is the in trust may possibly apply to resulting trusts as long as the trustee
other way; as to the dead, who are not here to answer for has not repudiated the trust (Heirs of Candelaria vs. Romero, 109
themselves, it would be the height of injustice and cruelty, to Phil. 500, 502-3; Martinez vs. Graño, 42 Phil. 35; Buencamino vs.
disturb their ashes, and violate the sanctity of the grave, unless the Matias, 63 O. G. 11033, 16 SCRA 849).
evidence of fraud be clear, beyond a reasonable doubt (Prevost vs. x
Gratz, 6 Wheat. [U.S.], 481, 498). o The rule of imprescriptibility was misapplied to constructive trusts
x Moreover, even if Donata’s allegation that she was Maximino’s sole heir does (Geronimo and Isidoro vs. Nava and Aquino, 105 Phil. 145, 153.
constitute fraud, it is insufficient to justify abandonment of the CFI Order, dated Compare with Cuison vs. Fernandez and Bengzon, 105 Phil. 135,
15 January 1960,22 considering the nature of intestate proceedings as being in 139; De Pasion vs. De Pasion, 112 Phil. 403, 407).
rem and the disputable presumptions of the regular performance of official duty o Acquisitive prescription may bar the action of the beneficiary
and lawful exercise of jurisdiction by the CFI in rendering the questioned Order, against the trustee in an express trust for the recovery of the
dated 15 January 1960, in Special Proceedings No. 928-R. property held in trust where (a) the trustee has performed
unequivocal acts of repudiation amounting to an ouster of the
ISSUE # 2: Whether the right of the Heirs of Maximino to recover their shares from the latter’s cestui qui trust; (b) such positive acts of repudiation have been
estate is imprescriptible. made known to the cestui qui trust and (c) the evidence thereon is
clear and conclusive (Laguna vs. Levantino, supra; Salinas vs.
HELD # 2: NO. Tuason, 55 Phil. 729. Compare with the rule regarding co-owners
x Assuming, for the sake of argument, that Donata’s misrepresentation constitutes found in the last paragraph of Article 494, Civil Code; Casañas vs.
fraud that would impose upon her the implied trust provided in Article 1456 of Rosello, 50 Phil. 97; Gerona vs. De Guzman, L-19060, May 29,
the Civil Code, this Court still cannot sustain respondents’ contention that their 1964, 11 SCRA 153, 157).
right to recover their shares in Maximino’s estate is imprescriptible. It is already o With respect to constructive trusts, the rule is different. The
settled in jurisprudence that an implied trust, as opposed to an express trust, is prescriptibility of an action for reconveyance based on constructive
subject to prescription and laches. trust is now settled (Alzona vs. Capunitan, L-10228, February 28,
x The case of Ramos v. Ramos23 already provides an elucidating discourse on the 1962, 4 SCRA 450; Gerona vs. De Guzman, supra; Claridad vs.
matter, to wit - Henares, 97 Phil. 973; Gonzales vs. Jimenez, L-19073, January 30,
o "Trusts are either express or implied. Express trusts are created by 1965, 13 SCRA 80; Boñaga vs. Soler, 112 Phil. 651; J. M. Tuason &
the intention of the trustor or of the parties. Implied trusts come Co., vs. Magdangal, L-15539, January 30, 1962, 4 SCRA 84).
into being by operation of law" (Art. 1441, Civil Code). "No express Prescription may supervene in an implied trust (Bueno vs. Reyes, L-
trusts concerning an immovable or any interest therein may be 22587, April 28, 1969, 27 SCRA 1179; Fabian vs. Fabian, L-20449,
proven by oral evidence. An implied trust may be proven by oral January 29, 1968; Jacinto vs. Jacinto, L-17957, May 31, 1962, 5
evidence" (Ibid; Arts. 1443 and 1457). SCRA 371).
o "No particular words are required for the creation of an express o And whether the trust is resulting or constructive, its enforcement
trust, it being sufficient that a trust is clearly intended" (Ibid; Art. may be barred by laches (90 C.J.S. 887-889; 54 Am Jur. 449-450;
1444; Tuason de Perez vs. Caluag, 96 Phil. 981; Julio vs. Dalandan, Diaz vs. Gorricho and Aguado, supra; Compare with Mejia vs.
L-19012, October 30, 1967, 21 SCRA 543, 546). "Express trusts are Gampona, 100 Phil. 277). [Emphases supplied.]
those which are created by the direct and positive acts of the x A present reading of the Quion24 and Sevilla25 cases, invoked by respondents,
parties, by some writing or deed, or will, or by words either must be made in conjunction with and guided accordingly by the principles
expressly or impliedly evincing an intention to create a trust" (89 established in the afore-quoted case. Thus, while respondents’ right to
C.J. S. 122). inheritance was transferred or vested upon them at the time of Maximino’s death,
o "Implied trusts are those which, without being expressed, are their enforcement of said right by appropriate legal action may be barred by the
deducible from the nature of the transaction as matters of intent, prescription of the action.
or which are superinduced on the transaction by operation of law x Prescription of the action for reconveyance of the disputed properties based on
as matters of equity, independently of the particular intention of implied trust is governed by Article 1144 of the New Civil Code, which reads -
the parties" (89 C.J.S. 724). They are ordinarily subdivided into o ART. 1144. The following actions must be brought within ten years
resulting and constructive trusts (89 C.J.S. 722). from the time the right of action accrues:
o "A resulting trust is broadly defined as a trust which is raised or ƒ (1) Upon a written contract;
created by the act or construction of law, but in its more restricted ƒ (2) Upon an obligation created by law;
sense it is a trust raised by implication of law and presumed always ƒ (3) Upon a judgment.
to have been contemplated by the parties, the intention as to which x Since an implied trust is an obligation created by law (specifically, in this case, by
is to be found in the nature of their transaction, but not expressed in Article 1456 of the New Civil Code), then respondents had 10 years within which
the deed or instrument of conveyance" (89 C.J.S. 725). to bring an action for reconveyance of their shares in Maximino’s properties. The
Examples of resulting trusts are found in Article 1448 to 1455 of next question now is when should the ten-year prescriptive period be reckoned
the Civil Code. See Padilla vs. Court of Appeals, L-31569, from. The general rule is that an action for reconveyance of real property based
September 28, 1973, 53 SCRA 168, 179). on implied trust prescribes ten years from registration and/or issuance of the title
o On the other hand, a constructive trust is a trust "raised by to the property,26 not only because registration under the Torrens system is a
construction of law, or arising by operation of law." In a more constructive notice of title,27 but also because by registering the disputed
restricted sense and as contradistinguished from a resulting trust, a properties exclusively in her name, Donata had already unequivocally repudiated
constructive trust is "a trust not created by any words, either any other claim to the same.
expressly or impliedly evincing a direct intention to create a trust, x By virtue of the CFI Order, dated 15 January 1960, in Special Proceedings No.
but by the construction of equity in order to satisfy the demands of 928-R, Donata was able to register and secure certificates of title over the
justice. It does not arise by agreement or intention but by disputed properties in her name on 27 June 1960. The respondents filed with the
operation of law." (89 C.J.S. 726-727). "If a person obtains legal RTC their Complaint for partition, annulment, and recovery of possession of the
title to property by fraud or concealment, courts of equity will disputed real properties, docketed as Civil Case No. CEB-5794, only on 3 March
impress upon the title a so-called constructive trust in favor of the 1987, almost 27 years after the registration of the said properties in the name of
defrauded party." A constructive trust is not a trust in the technical Donata. Therefore, respondents’ action for recovery of possession of the
sense (Gayondato vs. Treasurer of the P.I., 49 Phil. 244; See Art. disputed properties had clearly prescribed.
1456, Civil Code). x Moreover, even though respondents’ Complaint before the RTC in Civil Case No.
o There is a rule that a trustee cannot acquire by prescription the CEB-5794 also prays for partition of the disputed properties, it does not make
ownership of property entrusted to him (Palma vs. Cristobal, 77 their action to enforce their right to the said properties imprescriptible. While as
Phil. 712), or that an action to compel a trustee to convey property a general rule, the action for partition among co-owners does not prescribe so
registered in his name in trust for the benefit of the cestui qui trust long as the co-ownership is expressly or impliedly recognized, as provided for in
does not prescribe (Manalang vs. Canlas, 94 Phil. 776; Cristobal vs. Article 494, of the New Civil Code, it bears to emphasize that Donata had never
Gomez, 50 Phil. 810), or that the defense of prescription cannot be recognized respondents as co-owners or co-heirs, either expressly or impliedly.28
set up in an action to recover property held by a person in trust for Her assertion before the CFI in Special Proceedings No. 928-R that she was
the benefit of another (Sevilla vs. De los Angeles, 97 Phil. 875), or Maximino’s sole heir necessarily excludes recognition of some other co-owner or
that property held in trust can be recovered by the beneficiary co-heir to the inherited properties; Consequently, the rule on non-prescription of
regardless of the lapse of time (Marabilles vs. Quito, 100 Phil. 64; action for partition of property owned in common does not apply to the case at
Bancairen vs. Diones, 98 Phil. 122, 126; Juan vs. Zuñiga, 62 O.G. bar.
1351; 4 SCRA 1221; Jacinto vs. Jacinto, L-17957, May 31, 1962.
See Tamayo vs. Callejo, 147 Phil. 31, 37). ISSUE # 3: Whether such right of the Heirs of Maximino is likewise limited by the principle of
o That rule applies squarely to express trusts. The basis of the rule is laches.
that the possession of a trustee is not adverse. Not being adverse,
98
RECTO, GAYLE ANGELI M.
2011-0008 | AUSL
Personal Notes on Remedial Law 2 Review (based on the syllabus of Prof. Henedino M. Brondial)

HELD # 3: YES. it will be sustained by the ordinary presumptions of regularity, and it


x Other than prescription of action, respondents’ right to recover possession of the is not open to impeachment in any collateral action. * * *"
disputed properties, based on implied trust, is also barred by laches. The defense o But it is otherwise when the judgment is void. "A void judgment is
of laches, which is a question of inequity in permitting a claim to be enforced, in legal effect no judgment. By it no rights are divested. From it no
applies independently of prescription, which is a question of time. Prescription is rights can be obtained. Being worthless in itself, all proceedings
statutory; laches is equitable.29 founded upon it are equally worthless. It neither binds nor bars any
x Laches is defined as the failure to assert a right for an unreasonable and one. All acts performed under it and all claims flowing out of it are
unexplained length of time, warranting a presumption that the party entitled to void. The parties attempting to enforce it may be responsible as
assert it has either abandoned or declined to assert it. This equitable defense is trespassers. The purchaser at a sale by virtue of its authority finds
based upon grounds of public policy, which requires the discouragement of stale himself without title and without redress." (Freeman on Judgments,
claims for the peace of society.30 sec. 117, citing Campbell vs. McCahan, 41 Ill., 45; Roberts vs.
x This Court has already thoroughly discussed in its Decision the basis for barring Stowers, 7 Bush, 295, Huls vs. Buntin, 47 Ill., 396; Sherrell vs.
respondents’ action for recovery of the disputed properties because of laches. Goodrum, 3 Humph., 418; Andrews vs. State, 2 Sneed, 549;
This Court pointed out therein31 that - Hollingsworth vs. Bagley, 35 Tex., 345; Morton vs. Root, 2 Dill.,
312; Commercial Bank of Manchester vs. Martin, 9 Smedes & M.,
o In further support of their contention of fraud by Donata, the heirs 613; Hargis vs. Morse, 7 Kan., 259. See also Cornell vs. Barnes, 7
of Maximino even emphasized that Donata lived along the same Hill, 35; Dawson and Another vs. Wells, 3 Ind., 399; Meyer vs.
street as some of the siblings of Maximino and, yet, she failed to Mintonye, 106 Ill., 414; Olson vs. Nunnally, 47 Kan., 391; White vs.
inform them of the CFI Order, dated [15 January 1960], in Special Foote L. & M. Co., 29 W. Va., 385.)
Proceedings No. 928-R, and the issuance in her name of new TCTs o It is not always easy to draw the line of demarcation between a
covering the real properties which belonged to the estate of void judgment and a voidable one, but all authorities agree that
Maximino. This Court, however, appreciates such information jurisdiction over the subject-matter is essential to the validity of a
differently. It actually works against the heirs of Maximino. Since judgment and that want of such jurisdiction renders it void and a
they only lived nearby, Maximino’s siblings had ample opportunity mere nullity. In the eye of the law it is non-existent. (Fisher vs.
to inquire or discuss with Donata the status of the estate of their Harnden, 1 Paine, 55; Towns vs. Springer, 9 Ga., 130; Mobley vs.
deceased brother. Some of the real properties, which belonged to Mobley, 9 Ga., 247; Beverly and McBride vs. Burke, 9 Ga., 440;
the estate of Maximino, were also located within the same area as Central Bank of Georgia vs. Gibson, 11 Ga., 453; Johnson vs.
their residences in Cebu City, and Maximino’s siblings could have Johnson, 30 Ill., 215; St. Louis and Sandoval Coal and Mining Co.
regularly observed the actions and behavior of Donata with regard vs. Sandoval Coal and Mining Co., 111 Ill., 32; Swiggart vs. Harber,
to the said real properties. It is uncontested that from the time of 4 Scam., 364; Miller vs. Snyder, 6 Ind., 1; Seely vs. Reid, 3 Greene
Maximino’s death on 1 May 1952, Donata had possession of the [Iowa], 374.)34
real properties. She managed the real properties and even x The fraud and misrepresentation fostered by Donata on the CFI in Special
collected rental fees on some of them until her own death on 1 Proceedings No. 928-R did not deprive the trial court of jurisdiction over the
November 1977. After Donata’s death, Erlinda took possession of subject-matter of the case, namely, the intestate estate of Maximino. Donata’s
the real properties, and continued to manage the same and collect fraud and misrepresentation may have rendered the CFI Order, dated 15 January
the rental fees thereon. Donata and, subsequently, Erlinda, were so 1960, voidable, but not void on its face. Hence, the said Order, which already
obviously exercising rights of ownership over the real properties, in became final and executory, can only be set aside by direct action to annul and
exclusion of all others, which must have already put the heirs of enjoin its enforcement.35 It cannot be the subject of a collateral attack as is
Maximino on guard if they truly believed that they still had rights being done in this case. Note that respondents’ Complaint before the RTC in Civil
thereto. Case No. CEB-5794 was one for partition, annulment, and recovery of possession
o The heirs of Maximino knew he died on 1 May 1952. They even of the disputed properties. The annulment sought in the Complaint was not that
attended his wake. They did not offer any explanation as to why of the CFI Order, dated 15 January 1960, but of the certificates of title over the
they had waited 33 years from Maximino’s death before one of properties issued in Donata’s name. So until and unless respondents bring a
them, Silverio, filed a Petition for Letters of Administration for the direct action to nullify the CFI Order, dated 15 January 1960, in Special
intestate estate of Maximino on 21 January 1985. After learning Proceedings No. 928-R, and attain a favorable judgment therein, the assailed
that the intestate estate of Maximino was already settled in Special Order remains valid and binding.
Proceedings No. 928-R, they waited another two years, before x Nonetheless, this Court also points out that an action to annul an order or
instituting, on 3 March 1987, Civil Case No. CEB-5794, the judgment based on fraud must be brought within four years from the discovery
Complaint for partition, annulment and recovery of the real of the fraud.36 If it is conceded that the respondents came to know of Donata’s
property belonging to the estate of Maximino. x x x fraudulent acts only in 1985, during the course of the RTC proceedings which
x Considering the circumstances in the afore-quoted paragraphs, as well as they instituted for the settlement of Maximino’s estate, then their right to file an
respondents’ conduct before this Court, particularly the belated submission of action to annul the CFI Order, dated 15 January 1960, in Special Proceedings No.
evidence and argument of new issues, respondents are consistently displaying a 928-R (earlier instituted by Donata for the settlement of Maximino’s estate), has
penchant for delayed action, without any proffered reason or justification for likewise prescribed by present time.
such delay.
x It is well established that the law serves those who are vigilant and diligent and
not those who sleep when the law requires them to act. The law does not RODOLFO C. SABIDONG, Complainant, vs. NICOLASITO S. SOLAS (Clerk of Court IV),
encourage laches, indifference, negligence or ignorance. On the contrary, for a Respondent.
party to deserve the considerations of the courts, he must show that he is not A.M. No. P-01-1448 June 25, 2013
guilty of any of the aforesaid failings EN BANC

ISSUE # 4: Whether the subject CFI Order dated January 15, 1960 declaring Donata as the FACTS:
sole heir of Maximino is VOID. x Trinidad Sabidong, complainant Rodolfo’s mother, is one of the longtime
occupants of the subject property
HELD # 4: NO. x The said lot was part of the and subject of an ejectment suit filed by the Hodges
x Respondents presented only in their Reply and Supplemental Reply to the Estate which was pending before the MTCC Iloilo Br 4 (where Solas was the CoC)
petitioners’ Opposition to their Motion for Reconsideration the argument that the o Sabidongs Æ among the named defendants therein
CFI Order, dated 15 January 1960, in Special Proceedings No. 928-R is void and, x MTCC Æ ordered defendants to vacate and pay
thus, it cannot have any legal effect. Consequently, the registration of the x Sola Æsubmitted an Offer to Purchase on installment Lots 11 and 12
disputed properties in the name of Donata pursuant to such Order was likewise o Hodges Estate Æ rejected
void. ƒ in view of an application to purchase already filed by
o This Court is unconvinced. the actual occupant of Lot 12, "in line with the policy
x In the jurisprudence referred to by the respondents,33 an order or judgment is of the Probate Court to give priority to the actual
considered void when rendered by the court without or in excess of its occupants in awarding approval of Offers".
jurisdiction or in violation of a mandatory duty, circumstances which are not x Sola Æsubmitted another offer Purchase on installment Lot 11
present in the case at bar. o approved upon the court’s observation that the occupants of the
x Distinction must be made between a void judgment and a voidable one, thus - subject lots "have not manifested their desire to purchase the lots
o "* * * A voidable judgment is one which, though not a mere nullity, they are occupying up to this date and considering time restraint
is liable to be made void when a person who has a right to proceed and considering further, that the sales in favor of the x x x offerors
in the matter takes the proper steps to have its invalidity declared. are most beneficial to the estate x x x".
It always contains some defect which may become fatal. It carries x Lot 11 was subsequently conveyed to Sola for the total purchase price of
within it the means of its own overthrow. But unless and until it is P50,000
duly annulled, it is attended with all the ordinary consequences of a o TCT was then issued
legal judgment. The party against whom it is given may escape its x MTCC Æ issued a writ of demolition (to execute its decision)
effect as a bar or an obligation, but only by a proper application to x Rodolfo Æ filed a complaint against Sola before SC
have it vacated or reversed. Until that is done, it will be efficacious o asserting that as court employee respondent cannot buy property
as a claim, an estoppel, or a source of title. If no proceedings are in litigation (consequently he is not a buyer in good faith), commit
ever taken against it, it will continue throughout its life to all intents deception, dishonesty, oppression and grave abuse of authority
a valid sentence. If emanating from a court of general jurisdiction, o The respondent Nicolasito Solas, then Clerk of Court III, MTCC,
Iloilo City, has knowledge, by reason of his position that in 1983
99
RECTO, GAYLE ANGELI M.
2011-0008 | AUSL
Personal Notes on Remedial Law 2 Review (based on the syllabus of Prof. Henedino M. Brondial)

Hodges Estate was ejecting occupants of its land. x x x Taking and terminated.37 The probate court loses jurisdiction of an estate under
advantage of this inside information that the land subject of an administration only after the payment of all the debts and the remaining estate
ejectment case in the Municipal Trial Court in Cities, Iloilo City, delivered to the heirs entitled to receive the same.38 Since there is no evidence
whom respondent is a Clerk of Court III, the respondent to show that Sp. Proc. No. 1672 in the RTC of Iloilo, Branch 27, had already
surreptitiously offered to buy the said lot in litigation been closed and terminated at the time of the execution of the Deed of Sale
o respondent met with the family of the complainant and negotiated With Mortgage dated November 21, 1994, Lot 11 is still deemed to be "in
for the sale of the property and transfer of the title in favor of the litigation" subject to the operation of Article 1491 (5) of the Civil Code.
latter. Respondent made the complainant and his family believed x This notwithstanding, we hold that the sale of Lot 11 in favor of respondent did
that he is the representative of the estate and that he needed a not violate the rule on disqualification to purchase property because Sp. Proc. No.
downpayment right away. All the while, the Sabidong family (who 1672 was then pending before another court (RTC) and not MTCC where he was
were carpenters, laundrywomen, a janitor, persons who belong to Clerk of Court.
the underprivileged) relied on the representations of the x On the charges against the respondent, we find him liable for dishonesty and
respondent that he was authorized to facilitate the sale, with more grave misconduct.
reason that respondent represented himself as the City Sheriff; x Misconduct is a transgression of some established and definite rule of action,
x Rodolfo Æ instituted criminal case for estafa AND civil case for annulment of more particularly, unlawful behavior as well as gross negligence by a public
title, damages and injunction officer. To warrant dismissal from service, the misconduct must be grave,
o BOTH dismissed serious, important, weighty, momentous and not trifling. The misconduct must
x OCA Æ found Sola guilty of violating the rule under Art 1491 CC (re prohibited imply wrongful intention and not a mere error of judgment. The misconduct
sales); recommended suspension must also have a direct relation to and be connected with the performance of the
x SC Æ referred to administrative case to Executive Judge Hortillo (RTC Iloilo) public officer’s official duties amounting either to maladministration or willful,
x EJ Æ required Sola to file memorandum intentional neglect, or failure to discharge the duties of the office.39
x Sola Æ filed memorandum x Dishonesty is the "disposition to lie, cheat, deceive, defraud or betray;
o maintained that his purchase of the subject land is not covered by untrustworthiness; lack of integrity; lack of honesty, probity, or integrity in
the prohibition in paragraph 5, Article 1491 of the Civil Code. He principle; and lack of fairness and straightforwardness."40
pointed out that he bought Lot 11-A a decade after the MTCC of x In this case, respondent deceived complainant’s family who were led to believe
Iloilo, Branch 3, had ordered the ejectment of Priscila Saplagio and that he is the legal representative of the Hodges Estate, or at least possessed of
Trinidad Sabidong from the subject lot. He insisted that public trust such power to intercede for overstaying occupants of the estate’s properties like
was observed when complainant was accorded his right of first complainant. Boasting of his position as a court officer, a City Sheriff at that,
refusal in the purchase of Lot 11-A, albeit the latter failed to avail complainant’s family completely relied on his repeated assurance that they will
said right. Asserting that he is a buyer in good faith and for value, not be ejected from the premises. Upon learning that the lot they were
respondent cited the dismissal of the cases for Estafa and occupying was for sale and that they had to negotiate for it through respondent,
annulment of title and damages which complainant filed against complainant’s family readily gave the amounts he demanded and, along with
him Saplagio, complied with the requirements for a loan application with PAG-IBIG.
x Sola then compulsory retired BUT his benefits were not released All the while and unknown to complainant’s family, respondent was actually
x OCA Æ found respondent liable for serious and grave misconduct and dishonesty working to acquire Lot 11 for himself.
and recommended the forfeiture of respondent’s salary for six months, which x Thus, while respondent was negotiating with the Hodges Estate for the sale of
shall be deducted from his retirement benefits the property to him, he collected as down payment P5,000 from complainant’s
o his unilateral acts of extinguishing the contract to sell and forfeiting family in July 1986. Four months later, on November 18, 1986, the probate court
the amounts he received from complainant and Saplagio without approved respondent’s offer to purchase Lot 11. The latter received further down
due notice, respondent failed to act with justice and equity. He payment from complainant in the amount of P10,000 between 1992 and 1993, or
found respondent’s denial to be anchored merely on the fact that before the Deed of Sale with Mortgage41 dated November 21, 1994 could be
he had not issued receipts which was belied by his admission that executed in respondent’s favor.
he had asked money for the expenses of partitioning Lot 11 from x Thereafter, respondent demanded P3,000 from complainant supposedly for the
complainant and Saplagio. Since their PAG-IBIG loan applications subdivision of Lot 11 between the latter and the Saplagios. Yet, it was not until
did not materialize, complainant should have returned the amounts respondent obtained title over said lot that the same was subdivided into Lots
given to him by complainant and Saplagio 11-A and 11-B. The records42 of the case show that the Subdivision Plan dated
April 25, 1996, duly approved by the Land Management Services (DENR)
ISSUE: Whether the subject sale is among those prohibited under Art 1491 of the CC, subdividing Lot 11 into sublots 11-A and 11-B, was inscribed on February 28,
considering the position of Sola that the subject matter thereof is “no longer in litigation”. 1997 - two years after TCT No. T-107519 covering Lot 11 was issued in
respondent’s name on December 5, 1994.
HELD: YES. x Finally, in 1995, respondent received the amount of P2,000 to defray the
x Article 1491, paragraph 5 of the Civil Code prohibits court officers such as clerks expenses for documentation and transfer of title in complainant’s name. In the
of court from acquiring property involved in litigation within the jurisdiction or latter instance, while it may be argued that respondent already had the capacity
territory of their courts. Said provision reads: to sell the subject property, the sum of all the circumstances belie an honest
o Article 1491. The following persons cannot acquire by purchase, intention on his part to convey Lot 11-A to complainant. We note the inscription in
even at a public or judicial auction, either in person or through the TCT No. T-1183643 in the name of C.N. Hodges that respondent executed a
mediation of another: Request dated February 19, 1997 "for the issuance of separate titles in the name
ƒ (5) Justices, judges, prosecuting attorneys, clerks of of the registered owner."44 Soon after, TCT No. T-11646745 covering Lot 11-A
superior and inferior courts, and other officers and and TCT No. T-11646846 covering Lot 11-B were issued in the name of
employees connected with the administration of respondent on February 28, 1997 - only eight months after he executed the
justice, the property and rights in litigation or levied Contract to Sell47 in favor of complainant on June 3, 1996.
upon an execution before the court within whose x Respondent’s bare denials were correctly disregarded by the Court Administrator
jurisdiction or territory they exercise their respective in the light of his own admission that he indeed asked money from both
functions; this prohibition includes the act of acquiring complainant and Saplagio. The evidence on record clearly established that by
by assignment and shall apply to lawyers, with misrepresenting himself as the estate’s representative and as a court officer
respect to the property and rights which may be the having the power to protect complainant’s family from eviction, respondent was
object of any litigation in which they may take part by able to collect sums totaling P20,000 from complainant’s family. Even after the
virtue of their profession. latter realized they were duped since respondent was already the owner of Lot
x The rationale advanced for the prohibition is that public policy disallows the 11, they still offered to buy the property from him. Respondent, however,
transactions in view of the fiduciary relationship involved, i.e., the relation of changed his mind and no longer wanted to sell the property after nothing
trust and confidence and the peculiar control exercised by these persons.32 "In happened to the loan applications of complainant and Saplagio. This subsequent
so providing, the Code tends to prevent fraud, or more precisely, tends not to unilateral cancellation by respondent of the contract to sell with complainant may
give occasion for fraud, which is what can and must be done."33 have been an afterthought, and plainly unjustified, based merely on his own
x For the prohibition to apply, the sale or assignment of the property assumption that complainant could not make full payment. But it did not negate
must take place during the pendency of the litigation involving the the deception and fraudulent acts perpetrated against complainant’s family who
property.34 Where the property is acquired after the termination of the case, were forced into submission by the constant threat of eviction. Such acts
no violation of paragraph 5, Article 1491 of the Civil Code attaches.35 constitute grave misconduct for which respondent should be held answerable.
x In the case at bar, when respondent purchased Lot 11-A on November 21, 1994, x Since respondent had compulsorily retired from service on September 10, 2007,
the Decision in Civil Case No. 14706 which was promulgated on May 31, 1983 for this additional administrative case he should be fined in an amount equivalent
had long become final. Be that as it may, it can not be said that the to his salary for six months which shall likewise be deducted from his retirement
property is no longer "in litigation" at that time considering that it was benefits.
part of the Hodges Estate then under settlement proceedings (Sp. Proc.
No. 1672). B. Escheats [Rule 91]
x A thing is said to be in litigation not only if there is some contest or
litigation over it in court, but also from the moment that it becomes Section 1. When an by whom petition filed. — When a person dies intestate, seized of
subject to the judicial action of the judge.36 A property forming part of the real property in the Philippines, leaving no heir or person by law entitled to the same, the
estate under judicial settlement continues to be subject of litigation until the Solicitor General or his representative in behalf of the Republic of the Philippines, may file a
probate court issues an order declaring the estate proceedings closed and petition in the Court of First Instance of the province where the deceased last resided or in
terminated. The rule is that as long as the order for the distribution of the estate which he had estate, if he resided out of the Philippines, setting forth the facts, and praying
has not been complied with, the probate proceedings cannot be deemed closed that the estate of the deceased be declared escheated.
100
RECTO, GAYLE ANGELI M.
2011-0008 | AUSL
Personal Notes on Remedial Law 2 Review (based on the syllabus of Prof. Henedino M. Brondial)

x Petitioner further contests the CA ruling that declared as a private document said
Section 2. Order for hearing. — If the petition is sufficient in form and substance, the Deed of Donation dated January 4, 1984, despite the fact that a certified true
court, by an order reciting the purpose of the petition, shall fix a date and place for the and correct copy of the same was obtained from the Notarial Records Office,
hearing thereof, which date shall be not more than six (6) months after the entry of the order, Regional Trial Court, Cebu City on June 11, 1993 and acknowledged before Atty.
and shall direct that a copy of the order be published before the hearing at least once a week Numeriano Capangpangan, then Notary Public for Cebu.24
for six (6) successive weeks in some newspaper of general circulation published in the x Given the circumstances in this case and the contentions of the parties, we find
province, as the court shall be deem best. that no reversible error was committed by the appellate court in holding that
herein petitioner's complaint against respondent should be dismissed. The
Section 3. Hearing and judgment. — Upon satisfactory proof in open court on the date evidence on record and the applicable law indubitably favor respondent.
fixed in the order that such order has been published as directed and that the person died x Petitioner principally relies on Articles 744 and 1544 of the New Civil Code, which
intestate, seized of real or personal property in the Philippines, leaving no heir or person provide:
entitled to the same, and no sufficient cause being shown to the contrary, the court shall o Art. 744. Donations of the same thing to two or more different
adjudge that the estate of the estate of the deceased in the Philippines, after the payment of donees shall be governed by the provisions concerning the sale of
just debts and charges, shall escheat; and shall, pursuant to law, assign the personal estate to the same thing to two or more different persons.
the municipality or city where he last resided in the Philippines, and the real estate to the o Art. 1544. If the same thing should have been sold to different
municipalities or cities, respectively, in which the same is situated. If the deceased never vendees, the ownership shall be transferred to the person who may
resided in the Philippines, the whole estate may be assigned to the respective municipalities or have first taken possession thereof in good faith, if it should be
cities where the same is located. Shall estate shall be for the benefit of public schools, and movable property.
public charitable institutions and centers in said municipalities or cities. ƒ Should it be immovable property, the ownership shall
belong to the person acquiring it who in good faith
The court, at the instance of an interested party, or on its own motion, may order the first recorded it in the Registry of Property.
establishment of a permanent trust, so that the only income from the property shall be used. ƒ Should there be no inscription, the ownership shall
pertain to the person who in good faith was first in
Section 4. When and by whom claim to estate filed. — If a devisee, legatee, heir, the possession; and, in the absence thereof, to the
widow, widower, or other person entitled to such estate appears and files a claim thereto with person who presents the oldest title, provided there is
the court within five (5) years from the date of such judgment, such person shall have good faith. (Emphasis supplied.)
possession of and title to the same, or if sold, the municipality or city shall be accountable to x Petitioner claims that respondent was in bad faith when she registered the land
him for the proceeds after deducting reasonable charges for the care of the estate; but a claim in her name and, based on the abovementioned rules, he has a better right over
not made within the said time shall be forever barred. the property because he was first in material possession in good faith. However,
this allegation of bad faith on the part of Amelita Sola in acquiring the title is
1. Definition devoid of evidentiary support. For one, the execution of public documents, as in
the case of Affidavits of Adjudication, is entitled to the presumption of regularity,
2. Historical Background and Legal Basis hence convincing evidence is required to assail and controvert them.25 Second,
it is undisputed that OCT No. 3439 was issued in 1989 in the name of Amelita. It
requires more than petitioner's bare allegation to defeat the Original Certificate
3. Actions for Reversions of Title which on its face enjoys the legal presumption of regularity of
issuance.26 A Torrens title, once registered, serves as notice to the whole world.
Section 5. Other actions for escheat. — Until otherwise provided by law, actions All persons must take notice and no one can plead ignorance of its
reversion or escheat of properties alienated in violation of the Constitution or of any statute registration.27
shall be governed by this rule, except that the action shall be instituted in the province where
the land lies in whole or in part. ISSUE # 2: Whether Castorio corrected resorted to an action for reversion.

CASTORIO ALVARICO, petitioner, vs. AMELITA L. SOLA, respondent. HELD # 2: NO.


G.R. No. 138953 June 6, 2002 x Even assuming that respondent Amelita Sola acquired title to the
SECOND DIVISION disputed property in bad faith, only the State can institute reversion
proceedings under Sec. 101 of the Public Land Act.28 Thus:
FACTS: o Sec. 101.—All actions for reversion to the Government of lands of
x Petitioner Castorio Alvarico is the natural father of respondent Amelita Sola while the public domain or improvements thereon shall be instituted by
Fermina Lopez is petitioner's aunt, and also Amelita's adoptive mother the Solicitor General or the officer acting in his stead, in the proper
x Bureau of Lands approved and granted the Miscellaneous Sales Application courts, in the name of the Republic of the Philippines.
(MSA) of Fermina over the subject land in Waterfront, Cebut City x In other words, a private individual may not bring an action for reversion or any
x Fermina Æ executed a Deed of Self-Adjudication and Transfer of Rights3 over action which would have the effect of canceling a free patent and the
Lot 5 in favor of Amelita, who agreed to assume all the obligations, duties, and corresponding certificate of title issued on the basis thereof, such that the land
conditions imposed upon Fermina under the MSA covered thereby will again form part of the public domain. Only the Solicitor
x Amelita Æ then paid to the Bureau of Lands the amount of P282,900 General or the officer acting in his stead may do so.29 Since Amelita Sola's
x BL Æ issued an order approving the transfer of rights and granting the title originated from a grant by the government, its cancellation is a
amendment of the application from Fermina to Amelita matter between the grantor and the grantee.30 Clearly then, petitioner
o OCT was issued has no standing at all to question the validity of Amelita's title. It
x Castorio Æ filed an action for reconveyance against Amelita. follows that he cannot "recover" the property because, to begin with,
o He claimed that on January 4, 1984, Fermina donated the land to he has not shown that he is the rightful owner thereof.1âwphi1.nêt
him x Anent petitioner's contention that it was the intention of Fermina for Amelita to
o immediately thereafter, he took possession of the same. hold the property in trust for him, we held that if this was really the intention of
o He averred that the donation to him had the effect of withdrawing Fermina, then this should have been clearly stated in the Deed of Self-
the earlier transfer to Amelita Adjudication executed in 1983, in the Deed of Donation executed in 1984, or in a
x Amelita Æ filed Answer subsequent instrument. Absent any persuasive proof of that intention in any
o maintained that the donation to petitioner is void because Fermina written instrument, we are not prepared to accept petitioner's bare allegation
was no longer the owner of the property when it was allegedly concerning the donor's state of mind.
donated to petitioner, the property having been transferred earlier
to her.13 _________________________________
o She added that the donation was void because of lack of approval
from the Bureau of Lands, and that she had validly acquired the
GUARDIANS AND GUARDIANSHIP
land as Fermina's rightful heir.
o She also denied that she is a trustee of the land for petitioner. [RULES 92-100, as amended by AM 03-02-05-SC]
x RTC Æ in favor of Castorio
x CA Æ reversed RTC 1. Venue vs. Jurisdiction [Rule 92]
x Castorio Æ filed a Rule 45 before the SC
Section 1. Where to institute proceedings. — Guardianship of a person or estate of a
ISSUE # 1: Who between Castorio and Amelita has a better claim to the land? minor or incompetent may be instituted in the Court of First Instance of the province, or in the
justice of the peace court of the municipality, or in the municipal court chartered city where
HELD # 1: Amelita. the minor or incompetent persons resides, and if he resides in a foreign country, in the Court
x To prove she has a better claim, respondent Amelita Sola submitted a copy of of First Instance of the province wherein his property or the party thereof is situated;
OCT No. 3439 in her name and her husband's,19 a Deed of Self-Adjudication and provided, however, that where the value of the property of such minor or incompetent
Transfer of Rights20 over the property dated 1983 executed by Fermina in her exceeds that jurisdiction of the justice of the peace or municipal court, the proceedings shall
favor, and a certification from the municipal treasurer that she had been be instituted in the Court of First Instance.
declaring the land as her and her husband's property for tax purposes since
1993.21 In the City of Manila the proceedings shall be instituted in the Juvenile and Domestic Relations
x For his part, petitioner Castorio Alvarico presented a Deed of Donation22 dated Court.
January 4, 1984, showing that the lot was given to him by Fermina and
according to him, he immediately took possession in 1985 and continues in Section 2. Meaning of word "incompetent." — Under this rule, the word "incompetent"
possession up to the present.23 includes persons suffering the penalty of civil interdiction or who are hospitalized lepers,
101
RECTO, GAYLE ANGELI M.
2011-0008 | AUSL
Personal Notes on Remedial Law 2 Review (based on the syllabus of Prof. Henedino M. Brondial)

prodigals, deaf and dumb who are unable to read and write, those who are of unsound mind, (a) To make and return to the court, within three (3) months, a true and complete
even though they have lucid intervals, and persons not being of unsound mind, but by reason of inventory of all the estate, real and personal, of his ward which shall come to his possession or
age, disease, weak mind, and other similar causes, cannot, without outside aid, take care of knowledge of any other person for him;
themselves and manage their property, becoming thereby an easy prey for deceit and
exploitation. (b) To faithfully execute the duties of his trust, to manage and dispose of the estate
according to these rules for the best interests of the ward, and to provide for the proper care,
Section 3. Transfer of venue. — The court taking cognizance of a guardianship custody, and education of the ward;
proceeding, may transfer the same to the court of another province or municipality wherein the
ward has acquired real property, if he has transferred thereto his bona-fide residence, and the (c) To render a true and just account of all the estate of the ward in his hands, and
latter court shall have full jurisdiction to continue the proceedings, without requiring of all proceeds or interest derived therefrom, and of the management and disposition of the
payment of additional court fees. same, at the time designated by these rules and such other times as the courts directs, and at
the expiration of his trust to settle his accounts with the court and deliver and pay over all the
estate, effects, and moneys remaining in his hands, or due from him on such settlement, to the
person lawfully entitled thereto;
2. Appointments, Kinds, Qualifications [Rule 93]
(d) To perform all orders of the court by him to be performed.
Section 1. Who may petition for appointment of guardian for resident. — Any
relative, friend, or other person on behalf of a resident minor or incompetent who has no Section 2. When new bond may be required and old sureties discharged. —
parent or lawful guardian, or the minor himself if fourteen years of age or over, may petition Whenever it is deemed necessary, the court may require a new bond to be given by the
the court having jurisdiction for the appointment of a general guardian for the person or guardian, and may discharge the sureties on the old bond from further liability, after due
estate, or both, of such minor or incompetent. An officer of the Federal Administration of the notice to interested persons, when no injury can result therefrom to those interested in the
United States in the Philippines may also file a petition in favor of a ward thereof, and the estate.
Director of Health, in favor of an insane person who should be hospitalized, or in favor of an
isolated leper. Section 3. Bonds to be filed. Actions thereon. — Every bond given by a guardian shall be
filed in the office of the clerk of the court, and, in case of the breach of a condition thereof, may
Section 2. Contents of petition. — A petition for the appointment of a general guardian be prosecuted in the same proceeding or in a separate action for the use and benefit of the
must show, so far as known to the petitioner: ward or of any other person legally interested in the estate.

(a) The jurisdiction facts;


4. Power and Duties [Rule 96]
(b) The minority or incompetency rendering the appointment necessary or
convenient; Section 1. To what guardianship shall extend. — A guardian appointed shall have the
care and custody of the person of his ward, and the management of his estate, or the
(c) The names, ages, and residence of the relatives of the minor or incompetent, manangement of the estate only, as the case may be. The guardian of the estate of a non-
and of the person having him in their care; resident shall have the management of all the estate of the ward within the Philippines, and
no court other than that in which such guardian was appointed shall have jurisdiction over the
(d) The probable value and character of his estate; guardianship.

(e) The name of the person for whom letters of guardianship. Section 2. Guardian to pay debts of ward. — Every guardian must pay the ward's just
debts out of his personal estate and the income of his real estate, if sufficient; if not, then out
The petition shall be verified; but no defect in the petition or verification shall render void the of his real estate upon obtaining an order for the sale or encumbrance thereof.
issuance of letters of guardianship.
Section 3. Guardian to settle accounts, collect debts, and appear in actions for
Section 3. Court to set time for hearing. Notice thereof. — When a petition for the ward. — A guardian must settle all accounts of his ward, and demand, sue for, and receive all
appointment of a general guardian is filed, the court shall fix a time and place for hearing the debts due him, or may, with the approval of the court, compound for the same and give
same, and shall cause reasonable notice thereof to be given to the persons mentioned in the discharges to the debtor, on receiving a fair and just dividend of the estate and effects; and he
petition residing in the province, including the minor if above 14 years of age or the shall appear for and represent his ward in all actions and special proceedings, unless another
incompetent himself, and may direct other general or special notice thereof to be given. person be appointed for that purpose.

Section 4. Opposition to petition. — Any interested person may, by filing a written Section 4. Estate to be managed frugally, and proceeds applied to maintenance of
opposition, contest the petition on the ground of majority of the alleged minor, competency of ward. — A guardian must manage the estate of his ward frugally and without the waste, and
the alleged incompetent, or the insuitability of the person for whom letters are prayed, and may apply the income and profits thereof, so far as may be necessary, to the comfortable and
pray that the petition be dismissed, or that letters of guardianship issue to himself, or to any suitable maintenance of the ward and his family, if there be any; and if such income and
suitable person named in the opposition. profits be insufficient for that purpose, the guardian may sell or encumber the real estate,
upon being authorized by order so to do, and apply to such of the proceeds as may be
Section 5. Hearing and order for letters to issue. — At the hearing of the petition the necessary to such maintenance.
alleged in competent must be present if able to attend, and it must be shown that the
required notice has been given. Thereupon the courts shall hear the evidence of the parties in Section 5. Guardian may be authorized to join in partition proceedings after
support of their respective allegations, and, if the person in question is a minor, or hearing. — The court may authorized the guardian to join in an assent to a partition of real
incompetent it shall be appoint a suitable guardian of his person or estate, or both, with the or personal estate held by the ward jointly or in common with others, but such authority shall
powers and duties hereinafter specified. only be granted after hearing, upon such notice to relatives of the ward as the court may
direct, and a careful investigation as to the necessity and propriety of the proposed action.
Section 6. When and how guardian for non-resident appointed. Notice. — When a
person liable to be put under guardianship resides without the Philippines but the estate Section 6. Proceedings when the person suspected of embezzling or concealing
therein, any relative or friend of such person, or any one interested in his estate, in property of ward. — Upon complaint of the guardian or ward, or of any person having
expectancy or otherwise, may petition a court having jurisdiction for the appointment of a actual or prospective interest in the estate of the ward as creditor, heir, or otherwise, that
guardian for the estate, and if, after notice given to such person and in such manner as the anyone is suspected of having embezzled, concealed, or conveyed away any money, goods, or
court deems proper, by publication or otherwise, and hearing, the court is satisfied that such interest, or a written instrument, belonging to the ward or his estate, the court may cite the
non-resident is a minor or incompetent rendering a guardian necessary or convenient, it may suspected person to appear for examination touching such money, goods, interest, or
appoint a guardian for such estate. instrument, and make such orders as will secure the estate against such embezzlement,
concealment or conveyance.
Section 7. Parents as guardians. — When the property of the child under parental
authority is worth two thousand pesos or less, the father of the mother, without the necessity Section 7. Inventories and accounts of guardians, and appraisement of estates. — A
of court appointment, shall be his legal guardian. When the property of the child is worth more guardian must render to the court an inventory of the estate of his ward within three (3)
than two thousand pesos, the father or the mother shall be considered guardian of the child's months after his appointment, and annually after such appointment an inventory and account,
property, with the duties and obligations of guardians under this rules, and shall file the the rendition of any of which may be compelled upon the application of an interested person.
petition required by section 2 hereof. For good reasons the court may, however, appoint Such inventories and accounts shall be sworn to by the guardian. All the estate of the ward
another suitable person. described in the first inventory shall be appraised. In the appraisement the court may request
the assistance of one or more of the inheritance tax appraisers. And whenever any property of
Section 8. Service of judgment. — Final orders or judgments under this rule shall be the ward not included in an inventory already rendered is discovered, or suceeded to, or
served upon the civil registrar of the municipality or city where the minor or incompetent acquired by the ward, like proceedings shall be had for securing an inventory and
person resides or where his property or part thereof is situated. appraisement thereof within three (3) months after such discovery, succession, or acquisition.

Section 8. When guardian's accounts presented for settlement. Expenses and


3. Requirement [Rule 94]
compensation allowed. — Upon the expiration of a year from the time of his appointment,
and as often thereafter as may be required, a guardian must present his account to the court
Section 1. Bond to be given before issuance of letters. Amount. Condition. — for settlement and allowance. In the settlement of the account, the guardian, other than a
Before a guardian appointed enters upon the execution of his trust, or letters of guardianship parent, shall be allowed the amount of his reasonable expenses incurred in the execution of
issue, he shall give a bond, in such sum as the court directs, conditioned as follows: his trust and also such compensation for his services as the court deems just, not exceeding
fifteen per centum of the net income of the ward.

102
RECTO, GAYLE ANGELI M.
2011-0008 | AUSL
Personal Notes on Remedial Law 2 Review (based on the syllabus of Prof. Henedino M. Brondial)

(a) the surviving grandparent and In case several grandparents survive, the court shall select
5. Termination [Rule 97] any of them taking Into account all relevant considerations;
(b) the oldest brother or sister of the minor over twenty-one years of age, unless unfit or
disqualified;
Section 1. Petition that competency of ward be adjudged, and proceedings
(c) the actual custodian of the minor over twenty-one years of age, unless unfit or disqualified;
thereupon. — A person who has been declared incompetent for any reason, or his guardian,
and
relative, or friend, may petition the court to have his present competency judicially
(d) any other person, who in the sound discretion of the court, would serve the best interests
determined. The petition shall be verified by oath, and shall state that such person is then
of the minor.
competent. Upon receiving the petition, the court shall fix a time for hearing the questions
raised thereby, and cause reasonable notice thereof to be given to the guardian of the person
Sec. 7. Contents of petition. - A petition for the appointment of a general guardian must
so declared incompetent, and to the ward. On the trial, the guardian or relatives of the ward,
allege the following:
and, in the discretion of the court, any other person, may contest the right to the relief
demanded, and witnesses may be called and examined by the parties or by the court on its
(a) The jurisdictional facts;
own motion. If it be found that the person is no longer incompetent, his competency shall be
(b) The name, age and residence of the prospective ward;
adjudged and the guardianship shall cease.
(c) The ground rendering the appointment necessary or convenient; library
(d) The death of the parents of the minor or the termination, deprivation or suspension of
Section 2. When the guardian removed or allowed to resign. New appointment. —
their parental authority;
When a guardian becomes insane or otherwise incapable of discharging his trust or unsuitable
(e) The remarriage of the minor’s surviving parent;
therefor, or has wasted or mismanaged the estate, or failed for thirty (30) days after it is due to
(f) The names, ages, and residences of relatives within the 4th civil degree of the minor, and
render an account or make a return, the court may, upon reasonable notice to the
of persons having him in their care and custody;
guardian, remove him, and compel him to surrender the estate of the ward to the person
(g) The probable value, character and location of the property of the minor; and
found to be lawfully entitled thereto. A guardian may resign when it appears proper to allow
(h) The name, age and residence of the person for whom letters of guardianship are prayed.
the same; and upon his resignation or removal the court may appoint another in his place.
The petition shall be verified and accompanied by a certification against forum shopping.
Section 3. Other termination of guardianship. — The marriage or voluntary
However, no defect in the petition or verification shall render void the issuance of letters of
emancipation of a minor ward terminates the guardianship of the peson of the ward, and shall
guardianship.
enable the minor to administer his property as though he were of age, but he cannot borrow
the money or alienate or encumber real property without the consent of his father or mother,
or guardian. He can sue and be sued in court only with the assistance of his father, mother or Sec. 8. Time and notice of hearing. - When a petition for the appointment of a general
guardian is filed, the court shall fix a time and place for its hearing, and shall cause reasonable
guardian. The guardian of any person may be discharged by the court when it appears, upon
notice to be given to the persons mentioned in the petition, including the minor if he is
the application of the ward or otherwise, that the guardianship is no longer necessary.
fourteen years of age or over, and may direct other general or special notice to be given.
Section 4. Record to be kept by the justice of the peace or municipal judge. —
When a justice of the peace or municipal court takes cognizance of the proceedings in Sec. 9. Case study report. - The court shall order a social worker to conduct a case study
of the minor and all the prospective guardians and submit his report and recommendation to
pursuance of the provisions of these rules, the record of the proceedings shall be kept as in
the court for its guidance before the scheduled hearing. The social worker may intervene on
the Court of First Instance.
behalf of the minor if he finds that the petition for guardianship should be denied.
Section 5. Service of judgment. — Final orders of judgments under this rule shall be
Sec. 10. Opposition to petition. - Any interested person may contest the petition by filing a
served upon the civil registrar of the municipality or city where the minor or incompetent
written opposition based on such grounds as the majority of the minor or the unsuitability of
person resides or where his property or part thereof is situated.
the person for whom letters are prayed, and pray that the petition be denied, or that letters of
guardianship issue to himself, or to any suitable person named in the opposition.
AM 03-02-05-SC
RULE ON GUARDIANSHIP OF MINORS Sec. 11. Hearing and order for letters to issue. - At the hearing of the petition, it must
be shown that the requirement of notice has been complied with. The prospective ward shall
Section 1. Applicability of the Rule. - This Rule shall apply to petitions for guardianship be presented to the court. The court shall hear the evidence of the parties in support of their
over the person or property, or both, of a minor. chan robles virtual law library respective allegations. If warranted, the court shall appoint a suitable guardian of the person
The father and the mother shall jointly exercise legal guardianship over the person and or property, or both, of the minor.
property of their unemancipated common child without the necessity of a court appointment.
In such case, this Rule shall be suppletory to the provisions of the Family Code on At the discretion of the court, the hearing on guardianship may be closed to the public and the
guardianship. records of the case shall not be released without its approval.

Sec. 2. Who may petition for appointment of guardian. - On grounds authorized by Sec. 12. When and how a guardian of the property for non-resident minor is
law, any relative or other person on behalf of a minor, or the minor himself if fourteen years of appointed; notice. - When the minor resides outside the Philippines but has property in the
age or over, may petition the Family Court for the appointment of a general guardian over the Philippines, any relative or friend of such minor, or any one interested in his property, in
person or property, or both, of such minor. The petition may also be filed by the Secretary of expectancy or otherwise, may petition the Family Court for the appointment of a guardian over
Social Welfare and Development and by the Secretary of Health in the case of an insane minor the property.
who needs to be hospitalized.
Notice of hearing of the petition shall be given to the minor by publication or any other means
Sec. 3. Where to file petition. - A petition for guardianship over the person or property, or as the court may deem proper. The court may dispense with the presence of the non-resident
both, of a minor may be filed in the Family Court of the province or city where the minor minor.
actually resides. If he resides in a foreign country, the petition shall be flied with the Family
Court of the province or city where his property or any part thereof is situated. If after hearing the court is satisfied that such non-resident is a minor and a guardian is
necessary or convenient, it may appoint a guardian over his property.
Sec. 4. Grounds of petition. - The grounds for the appointment of a guardian over the
person or property, or both, of a minor are the following: Sec. 13. Service of final and executory judgment or order. - The final and executory
judgment or order shall be served upon the Local Civil Registrar of the municipality or city
(a) death, continued absence, or incapacity of his parents; where the minor resides and the Register of Deeds of the place where his property or part
(b) suspension, deprivation or termination of parental authority; chan robles virtual law library thereof is situated shall annotate the same in the corresponding title, and report to the court
(c) remarriage of his surviving parent, if the latter Is found unsuitable to exercise parental his compliance within fifteen days from receipt of the order.
authority; or
(d) when the best interests of the minor so require.cralaw Sec. 14. Bond of guardian; amount; conditions. - Before he enters upon the execution of
his trust, or letters of guardianship issue, an appointed guardian may be required to post a
Sec. 5. Qualifications of guardians. - In appointing a guardian, the court shall consider bond in such sum as the court shall determine and conditioned as follows:
the guardian’s:
(a) To make and return to the court, within three months after the issuance of his letters of
(a) moral character; chan robles virtual law library guardianship, a true and complete Inventory of all the property, real and personal, of his ward
(b) physical, mental and psychological condition; which shall come to his possession or knowledge or to the possession or knowledge of any
(c) financial status; other person in his behalf;
(d) relationship of trust with the minor; (b) To faithfully execute the duties of his trust, to manage and dispose of the property
(e) availability to exercise the powers and duties of a guardian for the full period of the according to this rule for the best interests of the ward, and to provide for his proper care,
guardianship; custody and education;
(f) lack of conflict of interest with the minor; and (c) To render a true and Just account of all the property of the ward in his hands, and of all
(g) ability to manage the property of the minor. proceeds or interest derived therefrom, and of the management and disposition of the same,
at the time designated by this rule and such other times as the court directs; and at the
Sec. 6. Who may be appointed guardian of the person or property, or both, of a expiration of his trust, to settle his accounts with the court and deliver and pay over all the
minor. - In default of parents or a court-appointed guardian, the court may appoint a property, effects, and monies remaining in his hands, or due from him on such settlement, to
guardian of the person or property, or both, of a minor, observing as far as practicable, the the person lawfully entitled thereto; and
following order of preference: (d) To perform all orders of the court and such other duties as may be required by law.

103
RECTO, GAYLE ANGELI M.
2011-0008 | AUSL
Personal Notes on Remedial Law 2 Review (based on the syllabus of Prof. Henedino M. Brondial)

Sec. 15. Where to file the bond; action thereon. - The bond posted by a guardian shall conditions as to the time and manner of payment, and security where a part of the payment is
be filed in the Family Court and, In case of breach of any of its conditions, the guardian may deferred. The original bond of the guardian shall stand as security for the proper appropriation
be prosecuted in the same proceeding for the benefit of the ward or of any other person of the proceeds of the sale or encumbrance, but the court may, if deemed expedient, require an
legally interested in the property. additional bond as a condition for the sale or encumbrance. The authority to sell or
encumber shall not extend beyond one year, unless renewed by the court.
Whenever necessary, the court may require the guardian to post a new bond and may
discharge from further liability the sureties on the old bond after due notice to interested Sec. 23. Court may order investment of proceeds and direct management of
persons, if no injury may result therefrom to those interested in the property. property. - The court may authorize and require the guardian to invest the proceeds of sales
or encumbrances, and any other money of his ward in his hands, in real or personal property,
Sec. 16. Bond of parents as guardians of property of minor. - If the market value of for the best interests of the ward, and may make such other orders for the management,
the property or the annual Income of the child exceeds P50,000.00, the parent concerned investment, and disposition of the property and effects, as circumstances may warrant.
shall furnish a bond In such amount as the court may determine, but in no case less than ten
per centurn of the value of such property or annual income, to guarantee the performance of Sec. 24. Grounds for removal or resignation of guardian. - When a guardian becomes
the obligations prescribed for general guardians. insane or otherwise incapable of discharging his trust or is found thereafter to be unsuitable,
or has wasted or mismanaged the property of the ward, or has failed to render an account or
A verified petition for approval of the bond shall be flied in the Family Court of the place where make a return for thirty days after it is due, the court may, upon reasonable notice to the
the child resides or, if the child resides in a foreign country, in the Family Court of the place guardian, remove him as such and require him to surrender the property of the ward to the
where the property or any part thereof is situated. person found to be lawfully entitled thereto.

The petition shall be docketed as a summary special proceeding In which all incidents and The court may allow the guardian to resign for justifiable causes.
issues regarding the performance of the obligations of a general guardian shall be heard and
resolved. Upon the removal or resignation of the guardian, the court shall appoint a new one.

Sec. 17. General duties of guardian. - A guardian shall have the care and custody of the No motion for removal or resignation shall be granted unless the guardian has submitted the
person of his ward and the management of his property, or only the management of his proper accounting of the property of the ward and the court has approved the same.
property. The guardian of the property of a nonresident minor shall have the management of
all his property within the Philippines. Sec. 25. Ground for termination of guardianship. - The court motu proprio or upon
verified motion of any person allowed to file a petition for guardianship may terminate the
A guardian shall perform the following duties: guardianship on the ground that the ward has come of age or has died. The guardian shall
notify the court of such fact within ten days of its occurrence.
(a) To pay the just debts of the ward out of the personal property and the income of the real
property of the ward, If the same is sufficient; otherwise, out of the real property of the ward Sec. 26. Service of final and executory judgment or order. - The final and executory
upon obtaining an order for its sale or encumbrance; judgment or order shall be served upon the Local Civil Registrar of the municipality or city
(b) To settle all accounts of his ward, and demand, sue for, receive all debts due him, or may, where the minor resides and the Register of Deeds of the province or city where his property
with the approval of the court, compound for the same and give discharges to the debtor on or any part thereof is situated. Both the Local Civil Registrar and’ the Register of Deeds shall
receiving a fair and just dividend of the property and effects; and to appear for and represent enter the final and executory judgment or order in the appropriate books in their offices.
the ward in all actions and special proceedings, unless another person is appointed for that
purpose; Sec. 27. Effect of the rule. - This Rule amends Rules 92 to 97 inclusive of the Rules of
(c) To manage the property of the ward frugally and without waste, and apply the income and Court on guardianship of minors. Guardianship of incompetents who are not minors shall
profits thereon, insofar as may be necessary, to the comfortable and suitable maintenance of continue to be under the jurisdiction of the regular courts and governed by the Rules of Court.
the ward; and if such income and profits be insufficient for that purpose, to sell or encumber
the real or personal property, upon being authorized by the court to do so; Sec. 28. Effectivity. - This Rule shall take effect on May 1, 2003 following its publication in
(d) To consent to a partition of real or personal property owned by the ward jointly or in a newspaper of general circulation not later than April 15, 2003.
common with others upon authority granted by the court after hearing, notice to relatives of
the ward, and a careful investigation as to the necessity and propriety of the proposed action; OLD LECTURE
(e) To submit to the court a verified inventory of the property of his ward within three months
after his appointment, and annually thereafter, the rendition of which may be required upon
Rule 92 Guardianship
the application of an interested person;
(f) To report to the court any property of the ward not included in the inventory which is The fundamental difference is that in guardianship, the subject is still alive. In
discovered, or succeeded to, or acquired by the ward within three months after such settlement of estate, the subject matter is already dead.
discovery, succession, or acquisition; and
(g) To render to the court for its approval an accounting of the property one year from his Three Kinds of Guardians:
appointment, and every year thereafter or as often as may be required. Judicial guardian - that appointed by the court in a judicial proceeding for legal
Sec. 18. Power and duty of the court - The court may: guardianship
(a) Request the assistance of one or more commissioners in the appraisal of the property of Legal guardian— guardian by operation of law; not just appointed by any court.
the ward reported in the initial and subsequent inventories;
(ex.: parents of minor children)
(b) Authorize reimbursement to the guardian, other than a parent, of reasonable expenses
incurred in the execution of his trust, and allow payment of compensation for his services as Guardian ad litem- that which is appointed by the court not necessarily in a
the court may deem just, not exceeding ten per centum of the net income of the ward, if any; guardianship proceeding, because this guardian ad litem is only on a temporary
otherwise, in such amount the court determines to be a reasonable compensation for his basis with a specific duty to perform.
services; and
(c) Upon complaint of the guardian or ward, or of any person having actual or prospective Q: Which court has jurisdiction over a petition for guardianship?
interest in the property at the ward, require any person suspected of having embezzled, A: Exclusively and originally cognizable by the Family Court (RTC specifically
concealed, or disposed of any money, goods or interest, or a written instrument belonging to designated as a Family Court because of RA 8369 ). There is no inferior court
the ward or his property to appear for examination concerning any thereof and issue such here.
orders as would secure the property against such embezzlement, concealment or conveyance.

Sec. 19. Petition to sell or encumber property. - When the income of a property under However, in settlement of estate, jurisdiction may be lodged in inferior courts
guardianship is insufficient to maintain and educate the ward, or when it is for his benefit that depending on the gross value of the estate.
his personal or real property or any part thereof be sold, mortgaged or otherwise encumbered,
and the proceeds invested in safe and productive security, or in the improvement or security The venue is the residence of the ward. If the ward has no residence and
of other real property, the guardian may file a verified petition setting forth such facts, and the guardianship application is over the property of the ward, the venue is
praying that an order issue authorizing the sale or encumbrance of the property. where the property is situated/ located.
Sec. 20. Order to show cause. - If the sale or encumbrance is necessary or would be Take note that the venue here is the residence of the ward as distinguished from
beneficial to the ward, the court shall order his next of kin and all person/s interested in the
the venue in adoption, which is the residence of the adopter. Bakit? Because in
property to appear at a reasonable time and place therein specified and show cause why the
petition should not be granted. adoption, the result is that the adoptee will become the child of the adopter for all
legal intents and purposes. But here, it is the guardian that goes to the ward as
Sec. 21. Hearing on return of order; costs. - At the time and place designated in the the relation here is only of a temporary character.
order to show cause, the court shall hear the allegations and evidence of the petitioner and
next of kin, and other persons interested, together with their witnesses, and grant or deny the You file a guardianship proceeding only on 2 grounds:
petition as the best interests of the ward may require. Minority; and
Incompetency.
Sec. 22. Contents of order for sale or encumbrance and its duration; bond. - If, after
full examination, it is necessary, or would be beneficial to the ward, to sell or encumber the
The term “incompetent” here is different from that in settlement of estate. Here,
property, or some portion of it, the court shall order such sale or encumbrance the proceeds of
which shall be expended for the maintenance or the education of the ward, or invested as the “incompetent” refers to:
circumstances may require. The order shall specify the grounds for the sale or encumbrance Sec. 2. Meaning of the word “incompetent”. Incompetent includes:
and may direct that the property ordered sold be disposed of at public sale, subject to such
104
RECTO, GAYLE ANGELI M.
2011-0008 | AUSL
Personal Notes on Remedial Law 2 Review (based on the syllabus of Prof. Henedino M. Brondial)

Those suffering from the penalty of civil interdiction (a penalty attached to A: Yes if the property of the war, which is their own children, is worth more than
conviction); P50T. If they sell the property of their ward, even if they are the legal guardians,
Hospitalized lepers; the sale of the property is void.
Prodigals (one who is a spendthrift; wastes money or property on things without
reserving any for himself and before you know it, he is not only a prodigal son but Q: Distinction between a guardian and a trustee
also a grasa man.); A: The distinction between a guardian and a trustee is that the latter has the legal
Deaf and Dumb unable to read and write; title while the guardian has no legal title of the property. So that the trustee can
Those of unsound mind although they have lucid intervals; negotiate and encumber the property under trust. Although the same trustee may
Persons not of unsound mind but by reason of age, disease, weak mind, and either be a trustee and a beneficiary at the same time.
other similar causes, cannot, without outside aid, take care of themselves or
manage their property. A trust relationship can either be express if there is a trust agreement between
parties or implied if it is by operation of law. Examples of trust by operation of law
A minor, under the age of 18, can be the subject of guardianship. is when parents die without a will and then some children are still minors,
whoever is the guardian of the minor holds the property for and in behalf of the
Case of Evangelista: The petition for guardianship was granted by the court minors in trust. So that you will note that the guardian here, being the trustee,
and Caniza was appointed as the guardian of Evangelista. As a guardian, may not be a party to a written trust agreement but he cannot dispose the minor's
remember the rights, the guardian can sue and be sued, can collect debts, can property without consent of the court for the reason that he is only in trust for the
manage the properties of the ward. So one of his actions here was to ask the ces qui trust. Even in the absence of a written contract, there is a trust
Estradas to vacate the premises owned by the ward. Judgment was rendered in relationship by operation of law.
favor of plaintiff, but on appeal, it was reversed and on appeal again from the
order of reversal, it was sustained. That is why it went up to the Supreme Court.
But pending the appeal with the CA, the ward died. This is a case for ejectment.
CASES
The issue here is that considering that there is no more guardianship because
death terminates guardianship, hence, the case must be dismissed because the
party appellant is not the proper party-in-interest. The SC said No. Even if death PILAR Y. GOYENA, petitioner, vs. AMPARO LEDESMA-GUSTILO, respondent.
terminates guardianship, in this particular case, it is contrary to the principle of G. R. No. 147148. January 13, 2003
equity of justice if we have to start all over again. The case is already with us, so THIRD DIVISION
remand of the case to the lower court would be a waste of time and more
importantly, they found from the record that Caniza was one of the heirs of the FACTS:
ward. Hence, there is still a party-in-interest even if there is no settlement of the x Amparo Æ filed with RTC Makati PETITION FOR LETTERS OF GUARDIANSHIP[1]
estate. Ang importante ay pag guardian ka tapos heir ka din, there is no need for over the person and properties of her sister Julieta
o Julieta Ledesma has been a patient in the Makati Medical Center
the settlement of the estate.
where she is under medical attention for old age, general debility,
and a mini-stroke
NOTE: Remember that you find that also in Sec. 16 of Rule 3: Substitution of x Pilar Æ filed an opposition
Parties (“without a need of appointing an administrator or executor of the estate o petition lacked factual and legal basis in that Julieta Ledesma is
in the substitution of parties.”). That is the doctrine laid down in this case. competent and sane and there is absolutely no need to appoint a
guardian to take charge of her person/property. She is very able to
Q: Who can initiate a petition for guardianship? take charge of her affairs, and this is clearly evident from her
A: Anybody who has interest in the person of or property of the ward. If you letters to the petitioner
cannot establish any interest in the person of or in the property of the ward, you o Amparo is unfit to be a guardian since her and Julieta’s interests
are antagonistic
cannot file a petition for guardianship.
x RTC Æ found Julieta incapable; appointed AMPARO as guardian
o A perusal of the records shows that petitioner (Amparo) is 72 years
Buyena vs. Ledesma: In this case, they were able to establish interest. They of age, the youngest sister of Julieta. Admittedly, the Oppositor
were both single and they were living together. Pilar Goyena, 90 years of age has been the close friend and
companion of Julieta for 61 years. Julieta was with Oppositor when
You have to establish interest. The Rule says, friends, relatives, or any person she suffered her first stroke in Makati in 1991 which was the
who has interest. reason why Julieta had to give up the management of their
hacienda in Bacolod. It is also not disputed that Julieta was with Pilar
Q: What is the procedure? when she had her second stroke in the U.S. In short, the
special bond of friendship existing between Julieta and the
A: File a petition with a court of competent jurisdiction in the proper venue. And
Oppositor cannot be denied. Now that Julieta is unable to manage
the court will issue an order setting the case for hearing. If there are oppositors, her personal life and business concerns due to senility and vascular
then they can file their opposition. Remember, this is a special proceeding and dementia, the oppositor wants to be appointed her guardian or else
jurisdiction is always acquired through publication. After trial, there will be Bart Lacson, Fely Montelibano and Jose T. Revilla.
decision either allowing or disallowing guardianship. In other word, appointing a o It is interesting to note that the oppositor has interposed her
guardian or not appointing a guardian. objection to the appointment of Amparo as guardian because she
The guardian now enters into the guardianship after he has filed the necessary thinks that the latter dislikes her. She further added that there
bond. The duties and responsibilities of a guardian are similar to that of an were a number of letters allegedly written by Julieta to Amparo
executor or administrator . which showed Julietas sentiments regarding certain matters.
Nevertheless, not one of the nearest of kin of Julieta opposed the
petition. As a matter of fact, her sisters signified their conformity
Q: Can he sell the properties of his ward? thereto. Thus, Ms. Goyenas mere conjecture that Amparo dislikes
A: Yes. Even without permission from the court if the property involved is her is no sufficient reason why the petition should be denied.
personal property. But, if it is real property, just like, an executor or Neither does it make Amparo unsuitable and unfit to perform the
administrator, he has to get permission from the court. Only with the permission duties of a guardian. On the contrary, it is Ms. Goyena who could
of the court will the sale of real property be considered a valid sale. It is not only be considered as to have an adverse interest to that of Julieta if it
permission of the court which is required, but also notice to all interested parties. is true that 50% of Julietas holdings at the Makati Medical Center
In guardianship, the absence of notice, even with permission of the court, still has been transferred to her as alleged in Exhibit 1 and Exhibit A.
renders the sale void. o By and large, the qualification of Amparo to act as guardian over
the person and properties of Julieta has been duly established. As a
sister, she can best take care of Julietas concerns and well being.
Q: In what instances may guardianship be terminated? Now that Julieta is in the twilight of her life, her family should be
A: Death also terminates guardianship, but the general rule is that if the ground given the opportunity to show their love and affection for her
for the appointment of the guardian ceases, then guardianship ceases as well.. If without however denying Pilar Goyena access to her considering
the minor is already of age (remember, even the minor himself can ask for the special bond of friendship between the two. Needless to say,
guardianship). This is an exception regarding lack of a minor’s legal personality the oppositor at 90 years of age could not be said to be physically
to sue. If the ground is based on incompetency and it is established that the fit to attend to all the needs of Julieta.
x RTC Æ denied ensuing MR
ward is already competent, or that he was insane, but now, he is no longer x Pilar Æ appealed to CA
insane; a petition for termination of guardianship may be filed.
x CA Æ affirmed RTC
o Indeed, oppositor-appellant (Pilar) has not shown the authenticity
Q: Should the legal guardian file guardianship proceedings over the person and and due execution of the letters which purport to show the
property of their minor children? existence of a rift between Julieta and her family and dissatisfaction
as to how the businesses were managed. At any rate, while it is
correct to say that no person should be appointed guardian if his
105
RECTO, GAYLE ANGELI M.
2011-0008 | AUSL
Personal Notes on Remedial Law 2 Review (based on the syllabus of Prof. Henedino M. Brondial)

interest conflict with those of the ward (Guerrero vs. Teran, 13 Phil. capital each of us to work, and keep the Hda, for [sic] generation
212), there are really no antagonistic interests to speak of between to generation.
petitioner [Amparo] and Julieta, they being co-owners of certain o For the last time I will repeat even if I have to kneel before you
properties. There is also no showing that petitioners business and Carlos I have no interest anymore in any future investment due
decisions in the past had resulted in the prejudice of Julieta. to my age and being single and alone in life. I would like to be able
o While the oppositor may have been very close to Julieta, there is to enjoy whatever monies that correspond to me. I would like to
no sufficient showing that petitioner is hostile to the best interests have enough money as a reserve for any future need that I might
of the latter. On the contrary, it was the petitioner who, realizing have like hospitalization, travel, buying whatever I like, etc. etc.
the need for the appointment of a person to guard her sisters (Letter to appellee; Exhibit 2)
interests, initiated the petition for guardianship. We see no x merely shows Julietas lack of interest in future investments, not
indication that petitioner is animated by a desire to prejudice necessarily a business disagreement, and certainly not per se
Julietas health as well as financial interests. In point of fact, it was amounting to antagonistic interests between her and respondent to
oppositor-appellant who had initially concealed the deteriorating render the latter unsuitable for appointment as guardian.
state of mind of Julieta from the court. Oppositors advanced age of x The second letter[18] which reads:
90 years also militate against her assuming the guardianship of the o My mind is still clear to tell you about Fortuna when I had my
incompetent. The oppositor has declared that she is not interested stroke I was confined in MMC for one month. If I am not mistaken
to be appointed legal guardian (p.21[,] Appellants Brief, Rollo, p. you did not visit me. One day Carlos came to visit me and asked me
59). But the persons that she points to as being better choices as this question. Do you think you will be able to continue
Julietas guardian over the appellee have not acted, nor even managing the Hda? I answered him I dont know it all depends on
indicated, their desire to act as such. In any case, We see no my sickness. Carlos said who do you want to take your place? I
cogent reason why We should reverse the well-reasoned said I want Cheling Zabaljauregui. Then Carlos said O.K. He asked
disquisition of the trial court. Pilar can you contact Cheling? Tell him to call me or see me. The
x CA Æ denied ensuing MR nephew of Cheling was a resident in MMC through him Pilar was
x Pilar Æ filed a Rule 45 before the SC able to contact Cheling and gave him Carlos message. So I thought
all the time it was agreeable. I left for USA for treatment. To my
ISSUE: Whether the alleged “antagonistic interests” render Amparo unfit to be appointed as surprise when I came back from USA it was not Cheling, but you
guardian of Julieta. (appellee) took over the management as you requested. Carlos did
not tell me but decided in your favor. x x x (Letter to appellee;
HELD: NO. Exhibit 3; emphasis supplied)
x Clearly, the issues raised and arguments in support of petitioners position require x shows that: 1) respondent did not visit Julieta when she was confined at the
a review of the evidence, hence, not proper for consideration in the petition at Makati Medical Center on account of her stroke, 2) there was disagreement as to
bar. This Court cannot thus be tasked to go over the proofs presented by the who should run the hacienda, with Julieta favoring a certain Cheling
parties and analyze, assess, and weigh them to ascertain if the trial court and Zabaljaurigue, and 3) respondent took over management of the hacienda with
appellate court were correct in according them superior credit.[10] their brother Carlos (Ledesma) supporting her. No inference as to the existence
x That the issues raised are factual is in fact admitted by petitioner in her Reply of antagonistic interests between respondent and Julieta can thus be made.
dated August 30, 2001:[11] x The third letter[19] which reads:
o Although the general rule is that this Honorable Court is not a trier o x x x Carlos went to the house before I left and asked from me
of facts, its jurisdiction being limited to reviewing and revising only twenty thousand (20,000) shares of San Carlos Milling which you
errors of law, it is nonetheless subject to the following exceptions gave because I wanted to sell all.xxx If he does not sell or cannot
which have been laid down in a number of decisions of this sell, just arrange to send them back to me. Amparing since I came
Honorable Court: here to America and Vancouver my requests have been ignored.
ƒ (1) When the conclusion is a finding grounded Everyone is suspecting that Pilar is the one ordering or
entirely on speculation, surmises and conjectures; (2) commanding me that is not true. What I asked from Julio is just to
When the inference made is manifestly mistaken, report to me or send me reports so I can follow up from here. But
absurd or impossible; (3) When there is grave abuse up to now he has ignored my requests x x x. (Letter to appellee
of discretion; (4) When the judgment is based on a Exhibit 4)
misapprehension of facts; (5) When the findings of x has no relevance to the issue of whether or not the lower courts erred in finding
facts are conflicting; (6) When the Court of Appeals, that respondent is not unsuitable for appointment as guardian. The letter in fact
in making its findings, went beyond the issues of the discloses, that it was Julietas nephew Julio Ledesma, and not respondent, who
case and the same is contrary to the admissions of ignored the request.
both appellants and appellee; (7) When the findings x As for the fourth letter[20] which reads:
of the Court of Appeals are contrary to those of the o I want all of you to know that whatever decision now and in the
trial court; (8) When the findings of facts are future I want to do nobody can stop me especially regarding my
conclusions without citation of specific evidence on properties, money, etc. I will be the only one to dispose of it
which they are based; (9) When the facts set forth in because it is mine. You said to Raul you are going to court, you are
the petition as well as in the petitioners main and most welcome x x x. (Letter to Connie, Exhibit 5)
reply briefs are not disputed by the respondents; and x it has also no relevance to the issue in the case at bar. The letter is not even
(10) When the findings of fact of the Court of Appeals addressed to respondent but to a certain Connie (a sister-in-law of Julieta).
is premised on the supposed absence of evidence and x Petitioners assertion that respondents intent in instituting the guardianship
is contradicted by the evidence on record (Emphasis proceedings is to take control of Julietas properties and use them for her own
supplied); (Rollo, 350-351) benefit[21] is purely speculative and finds no support form the records.
x Petitioner claims that there is no doubt that the instant petition falls within the x The claim that respondent is hostile to the best interests of Julieta also lacks
above-stated exceptions because the findings of the Court of Appeals are clearly merit. That respondent removed Julieta from the Makati Medical Center where
belied by the evidence on record.[12] she was confined after she suffered a stroke does not necessarily show her
x In the selection of a guardian, a large discretion must be allowed the hostility towards Julieta, given the observation by the trial court, cited in the
judge who deals directly with the parties.[13] As this Court said: present petition, that Julieta was still placed under the care of doctors[22] after
o As a rule, when it appears that the judge has exercised care and she checked out and was returned to the hospital when she suffered another
diligence in selecting the guardian, and has given due consideration stroke.
to the reasons for and against his action which are urged by the x Finally, this Court notes two undisputed facts in the case at bar, to wit: 1)
interested parties, his action should not be disturbed unless it is Petitioner opposed the petition for the appointment of respondent as guardian
made very clear that he has fallen into grievous error.[14] before the trial court because, among other reasons, she felt she was disliked by
x In the case at bar, petitioner has not shown that the lower courts committed any respondent,[23] a ground which does not render respondent unsuitable for
error. appointment as guardian, and 2) Petitioner concealed the deteriorating state of
x Petitioner cannot rely on Garchitorena v. Sotelo[15] with respect to the mind of Julieta before the trial court,[24] which is reflective of a lack of good
existence of antagonistic interests between respondent and Julieta. In faith.
that case, the interest of Perfecto Gabriel as creditor and mortgagee of the x Discussion of the third argument is unnecessary, the suitability of Amparo for
minor-wards properties (a house and lot) is antagonistic to the interest of the appointment as guardian not having been successfully contested.
wards as mortgagors, hence, Gabriels appointment as guardian was erroneous.
For while he sought to foreclose the wards properties as creditor and mortgagee
on one hand, he had to, on the other hand, endeavor to retain them for the The Incompetent, CARMEN CAIZA, represented by her legal guardian, AMPARO
wards as their guardian. Added to that was Gabriels appointment as guardian EVANGELISTA, petitioner, vs. COURT OF APPEALS (SPECIAL FIRST DIVISION), PEDRO
without him informing the guardianship court that he held a mortgage on the ESTRADA and his wife, LEONORA ESTRADA, respondents.
properties. Furthermore, he deliberately misinformed the said court that the first G.R. No. 110427. February 24, 1997
mortgagee was the Santa Clara Monastery when it was him. None of the said THIRD DIVISION
circumstances obtain in the present case.
x Petitioner can neither rely on certain letters of Julieta to establish her claim that FACTS:
there existed[16] a rift between the two which amounts to antagonistic interests. x RTC QC Br. 107 Æ declared Carmen, a 94 yo retired pharmacist and UP
The first letter[17] sent by Julieta to respondent which reads: professor, in a guardianship proceeding instituted by her niece, Amparo
o x x x So if you (appellee) do not agree with me (Julieta) my
decision is right to let us divide as soon as possible, so we will have
106
RECTO, GAYLE ANGELI M.
2011-0008 | AUSL
Personal Notes on Remedial Law 2 Review (based on the syllabus of Prof. Henedino M. Brondial)

o She was so adjudged because of her advanced age and physical such income and profits be insufficient for that purpose, the
infirmities which included cataracts in both eyes and senile guardian may sell or encumber the real estate, upon being
dementia authorized by order to do so, and apply to such of the proceeds as
o Amparo was appointed as guardian may be necessary to such maintenance."
x Amparo (in her capacity as guardian of Carmen) Æ filed an ejectment suit before x Finally, it may be pointed out in relation to the Estradas's defenses in the
MTC QC Br. 35 against spouses Pedro and Leonora Estrada ejectment action, that as the law now stands, even when, in forcible entry and
x Estradas Æ filed their answer unlawful detainer cases, the defendant raises the question of ownership in his
o declared that they had been living in Caiza's house since the pleadings and the question of possession cannot be resolved without deciding
1960's; that in consideration of their faithful service they had been the issue of ownership, the Metropolitan Trial Courts, Municipal Trial Courts, and
considered by Caiza as her own family, and the latter had in fact Municipal Circuit Trial Courts nevertheless have the undoubted competence to
executed a holographic will on September 4, 1988 by which she resolve. "the issue of ownership ** only to determine the issue of possession."
"bequeathed" to the Estradas the house and lot in question.
x MTC Æ decided in favor of Amparo (and Carmen) ISSUE # 2: Whether Carmen’s death rendered Amparo incapacitated to continue litigating for
x Estradas Æ appealed to RTC said deceased.
x RTC Æ reversed MTC
o held that the "action by which the issue of defendants' possession HELD # 2: NO. (Amparo may still pursue the case NOT as a guardian BUT in substitution for
should be resolved is accion publiciana, the obtaining factual and the deceased)
legal situation ** demanding adjudication by such plenary action x As already stated, Carmen Caiza passed away during the pendency of this
for recovery of possession cognizable in the first instance by the appeal. The Estradas thereupon moved to dismiss the petition, arguing that
Regional Trial Court." Caiza's death automatically terminated the guardianship, Amaparo Evangelista
x Amparo (in her capacity as guardian of Carmen) Æ appealed to CA lost all authority as her judicial guardian, and ceased to have legal personality to
x CA Æ affirmed RTC in toto represent her in the present appeal. The motion is without merit.
o Caiza sought to have the Court of Appeals reverse the decision of x While it is indeed well-established rule that the relationship of
October 21, 1992, but failed in that attempt. In a decision[10] guardian and ward is necessarily terminated by the death of either the
promulgated on June 2, 1993, the Appellate Court[11] affirmed the guardian or the ward,[38] the rule affords no advantage to the
RTC's judgment in toto. It ruled that (a) the proper remedy for Estradas. Amparo Evangelista, as niece of Carmen Caiza, is one of the latter's
Caiza was indeed an accion publiciana in the RTC, not an accion only two (2) surviving heirs, the other being Caiza's nephew, Ramon C. Nevado.
interdictal in the MetroTC, since the "defendants have not been in On their motion and by Resolution of this Court[39] of June 20, 1994, they were
the subject premises as mere tenants or occupants by tolerance, in fact substituted as parties in the appeal at bar in place of the deceased, in
they have been there as a sort of adopted family of Carmen Caiza," accordance with Section 17, Rule 3 of the Rules of Court, viz.:[40]
as evidenced by what purports to be the holographic will of the o "SEC. 18. Death of a party. After a party dies and the claim is not
plaintiff; and (b) while "said will, unless and until it has passed thereby extinguished, the court shall order, upon proper notice, the
probate by the proper court, could not be the basis of defendants' legal representative of the deceased to appear and be substituted
claim to the property, ** it is indicative of intent and desire on the for the deceased within a period of thirty (30) days, or within such
part of Carmen Caiza that defendants are to remain and are to time as may be granted. If the legal representative fails to appear
continue in their occupancy and possession, so much so that within said time, the court may order the opposing party to procure
Caiza's supervening incompetency can not be said to have vested the appointment of a legal representative of the deceased within a
in her guardian the right or authority to drive the defendants out." time to be specified by the court, and the representative shall
x Amparo (in her capacity as guardian of Carmen) Æ filed a Rule 45 before the SC immediately appear for and on behalf of the interest of the
x Carmen died during the pendency of the appeal deceased. The court charges involved in procuring such
o aforementioned guardian, Amparo Evangelista, and Ramon C. appointment, if defrayed by the opposing party, may be recovered
Nevado, her niece and nephew, respectively -- were by this Court's as costs. The heirs of the deceased may be allowed to be
leave, substituted for her substituted for the deceased, without requiring the appointment of
an executor or administrator and the court may appoint guardian ad
ISSUE # 1: Whether Amparo had the capacity to file the ejectment suit against the Estradas litem for the minor heirs.
considering the allegation that Carmen has already devised to the Estradas the subject house. x To be sure, an ejectment case survives the death of a party. Caiza's demise did
not extinguish the desahucio suit instituted by her through her guardian.[41] That
HELD # 1: YES. action, not being a purely personal one, survived her death; her heirs have taken
x The Estradas insist that the devise of the house to them by Caiza clearly denotes her place and now represent her interests in the appeal at bar.
her intention that they remain in possession thereof, and legally incapacitated
her judicial guardian, Amparo Evangelista, from evicting them therefrom, since
their ouster would be inconsistent with the ward's will. ONG vs. CA 401 SCRA
x A will is essentially ambulatory; at any time prior to the testator's death, it may
be changed or revoked;[29] and until admitted to probate, it has no effect NAPOLEON D. NERI, ALICIA D. NERI-MONDEJAR, VISMINDA D. NERI-CHAMBERS, ROSA D.
whatever and no right can be claimed thereunder, the law being quite explicit: NERI-MILLAN, DOUGLAS D. NERI, EUTROPIA D. ILLUT-COCKINOS AND VICTORIA D. ILLUT-
"No will shall pass either real or personal property unless it is proved and allowed PIALA, Petitioners, vs. HEIRS OF HADJI YUSOP UY AND JULPHA* IBRAHIM UY,
in accordance with the Rules of Court" (ART. 838, id.).[30] An owner's intention Respondents.
to confer title in the future to persons possessing property by his tolerance, is G.R. No. 194366 October 10, 2012
not inconsistent with the former's taking back possession in the meantime for SECOND DIVISION
any reason deemed sufficient. And that in this case there was sufficient cause for
the owner's resumption of possession is apparent: she needed to generate FACTS:
income from the house on account of the physical infirmities afflicting her, x Anuncacion contracted 2 marriages during her lifetime:
arising from her extreme age. o With Gonzalo Æ 2 children
x Amparo Evangelista was appointed by a competent court the general guardian of ƒ Eutropia and Victoria
both the person and the estate of her aunt, Carmen Caiza. Her Letters of o With Enrique Æ 5 children
Guardianship[31] dated December 19, 1989 clearly installed her as the "guardian ƒ Napoleon, Alicia, Visminda, Douglas and Rosa
over the person and properties of the incompetent CARMEN CAIZA with full x Anuncacion died intestate
authority to take possession of the property of said incompetent in any province x Enrique (in his capacity and as GUARDIAN of minor children Rosa and Douglas)
or provinces in which it may be situated and to perform all other acts necessary AND Napoleon, Alicia, and Visminda Æ executed an EJ settlement
for the management of her properties ** "[32] By that appointment, it o adjudicating among themselves the said homestead properties, and
became Evangelista's duty to care for her aunt's person, to attend to thereafter, conveying themto the late spouses Hadji Yusop Uy and
her physical and spiritual needs, to assure her well-being, with right to Julpha Ibrahim Uy (spouses Uy)for a consideration of P 80,000.00.
custody of her person in preference to relatives and friends.[33] It also x Children of Enrique Æ filed a complaint for annulment of sale of the said
became her right and duty to get possession of, and exercise control homestead properties against spouses Uy (later substituted by their heirs) before
over, Caiza's property, both real and personal, it being recognized the RTC
principle that the ward has no right to possession or control of his x Heirs of Uy Æ filed their Answer
property during her incompetency.[34] That right to manage the o countered that the sale took place beyond the 5-year prohibitory
ward's estate carries with it the right to take possession thereof and period from the issuance of the homestead patents.
recover it from anyone who retains it,[35] and bring and defend such o They also denied knowledge of Eutropia and Victoria’s
actions as may be needful for this purpose. [36] exclusionfrom the extrajudicial settlement and sale of the subject
properties, and interposed further the defenses of prescription and
x Actually, in bringing the action of desahucio, Evangelista was merely laches.
discharging the duty to attend to "the comfortable and suitable x RTC Æ in favor of the CHILDREN OF ENRIQUE
maintenance of the ward" explicitly imposed on her by Section 4, Rule o ruled that while the sale occurred beyond the 5-year prohibitory
96 of the Rules of Court, viz.: period, the sale is still void because Eutropia and Victoria were
o "SEC. 4. Estate to be managed frugally, and proceeds applied to deprived of their hereditary rights and that Enrique had no judicial
maintenance of ward. A guardian must manage the estate of his authority to sell the shares of his minor children, Rosa and Douglas.
ward frugally and without waste, and apply the income and profits x CA Æ reversed RTC
thereof, so far as maybe necessary, to the comfortable and suitable o It held that, while Eutropia and Victoria had no knowledge of the
maintenance of the ward and his family, if there be any; and if extrajudicial settlement and sale of the subject properties and as
107
RECTO, GAYLE ANGELI M.
2011-0008 | AUSL
Personal Notes on Remedial Law 2 Review (based on the syllabus of Prof. Henedino M. Brondial)

such, were not bound by it, the CA found it unconscionable to the party so making the ratification.16 Once ratified, expressly or
permit the annulment of the sale considering spouses Uy’s impliedly such as when the person knowingly received benefits from it,
possession thereof for 17 years, and that Eutropia and Victoria the contract is cleansed from all its defects from the moment it was
belatedly filed their action in 1997, or more than two years from constituted,17 as it has a retroactive effect.
knowledge of their exclusion as heirs in 1994 when their stepfather x Records, however, show that Rosa had ratified the extrajudicial settlement of the
died. It, however, did not preclude the excluded heirs from estate with absolute deed of sale. In Napoleon and Rosa’s Manifestation18
recovering their legitimes from their co-heirs. before the RTC dated July 11, 1997,they stated:
o declared the extrajudicial settlement and the subsequent sale as o "Concerning the sale of our parcel of land executed by our father,
valid and binding with respect to Enrique and hischildren, holding Enrique Neri concurred in and conformed to by us and our other
that as co-owners, they have the right to dispose of their two sisters and brother (the other plaintiffs), in favor of Hadji
respective shares as they consider necessary or fit. While Yusop Uy and his spouse Hadja Julpa Uy on July 7, 1979, we both
recognizing Rosa and Douglas to be minors at that time, they were confirmed that the same was voluntary and freely made by all of us
deemed to have ratified the sale when they failed to question it and therefore the sale was absolutely valid and enforceable as far
upon reaching the age of majority. It also found laches to have set as we all plaintiffs in this case are concerned;" (Underscoring
in because of their inaction for a long period of time. supplied)
x In their June 30, 1997 Joint-Affidavit,19 Napoleon and Rosa also alleged:
ISSUE: Whether the sale made by Enrique was valid considering that the said sale was o "That we are surprised that our names are included in this case
purportedly made in behalf of the latter’s children. since we do not have any intention to file a case against Hadji
Yusop Uy and Julpha Ibrahim Uy and their family and we respect
HELD: YES except insofar as the shares of Eutropia, Victoria and Douglas are concerned and acknowledge the validity of the Extra-Judicial Settlement of the
because (1) Eutropia and Victoria were not under Enrique’s guardianship AND (2) although Estate with Absolute Deed of Sale dated July 7, 1979;"
Douglas was under Enrique’s guardianship, he did not ratify the said sale upon reaching (Underscoring supplied)
majority age. x Clearly, the foregoing statements constituted ratification of the settlement of the
x However, while the settlement of the estate is null and void, the subsequent sale estate and the subsequent sale, thus, purging all the defects existing at the time
of the subject properties made by Enrique and his children, Napoleon, Alicia and of its execution and legitimizing the conveyance of Rosa’s 1/16 share in the
Visminda, in favor of the respondents is valid but only with respect to their estate of Anunciacion to spouses Uy. The same, however, is not true with
proportionate shares therein. It cannot be denied that these heirs have respect to Douglas for lack of evidence showing ratification.
acquired their respective shares in the properties of Anunciacion from x Considering, thus, that the extrajudicial settlement with sale is invalid and
the moment of her death11and that, as owners thereof, they can very therefore, not binding on Eutropia, Victoria and Douglas, only the shares of
well sell their undivided share in the estate.12 Enrique, Napoleon, Alicia, Visminda and Rosa in the homestead properties have
x With respect to Rosa and Douglas who were minors at the time of the execution effectively been disposed in favor of spouses Uy. "A person can only sell what
of the settlement and sale, their natural guardian and father, Enrique, he owns, or is authorized to sell and the buyer can as a consequence
represented them in the transaction. However, on the basis of the laws acquire no more than what the seller can legally transfer."20 On this score,
prevailing at that time, Enrique was merely clothed with powers of administration Article 493 of the Civil Code is relevant, which provides:
and bereft of any authority to dispose of their 2/16 shares in the estate of their o Each co-owner shall have the full ownership of his part and of the
mother, Anunciacion. fruits and benefits pertaining thereto, and he may therefore
x Articles 320 and 326 of the Civil Code, the laws in force at the time of the alienate, assign or mortgage it, and even substitute another person
execution of the settlement and sale, provide: in its enjoyment, except when personal rights are involved. But the
o ART. 320. The father, or in his absence the mother, is the legal effect of the alienation or the mortgage, with respect to the co-
administrator of the property pertaining to the child under parental owners, shall be limited to the portion which may be allotted to him
authority. If the property is worth more than two thousand pesos, in the division upon the termination of the co-ownership.
the father or mother shall give a bond subject to the approval of the x Consequently, spouses Uy or their substituted heirs became pro indiviso co-
Court of First Instance. owners of the homestead properties with Eutropia, Victoria and Douglas, who
o ART. 326. When the property of the child is worth more than two retained title to their respective 1/16 shares. They were deemed to be holding
thousand pesos, the father or mother shall be considered a the 3/16 shares of Eutropia, Victoria and Douglas under an implied constructive
guardian of the child’s property, subject to the duties and trust for the latter’s benefit, conformably with Article 1456 of the Civil Code
obligations of guardians under the Rules of Court. which states:"if property is acquired through mistake or fraud, the person
x Corollarily, Section 7, Rule 93 of the Rules of Court also provides: obtaining it is, by force of law, considered a trustee of an implied trust for the
o SEC. 7. Parents as Guardians. - When the property of the child benefit of the person from whom the property comes." As such, it is only fair,
under parental authority is worth two thousand pesos or less, the just and equitable that the amount paid for their shares equivalent to P
father or the mother, without the necessity of court appointment, 5,000.0021 each or a total of P 15,000.00 be returned to spouses Uy with legal
shall be his legal guardian. When the property of the child is worth interest.
more than two thousand pesos, the father or the mother shall be x On the issue of prescription, the Court agrees with petitioners that the present
considered guardian of the child’s property, with the duties and action has not prescribed in so far as it seeks to annul the extrajudicial
obligations of guardians under these Rules, and shall file the settlement of the estate. Contrary to the ruling of the CA, the prescriptive period
petition required by Section 2 hereof. For good reasons, the court of 2 years provided in Section 1 Rule 74 of the Rules of Court reckoned from the
may, however, appoint another suitable persons. execution of the extrajudicial settlement finds no application to petitioners
x Administration includes all acts for the preservation of the property Eutropia, Victoria and Douglas, who were deprived of their lawful participation in
and the receipt of fruits according to the natural purpose of the thing. the subject estate. Besides, an "action or defense for the declaration of the
Any act of disposition or alienation, or any reduction in the substance inexistence of a contract does not prescribe" in accordance with Article 1410 of
of the patrimony of child, exceeds the limits of administration.13 Thus, the Civil Code.
a father or mother, as the natural guardian of the minor under parental x However, the action to recover property held in trust prescribes after 10 years
authority, does not have the power to dispose or encumber the from the time the cause of action accrues,22 which is from the time of actual
property of the latter. Such power is granted by law only to a judicial notice in case of unregistered deed.23 In this case, Eutropia, Victoria and
guardian of the ward’s property and even then only with courts’ prior Douglas claimed to have knowledge of the extrajudicial settlement with sale after
approval secured in accordance with the proceedings set forth by the the death of their father, Enrique, in 1994 which spouses Uy failed to refute.
Rules of Court.14 Hence, the complaint filed in 1997 was well within the prescriptive period of 10
x Consequently, the disputed sale entered into by Enrique in behalf of his minor years.
children without the proper judicial authority, unless ratified by them upon
reaching the age of majority,15 is unenforceable in accordance with Articles NILO OROPESA, Petitioner, vs. CIRILO OROPESA, Respondent.
1317 and 1403(1) of the Civil Code which provide: G.R. No. 184528 April 25, 2012
o ART. 1317. No one may contract in the name of another without FIRST DIVISION
being authorized by the latter or unless he has by law a right to
represent him. FACTS:
ƒ A contract entered into in the name of another by one x Nilo Æ filed before the RTC a petition to declare him and one Ms. Louie Ginez as
who has no authority or legal representation, or who guardians over the property of his father, the (respondent) Cirilo Oropesa
has acted beyond his powers, shall be unenforceable, o the (respondent) has been afflicted with several maladies and has
unless it is ratified, expressly or impliedly, by the been sickly for over ten (10) years already having suffered a stroke
person on whose behalf it has been executed, before on April 1, 2003 and June 1, 2003, that his judgment and memory
it is revoked by the other contracting party. [were] impaired and such has been evident after his
o ART. 1403. The following contracts are unenforceable, unless they hospitalization; that even before his stroke, the (respondent) was
are ratified: observed to have had lapses in memory and judgment, showing
ƒ (1) Those entered into the name of another person by signs of failure to manage his property properly; that due to his
one who has been given no authority or legal age and medical condition, he cannot, without outside aid, manage
representation, or who has acted beyond his powers; his property wisely, and has become an easy prey for deceit and
x Ratification means that one under no disability voluntarily adopts and exploitation by people around him, particularly Ms. Ma. Luisa
gives sanction to some unauthorized act or defective proceeding, Agamata, his girlfriend.
which without his sanction would not be binding on him. It is this x RTC Æ ordered a social worker to study the case
voluntary choice, knowingly made, which amounts to a ratification of x Social worker Æ submitted his findings BUT WITHOUT findings on respondent
what was theretofore unauthorized, and becomes the authorized act of Cirilo, as the latter refused to talk to the social worker
108
RECTO, GAYLE ANGELI M.
2011-0008 | AUSL
Personal Notes on Remedial Law 2 Review (based on the syllabus of Prof. Henedino M. Brondial)

x Cirilo Æ then filed an Opposition to the petition for guardianship x Respondent denied the allegations made by petitioner and cited petitioner’s lack
x Nilo (after presenting his witnesses) Æ FAILED to file a Formal Offer of Evidence of material evidence to support his claims. According to respondent, petitioner
x Cirilo Æ filed an Omnibus Motion (1) to Declare the petitioner to have waived the did not present any relevant documentary or testimonial evidence that would
presentation of his Offer of Exhibits and the presentation of his Evidence Closed attest to the veracity of his assertion that respondent is incompetent largely due
since they were not formally offered; (2) To Expunge the Documents of the to his alleged deteriorating medical and mental condition. In fact, respondent
Petitioner from the Record; and (3) To Grant leave to the Oppositor to File points out that the only medical document presented by petitioner proves that he
Demurrer to Evidence is indeed competent to run his personal affairs and administer his properties.
o RTC Æ granted Portions of the said document, entitled "Report of Neuropsychological
x Cirilo Æ filed a demurrer Screening,"15 were quoted by respondent in his Memorandum16 to illustrate
o RTC Æ granted; denied ensuing MR that said report in fact favored respondent’s claim of competence, to wit:
x Nilo Æ appealed to CA x General Oropesa spoke fluently in English and Filipino, he enjoyed and
x CA Æ affirmed RTC; denied ensuing MR participated meaningfully in conversations and could be quite elaborate in his
x Nilo Æ filed a Rule 45 before the SC responses on many of the test items. He spoke in a clear voice and his
articulation was generally comprehensible. x x x.
ISSUE: Whether Cirilo may be considered as an “incompetent” person within the x General Oropesa performed in the average range on most of the domains that
contemplation of Section 2 Rule 92. were tested. He was able to correctly perform mental calculations and keep track
of number sequences on a task of attention. He did BEST in visuo-constructional
HELD: NO. tasks where he had to copy geometrical designs using tiles. Likewise, he was
x Petitioner comes before the Court arguing that the assailed rulings of the Court able to render and read the correct time on the Clock Drawing Test. x x x.
of Appeals should be set aside as it allegedly committed grave and reversible x x x x Reasoning abilities were generally intact as he was able to suggest effective
error when it affirmed the erroneous decision of the trial court which purportedly solutions to problem situations. x x x.17
disregarded the overwhelming evidence presented by him showing respondent’s x With the failure of petitioner to formally offer his documentary evidence, his
incompetence. proof of his father’s incompetence consisted purely of testimonies given by
x In Francisco v. Court of Appeals,10 we laid out the nature and purpose of himself and his sister (who were claiming interest in their father’s real and
guardianship in the following wise: personal properties) and their father’s former caregiver (who admitted to be
o A guardianship is a trust relation of the most sacred acting under their direction). These testimonies, which did not include any expert
character, in which one person, called a "guardian" acts for medical testimony, were insufficient to convince the trial court of petitioner’s
another called the "ward" whom the law regards as cause of action and instead lead it to grant the demurrer to evidence that was
incapable of managing his own affairs. A guardianship is filed by respondent.
designed to further the ward’s well-being, not that of the x Even if we were to overlook petitioner’s procedural lapse in failing to make a
guardian. It is intended to preserve the ward’s property, as formal offer of evidence, his documentary proof were comprised mainly of
well as to render any assistance that the ward may certificates of title over real properties registered in his, his father’s and his
personally require. It has been stated that while custody sister’s names as co-owners, tax declarations, and receipts showing payment of
involves immediate care and control, guardianship real estate taxes on their co-owned properties, which do not in any way relate to
indicates not only those responsibilities, but those of one in his father’s alleged incapacity to make decisions for himself. The only medical
loco parentis as well.11 document on record is the aforementioned "Report of Neuropsychological
x In a guardianship proceeding, a court may appoint a qualified guardian if the Screening" which was attached to the petition for guardianship but was never
prospective ward is proven to be a minor or an incompetent. identified by any witness nor offered as evidence. In any event, the said report,
x A reading of Section 2, Rule 92 of the Rules of Court tells us that persons who, as mentioned earlier, was ambivalent at best, for although the report had
though of sound mind but by reason of age, disease, weak mind or negative findings regarding memory lapses on the part of respondent, it also
other similar causes, are incapable of taking care of themselves and contained findings that supported the view that respondent on the average was
their property without outside aid are considered as incompetents who indeed competent.
may properly be placed under guardianship. The full text of the said x In an analogous guardianship case wherein the soundness of mind of the
provision reads: proposed ward was at issue, we had the occasion to rule that "where the sanity
o Sec. 2. Meaning of the word "incompetent." - Under this rule, the of a person is at issue, expert opinion is not necessary [and that] the
word "incompetent" includes persons suffering the penalty of civil observations of the trial judge coupled with evidence establishing the person’s
interdiction or who are hospitalized lepers, prodigals, deaf state of mental sanity will suffice."18
and dumb who are unable to read and write, those who are x Thus, it is significant that in its Order dated November 14, 2006 which denied
of unsound mind, even though they have lucid intervals, petitioner’s motion for reconsideration on the trial court’s unfavorable September
and persons not being of unsound mind, but by reason of 27, 2006 ruling, the trial court highlighted the fatal role that petitioner’s own
age, disease, weak mind, and other similar causes, cannot, documentary evidence played in disproving its case and, likewise, the trial court
without outside aid, take care of themselves and manage made known its own observation of respondent’s physical and mental state, to
their property, becoming thereby an easy prey for deceit wit:
and exploitation. o The Court noted the absence of any testimony of a medical expert
x We have held in the past that a "finding that a person is incompetent which states that Gen. Cirilo O. Oropesa does not have the mental,
should be anchored on clear, positive and definite evidence."12 We emotional, and physical capacity to manage his own affairs. On the
consider that evidentiary standard unchanged and, thus, must be applied in the contrary, Oppositor’s evidence includes a Neuropsychological
case at bar. Screening Report which states that Gen. Oropesa, (1) performs on
x In support of his contention that respondent is incompetent and, therefore, the average range in most of the domains that were tested; (2) is
should be placed in guardianship, petitioner raises in his Memorandum13 the capable of mental calculations; and (3) can provide solutions to
following factual matters: problem situations. The Report concludes that Gen. Oropesa
o a. Respondent has been afflicted with several maladies and has possesses intact cognitive functioning, except for mildly impaired
been sickly for over ten (10) years already; abilities in memory, reasoning and orientation. It is the observation
o b. During the time that respondent was hospitalized at the St. of the Court that oppositor is still sharp, alert and able.19 (Citation
Luke’s Medical Center after his stroke, he purportedly requested omitted; emphasis supplied.)
one of his former colleagues who was visiting him to file a loan x It is axiomatic that, as a general rule, "only questions of law may be raised in a
application with the Armed Forces of the Philippines Savings and petition for review on certiorari because the Court is not a trier of facts."20 We
Loan Association, Inc. (AFPSLAI) for payment of his hospital bills, only take cognizance of questions of fact in certain exceptional circumstances;21
when, as far as his children knew, he had substantial amounts of however, we find them to be absent in the instant case. It is also long settled
money in various banks sufficient to cover his medical expenses; that "factual findings of the trial court, when affirmed by the Court of Appeals,
o c. Respondent’s residence allegedly has been left dilapidated due to will not be disturbed by this Court. As a rule, such findings by the lower courts
lack of care and management; are entitled to great weight and respect, and are deemed final and conclusive on
o d. The realty taxes for respondent’s various properties remain this Court when supported by the evidence on record."22 We therefore adopt the
unpaid and therefore petitioner and his sister were supposedly factual findings of the lower court and the Court of Appeals and rule that the
compelled to pay the necessary taxes; grant of respondent’s demurrer to evidence was proper under the circumstances
o e. Respondent allegedly instructed petitioner to sell his Nissan obtaining in the case at bar.
Exalta car for the reason that the former would be purchasing x Section 1, Rule 33 of the Rules of Court provides:
another vehicle, but when the car had been sold, respondent did o Section 1. Demurrer to evidence. - After the plaintiff has completed
not procure another vehicle and refused to account for the money the presentation of his evidence, the defendant may move for
earned from the sale of the old car; dismissal on the ground that upon the facts and the law the
o f. Respondent withdrew at least $75,000.00 from a joint account plaintiff has shown no right to relief. If his motion is denied, he
under his name and his daughter’s without the latter’s knowledge shall have the right to present evidence. If the motion is granted but
or consent; on appeal the order of dismissal is reversed he shall be deemed to
o g. There was purportedly one occasion where respondent took a have waived the right to present evidence.
kitchen knife to stab himself upon the "orders" of his girlfriend x A demurrer to evidence is defined as "an objection by one of the parties in an
during one of their fights; action, to the effect that the evidence which his adversary produced is
o h. Respondent continuously allows his girlfriend to ransack his insufficient in point of law, whether true or not, to make out a case or sustain
house of groceries and furniture, despite protests from his the issue."23 We have also held that a demurrer to evidence "authorizes a
children.14 judgment on the merits of the case without the defendant having to submit

109
RECTO, GAYLE ANGELI M.
2011-0008 | AUSL
Personal Notes on Remedial Law 2 Review (based on the syllabus of Prof. Henedino M. Brondial)

evidence on his part, as he would ordinarily have to do, if plaintiff’s evidence qualifications for a guardian as provided by law. Further, he was not given the
shows that he is not entitled to the relief sought."241âwphi1 opportunity to submit evidence to controvert Biason’s appointment.16
x There was no error on the part of the trial court when it dismissed the petition x Abad also bewails his disqualification as guardian on the sole basis of his
for guardianship without first requiring respondent to present his evidence residence. He emphasizes that it is not a requirement for a guardian to be a
precisely because the effect of granting a demurrer to evidence other than resident of the same locality as the ward, or to be living with the latter under the
dismissing a cause of action is, evidently, to preclude a defendant from same roof in order to qualify for the appointment. The more significant
presenting his evidence since, upon the facts and the law, the plaintiff has shown considerations are that the person to be appointed must be of good moral
no right to relief. character and must have the capability and sound judgment in order that he may
be able to take care of the ward and prudently manage his assets.17
EDUARDO T. ABAD, Petitioner, vs. LEONARDO BIASON and GABRIEL A. MAGNO,
Respondents. x Unfortunately, pending the resolution of the instant petition, Biason died. On May
G.R. No. 191993 December 5, 2012 11, 2012, Maura filed a Manifestation and Motion,18 informing this Court that
FIRST DIVISION Biason passed away on April 3, 2012 at SDS Medical Center, Marikina City due to
multiple organ failure, septic shock, community acquired pneumonia high risk,
FACTS: prostate CA with metastasis, and attached a copy of his Death Certificate.19
x Abad (Maura’s nephew)Æ filed a petition for guardianship over the person and Maura averred that Biason’s death rendered moot and academic the issues
properties of Maura B. Abad (Maura) with RTC Dagupan raised in the petition. She thus prayed that the petition be dismissed and the
o Maura, who is single, more than ninety (90) years old and a guardianship be terminated.
resident of Rizal Street, Poblacion, Mangaldan, Pangasinan, is in x On June 20, 2012, this Court issued a Resolution,20 requiring Abad to comment
dire need of a guardian who will look after her and her business on the manifestation filed by Maura. Pursuant to the Resolution, Abad filed his
affairs. Due to her advanced age, Maura is already sickly and can Comment21 on August 9, 2012 and expressed his acquiescence to Maura’s
no longer manage to take care of herself and her properties motion to dismiss the petition. He asseverated that the issues raised in the
unassisted thus becoming an easy prey of deceit and exploitation petition pertain to the irregularity in the appointment of Biason as guardian
x Atty. Gabriel Magno Æ filed a Motion for Leave to Intervene, together with an which he believed had been rendered moot and academic by the latter’s death.
Opposition-in- Intervention He also supported Maura’s prayer for the termination of the guardianship by
x Leonardo Biason (also a nephew) Æ filed a Motion for Leave to File Opposition to asseverating that her act of filing of a petition-in-intervention is indicative of the
the Petition and attached therewith his Opposition to the Appointment of fact that she is of sound mind and that she can competently manage her
Eduardo Abad as Guardian of the Person and Properties of Maura B. Abad business affairs.
o alleged that he is also a nephew of Maura and that he was not o We find Maura’s motion meritorious.
notified of the pendency of the petition for the appointment of the x An issue or a case becomes moot and academic when it ceases to
latter’s guardian. He vehemently opposed the appointment of Abad present a justiciable controversy, so that a determination of the issue
as Maura’s guardian as he cannot possibly perform his duties as would be without practical use and value. In such cases, there is no
such since he resides in Quezon City while Maura maintains her actual substantial relief to which the petitioner would be entitled and
abode in Mangaldan, Pangasinan. Biason prayed that he be which would be negated by the dismissal of the petition.22
appointed as Maura’s guardian since he was previously granted by x In his petition, Abad prayed for the nullification of the CA Decision dated August
the latter with a power of attorney to manage her properties 28, 2009 and Resolution dated April 19, 2010, which dismissed his appeal from
x RTC Æ denied Abad’s petition; appointed BIASON as Maura’s guardian the Decision dated September 26, 2007 of the RTC and denied his motion for
x Abad Æ filed an MR reconsideration, respectively. Basically, he was challenging Biason’s qualifications
o RTC Æ denied and the procedure by which the RTC appointed him as guardian for Maura.
x Abad Æ appealed to CA However, with Biason’s demise, it has become impractical and futile to
o argued that the RTC erred in disqualifying him from being proceed with resolving the merits of the petition. It is a well-
appointed as Maura’s guardian despite the fact that he has all the established rule that the relationship of guardian and ward is
qualifications stated under the Rules. That he was not a resident of necessarily terminated by the death of either the guardian or the
Mangaldan, Pangasinan should not be a ground for his ward.23 The supervening event of death rendered it pointless to delve
disqualification as he had actively and efficiently managed the into the propriety of Biason’s appointment since the juridical tie
affairs and properties of his aunt even if he is residing in Metro between him and Maura has already been dissolved. The petition,
Manila. Moreover, he was expressly chosen by Maura to be her regardless of its disposition, will not afford Abad, or anyone else for
guardian that matter, any substantial relief.1âwphi1
o averred that no hearing was conducted to determine the x Moreover, Abad, in his Comment, shared Maura’s belief that the petition has lost
qualifications of Biason prior to his appointment as guardian. He its purpose and even consented to Maura’s prayer for the dismissal of the
claimed that the RTC also overlooked Maura’s express objection to petition.
Biason’s appointment
x CA Æ affirmed RTC
o The petitioner-appellant may have been correct in arguing that TRUSTEES [RULE 98]
there is no legal requirement that the guardian must be residing in
the same dwelling place or municipality as that of the ward or 1. Parties
incompetent, and that the Vancil vs. Belmes case cited by the court
a quo which held that "courts should not appoint as guardians
persons who are not within the jurisdiction of our courts" pertains 2. Kinds/ Classes
to persons who are not residents of the country.
o However, we do not find that the court a quo, by deciding to Section 1. Where trustee appointed. — A trustee necessary to carry into effect the
appoint the oppositor-appellee as guardian, has fallen into grievous provisions of a will on written instrument shall be appointed by the Court of First Instance in
error. which the will was allowed, if it be a will allowed in the Philippines, otherwise by the Court of
o For one, the oppositor-appellee, like petitioner-appellant, is also a First Instance of the province in which the property, or some portion thereof, affected by the
relative, a nephew of the incompetent. There are no vices of trust is situated.
character which have been established as to disqualify him from
being appointed as a guardian. Section 2. Appointment and powers of trustees under will. Executor of former
o Anent the claim of the petitioner-appellant that he has been trustee need not administer trust. — If a testator has omitted in his will to appoint a
expressly chosen by her aunt to be her guardian as evidenced by trustee in the Philippines, and if such appointment is necessary to carry into effect the
her testimony, although it could be given weight, the same could provisions of the will, the proper Court of First Instance may, after notice to all persons
not be heavily relied upon, especially considering the alleged interested, appoint a trustee who shall have the same rights, powers, and duties, and in whom
mental state of the incompetent due to her advanced age. the estate shall vest, as if he had been appointed by the testator. No person succeeding to a
x Abad Æ filed an MR trust as executor or administrator of a former trustee shall be required to accept such trust.
o CA Æ denied
x Abad Æ filed a Rule 45 before the SC Section 3. Appointment and powers of new trustee under written instrument. —
x Maura Æ filed a Motion for Leave to Intervene together with a Petition-in- When a trustee under a written instrument declines, resigns, dies or removed before the
Intervention objects of the trust are accomplished, and no adequate provision is made in such instrument
o He asseverated that the issues raised in the petition pertain to the for supplying the vacancy, the proper Court of First Instance may, after due notice to all
irregularity in the appointment of Biason as guardian which he persons interested, appoint a new trustee to act alone or jointly with the others, as the case
believed had been rendered moot and academic by the latter’s may be. Such new trustee shall have and exercise the same powers, right, and duties as if he
death. He also supported Maura’s prayer for the termination of the had been originally appointed, and the trust estate shall vest in him in like manner as it had
guardianship by asseverating that her act of filing of a petition-in- vested or would have vested, in the trustee in whose place he is substituted and the court
intervention is indicative of the fact that she is of sound mind and may order such conveyance to be made by the former trustee or his representatives, or by the
that she can competently manage her business affairs. other remaining trustees, as may be necessary or proper to vest the trust estate in the new
trustee, either or jointly with the others.
ISSUE: Whether Abad may still question Biason’s qualifications as guardian of Maura.
Section 4. Proceedings where trustee appointed abroad. — When land in the
HELD: NO. Philippines is held in trust for persons resident here by a trustee who derives his authority
x Abad contends that that CA erred in affirming the RTC’s decision despite the fact from without the Philippines, such trustee shall, on petition filed in the Court of First Instance
that it did not hold any hearing to determine whether Biason possessed all the of the province where the land is situated, and after due notice to all persons interested, be
110
RECTO, GAYLE ANGELI M.
2011-0008 | AUSL
Personal Notes on Remedial Law 2 Review (based on the syllabus of Prof. Henedino M. Brondial)

ordered to apply to the court for appointment as trustee; and upon his neglect or refusal to x Alcantaras Æ filed an Opposition
comply with such order, the court shall declare such trust vacant, and shall appoint a new o claimed that the money in the trust account belonged to them
trustee in whom the trust estate shall vest in like manner as if he had been originally under their Trust Agreement8 with Advent Capital
appointed by such court. o Atty. Concepcion could not claim any right or interest in the
dividends generated by their investments since Advent Capital
Section 5. Trustee must file bond. — Before entering on the duties of his trust, a trustee merely held these in trust for the Alcantaras, the trustors-
shall file with the clerk of the court having jurisdiction of the trust a bond in the amount fixed beneficiaries
by the judge of said court, payable to the Government of the Philippines and sufficient and o IN OTHER WORDS, the rehabilitation court DOES NOT have
available for the protection of any party in interest, and a trustee who neglects to file such jurisdiction over the subject dividends
bond shall be considered to have declined or resigned the trust; but the court may until x RTC Æ granted Atty. Concepcion’s motion
further order exempt a trustee under a will from giving a bond when the testator has directed o under Rule 59, Section 6 of the Rules of Court, a receiver has the
or requested such exemption and may so exempt any trustee when all persons beneficially duty to immediately take possession of all of the corporation’s
interested in the trust, being of full age, request the exemption. Such exemption may be assets and administer the same for the benefit of corporate
cancelled by the court at any time and the trustee required to forthwith file a bond. creditors. He has the duty to collect debts owing to the corporation,
which debts form part of its assets. Complying with the
Section 6. Conditions included in bond. — The following conditions shall be deemed to rehabilitation court’s order and Atty. Concepcion’s demand letter,
be part of the bond whether written therein or not; Belson turned over the subject dividends to him
x Alcantaras Æ filed a Rule 65 before the CA
(a) That the trustee will make and return to the court, at such time as it may order, x CA Æ granted the petition
a true inventory of all the real and personal estate belonging to him as trustee, which at the o ruled that the Alcantaras owned those dividends. They did not form
time of the making of such inventory shall have come to his possession or knowledge; part of Advent Capital’s assets as contemplated under the Interim
Rules of Procedure on Corporate Rehabilitation (Interim Rules).
(b) That he will manage and dispose of all such estate, and faithfully discharge his o the rehabilitation proceedings in this case referred only to the
trust in relation thereto, according to law and the will of the testator or the provisions of the assets and liabilities of the company proper, not to those of its
instrument or order under which he is appointed; Trust Department which held assets belonging to other people.
o Moreover, even if the Trust Agreement provided that Advent
(c) That he will render upon oath at least once a year until his trust is fulfilled, Capital, as trustee, shall have first lien on the Alcantara’s financial
unless he is excused therefrom in any year by the court, a true account of the property in his portfolio for the payment of its trust fees, the cash dividends in
hands and the management and disposition thereof, and will render such other accounts as Belson’s care cannot be summarily applied to the payment of such
the court may order; charges.
o To enforce its lien, Advent Capital has to file a collection suit.
(d) That at the expiration of his trust he will settle his account in court and pay over o The rehabilitation court cannot simply enforce the latter’s claim by
and deliver all the estate remaining in his hands, or due from him on such settlement, to the ordering Belson to deliver the money to it
person or persons entitled to thereto. x Atty. Concepcion Æ filed an MR
o CA Æ denied
But when the trustee is appointed as a successor to a prior trustee, the court may dispense with x Atty. Concepcion Æ filed a Rule 45 before the SC
the making and return of an inventory, if one has already been filed, and in such case the
condition of the bond shall be deemed to be altered accordingly. ISSUE: Whether Atty. Concepcion, in his capacity as Advent Capital’s receiver, can rightfully
collect the subject dividends which are allegedly the subject matter of the trust agreement
Section 7. Appraisal. Compensation of trustee. — When an inventory is required to be between the latter and the Alcantaras.
returned by a trustee, the estate and effects belonging to the trust shall be appraised and the
court may order one or more inheritance tax appraisers to assist in the appraisement. The HELD: NO.
compensation of the trustee shall be fixed by the court, if it be not determined in the x Advent Capital asserts that the cash dividends in Belson’s possession formed part
instrument creating the trust. of its assets based on paragraph 9 of its Trust Agreement with the Alcantaras,
which states:
Section 8. Removal or resignation of trustee. — The proper Court of First Instance may, o 9. Trust Fee: Other Expenses - As compensation for its services
upon petition of the parties beneficially interested and after due notice to the trustee and hereunder, the TRUSTEE shall be entitled to a trust or management
hearing, remove a trustee if such removal appears essential in the interest of the petitioner. The fee of 1 (one) % per annum based on the quarterly average
court may also, after due notice to all persons interested, remove a trustee who is insane or market value of the Portfolio or a minimum annual fee of
otherwise incapable of discharging his trust or evidently unsuitable therefor. A trustee, whether P5,000.00, whichever is higher. The said trust or management fee
appointed by the court or under a written instrument, may resign his trust if it shall automatically be deducted from the Portfolio at the end of
appears to the court proper to allow such resignation. each calendar quarter. The TRUSTEE shall likewise be reimbursed
for all reasonable and necessary expenses incurred by it in the
Section 9. Proceedings for sale or encumbrance of trust estate. — When the sale or discharge of its powers and duties under this Agreement, and in all
encumbrance of any real or personal estate held in trust is necessary or expedient, the court cases, the TRUSTEE shall have a first lien on the Portfolio for the
having jurisdiction of the trust may, on petition and after due notice and hearing, order such payment of the trust fees and other reimbursable expenses.
sale or encumbrance to be made, and the re-investment and application of the proceeds x According to Advent Capital, it could automatically deduct its management fees
thereof in such manner as will best effect the objects of the trust. The petition, notice, from the Alcantaras’ portfolio that they entrusted to it. Paragraph 9 of the Trust
hearing, order of sale or encumbrance, and record of proceedings, shall conform as nearly as Agreement provides that Advent Capital could automatically deduct its trust fees
may be to the provisions concerning the sale or encumbrance by guardians of the property of from the Alcantaras’ portfolio, "at the end of each calendar quarter," with the
minors or other wards. corresponding duty to submit to the Alcantaras a quarterly accounting report
within 20 days after.13
CASES x But the problem is that the trust fees that Advent Capital’s receiver
was claiming were for past quarters. Based on the stipulation, these
ADVENT CAPITAL AND FINANCE CORPORATION, Petitioner, vs. NICASIO I. should have been deducted as they became due. As it happened, at the
ALCANTARA and EDITHA I. ALCANTARA, Respondents. time Advent Capital made its move to collect its supposed management fees, it
G.R. No. 183050 January 25, 2012 neither had possession nor control of the money it wanted to apply to its claim.
THIRD DIVISION Belson, a third party, held the money in the Alcantaras’ names. Whether it
should deliver the same to Advent Capital or to the Alcantaras is not clear. What
FACTS: is clear is that the issue as to who should get the same has been seriously
x Advent Capital Æ filed a petition for rehabilitation before RTC Makati contested.
x RTC Æ appointed one Atty. Danilo L. Concepcion as rehabilitation receiver x The practice in the case of banks is that they automatically collect their
x Atty. Concepcion Æ conducted an audit management fees from the funds that their clients entrust to them for
o found that respondents Nicasio and Editha Alcantara (collectively, investment or lending to others. But the banks can freely do this since it holds or
the Alcantaras) owed Advent Capital P27,398,026.59, representing has control of their clients’ money and since their trust agreement authorized the
trust fees that it supposedly earned for managing their several trust automatic collection. If the depositor contests the deduction, his remedy is to
accounts bring an action to recover the amount he claims to have been illegally deducted
x Atty. Concepcion Æ then requested Belson Securities, Inc. (Belson) to deliver to from his account.
him, as Advent Capital’s rehabilitation receiver, the P7,635,597.50 in cash x Here, Advent Capital does not allege that Belson had already deducted the
dividends that Belson held under the Alcantaras’ Trust Account 95-013 management fees owing to it from the Alcantaras’ portfolio at the end of each
o claimed that the dividends, as trust fees, formed part of Advent calendar quarter. Had this been done, it may be said that the money in Belson’s
Capital’s assets possession would technically be that of Advent Capital. Belson would be holding
x Belson Æ refused such amount in trust for the latter. And it would be for the Alcantaras to institute
o citing the Alcantaras’ objections as well as the absence of an an action in the proper court against Advent Capital and Belson for misuse of its
appropriate order from the rehabilitation court funds.
x Atty. Concepcion Æ filed a motion before the rehabilitation court to direct Belson x But the above did not happen. Advent Capital did not exercise its right to cause
to release the money to him. the automatic deduction at the end of every quarter of its supposed management
o As rehabilitation receiver, he had the duty to take custody and fee when it had full control of the dividends. That was its fault. For their part, the
control of Advent Capital’s assets, such as the sum of money that Alcantaras had the right to presume that Advent Capital had
Belson held on behalf of Advent Capital’s Trust Department
111
RECTO, GAYLE ANGELI M.
2011-0008 | AUSL
Personal Notes on Remedial Law 2 Review (based on the syllabus of Prof. Henedino M. Brondial)

deducted its fees in the manner stated in the contract. The burden of proving o alleged that ACDC acted as a subcontractor for government
that the fees were not in fact collected lies with Advent Capital. projects such as the Metro Rail Transit, the Clark Centennial
x Further, Advent Capital or its rehabilitation receiver cannot unilaterally decide to Exposition and the Quezon Power Plant in Mauban, Quezon.
apply the entire amount of cash dividends retroactively to cover the accumulated ƒ Its clients for the construction projects, which were
trust fees. Advent Capital merely managed in trust for the benefit of the the general contractors of these projects, have not
Alcantaras the latter’s portfolio, which under Paragraph 214 of the Trust yet paid them;
Agreement, includes not only the principal but also its income or proceeds. The ƒ thus, ACDC had yet to receive the proceeds of the
trust property is only fictitiously attributed by law to the trustee "to the extent materials that were the subject of the trust receipts
that the rights and powers vested in a nominal owner shall be used by him on and were allegedly used for these constructions.
behalf of the real owner."15 ƒ As there were no proceeds received from these
x The real owner of the trust property is the trustor-beneficiary. In this clients, no misappropriation thereof could have taken
case, the trustors-beneficiaries are the Alcantaras. Thus, Advent place.
Capital could not dispose of the Alcantaras’ portfolio on its own. The x Makati Prosecutor Æ dismissed the complaint
income and principal of the portfolio could only be withdrawn upon the o He pointed out that the evidence presented by LBP failed to state
Alcantaras’ written instruction or order to Advent Capital.16 The latter the date when the goods described in the letters of credit were
could not also assign or encumber the portfolio or its income without actually released to the possession of the respondents.
the written consent of the Alcantaras.17 All these are stipulated in the Trust o Section 4 of P.D. 115 requires that the goods covered by trust
Agreement. receipts be released to the possession of the entrustee after the
x Ultimately, the issue is what court has jurisdiction to hear and adjudicate the latter’s execution and delivery to the entruster of a signed trust
conflicting claims of the parties over the dividends that Belson held in trust for receipt.
their owners. Certainly, not the rehabilitation court which has not been given the o He adds that LBP’s evidence also fails to show the date when the
power to resolve ownership disputes between Advent Capital and third parties. trust receipts were executed since all the trust receipts are undated
Neither Belson nor the Alcantaras are its debtors or creditors with interest in the x Landbank Æ filed an MR
rehabilitation. o Makati Prosecutor Æ denied
x Advent Capital must file a separate action for collection to recover the x Landbank Æ appealed to DOJ
trust fees that it allegedly earned and, with the trial court’s x DOJ Æ reversed Makati Prosecutor
authorization if warranted, put the money in escrow for payment to o pointed out that there was no question that the goods covered by
whoever it rightly belongs. Having failed to collect the trust fees at the the trust receipts were received by ACDC. He likewise adopted
end of each calendar quarter as stated in the contract, all it had against LBP’s argument that while the subjects of the trust receipts were
the Alcantaras was a claim for payment which is a proper subject for not mentioned in the trust receipts, they were listed in the letters
an ordinary action for collection. It cannot enforce its money claim by of credit referred to in the trust receipts.
simply filing a motion in the rehabilitation case for delivery of money o He also noted that the trust receipts contained maturity dates and
belonging to the Alcantaras but in the possession of a third party. clearly set out their stipulations.
x Rehabilitation proceedings are summary and non-adversarial in nature, o He further rejected the respondents’ defense that ACDC failed to
and do not contemplate adjudication of claims that must be threshed remit the payments to LBP due to the failure of the clients of ACDC
out in ordinary court proceedings. Adversarial proceedings similar to that in to pay them
ordinary courts are inconsistent with the commercial nature of a rehabilitation x Respondents Æ filed an MR
case. The latter must be resolved quickly and expeditiously for the sake of the o DOJ Æ denied
corporate debtor, its creditors and other interested parties. Thus, the Interim x Respondents Æ filed a petition for review before the CA
Rules "incorporate the concept of prohibited pleadings, affidavit evidence in lieu x CA Æ reversed DOJ
of oral testimony, clarificatory hearings instead of the traditional approach of o Applying the doctrine in Colinares, it ruled that this case did not
receiving evidence, and the grant of authority to the court to decide the case, or involve a trust receipt transaction, but a mere loan. It emphasized
any incident, on the basis of affidavits and documentary evidence."18 that construction materials, the subject of the trust receipt
x Here, Advent Capital’s claim is disputed and requires a full trial on the transaction, were delivered to ACDC even before the trust receipts
merits.1âwphi1 It must be resolved in a separate action where the Alcantaras’ were executed. It noted that LBP did not offer proof that the goods
claim and defenses may also be presented and heard. Advent Capital cannot say were received by ACDC, and that the trust receipts did not contain
that the filing of a separate action would defeat the purpose of corporate a description of the goods, their invoice value, the amount of the
rehabilitation. In the first place, the Interim Rules do not exempt a company draft to be paid, and their maturity dates. It also adopted ACDC’s
under rehabilitation from availing of proper legal procedure for collecting debt argument that since no payment for the construction projects had
that may be due it. Secondly, Court records show that Advent Capital had in fact been received by ACDC, its officers could not have been guilty of
sought to recover one of its assets by filing a separate action for replevin misappropriating any payment.
involving a car that was registered in its name. x Landbank Æ filed a Rule 45 before the SC
x Respondents (while the case was pending before the SC) Æ filed an MD
LAND BANK OF THE PHILIPPINES, Petitioner, vs. LAMBERTO C. PEREZ, NESTOR C. o informed the Court that LBP had already assigned to Philippine
KUN, MA. ESTELITA P. ANGELES-PANLILIO, and NAPOLEON O. GARCIA, Respondents. Opportunities for Growth and Income, Inc. all of its rights, title and
G.R. No. 166884 June 13, 2012 interests in the loans subject of this case in a Deed of Absolute Sale
SECOND DIVISION dated June 23, 2005
o also stated that Avent Holdings Corporation, in behalf of ACDC, had
FACTS: already settled ACDC’s obligation to LBP on October 8, 2009.
x Respondents are the officers and representatives of Asian Construction and Included as Annex "A" in this motion was a certification21 issued by
Development Corporation (ACDC), a corporation incorporated under Philippine the Philippine Opportunities for Growth and Income, Inc.,
law and engaged in the construction business stating that it was LBP’s successor-in-interest insofar as the trust
x Landbank Æ filed a complaint for estafa against respondents before the receipts in this case are concerned and that Avent Holdings
Prosecutor’s Office Makati Corporation had already settled the claims of LBP or obligations of
o the LBP’s Account Officer for the Account Management ACDC arising from these trust receipts
Development, Edna L. Juan, stated that LBP extended a credit
accommodation to ACDC through the execution of an Omnibus ISSUE: Whether the subject transactions may properly be considered as trust receipts so as to
Credit Line Agreement (Agreement)6 between LBP and ACDC on render the herein respondents liable for estafa.
October 29, 1996
o ACDC used the Letters of Credit/Trust Receipts Facility of the HELD: NO.
Agreement to buy construction materials x Section 4 of P.D. 115 defines a trust receipt transaction in this manner:
o respondents, as officers and representatives of ACDC, executed o Section 4. What constitutes a trust receipt transaction. A trust
trust receipts7 in connection with the construction materials, with a receipt transaction, within the meaning of this Decree, is any
total principal amount of P52,344,096.32. transaction by and between a person referred to in this Decree as
o The trust receipts matured, but ACDC failed to return to LBP the the entruster, and another person referred to in this Decree as
proceeds of the construction projects or the construction materials entrustee, whereby the entruster, who owns or holds absolute title
subject of the trust receipts. or security interests over certain specified goods, documents or
o LBP sent ACDC a demand letter,8 dated May 4, 1999, for the instruments, releases the same to the possession of the entrustee
payment of its debts, including those under the Trust Receipts upon the latter's execution and delivery to the entruster of a signed
Facility in the amount of P66,425,924.39. document called a "trust receipt" wherein the entrustee binds
o When ACDC failed to comply with the demand letter, LBP filed the himself to hold the designated goods, documents or instruments in
affidavit-complaint. trust for the entruster and to sell or otherwise dispose of the
x Respondents Æ filed a joint affidavit goods, documents or instruments with the obligation to turn over
o they signed the trust receipt documents on or about the same time to the entruster the proceeds thereof to the extent of the amount
LBP and ACDC executed the loan documents; owing to the entruster or as appears in the trust receipt or the
o their signatures were required by LBP for the release of the loans. goods, documents or instruments themselves if they are unsold or
o The trust receipts in this case do not contain (1) a description of not otherwise disposed of, in accordance with the terms and
the goods placed in trust, (2) their invoice values, and (3) their conditions specified in the trust receipt, or for other purposes
maturity dates, in violation of Section 5(a) of P.D. 115. substantially equivalent to any of the following:

112
RECTO, GAYLE ANGELI M.
2011-0008 | AUSL
Personal Notes on Remedial Law 2 Review (based on the syllabus of Prof. Henedino M. Brondial)

ƒ 1. In the case of goods or documents, (a) to sell the possession of the trustee until they are sold. But the goods and the materials
goods or procure their sale; or (b) to manufacture or that are used for a construction project are often placed under the control and
process the goods with the purpose of ultimate sale: custody of the clients employing the contractor, who can only be compelled to
Provided, That, in the case of goods delivered under return the materials if they fail to pay the contractor and often only after the
trust receipt for the purpose of manufacturing or requisite legal proceedings. The contractor’s difficulty and uncertainty in claiming
processing before its ultimate sale, the entruster shall these materials (or the buildings and structures which they become part of), as
retain its title over the goods whether in its original or soon as the bank demands them, disqualify them from being covered by trust
processed form until the entrustee has complied fully receipt agreements.
with his obligation under the trust receipt; or (c) to x Based on these premises, we cannot consider the agreements between the
load, unload, ship or tranship or otherwise deal with parties in this case to be trust receipt transactions because (1) from the start,
them in a manner preliminary or necessary to their the parties were aware that ACDC could not possibly be obligated to reconvey to
sale[.] LBP the materials or the end product for which they were used; and (2) from the
x There are two obligations in a trust receipt transaction. The first is moment the materials were used for the government projects, they became
covered by the provision that refers to money under the obligation to public, not LBP’s, property.
deliver it (entregarla) to the owner of the merchandise sold. The x Since these transactions are not trust receipts, an action for estafa should not be
second is covered by the provision referring to merchandise received brought against the respondents, who are liable only for a loan. In passing, it is
under the obligation to return it (devolvera) to the owner. Thus, under useful to note that this is the threat held against borrowers that Retired Justice
the Trust Receipts Law,22 intent to defraud is presumed when (1) the Claudio Teehankee emphatically opposed in his dissent in People v. Cuevo,32
entrustee fails to turn over the proceeds of the sale of goods covered by restated in Ong v. CA, et al.:33
the trust receipt to the entruster; or (2) when the entrustee fails to o The very definition of trust receipt x x x sustains the lower
return the goods under trust, if they are not disposed of in accordance court’s rationale in dismissing the information that the
with the terms of the trust receipts.23 contract covered by a trust receipt is merely a secured
x In all trust receipt transactions, both obligations on the part of the trustee exist loan. The goods imported by the small importer and retail
in the alternative - the return of the proceeds of the sale or the return or dealer through the bank’s financing remain of their own
recovery of the goods, whether raw or processed.24 When both parties enter property and risk and the old capitalist orientation of
into an agreement knowing that the return of the goods subject of the putting them in jail for estafa for non-payment of the
trust receipt is not possible even without any fault on the part of the secured loan (granted after they had been fully
trustee, it is not a trust receipt transaction penalized under Section 13 investigated by the bank as good credit risks) through the
of P.D. 115; the only obligation actually agreed upon by the parties fiction of the trust receipt device should no longer be
would be the return of the proceeds of the sale transaction. This permitted in this day and age.
transaction becomes a mere loan,25 where the borrower is obligated x As the law stands today, violations of Trust Receipts Law are criminally
to pay the bank the amount spent for the purchase of the goods. punishable, but no criminal complaint for violation of Article 315, paragraph 1(b)
x Article 1371 of the Civil Code provides that "[i]n order to judge the intention of of the Revised Penal Code, in relation with P.D. 115, should prosper against a
the contracting parties, their contemporaneous and subsequent acts shall be borrower who was not part of a genuine trust receipt transaction.
principally considered." Under this provision, we can examine the x Misappropriation or abuse of confidence is absent in this case.
contemporaneous actions of the parties rather than rely purely on the trust x Even if we assume that the transactions were trust receipts, the complaint
receipts that they signed in order to understand the transaction through their against the respondents still should have been dismissed. The Trust Receipts
intent. Law punishes the dishonesty and abuse of confidence in the handling of money
x We note in this regard that at the onset of these transactions, LBP knew that or goods to the prejudice of another, regardless of whether the latter is the
ACDC was in the construction business and that the materials that it sought to owner or not. The law does not singularly seek to enforce payment of the loan,
buy under the letters of credit were to be used for the following projects: the as "there can be no violation of [the] right against imprisonment for non-
Metro Rail Transit Project and the Clark Centennial Exposition Project.26 LBP had payment of a debt."34
in fact authorized the delivery of the materials on the construction sites for these x In order that the respondents "may be validly prosecuted for estafa under Article
projects, as seen in the letters of credit it attached to its complaint.27 Clearly, 315, paragraph 1(b) of the Revised Penal Code,35 in relation with Section 13 of
they were aware of the fact that there was no way they could recover the the Trust Receipts Law, the following elements must be established: (a) they
buildings or constructions for which the materials subject of the alleged trust received the subject goods in trust or under the obligation to sell the same and
receipts had been used. Notably, despite the allegations in the affidavit- to remit the proceeds thereof to [the trustor], or to return the goods if not sold;
complaint wherein LBP sought the return of the construction materials,28 its (b) they misappropriated or converted the goods and/or the proceeds of the sale;
demand letter dated May 4, 1999 sought the payment of the balance but failed (c) they performed such acts with abuse of confidence to the damage and
to ask, as an alternative, for the return of the construction materials or the prejudice of Metrobank; and (d) demand was made on them by [the trustor] for
buildings where these materials had been used.29 the remittance of the proceeds or the return of the unsold goods."36
x The fact that LBP had knowingly authorized the delivery of construction materials x In this case, no dishonesty or abuse of confidence existed in the handling of the
to a construction site of two government projects, as well as unspecified construction materials.
construction sites, repudiates the idea that LBP intended to be the owner of x In this case, the misappropriation could be committed should the entrustee fail
those construction materials. As a government financial institution, LBP should to turn over the proceeds of the sale of the goods covered by the trust receipt
have been aware that the materials were to be used for the construction of an transaction or fail to return the goods themselves. The respondents could not
immovable property, as well as a property of the public domain. As an have failed to return the proceeds since their allegations that the clients of ACDC
immovable property, the ownership of whatever was constructed with those had not paid for the projects it had undertaken with them at the time the case
materials would presumably belong to the owner of the land, under Article 445 was filed had never been questioned or denied by LBP. What can only be
of the Civil Code which provides: attributed to the respondents would be the failure to return the goods subject of
o Article 445. Whatever is built, planted or sown on the land of the trust receipts.
another and the improvements or repairs made thereon, belong to x We do not likewise see any allegation in the complaint that ACDC had used the
the owner of the land, subject to the provisions of the following construction materials in a manner that LBP had not authorized. As earlier
articles. pointed out, LBP had authorized the delivery of these materials to these project
x Even if we consider the vague possibility that the materials, consisting of sites for which they were used. When it had done so, LBP should have been
cement, bolts and reinforcing steel bars, would be used for the construction of a aware that it could not possibly recover the processed materials as they would
movable property, the ownership of these properties would still pertain to the become part of government projects, two of which (the Metro Rail Transit
government and not remain with the bank as they would be classified as Project and the Quezon Power Plant Project) had even become part of the
property of the public domain, which is defined by the Civil Code as: operations of public utilities vital to public service. It clearly had no intention of
o Article 420. The following things are property of public dominion: getting these materials back; if it had, as a primary government lending
ƒ (1) Those intended for public use, such as roads, institution, it would be guilty of extreme negligence and incompetence in not
canals, rivers, torrents, ports and bridges constructed foreseeing the legal complications and public inconvenience that would arise
by the State, banks, shores, roadsteads, and others of should it decide to claim the materials. ACDC’s failure to return these materials
similar character; or their end product at the time these "trust receipts" expired could not be
ƒ (2) Those which belong to the State, without being attributed to its volition. No bad faith, malice, negligence or breach of contract
for public use, and are intended for some public has been attributed to ACDC, its officers or representatives. Therefore, absent
service or for the development of the national wealth. any abuse of confidence or misappropriation on the part of the respondents, the
x In contrast with the present situation, it is fundamental in a trust criminal proceedings against them for estafa should not prosper.
receipt transaction that the person who advanced payment for the x In Metropolitan Bank,37 we affirmed the city prosecutor’s dismissal of a
merchandise becomes the absolute owner of said merchandise and complaint for violation of the Trust Receipts Law. In dismissing the complaint, we
continues as owner until he or she is paid in full, or if the goods had took note of the Court of Appeals’ finding that the bank was interested only in
already been sold, the proceeds should be turned over to him or to collecting its money and not in the return of the goods. Apart from the bare
her.30 allegation that demand was made for the return of the goods (raw materials that
x Thus, in concluding that the transaction was a loan and not a trust were manufactured into textiles), the bank had not accompanied its complaint
receipt, we noted in Colinares that the industry or line of work that the with a demand letter. In addition, there was no evidence offered that the
borrowers were engaged in was construction. We pointed out that the borrowers respondents therein had misappropriated or misused the goods in question.
were not importers acquiring goods for resale.31 Indeed, goods sold in retail are x The petition should be dismissed because the OSG did not file it and the civil
often within the custody or control of the trustee until they are purchased. In the liabilities have already been settled.
case of materials used in the manufacture of finished products, these finished x The proceedings before us, regarding the criminal aspect of this case, should be
products - if not the raw materials or their components - similarly remain in the dismissed as it does not appear from the records that the complaint was filed
113
RECTO, GAYLE ANGELI M.
2011-0008 | AUSL
Personal Notes on Remedial Law 2 Review (based on the syllabus of Prof. Henedino M. Brondial)

with the participation or consent of the Office of the Solicitor General (OSG). general guardian or guardian ad litem of the child, or if the child is in the custody of an orphan
Section 35, Chapter 12, Title III, Book IV of the Administrative Code of 1987 asylum, children's home, or benevolent society or person, by the proper officer of such
provides that: asylum, home, or society, or by such person; but if the child is illegitimate and has not been
o Section 35. Powers and Functions. — The Office of the Solicitor recognized, the consent of its father to the adoption shall not be required.
General shall represent the Government of the Philippines, its
agencies and instrumentalities and its officials and agents in any If the person to be adopted is of age, only his or her consent and that of the spouse, if any,
litigation, proceedings, investigation or matter requiring the shall be required.
services of lawyers. x x x It shall have the following specific powers
and functions: Section 4. Order for hearing. — If the petition and consent filed are sufficient in form and
ƒ (1) Represent the Government in the Supreme Court substance, the court, by an order reciting the purpose of the petition, shall fix a date and place
and the Court of Appeals in all criminal proceedings; for the hearing thereof, which shall not be more than six (6) months after the entry of the
represent the Government and its officers in the order, and shall direct that a copy of the order be published before the hearing at least once a
Supreme Court, the Court of Appeals and all other week for three (3) successive weeks in some newspaper of general circulation published in the
courts or tribunals in all civil actions and special province, as the court shall deem best.
proceedings in which the Government or any officer
thereof in his official capacity is a party. (Emphasis Section 5. Hearing and judgment. — Upon satisfactory proof in open court on the date
provided.) fixed in the order that such order has been published as directed, that the allegations of
x In Heirs of Federico C. Delgado v. Gonzalez,38 we ruled that the preliminary petition are true, and that it is a proper case for adoption and the petitioner or petitioners are
investigation is part of a criminal proceeding. As all criminal proceedings before able to bring up and educate the child properly, the court shall adjudge that thenceforth the
the Supreme Court and the Court of Appeals may be brought and defended by child is free from all legal obligations of obedience and maintenance with respect to its natural
only the Solicitor General in behalf of the Republic of the Philippines, a criminal parents, except the mother when the child is adopted by her husband, and is, to all legal
action brought to us by a private party alone suffers from a fatal defect. The intents and purposes, the child of the petitioner or petitioners, and that its surname is changed
present petition was brought in behalf of LBP by the Government Corporate to that the petitioner or petitioners. The adopted person or child shall thereupon become the
Counsel to protect its private interests. Since the representative of the "People of legal heir of his parents by adoption and shall also remain the legal heir of his natural parents.
the Philippines" had not taken any part of the case, it should be In case of the death of the adopted person or child, his parents and relatives by nature, and not
dismissed.1âwphi1 by adoption, shall be his legal heirs.
x On the other hand, if we look at the mandate given to the Office of the
Government Corporate Counsel, we find that it is limited to the civil Section 6. Proceedings as to the child whose parents are separated. Appeal. —
liabilities arising from the crime, and is subject to the control and When husband and wife are divorce or living separately and apart from each other, and the
supervision of the public prosecutor. Section 2, Rule 8 of the Rules question as to the care, custody, and control of a child or children of their marriage is brought
Governing the Exercise by the Office of the Government Corporate Counsel of its before a Court of First Instance by petition or as an incident to any other proceeding, the
Authority, Duties and Powers as Principal Law Office of All Government Owned or court, upon hearing the testimony as may be pertinent, shall award the care, custody, and
Controlled Corporations, filed before the Office of the National Administration control of each such child as will be for its best interest, permitting the child to choose which
Register on September 5, 2011, reads: parent it prefers to live with if it be over ten years of age, unless the parent so chosen be unfit
o Section 2. Extent of legal assistance - The OGCC shall represent to take charge of the child by the reason of moral depravity, habitual drunkenness, incapacity,
the complaining GOCC in all stages of the criminal proceedings. The or poverty. If, upon such hearing, it appears that both parents are improper persons to have
legal assistance extended is not limited to the preparation of the care, custody, and control of the child, the court may either designate the paternal or
appropriate sworn statements but shall include all aspects of an maternal grandparent of the child, or his oldest brother or sister, or some reputable and
effective private prosecution including recovery of civil liability discreet person to take charge of such child, or commit it to any suitable asylum, children's
arising from the crime, subject to the control and supervision of the home, or benevolent society. The court may in conformity with the provisions of the Civil Code
public prosecutor. order either or both parents to support or help support said child, irrespective of who may be
x Based on jurisprudence, there are two exceptions when a private party its custodian, and may make any order that is just and reasonable permitting the parent who
complainant or offended party in a criminal case may file a petition is deprived of its care and custody to visit the child or have temporary custody thereof. Either
with this Court, without the intervention of the OSG: (1) when there is parent may appeal from an order made in accordance with the provisions of this section. No
denial of due process of law to the prosecution, and the State or its child under seven years of age shall be separated from its mother, unless the court finds there
agents refuse to act on the case to the prejudice of the State and the are compelling reasons thereof.
private offended party;39 and (2) when the private offended party
questions the civil aspect of a decision of the lower court.40 Section 7. Proceedings as to vagrant or abused child. — When the parents of any
x In this petition, LBP fails to allege any inaction or refusal to act on the part of the minor child are dead, or by reason of long absence or legal or physical disability have
OSG, tantamount to a denial of due process. No explanation appears as to why abandoned it, or cannot support it through vagrancy, negligence, or misconduct, or neglect or
the OSG was not a party to the case. Neither can LBP now question the civil refuse to support it, or treat it with excessive harshness or give it corrupting orders, counsels,
aspect of this decision as it had already assigned ACDC’s debts to a third person, or examples, or cause or allow it to engage in begging, or to commit offenses against the law,
Philippine Opportunities for Growth and Income, Inc., and the civil liabilities the proper Court of First Instance, upon petition filed by some reputable resident of the
appear to have already been settled by Avent Holdings Corporation, in behalf of province setting forth the facts, may issue an order requiring such parents to show cause, or,
ACDC. These facts have not been disputed by LBP. Therefore, we can reasonably if the parents are dead or cannot be found, requiring the fiscal of the province to show cause,
conclude that LBP no longer has any claims against ACDC, as regards the subject at a time and place fixed in the order, why the child should not be taken from its parents, if
matter of this case, that would entitle it to file a civil or criminal action. living; and if upon the hearing it appears that the allegations of the petition are true, and that
it is order taking it from its parents, if living; and committing it to any suitable orphan asylum,
children's home, or benevolent society or person to be ultimately placed, by adoption or
ADOPTION AND CUSTODY OF MINORS otherwise, in a home found for it by such asylum, children's home, society, or person.
[RULES 99-100]
Section 8. Service of judgment. — Final orders or judgments under this rule shall be
served by the clerk upon the civil registrar of the city or municipality wherein the court issuing
RULE 99 (Adoption and Custody) the same is situated.
Section 1. Venue. — A person desiring to adopt another or have the custody of a minor
shall present his petition to the Court of First Instance of the province, or the municipal or RULE 100 (Rescission and Revocation)
justice of the peace court of the city or municipality in which he resides.
Section 1. Who may file petition; grounds. — A minor or other incapacitated person may,
In the City of Manila, the proceedings shall be instituted in the Juvenile and Domestic through a guardian or guardian ad litem, petition for the rescission or revocation of his or her
Relations Court. adoption for the same causes that authorize the deprivation of parental authority.
Section 2. Contents of petition. — The petition for adoption shall contain the same The adopter may, likewise, petition the court for the rescission of revocation of the adoption in
allegations required in a petition for guardianship, to wit: any of these cases:
(a) The jurisdictional facts; (a) If the adopted person has attempted against the file of the adopter;
(b) The qualification of the adopter; (b) When the adopted minor has abandoned the home of the adopter for more than
three (3) years;
(c) That the adopter is not disqualified by law;
(c) When by other acts the adopted person has repudiated the adoption.
(d) The name, age, and residence of the person to be adopted and of his relatives of
the persons who have him under their care;
Section 2. Order to answer. — The court in which the petition is filed shall issue an order
requiring the adverse party to answer the petition within fifteen (15) days from receipt of a
(e) The probable value and character of the estate of the person to be adopted. copy thereof. The order and a copy of the petition shall be served on the adverse party in such
manner as the court may direct
Section 3. Consent of adoption. — There shall be filed with the petition a written consent
to the adoption signed by the child, if fourteen years of age or over and not incompetent, and Section 3. Judgment. — If upon trial, on the day set therefor, the court finds that the
by the child's spouse, if any, and by each of its known living parents who is not insane or allegations of the petition are true, it shall render judgment ordering the rescission or
hopelessly intemperate or has not abandoned such child, or if there is no such parents by the revocation of the adoption, with or without costs, as justice requires.
114
RECTO, GAYLE ANGELI M.
2011-0008 | AUSL
Personal Notes on Remedial Law 2 Review (based on the syllabus of Prof. Henedino M. Brondial)

(e) "Abandoned child" refers to one who has no proper parental care or guardianship or whose
Section 4. Service of Judgment. — A certified copy of the judgment rendered in parent(s) has deserted him/her for a period of at least six (6) continuous months and has
accordance with the next preceding section shall be served upon the civil registrar concerned, been judicially declared as such.
within thirty (30) days from rendition thereof, who shall forthwith enter the action taken by the
court in the register. (f) "Supervised trial custody" is a period of time within which a social worker oversees the
adjustment and emotional readiness of both adopter(s) and adoptee in stabilizing their filial
Section 5. Time within which to file petition. — A minor or other incapacitated person relationship.
must file the petition for rescission or revocation of adoption within the five (5) years following
his majority, or if he was incompetent at the time of the adoption, within the five (5) years (g) "Department" refers to the Department of Social Welfare and Development.
following the recovery from such incompetency.
(h) "Child-placing agency" is a duly licensed and accredited agency by the Department to
The adopter must also file the petition to set aside the adoption within five (5) years from the provide comprehensive child welfare services including, but not limited to, receiving
time the cause or causes giving rise to the rescission or revocation of the same took place. applications for adoption, evaluating the prospective adoptive parents, and preparing the
adoption home study.
1. The Domestic Adoption Act of 1998
(i) "Child-caring agency" is a duly licensed and accredited agency by the Department that
Republic Act No. 8552 February 25, 1998 provides twenty four (24)-hour residential care services for abandoned, orphaned, neglected,
or voluntarily committed children.
AN ACT ESTABLISHING THE RULES AND POLICIES ON THE DOMESTIC ADOPTION
OF FILIPINO CHILDREN AND FOR OTHER PURPOSES (j) "Simulation of birth" is the tampering of the civil registry making it appear in the birth
records that a certain child was born to a person who is not his/her biological mother, causing
Be it enacted by the Senate and House of Representatives of the Philippines in Congress such child to lose his/her true identity and status.
assembled::
ARTICLE II
ARTICLE I PRE-ADOPTION SERVICES
GENERAL PROVISIONS
Section 4. Counseling Service. - The Department shall provide the services of licensed
Section 1. Short Title. - This Act shall be known as the "Domestic Adoption Act of 1998." social workers to the following:

Section 2. Declaration of Policies. - (a) It is hereby declared the policy of the State to (a) Biological Parent(s) - Counseling shall be provided to the parent(s) before and after the
ensure that every child remains under the care and custody of his/her parent(s) and be birth of his/her child. No binding commitment to an adoption plan shall be permitted before
provided with love, care, understanding and security towards the full and harmonious the birth of his/her child. A period of six (6) months shall be allowed for the biological
development of his/her personality. Only when such efforts prove insufficient and no parent(s) to reconsider any decision to relinquish his/her child for adoption before the decision
appropriate placement or adoption within the child's extended family is available shall adoption becomes irrevocable. Counseling and rehabilitation services shall also be offered to the
by an unrelated person be considered. biological parent(s) after he/she has relinquished his/her child for adoption.

(b) In all matters relating to the care, custody and adoption of a child, his/her interest shall be Steps shall be taken by the Department to ensure that no hurried decisions are made and all
the paramount consideration in accordance with the tenets set forth in the United Nations alternatives for the child's future and the implications of each alternative have been provided.
(UN) Convention on the Rights of the Child; UN Declaration on Social and Legal Principles
Relating to the Protection and Welfare of Children with Special Reference to Foster Placement (b) Prospective Adoptive Parent(s) - Counseling sessions, adoption fora and seminars, among
and Adoption, Nationally and Internationally; and the Hague Convention on the Protection of others, shall be provided to prospective adoptive parent(s) to resolve possible adoption issues
Children and Cooperation in Respect of Intercountry Adoption. Toward this end, the State shall and to prepare him/her for effective parenting.
provide alternative protection and assistance through foster care or adoption for every child who
is neglected, orphaned, or abandoned. (c) Prospective Adoptee - Counseling sessions shall be provided to ensure that he/she
understands the nature and effects of adoption and is able to express his/her views on
(c) It shall also be a State policy to: adoption in accordance with his/her age and level of maturity.

(i) Safeguard the biological parent(s) from making hurried decisions to relinquish his/her Section 5. Location of Unknown Parent(s). - It shall be the duty of the Department or
parental authority over his/her child; the child-placing or child-caring agency which has custody of the child to exert all efforts to
locate his/her unknown biological parent(s). If such efforts fail, the child shall be registered as
(ii) Prevent the child from unnecessary separation from his/her biological parent(s); a foundling and subsequently be the subject of legal proceedings where he/she shall be
declared abandoned.
(iii) Protect adoptive parent(s) from attempts to disturb his/her parental authority and custody
over his/her adopted child. Section 6. Support Services. - The Department shall develop a pre-adoption program
which shall include, among others, the above mentioned services.
Any voluntary or involuntary termination of parental authority shall be administratively or
judicially declared so as to establish the status of the child as "legally available for adoption" ARTICLE III
and his/her custody transferred to the Department of Social Welfare and Development or to ELIGIBILITY
any duly licensed and accredited child-placing or child-caring agency, which entity shall be
authorized to take steps for the permanent placement of the child; Section 7. Who May Adopt. - The following may adopt:

(iv) Conduct public information and educational campaigns to promote a positive environment (a) Any Filipino citizen of legal age, in possession of full civil capacity and legal rights, of good
for adoption; moral character, has not been convicted of any crime involving moral turpitude, emotionally
and psychologically capable of caring for children, at least sixteen (16) years older than the
(v) Ensure that sufficient capacity exists within government and private sector agencies to adoptee, and who is in a position to support and care for his/her children in keeping with the
handle adoption inquiries, process domestic adoption applications, and offer adoption-related means of the family. The requirement of sixteen (16) year difference between the age of the
services including, but not limited to, parent preparation and post-adoption education and adopter and adoptee may be waived when the adopter is the biological parent of the adoptee,
counseling; and or is the spouse of the adoptee's parent;

(vi) Encourage domestic adoption so as to preserve the child's identity and culture in his/her (b) Any alien possessing the same qualifications as above stated for Filipino nationals:
native land, and only when this is not available shall intercountry adoption be considered as a Provided, That his/her country has diplomatic relations with the Republic of the Philippines,
last resort. that he/she has been living in the Philippines for at least three (3) continuous years prior to
the filing of the application for adoption and maintains such residence until the adoption
Section 3. Definition of Terms. - For purposes of this Act, the following terms shall be decree is entered, that he/she has been certified by his/her diplomatic or consular office or
defined as: any appropriate government agency that he/she has the legal capacity to adopt in his/her
country, and that his/her government allows the adoptee to enter his/her country as his/her
adopted son/daughter: Provided, Further, That the requirements on residency and certification
(a) "Child" is a person below eighteen (18) years of age.
of the alien's qualification to adopt in his/her country may be waived for the following:
(b) "A child legally available for adoption" refers to a child who has been voluntarily or
(i) a former Filipino citizen who seeks to adopt a relative within the fourth (4th) degree of
involuntarily committed to the Department or to a duly licensed and accredited child-placing or
consanguinity or affinity; or
child-caring agency, freed of the parental authority of his/her biological parent(s) or guardian or
adopter(s) in case of rescission of adoption.
(ii) one who seeks to adopt the legitimate son/daughter of his/her Filipino spouse; or
(c) "Voluntarily committed child" is one whose parent(s) knowingly and willingly relinquishes
parental authority to the Department. (iii) one who is married to a Filipino citizen and seeks to adopt jointly with his/her spouse a
relative within the fourth (4th) degree of consanguinity or affinity of the Filipino spouse; or
(d) "Involuntarily committed child" is one whose parent(s), known or unknown, has been
permanently and judicially deprived of parental authority due to abandonment; substantial, (c) The guardian with respect to the ward after the termination of the guardianship and
clearance of his/her financial accountabilities.
continuous, or repeated neglect; abuse; or incompetence to discharge parental responsibilities.

115
RECTO, GAYLE ANGELI M.
2011-0008 | AUSL
Personal Notes on Remedial Law 2 Review (based on the syllabus of Prof. Henedino M. Brondial)

Husband and wife shall jointly adopt, except in the following cases: adopter(s) shall enjoy all the benefits to which biological parent(s) is entitled from the date the
adoptee is placed with the prospective adopter(s).
(i) if one spouse seeks to adopt the legitimate son/daughter of the other; or
Section 13. Decree of Adoption. - If, after the publication of the order of hearing has been
(ii) if one spouse seeks to adopt his/her own illegitimate son/daughter: Provided, However, complied with, and no opposition has been interposed to the petition, and after consideration
that the other spouse has signified his/her consent thereto; or of the case studies, the qualifications of the adopter(s), trial custody report and the evidence
submitted, the court is convinced that the petitioners are qualified to adopt, and that the
(iii) if the spouses are legally separated from each other. adoption would redound to the best interest of the adoptee, a decree of adoption shall be
entered which shall be effective as of the date the original petition was filed. This provision
In case husband and wife jointly adopt, or one spouse adopts the illegitimate son/daughter of shall also apply in case the petitioner(s) dies before the issuance of the decree of adoption to
the other, joint parental authority shall be exercised by the spouses. protect the interest of the adoptee. The decree shall state the name by which the child is to be
known.
Section 8. Who May Be Adopted. - The following may be adopted:
Section 14. Civil Registry Record. - An amended certificate of birth shall be issued by the
(a) Any person below eighteen (18) years of age who has been administratively or judicially Civil Registry, as required by the Rules of Court, attesting to the fact that the adoptee is the
declared available for adoption; child of the adopter(s) by being registered with his/her surname. The original certificate of
birth shall be stamped "cancelled" with the annotation of the issuance of an amended birth
(b) The legitimate son/daughter of one spouse by the other spouse; certificate in its place and shall be sealed in the civil registry records. The new birth certificate
to be issued to the adoptee shall not bear any notation that it is an amended issue.
(c) An illegitimate son/daughter by a qualified adopter to improve his/her status to that of
legitimacy; Section 15. Confidential Nature of Proceedings and Records. - All hearings in adoption
cases shall be confidential and shall not be open to the public. All records, books, and papers
(d) A person of legal age if, prior to the adoption, said person has been consistently relating to the adoption cases in the files of the court, the Department, or any other agency or
considered and treated by the adopter(s) as his/her own child since minority; institution participating in the adoption proceedings shall be kept strictly confidential.

(e) A child whose adoption has been previously rescinded; or If the court finds that the disclosure of the information to a third person is necessary for
purposes connected with or arising out of the adoption and will be for the best interest of the
(f) A child whose biological or adoptive parent(s) has died: Provided, That no proceedings shall adoptee, the court may merit the necessary information to be released, restricting the
be initiated within six (6) months from the time of death of said parent(s). purposes for which it may be used.

Section 9. Whose Consent is Necessary to the Adoption. - After being properly ARTICLE V
counseled and informed of his/her right to give or withhold his/her approval of the adoption, EFFECTS OF ADOPTION
the written consent of the following to the adoption is hereby required:
Section 16. Parental Authority. - Except in cases where the biological parent is the spouse
(a) The adoptee, if ten (10) years of age or over; of the adopter, all legal ties between the biological parent(s) and the adoptee shall be severed
and the same shall then be vested on the adopter(s).
(b) The biological parent(s) of the child, if known, or the legal guardian, or the proper
government instrumentality which has legal custody of the child; Section 17. Legitimacy. - The adoptee shall be considered the legitimate son/daughter of
the adopter(s) for all intents and purposes and as such is entitled to all the rights and
(c) The legitimate and adopted sons/daughters, ten (10) years of age or over, of the obligations provided by law to legitimate sons/daughters born to them without discrimination of
adopter(s) and adoptee, if any; any kind. To this end, the adoptee is entitled to love, guidance, and support in keeping with the
means of the family.
(d) The illegitimate sons/daughters, ten (10) years of age or over, of the adopter if living with
said adopter and the latter's spouse, if any; and Section 18. Succession. - In legal and intestate succession, the adopter(s) and the adoptee
shall have reciprocal rights of succession without distinction from legitimate filiation. However, if
(e) The spouse, if any, of the person adopting or to be adopted. the adoptee and his/her biological parent(s) had left a will, the law on testamentary
succession shall govern.
ARTICLE IV
PROCEDURE ARTICLE VI
RESCISSION OF ADOPTION
Section 10. Hurried Decisions. - In all proceedings for adoption, the court shall require
proof that the biological parent(s) has been properly counseled to prevent him/her from Section 19. Grounds for Rescission of Adoption. - Upon petition of the adoptee, with the
making hurried decisions caused by strain or anxiety to give up the child, and to sustain that all assistance of the Department if a minor or if over eighteen (18) years of age but is
measures to strengthen the family have been exhausted and that any prolonged stay of the incapacitated, as guardian/counsel, the adoption may be rescinded on any of the following
child in his/her own home will be inimical to his/her welfare and interest. grounds committed by the adopter(s): (a) repeated physical and verbal maltreatment by the
adopter(s) despite having undergone counseling; (b) attempt on the life of the adoptee; (c)
Section 11. Case Study. - No petition for adoption shall be set for hearing unless a licensed sexual assault or violence; or (d) abandonment and failure to comply with parental obligations.
social worker of the Department, the social service office of the local government unit, or any
child-placing or child-caring agency has made a case study of the adoptee, his/her biological Adoption, being in the best interest of the child, shall not be subject to rescission by the
parent(s), as well as the adopter(s), and has submitted the report and recommendations on the adopter(s). However, the adopter(s) may disinherit the adoptee for causes provided in Article
matter to the court hearing such petition. 919 of the Civil Code.

At the time of preparation of the adoptee's case study, the concerned social worker shall Section 20. Effects of Rescission. - If the petition is granted, the parental authority of the
confirm with the Civil Registry the real identity and registered name of the adoptee. If the adoptee's biological parent(s), if known, or the legal custody of the Department shall be
birth of the adoptee was not registered with the Civil Registry, it shall be the responsibility of restored if the adoptee is still a minor or incapacitated. The reciprocal rights and obligations of
the concerned social worker to ensure that the adoptee is registered. the adopter(s) and the adoptee to each other shall be extinguished.

The case study on the adoptee shall establish that he/she is legally available for adoption and The court shall order the Civil Registrar to cancel the amended certificate of birth of the
that the documents to support this fact are valid and authentic. Further, the case study of the adoptee and restore his/her original birth certificate.
adopter(s) shall ascertain his/her genuine intentions and that the adoption is in the best
interest of the child. Succession rights shall revert to its status prior to adoption, but only as of the date of
judgment of judicial rescission. Vested rights acquired prior to judicial rescission shall be
The Department shall intervene on behalf of the adoptee if it finds, after the conduct of the respected.
case studies, that the petition should be denied. The case studies and other relevant
documents and records pertaining to the adoptee and the adoption shall be preserved by the All the foregoing effects of rescission of adoption shall be without prejudice to the penalties
Department. imposable under the Penal Code if the criminal acts are properly proven.

Section 12. Supervised Trial Custody. - No petition for adoption shall be finally granted ARTICLE VII
until the adopter(s) has been given by the court a supervised trial custody period for at least VIOLATIONS AND PENALTIES
six (6) months within which the parties are expected to adjust psychologically and emotionally
to each other and establish a bonding relationship. During said period, temporary parental Section 21. Violations and Penalties. - (a) The penalty of imprisonment ranging from six
authority shall be vested in the adopter(s). (6) years and one (1) day to twelve (12) years and/or a fine not less than Fifty thousand
pesos (P50,000.00), but not more than Two hundred thousand pesos (P200,000.00) at the
The court may motu proprio or upon motion of any party reduce the trial period if it finds the discretion of the court shall be imposed on any person who shall commit any of the following
same to be in the best interest of the adoptee, stating the reasons for the reduction of the acts:
period. However, for alien adopter(s), he/she must complete the six (6)-month trial custody
except for those enumerated in Sec. 7 (b) (i) (ii) (iii). (i) obtaining consent for an adoption through coercion, undue influence, fraud, improper
material inducement, or other similar acts;
If the child is below seven (7) years of age and is placed with the prospective adopter(s)
through a pre-adoption placement authority issued by the Department, the prospective (ii) non-compliance with the procedures and safeguards provided by the law for adoption; or
116
RECTO, GAYLE ANGELI M.
2011-0008 | AUSL
Personal Notes on Remedial Law 2 Review (based on the syllabus of Prof. Henedino M. Brondial)

AN ACT ESTABLISHING THE RULES TO GOVERN INTER-COUNTRY ADOPTION OF


(iii) subjecting or exposing the child to be adopted to danger, abuse, or exploitation. FILIPINO CHILDREN, AND FOR OTHER PURPOSES.

(b) Any person who shall cause the fictitious registration of the birth of a child under the ARTICLE I
name(s) of a person(s) who is not his/her biological parent(s) shall be guilty of simulation of GENERAL PROVISIONS
birth, and shall be punished by prision mayor in its medium period and a fine not exceeding
Fifty thousand pesos (P50,000.00). Sec. 1. Short Title. - This Act shall be known as the "Inter-Country Adoption Act of 1995."

Any physician or nurse or hospital personnel who, in violation of his/her oath of office, shall Sec. 2. Declaration of Policy. - It is hereby declared the policy of the State to provide
cooperate in the execution of the abovementioned crime shall suffer the penalties herein every neglected and abandoned child with a family that will provide such child with love and
prescribed and also the penalty of permanent disqualification. care as well as opportunities for growth and development. Towards this end, efforts shall be
exerted to place the child with an adoptive family in the Philippines. However, recognizing
Any person who shall violate established regulations relating to the confidentiality and integrity that inter-country adoption may be considered as allowing aliens not presently allowed by law
of records, documents, and communications of adoption applications, cases, and processes to adopt Filipino children if such children cannot be adopted by qualified Filipino citizens or
shall suffer the penalty of imprisonment ranging from one (1) year and one (1) day to two (2) aliens, the State shall take measures to ensure that inter-country adoptions are allowed when
years, and/or a fine of not less than Five thousand pesos (P5,000.00) but not more than Ten the same shall prove beneficial to the child's best interests, and shall serve and protect his/her
thousand pesos (P10,000.00), at the discretion of the court. fundamental rights. cdt

A penalty lower by two (2) degrees than that prescribed for the consummated offense under Sec. 3. Definition of Terms. - As used in this Act. the term:
this Article shall be imposed upon the principals of the attempt to commit any of the acts
herein enumerated. Acts punishable under this Article, when committed by a syndicate or a) Inter-country adoption refers to the socio-legal process of adopting a Filipino child by
where it involves two (2) or more children shall be considered as an offense constituting child a foreigner or a Filipino citizen permanently residing abroad where the petition is filed, the
trafficking and shall merit the penalty of reclusion perpetua. supervised trial custody is undertaken, and the decree of adoption is issued outside the
Philippines.
Acts punishable under this Article are deemed committed by a syndicate if carried out by a
group of three (3) or more persons conspiring and/or confederating with one another in b) Child means a person below fifteen (15) years of age unless sooner emancipated by
carrying out any of the unlawful acts defined under this Article. Penalties as are herein law.
provided, shall be in addition to any other penalties which may be imposed for the same acts
punishable under other laws, ordinances, executive orders, and proclamations. c) Department refers to the Department of Social Welfare and Development of the
Republic of the Philippines.
When the offender is an alien, he/she shall be deported immediately after service of sentence
and perpetually excluded from entry to the country. d) Secretary refers to the Secretary of the Department of Social Welfare and
Development. acd
Any government official, employee or functionary who shall be found guilty of violating any of
the provisions of this Act, or who shall conspire with private individuals shall, in addition to the e) Authorized and accredited agency refers to the State welfare agency or a licensed
above-prescribed penalties, be penalized in accordance with existing civil service laws, rules and adoption agency in the country of the adopting parents which provide comprehensive social
regulations: Provided, That upon the filing of a case, either administrative or criminal, said services and which is duly recognized by the Department.
government official, employee, or functionary concerned shall automatically suffer suspension
until the resolution of the case. f) Legally-free child means a child who has been voluntarily or involuntarily committed
to the Department, in accordance with the Child and Youth Welfare Code.
Section 22. Rectification of Simulated Births. - A person who has, prior to the effectivity
of this Act, simulated the birth of a child shall not be punished for such act: Provided, That the g) Matching refers to the judicious pairing of the adoptive child and the applicant to
simulation of birth was made for the best interest of the child and that he/she has been promote a mutually satisfying parent-child relationship.
consistently considered and treated by that person as his/her own son/daughter: Provided,
further, That the application for correction of the birth registration and petition for adoption h) Board refers to the Inter-country Adoption Board.
shall be filed within five (5) years from the effectivity of this Act and completed thereafter:
Provided, finally, That such person complies with the procedure as specified in Article IV of this ARTICLE II
Act and other requirements as determined by the Department. THE INTER-COUNTRY ADOPTION BOARD

ARTICLE VIII Sec. 4. The Inter-Country Adoption Board. - There is hereby created the Inter-Country
FINAL PROVISIONS Adoption Board, hereinafter referred to as the Board to act as the central authority in matters
relating to inter-country adoption. It shall act as the policy-making body for purposes of
Section 23. Adoption Resource and Referral Office. - There shall be established an carrying out the provisions of this Act, in consultation and coordination with the Department,
Adoption Resources and Referral Office under the Department with the following functions: (a) the different child-care and placement agencies, adoptive agencies, as well as non-
monitor the existence, number, and flow of children legally available for adoption and governmental organizations engaged in child-care and placement activities. As such, it shall:
prospective adopter(s) so as to facilitate their matching; (b) maintain a nationwide information
and educational campaign on domestic adoption; (c) keep records of adoption proceedings; a) Protect the Filipino child from abuse, exploitation, trafficking and/or sale or any other
(d) generate resources to help child-caring and child-placing agencies and foster homes practice in connection with adoption which is harmful, detrimental, or prejudicial to the child;
maintain viability; and (e) do policy research in collaboration with the Intercountry Adoption
Board and other concerned agencies. The office shall be manned by adoption experts from the b) Collect, maintain, and preserve confidential information about the child and the
public and private sectors. adoptive parents;

Section 24. Implementing Rules and Regulations. - Within six (6) months from the c) Monitor, follow up, and facilitate completion of adoption of the child through
promulgation of this Act, the Department, with the Council for the Welfare of Children, the authorized and accredited agency;
Office of Civil Registry General, the Department of Justice, Office of the Solicitor General, and
two (2) private individuals representing child-placing and child-caring agencies shall formulate d) Prevent improper financial or other gain in connection with an adoption and deter
the necessary guidelines to make the provisions of this Act operative. improper practices contrary to this Act;

Section 25. Appropriations. - Such sum as may be necessary for the implementation of the e) Promote the development of adoption services including post-legal adoption; cd i
provisions of this Act shall be included in the General Appropriations Act of the year following its
enactment into law and thereafter. f) License and accredit child-caring/placement agencies and collaborate with them in
the placement of Filipino children;
Section 26. Repealing Clause. - Any law, presidential decree or issuance, executive order,
letter of instruction, administrative order, rule, or regulation contrary to, or inconsistent with g) Accredit and authorize foreign adoption agency in the placement of Filipino children
the provisions of this Act is hereby repealed, modified, or amended accordingly. in their own country; and

Section 27. Separability Clause. - If any provision of this Act is held invalid or h) Cancel the license to operate and blacklist the child-caring and placement agency or
unconstitutional, the other provisions not affected thereby shall remain valid and subsisting. adoptive agency involved from the accreditation list of the Board upon a finding of violation of
any provision under this Act.
Section 28. Effectivity Clause. - This Act shall take effect fifteen (15) days following its
complete publication in any newspaper of general circulation or in the Official Gazette. Sec. 5. Composition of the Board. - The Board shall be composed of the Secretary of the
Department as ex officio Chairman, and six (6) other members to be appointed by the
Approved: February 25, 1998 President for a nonrenewable term of six (6) years: Provided, That there shall be appointed
one (1) psychiatrist or psychologist, two (2) lawyers who shall have at least the qualifications
2. Inter-Country Adoption Act of 1995 of a regional trial court judge, one (1) registered social worker and two (2) representatives
from non-governmental organizations engaged in child-caring and placement activities. The
REPUBLIC ACT NO. 8043 members of the Board shall receive a per diem allowance of One thousand five hundred pesos
(P1,500) for each meeting attended by them: Provided, further, That no compensation shall be
paid for more than four (4) meetings a month.

117
RECTO, GAYLE ANGELI M.
2011-0008 | AUSL
Personal Notes on Remedial Law 2 Review (based on the syllabus of Prof. Henedino M. Brondial)

Sec. 6. Powers and Functions of the Board. - The Board shall have the following
powers and functions: f) is in a position to provide the proper care and support and to give the necessary
moral values and example to all his children, including the child to be adopted;
a) to prescribe rules and regulations as it may deem reasonably necessary to carry out
the provisions of this Act, after consultation and upon favorable recommendation of the g) agrees to uphold the basic rights of the child as embodied under Philippine laws, the
different agencies concerned with the child-caring, placement, and adoption; U.N. Convention on the Rights of the Child, and to abide by the rules and regulations issued to
implement the provisions of this Act;
b) to set the guidelines for the convening of an Inter-country Adoption Placement
Committee which shall be under the direct supervision of the Board; h) comes from a country with whom the Philippines has diplomatic relations and whose
government maintains a similarly authorized and accredited agency and that adoption is
c) to set the guidelines for the manner by which selection/matching of prospective allowed under his/her national laws; and
adoptive parents and adoptive child can be made;
i) possesses all the qualifications and none of the disqualifications provided herein and
d) to determine a reasonable schedule of fees and charges to be exacted in connection in other applicable Philippine laws.
with the application for adoption;
Sec. 10. Where to File Application. - An application to adopt a Filipino child shall be filed
e) to determine the form and contents of the application for inter-country adoption; either with the Philippine Regional Trial Court having jurisdiction over the child, or with the
Board, through an intermediate agency, whether governmental or an authorized and
g) to institute systems and procedures to prevent improper financial gain in connection accredited agency, in the country of the prospective adoptive parents, which application shall
with adoption and deter improper practices which are contrary to this Act; be in accordance with the requirements as set forth in the implementing rules and regulations
to be promulgated by the Board. cdasia
h) to promote the development of adoption services, including post-legal adoption
services, cd The application shall be supported by the following documents written and officially translated
in English.
i) to accredit and authorize foreign private adoption agencies which have demonstrated
professionalism, competence and have consistently pursued non-profit objectives to engage in a) Birth certificate of applicant(s);
the placement of Filipino children in their own country: Provided, That such foreign private
agencies are duly authorized and accredited by their own government to conduct inter-country b) Marriage contract, if married, and divorce decree, if applicable;
adoption: Provided, however, That the total number of authorized and accredited foreign
private adoption agencies shall not exceed one hundred (100) a year; c) Written consent of their biological or adoptive children above ten (10) years of age,
in the form of sworn statement;
j) to take appropriate measures to ensure confidentiality of the records of the child, the
natural parents and the adoptive parents at all times; d) Physical, medical and psychological evaluation by a duly licensed physician and
psychologist;
k) to prepare, review or modify, and thereafter, recommend to the Department of
Foreign Affairs, Memoranda of Agreement respecting inter-country adoption consistent with e) Income tax returns or any document showing the financial capability of the
the implementation of this Act and its stated goals, entered into, between and among foreign applicant(s);
governments, international organizations and recognized international non-governmental
organizations; f) Police clearance of applicant(s);

l) to assist other concerned agencies and the courts in the implementation of this Act, g) Character reference from the local church/minister, the applicant's employer and a
particularly as regards coordination with foreign persons, agencies and other entities involved member of the immediate community who have known the applicant(s) for at least five (5)
in the process of adoption and the physical transfer of the child; and years; and

m) to perform such other functions on matters relating to inter-country adoption as may h) Recent postcard-size pictures of the applicant(s) and his immediate family;
be determined by the President.
The Rules of Court shall apply in case of adoption by judicial proceedings.
ARTICLE III
PROCEDURE Sec. 11. Family Selection/Matching. - No child shall be matched to a foreign adoptive
family unless it is satisfactorily shown that the child cannot be adopted locally. The clearance,
Sec. 7. Inter-Country Adoption as the Last Resort. - The Board shall ensure that all as issued by the Board, with the copy of the minutes of the meetings, shall form part of the
possibilities for adoption of the child under the Family Code have been exhausted and that records of the child to be adopted. When the Board is ready to transmit the Placement
inter-country adoption is in the best interest of the child. Towards this end, the Board shall Authority to the authorized and accredited inter-country adoption agency and all the travel
set up the guidelines to ensure that steps will be taken to place the child in the Philippines documents of the child are ready, the adoptive parents, or any one of them, shall personally
before the child is placed for inter-country adoption: Provided, however, That the maximum fetch the child in the Philippines.
number that may be allowed for foreign adoption shall not exceed six hundred (600) a year for
the first five (5) years. Sec. 12. Pre-adoptive Placement Costs. - The applicant(s) shall bear the following costs
incidental to the placement of the child;
Sec. 8. Who May be Adopted. - Only a legally free child may be the subject of inter-
country adoption. In order that such child may be considered for placement, the following a) The cost of bringing the child from the Philippines to the residence of the applicant(s)
documents must be submitted to the Board: cdasia abroad, including all travel expenses within the Philippines and abroad; and

a) Child study; b) The cost of passport, visa, medical examination and psychological evaluation
required, and other related expenses.
b) Birth certificate/foundling certificate;
Sec. 13. Fees, Charges and Assessments. - Fees, charges, and assessments collected by
c) Deed of voluntary commitment/decree of abandonment/death certificate of parents; the Board in the exercise of its functions shall be used solely to process applications for inter-
country adoption and to support the activities of the Board.
d) Medical evaluation /history;
Sec. 14. Supervision of Trial Custody. - The governmental agency or the authorized and
e) Psychological evaluation, as necessary; and accredited agency in the country of the adoptive parents which filed the application for inter-
country adoption shall be responsible for the trial custody and the care of the child. It shall
f) Recent photo of the child. also provide family counseling and other related services. The trial custody shall be for a
period of six (6) months from the time of placement. Only after the lapse of the period of trial
Sec. 9. Who May Adopt. - An alien or a Filipino citizen permanently residing abroad may custody shall a decree of adoption be issued in the said country a copy of which shall be sent to
file an application for inter-country adoption of a Filipino child if he/she: the Board to form part of the records of the child.

a) is at least twenty-seven (27) years of age and at least sixteen (16) years older than During the trial custody, the adopting parent(s) shall submit to the governmental agency or
the child to be adopted, at the time of application unless the adoptor is the parent by nature the authorized and accredited agency, which shall in turn transmit a copy to the Board, a
of the child to be adopted or the spouse of such parent: progress report of the child's adjustment. The progress report shall be taken into consideration
in deciding whether or not to issue the decree of adoption. aisa dc
b) if married, his/her spouse must jointly file for the adoption;
The Department of Foreign Affairs shall set up a system by which Filipino children sent abroad
c) has the capacity to act and assume all rights and responsibilities of parental authority for trial custody are monitored and checked as reported by the authorized and accredited
under his national laws, and has undergone the appropriate counseling from an accredited inter-country adoption agency as well as the repatriation to the Philippines of a Filipino child
counselor in his/her country; whose adoption has not been approved.

d) has not been convicted of a crime involving moral turpitude; Sec. 15. Executive Agreements. - The Department of Foreign Affairs, upon representation
of the Board, shall cause the preparation of Executive Agreements with countries of the
e) is eligible to adopt under his/her national law;
118
RECTO, GAYLE ANGELI M.
2011-0008 | AUSL
Personal Notes on Remedial Law 2 Review (based on the syllabus of Prof. Henedino M. Brondial)

foreign adoption agencies to ensure the legitimate concurrence of said countries in upholding A. Domestic Adoption
the safeguards provided by this Act.
Section 1. Applicability of the Rule. - This Rule covers the domestic adoption of Filipino
ARTICLE IV children.
PENALTIES
Sec. 2. Objectives. - (a) The best interests of the child shall be the paramount consideration
Sec. 16. Penalties. - a) Any person who shall knowingly participate in the conduct or in all matters relating to his care, custody and adoption, in accordance with Philippine laws,
carrying out of an illegal adoption, in violation of the provisions of this Act, shall be punished the United Nations (UN) Convention on the Rights of the Child, UN Declaration on Social and
with a penalty of imprisonment ranging from six (6) years and one (1) day to twelve (12) Legal Principles Relating to the Protection and Welfare of Children with Special Reference to
years and/or a fine of not less than Fifty thousand pesos (P50,000), but not more than Two Foster Placement and Adoption, Nationally and Internationally, and the Hague Convention on
hundred thousand pesos (P200.000), at the discretion of the court. For purposes of this Act, the Protection of Children and Cooperation in Respect of Inter-country Adoption.
an adoption is illegal if it is effected in any manner contrary to the provisions of this Act or
established State policies, its implementing rules and regulations, executive agreements, and (b) The State shall provide alternative protection and assistance through foster care or
other laws pertaining to adoption. Illegality may be presumed from the following acts: adoption for every child who is a foundling, neglected, orphaned, or abandoned. To this end,
the State shall:
1) consent for an adoption was acquired through, or attended by coercion, fraud,
improper material inducement; (i) ensure that every child remains under the care and custody of his parents and is provided
with love, care, understanding and security for the full and harmonious development of his
2) there is no authority from the Board to effect adoption; personality. Only when such efforts prove insufficient and no appropriate placement or
adoption within the child’s extended family is available shall adoption by an unrelated person
3) the procedures and safeguards placed under the law for adoption were not complied be considered.
with; and
(ii) safeguard the biological parents from making hasty decisions in relinquishing their parental
4) the child to be adopted is subjected to, or exposed to danger, abuse and exploitation. authority over their child;

b) Any person who shall violate established regulations relating to the confidentiality and (iii) prevent the child from unnecessary separation from his biological parents;
integrity of records, documents and communications of adoption applications, cases and
processes shall suffer the penalty of imprisonment ranging from one (1) year and one (1) day (iv) conduct public information and educational campaigns to promote a positive environment
to two (2) years, and/or a fine of not less than Five thousand pesos (P5,000), but not more for adoption;
than Ten thousand pesos (P10,000), at the discretion of the court.
(v) ensure that government and private sector agencies have the capacity to handle adoption
A penalty lower by two (2) degrees than that prescribed for the consummated felony under inquiries, process domestic adoption applications and offer adoption-related services including,
this Article shall be imposed upon the principals of the attempt to commit any of the acts but not limited to, parent preparation and post-adoption education and counseling;
herein enumerated. cdasia
(vi) encourage domestic adoption so as to preserve the child’s identity and culture in his native
Acts punishable under this Article, when committed by a syndicate or where it involves two or land, and only when this is not available shall inter-country adoption be considered as a last
more children shall be considered as an offense constituting child trafficking and shall merit the resort; and
penalty of reclusion perpetua.
(vii) protect adoptive parents from attempts to disturb their parental authority and custody
Acts punishable under this Article are deemed committed by a syndicate if carried out by a over their adopted child.
group of three (3) or more persons conspiring and/or confederating with one another in Any voluntary or involuntary termination of parental authority shall be administratively or
carrying out any of the unlawful acts defined under this Article. Penalties as are herein judicially declared so as to establish the status of the child as “legally available for adoption”
provided shall be in addition to any other penalties which may be imposed for the same acts and his custody transferred to the Department of Social Welfare and Development or to any
punishable under other laws, ordinances, executive orders, and proclamations. duly licensed and accredited child-placing or child-caring agency, which entity shall be
authorized to take steps for the permanent placement of the child.
Sec. 17. Public Officers as Offenders. - Any government official, employee or functionary
who shall be found guilty of violating any of the provisions of this Act, or who shall conspire Sec. 3. Definition of Terms. - For purposes of this Rule:
with private individuals shall, in addition to the above-prescribed penalties, be penalized in
accordance with existing civil service laws, rules and regulations: Provided, That upon the (a) “Child” is a person below eighteen (18) years of age at the time of the filing of the petition
filing of a case, either administrative or criminal, said government official, employee or for adoption.
functionary concerned shall automatically suffer suspension until the resolution of the case.
(b) “A child legally available for adoption” refers to a child who has been voluntarily or
ARTICLE V involuntarily committed to the Department or to a duly licensed and accredited child-placing or
FINAL PROVISIONS child-caring agency, freed of the parental authority of his biological parents, or in case of
rescission of adoption, his guardian or adopter(s).
Sec. 18. Implementing Rules and Regulations. - The Inter-country Adoption Board, in
coordination with the Council for the Welfare of Children, the Department of Foreign Affairs, (c) “Voluntarily committed child” is one whose parents knowingly and willingly relinquish
and the Department of Justice, after due consultation with agencies involved in child-care and parental authority over him in favor of the Department.
placement, shall promulgate the necessary rules and regulations to implement the provisions of
this Act within six (6) months after its effectivity. (d) “Involuntarily committed child” is one whose parents, known or unknown, have been
permanently and judicially deprived of parental authority over him due to abandonment;
Sec. 19. Appropriations. - The amount of Five million pesos (P5,000,000) is hereby substantial, continuous or repeated neglect and abuse; or incompetence to discharge parental
appropriated from the proceeds of the Lotto for the initial operations of the Board and responsibilities.
subsequently the appropriations of the same shall be included in the General Appropriations
Act for the year following its enactment. (e) “Foundling” refers to a deserted or abandoned infant or child whose parents, guardian or
relatives are unknown; or a child committed to an orphanage or charitable or similar institution
Sec. 20. Separability Clause. - If any provision, or part hereof is held invalid or with unknown facts of birth and parentage and registered in the Civil Register as a “foundling.”
unconstitutional, the remainder of the law or the provision not otherwise affected, shall remain
valid and subsisting. cd i5 (f) “Abandoned child” refers to one who has no proper parental care or guardianship or whose
parents have deserted him for a period of at least six (6) continuous months and has been
Sec. 21. Repealing Clause. - Any law, decree, executive order, administrative order or judicially declared as such.
rules and regulations contrary to, or inconsistent with the provisions of this Act are hereby
repealed, modified or amended accordingly. (g) “Dependent child” refers to one who is without a parent, guardian or custodian or one
whose parents, guardian or other custodian for good cause desires to be relieved of his care
Sec. 22. Effectivity Clause. - This Act shall take effect fifteen (15) days after its publication and custody and is dependent upon the public for support.
in two (2) newspapers of general circulation.
(h) “Neglected child” is one whose basic needs have been deliberately not attended to or
Approved, June 7, 1995. inadequately attended to, physically or emotionally, by his parents or guardian.

3. Rules Of Adoption [AM No. 02-6-02, dated July 31, 2002 effective (i) “Physical neglect” occurs when the child is malnourished, ill-clad and without proper
shelter.
August 22, 2002]
(j) “Emotional neglect” exists when a child is raped, seduced, maltreated, exploited,
a. Who may adopt? [Section 4] overworked or made to work under conditions not conducive to good health or made to beg in
the streets or public places, or placed in moral danger, or exposed to drugs, alcohol, gambling,
b. Who may be adopted? [Section 5] prostitution and other vices.

(k) “Child-placement agency” refers to an agency duly licensed and accredited by the
c. Venue and Jurisdiction [Sections 6 & 20]
Department to provide comprehensive child welfare services including, but not limited to,

119
RECTO, GAYLE ANGELI M.
2011-0008 | AUSL
Personal Notes on Remedial Law 2 Review (based on the syllabus of Prof. Henedino M. Brondial)

receiving applications for adoption, evaluating the prospective adoptive parents and preparing (ii) if one spouse seeks to adopt his own illegitimate child: Provided, however, That the other
the adoption home study report. spouse has signified his consent thereto; or

(l) “Child-caring agency” refers to an agency duly licensed and accredited by the Department (iii) if the spouses are legally separated from each other.
that provides 24-hour residential care services for abandoned, orphaned, neglected or In case husband and wife jointly adopt or one spouse adopts the illegitimate child of the other,
voluntarily committed children. joint parental authority shall be exercised by the spouses.

(m) “Department” refers to the Department of Social Welfare and Development. SEC. 5. Who may be adopted. - The following may be adopted:

(n) "Deed of Voluntary Commitment” refers to the written and notarized instrument (1) Any person below eighteen (18) years of age who has been voluntarily committed to the
relinquishing parental authority and committing the child to the care and custody of the Department under Articles 154, 155 and 156 of P.D. No. 603 or judicially declared available for
Department executed by the child’s biological parents or in their absence, mental incapacity or adoption;
death, by the child’s legal guardian, to be witnessed by an authorized representative of the
Department after counseling and other services have been made available to encourage the (2) The legitimate child of one spouse, by the other spouse;
biological parents to keep the child.
(3) An illegitimate child, by a qualified adopter to raise the status of the former to that of
(o) “Child Study Report” refers to a study made by the court social worker of the child’s legal legitimacy;
status, placement history, psychological, social, spiritual, medical, ethno-cultural background
and that of his biological family needed in determining the most appropriate placement for (4) A person of legal age regardless of civil status, if, prior to the adoption, said person has
him. been consistently considered and treated by the adopters as their own child since minority;

(p) “Home Study Report” refers to a study made by the court social worker of the motivation (5) A child whose adoption has been previously rescinded; or
and capacity of the prospective adoptive parents to provide a home that meets the needs of a
child. (6) A child whose biological or adoptive parents have died: Provided, That no proceedings
shall be initiated within six (6) months from the time of death of said parents.
(q) “Supervised trial custody” refers to the period of time during which a social worker
oversees the adjustment and emotional readiness of both adopters and adoptee in stabilizing (7) A child not otherwise disqualified by law or these rules.
their filial relationship.
Sec. 6. Venue. - The petition for adoption shall be filed with the Family Court of the province
(r) “Licensed Social Worker” refers to one who possesses a degree in bachelor of science in or city where the prospective adoptive parents reside.
social work as a minimum educational requirement and who has passed the government
licensure examination for social workers as required by Republic Act No. 4373. Sec. 7. Contents of the Petition. - The petition shall be verified and specifically state at the
heading of the initiatory pleading whether the petition contains an application for change of
(s) “Simulation of birth” is the tampering of the civil registry to make it appear in the birth name, rectification of simulated birth, voluntary or involuntary commitment of children, or
records that a certain child was born to a person who is not his biological mother, thus causing declaration of child as abandoned, dependent or neglected.
such child to lose his true identity and status.
1) If the adopter is a Filipino citizen, the petition shall allege the following:
(t) “Biological Parents” refer to the child’s mother and father by nature.
(a) The jurisdictional facts;
(u) “Pre-Adoption Services” refer to psycho-social services provided by professionally-trained
social workers of the Department, the social services units of local governments, private and (b) That the petitioner is of legal age, in possession of full civil capacity and legal rights; is of
government health facilities, Family Courts, licensed and accredited child-caring and child- good moral character; has not been convicted of any crime involving moral turpitude; is
placement agencies and other individuals or entities involved in adoption as authorized by the emotionally and psychologically capable of caring for children; is at least sixteen (16) years
Department. older than the adoptee, unless the adopter is the biological parent of the adoptee or is the
spouse of the adoptee’s parent; and is in a position to support and care for his children in
(v) “Residence” means a person’s actual stay in the Philippines for three (3) continuous years keeping with the means of the family and has undergone pre-adoption services as required by
immediately prior to the filing of a petition for adoption and which is maintained until the Section 4 of Republic Act No. 8552.
adoption decree is entered. Temporary absences for professional, business, health, or
emergency reasons not exceeding sixty (60) days in one (1) year does not break the 2) If the adopter is an alien, the petition shall allege the following:
continuity requirement. (a) The jurisdictional facts;

(w) “Alien” refers to any person, not a Filipino citizen, who enters and remains in the (b) Sub-paragraph 1(b) above;
Philippines and is in possession of a valid passport or travel documents and visa.
(c) (c) That his country has diplomatic relations with the Republic of the Philippines;
SEC. 4. Who may adopt. - The following may adopt:
(d) That he has been certified by his diplomatic or consular office or any appropriate
(1) Any Filipino citizen of legal age, in possession of full civil capacity and legal rights, of good government agency to have the legal capacity to adopt in his country and his government
moral character, has not been convicted of any crime involving moral turpitude; who is allows the adoptee to enter his country as his adopted child and reside there permanently as
emotionally and psychologically capable of caring for children, at least sixteen (16) years older an adopted child; and
than the adoptee, and who is in a position to support and care for his children in keeping with
the means of the family. The requirement of a 16-year difference between the age of the (e) That he has been living in the Philippines for at least three (3) continuous years prior to
adopter and adoptee may be waived when the adopter is the biological parent of the adoptee the filing of the petition and he maintains such residence until the adoption decree is entered.
or is the spouse of the adoptee’s parent; The requirements of certification of the alien’s qualification to adopt in his country and of
residency may be waived if the alien:
(2) Any alien possessing the same qualifications as above-stated for Filipino nationals:
Provided, That his country has diplomatic relations with the Republic of the Philippines, that he (i) is a former Filipino citizen who seeks to adopt a relative within the fourth degree of
has been living in the Philippines for at least three (3) continuous years prior to the filing of consanguinity or affinity; or
the petition for adoption and maintains such residence until the adoption decree is entered,
that he has been certified by his diplomatic or consular office or any appropriate government (ii) seeks to adopt the legitimate child of his Filipino spouse; or
agency to have the legal capacity to adopt in his country, and that his government allows the
adoptee to enter his country as his adopted child. Provided, further, That the requirements on (iii) is married to a Filipino citizen and seeks to adopt jointly with his spouse a relative within
residency and certification of the alien’s qualification to adopt in his country may be waived for the fourth degree of consanguinity or affinity of the Filipino spouse.
the following:
3) If the adopter is the legal guardian of the adoptee, the petition shall allege that
(i) a former Filipino citizen who seeks to adopt a relative within the fourth (4th) degree of guardianship had been terminated and the guardian had cleared his financial accountabilities.
consanguinity or affinity; or
4) If the adopter is married, the spouse shall be a co-petitioner for joint adoption except if:
(ii) one who seeks to adopt the legitimate child of his Filipino spouse; or
(a) one spouse seeks to adopt the legitimate child of the other, or
(iii) one who is married to a Filipino citizen and seeks to adopt jointly with his spouse a relative
within the fourth (4th) degree of consanguinity or affinity of the Filipino spouse. (b) if one spouse seeks to adopt his own illegitimate child and the other spouse signified
written consent thereto, or
(3) The guardian with respect to the ward after the termination of the guardianship and
clearance of his financial accountabilities. (c) if the spouses are legally separated from each other.
5) If the adoptee is a foundling, the petition shall allege the entries which should appear in his
Husband and wife shall jointly adopt, except in the following cases: birth certificate, such as name of child, date of birth, place of birth, if known; sex, name and
citizenship of adoptive mother and father, and the date and place of their marriage.
(i) if one spouse seeks to adopt the legitimate child of one spouse by the other spouse; or
6) If the petition prays for a change of name, it shall also state the cause or reason for the
change of name.
120
RECTO, GAYLE ANGELI M.
2011-0008 | AUSL
Personal Notes on Remedial Law 2 Review (based on the syllabus of Prof. Henedino M. Brondial)

In all petitions, it shall be alleged: (1) the registered name of the adoptee in the birth certificate and the names by which the
adoptee has been known which shall be stated in the caption;
(a) The first name, surname or names, age and residence of the adoptee as shown by his
record of birth, baptismal or foundling certificate and school records. (2) the purpose of the petition;

(b) That the adoptee is not disqualified by law to be adopted. (3) the complete name which the adoptee will use if the petition is granted;

(c) The probable value and character of the estate of the adoptee. (4) the date and place of hearing which shall be set within six (6) months from the date of the
issuance of the order and shall direct that a copy thereof be published before the date of
(d) The first name, surname or names by which the adoptee is to be known and registered in hearing at least once a week for three successive weeks in a newspaper of general circulation
the Civil Registry. in the province or city where the court is situated; Provided, that in case of application for
change of name, the date set for hearing shall not be within four (4) months after the last
A certification of non-forum shopping shall be included pursuant to Section 5, Rule 7 of the publication of the notice nor within thirty (30) days prior to an election.
1997 Rules of Civil Procedure.
The newspaper shall be selected by raffle under the supervision of the Executive Judge.
Sec. 8. Rectification of Simulated Birth. - In case the petition also seeks rectification of a
simulated of birth, it shall allege that: (5) a directive to the social worker of the court, the social service office of the local government
unit or any child-placing or child-caring agency, or the Department to prepare and submit child
(a) Petitioner is applying for rectification of a simulated birth; and home study reports before the hearing if such reports had not been attached to the petition
due to unavailability at the time of the filing of the latter; and
(b) The simulation of birth was made prior to the date of effectivity of Republic Act No. 8552
and the application for rectification of the birth registration and the petition for adoption were (6) a directive to the social worker of the court to conduct counseling sessions with the
filed within five years from said date; biological parents on the matter of adoption of the adoptee and submit her report before the
date of hearing.
(c) The petitioner made the simulation of birth for the best interests of the adoptee; and At the discretion of the court, copies of the order of hearing shall also be furnished the Office
of the Solicitor General through the provincial or city prosecutor, the Department and the
(d) The adoptee has been consistently considered and treated by petitioner as his own child. biological parents of the adoptee, if known.

Sec. 9. Adoption of a foundling, an abandoned, dependent or neglected child. - In If a change in the name of the adoptee is prayed for in the petition, notice to the Solicitor
case the adoptee is a foundling, an abandoned, dependent or neglected child, the petition General shall be mandatory.
shall allege:
Sec. 13. Child and Home Study Reports. - In preparing the child study report on the
(a) The facts showing that the child is a foundling, abandoned, dependent or neglected; adoptee, the concerned social worker shall verify with the Civil Registry the real identity and
registered name of the adoptee. If the birth of the adoptee was not registered with the Civil
(b) The names of the parents, if known, and their residence. If the child has no known or Registry, it shall be the responsibility of the social worker to register the adoptee and secure a
living parents, then the name and residence of the guardian, if any; certificate of foundling or late registration, as the case may be.

(c) The name of the duly licensed child-placement agency or individual under whose care the The social worker shall establish that the child is legally available for adoption and the
child is in custody; and documents in support thereof are valid and authentic, that the adopter has sincere intentions
and that the adoption shall inure to the best interests of the child.
(d) That the Department, child-placement or child-caring agency is authorized to give its
consent. In case the adopter is an alien, the home study report must show the legal capacity to adopt
and that his government allows the adoptee to enter his country as his adopted child in the
Sec. 10. Change of name. - In case the petition also prays for change of name, the title or absence of the certification required under Section 7(b) of Republic Act No. 8552.
caption must contain:
If after the conduct of the case studies, the social worker finds that there are grounds to deny
(a) The registered name of the child; the petition, he shall make the proper recommendation to the court, furnishing a copy thereof
to the petitioner.
(b) Aliases or other names by which the child has been known; and
Sec. 14. Hearing. - Upon satisfactory proof that the order of hearing has been published and
(c) The full name by which the child is to be known. jurisdictional requirements have been complied with, the court shall proceed to hear the
petition. The petitioner and the adoptee must personally appear and the former must testify
Sec. 11. Annexes to the Petition. - The following documents shall be attached to the before the presiding judge of the court on the date set for hearing.
petition:
The court shall verify from the social worker and determine whether the biological parent has
A. Birth, baptismal or foundling certificate, as the case may be, and school records showing been properly counseled against making hasty decisions caused by strain or anxiety to give up
the name, age and residence of the adoptee; the child; ensure that all measures to strengthen the family have been exhausted; and
ascertain if any prolonged stay of the child in his own home will be inimical to his welfare and
B. Affidavit of consent of the following: interest.
1. The adoptee, if ten (10) years of age or over;
Sec. 15. Supervised Trial Custody. - Before issuance of the decree of adoption, the court
2. The biological parents of the child, if known, or the legal guardian, or the child-placement shall give the adopter trial custody of the adoptee for a period of at least six (6) months within
agency, child-caring agency, or the proper government instrumentality which has legal custody which the parties are expected to adjust psychologically and emotionally to each other and
of the child; establish a bonding relationship. The trial custody shall be monitored by the social worker of
the court, the Department, or the social service of the local government unit, or the child-
3. The legitimate and adopted children of the adopter and of the adoptee, if any, who are ten placement or child-caring agency which submitted and prepared the case studies. During said
(10) years of age or over; period, temporary parental authority shall be vested in the adopter.

4. The illegitimate children of the adopter living with him who are ten (10) years of age or The court may, motu proprio or upon motion of any party, reduce the period or exempt the
over; and parties if it finds that the same shall be for the best interests of the adoptee, stating the
reasons therefor.
5. The spouse, if any, of the adopter or adoptee.
C. Child study report on the adoptee and his biological parents; An alien adopter however must complete the 6-month trial custody except the following:

D. If the petitioner is an alien, certification by his diplomatic or consular office or any a) a former Filipino citizen who seeks to adopt a relative within the fourth (4th) degree of
appropriate government agency that he has the legal capacity to adopt in his country and that consanguinity or affinity; or
his government allows the adoptee to enter his country as his own adopted child unless
exempted under Section 4(2); b) one who seeks to adopt the legitimate child of his Filipino spouse; or

E. Home study report on the adopters. If the adopter is an alien or residing abroad but c) one who is married to a Filipino citizen and seeks to adopt jointly with his or her spouse the
qualified to adopt, the home study report by a foreign adoption agency duly accredited by the latter’s relative within the fourth (4th) degree of consanguinity or affinity.
Inter-Country Adoption Board; and If the child is below seven (7) years of age and is placed with the prospective adopter through
a pre-adoption placement authority issued by the Department, the court shall order that the
F. Decree of annulment, nullity or legal separation of the adopter as well as that of the prospective adopter shall enjoy all the benefits to which the biological parent is entitled from
biological parents of the adoptee, if any. the date the adoptee is placed with him.

Sec. 12. Order of Hearing. - If the petition and attachments are sufficient in form and The social worker shall submit to the court a report on the result of the trial custody within
substance, the court shall issue an order which shall contain the following: two weeks after its termination.

121
RECTO, GAYLE ANGELI M.
2011-0008 | AUSL
Personal Notes on Remedial Law 2 Review (based on the syllabus of Prof. Henedino M. Brondial)

Sec. 16. Decree of Adoption. - If the supervised trial custody is satisfactory to the parties The court shall order that the parental authority of the biological parent of the adoptee, if
and the court is convinced from the trial custody report and the evidence adduced that the known, or the legal custody of the Department shall be restored if the adoptee is still a minor
adoption shall redound to the best interests of the adoptee, a decree of adoption shall be or incapacitated and declare that the reciprocal rights and obligations of the adopter and the
issued which shall take effect as of the date the original petition was filed even if the adoptee to each other shall be extinguished.
petitioners die before its issuance.
The court shall further declare that successional rights shall revert to its status prior to
The decree shall: adoption, as of the date of judgment of judicial rescission. Vested rights acquired prior to
judicial rescission shall be respected.
A. State the name by which the child is to be known and registered;
It shall also order the adoptee to use the name stated in his original birth or foundling
B. Order: certificate.

1) the Clerk of Court to issue to the adopter a certificate of finality upon expiration of the 15- The court shall further order the Civil Registrar where the adoption decree was registered to
day reglementary period within which to appeal; cancel the new birth certificate of the adoptee and reinstate his original birth or foundling
certificate.
2) the adopter to submit a certified true copy of the decree of adoption and the certificate of
finality to the Civil Registrar where the child was originally registered within thirty (30) days Sec. 24. Service of Judgment. - A certified true copy of the judgment together with a
from receipt of the certificate of finality. In case of change of name, the decree shall be certificate of finality issued by the Branch Clerk of the Court which rendered the decision in
submitted to the Civil Registrar where the court issuing the same is situated. accordance with the preceding Section shall be served by the petitioner upon the Civil
Registrar concerned within thirty (30) days from receipt of the certificate of finality. The Civil
3) the Civil Registrar of the place where the adoptee was registered: Registrar shall forthwith enter the rescission decree in the register and submit proof of
compliance to the court issuing the decree and the Clerk of Court within thirty (30) days from
a. to annotate on the adoptee’s original certificate of birth the decree of adoption within thirty receipt of the decree.
(30) days from receipt of the certificate of finality;
The Clerk of Court shall enter the compliance in accordance with Section 17 hereof.
b. to issue a certificate of birth which shall not bear any notation that it is a new or amended
certificate and which shall show, among others, the following: registry number, date of SEC. 25. Repeal. - This supersedes Rule 99 on Adoption and Rule 100 of the Rules
registration, name of child, sex, date of birth, place of birth, name and citizenship of adoptive of Court.
mother and father, and the date and place of their marriage, when applicable;

c. to seal the original certificate of birth in the civil registry records which can be opened only B. Inter-Country Adoption
upon order of the court which issued the decree of adoption; and
Sec. 26. Applicability. - The following sections apply to inter-country adoption of Filipino
d. to submit to the court issuing the decree of adoption proof of compliance with all the children by foreign nationals and Filipino citizens permanently residing abroad.
foregoing within thirty days from receipt of the decree.
If the adoptee is a foundling, the court shall order the Civil Registrar where the foundling was SEC. 27. Objectives. - The State shall:
registered, to annotate the decree of adoption on the foundling certificate and a new birth
certificate shall be ordered prepared by the Civil Registrar in accordance with the decree. a) a) consider inter-country adoption as an alternative means of child care, if the child cannot
be placed in a foster or an adoptive family or cannot, in any suitable manner, be cared for in
Sec. 17. Book of Adoptions. - The Clerk of Court shall keep a book of adoptions showing the the Philippines;
date of issuance of the decree in each case, compliance by the Civil Registrar with Section
16(B)(3) and all incidents arising after the issuance of the decree. b) ensure that the child subject of inter-country adoption enjoys the same protection accorded
to children in domestic adoption; and
Sec. 18. Confidential Nature of Proceedings and Records. - All hearings in adoption
cases, after compliance with the jurisdictional requirements shall be confidential and shall not c) take all measures to ensure that the placement arising therefrom does not result in
be open to the public. All records, books and papers relating to the adoption cases in the files improper financial gain for those involved.
of the court, the Department, or any other agency or institution participating in the adoption
proceedings shall be kept strictly confidential. Sec. 28. Where to File Petition. - A verified petition to adopt a Filipino child may be filed
by a foreign national or Filipino citizen permanently residing abroad with the Family Court
If the court finds that the disclosure of the information to a third person is necessary for having jurisdiction over the place where the child resides or may be found.
security reasons or for purposes connected with or arising out of the adoption and will be for
the best interests of the adoptee, the court may, upon proper motion, order the necessary It may be filed directly with the Inter-Country Adoption Board.
information to be released, restricting the purposes for which it may be used.
Sec. 29. Who may be adopted. - Only a child legally available for domestic adoption may
Sec. 19. Rescission of Adoption of the Adoptee. - The petition shall be verified and filed be the subject of inter-country adoption.
by the adoptee who is over eighteen (18) years of age, or with the assistance of the
Department, if he is a minor, or if he is over eighteen (18) years of age but is incapacitated, Sec. 30. Contents of Petition. - The petitioner must allege:
by his guardian or counsel.
a) his age and the age of the child to be adopted, showing that he is at least twenty-seven
The adoption may be rescinded based on any of the following grounds committed by the (27) years of age and at least sixteen (16) years older than the child to be adopted at the time
adopter: of application, unless the petitioner is the parent by nature of the child to be adopted or the
spouse of such parent, in which case the age difference does not apply;
1) repeated physical and verbal maltreatment by the adopter despite having undergone
counseling; b) if married, the name of the spouse who must be joined as co-petitioner except when the
adoptee is a legitimate child of his spouse;
2) attempt on the life of the adoptee;
c) that he has the capacity to act and assume all rights and responsibilities of parental
3) sexual assault or violence; or authority under his national laws, and has undergone the appropriate counseling from an
accredited counselor in his country;
4) abandonment or failure to comply with parental obligations.
Adoption, being in the best interests of the child, shall not be subject to rescission by the d) that he has not been convicted of a crime involving moral turpitude;
adopter. However, the adopter may disinherit the adoptee for causes provided in Article 919 of
the Civil Code. e) that he is eligible to adopt under his national law;

Sec. 20. Venue. - The petition shall be filed with the Family Court of the city or province f) that he can provide the proper care and support and instill the necessary moral values and
where the adoptee resides. example to all his children, including the child to be adopted;

Sec. 21. Time within which to file petition. - The adoptee, if incapacitated, must file the g) that he agrees to uphold the basic rights of the child, as embodied under Philippine laws and
petition for rescission or revocation of adoption within five (5) years after he reaches the age the U. N. Convention on the Rights of the Child, and to abide by the rules and regulations
of majority, or if he was incompetent at the time of the adoption, within five (5) years after issued to implement the provisions of Republic Act No. 8043;
recovery from such incompetency.
h) that he comes from a country with which the Philippines has diplomatic relations and whose
Sec. 22. Order to Answer. - The court shall issue an order requiring the adverse party to government maintains a similarly authorized and accredited agency and that adoption of a
answer the petition within fifteen (15) days from receipt of a copy thereof. The order and copy Filipino child is allowed under his national laws; and
of the petition shall be served on the adverse party in such manner as the court may direct.
i) that he possesses all the qualifications and none of the disqualifications provided in this
Sec. 23. Judgment. - If the court finds that the allegations of the petition are true, it shall Rule, in Republic Act No. 8043 and in all other applicable Philippine laws.
render judgment ordering the rescission of adoption, with or without costs, as justice requires.
Sec. 31. Annexes. - The petition for adoption shall contain the following annexes written and
officially translated in English:
122
RECTO, GAYLE ANGELI M.
2011-0008 | AUSL
Personal Notes on Remedial Law 2 Review (based on the syllabus of Prof. Henedino M. Brondial)

We are only going to study 2 fundamental questions on adoption. Who may be


a) Birth certificate of petitioner; adopted and who may adopt. Once you perfect that we can go to another
lesson.
b) Marriage contract, if married, and, if applicable, the divorce decree, or judgment dissolving
the marriage;
Q: Who may adopt? Or who is a qualified prospective adopter?
c) Sworn statement of consent of petitioner’s biological or adopted children above ten (10) A:
years of age; 1. Any Filipino Citizen
2. Any alien
d) Physical, medical and psychological evaluation of the petitioner certified by a duly licensed
3. Guardian, with respect to the ward. And remember, there is no
physician and psychologist;
qualification as to the citizenship of the guardian. The only qualification is that
e) Income tax returns or any authentic document showing the current financial capability of the guardianship must have ended AND his accountability as a guardian has
the petitioner; been completed. Yun lang ang qualification.

f) Police clearance of petitioner issued within six (6) months before the filing of the petitioner; Q: Now let's go to the Filipino, what are the qualifications?
A:
g) Character reference from the local church/minister, the petitioner’s employer and a member
of the immediate community who have known the petitioner for at least five (5) years; 1. of legal age;
2. he must possess full civil capacity and legal rights;
h) Full body postcard-size pictures of the petitioner and his immediate family taken at least six 3. of good moral character;
(6) months before the filing of the petition. 4. has not been convicted of any crime involving moral turpitude;
5. emotionally and psychologically capable of caring or children;
Sec. 32. Duty of Court. - The court, after finding that the petition is sufficient in form and
6. financially capable or economically capable;
substance and a proper case for inter-country adoption, shall immediately transmit the petition
to the Inter-Country Adoption Board for appropriate action. 7. at least 16 years older than the adoptee.

SEC. 33. Effectivity. - This Rule shall take effect on August 22, 2002 following its publication NOTE: When one is of legal age, a minor cannot adopt. Because he must be
in a newspaper of general circulation. capable of unquestionable demonstration. Under the Rules on Evidence, the
court can either take judicial notice of that because he is capable of
OLD LECTURE unquestionable demonstration.

ADOPTION Q: What does possession of full civil capacity and legal rights mean?
Background of adoption: The rule on adoption has been amended several times A: Give me a person who is not possessed of full civil capacity and legal right. A
and even the rules on procedure, which you find in 99 and 100 are no longer convicted person which has received an additional penalty of civil interdiction.
applicable. They have been repealed expressly. As early as the mid-70s when the
Child and Youth Welfare Code (PD 603) was enacted, it already amended the Q: How about a deaf-mute? Is a deaf-mute in possession of full civil capacity?
rules on adoption. PD 603 was also amended by the Family Code. But all these A: Under the rules, a deaf-mute who is not able to read and write is not in
are no longer applicable in toto because of the new laws on adoption which possession of full civil capacity, thus it is incapable of contracting obligations.
should be the subject matter of today's section. The laws applicable now is not
even the Civil Code per se, but rather it should be RA 8562, the Domestic Q: A person caught urinating, and charged of urinating in public. Is this not
Adoption Law of 1998 and the Inter Country Adoption Law of 1995 (RA 8043). moral turpitude?
These are the substantive law governing adoption. Our concern is the rule on A: Urinating or defecating in public is an offense and not a crime. The
adoption. requirement speaks of one who has not been convicted of a crime, which is
punishable by the Revised Penal Code.
AM 02-6-02SC, dated July 2002, is the new rule on Adoption.
Q: But if you are charged of Rape, can you not adopt?
Q: Who may be adopted only? A: No. Because it speaks of conviction. You must first be convicted.
A: General Rule: one who is legally available for adoption.
But if you are convicted of homicide, recent jurisprudence say, you can still adopt
Q: Who are legally available for adoption? because it is not a crime of moral turpitude. So tatlo, there must be a crime,
A: Those who are voluntarily committed and/or involuntarily committed. there must be conviction, and it must be of moral turpitude.

The parents or guardians of these persons have voluntarily surrendered their Q: How do you establish your emotional and psychological capacity?
parental or guardianship authority to the DSWD. A: Clearances to show that you have not been convicted from a crime of moral
turpitude. Clearances from RTC, police, PNP, barangay, MTC, etc. And these
Q: Who are those involuntarily committed? clearances will ought to show that you are of good moral character, that you are
A: Judicially or administratively deprived. Minors, whose parents or guardians are emotionally and psychologically capacitated, subject of course to presentation of
administratively or judicially deprived of their authority over these persons. They evidence and cross-examination. Wala namang summons sa adoption.
are involuntarily committed. And within that concept, you have the abandoned,
the dependents, and neglected children. Q: How do you establish your financial capacity?
A: Certificate of employment, income tax returns.
Q: What is a child-placement agency as distinguished from a child-caring
agency? Q: When we speak of age gap, the miracle number is 16, why? What is sought
A: The distinction is in the services that they cater. Pag placement, it is to to be avoided by this age gap?
provide comprehensive child welfare services including, but not limited to, A: To avoid temptation. You look at the history of the Civil Code of the
receiving applications for adoption, evaluating the prospective adoptive parents, Philippines which was adopted from the Civil Code of Spain. And being of
and preparing the adoption home study while pag caring, it is like Hospicio de European origin, there is that also a 16 year old gap. Malalaking bulas ang mga
San Jose, which provides 24 hour residential care services for abandoned, european and they seek adoption as a means of having mistresses.
orphaned, neglected or voluntarily committed therein.
Q: Who may be adopted?
Q: Which court has jurisdiction over a petition for adoption? A: General Rule: one who is legally available for adoption.
A: It is the Family Court. RTC is not even a totally and perfectly correct answer
because it may be another RTC. Q: Who are legally available for adoption?
A: There are 7. So that minority is not an absolute requirement to be adopted.
Q: What is the venue? Relationship is not even a disqualification on the part of the adoptee. because
A: The place where the adoptive parents reside. they can be adopted.
NOTE: Here, the meaning of residence is the actual habituation of the petitioner.
What I would like to emphasize is the doctrine laid down in Cang v CA, which is
that consent must be given either by the prospective adoptee, if he is at least 10

123
RECTO, GAYLE ANGELI M.
2011-0008 | AUSL
Personal Notes on Remedial Law 2 Review (based on the syllabus of Prof. Henedino M. Brondial)

years old, or parents, or guardians, or the DSWD. Without that consent, as laid another decree of adoption. Ang finality dito is not the finality of the judgment
down in this doctrine, adoption is null and void. but rather the finality of the decree of adoption.
So you can adopt even your own child. You can even adopt your own
grandchildren just like what Juan Ponce Enrile did to his grandchildren. Q: When is the decree of adoption issued?
A: After complying with the 6 month trial custody. Again, this is another
Q: Another person who may adopt is a foreigner, and he may adopt under the undertaking of the DSWD. Tignan niyo, the home or case study report is before
domestic adoption law or the inter-country adoption law. What are the the decision is rendered, but before a decree of adoption is rendered kailangan
requirements? pa ng isa pang report which is the trial custody report.
A: So all the requirements applied to a Filipino prospective adopter are also
required of an alien. Therefore, he must be of legal age, in possession of full civil Q: For how long?
capacity and legal rights, of good moral character, not convicted of a crime A: General Rule: 6 months.
involving moral turpitude, etc. Exception: when it is in the best interest of the child; the petition during the
Over and above this, an alien is also required: hearing is that the prospective adoptee is already living with the adopter.
1. he must have continuously resided in the Philippines 3 years prior to
the filing of the adoption; Q: After the decree of adoption has become final and executory, what is the
2. he must have a certification from his consular official that he is in next?
possession of full civil capacity; A: There should be a new birth certificate issued. Remember that one of the
3. his country of origin must have diplomatic relations with our country; rights of a party in litigation is the right to a speedy and public trial. But this
4. his country of origin must allow the adoptee to become a citizen of adoption proceeding is exceptional because even the documents there are public
his country; records but are not open to the public, not anybody can get it including the new
5. that his country of origin allows the adoptee to enter the country of birth certificate issued to the adopter.
origin of the adopter;
NOTE: But take note that under the rule on rescission of adoption, the issued
Q: Where should he file this petition for adoption? (Section 28) certificate of the civil registrar's office is cancelled in favor of the old birth
A: certificate, which means that the old birth certificate is not deleted, but remains
1. It may be filed by a foreign national or Filipino citizen permanently in the record. And this is the reason of the confidential nature of the
residing abroad with the Family Court having jurisdiction over the place where proceedings.
the child resides or may be found;
2. It may be filed directly with the Inter-Country Adoption Board. Q: What is adoption?
A: It is a judicial proceeding whereby the relationship of paternity and filiation is
When an alien files a petition before the Inter-Country Adoption Board, the alien established. AS simple as that.
will come here in the Philippines and will bring the adoptee and the social worker
back to his country of origin for the trial custody of 6 months will happen there. According to a noted professor, when a decree of adoption is final and
executory, there is the total and absolute cut of any legal relationship between
This is purposely done by the law, because there is an objective of the natural parent and the child.
discouragement. To discourage the adoption of Filipino citizens by aliens. In fact
that is only allowed if the prospective adoptee cannot be adopted by a Filipino. Under the new rules, the adopter cannot rescind the adoption but only disinherit
Filipinos have the preference of adopting. the adopted child. An adopted child has all the rights of a legitimate child.

Q: What constitutes the allegations?


A: CASES
1. Jurisdictional facts;
2. legal capacity of the prospective adopter and the adoptee; HERBERT CANG, petitioner, vs. COURT OF APPEALS and Spouses RONALD V. CLAVANO
and MARIA CLARA CLAVANO, respondents.
G.R. No. 105308 September 25, 1998
In the case of Republic v. Hernandez (1997), the SC said that the petition for
THIRD DIVISION
adoption does not carry with it the change of name of the adoptee. With the
present rules on adoption specifically under Section 10, this doctrine does not FACTS:
apply anymore. This is a good bar question. So you can now have joinder of x Herbert and Anna Marie Clavano who were married on Jan 27, 1973, begot 3
causes of action in special proceedings, in effect. Dati wala. So you can now join children,
o Keith, Charmaine, and Joseph Anthony
in your prayer, asking for a change of name and for adoption. Let me emphasize, x Anna Marie learned of her husband's alleged extramarital affair with Wilma Soco
it is a change of name and not a change of surname because change of surname
x Anna Marie Æ filed a petition for legal separation
is an automatic effect of adoption. o RTC Æ granted
x Herbert left for the US
Q: After you have filed a petition for adoption, what happens now? o Was granted a divorce decree there
A: There would be a case study, where it is immediately assigned to a social o also granted sole custody of the three minor children to Anna Marie
worker. Under present dispensation in our jurisdiction, every family courts is now x Ronald Clavano and Maria Clara Diago Clavano (respectively the brother and
provided with a social worker. Because a petition for adoption is exclusively sister-in-law of Anna Marie) Æ filed for the adoption of the 3 minor Cang children
o Their petition bears the signature of then 14-year-old Keith
cognizable by the Family Courts. signifying consent to his adoption; Anna Marie likewise filed an
affidavit of consent alleging that her husband had "evaded his legal
obligation to support" his children
Q: For what purpose is this? x Herbert learned about the petition and went back to PH
A: One is the home study report and the other is the case study report. The x Herbert Æ filed an Opposition to the petition
home study report is about assigning to the social worker of the respective o Although the Clavanos were financially capable of supporting the
children while his finances were "too meager" compared to theirs,
adoptee and the case study report is about the prospective adopter. he could not "in conscience, allow anybody to strip him of his
parental authority over his beloved children
The general objective of adoption, why it is allowed under the rules, is that x RTC Æ issued an Order
which leads to the benefit of the adoptee. o finding that Anna Marie had, in effect, relinquished custody over
the children and, therefore, such custody should be transferred to
Under the rules, it is mandatory that the petitioner must appear and testify. the father
o directed the Clavanos to deliver custody over the minors to
Similar to an annulment of marriage case, there can be no stipulation of facts petitioner
here or confessions of judgment. There must be actual presentation of evidence x RTC Æ then rendered a decision AGAINST HERBERT
here. And basic requirement here is the consent given by the adoptee, the o the oppositor Herbert Cang has abandoned his children. And
parents or the guardians of the adoptee. abandonment of a child by its parent is commonly specified by
statute as a ground for dispensing with his consent to its adoption.
Q: After the hearing, may the court render judgment now? o Indeed, in such case, adoption will be allowed not only without the
consent of the parent, but even against his opposition
A: The judgment referred to here is not similar to the judgment that we have x Herbert Æ appealed to CA
ordinarily because the judgment here is always subject to the issuance of x CA Æ affirmed RTC
124
RECTO, GAYLE ANGELI M.
2011-0008 | AUSL
Personal Notes on Remedial Law 2 Review (based on the syllabus of Prof. Henedino M. Brondial)

o Art. 188 of the Family Code requires the written consent of the x As clearly inferred from the foregoing provisions of law, the written consent of
natural parents of the child to be adopted. It has been held the natural parent is indispensable for the validity of the decree of adoption.
however that the consent of the parent who has abandoned the Nevertheless, the requirement of written consent can be dispensed with if the
child is not necessary (Dayrit vs. Piccio; Santos vs. Ananzanso). In parent has abandoned the child 13 or that such parent is "insane or hopelessly
adoption cases, abandonment connotes any conduct on the part of intemperate." The court may acquire jurisdiction over the case even, without the
the parent to forego parental duties and relinquish parental claims written consent of the parents or one of the parents provided that the petition for
to the child, or the neglect or refusal to perform the natural and adoption alleges facts sufficient to warrant exemption from compliance
legal obligations which parents owe their children (Santos vs. therewith. This is in consonance with the liberality with which this Court treats
Ananzanso, supra), or the withholding of the parent's presence, his the procedural aspect of adoption. Thus, the Court declared:
care and the opportunity to display voluntary affection. The issue of o . . . . The technical rules of pleading should not be
abandonment is amply covered by the discussion of the first error. stringently applied to adoption proceedings, and it is
x Herbert Æ filed an MR deemed more important that the petition should contain
o CA Æ denied facts relating to the child and its parents, which may give
x Herbert Æ filed a Rule 45 before the SC information to those interested, than that it should be
formally correct as a pleading. Accordingly, it is generally held
ISSUE # 1: Whether RTC acquired jurisdiction over the subject petition for adoption, that a petition will confer jurisdiction if it substantially complies with
considering the lack of consent of Herbert the adoption statute, alleging all facts necessary to give the court
jurisdiction. 14
HELD # 1: YES. x In the instant case, only the affidavit of consent of the natural mother was
x Art. 31 of P.D. No. 603 provides — attached to the petition for adoption. Petitioner's consent, as the natural
o Art. 31. Whose Consent is Necessary. — The written consent father is lacking. Nonetheless, the petition sufficiently alleged the fact
of the following to the adoption shall be necessary: of abandonment of the minors for adoption by the natural father as
ƒ (1) The person to be adopted, if fourteen follows:
years of age or, over; o 3. That the children's mother, sister of petitioner
ƒ (2) The natural parents of the child or his RONALD V. CLAVANO, has given her express consent to this
legal guardian of the Department of Social Welfare or adoption, as shown by Affidavit of Consent, Annex "A". Likewise,
any duly licensed child placement agency under the written consent of Keith Cang, now 14 years of age appears on
whose care the child may be; page 2 of this petition; However, the father of the children, Herbert
ƒ (3) The natural children, fourteen years and Cang, had already left his wife and children and had already
above, of the adopting parents. (Emphasis supplied) divorced the former, as evidenced by the xerox copy of the
x On December 17, 1986, then President Corazon C. Aquino issued DECREE OF DIVORCE issued by the County of Washoe, State of
Executive Order No. 91 amending Articles 27, 28, 29, 31, 33 and 35 of Nevada, U.S.A. (Annex "B") which was filed at the instance of Mr.
the Child and Youth Welfare Code. As thus amended, Article 31 read: Cang, not long after he abandoned his family to live in the United
o Art. 31. Whose Consent is Necessary. — The written consent States as an illegal immigrant. 15
of the following to the adoption shall be necessary: x The allegations of abandonment in the petition for adoption, even
ƒ (1) The person to be adopted, if fourteen absent the written consent of petitioner, sufficiently vested the lower
years of age or over; court with jurisdiction since abandonment of the child by his natural
ƒ (2) The natural parents of the child or his parents is one of the circumstances under which our statutes and
legal guardian after receiving counselling and jurisprudence 16 dispense with the requirement of written consent to
appropriate social services from the Ministry of Social the adoption of their minor children.
Services and Development or from a duly licensed
child-placement agency;
ƒ (3) The Ministry of Social Services and ISSUE # 2: Whether Herbert abandoned his minor children so as to dispense with his consent
Development or any duly licensed child-placement in the petition for the adoption of said children.
agency under whose care and legal custody the child
may be; HELD # 2: NO.
ƒ (4) The natural children, fourteen years and x However, in cases where the father opposes the adoption primarily because his
above, of the adopting parents. (Emphasis supplied) consent thereto was not sought, the matter of whether he had abandoned his
x Jurisdiction being a matter of substantive law, the established rule is child becomes a proper issue for determination. The issue of abandonment
that the statute in force at the time of the commencement of the action by the oppositor natural parent is a preliminary issue that an adoption
determines the jurisdiction of the court. 12 As such, when private court must first confront. Only upon, failure of the oppositor natural
respondents filed the petition for adoption on September 25, 1987, the father to prove to the satisfaction of the court that he did not abandon
applicable law was the Child and Youth Welfare Code, as amended by Executive his child may the petition for adoption be considered on its merits.
Order No. 91. x As a rule, factual findings of the lower courts are final and binding upon this
x During the pendency of the petition for adoption or on August 3, 1988, Court. 17 This Court is not expected nor required to examine or contrast the oral
the Family Code which amended the Child and Youth Welfare Code and documentary evidence submitted by the parties. 18 However, although this
took effect. Article 256 of the Family Code provides for its retroactivity Court is not a trier of facts, it has the authority to review and reverse the factual
"insofar as it does not prejudice or impair vested or acquired rights in findings of the lower courts if it that these do not conform to the evidence on
accordance with the Civil Code or other laws." As amended by the Family record. 19
Code, the statutory provision on consent for adoption now reads: x In Reyes v. Court of Appeals, 20 this Court has held that the exceptions to the
o Art. 188. The written consent of the following to the adoption rule that factual findings of the trial court are final and conclusive and may not
shall be necessary: be reviewed on appeal are the following:
ƒ (1) The person to be adopted, if ten years o (1) when the inference made is manifestly mistaken, absurd or
of age or over; impossible;
ƒ (2) The parents by nature of the child, the o (2) when there is a grave abuse of discretion;
legal guardian, or the proper government o (3) when the finding is grounded entirely on speculations, surmises
instrumentality; or conjectures;
ƒ (3) The legitimate and adopted children, ten o (4) when the judgment of the Court of Appeals is based on
years of age or over, of the adopting parent or misapprehension of facts;
parents; o (5) when the findings of fact are conflicting;
ƒ (4) The illegitimate children, ten years of o (6) when the Court of Appeals, in making its findings, went beyond
age or over, of the adopting parents, if living with the issues of the case and the same is contrary to the admissions
said parent and the latter's spouse, if any; and of both appellant and appellee;
ƒ (5) The spouse, if any, of the person o (7) when the findings of the Court of Appeals are contrary to those
adopting or to be adopted. (Emphasis supplied) of the trial court;
x Based on the foregoing, it is thus evident that notwithstanding the amendments o (8) when the findings of fact are conclusions without citation of
to the law, the written consent of the natural parent to the adoption has specific evidence on which they are based;
remained a requisite for its validity. Notably, such requirement is also o (9) when the Court of Appeals manifestly overlooked certain
embodied in Rule 99 of the Rules of Court as follows: relevant facts not disputed by the parties and which, if properly
o Sec. 3. Consent to adoption. — There shall be filed with the considered, would justify a different conclusion and
petition a written consent to the adoption signed by the child, if o (10) when the findings of fact of the Court of Appeals are premised
fourteen years of age or over and not incompetent, and by the on the absence of evidence and are contradicted by the evidence
child's spouse, if any, and by each of its known living parents who on record.
is not insane or hopelessly intemperate or has not abandoned the x This Court finds that both the lower court and the Court of Appeals failed to
child, or if the child is in the custody of an orphan asylum, appreciate facts and circumstances that should have elicited a different
children's home, or benevolent society or person, by the proper conclusion 21 on the issue of whether petitioner has so abandoned his children,
officer or officers of such asylum, home, or society, or by such thereby making his consent to the adoption unnecessary.
persons; but if the child is illegitimate and has not been recognized, x In its ordinary sense, the word "abandon'' means to forsake entirely, to
the consent of its father to the adoption shall not be required. forsake or renounce utterly. The dictionaries trace this word to the root
(Emphasis supplied) idea of "putting under a ban." The emphasis is on the finality and publicity
125
RECTO, GAYLE ANGELI M.
2011-0008 | AUSL
Personal Notes on Remedial Law 2 Review (based on the syllabus of Prof. Henedino M. Brondial)

with which a thing or body is thus put in the control of another, hence, the One cannot say that his or her suffering is greater than that of the
meaning of giving up absolutely, with intent never to resume or claim one's other parent. It is not so much the suffering, pride, and other
rights or interests. 22 In reference to abandonment of a child by his feelings of either parent but the welfare of the child which is the
parent, the act of abandonment imports "any conduct of the parent paramount consideration. (Emphasis supplied) 29
which evinces a settled purpose to forego all parental duties and x Indeed, it would be against the spirit of the law if financial
relinquish all parental claims to the child." It means "neglect or refusal consideration were to be the paramount consideration in deciding
to perform the natural and legal obligations of care and support which whether to deprive a person of parental authority over his children.
parents owe their children." 23 There should be a holistic approach to the matter, taking into account
x In the instant case, records disclose that petitioner's conduct did not manifest a the physical, emotional, psychological, mental, social and spiritual
settled purpose to forego all parental duties and relinquish all parental claims needs of the child. 30 The conclusion of the courts below that petitioner
over his children as to, constitute abandonment. Physical estrangement alone, abandoned his family needs more evidentiary support other than his inability to
without financial and moral desertion, is not tantamount to abandonment. 24 provide them the material comfort that his admittedly affluent in-laws could
While admittedly, petitioner was physically absent as he was then in the United provide. There should be proof that he had so emotionally abandoned them that
States, he was not remiss in his natural and legal obligations of love, care and his children would not miss his guidance and counsel if they were given to
support for his children. He maintained regular communication with his wife and adopting parents. The letters he received from his children prove that petitioner
children through letters and telephone. He used to send packages by mail and maintained the more important emotional tie between him and his children. The
catered to their whims. children needed him not only because he could cater to their whims but also
x Aside from these letters, petitioner also presented certifications of banks in the because he was a person they could share with their daily activities, problems
U.S.A. showing that even prior to the filing of the petition for adoption, he had and triumphs.
deposited amounts for the benefit of his children. 25 Exhibits 24 to 45 are copies x The Court is thus dismayed that the courts below did not look beyond petitioner's
of checks sent by petitioner to the children from 1985 to 1989. "meager" financial support to ferret out other indications on whether petitioner
x These pieces of evidence are all on record. It is, therefore, quite surprising why had in fact abandoned his family. The omission of said courts has led us to
the courts below simply glossed over these, ignoring not only evidence on examine why the children were subjected to the process of adoption,
financial support but also the emotional exchange of sentiments between notwithstanding the proven ties that bound them to their father. To our
petitioner and his family. Instead, the courts below emphasized the meagerness consternation, the record of the case bears out the fact that the welfare of the
of the amounts he sent to his children and the fact that, as regards the bank children was not exactly the "paramount consideration" that impelled Anna Marie
deposits, these were "withdrawable by him alone." Simply put, the courts below to consent to their adoption.
attached a high premium to the prospective adopters' financial status but totally x In her affidavit of consent, Anna Marie expressly said that leaving the children in
brushed aside the possible repercussion of the adoption on the emotional and the country, as she was wont to travel abroad often, was a problem that would
psychological well-being of the children. naturally hamper her job-seeking abroad. In other words, the adoption appears
x True, Keith had expressed his desire to be adopted by his uncle and aunt. to be a matter of convenience for her because Anna Marie herself is financially
However, his seeming steadfastness on the matter as shown by his testimony is capable of supporting her children. 31 In his testimony, private respondent
contradicted by his feelings towards his father as revealed in his letters to him. It Ronald swore that Anna Marie had been out of the country for two years and
is not at all farfetched to conclude that Keith's testimony was actually the effect came home twice or three times, 32 thereby manifesting the fact that it was she
of the filing of the petition for adoption that would certainly have engendered who actually left her children to the care of her relatives. It was bad enough that
confusion in his young mind as to the capability of his father to sustain the their father left their children when he went abroad, but when their mother
lifestyle he had been used to. followed suit for her own reasons, the situation worsened. The Clavano family
x The courts below emphasized respondents' emotional attachment to the children. must have realized this. Hence, when the family first discussed the adoption of the
This is hardly surprising for, from the very start of their young lives, the children children, they decided that the prospective adopter should be Anna Marie's brother
were used to their presence. Such attachment had persisted and certainly, the Jose. However, because he had children of his own, the family decided to devolve
young ones' act of snuggling close to private respondent Ronald Clavano was not the task upon private respondents. 33
indicative of their emotional detachment from their father. Private respondents, x This couple, however, could not always be in Cebu to care for the children. A
being the uncle and aunt of the children, could not but come to their succor businessman, private respondent Ronald Clavano commutes between Cebu and
when they needed help as when Keith got sick and private respondent Ronald Manila while his wife, private respondent Maria Clara, is an international flight
spent for his hospital bills. stewardess. 34 Moreover, private respondent Ronald claimed that he could "take
x In a number of cases, this Court has held that parental authority care of the children while their parents are away," 35 thereby indicating the
cannot be entrusted to a person simply because he could give the child evanescence of his intention. He wanted to have the children's surname changed
a larger measure of material comfort than his natural parent. Thus, in to Clavano for the reason that he wanted to take them to the United States as it
David v. Court of Appeals, 26 the Court awarded custody of a minor illegitimate would be difficult for them to get a visa if their surname were different from his.
child to his mother who was a mere secretary and market vendor instead of to his 36 To be sure, he also testified that he wanted to spare the children the stigma
affluent father who was a married man, not solely because the child opted to go of being products of a broken home.
with his mother. The Court said: x Nevertheless, a close analysis of the testimonies of private respondent Ronald,
o Daisie and her children may not be enjoying a life of affluence that his sister Anna Marie and their brother Jose points to the inescapable conclusion
private respondent promises if the child lives with him. It is that they just wanted to keep the children away from their father. One of the
enough, however, that petitioner is earning a decent living and is overriding considerations for the adoption was allegedly the state of Anna Marie's
able to support her children according to her means. health — she was a victim of an almost fatal accident and suffers from a heart
x In Celis v. Cafuir 27 where the Court was confronted with the issue of whether to ailment. However, she herself admitted that her health condition was not that
award custody of a child to the natural mother or to a foster mother, this Court serious as she could still take care of the children. 37 An eloquent evidence of
said: her ability to physically care for them was her employment at the Philippine
o This court should avert the tragedy in the years to come of having Consulate in Los Angeles 38 — she could not have been employed if her health
deprived mother and son of the beautiful associations and tender, were endangered. It is thus clear that the Clavanos' attempt at depriving
imperishable memories engendered by the relationship of parent petitioner of parental authority apparently stemmed from their notion that he was
and child. We should not take away from a mother the opportunity an inveterate womanizer. Anna Marie in fact expressed fear that her children
of bringing up her own child even at the cost of extreme sacrifice would "never be at ease with the wife of their father." 39
due to poverty and lack of means; so that afterwards, she may be x Petitioner, who described himself as single in status, denied being a womanizer
able to look back with pride and a sense of satisfaction at her and father to the sons of Wilma Soco. 40 As to whether he was telling the truth
sacrifices and her efforts, however humble, to make her dreams of is beside the point. Philippine society, being comparatively conservative and
her little boy come true. We should not forget that the relationship traditional, aside from being Catholic in orientation, it does not countenance
between a foster mother and a child is not natural but artificial. If womanizing on the part of a family man, considering the baneful effects such
the child turns out to be a failure or forgetful of what its foster irresponsible act visits on his family. Neither may the Court place a premium on
parents had done for him, said parents might yet count and the inability of a man to distinguish between siring children and parenting them.
appraise (sic) all that they have done and spent for him and with Nonetheless, the actuality that petitioner carried on an affair with a paramour
regret consider all of it as a dead loss, and even rue the day they cannot be taken as sufficient basis for the conclusion that petitioner was
committed the blunder of taking the child into their hearts and their necessarily an unfit father. 41 Conventional wisdom and common human
home. Not so with a real natural mother who never counts the cost experience show that a "bad" husband does not necessarily make a "bad" father.
and her sacrifices, ever treasuring memories of her associations That a husband is not exactly an upright man is not, strictly speaking, a sufficient
with her child, however unpleasant and disappointing. Flesh and ground to deprive him as a father of his inherent right to parental authority over
blood count ......... the children. 42 Petitioner has demonstrated his love and concern for his children
x In Espiritu v. Court of Appeals, 28 the Court stated that "(I)n ascertaining the when he took the trouble of sending a telegram 43 to the lower court expressing
welfare and best interests of the child, courts are mandated by the Family Code his intention to oppose the adoption immediately after learning about it. He
to take into account all relevant considerations." Thus, in awarding custody of traveled back to this country to attend to the case and to testify about his love
the child to the father, the Court said: for his children and his desire to unite his family once more in the United States.
o A scrutiny of the pleadings in this case indicates that Teresita, or at 44
least, her counsel are more intent on emphasizing the "torture and x Private respondents themselves explained why petitioner failed to abide by the
agony" of a mother separated from her children and the humiliation agreement with his wife on the support of the children. Petitioner was an illegal
she suffered as a, result of her character being made a key issue in alien in the United States. As such, he could not have procured gainful
court rather than the feelings and future, the best interests and employment. Private respondents failed to refute petitioner's testimony that he
welfare of her children. While the bonds between a mother and her did not receive his share from the sale of the conjugal home, 45 pursuant to
small child are special in nature, either parent, whether father or their manifestation/compromise agreement in the legal separation case. Hence, it
mother, is bound to suffer agony and pain if deprived of custody. can be reasonably presumed that the proceeds of the sale redounded to the
126
RECTO, GAYLE ANGELI M.
2011-0008 | AUSL
Personal Notes on Remedial Law 2 Review (based on the syllabus of Prof. Henedino M. Brondial)

benefit of his family, particularly his children. The proceeds may not have lasted Appeals, 57 a case involving the visitorial rights of an illegitimate parent over his
long but there is ample evidence to show that thereafter, petitioner tried to abide child, the Court expressed the opinion that:
by his agreement with his wife and sent his family money, no matter how o Parents have the natural right, as well as the moral and legal duty,
"meager." to care for their children, see to their upbringing and safeguard
x The liberality with which this Court treats matters leading to adoption their best interest and welfare. This authority and responsibility
insofar as it carries out the beneficent purposes of the law to ensure may not be unduly denied the parents; neither may it be
the rights and privileges of the adopted child arising therefrom, ever renounced by them. Even when the parents are estranged and their
mindful that the paramount consideration is the overall benefit and affection for each other is lost, the attachment and feeling for their
interest of the adopted child, should be understood in its proper offsprings invariably remain unchanged. Neither the law not the
context and perspective. The Court's position, should not be misconstrued or courts allow this affinity to suffer absent, of course, any real, grave
misinterpreted as to extend to inferences beyond the contemplation of law and and imminent threat to the well being of the child.
jurisprudence. 46 The discretion to approve adoption proceedings is not to be x Since the incorporation of the law concerning adoption in the Civil Code, there
anchored solely on best interests of the child but likewise, with due regard to the has been a pronounced trend to place emphasis in adoption proceedings, not so
natural rights of the parents over the child. 47 much on the need of childless couples for a child, as on the paramount interest,
x In this regard, this Court notes private respondents' reliance on the of a child who needs the love and care of parents. After the passage of the Child
manifestation/compromise agreement between petitioner and Anna Marie which and Youth Welfare Code and the Family Code, the discernible trend has impelled
became the basis of the decree of legal separation. According to private the enactment of Republic Act No. 8043 on Intercountry, Adoption 58 and
respondents' counsel, 48 the authority given to Anna Marie by that decree to Republic Act No. 8552 establishing the rules on the domestic adoption of Filipino
enter into contracts as a result of the legal separation was "all embracing" 49 children. 59
and, therefore, included giving her sole consent to the adoption. This conclusion x The case at bar applies the relevant provisions of these recent laws,
is however, anchored on the wrong premise that the authority given to the such as the following policies in the "Domestic Adoption Act of 1998":
innocent spouse to enter into contracts that obviously refer to their conjugal o (a) To ensure that every child remains under the care
properties, shall include entering into agreements leading to the adoption of the and custody of his/her parent(s) and be provided with love, care,
children. Such conclusion is as devoid of a legal basis as private respondents' understanding and security towards the full and harmonious
apparent reliance on the decree of legal separation for doing away with development of his/her personality. 60
petitioner's consent to the adoption. o (b) In all matters relating to the care, custody and
x The transfer of custody over the children to Anna Marie by virtue of the decree adoption of a child, his/her interest shall be the paramount
of legal separation did not, of necessity; deprive petitioner of parental authority consideration in accordance with the tenets set forth in the United
for the purpose of placing the children up for adoption. Article 213 of the Family Nations (UN) Convention on the Rights of the Child. 61
Code states: ". . . in case of legal separation of parents, parental authority shall o (c) To prevent the child from unnecessary separation
be exercised by the parent designated by the court." In awarding custody, the from his/her biological parent(s). 62
court shall take into account "all relevant considerations, especially the choice of x Inasmuch as the Philippines is a signatory to the United Nations
the child over seven years of age, unless the parent chosen is unfit." Convention on the Rights of the Child, the government and its officials
x If should be noted, however, that the law only confers on the innocent spouse are duty bound to comply with its mandates. Of particular relevance to
the "exercise" of parental authority. Having custody of the child, the innocent instant case are the following provisions:
spouse shall implement the sum of parental rights with respect to his rearing and o States Parties shall respect the responsibilities, rights and duties of
care. The innocent spouse shall have the right to the child's services and earnings, parents . . . to provide, in a manner consistent with the evolving
and the right to direct his activities and make decisions regarding his care and capacities of the child, appropriate direction and guidance in the
control, education, health and religion. 50 exercise by the child of the rights recognized in the present
x In a number of cases, this Court has considered parental authority, the joint Convention. 63
exercise of which is vested by the law upon the parents, 51 as . . . a mass of o States Parties shall respect the right of the child who is separated
rights and obligations which the law grants to parents for the purpose from one or both parents to maintain personal relations and direct
of the children's physical preservation and development, as well as the contact with both parents on a regular basis, except if it is contrary
cultivation of their intellect and the education of their hearts and to the child's best interests. 64
senses. As regards parental authority, "there is no power, but a task; o A child whose parents reside in different States shall have the right
no complex of rights, but a sum of duties; no sovereignty but a sacred to maintain on a regular basis, save in exceptional circumstances
trust for the welfare of the minor." personal relations and direct contacts with both parents . . . 65
o Parental authority and responsibility are inalienable and may not be o States Parties shall respect the rights and duties of the parents . . .
transferred or renounced except in cases authorized by law. The to provide direction to the child in the exercise of his or her right in
right attached to parental authority, being purely personal, the law a manner consistent with the evolving capacities of the child. 66
allows a waiver of parental authority only in cases of adoption, x Underlying the policies and precepts in international conventions and
guardianship and surrender to a children's home or an orphan the domestic statutes with respect to children is the overriding
institution. When a parent entrusts the custody of a minor to principle that all actuations should be in the best interests of the child.
another, such as a friend or godfather, even in a document, what is This is not, however, to be implemented in derogation of the primary
given is merely temporary custody and it does not constitute a right of the parent or parents to exercise parental authority over him.
renunciation of parental authority. Even if a definite renunciation is The rights of parents vis-à-vis that of their children are not antithetical
manifest, the law still disallows the same. to each other, as in fact, they must be respected and harmonized to the
o The father and mother, being the natural guardians of fullest extent possible.
unemancipated children, are duty-bound and entitled to keep them x Keith, Charmaine and Joseph Anthony have all grown up. Keith and Charmaine
in their custody and company. 52 (Emphasis supplied) are now of legal age while Joseph Anthony is approaching eighteen, the age of
x As such, in instant case, petitioner may not be deemed as having been majority. For sure, they shall be endowed with the discretion to lead lives
completely deprived of parental authority, notwithstanding the award of custody independent of their parents. This is not to state that this case has been
to Anna Marie in the legal separation case. To reiterate, that award was arrived at rendered moot and academic, for their welfare and best interests regarding their
by the lower court on the basis of the agreement of the spouses. adoption, must be determined as of the time that the petition for adoption was
x While parental authority may be waived, as in law it may be subject to filed. 67 Said petition must be denied as it was filed without the required consent
a compromise, 53 there was no factual finding in the legal separation of their father who, by law and under the facts of the case at bar, has not
case that petitioner was such an irresponsible person that he should be abandoned them.
deprived of custody of his children or that there are grounds under the
law that could deprive him of parental authority. In fact, in the legal TOMASA VDA. DE JACOB, as Special Administratrix of the Intestate Estate of Deceased
separation case, the court thereafter ordered the transfer of custody over the Alfredo E. Jacob, petitioner, vs. COURT OF APPEALS, PEDRO PILAPIL, THE REGISTER OF
children from Anna Marie back to petitioner. The order was not implemented DEEDS for the Province of Camarines Sur, and JUAN F. TRIVINO as publisher of "Balalong,"
because of Anna Marie's motion for reconsideration thereon. The Clavano family respondents.
also vehemently objected to the transfer of custody to the petitioner, such that G.R. No. 135216 August 19, 1999
the latter was forced to file a contempt charge against them. 54 THIRD DIVISION
x The law is clear that either parent may lose parental authority over the child only
for a valid reason. No such reason was established in the legal separation case. FACTS:
In the instant case for adoption, the issue is whether or not petitioner had x Tomasa, being the surviving spouse of Dr. Alfredo E. Jacob was appointed as
abandoned his children as to warrant dispensation of his consent to their special administratix of the estate of said deceased in the settlement proceedings
adoption. Deprivation of parental authority is one of the effects of a decree of therefor before RTC Tiagon, CamSur
adoption. 55 But there cannot be a valid decree of adoption in this case precisely x Pedro (claiming to be the legally-adopted son of Alfredo) Æ filed a motion for
because, as this Court has demonstrated earlier, the finding of the courts below intervention
on the issue of petitioner's abandonment of his family was based on a o Claimed that he is the sole surviving heir
misappreciation that was tantamount to non-appreciation, of facts on record. o questioned the validity of the marriage between appellant Tomasa
x As regards the divorce obtained in the United States, this Court has ruled in and his adoptive father Alfredo.
Tenchavez v. Escaño 56 that a divorce obtained by Filipino citizens after the x Tomasa Æ filed an opposition to said motion
effectivity of the Civil Code is not recognized in this jurisdiction as it is contrary to x Tomasa Æ likewise filed an action for injunction with damages
State policy. While petitioner is now an American citizen, as regards Anna Marie o questioning appellee's claim as the legal heir of Alfredo
who has apparently remained a Filipino citizen, the divorce has no legal effect. x RTC (settlement proceedings) Æ rendered a decision IN FAVOR OF PEDRO
x Parental authority is a constitutionally protected State policy borne out of x Tomasa Æ appealed to CA
established customs and tradition of our people. Thus, in Silva v. Court of x CA Æ affirmed RTC in toto
127
RECTO, GAYLE ANGELI M.
2011-0008 | AUSL
Personal Notes on Remedial Law 2 Review (based on the syllabus of Prof. Henedino M. Brondial)

x Tomasa Æ filed a Rule 45 before the SC Dr. Jacob. Taken together, these circumstances inexorably negate the alleged
adoption of respondent.49
ISSUE: Whether Pedro was able to establish that he was the legally-adopted son of the x The burden of proof in establishing adoption is upon the person
deceased. claiming such relationship.50 This Respondent Pilapil failed to do.
Moreover, the evidence presented by petitioner shows that the alleged
HELD: NO. adoption is a sham.
x In ruling that Respondent Pedro Pilapil was adopted by Dr. Jacob and that the
signature of Judge Moya appearing on the Adoption Order was valid, the Court of
Appeals relied on the presumption that the judge had acted in the regular REPUBLIC OF THE PHILIPPINES, petitioner, vs. HON. JOSE R. HERNANDEZ, in his
performance of his duties. The appellate court also gave credence to the capacity as Presiding Judge, Regional Trial Court, Branch 158, Pasig City and SPOUSES VAN
testimony of respondent’s handwriting expert, for "the assessment of the MUNSON y NAVARRO and REGINA MUNSON y ANDRADE, respondents.
credibility of such expert witness rests largely on the discretion of the trial court . G.R. No. 117209 February 9, 1996
. . "35 SECOND DIVISION
o We disagree.
x As a rule, the factual findings of the trial court are accorded great weight and FACTS:
respect by appellate courts, because it had the opportunity to observe the x Spouses Van Munson y Navarro and Regina Munson y Andrade Æ filed a petition
demeanor of witnesses and to note telltale signs indicating the truth or the falsity to adopt the minor Kevin Earl Bartolome Moran before RTC Pasig
of a testimony. The rule, however, is not applicable to the present case, because o In the very same petition, private respondents prayed for the
it was Judge Augusto O. Cledera, not the ponente, who heard the testimonies of change of the first name or said minor adoptee to Aaron Joseph,
the two expert witnesses. Thus, the Court examined the records and found that the same being the name with which he was baptized in keeping
the Court of Appeals and the trial court "failed to notice certain relevant facts with religious tradition and by which he has been called by his
which, if properly considered, will justify a different conclusion."36 Hence, the adoptive family, relatives and friends since May 6, 1993 when he
present case is an exception to the general rule that only questions of law may arrived at private respondents' residence
be reviewed in petitions under Rule 45.37 x Republic Æ filed an opposition
x Central to the present question is the authenticity of Judge Moya's signature on o opposed the inclusion of the relief for change of name in the same
the questioned Order of Adoption. To enlighten the trial court on this matter, two petition for adoption
expert witnesses were presented, one for petitioner and one for Respondent x RTC Æ decided IN FAVOR of the SPOUSES
Pilapil. The trial court relied mainly on respondent’s expert and brushed aside the o WHEREFORE, minor child Kevin Earl Bartolome Moran is freed from
Deposition of Judge Moya himself.38 Respondent Pilapil justifies the trial judge’s all legal obligations of obedience and maintenance with respect to
action by arguing that the Deposition was ambiguous. He contends that Judge his natural parents, and for all legal intents and purposes shall be
Moya could not remember whether the signature on the Order was his and cites known as Aaron Joseph Munson y Andrade, the legally adopted
the following portion as proof:39 child of Van Munson and Regina Munson effective upon the filing of
o Q. What was you[r] response, sir? the petition on March 10, 1994. As soon as the decree of adoption
o A: I said I do not remember. becomes final and executory, it shall be recorded in the Office of
x Respondent Pilapil's argument is misleading, because it took the judge's the Local Civil Registrar of Pasig, Metro Manila pursuant to Section
testimony out of its context. Considered with the rest of the Deposition, Judge 8, Rule 99 and Section 6, Rule 103, respectively, of the Rules of
Moya's statements contained no ambiguity. He was clear when he answered the Court, and shall be annotated in the record of birth of the adopted
queries in the following manner: child, which in this case is in Valenzuela, Metro Manila, where the
o Atty. Benito P. Fabie child was born. Likewise, send a copy of this Order to the National
o Q. What else did she tell you[?] Census and Statistics Office, Manila, for its appropriate action
o A. And she ask[ed] me if I remembered having issued consisten(t) herewith
the order. x Republic Æ directly appealed to SC via Rule 45
o Q. What was your response sir[?]
o A. I said I do not remember.40 ISSUE # 1: Whether the LAST name of the adoptee may be changed in the same petition for
x The answer "I do not remember" did not suggest that Judge Moya was unsure of adoption.
what he was declaring. In fact, he was emphatic and categorical in the
subsequent exchanges during the Deposition: HELD # 1: YES.
o Atty. Benito P. Fabie x It is the position of petitioner that respondent judge exceeded his jurisdiction
o Q. I am showing to you this Order, Exh. "A" deposition[;] when he additionally granted the prayer for the change of the given or proper
will you please recall whether you issued this Order and whether name of the adoptee in a petition for adoption.
the facsimile of the signature appearing thereon is your signature. x Petitioner argues that a petition for adoption and a petition for change of name
o A. As I said, I do not remember having issued such an are two special proceedings which, in substance and purpose, are different from
order and the signature reading Jose[;] I can’t make out clearly and are not related to each other, being respectively governed by distinct sets of
what comes after the name[;] Jose Moya is not my signature.41 law and rules. In order to be entitled to both reliefs, namely, a decree of
x Clearly, Judge Moya could not recall having ever issued the Order of adoption and an authority to change the giver or proper name of the adoptee,
Adoption. More importantly, when shown the signature over his name, the respective proceedings for each must be instituted separately, and the
he positively declared that it was not his. substantive and procedural requirements therefor under Articles 183 to 193 of the
x The fact that he had glaucoma when his Deposition was taken does not discredit Family Code in relation to Rule 99 of the Rules of Court for adoption, and Articles
his statements. At the time, he could with medication still read the newspapers; 364 to 380 of the Civil Code in relation to Rule 103 of the Rules of Court for
upon the request of the defense counsel, he even read a document shown to change of name, must correspondingly be complied with. 10
him.42 Indeed, we find no reason - and the respondent has not presented any - x A perusal of the records, according to petitioner, shows that only the laws and
to disregard the Deposition of Judge Moya. rules on adoption have been observed, but not those for a petition for change of
x Judge Moya's declaration was supported by the expert testimony of NBI name. 11 Petitioner further contends that what the law allows is the change of
Document Examiner Bienvenido Albacea, who declared: the surname of the adoptee, as a matter of right, to conform with that of the
o Atty. Paraiso adopter and as a natural consequence of the adoption thus granted. If what is
o Q And were you able to determine [w]hat purpose you had in sought is the change of the registered given or proper name, and since this
your examination of this document? would involve a substantial change of one's legal name, a petition for change of
o A Yes sir, [based on] my conclusion, [I] stated that the name under Rule 103 should accordingly be instituted, with the substantive and
questioned and the standard signature Jose L. Moya were not adjective requisites therefor being conformably satisfied. 12
written by one and the same person. On the basis of my findings x Private respondents, on the contrary, admittedly filed the petition for adoption
that I would point out in detail, the difference in the writing with a prayer for change of name predicated upon Section 5, Rule 2 which allows
characteristics [was] in the structural pattern of letters which is permissive joinder of causes of action in order to avoid multiplicity of suits and in
very apparent as shown in the photograph as the capital letter line with the policy of discouraging protracted and vexatious litigations. It is
"J".43 argued that there is no prohibition in the Rules against the joinder of adoption
x It is noteworthy that Mr. Albacea is a disinterested party, his services having and change of name being pleaded as two separate but related causes of action
been sought without any compensation. Moreover, his competence was in a single petition. Further, the conditions for permissive joinder of causes of
recognized even by Respondent Pilapil’s expert witness, Atty. Desiderio Pagui.44 action, i.e., jurisdiction of the court, proper venue and joinder of parties, have
x Other considerations also cast doubt on the claim of respondent. The alleged been met. 13
Order was purportedly made in open court. In his Deposition, however, Judge x Corollarily, petitioner insists on strict adherence to the rule regarding change of
Moya declared that he did not dictate decisions in adoption cases. The only name in view of the natural interest of the State in maintaining a system of
decisions he made in open court were criminal cases, in which the accused identification of its citizens and in the orderly administration of justice. 14 Private
pleaded guilty.45 Moreover, Judge Moya insisted that the branch where he was respondents argue otherwise and invoke a liberal construction and application of
assigned was always indicated in his decisions and orders; yet the questioned the Rules, the welfare and interest of the adoptee being the primordial concern
Order did not contain this information. Furthermore, Pilapil’s conduct gave no that should be addressed in the instant proceeding. 15
indication that he recognized his own alleged adoption, as shown by the x On this score, the trial court adopted a liberal stance in holding that -
documents that he signed and other acts that he performed thereafter.46 In the o Furthermore, the change of name of the child from Kevin
same vein, no proof was presented that Dr. Jacob had treated him as an adopted Earl Bartolome to Aaron Joseph should not be treated
child. Likewise, both the Bureau of Records Management47 in Manila and the strictly, it appearing that no rights have been prejudiced by
Office of the Local Civil Registrar of Tigaon, Camarines Sur,48 issued said change of name. The strict and meticulous observation
Certifications that there was no record that Pedro Pilapil had been adopted by of the requisites set forth by Rule 103 of the Rules of Court
128
RECTO, GAYLE ANGELI M.
2011-0008 | AUSL
Personal Notes on Remedial Law 2 Review (based on the syllabus of Prof. Henedino M. Brondial)

is indubitably for the purpose of preventing fraud, ensuring adoption to effect a change of name in the absence of the corresponding petition
that neither State nor any third person should be for the latter relief at law.
prejudiced by the grant of the petition for change of name x Neither can the allowance of the subject petition, by any stretch of imagination
under said rule, to a petitioner of discernment. and liberality, be justified under the rule allowing permissive joinder of causes of
x The first name sought to be changed belongs to an infant barely over a year old. action. Moreover, the reliance by private respondents on the pronouncements in
Kevin Earl has not exercised full civil rights nor engaged in any contractual Briz vs. Brit, et al. 21 and Peyer vs. Martinez, et al. 22 is misplaced. A
obligations. Neither can he nor petitioners on his behalf, be deemed to have any restatement of the rule and jurisprudence on joinder of causes of action would,
immoral, criminal or illicit purpose for seeking said cha(n)ge of name. It stands therefore, appear to be called for.
to reason that there is no way that the state or any person may be so x By a joinder of actions, or more properly, a joinder of causes of action, is meant
prejudiced by the action for change of Kevin Earl's first name. In fact, to the uniting of two or more demands or rights of action in one action; the
obviate any possible doubts on the intent of petitioners, the prayer for change of statement of more than one cause of action in a declaration. 23 It is the union of
name was caused to be published together with the petition for adoption. 16 two or more civil causes of action, each of which could be made the basis of a
x Art. 189 of the Family Code enumerates in no uncertain terms the legal effects of separate suit, in the same complaint, declaration or petition. A plaintiff may under
adoption: certain circumstances join several distinct demands, controversies or rights of
o (1) For civil purposes, the adopted shall be deemed to be action in one declaration, complaint or petition. 24
a legitimate child of the adopters and both shall acquire the x As can easily be inferred from the above definitions, a party is generally not
reciprocal rights and obligations arising from the relationship of required to join in one suit several distinct causes of action. The joinder of
parent and child, including the right of the adopted to use the separate causes of action, where allowable, is permissive and not mandatory in
surname of the adopters; the absence of a contrary statutory provision, even though the causes of action
o (2) The parental authority of the parents by nature over arose from the same factual setting and might under applicable joinder rules be
the adopted shall terminate and be vested in the adopters, except joined. 25 Modern statutes and rules governing joinders are intended to avoid a
that if the adopter is the spouse of the parent by nature of the multiplicity of suits and to promote the efficient administration of justice
adopted, parental authority over the adopted shall be exercised wherever this may be done without prejudice to the rights of the litigants. To
jointly by both spouses; and achieve these ends, they are liberally construed. 26
o (3) The adopted shall remain an intestate heir of his x While joinder of causes of action is largely left to the option of a party
parents and other blood relatives. litigant, Section 5, Rule 2 of our present Rules allows causes of action
x Clearly, the law allows the adoptee, as a matter of right and obligation, to be joined in one complaint conditioned upon the following
to bear the surname of the adopter, upon issuance of the decree of requisites: (a) it will not violate the rules on jurisdiction, venue and
adoption. It is the change of the adoptee's surname to follow that of joinder of parties; and (b) the causes of action arise out of the same
the adopter which is the natural and necessary consequence of a grant contract, transaction or relation between the parties, or are for
of adoption and must specifically be contained in the order of the court, demands for money or are of the same nature and character. x The
in fact, even if not prayed for by petitioner. objectives of the rule or provision are to avoid a multiplicity of suits where
the same parties and subject matter are to be dealt with by effecting in one
ISSUE # 2: Whether a petition to change the FIRST name of the prospective adoptee and a action a complete determination of all matters in controversy and litigation
petition for adoption may correctly be joined. between the parties involving one subject matter, and to expedite the disposition
of litigation at minimum cost. The provision should be construed so as to avoid
HELD # 2: NO. such multiplicity, where possible, without prejudice to the rights of the litigants.
x However, the given or proper name, also known as the first or Christian name, of Being of a remedial nature, the provision should be liberally construed, to the
the adoptee must remain as it was originally registered in the civil register. The end that related controversies between the same parties may be adjudicated at
creation of an adoptive relationship does not confer upon the adopter a one time; and it should be made effectual as far as practicable, 27 with the end
license to change the adoptee's registered Christian or first name. The in view of promoting the efficient administration of justice. 28
automatic change thereof, premised solely upon the adoption thus x The statutory intent behind the provisions on joinder of causes of action is to
granted, is beyond the purview of a decree of adoption. Neither is it a encourage joinder of actions which could reasonably be said to involve kindred
mere incident in nor an adjunct of an adoption proceeding, such that a rights and wrongs, although the courts have not succeeded in giving a standard
prayer therefor furtively inserted in a petition for adoption, as in this definition of the terms used or in developing a rule of universal application. The
case, cannot properly be granted. dominant idea is to permit joinder of causes of action, legal or equitable, where
x The name of the adoptee as recorded in the civil register should be used in the there is some substantial unity between them. 29 While the rule allows a plaintiff
adoption proceedings in order to vest the court with jurisdiction to hear and to join as many separate claims as he may have, there should nevertheless be
determine the same, 17 and shall continue to be so used until the court orders some unity in the problem presented and a common question of law and fact
otherwise. Changing the given or proper name of a person as recorded in the involved, subject always to the restriction thereon regarding jurisdiction, venue
civil register is a substantial change in one's official or legal name and cannot be and joinder of parties. Unlimited joinder is not authorized. 30
authorized without a judicial order. The purpose of the statutory procedure x Our rule on permissive joinder of causes of action, with the proviso subjecting it
authorizing a change of name is simply to have, wherever possible, a record of to the correlative rules on jurisdiction, venue and joinder of parties 31 and
the change, and in keeping with the object of the statute, a court to which the requiring a conceptual unity in the problems presented, effectively disallows
application is made should normally make its decree recording such change. 18 unlimited joinder. 32
x The official name of a person whose birth is registered in the civil register is the x Turning now to the present petition, while it is true that there is no express
name appearing therein. If a change in one's name is desired, this can only be prohibition against the joinder of a petition for adoption and for change of name,
done by filing and strictly complying with the substantive and procedural we do not believe that there is any relation between these two petitions, nor are
requirements for a special proceeding for change of name under Rule 103 of the they of the same nature or character, much less do they present any common
Rules of Court, wherein the sufficiency of the reasons or grounds therefor can be question of fact or law, which conjointly would warrant their joinder. In short,
threshed out and accordingly determined. these petitions do not rightly meet the underlying test of conceptual unity
x Under Rule 103, a petition for change of name shall be filed in the regional trial demanded to sanction their joinder under our Rules.
court of the province where the person desiring to change his name resides. It x As keenly observed and correctly pointed out by the Solicitor General -
shall be signed and verified by the person desiring his name to be changed or by o A petition for adoption and a petition for change of name
some other person in his behalf and shall state that the petitioner has been a are two special proceedings which, in substance and
bona fide resident of the province where the petition is filed for at least three purpose, are different from each other. Each action is
years prior to such filing, the cause for which the change of name is sought, and individually governed by particular sets of laws and rules.
the name asked for. An order for the date and place of hearing shall be made These two proceedings involve disparate issues. In a
and published, with the Solicitor General or the proper provincial or city petition for adoption, the court is called upon to evaluate
prosecutor appearing for the Government at such hearing. It is only upon the proposed adopter's fitness and qualifications to bring
satisfactory proof of the veracity of the allegations in the petition and the up and educate the adoptee properly (Prasnick vs. Republic,
reasonableness of the causes for the change of name that the court may 99 Phil. 665). On the other hand, in a petition for change of
adjudge that the name be changed as prayed for in the petition, and shall name, no family relations are created or affected for what
furnish a copy of said judgment to the civil registrar of the municipality is looked into is the propriety and reasonableness of the
concerned who shall forthwith enter the same in the civil register. grounds supporting the proposed change of name (Yu vs.
x A petition for change of name being a proceeding in rem, strict Republic, 17 SCRA 253).
compliance with all the requirements therefor is indispensable in order o . . . Hence, the individual merits of each issue must be separately
to vest the court with jurisdiction for its adjudication. 19 It is an assessed and determined for neither action is dependent on the
independent and discrete special proceeding, in and by itself, governed other. 33
by its own set of rules. A fortiori, it cannot be granted by means of any x The rule on permissive joinder of: causes of action is clear. Joinder may
other proceeding. To consider it as a mere incident or an offshoot of be allowed only if the actions show a commonality of relationship and
another special proceeding would be to denigrate its role and conform to the rules on jurisdiction, venue and joinder of parties
significance as the appropriate remedy available under our remedial (Section 5, Rule 2, Rules of Court).
law system. x These conditions are wanting in the instant case. As already pointed
x The Solicitor General correctly points out the glaring defects of the subject out in our Petition (pp. 9-10), an action for adoption and an action for
petition insofar as it seeks the change of name of the adoptee, 20 all of which change of name are, in nature and purpose, not related to each other
taken together cannot but lead to the conclusion that there was no petition and do not arise out of the same relation between the parties. While
sufficient in form and substance for change of name as would rightfully deserve what is cogent in an adoption proceeding is the proposed adopter's fitness and
an order therefor. It would be procedurally erroneous to employ a petition for qualifications to adopt, a petition for change of first name may only prosper upon
proof of reasonable and compelling grounds supporting the change requested.
129
RECTO, GAYLE ANGELI M.
2011-0008 | AUSL
Personal Notes on Remedial Law 2 Review (based on the syllabus of Prof. Henedino M. Brondial)

Fitness to adopt is not determinative of the sufficiency of reasons justifying a rights, be it the Constitution itself or only a statute or a rule of
change of name. And similarly, a change of first name cannot be justified in view court.
of a finding that the proposed adopter was found fit to adopt. There is just no way o . . . (T)hey are required to be followed except only when for the
that the two actions can connect and find a common ground, thus the most persuasive of reasons they may be relaxed to relieve a litigant
joinder would be improper. of an injustice not commensurate with the degree of his
x In contending that adoption and change of name may be similarly sought in one thoughtlessness in not complying with the procedure prescribed .....
petition, private respondents rely upon Peyer vs. Martinez and Briz vs. Briz (p. 4, . While it is true that a litigation is not a game of technicalities, this
Comment) does not mean that the Rules of Court may be ignored at will and
x We however submit that these citations are non sequitur. In both cases, the fact at random to the prejudice of the orderly presentation and
of intimacy and relatedness of the issues is so pronounced. In Peyer, an assessment of the issues and their just resolution. Justice eschews
application to pronounce the husband an absentee is obviously intertwined with anarchy.
the action to transfer the management of conjugal assets to the wife. In Briz, an o Only exceptionally in very extreme circumstances, when a rule
action for declaration of heirship was deemed a clear condition precedent to an deserts its proper office as an aid to justice and becomes its great
action to recover the land subject of partition and distribution proceeding. hindrance and chief enemy such that rigid application thereof
However, the commonality of relationship which stands out in both cases does frustrates rather than promotes substantial justice, will
not characterize the present action for adoption and change of name. Thus the technicalities deserve scant consideration from the court. In such
rulings in Peyer and Briz find no place in the case at bar. situations, the courts are empowered, even obligated, to suspend
x Besides, it is interesting to note that although a joinder of the two actions was, the operation of the rules. 41
in Briz, declared feasible, the Supreme Court did not indorse an automatic x We do not perceive any injustice that can possibly be visited upon private
joinder and instead remanded the matter for further proceedings, granting leave respondents by following the reglementary procedure for the change in the
to amend the pleadings and implead additional parties-defendants for a complete proper or given name that they seek for their adopted child. We are hard put to
determination of the controversy (Briz vs. Briz, 43 Phil. 763, 770). Such descry the indispensability of a change of the first name of the adoptee to his
cautionary stance all the more emphasizes that although joinders are generally welfare and benefit. Nor is the said change of such urgency that would justify an
accepted, they are not allowed where the conditions are not satisfactorily met. exemption from or a relaxation of the Rules. It is the State that stands to be
34 prejudiced by a wanton disregard of Rule 103 in this case, considering its natural
x It furthermore cannot be said that the proposed joinder in this instance will make interest in the methodical administration of justice and in the efficacious
for a complete determination of all matters pertaining to the coetaneous grant of maintenance of a system of identification of its citizens.
adoption and change of name of the adoptee in one petition. As already stated, x The danger wrought by non-observance of the Rules is that the violation of or
the subject petition was grossly insufficient in form and substance with respect to failure to comply with the procedure prescribed by law prevents the proper
the prayer for change of name of the adoptee. The policy of avoiding determination of the questions raised by the parties with respect to the merits of
multiplicity of suits which underscores the rule on permissive joinder of causes of the case and makes it necessary to decide, in the first place, such questions as
action is addressed to suits that are intimately related and also present relate to the form of the action. The rules and procedure laid down for the trial
interwoven and dependent issues which can be most expeditiously and court and the adjudication of cases are matters of public policy. 42 They are
comprehensively settled by having just one judicial proceeding, but not to suits matters of public order and interest which can in no wise be changed or
or actions whose subject matters or corresponding reliefs are unrelated or regulated by agreements between or stipulations by parties to an action for their
diverse such that they are best taken up individually. singular convenience. 43
x In Nabus vs. Court of Appeals, et al., 35 the Court clarified the rule on x In Garcia vs. Republic, 44 we are reminded of the definiteness in the application
permissive joinder of causes of action: of the Rules and the importance of seeking relief under the appropriate
o The rule is clearly permissive. It does not constitute an obligatory proceeding:
rule, as there is no positive provision of law or any rule of o . . . The procedure set by law should be delimited. One should not
jurisprudence which compels a party to join all his causes of action confuse or misapply one procedure for another lest we create
and bring them at one and the same time. Under the present rules, confusion in the application of the proper remedy.
the provision is still that the plaintiff may, and not that he must, x Respondent judge's unmindful disregard of procedural tenets aimed at achieving
unite several causes of action although they may be included in stability of procedure is to be deplored. He exceeded his prerogatives by granting
one of the classes specified. This, therefore, leaves it to the the prayer for change of name, his order being unsupported by both statutory
plaintiff's option whether the causes of action shall be joined in the and case law. The novel but unwarranted manner in which he adjudicated this
same action, and no unfavorable inference may be drawn from his case may be characterized as a regrettable abdication of the duty to uphold the
failure or refusal to do so. He may always file another action based teachings of remedial law and jurisprudence.
on the remaining cause or causes of action within the prescriptive
period therefor. (Emphasis supplied.) ISSUE # 3: Whether there is ground to change the FIRST name of the adoptee.
x The situation presented in this case does not warrant exception from the Rules
under the policy of liberal construction thereof in general, and for change of HELD # 3: NO.
name in particular, as proposed by private respondents and adopted by x Petitioner avers that it was error for the lower court to grant the petition for
respondent judge. Liberal construction of the Rules may be invoked in situations change of name without citing or proving any lawful ground. Indeed, the only
wherein there may be some excusable formal deficiency or error in a pleading, justification advanced for the change of name was the fact of the adoptee's
provided that the same does not subvert the essence of the proceeding and baptism under the name Aaron Joseph and by which he has been known since
connotes at least a reasonable attempt at compliance with the Rules. Utter he came to live with private respondents. 45
disregard of the Rules cannot justly be rationalized by harking on the policy of x Private respondents, through a rather stilted ratiocination, assert that upon the
liberal construction. grant of adoption, the subject minor adoptee ipso facto assumed a new
x The Court is not impervious to the frustration that litigants and lawyers alike identification and designation, that is, Aaron Joseph which was the name given
would at times encounter in procedural bureaucracy but imperative justice to him during the baptismal rites. Allowing the change of his first name as
requires correct observance of indispensable technicalities precisely designed to prayed for in the petition, so they claim, merely confirms the designation by
ensure its proper dispensation. 36 It has long been recognized that strict which he is known and called in the community in which he lives. This largely
compliance with the Rules of Court is indispensable for the prevention of echoes the opinion of the lower court that naming the child Aaron Joseph was
needless delays and for the orderly and expeditious dispatch of judicial business. symbolic of naming him at birth, and that they, as adoptive parents, have as
37 much right as the natural parents to freely select the first name of their adopted
x Procedural rules are not to be disdained as mere technicalities that may be child. 46
ignored at will to suit the convenience of a party. Adjective law is important in x The lower court was sympathetic to herein private respondents and ruled on this
ensuring the effective enforcement of substantive rights through the orderly and point in this manner:
speedy administration of justice. These rules are not intended to hamper litigants o As adoptive parents, petitioner like other parents may freely select
or complicate litigation but, indeed to provide for a system under which a suitor the first name given to his/her child as it is only the surname to
may be heard in the correct form and manner and at the prescribed time in a which the child is entitled that is fixed by law .......
peaceful confrontation before a judge whose authority they acknowledge. 38 o The given name of the minor was Kevin Earl, a name given for no
x It cannot be overemphasized that procedural rules have their own wholesome other purpose than for identification purposes in a birth certificate
rationale in the orderly administration of justice. Justice has to be administered by a woman who had all intentions of giving him away. The naming
according to the Rules in order to obviate arbitrariness, caprice, or whimsicality. of the minor as Aaron Joseph by petitioners upon the grant of their
39 We have been cautioned and reminded in Limpot vs. CA, et al. that: 40 petition for adoption is symbolic of naming the minor at birth. 47
o Rules of procedure are intended to ensure the orderly x We cannot fathom any legal or jurisprudential basis for this attenuated ruling of
administration of justice and the protection of substantive rights in respondent judge and must thus set it aside.
judicial and extrajudicial proceedings. It is a mistake to propose x It is necessary to reiterate in this discussion that a person's name is a word or
that substantive law and adjective law are contradictory to each combination of words by which he is known and identified, and distinguished
other or, as has often been suggested, that enforcement of from others, for the convenience of the world at large in addressing him, or in
procedural rules should never be permitted if it will result in speaking of or dealing with him. It is both of personal as well as public interest
prejudice to the substantive rights of the litigants. This is not that every person must have a name. The name of an individual has two parts:
exactly true; the concept is much misunderstood. As a matter of the given or proper name and the surname or family name. The giver or proper
fact, the policy of the courts is to give both kinds of law, as name is that which is given to the individual at birth or at baptism, to distinguish
complementing each other, in the just and speedy resolution of the him from other individuals. The surname or family name is that which identifies
dispute between the parties. Observance of both substantive rights the family to which he belongs and is continued from parent to child. The given
is equally guaranteed by due process, whatever the source of such name may be freely selected by the parents for the child, but the surname to
which the child is entitled is fixed by law. 48
130
RECTO, GAYLE ANGELI M.
2011-0008 | AUSL
Personal Notes on Remedial Law 2 Review (based on the syllabus of Prof. Henedino M. Brondial)

x By Article 408 of the Civil Code, a person's birth must be entered in the civil x It should be noted that in said case the change of surname, not the given name,
register. The official name of a person is that given him in the civil register. That and the legal consequences thereof in view of the adoption were at issue. That it
is his name in the eyes of the law. 49 And once the name of a person is officially was sought in a petition duly and precisely filed for that purpose with ample
entered in the civil register, Article 376 of the same Code seals that identity with proof of the lawful grounds therefor only serves to reinforce the imperative
its precise mandate: no person can change his name or surname without judicial necessity of seeking relief under and through the legally prescribed procedures.
authority. This statutory restriction is premised on the interest of the State in x Here, the Solicitor General meritoriously explained that:
names borne by individuals and entities for purposes of identification. 50 o Respondent Judge failed to distinguish between a situation wherein
x By reason thereof, the only way that the name of person can be changed legally a child is being named for the first time by his natural parent, as
is through a petition for change of name under Rule 103 of the Rules of Court. against one wherein, a child is previously conferred a first name by
51 For purposes of an application for change of name under Article 376 of the his natural parent, and such name is subsequently sought to be
Civil Code and correlatively implemented by Rule 103, the only name that may disregarded and changed by the adoptive parents. In the first case,
be changed is the true or official name recorded in the civil register. As earlier there is no dispute that natural parents have the right to freely
mentioned, a petition for change of name being a proceeding in rem, impressed select and give the child's first name for every person, including
as it is with public interest, strict compliance with all the requisites therefor in juridical persons, must have a name (Tolentino, A., Commentaries
order to vest the court with jurisdiction is essential, and failure therein renders and Jurisprudence on the Civil Code, Vo. I, 1987 edition, page
the proceedings a nullity. 52 721). In the second case, however, as in the case at bar, private
x It must likewise be stressed once again that a change of name is a privilege, not respondents, in their capacities as adopters, cannot claim a right to
a matter of right, addressed to the sound discretion of the court which has the name the minor adoptee after such right to name the child had
duty to consider carefully the consequences of a change of name and to deny already been exercised by the natural parent. Adopting parents
the same unless weighty reasons are shown. Before a person can be authorized have not been conferred such right by law, hence, the right
to change his name, that is, his true or official name or that which appears in his assertes by private respondents herein remains but illusory.
birth certificate or is entered in the civil register, he must show proper and Renaming the adoptee cannot be claimed as a right. It is merely a
reasonable cause or any convincing reason which may justify such change. 53 privilege necessitating judicial consent upon compelling grounds.
x Jurisprudence has recognized, inter alia, the following grounds as being sufficient 61
to warrant a change of name: x The liberality with which this Court treats matters leading up to adoption insofar
o (a) when the name is ridiculous, dishonorable or extremely difficult as it carries out the beneficent purposes of adoption and ensures to the adopted
to write or pronounce; child the rights and privileges arising therefrom, ever mindful that the paramount
o (b) when the change results as a legal consequence of legitimation consideration is the overall benefit and interest of the adopted child, 62 should be
or adoption; understood in its proper context. It should not be misconstrued or
o (c) when the change will avoid confusion; misinterpreted to extend to inferences beyond the contemplation of law and
o (d) when one has continuously used and been known since jurisprudence.
childhood by a Filipino name and was unaware of alien parentage; x The practically unrestricted freedom of the natural parent to select the proper or
o (e) when the change is based on a sincere desire to adopt a Filipino given name of the child presupposes that no other name for it has theretofore
name to erase signs of former alienage, all in good faith and been entered in the civil register. Once such name is registered, regardless of
without prejudice to anybody; and the reasons for such choice and even if it be solely for the purpose of
o (f) when the surname causes embarrassment and there is no identification, the same constitutes the official name. This effectively
showing that the desired change of name was for a fraudulent authenticates the identity of the person and must remain unaltered save when,
purpose or that the change of name would prejudice public for the most compelling reasons shown in an appropriate proceeding, its change
interest. 54 may merit judicial approval.
x Contrarily, a petition for change of name grounded on the fact that one was x While the right of a natural parent to name the child is recognized, guaranteed
baptized by another name, under which he has been known and which he used, and protected under the law, the so-called right of an adoptive parent to re-
has been denied inasmuch as the use of baptismal names is not sanctioned. 55 name an adopted child by virtue or as a consequence of adoption, even for the
For, in truth, baptism is not a condition sine qua non to a change of name. 56 most noble intentions and moving supplications, is unheard of in law and
Neither does the fact that the petitioner has been using a different name and has consequently cannot be favorably considered. To repeat, the change of the
become known by it constitute proper and reasonable cause to legally authorize a surname of the adoptee as a result of the adoption and to follow that of the
change of name. 57 A name given to a person in the church records or adopter does not lawfully extend to or include the proper or given name.
elsewhere or by which be is known in the community - when at variance with Furthermore, factual realities and legal consequences, rather than sentimentality
that entered in the civil register - is unofficial and cannot be recognized as his and symbolisms, are what are of concern to the Court.
real name. 58 x Finally, it is understood that this decision does not entirely foreclose and is
x The instant petition does not sufficiently persuade us to depart from such rulings without prejudice to, private respondents' privilege to legally change the proper
of long accepted wisdom and applicability. The only grounds offered to justify or given name of their adopted child, provided that the same is exercised, this
the change of name prayed for was that the adopted child had been baptized as time, via a proper petition for change of name. Of course, the grant thereof is
Aaron Joseph in keeping with the religious faith of private respondents and that conditioned on strict compliance with all jurisdictional requirements and
it was the name by which he had been called and known by his family, relatives satisfactory proof of the compelling reasons advanced therefor.
and friends from, the time he came to live with private respondents. 59 Apart
from suffusing their pleadings with sanctimonious entreaties for compassion, REPUBLIC OF THE PHILIPPINES, petitioner, vs. THE COURT OF APPEALS, JAIME B.
none of the justified grounds for a change of name has been alleged or CARANTO, and ZENAIDA P. CARANTO, respondents.
established by private respondents. The legal bases chosen by them to bolster G.R. No. 103695. March 15, 1996
their cause have long been struck down as unavailing for their present purposes. SECOND DIVISION
For, to allow the adoptee herein to use his baptismal name, instead of his name
registered in the civil register, would be to countenance or permit that which has FACTS:
always been frowned upon. 60 x Carantos Æ filed a petition to adopt Midael C. Mazon with prayer for the
x The earlier quoted posturing of respondent judge, as expressed in his assailed correction of the minors first name Midael to Michael before RTC Cavite
order that - o Michael was then fifteen years old, who had been living with
o (a)s adoptive parents, petitioners like other parents may freely private respondent Jaime B. Caranto since he was seven years old
select the first name given to his/her child as it is only the surname x SolGen Æ filed an opposition
to which the child is entitled that is fixed by law ...... o insofar as it sought the correction of the name of the child from
ƒ The given name of the minor was Kevin Earl, a name Midael to Michael.
given for no other purpose than for identification o He argued that although the correction sought concerned only a
purposes in a birth certificate by a woman who had clerical and innocuous error, it could not be granted because the
all the intentions of giving him away. The naming of petition was basically for adoption, not the correction of an entry in
the minor as Aaron Joseph by petitioners upon grant the civil registry under Rule 108 of the Rules of Court.
of their petition for adoption is symbolic of naming x RTC Æ ruled IN FAVOR of the CARANTOS
the minor at birth. o dismissed the opposition of the Solicitor General. on the ground
o and supposedly based on the authority of Republic vs. Court of that Rule 108 of the Rules of Court (Cancellation or Correction of
Appeals and Maximo Wong, supra, painfully misapplies the ruling Entries in the Civil Registry) applies only to the correction of entries
therein enunciated. concerning the civil status of persons.
x The factual backdrop of said case is not at all analogous to that of the case at o It cited Rule 108 which provides that any person interested in an
bar. In the Wong case, therein petitioner Maximo Wong sought the change of his act, event, order or decree concerning the civil status of the
surname which he acquired by virtue of the decree of adoption granted in favor persons which has been recorded in the civil register, may file a
of spouses Hoong Wong and Concepcion Ty Wong. Upon reaching the age of verified petition for the cancellation or correction of any entry
majority, he filed a petition in court to change his surname from Wong to Alcala, relating thereto.
which was his surname prior to the adoption. He adduced proof that the use of o It held that the correction of names in the civil registry is not one
the surname Wong caused him embarrassment and isolation from friends and of the matters enumerated in Rule 108, 2 as entries subject to
relatives in view of a suggested Chinese ancestry when in reality he is a Muslim cancellation or correction.
Filipino residing in a Muslim community, thereby hampering his business and o According to the trial court, the error could be corrected in the
social life, and that his surviving adoptive mother consented to the change of same proceeding for adoption to prevent multiplicity of actions, and
name sought. This Court granted the petition and regarded the change of the inconvenience to the petitioners.
surname as a mere incident in, rather than the object of, the adoption. x SolGen Æ appealed to CA
x CA Æ affirmed RTC in toto
131
RECTO, GAYLE ANGELI M.
2011-0008 | AUSL
Personal Notes on Remedial Law 2 Review (based on the syllabus of Prof. Henedino M. Brondial)

o ruled that the case of Cruz v. Republic,[2] invoked by the petitioner x Nor was notice of the petition for correction of entry published as required by
in support of its plea that the trial court did not acquire jurisdiction Rule 108, 4 which reads:
over the case, was inapplicable because that case involved a o 4. Notice and publication. - Upon filling of the petition, the court
substantial error. shall, by an order, fix the time and place for the hearing of the
o Like the trial court, it held that to require the petitioners to file a same, and cause reasonable notice thereof to be given to the
separate petition for correction of name would entail additional persons named in the petition. The court shall also cause the order
time and expenses for them as. well as for the Government and the to be published once a week for three (3) consecutive weeks in a
Courts. newspaper of general circulation in the province.
x SolGen Æ filed a Rule 45 before the SC x While there was notice given by publication in this case, it was notice of the
petition for adoption made in compliance with Rule 99, 4. In that notice only the
ISSUE # 1: Whether the RTC has acquired jurisdiction over the subject matter of the case, prayer for adoption of the minor was stated. Nothing was mentioned that in
considering that the subject petition for adoption includes prayer to change the first name of addition the correction of his name in the civil registry was also being sought.
the prospective adoptee. The local civil registrar was thus deprived of notice and, consequently, of the
opportunity to be heard.
HELD #1: YES. x The necessary consequence of the failure to implead the civil registrar as an
x The first issue is whether on the facts stated, the RTC acquired jurisdiction over indispensable party and to give notice by publication of the petition for correction
the private respondents petition for adoption. Petitioners contention is that the of entry was to render the proceeding of the trial court, so far as the correction of
trial court did not acquire jurisdiction over the petition for adoption because the entry was concered, null and void for lack of jurisdiction both as to party and as to
notice by publication did not state the true name of the minor child. Petitioner the subject matter
invokes the ruling in Cruz v. Republic.[3] There the petition for adoption and the
notice published in the newspaper gave the baptismal name of the child -
(Rosanna E. Cruz) instead of her name in the record of birth (Rosanna E. Bucoy). EUGENIO R. REYES, joined by TIMOTHY JOSEPH M. REYES, MA. GRACIA S. REYES, ROMAN
it was held that this was a substantial defect in the petition and the published GABRIEL M. REYES, and MA. ANGELA S. REYES, Petitioners, vs. LIBRADA F. MAURICIO
order of hearing. Indeed there was a question of identity involved in that case. (deceased) and LEONIDA F. MAURICIO, Respondents.
Rosanna E. Cruz could very well be a different person from Rosanna E. Bucoy, as G.R. No. 175080 November 24, 2010
common experience would indicate. FIRST DIVISION
x The present case is different. It involves an obvious clerical error in the
name of the child sought to be adopted. In this case the correction FACTS:
involves merely the substitution of the letters ch for the letter d, so x Eugenio is the registered owner of the subject property located in Bulacan, which
that what appears as Midael as given name would read Michael. Even has been adjudicated to him by virtue of an extrajudicial settlement among the
the Solicitor General admits that the error is a plainly clerical one. heirs following the death of his parents.
Changing the name of the child from Midael C. Mazon to Michael C Mazon cannot x Mauricios Æ filed a complaint before the DARAB
possibly cause any confusion, because both names can be read and pronounced o alleging that they were the legal heirs of one Godofredo Mauricio,
with the same rhyme (tugma) and tone (tono, tunog, himig). The purpose of the the lawful and registered tenant of Eugenio through his
publication requirement is to give notice so that those who have any objection to predecessors-in-interest to the subject land
the adoption can make their objection known. That purpose has been served by o Contended that Eugenio caused the preparation of a document
publication of notice in this case. denominated as Kasunduan dated 28 September 1994 to eject
x For this reason we hold that the RTC correctly granted the petition for adoption respondents from the subject property but such Kasunduan was
of the minor Midael C. Mazon and the Court of Appeals, in affirming the decision void, considering that Librada was illiterate and said Kasunduan
of the trial court, correctly did so. was neither read nor explained to her
x Eugenio Æ filed his answer
ISSUE # 2: Whether the RTC correctly granted the prayer to change the name of said o contended, among others, that no tenancy relationship existed
prospective adoptee. between him and respondents and that Leonida had no legal
personality to file the present suit.
HELD #2: NO. x Provincial Adjudicator Æ in favor of the MAURICIOS
x With regard to the second assignment of error in the petition, we hold that both o concluded that Godofredo was the tenant of Eugenio, and Librada,
the Court of Appeals and the trial court erred in granting private respondents being the surviving spouse, should be maintained in peaceful
prayer for the correction of the name of the child in the civil registry. possession of the subject land.
x Contrary to what the trial court thought, Rule 108 of the Rules of Court applies x DARAB Æ in favor of the MAURICIOS
to this case and because its provision was not complied with, the decision of the o banked on the Kasunduang Buwisan sa Sakahan or the leasehold
trial court, insofar as it ordered the correction of the name of the minor, is void contract executed by Susana in favor of Godofredo to support the
and without force or effect. tenancy relationship. Furthermore, the DARAB declared the other
x The trial court was clearly in error in holding Rule 108 to be applicable only to Kasunduan as void by relying on the evaluation of the Provincial
the correction of errors concerning the civil status of persons. Rule 108, 2 plainly Adjudicator as to the legal incapacity of Librada to enter into such a
states: contract.
o 2. Entries subject to cancellation or correction. - Upon good and x Eugenio Æ filed an MR
valid grounds, the following entries in the civil register may be o DARAB Æ denied
cancelled or corrected: x Eugenio Æ appealed to CA
ƒ (a) births; (b) marriages; (c) deaths; (d) legal o CA Æ affirmed DARAB
separation; (e) judgments of annulments of marriage; x Eugenio Æ filed a Rule 45 before the SC
(f) judgments declaring marriages void from the o contended that Leonida is a mere ward of Godofredo and
beginning; (g) legitimations; (h) adoptions; (i) Librada, thus, not a legal heir
acknowledgments of natural children; (j)
naturalization; (k) election, loss or recovery of ISSUE: Whether Eugenio correctly questioned Leonida’s status as an adoptive child of
citizenship; (l) civil interdiction; (m) judicial Godofredo in the present Rule 45 petition that stemmed from a proceeding on tenancy before
determination of filiation; (n) voluntary emancipation the DARAB.
of a minor; and (o) changes of name.
x This case falls under letter (o), referring to changes of name. Indeed, it has been HELD: NO.
the uniform ruling of this Court that Art. 412 of the Civil Code - to implement x It is settled law that filiation cannot be collaterally attacked.20 Well-
which Rule 108 was inserted in the rules of Court in 1964 - covers those known civilista Dr. Arturo M. Tolentino, in his book "Civil Code of the Philippines,
harmless and innocuous changes, such as correction of a name that is clearly Commentaries and Jurisprudence," noted that the aforecited doctrine is rooted
misspelled.[4] Thus, in Yu vs. Republic[5] it was held that to change Sincio to from the provisions of the Civil Code of the Philippines. He explained thus:
Sencio which merely involves the substitution of the first vowel i in the first name o The legitimacy of the child cannot be contested by way of defense
into the vowel e amounts merely to the righting of a clerical error. In Labayo- or as a collateral issue in another action for a different purpose.
Rowe v. Republic[6] it was held that the change of petitioners name from Beatriz The necessity of an independent action directly impugning the
Labayo/Beatriz Labayu to Emperatriz Labayo is a mere innocuous alteration legitimacy is more clearly expressed in the Mexican code (article
wherein a summary proceeding is appropriate. 335) which provides: "The contest of the legitimacy of a child by
x Rule 108 thus applies to the present proceeding. Now 3 of this Rule provides: the husband or his heirs must be made by proper complaint before
o 3. Parties. - When cancellation or correction of an entry in the civil the competent court; any contest made in any other way is void."
register is sought, the civil registrar and all persons who have or This principle applies under our Family Code. Articles 170 and 171
claim any interest which would be affected thereby shall be made of the code confirm this view, because they refer to "the action to
parties to the proceeding. impugn the legitimacy." This action can be brought only by the
x The local civil registrar is thus required to be made a party to the husband or his heirs and within the periods fixed in the present
proceeding. He is an indispensable party, without whom no final articles.21
determination of the case can be had.[7] As he was not impleaded in this x In Braza v. City Civil Registrar of Himamaylan City, Negros Occidental,22 the
case much less given notice of the proceeding, the decision of the trial court, Court stated that legitimacy and filiation can be questioned only in a
insofar as it granted the prayer for the correction of entry, is void. The absence direct action seasonably filed by the proper party, and not through
of an indenpensable party in a case renders ineffectual all the proceeding collateral attack.23
subsequent to the filling of the complaint including the judgment.[8] x The same rule is applied to adoption such that it cannot also be made
subject to a collateral attack. In Reyes v. Sotero,24 this Court reiterated that
132
RECTO, GAYLE ANGELI M.
2011-0008 | AUSL
Personal Notes on Remedial Law 2 Review (based on the syllabus of Prof. Henedino M. Brondial)

adoption cannot be assailed collaterally in a proceeding for the settlement of a o "Art. 364. Legitimate and legitimated children shall principally use
decedent’s estate.25 Furthermore, in Austria v. Reyes,26 the Court declared that the surname of the father.
the legality of the adoption by the testatrix can be assailed only in a separate o Art. 365. An adopted child shall bear the surname of the adopter.
action brought for that purpose and cannot be subject to collateral attack.27 o Art. 369. Children conceived before the decree annulling a voidable
x Against these jurisprudential backdrop, we have to leave out the status of marriage shall principally use the surname of the father.
Leonida from the case for annulment of the "Kasunduan" that supposedly favors o Art. 370. A married woman may use:
petitioners’ cause. ƒ (1) Her maiden first name and surname and add her
husband's surname, or
IN THE MATTER OF THE ADOPTION OF STEPHANIE NATHY ASTORGA GARCIA ƒ (2) Her maiden first name and her husband's
HONORATO B. CATINDIG, petitioner. surname or
G.R. No. 148311. March 31, 2005 ƒ (3) Her husband's full name, but prefixing a word
THIRD DIVISION indicating that she is his wife, such as ‘Mrs.’
o Art. 371. In case of annulment of marriage, and the wife is the
FACTS: guilty party, she shall resume her maiden name and surname. If
x Honorato Æ filed a petition to adopt his minor illegitimate child Stephanie Nathy she is the innocent spouse, she may resume her maiden name and
Astorga Garcia; the allegations are: surname. However, she may choose to continue employing her
o Stephanie was born on June 26, 1994;2 former husband's surname, unless:
o that her mother is Gemma Astorga Garcia; ƒ (1) The court decrees otherwise, or
o that Stephanie has been using her mother’s middle name and ƒ (2) She or the former husband is married again to
surname; and another person.
o that he is now a widower and qualified to be her adopting parent. o Art. 372. When legal separation has been granted, the wife shall
He prayed that Stephanie’s middle name Astorga be changed to continue using her name and surname employed before the legal
"Garcia," her mother’s surname, and that her surname "Garcia" be separation.
changed to "Catindig," his surname o Art. 373. A widow may use the deceased husband's surname as
x RTC Æ granted the petition though he were still living, in accordance with Article 370.
o Pursuant to Article 189 of the Family Code of the Philippines, the o Art. 374. In case of identity of names and surnames, the younger
minor shall be known as STEPHANIE NATHY CATINDIG. person shall be obliged to use such additional name or surname as
x Honorato Æ moved for clarification and/or reconsideration will avoid confusion.
o praying that Stephanie should be allowed to use the surname of o Art. 375. In case of identity of names and surnames between
her natural mother (GARCIA) as her middle name ascendants and descendants, the word ‘Junior’ can be used only by
x RTC Æ denied a son. Grandsons and other direct male descendants shall either:
x Honorato Æ filed directly a Rule 45 before the SC ƒ (1) Add a middle name or the mother's surname,
ƒ (2) Add the Roman numerals II, III, and so on.
ISSUE: Whether an illegitimate child, upon adoption by her natural father, use the surname of o Law Is Silent As To The Use Of Middle Name -
her natural mother as her middle name. x As correctly submitted by both parties, there is no law regulating the
use of a middle name. Even Article 176 of the Family Code, as amended by
HELD: YES. Republic Act No. 9255, otherwise known as "An Act Allowing Illegitimate Children
x Petitioner submits that the trial court erred in depriving Stephanie of a middle To Use The Surname Of Their Father," is silent as to what middle name a child
name as a consequence of adoption because: (1) there is no law prohibiting an may use.
adopted child from having a middle name in case there is only one adopting x The middle name or the mother’s surname is only considered in Article 375(1),
parent; (2) it is customary for every Filipino to have as middle name the surname quoted above, in case there is identity of names and surnames between
of the mother; (3) the middle name or initial is a part of the name of a person; ascendants and descendants, in which case, the middle name or the mother’s
(4) adoption is for the benefit and best interest of the adopted child, hence, her surname shall be added.
right to bear a proper name should not be violated; (5) permitting Stephanie to x Notably, the law is likewise silent as to what middle name an adoptee may use.
use the middle name "Garcia" (her mother’s surname) avoids the stigma of her Article 365 of the Civil Code merely provides that "an adopted child shall bear the
illegitimacy; and; (6) her continued use of "Garcia" as her middle name is not surname of the adopter." Also, Article 189 of the Family Code, enumerating the
opposed by either the Catindig or Garcia families. legal effects of adoption, is likewise silent on the matter, thus:
x The Republic, through the Office of the Solicitor General (OSG), agrees with o "(1) For civil purposes, the adopted shall be deemed to be a
petitioner that Stephanie should be permitted to use, as her middle name, the legitimate child of the adopters and both shall acquire the
surname of her natural mother for the following reasons: reciprocal rights and obligations arising from the relationship of
o First, it is necessary to preserve and maintain Stephanie’s filiation parent and child, including the right of the adopted to use the
with her natural mother because under Article 189 of the Family surname of the adopters;
Code, she remains to be an intestate heir of the latter. Thus, to x However, as correctly pointed out by the OSG, the members of the Civil Code
prevent any confusion and needless hardship in the future, her and Family Law Committees that drafted the Family Code recognized the Filipino
relationship or proof of that relationship with her natural mother custom of adding the surname of the child’s mother as his middle name. In the
should be maintained. Minutes of the Joint Meeting of the Civil Code and Family Law Committees, the
o Second, there is no law expressly prohibiting Stephanie to use the members approved the suggestion that the initial or surname of the mother
surname of her natural mother as her middle name. What the law should immediately precede the surname of the father, thus:
does not prohibit, it allows. o "Justice Caguioa commented that there is a difference
o Last, it is customary for every Filipino to have a middle name, between the use by the wife of the surname and that of the
which is ordinarily the surname of the mother. This custom has child because the father’s surname indicates the family to
been recognized by the Civil Code and Family Code. In fact, the which he belongs, for which reason he would insist on the
Family Law Committees agreed that "the initial or surname of the use of the father’s surname by the child but that, if he
mother should immediately precede the surname of the father so wants to, the child may also use the surname of the
that the second name, if any, will be before the surname of the mother.
mother."7 o Justice Puno posed the question: If the child chooses to use the
x We find merit in the petition. surname of the mother, how will his name be written? Justice
o Use Of Surname Is Fixed By Law - Caguioa replied that it is up to him but that his point is that it
x For all practical and legal purposes, a man's name is the designation by which he should be mandatory that the child uses the surname of the father
is known and called in the community in which he lives and is best known. It is and permissive in the case of the surname of the mother.
defined as the word or combination of words by which a person is distinguished x Prof. Baviera remarked that Justice Caguioa’s point is covered by the present
from other individuals and, also, as the label or appellation which he bears for Article 364, which reads:
the convenience of the world at large addressing him, or in speaking of or o Legitimate and legitimated children shall principally use the
dealing with him.8 It is both of personal as well as public interest that every surname of the father.
person must have a name. o Justice Puno pointed out that many names change through no
x The name of an individual has two parts: choice of the person himself precisely because of this
o (1) the given or proper name and misunderstanding. He then cited the following example: Alfonso
o (2) the surname or family name. Ponce Enrile’s correct surname is Ponce since the mother’s surname
ƒ The given or proper name is that which is given to is Enrile but everybody calls him Atty. Enrile. Justice Jose Gutierrez
the individual at birth or at baptism, to distinguish David’s family name is Gutierrez and his mother’s surname is David
him from other individuals. The surname or family but they all call him Justice David.
name is that which identifies the family to which he o Justice Caguioa suggested that the proposed Article (12) be
belongs and is continued from parent to child. The modified to the effect that it shall be mandatory on the child to use
given name may be freely selected by the parents for the surname of the father but he may use the surname of the
the child, but the surname to which the child is mother by way of an initial or a middle name. Prof. Balane stated
entitled is fixed by law.9 that they take note of this for inclusion in the Chapter on Use of
x Thus, Articles 364 to 380 of the Civil Code provides the substantive rules which Surnames since in the proposed Article (10) they are just
regulate the use of surname10 of an individual whatever may be his status in enumerating the rights of legitimate children so that the details can
life, i.e., whether he may be legitimate or illegitimate, an adopted child, a be covered in the appropriate chapter.
married woman or a previously married woman, or a widow, thus:
133
RECTO, GAYLE ANGELI M.
2011-0008 | AUSL
Personal Notes on Remedial Law 2 Review (based on the syllabus of Prof. Henedino M. Brondial)

o Justice Puno remarked that there is logic in the simplification thereof, and copy of such order shall be served on the person alleged to be insane, and to the
suggested by Justice Caguioa that the surname of the father should one having charge him, or on such of his relatives residing in the province or city as the judge
always be last because there are so many traditions like the may deem proper. The court shall furthermore order the sheriff to produce the alleged insane
American tradition where they like to use their second given name person, if possible, on the date of the hearing.
and the Latin tradition, which is also followed by the Chinese
wherein they even include the Clan name. Section 3. Hearing and judgment. — Upon satisfactory proof, in open court on the date
o Justice Puno suggested that they agree in principle that in the fixed in the order, that the commitment applied for is for the public welfare or for the welfare
Chapter on the Use of Surnames, they should say that initial or of the insane person, and that his relatives are unable for any reason to take proper custody
surname of the mother should immediately precede the surname of and care of him, the court shall order his commitment to such hospital or other place for the
the father so that the second name, if any, will be before the insane as may be recommended by the Director of Health. The court shall make proper
surname of the mother. Prof. Balane added that this is really the provisions for the custody of property or money belonging to the insane until a guardian be
Filipino way. The Committee approved the suggestion."12 properly appointed.
(Emphasis supplied)
x In the case of an adopted child, the law provides that "the adopted shall bear Section 4. Discharge of insane. — When, in the opinion of the Director of Health, the
the surname of the adopters."13 Again, it is silent whether he can use a middle person ordered to be committed to a hospital or other place for the insane is temporarily or
name. What it only expressly allows, as a matter of right and obligation, is for permanently cured, or may be released without danger he may file the proper petition with
the adoptee to bear the surname of the adopter, upon issuance of the decree of the Court of First Instance which ordered the commitment.
adoption.14
o The Underlying Intent of Adoption Is In Favor of the Adopted Child Section 5. Assistance of fiscal in the proceeding. — It shall be the duty of the
x Adoption is defined as the process of making a child, whether related provincial fiscal or in the City of Manila the fiscal of the city, to prepare the petition for the
or not to the adopter, possess in general, the rights accorded to a Director of Health and represent him in court in all proceedings arising under the provisions of
legitimate child.15 It is a juridical act, a proceeding in rem which creates this rule.
between two persons a relationship similar to that which results from
legitimate paternity and filiation.16 The modern trend is to consider
adoption not merely as an act to establish a relationship of paternity and filiation, HABEAS CORPUS
but also as an act which endows the child with a legitimate status.17 This was,
[RULE 102]
indeed, confirmed in 1989, when the Philippines, as a State Party to the
Convention of the Rights of the Child initiated by the United Nations, accepted
the principle that adoption is impressed with social and moral responsibility, and 1. Definition and Nature or Scope
that its underlying intent is geared to favor the adopted child.18 Republic Act No.
8552, otherwise known as the "Domestic Adoption Act of 1998,"19 secures these Section 1. To what habeas corpus extends. — Except as otherwise expressly provided by
rights and privileges for the adopted.20 law, the writ of habeas corpus shall extend to all cases of illegal confinement or detention by
x One of the effects of adoption is that the adopted is deemed to be a legitimate which any person is deprived of his liberty, or by which the rightful custody of any person is
child of the adopter for all intents and purposes pursuant to Article 18921 of the withheld from the person entitled thereto.
Family Code and Section 1722 Article V of RA 8552.23
x Being a legitimate child by virtue of her adoption, it follows that 2. Requisites for Application
Stephanie is entitled to all the rights provided by law to a legitimate
child without discrimination of any kind, including the right to bear the
Section 3. Requisites of application therefor. — Application for the writ shall be by
surname of her father and her mother, as discussed above. This is
petition signed and verified either by the party for whose relief it is intended, or by some
consistent with the intention of the members of the Civil Code and
person on his behalf, and shall set forth:
Family Law Committees as earlier discussed. In fact, it is a Filipino
custom that the initial or surname of the mother should immediately
(a) That the person in whose behalf the application is made is imprisoned or
precede the surname of the father.
restrained on his liberty;
x Additionally, as aptly stated by both parties, Stephanie’s continued use of her
mother’s surname (Garcia) as her middle name will maintain her maternal
(b) The officer or name of the person by whom he is so imprisoned or restrained; or,
lineage. It is to be noted that Article 189(3) of the Family Code and Section
if both are unknown or uncertain, such officer or person may be described by an assumed
1824, Article V of RA 8552 (law on adoption) provide that the adoptee remains an
appellation, and the person who is served with the writ shall be deemed the person intended;
intestate heir of his/her biological parent. Hence, Stephanie can well assert or
claim her hereditary rights from her natural mother in the future.
(c) The place where he is so imprisoned or restrained, if known;
x Moreover, records show that Stephanie and her mother are living together in the
house built by petitioner for them at 390 Tumana, San Jose, Baliuag, Bulacan.
(d) A copy of the commitment or cause of detention of such person, if it can be
Petitioner provides for all their needs. Stephanie is closely attached to both her
procured without impairing the efficiency of the remedy; or, if the imprisonment or restraint is
mother and father. She calls them "Mama" and "Papa". Indeed, they are one
without any legal authority, such fact shall appear.
normal happy family. Hence, to allow Stephanie to use her mother’s surname as
her middle name will not only sustain her continued loving relationship with her
mother but will also eliminate the stigma of her illegitimacy. 3. Disallowance or Discharge of Writ
o Liberal Construction of Adoption Statutes In Favor Of Adoption -
x It is a settled rule that adoption statutes, being humane and salutary, Section 4. When writ not allowed or discharge authorized. — If it appears that the
should be liberally construed to carry out the beneficent purposes of person alleged to be restrained of his liberty is in the custody of an officer under process
adoption.25 The interests and welfare of the adopted child are of primary and issued by a court or judge or by virtue of a judgment or order of a court of record, and that
paramount consideration,26 hence, every reasonable intendment should be the court or judge had jurisdiction to issue the process, render the judgment, or make the
sustained to promote and fulfill these noble and compassionate objectives of the order, the writ shall not be allowed; or if the jurisdiction appears after the writ is allowed, the
law.27 person shall not be discharged by reason of any informality or defect in the process,
x Lastly, Art. 10 of the New Civil Code provides that: judgment, or order. Not shall anything in this rule be held to authorize the discharge of a
o "In case of doubt in the interpretation or application of laws, it is person charged with or convicted of an offense in the Philippines, or of a person suffering
presumed that the lawmaking body intended right and justice to imprisonment under lawful judgment.
prevail."
x This provision, according to the Code Commission, "is necessary so that it may 4. Preliminary Citation vs. Writ
tip the scales in favor of right and justice when the law is doubtful or obscure. It
will strengthen the determination of the courts to avoid an injustice which may
Section 6. To whom writ directed, and what to require. — In case of imprisonment or
apparently be authorized by some way of interpreting the law."28
restraint by an officer, the writ shall be directed to him, and shall command him to have the
x Hence, since there is no law prohibiting an illegitimate child adopted by her
body of the person restrained of his liberty before the court or judge designated in the writ at
natural father, like Stephanie, to use, as middle name her mother’s surname, we
the time and place therein specified. In case of imprisonment or restraint by a person not an
find no reason why she should not be allowed to do so.
officer, the writ shall be directed to an officer, and shall command him to take and have the
body of the person restrained of his liberty before the court or judge designated in the writ at
________________________________________________
the time and place therein specified, and to summon the person by whom he is restrained
then and there to appear before said court or judge to show the cause of the imprisonment or
PROCEEDINGS FOR HOSPITALIZATION OF INSANE PERSONS restraint.
[RULE 101]
5. The Return: When Evidence; When Plea
Section 1. Venue, Petition for commitment. — A petition for the commitment of a
person to a hospital or other place for the insane may be filed with the Court of First Instance
of the province where the person alleged to be insane is found. The petition shall be filed by Section 10. Contents of return. — When the person to be produced is imprisoned or
the Director of Health in all cases where, in his opinion, such commitment is for the public restrained by an officer, the person who makes the return shall state therein, and in other
welfare, or for the welfare of said person who, in his judgment, is insane and such person or cases the person in whose custody the prisoner is found shall state, in writing to the court or
the one having charge of him is opposed to his being taken to a hospital or other place for the judge before whom the writ is returnable, plainly and unequivocably:
insane.
(a) Whether he has or has not the party in his custody or power, or under restraint;
Section 2. Order for hearing. — If the petition filed is sufficient in form and substance,
the court, by an order reciting the purpose of the petition, shall fix a date for the hearing
134
RECTO, GAYLE ANGELI M.
2011-0008 | AUSL
Personal Notes on Remedial Law 2 Review (based on the syllabus of Prof. Henedino M. Brondial)

(b) If he has the party in his custody or power, or under restraint, the authority and In preliminary citation, it is not mandatory where the court may dispense with the
the true and whole cause thereof, set forth at large, with a copy of the writ, order execution, issuance of the preliminary citation and go directly to the issuance of the writ of
or other process, if any, upon which the party is held;
habeas corpus.
(c) If the party is in his custody or power or is restrained by him, and is not
produced, particularly the nature and gravity of the sickness or infirmity of such party by A person in custody of another, restraining the liberty of another, must give a
reason of which he cannot, without danger, be bought before the court or judge; return upon receipt of the writ of habeas corpus. That return can either be prima
facie evidence of the detention or a plea of the facts stated therein, in the
(d) If he has had the party in his custody or power, or under restraint, and has
transferred such custody or restraint to another, particularly to whom, at what time, for what
return.
cause, and by what authority such transfer was made.
Q: What does this mean? (Section 13)
Section 11. Return to be signed and sworn to. — The return or statement shall be A: The content of the return as to whether it is a plea only or prima facie
signed by the person who makes it; and shall also be sworn by him if the prisoner is not evidence of detention, they distinguish as to who has burden of proof. If the
produced, and in all other cases unless the return is made and signed by a sworn public officer return contains prima facie evidence of the detention, then petitioner has the
in his official capacity. burden of proof to show that the detention is illegal. But if it is only a plea of the
facts stated in the return, then the one who has the custody of the person has
Section 12. Hearing on return. Adjournments. — When the writ is returned before one
judge, at a time when the court is in session, he may forthwith adjourn the case into the the burden of proof.
court, there to be heard and determined. The court or judge before whom the writ is returned
or adjourned must immediately proceed to hear and examine the return, and such other So a writ of habeas corpus is directed to a jail warden. When the jail warden
matters as are properly submitted for consideration, unless for good cause shown the hearing prepares the return, which is brought to the court, he says that this person is
is adjourned, in which event the court or judge shall make such order for the safekeeping of under a commitment order, merong desisyon ang korte that this one should be
the person imprisoned or restrained as the nature of the case requires. If the person
imprisoned because he was denied bail although is appeal is still pending. That is
imprisoned or restrained is not produced because of his alleged sickness or infirmity, the court
or judge must be satisfied that it is so grave that such person cannot be produced without prima facie evidence of the cause of his detention, and when that is submitted to
danger, before proceeding to hear and dispose of the matter. On the hearing the court or the court, the applicant has the burden to establish that that commitment order
judge shall disregard matters of form and technicalities in respect to any warrant or order of is illegal.
commitment of a court or officer authorized to commit by law.
But if the return, it says well i am taking custody of this child because i am the
Section 13. When the return evidence, and when only a plea. — If it appears that the
father, that is not a commitment order or judicial order, that is coming from a
prisoner is in custody under a warrant of commitment in pursuance of law, the return shall be
considered prima facie evidence of the cause of restraint, but if he is restrained of his liberty private person. Therefore, that is not prima facie evidence but only a plea of the
by any alleged private authority, the return shall be considered only as a plea of the facts facts stated therein. Hence, the father will show that he has the right to take
therein set forth, and the party claiming the custody must prove such facts. custody of the child and not anymore the petitioner.

OLD LECTURE CASES

Q: What is the constitutional provision about habeas corpus? ERLINDA K. ILUSORIO, petitioner, vs. ERLINDA I. BILDNER and SYLVIA K. ILUSORIO,
A: The privilege of the writ of habeas corpus shall not be suspended except in JOHN DOE and JANE DOE, respondents.
cases of invasion or when public safety requires it. It is not the writ which is G.R. No. 139789 May 12, 2000
FIRST DIVISION
suspended but the privilege.
FACTS:
Q: What is a writ of habeas corpus? x Erlinda Kalaw and Potenciano Ilusorio contracted matrimony and lived together
A: It is an order or judicial process directed to the person to show cause for the for a period 30 years. In 1972, they separated from bed and board for
reason of detention. undisclosed reasons. Out of their marriage, the spouses had 6 children.
x The children, Sylvia and Erlinda alleged that during the time Potenciano arrived
from US and lived with Erlinda for 5 months, their mother gave Potenciano an
Q: By that definition, it is directed in two conditions, which are:
overdose of 200 mg instead of 100 mg Zoloft, an antidepressant drug prescribed
A: by his doctor in New York, U.S.A. As a consequence, Potenciano’s health
1. illegal detention or confinement;
deteriorated.
2. illegal withholding of custody from a person entitled thereto. x Erlinda then filed with RTC a petition for guardianship over the person and
It is directed to someone who commits an act of either illegal detention or property of Potenciano Ilusorio due to the latter’s advanced age, frail health,
confinement or illegal withholding of custody from a person entitled thereto. poor eyesight and impaired judgment.
x Subsequently, after attending a corporate meeting in Baguio City, Potenciano
Ilusorio did not return to Antipolo City and instead lived at Cleveland
Q: What is an example of the first instance? Condominium, Makati.
A: Violation of the constitution, whereby a person is deprived or restrained of his x Erlinda filed with the CA a petition for habeas corpus. She alleged that
liberty or not afforded a right to due process, or an order from an authority respondents refused petitioner’s demands to see and visit her husband and
which has no jurisdiction. prohibited Potenciano from returning to Antipolo City.
x CA allowed visitation rights to Erlinda but ordered that the writ of habeas corpus
previously issued be recalled and the herein petition for habeas corpus be
Q: What is an example of the second instance?
dismissed for lack of unlawful restraint or detention of the subject of the petition.
A: Malimit itong mangyari within the family, where the mother or father fights
over for the custody of the child. ISSUE: Whether a wife may compel her husband to live with her in conjugal bliss by way writ
of a writ of habeas corpus.
Q: If a woman leaves the parental home in order to stay with her
paramour, can the parents file a petition for a writ of habeas corpus? HELD: NO.
A: It depends. If the child is a minor, a writ of habeas corpus is available. But x The evidence shows that there was no actual and effective detention or
deprivation of lawyer Potenciano Ilusorio’s liberty that would justify the issuance
when a child comes of age, the writ of habeas corpus is not available. of the writ. The fact that lawyer Potenciano Ilusorio is about 86 years of age, or
under medication does not necessarily render him mentally incapacitated.
Q: Suppose a judge renders a judgment penalizing a person with Soundness of mind does not hinge on age or medical condition but on the
imprisonment of 6 years 1 month and 1 day, and then the convict has capacity of the individual to discern his actions.
x After due hearing, the Court of Appeals concluded that there was no unlawful
already stayed for more than that period of time, is the writ available?
A: Yes, the writ is available because that is the immediate remedy. Although you restraint on his liberty.
x The Court of Appeals also observed that lawyer Potenciano Ilusorio did not
can also file certiorari but it is no longer immediate here. Because what do you request the administrator of the Cleveland Condominium not to allow his wife
intend to annul there? wala. and other children from seeing or visiting him. He made it clear that he did not
object to seeing them.
Q: What is a preliminary citation as compared to the writ of habeas x As to lawyer Potenciano Ilusorio’s mental state, the Court of Appeals observed
corpus or the peremptory writ? that he was of sound and alert mind, having answered all the relevant questions
A: Preliminary citation is a citation to the government officer having the person to the satisfaction of the court.
x Being of sound mind, he is thus possessed with the capacity to make choices. In
in his custody to show cause why the writ of habeas corpus should not issue this case, the crucial choices revolve on his residence and the people he opts to
(detention not patently illegal). see or live with. The choices he made may not appeal to some of his family
Preemptory writ is issued when the cause of the detention appears to be members but these are choices which exclusively belong to Potenciano. He made
patently illegal and the non-compliance wherewith is punishable. it clear before the Court of Appeals that he was not prevented from leaving his

135
RECTO, GAYLE ANGELI M.
2011-0008 | AUSL
Personal Notes on Remedial Law 2 Review (based on the syllabus of Prof. Henedino M. Brondial)

house or seeing people. With that declaration, and absent any true restraint on
his liberty, we have no reason to reverse the findings of the Court of Appeals.
x With his full mental capacity coupled with the right of choice, Potenciano Ilusorio FACTS:
may not be the subject of visitation rights against his free choice. Otherwise, we x This case stemmed from a plunder case filed by the People against Serapio
will deprive him of his right to privacy. Needless to say, this will run against his among others
fundamental constitutional right. x Serapio (member of the Board of Trustees and the Legal Counsel of the Erap
x The Court of Appeals exceeded its authority when it awarded visitation rights in a Muslim Youth Foundation, a non-stock, non-profit foundation established in
petition for habeas corpus where Erlinda never even prayed for such right. The February 2000 ostensibly for the purpose of providing educational opportunities
ruling is not consistent with the finding of subject’s sanity. for the poor and underprivileged but deserving Muslim youth and students, and
x When the court ordered the grant of visitation rights, it also emphasized that the support to research and advance studies of young Muslim educators and
same shall be enforced under penalty of contempt in case of violation or refusal scientists) Æ received on its behalf a donation in the amount of Two Hundred
to comply. Such assertion of raw, naked power is unnecessary. Million Pesos (P200 Million) from Ilocos Sur Governor Luis Chavit Singson through
x The Court of Appeals missed the fact that the case did not involve the right of a the latters assistant Mrs. Yolanda Ricaforte
parent to visit a minor child but the right of a wife to visit a husband. In case the o He then turned over the said amount to the Foundations treasurer
husband refuses to see his wife for private reasons, he is at liberty to do so who later deposited it in the Foundations account with the
without threat of any penalty attached to the exercise of his right. Equitable PCI Bank
x No court is empowered as a judicial authority to compel a husband to live with x Gov. Singson Æ then publicly accused then President Joseph E. Estrada and his
his wife. Coverture cannot be enforced by compulsion of a writ of habeas corpus cohorts of engaging in several illegal activities, including its operation on the
carried out by sheriffs or by any other mesne process. That is a matter beyond illegal numbers game known as jueteng
judicial authority and is best left to the man and woman’s free choice. o Several criminal complaints were then filed against several persons
including Serapio
IN THE MATTER OF THE PETITION FOR HABEAS CORPUS OF POTENCIANO x Ombudsman Æ issued a joint resolution recommending, inter alia, that Joseph
ILUSORIO Estrada, petitioner and several others be charged with the criminal offense of
ERLINDA K. ILUSORIO, petitioner, vs. ERLINDA K. ILUSORIO-BILDNER, SYLVIA K. plunder
ILUSORIO-YAP, JOHN DOES and JANE DOES, respondents. o Serapio received a copy of the resolution on April 5, 2001
G.R. No. 139789 July 19, 2001 x Serapio (the ff day) Æ filed an MR before the Office of the Ombudsman (OO)
FIRST DIVISION o likewise filed on said date, this time with the Sandiganbayan, an
Urgent Omnibus Motion:
FACTS: ƒ (a) To Hold in Abeyance the Issuance of Warrant of
x This is an MR to the decision in the 2000 case above. Arrest and Further Proceedings;
ƒ (b) To Conduct a Determination of Probable Cause;
ISSUE: Whether Article XII of the 1987 Constitution and Articles 68 and 69 of the Family Code ƒ (c) For Leave to File Accused's Motion for
may be used by petitioner-movant as leverage so that the Court may declare that she and Reconsideration and/or Reinvestigation; and
deceased spouse should have lived together. ƒ (d) To Direct the Ombudsman to Conduct a
Reinvestigation of the Charges against accused
HELD: NO. Edward Serapio
x One reason why Erlinda K. Ilusorio sought custody of her husband was that x OO Æ denied the MR on the ground that the Information has already been filed
respondents Lin and Sylvia were illegally restraining Potenciano Ilusorio to with the SB
fraudulently deprive her of property rights out of pure greed.[14] She claimed x SB Æ issued warrants of arrest
that her two children were using their sick and frail father to sign away x Serapio Æ filed an application for bail
Potenciano and Erlinda' s property to companies controlled by Lin and Sylvia. x BUT before the same could actually be heard, SB ordered the arraignment of the
She also argued that since Potenciano retired as director and officer of Baguio other accused
Country Club and Philippine Oversees Telecommunications, she would logically x Serapio (BEFORE SB could resolve the pending motions [for the quashal of
assume his position and control. Yet, Lin and Sylvia were the ones controlling information, for fixing of bail, for arraignment etc]) Æ filed with the SC a Petition
the corporations.[15] for Habeas Corpus and Certiorari
x The fact of illegal restraint has not been proved during the hearing at the Court o praying that the Court declare void the questioned orders,
of Appeals on March 23, 1999.[16] Potenciano himself declared that he was not resolutions and actions of the Sandiganbayan on his claim that he
prevented by his children from seeing anybody and that he had no objection to was thereby effectively denied of his right to due process.
seeing his wife and other children whom he loved. o Petitioner likewise prayed for the issuance of a writ of
x Erlinda highlighted that her husband suffered from various ailments. Thus, habeas corpus; that the People be declared to have waived their
Potenciano Ilusorio did not have the mental capacity to decide for himself. right to present evidence in opposition to his petition for bail; and,
Hence, Erlinda argued that Potenciano be brought before the Supreme Court so premised on the failure of the People to adduce strong evidence of
that we could determine his mental state. petitioner's guilt of plunder, that he be granted provisional liberty on
x We were not convinced that Potenciano Ilusorio was mentally incapacitated to bail after due proceedings.
choose whether to see his wife or not. Again, this is a question of fact that has x SB Æ denied the motion to quash
been decided in the Court of Appeals. x Serapio Æ filed a Rule 65 before the SC
x As to whether the children were in fact taking control of the corporations, these
are matters that may be threshed out in a separate proceeding, irrelevant in ISSUE: Whether petitioner was deprived of his right to due process in the plunder case and
habeas corpus. should thus be released from detention via a writ of habeas corpus.
x The law provides that the husband and the wife are obliged to live together,
observe mutual love, respect and fidelity.[20] The sanction therefor is the HELD: NO.
"spontaneous, mutual affection between husband and wife and not any legal x On the first issue, petitioner contends that the Sandiganbayan committed a
mandate or court order" to enforce consortium.[21] grave abuse of its discretion amounting to excess or lack of jurisdiction when it
x Obviously, there was absence of empathy between spouses Erlinda and deferred the hearing of his petition for bail to July 10, 2001, arraigned him on
Potenciano, having separated from bed and board since 1972. We defined said date and entered a plea of not guilty for him when he refused to be
empathy as a shared feeling between husband and wife experienced not only by arraigned. He insists that the Rules on Criminal Procedure, as amended, does not
having spontaneous sexual intimacy but a deep sense of spiritual communion. require that he be arraigned first prior to the conduct of bail hearings since the
Marital union is a two-way process. latter can stand alone and must, of necessity, be heard immediately.55 Petitioner
x Marriage is definitely for two loving adults who view the relationship with "amor maintains that his arraignment before the bail hearings are set is not necessary
gignit amorem” respect, sacrifice and a continuing commitment to togetherness, since he would not plead guilty to the offense charged, as is evident in his earlier
conscious of its value as a sublime social institution statements insisting on his innocence during the Senate investigation of the
jueteng scandal and the preliminary investigation before the Ombudsman.56
Neither would the prosecution be prejudiced even if it would present all its
evidence before his arraignment because, under the Revised Penal Code, a
voluntary confession of guilt is mitigating only if made prior to the presentation
of evidence for the prosecution,57 and petitioner admitted that he cannot
repudiate the evidence or proceedings taken during the bail hearings because
ATTY. EDWARD SERAPIO, petitioner, vs. SANDIGANBAYAN (THIRD DIVISION), Rule 114, Section 8 of the Revised Rules of Court expressly provides that
PEOPLE OF THE PHILIPPINES, and PHILIPPINE NATIONAL POLICE DIRECTOR-GENERAL evidence present during bail hearings are automatically reproduced during the
LEANDRO MENDOZA, respondents. trial.58 Petitioner likewise assures the prosecution that he is willing to be
G.R. No. 148468. January 28, 2003 arraigned prior to the posting of a bail bond should he be granted bail.59
EN BANC x The People insist that arraignment is necessary before bail hearings may be
commenced, because it is only upon arraignment that the issues are joined. The
EDWARD S. SERAPIO, petitioner, vs. HONORABLE SANDIGANBAYAN and PEOPLE OF THE People stress that it is only when an accused pleads not guilty may he file a
PHILIPPINES, respondents. petition for bail and if he pleads guilty to the charge, there would be no more
G.R. No. 148769. January 28, 2003 need for him to file said petition. Moreover, since it is during arraignment that
the accused is first informed of the precise charge against him, he must be
EDWARD S. SERAPIO, petitioner, vs. HONORABLE SANDIGANBAYAN (THIRD DIVISION) and arraigned prior to the bail hearings to prevent him from later assailing the
PEOPLE OF THE PHILIPPINES, respondents. validity of the bail hearings on the ground that he was not properly informed of
G.R. No. 149116. January 28, 2003 the charge against him, especially considering that, under Section 8, Rule 114 of
136
RECTO, GAYLE ANGELI M.
2011-0008 | AUSL
Personal Notes on Remedial Law 2 Review (based on the syllabus of Prof. Henedino M. Brondial)

the Revised Rules of Court, evidence presented during such proceedings are preclude his right to assail the validity of the Information charging him with such
considered automatically reproduced at the trial.60 Likewise, the arraignment of offense. It must be conceded, however, that if a motion to quash a criminal
accused prior to bail hearings diminishes the possibility of an accused's flight complaint or Information on the ground that the same does not charge any
from the jurisdiction of the Sandiganbayan because trial in absentia may be had offense is granted and the case is dismissed and the accused is ordered
only if an accused escapes after he has been arraigned.61 The People also released, the petition for bail of an accused may become moot and academic.
contend that the conduct of bail hearings prior to arraignment would extend to x We now resolve the issue of whether or not it is mandatory that the hearings on
an accused the undeserved privilege of being appraised of the prosecution's the petitions for bail of petitioner and accused Jose "Jinggoy" Estrada in Criminal
evidence before he pleads guilty for purposes of penalty reduction.62 Case No. 26558 and the trial of the said case as against former President Joseph
x Although petitioner had already been arraigned on July 10, 2001 and a plea of E. Estrada be heard jointly.
not guilty had been entered by the Sandiganbayan on his behalf, thereby x Petitioner argues that the conduct of joint bail hearings would negate his right to
rendering the issue as to whether an arraignment is necessary before the have his petition for bail resolved in a summary proceeding since said hearings
conduct of bail hearings in petitioner's case moot, the Court takes this might be converted into a full blown trial on the merits by the prosecution.76
opportunity to discuss the controlling precepts thereon pursuant to its symbolic x For their part, the People claim that joint bail hearings will save the court from
function of educating the bench and bar.63 having to hear the same witnesses and the parties from presenting the same
x The contention of petitioner is well-taken. The arraignment of an accused is evidence where it would allow separate bail hearings for the accused who are
not a prerequisite to the conduct of hearings on his petition for bail. A charged as co-conspirators in the crime of plunder.77
person is allowed to petition for bail as soon as he is deprived of his x In issuing its June 1, 2001 Order directing all accused in Criminal Case No. 26558
liberty by virtue of his arrest or voluntary surrender.64 An accused to participate in the bail hearings, the Sandiganbayan explained that the directive
need not wait for his arraignment before filing a petition for bail. x In was made was in the interest of the speedy disposition of the case. It stated:
Lavides vs. Court of Appeals,65 this Court ruled on the issue of whether an o " x x x The obvious fact is, if the rest of the accused other than the
accused must first be arraigned before he may be granted bail. Lavides involved accused Serapio were to be excused from participating in the
an accused charged with violation of Section 5(b) Republic Act No. 7610 (The hearing on the motion for bail of accused Serapio, under the
Special Protection of Children Against Abuse, Exploitation and Discrimination pretext that the same does not concern them and that they will
Act), an offense punishable by reclusion temporal in its medium period to participate in any hearing where evidence is presented by the
reclusion perpetua. The accused therein assailed, inter alia, the trial court's prosecution only if and when they will already have filed their
imposition of the condition that he should first be arraigned before he is allowed petitions for bail, or should they decide not to file any, that they
to post bail. We held therein that "in cases where it is authorized, bail should be will participate only during the trial proper itself, then everybody
granted before arraignment, otherwise the accused may be precluded from filing will be faced with the daunting prospects of having to go through
a motion to quash."66 the process of introducing the same witness and pieces of evidence
x However, the foregoing pronouncement should not be taken to mean two times, three times or four times, as many times as there are
that the hearing on a petition for bail should at all times precede petitions for bail filed. Obviously, such procedure is not conducive to
arraignment, because the rule is that a person deprived of his liberty the speedy termination of a case. Neither can such procedure be
by virtue of his arrest or voluntary surrender may apply for bail as soon characterized as an orderly proceeding."78
as he is deprived of his liberty, even before a complaint or information x There is no provision in the Revised Rules of Criminal Procedure or the Rules of
is filed against him.67 The Court's pronouncement in Lavides should be Procedure of the Sandiganbayan governing the hearings of two or more petitions
understood in light of the fact that the accused in said case filed a petition for for bail filed by different accused or that a petition for bail of an accused be
bail as well as a motion to quash the informations filed against him. Hence, we heard simultaneously with the trial of the case against the other accused. The
explained therein that to condition the grant of bail to an accused on his matter of whether or not to conduct a joint hearing of two or more petitions for
arraignment would be to place him in a position where he has to choose bail filed by two different accused or to conduct a hearing of said petition jointly
between (1) filing a motion to quash and thus delay his release on bail because with the trial against another accused is addressed to the sound discretion of the
until his motion to quash can be resolved, his arraignment cannot be held, and trial court. Unless grave abuse of discretion amounting to excess or lack of
(2) foregoing the filing of a motion to quash so that he can be arraigned at once jurisdiction is shown, the Court will not interfere with the exercise by the
and thereafter be released on bail. This would undermine his constitutional right Sandiganbayan of its discretion.
not to be put on trial except upon a valid complaint or Information sufficient to x It may be underscored that in the exercise of its discretion, the Sandiganbayan
charge him with a crime and his right to bail.68 must take into account not only the convenience of the State, including the
x It is therefore not necessary that an accused be first arraigned before prosecution, but also that of the accused and the witnesses of both the
the conduct of hearings on his application for bail. For when bail is a prosecution and the accused and the right of accused to a speedy trial. The
matter of right, an accused may apply for and be granted bail even Sandiganbayan must also consider the complexities of the cases and of the
prior to arraignment. The ruling in Lavides also implies that an factual and legal issues involving petitioner and the other accused. After all, if
application for bail in a case involving an offense punishable by this Court may echo the observation of the United States Supreme Court, the
reclusion perpetua to death may also be heard even before an accused State has a stake, with every citizen, in his being afforded our historic individual
is arraigned. Further, if the court finds in such case that the accused is protections, including those surrounding criminal prosecutions. About them, this
entitled to bail because the evidence against him is not strong, he may Court dares not become careless or complacent when that fashion has become
be granted provisional liberty even prior to arraignment; for in such a rampant over the earth.79
situation, bail would be "authorized" under the circumstances. In fine, x It must be borne in mind that in Ocampo vs. Bernabe,80 this Court held that in a
the Sandiganbayan committed a grave abuse of its discretion petition for bail hearing, the court is to conduct only a summary hearing,
amounting to excess of jurisdiction in ordering the arraignment of meaning such brief and speedy method of receiving and considering the
petitioner before proceeding with the hearing of his petition for bail. evidence of guilt as is practicable and consistent with the purpose of the hearing
x With respect to the second issue of whether petitioner may file a motion to which is merely to determine the weight of evidence for purposes of bail. The
quash during the pendency of his petition for bail, petitioner maintains that a court does not try the merits or enter into any inquiry as to the weight that
motion to quash and a petition for bail are not inconsistent, and may proceed ought to be given to the evidence against the accused, nor will it speculate on
independently of each other. While he agrees with the prosecution that a motion the outcome of the trial or on what further evidence may be offered therein. It
to quash may in some instances result in the termination of the criminal may confine itself to receiving such evidence as has reference to substantial
proceedings and in the release of the accused therein, thus rendering the matters, avoiding unnecessary thoroughness in the examination and cross-
petition for bail moot and academic, he opines that such is not always the case; examination of witnesses, and reducing to a reasonable minimum the amount of
hence, an accused in detention cannot be forced to speculate on the outcome of corroboration particularly on details that are not essential to the purpose of the
a motion to quash and decide whether or not to file a petition for bail or to hearing.
withdraw one that has been filed.69 He also insists that the grant of a motion to x A joint hearing of two separate petitions for bail by two accused will of course
quash does not automatically result in the discharge of an accused from avoid duplication of time and effort of both the prosecution and the courts and
detention nor render moot an application for bail under Rule 117, Section 5 of minimizes the prejudice to the accused, especially so if both movants for bail are
the Revised Rules of Court.70 charged of having conspired in the commission of the same crime and the
x The Court finds that no such inconsistency exists between an application of an prosecution adduces essentially the same evident against them. However, in the
accused for bail and his filing of a motion to quash. Bail is the security given for cases at bar, the joinder of the hearings of the petition for bail of petitioner with
the release of a person in the custody of the law, furnished by him or a the trial of the case against former President Joseph E. Estrada is an entirely
bondsman, to guarantee his appearance before any court as required under the different matter. For, with the participation of the former president in the hearing
conditions set forth under the Rules of Court.71 Its purpose is to obtain the of petitioner's petition for bail, the proceeding assumes a completely different
provisional liberty of a person charged with an offense until his conviction while dimension. The proceedings will no longer be summary. As against former
at the same time securing his appearance at the trial.72 As stated earlier, a President Joseph E. Estrada, the proceedings will be a full-blown trial which is
person may apply for bail from the moment that he is deprived of his liberty by antithetical to the nature of a bail hearing. Moreover, following our ruling in Jose
virtue of his arrest or voluntary surrender.73 Estrada vs. Sandiganbayan, supra where we stated that Jose "Jinggoy" Estrada
x On the other hand, a motion to quash an Information is the mode by which an can only be charged with conspiracy to commit the acts alleged in sub-paragraph
accused assails the validity of a criminal complaint or Information filed against (a) of the amended Information since it is not clear from the latter if the accused
him for insufficiency on its face in point of law, or for defects which are apparent in sub-paragraphs (a) to (d) thereof conspired with each other to assist Joseph
in the face of the Information.74 An accused may file a motion to quash the Estrada to amass ill-gotten wealth, we hold that petitioner can only be charged
Information, as a general rule, before arraignment.75 with having conspired with the other co-accused named in sub-paragraph (a) by
x These two reliefs have objectives which are not necessarily antithetical to each "receiving or collecting, directly or indirectly, on several instances, money x x x
other. Certainly, the right of an accused right to seek provisional liberty when from illegal gambling, x x x in consideration of toleration or protection of illegal
charged with an offense not punishable by death, reclusion perpetua or life gambling.81 Thus, with respect to petitioner, all that the prosecution needs to
imprisonment, or when charged with an offense punishable by such penalties but adduce to prove that the evidence against him for the charge of plunder is
after due hearing, evidence of his guilt is found not to be strong, does not strong are those related to the alleged receipt or collection of money from illegal
137
RECTO, GAYLE ANGELI M.
2011-0008 | AUSL
Personal Notes on Remedial Law 2 Review (based on the syllabus of Prof. Henedino M. Brondial)

gambling as described in sub-paragraph (a) of the amended Information. With is strong, it is the court's duty to deny the application for bail. However, when
the joinder of the hearing of petitioner's petition for bail and the trial of the the evidence of guilt is not strong, bail becomes a matter of right.94
former President, the latter will have the right to cross-examine intensively and x In this case, petitioner is not entitled to bail as a matter of right at this
extensively the witnesses for the prosecution in opposition to the petition for bail stage of the proceedings. Petitioner's claim that the prosecution had refused
of petitioner. If petitioner will adduce evidence in support of his petition after the to present evidence to prove his guilt for purposes of his bail application and that
prosecution shall have concluded its evidence, the former President may insist on the Sandiganbayan has refused to grant a hearing thereon is not borne by the
cross-examining petitioner and his witnesses. The joinder of the hearing of records. The prosecution did not waive, expressly or even impliedly, its right to
petitioner's bail petition with the trial of former President Joseph E. Estrada will adduce evidence in opposition to the petition for bail of petitioner. It must be
be prejudicial to petitioner as it will unduly delay the determination of the issue noted that the Sandiganbayan had already scheduled the hearing dates for
of the right of petitioner to obtain provisional liberty and seek relief from this petitioner's application for bail but the same were reset due to pending incidents
Court if his petition is denied by the respondent court. The indispensability of the raised in several motions filed by the parties, which incidents had to be resolved
speedy resolution of an application for bail was succinctly explained by Cooley in by the court prior to the bail hearings. The bail hearing was eventually scheduled
his treatise Constitutional Limitations, thus: by the Sandiganbayan on July 10, 2001 but the hearing did not push through due
o "For, if there were any mode short of confinement which would to the filing of this petition on June 29, 2001.
with reasonable certainty insure the attendance of the accused to x The delay in the conduct of hearings on petitioner's application for bail is
answer the accusation, it would not be justifiable to inflict upon him therefore not imputable solely to the Sandiganbayan or to the prosecution.
that indignity, when the effect is to subject him in a greater or Petitioner is also partly to blame therefor, as is evident from the following list of
lesser degree, to the punishment of a guilty person, while as yet it motions filed by him and by the prosecution:
is not determined that he has not committed any crime."82 x Furthermore, the Court has previously ruled that even in cases where the
x While the Sandiganbayan, as the court trying Criminal Case No. 26558, is prosecution refuses to adduce evidence in opposition to an application for bail by
empowered "to proceed with the trial of the case in the manner it determines an accused charged with a capital offense, the trial court is still under duty to
best conducive to orderly proceedings and speedy termination of the case,"83 conduct a hearing on said application.101 The rationale for such requirement
the Court finds that it gravely abused its discretion in ordering that the petition was explained in Narciso vs. Sta. Romana-Cruz (supra), citing Basco vs.
for bail of petitioner and the trial of former President Joseph E. Estrada be held Rapatalo:102
jointly. It bears stressing that the Sandiganbayan itself acknowledged in its May o "When the grant of bail is discretionary, the prosecution has the
4, 2001 Order the "pre-eminent position and superiority of the rights of burden of showing that the evidence of guilt against the accused is
[petitioner] to have the matter of his provisional liberty resolved . . . without strong. However, the determination of whether or not the evidence
unnecessary delay,"84 only to make a volte face and declare that after all the of guilt is strong, being a matter of judicial discretion, remains with
hearing of petition for bail of petitioner and Jose "Jinggoy" Estrada and the trial the judge. This discretion by the very nature of things, may rightly
as against former President Joseph E. Estrada should be held simultaneously. In be exercised only after the evidence is submitted to the court at
ordering that petitioner's petition for bail to be heard jointly with the trial of the the hearing. Since the discretion is directed to the weight of the
case against his co-accused former President Joseph E. Estrada, the evidence and since evidence cannot properly be weighed if not duly
Sandiganbayan in effect allowed further and unnecessary delay in the resolution exhibited or produced before the court, it is obvious that a proper
thereof to the prejudice of petitioner. In fine then, the Sandiganbayan committed exercise of judicial discretion requires that the evidence of guilt be
a grave abuse of its discretion in ordering a simultaneous hearing of petitioner's submitted to the court, the petitioner having the right of cross-
petition for bail with the trial of the case against former President Joseph E. examination and to introduce his own evidence in rebuttal."103
Estrada on its merits. x Accordingly, petitioner cannot be released from detention until the
x With respect to petitioner's allegations that the prosecution tried to delay the bail Sandiganbayan conducts a hearing of his application for bail and
hearings by filing dilatory motions, the People aver that it is petitioner and his resolve the same in his favor. Even then, there must first be a finding
co-accused who caused the delay in the trial of Criminal Case No. 26558 by their that the evidence against petitioner is not strong before he may be
filing of numerous manifestations and pleadings with the Sandiganbayan.85 They granted bail.
assert that they filed the motion for joint bail hearing and motion for earlier x Anent the issue of the propriety of the issuance of a writ of habeas corpus for
arraignment around the original schedule for the bail hearings which was on May petitioner, he contends that he is entitled to the issuance of said writ because
21-25, 2001.86 the State, through the prosecution's refusal to present evidence and by the
x They argue further that bail is not a matter of right in capital offenses.87 In Sandiganbayan's refusal to grant a bail hearing, has failed to discharge its
support thereof, they cite Article III, Sec 13 of the Constitution, which states that burden of proving that as against him, evidence of guilt for the capital offense of
— plunder is strong. Petitioner contends that the prosecution launched "a seemingly
o "All persons, except those charged with offenses punishable by endless barrage of obstructive and dilatory moves" to prevent the conduct of bail
reclusion perpetua when evidence of guilt is strong, shall before hearings. Specifically, the prosecution moved for petitioner's arraignment before
conviction be bailable by sufficient sureties, or be released on the commencement of bail hearings and insisted on joint bail hearings for
recognizance as may be provided by law. The right to bail shall not petitioner, Joseph Estrada and Jinggoy Estrada despite the fact that it was only
be impaired even when the privilege of the writ of habeas corpus is petitioner who asked for a bail hearing; manifested that it would present its
suspended. Excessive bail shall not be required."88 evidence as if it is the presentation of the evidence in chief, meaning that the
x The People also cited Rule 114, Secs. 7 and 4 of the Revised Rules of Court bail hearings would be concluded only after the prosecution presented its entire
which provide: case upon the accused; and argued that petitioner's motion to quash and his
o "Sec. 7 Capital offense or an offense punishable by reclusion petition for bail are inconsistent, and therefore, petitioner should choose to
perpetua or life imprisonment, not bailable. — No person charged pursue only one of these two remedies.104 He further claims that the
with a capital offense, or an offense punishable by reclusion Sandiganbayan, through its questioned orders and resolutions postponing the
perpetua or life imprisonment, shall be admitted to bail when bail hearings effectively denied him of his right to bail and to due process of
evidence of guilt is strong, regardless of the stage of the criminal law.105
prosecution. x Petitioner also maintains that the issuance by the Sandiganbayan of new orders
o Sec. 4 Bail, a matter of right, exception. — All persons in custody canceling the bail hearings which it had earlier set did not render moot and
shall be admitted to bail as a matter of right, with sufficient academic the petition for issuance of a writ of habeas corpus, since said orders
sureties, or released on recognizance as prescribed by law or this have resulted in a continuing deprivation of petitioner's right to bail.106 He
Rule x x x (b) and before conviction by the Regional Trial Court of argues further that the fact that he was arrested and is detained pursuant to
an offense not punishable by death, reclusion perpetua or life valid process does not by itself negate the efficacy of the remedy of habeas
imprisonment."89 corpus. In support of his contention, petitioner cites Moncupa vs. Enrile,107
x Irrefragably, a person charged with a capital offense is not absolutely denied the where the Court held that habeas corpus extends to instances where the
opportunity to obtain provisional liberty on bail pending the judgment of his detention, while valid from its inception, has later become arbitrary.108
case. However, as to such person, bail is not a matter of right but is x However, the People insist that habeas corpus is not proper because
discretionary upon the court.90 Had the rule been otherwise, the Rules would petitioner was arrested pursuant to the amended information which
not have provided for an application for bail by a person charged with a capital was earlier filed in court,109 the warrant of arrest issuant pursuant
offense under Rule 114, Section 8 which states: thereto was valid, and petitioner voluntarily surrendered to the
o "Sec. 8 Burden of proof in bail application. — At the hearing of an authorities.110
application for bail filed by a person who is in custody for the x As a general rule, the writ of habeas corpus will not issue where the
commission of an offense punishable by death, reclusion perpetua, or person alleged to be restrained of his liberty in custody of an officer
life imprisonment, the prosecution has the burden of showing that under a process issued by the court which jurisdiction to do so.111 In
the evidence of guilt is strong. The evidence presented during the exceptional circumstances, habeas corpus may be granted by the
bail hearing shall be considered automatically reproduced at the trial courts even when the person concerned is detained pursuant to a valid
but, upon motion of either party, the court may recall any witness for arrest or his voluntary surrender, for this writ of liberty is recognized
additional examination unless the latter is dead, outside the as "the fundamental instrument for safeguarding individual freedom
Philippines, or otherwise unable to testify."91 against arbitrary and lawless state action" due to "its ability to cut
x Under the foregoing provision, there must be a showing that the evidence of through barriers of form and procedural mazes."112 Thus, in previous
guilt against a person charged with a capital offense is not strong for the court to cases, we issued the writ where the deprivation of liberty, while initially
grant him bail. Thus, upon an application for bail by the person charged with a valid under the law, had later become invalid,113 and even though the
capital offense, a hearing thereon must be conducted, where the prosecution persons praying for its issuance were not completely deprived of their
must be accorded an opportunity to discharge its burden of proving that the liberty.114
evidence of guilt against an accused is strong.92 The prosecution shall be x The Court finds no basis for the issuance of a writ of habeas corpus in favor of
accorded the opportunity to present all the evidence it may deem necessary for petitioner. The general rule that habeas corpus does not lie where the person
this purpose.93 When it is satisfactorily demonstrated that the evidence of guilt alleged to be restrained of his liberty is in the custody of an officer under process
138
RECTO, GAYLE ANGELI M.
2011-0008 | AUSL
Personal Notes on Remedial Law 2 Review (based on the syllabus of Prof. Henedino M. Brondial)

issued by a court which had jurisdiction to issue the same115 applies, because ISSUE # 1: Whether the application for the issuance of writ of HC is proper.
petitioner is under detention pursuant to the order of arrest issued by the
Sandiganbayan on April 25, 2001 after the filing by the Ombudsman of the HELD # 1: NO.
amended information for plunder against petitioner and his co-accused. x In connection with their alleged impending warrantless arrest, petitioners
Petitioner had in fact voluntarily surrendered himself to the authorities on April Lacson, Aquino, and mancao pray that the "appropriate court before whom the
25, 2001 upon learning that a warrant for his arrest had been issued. informations against petitioners are filed be directed to desist from arraigning
x The ruling in Moncupa vs. Enrile116 that habeas corpus will lie where the and proceeding with the trial of the case, until the instant petition is finally
deprivation of liberty which was initially valid has become arbitrary in view of resolved." This relief is clearly premature considering that as of this date, no
subsequent developments finds no application in the present case because complaints or charges have been filed against any of the petitioners for any
the hearing on petitioner's application for bail has yet to commence. As crime. And in the event that the same are later filed, this Court cannot enjoin
stated earlier, the delay in the hearing of petitioner's petition for bail criminal prosecution conducted in accordance with the Rules of Court, for by that
cannot be pinned solely on the Sandiganbayan or on the prosecution time any arrest would have been in pursuant of a duly issued warrant.
for that matter. Petitioner himself is partly to be blamed. Moreover, a petition x As regards petitioners' prayer that the hold departure orders issued against them
for habeas corpus is not the appropriate remedy for asserting one's be declared null and void ab initio, it is to be noted that petitioners are not
right to bail.117 It cannot be availed of where accused is entitled to bail directly assailing the validity of the subject hold departure orders in their petition.
not as a matter of right but on the discretion of the court and the latter They are not even expressing intention to leave the country in the near future.
has not abused such discretion in refusing to grant bail,118 or has not The prayer to set aside the same must be made in proper proceedings initiated
even exercised said discretion. The proper recourse is to file an for that purpose.
application for bail with the court where the criminal case is pending x Anent petitioners' allegations ex abundante ad cautelam in support of
and to allow hearings thereon to proceed. their application for the issuance of a writ of habeas corpus, it is
x The issuance of a writ of habeas corpus would not only be unjustified manifest that the writ is not called for since its purpose is to relieve
but would also preempt the Sandiganbayan's resolution of the pending petitioners from unlawful restraint (Ngaya-an v. Balweg, 200 SCRA 149
application for bail of petitioner. The recourse of petitioner is to [1991]), a matter which remains speculative up to this very day.
forthwith proceed with the hearing on his application for bail.
ISSUE # 2: Whether mandamus is a proper remedy.

HELD # 2: NO.
PANFILO LACSON, MICHAEL RAY B. AQUINO and CESAR O. MANCAO, petitioners, vs. x The petition herein is denominated by petitioner Defensor-Santiago as one for
SECRETARY HERNANDO PEREZ, P/DIRECTOR LEANDRO MENDOZA, and P/SR. SUPT. mandamus. It is basic in matters relating to petitions for mandamus that the legal
REYNALDO BERROYA, respondents. right of the petitioner to the performance of a particular act which is sought to be
G.R. No. 147780 May 10, 2001 compelled must be clear and complete. Mandamus will not issue unless the right to
relief is clear at the time of the award (Palileo v. Ruiz Castro, 85 Phil. 272). Up to
MIRIAM DEFENSOR-SANTIAGO, petitioner, vs. ANGELO REYES, Secretary of National Defense, the present time, petitioner Defensor Santiago has not shown that she is
ET AL., respondents. in imminent danger of being arrested without a
G.R. No. 147781 May 10, 2001 warrant. In point of fact, the authorities have categorically stated that
petitioner will not be arrested without a warrant.
RONALDO A. LUMBAO, petitioner, vs. SECRETARY HERNANDO PEREZ, GENERAL DIOMEDIO
VILLANUEVA, P/DIRECTOR LEANDRO MENDOZA, and P/SR. SUPT. REYNALDO BERROYA,
respondents. DISCOURSE ON HABEAS CORPUS AND WARRANTLESS ARRESTS (Dissenting
G.R. No. 147799 May 10, 2001 Opinion of Justice Kapunan in Lacson vs. Perez)

THE LABAN NG DEMOKRATIKONG PILIPINO, petitioner, vs. THE DEPARTMENT OF JUSTICE,


SECRETARY HERNANDO PEREZ, THE ARMED FORCES OF THE PHILIPPINES, GENERAL x Section 18 grants the President, as Commander-in-Chief, the power to call out
DIOMEDIO VILLANUEVA, THE PHILIPPINE NATIONAL POLICE, and DIRECTOR GENERAL the armed forces in cases of (1) lawless violence, (2) rebellion and (3) invasion.9
LEANDRO MENDOZA, respondents. In the latter two cases, i.e., rebellion or invasion, the President may, when public
G.R. No. 147810 May 10, 2001 safety requires, also (a) suspend the privilege of the writ of habeas corpus, or
EN BANC (b) place the Philippines or any part thereof under martial law. However, in the
exercise of this calling out power as Commander-in-Chief of the armed forces,
FACTS: the Constitution does not require the President to make a declaration of a "state
x President GMA Æ issued Proclamation No. 38 declaring that there was a state of of rebellion" (or, for that matter, of lawless violence or invasion). The term "state
rebellion in the National Capital Region of rebellion" has no legal significance. It is vague and amorphous and does not
o Likewise issued General Order No. 1 directing the Armed Forces of give the President more power than what the Constitution says, i. e, whenever it
the Philippines and the Philippine National Police to suppress the becomes necessary, he may call out such armed forces to prevent or suppress
rebellion in the National Capital Region. Warrantless arrests of lawless violence, invasion or rebellion. As Justice Mendoza observed during the
several alleged leaders and promoters of the "rebellion" were hearing of this case, such a declaration is "legal surplusage." But whatever the
thereafter effected term means, it cannot diminish or violate constitutionally-protected rights, such
ƒ This AROSE from an incident when she was faced by as the right to due process,10 the rights to free speech and peaceful assembly to
an "angry and violent mob armed with explosives, petition the government for redress of grievances,11 and the right against
firearms, bladed weapons, clubs, stones and other unreasonable searches and seizures,12 among others.
deadly weapons" assaulting and attempting to break x In Integrated Bar of the Philippines vs. Zamora, et al.,13 the Court held that:
into Malacañang o x x x [T]he distinction (between the calling out power, on
x This case is a consolidation of 4 petitions, viz: one hand, and the power to suspend the privilege of the
o (1) G. R. No. 147780 for prohibition, injunction, mandamus, and write of habeas corpus and to declare martial law, on the
habeas corpus (with an urgent application for the issuance of other hand) places the calling out power in a different
temporary restraining order and/or writ of preliminary injunction) category from the power to declare martial law and the
filed by Panfilio M. Lacson, Michael Ray B. Aquino, and Cezar O. power to suspend the privilege of the writ of habeas
Mancao; corpus, otherwise, the framers of the Constitution would
o (2) G. R. No. 147781 for mandamus and/or review of the have simply lumped together the three powers and
factual basis for the suspension of the privilege of the writ provided for their revocation and review without any
of habeas corpus, with prayer for the suspension of the qualification. Expressio unius est exclusio alterius.
privilege of the writ of habeas corpus, with prayer for a o The reason for the difference in the treatment of the
temporary restraining order filed by Miriam Defensor- aforementioned powers highlights the intent to grant the President
Santiago; the widest leeway and broadest discretion in using the "calling out"
o (3) G. R. No. 147799 for prohibition and injunction with prayer for power because it is considered as the lesser and more benign
a writ of preliminary injunction and/or restraining order filed by power compared to the power to suspend the privilege of the writ
Ronaldo A. Lumbao; and of habeas corpus and the power to impose martial law, both of
o (4) G. R. No. 147810 for certiorari and prohibition filed by the which involve the curtailment and suppression of certain basic civil
political party Laban ng Demokratikong Pilipino. rights and individual freedoms, and thus necessitating affirmation
x President GMA Æ then ordered the lifting of the declaration of a "state of by Congress and, in appropriate cases, review by this Court.
rebellion" in Metro Manila. x On the other hand, if the motive behind the declaration of a "state of rebellion"
o Accordingly, the instant petitions have been rendered moot and is to arrest persons without warrant and detain them without bail and, thus, skirt
academic the Constitutional safeguards for the citizens' civil liberties, the so-called "state of
x DOJ Sec Æ claimed that the “warrantless arrests” were grounded on general rebellion" partakes the nature of martial law without declaring on its face, yet, if it
instructions to law enforcement officers and military agencies to implement is applied and administered by public authority with an evil eye so as to
Proclamation No. 38 practically make it unjust and oppressive, it is within the prohibition of the
o [I]t is already the declared intention of the Justice Department and Constitution.14 In an ironic sense, a "state of rebellion" declared as a subterfuge
police authorities to obtain regular warrants of arrests from the to effect warrantless arrest and detention for an unbailable offense places a
courts for all acts committed prior to and until May 1, 2001 which heavier burden on the people's civil liberties than the suspension of the privilege
means that preliminary investigations will henceforth be conducted. of the writ of habeas corpus the declaration of martial law because in the latter
case, built-in safeguards are automatically set on motion: (1) The period for
139
RECTO, GAYLE ANGELI M.
2011-0008 | AUSL
Personal Notes on Remedial Law 2 Review (based on the syllabus of Prof. Henedino M. Brondial)

martial law or suspension is limited to a period not exceeding sixty day; (2) The x Petitioners were arrested or sought to be arrested without warrant for acts of
President is mandated to submit a report to Congress within forty-eight hours rebellion ostensibly under Section 5 of Rule 113. Respondents' theory is based on
from the proclamation or suspension; (3) The proclamation or suspension is Umil vs. Ramos,17 where this Court held:
subject to review by Congress, which may revoke such proclamation or o The crimes of rebellion, subversion, conspiracy or proposal to
suspension. If Congress is not in session, it shall convene in 24 hours without commit such crimes, and crimes or offenses committed in
need for call; and (4) The sufficiency of the factual basis thereof or its extension furtherance thereof or in connection therewith constitute direct
is subject to review by the Supreme Court in an appropriate proceeding.15 assault against the State and are in the nature of continuing
x No right is more fundamental than the right to life and liberty. Without these crimes.18
rights, all other individual rights may not exist. Thus, the very first section in our x Following this theory, it is argued that under Section 5(a), a person who "has
Constitution's Bill of Rights, Article III, reads: committed, is actually committing, or is attempting to commit" rebellion and may
o SECTION 1. No person shall be deprived of life, liberty, or property be arrested without a warrant at any time so long as the rebellion persists.
without due process of law, nor shall any person be denied the x Reliance on Umil is misplaced. The warrantless arrests therein, although effected
equal protection of the laws. a day or days after the commission of the violent acts of petitioners therein,
x And to assure the fullest protection of the right, more especially against were upheld by the Court because at the time of their respective arrests, they
government impairment, Section 2 thereof provides: were members of organizations such as the Communist Party of the Philippines,
o SEC. 2. The right of the people to be secure in their persons, the New Peoples Army and the National United Front Commission, then outlawed
houses, papers, and effects against unreasonable searches and groups under the Anti-Subversion Act. Their mere membership in said illegal
seizures of whatever nature and for any purpose shall be inviolable, organizations amounted to committing the offense of subversion19 which
and no search warrant or warrant of arrest shall issue except upon justified their arrests without warrants.
probable cause to be determined personally by the judge after x In contrast, it has not been alleged that the persons to be arrested for their
examination under oath or affirmation of the complainant and the alleged participation in the "rebellion" on May 1, 2001 are members of an
witnesses he may produce, and particularly describing the place to outlawed organization intending to overthrow the government. Therefore, to
be searched and the persons or things to be seized. justify a warrantless arrest under Section 5(a), there must be a showing that the
x Indeed, there is nothing in Section 18 which authorizes the President persons arrested or to be arrested has committed, is actually committing or is
or any person acting under her direction to make unwarranted arrests. attempting to commit the offense of rebellion.20 In other words, there must be
The existence of "lawless violence, invasion or rebellion" only an overt act constitutive of rebellion taking place in the presence of the arresting
authorizes the President to call out the "armed forces to prevent or officer. In United States vs. Samonte,21 the term" in his [the arresting officer's]
suppress lawless violence, invasion or rebellion." presence" was defined thus:
x Not even the suspension of the privilege of the writ of habeas corpus or the o An offense is said to be committed in the presence or within the
declaration of martial law authorizes the President to order the arrest of any view of an arresting officer or private citizen when such officer or
person. The only significant consequence of the suspension of the writ of habeas person sees the offense, even though at a distance, or hears the
corpus is to divest the courts of the power to issue the writ whereby the detention disturbance created thereby and proceeds at once to the scene
of the person is put in issue. It does not by itself authorize the President to order thereof; or the offense is continuing, or has not been
the arrest of a person. And even then, the Constitution in Section 18, Article VII consummated, at the time the arrest is made.2
makes the following qualifications: x This requirement was not complied with particularly in the arrest of Senator
o The suspension of the privilege of the writ shall apply only Enrile. In the Court's Resolution of May 5, 2001 in the petition for habeas corpus
to persons judicially charged for rebellion or offenses filed by Senator Enrile, the Court noted that the sworn statements of the
inherent in or directly connected with invasion. policemen who purportedly arrested him were hearsay.23 Senator Enrile was
o During the suspension of the privilege of the writ, any person thus arrested two (2) days after he delivered allegedly seditious speeches.
arrested or detained shall be judicially charged within three days, Consequently, his arrest without warrant cannot be justified under Section 5(b)
otherwise he shall be released. which states that an arrest without a warrant is lawful when made after an
x In the instant case, the President did not suspend the writ of habeas offense has just been committed and the arresting officer or private person has
corpus. Nor did she declare martial law. A declaration of a "state of probable cause to believe based on personal knowledge of facts and
rebellion," at most, only gives notice to the nation that it exists, and circumstances that the person arrested has committed the offense.
that the armed forces may be called to prevent or suppress it, as in fact x At this point, it must be stressed that apart from being inapplicable to the cases
she did. Such declaration does not justify any deviation from the Constitutional at bar, Umil is not without any strong dissents. It merely re-affirmed Garcia-
proscription against unreasonable searches and seizures. Padilla vs. Enrile,24 a case decided during the Marcos martial law regime.25 It
x As a general rule, an arrest may be made only upon a warrant issued by a court. cannot apply when the country is supposed to be under the regime of freedom
In very circumscribed instances, however, the Rules of Court allow warrantless and democracy. The separate opinions of the following Justices in the motion for
arrests. Section 5, Rule 113 provides: reconsideration of said case26 are apropos:
o SEC. 5. Arrest without warrant; when lawful. - A police officer or a o FERNAN C.J., concurring and dissenting:
private person may, without a warrant, arrest a person: ƒ Secondly, warrantless arrests may not be allowed if
ƒ (a) When, in his presence, the person to be arrested the arresting officers are not sure what particular
has committed, is actually committing, or is provision of law had been violated by the person
attempting to commit an offense; arrested. True it is that law enforcement agents and
ƒ (b) When an offense has just been committed and he even prosecutors are not all adept at the law.
has probable cause to believe based on personal However, erroneous perception, not to mention
knowledge of facts or circumstances that the person ineptitude among their ranks, especially if it would
to be arrested has committed it; and result in the violation of any right of a person, may
o In cases falling under paragraphs (a) and (b) above, the person not be tolerated. That the arrested person has the
arrested without a warrant shall be forthwith delivered to the "right to insist during the pre-trial or trial on the
nearest police station or jail and shall be proceeded against in merits" (Resolution, p. 18) that he was exercising a
accordance with section 7 of Rule 112. right which the arresting officer considered as
x It must be noted that the above are exceptions to the constitutional norm contrary to law, is beside the point. No person should
enshrined in the Bill of Rights that a person may only be arrested on the strength be subjected to the ordeal of a trial just because the
of a warrant of arrest issued by a "judge" after determining "personally" the law enforcers wrongly perceived his action.27
existence of "probable cause" after examination under oath or affirmation of the (Underscoring supplied)
complainant and the witnesses he may produce. Its requirements should, o GUTIERREZ, JR., J., concurring and dissenting opinion
therefore, be scrupulously met: ƒ Insofar as G.R. NO. 81567 is concerned, I joint the
o The right of a person to be secure against any unreasonable other dissenting Justices in their observations
seizure of his body and any deprivation of his liberty is a most basic regarding "continuing offenses." To base warrantless
and fundamental one. The statute or rule which allows exceptions arrests on the doctrine of continuing offense is to give
to the requirement of warrants of arrests is strictly construed. Any a license for the illegal detention of persons on pure
exception must clearly fall within the situations when securing a suspicion. Rebellion, insurrection, or sedition are
warrant would be absurd or is manifestly unnecessary as provided political offenses where the line between overt acts
by the Rule. We cannot liberally construe the rule on arrests and simple advocacy or adherence to a belief is
without warrant or extend its application beyond the cases extremely thin. If a court has convicted an accused of
specifically provided by law. To do so would infringe upon personal rebellion and he is found roaming around, he may be
liberty and set back a basic right so often violated and so deserving arrested. But until a person is proved guilty, I fail to
of full protection.16 see how anybody can jump to a personal conclusion
x A warrantless arrest may be justified only if the police officer had facts and that the suspect is indeed a rebel and must be picked
circumstances before him which, had they been before a judge, would constitute up on sight whenever seen. The grant of authority in
adequate basis for a finding of probable cause of the commission of an offense the majority opinion is too broad. If warrantless
and that the person arrested is probably guilty of committing the offense. That is searches are to be validated, it should be Congress
why the Rules of Criminal Procedure require that when arrested, the person and not this Court which should draw strict and
"arrested has committed, is actually committing, or is attempting to commit an narrow standards. Otherwise, the non-rebels who are
offense" in the presence of the arresting officer. Or if it be a case of an offense critical, noisy, or obnoxious will be indiscriminately
which had "just been committed," that the police officer making the arrest "has lumped up with those actually taking up arms against
personal knowledge of facts or circumstances that the person to be arrested has the Government.
committed it."
140
RECTO, GAYLE ANGELI M.
2011-0008 | AUSL
Personal Notes on Remedial Law 2 Review (based on the syllabus of Prof. Henedino M. Brondial)

ƒ The belief of law enforcement authorities, no matter x It is observed that a sufficient period has lapsed between the fateful day of May
how well-grounded on past events, that the petitioner 1, 2001 up to the present. If respondents have ample evidence against
would probably shoot other policemen whom he may petitioners, then they should forthwith file the necessary criminal complaints in
meet does not validate warrantless arrests. I cannot order that the regular procedure can be followed and the warrants of arrest
understand why the authorities preferred to bide their issued by the courts in the normal course. When practicable, resort to the
time, await the petitioner's surfacing from warrant process is always to be preferred because "it interposes an orderly
underground, and ounce on him with no legal authority procedure involving 'judicial impartiality' whereby a neutral and detached
instead of securing warrants of arrest for his magistrate can make informed and deliberate determinations on the issue of
apprehension.28 (Underscoring supplied) probable cause."31
o CRUZ, J., concurring and dissenting: x The neutrality, detachment and independence that judges are supposed to
ƒ I submit that the affirmation by this Court of the possess is precisely the reason the framers of the 1987 Constitution have
Garcia-Padilla decision to justify the illegal arrests reposed upon them alone the power to issue warrants of arrest. To vest the
made in the cases before us is a step back to that same to a branch of government, which is also charged with prosecutorial
shameful past when individual rights were wantonly powers, would make such branch the accused's adversary and accuser, his judge
and systematically violated by the Marcos and jury.32
dictatorship. It seem some of us have short memories x A declaration of a state of rebellion does not relieve the State of its burden of
of that repressive regime, but I for one am not one to proving probable cause. The declaration does not constitute a substitute for
forget so soon. As the ultimate defender of the proof. It does not in any way bind the courts, which must still judge for itself the
Constitution, this Court should not gloss over the existence of probable cause. Under Section 18, Article VII, the determination of
abuses of those who, out of mistaken zeal, would the existence of a state of rebellion for purposes of proclaiming martial law or
violate individual liberty in the dubious name of the suspension of the privilege of the writ of habeas corpus rests for which the
national security. Whatever their ideology and even if President is granted ample, though not absolute, discretion. Under Section 2,
it be hostile to ours, the petitioners are entitled to the Article III, the determination of probable cause is a purely legal question of
protection of the Bill of Rights, no more and no less which courts are the final arbiters.
than any other person in this country. That is what x Justice Secretary Hernando Perez is reported to have announced that the lifting
democracy is all about.29 (Underscoring supplied) of the "state of rebellion" on May 7, 2001 does not stop the police from making
o FELICIANO, J., concurring and dissenting: warrantless arrests.33 If this is so, the pernicious effects of the declaration on
ƒ 12. My final submission, is that, the doctrine of the people's civil liberties have not abated despite the lifting thereof. No one
"continuing crimes," which has its own legitimate exactly knows who are in the list or who prepared the list of those to be arrested
function to serve in our criminal law jurisprudence, for alleged complicity in the "continuing" crime of "rebellion" defined as such by
cannot be invoked for weakening and dissolving the executive fiat. The list of the perceived leaders, financiers and supporters of the
constitutional guarantee against warrantless arrest. "rebellion" to be arrested and incarcerated could expand depending on the
Where no overt acts comprising all or some of the appreciation of the police. The coverage and duration of effectivity of the orders
elements of the offense charged are shown to have of arrest are thus so open-ended and limitless as to place in constant and
been committed by the person arrested without continuing peril the people's Bill of Rights. It is of no small significance that four
warrant, the "continuing crime" doctrine should not be of he petitioners are opposition candidates for the Senate. Their campaign
used to dress up the pretense that a crime, begun or activities have been to a large extent immobilized. If the arrests and orders of
committed elsewhere, continued to be committed by arrest against them are illegal, then their Constitutional right to seek public
the person arrested in the presence of the office, as well as the right of he people to choose their officials, is violated.
arresting officer. The capacity for mischief of such a x In view of the transcendental importance and urgency of the issues raised in
utilization of the "continuing crimes" doctrine, is these cases affecting as they do the basic liberties of the citizens enshrined in
infinitely increased where the crime charged does not our Constitution, it behooves us to rule thereon now, instead of relegating the
consist of unambiguous criminal acts with a definite cases to trial courts which unavoidably may come up with conflicting
beginning and end in time and space (such as the dispositions, the same to reach this Court inevitably for final ruling. As we aptly
killing or wounding of a person or kidnapping and pronounced in Salonga vs. Cruz Paño:34
illegal detention or arson) but rather or such o The Court also has the duty to formulate guiding and controlling
problematic offenses as membership in or affiliation constitutional principles, precepts, doctrines, or rules. It has the
with or becoming a member of, a subversive symbolic function of educating bench and bar on the extent of
association or organization. For in such cases, the protection given by constitutional guarantees.
overt constitutive acts may be morally neutral in x Petitioners look up in urgent supplication to the Court, considered the last
themselves, and the unlawfulness of the acts a bulwark of democracy, for relief. If we do not act promptly, justly and fearlessly,
function of the aims or objectives of the organization to whom will they turn to?
involved. Note, for instance, the following acts which
constitute prima facie evidence of "membership in any
subversive association:" ANISAH IMPAL SANGCA, petitioner, vs. THE CITY PROSECUTOR OF CEBU CITY and
x a) Allowing himself to be listed as a THE PRESIDING JUDGE, Regional Trial Court, Branch 58, Cebu City, respondents.
member in any book or any of the lists, G.R. No. 175864 June 8, 2007
records, correspondence, or any other THIRD DIVISION
document of the organization;
x b) Subjecting himself to the discipline of FACTS:
such or association or organization in x PDEA Regional Office VII Æ received information that Lovely Impal Adam was
any form whatsoever; engaged in illegal drug trafficking activities in Cebu City and neighboring cities
x c) Giving financial contribution to such and municipalities
association or organization in dues, x PDEA conducted an entrapment operation
assessments, loans or in any other x Adam was then detained in the Cebu City Jail for alleged violation of Section 5,
forms; Article 2 of Republic Act (R.A.) No. 9165, otherwise known as the Dangerous
x f) Conferring with officers or other Drugs Act of 2002
members of such association or x Inquest prosecutor Æ recommended the dismissal of the case
organization in furtherance of any plan x City Prosecutor Æ disapproved
or enterprise thereof; x DOJ Sec Æ found no probable cause to hold Adam liable for the offense charged
x g) Preparing documents, pamphlets, o directed the City Prosecutor of Cebu City to withdraw the
leaflets, books, or any other type of information
publication to promote the objectives ƒ A very thorough and careful scrutiny of the records,
and purposes of such association or particularly the affidavit of arrest, reveals that no
organization; payment was ever made by the police officers for the
x k) Participating in any way in the supposed object of the buy-bust operations. The
activities, planning action, objectives, or police officers have not even alleged in their affidavits
purposes of such association or that payment was made to respondent in exchange
organization. for the shabu. No buy-bust money was ever
x It may well be, as the majority implies, that the constitutional rule against presented. The certificate of inventory does not show
warrantless arrests and seizures makes the law enforcement work of police any buy-bust money. These stick out like a sore
agencies more difficult to carry out. It is not our Court's function, however, and thumb in the case at bar.
the Bill of Rights was not designed, to make life easy for police forces but rather ƒ Suffice it to say that one of the essential elements to
to protect the liberties of private individuals. Our police forces must simply learn be established in the prosecution of the drug "buy-
to live with the requirements of the Bill of Rights, to enforce the law by bust" cases, that is, "the delivery of the thing sold
modalities which themselves comply with the fundamental law. Otherwise they and the payment therefore" is wanting. It was aptly
are very likely to destroy, whether through sheer ineptness or excess of zeal, the said in the case of People v. Alilin, 206 SCRA 773,
very freedoms which make our policy worth protecting and saving.30 that: "To sustain a conviction for selling prohibited
(Underscoring supplied) drugs, the same must be clearly and unmistakably
established."
141
RECTO, GAYLE ANGELI M.
2011-0008 | AUSL
Personal Notes on Remedial Law 2 Review (based on the syllabus of Prof. Henedino M. Brondial)

criminal prosecution and secure freedom. When a prisoner petitions


x PDEA Æ filed an MR for a writ of habeas corpus, he thereby commences a suit and
o DOJ Sec Æ denied prosecutes a case in that court.
x Judge Ingles (PJ of RTC Cebu Br 58) Æ GRANTED the Motion to Withdraw x Habeas corpus is not in the nature of a writ of error; nor intended as
Information and ordered the RELEASE of the accused substitute for the trial court’s function. It cannot take the place of
x Anisah Impal Sangca Æ filed the instant petition before the SC praying for the appeal, certiorari or writ of error. The writ cannot be used to investigate and
issuance of a writ of habeas corpus and the release of Adam consider questions of error that might be raised relating to procedure or on the
merits. The inquiry in a habeas corpus proceeding is addressed to the question
ISSUE: Whether a writ of HC should still be issued considering that RTC Cebu has already of whether the proceedings and the assailed order are, for any reason, null and
ordered Amad’s release. void. The writ is not ordinarily granted where the law provides for other
remedies in the regular course, and in the absence of exceptional circumstances.
HELD: NO. Moreover, habeas corpus should not be granted in advance of trial. The orderly
x A writ of habeas corpus extends to all cases of illegal confinement or detention in course of trial must be pursued and the usual remedies exhausted before
which any person is deprived of his liberty, or in which the rightful custody of resorting to the writ where exceptional circumstances are extant. In another
any person is withheld from the person entitled to it. Its essential object and case, it was held that habeas corpus cannot be issued as a writ of error or as a
purpose is to inquire into all manner of involuntary restraint and to relieve a means of reviewing errors of law and irregularities not involving the questions of
person from it if such restraint is illegal. The singular function of a petition jurisdiction occurring during the course of the trial, subject to the caveat that
for habeas corpus is to protect and secure the basic freedom of constitutional safeguards of human life and liberty must be preserved, and not
physical liberty.7 destroyed. It has also been held that where restraint is under legal process,
x In the instant case, records show that Adam has been released upon mere errors and irregularities, which do not render the proceedings void, are not
order of the trial judge on January 26, 2007. Therefore, the petition has grounds for relief by habeas corpus because in such cases, the restraint is not
become moot illegal.
x Habeas corpus is a summary remedy. It is analogous to a proceeding in
rem when instituted for the sole purpose of having the person of restraint
ANITA MANGILA, Petitioner, vs. JUDGE HERIBERTO M. PANGILINAN, ASST. CITY presented before the judge in order that the cause of his detention may be
PROSECUTOR II LUCIA JUDY SOLINAP, and NATIONAL BUREAU OF INVESTIGATION inquired into and his statements final. The writ of habeas corpus does not act
(DIRECTOR REYNALDO WYCOCO), Respondents. upon the prisoner who seeks relief, but upon the person who holds him
G.R. No. 160739 July 17, 2013 in what is alleged to be the unlawful authority. Hence, the only parties
FIRST DIVISION before the court are the petitioner (prisoner) and the person holding the
petitioner in custody, and the only question to be resolved is whether the
FACTS: custodian has authority to deprive the petitioner of his liberty. The writ may be
x Mangila (and 7 others) Æ were charged with syndicated estafa in violation of denied if the petitioner fails to show facts that he is entitled thereto ex merito
Article 315 of the Revised Penal Code AND with violations of Section 7(b) of justicias.
Republic Act No. 8042 (Migrant Workers and Overseas Filipino Act of 1995) filed x A writ of habeas corpus, which is regarded as a "palladium of liberty," is a
before MTCC Puerto Princesa prerogative writ which does not issue as a matter of right but in the
o The complaints arose from the recruiting and promising of sound discretion of the court or judge. It is, however, a writ of right on
employment by Mangila and the others to the private complainants proper formalities being made by proof. Resort to the writ is not to inquire
as overseas contract workers in Toronto, Canada, and from the into the criminal act of which a complaint is made but unto the right of liberty,
collection of visa processing fees, membership fees and on-line notwithstanding the act, and the immediate purpose to be served is relief from
application the private complainants without lawful authority from illegal restraint. The primary, if not the only object of the writ of habeas corpus
the Philippine Overseas Employment Administration (POEA) ad subjuciendum, is to determine the legality of the restraint under which a
x Judge P Æ conducted PI on June 17, 2003 person is held.11 (Bold underscoring supplied for emphasis)
o Found probable cause x The object of the writ of habeas corpus is to inquire into the legality of the
o Issued warrants of arrest against Mangila et al without bail detention, and, if the detention is found to be illegal, to require the release of
x Mangila Æ arrested and detained in NBI on June 18, 2003 the detainee. Equally well-settled however, is that the writ will not issue
x Mangila Æ then filed a petition for HC before the SC where the person in whose behalf the writ is sought is out on bail, or is
o Claiming that Judge Pangilinan did not have the authority to in the custody of an officer under process issued by a court or judge
conduct the preliminary investigation; that the preliminary with jurisdiction or by virtue of a judgment or order of a court of
investigation he conducted was not yet completed when he issued record.12
the warrant of arrest; and that the issuance of the warrant of arrest x There is no question that when the criminal complaints were lodged against
was without sufficient justification or without a prior finding of Mangila and her cohorts on June 16, 2003,Judge Pangilinan, as the Presiding
probable cause Judge of the MTCC, was empowered to conduct preliminary investigations
x CA Æ denied petition involving "all crimes cognizable by the proper court in their respective territorial
o As a general rule, a writ of habeas corpus will not be granted jurisdictions." His authority was expressly provided in Section 2, Rule 112 of the
where relief may be had or could have been procured by resort to Revised Rules of Criminal Procedure, to wit:
another general remedy. As pointed out in Luna vs. Plaza, if o Section 2.Officers authorized to conduct preliminary investigations.
petitioner is detained by virtue of a warrant of arrest, which is - The following may conduct preliminary investigations:
allegedly invalid, the remedy available to her is not a petition for ƒ (a) Provincial or City Prosecutors and their assistants;
habeas corpus but a petition to quash the warrant of arrest or a ƒ (b) Judges of the Municipal Trial Courts and Municipal
petition for a reinvestigation of the case by the Municipal Judge or Circuit Trial Courts;
by the Provincial Fiscal. ƒ (c) National and Regional State Prosecutors; and
o Section 5, Rule 112 of the Revised Rules of Criminal Procedure ƒ (d) Other officers as may be authorized by law.
provides that the Municipal Judge who conducted the preliminary o Their authority to conduct preliminary investigations shall include
investigation shall transmit his resolution, together with the record all crimes cognizable by the proper court in their respective
of the case, including the warrant of arrest, to the Provincial territorial jurisdictions. (2a)
Prosecutor, who shall review the same and order the release of an x Under Section 6(b) of Rule 112of the Revised Rules of Criminal
accused who is detained if no probable cause is found against him. Procedure, the investigating judge could issue a warrant of arrest
Thus, the proper remedy available to petitioner is for her to file during the preliminary investigation even without awaiting its
with the Provincial Prosecutor a motion to be released from conclusion should he find after an examination in writing and under
detention on the grounds alleged in the instant petition. oath of the complainant and the witnesses in the form of searching
questions and answers that a probable cause existed, and that there
ISSUE: Whether a petition for HC is the proper remedy in the present case considering that was a necessity of placing the respondent under immediate custody in
Mangila was detained by virtue of a warrant of arrest issued by MTCC Puerto Princesa order not to frustrate the ends of justice.1âwphi1 In the context of this
rule, Judge Pangilinan issued the warrant of arrest against Mangila and
HELD: NO. her cohorts. Consequently, the CA properly denied Mangila’s petition for habeas
x Restraint that is lawful and pursuant to a court process cannot be inquired into corpus because she had been arrested and detained by virtue of the warrant
through habeas corpus. issued for her arrest by Judge Pangilinan, a judicial officer undeniably possessing
x The high prerogative writ of habeas corpus has been devised as a the legal authority to do so.
speedy and effective remedy to relieve persons from unlawful restraint. x It is relevant to point out at this juncture that the authority of the MTC
In Caballes v. Court of Appeals,10 the Court discoursed on the nature of the and MTCC judges to conduct preliminary investigations was removed
special proceeding of habeas corpus in the following manner: only effective on October 3, 2005 pursuant to A.M. No. 05-8-26-SC.
o A petition for the issuance of a writ of habeas corpus is a special x With Mangila’s arrest and ensuing detention being by virtue of the order lawfully
proceeding governed by Rule 102 of the Rules of Court, as issued by Judge Pangilinan, the writ of habeas corpus was not an appropriate
amended. In Ex Parte Billings, it was held that habeas remedy to relieve her from the restraint on her liberty. This is because the
corpus is that of a civil proceeding in character. It seeks the restraint, being lawful and pursuant to a court process, could not be
enforcement of civil rights. Resorting to the writ is not to inquired into through habeas corpus. To quote the dictum enunciated by
inquire into the criminal act of which the complaint is Justice Malcolm in Quintos v. Director of Prisons:13
made, but into the right of liberty, notwithstanding the act o The writ of habeas corpus secures to a prisoner the right to have
and the immediate purpose to be served is relief from the cause of his detention examined and determined by a court of
illegal restraint. The rule applies even when instituted to arrest a justice, and to have ascertained if he is held under lawful authority.
142
RECTO, GAYLE ANGELI M.
2011-0008 | AUSL
Personal Notes on Remedial Law 2 Review (based on the syllabus of Prof. Henedino M. Brondial)

The function of habeas corpus, where the party who has appealed to least once a week for three (3) successive weeks in some newspaper of general circulation
its aid is in custody under process, does not extend beyond an published in the province, as the court shall deem best. The date set for the hearing shall not
inquiry into the jurisdiction of the court by which it was issued and be within thirty (30) days prior to an election nor within four (4) month after the last
the validity of the process upon its face. It is not a writ of error. xxx publication of the notice.
(Bold underscoring supplied for emphasis)
x Accordingly, Section 4, Rule 102 of the Rules of Court explicitly states: Section 4. Hearing. — Any interested person may appear at the hearing and oppose the
o Section 4.When writ not allowed or discharge authorized. — If it petition. The Solicitor General or the proper provincial or city fiscal shall appear on behalf of
appears that the person alleged to be restrained of his liberty is in the Government of the Republic.
the custody of an officer under process issued by a court or judge
or by virtue of a judgment or order of a court of record, and that Section 5. Judgment. — Upon satisfactory proof in open court on the date fixed in the
the court or judge had jurisdiction to issue the process, render the order that such order has been published as directed and that the allegations of the petition
judgment, or make the order, the writ shall not be allowed; or if are true, the court shall, if proper and reasonable cause appears for changing the name of the
the jurisdiction appears after the writ is allowed, the person shall petitioner, adjudge that such name be changed in accordance with the prayer of the petition.
not be discharged by reason of any informality or defect in the
process, judgment, or order. Nor shall anything in this rule be held Section 6. Service of judgment. — Judgments or orders rendered in connection with this
to authorize the discharge of a person charged with or convicted of rule shall be furnished the civil registrar of the municipality or city where the court issuing the
an offense in the Philippines, or of a person suffering imprisonment same is situated, who shall forthwith enter the same in the civil register.
under lawful judgment. (Bold underscoring supplied for emphasis)
x Still, Mangila harps on the procedural flaws supposedly committed by Judge RULE 108
Pangilinan in her attempt to convince the Court on her entitlement to the issuance Cancellation Or Correction Of Entries In The Civil Registry
of the writ of habeas corpus. She insists that the illegality and invalidity of the
warrant of arrest because of its having been issued without an exhaustive Section 1. Who may file petition. — Any person interested in any act, event, order or
examination of the complainants and the witnesses in writing and under oath; decree concerning the civil status of persons which has been recorded in the civil register, may
without a prior finding of probable cause; and without consideration of the file a verified petition for the cancellation or correction of any entry relating thereto, with the
necessity for its issuance in order not to frustrate the ends of justice were enough Court of First Instance of the province where the corresponding civil registry is located.
reasons for granting the writ of habeas corpus.14
o Mangila fails to persuade. Section 2. Entries subject to cancellation or correction. — Upon good and valid
x To begin with, Judge Pangilinan issued the order of arrest after examining grounds, the following entries in the civil register may be cancelled or corrected: (a) births: (b)
Palayon, one of the complainants against Mangila and her cohorts. If he, as the marriage; (c) deaths; (d) legal separations; (e) judgments of annulments of marriage; (f)
investigating judge, considered Palayon’s evidence sufficient for finding probable judgments declaring marriages void from the beginning; (g) legitimations; (h) adoptions; (i)
cause against her and her cohorts, which finding the Court justifiably presumes acknowledgments of natural children; (j) naturalization; (k) election, loss or recovery of
from his act of referring the case and its records to the Office of the City citizenship; (l) civil interdiction; (m) judicial determination of filiation; (n) voluntary
Prosecutor on the day immediately following the preliminary investigation he emancipation of a minor; and (o) changes of name.
conducted, her petition for habeas corpus could not be the proper remedy by
which she could assail the adequacy of the adverse finding. Even granting that Section 3. Parties. — When cancellation or correction of an entry in the civil register is
there was a failure to adhere to the law or rule, such failure would not be the sought, the civil registrar and all persons who have or claim any interest which would be
equivalent of a violation of her constitutional rights.15 affected thereby shall be made parties to the proceeding.
x Secondly, it was not procedurally correct for her to impugn the issuance of the
warrant of arrest by hinting that the investigating judge did not at all consider Section 4. Notice and publication. — Upon the filing of the petition, the court shall, by
the necessity of determining the existence of probable cause for its issuance due an order, fix the time and place for the hearing of the same, and cause reasonable notice
to time constraints and in order not to frustrate the ends of justice, for that thereof to be given to the persons named in the petition. The court shall also cause the order
consideration was presumed. to be published once a week for three (3) consecutive weeks in a newspaper of general
x And, lastly, it was clear that under Section 5,16 Rule 112 of the Revised Rules of circulation in the province.
Criminal Procedure, the resolution of the investigating judge was not final but
was still subject to the review by the public prosecutor who had the power to Section 5. Opposition. — The civil registrar and any person having or claiming any interest
order the release of the detainee if no probable cause should beultimately found under the entry whose cancellation or correction is sought may, within fifteen (15) days from
against her. In the context of the rule, Mangila had no need to seek the issuance notice of the petition, or from the last date of publication of such notice, file his opposition
of the writ of habeas corpus to secure her release from detention. Her proper thereto.
recourse was to bring the supposed irregularities attending the
conduct of the preliminary investigation and the issuance of the Section 6. Expediting proceedings. — The court in which the proceeding is brought may
warrant for her arrest to the attention of the City Prosecutor, who had make orders expediting the proceedings, and may also grant preliminary injunction for the
been meanwhile given the most direct access to the entire records of preservation of the rights of the parties pending such proceedings.
the case, including the warrant of arrest, following Judge Pangilinan’s
transmittal of them to the City Prosecutor for appropriate action.17 We Section 7. Order. — After hearing, the court may either dismiss the petition or issue an
agree with the CA, therefore, that the writ of habeas corpus could not order granting the cancellation or correction prayed for. In either case, a certified copy of the
be used as a substitute for another available remedy. judgment shall be served upon the civil registrar concerned who shall annotated the same in
his record.

CHANGE OF NAME [RULE 103] vs. CORRECTION/ CANCELLATION OF R.A. 9048


ENTRIES [RULE 108]
REPUBLIC ACT NO. 9048 March 22, 2001
1. Venue vs. Jurisdiction
2. Contents of Petition/ Grounds AN ACT AUTHORIZING THE CITY OR MUNICIPAL CIVIL REGISTRAR OR THE CONSUL
3. Hearing GENERAL TO CORRECT A CLERICAL OR TYPOGRAPHICAL ERROR IN AN ENTRY
AND/OR CHANGE OF FIRST NAME OR NICKNAME IN THE CIVIL REGISTER
4. Judgment
WITHOUT NEED OF A JUDICIAL ORDER, AMENDING FOR THIS PURPOSE ARTICLES
5. R.A. 9048 (as amended by R.A. 10172) and IRRs 376 AND 412 OF THE CIVIL CODE OF THE PHILIPPINES

RULE 103 Be it enacted by the Senate and the House of Representatives of the Philippines in Congress
Change of Name assembled:

Section 1. Venue. — A person desiring to change his name shall present the petition to the Section 1. Authority to Correct Clerical or Typographical Error and Change of First
Court of First Instance of the province in which he resides, or, in the City of Manila, to the Name or Nickname - No entry in a civil register shall be changed or corrected without a
Juvenile and Domestic Relations Court. judicial order, except for clerical or typographical errors and change of first name or nickname
which can be corrected or changed by the concerned city or municipal civil registrar or consul
Section 2. Contents of petition. — A petition for change of name shall be signed and general in accordance with the provisions of this Act and its implementing rules and
verified by the person desiring his name changed, or some other person on his behalf, and regulations.
shall set forth:
Section 2. Definition of Terms - As used in this Act, the following terms shall mean:
(a) That the petitioner has been a bona fide resident of the province where the
petition is filed for at least three (3) years prior to the date of such filing; (1) "City or Municipal civil registrar" refers to the head of the local civil registry office of the
city or municipality, as the case may be, who is appointed as such by the city or municipal
(b) The cause for which the change of the petitioner's name is sought; mayor in accordance with the provisions of existing laws.

(c) The name asked for. (2) "Petitioner" refers to a natural person filing the petition and who has direct and personal
interest in the correction of a clerical or typographical error in an entry or change of first name
Section 3. Order for hearing. — If the petition filed is sufficient in form and substance, or nickname in the civil register.
the court, by an order reciting the purpose of the petition, shall fix a date and place for the
hearing thereof, and shall direct that a copy of the order be published before the hearing at

143
RECTO, GAYLE ANGELI M.
2011-0008 | AUSL
Personal Notes on Remedial Law 2 Review (based on the syllabus of Prof. Henedino M. Brondial)

(3) "Clerical or typographical error" refers to a mistake committed in the performance of


clerical work in writing, copying, transcribing or typing an entry in the civil register that is Section 7. Duties and Powers of the Civil Registrar General. - The civil registrar
harmless and innocuous, such as misspelled name or misspelled place of birth or the like, general shall, within ten (10) working days from receipt of the decision granting a petition,
which is visible to the eyes or obvious to the understanding, and can be corrected or changed exercise the power to impugn such decision by way of an objection based on the following
only by reference to other existing record or records: Provided, however, That no correction grounds:
must involve the change of nationality, age, status or sex of the petitioner.
(1) The error is not clerical or typographical;
(4) "Civil Register" refers to the various registry books and related certificates and documents
kept in the archives of the local civil registry offices, Philippine Consulates and of the Office of (2) The correction of an entry or entries in the civil register is substantial or controversial as it
the Civil Registrar General. affects the civil status of a person; or

(5) "Civil registrar general" refers to the Administrator of the National Statistics Office which is (3) The basis used in changing the first name or nickname of a person does not fall under
the agency mandated to carry out and administer the provision of laws on civil registration. Section 4.

(6) "First name" refers to a name or nickname given to a person which may consist of one or The civil registrar general shall immediately notify the city or municipal civil registrar or the
more names in addition to the middle and last names. consul general of the action taken on the decision. Upon receipt of the notice thereof, the city
or municipal civil registrar or the consul general shall notify the petitioner of such action.
Section 3. Who May File the Petition and Where. - Any person having direct and personal
interest in the correction of a clerical or typographical error in an entry and/or change of first The petitioner may seek reconsideration with the civil registrar general or file the appropriate
name or nickname in the civil register may file, in person, a verified petition with the local petition with the proper court.
civil registry office of the city or municipality where the record being sought to be
corrected or changed is kept. If the civil registrar general fails to exercise his power to impugn the decision of the city or
municipal civil registrar or of the consul general within the period prescribed herein, such
In case the petitioner has already migrated to another place in the country and it would not be decision shall become final and executory.
practical for such party, in terms of transportation expenses, time and effort to appear in
person before the local civil registrar keeping the documents to be corrected or changed, the Where the petition is denied by the city or municipal civil registrar or the consul general, the
petition may be filed, in person, with the local civil registrar of the place where the interested petitioner may either appeal the decision to the civil registrar general or file the appropriate
party is presently residing or domiciled. The two (2) local civil registrars concerned will then petition with the proper court.
communicate to facilitate the processing of the petition.
Section 8. Payment of Fees. - The city or municipal civil registrar or the consul general
Citizens of the Philippines who are presently residing or domiciled in foreign countries may file shall be authorized to collect reasonable fees as a condition for accepting the petition. An
their petition, in person, with the nearest Philippine Consulates. indigent petitioner shall be exempt from the payment of the said fee.

The petitions filed with the city or municipal civil registrar or the consul general shall be Section 9. Penalty Clause. - A person who violates any of the provisions of this Act shall,
processed in accordance with this Act and its implementing rules and regulations. upon conviction, be penalized by imprisonment of not less than six (6) years but not more
than twelve (12) years, or a fine of not less than Ten thousand pesos (P10,000.00) but not
All petitions for the clerical or typographical errors and/or change of first names or nicknames more than One Hundred Thousand pesos (P100,000.00), or both, at the discretion of the
may be availed of only once. court.

Section 4. Grounds for Change of First Name or Nickname. - The petition for change of In addition, if the offender is a government official or employee he shall suffer the penalties
first name or nickname may be allowed in any of the following cases: provided under civil service laws, rules and regulations.

(1) The petitioner finds the first name or nickname to be ridiculous, tainted with dishonor or Section 10. Implementing Rules and Regulations. - The civil registrar general shall, in
extremely difficult to write or pronounce. consultation with the Department of Justice, the Department of Foreign Affairs, the Office of
the Supreme Court Administrator, the University of the Philippines Law Center and the
(2) The new first name or nickname has been habitually and continuously used by the Philippine Association of Civil Registrars, issue the necessary rules and regulations for the
petitioner and he has been publicly known by that by that first name or nickname in the effective implementation of this Act not later than three (3) months from the effectivity of this
community: or law.

(3) The change will avoid confusion. Section 11. Retroactivity Clause. - This Act shall have retroactive effect insofar as it does
not prejudice or impair vested or acquired rights in accordance with the Civil Code and other
Section 5. Form and Contents of the Petition. - The petition shall be in the form of an laws.
affidavit, subscribed and sworn to before any person authorized by the law to administer
oaths. The affidavit shall set forth facts necessary to establish the merits of the petition and Section 12. Separability Clause. - If any portion or provision of this Act is declared void or
shall show affirmatively that the petitioner is competent to testify to the matters stated. The unconstitutional, the remaining portions or provisions thereof shall not be affected by such
petitioner shall state the particular erroneous entry or entries, which are sought to be declaration.
corrected and/or the change sought to be made.
Section 13. Repealing Clause - All laws, decrees, orders, rules and regulations, other
The petition shall be supported with the following documents: issuances, or parts thereof inconsistent with the provisions of this Act are hereby repealed or
modified accordingly.
(1) A certified true machine copy of the certificate or of the page of the registry book
containing the entry or entries sought to be corrected or changed. Section 14. Effectivity Clause. - This Act shall take effect fifteen (15) days after its
complete publication in at least two (2) national newspapers of general circulation.
(2) At least two (2) public or private documents showing the correct entry or entries upon
which the correction or change shall be based; and Approved: March 22, 2001

(3) Other documents which the petitioner or the city or municipal civil registrar or the consul R.A. 10172
general may consider relevant and necessary for the approval of the petition.
REPUBLIC ACT NO. 10172
In case of change of first name or nickname, the petition shall likewise be supported with the
documents mentioned in the immediately preceding paragraph. In addition, the petition shall
AN ACT FURTHER AUTHORIZING THE CITY OR MUNICIPAL CIVIL REGISTRAR OR
be published at least once a week for two (2) consecutive weeks in a newspaper of general
THE CONSUL GENERAL TO CORRECT CLERICAL OR TYPOGRAPHICAL ERRORS IN
circulation. Furthermore, the petitioner shall submit a certification from the appropriate law
THE DAY AND MONTH IN THE DATE OF BIRTH OR SEX OF A PERSON APPEARING IN
enforcement agencies that he has no pending case or no criminal record.
THE CIVIL REGISTER WITHOUT NEED OF A JUDICIAL ORDER, AMENDING FOR
THIS PURPOSE REPUBLIC ACT NUMBERED NINETY FORTY-EIGHT
The petition and its supporting papers shall be filed in three (3) copies to be distributed as
follows: first copy to the concerned city or municipal civil registrar, or the consul general;
Be it enacted by the Senate and House of Representatives of the Philippines in Congress
second copy to the Office of the Civil Registrar General; and third copy to the petitioner.
assembled:
Section 6. Duties of the City or Municipal Civil Registrar or the Consul General. -
Section 1. Section 1 of Republic Act No. 9048, hereinafter referred to as the Act, is hereby
The city or municipal civil registrar or the consul general to whom the petition is presented
amended to read as follows:
shall examine the petition and its supporting documents. He shall post the petition in a
conspicuous place provided for that purpose for ten (10) consecutive days after he finds the
"SECTION 1. Authority to Correct Clerical or Typographical Error and Change of First
petition and its supporting documents sufficient in form and substance.
Name or Nickname. - No entry in a civil register shall be changed or corrected without a
judicial order, except for clerical or typographical errors and change of first name or nickname,
The city or municipal civil registrar or the consul general shall act on the petition and shall
the day and month in the date of birth or sex of a person where it is patently clear that there
render a decision not later than five (5) working days after the completion of the posting
was a clerical or typographical error or mistake in the entry, which can be corrected or changed
and/or publication requirement. He shall transmit a copy of his decision together with the
by the concerned city or municipal civil registrar or consul general in accordance with the
records of the proceedings to the Office of the Civil Registrar General within five (5) working
provisions of this Act and its implementing rules and regulations."
days from the date of the decision.
144
RECTO, GAYLE ANGELI M.
2011-0008 | AUSL
Personal Notes on Remedial Law 2 Review (based on the syllabus of Prof. Henedino M. Brondial)

Section 2. Section 2, paragraph (3) of the Act is likewise amended to read as follows:

"SEC. 2. Definition of Terms. - As used in this Act, the following terms shall mean:

(1) xxx xxx IMPLEMENTING RULES AND REGULATIONS

(2) xxx xxx ADMINISTRATIVE ORDER NO. 1, SERIES OF 2001

(3) ‘Clerical or typographical error’ refers to a mistake committed in the performance of clerical Subject: RULES AND REGULATIONS GOVERNING THE IMPLEMENTATION OF
work in writing, copying, transcribing or typing an entry in the civil register that is harmless REPUBLIC ACT NO. 9048
and innocuous, such as misspelled name or misspelled place of birth, mistake in the entry of
day and month in the date of birth or the sex of the person or the like, which is visible to the Pursuant to Section 10 of R.A. No. 9048, which took effect on 22 April 2001, the following rules
eyes or obvious to the understanding, and can be corrected or changed only by reference to and regulations are hereby promulgated for the information, guidance and compliance of all
other existing record or records: Provided, however, That no correction must involve the concerned parties.
change of nationality, age, or status of the petitioner."
PRELIMINARY STATEMENT
Section 3. Section 5 of the Act is hereby amended to read as follows:
Article 376 of the Civil Code provides that "No person can change his name or surname
"SEC. 5. Form and Contents of the Petition. - The petition for correction of a clerical or without judicial authority.". Article 412 of the same Code provides that "No entry in a civil
typographical error, or for change of first name or nickname, as the case may be, shall be in register shall be changed or corrected, without a judicial order.".
the form of an affidavit, subscribed and sworn to before any person authorized by law to
administer oaths. The affidavit shall set forth facts necessary to establish the merits of the Republic Act No. 9048 amended Articles 376 and 412. Section 1 of this amendatory law
petition and shall show affirmatively that the petitioner is competent to testify to the matters provides: "No entry in a civil register shall be changed or corrected without a judicial order,
stated. The petitioner shall state the particular erroneous entry or entries, which are sought to except for clerical or typographical errors and change of first name or nickname which can be
be corrected and/or the change sought to be made. corrected or changed by the concerned city or municipal civil registrar or consul general in
accordance with the provisions of this Act and its implementing rules and regulations.".
The petition shall be supported with the following documents:
As provided under Section 10 of Republic Act No. 9048, the Civil Registrar General
(1) A certified true machine copy of the certificate or of the page of the registry book promulgated these rules and regulations, in consultation with the representatives from the
containing the entry or entries sought to be corrected or changed; Department of Justice, Department of Foreign Affairs, Office of the Supreme Court
Administrator, University of the Philippines Law Center and Philippine Association of Civil
(2) At least two (2) public or private documents showing the correct entry or entries upon Registrars.
which the correction or change shall be based; and
IMPLEMENTING RULES AND REGULATIONS
(3) Other documents which the petitioner or the city or municipal civil registrar or the consul
general may consider relevant and necessary for the approval of the petition. Rule 1. Authority to correct clerical or typographical error and to change first name
or nickname. - The city/municipal civil registrar, Consul General, including the Clerk of the
No petition for correction of erroneous entry concerning the date of birth or the sex of a Shari'a Court in his capacity as District or Circuit Registrar of Muslim Marriages, Divorces,
person shall be entertained except if the petition is accompanied by earliest school record or Revocations of Divorces and Conversions, are hereby authorized to correct clerical or
earliest school documents such as, but not limited to, medical records, baptismal certificate typographical error and to change first name or nickname in the civil register.
and other documents issued by religious authorities; nor shall any entry involving change of
gender corrected except if the petition is accompanied by a certification issued by an Rule 2. Definition of terms. - As used in this Order, the following terms shall mean:
accredited government physician attesting to the fact that the petitioner has not undergone
sex change or sex transplant. The petition for change of first name or nickname, or for 2.1. City or Municipal Civil Registrar (C/MCR) - Refers to the head of the local civil
correction of erroneous entry concerning the day and month in the date of birth or the sex of registry office (LCRO) of the city or municipality, as the case may be, who is appointed by the
a person, as the case may be, shall be published at least once a week for two (2) consecutive city or municipal mayor in accordance with the provisions of existing laws.
weeks in a newspaper of general circulation.
2.2. Consul General (CG) - Refers to an official of the Department of Foreign Affairs
Furthermore, the petitioner shall submit a certification from the appropriate law enforcements, who has been issued the consular commissions by the President and/or the Secretary of
agencies that he has no pending case or no criminal record. Foreign Affairs. In a foreign service establishment of the Philippines where there is no Consul
General, the civil registration function and duties herein provided for the Consul General shall
The petition and its supporting papers shall be filed in three (3) copies to be distributed as be exercised and performed by the Consul or Vice Consul who should be similarly issued
follows: first copy to the concerned city or municipal civil registrar, or the consul general; consular commissions by the President and/or the Secretary of Foreign Affairs.
second copy to the Office of the Civil Registrar General; and third copy to the petitioner."
2.3. District/Circuit Registrar (D/CR) - Refers to the Clerk of the Shari'a District or
Section 4. Section 8 of the Act is hereby amended to read as follows: Circuit Court acting in the performance of its civil registration function with regard to Muslim
Marriages, Divorces, Revocations of Divorces and Conversions under Title VI, Book Two of
"SEC. 8. Payment of Fees. - The city or municipal civil registrar or the consul general shall Presidential Decree No. 1083 which is otherwise known as the Code of Muslim Personal Laws.
be authorized to collect reasonable fees as a condition for accepting the petition. An indigent
petitioner shall be exempt from the payment of the said fee. 2.4. Civil Registrar General (CRG) - Refers to the Administrator of the National
Statistics Office (NSO) which is the agency mandated to carry out and administer the
The fees collected by the city or municipal civil registrar or the consul general pursuant to this provisions of laws on civil registration.
Act shall accrue to the funds of the Local Civil Registry Office concerned or the Office of the
Consul General for modernization of the office and hiring of new personnel and procurement of 2.5. Local Civil Registry Office (LCRO) - Refers to an office or department in the city
supplies, subject to government accounting and auditing rules." or municipal government that is mandated to perform civil registration function.

Section 5. Separability Clause. - If any provision of this Act shall at any time be found to 2.6. Petitioner - Refers to a natural person filing the petition and who has direct and
be unconstitutional or invalid, the remainder thereof not affected by such declaration shall personal interest in the correction of a clerical or typographical error in an entry or change of
remain in full force and effect. first name or nickname in the civil register.

Section 6. Repealing Clause. - Any laws, decrees, rules or regulations inconsistent with the 2.7. Indigent petitioner - Refers to a destitute, needy and poor individual who is
provisions of this Act are hereby repealed or modified accordingly. certified as such by the social welfare and development office of the city/municipal
government.
Section 7. Effectivity Clause. - This Act shall take effect fifteen (15) days after its
publication in the Official Gazette or in at least two (2) newspapers of general circulation. 2.8. Clerical or typographical error - Refers to a mistake committed in the
performance of clerical work in writing, copying, transcribing or typing an entry in the civil
Approved: AUG 15 2012 register that is harmless and innocuous, such as misspelled name or misspelled place of birth
or the like, which is visible to the eyes or obvious to the understanding, and can be corrected
or changed only by reference to other existing record or records: Provided, however, That no
correction must involve the change of nationality, age, status or sex of the petitioner.

2.9. First name - Refers to the name or nickname given to a person which may
consist of one or more names in addition to the middle and last names.

2.10. Civil Register - Refers to the various registry books and related certificates and
documents kept in the archives of the LCROs, Philippine Consulates, Office of the Civil
Registrar General, and Shari'a District/Circuit Courts.

145
RECTO, GAYLE ANGELI M.
2011-0008 | AUSL
Personal Notes on Remedial Law 2 Review (based on the syllabus of Prof. Henedino M. Brondial)

2.11. Newspaper of general circulation - Refers to a newspaper that is published for the 5.7 Act on the petition within five (5) working days after the completion of the
dissemination of local news and general information; that has a bona fide subscription list of posting and/or publication requirement. In case the C/MCR, CG or D/CR approves the petition,
paying subscribers; and that is published at regular intervals. he shall render his decision in a prescribed form in triplicate copies, indicating therein the entry
sought to be corrected or the first name sought to be changed in the civil register, and the
2.12. Record-keeping civil registrar (RKCR) - Refers to the C/MCR in whose archive is kept the corresponding correction or change made.
record, which contains the error to be corrected or the first name to be changed. This term
shall be used only in cases involving migrant petitioner. 5.8 Deny the petition for correction of clerical or typographical error based on any of
the following grounds:
2.13. Petition-receiving civil registrar (PRCR) - Refers to the C/MCR of the city or municipality
where the petitioner resides or is domiciled and who receives the petition on behalf of the 5.8.1. The supporting documents are not authentic and genuine.
RKCR in the case of a migrant petitioner.
5.8.2. The C/MCR has personal knowledge that a similar petition is filed or pending in court or
2.14. Migrant petitioner (MP) - Refers to a petitioner whose present residence or domicile is in any other LCRO.
different from the place where the civil registry record to be corrected was registered.
5.8.3. The petition involves the same entry in the same document, which was previously
2.15. Spouse - Refers to one's legal wife or legal husband. corrected or changed under this Order.

2.16. Guardian - Refers to a person lawfully invested with the power, and charged with the 5.8.4. The petition involves the change of the status, sex, age or nationality of the petitioner
duty, of taking care of the person and managing the property and rights of another person, or of any person named in the document.
who, for defect of age, understanding, or self-control, is considered incapable of administering
his own affairs. This term may refer also to those who, under Article 216 of the Family Code, 5.8.5. Such other grounds as the C/MCR may deem not proper for correction.
are authorized to exercise substitute parental authority over the child in default of parents or a
judicially appointed guardian. These persons are the following: 5.9. In the case of petition for change of first name or nickname, the C/MCR shall
deny the petition based on any of the following grounds, in addition to Rule 5.8.1 to Rule
2.16.1. The surviving grandparent, as provided in Article 214 of the Family Code; 5.8.3:

2.16.2. The oldest brother or sister, over twenty-one years of age, unless unfit or disqualified; 5.9.1. The first name or nickname sought to be changed is neither ridiculous, nor tainted with
and dishonor nor extremely difficult to write or pronounce.

2.16.3. The child's actual custodian, over twenty-one years of age, unless unfit or disqualified. 5.9.2. The new first name or nickname sought to be adopted has not been habitually and
continuously used by the petitioner, and he has not been publicly known by that first name or
Rule 3. Who may file the petition. - Any person of legal age, having direct and personal nickname in the community.
interest in the correction of a clerical or typographical error in an entry and/or change of first
name or nickname in the civil register, may file the petition. A person is considered to have 5.9.3. There is no confusion to be avoided or created with the use of the registered first name
direct and personal interest when he is the owner of the record, or the owner's spouse, or nickname of the petitioner.
children, parents, brothers, sisters, grandparents, guardian, or any other person duly
authorized by law or by the owner of the document sought to be corrected: Provided, 5.10. Record the decision in the appropriate record book as mentioned in Rule 5.6, and shall
however, That when a person is a minor or physically or mentally incapacitated, the petition transmit said decision together with the records of proceedings to the OCRG within five (5)
may be filed on his behalf by his spouse, or any of his children, parents, brothers, sisters, working days after the date of decision.
grandparents, guardians, or persons duly authorized by law.
Insofar as applicable, Rule 5 shall be observed also by the CG and D/CR.
Rule 4. Where to file the petition. - The verified petition may be filed, in person, with the
LCRO of the city or municipality or with the Office of the Clerk of the Shari'a Court, as the case Rule 6. Procedures for migrant petitioner. - When the petition is for or from a person
may be, where the record containing the clerical or typographical error to be corrected, or first who is a resident or domiciled in a place different from the place where the document sought
name to be changed, is registered. to be corrected was registered, the following procedures shall be observed:

When the petitioner had already migrated to another place within the Philippines and it would 6.1. The PRCR shall perform the following:
not be practical for such party, in terms of transportation expenses, time and effort to appear
in person before the RKCR, the petition may be filed, in person, with the PRCR of the place 6.1.1. Examine the petition as to completeness of requirements and supporting documents as
where the migrant petitioner is residing or domiciled. required under Rule 8.

Any person whose civil registry record was registered in the Philippines, or in any Philippine 6.1.2. Receive the petition upon payment by the petitioner of prescribed fees as required
Consulate, but who is presently residing or domiciled in a foreign country, may file the under Rule 18.
petition, in person, with the nearest Philippine Consulate, or in accordance with Rule 3.
6.1.3. Ensure that posting or publication of the petition as required under Rule 9 is complied
Rule 5. Processing of the petition. - The C/MCR shall: with.

5.1. Examine the petition as to completeness of requirements and supporting 6.1.4. Endorse the petition and its supporting documents, including the filing fee in postal
documents as required under Rule 8. money order or in any other mode of payment to the RKCR.

5.2. Determine whether or not the civil registry document, which is the subject of the 6.2. The RKCR shall perform the following:
petition, forms part of the civil register of his office. If it is part of the civil register of his office,
he shall assume jurisdiction, otherwise, Rule 6 shall apply. 6.1.1. Examine the petition as to completeness of requirements and supporting documents as
required under Rule 8 and as transmitted by the PRCR.
5.3. Receive the petition upon payment of the prescribed fees by the petitioner.
6.1.2. Observe the procedures under Rule 5.5 to Rule 5.9.
5.4. Ensure that posting or publication requirement is complied with in accordance
with Rule 9. Insofar as applicable, Rule 6 shall be observed also by the CG and D/CR.

5.5. Investigate and consider any third party intervention to the petition. Rule 7. Availment of the privilege. - The correction of clerical or typographical error shall
be availed of only once with respect to a particular entry or entries in the same civil registry
5.6. Enter all petitions in the appropriate record book, as may be prescribed by the record. However, with regard to the change of first name or nickname in the birth certificate,
CRG, indicating therein, among others, the following information: the privilege shall be availed of only once subject to Rule 12 hereunder.

5.6.1. Petition number Rule 8. Form and content of the petition. - The petition shall be in the prescribed form of
an affidavit, subscribed and sworn to before any person authorized by law to administer oath.
5.6.2. Name of petitioner The affidavit shall set forth facts necessary to establish the merits of the petition and shall
show affirmatively that the petitioner is competent to testify to the matters stated. The
5.6.3. Type of petition petitioner shall state the particular erroneous entry or entries sought to be corrected or the
first name sought to be changed, and the correction or change to be made.
5.6.4. Date of petition
8.1. The petition for the correction of clerical or typographical error shall be
5.6.5. Date of receipt supported with the following documents:

5.6.6. Entry sought to be corrected/changed 8.1.1. A certified true machine copy of the certificate or of the page of the registry book
containing the entry or entries sought to be corrected or changed;
5.6.7. Correction/Change made
8.1.2. At least two (2) public or private documents showing the correct entry or entries upon
5.6.8. Action taken or decision which the correction or change shall be based;

146
RECTO, GAYLE ANGELI M.
2011-0008 | AUSL
Personal Notes on Remedial Law 2 Review (based on the syllabus of Prof. Henedino M. Brondial)

8.1.3. Notice or certification of posting 11.1.3. The petition for correction of clerical or typographical error was not posted, or the
petition for change of first name was not published as required under Rule 9.
8.1.4. Other documents which the petitioner or the C/MCR, or the CG, or D/CR may consider
relevant and necessary for the approval of the petition. 11.1.4. The basis used in changing the first name or nickname of the person does not fall
under any of the following circumstances:
8.2. In case of change of first name or nickname, the petition shall be supported with
the following documents and shall comply with the following requirements: 11.1.4.1. The name or nickname is ridiculous, tainted with dishonor or extremely difficult to
write or pronounce.
8.2.1. Documents required under Rule 8.1.
11.1.4.2. The new first name or nickname has been habitually and continuously used by the
8.2.2. A clearance or a certification that the owner of the document has no pending petitioner and he has been publicly known by that first name or nickname in the community.
administrative, civil or criminal case, or no criminal record, which shall be obtained from the
following: 11.1.4.3. The change of first name or nickname will avoid confusion.

8.2.2.1. Employer, if employed 11.1.5. The C/MCR does not have authority to take cognizance of the case.

8.2.2.2. National Bureau of Investigation 11.2. Notify the C/MCR or the CG or the D/CR of the action taken on the decision not later
than ten (10) working days from the date of impugning or approving the decision.
8.2.2.3. Philippine National Police
11.3. Act on all appeals or reconsideration duly filed by the petitioner.
8.2.3. Affidavit of publication from the publisher and a copy of the newspaper clipping.
11.4. Devise or cause to be devised the forms necessary or required for the effective
8.3. The C/MCR, CG or D/CR shall not accept a petition unless all requirements and implementation of this Order.
supporting documents are complied with by the petitioner.
11.5. Perform such other duties and functions as may be necessary to carry out the provisions
8.4. The petition and its supporting documents shall be filed in three (3) copies, and of R.A. 9048.
upon acceptance, shall be distributed as follows:
Rule 12. Effect of approving the petition for change of name. - When the petition for a
8.4.1. First copy to the concerned C/MCR, CG or D/CR, change of first name is approved by the C/MCR or CG or D/CR and such decision has not been
impugned by the CRG, the change shall be reflected in the birth certificate by way of marginal
8.4.2. Second copy to the OCRG, and annotation. In case there are other civil registry records of the same person which are affected
by such change, the decision of approving the change of first name in the birth certificate,
8.4.3. Third copy to the petitioner. upon becoming final and executory, shall be sufficient to be used as basis in changing the first
name of the same person in his other affected records without need for filing a similar petition.
Rule 9. Posting and publication of the petition. - The petition shall be posted by the In such a case, the successful petitioner shall file a request in writing with the concerned
concerned C/MCR, CG or D/CR in a conspicuous place provided for that purpose for ten (10) C/MCR, CG or D/CR to make such marginal annotation, attaching thereto a copy of the
consecutive days after he finds the petition and its supporting documents sufficient in form decision.
and substance.
Rule 13. Effect of denying the petition. - Where the petition is not granted by the C/MCR,
For a change of first name, the petition shall, in addition to the above-stated posting CG or D/CR, as the case may be, the petitioner may either appeal the decision to the CRG
requirements, be published at least once a week for two (2) consecutive weeks in a within ten (10) working days from receipt of the decision, or file the appropriate petition with
newspaper of general circulation. As proof of publication, the petitioner shall attach to the the proper court. In case the petitioner opts to appeal the decision to the CRG, the latter shall
petition a clipping of the publication and an affidavit of publication from the publisher of the render decision within thirty (30) calendar days after receipt of the appeal. The CRG shall
newspaper where publication was made. furnish the C/MCR, CG or D/CR a copy of the decision not later than ten (10) working days
after the date of the decision.
In the case of migrant petitioner, the petition shall be posted first at the office of the PRCR for
ten (10) consecutive days before sending it to the RKCR. Upon receipt, the RKCR shall post Rule 14. Appeal. - When the petition is denied by the C/MCR, the petitioner may appeal the
again the petition in his office for another ten (10) consecutive days. When the petition is for a decision to the CRG, in which case, the following guidelines shall be observed:
change of first name, the migrant petitioner shall publish the petition in a newspaper of
general and national circulation. 14.1. The adversely affected petitioner shall file the notice of appeal to the concerned C/MCR
within ten (10) working days after the receipt of the latter's decision.
In the case where a person's civil registry record or records were registered in the Philippines
or in any of the Philippine Consulates, but the persons presently resides or is domiciled in a 14.2. The C/MCR shall, within five (5) working days after the receipt of the notice of appeal
foreign country, posting and/or publication, as the case may be, shall be done in the place from the petitioner, submit the petition and all supporting documents to the CRG.
where the petition is filed and in the place where the record sought to be corrected is kept.
14.3. The CRG shall render decision on the appeal within thirty (30) calendar days after receipt
Rule 10. Duties of the C/MCR. - The C/MCR shall have the following duties: thereof. The decision of the CRG shall be transmitted to the concerned C/MCR within ten (10)
working days after the date of the decision. Within ten (10) working days after receipt of the
10.1. Examine the petition and its supporting documents. decision, the C/MCR shall notify the petitioner and shall carry out the decision.

10.2. If necessary, conduct investigation by interviewing and asking probing questions to the 14.4. When the petitioner fails to seasonably file the appeal, the decision of the C/MCR
petitioner. disapproving the petition shall become final and executory, and the only option left for the
petitioner shall be to file the appropriate petition with the proper court.
10.3. Post the petition in a conspicuous place provided for that purpose for ten (10)
consecutive days after he finds the petition and its supporting documents sufficient in form 14.5. The petitioner may file the appeal to the CRG on any of the following grounds:
and substance.
14.5.1. A new evidence is discovered, which when presented, shall materially affect, alter,
10.4. Act on the petition and render a decision not later than five (5) working days after the modify or reverse the decision of the C/MCR.
completion of the posting and/or publication requirement.
14.5.2. The denial of the C/MCR is erroneous or not supported with evidence.
10.5. Transmit a copy of the decision together with the records of the proceedings to the
OCRG within five (5) working days after the date of the decision. 14.5.3. The denial of the C/MCR is done with grave abuse of authority or discretion.

10.6. Perform such other duties and functions as may be necessary to carry out the provisions Insofar as applicable, Rule 14 shall be observed in the case of a petition denied by the CG or
of R.A. 9048. D/CR.

Insofar as applicable, the CG and the D/CR shall perform the duties of the C/MCR as provided Rule 15. Failure of the CRG to impugn. - If the CRG fails to impugn the decision of the
for under this Rule. C/MCR, CG or D/CR within ten (10) working days after receipt of the decision granting the
petition, such decision shall become final and executory.
Rule 11. Duties and powers of the CRG. - The CRG shall have the following duties and
powers: Rule 16. Effect of impugning the decision. - Where the decision of the C/MCR, CG or
D/CR is impugned by the CRG, the petitioner may appeal the decision by way of
11.1. Impugn the decision of the C/MCR or CG or D/CR within ten (10) working days after reconsideration with the latter within fifteen (15) working days from receipt of the decision
receipt of the decision granting the petition based on any of the following grounds: and shall be based only on the ground of new evidence discovered, or file the appropriate
petition with the proper court. The decision which shall be rendered by the CRG within thirty
11.1.1. The error is not clerical or typographical. (30) calendar days after receipt of the appeal shall be final and executory.

11.1.2. The correction of an entry in the civil register is substantial or controversial as it Rule 17. Recording, filing and retrieval of decision. - The CRG shall prescribe the proper
involves the change of the age, sex, nationality or civil status of a person. recording, filing and retrieval system of the decisions.

147
RECTO, GAYLE ANGELI M.
2011-0008 | AUSL
Personal Notes on Remedial Law 2 Review (based on the syllabus of Prof. Henedino M. Brondial)

Rule 18. Authority to collect filing and other fees. - The C/MCR or the D/CR is hereby The same grounds. Under RA 9048, the same grounds:
authorized to collect from every petitioner a filing fee in the amount of one thousand pesos 1. That the name is ridiculous. If your name is Lucifer, you can have it
(P1,000.00) for the correction of clerical or typographical error, and three thousand pesos
changed under Rule 103 or RA 9048.
(P3,000.00) for change of first name or nickname. An indigent petitioner as defined under Rule
2.7, shall be exempt from the payment of said fee. 2. Tainted with dishonor. If your name is Osama Bin Laden, you can
have it changed.
In the case of a petition filed with the CG, a filing fee of fifty U.S. dollars ($50.00) or its 3. Your name is very difficult to pronounce or write. If your name is
equivalent value in local currency for the correction of clerical or typographical error, and one supercalifragilisticexpialidocious.
hundred fifty U.S. dollars ($150.00) or its equivalent value in local currency for the change of 4. That you are known by the name in the community.
first name, shall be collected.
5. To avoid confusion.
In the case of a migrant petitioner for correction of clerical or typographical error, there shall 6. To avoid foreign alienage.
be a service fee of five hundred pesos (P500.00) to be collected by the PRCR. In case the So the same grounds either under Rule 103 or RA 9048.
petition is for change of name, the service fee is one thousand pesos (P1,000.00). The service
fee shall accrue to the city or municipal government of the PRCR. The PRCR shall also collect Under Rule 108, you have to correct entries in a document. And the enumerated
the filing fee from the migrant petitioner, which shall be in the form of postal money order or public documents where entries have had are as many as possible. Those which
other form of payment which shall be payable to and transmitted to the RKCR, together with
are in the custody of the register of deeds. If the change are substantial, you
the petition and supporting documents.
cannot avail RA 9048 but Rule 108. You can only avail of RA 9048 if the change
When the petitioner files petition for correction of clerical or typographical error, sought is clerical or typographical in nature.
simultaneously with a petition for change of first name, and the same document is involved,
the petitioner shall pay only the amount corresponding to the fee for the petition for change of Q: Name is Maria Cecilia when in her birth certificate it shows as Ma.
first name. Cecilia, is that typographical or clerical error?
The local legislative body shall ratify the fees herein prescribed upon effectivity of this Order.
A: There is no error there. What you have to seek is change of name because Ma
Prior to ratification by the local legislative body, all fees collected in connection with this Order is different from Maria Cecilia. There is no error there.
shall go to the LCRO trust fund: Provided, however, That the fees prescribed herein shall be
uniform in all cities and municipalities in the country, and in all Philippine Consulates. You cannot avail of RA 9048 if it will change sex, status, or nationality. If there is
error you can change it under Rule 108 and not RA 9048.
Rule 19. Penalty clause. - A person who violates any of the provisions of R.A. No. 9048 and
of this Order shall, upon conviction, be penalized by imprisonment of not less than six (6)
years but not more than twelve (12) years, or a fine of not less than ten thousand pesos
Q: Suppose you do it under Rule 108, and it was denied. Where do you
(P10,000.00) but not more than one hundred thousand pesos (P100,000.00), or both, at the appeal or what is the remedy?
discretion of the court. In addition, if the offender is a government official or employee, he A: The remedy is to go to the Civil Registrar General who is the Manager of the
shall suffer the penalties provided under existing civil service laws, rules and regulations. National Census Administrative Office. It is not an appeal but a motion for
reconsideration. If the civil registrar affirms the denial, you file a PETITION
Rule 20. Periodic review. - The Civil Registrar General may call for periodic review of the under Rule 108.
IRR as may be necessary.

Rule 21. Retroactivity clause. - This Order shall have retroactive effect insofar as it does Q: Going back to RA 9048, A was born in Quezon City, he now resides
not prejudice or impair vested or acquired rights in accordance with the Civil Code and other in Baguio City. Should he go to Quezon City in order to file under RA
laws. 9048?
A: No, he has to file it in Baguio and under the rules the two civil registrar, that
Rule 22. Separability clause. - If any portion or provision of this Order is declared void or of Baguio and Quezon city will coordinate. It also can be done abroad, by filing
unconstitutional, the remaining portions or provisions thereof shall not be affected by such
declaration.
the same with the Philippine Consulate, it is a matter of communication.

Rule 23. Repealing clause. - All circulars, memoranda, rules and regulations or parts NOTE: Appeal in RA 9048 is with the Civil Registrar General. Although it is not
thereof inconsistent with the provisions of this Order are hereby repealed or modified really an appeal, it is a motion for reconsideration. Appeal under Rule 103 or 108
accordingly. is ordinary/ regular appeal. In RA 9048, it is not really an appeal, wala naman
kasi appeal sa administrative remedies, you use the word appeal for facilitation
Rule 24. Effectivity clause. - This Order shall take effect fifteen (15) days after its
publication in a newspaper of general circulation.
of better understanding. But that is not an appeal, motion for reconsideration
siguro.
APPROVED this 24th day of July 2001
CASES

OLD LECTURE MA. LOURDES BARRIENTOS ELEOSIDA, for and in behalf of her minor child, CHARLES
CHRISTIAN ELEOSIDA, petitioner, vs. LOCAL CIVIL REGISTRAR OF QUEZON CITY, and
RULE 103 (CHANGE OF NAME) and 108 (CORRECTION OF ENTRIES) are still CARLOS VILLENA BORBON, respondents.
applicable. They have only been amended but are not yet repealed by RA 9048. G.R. No. 130277 May 9, 2002
FIRST DIVISION
Therefore, an affected person can avail of Rule 103 or Rule 108 without RA 9048
being violated. FACTS:
x Lourdes Æ filed a petition before RTC QC seeking to correct the following entries
RA 9048 is known as administrative procedure of changing one's name or nick in the birth certificate of her son, Charles Christian:
name or administrative procedure of correcting an entry in a document. o first, the surname "Borbon" should be changed to "Eleosida;"
Therefore, being administrative, it is extrajudicial. In case of Rules 103 and 108, o second, the date of the parents' wedding should be left blank; and
o third, the informant's name should be "Ma. Lourdes B. Eleosida,"
they are judicial processes. instead of "Ma. Lourdes E. Borbon."
x Lourdes’s arguments:
Q: Which courts have jurisdiction over Rules 103 and 108? o she gave birth to her son out of wedlock on May 24, 1992;
A: RTC o that she and the boy's father, Carlos Borbon, were never married;
o and that the child is therefore illegitimate and should follow the
Q: Venue? mother's surname.
x The petition impleaded the Local Registrar of Quezon City and Carlos Villena
A: Residence in case of change of name. Where the registry is located in case of Borbon as respondents.
correction of entries. x RTC Æ issued 1st order
o Let copies of this notice be furnished the petitioner, and together
Q: What are the amendments to these two rules brought about by RA with copies of the petition, respondent Carlos Villena Borbon; the
9048? Offices of the Local Civil Registrar of Quezon City and the Solicitor
A: Remember that RA 9048 speaks only of names and nickname but when Rule General, who are given fifteen (15) days from notice of the
petition, or from the last date of publication of such notice, within
103 says change of name, it does not only refer to name or nick name but also which to file their opposition thereto, if any. In the event that the
to family name. So if you want to change the family name you cannot avail of RA Solicitor General may not be able to appear on the scheduled
9048. hearing, to designate the City Prosecutor of Quezon City to appear
for and in behalf of the State.
Q: What are the grounds? x SolGen Æ DID NOT file opposition
A: x RTC Æ issued 2nd order

148
RECTO, GAYLE ANGELI M.
2011-0008 | AUSL
Personal Notes on Remedial Law 2 Review (based on the syllabus of Prof. Henedino M. Brondial)

o Considering that there is no opposition filed despite notice to the o Thus, the persons who must be made parties to a proceeding
Solicitor General as contained in the notice of hearing dated April concerning the cancellation or correction of an entry in the civil
23, 1997 requiring that office to file their opposition, if any, to the register are—
petition for correction of entries in the birth certificate of minor ƒ (1) the civil registrar, and
child Charles Christian Eleosida, the petitioner will be allowed to ƒ (2) all persons who have or claim any interest which
present compliance with the jurisdictional requirements and at the would be affected thereby.
same time initially present evidence on July 23, 1997, at 8:30 o Upon the filing of the petition, it becomes the duty of the court to
o'clock in the morning ƒ (1) issue an order fixing the time and place for the
x RTC Æ then dismissed the petition hearing of the petition, and
o It is an established jurisprudence that, only CLERICAL ERRORS OF ƒ (2) cause the order for hearing to be published once
A HARMLESS AND INNOCUOUS NATURE like: misspelled name, a week for three (3) consecutive weeks in a
occupation of the parents, etc., may be the subject of a judicial newspaper of general circulation in the province.
order (contemplated under Article 412 of the New Civil Code), o The following are likewise entitled to oppose the petition:--(1) the
authorizing changes or corrections and: NOT as may affect the civil registrar, and (2) any person having or claiming any interest
CIVIL STATUS, NATIONALITY OR CITIZENSHIP OF THE PERSONS under the entry whose cancellation or correction is sought.
INVOLVED. o If all these procedural requirements have been followed, a
o In the present case, it is very clear that the changes desired by the petition for correction and/or cancellation of entries in the
petitioner will ultimately affect the CIVIL STATUS OF CHARLES record of birth even if filed and conducted under Rule 108 of
CHRISTIAN, as she wants the Court to Direct the Civil Registrar of the Revised Rules of Court can no longer be described as
Quezon City to substitute her maiden name, ELEOSIDA, with that 'summary'. xxx"12
of BORBON; to delete the information supplied in ITEM 12, x It is true in the case at bar that the changes sought to be made by petitioner are
respecting the date and place of marriage of parents, on the not merely clerical or harmless errors but substantial ones as they would affect
ground that she was never married to respondent CARLOS the status of the marriage between petitioner and Carlos Borbon, as well as the
VILLENA BORBON and amend the information in ITEM 14, legitimacy of their son, Charles Christian. Changes of such nature, however,
respecting the name of the informant, from MA. LOURDES E. are now allowed under Rule 108 in accordance with our ruling in
BORBON to MA. LOURDES B. ELEOSIDA, and is indicative of Republic vs. Valencia provided that the appropriate procedural
petitioner's intention and device to establish that CHARLES requirements are complied with. The records show that upon receipt of the
CHRISTIAN's civil status as ILLEGITIMATE. petition, the trial court issued a notice of hearing setting the hearing on June 26,
o With the petition's ultimate purpose on the part of petitioner to 1997 at 8:30 in the morning at Room 118, Hall of Justice, Quezon City. The trial
secure judicial order, which would authorize a change in the civil court likewise ordered the publication of said notice once a week for three (3)
status of CHARLES CHRISTIAN, this Court, finds the action consecutive weeks in a newspaper of general circulation and its posting in
improper. The matters desired to be cancelled and/or changed by selected places in Metro Manila. The notice stated that the petitioner shall prove
petitioner cannot be considered falling under the ambit of the her petition during said hearing and all other persons having or claiming any
words 'clerical errors of a harmless and innocuous nature.' interest thereon shall also appear and show if there is any reason why the
x Lourdes Æ filed a Rule 45 before the SC petition should not be granted. Respondents Carlos Villena Borbon, the Local
x SC Æ required the Civil Registrar AND BORBON to Comment Civil Registrar of Quezon City and the Solicitor General were all furnished with a
x SolGen Æ filed a manifestation in lieu of a comment copy of the notice of hearing together with a copy of the petition. On June 26,
o The OSG submitted that even substantial errors in the civil registry 1997, the trial court issued a second order giving the petitioner an opportunity to
may be corrected provided that the parties aggrieved by the error show compliance with the jurisdictional requirements and to present evidence
avail themselves of the appropriate adversary proceeding. Thus it during the hearing set on July 23, 1997. The foregoing satisfy all the
argued that even if the petition seeks the correction and eventual requirements of Rule 108 to make it an adversary proceeding. It was therefore
change in the civil status of Charles Christian, the same can be an error for the trial court to dismiss the petition motu proprio without allowing
ordered by the court as long as all the parties who may be affected the petitioner to present evidence to support her petition and all the other
by the entries are notified and represented. persons who have an interest over the matter to oppose the same
x Borbon Æ DID NOT file his comment

ISSUE: Whether corrections of entries in the certificate of live birth pursuant to Article 412 of REPUBLIC OF THE PHILIPPINES, petitioner, vs. CARLITO I. KHO, MICHAEL KHO, MERCY
the Civil Code, in relation to Rule 108 of the Rules of Court, may be allowed even if the errors NONA KHO-FORTUN, HEDDY MOIRA KHO-SERRANO, KEVIN DOGMOC KHO (Minor), and
to be corrected are substantial and not merely clerical errors of a harmless and innocuous KELLY DOGMOC KHO (Minor), respondents.
nature. G.R. No. 170340 June 29, 2007
SECOND DIVISION
HELD: YES.
x We find merit in the petition. FACTS:
x Rule 108 of the Revised Rules of Court provides the procedure for cancellation or x Carlito (and his siblings Michael, Mercy Nona and Heddy Moira) Æ filed before
correction of entries in the civil registry. The proceedings under said rule the RTC of Butuan City a verified petition for correction of entries in the civil
may either be summary or adversary in nature. If the correction sought to registry of Butuan City to effect changes in their respective birth certificates and
be made in the civil register is clerical, then the procedure to be adopted is impleading therein the Local Civil Registrar of Butuan City
summary. If the rectification affects the civil status, citizenship or nationality of o Carlito also asked the court in behalf of his minor children, Kevin
a party, it is deemed substantial, and the procedure to be adopted is and Kelly, to order the correction of some entries in their birth
adversary.10 This is our ruling in Republic vs. Valencia11 where we held that certificates.
even substantial errors in a civil registry may be corrected and the true o he requested the correction in his birth certificate of the citizenship
facts established under Rule 108 provided the parties aggrieved by the of his mother to "Filipino" instead of "Chinese," as well as the
error avail themselves of the appropriate adversary proceeding. An deletion of the word "married" opposite the phrase "Date of
appropriate adversary suit or proceeding is one where the trial court has marriage of parents" because his parents, Juan Kho and Epifania
conducted proceedings where all relevant facts have been fully and properly Inchoco (Epifania), were allegedly not legally married.
developed, where opposing counsel have been given opportunity to demolish the o same request to delete the "married" status of their parents from
opposite party's case, and where the evidence has been thoroughly weighed and their respective birth certificates was made by Carlito’s siblings
considered. The Court further laid down the procedural requirements to make Michael, Mercy Nona, and Heddy Moira
the proceedings under Rule 108 adversary, thus: o with respect to his children’s birth certificate, he prayed that the
o "The pertinent sections of Rule 108 provide: date of his and his wife’s marriage be corrected from April 27, 1989
ƒ SEC. 3. Parties.—When cancellation or correction of to January 21, 2000, the date appearing in their marriage
an entry in the civil register is sought, the civil certificate.
registrar and all persons who have or claim any o Additional prayers in AMENDED PETITION:
interest which would be affected thereby shall be ƒ Carlito’s second name of "John" be deleted from his
made parties to the proceeding.1âwphi1.nêt record of birth; and
ƒ SEC. 4. Notice and publication.—Upon the filing of the ƒ that the name and citizenship of Carlito’s father in his
petition, the court shall, by an order, fix the time and (Carlito’s) marriage certificate be corrected from
place for the hearing of the same, and cause "John Kho" to "Juan Kho" and "Filipino" to "Chinese,"
reasonable notice thereof to be given to the persons respectively
named in the petition. The court shall also cause the x The petition was published for three consecutive weeks4 in Mindanao Daily
order to be published once in a week for three 93) Patrol-CARAGA, a newspaper of general circulation
consecutive weeks in a newspaper of general x Civil Registrar Æ stated her observations and suggestions to the proposed
circulation in the province. corrections in the birth records of Carlito and his siblings but interposed no
ƒ SEC. 5. Opposition.—The civil registrar and any objections to the other amendments
person having or claiming any interest under the x RTC Æ directed the local civil registrar of Butuan City to correct the entries in the
entry whose cancellation or correction is sought may, record of birth of Carlito, as follows:
within fifteen (15) days from notice, file his opposition o (1) change the citizenship of his mother from "Chinese" to
thereto. "Filipino";
o (2) delete "John" from his name; and

149
RECTO, GAYLE ANGELI M.
2011-0008 | AUSL
Personal Notes on Remedial Law 2 Review (based on the syllabus of Prof. Henedino M. Brondial)

o (3) delete the word "married" opposite the date of marriage of his would be set open, the consequence of which might be
parents. detrimental and far reaching. x x x (Emphasis supplied)
o The last correction was ordered to be effected likewise in the birth
certificates of respondents Michael, Mercy Nona, and Heddy Moira ISSUE # 2: Whether a petition under Rule 108 may prosper, considering that the said changes
o ordered the correction of the birth certificates of the minor children are substantial in nature.
of Carlito to reflect the date of marriage of Carlito and Marivel
Dogmoc (Marivel) as January 21, 2000, instead of April 27, 1989, HELD # 2: YES.
and the name "Maribel" as "Marivel." x In Republic v. Valencia,13 however, this Court ruled, and has since
o the corrections ordered pertained to the alteration of the name of repeatedly ruled, that even substantial errors in a civil registry may be
Carlito’s father from "John Kho" to "Juan Kho" and the latter’s corrected through a petition filed under Rule 108.14
citizenship from "Filipino" to "Chinese." x It is undoubtedly true that if the subject matter of a petition is not for the
x RepublicÆ appealed to CA correction of clerical errors of a harmless and innocuous nature, but one
o faulting the trial court in granting the petition for correction of involving nationality or citizenship, which is indisputably substantial as well as
entries in the subject documents despite the failure of respondents controverted, affirmative relief cannot be granted in a proceeding summary in
to implead the minors’ mother, Marivel, as an indispensable party nature. However, it is also true that a right in law may be enforced and a wrong
and to offer sufficient evidence to warrant the corrections with may be remedied as long as the appropriate remedy is used. This Court adheres
regard to the questioned "married" status of Carlito and his siblings’ to the principle that even substantial errors in a civil registry may be corrected
parents, and the latter’s citizenship. and the true facts established provided the parties aggrieved by the error avail
o also faulted the trial court for ordering the change of the name themselves of the appropriate adversary proceeding.
"Carlito John Kho" to "Carlito Kho" for non-compliance with x What is meant by "appropriate adversary proceeding?" Black’s Law Dictionary
jurisdictional requirements for a change of name under Rule 103 of defines "adversary proceeding["] as follows:
the Rules of Court. o One having opposing parties; contested, as distinguished
x CA Æ affirmed RTC from an ex parte application, one of which the party seeking
o The CA found that Rule 108 of the Revised Rules of Court, which relief has given legal warning to the other party, and
outlines the proper procedure for cancellation or correction of afforded the latter an opportunity to contest it. x x x 15
entries in the civil registry, was observed in the case. (Emphasis, italics and underscoring supplied)
o Regarding Carlito’s minor children Kevin and Kelly, the appellate x The enactment in March 2001 of Republic Act No. 9048, otherwise known as
court held that the correction of their mother’s first name from "An Act Authorizing the City or Municipal Civil Registrar or the Consul General to
"Maribel" to "Marivel" was made to rectify an innocuous error. Correct a Clerical or Typographical Error in an Entry and/or Change of First Name
o As for the change in the date of the marriage of Carlito and or Nickname in the Civil Register Without Need of Judicial Order," has been
Marivel, albeit the CA conceded that it is a substantial alteration, it considered to lend legislative affirmation to the judicial precedence
held that the date would not affect the minors’ filiation from that substantial corrections to the civil status of persons recorded in
"legitimate" to "illegitimate" considering that at the time of their the civil registry may be effected through the filing of a petition under
respective births in 1991 and 1993, their father Carlito’s first Rule 108.16
marriage was still subsisting as it had been annulled only in 1999. x Thus, this Court in Republic v. Benemerito17 observed that the obvious effect
o In light of Carlito’s legal impediment to marry Marivel at the time of Republic Act No. 9048 is to make possible the administrative
they were born, their children Kevin and Kelly were illegitimate. It correction of clerical or typographical errors or change of first name or
followed, the CA went on to state, that Marivel was not an nickname in entries in the civil register, leaving to Rule 108 the
indispensable party to the case, the minors having been correction of substantial changes in the civil registry in appropriate
represented by their father as required under Section 5 of Rule 39 adversarial proceedings.
of the Revised Rules of Court. x When all the procedural requirements under Rule 108 are thus followed, the
o Further, the CA ruled that although Carlito failed to observe the appropriate adversary proceeding necessary to effect substantial corrections to the
requirements of Rule 103 of the Rules of Court, he had complied entries of the civil register is satisfied.18 The pertinent provisions of Rule 108 of
nonetheless with the jurisdictional requirements for correction of the Rules of Court read:
entries in the civil registry under Rule 108 of the Rules of Court. o SEC. 3. Parties. - When cancellation or correction of an entry in the
The petition for correction of entry in Carlito’s birth record, it noted, civil registrar is sought, the civil registrar and all persons who have
falls under letter "o" of the enumeration under Section 2 of Rule or claim any interest which would be affected thereby shall be
108. made parties to the proceeding.
x Republic Æ filed a Rule 45 before the SC o SEC. 4. Notice and publication. — Upon the filing of the petition,
o since the changes sought by respondents were substantial in the court shall, by an order, fix the time and place for the hearing
nature, they could only be granted through an adversarial of the same, and cause reasonable notice thereof to be given to
proceeding in which indispensable parties, such as Marivel and the persons named in the petition. The court shall also cause the
respondents’ parents, should have been notified or impleaded. order to be published once in a week for three (3) consecutive
o further contends that the jurisdictional requirements to change weeks in a newspaper of general circulation in the province.
Carlito’s name under Section 2 of Rule 103 of the Rules of Court o SEC. 5. Opposition. — The civil registrar and any person having or
were not satisfied because the Amended Petition failed to allege claiming any interest under the entry whose cancellation or correction
Carlito’s prior three-year bona fide residence in Butuan City, and is sought may, within fifteen (15) days from notice of the petition, or
that the title of the petition did not state Carlito’s aliases and his from the last date of publication of such notice, file his opposition
true name as "Carlito John I. Kho." thereto. (Emphasis and underscoring supplied)
o Petitioner concludes that the same jurisdictional defects attached to x There is no dispute that the trial court’s Order19 setting the petition for hearing
the change of name of Carlito’s father. and directing any person or entity having interest in the petition to oppose it was
posted20 as well as published for the required period; that notices of hearings
ISSUE # 1: Whether the changes sought in the present petition are substantial in nature were duly served on the Solicitor General, the city prosecutor of Butuan and the
thereby warranting an adversary proceeding. local civil registrar; and that trial was conducted on January 31, 2002 during
which the public prosecutor, acting in behalf of the OSG, actively participated by
HELD # 1: YES. cross-examining Carlito and Epifania.
x It can not be gainsaid that the petition, insofar as it sought to change the
citizenship of Carlito’s mother as it appeared in his birth certificate and delete the ISSUE # 3: Whether the failure to implead Marivel and Carlito’s parents is fatal to the cause of
"married" status of Carlito’s parents in his and his siblings’ respective birth Kho et al.
certificates, as well as change the date of marriage of Carlito and Marivel
involves the correction of not just clerical errors of a harmless and innocuous HELD # 3: NO.
nature.10 Rather, the changes entail substantial and controversial amendments. x A similar issue was earlier raised in Barco v. Court of Appeals.21 That case
x For the change involving the nationality of Carlito’s mother as reflected in his stemmed from a petition for correction of entries in the birth certificate of a
birth certificate is a grave and important matter that has a bearing and effect on minor, June Salvacion Maravilla, to reflect the name of her real father (Armando
the citizenship and nationality not only of the parents, but also of the Gustilo) and to correspondingly change her surname. The petition was granted
offspring.11 by the trial court.
x Further, the deletion of the entry that Carlito’s and his siblings’ parents were o Barco, whose minor daughter was allegedly fathered also by
"married" alters their filiation from "legitimate" to "illegitimate," with significant Gustilo, however, sought to annul the trial court’s decision, claiming
implications on their successional and other rights. that she should have been made a party to the petition for
x Clearly, the changes sought can only be granted in an adversary proceeding. correction. Failure to implead her deprived the RTC of jurisdiction,
Labayo-Rowe v. Republic12 explains the raison d etre: she contended.
o x x x. The philosophy behind this requirement lies in the o In dismissing Barco’s petition, this Court held that the publication of
fact that the books making up the civil register and all the order of hearing under Section 4 of Rule 108 cured the failure
documents relating thereto shall be prima facie evidence of to implead an indispensable party.
the facts therein contained. If the entries in the civil o The essential requisite for allowing substantial corrections of entries
register could be corrected or changed through mere in the civil registry is that the true facts be established in an
summary proceedings and not through appropriate action appropriate adversarial proceeding. This is embodied in Section 3,
wherein all parties who may be affected by the entries are Rule 108 of the Rules of Court, which states:
notified or represented, the door to fraud or other mischief ƒ Section 3. Parties. - When cancellation or correction
of an entry in the civil register is sought, the civil
150
RECTO, GAYLE ANGELI M.
2011-0008 | AUSL
Personal Notes on Remedial Law 2 Review (based on the syllabus of Prof. Henedino M. Brondial)

registrar and all persons who have or claim any was not respondents’ fault, does not in any way change the adversarial nature of
interest which would be affected thereby shall be the proceedings.
made parties to the proceeding. x Also significant to note is that the birth certificates of Carlito’s siblings uniformly
o Undoubtedly, Barco is among the parties referred to in Section 3 of stated the citizenship of Epifania as "Filipino." To disallow the correction in
Rule 108. Her interest was affected by the petition for correction, Carlito’s birth record of his mother’s citizenship would perpetuate an
as any judicial determination that June was the daughter of inconsistency in the natal circumstances of the siblings who are unquestionably
Armando would affect her ward’s share in the estate of her father. born of the same mother and father.
x x x. x Outside the ambit of substantial corrections, of course, is the correction of the
o Yet, even though Barco was not impleaded in the petition, name of Carlito’s wife from "Maribel" to "Marivel." The mistake is clearly clerical
the Court of Appeals correctly pointed out that the defect or typographical, which is not only visible to the eyes, but is also obvious to the
was cured by compliance with Section 4, Rule 108, which understanding34 considering that the name reflected in the marriage certificate
requires notice by publication x x x. of Carlito and his wife is "Marivel."
x Apropos is Yu v. Republic35 which held that changing the appellant’s Christian
o The purpose precisely of Section 4, Rule 108 is to bind the name of "Sincio" to "Sencio" amounts merely to the righting of a clerical error. The
whole world to the subsequent judgment on the petition. change of name from Beatriz Labayo/Beatriz Labayu to Emperatriz Labayo was
The sweep of the decision would cover even parties who also held to be a mere innocuous alteration, which can be granted through a
should have been impleaded under Section 3, Rule 108, but summary proceeding.36 The same ruling holds true with respect to the
were inadvertently left out. x x x correction in Carlito’s marriage certificate of his father’s name from "John Kho" to
o Verily, a petition for correction is an action in rem, an action "Juan Kho." Except in said marriage certificate, the name "Juan Kho" was
against a thing and not against a person. The decision on the uniformly entered in the birth certificates of Carlito and of his siblings.
petition binds not only the parties thereto but the whole world. An
in rem proceeding is validated essentially through publication. IN RE: PETITION FOR CHANGE OF NAME AND/OR CORRECTION/CANCELLATION
Publication is notice to the whole world that the proceeding has for OF ENTRY IN CIVIL REGISTRY OF JULIAN LIN CARULASAN WANG also known as
its object to bar indefinitely all who might be minded to make an JULIAN LIN WANG, to be amended/corrected as JULIAN LIN WANG, JULIAN LIN WANG, duly
objection of any sort against the right sought to be established. It represented by his mother ANNA LISA WANG, Petitioners, vs. CEBU CITY CIVIL REGISTRAR,
is the publication of such notice that brings in the whole world as a duly represented by the Registrar OSCAR B. MOLO, Respondents.
party in the case and vests the court with jurisdiction to hear and G.R. No. 159966. March 30, 2005
decide it.22 SECOND DIVISION
x Given the above ruling, it becomes unnecessary to rule on whether Marivel or
respondents’ parents should have been impleaded as parties to the proceeding. It FACTS:
may not be amiss to mention, however, that during the hearing on January 31, x Julian Lin Carulasan Wang was born to Anna Lisa Wang and Sing-Foe Wang who
2002, the city prosecutor who was acting as representative of the OSG were SUBSEQUENTLY married to each other and executed a deed of legitimation
did not raise any objection to the non-inclusion of Marivel and Carlito’s of their son so that the child’s name was changed from Julian Lin Carulasan to
parents as parties to the proceeding. Julian Lin Carulasan Wang
x Parenthetically, it seems highly improbable that Marivel was unaware of the x They planned to stay in SG where Julian would be studying
proceedings to correct the entries in her children’s birth certificates, especially x Julian (represented by mother Anna Lisa) Æ filed a petition for change of name
since the notices, orders and decision of the trial court eHe were all sent to the and/or correction/cancellation of entry in the Civil Registry of Julian Lin Carulasan
residence23 she shared with Carlito and the children. Wang PRAYING that his middle name be dropped
x It is also well to remember that the role of the court in hearing a petition to o Since it is a practice in SG that middle names or the maiden
correct certain entries in the civil registry is to ascertain the truth about the facts surname of the mother are not carried in a person’s name
recorded therein.24 o out of fear that Julian will be discriminated against
x With respect to the date of marriage of Carlito and Marivel, their certificate of o Julian and his sister might also be asking whether they are brother
marriage25 shows that indeed they were married on January 21, 2000, not on and sister since they have different surnames.
April 27, 1989. Explaining the error, Carlito declared that the date "April 27, o Carulasan sounds funny in Singapore’s Mandarin language since
1989" was supplied by his helper, adding that he was not married to Marivel at they do not have the letter "R" but if there is, they pronounce it as
the time his sons were born because his previous marriage was annulled only in "L."
1999.26 Given the evidence presented by respondents, the CA observed that the x RTC Æ denied the petition
minors were illegitimate at birth, hence, the correction would bring about no o found that the reason given for the change of name sought in the
change at all in the nature of their filiation. petition—that is, that petitioner Julian may be discriminated against
x With respect to Carlito’s mother, it bears noting that she declared at the witness when studies in Singapore because of his middle name—did not fall
stand that she was not married to Juan Kho who died in 1959.27 Again, that within the grounds recognized by law.
testimony was not challenged by the city prosecutor. o ruled that the change sought is merely for the convenience of the
x The documentary evidence supporting the deletion from Carlito’s and his siblings’ child.
birth certificates of the entry "Married" opposite the date of marriage of their o Since the State has an interest in the name of a person, names
parents, moreover, consisted of a certification issued on November 24, 1973 by cannot be changed to suit the convenience of the bearers.
St. Joseph (Butuan City) Parish priest Eugene van Vught stating that Juan Kho ƒ Under Article 174 of the Family Code, legitimate
and Epifania had been living together as common law couple since 1935 but children have the right to bear the surnames of the
have never contracted marriage legally.28 father and the mother, and there is no reason why
x A certification from the office of the city registrar, which was appended to this right should now be taken from petitioner Julian,
respondents’ Amended Petition, likewise stated that it has no record of marriage considering that he is still a minor. The trial court
between Juan Kho and Epifania.29 Under the circumstances, the deletion of the added that when petitioner Julian reaches the age of
word "Married" opposite the "date of marriage of parents" is warranted. majority, he could then decide whether he will change
x With respect to the correction in Carlito’s birth certificate of his name from his name by dropping his middle name
"Carlito John" to "Carlito," the same was properly granted under Rule 108 of the x Julian Æ filed an MR
Rules of Court. As correctly pointed out by the CA, the cancellation or correction x RTC Æ denied
of entries involving changes of name falls under letter "o" of the following o The dropping of the middle name would be tantamount to giving
provision of Section 2 of Rule 108:30 due recognition to or application of the laws of Singapore instead
o Section 2. Entries subject to cancellation or correction. — Upon of Philippine law which is controlling. That the change of name
good and valid grounds, the following entries in the civil register would not prejudice public interest or would not be for a fraudulent
may be cancelled or corrected: (a) births; (b) marriages; (c) purpose would not suffice to grant the petition if the reason for the
deaths; (d) legal separation; (e) judgments of annulment of change of name is itself not reasonable
marriage; (f) judgments declaring marriages void from the x Julian Æ filed a Rule 45 before the SC
beginning; (g) legitimations; (h) adoptions; (i) acknowledgments of o arguing that the trial court has decided a question of substance not
natural children; (j) naturalization; (k) election, loss or recovery of theretofore determined by the Court, that is: whether or not
citizenship; (l) civil interdiction; (m) judicial determination of dropping the middle name of a minor child is contrary to Article
filiation; (n) voluntary emancipation of a minor; and (o) changes of 1747 of the Family Code.
name. (Emphasis and underscoring supplied) o Petitioner contends that "[W]ith globalization and mixed marriages,
x Hence, while the jurisdictional requirements of Rule 103 (which there is a need for the Supreme Court to rule on the matter of
governs petitions for change of name) were not complied with, dropping of family name for a child to adjust to his new
observance of the provisions of Rule 108 suffices to effect the environment, for consistency and harmony among siblings, taking
correction sought for. into consideration the "best interest of the child."8
x More importantly, Carlito’s official transcript of record from the Urious College in o It is argued that convenience of the child is a valid reason for
Butuan City,31 certificate of eligibility from the Civil Service Commission,32 and changing the name as long as it will not prejudice the State and
voter registration record33 satisfactorily show that he has been known by his others. Petitioner points out that the middle name "Carulasan" will
first name only. No prejudice is thus likely to arise from the dropping of the cause him undue embarrassment and the difficulty in writing or
second name. pronouncing it will be an obstacle to his social acceptance and
x The correction of the mother’s citizenship from Chinese to Filipino as appearing integration in the Singaporean community. Petitioner also alleges
in Carlito’s birth record was also proper. Of note is the fact that during the cross that it is error for the trial court to have denied the petition for
examination by the city prosecutor of Epifania, he did not deem fit to question change of name until he had reached the age of majority for him to
her citizenship. Such failure to oppose the correction prayed for, which certainly
151
RECTO, GAYLE ANGELI M.
2011-0008 | AUSL
Personal Notes on Remedial Law 2 Review (based on the syllabus of Prof. Henedino M. Brondial)

decide the name to use, contrary to previous cases9 decided by and it has frequently been held that, when identity is certain, a
this Court that allowed a minor to petition for change of name. variance in, or misspelling of, the name is immaterial.
x SC Æ required SolGen to comment x The names of individuals usually have two parts: the given name or proper
x SolGen Æ filed comment name, and the surname or family name.
o positing that the trial court correctly denied the petition for change o The given or proper name is that which is given to the individual at
of name. The OSG argues that under Article 174 of the Family birth or baptism, to distinguish him from other individuals.
Code, legitimate children have the right to bear the surnames of o The name or family name is that which identifies the family to
their father and mother, and such right cannot be denied by the which he belongs and is continued from parent to child. The given
mere expedient of dropping the same. name may be freely selected by the parents for the child; but the
o According to the OSG, there is also no showing that the dropping of surname to which the child is entitled is fixed by law.
the middle name "Carulasan" is in the best interest of petitioner, x A name is said to have the following characteristics:
since mere convenience is not sufficient to support a petition for o (1) It is absolute, intended to protect the individual from being
change of name and/or cancellation of entry.12 confused with others.
o The OSG also adds that the petitioner has not shown any o (2) It is obligatory in certain respects, for nobody can be without
compelling reason to justify the change of name or the dropping of a name.
the middle name, for that matter. Petitioner’s allegation that the o (3) It is fixed, unchangeable, or immutable, at least at the
continued use of the middle name may result in confusion and start, and may be changed only for good cause and by judicial
difficulty is allegedly more imaginary than real. The OSG reiterates proceedings.
its argument raised before the trial court that the dropping of the o (4) It is outside the commerce of man, and, therefore,
child’s middle name could only trigger much deeper inquiries inalienable and intransmissible by act inter vivos or mortis causa.
regarding the true parentage of petitioner. o (5) It is imprescriptible.19
o Hence, while petitioner Julian has a sister named Jasmine Wei x This citation does not make any reference to middle names, but this does not
Wang, there is no confusion since both use the surname of their mean that middle names have no practical or legal significance. Middle names
father, Wang. Even assuming that it is customary in Singapore to serve to identify the maternal lineage or filiation of a person as well as
drop the middle name, it has also not been shown that the use of further distinguish him from others who may have the same given
such middle name is actually proscribed by Singaporean law name and surname as he has.
x Our laws on the use of surnames state that legitimate and legitimated children
ISSUE # 1: Whether the grounds cited by Julian are sufficient in granting his petition. shall principally use the surname of the father.20 The Family Code gives
legitimate children the right to bear the surnames of the father and the
HELD # 1: NO. mother,21 while illegitimate children shall use the surname of their mother,
x The Court has had occasion to express the view that the State has an interest in unless their father recognizes their filiation, in which case they may bear the
the names borne by individuals and entities for purposes of identification, and father’s surname.22
that a change of name is a privilege and not a right, so that before a person can x Applying these laws, an illegitimate child whose filiation is not recognized by the
be authorized to change his name given him either in his certificate of birth or father bears only a given name and his mother’s surname, and does not have a
civil registry, he must show proper or reasonable cause, or any compelling middle name. The name of the unrecognized illegitimate child therefore identifies
reason which may justify such change. Otherwise, the request should be him as such. It is only when the illegitimate child is legitimated by the
denied.14 subsequent marriage of his parents or acknowledged by the father in a public
x The touchstone for the grant of a change of name is that there be document or private handwritten instrument that he bears both his mother’s
‘proper and reasonable cause’ for which the change is sought.15 To surname as his middle name and his father’s surname as his surname, reflecting
justify a request for change of name, petitioner must show not only some his status as a legitimated child or an acknowledged illegitimate child.
proper or compelling reason therefore but also that he will be x Accordingly, the registration in the civil registry of the birth of such individuals
prejudiced by the use of his true and official name. requires that the middle name be indicated in the certificate. The registered
x Among the grounds for change of name which have been held valid are: name of a legitimate, legitimated and recognized illegitimate child thus contains a
o (a) when the name is ridiculous, dishonorable or extremely difficult given or proper name, a middle name, and a surname.
to write or pronounce; x Petitioner theorizes that it would be for his best interest to drop his middle name
o (b) when the change results as a legal consequence, as in as this would help him to adjust more easily to and integrate himself into
legitimation; Singaporean society. In support, he cites Oshita v. Republic23 and Calderon v.
o (c) when the change will avoid confusion; Republic,24 which, however, are not apropos both.
o (d) when one has continuously used and been known since x In Oshita, the petitioner therein, a legitimate daughter of a Filipino mother,
childhood by a Filipino name, and was unaware of alien parentage; Buena Bartolome, and a Japanese father, Kishimatsu Oshita, sought to change her
o (e) a sincere desire to adopt a Filipino name to erase signs of name from Antonina B. Oshita to Antonina Bartolome. The Court granted her
former alienage, all in good faith and without prejudicing anybody; petition based on the following considerations: she had elected Philippine
and citizenship upon reaching the age of majority; her other siblings who had also
o (f) when the surname causes embarrassment and there is no elected Philippine citizenship have been using their mother’s surname; she was
showing that the desired change of name was for a fraudulent embarrassed to bear a Japanese surname there still being ill feeling against the
purpose or that the change of name would prejudice public Japanese due to the last World War; and there was no showing that the change
interest.16 of name was motivated by a fraudulent purpose or that it will prejudice public
x In granting or denying petitions for change of name, the question of proper and interest.
reasonable cause is left to the sound discretion of the court. The evidence x In Calderon, the Court allowed petitioner Gertrudes Josefina del Prado, an
presented need only be satisfactory to the court and not all the best illegitimate minor child acting through her mother who filed the petition in her
evidence available. What is involved is not a mere matter of allowance or behalf, to change her name to Gertudes Josefina Calderon, taking the surname
disallowance of the request, but a judicious evaluation of the sufficiency and of her stepfather, Romeo C. Calderon, her mother’s husband. The Court held
propriety of the justifications advanced in support thereof, mindful of the that a petition for change of name of an infant should be granted where to do is
consequent results in the event of its grant and with the sole prerogative for clearly for the best interest of the child. The Court took into consideration the
making such determination being lodged in the courts.17 opportunity provided for the minor petitioner to eliminate the stigma of
illegitimacy which she would carry if she continued to use the surname of her
ISSUE # 2: Whether a person may be allowed to drop his middle name. illegitimate father. The Court pronounced that justice dictates that every person
be allowed to avail of any opportunity to improve his social standing as long as
HELD # 2: NO. doing so he does not cause prejudice or injury to the interests of the State or of
x The petition before us is unlike other petitions for change of name, as it does not other people.
simply seek to change the name of the minor petitioner and adopt another, but x Petitioner cites Alfon v. Republic,25 in arguing that although Article 174 of the
instead seeks to drop the middle name altogether. Decided cases in this Family Code gives the legitimate child the right to use the surnames of the father
jurisdiction involving petitions for change of name usually deal with requests for and the mother, it is not mandatory such that the child could use only one family
change of surname. There are only a handful of cases involving requests name, even the family name of the mother. In Alfon, the petitioner therein, the
for change of the given name18 and none on requests for changing or legitimate daughter of Filomeno Duterte and Estrella Alfon, sought to change her
dropping of the middle name. Does the law allow one to drop the name from Maria Estrella Veronica Primitiva Duterte (her name as registered in
middle name from his registered name? We have to answer in the the Local Civil Registry) to Estrella S. Alfon (the name she had been using since
negative. childhood, in her school records and in her voter’s registration). The trial court
x A discussion on the legal significance of a person’s name is relevant at this point. denied her petition but this Court overturned the denial, ruling that while Article
We quote, thus: 364 of the Civil Code states that she, as a legitimate child, should principally use
o …For all practical and legal purposes, a man's name is the the surname of her father, there is no legal obstacle for her to choose to use the
designation by which he is known and called in the community in surname of herm other to which she is entitled. In addition, the Court found that
which he lives and is best known. It is defined as the word or there was ample justification to grant her petition, i.e., to avoid confusion.
combination of words by which a person is distinguished from other x Weighing petitioner’s reason of convenience for the change of his name against
individuals and, also, as the label or appellation which he bears for the standards set in the cases he cites to support his contention would show that
the convenience of the world at large addressing him, or in his justification is amorphous, to say the least, and could not warrant favorable
speaking of or dealing with him. Names are used merely as one action on his petition.
method of indicating the identity of persons; they are descriptive of x The factual antecedents and unique circumstances of the cited cases are not at
persons for identification, since, the identity is the essential thing all analogous to the case at bar. The instant case is clearly distinguishable from
the cases of Oshita and Alfon, where the petitioners were already of age when
152
RECTO, GAYLE ANGELI M.
2011-0008 | AUSL
Personal Notes on Remedial Law 2 Review (based on the syllabus of Prof. Henedino M. Brondial)

they filed their petitions for change of name. Being of age, they are considered HELD: NO.
to have exercised their discretion and judgment, fully knowing the effects of their x Rule 108 of the Rules of Court vis a vis Article 412 of the Civil Code15 charts the
decision to change their surnames. It can also be unmistakably observed that the procedure by which an entry in the civil registry may be cancelled or corrected.
reason for the grant of the petitions for change of name in these two cases was The proceeding contemplated therein may generally be used only to correct
the presence of reasonable or compelling grounds therefore. The Court, in clerical, spelling, typographical and other innocuous errors in the civil registry. A
Oshita, recognized the tangible animosity most Filipinos had during that time clerical error is one which is visible to the eyes or obvious to the
against the Japanese as a result of World War II, in addition to the fact of understanding; an error made by a clerk or a transcriber; a mistake in
therein petitioner’s election of Philippine citizenship. In Alfon, the Court granted copying or writing, or a harmless change such as a correction of name
the petition since the petitioner had been known since childhood by a name that is clearly misspelled or of a misstatement of the occupation of the
different from her registered name and she had not used her registered name in parent. Substantial or contentious alterations may be allowed only in
her school records and voter’s registration records; thus, denying the petition adversarial proceedings, in which all interested parties are impleaded
would only result to confusion. and due process is properly observed.16
x Calderon, on the other hand, granted the petition for change of name filed by a x The allegations of the petition filed before the trial court clearly show that
mother in behalf of her illegitimate minor child. Petitioner cites this case to petitioners seek to nullify the marriage between Pablo and Lucille on the ground
buttress his argument that he does not have to reach the age of majority to that it is bigamous and impugn Patrick’s filiation in connection with which they
petition for change of name. However, it is manifest in Calderon that the Court, ask the court to order Patrick to be subjected to a DNA test.
in granting the petition for change of name, gave paramount consideration to x Petitioners insist, however, that the main cause of action is for the correction of
the best interests of the minor petitioner therein. Patrick’s birth records17 and that the rest of the prayers are merely incidental
x In the case at bar, the only reason advanced by petitioner for the thereto.
dropping his middle name is convenience. However, how such change x Petitioners’ position does not lie. Their cause of action is actually to seek
of name would make his integration into Singaporean society easier the declaration of Pablo and Lucille’s marriage as void for being
and convenient is not clearly established. That the continued use of his bigamous and impugn Patrick’s legitimacy, which causes of action are
middle name would cause confusion and difficulty does not constitute governed not by Rule 108 but by A.M. No. 02-11-10-SC which took
proper and reasonable cause to drop it from his registered complete effect on March 15, 2003, and Art. 17118 of the Family Code,
name. respectively, hence, the petition should be filed in a Family Court as
x In addition, petitioner is only a minor. Considering the nebulous foundation expressly provided in said Code.1avvphi1
on which his petition for change of name is based, it is best that the matter of x It is well to emphasize that, doctrinally, validity of marriages as well as
change of his name be left to his judgment and discretion when he reaches the legitimacy and filiation can be questioned only in a direct action
age of majority.26 As he is of tender age, he may not yet understand and seasonably filed by the proper party, and not through collateral attack
appreciate the value of the change of his name and granting of the same at this such as the petition filed before the court a quo.
point may just prejudice him in his rights under our laws. o Petitioners’ reliance on the cases they cited is misplaced.
x Cariño v. Cariño was an action filed by a second wife against the first wife for
MA. CRISTINA TORRES BRAZA, PAOLO JOSEF T. BRAZA and JANELLE ANN T. BRAZA, the return of one-half of the death benefits received by the first after the death
Petitioners, vs. THE CITY CIVIL REGISTRAR OF HIMAMAYLAN CITY, NEGROS of the husband. Since the second wife contracted marriage with the husband
OCCIDENTAL, minor PATRICK ALVIN TITULAR BRAZA, represented by LEON TITULAR, while the latter’s marriage to the first wife was still subsisting, the Court ruled on
CECILIA TITULAR and LUCILLE C. TITULAR, Respondents. the validity of the two marriages, it being essential to the determination of who is
G.R. No. 181174 December 4, 2009 rightfully entitled to the death benefits.
FIRST DIVISION x In Lee v. Court of Appeals, the Court held that contrary to the contention that
the petitions filed by the therein petitioners before the lower courts were actions
FACTS: to impugn legitimacy, the prayer was not to declare that the petitioners are
x Cristina and Pablo were married on January 4, 1978 illegitimate children of Keh Shiok Cheng as stated in their records of birth but to
o 2 children: Paolo and Janelle establish that they are not the latter’s children, hence, there was nothing to
x Pablo died in an accident impugn as there was no blood relation at all between the petitioners and Keh
x Lucille Æ went to the wake AND began introducing “Patrick Alvin Titular Braza” Shiok Cheng. That is why the Court ordered the cancellation of the name of Keh
as her son with Pablo Shiok Cheng as the petitioners’ mother and the substitution thereof with "Tiu
x Cristina Æ found Patrick’s birth certificate Chuan" who is their biological mother. Thus, the collateral attack was allowed
o Name of Child : PATRICK ALVIN CELESTIAL TITULAR and the petition deemed as adversarial proceeding contemplated under Rule
o Annotation/Remarks : "Acknowledge (sic) by the father Pablito 108.
Braza on January 13, 1997" x In Republic v. Kho, it was the petitioners themselves who sought the
o Remarks : Legitimated by virtue of subsequent marriage of correction of the entries in their respective birth records to reflect that they were
parents on April 22, 1998 at Manila. Henceforth, the child shall be illegitimate and that their citizenship is "Filipino," not Chinese, because their
known as Patrick Alvin Titular Braza (Emphasis and underscoring parents were never legally married. Again, considering that the changes sought to
supplied) be made were substantial and not merely innocuous, the Court, finding the
x Cristina Æ likewise found a marriage certificate showing that Lucille and Pablo proceedings under Rule 108 to be adversarial in nature, upheld the lower court’s
were married on April 22, 1998 grant of the petition.
x Cristina (and children) Æ filed a petition to correct the entries in the birth record x It is thus clear that the facts in the above-cited cases are vastly different from
of Patrick in the Local Civil Register before RTC Himamaylan City, Negros those obtaining in the present case.
Occidental
o Contending that Patrick could not have been legitimated by the
supposed marriage between Lucille and Pablo, said marriage being ROMMEL JACINTO DANTES SILVERIO, petitioner, vs. REPUBLIC OF THE
bigamous on account of the valid and subsisting marriage between PHILIPPINES, respondent.
Ma. Cristina and Pablo, petitioners prayed for G.R. No. 174689 October 22, 2007
ƒ (1) the correction of the entries in Patrick's birth FIRST DIVISION
record with respect to his legitimation, the name of
the father and his acknowledgment, and the use of FACTS:
the last name "Braza"; x Silverio was born a male with Rommel Jacinto Dantes Silverio as his registered
ƒ (2) a directive to Leon, Cecilia and Lucille, all name
surnamed Titular, as guardians of the minor Patrick, x He is a male transsexual
to submit Parick to DNA testing to determine his o underwent psychological examination, hormone treatment and
paternity and filiation; and breast augmentation in US
ƒ (3) the declaration of nullity of the legitimation of o underwent sex reassignment surgery2 in Bangkok, Thailand
Patrick as stated in his birth certificate and, for this o was thereafter examined by Dr. Marcelino Reysio-Cruz, Jr., a plastic
purpose, the declaration of the marriage of Lucille and reconstruction surgeon in the Philippines, who issued a medical
and Pablo as bigamous. certificate attesting that he (petitioner) had in fact undergone the
x Patrick Æ filed an MD on the ground of lack of jurisdiction procedure
x RTC Æ granted MD; dismissed the case x He then lived as a female and was engaged to be married
o dismissed the petition without prejudice, it holding that in a special x Silverio Æ filed a petition for the change of his first name and sex in his birth
proceeding for correction of entry, the court, which is not acting as certificate before RTC Manila Br. 8
a family court under the Family Code, has no jurisdiction over an o sought to have his name in his birth certificate changed from
action to annul the marriage of Lucille and Pablo, impugn the "Rommel Jacinto" to "Mely," and his sex from "male" to "female."
legitimacy of Patrick, and order Patrick to be subjected to a DNA x RTC Æ GRANTED
test, hence, the controversy should be ventilated in an ordinary o Firstly, the [c]ourt is of the opinion that granting the petition would
adversarial action. be more in consonance with the principles of justice and equity.
x Cristina (and children) Æ filed an MR With his sexual [re-assignment], petitioner, who has always felt,
o RTC Æ denied thought and acted like a woman, now possesses the physique of a
x Cristina (and children) Æ filed a Rule 45 before the SC female. Petitioner’s misfortune to be trapped in a man’s body is not
his own doing and should not be in any way taken against him.
ISSUE: Whether the RTC may correctly nullify marriages and rule on legitimacy and filiation in o Likewise, the [c]ourt believes that no harm, injury [or] prejudice
a petition under Rule 108. will be caused to anybody or the community in granting the
petition. On the contrary, granting the petition would bring the
153
RECTO, GAYLE ANGELI M.
2011-0008 | AUSL
Personal Notes on Remedial Law 2 Review (based on the syllabus of Prof. Henedino M. Brondial)

much-awaited happiness on the part of the petitioner and her the use of his true and official name does not prejudice him at all. For all these
[fiancé] and the realization of their dreams. reasons, the Court of Appeals correctly dismissed petitioner’s petition in so far as
o Finally, no evidence was presented to show any cause or ground to the change of his first name was concerned.
deny the present petition despite due notice and publication
thereof. Even the State, through the [OSG] has not seen fit to ISSUE # 2: Whether Silverio’s sex in his birth certificate may be changed under RA 9048.
interpose any [o]pposition.
x Republic Æ filed a Rule 65 before CA HELD # 2: NO.
o It alleged that there is no law allowing the change of entries in the x The determination of a person’s sex appearing in his birth certificate is
birth certificate by reason of sex alteration. a legal issue and the court must look to the statutes.21 In this
x CA Æ reversed RTC connection, Article 412 of the Civil Code provides:
o It ruled that the trial court’s decision lacked legal basis. There is no o ART. 412. No entry in the civil register shall be changed or
law allowing the change of either name or sex in the certificate of corrected without a judicial order.
birth on the ground of sex reassignment through surgery. Thus, the x Together with Article 376 of the Civil Code, this provision was amended by RA
Court of Appeals granted the Republic’s petition, set aside the 9048 in so far as clerical or typographical errors are involved. The correction or
decision of the trial court and ordered the dismissal of SP Case No. change of such matters can now be made through administrative proceedings
02-105207. Petitioner moved for reconsideration but it was denied. and without the need for a judicial order. In effect, RA 9048 removed from the
ambit of Rule 108 of the Rules of Court the correction of such errors.22 Rule
ISSUE # 1: Whether Silverio’s name may be changed in view of his sex reassignment. 108 now applies only to substantial changes and corrections in entries
in the civil register.23
HELD # 1: NO. x Section 2(c) of RA 9048 defines what a "clerical or typographical error" is:
x Petitioner invoked his sex reassignment as the ground for his petition for change x SECTION 2. Definition of Terms. - As used in this Act, the following terms shall
of name and sex. As found by the trial court: mean:
o Petitioner filed the present petition not to evade any law or o (3) "Clerical or typographical error" refers to a mistake
judgment or any infraction thereof or for any unlawful motive but committed in the performance of clerical work in writing, copying,
solely for the purpose of making his birth records compatible with transcribing or typing an entry in the civil register that is harmless
his present sex. (emphasis supplied) and innocuous, such as misspelled name or misspelled place of
x Petitioner believes that after having acquired the physical features of a female, birth or the like, which is visible to the eyes or obvious to the
he became entitled to the civil registry changes sought. understanding, and can be corrected or changed only by reference
o We disagree. to other existing record or records: Provided, however, That no
x The State has an interest in the names borne by individuals and correction must involve the change of nationality, age, status or
entities for purposes of identification.11 A change of name is a sex of the petitioner. (emphasis supplied)
privilege, not a right.12 Petitions for change of name are controlled by x Under RA 9048, a correction in the civil registry involving the change of
statutes.13 In this connection, Article 376 of the Civil Code provides: sex is not a mere clerical or typographical error. It is a substantial
o ART. 376. No person can change his name or surname without change for which the applicable procedure is Rule 108 of the Rules of
judicial authority. Court.
x This Civil Code provision was amended by RA 9048 (Clerical Error Law). In x The entries envisaged in Article 412 of the Civil Code and correctable under Rule
particular, Section 1 of RA 9048 provides: 108 of the Rules of Court are those provided in Articles 407 and 408 of the Civil
o SECTION 1. Authority to Correct Clerical or Typographical Error and Code:24
Change of First Name or Nickname. - No entry in a civil register o ART. 407. Acts, events and judicial decrees concerning the civil
shall be changed or corrected without a judicial order, except for status of persons shall be recorded in the civil register.
clerical or typographical errors and change of first name or o ART. 408. The following shall be entered in the civil register:
nickname which can be corrected or changed by the concerned city ƒ (1) Births;
or municipal civil registrar or consul general in accordance with the ƒ (2) marriages;
provisions of this Act and its implementing rules and regulations. ƒ (3) deaths;
x RA 9048 now governs the change of first name.14 It vests the power and ƒ (4) legal separations;
authority to entertain petitions for change of first name to the city or municipal ƒ (5) annulments of marriage;
civil registrar or consul general concerned. Under the law, therefore, ƒ (6) judgments declaring marriages void from the
jurisdiction over applications for change of first name is now primarily beginning;
lodged with the aforementioned administrative officers. The intent and ƒ (7) legitimations;
effect of the law is to exclude the change of first name from the ƒ (8) adoptions;
coverage of Rules 103 (Change of Name) and 108 (Cancellation or ƒ (9) acknowledgments of natural children;
Correction of Entries in the Civil Registry) of the Rules of Court, until ƒ (10) naturalization;
and unless an administrative petition for change of name is first filed ƒ (11) loss, or
and subsequently denied.15 It likewise lays down the corresponding ƒ (12) recovery of citizenship;
venue,16 form17 and procedure. In sum, the remedy and the proceedings ƒ (13) civil interdiction;
regulating change of first name are primarily administrative in nature, ƒ (14) judicial determination of filiation;
not judicial. ƒ (15) voluntary emancipation of a minor; and
x RA 9048 likewise provides the grounds for which change of first name may be ƒ (16) changes of name.
allowed: x The acts, events or factual errors contemplated under Article 407 of the Civil
o SECTION 4. Grounds for Change of First Name or Nickname. - The Code include even those that occur after birth.25 However, no reasonable
petition for change of first name or nickname may be allowed in interpretation of the provision can justify the conclusion that it covers the
any of the following cases: correction on the ground of sex reassignment.
ƒ (1) The petitioner finds the first name or nickname to x To correct simply means "to make or set aright; to remove the faults or
be ridiculous, tainted with dishonor or extremely error from" while to change means "to replace something with
difficult to write or pronounce; something else of the same kind or with something that serves as a
ƒ (2) The new first name or nickname has been substitute."26 The birth certificate of petitioner contained no error. All entries
habitually and continuously used by the petitioner and therein, including those corresponding to his first name and sex, were all correct.
he has been publicly known by that first name or No correction is necessary.
nickname in the community; or x Article 407 of the Civil Code authorizes the entry in the civil registry of certain
ƒ (3) The change will avoid confusion. acts (such as legitimations, acknowledgments of illegitimate children and
x Petitioner’s basis in praying for the change of his first name was his sex naturalization), events (such as births, marriages, naturalization and deaths) and
reassignment. He intended to make his first name compatible with the sex he judicial decrees (such as legal separations, annulments of marriage, declarations
thought he transformed himself into through surgery. However, a change of of nullity of marriages, adoptions, naturalization, loss or recovery of citizenship,
name does not alter one’s legal capacity or civil status.18 RA 9048 does not civil interdiction, judicial determination of filiation and changes of name). These
sanction a change of first name on the ground of sex reassignment. acts, events and judicial decrees produce legal consequences that touch upon
Rather than avoiding confusion, changing petitioner’s first name for his declared the legal capacity, status and nationality of a person. Their effects are expressly
purpose may only create grave complications in the civil registry and the public sanctioned by the laws. In contrast, sex reassignment is not among those acts or
interest. events mentioned in Article 407. Neither is it recognized nor even mentioned by
x Before a person can legally change his given name, he must present proper any law, expressly or impliedly.
or reasonable cause or any compelling reason justifying such change.19 x "Status" refers to the circumstances affecting the legal situation (that is, the sum
In addition, he must show that he will be prejudiced by the use of his total of capacities and incapacities) of a person in view of his age, nationality and
true and official name.20 In this case, he failed to show, or even allege, any his family membership.27
prejudice that he might suffer as a result of using his true and official name. x The status of a person in law includes all his personal qualities and relations,
x In sum, the petition in the trial court in so far as it prayed for the change of more or less permanent in nature, not ordinarily terminable at his own will, such
petitioner’s first name was not within that court’s primary jurisdiction as the as his being legitimate or illegitimate, or his being married or not. The
petition should have been filed with the local civil registrar concerned, assuming comprehensive term status… include such matters as the beginning and end of
it could be legally done. It was an improper remedy because the proper remedy legal personality, capacity to have rights in general, family relations, and its
was administrative, that is, that provided under RA 9048. It was also filed in the various aspects, such as birth, legitimation, adoption, emancipation, marriage,
wrong venue as the proper venue was in the Office of the Civil Registrar of divorce, and sometimes even succession.28 (emphasis supplied)
Manila where his birth certificate is kept. More importantly, it had no merit since
154
RECTO, GAYLE ANGELI M.
2011-0008 | AUSL
Personal Notes on Remedial Law 2 Review (based on the syllabus of Prof. Henedino M. Brondial)

x A person’s sex is an essential factor in marriage and family relations. It who has undergone sex reassignment the privilege to change his name and sex
is a part of a person’s legal capacity and civil status. In this connection, to conform with his reassigned sex, it has to enact legislation laying down the
Article 413 of the Civil Code provides: guidelines in turn governing the conferment of that privilege.
o ART. 413. All other matters pertaining to the registration of civil x It might be theoretically possible for this Court to write a protocol on when a
status shall be governed by special laws. person may be recognized as having successfully changed his sex. However, this
x But there is no such special law in the Philippines governing sex Court has no authority to fashion a law on that matter, or on anything else. The
reassignment and its effects. This is fatal to petitioner’s cause. x Court cannot enact a law where no law exists. It can only apply or interpret the
Moreover, Section 5 of Act 3753 (the Civil Register Law) provides: written word of its co-equal branch of government, Congress.
o SEC. 5. Registration and certification of births. - The declaration of x Petitioner pleads that "[t]he unfortunates are also entitled to a life of happiness,
the physician or midwife in attendance at the birth or, in default contentment and [the] realization of their dreams." No argument about that. The
thereof, the declaration of either parent of the newborn child, shall Court recognizes that there are people whose preferences and orientation do not
be sufficient for the registration of a birth in the civil register. Such fit neatly into the commonly recognized parameters of social convention and
declaration shall be exempt from documentary stamp tax and shall that, at least for them, life is indeed an ordeal. However, the remedies petitioner
be sent to the local civil registrar not later than thirty days after the seeks involve questions of public policy to be addressed solely by the legislature,
birth, by the physician or midwife in attendance at the birth or by not by the courts.
either parent of the newborn child.
o In such declaration, the person above mentioned shall certify to the
following facts: (a) date and hour of birth; (b) sex and nationality REPUBLIC OF THE PHILIPPINES, Petitioner, vs. JENNIFER B. CAGANDAHAN,
of infant; (c) names, citizenship and religion of parents or, in case Respondent
the father is not known, of the mother alone; (d) civil status of G.R. No. 166676 September 12, 2008
parents; (e) place where the infant was born; and (f) such other SECOND DIVISION
data as may be required in the regulations to be issued.
x Under the Civil Register Law, a birth certificate is a historical record of the facts FACTS:
as they existed at the time of birth.29 Thus, the sex of a person is x Jennifer Cagandahan was born a female
determined at birth, visually done by the birth attendant (the physician o She then developed male characteristics AND was diagnosed with
or midwife) by examining the genitals of the infant. Considering that Congenital Adrenal Hyperplasia (CAH) which is a condition where
there is no law legally recognizing sex reassignment, the determination persons thus afflicted possess both male and female characteristics
of a person’s sex made at the time of his or her birth, if not attended by o her ovarian structures had minimized, she has stopped growing
error,30 is immutable.31 and she has no breast or menstrual development
x When words are not defined in a statute they are to be given their common and o for all interests and appearances as well as in mind and emotion,
ordinary meaning in the absence of a contrary legislative intent. The words she has become a male person
"sex," "male" and "female" as used in the Civil Register Law and laws concerning x Cagandahan Æ filed a petition for Correction of Entries in Birth Certificate before
the civil registry (and even all other laws) should therefore be understood in their RTC Siniloan, Laguna Br 33
common and ordinary usage, there being no legislative intent to the contrary. In x RTC Æ GRANTED
this connection, sex is defined as "the sum of peculiarities of structure and o The Court is convinced that petitioner has satisfactorily shown that
function that distinguish a male from a female"32 or "the distinction between he is entitled to the reliefs prayed [for]. Petitioner has adequately
male and female."33 Female is "the sex that produces ova or bears young"34 presented to the Court very clear and convincing proofs for the
and male is "the sex that has organs to produce spermatozoa for fertilizing granting of his petition. It was medically proven that petitioner’s
ova."35 Thus, the words "male" and "female" in everyday understanding do not body produces male hormones, and first his body as well as his
include persons who have undergone sex reassignment. Furthermore, "words action and feelings are that of a male. He has chosen to be male.
that are employed in a statute which had at the time a well-known meaning are He is a normal person and wants to be acknowledged and
presumed to have been used in that sense unless the context compels to the identified as a male.
contrary."36 Since the statutory language of the Civil Register Law was enacted x Republic Æ filed a Rule 45 before SC
in the early 1900s and remains unchanged, it cannot be argued that the term
"sex" as used then is something alterable through surgery or something that ISSUE: Whether Cangandahan’s petition should prosper considering the circumstances
allows a post-operative male-to-female transsexual to be included in the obtaining.
category "female."
x For these reasons, while petitioner may have succeeded in altering his HELD: YES.
body and appearance through the intervention of modern surgery, no x The OSG argues that the petition below is fatally defective for non-compliance
law authorizes the change of entry as to sex in the civil registry for that with Rules 103 and 108 of the Rules of Court because respondent’s petition did
reason. Thus, there is no legal basis for his petition for the correction or not implead the local civil registrar. Section 3, Rule 108 provides that the civil
change of the entries in his birth certificate. registrar and all persons who have or claim any interest which would be affected
thereby shall be made parties to the proceedings. Likewise, the local civil
ISSUE # 3: Whether equity may be invoked by Silverio in seeking the change of his name and registrar is required to be made a party in a proceeding for the correction of
sex. name in the civil registry. He is an indispensable party without whom no final
determination of the case can be had.[12] Unless all possible indispensable
HELD # 3: NO. parties were duly notified of the proceedings, the same shall be considered as
x The trial court opined that its grant of the petition was in consonance with the falling much too short of the requirements of the rules.13 The corresponding
principles of justice and equity. It believed that allowing the petition would cause petition should also implead as respondents the civil registrar and all other
no harm, injury or prejudice to anyone. persons who may have or may claim to have any interest that would be affected
o This is wrong. thereby.14 Respondent, however, invokes Section 6,[15] Rule 1 of the Rules of
x The changes sought by petitioner will have serious and wide-ranging legal and Court which states that courts shall construe the Rules liberally to promote their
public policy consequences. First, even the trial court itself found that the objectives of securing to the parties a just, speedy and inexpensive disposition of
petition was but petitioner’s first step towards his eventual marriage to his male the matters brought before it. We agree that there is substantial compliance with
fiancé. However, marriage, one of the most sacred social institutions, is a special Rule 108 when respondent furnished a copy of the petition to the local civil
contract of permanent union between a man and a woman.37 One of its registrar.
essential requisites is the legal capacity of the contracting parties who must be a x The determination of a person’s sex appearing in his birth certificate is a legal
male and a female.38 To grant the changes sought by petitioner will issue and the court must look to the statutes. In this connection, Article 412 of
substantially reconfigure and greatly alter the laws on marriage and family the Civil Code provides:
relations. It will allow the union of a man with another man who has undergone o ART. 412. No entry in a civil register shall be changed or corrected
sex reassignment (a male-to-female post-operative transsexual). Second, there without a judicial order.
are various laws which apply particularly to women such as the provisions of the x Together with Article 376[16] of the Civil Code, this provision was amended by
Labor Code on employment of women,39 certain felonies under the Revised Republic Act No. 9048[17] in so far as clerical or typographical errors are
Penal Code40 and the presumption of survivorship in case of calamities under involved. The correction or change of such matters can now be made through
Rule 131 of the Rules of Court,41 among others. These laws underscore the administrative proceedings and without the need for a judicial order. In effect,
public policy in relation to women which could be substantially affected if Rep. Act No. 9048 removed from the ambit of Rule 108 of the Rules of Court the
petitioner’s petition were to be granted. correction of such errors. Rule 108 now applies only to substantial changes and
x It is true that Article 9 of the Civil Code mandates that "[n]o judge or corrections in entries in the civil register.18
court shall decline to render judgment by reason of the silence, x Under Rep. Act No. 9048, a correction in the civil registry involving the change of
obscurity or insufficiency of the law." However, it is not a license for courts to sex is not a mere clerical or typographical error. It is a substantial change for
engage in judicial legislation. The duty of the courts is to apply or interpret the which the applicable procedure is Rule 108 of the Rules of Court.19
law, not to make or amend it. x The entries envisaged in Article 412 of the Civil Code and correctable under Rule
x In our system of government, it is for the legislature, should it choose to do so, 108 of the Rules of Court are those provided in Articles 407 and 408 of the Civil
to determine what guidelines should govern the recognition of the effects of sex Code:
reassignment. The need for legislative guidelines becomes particularly important o ART. 407. Acts, events and judicial decrees concerning the civil
in this case where the claims asserted are statute-based. status of persons shall be recorded in the civil register.
x To reiterate, the statutes define who may file petitions for change of first name o ART. 408. The following shall be entered in the civil register:
and for correction or change of entries in the civil registry, where they may be ƒ (1) Births; (2) marriages; (3) deaths; (4) legal
filed, what grounds may be invoked, what proof must be presented and what separations; (5) annulments of marriage; (6)
procedures shall be observed. If the legislature intends to confer on a person judgments declaring marriages void from the
155
RECTO, GAYLE ANGELI M.
2011-0008 | AUSL
Personal Notes on Remedial Law 2 Review (based on the syllabus of Prof. Henedino M. Brondial)

beginning; (7) legitimations; (8) adoptions; (9) Life is already difficult for the ordinary person. We cannot but respect how
acknowledgments of natural children; (10) respondent deals with his unordinary state and thus help make his life easier,
naturalization; (11) loss, or (12) recovery of considering the unique circumstances in this case.
citizenship; (13) civil interdiction; (14) judicial x As for respondent’s change of name under Rule 103, this Court has
determination of filiation; (15) voluntary emancipation held that a change of name is not a matter of right but of judicial
of a minor; and (16) changes of name. discretion, to be exercised in the light of the reasons adduced and the
x The acts, events or factual errors contemplated under Article 407 of the Civil consequences that will follow.[28] The trial court’s grant of
Code include even those that occur after birth.20 respondent’s change of name from Jennifer to Jeff implies a change of
x Respondent undisputedly has CAH. This condition causes the early or a feminine name to a masculine name. Considering the consequence
"inappropriate" appearance of male characteristics. A person, like that respondent’s change of name merely recognizes his preferred
respondent, with this condition produces too much androgen, a male gender, we find merit in respondent’s change of name. Such a change
hormone. A newborn who has XX chromosomes coupled with CAH usually has a will conform with the change of the entry in his birth certificate from
(1) swollen clitoris with the urethral opening at the base, an ambiguous genitalia female to male.
often appearing more male than female; (2) normal internal structures of the
female reproductive tract such as the ovaries, uterus and fallopian tubes; as the
child grows older, some features start to appear male, such as deepening of the REPUBLIC OF THE PHILIPPINES, PETITIONER, vs. DR. NORMA S. LUGSANAY UY,
voice, facial hair, and failure to menstruate at puberty. About 1 in 10,000 to RESPONDENT.
18,000 children are born with CAH. G.R. No. 198010 August 12, 2013
x CAH is one of many conditions[21] that involve intersex anatomy. During the THIRD DIVISION
twentieth century, medicine adopted the term "intersexuality" to apply to human
beings who cannot be classified as either male or female.[22] The term is now of FACTS:
widespread use. According to Wikipedia, intersexuality "is the state of a living x Uy’s birth certificate shows that her full name is "Anita Sy" when in fact she is
thing of a gonochoristic species whose sex chromosomes, genitalia, and/or allegedly known to her family and friends as "Norma S. Lugsanay."
secondary sex characteristics are determined to be neither exclusively male nor o Her passport and other IDs all bear the name "Norma S.
female. An organism with intersex may have biological characteristics of both Lugsanay."
male and female sexes." x Uy Æ then filed Petition for Correction of Entry in her Certificate of Live Birth (for
x Intersex individuals are treated in different ways by different cultures. In most correction of NAME) before RTC Gingoog City
societies, intersex individuals have been expected to conform to either a male or o allegedly filed earlier a petition for correction of entries with the
female gender role.[23] Since the rise of modern medical science in Western Office of the Local Civil Registrar of Gingoog City to effect the
societies, some intersex people with ambiguous external genitalia have had their corrections on her name and citizenship which was supposedly
genitalia surgically modified to resemble either male or female genitals.[24] More granted
commonly, an intersex individual is considered as suffering from a "disorder" o BUT NSO did not make the necessary corrections
which is almost always recommended to be treated, whether by surgery and/or by o She also alleged that she is an illegitimate child considering that
taking lifetime medication in order to mold the individual as neatly as possible into her parents were never married, so she had to follow the surname
the category of either male or female. of her mother.10
x In deciding this case, we consider the compassionate calls for o She also contended that she is a Filipino citizen and not Chinese,
recognition of the various degrees of intersex as variations which and all her siblings bear the surname Lugsanay and are all Filipinos
should not be subject to outright denial. "It has been suggested that there x RTC Æ granted
is some middle ground between the sexes, a ‘no-man’s land’ for those individuals o The RTC concluded that respondent’s petition would neither
who are neither truly ‘male’ nor truly ‘female’."[25] The current state of prejudice the government nor any third party. It also held that the
Philippine statutes apparently compels that a person be classified names "Norma Sy Lugsanay" and "Anita Sy" refer to one and the
either as a male or as a female, but this Court is not controlled by mere same person, especially since the Local Civil Registrar of Gingoog
appearances when nature itself fundamentally negates such rigid City has effected the correction. Considering that respondent has
classification. continuously used and has been known since childhood as "Norma
x In the instant case, if we determine respondent to be a female, then there is no Sy Lugsanay" and as a Filipino citizen, the RTC granted the petition
basis for a change in the birth certificate entry for gender. But if we determine, to avoid confusion
based on medical testimony and scientific development showing the respondent x Republic Æ appealed to CA
to be other than female, then a change in the subject’s birth certificate entry is in x CA Æ affirmed RTC in toto
order. o The CA held that respondent’s failure to implead other
x Biologically, nature endowed respondent with a mixed (neither consistently and indispensable parties was cured upon the publication of the Order
categorically female nor consistently and categorically male) composition. setting the case for hearing in a newspaper of general circulation
Respondent has female (XX) chromosomes. However, respondent’s body system for three (3) consecutive weeks and by serving a copy of the notice
naturally produces high levels of male hormones (androgen). As a result, to the Local Civil Registrar, the OSG and the City Prosecutor’s
respondent has ambiguous genitalia and the phenotypic features of a male. Office.17
x Ultimately, we are of the view that where the person is biologically or o As to whether the petition is a collateral attack on respondent’s
naturally intersex the determining factor in his gender classification filiation, the CA ruled in favor of respondent, considering that her
would be what the individual, like respondent, having reached the age parents were not legally married and that her siblings’ birth
of majority, with good reason thinks of his/her sex. Respondent here certificates uniformly state that their surname is Lugsanay and their
thinks of himself as a male and considering that his body produces high citizenship is Filipino
levels of male hormones (androgen) there is preponderant biological x Republic Æ filed an MR
support for considering him as being male. Sexual development in o CA Æ denied
cases of intersex persons makes the gender classification at birth x Republic Æ filed a Rule 45 before the SC
inconclusive. It is at maturity that the gender of such persons, like
respondent, is fixed. ISSUE: Whether Uy’s failure to implead indispensable parties, viz, her parents and siblings,
x Respondent here has simply let nature take its course and has not taken was cured by the subsequent publication.
unnatural steps to arrest or interfere with what he was born with. And
accordingly, he has already ordered his life to that of a male. Respondent could HELD: NO.
have undergone treatment and taken steps, like taking lifelong medication,[26] x In this case, respondent sought the correction of entries in her birth certificate,
to force his body into the categorical mold of a female but he did not. He chose particularly those pertaining to her first name, surname and citizenship. She
not to do so. Nature has instead taken its due course in respondent’s sought the correction allegedly to reflect the name which she has been known
development to reveal more fully his male characteristics. for since childhood, including her legal documents such as passport and school
x In the absence of a law on the matter, the Court will not dictate on respondent and professional records. She likewise relied on the birth certificates of her full
concerning a matter so innately private as one’s sexuality and lifestyle blood siblings who bear the surname "Lugsanay" instead of "Sy" and citizenship
preferences, much less on whether or not to undergo medical treatment to of "Filipino" instead of "Chinese." The changes, however, are obviously not mere
reverse the male tendency due to CAH. The Court will not consider respondent clerical as they touch on respondent’s filiation and citizenship. In changing her
as having erred in not choosing to undergo treatment in order to become or surname from "Sy" (which is the surname of her father) to "Lugsanay" (which is
remain as a female. Neither will the Court force respondent to undergo the surname of her mother), she, in effect, changes her status from legitimate to
treatment and to take medication in order to fit the mold of a female, as society illegitimate; and in changing her citizenship from Chinese to Filipino, the same
commonly currently knows this gender of the human species. Respondent is the affects her rights and obligations in this country. Clearly, the changes are
one who has to live with his intersex anatomy. To him belongs the human right substantial.
to the pursuit of happiness and of health. Thus, to him should belong the x It has been settled in a number of cases starting with Republic v. Valencia20 that
primordial choice of what courses of action to take along the path of his sexual even substantial errors in a civil registry may be corrected and the true facts
development and maturation. In the absence of evidence that respondent is an established provided the parties aggrieved by the error avail themselves of the
"incompetent"[27] and in the absence of evidence to show that classifying appropriate adversary proceeding.21 The pronouncement of the Court in that
respondent as a male will harm other members of society who are equally case is illuminating:
entitled to protection under the law, the Court affirms as valid and justified the o It is undoubtedly true that if the subject matter of a petition is not
respondent’s position and his personal judgment of being a male. for the correction of clerical errors of a harmless and innocuous
x In so ruling we do no more than give respect to (1) the diversity of nature; and nature, but one involving nationality or citizenship, which is
(2) how an individual deals with what nature has handed out. In other words, we indisputably substantial as well as controverted, affirmative relief
respect respondent’s congenital condition and his mature decision to be a male. cannot be granted in a proceeding summary in nature. However, it
156
RECTO, GAYLE ANGELI M.
2011-0008 | AUSL
Personal Notes on Remedial Law 2 Review (based on the syllabus of Prof. Henedino M. Brondial)

is also true that a right in law may be enforced and a wrong may be x In Labayo-Rowe v. Republic,36 petitioner filed a petition for the correction of
remedied as long as the appropriate remedy is used. This Court entries in the birth certificates of her children, specifically to change her name from
adheres to the principle that even substantial errors in a civil Beatriz V. Labayu/Beatriz Labayo to Emperatriz Labayo, her civil status from
registry may be corrected and the true facts established provided "married" to "single," and the date and place of marriage from "1953-Bulan" to "No
the parties aggrieved by the error avail themselves of the marriage." The Court modified the trial court’s decision by nullifying the portion
appropriate adversary proceeding. x x x thereof which directs the change of petitioner’s civil status as well as the filiation of
x What is meant by "appropriate adversary proceeding?" Black’s Law Dictionary her child, because it was the OSG only that was made respondent and the
defines "adversary proceeding" as follows: proceedings taken was summary in nature which is short of what is required in
o One having opposing parties; contested, as distinguished from an cases where substantial alterations are sought.
ex parte application, one of which the party seeking relief has given x Respondent’s birth certificate shows that her full name is Anita Sy, that she is a
legal warning to the other party, and afforded the latter an Chinese citizen and a legitimate child of Sy Ton and Sotera Lugsanay. In filing
opportunity to contest it. Excludes an adoption proceeding.22 the petition, however, she seeks the correction of her first name and surname,
x In sustaining the RTC decision, the CA relied on the Court’s conclusion in her status from "legitimate" to "illegitimate" and her citizenship from "Chinese"
Republic v. Kho,23 Alba v. Court of Appeals,24 and Barco v. Court of Appeals,25 to "Filipino." Thus, respondent should have impleaded and notified not
that the failure to implead indispensable parties was cured by the publication of only the Local Civil Registrar but also her parents and siblings as the
the notice of hearing pursuant to the provisions of Rule 108 of the Rules of persons who have interest and are affected by the changes or
Court. In Republic v. Kho,26 petitioner therein appealed the RTC decision corrections respondent wanted to make.
granting the petition for correction of entries despite respondents’ failure to x The fact that the notice of hearing was published in a newspaper of
implead the minor’s mother as an indispensable party. The Court, however, did general circulation and notice thereof was served upon the State will
not strictly apply the provisions of Rule 108, because it opined that it was highly not change the nature of the proceedings taken.37
improbable that the mother was unaware of the proceedings to correct the x A reading of Sections 4 and 5, Rule 108 of the Rules of Court shows that the
entries in her children’s birth certificates especially since the notices, orders and Rules mandate two sets of notices to different potential oppositors:
decision of the trial court were all sent to the residence she shared with them.27 o one given to the persons named in the petition and
x In Alba v. Court of Appeals,28 the Court found nothing wrong with the trial o another given to other persons who are not named in the
court’s decision granting the petition for correction of entries filed by respondent petition but nonetheless may be considered interested or affected
although the proceedings was not actually known by petitioner. In that case, parties.38
petitioner’s mother and guardian was impleaded in the petition for correction of ƒ Summons must, therefore, be served not for
entries, and notices were sent to her address appearing in the subject birth the purpose of vesting the courts with
certificate. However, the notice was returned unserved, because apparently she jurisdiction but to comply with the
no longer lived there. Thus, when she allegedly learned of the granting of the requirements of fair play and due process to
petition, she sought the annulment of judgment which the Court denied. afford the person concerned the opportunity to
Considering that the petition for correction of entries is a proceeding in rem, the protect his interest if he so chooses.39
Court held that acquisition of jurisdiction over the person of the petitioner is, x While there may be cases where the Court held that the failure to implead and
therefore, not required and the absence of personal service was cured by the notify the affected or interested parties may be cured by the publication of the
trial court’s compliance with Rule 108 which requires notice by publication.29 notice of hearing, earnest efforts were made by petitioners in bringing to court
x In Barco v. Court of Appeals,30 the Court addressed the question of whether all possible interested parties.40 Such failure was likewise excused where the
the court acquired jurisdiction over petitioner and all other indispensable parties interested parties themselves initiated the corrections proceedings;41 when there
to the petition for correction of entries despite the failure to implead them in said is no actual or presumptive awareness of the existence of the interested
case. While recognizing that petitioner was indeed an indispensable party, the parties;42 or when a party is inadvertently left out.43
failure to implead her was cured by compliance with Section 4 of Rule 108 which x It is clear from the foregoing discussion that when a petition for cancellation or
requires notice by publication. In so ruling, the Court pointed out that the correction of an entry in the civil register involves substantial and controversial
petitioner in a petition for correction cannot be presumed to be aware of all the alterations, including those on citizenship, legitimacy of paternity or filiation, or
parties whose interests may be affected by the granting of a petition. It legitimacy of marriage, a strict compliance with the requirements of Rule 108
emphasized that the petitioner therein exerted earnest effort to comply with the ofthe Rules of Court is mandated.44 If the entries in the civil register could be
provisions of Rule 108. Thus, the publication of the notice of hearing was corrected or changed through mere summary proceedings and not through
considered to have cured the failure to implead indispensable parties. appropriate action wherein all parties who may be affected by the entries are
x In this case, it was only the Local Civil Registrar of Gingoog City who notified or represented, the door to fraud or other mischief would be set open,
was impleaded as respondent in the petition below. This, the consequence of which might be detrimental and far reaching
notwithstanding, the RTC granted her petition and allowed the correction sought
by respondent, which decision was affirmed in toto by the CA.
o We do not agree with the RTC and the CA. MINORU FUJIKI, PETITIONER, vs. MARIA PAZ GALELA MARINAY, SHINICHI MAEKARA,
x This is not the first time that the Court is confronted with the issue involved in LOCAL CIVIL REGISTRAR OF QUEZON CITY, AND THE ADMINISTRATOR AND CIVIL
this case. Aside from Kho, Alba and Barco, the Court has addressed the same in REGISTRAR GENERAL OF THE NATIONAL STATISTICS OFFICE, RESPONDENTS.
Republic v. Coseteng-Magpayo,31 Ceruila v. Delantar,32 and Labayo-Rowe v. G.R. No. 196049 June 26, 2013
Republic.33 SECOND DIVISION
x In Republic v. Coseteng-Magpayo,34 claiming that his parents were never
legally married, respondent therein filed a petition to change his name from FACTS:
"Julian Edward Emerson Coseteng Magpayo," the name appearing in his birth x Petitioner Minoru Fujiki is a Japanese national who married respondent Maria Paz
certificate to "Julian Edward Emerson Marquez Lim Coseteng." The notice setting Galela Marinay in the Philippines on 23 January 2004.
the petition for hearing was published and there being no opposition thereto, the o The marriage did not sit well with petitioner’s parents. Thus, Fujiki
trial court issued an order of general default and eventually granted respondent’s could not bring his wife to Japan where he resides.
petition deleting the entry on the date and place of marriage of parties; x In 2008, Marinay met another Japanese, Shinichi Maekara.
correcting his surname from "Magpayo" to "Coseteng"; deleting the entry x Without the first marriage being dissolved, Marinay and Maekara were married
"Coseteng" for middle name; and deleting the entry "Fulvio Miranda Magpayo, on 15 May 2008 in Quezon City, Philippines.
Jr." in the space for his father. The Republic of the Philippines, through the OSG, x Fujiki and Marinay met in Japan and they were able to reestablish their
assailed the RTC decision on the grounds that the corrections made on relationship.
respondent’s birth certificate had the effect of changing the civil status from x In 2010, Fujiki helped Marinay obtain a judgment from a family court in Japan
legitimate to illegitimate and must only be effected through an appropriate which declared the marriage between Marinay and Maekara void on the ground
adversary proceeding. The Court nullified the RTC decision for respondent’s of bigamy.
failure to comply strictly with the procedure laid down in Rule 108 of the Rules of x Fujiki Æ then filed a petition in the RTC entitled: "Judicial Recognition of Foreign
Court. Aside from the wrong remedy availed of by respondent as he filed a Judgment (or Decree of Absolute Nullity of Marriage)."
petition for Change of Name under Rule 103 of the Rules of Court, assuming that x RTC Æ dismissed the petition
he filed a petition under Rule 108 which is the appropriate remedy, the petition o citing the provisions of the Rule on Declaration of Absolute Nullity
still failed because of improper venue and failure to implead the Civil Registrar of of Void Marriages and Annulment of Voidable Marriages (A.M. No.
Makati City and all affected parties as respondents in the case. 02-11-10-SC) which provides that a petition for declaration of
x In Ceruila v. Delantar,35 the Ceruilas filed a petition for the cancellation and absolute nullity of void marriage may be filed solely by the husband
annulment of the birth certificate of respondent on the ground that the same or the wife, in this case either Maekara or Marinay.
was made as an instrument of the crime of simulation of birth and, therefore, x Fujiki Æ filed an MR
invalid and spurious, and it falsified all material entries therein. The RTC issued o argued that Rule 108 (Cancellation or Correction of Entries in the
an order setting the case for hearing with a directive that the same be published Civil Registry) of the Rules of Court is applicable. Rule 108 is the
and that any person who is interested in the petition may interpose his comment "procedural implementation" of the Civil Register Law (Act No.
or opposition on or before the scheduled hearing. Summons was likewise sent to 3753) in relation to Article 413 of the Civil Code.
the Civil Register of Manila. After which, the trial court granted the petition and o The Civil Register Law imposes a duty on the "successful petitioner
nullified respondent’s birth certificate. Few months after, respondent filed a for divorce or annulment of marriage to send a copy of the final
petition for the annulment of judgment claiming that she and her guardian were decree of the court to the local registrar of the municipality where
not notified of the petition and the trial court’s decision, hence, the latter was the dissolved or annulled marriage was solemnized."
issued without jurisdiction and in violation of her right to due process. The Court o Section 2 of Rule 108 provides that entries in the civil registry
annulled the trial court’s decision for failure to comply with the requirements of relating to "marriages," "judgments of annulments of marriage"
Rule 108, especially the non-impleading of respondent herself whose birth and "judgments declaring marriages void from the beginning" are
certificate was nullified.1âwphi1 subject to cancellation or correction. The petition in the RTC sought
157
RECTO, GAYLE ANGELI M.
2011-0008 | AUSL
Personal Notes on Remedial Law 2 Review (based on the syllabus of Prof. Henedino M. Brondial)

(among others) to annotate the judgment of the Japanese Family HELD # 2: YES.
Court on the certificate of marriage between Marinay and Maekara. x Since the recognition of a foreign judgment only requires proof of fact of the
x RTC Æ denied MR judgment, it may be made in a special proceeding for cancellation or correction
o The RTC considered the petition as a collateral attack on the of entries in the civil registry under Rule 108 of the Rules of Court. Rule 1,
validity of marriage between Marinay and Maekara. The trial court Section 3 of the Rules of Court provides that "[a] special proceeding is a remedy
held that this is a "jurisdictional ground" to dismiss the petition.28 by which a party seeks to establish a status, a right, or a particular fact."
Moreover, the verification and certification against forum shopping of x Rule 108 creates a remedy to rectify facts of a person’s life which are recorded
the petition was not authenticated as required under Section 529 of by the State pursuant to the Civil Register Law or Act No. 3753. These are facts
A.M. No. 02-11-10-SC. Hence, this also warranted the of public consequence such as birth, death or marriage, which the State has an
"immediate dismissal" of the petition under the same provision. interest in recording. As noted by the Solicitor General, in Corpuz v. Sto. Tomas
x Fujiki Æ filed a Rule 45 before the SC this Court declared that "[t]he recognition of the foreign divorce decree may be
x SolGen Æ filed a Comment made in a Rule 108 proceeding itself, as the object of special proceedings (such
o contended that the petition to recognize the Japanese Family Court as that in Rule 108 of the Rules of Court) is precisely to establish the status or
judgment may be made in a Rule 108 proceeding. right of a party or a particular fact."
o Corpuz v. Santo Tomas: The recognition of the foreign divorce x Rule 108, Section 1 of the Rules of Court states:
decree may be made in a Rule 108 proceeding itself, as the object o Sec. 1. Who may file petition. — Any person interested in any
of special proceedings (such as that in Rule 108 of the Rules of act, event, order or decree concerning the civil status of persons
Court) is precisely to establish the status or right of a party or a which has been recorded in the civil register, may file a verified
particular fact. petition for the cancellation or correction of any entry relating
o While Corpuz concerned a foreign divorce decree, in the present thereto, with the Regional Trial Court of the province where the
case the Japanese Family Court judgment also affected the civil corresponding civil registry is located. (Emphasis supplied)
status of the parties, especially Marinay, who is a Filipino citizen. x Fujiki has the personality to file a petition to recognize the Japanese Family Court
o The Solicitor General asserted that Rule 108 of the Rules of Court is judgment nullifying the marriage between Marinay and Maekara on the ground of
the procedure to record "[a]cts, events and judicial decrees bigamy because the judgment concerns his civil status as married to Marinay. For
concerning the civil status of persons" in the civil registry as the same reason he has the personality to file a petition under Rule 108 to cancel
required by Article 407 of the Civil Code. In other words, "[t]he law the entry of marriage between Marinay and Maekara in the civil registry on the
requires the entry in the civil registry of judicial decrees that basis of the decree of the Japanese Family Court.
produce legal consequences upon a person’s legal capacity and x There is no doubt that the prior spouse has a personal and material interest in
status x x x." The Japanese Family Court judgment directly bears maintaining the integrity of the marriage he contracted and the property
on the civil status of a Filipino citizen and should therefore be relations arising from it. There is also no doubt that he is interested in the
proven as a fact in a Rule 108 proceeding. cancellation of an entry of a bigamous marriage in the civil registry, which
compromises the public record of his marriage. The interest derives from the
ISSUE # 1: Whether the Rule on Declaration of Absolute Nullity of Void Marriages and substantive right of the spouse not only to preserve (or dissolve, in limited
Annulment of Voidable Marriages (A.M. No. 02-11-10-SC) is applicable in the present case. instances68) his most intimate human relation, but also to protect his property
interests that arise by operation of law the moment he contracts marriage. These
HELD # 1: NO. property interests in marriage include the right to be supported "in keeping with
x The Rule on Declaration of Absolute Nullity of Void Marriages and Annulment of the financial capacity of the family" and preserving the property regime of the
Voidable Marriages (A.M. No. 02-11-10-SC) does not apply in a petition to marriage.
recognize a foreign judgment relating to the status of a marriage where one of x The case of Braza v. The City Civil Registrar of Himamaylan City, Negros
the parties is a citizen of a foreign country. Occidental where the Court held that a "trial court has no jurisdiction to nullify
x For Philippine courts to recognize a foreign judgment relating to the status of a marriages" in a special proceeding for cancellation or correction of entry under
marriage where one of the parties is a citizen of a foreign country, the petitioner Rule 108 of the Rules of Court is not applicable in the case at bar because Braza
only needs to prove the foreign judgment as a fact under the Rules of Court. To does not involve a recognition of a foreign judgment nullifying a bigamous
be more specific, a copy of the foreign judgment may be admitted in evidence marriage where one of the parties is a citizen of the foreign country.
and proven as a fact under Rule 132, Sections 24 and 25, in relation to Rule 39, x To be sure, a petition for correction or cancellation of an entry in the civil registry
Section 48(b) of the Rules of Court. cannot substitute for an action to invalidate a marriage. A direct action is
x A foreign judgment relating to the status of a marriage affects the civil status, necessary to prevent circumvention of the substantive and procedural safeguards
condition and legal capacity of its parties. However, the effect of a foreign of marriage under the Family Code, A.M. No. 02-11-10-SC and other related
judgment is not automatic. To extend the effect of a foreign judgment in the laws.
Philippines, Philippine courts must determine if the foreign judgment is consistent x A direct action for declaration of nullity or annulment of marriage is also
with domestic public policy and other mandatory laws. necessary to prevent circumvention of the jurisdiction of the Family Courts under
x A petition to recognize a foreign judgment declaring a marriage void does not the Family Courts Act of 1997 (Republic Act No. 8369), as a petition for
require relitigation under a Philippine court of the case as if it were a new cancellation or correction of entries in the civil registry may be filed in the
petition for declaration of nullity of marriage. Philippine courts cannot presume to Regional Trial Court "where the corresponding civil registry is located." In other
know the foreign laws under which the foreign judgment was rendered. They words, a Filipino citizen cannot dissolve his marriage by the mere expedient of
cannot substitute their judgment on the status, condition and legal capacity of the changing his entry of marriage in the civil registry.
foreign citizen who is under the jurisdiction of another state. Thus, x However, this does not apply in a petition for correction or cancellation of a civil
Philippine courts can only recognize the foreign judgment as a fact registry entry based on the recognition of a foreign judgment annulling a
according to the rules of evidence. marriage where one of the parties is a citizen of the foreign country. There is
x Section 48(b), Rule 39 of the Rules of Court provides that a foreign judgment or neither circumvention of the substantive and procedural safeguards of marriage
final order against a person creates a "presumptive evidence of a right as under Philippine law, nor of the jurisdiction of Family Courts under R.A. No.
between the parties and their successors in interest by a subsequent title." 8369. A recognition of a foreign judgment is not an action to nullify a marriage.
Moreover, Section 48 of the Rules of Court states that "the judgment or final order It is an action for Philippine courts to recognize the effectivity of a foreign
may be repelled by evidence of a want of jurisdiction, want of notice to the party, judgment, which presupposes a case which was already tried and decided under
collusion, fraud, or clear mistake of law or fact." Thus, Philippine courts exercise foreign law. The procedure in A.M. No. 02-11-10-SC does not apply in a petition
limited review on foreign judgments. Courts are not allowed to delve into the to recognize a foreign judgment annulling a bigamous marriage where one of the
merits of a foreign judgment. parties is a citizen of the foreign country. Neither can R.A. No. 8369 define the
x Since 1922 in Adong v. Cheong Seng Gee,63 Philippine courts have recognized jurisdiction of the foreign court.
foreign divorce decrees between a Filipino and a foreign citizen if they are
successfully proven under the rules of evidence.64 Divorce involves the ISSUE # 3: Whether RTC can recognize the foreign judgment in a proceeding for cancellation
dissolution of a marriage, but the recognition of a foreign divorce decree does or correction of entries in the Civil Registry under Rule 108 of the Rules of Court.
not involve the extended procedure under A.M. No. 02-11-10-SC or the rules of
ordinary trial. While the Philippines does not have a divorce law, Philippine courts HELD # 3: YES.
may, however, recognize a foreign divorce decree under the second paragraph of x In Braza v. The City Civil Registrar of Himamaylan City, Negros Occidental, this
Article 26 of the Family Code, to capacitate a Filipino citizen to remarry when his Court held that a "trial court has no jurisdiction to nullify marriages" in a special
or her foreign spouse obtained a divorce decree abroad proceeding for cancellation or correction of entry under Rule 108 of the Rules of
x There is therefore no reason to disallow Fujiki to simply prove as a fact the Court.81 Thus, the "validity of marriage[] x x x can be questioned only in a direct
Japanese Family Court judgment nullifying the marriage between Marinay and action" to nullify the marriage.82 The RTC relied on Braza in dismissing the
Maekara on the ground of bigamy. While the Philippines has no divorce law, the petition for recognition of foreign judgment as a collateral attack on the marriage
Japanese Family Court judgment is fully consistent with Philippine public policy, between Marinay and Maekara.
as bigamous marriages are declared void from the beginning under Article 35(4) x Braza is not applicable because Braza does not involve a recognition of a foreign
of the Family Code. Bigamy is a crime under Article 349 of the Revised Penal judgment nullifying a bigamous marriage where one of the parties is a citizen of
Code. Thus, Fujiki can prove the existence of the Japanese Family Court the foreign country.
judgment in accordance with Rule 132, Sections 24 and 25, in relation to Rule x To be sure, a petition for correction or cancellation of an entry in the
39, Section 48(b) of the Rules of Court. civil registry cannot substitute for an action to invalidate a marriage. A
direct action is necessary to prevent circumvention of the substantive and
ISSUE # 2: Whether the Regional Trial Court can recognize the foreign judgment in a procedural safeguards of marriage under the Family Code, A.M. No. 02-11-10-SC
proceeding for cancellation or correction of entries in the Civil Registry under Rule 108 of the and other related laws. Among these safeguards are the requirement of proving
Rules of Court. the limited grounds for the dissolution of marriage,83 support pendente lite of
the spouses and children,84 the liquidation, partition and distribution of the
158
RECTO, GAYLE ANGELI M.
2011-0008 | AUSL
Personal Notes on Remedial Law 2 Review (based on the syllabus of Prof. Henedino M. Brondial)

properties of the spouses,85 and the investigation of the public prosecutor to already "presumptive evidence of a right between the parties." Upon
determine collusion.86 A direct action for declaration of nullity or annulment of recognition of the foreign judgment, this right becomes conclusive and the
marriage is also necessary to prevent circumvention of the jurisdiction of the judgment serves as the basis for the correction or cancellation of entry in the
Family Courts under the Family Courts Act of 1997 (Republic Act No. 8369), as a civil registry. The recognition of the foreign judgment nullifying a bigamous
petition for cancellation or correction of entries in the civil registry may be filed in marriage is a subsequent event that establishes a new status, right and fact92
the Regional Trial Court "where the corresponding civil registry is located."87 In that needs to be reflected in the civil registry. Otherwise, there will be an
other words, a Filipino citizen cannot dissolve his marriage by the mere inconsistency between the recognition of the effectivity of the foreign judgment
expedient of changing his entry of marriage in the civil registry. and the public records in the Philippines.1âwphi1
x However, this does not apply in a petition for correction or cancellation x However, the recognition of a foreign judgment nullifying a bigamous
of a civil registry entry based on the recognition of a foreign judgment marriage is without prejudice to prosecution for bigamy under Article
annulling a marriage where one of the parties is a citizen of the foreign 349 of the Revised Penal Code.93 The recognition of a foreign
country. There is neither circumvention of the substantive and procedural judgment nullifying a bigamous marriage is not a ground for extinction
safeguards of marriage under Philippine law, nor of the jurisdiction of Family of criminal liability under Articles 89 and 94 of the Revised Penal Code.
Courts under R.A. No. 8369. A recognition of a foreign judgment is not an action Moreover, under Article 91 of the Revised Penal Code, "[t]he term of
to nullify a marriage. It is an action for Philippine courts to recognize the prescription [of the crime of bigamy] shall not run when the offender is
effectivity of a foreign judgment, which presupposes a case which was absent from the Philippine archipelago."
already tried and decided under foreign law. The procedure in A.M. No. 02- x Since A.M. No. 02-11-10-SC is inapplicable, the Court no longer sees the need to
11-10-SC does not apply in a petition to recognize a foreign judgment annulling address the questions on venue and the contents and form of the petition under
a bigamous marriage where one of the parties is a citizen of the foreign country. Sections 4 and 5, respectively, of A.M. No. 02-11-10-SC.
Neither can R.A. No. 8369 define the jurisdiction of the foreign court.
x Article 26 of the Family Code confers jurisdiction on Philippine courts to extend
the effect of a foreign divorce decree to a Filipino spouse without undergoing REPUBLIC OF THE PHILIPPINES, Petitioner, vs. MERLINDA L. OLAYBAR, Respondent.
trial to determine the validity of the dissolution of the marriage. The second G.R. No. 189538 February 10, 2014
paragraph of Article 26 of the Family Code provides that "[w]here a marriage THIRD DIVISION
between a Filipino citizen and a foreigner is validly celebrated and a divorce is
thereafter validly obtained abroad by the alien spouse capacitating him or her to FACTS:
remarry, the Filipino spouse shall have capacity to remarry under Philippine law." x Upon requesting for a Certificate of No Marriage from the NSO, respondent
In Republic v. Orbecido,88 this Court recognized the legislative intent of the learned that she was allegedly married to a Korean national named Ye Son Sune
second paragraph of Article 26 which is "to avoid the absurd situation where the before the MTCC.
Filipino spouse remains married to the alien spouse who, after obtaining a x Denying having contracted said marriage and personally knowing said Ye Son
divorce, is no longer married to the Filipino spouse"89 under the laws of his or Sune, respondent filed with the RTC Cebu a Petition for Cancellation of Entries in
her country. The second paragraph of Article 26 of the Family Code only the Marriage Contract, especially the entries in the wife portion thereof,
authorizes Philippine courts to adopt the effects of a foreign divorce decree impleading both Local Civil Registrar of Cebu and the alleged husband
precisely because the Philippines does not allow divorce. Philippine courts cannot x RTC Æ granted respondent’s petition
try the case on the merits because it is tantamount to trying a case for divorce. o Finding that the signature appearing in the subject marriage
x The second paragraph of Article 26 is only a corrective measure to address the contract was not that of respondent, the court found basis in
anomaly that results from a marriage between a Filipino, whose laws do not granting the latter’s prayer to straighten her record and rectify the
allow divorce, and a foreign citizen, whose laws allow divorce. The anomaly terrible mistake
consists in the Filipino spouse being tied to the marriage while the foreign x Republic Æ filed an MR on the ff grounds:
spouse is free to marry under the laws of his or her country. The correction is o (1) there was no clerical spelling, typographical and other
made by extending in the Philippines the effect of the foreign divorce decree, innocuous errors in the marriage contract for it to fall within the
which is already effective in the country where it was rendered. The second provisions of Rule 108 of the Rules of Court; and
paragraph of Article 26 of the Family Code is based on this Court’s decision in o (2) granting the cancellation of all the entries in the wife portion of
Van Dorn v. Romillo90 which declared that the Filipino spouse "should not be the alleged marriage contract is, in effect, declaring the marriage
discriminated against in her own country if the ends of justice are to be void ab initio
served."91 x RTC Æ denied MR
x The principle in Article 26 of the Family Code applies in a marriage between a o RTC held that it had jurisdiction to take cognizance of cases for
Filipino and a foreign citizen who obtains a foreign judgment nullifying the correction of entries even on substantial errors under Rule 108 of
marriage on the ground of bigamy. The Filipino spouse may file a petition abroad the Rules of Court being the appropriate adversary proceeding
to declare the marriage void on the ground of bigamy. The principle in the required. Considering that respondent’s identity was used by an
second paragraph of Article 26 of the Family Code applies because the foreign unknown person to contract marriage with a Korean national, it
spouse, after the foreign judgment nullifying the marriage, is capacitated to would not be feasible for respondent to institute an action for
remarry under the laws of his or her country. If the foreign judgment is not declaration of nullity of marriage since it is not one of the void
recognized in the Philippines, the Filipino spouse will be discriminated—the marriages under Articles 35 and 36 of the Family Code
foreign spouse can remarry while the Filipino spouse cannot remarry. x Republic Æ filed a Rule 45 before the SC
x Under the second paragraph of Article 26 of the Family Code, Philippine courts
are empowered to correct a situation where the Filipino spouse is still tied to the ISSUE # 1: Whether Rule 108 applies only in cases where there are errors in entries sought to
marriage while the foreign spouse is free to marry. Moreover, notwithstanding be cancelled or corrected.
Article 26 of the Family Code, Philippine courts already have jurisdiction to
extend the effect of a foreign judgment in the Philippines to the extent that the HELD # 1: NO.
foreign judgment does not contravene domestic public policy. A critical difference
between the case of a foreign divorce decree and a foreign judgment nullifying a ISSUE # 2: Whether the cancellation of all entries in the wife portion of the alleged marriage
bigamous marriage is that bigamy, as a ground for the nullity of marriage, is fully contract results in the declaration of nullity of the subject marriage.
consistent with Philippine public policy as expressed in Article 35(4) of the Family
Code and Article 349 of the Revised Penal Code. The Filipino spouse has the HELD # 2: NO.
option to undergo full trial by filing a petition for declaration of nullity of x It is true that in special proceedings, formal pleadings and a hearing may be
marriage under A.M. No. 02-11-10-SC, but this is not the only remedy available dispensed with, and the remedy [is] granted upon mere application or motion.
to him or her. Philippine courts have jurisdiction to recognize a foreign judgment However, a special proceeding is not always summary. The procedure laid
nullifying a bigamous marriage, without prejudice to a criminal prosecution for down in Rule 108 is not a summary proceeding per se. It requires
bigamy. publication of the petition; it mandates the inclusion as parties of all
x In the recognition of foreign judgments, Philippine courts are incompetent to persons who may claim interest which would be affected by the
substitute their judgment on how a case was decided under foreign law. They cancellation or correction; it also requires the civil registrar and any
cannot decide on the "family rights and duties, or on the status, condition and person in interest to file their opposition, if any; and it states that
legal capacity" of the foreign citizen who is a party to the foreign judgment. although the court may make orders expediting the proceedings, it is
Thus, Philippine courts are limited to the question of whether to extend the after hearing that the court shall either dismiss the petition or issue an
effect of a foreign judgment in the Philippines. In a foreign judgment relating to order granting the same. Thus, as long as the procedural requirements
the status of a marriage involving a citizen of a foreign country, Philippine courts in Rule 108 are followed, it is the appropriate adversary proceeding to
only decide whether to extend its effect to the Filipino party, under the rule of lex effect substantial corrections and changes in entries of the civil
nationalii expressed in Article 15 of the Civil Code. register.22
x For this purpose, Philippine courts will only determine x In this case, the entries made in the wife portion of the certificate of marriage
o (1) whether the foreign judgment is inconsistent with an are admittedly the personal circumstances of respondent. The latter, however,
overriding public policy in the Philippines; and claims that her signature was forged and she was not the one who contracted
o (2) whether any alleging party is able to prove an extrinsic marriage with the purported husband. In other words, she claims that no such
ground to repel the foreign judgment, i.e. want of jurisdiction, marriage was entered into or if there was, she was not the one who entered into
want of notice to the party, collusion, fraud, or clear mistake of law such contract. It must be recalled that when respondent tried to obtain a
or fact. CENOMAR from the NSO, it appeared that she was married to a certain Ye Son
x If there is neither inconsistency with public policy nor adequate proof Sune. She then sought the cancellation of entries in the wife portion of the
to repel the judgment, Philippine courts should, by default, recognize marriage certificate.
the foreign judgment as part of the comity of nations. Section 48(b), x In filing the petition for correction of entry under Rule 108, respondent made the
Rule 39 of the Rules of Court states that the foreign judgment is Local Civil Registrar of Cebu City, as well as her alleged husband Ye Son Sune, as
159
RECTO, GAYLE ANGELI M.
2011-0008 | AUSL
Personal Notes on Remedial Law 2 Review (based on the syllabus of Prof. Henedino M. Brondial)

parties-respondents. It is likewise undisputed that the procedural requirements (c) The names and residences of creditors and others who may have any adverse interest over
set forth in Rule 108 were complied with. The Office of the Solicitor General was the property of the absentee;
likewise notified of the petition which in turn authorized the Office of the City
Prosecutor to participate in the proceedings. More importantly, trial was (d) The probable value, location and character of the property belonging to the absentee.
conducted where respondent herself, the stenographer of the court where the
alleged marriage was conducted, as well as a document examiner, testified. Section 4. Time of hearing; notice and publication thereof. — When a petition for the
Several documents were also considered as evidence. With the testimonies and appointment of a representative, or for the declaration of absence and the appointment of a
other evidence presented, the trial court found that the signature appearing in trustee or administrator, is filed, the court shall fix a date and place for the hearing thereof
the subject marriage certificate was different from respondent’s signature where all concerned may appear to contest the petition.
appearing in some of her government issued identification cards.23 The court
thus made a categorical conclusion that respondent’s signature in the marriage Copies of the notice of the time and place fixed for the hearing shall be served upon the known
certificate was not hers and, therefore, was forged. Clearly, it was established heirs, legatees, devisees, creditors and other interested persons, at least ten (10) days before
that, as she claimed in her petition, no such marriage was celebrated. the day of the hearing, and shall be published once a week for three (3) consecutive weeks prior
x Indeed the Court made a pronouncement in the recent case of Minoru Fujiki v. to the time designated for the hearing, in a newspaper of general circulation in the province or
Maria Paz Galela Marinay, Shinichi Maekara, Local Civil Registrar of Quezon City, city where the absentee resides, as the court shall deem best.
and the Administrator and Civil Registrar General of the National Statistics
Office24 that: Section 5. Opposition. — Anyone appearing to contest the petition shall state in writing his
o To be sure, a petition for correction or cancellation of an grounds therefor, and serve a copy thereof on the petitioner and other interested parties on or
entry in the civil registry cannot substitute for an action to before the date designated for the hearing.
invalidate a marriage. A direct action is necessary to
prevent circumvention of the substantive and procedural Section 6. Proof at hearing; order. — At the hearing, compliance with the provisions of
safeguards of marriage under the Family Code, A.M. No. section 4 of this rule must first be shown. Upon satisfactory proof of the allegations in the
02-11-10-SC and other related laws. Among these petition, the court shall issue an order granting the same and appointing the representative,
safeguards are the requirement of proving the limited trustee or administrator for the absentee. The judge shall take the necessary measures to
grounds for the dissolution of marriage, support pendente safeguard the rights and interests of the absentee and shall specify the powers, obligations
lite of the spouses and children, the liquidation, partition and remuneration of his representative, trustee or administrator, regulating them by the rules
and distribution of the properties of the spouses and the concerning guardians.
investigation of the public prosecutor to determine
collusion. A direct action for declaration of nullity or annulment of In case of declaration of absence, the same shall not take effect until six (6) months after its
marriage is also necessary to prevent circumvention of the publication in a newspaper of general circulation designated by the court and in the Official
jurisdiction of the Family Courts under the Family Courts Act of Gazette.
1997 (Republic Act No. 8369), as a petition for cancellation or
correction of entries in the civil registry may be filed in the Regional Section 7. Who may be appointed. — In the appointment of a representative, the spouse
Trial Court where the corresponding civil registry is located. In present shall be preferred when there is no legal separation. If the absentee left no spouse, or
other words, a Filipino citizen cannot dissolve his marriage by the if the spouse present is a minor or otherwise incompetent, any competent person may be
mere expedient of changing his entry of marriage in the civil appointed by the court.
registry.
x Aside from the certificate of marriage, no such evidence was presented to show In case of declaration of absence, the trustee or administrator of the absentee's property shall
the existence of marriage. Rather, respondent showed by overwhelming be appointed in accordance with the preceding paragraph.
evidence that no marriage was entered into and that she was not even aware of
such existence. The testimonial and documentary evidence clearly established Section 8. Termination of administration. — The trusteeship or administration of the
that the only "evidence" of marriage which is the marriage certificate was a property of the absentee shall cease upon order of the court in any of the following cases:
forgery. While we maintain that Rule 108 cannot be availed of to determine the
validity of marriage, we cannot nullify the proceedings before the trial court (a) When the absentee appears personally or by means of an agent;
where all the parties had been given the opportunity to contest the allegations of
respondent; the procedures were followed, and all the evidence of the parties (b) When the death of the absentee is proved and his testate or
had already been admitted and examined. Respondent indeed sought, not the intestate heirs appear;
nullification of marriage as there was no marriage to speak of, but the correction
of the record of such marriage to reflect the truth as set forth by the evidence. (c) When a third person appears, showing by a proper document that
Otherwise stated, in allowing the correction of the subject certificate of marriage he has acquired the absentee's property by purchase or other title.
by cancelling the wife portion thereof, the trial court did not, in any way, declare
the marriage void as there was no marriage to speak of. In these cases the trustee or administrator shall cease in the performance of his office, and
the property shall be placed at the disposal of whose may have a right thereto.
ABSENTEES
[RULE 107]
______________________________________
Section 1. Appointment of representative. — When a person disappears from his
domicile, his whereabouts being unknown, and without having left an agent to administer his
property, or the power conferred upon the agent has expired, any interested party, relative or PREROGATIVE WRITS
friend may petition the Court of First Instance of the place where the absentee resided before
his dis-appearance, for the appointment of a person to represent him provisionally in all that 1. Writ of Amparo
may be necessary. In the City of Manila, the petition shall be filed in the Juvenile and Domestic
Relations Court. A.M. No. 07-9-12-SC
(25 September 2007)
Section 2. Declaration of absence; who may petition. — After the lapse of two (2) years
from his disapperance and without any news about the absentee or since the receipt of the THE RULE ON THE WRIT OF AMPARO
last news, or of five (5) years in case the absentee has left a person in charge of the
administration of his property, the declaration of his absence and appointment of a trustee or SECTION 1. Petition. - The petition for a writ of amparo is a remedy available to any person
administrative may be applied for by any of the following: whose right to life, liberty and security is violated or threatened with violation by an unlawful act
or omission of a public official or employee, or of a private individual or entity.
(a) The spouse present;
The writ shall cover extralegal killings and enforced disappearances or threats thereof.
(b) The heirs instituted in a will, who may present an authentic copy of the same.
SEC. 2. Who May File. - The petition may be filed by the aggrieved party or by any qualified
(c) The relatives who would succeed by the law of intestacy; and person or entity in the following order:

(d) Those who have over the property of the absentee some right subordinated to the Any member of the immediate family, namely: the spouse, children and parents of the
condition of his death. aggrieved party;

Section 3. Contents of petition. — The petition for the appointment of a representative, or Any ascendant, descendant or collateral relative of the aggrieved party within the fourth civil
for the declaration of absence and the appointment of a trustee or an administrator, must degree of consanguinity or affinity, in default of those mentioned in the preceding paragraph;
show the following: or

(a) The jurisdictional facts; Any concerned citizen, organization, association or institution, if there is no known member of
the immediate family or relative of the aggrieved party.
(b) The names, ages, and residences of the heirs instituted in the will, copy of which shall be
presented, and of the relatives who would succeed by the law of intestacy; The filing of a petition by the aggrieved party suspends the right of all other authorized parties
to file similar petitions. Likewise, the filing of the petition by an authorized party on behalf of
the aggrieved party suspends the right of all others, observing the order established herein.

160
RECTO, GAYLE ANGELI M.
2011-0008 | AUSL
Personal Notes on Remedial Law 2 Review (based on the syllabus of Prof. Henedino M. Brondial)

to determine the cause, manner, location and time of death or disappearance as well as any
SEC. 3. Where to File. - The petition may be filed on any day and at any time with the
pattern or practice that may have brought about the death or disappearance; to identify and
Regional Trial Court of the place where the threat, act or omission was committed or any of its
elements occurred, or with the Sandiganbayan, the Court of Appeals, the Supreme Court, or any apprehend the person or persons involved in the death or disappearance; and to bring the
justice of such courts. The writ shall be enforceable anywhere in the Philippines.
suspected offenders before a competent court.
When issued by a Regional Trial Court or any judge thereof, the writ shall be returnable before
such court or judge. The return shall also state other matters relevant to the investigation, its resolution and the
prosecution of the case.
When issued by the Sandiganbayan or the Court of Appeals or any of their justices, it may be
returnable before such court or any justice thereof, or to any Regional Trial Court of the place A general denial of the allegations in the petition shall not be allowed.
where the threat, act or omission was committed or any of its elements occurred.
SEC. 10. Defenses not Pleaded Deemed Waived. — All defenses shall be raised in the
When issued by the Supreme Court or any of its justices, it may be returnable before such return, otherwise, they shall be deemed waived.
Court or any justice thereof, or before the Sandiganbayan or the Court of Appeals or any of
their justices, or to any Regional Trial Court of the place where the threat, act or omission was SEC. 11. Prohibited Pleadings and Motions. - The following pleadings and motions are
committed or any of its elements occurred. prohibited:

SEC. 4. No Docket Fees. - The petitioner shall be exempted from the payment of the docket Motion to dismiss;
and other lawful fees when filing the petition. The court, justice or judge shall docket the
petition and act upon it immediately. Motion for extension of time to file return, opposition, affidavit, position paper and other
pleadings;
SEC. 5. Contents of Petition. - The petition shall be signed and verified and shall allege the
following: Dilatory motion for postponement;

The personal circumstances of the petitioner; Motion for a bill of particulars;

The name and personal circumstances of the respondent responsible for the threat, act or Counterclaim or cross-claim;
omission, or, if the name is unknown or uncertain, the respondent may be described by an
assumed appellation; Third-party complaint;

The right to life, liberty and security of the aggrieved party violated or threatened with violation Reply;
by an unlawful act or omission of the respondent, and how such threat or violation is
committed with the attendant circumstances detailed in supporting affidavits; Motion to declare respondent in default;

The investigation conducted, if any, specifying the names, personal circumstances, and Intervention;
addresses of the investigating authority or individuals, as well as the manner and conduct of
the investigation, together with any report; Memorandum;

The actions and recourses taken by the petitioner to determine the fate or whereabouts of the Motion for reconsideration of interlocutory orders or interim relief orders; and
aggrieved party and the identity of the person responsible for the threat, act or omission; and
Petition for certiorari, mandamus or prohibition against any interlocutory order.
The relief prayed for.
SEC. 12. Effect of Failure to File Return. — In case the respondent fails to file a return,
The petition may include a general prayer for other just and equitable reliefs. the court, justice or judge shall proceed to hear the petition ex parte.

SEC. 6. Issuance of the Writ. - Upon the filing of the petition, the court, justice or judge SEC. 13. Summary Hearing. — The hearing on the petition shall be summary. However, the
shall immediately order the issuance of the writ if on its face it ought to issue. The clerk of court, justice or judge may call for a preliminary conference to simplify the issues and
court shall issue the writ under the seal of the court; or in case of urgent necessity, the justice determine the possibility of obtaining stipulations and admissions from the parties.
or the judge may issue the writ under his or her own hand, and may deputize any officer or
person to serve it. The hearing shall be from day to day until completed and given the same priority as petitions
for habeas corpus.
The writ shall also set the date and time for summary hearing of the petition which shall not
be later than seven (7) days from the date of its issuance. SEC. 14. Interim Reliefs. — Upon filing of the petition or at anytime before final judgment,
the court, justice or judge may grant any of the following reliefs:
SEC. 7. Penalty for Refusing to Issue or Serve the Writ. - A clerk of court who refuses
to issue the writ after its allowance, or a deputized person who refuses to serve the same, (a) Temporary Protection Order. - The court, justice or judge, upon motion or motu proprio,
shall be punished by the court, justice or judge for contempt without prejudice to other may order that the petitioner or the aggrieved party and any member of the immediate family
disciplinary actions. be protected in a government agency or by an accredited person or private institution capable
of keeping and securing their safety. If the petitioner is an organization, association or
SEC. 8. How the Writ is Served. - The writ shall be served upon the respondent by a institution referred to in Section 3(c) of this Rule, the protection may be extended to the
judicial officer or by a person deputized by the court, justice or judge who shall retain a copy officers involved.
on which to make a return of service. In case the writ cannot be served personally on the
respondent, the rules on substituted service shall apply. The Supreme Court shall accredit the persons and private institutions that shall extend
temporary protection to the petitioner or the aggrieved party and any member of the
SEC. 9. Return; Contents. - Within seventy-two (72) hours after service of the writ, the immediate family, in accordance with guidelines which it shall issue.
respondent shall file a verified written return together with supporting affidavits which shall,
among other things, contain the following: The accredited persons and private institutions shall comply with the rules and conditions that
may be imposed by the court, justice or judge.
The lawful defenses to show that the respondent did not violate or threaten with violation the
right to life, liberty and security of the aggrieved party, through any act or omission; (b) Inspection Order. — The court, justice or judge, upon verified motion and after due hearing,
may order any person in possession or control of a designated land or other property, to permit
The steps or actions taken by the respondent to determine the fate or whereabouts of the entry for the purpose of inspecting, measuring, surveying, or photographing the property or any
aggrieved party and the person or persons responsible for the threat, act or omission; relevant object or operation thereon.

All relevant information in the possession of the respondent pertaining to the threat, act or The motion shall state in detail the place or places to be inspected. It shall be supported by
omission against the aggrieved party; and affidavits or testimonies of witnesses having personal knowledge of the enforced
disappearance or whereabouts of the aggrieved party.
If the respondent is a public official or employee, the return shall further state the actions that
have been or will still be taken: If the motion is opposed on the ground of national security or of the privileged nature of the
information, the court, justice or judge may conduct a hearing in chambers to determine the
to verify the identity of the aggrieved party; merit of the opposition.

to recover and preserve evidence related to the death or disappearance of the person The movant must show that the inspection order is necessary to establish the right of the
identified in the petition which may aid in the prosecution of the person or persons aggrieved party alleged to be threatened or violated.
responsible;
The inspection order shall specify the person or persons authorized to make the inspection and
to identify witnesses and obtain statements from them concerning the death or the date, time, place and manner of making the inspection and may prescribe other conditions
disappearance; to protect the constitutional rights of all parties. The order shall expire five (5) days after the
date of its issuance, unless extended for justifiable reasons.
161
RECTO, GAYLE ANGELI M.
2011-0008 | AUSL
Personal Notes on Remedial Law 2 Review (based on the syllabus of Prof. Henedino M. Brondial)

SEC. 24. Substantive Rights. — This Rule shall not diminish, increase or modify substantive
(c) Production Order. - The court, justice or judge, upon verified motion and after due rights recognized and protected by the Constitution.
hearing, may order any person in possession, custody or control of any designated documents,
papers, books, accounts, letters, photographs, objects or tangible things, or objects in digitized SEC. 25. Suppletory Application of the Rules of Court. - The Rules of Court shall apply
or electronic form, which constitute or contain evidence relevant to the petition or the return, suppletorily insofar as it is not inconsistent with this Rule.
to produce and permit their inspection, copying or photographing by or on behalf of the
movant. SEC. 26. Applicability to Pending Cases. - This Rule shall govern cases involving
extralegal killings and enforced disappearances or threats thereof pending in the trial and
The motion may be opposed on the ground of national security or of the privileged nature of appellate courts.
the information, in which case the court, justice or judge may conduct a hearing in chambers
to determine the merit of the opposition. SEC. 27. Effectivity. - This Rule shall take effect on October 24, 2007, following its
publication in three (3) newspapers of general circulation.
The court, justice or judge shall prescribe other conditions to protect the constitutional rights
of all the parties. NOTES (Based on the Annotations of Riguera and Albano; includes
(d) Witness Protection Order. - The court, justice or judge, upon motion or motu proprio, may
Author’s own comments and research)
refer the witnesses to the Department of Justice for admission to the Witness Protection,
Security and Benefit Program, pursuant to Republic Act No. 6981.
x Amparo Æ protection
o Borrowed from the Mexican law
The court, justice or judge may also refer the witnesses to other government agencies, or to x BASIS and NATURE: The "amparo de libertad" transcends the
accredited persons or private institutions capable of keeping and securing their safety. protection of habeas corpus. Once a lawsuit is filed under Habeas
SEC. 15. Availability of Interim Reliefs to Respondent. - Upon verified motion of the
Corpus, Rule 102, Rules of Court, the defendants, government
respondent and after due hearing, the court, justice or judge may issue an inspection order or officers would merely submit the usual defense of alibi or non-
production order under paragraphs (b) and (c) of the preceding section. custody of the body sought to be produced.
o The 1987 Philippine Constitution, however, empowers the
A motion for inspection order under this section shall be supported by affidavits or testimonies Supreme Court of the Philippines to promulgate amparo
of witnesses having personal knowledge of the defenses of the respondent. and Habeas Data, as part of the Rules of Court expressly:
SEC. 16. Contempt. - The court, justice or judge may order the respondent who refuses to
“Promulgate rules concerning the protection and
make a return, or who makes a false return, or any person who otherwise disobeys or resists a enforcement of constitutional rights.”
lawful process or order of the court to be punished for contempt. The contemnor may be x Grounds:
imprisoned or imposed a fine. o Extralegal killings
o Enforced disappearances
SEC. 17. Burden of Proof and Standard of Diligence Required. - The parties shall
establish their claims by substantial evidence.
Writ of Amparo Writ of Habeas Corpus
The respondent who is a private individual or entity must prove that ordinary diligence as
required by applicable laws, rules and regulations was observed in the performance of duty. the confinement OR
there is no color of withholding of restraint has
The respondent who is a public official or employee must prove that extraordinary diligence as legality as to the a color of legality in that it
disappearance/ killing of is usually done under a
required by applicable laws, rules and regulations was observed in the performance of duty. the party process or commitment
order
The respondent public official or employee cannot invoke the presumption that official duty
has been regularly performed to evade responsibility or liability.

SEC. 18. Judgment. — The court shall render judgment within ten (10) days from the time Writ of Amparo Search Warrant
the petition is submitted for decision. If the allegations in the petition are proven by protective; to protect
substantial evidence, the court shall grant the privilege of the writ and such reliefs as may be as to purpose one's right to L,L,S criminal prosecution
proper and appropriate; otherwise, the privilege shall be denied.
MTC/RTC where the crime
SEC. 19. Appeal. - Any party may appeal from the final judgment or order to the Supreme as to concurrent RTC, SB, CA, was committed (venue
jurisdiction SC being jurisdictional in
Court under Rule 45. The appeal may raise questions of fact or law or both.
criminal cases)

The period of appeal shall be five (5) working days from the date of notice of the adverse where the threat or act where the crime was
judgment. as to venue
was committed committed

as to person
The appeal shall be given the same priority as in habeas corpus cases. private/ public person the police
initiating

SEC. 20. Archiving and Revival of Cases. - The court shall not dismiss the petition, but shall personalty shall be seized if
as to seizure personalty is merely
archive it, if upon its determination it cannot proceed for a valid cause such as the failure of of personalty inspected or copied
within the contemplation of
petitioner or witnesses to appear due to threats on their lives. Section 3 Rule 126

A periodic review of the archived cases shall be made by the amparo court that shall, motu
proprio or upon motion by any party, order their revival when ready for further proceedings. x May be enforced anywhere
The petition shall be dismissed with prejudice upon failure to prosecute the case after the x CANNOT implead the President of the PH pursuant to the doctrine of
lapse of two (2) years from notice to the petitioner of the order archiving the case. presidential immunity (Rubrico vs. Pres GMA)
x MAY include the AFP Chief of Staff and the PNP Director,
The clerks of court shall submit to the Office of the Court Administrator a consolidated list of notwithstanding their clear non-involvement in the extralegal killing/
archived cases under this Rule not later than the first week of January of every year.
disappearance for the purpose of determining who the author is
SEC. 21. Institution of Separate Actions. — This Rule shall not preclude the filing of (Rubrico vs. Pres GMA)
separate criminal, civil or administrative actions. x Return must be made within 5 working days from receipt of the writ
o General denial by the respondent is NOT allowed x
SEC. 22. Effect of Filing of a Criminal Action. - When a criminal action has been Interim Reliefs available:
commenced, no separate petition for the writ shall be filed. The reliefs under the writ shall be o Temporary Protection Order (TPO)
available by motion in the criminal case.
ƒ By motion or motu proprio
The procedure under this Rule shall govern the disposition of the reliefs available under the ƒ So that the aggrieved party or his immediate
writ of amparo. family may be protected by a government
institution OR an accredited person or agency
SEC. 23. Consolidation. - When a criminal action is filed subsequent to the filing of a o Inspection Order (IO)
petition for the writ, the latter shall be consolidated with the criminal action. ƒ Upon verified motion AFTER hearing (litigated
motion)
When a criminal action and a separate civil action are filed subsequent to a petition for a writ
of amparo, the latter shall be consolidated with the criminal action. ƒ For the inspection and photographing of the
property or relevant object or operation
After consolidation, the procedure under this Rule shall continue to apply to the disposition of thereto
the reliefs in the petition. ƒ DEFENSE of respondent: national security or
evidentiary privilege
162
RECTO, GAYLE ANGELI M.
2011-0008 | AUSL
Personal Notes on Remedial Law 2 Review (based on the syllabus of Prof. Henedino M. Brondial)

x REMEDY: conduct an “in camera” ƒ REMEDY of the petitioner: motion to


hearing consolidate the two actions before the court
o In camera hearing Æ where the criminal action is pending
one conducted inside o BUT this is without prejudice to the filing of a separate
the chambers of the civil action if grounded on Article 32 of the Civil Code
judge whereby the x WA shall not lie against a Hold Departure Order where a criminal
public is excluded action is already pending and the petition for WA was filed
o Production Order (PO) separately. The remedy of the petitioner is to file a motion to lift the
ƒ Upon verified motion AFTER hearing (litigated HDO before the court where the criminal action is pending. (Reyes
motion) vs. Gonzales)
ƒ For the production of designated objects for x NOTA BENE: The Rule on Writ of Amparo is SC’s answer to the
inspection or copying absence of any legislative measure to curb extrajudicial killings and
ƒ DEFENSE of respondent: national security or enforced disappearances
evidentiary privilege o HOWEVER, on DEC 21, 2012 Pres. NoyNoy approved an
x REMEDY: conduct an “in camera” SB/HB and passed the same as REPUBLIC ACT NO.
hearing 10353 entitled AN ACT DEFINING AND PENALIZING
o Witness Protection Order (WPO) ENFORCED OR INVOLUNTARY DISAPPEARANCE
ƒ By motion or motu proprio
ƒ For the referral of the witness to DOJ’s WPO REPUBLIC ACT NO. 10353
or other government agencies or an
AN ACT DEFINING AND PENALIZING ENFORCED OR INVOLUNTARY
accredited person or agency
DISAPPEARANCE
x These interim reliefs may only be granted by the court BEFORE the
determination of the amparo petition. Thus, after absolving the Be it enacted by the Senate and House of Representatives of the Philippines in Congress
respondents in the petition, the court can no longer grant any interim assembled:
relief in favor of the petitioner. (Yano vs. Sanchez)
x NATURE Of proceedings Æ SUMMARY Section 1. Short Title. -This Act shall be known as the "Anti-Enforced or Involuntary
Disappearance Act of 2012″.
o BUT this petition is NOT governed by the Rule on
Summary Procedure (RSP), thus: Section 2. Declaration of Policy. -The State values the dignity of every human person and
ƒ An MD on any ground is PROHIBITED (vs. in guarantees full respect for human rights for which highest priority shall be given to the
RSP where MD may be allowed on the ground enactment of measures for the enhancement of the right of all people to human dignity, the
of lack of jurisdiction or failure to undergo prohibition against secret detention places, solitary confinement, incommunicado, or other
barangay conciliation proceedings) similar forms of detention, the provision for penal and civil sanctions for such violations, and
compensation and rehabilitation for the victims and their families, particularly with respect to
ƒ Any counterclaim is PROHIBITED (vs. in RSP
the use of torture, force, violence, threat, intimidation or any other means which vitiate the
where compulsory counterclaims are allowed) free will of persons abducted, arrested, detained, disappeared or otherwise removed from the
ƒ Petition for Relief is ALLOWED (vs. in RSP effective protection of the law.
where it is prohibited)
ƒ An MR of an interlocutory order is Furthermore, the State adheres to the principles and standards on the absolute condemnation
PROHIBITED while an MR of a final order/ of human rights violations set by the 1987 Philippine Constitution and various international
instruments such as, but not limited to, the International Covenant on Civil and Political Rights
judgment is ALLOWED (vs. in RSP where it is
(ICCPR), and the Convention Against Torture and Other Cruel, Inhuman or Degrading
the reverse) Treatment or Punishment (CAT), to which the Philippines is a State party.
x A writ of amparo is not the proper remedy in child custody cases
where the biological mother, after surrendering her child to the Section 3. Definitions. -For purposes of this Act, the following terms shall be defined as
DSWD for purposes of adoption and after the said child’s adoption follows:
has already been granted, changes her mind and seeks custody of
(a) Agents of the State refer to persons who, by direct provision of the law, popular election or
the child. (Caram vs. Segui) appointment by competent authority, shall take part in the performance of public functions in
x REMEDY against an order issuing WA Æ APPEAL the government, or shall perform in the government or in any of its branches public duties as an
o Within 5 working days from receipt of the order employee, agent or subordinate official, of any rank or class.
o OTHER AVAILABLE REMEDIES Æ MR or MNT
ƒ Appeal in such cases should be filed within 5 (b) Enforced or involuntary disappearance refers to the arrest, detention, abduction or any
working days from receipt of order denying other form of deprivation of liberty committed by agents of the State or by persons or groups
of persons acting with the authorization, support or acquiescence of the State, followed by a
the MR/MNT refusal to acknowledge the deprivation of liberty or by concealment of the fate or whereabouts
x QUANTUM OF EVIDENCE (for both petitioner and respondent) Æ of the disappeared person, which places such person outside the protection of the law.
SUBSTANTIAL EVIDENCE
o Definition: that amount of relevant evidence which a (c) Order of Battle refers to a document made by the military, police or any law enforcement
reasonable mind might accept as adequate to support a agency of the government, listing the names of persons and organizations that it perceives to
conclusion be enemies of the State and which it considers as legitimate targets as combatants that it
could deal with, through the use of means allowed by domestic and international law.
x DILIGENCE required on the part of the respondent in performing his
duty: (d) Victim refers to the disappeared person and any individual who has suffered harm as a
o Ordinary Æ if the respondent is a private individual direct result of an enforced or involuntary disappearance as defined in letter (b) of this
o Extraordinary Æ if the respondent is a public officer/ Section.
employee
ƒ THUS, the presumption of regularity cannot Section 4. Nonderogability of the Right Against Enforced or Involuntary
Disappearance. -The right against enforced or involuntary disappearance and the
be invoked by him fundamental safeguards for its prevention shall not be suspended under any circumstance
x If the respondent failed to discharge his burden of substantial
including political instability, threat of war, state of war or other public emergencies.
evidence, will the WA automatically issue?
o No. The petitioner must be able to discharge his own Section 5. "Order of Battle" or Any Order of Similar Nature, Not Legal Ground, for
burden of proof. (Yano vs. Sanchez) Enforced or Involuntary Disappearance. - An "Order of Battle" or any order of similar
x CRIMINAL BAR RULE nature, official or otherwise, from a superior officer or a public authority causing the
commission of enforced or involuntary disappearance is unlawful and cannot be invoked as a
o Where a criminal case has already been instituted, a justifying or exempting circumstance. Any person receiving such an order shall have the right
petition for issuance of WA cannot be filed separately. to disobey it.
ƒ (Author’s note) REMEDY of the respondent in
the petition: raise litis pendentia as an Section 6. Right of Access to Communication. - It shall be the absolute right of any person
affirmative defense (considering that an MD is deprived of liberty to have immediate access to any form of communication available in order
a prohibited motion) for him or her to inform his or her family, relative, friend, lawyer or any human rights
organization on his or her whereabouts and condition.
o Corollarily, when the petition for WA has been filed, a
separate criminal case may still be filed Section 7. Duty to Report Victims of Enforced or Involuntary Disappearance. - Any
person, not being a principal, accomplice or accessory, who has an information of a case of
163
RECTO, GAYLE ANGELI M.
2011-0008 | AUSL
Personal Notes on Remedial Law 2 Review (based on the syllabus of Prof. Henedino M. Brondial)

enforced or involuntary disappearance or who shall learn of such information or that a person habeas corpus, amparo and habeas data shall be dispensed with expeditiously. As such, all
is a victim of enforced or involuntary disappearance, shall immediately report in writing the courts and other concerned agencies of government shall give priority to such proceedings.
circumstances and whereabouts of the victim to any office, detachment or division of the
Department of the Interior and Local Government (DILG), the Department of National Defense Moreover, any order issued or promulgated pursuant to such writs or their respective
(DND), the Philippine National Police (PNP), the Armed Forces of the Philippines (AFP), the proceedings shall be executed and complied with immediately.
National Bureau of Investigation (NBI), the City or Provincial Prosecutor, the Commission on
Human Rights (CHR) or any human rights organization and, if known, the victim’s family, Section 13. Visitation /Inspection of Places of Detention and, Confinement. -The
relative, or lawyer. CHR or its duly authorized representatives are hereby mandated and authorized to conduct
regular, independent, unannounced and unrestricted visits to or inspection of all places of
Section 8. Duty to Certify in Writing on the Results of Inquiry into a Reported detention and confinement.
Disappeared Person’s Whereabouts. -In case a family member, relative, lawyer,
representative of a human rights organization or a member of the media inquires with a Section 14. Liability of Commanding Officer or Superior. - The immediate commanding
member or official of any police or military detention center, the PNP or any of its agencies, officer of the unit concerned of the AFP or the immediate senior official of the PNP and other
the AFP or any of its agencies, the NBI or any other agency or instrumentality of the law enforcement agencies shall be held liable as a principal to the crime of enforced or
government, as well as any hospital or morgue, public or private, on the presence or involuntary disappearance for acts committed by him or her that shall have led, assisted,
whereabouts of a reported victim of enforced or involuntary disappearance, such member or abetted or allowed, whether directly or indirectly, the commission thereof by his or her
official shall immediately issue a certification in writing to the inquiring person or entity on the subordinates. If such commanding officer has knowledge of or, owing to the circumstances at
presence or absence and/or information on the whereabouts of such disappeared person, the time, should have known that an enforced or involuntary disappearance is being
stating, among others, in clear and unequivocal manner the date and time of inquiry, details of committed, or has been committed by subordinates or by others within the officer’s area of
the inquiry and the response to the inquiry. responsibility and, despite such knowledge, did not take preventive or coercive action either
before, during or immediately after its commission, when he or she has the authority to
Section 9. Duty of Inquest/Investigating Public Prosecutor or any Judicial or prevent or investigate allegations of enforced or involuntary disappearance but failed to
Quasi-Judicial Official or Employee. -Any inquest or investigating public prosecutor, or prevent or investigate such allegations, whether deliberately or due to negligence, shall also
any judicial or quasi-judicial official or employee who learns that the person delivered for be held liable as principal.
inquest or preliminary investigation or for any other judicial process is a victim of enforced or
involuntary disappearance shall have the duty to immediately disclose the victim’s Section 15. Penal Provisions. - (a) The penalty of reclusion perpetua and its accessory
whereabouts to his or her immediate family, relatives, lawyer/s or to a human rights penalties shall be imposed upon the following persons:
organization by the most expedient means.
(1) Those who directly committed the act of enforced or involuntary disappearance;
Section 10. Official Up-to-Date Register of All Persons Detained or Confined. - All
persons detained or confined shall be placed solely in officially recognized and controlled (2) Those who directly forced, instigated, encouraged or induced others to commit the act of
places of detention or confinement where an official up-to-date register of such persons shall enforced or involuntary disappearance;
be maintained. Relatives, lawyers, judges, official bodies and all persons who have legitimate
interest in the whereabouts and condition of the persons deprived of liberty shall have free (3) Those who cooperated in the act of enforced or involuntary disappearance by committing
access to the register. another act without which the act of enforced or involuntary disappearance would not have
been consummated;
The following details, among others, shall be recorded, in the register:
(4) Those officials who allowed the act or abetted in the consummation of enforced or
(a) The identity or name, description and address of the person deprived of liberty; involuntary disappearance when it is within their power to stop or uncover the commission
thereof; and
(b) The date, time and location where the person was deprived of liberty and the identity of
the person who made such deprivation of liberty; (5) Those who cooperated in the execution of the act of enforced or involuntary disappearance
by previous or simultaneous acts.
(c) The authority who decided the deprivation of liberty and the reasons for the deprivation of
liberty or the crime or offense committed; (b) The penalty of reclusion temporal and its accessory penalties shall be imposed upon those
who shall commit the act of enforced or involuntary disappearance in the attempted stage as
(d) The authority controlling the deprivation of liberty; provided for and defined under Article 6 of the Revised Penal Code.

(e) The place of deprivation of liberty, the date and time of admission to the place of (c) The penalty of reclusion temporal and its accessory penalties shall also be imposed upon
deprivation of liberty and the authority responsible for the place of deprivation of liberty; persons who, having knowledge of the act of enforced or involuntary disappearance and
without having participated therein, either as principals or accomplices, took part subsequent
(f) Records of physical, mental and psychological condition of the detained or confined person to its commission in any of the following manner:
before and after the deprivation of liberty and the name and address of the physician who
examined him or her physically, mentally and medically; (1) By themselves profiting from or assisting the offender to profit from the effects of the act
of enforced or involuntary disappearance;
(g) The date and time of release or transfer of the detained or confined person to another
place of detention, the destination and the authority responsible for the transfer; (2) By concealing the act of enforced or involuntary disappearance and/or destroying the
effects or instruments thereof in order to prevent its discovery; or
(h) The date and time of each removal of the detained or confined person from his or her cell,
the reason or purpose for such removal and the date and time of his or her return to his or her (3) By harboring, concealing or assisting in the escape of the principal/s in the act of enforced
cell; or involuntary disappearance, provided such accessory acts are done with the abuse of official
functions.
(i) A summary of the physical, mental and medical findings of the detained or confined person
after each interrogation; (d) The penalty of prision correctional and its accessory penalties shall be imposed against
persons who defy, ignore or unduly delay compliance with any order duly issued or
(j) The names and addresses of the persons who visit the detained or confined person and the promulgated pursuant to the writs of habeas corpus, amparo and habeas data or their
date and time of such visits and the date and time of each departure; respective proceedings.

(k) In the event of death during the deprivation of liberty, the identity, the circumstances and (e) The penalty of arresto mayor and its accessory penalties shall be imposed against any
cause of death of the victim as well as the destination of the human remains; and person who shall violate the provisions of Sections 6, 7, 8, 9 and 10 of this Act.

(l) All other important events bearing on and all relevant details regarding the treatment of the Section 16. Preventive Suspension/Summary Dismissal. -Government officials and
detained or confined person. personnel who are found to be perpetrators of or participants in any manner in the
commission of enforced or involuntary disappearance as a result of a preliminary investigation
Provided, That the details required under letters (a) to (f) shall be entered immediately in the conducted for that purpose shall be preventively suspended or summarily dismissed from the
register upon arrest and/or detention. service, depending on the strength of the evidence so presented and gathered in the said
preliminary investigation or as may be recommended by the investigating authority.
All information contained in the register shall be regularly or upon request reported to the CHR
or any other agency of government tasked to monitor and protect human rights and shall be Section 17. Civil Liability. -The act of enforced or involuntary disappearance shall render
made available to the public. its perpetrators and the State agencies which organized, acquiesced in or tolerated such
disappearance liable under civil law.
Section 11. Submission of List of Government Detention Facilities. -Within six (6)
months from the effectivity of this Act and as may be requested by the CHR thereafter, all Section 18. Independent Liability. -The criminal liability of the offender under this Act
government agencies concerned shall submit an updated inventory or list of all officially shall be independent of or without prejudice to the prosecution and conviction of the said
recognized and controlled detention or confinement facilities, and the list of detainees or offender for any violation of Republic Act No. 7438, otherwise known as "An Act Defining
persons deprived of liberty under their respective jurisdictions to the CHR. Certain Rights of Person Arrested, Detained or Under Custodial Investigation as well as the
Duties of the Arresting, Detaining, and Investigating Officers, and Providing Penalties for
Section 12. Immediate Issuance and Compliance of the Writs of Habeas Corpus, Violations Thereof’; Republic Act No. 9745, otherwise known as "An Act Penalizing Torture and
Amparo and Habeas Data. - All proceedings pertaining to the issuance of the writs of Other Cruel, Inhuman and Degrading Treatment or Punishment, and Prescribing Penalties
Therefor"; and applicable provisions of the Revised Penal Code.
164
RECTO, GAYLE ANGELI M.
2011-0008 | AUSL
Personal Notes on Remedial Law 2 Review (based on the syllabus of Prof. Henedino M. Brondial)

Section 19. Nonexclusivity or Double Jeopardy Under International Law. - Any Section 31. Separability Clause. -If for any reason, any section or provision of this Act is
investigation, trial and decision in any Philippines court, or body for any violation of this Act declared unconstitutional or invalid, such other sections or provisions not affected thereby
shall; be without prejudice to any investigation, trial, decision or any other legal or shall remain in full force and effect.
administrative process before any appropriate international court or agency under applicable
international human rights and humanitarian law. Section 32. Repealing Clause. - All laws, decrees, executive orders, rules and regulations
and other issuances or parts thereof inconsistent with the provisions of this Act are hereby
Section 20. Exemption from Prosecution. - Any offender who volunteers information that repealed, amended or modified accordingly.
leads to the discovery of the victim of enforced or involuntary disappearance or the
prosecution of the offenders without the victim being found shall be exempt from any criminal Section 33. Effectivity Clause. - This Act shall take effect fifteen (15) days after its
and/or civil liability under this Act: Provided, That said offender does not appear to be the publication in at least two (2) newspapers of general circulation or the Official Gazette, which
most guilty. shall not be later than seven (7) days after the approval thereof.

Section 21. Continuing Offense. - An act constituting enforced or involuntary CASES


disappearance shall be considered a continuing offense as long as the perpetrators continue to
conceal the fate and whereabouts of the disappeared person and such circumstances have not
DANIEL MASANGKAY TAPUZ, AURORA TAPUZ-MADRIAGA, LIBERTY M. ASUNCION,
been determined with certainty.
LADYLYN BAMOS MADRIAGA, EVERLY TAPUZ MADRIAGA, EXCEL TAPUZ, IVAN TAPUZ AND
MARIAN TIMBAS, petitioners, vs. HONORABLE JUDGE ELMO DEL ROSARIO, in his
Section 22. Statue of Limitations Exemption. - The prosecution of persons responsible
capacity as Presiding Judge of RTC Br. 5 Kalibo, SHERIFF NELSON DELA CRUZ, in his capacity
for enforced or involuntary disappearance shall not prescribe unless the victim surfaces alive.
as Sheriff of the RTC, THE PHILIPPINE NATIONAL POLICE stationed in Boracay Island,
In which case, the prescriptive period shall be twenty-five (25) years from the date of such
represented by the PNP STATION COMMANDER, THE HONORABLE COURT OF APPEALS IN
reappearance.
CEBU 18th DIVISION, SPOUSES GREGORIO SANSON & MA. LOURDES T. SANSON,
respondents.
Section 23. Special Amnesty Law Exclusion. - Persons who are changed with and/or
G.R. No. 182484 June 17, 2008
guilty of the act of enforced or involuntary disappearance shall not benefit from any special
EN BANC
amnesty law or other similar executive measures that shall exempt them from any penal
proceedings or sanctions.
FACTS:
x Spouses Sanson Æ filed before the MCTC Buruanga-Malay, Aklan a complaint for
Section 24. State Protection - The State, through its appropriate agencies, shall ensure
forcible entry and damages with a prayer for the issuance of a writ of preliminary
the safety of all persons involved in the search, investigation and prosecution of enforced or
mandatory injunction against the herein petitioners
involuntary disappearance including, but not limited to, the victims, their families,
o (1) they are the registered owners under TCT No. 35813 of a
complainants, witnesses, legal counsel and representatives of human rights organizations and
1.0093-hectare parcel of land located at Sitio Pinaungon, Balabag,
media. They shall likewise be protected from any intimidation or reprisal.
Boracay, Malay, Aklan (the "disputed land");
o (2) they were the disputed land's prior possessors when the
Section 25. Applicability of Refouler. -No person shall be expelled, returned or extradited
petitioners - armed with bolos and carrying suspected firearms and
to another State where there are substantial grounds to believe that such person shall be in
danger of being subjected to enforced or involuntary disappearance. For purposes of together with unidentified persons numbering 120 - entered the
determining whether such grounds exist, the Secretary of the Department, of Foreign Affairs disputed land by force and intimidation, without the private
(DFA) and the Secretary of the Department of Justice (DOJ) in coordination with the respondents' permission and against the objections of the private
Chairperson of the CHR, shall take into account all relevant considerations including where respondents' security men, and built thereon a nipa and bamboo
applicable and not limited to, the existence in the requesting State of a consistent pattern of structure.
x MCTC Æ ruled in favor of Spouses Sanson
gross, flagrant or mass violations of human rights.
x Petitioners Æ appealed to RTC
x Spouses Sanson Æ prayed for issuance of WPI
Section 26. Restitution and Compensation to Victims of Enforced or Involuntary
Disappearance and/or Their Immediate Relatives. -The victims of enforced or o RTC Æ granted
x Petitioners Æ filed an MR
involuntary disappearance who surface alive shall be entitled to monetary compensation,
rehabilitation and restitution of honor and reputation. Such restitution of honor and reputation o RTC Æ denied
shall include immediate expunging or rectification of any derogatory record, information or x Spouses Sanson Æ filed a motion for demolition
public declaration/statement on his or her person, personal circumstances, status, and/or o RTC Æ granted
organizational affiliation by the appropriate government or private agency or agencies ƒ issued via a Special Order10 a writ of demolition to be
concerned. implemented fifteen (15) days after the Sheriff's
written notice to the petitioners to voluntarily
The immediate relatives of a victim of enforced or involuntary disappearance, within the fourth demolish their house/s to allow the private
civil degree of consanguinity or affinity, may also claim for compensation as provided for under respondents to effectively take actual possession of
Republic Act No. 7309, entitled "An Act Creating a Board of Claims under the Department of the land.
Justice for Victims of Unjust Imprisonment or Detention and Victims of Violent Crimes and For x Sheriff of RTC Æ issued the Notice to Vacate and for Demolition
Other Purposes", and other relief programs of the government. x Petitioners Æ filed this petition before the SC praying for 3 remedies:
o a petition for certiorari under Rule 65 of the Revised Rules of Court;
The package of indemnification for both the victims and the immediate relatives within the o the issuance of a writ of habeas data under the Rule on the Writ of
fourth civil degree of consanguinity or affinity shall be without prejudice to other legal Habeas Data; and finally,
remedies that may be available to them. o the issuance of the writ of amparo under the Rule on the Writ of
Amparo.
Section 27. Rehabilitation of Victims and/or Their Immediate Relatives, and x Petitioners’ contentions:
Offenders. - In order that the victims of enforced or involuntary disappearance who surfaced o "29. On April 29, 2006 at about 9:20 a.m. armed men sporting 12
alive and/or their immediate relatives within the fourth civil degree of consanguinity or affinity, gauge shot guns intruded into the property of the defendants [the
may be effectively reintegrated into the mainstream of society and in the process of land in dispute]. They were not in uniform. They fired their
development, the State, through the CHR, in coordination with the Department of Health, the shotguns at the defendants. Later the following day at 2:00 a.m.
Department of Social Welfare and Development (DSWD) and the concerned nongovernment two houses of the defendants were burned to ashes.
organization/s, shall provide them with appropriate medical care and rehabilitation free of o 30. These armed men [without uniforms] removed the barbed wire
charge. fence put up by defendants to protect their property from
intruders. Two of the armed men trained their shotguns at the
Toward the attainment of restorative justice, a parallel rehabilitation program for persons who defendants who resisted their intrusion. One of them who was
have committed enforced or involuntary disappearance shall likewise be implemented without identified as SAMUEL LONGNO y GEGANSO, 19 years old, single,
cost to such offenders. and a resident of Binun-an, Batad, Iloilo, fired twice.
o 31. The armed men torched two houses of the defendants reducing
Section 28. Implementing Rules and Regulations. - Within thirty (30) days from the them to ashes. [...]
effectivity of this Act, the DOJ, the DSWD, the CHR, the Families of Victims of Involuntary o 32. These acts of TERRORISM and (heinous crime) of ARSON were
Disappearance (FIND) and the Families of Desaparecidos for Justice (Desaparecidos), in reported by one of the HEIRS OF ANTONIO TAPUZ [...]. The
consultation with other human rights organizations, shall jointly promulgate the rules and terrorists trained their shotguns and fired at minors namely IVAN
regulations for the effective implementation of this Act and shall ensure the full dissemination GAJISAN and MICHAEL MAGBANUA, who resisted their intrusion.
of the same to the public. Their act is a blatant violation of the law penalizing Acts of Violence
against women and children, which is aggravated by the use of
Section 29. Suppletory Applications. - The applicable provisions of the Revised Penal high-powered weapons.
Code shall have suppletory application insofar as they are consistent with the provisions of this
Act. ISSUE # 1: Whether a writ of amparo should issue.

Section 30. Appropriations. -The amount of Ten million pesos (P10,000,000.00) is hereby HELD # 1: NO.
appropriated for the initial implementation of this Act by the CHR. Subsequent fluids for the x To start off with the basics, the writ of amparo was originally conceived as
continuing implementation of this Act shall be included in the respective budgets of the CHR a response to the extraordinary rise in the number of killings and
and the DOJ in the annual General Appropriations Act. enforced disappearances, and to the perceived lack of available and

165
RECTO, GAYLE ANGELI M.
2011-0008 | AUSL
Personal Notes on Remedial Law 2 Review (based on the syllabus of Prof. Henedino M. Brondial)

effective remedies to address these extraordinary concerns. It is that, if decided as the petitioners advocate, may render the pending RTC appeal
intended to address violations of or threats to the rights to life, liberty moot.
or security, as an extraordinary and independent remedy beyond those x Under these legal and factual situations, we are far from satisfied with the prima
available under the prevailing Rules, or as a remedy supplemental to facie existence of the ultimate facts that would justify the issuance of a writ of
these Rules. What it is not, is a writ to protect concerns that are purely amparo. Rather than acts of terrorism that pose a continuing threat to
property or commercial. Neither is it a writ that we shall issue on the persons of the petitioners, the violent incidents alleged appear to
amorphous and uncertain grounds. Consequently, the Rule on the Writ of us to be purely property-related and focused on the disputed land.
Amparo - in line with the extraordinary character of the writ and the reasonable Thus, if the petitioners wish to seek redress and hold the alleged
certainty that its issuance demands - requires that every petition for the issuance perpetrators criminally accountable, the remedy may lie more in the
of the writ must be supported by justifying allegations of fact, to wit: realm of ordinary criminal prosecution rather than on the use of the
o "(a) The personal circumstances of the petitioner; extraordinary remedy of the writ of amparo.
o (b) The name and personal circumstances of the respondent x Nor do we believe it appropriate at this time to disturb the MCTC findings, as our
responsible for the threat, act or omission, or, if the name is action may carry the unintended effect, not only of reversing the MCTC ruling
unknown or uncertain, the respondent may be described by an independently of the appeal to the RTC that is now in place, but also of nullifying
assumed appellation the ongoing appeal process. Such effect, though unintended, will obviously wreak
o (c) The right to life, liberty and security of the aggrieved havoc on the orderly administration of justice, an overriding goal that the Rule on
party violated or threatened with violation by an unlawful the Writ of Amparo does not intend to weaken or negate.
act or omission of the respondent, and how such threat or x Separately from these considerations, we cannot fail but consider too at this
violation is committed with the attendant circumstances detailed point the indicators, clear and patent to us, that the petitioners' present recourse
in supporting affidavits; via the remedy of the writ of amparo is a mere subterfuge to negate the assailed
o (d) The investigation conducted, if any, specifying the names, orders that the petitioners sought and failed to nullify before the appellate court
personal circumstances, and addresses of the investigating because of the use of an improper remedial measure. We discern this from the
authority or individuals, as well as the manner and conduct of the petitioners' misrepresentations pointed out above; from their obvious act of
investigation, together with any report; forum shopping; and from the recourse itself to the extraordinary remedies of
o (e) The actions and recourses taken by the petitioner to the writs of certiorari and amparo based on grounds that are far from forthright
determine the fate or whereabouts of the aggrieved party and and sufficiently compelling. To be sure, when recourses in the ordinary course of
the identity of the person responsible for the threat, act or law fail because of deficient legal representation or the use of improper remedial
omission; and measures, neither the writ of certiorari nor that of amparo - extraordinary though
o (f) The relief prayed for. they may be - will suffice to serve as a curative substitute. The writ of amparo,
ƒ The petition may include a general prayer for other particularly, should not issue when applied for as a substitute for the appeal or
just and equitable reliefs."22 certiorari process, or when it will inordinately interfere with these processes - the
ƒ The writ shall issue if the Court is preliminarily situation obtaining in the present case.
satisfied with the prima facie existence of the ultimate x While we say all these, we note too that the Rule on the Writ of Amparo provides
facts determinable from the supporting affidavits that for rules on the institution of separate actions,24 for the effect of earlier-
detail the circumstances of how and to what extent a filed criminal actions,25 and for the consolidation of petitions for the
threat to or violation of the rights to life, liberty and issuance of a writ of amparo with a subsequently filed criminal and civil
security of the aggrieved party was or is being action.26 These rules were adopted to promote an orderly procedure
committed. for dealing with petitions for the issuance of the writ of amparo when
x The issuance of the writ of amparo in the present case is anchored on the factual the parties resort to other parallel recourses.
allegations heretofore quoted,23 that are essentially repeated in paragraph 54 of x Where, as in this case, there is an ongoing civil process dealing directly
the petition. These allegations are supported by the following documents: with the possessory dispute and the reported acts of violence and
o "(a) Joint Affidavit dated 23 May 2006 of Rowena B. Onag, Apolsida harassment, we see no point in separately and directly intervening through a
Umambong, Ariel Gac, Darwin Alvarez and Edgardo Pinaranda, writ of amparo in the absence of any clear prima facie showing that the right to
supporting the factual positions of the petitioners, id., petitioners' life, liberty or security - the personal concern that the writ is intended to protect
prior possession, private respondents' intrusion and the illegal acts - is immediately in danger or threatened, or that the danger or threat is
committed by the private respondents and their security guards on continuing. We see no legal bar, however, to an application for the issuance of the
19 April 2006; writ, in a proper case, by motion in a pending case on appeal or on certiorari,
o (b) Unsubscribed Affidavit of Nemia Carmen y Tapuz, alleging the applying by analogy the provisions on the co-existence of the writ with a
illegal acts (firing of guns, etc.) committed by a security guard separately filed criminal case.
against minors - descendants of Antonio Tapuz;
o (c) Unsubscribed Affidavit of Melanie Tapuz y Samindao, essentially ISSUE # 2: Whether a writ of habeas data should issue.
corroborating Nemia's affidavit;
o (d) Certification dated 23 April 2006 issued by Police Officer HELD # 2: NO.
Jackson Jauod regarding the incident of petitioners' intrusion into x Section 6 of the Rule on the Writ of Habeas Data requires the following material
the disputed land; allegations of ultimate facts in a petition for the issuance of a writ of habeas
o (e) Certification dated 27 April 2006 issued by Police Officer Allan data:
R. Otis, narrating the altercation between the Tapuz family and the o "(a) The personal circumstances of the petitioner and the
security guards of the private respondents, including the gun- respondent;
poking and shooting incident involving one of the security guards; o (b) The manner the right to privacy is violated or threatened and
o (f) Certification issued by Police Officer Christopher R. Mendoza, how it affects the right to life, liberty or security of the aggrieved
narrating that a house owned by Josiel Tapuz, Jr., rented by a party;
certain Jorge Buenavente, was accidentally burned by a fire." o (c) The actions and recourses taken by the petitioner to secure the
x On the whole, what is clear from these statements - both sworn and unsworn - is data or information;
the overriding involvement of property issues as the petition traces its roots to o (d) The location of the files, registers or databases, the government
questions of physical possession of the property disputed by the private parties. office, and the person in charge, in possession or in control of the
If at all, issues relating to the right to life or to liberty can hardly be discerned data or information, if known;
except to the extent that the occurrence of past violence has been alleged. The o (e) The reliefs prayed for, which may include the updating,
right to security, on the other hand, is alleged only to the extent of the threats rectification, suppression or destruction of the database or
and harassments implied from the presence of "armed men bare to the waist" information or files kept by the respondent.
and the alleged pointing and firing of weapons. Notably, none of the ƒ In case of threats, the relief may include a prayer for
supporting affidavits compellingly show that the threat to the rights to an order enjoining the act complained of; and
life, liberty and security of the petitioners is imminent or is continuing. o (f) Such other relevant reliefs as are just and equitable."
x A closer look at the statements shows that at least two of them - the statements x Support for the habeas data aspect of the present petition only alleges that:
of Nemia Carreon y Tapuz and Melanie Tapuz are practically identical and o "1. [ … ] Similarly, a petition for a WRIT OF HABEAS DATA is
unsworn. The Certification by Police Officer Jackson Jauod, on the other hand, prayed for so that the PNP may release the report on the burning of
simply narrates what had been reported by one Danny Tapuz y Masangkay, and the homes of the petitioners and the acts of violence employed
even mentions that the burning of two residential houses was against them by the private respondents, furnishing the Court and
"accidental." the petitioners with copy of the same;
x As against these allegations are the cited MCTC factual findings in its decision in o 66. Petitioners apply for a WRIT OF HABEAS DATA commanding
the forcible entry case which rejected all the petitioners' factual claims. These the Philippine National Police [PNP] to produce the police report
findings are significantly complete and detailed, as they were made under a full- pertaining to the burning of the houses of the petitioners in the
blown judicial process, i.e., after examination and evaluation of the contending land in dispute and likewise the investigation report if an
parties' positions, evidence and arguments and based on the report of a court- investigation was conducted by the PNP."
appointed commissioner. x These allegations obviously lack what the Rule on Writ of Habeas Data
x We preliminarily examine these conflicting factual positions under the backdrop requires as a minimum, thus rendering the petition fatally deficient.
of a dispute (with incidents giving rise to allegations of violence or threat Specifically, we see no concrete allegations of unjustified or unlawful
thereof) that was brought to and ruled upon by the MCTC; subsequently brought violation of the right to privacy related to the right to life, liberty or
to the RTC on an appeal that is still pending; still much later brought to the security. The petition likewise has not alleged, much less demonstrated, any
appellate court without conclusive results; and then brought to us on need for information under the control of police authorities other than those it
interlocutory incidents involving a plea for the issuance of the writ of amparo has already set forth as integral annexes. The necessity or justification for the
166
RECTO, GAYLE ANGELI M.
2011-0008 | AUSL
Personal Notes on Remedial Law 2 Review (based on the syllabus of Prof. Henedino M. Brondial)

issuance of the writ, based on the insufficiency of previous efforts made to x Considering that there is no legal basis for its issuance, as in this case, the writ
secure information, has not also been shown. In sum, the prayer for the will not be issued and the petition will be dismissed outright.
issuance of a writ of habeas data is nothing more than the "fishing x This new remedy of writ of amparo which is made available by this Court is
expedition" that this Court - in the course of drafting the Rule on intended for the protection of the highest possible rights of any person, which is
habeas data - had in mind in defining what the purpose of a writ of his or her right to life, liberty and security. The Court will not spare any time or
habeas data is not. In these lights, the outright denial of the petition effort on its part in order to give priority to petitions of this nature. However, the
for the issuance of the writ of habeas data is fully in order. Court will also not waste its precious time and effort on matters not covered by
the writ.

ARMANDO Q. CANLAS, MIGUEL D. CAPISTRANO, MARRIETA PIA, petitioners, vs. NAPICO P/SUPT. FELIXBERTO CASTILLO, POLICE OFFICERS ROMEO BAGTAS, RUPERTO
HOMEOWNERS ASS’N., I - XIII, INC., ET AL., respondents. BORLONGAN, EDMUNDO DIONISIO, RONNIE MORALES, ARNOLD TRIA, and GILBERTO
G.R. No. 182795 June 5, 2008 PUNZALAN, ENGR. RICASOL P. MILLAN, ENGR. REDENTOR S. DELA CRUZ, MR. ANASTACIO L.
EN BANC BORLONGAN, MR. ARTEMIO ESGUERRA, "TISOY," and JOHN DOES, Petitioners, vs. DR.
AMANDA T. CRUZ, NIXON T. CRUZ, and FERDINAND T. CRUZ, Respondents.
FACTS: G.R. No. 182165 November 25, 2009
x Canlas et al are settlers in a certain parcel of land situated in Barangay EN BANC
Manggahan, Pasig City.
x They were the losing parties in the case of G.R. Nos. 177448, 180768, 177701, FACTS:
177038 x Spouses Amanda Cruz and Ferdinand Cruz leased a parcel of land in Barrio
o Their dwellings/houses have either been demolished as of the time Guinhawa, Malolos, Bulacan
of filing of the petition, or is about to be demolished pursuant to a x Provincial Government of Bulacan Æ demanded the Cruzes to leave the premises
court judgment. because the same shall be used for local projects
x Canlas et al Æ filed a petition for issuance of WA before the SC x Spouses refused
o Alleged that the subject land titles invoked against them are x Province Æ filed an unlawful detainer case before MTC Bulacan
spurious x MTC Æ in favor of Province
ƒ It is therefore the ardent initiatives of the herein o Judgment became final and executory
Petitioners, by way of the said prayer for the issuance x HOWEVER, Spouses refused to vacate
of the Writ of Amparo, that these unprincipled Land x Spouses Æ filed cases against the Province and the judges who presided over
Officials be summoned to answer their participation in the case
the issuances of these fraudulent and spurious titles, x One of the cases filed by the Spouses was an injunction case with a prayer for
NOW, in the hands of the Private Respondents. The WPI
Courts of Justice, including this Honorable Supreme x RTC Br 10 Æ issued WPI
Court, are likewise being made to believe that said o Ordered the REMAND of the determination of the issues raised by
titles in the possession of the Private Respondents the petitioners on the issued writ of demolition to the MTC of
were issued untainted with frauds Bulacan, Bulacan.
o BUT, in the same petition, they prayed that the judgment in G.R. o Finding that the fallo of the RTC July 19, 2005 Order treats, as a
Nos. 177448, 180768, 177701, 177038 be reversed suspensive condition for the lifting of the permanent injunction, the
ƒ That, Petitioners herein knew before hand that: there determination of the boundaries of the property, the Province
can be no motion for reconsideration for the second returned the issue for the consideration of the MTC. In a Geodetic
or third time to be filed before this Honorable Engineer’s Report submitted to the MTC on August 31, 2007, the
Supreme Court. As such therefore, Petitioners herein metes and bounds of the property were indicated.
are aware of the opinion that this present petition x MTC Æ approved the report re metes and bounds; declared that the WPI is
should not in any way be treated as such motions ineffective
fore reconsideration. Solely, this petition is only for x Province Æ filed another motion for demolition
the possible issuance of the writ of amparo, although o MTC Æ granted
it might affect the previous rulings of this Honorable x Spouses Æ filed a motion before Branch 10 of the RTC for the issuance of a
Supreme Court in these cases, G.R. Nos. 177448, temporary restraining order (TRO) which it set for hearing on January 25, 2008
180768, 177701 and 177038. Inherent in the powers o RTC Æ issued TRO (despite the earlier implementation of the writ
of the Supreme Court of the Philippines is to modify, of demolition)
reverse and set aside, even its own previous decision, x Spouses then took possession of the premises
that can not be thwarted nor influenced by any one, x Police Officer Castillo (deployed by Province) Æ entered the property as
but, only on the basis of merits and evidence. This is instructed by virtue of a memorandum issued by Governor Joselito R. Mendoza
the purpose of this petition for the Writ of Amparo. x Spouses refused to turn over the property
x Spouses Æ filed Respectful Motion-Petition for Writ of Amparo and Habeas Data
ISSUE: Whether the remedy of writ of amparo was properly resorted to. with RTC Br 10
o Respondents averred that despite the Permanent Injunction,
HELD: NO. petitioners unlawfully entered the property with the use of heavy
x The Rule on the Writ of Amparo provides: equipment, tore down the barbed wire fences and tents,6 and
o Section 1. Petition. - The petition for a writ of amparo is a remedy arrested them when they resisted petitioners’ entry; and that as
available to any person whose right to life, liberty and security is early as in the evening of February 20, 2008, members of the
violated or threatened with violation by an unlawful act or omission Philippine National Police had already camped in front of the
of a public official or employee, or of a private individual or entity. property.
ƒ The writ shall cover extralegal killings and enforced x RTC Æ issued the subject writs
disappearances or threats thereof. (Emphasis o Petitioners have shown by preponderant evidence that the facts
supplied.) and circumstances of the alleged offenses examined into on Writs
x The threatened demolition of a dwelling by virtue of a final judgment of Amparo and Habeas Data that there have been an on-going
of the court, which in this case was affirmed with finality by this Court hearings on the verified Petition for Contempt, docketed as Special
in G.R. Nos. 177448, 180768, 177701, 177038, is not included among Proceedings No. 306-M-2006, before this Court for alleged violation
the enumeration of rights as stated in the above-quoted Section 1 for by the respondents of the Preliminary Injunction Order dated July
which the remedy of a writ of amparo is made available. Their claim to 16, 2005 [sic] in Sp. Civil Action No. 833-M-2002, hearings were
their dwelling, assuming they still have any despite the final and held on January 25, 2008, February 12 and 19, 2008, where the
executory judgment adverse to them, does not constitute right to life, respondents prayed for an April 22, 2008 continuance, however, in
liberty and security. There is, therefore, no legal basis for the issuance of the the pitch darkness of February 20, 2008, police officers, some
writ of amparo. personnel from the Engineering department, and some civilians
x Besides, the factual and legal basis for petitioners’ claim to the land in question is proceeded purposely to the Pinoy Compound, converged therein
not alleged in the petition at all. The Court can only surmise that these rights and and with continuing threats of bodily harm and danger and stone-
interest had already been threshed out and settled in the four cases cited above. throwing of the roofs of the homes thereat from voices around its
No writ of amparo may be issued unless there is a clear allegation of the premises, on a pretext of an ordinary police operation when
supposed factual and legal basis of the right sought to be protected. enterviewed [sic] by the media then present, but at 8:00 a.m. to
x Under Section 6 of the same rules, the court shall issue the writ upon the filing late in the afternoon of February 21, 2008, zoomed in on the
of the petition, only if on its face, the court ought to issue said writ. petitioners, subjecting them to bodily harm, mental torture,
o Section 6. Issuance of the Writ. - Upon the filing of the petition, degradation, and the debasement of a human being, reminiscent of
the court, justice or judge shall immediately order the issuance of the martial law police brutality, sending chill in any ordinary citizen
the writ if on its face it ought to issue. The clerk of court shall issue o "WHEREFORE, premises considered, the Commitment Orders and
the writ under the seal of the court; or in case of urgent necessity, waivers in Crim. Cases Nos. 08-77 for Direct assault; Crim. Case
the justice or the judge may issue the writ under his or her own No. 08-77 for Other Forms of Trespass; and Crim. Case No. 08-78
hand, and may deputize any officer or person to serve it. for Light Threats are hereby DECLARED illegal, null and void, as
o The writ shall also set the date and time for summary hearing of petitioners were deprived of their substantial rights, induced by
the petition which shall not be later than seven (7) days from the duress or a well-founded fear of personal violence. Accordingly, the
date of its issuance.
167
RECTO, GAYLE ANGELI M.
2011-0008 | AUSL
Personal Notes on Remedial Law 2 Review (based on the syllabus of Prof. Henedino M. Brondial)

commitment orders and waivers are hereby SET ASIDE. The property disputed by the private parties. If at all,
temporary release of the petitioners is declared ABSOLUTE. issues relating to the right to life or to liberty can
x Province Æ filed Rue 45 before SC hardly be discerned except to the extent that the
occurrence of past violence has been alleged. The
ISSUE: Whether the RTC correctly issued the writs of amparo and habeas data, considering right to security, on the other hand, is alleged only to
that (1) the petition involves property rights and (2) there is already a pending criminal case the extent of the treats and harassments implied from
before MTC Br 1. the presence of "armed men bare to the waist" and
the alleged pointing and firing of weapons. Notably,
HELD: NO. none of the supporting affidavits compellingly show
x The Court is, under the Constitution, empowered to promulgate rules for the that the threat to the rights to life, liberty and security
protection and enforcement of constitutional rights.13 In view of the of the petitioners is imminent or continuing.18
heightening prevalence of extrajudicial killings and enforced (Emphasis in the original; underscoring supplied)
disappearances, the Rule on the Writ of Amparo was issued and took x It bears emphasis that respondents’ petition did not show any actual violation,
effect on October 24, 2007 which coincided with the celebration of imminent or continuing threat to their life, liberty and security. Bare allegations
United Nations Day and affirmed the Court’s commitment towards that petitioners "in unison, conspiracy and in contempt of court, there and then
internationalization of human rights. More than three months later or on willfully, forcibly and feloniously with the use of force and intimidation entered
February 2, 2008, the Rule on the Writ of Habeas Data was promulgated. and forcibly, physically manhandled the petitioners (respondents) and arrested
x Section 1 of the Rule on the Writ of Amparo provides: the herein petitioners (respondents)"19 will not suffice to prove entitlement to
o Section 1. Petition. - The petition for a writ of amparo is a remedy the remedy of the writ of amparo. No undue confinement or detention was
available to any person whose right to life, liberty and security is present. In fact, respondents were even able to post bail for the offenses a day
violated or threatened with violation by an unlawful act or omission after their arrest.20
of a public official or employee, or of a private individual or entity. x Although respondents’ release from confinement does not necessarily
The writ shall cover extralegal killings and enforced disappearances hinder supplication for the writ of amparo, absent any evidence or even
or threats thereof. (Emphasis and underscoring supplied) an allegation in the petition that there is undue and continuing
x Section 1 of the Rule on the Writ of Habeas Data provides: restraint on their liberty, and/or that there exists threat or intimidation
o Section 1. Habeas Data. - The writ of habeas data is a remedy that destroys the efficacy of their right to be secure in their persons,
available to any person whose right to privacy in life, liberty or the issuance of the writ cannot be justified.
security is violated or threatened by an unlawful act or omission of x That respondents are merely seeking the protection of their property rights is
a public official or employee or of a private individual or entity gathered from their Joint Affidavit, viz:
engaged in the gathering, collecting or storing of data or o 11. Kami ay humarang at humiga sa harap ng mga heavy
information regarding the person, family, home and equipment na hawak hawak ang nasabing kautusan ng RTC Branch
correspondence of the aggrieved party. (Emphasis and 10 (PERMANENT INJUNCTION at RTC ORDERS DATED February
underscoring supplied) 12, 17 at 19 2008) upang ipaglaban ang dignidad ng kautusan ng
x From the above-quoted provisions, the coverage of the writs is limited korte, ipaglaban ang prinsipyo ng "SELF-HELP" at batas ukol sa
to the protection of rights to life, liberty and security. And the writs cover "PROPERTY RIGHTS", Wala kaming nagawa ipagtanggol ang aming
not only actual but also threats of unlawful acts or omissions. karapatan sa lupa na 45 years naming "IN POSSESSION."
x Secretary of National Defense v. Manalo14 teaches: (Underscoring supplied)
o As the Amparo Rule was intended to address the intractable x Oddly, respondents also seek the issuance of a writ of habeas data when it is not
problem of "extralegal killings" and "enforced disappearances," its even alleged that petitioners are gathering, collecting or storing data or
coverage, in its present form, is confined to these two instances or information regarding their person, family, home and correspondence.
to threats thereof. "Extralegal killings" are "killings committed x As for respondents’ assertion of past incidents21 wherein the Province allegedly
without due process of law, i.e., without legal safeguards or judicial violated the Permanent Injunction order, these incidents were already raised in
proceedings." On the other hand, "enforced disappearances" are the injunction proceedings on account of which respondents filed a case for
"attended by the following characteristics: an arrest, detention or criminal contempt against petitioners.22
abduction of a person by a government official or organized groups x Before the filing of the petition for writs of amparo and habeas data, or on
or private individuals acting with the direct or indirect acquiescence February 22, 2008, petitioners even instituted a petition for habeas corpus which
of the government; the refusal of the State to disclose the fate or was considered moot and academic by Branch 14 of the Malolos RTC and was
whereabouts of the person concerned or a refusal to acknowledge accordingly denied by Order of April 8, 2008.
the deprivation of liberty which places such persons outside the x More. Respondent Amanda and one of her sons, Francisco Jr., likewise filed a
protection of law.15 (Underscoring supplied, citations omitted) petition for writs of amparo and habeas data before the Sandiganbayan, they
x To thus be covered by the privilege of the writs, respondents must alleging the commission of continuing threats by petitioners after the issuance of
meet the threshold requirement that their right to life, liberty and the writs by the RTC, which petition was dismissed for insufficiency and forum
security is violated or threatened with an unlawful act or omission. shopping.
Evidently, the present controversy arose out of a property dispute x It thus appears that respondents are not without recourse and have in
between the Provincial Government and respondents. Absent any fact taken full advantage of the legal system with the filing of civil,
considerable nexus between the acts complained of and its effect on criminal and administrative charges.231avvphi1
respondents’ right to life, liberty and security, the Court will not delve x It need not be underlined that respondents’ petitions for writs of
on the propriety of petitioners’ entry into the property. amparo and habeas data are extraordinary remedies which cannot be
x Apropos is the Court’s ruling in Tapuz v. Del Rosario:16 used as tools to stall the execution of a final and executory decision in
o To start off with the basics, the writ of amparo was originally a property dispute.
conceived as a response to the extraordinary rise in the number of x At all events, respondents’ filing of the petitions for writs of amparo and habeas
killings and enforced disappearances, and to the perceived lack of data should have been barred, for criminal proceedings against them had
available and effective remedies to address these extraordinary commenced after they were arrested in flagrante delicto and proceeded against
concerns. It is intended to address violations of or threats to the in accordance with Section 6, Rule 11224 of the Rules of Court. Validity of the
rights to life, liberty or security, as an extraordinary and arrest or the proceedings conducted thereafter is a defense that may be set up
independent remedy beyond those available under the prevailing by respondents during trial and not before a petition for writs of amparo and
Rules, or as a remedy supplemental to these Rules. What it is not, habeas data. The reliefs afforded by the writs may, however, be made available
is a writ to protect concerns that are purely property or to the aggrieved party by motion in the criminal proceedings
commercial. Neither is it a writ that we shall issue on amorphous
and uncertain grounds. Consequently, the Rule on the Writ of
Amparo - in line with the extraordinary character of the writ and
the reasonable certainty that its issuance demands - requires that GEN. AVELINO I. RAZON, JR., Chief, Philippine National Police (PNP); Police Chief
every petition for the issuance of the writ must be supported by Superintendent RAUL CASTAÑEDA, Chief, Criminal Investigation and Detection Group (CIDG);
justifying allegations of fact, to wit: Police Senior Superintendent LEONARDO A. ESPINA, Chief, Police Anti-Crime and Emergency
ƒ The writ shall issue if the Court is preliminarily Response (PACER); and GEN. JOEL R. GOLTIAO, Regional Director of ARMM, PNP, Petitioners,
satisfied with the prima facie existence of the ultimate vs. MARY JEAN B. TAGITIS, herein represented by ATTY. FELIPE P. ARCILLA, JR., Attorney-
facts determinable from the supporting affidavits that in-Fact, Respondent.
detail the circumstances of how and to what extent a G.R. No. 182498 December 3, 2009
threat to or violation of the rights to life, liberty and EN BANC
security of the aggrieved party was or is being
committed.17 (Emphasis and italics in the original, FACTS:
citation omitted) x Engineer Morced N. Tagitis (a consultant for the World Bank and the Senior
o Tapuz also arose out of a property dispute, albeit between private Honorary Counselor for the Islamic Development Bank (IDB) Scholarship
individuals, with the petitioners therein branding as "acts of Programme) Æ went to Jolo, Sulu with one Arsimin Kunnong (Kunnong), an IDB
terrorism" the therein respondents’ alleged entry into the disputed scholar, from a seminar in Zamboanga City.
land with armed men in tow. The Court therein held: x They checked-in at ASY Pension House
ƒ On the whole, what is clear from these statements - x Kunnong went on an errand but upon his return, Tagitis was no longer there
both sworn and unsworn - is the overriding x The receptionist said that Tagitis went out to buy food at around 12:30 in the
involvement of property issues as the petition traces afternoon and even left his room key with the desk; advised Kunnong to wait
its roots to questions of physical possession of the
168
RECTO, GAYLE ANGELI M.
2011-0008 | AUSL
Personal Notes on Remedial Law 2 Review (based on the syllabus of Prof. Henedino M. Brondial)

x Kunnong and Muhammad Abdulnazeir N. Matli, (UP professor of Muslim studies Chief Sr. Supt. Leonardo A. Espina to exert extraordinary diligence
and Tagitis’ fellow student counselor at the IDB) Æ reported Tagitis’ and efforts to protect the life, liberty and security of Tagitis, with
disappearance to the Jolo Police Station the obligation to provide monthly reports of their actions to the CA.
x Wife Mary Jane Tagitis (through Atty. Arcilla) Æ filed a petition for writ of At the same time, the CA dismissed the petition against the then
amparo with CA against Lt. Gen. Alexander Yano, Commanding General, respondents from the military, Lt. Gen Alexander Yano and Gen.
Philippine Army; Gen. Avelino I. Razon, Chief, Philippine National Police (PNP); Ruben Rafael, based on the finding that it was PNP-CIDG, not the
Gen. Edgardo M. Doromal, Chief, Criminal Investigation and Detention Group military, that was involved.
(CIDG); Sr. Supt. Leonardo A. Espina, Chief, Police Anti-Crime and Emergency x Razon et al Æ filed an MR
Response; Gen. Joel Goltiao, Regional Director, ARMM-PNP; and Gen. Ruben o CA Æ denied
Rafael, Chief, Anti-Terror Task Force Comet [collectively referred to as x Razon et al Æ filed a Rule 45 before the SC
petitioners]
o [The respondent] has exhausted all administrative avenues and ISSUE # 1: Whether the petition filed before the CA was sufficient in form and substance.
remedies but to no avail, and under the circumstances, [the
respondent] has no other plain, speedy and adequate remedy to HELD # 1: YES.
protect and get the release of subject Engr. Morced Tagitis from x In questioning the sufficiency in form and substance of the respondent’s Amparo
the illegal clutches of the [petitioners], their intelligence operatives petition, the petitioners contend that the petition violated Section 5(c), (d), and
and the like which are in total violation of the subject’s human and (e) of the Amparo Rule. Specifically, the petitioners allege that the respondent
constitutional rights, except the issuance of a WRIT OF AMPARO. failed to:
x CA Æ issued the writ o 1) allege any act or omission the petitioners committed in violation
o directed the petitioners to file their verified return within seventy- of Tagitis’ rights to life, liberty and security;
two (72) hours from service of the writ o 2) allege in a complete manner how Tagitis was abducted, the
x Razon et al Æ filed return persons responsible for his disappearance, and the respondent’s
o denied any involvement in or knowledge of Tagitis’ alleged source of information;
abduction. o 3) allege that the abduction was committed at the petitioners’
o They argued that the allegations of the petition were incomplete instructions or with their consent;
and did not constitute a cause of action against them; were o 4) implead the members of CIDG regional office in Zamboanga
baseless, or at best speculative; and were merely based on hearsay alleged to have custody over her husband;
evidence o 5) attach the affidavits of witnesses to support her accusations;
x The disappearance of Tagitis was admitted o 6) allege any action or inaction attributable to the petitioners in the
x CA Æ directed Gen. Goltiao - as the officer in command of the area of performance of their duties in the investigation of Tagitis’
disappearance - to form TASK FORCE TAGITIS disappearance; and
x CA Æ subsequently set three hearings to monitor whether TASK FORCE TAGITIS o 7) specify what legally available efforts she took to determine the
was exerting "extraordinary efforts" in handling the disappearance of Tagitis.20 fate or whereabouts of her husband.
o (1) the first hearing would be to mobilize the CIDG, Zamboanga x A petition for the Writ of Amparo shall be signed and verified and shall allege,
City; among others (in terms of the portions the petitioners cite):75
o (2) the second hearing would be to mobilize intelligence with Abu o (c) The right to life, liberty and security of the aggrieved party
Sayyaf and ARMM; and violated or threatened with violation by an unlawful act or omission
o (3) the third hearing would be to mobilize the Chief of Police of of the respondent, and how such threat or violation is committed
Jolo, Sulu and the Chief of Police of Zamboanga City and other with the attendant circumstances detailed in supporting affidavits;
police operatives o (d) The investigation conducted, if any, specifying the names,
x Task Force Æ submitted report personal circumstances, and addresses of the investigating
o [Based] on reliable information from the Office of Muslim Affairs in authority or individuals, as well as the manner and conduct of the
Manila, Tagitis has reportedly taken and carried away… more or investigation, together with any report;
less Five Million Pesos (P5,000,000.00) deposited and entrusted to o (e) The actions and recourses taken by the petitioner to determine
his … [personal] bank accounts by the Central Office of IDB, the fate or whereabouts of the aggrieved party and the identity of
Jeddah, Kingdom of Saudi Arabia, which [was] intended for the … the person responsible for the threat, act or omission; and
IDB Scholarship Fund. x The framers of the Amparo Rule never intended Section 5(c) to be
x CA Æ in favor of Tagitis complete in every detail in stating the threatened or actual violation of
o Confirming that the disappearance of Tagitis was an "enforced a victim’s rights. As in any other initiatory pleading, the pleader must of
disappearance" under the United Nations (UN) Declaration on the course state the ultimate facts constituting the cause of action,
Protection of All Persons from Enforced Disappearances omitting the evidentiary details.76 In an Amparo petition, however, this
o when military intelligence pinpointed the investigative arm of the requirement must be read in light of the nature and purpose of the
PNP (CIDG) to be involved in the abduction, the missing-person proceeding, which addresses a situation of uncertainty; the petitioner
case qualified as an enforced disappearance. may not be able to describe with certainty how the victim exactly disappeared, or
o The conclusion that the CIDG was involved was based on the who actually acted to kidnap, abduct or arrest him or her, or where the victim is
respondent’s testimony, corroborated by her companion, Mrs. detained, because these information may purposely be hidden or covered up by
Talbin. those who caused the disappearance. In this type of situation, to require
o The CA noted that the information that the CIDG, as the police the level of specificity, detail and precision that the petitioners
intelligence arm, was involved in Tagitis’ abduction came from no apparently want to read into the Amparo Rule is to make this Rule a
less than the military - an independent agency of government. token gesture of judicial concern for violations of the constitutional
o The CA thus greatly relied on the "raw report" from Col. Kasim’s rights to life, liberty and security.
asset, pointing to the CIDG’s involvement in Tagitis’ abduction. x To read the Rules of Court requirement on pleadings while addressing the unique
o The CA held that "raw reports" from an "asset" carried "great Amparo situation, the test in reading the petition should be to determine
weight" in the intelligence world. It also labeled as "suspect" Col. whether it contains the details available to the petitioner under the
Kasim’s subsequent and belated retraction of his statement that the circumstances, while presenting a cause of action showing a violation of the
military, the police, or the CIDG was involved in the abduction of victim’s rights to life, liberty and security through State or private party action.
Tagitis. The petition should likewise be read in its totality, rather than in terms of its
o characterized as "too farfetched and unbelievable" and "a bedlam isolated component parts, to determine if the required elements - namely, of the
of speculation" police theories painting the disappearance as disappearance, the State or private action, and the actual or threatened
"intentional" on the part of Tagitis. He had no previous brushes violations of the rights to life, liberty or security - are present.
with the law or any record of overstepping the bounds of any trust x In the present case, the petition amply recites in its paragraphs 4 to 11 the
regarding money entrusted to him; no student of the IDB circumstances under which Tagitis suddenly dropped out of sight after engaging
scholarship program ever came forward to complain that he or she in normal activities, and thereafter was nowhere to be found despite efforts to
did not get his or her stipend. locate him. The petition alleged, too, under its paragraph 7, in relation to
o The CA also found no basis for the police theory that Tagitis was paragraphs 15 and 16, that according to reliable information, police operatives
"trying to escape from the clutches of his second wife," on the were the perpetrators of the abduction. It also clearly alleged how Tagitis’ rights
basis of the respondent’s testimony that Tagitis was a Muslim who to life, liberty and security were violated when he was "forcibly taken and
could have many wives under the Muslim faith, and that there was boarded on a motor vehicle by a couple of burly men believed to be police
"no issue" at all when the latter divorced his first wife in order to intelligence operatives," and then taken "into custody by the respondents’ police
marry the second. intelligence operatives since October 30, 2007, specifically by the CIDG, PNP
o Finally, the CA also ruled out kidnapping for ransom by the Abu Zamboanga City, x x x held against his will in an earnest attempt of the police to
Sayyaf or by the ARMM paramilitary as the cause for Tagitis’ involve and connect [him] with different terrorist groups."77
disappearance, since the respondent, the police and the military x These allegations, in our view, properly pleaded ultimate facts within the
noted that there was no acknowledgement of Tagitis’ abduction or pleader’s knowledge about Tagitis’ disappearance, the participation by agents
demand for payment of ransom - the usual modus operandi of of the State in this disappearance, the failure of the State to release Tagitis or to
these terrorist groups. provide sufficient information about his whereabouts, as well as the actual
o Based on these considerations, the CA thus extended the privilege violation of his right to liberty. Thus, the petition cannot be faulted for any failure
of the writ to Tagitis and his family, and directed the CIDG Chief, in its statement of a cause of action.
Col. Jose Volpane Pante, PNP Chief Avelino I. Razon, Task Force x If a defect can at all be attributed to the petition, this defect is its lack
Tagitis heads Gen. Joel Goltiao and Col. Ahiron Ajirim, and PACER of supporting affidavit, as required by Section 5(c) of the Amparo Rule.
169
RECTO, GAYLE ANGELI M.
2011-0008 | AUSL
Personal Notes on Remedial Law 2 Review (based on the syllabus of Prof. Henedino M. Brondial)

Owing to the summary nature of the proceedings for the writ and to facilitate the o 15. According to reliable information received by the [respondent],
resolution of the petition, the Amparo Rule incorporated the requirement for subject Engr. Tagitis is in the custody of police intelligence
supporting affidavits, with the annotation that these can be used as the affiant’s operatives, specifically with the CIDG, PNP Zamboanga City, being
direct testimony.78 This requirement, however, should not be read as an held against his will in an earnest attempt of the police to involve
absolute one that necessarily leads to the dismissal of the petition if and connect Engr. Tagitis with the different terrorist groups;
not strictly followed. Where, as in this case, the petitioner has o 17. [The respondent] filed her complaint with the PNP Police
substantially complied with the requirement by submitting a verified Station at the ARMM in Cotobato and in Jolo, as suggested by her
petition sufficiently detailing the facts relied upon, the strict need for friends, seeking their help to find her husband, but [the
the sworn statement that an affidavit represents is essentially fulfilled. respondent’s] request and pleadings failed to produce any positive
We note that the failure to attach the required affidavits was fully cured when results
the respondent and her witness (Mrs. Talbin) personally testified in the CA o 20. Lately, [respondent] was again advised by one of the
hearings held on January 7 and 17 and February 18, 2008 to swear to and flesh [petitioners] to go to the ARMM Police Headquarters again in
out the allegations of the petition. Thus, even on this point, the petition cannot Cotobato City and also to the different Police Headquarters
be faulted. including the police headquarters in Davao City, in Zamboanga City,
x Section 5(d) of the Amparo Rule requires that prior investigation of an in Jolo, and in Camp Crame, Quezon City, and all these places have
alleged disappearance must have been made, specifying the manner been visited by the [respondent] in search for her husband, which
and results of the investigation. Effectively, this requirement seeks to entailed expenses for her trips to these places thereby resorting her
establish at the earliest opportunity the level of diligence the public to borrowings and beggings [sic] for financial help from friends and
authorities undertook in relation with the reported disappearance.79 relatives only to try complying to the different suggestions of these
o We reject the petitioners’ argument that the respondent’s petition police officers, despite of which, her efforts produced no positive
did not comply with the Section 5(d) requirements of the Amparo results up to the present time;
Rule, as the petition specifies in its paragraph 11 that Kunnong and o 25. [The respondent] has exhausted all administrative avenues and
his companions immediately reported Tagitis’ disappearance to the remedies but to no avail, and under the circumstances,
police authorities in Jolo, Sulu as soon as they were relatively [respondent] has no other plain, speedy and adequate remedy to
certain that he indeed had disappeared. The police, however, gave protect and get the release of subject Engr. Morced Tagitis from
them the "ready answer" that Tagitis could have been abducted by the illegal clutches of [the petitioners], their intelligence operatives
the Abu Sayyaf group or other anti-government groups. The and the like which are in total violation of the subject’s human and
respondent also alleged in paragraphs 17 and 18 of her petition constitutional rights, except the issuance of a WRIT OF AMPARO.
that she filed a "complaint" with the PNP Police Station in Cotobato x Based on these considerations, we rule that the respondent’s petition for the Writ
and in Jolo, but she was told of "an intriguing tale" by the police of Amparo is sufficient in form and substance and that the Court of Appeals had
that her husband was having "a good time with another woman." every reason to proceed with its consideration of the case.
The disappearance was alleged to have been reported, too, to no
less than the Governor of the ARMM, followed by the respondent’s ISSUE # 2: Whether Engr. Tagistis may have been subjected to extralegal killing and/or
personal inquiries that yielded the factual bases for her petition.80 enforced disappearance so as to render his case within the ambit of a writ of amparo.
x These allegations, to our mind, sufficiently specify that reports have been made
to the police authorities, and that investigations should have followed. That the HELD # 2: YES.
petition did not state the manner and results of the investigation that the x The present case is one of first impression in the use and application of the Rule
Amparo Rule requires, but rather generally stated the inaction of the police, their on the Writ of Amparo in an enforced disappearance situation. For a deeper
failure to perform their duty to investigate, or at the very least, their reported appreciation of the application of this Rule to an enforced disappearance
failed efforts, should not be a reflection on the completeness of the petition. To situation, a brief look at the historical context of the writ and enforced
require the respondent to elaborately specify the names, personal disappearances would be very helpful.
circumstances, and addresses of the investigating authority, as well x The phenomenon of enforced disappearance arising from State action first
the manner and conduct of the investigation is an overly strict attracted notice in Adolf Hitler’s Nact und Nebel Erlass or Night and Fog Decree
interpretation of Section 5(d), given the respondent’s frustrations in of December 7, 1941.82 The Third Reich’s Night and Fog Program, a State
securing an investigation with meaningful results. Under these policy, was directed at persons in occupied territories "endangering German
circumstances, we are more than satisfied that the allegations of the petition on security"; they were transported secretly to Germany where they disappeared
the investigations undertaken are sufficiently complete for purposes of bringing without a trace. In order to maximize the desired intimidating effect, the policy
the petition forward. prohibited government officials from providing information about the fate of
x Section 5(e) is in the Amparo Rule to prevent the use of a petition - these targeted persons.83
that otherwise is not supported by sufficient allegations to constitute a x In the mid-1970s, the phenomenon of enforced disappearances resurfaced,
proper cause of action - as a means to "fish" for evidence.81 The shocking and outraging the world when individuals, numbering anywhere from
petitioners contend that the respondent’s petition did not specify what "legally 6,000 to 24,000, were reported to have "disappeared" during the military regime
available efforts were taken by the respondent," and that there was an "undue in Argentina. Enforced disappearances spread in Latin America, and the issue
haste" in the filing of the petition when, instead of cooperating with authorities, became an international concern when the world noted its widespread and
the respondent immediately invoked the Court’s intervention. systematic use by State security forces in that continent under Operation
o We do not see the respondent’s petition as the petitioners view it. Condor84 and during the Dirty War85 in the 1970s and 1980s. The escalation of
x Section 5(e) merely requires that the Amparo petitioner (the the practice saw political activists secretly arrested, tortured, and killed as part of
respondent in the present case) allege "the actions and recourses governments’ counter-insurgency campaigns. As this form of political brutality
taken to determine the fate or whereabouts of the aggrieved party and became routine elsewhere in the continent, the Latin American media
the identity of the person responsible for the threat, act or omission." standardized the term "disappearance" to describe the phenomenon. The victims
The following allegations of the respondent’s petition duly outlined the actions of enforced disappearances were called the "desaparecidos,"86 which literally
she had taken and the frustrations she encountered, thus compelling her to file means the "disappeared ones."87
her petition. x In general, there are three different kinds of "disappearance" cases:
o 7. Soon after the student left the room, Engr. Tagitis went out of o 1) those of people arrested without witnesses or without
the pension house to take his early lunch but while out on the positive identification of the arresting agents and are never
street, a couple of burly men believed to be police intelligence found again;
operatives, forcibly took him and boarded the latter on a motor o 2) those of prisoners who are usually arrested without an
vehicle then sped away without the knowledge of his student, appropriate warrant and held in complete isolation for
Arsimin Kunnong; weeks or months while their families are unable to discover their
o 10. When Kunnong could not locate Engr. Tagitis, the former whereabouts and the military authorities deny having them in
sought the help of another IDB scholar and reported the matter to custody until they eventually reappear in one detention center or
the local police agency; another; and
o 11. Arsimin Kunnong, including his friends and companions in Jolo, o 3) those of victims of "salvaging" who have disappeared until
exerted efforts in trying to locate the whereabouts of Engr. Tagitis their lifeless bodies are later discovered.88
and when he reported the matter to the police authorities in Jolo, x In the Philippines, enforced disappearances generally fall within the first
he was immediately given a ready answer that Engr. Tagitis could two categories,89 and 855 cases were recorded during the period of martial
[have been] abducted by the Abu Sayyaf group and other groups law from 1972 until 1986. Of this number, 595 remained missing, 132 surfaced
known to be fighting against the government; alive and 127 were found dead. During former President Corazon C. Aquino’s
o 12. Being scared with these suggestions and insinuations of the term, 820 people were reported to have disappeared and of these, 612 cases
police officers, Kunnong reported the matter to the were documented. Of this number, 407 remain missing, 108 surfaced alive and
[respondent](wife of Engr. Tagitis) by phone and other responsible 97 were found dead. The number of enforced disappearances dropped during
officers and coordinators of the IDB Scholarship Programme in the former President Fidel V. Ramos’ term when only 87 cases were reported, while
Philippines who alerted the office of the Governor of ARMM who the three-year term of former President Joseph E. Estrada yielded 58 reported
was then preparing to attend the OIC meeting in Jeddah, Saudi cases. KARAPATAN, a local non-governmental organization, reports that as of
Arabia; March 31, 2008, the records show that there were a total of 193 victims of
o 13. [The respondent], on the other hand, approached some of her enforced disappearance under incumbent President Gloria M. Arroyo’s
co-employees with the Land Bank in Digos branch, Digos City, administration. The Commission on Human Rights’ records show a total of 636
Davao del Sur, who likewise sought help from some of their friends verified cases of enforced disappearances from 1985 to 1993. Of this number,
in the military who could help them find/locate the whereabouts of 406 remained missing, 92 surfaced alive, 62 were found dead, and 76 still have
her husband; undetermined status.90 Currently, the United Nations Working Group on
170
RECTO, GAYLE ANGELI M.
2011-0008 | AUSL
Personal Notes on Remedial Law 2 Review (based on the syllabus of Prof. Henedino M. Brondial)

Enforced or Involuntary Disappearance91 reports 619 outstanding cases of and, in the proper case, by the commencement of
enforced or involuntary disappearances covering the period December 1, 2007 to criminal action against the guilty parties
November 30, 2008 x Enforced Disappearances in International Law
x Enforced Disappearances in PH x From the International Law perspective, involuntary or enforced disappearance is
x The Amparo Rule expressly provides that the "writ shall cover extralegal killings considered a flagrant violation of human rights.101 It does not only violate
and enforced disappearances or threats thereof."93 We note that although the right to life, liberty and security of the desaparecido; it affects their families
the writ specifically covers "enforced disappearances," this concept is as well through the denial of their right to information regarding the
neither defined nor penalized in this jurisdiction. The records of the circumstances of the disappeared family member. Thus, enforced
Supreme Court Committee on the Revision of Rules (Committee) reveal that the disappearances have been said to be "a double form of torture," with "doubly
drafters of the Amparo Rule initially considered providing an elemental definition paralyzing impact for the victims," as they "are kept ignorant of their own fates,
of the concept of enforced disappearance:94 while family members are deprived of knowing the whereabouts of their detained
o JUSTICE MARTINEZ: I believe that first and foremost we should loved ones" and suffer as well the serious economic hardship and poverty that in
come up or formulate a specific definition [for] extrajudicial killings most cases follow the disappearance of the household breadwinner.102
and enforced disappearances. From that definition, then we can x The UN General Assembly first considered the issue of "Disappeared Persons" in
proceed to formulate the rules, definite rules concerning the same. December 1978 under Resolution 33/173. The Resolution expressed the General
o CHIEF JUSTICE PUNO: … As things stand, there is no law Assembly’s deep concern arising from "reports from various parts of the world
penalizing extrajudicial killings and enforced relating to enforced or involuntary disappearances," and requested the "UN
disappearances… so initially also we have to [come up Commission on Human Rights to consider the issue of enforced disappearances
with] the nature of these extrajudicial killings and enforced with a view to making appropriate recommendations."103
disappearances [to be covered by the Rule] because our x In 1992, in response to the reality that the insidious practice of enforced
concept of killings and disappearances will define the disappearance had become a global phenomenon, the UN General Assembly
jurisdiction of the courts. So we’ll have to agree among adopted the Declaration on the Protection of All Persons from Enforced
ourselves about the nature of killings and disappearances for Disappearance (Declaration).104 This Declaration, for the first time, provided in
instance, in other jurisdictions, the rules only cover state its third preambular clause a working description of enforced disappearance, as
actors. That is an element incorporated in their concept of follows:
extrajudicial killings and enforced disappearances. In other o Deeply concerned that in many countries, often in a persistent
jurisdictions, the concept includes acts and omissions not only of manner, enforced disappearances occur, in the sense that persons
state actors but also of non state actors. Well, more specifically in are arrested, detained or abducted against their will or otherwise
the case of the Philippines for instance, should these rules include deprived of their liberty by officials of different branches or levels of
the killings, the disappearances which may be authored by let us Government, or by organized groups or private individuals acting
say, the NPAs or the leftist organizations and others. So, again we on behalf of, or with the support, direct or indirect, consent or
need to define the nature of the extrajudicial killings and enforced acquiescence of the Government, followed by a refusal to disclose
disappearances that will be covered by these rules. [Emphasis the fate or whereabouts of the persons concerned or a refusal to
supplied] 95 acknowledge the deprivation of their liberty, which places such
x In the end, the Committee took cognizance of several bills filed in the House of persons outside the protection of the law. [Emphasis supplied]
Representatives96 and in the Senate97 on extrajudicial killings and enforced x Fourteen years after (or on December 20, 2006), the UN General Assembly
disappearances, and resolved to do away with a clear textual definition of these adopted the International Convention for the Protection of All Persons from
terms in the Rule. The Committee instead focused on the nature and Enforced Disappearance (Convention).105 The Convention was opened for
scope of the concerns within its power to address and provided the signature in Paris, France on February 6, 2007.106 Article 2 of the Convention
appropriate remedy therefor, mindful that an elemental definition may defined enforced disappearance as follows:
intrude into the ongoing legislative efforts.98 o For the purposes of this Convention, "enforced disappearance" is
x As the law now stands, extra-judicial killings and enforced considered to be the arrest, detention, abduction or any other form
disappearances in this jurisdiction are not crimes penalized separately of deprivation of liberty by agents of the State or by persons or
from the component criminal acts undertaken to carry out these groups of persons acting with the authorization, support or
killings and enforced disappearances and are now penalized under the acquiescence of the State, followed by a refusal to acknowledge the
Revised Penal Code and special laws.99 The simple reason is that the deprivation of liberty or by concealment of the fate or whereabouts
Legislature has not spoken on the matter; the determination of what of the disappeared person, which place such a person outside the
acts are criminal and what the corresponding penalty these criminal protection of the law. [Emphasis supplied]
acts should carry are matters of substantive law that only the x The Convention is the first universal human rights instrument to assert that there
Legislature has the power to enact under the country’s constitutional is a right not to be subject to enforced disappearance107 and that this right is
scheme and power structure. non-derogable.108 It provides that no one shall be subjected to enforced
x Even without the benefit of directly applicable substantive laws on extra-judicial disappearance under any circumstances, be it a state of war, internal political
killings and enforced disappearances, however, the Supreme Court is not instability, or any other public emergency. It obliges State Parties to codify
powerless to act under its own constitutional mandate to promulgate "rules enforced disappearance as an offense punishable with appropriate penalties
concerning the protection and enforcement of constitutional rights, pleading, under their criminal law.109 It also recognizes the right of relatives of the
practice and procedure in all courts,"100 since extrajudicial killings and enforced disappeared persons and of the society as a whole to know the truth on the fate
disappearances, by their nature and purpose, constitute State or private party and whereabouts of the disappeared and on the progress and results of the
violation of the constitutional rights of individuals to life, liberty and security. investigation.110 Lastly, it classifies enforced disappearance as a continuing
Although the Court’s power is strictly procedural and as such does not diminish, offense, such that statutes of limitations shall not apply until the fate and
increase or modify substantive rights, the legal protection that the Court can whereabouts of the victim are established.
provide can be very meaningful through the procedures it sets in addressing x To date, the Philippines has neither signed nor ratified the Convention, so that
extrajudicial killings and enforced disappearances. The Court, through its the country is not yet committed to enact any law penalizing enforced
procedural rules, can set the procedural standards and thereby directly compel disappearance as a crime. The absence of a specific penal law, however, is
the public authorities to act on actual or threatened violations of constitutional not a stumbling block for action from this Court, as heretofore
rights. To state the obvious, judicial intervention can make a difference - even if mentioned; underlying every enforced disappearance is a violation of
only procedurally - in a situation when the very same investigating public the constitutional rights to life, liberty and security that the Supreme
authorities may have had a hand in the threatened or actual violations of Court is mandated by the Constitution to protect through its rule-
constitutional rights. making powers.
x Lest this Court intervention be misunderstood, we clarify once again that we x Separately from the Constitution (but still pursuant to its terms), the Court is
do not rule on any issue of criminal culpability for the extrajudicial guided, in acting on Amparo cases, by the reality that the Philippines is a
killing or enforced disappearance. This is an issue that requires criminal member of the UN, bound by its Charter and by the various conventions we
action before our criminal courts based on our existing penal laws. Our signed and ratified, particularly the conventions touching on humans rights.
intervention is in determining whether an enforced disappearance has Under the UN Charter, the Philippines pledged to "promote universal respect for,
taken place and who is responsible or accountable for this and observance of, human rights and fundamental freedoms for all without
disappearance, and to define and impose the appropriate remedies to distinctions as to race, sex, language or religion."112 Although no universal
address it. The burden for the public authorities to discharge in these agreement has been reached on the precise extent of the "human rights and
situations, under the Rule on the Writ of Amparo, is twofold. fundamental freedoms" guaranteed to all by the Charter,113 it was the UN itself
o The first is to ensure that all efforts at disclosure and that issued the Declaration on enforced disappearance, and this Declaration
investigation are undertaken under pain of indirect states:114
contempt from this Court when governmental efforts are less than o Any act of enforced disappearance is an offence to dignity. It is
what the individual situations require. condemned as a denial of the purposes of the Charter of the United
o The second is to address the disappearance, so that the life of Nations and as a grave and flagrant violation of human rights and
the victim is preserved and his or her liberty and security restored. fundamental freedoms proclaimed in the Universal Declaration of
ƒ In these senses, our orders and directives relative to Human Rights and reaffirmed and developed in international
the writ are continuing efforts that are not truly instruments in this field. [Emphasis supplied]
terminated until the extrajudicial killing or enforced o As a matter of human right and fundamental freedom and as a
disappearance is fully addressed by the complete policy matter made in a UN Declaration, the ban on enforced
determination of the fate and the whereabouts of the disappearance cannot but have its effects on the country, given our
victim, by the production of the disappeared person own adherence to "generally accepted principles of international law
and the restoration of his or her liberty and security, as part of the law of the land."115
171
RECTO, GAYLE ANGELI M.
2011-0008 | AUSL
Personal Notes on Remedial Law 2 Review (based on the syllabus of Prof. Henedino M. Brondial)

x In the recent case of Pharmaceutical and Health Care Association of the "(m)embers can no longer contend that they do not
Philippines v. Duque III,116 we held that: know what human rights they promised in the Charter
o Under the 1987 Constitution, international law can become to promote." Moreover, a U.N. Declaration is,
part of the sphere of domestic law either by transformation according to one authoritative definition, "a formal
or incorporation. The transformation method requires that and solemn instrument, suitable for rare occasions
an international law be transformed into a domestic law when principles of great and lasting importance are
through a constitutional mechanism such as local being enunciated." Accordingly, it has been observed
legislation. The incorporation method applies when, by that the Universal Declaration of Human Rights "no
mere constitutional declaration, international law is longer fits into the dichotomy of ‘binding treaty’
deemed to have the force of domestic law. [Emphasis against ‘non-binding pronouncement,' but is rather an
supplied] authoritative statement of the international
x We characterized "generally accepted principles of international law" as norms of community." Thus, a Declaration creates an
general or customary international law that are binding on all states. We held expectation of adherence, and "insofar as the
further:117 expectation is gradually justified by State practice, a
o [G]enerally accepted principles of international law, by virtue of the declaration may by custom become recognized as
incorporation clause of the Constitution, form part of the laws of laying down rules binding upon the States." Indeed,
the land even if they do not derive from treaty obligations. The several commentators have concluded that the
classical formulation in international law sees those customary rules Universal Declaration has become, in toto, a part of
accepted as binding result from the combination [of] two elements: binding, customary international law. [Citations
the established, widespread, and consistent practice on the part of omitted]
States; and a psychological element known as the opinion juris sive o Fourth, in interpreting Article 2 (right to an effective domestic
necessitates (opinion as to law or necessity). Implicit in the latter remedy) of the International Convention on Civil and Political Rights
element is a belief that the practice in question is rendered (ICCPR), to which the Philippines is both a signatory and a State
obligatory by the existence of a rule of law requiring it. [Emphasis Party, the UN Human Rights Committee, under the Office of the
in the original] High Commissioner for Human Rights, has stated that the act of
x The most widely accepted statement of sources of international law today is enforced disappearance violates Articles 6 (right to life), 7
Article 38(1) of the Statute of the International Court of Justice, which provides (prohibition on torture, cruel, inhuman or degrading treatment or
that the Court shall apply "international custom, as evidence of a general punishment) and 9 (right to liberty and security of the person) of
practice accepted as law."118 The material sources of custom include State the ICCPR, and the act may also amount to a crime against
practice, State legislation, international and national judicial decisions, recitals in humanity.131
treaties and other international instruments, a pattern of treaties in the same o Fifth, Article 7, paragraph 1 of the 1998 Rome Statute establishing
form, the practice of international organs, and resolutions relating to legal the International Criminal Court (ICC) also covers enforced
questions in the UN General Assembly.119 Sometimes referred to as "evidence" disappearances insofar as they are defined as crimes against
of international law,120 these sources identify the substance and content of the humanity,132 i.e., crimes "committed as part of a widespread or
obligations of States and are indicative of the "State practice" and "opinio juris" systematic attack against any civilian population, with knowledge of
requirements of international law.121 We note the following in these respects: the attack." While more than 100 countries have ratified the Rome
o First, barely two years from the adoption of the Declaration, the Statute,133 the Philippines is still merely a signatory and has not
Organization of American States (OAS) General Assembly adopted yet ratified it. We note that Article 7(1) of the Rome Statute has
the Inter-American Convention on Enforced Disappearance of been incorporated in the statutes of other international and hybrid
Persons in June 1994.122 State parties undertook under this tribunals, including Sierra Leone Special Court, the Special Panels
Convention "not to practice, permit, or tolerate the forced for Serious Crimes in Timor-Leste, and the Extraordinary Chambers
disappearance of persons, even in states of emergency or in the Courts of Cambodia.134 In addition, the implementing
suspension of individual guarantees."123 One of the key provisions legislation of State Parties to the Rome Statute of the ICC has
includes the States’ obligation to enact the crime of forced given rise to a number of national criminal provisions also covering
disappearance in their respective national criminal laws and to enforced disappearance.135
establish jurisdiction over such cases when the crime was x While the Philippines is not yet formally bound by the terms of the Convention on
committed within their jurisdiction, when the victim is a national of enforced disappearance (or by the specific terms of the Rome Statute) and has
that State, and "when the alleged criminal is within its territory and not formally declared enforced disappearance as a specific crime, the above
it does not proceed to extradite him," which can be interpreted as recital shows that enforced disappearance as a State practice has been
establishing universal jurisdiction among the parties to the Inter- repudiated by the international community, so that the ban on it is now a
American Convention.124 At present, Colombia, Guatemala, generally accepted principle of international law, which we should consider a part
Paraguay, Peru and Venezuela have enacted separate laws in of the law of the land, and which we should act upon to the extent already
accordance with the Inter-American Convention and have defined allowed under our laws and the international conventions that bind us.
activities involving enforced disappearance to be x The following civil or political rights under the Universal Declaration of Human
criminal.1251avvphi1 Rights, the ICCPR and the International Convention on Economic, Social and
o Second, in Europe, the European Convention on Human Rights has Cultural Rights (ICESR) may be infringed in the course of a disappearance:136
no explicit provision dealing with the protection against enforced o 1) the right to recognition as a person before the law;
disappearance. The European Court of Human Rights (ECHR), o 2) the right to liberty and security of the person;
however, has applied the Convention in a way that provides ample o 3) the right not to be subjected to torture and other cruel, inhuman
protection for the underlying rights affected by enforced or degrading treatment or punishment;
disappearance through the Convention’s Article 2 on the right to o 4) the right to life, when the disappeared person is killed;
life; Article 3 on the prohibition of torture; Article 5 on the right to o 5) the right to an identity;
liberty and security; Article 6, paragraph 1 on the right to a fair o 6) the right to a fair trial and to judicial guarantees;
trial; and Article 13 on the right to an effective remedy. A leading o 7) the right to an effective remedy, including reparation and
example demonstrating the protection afforded by the European compensation;
Convention is Kurt v. Turkey,126 where the ECHR found a violation o 8) the right to know the truth regarding the circumstances of a
of the right to liberty and security of the disappeared person when disappearance.
the applicant’s son disappeared after being taken into custody by o 9) the right to protection and assistance to the family;
Turkish forces in the Kurdish village of Agilli in November 1993. It o 10) the right to an adequate standard of living;
further found the applicant (the disappeared person’s mother) to o 11) the right to health; and
be a victim of a violation of Article 3, as a result of the silence of o 12) the right to education [Emphasis supplied]
the authorities and the inadequate character of the investigations x Article 2 of the ICCPR, which binds the Philippines as a state party, provides:
undertaken. The ECHR also saw the lack of any meaningful o Article 2
investigation by the State as a violation of Article 13.127 o 3. Each State Party to the present Covenant undertakes:
o Third, in the United States, the status of the prohibition on ƒ (a) To ensure that any person whose rights or
enforced disappearance as part of customary international law is freedoms as herein recognized are violated shall have
recognized in the most recent edition of Restatement of the Law: an effective remedy, notwithstanding that the
The Third,128 which provides that "[a] State violates international violation has been committed by persons acting in an
law if, as a matter of State policy, it practices, encourages, or official capacity;
condones… (3) the murder or causing the disappearance of ƒ (b) To ensure that any person claiming such a
individuals."129 We significantly note that in a related matter that remedy shall have his right thereto determined by
finds close identification with enforced disappearance - the matter competent judicial, administrative or legislative
of torture - the United States Court of Appeals for the Second authorities, or by any other competent authority
Circuit Court held in Filartiga v. Pena-Irala130 that the prohibition provided for by the legal system of the State, and to
on torture had attained the status of customary international law. develop the possibilities of judicial remedy;
The court further elaborated on the significance of UN declarations, ƒ (c) To ensure that the competent authorities shall
as follows: enforce such remedies when granted. [Emphasis
ƒ These U.N. declarations are significant because they supplied]
specify with great precision the obligations of member x In General Comment No. 31, the UN Human Rights Committee opined that the
nations under the Charter. Since their adoption, right to an effective remedy under Article 2 of the ICCPR includes the obligation
172
RECTO, GAYLE ANGELI M.
2011-0008 | AUSL
Personal Notes on Remedial Law 2 Review (based on the syllabus of Prof. Henedino M. Brondial)

of the State to investigate ICCPR violations promptly, thoroughly, and effectively, x Before going into the issue of whether the respondent has discharged the burden
viz:137 of proving the allegations of the petition for the Writ of Amparo by the degree of
o 15. Article 2, paragraph 3, requires that in addition to effective proof required by the Amparo Rule, we shall discuss briefly the unique
protection of Covenant rights, States Parties must ensure that evidentiary difficulties presented by enforced disappearance cases;
individuals also have accessible and effective remedies to vindicate these difficulties form part of the setting that the implementation of
those rights… The Committee attaches importance to States the Amparo Rule shall encounter.
Parties' establishing appropriate judicial and administrative x These difficulties largely arise because the State itself - the party whose
mechanisms for addressing claims of rights violations under involvement is alleged - investigates enforced disappearances. Past experiences
domestic law… Administrative mechanisms are particularly required in other jurisdictions show that the evidentiary difficulties are generally
to give effect to the general obligation to investigate allegations of threefold.
violations promptly, thoroughly and effectively through independent o First, there may be a deliberate concealment of the
and impartial bodies. A failure by a State Party to investigate identities of the direct perpetrators.141 Experts note that
allegations of violations could in and of itself give rise to a separate abductors are well organized, armed and usually members of the
breach of the Covenant. Cessation of an ongoing violation is an military or police forces, thus:
essential element of the right to an effective remedy. [Emphasis ƒ The victim is generally arrested by the security forces
supplied] or by persons acting under some form of
x The UN Human Rights Committee further stated in the same General Comment governmental authority. In many countries the units
No. 31 that failure to investigate as well as failure to bring to justice the that plan, implement and execute the program are
perpetrators of ICCPR violations could in and of itself give rise to a separate generally specialized, highly-secret bodies within the
breach of the Covenant, thus:138 armed or security forces. They are generally directed
o 18. Where the investigations referred to in paragraph 15 reveal through a separate, clandestine chain of command,
violations of certain Covenant rights, States Parties must ensure but they have the necessary credentials to avoid or
that those responsible are brought to justice. As with failure to prevent any interference by the "legal" police forces.
investigate, failure to bring to justice perpetrators of such violations These authorities take their victims to secret
could in and of itself give rise to a separate breach of the detention centers where they subject them to
Covenant. These obligations arise notably in respect of those interrogation and torture without fear of judicial or
violations recognized as criminal under either domestic or other controls.142
international law, such as torture and similar cruel, inhuman and ƒ In addition, there are usually no witnesses to the
degrading treatment (article 7), summary and arbitrary killing crime; if there are, these witnesses are usually afraid
(article 6) and enforced disappearance (articles 7 and 9 and, to speak out publicly or to testify on the
frequently, 6). Indeed, the problem of impunity for these violations, disappearance out of fear for their own lives.143 We
a matter of sustained concern by the Committee, may well be an have had occasion to note this difficulty in Secretary of
important contributing element in the recurrence of the violations. Defense v. Manalo144 when we acknowledged that
When committed as part of a widespread or systematic attack on a "where powerful military officers are implicated, the
civilian population, these violations of the Covenant are crimes hesitation of witnesses to surface and testify against
against humanity (see Rome Statute of the International Criminal them comes as no surprise."
Court, article 7). [Emphasis supplied] o Second, deliberate concealment of pertinent evidence of
x In Secretary of National Defense v. Manalo,139 this Court, in ruling that the right the disappearance is a distinct possibility; the central piece of
to security of persons is a guarantee of the protection of one’s right by the evidence in an enforced disappearance - i.e., the corpus delicti or
government, held that: the victim’s body - is usually concealed to effectively thwart the
o The right to security of person in this third sense is a corollary of start of any investigation or the progress of one that may have
the policy that the State "guarantees full respect for human rights" begun.145 The problem for the victim’s family is the State’s virtual
under Article II, Section 11 of the 1987 Constitution. As the monopoly of access to pertinent evidence. The Inter-American
government is the chief guarantor of order and security, the Court of Human Rights (IACHR) observed in the landmark case of
Constitutional guarantee of the rights to life, liberty and security of Velasquez Rodriguez146 that inherent to the practice of enforced
person is rendered ineffective if government does not afford disappearance is the deliberate use of the State’s power to destroy
protection to these rights especially when they are under threat. the pertinent evidence. The IACHR described the concealment as a
Protection includes conducting effective investigations, organization clear attempt by the State to commit the perfect crime.147
of the government apparatus to extend protection to victims of o Third is the element of denial; in many cases, the State
extralegal killings or enforced disappearances (or threats thereof) authorities deliberately deny that the enforced
and/or their families, and bringing offenders to the bar of justice. disappearance ever occurred.148 "Deniability" is central to the
The Inter-American Court of Human Rights stressed the importance policy of enforced disappearances, as the absence of any proven
of investigation in the Velasquez Rodriguez Case, viz: disappearance makes it easier to escape the application of legal
ƒ (The duty to investigate) must be undertaken in a standards ensuring the victim’s human rights.149 Experience shows
serious manner and not as a mere formality that government officials typically respond to requests for
preordained to be ineffective. An investigation must information about desaparecidos by saying that they are not aware
have an objective and be assumed by the State as its of any disappearance, that the missing people may have fled the
own legal duty, not as a step taken by private country, or that their names have merely been invented.150
interests that depends upon the initiative of the victim x These considerations are alive in our minds, as these are the difficulties we
or his family or upon their offer of proof, without an confront, in one form or another, in our consideration of this case.
effective search for the truth by the government. x Sections 13, 17 and 18 of the Amparo Rule define the nature of an Amparo
[Emphasis supplied] proceeding and the degree and burden of proof the parties to the case carry, as
x Manalo significantly cited Kurt v. Turkey,140 where the ECHR interpreted the follows:
"right to security" not only as a prohibition on the State against arbitrary o Section 13. Summary Hearing. The hearing on the petition shall be
deprivation of liberty, but also as the imposition of a positive duty to afford summary. However, the court, justice or judge may call for a
protection to the right to liberty. The Court notably quoted the following ECHR preliminary conference to simplify the issues and determine the
ruling: possibility of obtaining stipulations and admissions from the parties.
o [A]ny deprivation of liberty must not only have been effected in o Section 17. Burden of Proof and Standard of Diligence Required. -
conformity with the substantive and procedural rules of national The parties shall establish their claims by substantial evidence.
law but must equally be in keeping with the very purpose of Article The respondent who is a private individual must
5, namely to protect the individual from arbitrariness... Having prove that ordinary diligence as required by applicable laws, rules
assumed control over that individual, it is incumbent on the and regulations was observed in the performance of duty.
authorities to account for his or her whereabouts. For this reason, The respondent who is a public official or employee
Article 5 must be seen as requiring the authorities to take effective must prove that extraordinary diligence as required by applicable
measures to safeguard against the risk of disappearance and to laws, rules and regulations was observed in the performance of
conduct a prompt effective investigation into an arguable claim that duty.
a person has been taken into custody and has not been seen since. The respondent public official or employee cannot
[Emphasis supplied] invoke the presumption that official duty has been regularly
x These rulings effectively serve as the backdrop for the Rule on the Writ of performed or evade responsibility or liability.
Amparo, which the Court made effective on October 24, 2007. Although the o Section 18. Judgment. - … If the allegations in the petition are
Amparo Rule still has gaps waiting to be filled through substantive law, as proven by substantial evidence, the court shall grant the privilege
evidenced primarily by the lack of a concrete definition of "enforced of the writ and such reliefs as may be proper and appropriate;
disappearance," the materials cited above, among others, provide ample otherwise, the privilege shall be denied. [Emphasis supplied]
guidance and standards on how, through the medium of the Amparo Rule, the x These characteristics - namely, of being summary and the use of
Court can provide remedies and protect the constitutional rights to life, liberty substantial evidence as the required level of proof (in contrast to the usual
and security that underlie every enforced disappearance. preponderance of evidence or proof beyond reasonable doubt in court
proceedings) - reveal the clear intent of the framers of the Amparo Rule
ISSUE # 3: What is the nature of writ of amparo proceedings? to have the equivalent of an administrative proceeding, albeit judicially
conducted, in addressing Amparo situations. The standard of diligence
HELD # 3: SUMMARY and AKIN TO ADMINISTRATIVE PROCEEDINGS. required - the duty of public officials and employees to observe extraordinary
173
RECTO, GAYLE ANGELI M.
2011-0008 | AUSL
Personal Notes on Remedial Law 2 Review (based on the syllabus of Prof. Henedino M. Brondial)

diligence - point, too, to the extraordinary measures expected in the protection of must be responsive to the evidentiary difficulties
constitutional rights and in the consequent handling and investigation of extra- faced. On the one hand, we cannot be arbitrary in the
judicial killings and enforced disappearance cases. admission and appreciation of evidence, as
x Thus, in these proceedings, the Amparo petitioner needs only to properly comply arbitrariness entails violation of rights and cannot be
with the substance and form requirements of a Writ of Amparo petition, as used as an effective counter-measure; we only
discussed above, and prove the allegations by substantial evidence. Once a compound the problem if a wrong is addressed by the
rebuttable case has been proven, the respondents must then respond and prove commission of another wrong. On the other hand, we
their defenses based on the standard of diligence required. The rebuttable case, cannot be very strict in our evidentiary rules and
of course, must show that an enforced disappearance took place under cannot consider evidence the way we do in the usual
circumstances showing a violation of the victim’s constitutional rights to life, criminal and civil cases; precisely, the proceedings
liberty or security, and the failure on the part of the investigating authorities to before us are administrative in nature where, as a
appropriately respond. rule, technical rules of evidence are not strictly
x The landmark case of Ang Tibay v. Court of Industrial Relations151 observed. Thus, while we must follow the substantial
provided the Court its first opportunity to define the substantial evidence evidence rule, we must observe flexibility in
required to arrive at a valid decision in administrative proceedings. To directly considering the evidence we shall take into account.
quote Ang Tibay: x The fair and proper rule, to our mind, is to consider all the pieces of
o Substantial evidence is more than a mere scintilla. It means evidence adduced in their totality, and to consider any evidence
such relevant evidence as a reasonable mind might accept otherwise inadmissible under our usual rules to be admissible if it is
as adequate to support a conclusion. [citations omitted] The consistent with the admissible evidence adduced. In other words, we
statute provides that ‘the rules of evidence prevailing in courts of reduce our rules to the most basic test of reason - i.e., to the relevance
law and equity shall not be controlling.’ The obvious purpose of this of the evidence to the issue at hand and its consistency with all other
and similar provisions is to free administrative boards from the pieces of adduced evidence. Thus, even hearsay evidence can be
compulsion of technical rules so that the mere admission of matter admitted if it satisfies this basic minimum test.
which would be deemed incompetent in judicial proceedings would x We note in this regard that the use of flexibility in the consideration of evidence
not invalidate the administrative order. [citations omitted] But this is not at all novel in the Philippine legal system. In child abuse cases, Section 28
assurance of a desirable flexibility in administrative procedure does of the Rule on Examination of a Child Witness157 is expressly recognized as an
not go so far as to justify orders without a basis in evidence having exception to the hearsay rule. This Rule allows the admission of the hearsay
rational probative force. [Emphasis supplied testimony of a child describing any act or attempted act of sexual abuse in any
x In Secretary of Defense v. Manalo,152 which was the Court’s first petition for a criminal or non-criminal proceeding, subject to certain prerequisites and the right
Writ of Amparo, we recognized that the full and exhaustive proceedings that the of cross-examination by the adverse party. The admission of the statement is
substantial evidence standard regularly requires do not need to apply due to the determined by the court in light of specified subjective and objective
summary nature of Amparo proceedings. We said: considerations that provide sufficient indicia of reliability of the child witness.158
o The remedy [of the writ of amparo] provides rapid judicial These requisites for admission find their counterpart in the present case under
relief as it partakes of a summary proceeding that requires the above-described conditions for the exercise of flexibility in the consideration
only substantial evidence to make the appropriate reliefs of evidence, including hearsay evidence, in extrajudicial killings and enforced
available to the petitioner; it is not an action to determine disappearance cases.
criminal guilt requiring proof beyond reasonable doubt, or liability
for damages requiring preponderance of evidence, or ISSUE # 4: Whether Mary Jane Tagitis was able to discharge her burden of proving by
administrative responsibility requiring substantial evidence that will substantial evidence the allegation of enforced disappearance.
require full and exhaustive proceedings. [Emphasis supplied]
x Not to be forgotten in considering the evidentiary aspects of Amparo petitions HELD # 4: YES.
are the unique difficulties presented by the nature of enforced disappearances, x The threshold question for our resolution is: was there an enforced
heretofore discussed, which difficulties this Court must frontally meet if the disappearance within the meaning of this term under the UN Declaration we
Amparo Rule is to be given a chance to achieve its objectives. These evidentiary have cited?
difficulties compel the Court to adopt standards appropriate and responsive to x The Convention defines enforced disappearance as "the arrest, detention,
the circumstances, without transgressing the due process requirements that abduction or any other form of deprivation of liberty by agents of the State or by
underlie every proceeding. persons or groups of persons acting with the authorization, support or
x In the seminal case of Velasquez Rodriguez,153 the IACHR - faced with a lack of acquiescence of the State, followed by a refusal to acknowledge the deprivation
direct evidence that the government of Honduras was involved in Velasquez of liberty or by concealment of the fate or whereabouts of the disappeared
Rodriguez’ disappearance - adopted a relaxed and informal evidentiary standard, person, which place such a person outside the protection of the law."159
and established the rule that presumes governmental responsibility for a x Under this definition, the elements that constitute enforced disappearance
disappearance if it can be proven that the government carries out a general are essentially fourfold:
practice of enforced disappearances and the specific case can be linked to that o (a) arrest, detention, abduction or any form of deprivation of
practice.154 The IACHR took note of the realistic fact that enforced liberty;
disappearances could be proven only through circumstantial or indirect evidence o (b) carried out by agents of the State or persons or groups of
or by logical inference; otherwise, it was impossible to prove that an individual persons acting with the authorization, support or acquiescence of
had been made to disappear. It held: the State;
o 130. The practice of international and domestic courts shows that o (c) followed by a refusal to acknowledge the detention, or a
direct evidence, whether testimonial or documentary, is not the concealment of the fate of the disappeared person; and
only type of evidence that may be legitimately considered in o (d) placement of the disappeared person outside the protection of
reaching a decision. Circumstantial evidence, indicia, and the law. [Emphasis supplied]
presumptions may be considered, so long as they lead to x We find no direct evidence indicating how the victim actually disappeared. The
conclusions consistent with the facts. direct evidence at hand only shows that Tagitis went out of the ASY Pension
o 131. Circumstantial or presumptive evidence is especially important House after depositing his room key with the hotel desk and was never seen nor
in allegations of disappearances, because this type of repression is heard of again. The undisputed conclusion, however, from all concerned - the
characterized by an attempt to suppress all information about the petitioner, Tagitis’ colleagues and even the police authorities - is that Tagistis
kidnapping or the whereabouts and fate of the victim. [Emphasis disappeared under mysterious circumstances and was never seen again. The
supplied] respondent injected the causal element in her petition and testimony, as we shall
ƒ In concluding that the disappearance of Manfredo discuss below.
Velásquez (Manfredo) was carried out by agents who x We likewise find no direct evidence showing that operatives of PNP CIDG
acted under cover of public authority, the IACHR Zamboanga abducted or arrested Tagitis. If at all, only the respondent’s
relied on circumstantial evidence including the allegation that Tagistis was under CIDG Zamboanga custody stands on record,
hearsay testimony of Zenaida Velásquez, the victim’s but it is not supported by any other evidence, direct or circumstantial.
sister, who described Manfredo’s kidnapping on the x In her direct testimony, the respondent pointed to two sources of information as
basis of conversations she had with witnesses who her bases for her allegation that Tagistis had been placed under government
saw Manfredo kidnapped by men in civilian clothes in custody (in contrast with CIDG Zamboanga custody). The first was an unnamed
broad daylight. She also told the Court that a former friend in Zamboanga (later identified as Col. Ancanan), who occupied a high
Honduran military official had announced that position in the military and who allegedly mentioned that Tagitis was in good
Manfredo was kidnapped by a special military hands. Nothing came out of this claim, as both the respondent herself and her
squadron acting under orders of the Chief of the witness, Mrs. Talbin, failed to establish that Col. Ancanan gave them any
Armed Forces.155 The IACHR likewise considered the information that Tagitis was in government custody. Col. Ancanan, for his part,
hearsay testimony of a second witness who asserted admitted the meeting with the respondent but denied giving her any information
that he had been told by a Honduran military officer about the disappearance.
about the disappearance, and a third witness who x The more specific and productive source of information was Col. Kasim, whom
testified that he had spoken in prison to a man who the respondent, together with her witness Mrs. Talbin, met in Camp Katitipan in
identified himself as Manfredo.156 Davao City. To quote the relevant portions of the respondent’s testimony:
ƒ Velasquez stresses the lesson that flexibility is o Q: Were you able to speak to other military officials regarding the
necessary under the unique circumstances that whereabouts of your husband particularly those in charge of any
enforced disappearance cases pose to the courts; to records or investigation?
have an effective remedy, the standard of evidence
174
RECTO, GAYLE ANGELI M.
2011-0008 | AUSL
Personal Notes on Remedial Law 2 Review (based on the syllabus of Prof. Henedino M. Brondial)

o A: I went to Camp Katitipan in Davao City. Then one military o Q: When he was reading it to you, was he reading it line by line or
officer, Col. Casim, told me that my husband is being abducted he was reading in a summary form?
[sic] because he is under custodial investigation because he is o A: Sometimes he was glancing to the report and talking to us,
allegedly "parang liason ng J.I.", sir. sir.165
o Q: What is J.I.? o Q: Were you informed as to the place where he was being kept
o A: Jema’ah Islamiah, sir. during that time?
o Q: Was there any information that was read to you during one of o A: He did not tell us where he [Tagitis] was being kept. But he
those visits of yours in that Camp? mentioned this Talipapao, Sulu, sir.
o A: Col. Casim did not furnish me a copy of his report because he o Q: After that incident, what did you do if any?
said those reports are highly confidential, sir. o A: We just left and as I’ve mentioned, we just waited because that
o Q: Was it read to you then even though you were not furnished a raw information that he was reading to us [sic] after the custodial
copy? investigation, Engineer Tagitis will be released. [Emphasis
o A: Yes, sir. In front of us, my friends. supplied]166
o Q: And what was the content of that highly confidential report? x Col. Kasim never denied that he met with the respondent and her friends, and
o A: Those alleged activities of Engineer Tagitis, sir.161 [Emphasis that he provided them information based on the input of an unnamed asset. He
supplied] simply claimed in his testimony that the "informal letter" he received from his
x She confirmed this testimony in her cross-examination: informant in Sulu did not indicate that Tagitis was in the custody of the CIDG. He
o Q: You also mentioned that you went to Camp Katitipan in Davao also stressed that the information he provided the respondent was merely a "raw
City? report" from "barangay intelligence" that still needed confirmation and "follow
o A: Yes, ma’am. up" as to its veracity.167
o Q: And a certain Col. Kasim told you that your husband was x To be sure, the respondent’s and Mrs. Talbin’s testimonies were far from perfect,
abducted and under custodial investigation? as the petitioners pointed out. The respondent mistakenly characterized Col.
o A: Yes, ma’am. Kasim as a "military officer" who told her that "her husband is being abducted
o Q: And you mentioned that he showed you a report? because he is under custodial investigation because he is allegedly ‘parang liason
o A: Yes, ma’am. ng J.I.’" The petitioners also noted that "Mrs. Talbin’s testimony imputing certain
o Q: Were you able to read the contents of that report? statements to Sr. Supt. Kasim that Engr. Tagitis is with the military, but he is not
o A: He did not furnish me a copy of those [sic] report because those certain whether it is the PNP or AFP is not worthy of belief, since Sr. Supt. Kasim
[sic] were highly confidential. That is a military report, ma’am. is a high ranking police officer who would certainly know that the PNP is not part
o Q: But you were able to read the contents? of the military."
o A: No. But he read it in front of us, my friends, ma’am. x Upon deeper consideration of these inconsistencies, however, what appears clear
o Q: How many were you when you went to see Col. Kasim? to us is that the petitioners never really steadfastly disputed or presented
o A: There were three of us, ma’am. evidence to refute the credibility of the respondent and her witness, Mrs. Talbin.
o Q: Who were your companions? The inconsistencies the petitioners point out relate, more than anything else, to
o A: Mrs. Talbin, tapos yung dalawang friends nya from Mati City, details that should not affect the credibility of the respondent and Mrs. Talbin;
Davao Oriental, ma’am.162 the inconsistencies are not on material points.168 We note, for example, that
o Q: When you were told that your husband is in good hands, what these witnesses are lay people in so far as military and police matters are
was your reaction and what did you do? concerned, and confusion between the police and the military is not unusual. As
o A: May binasa kasi sya that my husband has a parang meeting with a rule, minor inconsistencies such as these indicate truthfulness rather than
other people na parang mga terorista na mga tao. Tapos at the prevarication169and only tend to strengthen their probative value, in contrast to
end of the report is [sic] under custodial investigation. So I told him testimonies from various witnesses dovetailing on every detail; the latter cannot
"Colonel, my husband is sick. He is diabetic at nagmemaintain yun but generate suspicion that the material circumstances they testified to were
ng gamot. Pakisabi lang sa naghohold sa asawa ko na bigyan siya integral parts of a well thought of and prefabricated story.170
ng gamot, ma’am."163 x Based on these considerations and the unique evidentiary situation in enforced
o Q: You mentioned that you received information that Engineer disappearance cases, we hold it duly established that Col. Kasim informed the
Tagitis is being held by the CIDG in Zamboanga, did you go to respondent and her friends, based on the informant’s letter, that Tagitis,
CIDG Zamboanga to verify that information? reputedly a liaison for the JI and who had been under surveillance since January
o A: I did not go to CIDG Zamboanga. I went to Camp Karingal 2007, was "in good hands" and under custodial investigation for complicity with
instead. Enough na yun na effort ko because I know that they the JI after he was seen talking to one Omar Patik and a certain "Santos" of
would deny it, ma’am.164 Bulacan, a "Balik Islam" charged with terrorism. The respondent’s and Mrs.
x On February 11, 2008, the respondent presented Mrs. Talbin to corroborate her Talbin’s testimonies cannot simply be defeated by Col. Kasim’s plain denial and
testimony that her husband was abducted and held under custodial investigation his claim that he had destroyed his informant’s letter, the critical piece of
by the PNP-CIDG Zamboanga City, viz: evidence that supports or negates the parties’ conflicting claims. Col. Kasim’s
o Q: You said that you went to Camp Katitipan in Davao City admitted destruction of this letter - effectively, a suppression of this evidence -
sometime November 24, 2007, who was with you when you went raises the presumption that the letter, if produced, would be proof of what the
there? respondent claimed.171 For brevity, we shall call the evidence of what Col. Kasim
o A: Mary Jean Tagitis, sir. reported to the respondent to be the "Kasim evidence."
o Q: Only the two of you? x Given this evidence, our next step is to decide whether we can accept this
o A: No. We have some other companions. We were four at that evidence, in lieu of direct evidence, as proof that the disappearance of Tagitis was
time, sir. due to action with government participation, knowledge or consent and that he
o Q: Who were they? was held for custodial investigation. We note in this regard that Col. Kasim was
o A: Salvacion Serrano, Mini Leong, Mrs. Tagitis and me, sir. never quoted to have said that the custodial investigation was by the CIDG
o Q: Were you able to talk, see some other officials at Camp Zamboanga. The Kasim evidence only implies government intervention through
Katitipan during that time? the use of the term "custodial investigation," and does not at all point to CIDG
o A: Col. Kasim (PS Supt. Julasirim Ahadin Kasim) only, sir. Zamboanga as Tagitis’ custodian.
o Q: Were you able to talk to him? x Strictly speaking, we are faced here with a classic case of hearsay evidence -
o A: Yes, sir. i.e., evidence whose probative value is not based on the personal knowledge of
o Q: The four of you? the witnesses (the respondent, Mrs. Talbin and Col. Kasim himself) but on the
o A: Yes, sir. knowledge of some other person not on the witness stand (the informant).172
o Q: What information did you get from Col. Kasim during that time? x To say that this piece of evidence is incompetent and inadmissible evidence of
o A: The first time we met with [him] I asked him if he knew of the what it substantively states is to acknowledge - as the petitioners effectively
exact location, if he can furnish us the location of Engr. Tagitis. And suggest - that in the absence of any direct evidence, we should simply dismiss
he was reading this report. He told us that Engr. Tagitis is in good the petition. To our mind, an immediate dismissal for this reason is no different
hands. He is with the military, but he is not certain whether he is from a statement that the Amparo Rule - despite its terms - is ineffective, as it
with the AFP or PNP. He has this serious case. He was charged of cannot allow for the special evidentiary difficulties that are unavoidably present
terrorism because he was under surveillance from January 2007 up in Amparo situations, particularly in extrajudicial killings and enforced
to the time that he was abducted. He told us that he was under disappearances. The Amparo Rule was not promulgated with this intent or with
custodial investigation. As I’ve said earlier, he was seen under the intent to make it a token gesture of concern for constitutional rights. It was
surveillance from January. He was seen talking to Omar Patik, a promulgated to provide effective and timely remedies, using and profiting from
certain Santos of Bulacan who is also a Balik Islam and charged local and international experiences in extrajudicial killings and enforced
with terrorism. He was seen carrying boxes of medicines. Then we disappearances, as the situation may require. Consequently, we have no choice
asked him how long will he be in custodial investigation. He said but to meet the evidentiary difficulties inherent in enforced disappearances with
until we can get some information. But he also told us that he the flexibility that these difficulties demand.1avvphi1
cannot give us that report because it was a raw report. It was not x To give full meaning to our Constitution and the rights it protects, we hold that,
official, sir. as in Velasquez, we should at least take a close look at the available evidence to
o Q: You said that he was reading a report, was that report in determine the correct import of every piece of evidence - even of those usually
document form, in a piece of paper or was it in the computer or considered inadmissible under the general rules of evidence - taking into
what? account the surrounding circumstances and the test of reason that we can use as
o A: As far as I can see it, sir, it is written in white bond paper. I basic minimum admissibility requirement. In the present case, we should at least
don’t know if it was computerized but I’m certain that it was determine whether the Kasim evidence before us is relevant and meaningful to
typewritten. I’m not sure if it used computer, fax or what, sir.
175
RECTO, GAYLE ANGELI M.
2011-0008 | AUSL
Personal Notes on Remedial Law 2 Review (based on the syllabus of Prof. Henedino M. Brondial)

the disappearance of Tagistis and reasonably consistent with other evidence in disclosure that Tagitis was under custodial investigation for complicity in
the case. terrorism. Another distinctive trait that runs through these developments is the
x The evidence about Tagitis’ personal circumstances surrounded him with an air government’s dismissive approach to the disappearance, starting from the initial
of mystery. He was reputedly a consultant of the World Bank and a Senior response by the Jolo police to Kunnong’s initial reports of the disappearance, to
Honorary Counselor for the IDB who attended a seminar in Zamboanga and the responses made to the respondent when she herself reported and inquired
thereafter proceded to Jolo for an overnight stay, indicated by his request to about her husband’s disappearance, and even at Task Force Tagitis itself.
Kunnong for the purchase of a return ticket to Zamboanga the day after he x As the CA found through Task Force Tagitis, the investigation was at best
arrived in Jolo. Nothing in the records indicates the purpose of his overnight haphazard since the authorities were looking for a man whose picture they
sojourn in Jolo. A colleague in the IDB, Prof. Matli, early on informed the Jolo initially did not even secure. The returns and reports made to the CA fared no
police that Tagitis may have taken funds given to him in trust for IDB scholars. better, as the CIDG efforts themselves were confined to searching for custodial
Prof Matli later on stated that he never accused Tagitis of taking away money records of Tagitis in their various departments and divisions. To point out the
held in trust, although he confirmed that the IDB was seeking assistance in obvious, if the abduction of Tagitis was a "black" operation because it was
locating funds of IDB scholars deposited in Tagitis’ personal account. Other than unrecorded or officially unauthorized, no record of custody would ever appear in
these pieces of evidence, no other information exists in the records relating to the the CIDG records; Tagitis, too, would not be detained in the usual police or CIDG
personal circumstances of Tagitis. detention places. In sum, none of the reports on record contains any meaningful
x The actual disappearance of Tagitis is as murky as his personal circumstances. results or details on the depth and extent of the investigation made. To be sure,
While the Amparo petition recited that he was taken away by "burly men reports of top police officials indicating the personnel and units they directed to
believed to be police intelligence operatives," no evidence whatsoever was investigate can never constitute exhaustive and meaningful investigation, or
introduced to support this allegation. Thus, the available direct evidence is that equal detailed investigative reports of the activities undertaken to search for
Tagitis was last seen at 12.30 p.m. of October 30, 2007 - the day he arrived in Tagitis. Indisputably, the police authorities from the very beginning failed to
Jolo - and was never seen again. come up to the extraordinary diligence that the Amparo Rule requires.
x The Kasim evidence assumes critical materiality given the dearth of direct x Based on these considerations, we conclude that Col. Kasim’s disclosure,
evidence on the above aspects of the case, as it supplies the gaps that were made in an unguarded moment, unequivocally point to some
never looked into and clarified by police investigation. It is the evidence, too, government complicity in the disappearance. The consistent but
that colors a simple missing person report into an enforced disappearance case, unfounded denials and the haphazard investigations cannot but point to this
as it injects the element of participation by agents of the State and thus brings conclusion. For why would the government and its officials engage in
into question how the State reacted to the disappearance. their chorus of concealment if the intent had not been to deny what
x Denials on the part of the police authorities, and frustration on the part they already knew of the disappearance? Would not an in-depth and
of the respondent, characterize the attempts to locate Tagitis. Initially in thorough investigation that at least credibly determined the fate of Tagitis be a
Jolo, the police informed Kunnong that Tagitis could have been taken by the Abu feather in the government’s cap under the circumstances of the disappearance?
Sayyaf or other groups fighting the government. No evidence was ever offered From this perspective, the evidence and developments, particularly the Kasim
on whether there was active Jolo police investigation and how and why the Jolo evidence, already establish a concrete case of enforced disappearance that the
police arrived at this conclusion. The respondent’s own inquiry in Jolo yielded the Amparo Rule covers. From the prism of the UN Declaration, heretofore cited and
answer that he was not missing but was with another woman somewhere. Again, quoted,173 the evidence at hand and the developments in this case confirm the
no evidence exists that this explanation was arrived at based on an investigation. fact of the enforced disappearance and government complicity, under a
As already related above, the inquiry with Col. Ancanan in Zamboanga yielded background of consistent and unfounded government denials and haphazard
ambivalent results not useful for evidentiary purposes. Thus, it was only the handling. The disappearance as well effectively placed Tagitis outside the
inquiry from Col. Kasim that yielded positive results. Col. Kasim’s story, however, protection of the law - a situation that will subsist unless this Court acts.
confirmed only the fact of his custodial investigation (and, impliedly, his arrest or x This kind of fact situation and the conclusion reached are not without precedent
abduction), without identifying his abductor/s or the party holding him in in international enforced disappearance rulings. While the facts are not exactly the
custody. The more significant part of Col. Kasim’s story is that the abduction same, the facts of this case run very close to those of Timurtas v. Turkey,174 a
came after Tagitis was seen talking with Omar Patik and a certain Santos of case decided by ECHR. The European tribunal in that case acted on the basis of
Bulacan, a "Balik Islam" charged with terrorism. Mrs. Talbin mentioned, too, that the photocopy of a "post-operation report" in finding that Abdulvahap
Tagitis was being held at Talipapao, Sulu. None of the police agencies Timurtas (Abdulvahap) was abducted and later detained by agents (gendarmes)
participating in the investigation ever pursued these leads. Notably, Task Force of the government of Turkey. The victim's father in this case brought a claim
Tagitis to which this information was relayed did not appear to have lifted a against Turkey for numerous violations of the European Convention, including
finger to pursue these aspects of the case. the right to life (Article 2) and the rights to liberty and security of a person
x More denials were manifested in the Returns on the writ to the CA made by the (Article 5). The applicant contended that on August 14, 1993, gendarmes
petitioners. Then PNP Chief Gen. Avelino I. Razon merely reported the directives apprehended his son, Abdulvahap for being a leader of the Kurdish Workers’
he sent to the ARMM Regional Director and the Regional Chief of the CIDG on Party (PKK) in the Silopi region. The petition was filed in southeast Turkey nearly
Tagitis, and these reports merely reiterated the open-ended initial report of the six and one half years after the apprehension. According to the father,
disappearance. The CIDG directed a search in all of its divisions with negative gendarmes first detained Abdulvahap and then transferred him to another
results. These, to the PNP Chief, constituted the exhaustion "of all possible detainment facility. Although there was no eyewitness evidence of the
efforts." PNP-CIDG Chief General Edgardo M. Doromal, for his part, also reported apprehension or subsequent detainment, the applicant presented evidence
negative results after searching "all divisions and departments [of the CIDG] for corroborating his version of events, including a photocopy of a post-operation
a person named Engr. Morced N. Tagitis . . . and after a diligent and thorough report signed by the commander of gendarme operations in Silopi, Turkey. The
research, records show that no such person is being detained in the CIDG or any report included a description of Abdulvahap's arrest and the result of a
of its department or divisions." PNP-PACER Chief PS Supt. Leonardo A. Espina subsequent interrogation during detention where he was accused of being a
and PNP PRO ARMM Regional Director PC Superintendent Joel R. Goltiao did no leader of the PKK in the Silopi region. On this basis, Turkey was held responsible
better in their affidavits-returns, as they essentially reported the results of their for Abdulvahap’s enforced disappearance.
directives to their units to search for Tagitis. x Following the lead of this Turkish experience - adjusted to the Philippine legal
x The extent to which the police authorities acted was fully tested when the CA setting and the Amparo remedy this Court has established, as applied to the
constituted Task Force Tagitis, with specific directives on what to do. The unique facts and developments of this case - we believe and so hold that the
negative results reflected in the Returns on the writ were again replicated during government in general, through the PNP and the PNP-CIDG, and in particular,
the three hearings the CA scheduled. Aside from the previously mentioned the Chiefs of these organizations together with Col. Kasim, should be held fully
"retraction" that Prof. Matli made to correct his accusation that Tagitis took accountable for the enforced disappearance of Tagitis.
money held in trust for students, PS Supt. Ajirim reiterated in his testimony that x The PNP and CIDG are accountable because Section 24 of Republic Act
the CIDG consistently denied any knowledge or complicity in any abduction and No. 6975, otherwise known as the "PNP Law,"175 specifies the PNP as
said that there was no basis to conclude that the CIDG or any police unit had the governmental office with the mandate "to investigate and prevent
anything to do with the disappearance of Tagitis; he likewise considered it crimes, effect the arrest of criminal offenders, bring offenders to
premature to conclude that Tagitis simply ran away with the money in his justice and assist in their prosecution." The PNP-CIDG, as Col. Jose
custody. As already noted above, the Task Force notably did not pursue any Volpane Pante (then Chief of CIDG Region 9) testified, is the "investigative
investigation about the personal circumstances of Tagitis, his background in arm" of the PNP and is mandated to "investigate and prosecute all
relation to the IDB and the background and activities of this Bank itself, and the cases involving violations of the Revised Penal Code, particularly those
reported sighting of Tagistis with terrorists and his alleged custody in Talipapao, considered as heinous crimes."176 Under the PNP organizational structure,
Sulu. No attempt appears to have ever been made to look into the alleged IDB the PNP-CIDG is tasked to investigate all major crimes involving violations of the
funds that Tagitis held in trust, or to tap any of the "assets" who are Revised Penal Code and operates against organized crime groups, unless the
indispensable in investigations of this nature. These omissions and negative President assigns the case exclusively to the National Bureau of Investigation
results were aggravated by the CA findings that it was only as late as January (NBI).177 No indication exists in this case showing that the President ever
28, 2008 or three months after the disappearance that the police authorities directly intervened by assigning the investigation of Tagitis’ disappearance
requested for clear pictures of Tagitis. Col. Kasim could not attend the trial exclusively to the NBI.
because his subpoena was not served, despite the fact that he was designated x Given their mandates, the PNP and PNP-CIDG officials and members were
as Ajirim’s replacement in the latter’s last post. Thus, Col. Kasim was not then the ones who were remiss in their duties when the government completely
questioned. No investigation - even an internal one - appeared to have been failed to exercise the extral'>To fully enforce the Amparo remedy, we refer this
made to inquire into the identity of Col. Kasim’s "asset" and what he indeed case back to the CA for appropriate proceedings directed at the monitoring of the
wrote. PNP and the PNP-CIDG investigations and actions, and the validation of their
x We glean from all these pieces of evidence and developments a consistency in results through hearings the CA may deem appropriate to conduct. For purposes
the government’s denial of any complicity in the disappearance of Tagitis, of these investigations, the PNP/PNP-CIDG shall initially present to the CA a plan
disrupted only by the report made by Col. Kasim to the respondent at Camp of action for further investigation, periodically reporting the detailed results of its
Katitipan. Even Col. Kasim, however, eventually denied that he ever made the investigation to the CA for its consideration and action. On behalf of this Court,
176
RECTO, GAYLE ANGELI M.
2011-0008 | AUSL
Personal Notes on Remedial Law 2 Review (based on the syllabus of Prof. Henedino M. Brondial)

the CA shall pass upon: the need for the PNP and the PNP-CIDG to make the disappearance of Tagitis, the dismissive approach of the police
disclosures of matters known to them as indicated in this Decision and as further authorities to the report of the disappearance, as well as the
CA hearings may indicate; the petitioners’ submissions; the sufficiency of their haphazard investigations conducted that did not translate into any
investigative efforts; and submit to this Court a quarterly report containing its meaningful results, to be indicative of government complicity in the
actions and recommendations, copy furnished the petitioners and the disappearance of Tagitis (for purposes of the Rule on the Writ of
respondent, with the first report due at the end of the first quarter counted from Amparo).
the finality of this Decision. The PNP and the PNP-CIDG shall have one (1) full o We explained that although the Kasim evidence was patently
year to undertake their investigation. The CA shall submit its full report for the hearsay (and was thus incompetent and inadmissible under our
consideration of this Court at the end of the 4th quarter counted from the finality rules of evidence), the unique evidentiary difficulties posed by
of this Decision. enforced disappearance cases compel us to adopt standards that
x WHEREFORE, premises considered, we DENY the petitioners’ petition for review were appropriate and responsive to the evidentiary difficulties
on certiorari for lack of merit, and AFFIRM the decision of the Court of Appeals faced. We noted that while we must follow the substantial evidence
dated March 7, 2008 under the following terms: rule, we must also observe flexibility in considering the evidence
o a. Recognition that the disappearance of Engineer Morced N. that we shall take into account.
Tagitis is an enforced disappearance covered by the Rule on the x Razon et al’s contentions:
Writ of Amparo; o First, the petitioners argue that there was no sufficient evidence to
o b. Without any specific pronouncement on exact authorship and conclude that Col. Kasim’s disclosure unequivocally points to some
responsibility, declaring the government (through the PNP and the government complicity in the disappearance of Tagitis. Specifically,
PNP-CIDG) and Colonel Julasirim Ahadin Kasim accountable for the the petitioners contend that this Court erred in unduly relying on
enforced disappearance of Engineer Morced N. Tagitis; the raw information given to Col. Kasim by a personal intelligence
o c. Confirmation of the validity of the Writ of Amparo the Court of "asset" without any other evidence to support it. The petitioners
Appeals issued; also point out that the Court misapplied its cited cases (Secretary of
o d. Holding the PNP, through the PNP Chief, and the PNP-CIDG, Defense v. Manalo,7 Velasquez Rodriguez v. Honduras,8 and
through its Chief, directly responsible for the disclosure of material Timurtas v. Turkey9) to support its December 3, 2009 decision; in
facts known to the government and to their offices regarding the those cases, more than one circumstance pointed to the complicity
disappearance of Engineer Morced N. Tagitis, and for the conduct of the government and its agents. The petitioners emphasize that
of proper investigations using extraordinary diligence, with the in the present case, the respondent only presented a "token piece
obligation to show investigation results acceptable to this Court; of evidence" that points to Col. Kasim as the source of information
o e. Ordering Colonel Julasirim Ahadin Kasim impleaded in this case that Tagitis was under custodial investigation for having been
and holding him accountable with the obligation to disclose suspected as a "terrorist supporter." This, according to the
information known to him and to his "assets" in relation with the petitioners, cannot be equated to the substantial evidence required
enforced disappearance of Engineer Morced N. Tagitis; by the Rule on the Writ of Amparo.10
o f. Referring this case back to the Court of Appeals for appropriate o Second, the petitioners contend that Col. Kasim’s death renders
proceedings directed at the monitoring of the PNP and PNP-CIDG impossible compliance with the Court’s directive in its December 3,
investigations, actions and the validation of their results; the PNP 2009 decision that Col. Kasim be impleaded in the present case and
and the PNP-CIDG shall initially present to the Court of Appeals a held accountable with the obligation to disclose information known
plan of action for further investigation, periodically reporting their to him and to his "assets" on the enforced disappearance of Tagitis.
results to the Court of Appeals for consideration and action; The petitioners alleged that Col. Kasim was killed in an encounter
o g. Requiring the Court of Appeals to submit to this Court a with the Abu Sayaff Group on May 7, 2009. To prove Col. Kasim’s
quarterly report with its recommendations, copy furnished the death, the petitioners attached to their motion a copy of an article
incumbent PNP and PNP-CIDG Chiefs as petitioners and the entitled "Abus kill Sulu police director" published by the Philippine
respondent, with the first report due at the end of the first quarter Daily Inquirer on May 8, 2009.11 This article alleged that "Senior
counted from the finality of this Decision; Supt. Julasirim Kasim, his brother Rosalin, a police trainee, and two
o h. The PNP and the PNP-CIDG shall have one (1) full year to other police officers were killed in a fire fight with Abu Sayyaf
undertake their investigations; the Court of Appeals shall submit its bandits that started at about 1 p.m. on Thursday, May 7, 2009 at
full report for the consideration of this Court at the end of the 4th the boundaries of Barangays Kulasi and Bulabog in Maimbung
quarter counted from the finality of this Decision; town, Sulu." The petitioners also attached an official copy of
ƒ These directives and those of the Court of Appeals’ General Order No. 1089 dated May 15, 2009 issued by the PNP
made pursuant to this Decision shall be given to, and National Headquarters, indicating that "PS SUPT [Police Senior
shall be directly enforceable against, whoever may be Superintendent] Julasirim Ahadin Kasim 0-05530, PRO ARMM, is
the incumbent Chiefs of the Philippine National Police posthumously retired from PNP service effective May 8, 2009."12
and its Criminal Investigation and Detection Group, Additionally, the petitioners point out that the intelligence "assets"
under pain of contempt from this Court when the who supplied the information that Tagitis was under custodial
initiatives and efforts at disclosure and investigation investigation were personal to Col. Kasim; hence, the movants can
constitute less than the extraordinary diligence that no longer comply with this Court’s order to disclose any information
the Rule on the Writ of Amparo and the known to Col. Kasim and his "assets."
circumstances of this case demand. Given the unique
nature of Amparo cases and their varying attendant ISSUE # 1: Whether Col. Kasim should still be impleaded as a party to this case considering
circumstances, these directives - particularly, the his death.
referral back to and monitoring by the CA - are
specific to this case and are not standard remedies HELD # 1: NO.
that can be applied to every Amparo situation. x We hold that our directive to implead Col. Kasim as a party to the present case
ƒ The dismissal of the Amparo petition with respect to has been rendered moot and academic by his death. Nevertheless, we resolve to
General Alexander Yano, Commanding General, deny the petitioners’ motion for reconsideration for lack of merit.
Philippine Army, and General Ruben Rafael, Chief, x Paragraph (e) of the dispositive portion of our December 3, 2009 decision
Anti-Terrorism Task Force Comet, Zamboanga City, is directs:
hereby AFFIRMED. o e. Ordering Colonel Julasirim Ahadin Kasim impleaded in this case
and holding him accountable with the obligation to disclose
GEN. AVELINO I. RAZON, JR., Chief, Philippine National Police (PNP); Police Chief information known to him and to his "assets" in relation with the
Superintendent RAUL CASTAÑEDA, Chief, Criminal Investigation and Detection Group (CIDG); enforced disappearance of Engineer Morced N. Tagitis;
Police Senior Superintendent LEONARDO A. ESPINA, Chief, Police Anti-Crime and Emergency x Undisputably, this directive can no longer be enforced, and has been rendered
Response (PACER); and GEN. JOEL R. GOLTIAO, Regional Director of ARMM, PNP, Petitioners, moot and academic, given Col. Kasim's demise. His intervening death, however,
vs. MARY JEAN B. TAGITIS, herein represented by ATTY. FELIPE P. ARCILLA, JR., Attorney- does not necessarily signify the loss of the information Col. Kasim may have left
in-Fact, Respondent. behind, particularly the network of "assets" he utilized while he was in the
G.R. No. 182498 February 16, 2010 service. Intelligence gathering is not an activity conducted in isolation, and
EN BANC involves an interwoven network of informants existing on the basis of symbiotic
relationships with the police and the military. It is not farfetched that a
FACTS: resourceful investigator, utilizing the extraordinary diligence that the Rule on the
x This is an MR of the 2009 SC Decision Writ of Amparo requires,13 can still access or reconstruct the information Col.
o based, among other considerations, on the finding that Col. Kasim received from his "asset" or network of assets during his lifetime.
Julasirim Ahadin Kasim (Col. Kasim) informed the respondent Mary x The extinction of Col. Kasim’s personal accountability and obligation to disclose
Jean Tagitis (respondent) and her friends that her husband had material information, known to him and his assets, does not also erase the
been under surveillance since January 2007 because an informant burden of disclosure and investigation that rests with the PNP and the CIDG. Lest
notified the authorities, through a letter, that Tagitis was a liaison this Court be misunderstood, we reiterate that our holding in our December 3,
for the JI;5 that he was "in good hands" and under custodial 2009 Decision that the PNP -- through the incumbent PNP Chief; and the PNP-
investigation for complicity with the JI after he was seen talking to CIDG, through its incumbent Chief -- are directly responsible14 for the disclosure
one Omar Patik and a certain "Santos" of Bulacan, a "Balik Islam" of material facts known to the government and to their offices regarding the
charged with terrorism (Kasim evidence). disappearance of Tagitis; and that the conduct of proper investigation using
o We considered Col. Kasim’s information, together with the extraordinary diligence still subsists. These are continuing obligations that will
consistent denials by government authorities of any complicity in not truly be terminated until the enforced disappearance of the victim, Engr.
177
RECTO, GAYLE ANGELI M.
2011-0008 | AUSL
Personal Notes on Remedial Law 2 Review (based on the syllabus of Prof. Henedino M. Brondial)

Morced N. Tagitis, is fully addressed by the responsible or accountable parties, as conclusion that the Tagitis affair carried a "foul smell" indicative of government
we directed in our Decision. complicity or, at the very least, an attempt at cover-up and concealment. This is
the situation that the Writ of Amparo specifically seeks to address.
ISSUE # 2: Whether government complicity in the disappearance of Tagitis was sufficiently x Manalo, Velasquez Rodriguez and Timurtas, read in proper perspective, fully
shown by the Kasim evidence. support our findings and conclusions in this case.
x Manalo is different from Tagitis in terms of their factual settings, as enforced
HELD # 2: YES. disappearance was no longer a problem in that case. The enforced
x We see no merit in the petitioners’ submitted position that no sufficient evidence disappearance of the brothers Raymond and Reynaldo Manalo effectively ended
exists to support the conclusion that the Kasim evidence unequivocally points to when they escaped from captivity and surfaced, while Tagitis is still nowhere to
some government complicity in the disappearance. Contrary to the petitioners’ be found and remains missing more than two years after his reported
claim that our conclusions only relied on Col. Kasim’s report, our Decision plainly disappearance. An Amparo situation subsisted in Manalo, however, because of
and pointedly considered other evidence supporting our conclusion, particularly the continuing threat to the brothers’ right to security; the brothers claimed that
the consistent denials by government authorities of any complicity in the since the persons responsible for their enforced disappearance were still at large
disappearance of Tagitis; the dismissive approach of the police authorities to the and had not been held accountable, the former were still under the threat of
report of the disappearance; and the conduct of haphazard investigations that did being once again abducted, kept captive or even killed, which threat constituted
not translate into any meaningful results. We painstakingly ruled: a direct violation of their right to security of person. In ruling that substantial
x To give full meaning to our Constitution and the rights it protects, we hold that, evidence existed to support the conclusion that the respondents’ right to security
as in Velasquez, we should at least take a close look at the available evidence to had been violated, the Court not only considered the respondents’ affidavit and
determine the correct import of every piece of evidence - even of those usually testimony which positively identified the perpetrators, but also noted other
considered inadmissible under the general rules of evidence - taking into evidence showing the ineffective investigation and protection on the part of the
account the surrounding circumstances and the test of reason that we can use as military. The Court significantly found that:
basic minimum admissibility requirement. In the present case, we should at least o Next, the violation of the right to security as protection by the
determine whether the Kasim evidence before us is relevant and meaningful to government. Apart from the failure of military elements to provide
the disappearance of Tagistis and reasonably consistent with other evidence in protection to respondents by themselves perpetrating the
the case. abduction, detention, and torture, they also miserably failed in
x The Kasim evidence assumes critical materiality given the dearth of direct conducting an effective investigation of respondents’ abduction as
evidence on the above aspects of the case, as it supplies the gaps that were revealed by the testimony and investigation report of petitioners’
never looked into and clarified by police investigation. It is the evidence, too, own witness, Lt. Col. Ruben Jimenez, Provost Marshall of the 7th
that colors a simple missing person report into an enforced disappearance case, Infantry Division.
as it injects the element of participation by agents of the State and thus brings o The one-day investigation conducted by Jimenez was very limited,
into question how the State reacted to the disappearance. superficial, and one-sided. He merely relied on the Sworn
x We glean from all these pieces of evidence and developments a Statements of the six implicated members of the CAFGU and
consistency in the government’s denial of any complicity in the civilians whom he met in the investigation for the first time. He was
disappearance of Tagitis, disrupted only by the report made by Col. present at the investigation when his subordinate Lingad was
Kasim to the respondent at Camp Katitipan. Even Col. Kasim, however, taking the sworn statements, but he did not propound a single
eventually denied that he ever made the disclosure that Tagitis was under question to ascertain the veracity of their statements or their
custodial investigation for complicity in terrorism. Another distinctive trait that credibility. He did not call for other witnesses to test the alibis given
runs through these developments is the government’s dismissive approach by the six implicated persons nor for the family or neighbors of the
to the disappearance, starting from the initial response by the Jolo respondents.
police to Kunnong’s initial reports of the disappearance, to the o In his affidavit, petitioner Secretary of National Defense attested
responses made to the respondent when she herself reported and that in a Memorandum Directive dated October 31, 2007, he issued
inquired about her husband’s disappearance, and even at Task Force a policy directive addressed to the AFP Chief of Staff, that the AFP
Tagitis itself. should adopt rules of action in the event the writ of amparo is
x As the CA found through Task Force Tagitis, the investigation was at best issued by a competent court against any members of the AFP,
haphazard since the authorities were looking for a man whose picture which should essentially include verification of the identity of the
they initially did not even secure. The returns and reports made to the CA aggrieved party; recovery and preservation of relevant evidence;
fared no better, as the CIDG efforts themselves were confined to identification of witnesses and securing statements from them;
searching for custodial records of Tagitis in their various departments determination of the cause, manner, location and time of death or
and divisions. To point out the obvious, if the abduction of Tagitis was a disappearance; identification and apprehension of the person or
"black" operation because it was unrecorded or officially unauthorized, no record persons involved in the death or disappearance; and bringing of the
of custody would ever appear in the CIDG records; Tagitis, too, would not be suspected offenders before a competent court. Petitioner AFP Chief
detained in the usual police or CIDG detention places. In sum, none of the of Staff also submitted his own affidavit attesting that he received
reports on record contains any meaningful results or details on the depth and the above directive of respondent Secretary of National Defense
extent of the investigation made. To be sure, reports of top police officials and that acting on this directive, he immediately caused to be
indicating the personnel and units they directed to investigate can never issued a directive to the units of the AFP for the purpose of
constitute exhaustive and meaningful investigation, or equal detailed establishing the circumstances of the alleged disappearance and
investigative reports of the activities undertaken to search for Tagitis. the recent reappearance of the respondents, and undertook to
Indisputably, the police authorities from the very beginning failed to come up to provide results of the investigations to respondents. To this day,
the extraordinary diligence that the Amparo Rule requires. however, almost a year after the policy directive was issued by
x Likewise, we see no merit in the petitioners’ claim that the Kasim evidence does petitioner Secretary of National Defense on October 31, 2007,
not amount to substantial evidence required by the Rule on the Writ of Amparo. respondents have not been furnished the results of the
This is not a new issue; we extensively and thoroughly considered and resolved it investigation which they now seek through the instant petition for a
in our December 3, 2009 Decision. At this point, we need not go into another full writ of amparo.
discussion of the justifications supporting an evidentiary standard specific to the o Under these circumstances, there is substantial evidence to warrant
Writ of Amparo. Suffice it to say that we continue to adhere to the the conclusion that there is a violation of respondents’ right to
substantial evidence rule that the Rule on the Writ of Amparo requires, with security as a guarantee of protection by the government.
some adjustments for flexibility in considering the evidence presented. When we [Emphasis supplied]18
ruled that hearsay evidence (usually considered inadmissible under the general x Similarly in Velasquez Rodriguez, the Inter-American Court of Human Rights
rules of evidence) may be admitted as the circumstances of the case may (IACHR) acknowledged that when the Honduran Government carried out or
require, we did not thereby dispense with the substantial evidence rule; we tolerated enforced disappearances, the police customarily used a distinctive form
merely relaxed the evidentiary rule on the admissibility of evidence, maintaining of kidnapping. Consequently, the IACHR presumed that Velasquez disappeared at
all the time the standards of reason and relevance that underlie every evidentiary the "hands of or with the acquiescence of those officials within the framework of
situation. This, we did, by considering the totality of the obtaining situation and that practice." Moreover, the IACHR found that negative inferences may be drawn
the consistency of the hearsay evidence with the other available evidence in the from the fact that the government failed to investigate or to inquire into his
case. disappearance, and thwarted the attempts by the victim’s family to do so; these
x We also cannot agree with the petitioners’ contention that we misapplied according to the Court strongly suggested the government’s involvement in the
Secretary of Defense v. Manalo,15 Velasquez Rodriguez v. Honduras,16 and disappearance, even if there was no direct evidence indicating that the
Timurtas v. Turkey17 to support our December 3, 2009 decision. The petitioners government kidnapped Velasquez.19 The Court thus held:
make this claim with the view that in these cases, more than one circumstance o iii. In the case of Manfredo Velásquez, there were the same type of
pointed to the government or its agents as the parties responsible for the denials by his captors and the Armed Forces, the same omissions of
disappearance, while we can only point to the Kasim evidence. A close reading of the latter and of the Government in investigating and revealing his
our December 3, 2009 Decision shows that it rests on more than one basis. whereabouts, and the same ineffectiveness of the courts where
x At the risk of repetition, we stress that other pieces of evidence point the way three writs of HABEAS corpus and two criminal complaints were
towards our conclusion, particularly the unfounded and consistent denials by brought ( testimony of Miguel Angel Pavón Salazar, Ramón
government authorities of any complicity in the disappearance; the dismissive Custodio López, Zenaida Velásquez, press clippings and
approach of the police to the report of the disappearance; and the haphazard documentary evidence ).
handling of the investigation that did not produce any meaningful results. In o h. There is no evidence in the record that Manfredo Velásquez had
cruder but more understandable language, the run-around given to the disappeared in order to join subversive groups, other than a letter
respondent and the government responses to the request for meaningful from the Mayor of Langue, which contained rumors to that effect.
investigation, considered in the light of the Kasim evidence, pointed to the The letter itself shows that the Government associated him with
178
RECTO, GAYLE ANGELI M.
2011-0008 | AUSL
Personal Notes on Remedial Law 2 Review (based on the syllabus of Prof. Henedino M. Brondial)

activities it considered a threat to national security. However, the x She was finally released after 6 days BUT she was given a cellphone with a SIM
Government did not corroborate the view expressed in the letter card, among others, with which she was contacted thereafter by her abductors
with any other evidence. Nor is there any evidence that he was x She was also threatened against informing the group KARAPATAN of her
kidnapped by common criminals or other persons unrelated to the abduction and torture
practice of disappearances existing at that time." x Melissa Æ then filed an Amparo and Habeas Data petition before the CA with the
o 148. Based upon the above, the Court finds that the following facts ff prayers:
have been proven in this proceeding: (1) a practice of o (1) respondents be enjoined from harming or even approaching
disappearances carried out or tolerated by Honduran officials petitioner and her family;
existed between 1981 and 1984; ( 2) Manfredo Velásquez o (2) an order be issued allowing the inspection of detention areas in
disappeared at the hands of or with the acquiescence of those the 7th Infantry Division, Fort Magsaysay, Laur, Nueva Ecija;
officials within the framework of that practice; and (3) the o (3) respondents be ordered to produce documents relating to any
Government of Honduras failed to guarantee the human rights report on the case of petitioner including, but not limited to,
affected by that practice. intelligence report and operation reports of the 7th Infantry
x Finally, in Timurtas, the European Court of Human Rights (ECHR) altered the Division, the Special Operations Group of the Armed Forces of the
prevailing jurisprudence by permitting a lesser evidentiary burden in cases of Philippines (AFP) and its subsidiaries or branch/es prior to, during
enforced disappearances. The ECHR dismissed the need for direct evidence and subsequent to 19 May 2009;
previously held necessary in the leading case of Kurt v. Turkey,21 and instead o (4) respondents be ordered to expunge from the records of the
permitted the use of circumstantial evidence to establish a violation of the right respondents any document pertinent or connected to Melissa C.
to life. It stated that "whether the failure on the part of authorities to provide a Roxas, Melissa Roxas or any name which sounds the same; and
plausible explanation as to a detainee’s fate, in the absence of a body, might o (5) respondents be ordered to return to petitioner her journal,
raise issues under Article 2 of the Convention (right to life), will depend on the digital camera with memory card, laptop computer, external hard
circumstances of the case and, in particular, on the existence of sufficient disk, IPOD, wristwatch, sphygmomanometer, stethoscope,
circumstantial evidence based on concrete elements, from which it may be medicines and her P15,000.00 cash
concluded to the requisite standard of proof that the detainee must be presumed x SolGen Æ filed a Return for the respondents; prayed for DISMISSAL of the
to have died in custody."22 The ECHR found that: petition
o Noting that more than six and a half years has gone by since o (a) as against respondent President Gloria Macapagal-Arroyo, in
Abdulvahap Timurtas’ apprehension and having regard to all the particular, because of her immunity from suit,52 and
other circumstances of the case, the Court found that the o (b) as against all of the public respondents, in general, in view of
disappearance of Abdulvahap Timurtas after he had been taken the absence of any specific allegation in the petition that they had
into detention led, in the circumstances of this case, to a participated in, or at least authorized, the commission of such
presumption that he had died. No explanation having been atrocities
provided by the Government as to what had happened to him x CA Æ partly granted the petition
during his detention, the Government was liable for his death and o gave due weight and consideration to the petitioner’s version that
there was a violation of Article 2 of the Convention. [Emphasis she was indeed abducted and then subjected to torture for five (5)
supplied] straight days; CA believed that Melissa was telling the truth
o Significantly (in the context of the present case), the ECHR also o BUT was not convinced that the military or any other person acting
noted that the inadequacy of the investigation into the under the acquiescence of the government, were responsible for
disappearance of Timurtas also constituted a violation of his right the abduction and torture of the petitioner.89 The appellate court
to life under Article 2 of the European Convention on Human stressed that, judging by her own statements, the petitioner merely
Rights. "believed" that the military was behind her abduction.90 Thus, the
x Thus viewed, common threads that plainly run in the three cited cases Court of Appeals absolved the public respondents from any
are applicable to the present case. There is the evidence of ineffective complicity in the abduction and torture of petitioner.91 The petition
investigation in Manalo and Velasquez Rodriguez, while in all three was the was likewise dismissed as against public respondent President
recognition that the burden of proof must be lowered or relaxed (either Gloria Macapagal-Arroyo, in view of her immunity from suit.
through the use of circumstantial or indirect evidence or even by o petitioner’s prayers for the return of her personal belongings were
logical inference); the requirement for direct evidence to establish that an denied.93 Petitioner’s prayers for an inspection order and
enforced disappearance occurred -- as the petitioners effectively suggest -- production order also met the same fate
would render it extremely difficult, if not impossible, to prove that an x Melissa Æ filed a Rule 45 before the SC
individual has been made to disappear. In these lights, we emphasized in
our December 3, 2009 Decision that while the need for substantial ISSUE # 1: Whether then President GMA was correctly impleaded in the petition for amparo
evidence remains the rule, flexibility must be observed where and habeas data pursuant to the doctrine of command responsibility.
appropriate (as the Courts in Velasquez Rodriguez and Timurtas did)
for the protection of the precious rights to life, liberty and security. This HELD # 1: NO.
flexibility, we noted, requires that "we should take a close look at the x It must be stated at the outset that the use by the petitioner of the
available evidence to determine the correct import of every piece of doctrine of command responsibility as the justification in impleading the
evidence - even of those usually considered inadmissible under the public respondents in her amparo petition, is legally inaccurate, if not
general rules of evidence - taking into account the surrounding incorrect. The doctrine of command responsibility is a rule of substantive law
circumstances and the test of reason that we can use as basic that establishes liability and, by this account, cannot be a proper legal
minimum admissibility requirement." From these perspectives, we see no basis to implead a party-respondent in an amparo petition.100
error that we should rectify or reconsider. x The case of Rubrico v. Arroyo,101 which was the first to examine command
responsibility in the context of an amparo proceeding, observed that the doctrine
IN THE MATTER OF THE PETITION FOR THE WRIT OF AMPARO AND THE WRIT OF HABEAS is used to pinpoint liability. Rubrico notes that:102
DATA IN FAVOR OF MELISSA C. ROXAS, MELISSA C. ROXAS, Petitioner, vs. GLORIA o The evolution of the command responsibility doctrine finds its
MACAPAGAL-ARROYO, GILBERT TEODORO, GEN. VICTOR S. IBRADO, P/DIR. GEN. JESUS context in the development of laws of war and armed combats.
AME VERZOSA, LT. GEN. DELFIN N. BANGIT, PC/SUPT. LEON NILO A. DELA CRUZ, MAJ. GEN. According to Fr. Bernas, "command responsibility," in its simplest
RALPH VILLANUEVA, PS/SUPT. RUDY GAMIDO LACADIN, AND CERTAIN PERSONS WHO GO BY terms, means the "responsibility of commanders for crimes
THE NAME[S] DEX, RC AND ROSE, Respondents. committed by subordinate members of the armed forces or
G.R. No. 189155 September 7, 2010 other persons subject to their control in international wars
EN BANC or domestic conflict."103 In this sense, command responsibility is
properly a form of criminal complicity. The Hague
FACTS: Conventions of 1907 adopted the doctrine of command
x Melissa is an American citizen with Filipino descent. responsibility,104 foreshadowing the present-day precept of
x She enrolled in an exposure program of Bagong Alyansang Makabayan-United holding a superior accountable for the atrocities committed by his
States of America (BAYAN-USA) while she was still in the US subordinates should he be remiss in his duty of control over them.
x As such, she volunteered to join members of BAYAN-Tarlac6 in conducting an As then formulated, command responsibility is "an omission mode
initial health survey in La Paz, Tarlac for a future medical mission of individual criminal liability," whereby the superior is made
x While on mission, Melissa and her companions decided to rest in the house of responsible for crimes committed by his subordinates for failing to
one Paolo prevent or punish the perpetrators105 (as opposed to crimes he
x Suddenly, fifteen (15) heavily armed men forcibly opened the door, barged ordered). (Emphasis in the orginal, underscoring supplied)
inside and ordered petitioner and her companions to lie on the ground face x Since the application of command responsibility presupposes an
down.12 The armed men were all in civilian clothes and, with the exception of imputation of individual liability, it is more aptly invoked in a full-blown
their leader, were also wearing bonnets to conceal their faces criminal or administrative case rather than in a summary amparo
x Before being blindfolded, Melissa was able to see the faces of these men proceeding. The obvious reason lies in the nature of the writ itself:
x They were then taken to another place via a van by her abductors who o The writ of amparo is a protective remedy aimed at providing
introduced themselves as "Dex," "James" and "RC." judicial relief consisting of the appropriate remedial measures and
x Melissa was brought to a cell and was separated from her 2 companions directives that may be crafted by the court, in order to address
x She was then informed that she was being detained because of her alleged specific violations or threats of violation of the constitutional rights
affiliation with the CPP-NPA to life, liberty or security.106 While the principal objective of its
x She was tortured while in detention proceedings is the initial determination of whether an enforced
disappearance, extralegal killing or threats thereof had transpired—
179
RECTO, GAYLE ANGELI M.
2011-0008 | AUSL
Personal Notes on Remedial Law 2 Review (based on the syllabus of Prof. Henedino M. Brondial)

the writ does not, by so doing, fix liability for such disappearance, she managed to see. To the mind of this Court, these
killing or threats, whether that may be criminal, civil or cartographic sketches have the undeniable potential
administrative under the applicable substantive law.107 The of giving the greatest certainty as to the true identity
rationale underpinning this peculiar nature of an amparo writ has and affiliation of petitioner’s abductors. Unfortunately
been, in turn, clearly set forth in the landmark case of The for the petitioner, this potential has not been realized
Secretary of National Defense v. Manalo:108 in view of the fact that the faces described in such
ƒ x x x The remedy provides rapid judicial relief as it sketches remain unidentified, much less have been
partakes of a summary proceeding that requires only shown to be that of any military or police personnel.
substantial evidence to make the appropriate reliefs Bluntly stated, the abductors were not proven to be
available to the petitioner; it is not an action to part of either the military or the police chain of
determine criminal guilt requiring proof beyond command.
reasonable doubt, or liability for damages requiring o Second. The claim of the petitioner that she was taken to Fort
preponderance of evidence, or administrative Magsaysay was not adequately established by her mere estimate of
responsibility requiring substantial evidence that will the time it took to reach the place where she was detained and by
require full and exhaustive proceedings.109(Emphasis the sounds that she heard while thereat. Like the Court of Appeals,
supplied) We are not inclined to take the estimate and observations of the
x It must be clarified, however, that the inapplicability of the doctrine of petitioner as accurate on its face—not only because they were
command responsibility in an amparo proceeding does not, by any made mostly while she was in blindfolds, but also in view of the
measure, preclude impleading military or police commanders on the fact that she was a mere sojourner in the Philippines, whose
ground that the complained acts in the petition were committed with familiarity with Fort Magsaysay and the travel time required to
their direct or indirect acquiescence. In which case, commanders may reach it is in itself doubtful.116 With nothing else but obscure
be impleaded—not actually on the basis of command responsibility— observations to support it, petitioner’s claim that she was taken to
but rather on the ground of their responsibility, or at least Fort Magsaysay remains a mere speculation.
accountability. In Razon v. Tagitis,110 the distinct, but interrelated concepts of x In sum, the petitioner was not able to establish to a concrete point that her
responsibility and accountability were given special and unique significations in abductors were actually affiliated, whether formally or informally, with the
relation to an amparo proceeding, to wit: military or the police organizations. Neither does the evidence at hand prove that
o x x x Responsibility refers to the extent the actors have been petitioner was indeed taken to the military camp Fort Magsaysay to the exclusion
established by substantial evidence to have participated in of other places. These evidentiary gaps, in turn, make it virtually impossible to
whatever way, by action or omission, in an enforced determine whether the abduction and torture of the petitioner was in fact
disappearance, as a measure of the remedies this Court shall craft, committed with the acquiescence of the public respondents. On account of this
among them, the directive to file the appropriate criminal and civil insufficiency in evidence, a pronouncement of responsibility on the part of the
cases against the responsible parties in the proper courts. public respondents, therefore, cannot be made.
Accountability, on the other hand, refers to the measure of
remedies that should be addressed to those who exhibited ISSUE # 3: Whether the return of Melissa’s personal belongings was proper in a petition for
involvement in the enforced disappearance without bringing the amparo and habeas data.
level of their complicity to the level of responsibility defined above;
or who are imputed with knowledge relating to the enforced HELD # 3: NO.
disappearance and who carry the burden of disclosure; or those x In its decision, the Court of Appeals denied the above prayer of the petitioner by
who carry, but have failed to discharge, the burden of reason of the failure of the latter to prove that the public respondents were
extraordinary diligence in the investigation of the enforced involved in her abduction and torture.117 We agree with the conclusion of the
disappearance. Court of Appeals, but not entirely with the reason used to support it. To the mind
of this Court, the prayer of the petitioner for the return of her belongings is
ISSUE # 2: Whether there was sufficient evidence in impleading the herein public doomed to fail regardless of whether there is sufficient evidence to hold public
respondents. respondents responsible for the abduction of the petitioner.
x In the first place, an order directing the public respondents to return
HELD # 2: NO. the personal belongings of the petitioner is already equivalent to a
x At any rate, it is clear from the records of the case that the intent of the conclusive pronouncement of liability. The order itself is a substantial relief
petitioner in impleading the public respondents is to ascribe some form of that can only be granted once the liability of the public respondents has been
responsibility on their part, based on her assumption that they, in one way or the fixed in a full and exhaustive proceeding. As already discussed above, matters
other, had condoned her abduction and torture.111 of liability are not determinable in a mere summary amparo
x To establish such assumption, petitioner attempted to show that it was proceeding.118
government agents who were behind her ordeal. Thus, the petitioner calls x But perhaps the more fundamental reason in denying the prayer of the
attention to the circumstances surrounding her abduction and torture—i.e., the petitioner, lies with the fact that a person’s right to be restituted of his
forcible taking in broad daylight; use of vehicles with no license plates; utilization property is already subsumed under the general rubric of property
of blindfolds; conducting interrogations to elicit communist inclinations; and the rights—which are no longer protected by the writ of amparo.119 Section
infliction of physical abuse—which, according to her, is consistent with the way 1 of the Amparo Rule,120 which defines the scope and extent of the writ, clearly
enforced disappearances are being practiced by the military or other state excludes the protection of property rights.
forces.112
x Moreover, petitioner also claims that she was held inside the military camp Fort ISSUE # 4: Whether the inspection of the detention areas of Fort Magsaysay may properly
Magsaysay—a conclusion that she was able to infer from the travel time required ordered in the present case.
to reach the place where she was actually detained, and also from the sounds of
construction, gun-fire and airplanes she heard while thereat.113 HELD # 4: NO.
o We are not impressed. x Considering the dearth of evidence concretely pointing to any military
x The totality of the evidence presented by the petitioner does not inspire involvement in petitioner’s ordeal, this Court finds no error on the part of the
reasonable conclusion that her abductors were military or police personnel and Court of Appeals in denying an inspection of the military camp at Fort
that she was detained at Fort Magsaysay. Magsaysay. We agree with the appellate court that a contrary stance would be
o First. The similarity between the circumstances attending a equivalent to sanctioning a "fishing expedition," which was never intended by the
particular case of abduction with those surrounding previous Amparo Rule in providing for the interim relief of inspection order.122 Contrary to
instances of enforced disappearances does not, necessarily, carry the explicit position123 espoused by the petitioner, the Amparo Rule does not
sufficient weight to prove that the government orchestrated such allow a "fishing expedition" for evidence.
abduction. We opine that insofar as the present case is concerned, x An inspection order is an interim relief designed to give support or strengthen
the perceived similarity cannot stand as substantial evidence of the the claim of a petitioner in an amparo petition, in order to aid the court before
involvement of the government. making a decision.124 A basic requirement before an amparo court may
ƒ In amparo proceedings, the weight that may be grant an inspection order is that the place to be inspected is reasonably
accorded to parallel circumstances as evidence of determinable from the allegations of the party seeking the order. While
military involvement depends largely on the the Amparo Rule does not require that the place to be inspected be identified
availability or non-availability of other pieces of with clarity and precision, it is, nevertheless, a minimum for the issuance of
evidence that has the potential of directly proving the an inspection order that the supporting allegations of a party be
identity and affiliation of the perpetrators. Direct sufficient in itself, so as to make a prima facie case. This, as was shown
evidence of identity, when obtainable, must be above, petitioner failed to do.
preferred over mere circumstantial evidence based on x Since the very estimates and observations of the petitioner are not strong
patterns and similarity, because the former enough to make out a prima facie case that she was detained in Fort Magsaysay,
indubitably offers greater certainty as to the true an inspection of the military camp cannot be ordered. An inspection order cannot
identity and affiliation of the perpetrators. An amparo issue on the basis of allegations that are, in themselves, unreliable and doubtful.
court cannot simply leave to remote and hazy
inference what it could otherwise clearly and directly
ascertain.
ƒ In the case at bench, petitioner was, in fact, able to
include in her Offer of Exhibits,114 the cartographic
sketches115 of several of her abductors whose faces
180
RECTO, GAYLE ANGELI M.
2011-0008 | AUSL
Personal Notes on Remedial Law 2 Review (based on the syllabus of Prof. Henedino M. Brondial)

appropriate legal measures for the protection of human rights,"128 must be


tapped in order to fill certain investigative and remedial voids.
x Ironic as it seems, but part and parcel of the reason why the petitioner was not
ISSUE # 5: Whether the privilege of the writ of habeas data should be granted. able to adduce substantial evidence proving her allegations of government
complicity in her abduction and torture, may be attributed to the incomplete and
HELD # 5: NO. one-sided investigations conducted by the government itself. This "awkward"
x As earlier intimated, the Court of Appeals granted to the petitioner the privilege situation, wherein the very persons alleged to be involved in an enforced
of the writ of habeas data, by enjoining the public respondents from "distributing disappearance or extralegal killing are, at the same time, the very ones tasked by
or causing the distribution to the public any records in whatever form, reports, law to investigate the matter, is a unique characteristic of these proceedings and
documents or similar papers" relative to the petitioner’s "alleged ties with the is the main source of the "evidentiary difficulties" faced by any petitioner in any
CPP-NPA or pertinently related to her abduction and torture." Though not raised amparo case.129
as an issue in this appeal, this Court is constrained to pass upon and review this x Cognizant of this situation, however, the Amparo Rule placed a potent
particular ruling of the Court of Appeals in order to rectify, what appears to Us, safeguard—requiring the "respondent who is a public official or employee" to
an error infecting the grant prove that no less than "extraordinary diligence as required by applicable laws,
x For the proper appreciation of the rationale used by the Court of Appeals in rules and regulations was observed in the performance of duty."130 Thus, unless
granting the privilege of the writ of habeas data, We quote hereunder the and until any of the public respondents is able to show to the satisfaction of the
relevant portion125 of its decision: amparo court that extraordinary diligence has been observed in their
o Under these premises, Petitioner prayed that all the records, investigations, they cannot shed the allegations of responsibility despite the
intelligence reports and reports on the investigations conducted on prevailing scarcity of evidence to that effect.
Melissa C. Roxas or Melissa Roxas be produced and eventually x With this in mind, We note that extraordinary diligence, as required by the
expunged from the records. Petitioner claimed to be included in the Amparo Rule, was not fully observed in the conduct of the police and military
Government’s Order of Battle under Oplan Bantay Laya which listed investigations in the case at bar.
political opponents against whom false criminal charges were filed x A perusal of the investigation reports submitted by Task Group CAROJAN shows
based on made up and perjured information. modest effort on the part of the police investigators to identify the perpetrators
x Pending resolution of this petition and before Petitioner could testify before Us, of the abduction. To be sure, said reports are replete with background checks on
Ex-army general Jovito Palaparan, Bantay party-list, and Pastor Alcover of the the victims of the abduction, but are, at the same time, comparatively silent as
Alliance for Nationalism and Democracy party-list held a press conference where to other concrete steps the investigators have been taking to ascertain the
they revealed that they received an information from a female NPA rebel who authors of the crime. Although conducting a background investigation on the
wanted out of the organization, that Petitioner was a communist rebel. Alcover victims is a logical first step in exposing the motive behind the abduction—its
claimed that said information reached them thru a letter with photo of Petitioner necessity is clearly outweighed by the need to identify the perpetrators,
holding firearms at an NPA training camp and a video CD of the training especially in light of the fact that the petitioner, who was no longer in captivity,
exercises. already came up with allegations about the motive of her captors.
x Clearly, and notwithstanding Petitioner’s denial that she was the person in said x Instead, Task Group CAROJAN placed the fate of their investigations solely on
video, there were records of other investigations on Melissa C. Roxas or Melissa the cooperation or non-cooperation of the petitioner—who, they claim, was less
Roxas which violate her right to privacy. Without a doubt, reports of such than enthusiastic in participating in their investigative efforts.131 While it may be
nature have reasonable connections, one way or another, to conceded that the participation of the petitioner would have facilitated the
petitioner’s abduction where she claimed she had been subjected to progress of Task Group CAROJAN’s investigation, this Court believes that the
cruelties and dehumanizing acts which nearly caused her life precisely former’s reticence to cooperate is hardly an excuse for Task Group CAROJAN not
due to allegation of her alleged membership in the CPP-NPA. And if said to explore other means or avenues from which they could obtain relevant
report or similar reports are to be continuously made available to the public, leads.132 Indeed, while the allegations of government complicity by the
Petitioner’s security and privacy will certainly be in danger of being violated or petitioner cannot, by themselves, hold up as adequate evidence before a court of
transgressed by persons who have strong sentiments or aversion against law—they are, nonetheless, a vital source of valuable investigative leads that
members of this group. The unregulated dissemination of said unverified video must be pursued and verified, if only to comply with the high standard of
CD or reports of Petitioner’s alleged ties with the CPP-NPA indiscriminately made diligence required by the Amparo Rule in the conduct of investigations.
available for public consumption without evidence of its authenticity or veracity x Assuming the non-cooperation of the petitioner, Task Group CAROJAN’s reports
certainly violates Petitioner’s right to privacy which must be protected by this still failed to explain why it never considered seeking the assistance of Mr. Jesus
Court. We, thus, deem it necessary to grant Petitioner the privilege of the Writ of Paolo—who, along with the victims, is a central witness to the abduction. The
Habeas Data. (Emphasis supplied). reports of Task Group CAROJAN is silent in any attempt to obtain from Mr. Paolo, a
x The writ of habeas data was conceptualized as a judicial remedy enforcing the cartographic sketch of the abductors or, at the very least, of the one who, by
right to privacy, most especially the right to informational privacy of petitioner’s account, was not wearing any mask.1avvphi1
individuals.126 The writ operates to protect a person’s right to control x The recollection of Mr. Paolo could have served as a comparative material to the
information regarding himself, particularly in the instances where such sketches included in petitioner’s offer of exhibits that, it may be pointed out,
information is being collected through unlawful means in order to achieve were prepared under the direction of, and first submitted to, the CHR pursuant
unlawful ends. to the latter’s independent investigation on the abduction and torture of the
x Needless to state, an indispensable requirement before the privilege of the writ petitioner.133 But as mentioned earlier, the CHR sketches remain to be
may be extended is the showing, at least by substantial evidence, of an unidentified as of this date.
actual or threatened violation of the right to privacy in life, liberty or x In light of these considerations, We agree with the Court of Appeals that further
security of the victim.127 This, in the case at bench, the petitioner investigation under the norm of extraordinary diligence should be undertaken.
failed to do. This Court simply cannot write finis to this case, on the basis of an incomplete
x The main problem behind the ruling of the Court of Appeals is that there is investigation conducted by the police and the military. In a very real sense, the
actually no evidence on record that shows that any of the public respondents right to security of the petitioner is continuously put in jeopardy because of the
had violated or threatened the right to privacy of the petitioner. The act ascribed deficient investigation that directly contributes to the delay in bringing the real
by the Court of Appeals to the public respondents that would have violated or perpetrators before the bar of justice.
threatened the right to privacy of the petitioner, i.e., keeping records of x To add teeth to the appellate court’s directive, however, We find it fitting, nay,
investigations and other reports about the petitioner’s ties with the CPP-NPA, necessary to shift the primary task of conducting further investigations on the
was not adequately proven—considering that the origin of such records were abduction and torture of the petitioner upon the CHR.134 We note that the CHR,
virtually unexplained and its existence, clearly, only inferred by the appellate unlike the police or the military, seems to enjoy the trust and confidence of the
court from the video and photograph released by Representatives Palparan and petitioner—as evidenced by her attendance and participation in the hearings
Alcover in their press conference. No evidence on record even shows that any of already conducted by the commission.135 Certainly, it would be reasonable to
the public respondents had access to such video or photograph. assume from such cooperation that the investigations of the CHR have
x In view of the above considerations, the directive by the Court of Appeals advanced, or at the very least, bears the most promise of advancing farther, in
enjoining the public respondents from "distributing or causing the distribution to terms of locating the perpetrators of the abduction, and is thus, vital for a final
the public any records in whatever form, reports, documents or similar papers" resolution of this petition. From this perspective, We also deem it just and
relative to the petitioner’s "alleged ties with the CPP-NPA," appears to be devoid appropriate to relegate the task of affording interim protection to the petitioner,
of any legal basis. The public respondents cannot be ordered to refrain from also to the CHR.
distributing something that, in the first place, it was not proven to have. x x Hence, We modify the directive of the Court of the Appeals for further
Verily, until such time that any of the public respondents were found to be investigation, as follows—
actually responsible for the abduction and torture of the petitioner, any inference o 1.) Appointing the CHR as the lead agency tasked with conducting
regarding the existence of reports being kept in violation of the petitioner’s right further investigation regarding the abduction and torture of the
to privacy becomes farfetched, and premature. petitioner. Accordingly, the CHR shall, under the norm of
x For these reasons, this Court must, at least in the meantime, strike down the extraordinary diligence, take or continue to take the necessary
grant of the privilege of the writ of habeas data. steps: (a) to identify the persons described in the cartographic
x Our review of the evidence of the petitioner, while telling of its innate sketches submitted by the petitioner, as well as their whereabouts;
insufficiency to impute any form of responsibility on the part of the public and (b) to pursue any other leads relevant to petitioner’s abduction
respondents, revealed two important things that can guide Us to a proper and torture.
disposition of this case. One, that further investigation with the use of o 2.) Directing the incumbent Chief of the Philippine National Police
extraordinary diligence must be made in order to identify the perpetrators behind (PNP), or his successor, and the incumbent Chief of Staff of the
the abduction and torture of the petitioner; and two, that the Commission on AFP, or his successor, to extend assistance to the ongoing
Human Rights (CHR), pursuant to its Constitutional mandate to "investigate all investigation of the CHR, including but not limited to furnishing the
forms of human rights violations involving civil and political rights and to provide latter a copy of its personnel records circa the time of the
181
RECTO, GAYLE ANGELI M.
2011-0008 | AUSL
Personal Notes on Remedial Law 2 Review (based on the syllabus of Prof. Henedino M. Brondial)

petitioner’s abduction and torture, subject to reasonable regulations military or the police or are civilians; if they are civilians, whether
consistent with the Constitution and existing laws. they acted on their own or were following orders, and in the latter
o 3.) Further directing the incumbent Chief of the PNP, or his case, from whom
successor, to furnish to this Court, the Court of Appeals, and the o since the petitioner has established that the vehicle used in the
petitioner or her representative, a copy of the reports of its abduction was linked to a vehicle (with license plate number TAB
investigations and their recommendations, other than those that are 194) impounded at the headquarters of the 56th IB, it became the
already part of the records of this case, within ninety (90) days from burden of the AFP to exercise extraordinary diligence to determine
receipt of this decision. the why and the wherefore of the loss of the license plate in their
o 4.) Further directing the CHR to (a) furnish to the Court of Appeals custody and its appearance in a vehicle (a maroon Toyota Revo)
within ninety (90) days from receipt of this decision, a copy of the used in Jonas’ abduction
reports on its investigation and its corresponding o branded PNP-CIDG’s investigation as "rather shallow" and
recommendations; and to (b) provide or continue to provide "conducted haphazardly."
protection to the petitioner during her stay or visit to the o the petitions for habeas corpus and contempt as against President
Philippines, until such time as may hereinafter be determined by Gloria Macapagal-Arroyo must be dropped since she enjoys the
this Court. privilege of immunity from suit
ƒ Accordingly, this case must be referred back to the x Edita Æ filed a Rule 45 before the SC
Court of Appeals, for the purposes of monitoring x SCÆ rendered a decision (2010), among others:
compliance with the above directives and determining o DIRECT the Commission on Human Rights to conduct appropriate
whether, in light of any recent reports or investigative proceedings, including field investigations - acting as
recommendations, there would already be sufficient the Court’s directly commissioned agency for purposes of the Rule
evidence to hold any of the public respondents on the Writ of Amparo - with the tasks of: (a) ascertaining the
responsible or, at least, accountable. After making identities of the cartographic sketches of two of the abductors as
such determination, the Court of Appeals shall submit well as their whereabouts; (b) determining based on records, past
its own report with recommendation to this Court for and present, the identities and locations of the persons identified
final action. The Court of Appeals will continue to by State Prosecutor Velasco alleged to be involved in the abduction
have jurisdiction over this case in order to accomplish of Jonas namely: T/Sgt. Jason Roxas (Philippine Army), Cpl. Maria
its tasks under this decision. Joana Francisco (Philippine Air Force), M/Sgt. Aron Arroyo
(Philippine Air Force), and an alias T.L., all reportedly assigned with
Military Intelligence Group 15 of Intelligence Service of the Armed
EDITA T. BURGOS, Petitioner, vs. GEN. HERMOGENES ESPERON, JR., LT. GEN. ROMEO Forces of the Philippines; further proceedings and investigations, as
P. TOLENTINO, MAJ. GEN. JUANITO GOMEZ, MAJ. GEN. DELFIN BANGIT, LT. COL. NOEL may be necessary, should be made to pursue the lead allegedly
CLEMENT, LT. COL. MELQUIADES FELICIANO, and DIRECTOR GENERAL OSCAR CALDERON, provided by State Prosecutor Velasco on the identities of the
Respondents. possible abductors; (c) inquiring into the veracity of Lipio’s and
G.R. No. 178497 February 4, 2014 Manuel’s claims that Jonas was abducted by a certain @KA DANTE
and @KA ENSO of the CPP/NPA guerilla unit RYG; (d) determining
EDITA T. BURGOS, Petitioner, vs. GEN. HERMOGENES ESPERON, JR., LT. GEN. ROMEO P. based on records, past and present, as well as further
TOLENTINO, MAJ. GEN. JUANITO GOMEZ, MAJ. GEN. DELFIN BANGIT, LT. COL. NOEL investigation, the identities and whereabouts of @KA DANTE and
CLEMENT, LT. COL. MELQUIADES FELICIANO, and DIRECTOR GENERAL OSCAR CALDERON, @KA ENSO; and (e) undertaking all measures, in the investigation
Respondents. of the Burgos abduction, that may be necessary to live up to the
G.R. No. 183711 extraordinary measures we require in addressing an enforced
disappearance under the Rule on the Writ of Amparo;
EDITA T. BURGOS, Petitioner, vs. GEN. HERMOGENES ESPERON, JR., LT. GEN. ROMEO P. x CHR Æ rendered a report; submitted to CA
TOLENTINO, MAJ. GEN. JUANITO GOMEZ, LT. COL. MELQUIADES FELICIANO, and LT. COL. x CA Æ in favor of EDITA
NOEL CLEMENT, Respondents. o The CA held that the issue in the petition for habeas corpus is not
G.R. No. 183712 the illegal confinement or detention of Jonas, but his enforced
disappearance. Considering that Jonas was a victim of enforced
EDITA T. BURGOS, Petitioner, vs. CHIEF OF STAFF OF THE ARMED FORCES OF THE disappearance, the present case is beyond the ambit of a petition
PHILIPPINES, GEN. HERMOGENES ESPERON, JR.; Commanding General of the Philippine for habeas corpus.
Army, LT. GEN. ALEXANDER YANO; and Chief of the Philippine National Police, DIRECTOR o As far as the AFP was concerned, the CA held that the fact that Lt.
GENERAL AVELINO RAZON, JR., Respondents. Baliaga of the Philippine Army was positively identified as one of the
G.R. No. 183713 abductors of Jonas, coupled with the AFP’s lack of serious effort to
EN BANC conduct further investigation, spoke loudly of the AFP
leadership’s accountability.
FACTS: x Esperon et al Æ filed an MR
x Jonas Joseph T. Burgos - a farmer advocate and a member of Kilusang x Edita Æ filed an Urgent Ex Parte Motion Ex Abundanti Cautela
Magbubukid sa Bulacan (a chapter of the militant peasant organization Kilusang o (1) order the persons named in the sealed documents to be
Magbubukid ng Pilipinas) - was forcibly taken and abducted by a group of four impleaded in CA-G.R. SP No. 00008-WA and G.R. No. 183713;
(4) men and a woman from the extension portion of Hapag Kainan Restaurant, o (2) issue a writ of Amparo on the basis of the newly discovered
located at the ground floor of Ever Gotesco Mall, Commonwealth Avenue, evidence (the sealed attachment to the motion); and
Quezon City o (3) refer the cases to the CA for further hearing on the newly
x Jonas shouted at the manager “Maam aktibista lang po ako” discovered evidence.
x The security guard tried to intervene but upon seeing that the abductors were
police officers, he backed off ISSUE # 1: Whether the CHR should be allowed to view the subject documents which, as
o The guard was able to see that Jonas was forced into the rear respondents allege, are of sensitive and confidential nature.
portion of a plain maroon colored Toyota Revo with plate number
TAB 194 HELD # 1: YES.
x Edita, Jonas’ mother, held a press conference x The directive for the submission of the above-mentioned documents arose from
x It was later on discovered that plate number TAB 194 was registered to a 1991 our determination in our June 22, 2010 Resolution that the PNP-CIDG failed to
Isuzu XLT vehicle owned by a certain Mauro B. Mudlong. identify the cartographic sketches of two (one male and one female) of the five
o It was also later confirmed by employees of the Department of abductors of Jonas, based on their interview with eyewitnesses to the abduction.
Environment and Natural Resources (DENR) that Mudlong was For this reason, the Court directly commissioned the CHR to continue the
arrested and his 1991 Isuzu XLT vehicle was seized on June 24, investigation of Jonas’ abduction and the gathering of evidence.
2006 by Cpl. Castro Bugalan and Pfc. Jose Villeña of the 56th x Based on its March 15, 2011 Report, the CHR uncovered a lead - a claim made
Infantry Battalion (IB) of the Philippine Army for transporting by Eustaquio, Chairman of the Union Masses for Democracy and Justice, that the
timber without permit. male abductor of Jonas appearing in the cartographic sketch was among the
o As agreed upon by the DENR employees and officers of the 56th raiders who abducted him and four others, known as the "ERAP FIVE."
IB, the vehicle with the license plate no. TAB 194 was impounded in x This prompted the CHR to request copies of the documents embodied in par.
the 56th IB headquarters whose commanding officer at that time III(i) of the fallo of the Court’s July 5, 2011 Resolution from General Gilberto
was Lt. Col. Noel Clement Jose C. Roa of the Office of the Judge Advocate General, AFP. Gen. Roa initially
x Edita Æ filed consolidated petitions for the Issuance of the Writ of Habeas denied this request but eventually complied with the Court’s directive of July 5,
Corpus, for Contempt, and for the Issuance of a Writ of Amparo before the CA 2011 to submit the documents via the September 23, 2011 Manifestation and
x CA Æ dismissed the petition Motion and the June 7, 2013 Compliance. In the same July 5, 2011 Resolution,
o petitioner presented failed to establish her claimed direct the Court made it plain that these documents shall be released exclusively to the
connection between the abductors of Jonas and the military Court for its examination to determine their relevance to the present case and the
o the evidence does not show how license plate number TAB 194 advisability of their public disclosure.
(supposedly attached to the 1991 Isuzu XLT vehicle impounded at x Pursuant to the Court’s October 11, 2011 Resolution, the CHR submitted its
the 56th IB Headquarters) came to be attached to the getaway March 20, 2012 Progress Report on its continuing investigation of Jonas’
Toyota Revo on April 28, 2007, and whether the two license plates abduction. Attached to this Progress Report was Virgilio Eustaquio’s sworn
are one and the same at all. The CA emphasized that the evidence affidavit stating that: (1) he was one of the victims of the abduction incident on
does not indicate whether the abductors are members of the May 22, 2006, otherwise known as the "ERAP FIVE" incident; (2) as a result of
182
RECTO, GAYLE ANGELI M.
2011-0008 | AUSL
Personal Notes on Remedial Law 2 Review (based on the syllabus of Prof. Henedino M. Brondial)

this incident, they filed a case with the Ombudsman against Commodore Razon, Jr. v. Tagitis) for the enforced disappearance of Jonas. In its March 18,
Leonardo Calderon and other members of the Intelligence Service, AFP (ISAFP) 2013 decision, the CA found, by substantial evidence, that Lt. Baliaga
for arbitrary detention, unlawful arrest, maltreatment of prisoners, grave threats, participated in the abduction on the basis of Cabintoy’s positive identification that
incriminatory machination and robbery; and (3) the male abductor of Jonas he was one of the abductors of Jonas who told him not to interfere because the
appearing in the cartographic sketch shown to him by the CHR was among the latter had been under surveillance for drugs. In the same Decision, the CA also
raiders who abducted him and his four companions because it resembled the held the AFP and the PNP accountable for having failed to discharge the burden
cartographic sketch he described in relation to the ERAP FIVE incident on May of extraordinary diligence in the investigation of the enforced disappearance of
22, 2006. Jonas. Thus, the CA issued the following directives to address the enforced
x After reviewing the submissions of both the respondents20 and the CHR21 disappearance of Jonas:
pursuant to the Court’s July 5, 2011, August 23, 2011 and October 11, 2011 o (1) DIRECT the PNP through its investigative arm, the PNP-CIDG,
Resolutions, we resolve to grant the CHR access to these requested documents to identify and locate the abductors of Jonas Burgos who are still at
to allow them the opportunity to ascertain the true identities of the persons large and to establish the link between the abductors of Jonas
depicted in the cartographic sketches. Burgos and those involved in the ERAP 5 incident;
x At this point, we emphasize that the sworn affidavit of Eustaquio (that attests to o (2) DIRECT the incumbent Chief of Staff of the Armed Forces of the
the resemblance of one of Jonas’ abductors to the abductors of the ERAP FIVE) Philippines and the Director General of the Philippines National
constitutes the sought-after missing link that establishes the relevance of the Police, and their successors, to ensure the continuance of their
requested documents to the present case. We note that this lead may help the investigation and coordination on the enforced disappearance of
CHR ascertain the identities of those depicted in the cartographic sketches as Jonas Burgos until the persons found responsible are brought
two of Jonas’ abductors (one male and one female) who, to this day, remain before the bar of justice;
unidentified. o (3) DIRECT the Commission on Human Rights to continue with its
x In view of the sensitive and confidential nature of the requested documents, we own independent investigation on the enforced disappearance of
direct the Clerk of Court of the Supreme Court to allow the duly-authorized Jonas Burgos with the same degree of diligence required under the
representatives of the CHR to inspect the requested documents in camera within Rule on the Writ of Amparo;
five (5) days from receipt of this Resolution. o (4) DIRECT the Armed Forces of the Philippines and the Philippine
x The documents shall be examined and compared with the cartographic sketches National Police to extend full assistance to the Commission on
of the two abductors of Jonas, without copying and without bringing the Human Rights in the conduct of the latter’s investigation; and
documents outside the premises of the Office of the Clerk of Court of the o (5) DIRECT the Chief of Staff, Armed Forces of the Philippines, the
Supreme Court. The inspection of the documents shall be within office hours and Director General, Philippine National Police and the Chairman,
for a reasonable period of time sufficient to allow the CHR to comprehensively Commission on Human Rights to submit a quarterly report to the
investigate the lead provided by Eustaquio. Court on the results of their respective investigation.28
x To fully fulfill the objective of the Rule on the Writ of Amparo, further x We note that the respondents did not appeal the March 18, 2013 CA decision
investigation using the standard of extraordinary diligence should be undertaken and the May 23, 2013 CA resolution denying their motion for partial
by the CHR to pursue the lead provided by Eustaquio. We take judicial notice of reconsideration.
the ongoing investigation being conducted by the Department of Justice (DOJ), x Based on the above considerations, in particular, the final ruling of the CA that
through the NBI, on the disappearance of Jonas.22 In this regard, we direct the confirmed the validity of the issuance of the Writ of Amparo and its
NBI to coordinate and provide direct investigative assistance to the CHR as the determination of the entities responsible for the enforced disappearance of
latter may require, pursuant to the authority granted under the Court’s June 22, Jonas, we resolve to deny the petitioner’s prayer to issue the writ of Amparo
2010 Resolution. anew and to refer the case to the CA based on the newly discovered evidence.
x For this purpose, we require the CHR to submit a supplemental investigation We so conclude as the petitioner’s request for the reissuance of the writ and for
report to the DOJ, copy furnished the petitioner, the NBI, the incumbent Chiefs the rehearing of the case by the CA would be redundant and superfluous in light
of the AFP, the PNP and the PNP-CIDG, and all the respondents within sixty days of: (1) the ongoing investigation being conducted by the DOJ through the NBI;
(60) days from receipt of this Resolution. (2) the CHR investigation directed by the Court in this Resolution; and (3) the
continuing investigation directed by the CA in its March 18, 2013 decision.
ISSUE # 2: Whether Edita’s Urgent Ex Parte Motion Ex Abundanti Cautela should be granted in x We emphasize that while the Rule on the Writ of Amparo accords the
that the liability of the herein respondents should be determined. Court a wide latitude in crafting remedies to address an enforced
disappearance, it cannot (without violating the nature of the writ of
HELD # 2: NO. Amparo as a summary remedy that provides rapid judicial relief) grant
x After reviewing the newly discovered evidence submitted by the petitioner and remedies that would complicate and prolong rather than expedite the
considering all the developments of the case, including the March 18, 2013 CA investigations already ongoing. Note that the CA has already
decision that confirmed the validity of the issuance of the Writ of Amparo in the determined with finality that Jonas was a victim of enforced
present case, we resolve to deny the petitioner’s Urgent Ex Parte Motion Ex disappearance.
Abundanti Cautela. x We clarify that by denying the petitioner’s motion, we do not thereby rule on
x We note and conclude, based on the developments highlighted above, that the the admissibility or the merits of the newly discovered evidence
beneficial purpose of the Writ of Amparo has been served in the present case. As submitted by the petitioner. We likewise do not foreclose any
we held in Razon, Jr. v. Tagitis,23 the writ merely embodies the Court’s investigation by the proper investigative and prosecutory agencies of
directives to police agencies to undertake specified courses of action to the other entities whose identities and participation in the enforced
address the enforced disappearance of an individual. The Writ of Amparo disappearance of Jonas may be disclosed in future investigations and
serves both a preventive and a curative role. It is curative as it facilitates proceedings. Considering that the present case has already reached the
the subsequent punishment of perpetrators through the investigation prosecution stage, the petitioner’s motion should have been filed with the proper
and remedial action that it directs.24 The focus is on procedural investigative and prosecutory agencies of the government.
curative remedies rather than on the tracking of a specific criminal or x To expedite proceedings, we refer the petitioner’s motion, this Resolution and its
the resolution of administrative liabilities. The unique nature of covered cases to the DOJ for investigation, for the purpose of filing the
Amparo proceedings has led us to define terms or concepts specific to appropriate criminal charges in the proper courts against the proper parties, if
what the proceedings seek to achieve. In Razon Jr., v. Tagitis,25 we defined warranted, based on the gathered evidence. For this purpose, we direct the
what the terms "responsibility" and "accountability" signify in an Amparo case. petitioner to furnish the DOJ and the NBI copies of her Urgent Ex Parte Motion
We said: Ex Abundanti Cautela, together with the sealed attachments to the Motion,
o Responsibility refers to the extent the actors have been within five (5) days from receipt of this Resolution.
established by substantial evidence to have participated in x As mentioned, we take judicial notice of the ongoing investigation by the DOJ,
whatever way, by action or omission, in an enforced through the NBI, of the disappearance of Jonas. This DOJ investigation is without
disappearance, as a measure of the remedies this Court shall craft, prejudice to the Office of the Ombudsman’s exercise of its primary jurisdiction
among them, the directive to file the appropriate criminal and civil over the investigation of the criminal aspect of this case should the case be
cases against the responsible parties in the proper courts. determined to be cognizable by the Sandiganbayan.29
Accountability, on the other hand, refers to the measure of x As we direct below, further investigation for purposes of the present proceedings
remedies that should be addressed to those who exhibited shall continue to be undertaken by the CHR, in close coordination with the NBI,
involvement in the enforced disappearance without bringing the for the completion of the investigation under the terms of our June 22, 2010
level of their complicity to the level of responsibility defined above; Resolution and the additional directives under the present Resolution.
or who are imputed with knowledge relating to the enforced x As a final note, we emphasize that our ROLE in a writ of Amparo
disappearance and who carry the burden of disclosure; or those proceeding is merely
who carry, but have failed to discharge, the burden of o to determine whether an enforced disappearance has taken
extraordinary diligence in the investigation of the enforced place;
disappearance.26 o to determine who is responsible or accountable; and
x In the present case, while Jonas remains missing, the series of calculated o to define and impose the appropriate remedies to address
directives issued by the Court outlined above and the extraordinary diligence the the disappearance.1âwphi1
CHR demonstrated in its investigations resulted in the criminal prosecution of Lt. x As shown above, the beneficial purpose of the Writ of Amparo has been served
Baliaga. We take judicial notice of the fact that the Regional Trial Court, Quezon in the present case with the CA’s final determination of the persons responsible
City, Branch 216, has already found probable cause for arbitrary detention and accountable for the enforced disappearance of Jonas and the
against Lt. Baliaga and has ordered his arrest in connection with Jonas’ commencement of criminal action against Lt. Baliaga. At this stage, criminal,
disappearance.27 investigation and prosecution proceedings are already beyond the
x We also emphasize that the CA in its March 18, 2013 decision already ruled with reach of the Writ of Amparo proceeding now before us.
finality on the entities responsible and accountable (as these terms are defined in
183
RECTO, GAYLE ANGELI M.
2011-0008 | AUSL
Personal Notes on Remedial Law 2 Review (based on the syllabus of Prof. Henedino M. Brondial)

x Based on the above developments, we now hold that the full extent of the or that which has jurisdiction over the place where the data or information is gathered,
remedies envisioned by the Rule on the Writ of Amparo has been served and collected or stored.
exhausted.
x Considering the foregoing, the Court RESOLVES to: The writ of habeas data shall be enforceable anywhere in the Philippines.
o (1) DENY petitioner Edita Burgos’ Urgent Ex Parte Motion Ex
Abundanti Cautela; Sec. 5. Docket Fees. - No docket and other lawful fees shall be required from an indigent
o (2) REFER the petitioner’s Urgent Ex Parte Motion Ex Abundanti petitioner. The petition of the indigent shall be docked and acted upon immediately, without
Cautela, this Resolution and its covered cases to the Department of prejudice to subsequent submission of proof of indigency not later than fifteen (15) days from
Justice for investigation for the purpose of filing the appropriate the filing of the petition.
criminal charges in the proper courts against the proper parties if
such action is warranted by the gathered evidence. The referral to SEC. 6. Petition. - A verified written petition for a writ of habeas data should contain:
the Department of Justice is without prejudice to the Office of the
Ombudsman’s exercise of its primary jurisdiction over the (a) The personal circumstances of the petitioner and the respondent;
investigation should the case be determined to be cognizable by
the Sandiganbayan; (b) The manner the right to privacy is violated or threatened and how it affects the right to
o (3) DIRECT the petitioner to furnish the Department of Justice and life, liberty or security of the aggrieved party;
the National Bureau of Investigation copies of her Urgent Ex Parte
Motion Ex Abundanti Cautela, together with the sealed attachments (c) The actions and recourses taken by the petitioner to secure the data or information;
to the Motion, within five (5) days from receipt of this Resolution;
o (4) DIRECT the Clerk of Court of the Supreme Court to allow the (d) The location of the files, registers or databases, the government office, and the person in
duly-authorized representatives of the Commission on Human charge, in possession or in control of the data or information, if known;
Rights to inspect the requested documents in camera within five
(5) days from receipt of this Resolution. For this purpose, the (e) The reliefs prayed for, which may include the updating, rectification, suppression or
documents shall be examined and compared with the cartographic destruction of the database or information or files kept by the respondent.
sketches of the two abductors of Jonas Burgos without copying and
bringing the documents outside the premises of the Office of the In case of threats, the relief may include a prayer for an order enjoining the act
Clerk of Court of the Supreme Court. The inspection of the complained of; and
documents shall be conducted within office hours and for a
reasonable period of time that would allow the Commission on (f) Such other relevant reliefs as are just and equitable.
Human Rights to comprehensively investigate the lead provided by
Virgilio Eustaquio; SEC. 7. Issuance of the Writ. - Upon the filing of the petition, the court, justice or judge
o (5) DIRECT the National Bureau of Investigation to coordinate and shall immediately order the issuance of the writ if on its face it ought to issue. The clerk of
provide direct investigative assistance to the Commission on court shall issue the writ under the seal of the court and cause it to be served within three (3)
Human Rights as the latter may require, pursuant to the authority days from the issuance; or, in case of urgent necessity, the justice or judge may issue the writ
granted under the Court's June 22, 2010 Resolution. under his or her own hand, and may deputize any officer or person serve it.
o (6) REQUIRE the Commission on Human Rights to submit a
supplemental investigation report to the Department of Justice, copy The writ shall also set the date and time for summary hearing of the petition which shall not
furnished the petitioner, the National Bureau of Investigation, the be later than ten (10) work days from the date of its issuance.
incumbent Chiefs of the Armed Forces of the Philippines, the
Philippine National Police and the Philippine National Police-Criminal SEC. 8. Penalty for Refusing to Issue or Serve the Writ. - A clerk of court who refuses
Investigation and Detection Group, and all the respondents, within to issue the writ after its allowance, or a deputized person who refuses to serve the same,
sixty (60) days from receipt of this Resolution. shall be punished by the court, justice or judge for contempt without prejudice to other
o (7) DECLARE this Writ of Amparo proceeding closed and disciplinary actions.
terminated, without prejudice to the concerned parties' compliance
with the above directives and subject to the Court's continuing SEC. 9. How the Writ is Served. - The writ shall be served upon the respondent by a
jurisdiction to enforce compliance with this Resolution. judicial officer or by a person deputized by the court, justice or judge who shall retain a copy
on which to make a return of service. In case the writ cannot be served personally on the
2. Writ of Habeas Data respondent, the rules on substituted service shall apply.

A. M. No. 08-1-16-SC SEC. 10. Return; Contents. - The respondent shall file a verified written return together
January 22, 2008 with supporting affidavits within five (5) working days from service of the writ, which period
may be reasonably extended by the Court for justifiable reasons. The return shall, among
THE RULE ON THE WRIT OF HABEAS DATA other things, contain the following:

SECTION 1. Habeas Data. - The writ of habeas data is a remedy available to any person (a) The lawful defenses such as national security, state secrets, privileged communications,
whose right to privacy in life, liberty or security is violated or threatened by an unlawful act or confidentiality of the source of information of media and others;
omission of a public official or employee, or of a private individual or entity engaged in the
gathering, collecting or storing of data or information regarding the person, family, home and (b) In case of respondent in charge, in possession or in control of the data or information
correspondence of the aggrieved party. subject of the petition;

SEC. 2. Who May File. - Any aggrieved party may file a petition for the writ of habeas data. (i) a disclosure of the data or information about the petitioner, the nature of such data or
However, in cases of extralegal killings and enforced disappearances, the petition may be filed information, and the purpose for its collection;
by:
(ii) the steps or actions taken by the respondent to ensure the security and confidentiality of
(a) Any member of the immediate family of the aggrieved party, namely: the spouse, children the data or information; and,
and parents; or
(iii) the currency and accuracy of the data or information held; and,
(b) Any ascendant, descendant or collateral relative of the aggrieved party within the fourth
civil degree of consanguinity or affinity, in default of those mentioned in the preceding (c) Other allegations relevant to the resolution of the proceeding.
paragraph; or
A general denial of the allegations in the petition shall not be allowed.
SEC. 3. Where to File. - The petition may be filed with the Regional Trial Court where the
petitioner or respondent resides, or that which has jurisdiction over the place where the data SEC. 11. Contempt. - The court, justice or judge may punish with imprisonment or fine a
or information is gathered, collected or stored, at the option of the petitioner. respondent who commits contempt by making a false return, or refusing to make a return; or
any person who otherwise disobeys or resist a lawful process or order of the court.
The petition may also be filed with the Supreme Court or the Court of Appeals or the
Sandiganbayan when the action concerns public data files of government offices. SEC. 12. When Defenses May be Heard in Chambers. - A hearing in chambers may be
conducted where the respondent invokes the defense that the release of the data or
SEC. 4. Where Returnable; Enforceable. - When the writ is issued by a Regional Trial information in question shall compromise national security or state secrets, or when the data
Court or any judge thereof, it shall be returnable before such court or judge. or information cannot be divulged to the public due to its nature or privileged character.

When issued by the Court of Appeals or the Sandiganbayan or any of its justices, it may be Sec. 13. Prohibited Pleadings and Motions. - The following pleadings and motions are
returnable before such court or any justice thereof, or to any Regional Trial Court of the place prohibited:
where the petitioner or respondent resides, or that which has jurisdiction over the place where
the data or information is gathered, collected or stored. (a) Motion to dismiss;

When issued by the Supreme Court or any of its justices, it may be returnable before such (b) Motion for extension of time to file return, opposition, affidavit, position paper and other
Court or any justice thereof, or before the Court of Appeals or the Sandiganbayan or any of its pleadings;
justices, or to any Regional Trial Court of the place where the petitioner or respondent resides,
(c) Dilatory motion for postponement;
184
RECTO, GAYLE ANGELI M.
2011-0008 | AUSL
Personal Notes on Remedial Law 2 Review (based on the syllabus of Prof. Henedino M. Brondial)

NOTES (Based on the Annotations of Riguera and Albano; includes


(d) Motion for a bill of particulars; Author’s own comments and research)
(e) Counterclaim or cross-claim;
x Venue:
(f) Third-party complaint; o Where the petitioner resides OR
o Where the data or information was allegedly gathered,
(g) Reply; stored, or collected
ƒ AT THE OPTION of the petitioner
(h) Motion to declare respondent in default;
x Jurisdiction Æ RTC, SC, CA (concurrent)
(i) Intervention; x Who may file Æ the aggrieved party OR his immediate family or
relative within 4th degree
(j) Memorandum; x Return Æ filed within 5 working days OR as may be reasonably
extended by the issuing court for justifiable reasons
(k) Motion for reconsideration of interlocutory orders or interim relief orders; and x Effects of failure to file return:
o Proceedings to be heard ex parte
(l) Petition for certiorari, mandamus or prohibition against any interlocutory order.
o Respondent may be cited in indirect contempt x
SEC. 14. Return; Filing. - In case the respondent fails to file a return, the court, justice or Nature of proceedings, prohibited pleadings, mode of appeal,
judge shall proceed to hear the petition ex parte, granting the petitioner such relief as the quantum of evidence, hearing in camera, effect of filing of criminal
petition may warrant unless the court in its discretion requires the petitioner to submit action Æ same as in amparo petition
evidence. x Petition for habeas data is IMPROPER when what is sought to
SEC. 15. Summary Hearing. - The hearing on the petition shall be summary. However, the
protected are property rights such as employment. (Meralco vs.
court, justice or judge may call for a preliminary conference to simplify the issues and Gopez-Lim)
determine the possibility of obtaining stipulations and admissions from the parties. x The phrase “engaged in gathering, collecting, or storing data and
information” does NOT necessarily mean that the respondent must be
SEC. 16. Judgment. - The court shall render judgment within ten (10) days from the time in a business relating thereto. Otherwise, the writ of habeas data will
the petition is submitted for decision. If the allegations in the petition are proven by be unjustifiably limited to a very small group of persons. (Vivares vs.
substantial evidence, the court shall enjoin the act complained of, or order the deletion,
destruction, or rectification of the erroneous data or information and grant other relevant
St. Theresa’s College)
reliefs as may be just and equitable; otherwise, the privilege of the writ shall be denied. x When the petitioner has no right to expect privacy, such as when she
uploads her photos on Facebook, especially when her settings are on
Upon its finality, the judgment shall be enforced by the sheriff or any lawful officers as may be PUBLIC, she may not invoke the writ of habeas data. (Vivares vs. St.
designated by the court, justice or judge within five (5) working days. Theresa’s College)
SEC. 17. Return of Service. - The officer who executed the final judgment shall, within
HABEAS CORPUS AMPARO HABEAS DATA
three (3) days from its enforcement, make a verified return to the court. The return shall
contain a full statement of the proceedings under the writ and a complete inventory of the illegal confinement or violation of right to
database or information, or documents and articles inspected, updated, rectified, or deleted, detention OR withholding extralegal killings and privacy in life, liberty
ground enforced disappearances or
with copies served on the petitioner and the respondent. of the rightful custody of or security is violated
threats thereof
any person or threaten thereof
The officer shall state in the return how the judgment was enforced and complied with by the
respondent, as well as all objections of the parties regarding the manner and regularity of the aggrieved party or by any aggrieved party or by
party for whose relief it is
qualified person or entity in any person mentioned
service of the writ. petitioner intended OR by some
the ORDER set forth in in Section 2 of the
person on his behalf
Section 2 of Rule Rule
SEC. 18. Hearing on Officer’s Return. - The court shall set the return for hearing with due
notice to the parties and act accordingly.
public official or
SEC. 19. Appeal. - Any party may appeal from the final judgment or order to the Supreme employee, or of a
Court under Rule 45. The appeal may raise questions of fact or law or both. private individual or
public official or employee, entity engaged in the
officer or person who
The period of appeal shall be five (5) working days from the date of notice of the judgment or or of a private individual or gathering, collecting or
illegally detains and
respondent entity who committed the storing of data or
final order. confines OR unlawfully
violation or the threat to information regarding
withholds rightful custody
violate the person, family,
The appeal shall be given the same priority as in habeas corpus and amparo cases. home and
correspondence of the
SEC. 20. Institution of Separate Actions. - The filing of a petition for the writ of habeas aggrieved party
data shall not preclude the filing of separate criminal, civil or administrative actions.

SEC. 21. Consolidation. - When a criminal action is filed subsequent to the filing of a
petition for the writ, the latter shall be consolidated with the criminal action. RTC, SB, CA, SC
(concurrent without
RTC, SB, CA, SC
prejudice to the doctrine RTC, SB, CA, SC
When a criminal action and a separate civil action are filed subsequent to a petition for a writ (concurrent without
of hierarchy of courts); (concurrent without
jurisdiction prejudice to the
of habeas data, the petition shall be consolidated with the criminal action. MTC under its special prejudice to the doctrine of
doctrine of hierarchy
jurisdiction in the hierarchy of courts)
of courts)
absence of RTC judges in
After consolidation, the procedure under this Rule shall continue to govern the disposition of
the province or city
the reliefs in the petition.

SEC. 22. Effect of Filing of a Criminal Action. - When a criminal action has been
commenced, no separate petition for the writ shall be filed. The relief under the writ shall be where the petitioner or
available to an aggrieved party by motion in the criminal case. respondent resides, or
that which has
where the petitioner or where the threat, act or
jurisdiction over the
The procedure under this Rule shall govern the disposition of the reliefs available under the respondent resides at the omission was committed or
venue place where the data
writ of habeas data. option of the petitioner any of its elements
or information is
(application of Rule 4) occurred
gathered, collected or
SEC. 23. Substantive Rights. - This Rule shall not diminish, increase or modify substantive stored, at the option of
the petitioner
rights.

SEC. 24. Suppletory Application of the Rules of Court. - The Rules of Court shall apply PhP 500 [Section 7(g)
suppletorily insofar as it is not inconsistent with this Rule. PhP 500 [Section 7(g)
docket fees Exempt Rule 141); indigents
Rule 141)
are exempt
SEC. 25. Effectivity. - This Rule shall take effect on February 2, 2008, following its
publication in three (3) newspapers of general circulation. if issued by RTC/ MTC -
within the judicial region
extent of
[PUBLISHED IN THE MANILA BULLETIN, THE PHILIPPINE STAR AND THE PHILIPPINE DAILY where the court sits; if anywhere in PH anywhere in PH
enforceaility
issued by SB/ CA/ SC -
INQUIRER ON 25 JANUARY 2008]
anywhere in PH

185
RECTO, GAYLE ANGELI M.
2011-0008 | AUSL
Personal Notes on Remedial Law 2 Review (based on the syllabus of Prof. Henedino M. Brondial)

TPO, WPO, Production


interim reliefs none none (v) R.A. No. 9175, Chainsaw Act;
Order, Inspection Order

within 5 working days within 5 working days (w) R.A. No. 9275, Clean Water Act;
filing of at the time and date WHICH may NOT be WHICH MAY be
return provided for in the writ extended EXCEPT on highly extended reasonably
meritorious grounds for justifiable reasons (x) R.A. No. 9483, Oil Spill Compensation Act of 2007; and

within 5 working days (y) Provisions in C.A. No. 141, The Public Land Act; R.A. No. 6657, Comprehensive Agrarian
within 48 hours from within 5 working days from
from receipt of order Reform Law of 1988; R.A. No. 7160, Local Government Code of 1991; R.A. No. 7161, Tax
receipt of order receipt of order dismissing
dismissing the Laws Incorporated in the Revised Forestry Code and Other Environmental Laws (Amending the
appeal dismissing the petition/ the petition/ denying the
petition/ denying the NIRC); R.A. No. 7308, Seed Industry Development Act of 1992; R.A. No. 7900, High-Value
denying the MR; via MR; via Rule 45 directly to
MR; via Rule 45
Notice of Appeal SC
directly to SC Crops Development
primacy of
Rules of Procedure for Environmental Cases Act; R.A. No. 8048, Coconut Preservation Act;
criminal not applicable applicable applicable
action R.A. No. 8435, Agriculture and Fisheries Modernization Act of 1997; R.A. No. 9522, The
Philippine Archipelagic Baselines Law; R.A. No. 9593, Renewable Energy Act of 2008; R.A. No.
quantum of preponderance of
proof evidence
substantial evidence substantial evidence 9637, Philippine Biofuels Act; and other existing laws that relate to the conservation,
development, preservation, protection and utilization of the environment and natural
resources.
CASES
Section 3. Objectives. - The objectives of these Rules are:
x See Tapuz vs. del Rosario on page 6
x See Castillo vs. Cruz on page 7 (a) To protect and advance the constitutional right of the people to a balanced and healthful
x See Roxas vs. GMA on pag 21 ecology;

(b) To provide a simplified, speedy and inexpensive procedure for the enforcement of
3. Writ of Kalikasan
environmental rights and duties recognized under the Constitution, existing laws, rules and
regulations, and international agreements;
A.M. No. 09-6-8-SC
RULES OF PROCEDURE FOR ENVIRONMENTAL CASES (c) To introduce and adopt innovations and best practices ensuring the effective enforcement
of remedies and redress for violation of environmental laws; and
PART I
(d) To enable the courts to monitor and exact compliance with orders and judgments in
RULE 1 environmental cases.
GENERAL PROVISIONS
Section 4. Definition of Terms. -
Section 1. Title. — These Rules shall be known as "The Rules of Procedure for
Environmental Cases." (a) By-product or derivatives means any part taken or substance extracted from wildlife, in
raw or in processed form including stuffed animals and herbarium specimens. 1avvphi1
Section 2. Scope. — These Rules shall govern the procedure in civil, criminal and special civil
actions before the Regional Trial Courts, Metropolitan Trial Courts, Municipal Trial Courts in (b) Consent decree refers to a judicially-approved settlement between concerned parties
Cities, Municipal Trial Courts and Municipal Circuit Trial Courts involving enforcement or based on public interest and public policy to protect and preserve the environment.
violations of environmental and other related laws, rules and regulations such as but not
limited to the following: (c) Continuing mandamus is a writ issued by a court in an environmental case directing any
agency or instrumentality of the government or officer thereof to perform an act or series of
(a) Act No. 3572, Prohibition Against Cutting of Tindalo, Akli, and Molave Trees; acts decreed by final judgment which shall remain effective until judgment is fully satisfied.

(b) P.D. No. 705, Revised Forestry Code; (d) Environmental protection order (EPO) refers to an order issued by the court directing or
enjoining any person or government agency to perform or desist from performing an act in
(c) P.D. No. 856, Sanitation Code; order to protect, preserve or rehabilitate the environment.

(d) P.D. No. 979, Marine Pollution Decree; (e) Mineral refers to all naturally occurring inorganic substance in solid, gas, liquid, or any
intermediate state excluding energy materials such as coal, petroleum, natural gas, radioactive
(e) P.D. No. 1067, Water Code; materials and geothermal energy.

(f) P.D. No. 1151, Philippine Environmental Policy of 1977; (f) Precautionary principle states that when human activities may lead to threats of serious
and irreversible damage to the environment that is scientifically plausible but uncertain,
(g) P.D. No. 1433, Plant Quarantine Law of 1978; actions shall be taken to avoid or diminish that threat.

(h) P.D. No. 1586, Establishing an Environmental Impact Statement System Including Other (g) Strategic lawsuit against public participation (SLAPP) refers to an action whether civil,
Environmental Management Related Measures and for Other Purposes; criminal or administrative, brought against any person, institution or any government agency or
local government unit or its officials and employees, with the intent to harass, vex, exert undue
(i) R.A. No. 3571, Prohibition Against the Cutting, Destroying or Injuring of Planted or Growing pressure or stifle any legal recourse that such person, institution or government agency has
Trees, Flowering Plants and Shrubs or Plants of Scenic Value along Public Roads, in Plazas, taken or may take in the enforcement of environmental laws, protection of the
Parks, School Premises or in any Other Public Ground; environment or assertion of environmental rights.

(j) R.A. No. 4850, Laguna Lake Development Authority Act; (h) Wildlife means wild forms and varieties of flora and fauna, in all developmental stages
including those which are in captivity or are being bred or propagated.
(k) R.A. No. 6969, Toxic Substances and Hazardous Waste Act;
PART II
(l) R.A. No. 7076, People’s Small-Scale Mining Act; CIVIL PROCEDURE

(m) R.A. No. 7586, National Integrated Protected Areas System Act including all laws, decrees, RULE 2
orders, proclamations and issuances establishing protected areas; PLEADINGS AND PARTIES

(n) R.A. No. 7611, Strategic Environmental Plan for Palawan Act; Section 1. Pleadings and motions allowed. — The pleadings and motions that may be
filed are complaint, answer which may include compulsory counterclaim and cross-claim,
(o) R.A. No. 7942, Philippine Mining Act; motion for intervention, motion for discovery and motion for reconsideration of the judgment.

(p) R.A. No. 8371, Indigenous Peoples Rights Act; Motion for postponement, motion for new trial and petition for relief from judgment shall be
allowed in highly meritorious cases or to prevent a manifest miscarriage of justice.
(q) R.A. No. 8550, Philippine Fisheries Code;
Section 2. Prohibited pleadings or motions. — The following pleadings or motions shall
(r) R.A. No. 8749, Clean Air Act; not be allowed:

(s) R.A. No. 9003, Ecological Solid Waste Management Act; (a) Motion to dismiss the complaint;

(t) R.A. No. 9072, National Caves and Cave Resource Management Act; (b) Motion for a bill of particulars;

(u) R.A. No. 9147, Wildlife Conservation and Protection Act;


186
RECTO, GAYLE ANGELI M.
2011-0008 | AUSL
Personal Notes on Remedial Law 2 Review (based on the syllabus of Prof. Henedino M. Brondial)

(c) Motion for extension of time to file pleadings, except to file answer, the extension not to court officer or for justifiable reasons, by the counsel or representative of the plaintiff or any
exceed fifteen (15) days; suitable person authorized or deputized by the court issuing the summons.

(d) Motion to declare the defendant in default; Any private person who is authorized or deputized by the court to serve summons, orders and
other court processes shall for that purpose be considered an officer of the court.
(e) Reply and rejoinder; and
The summons shall be served on the defendant, together with a copy of an order informing all
(f) Third party complaint. parties that they have fifteen (15) days from the filing of an answer, within which to avail of
interrogatories to parties under Rule 25 of the Rules of Court and request for admission by
Section 3. Verified complaint. — The verified complaint shall contain the names of the adverse party under Rule 26, or at their discretion, make use of depositions under Rule 23 or
parties, their addresses, the cause of action and the reliefs prayed for. other measures under Rules 27 and 28.

The plaintiff shall attach to the verified complaint all evidence proving or supporting the cause of Should personal and substituted service fail, summons by publication shall be allowed. In the
action consisting of the affidavits of witnesses, documentary evidence and if possible, object case of juridical entities, summons by publication shall be done by indicating the names of the
evidence. The affidavits shall be in question and answer form and shall comply with the rules of officers or their duly authorized representatives.
admissibility of evidence.
Section 14. Verified answer. - Within fifteen (15) days from receipt of summons, the
The complaint shall state that it is an environmental case and the law involved. The complaint defendant shall file a verified answer to the complaint and serve a copy thereof on the
shall also include a certification against forum shopping. If the complaint is not an plaintiff. The defendant shall attach affidavits of witnesses, reports, studies of experts and all
environmental complaint, the presiding judge shall refer it to the executive judge for re-raffle. evidence in support of the defense.

Section 4. Who may file. — Any real party in interest, including the government and juridical Affirmative and special defenses not pleaded shall be deemed waived, except lack of
entities authorized by law, may file a civil action involving the enforcement or violation of any jurisdiction.
environmental law.
Cross-claims and compulsory counterclaims not asserted shall be considered barred. The
Section 5. Citizen suit. — Any Filipino citizen in representation of others, including minors or answer to counterclaims or cross-claims shall be filed and served within ten (10) days from
generations yet unborn, may file an action to enforce rights or obligations under service of the answer in which they are pleaded.
environmental laws. Upon the filing of a citizen suit, the court shall issue an order which shall
contain a brief description of the cause of action and the reliefs prayed for, requiring all Section 15. Effect of failure to answer. - Should the defendant fail to answer the
interested parties to manifest their interest to intervene in the case within fifteen (15) days complaint within the period provided, the court shall declare defendant in default and upon
from notice thereof. The plaintiff may publish the order once in a newspaper of a general motion of the plaintiff, shall receive evidence ex parte and render judgment based thereon and
circulation in the Philippines or furnish all affected barangays copies of said order. the reliefs prayed for.

Citizen suits filed under R.A. No. 8749 and R.A. No. 9003 shall be governed by their respective RULE 3
provisions. PRE-TRIAL

Section 6. Service of the complaint on the government or its agencies. - Upon the Section 1. Notice of pre-trial. - Within two (2) days from the filing of the answer to the
filing of the complaint, the plaintiff is required to furnish the government or the appropriate counterclaim or cross-claim, if any, the branch clerk of court shall issue a notice of the pre-trial
agency, although not a party, a copy of the complaint. Proof of service upon the government to be held not later than one (1) month from the filing of the last pleading.
or the appropriate agency shall be attached to the complaint.
The court shall schedule the pre-trial and set as many pre-trial conferences as may be
Section 7. Assignment by raffle. - If there is only one (1) designated branch in a multiple- necessary within a period of two (2) months counted from the date of the first pre-trial
sala court, the executive judge shall immediately refer the case to said branch. If there are conference.
two (2) or more designated branches, the executive judge shall conduct a special raffle on the
day the complaint is filed. Section 2. Pre-trial brief. - At least three (3) days before the pretrial, the parties shall
submit pre-trial briefs containing the following:
Section 8. Issuance of Temporary Environmental Protection Order (TEPO). - If it
appears from the verified complaint with a prayer for the issuance of an Environmental (a) A statement of their willingness to enter into an amicable settlement indicating the desired
Protection Order (EPO) that the matter is of extreme urgency and the applicant will suffer terms thereof or to submit the case to any of the alternative modes of dispute resolution;
grave injustice and irreparable injury, the executive judge of the multiple-sala court before
raffle or the presiding judge of a single-sala court as the case may be, may issue ex parte a (b) A summary of admitted facts and proposed stipulation of facts;
TEPO effective for only seventy-two (72) hours from date of the receipt of the TEPO by the
party or person enjoined. Within said period, the court where the case is assigned, shall (c) The legal and factual issues to be tried or resolved. For each factual issue, the parties shall
conduct a summary hearing to determine whether the TEPO may be extended until the state all evidence to support their positions thereon. For each legal issue, parties shall state the
termination of the case. applicable law and jurisprudence supporting their respective positions thereon;

The court where the case is assigned, shall periodically monitor the existence of acts that are (d) The documents or exhibits to be presented, including depositions, answers to
the subject matter of the TEPO even if issued by the executive judge, and may lift the same at interrogatories and answers to written request for admission by adverse party, stating the
any time as circumstances may warrant. purpose thereof;

The applicant shall be exempted from the posting of a bond for the issuance of a TEPO. (e) A manifestation of their having availed of discovery procedures or their intention to avail
themselves of referral to a commissioner or panel of experts;
Section 9. Action on motion for dissolution of TEPO. - The grounds for motion to
dissolve a TEPO shall be supported by affidavits of the party or person enjoined which the (f) The number and names of the witnesses and the substance of their affidavits;
applicant may oppose, also by affidavits.
(g) Clarificatory questions from the parties; and
The TEPO may be dissolved if it appears after hearing that its issuance or continuance would
cause irreparable damage to the party or person enjoined while the applicant may be fully (h) List of cases arising out of the same facts pending before other courts or administrative
compensated for such damages as he may suffer and subject to the posting of a sufficient agencies. Failure to comply with the required contents of a pre-trial brief may be a ground for
bond by the party or person enjoined. contempt.

Section 10. Prohibition against temporary restraining order (TRO) and preliminary Failure to file the pre-trial brief shall have the same effect as failure to appear at the pre-trial.
injunction. - Except the Supreme Court, no court can issue a TRO or writ of preliminary
injunction against lawful actions of government agencies that enforce environmental laws or Section 3. Referral to mediation. - At the start of the pre-trial conference, the court shall
prevent violations thereof. inquire from the parties if they have settled the dispute; otherwise, the court shall immediately
refer the parties or their counsel, if authorized by their clients, to the Philippine Mediation
Section 11. Report on TEPO, EPO, TRO or preliminary injunction. - The judge shall Center (PMC) unit for purposes of mediation. If not available, the court shall refer the case to
report any action taken on a TEPO, EPO, TRO or a preliminary injunction, including its the clerk of court or legal researcher for mediation.
modification and dissolution, to the Supreme Court, through the Office of the Court
Administrator, within ten (10) days from the action taken. Mediation must be conducted within a non-extendible period of thirty (30) days from receipt of
notice of referral to mediation.
Section 12. Payment of filing and other legal fees. - The payment of filing and other legal
fees by the plaintiff shall be deferred until after judgment unless the plaintiff is allowed to The mediation report must be submitted within ten (10) days from the expiration of the 30-
litigate as an indigent. It shall constitute a first lien on the judgment award. day period.

For a citizen suit, the court shall defer the payment of filing and other legal fees that shall Section 4. Preliminary conference. - If mediation fails, the court will schedule the
serve as first lien on the judgment award. continuance of the pre-trial. Before the scheduled date of continuance, the court may refer the
case to the branch clerk of court for a preliminary conference for the following purposes:
Section 13. Service of summons, orders and other court processes. - The summons,
orders and other court processes may be served by the sheriff, his deputy or other proper (a) To assist the parties in reaching a settlement;
187
RECTO, GAYLE ANGELI M.
2011-0008 | AUSL
Personal Notes on Remedial Law 2 Review (based on the syllabus of Prof. Henedino M. Brondial)

Section 7. Effect of failure to appear at pre-trial. - The court shall not dismiss the
(b) To mark the documents or exhibits to be presented by the parties and copies thereof to be complaint, except upon repeated and unjustified failure of the plaintiff to appear. The
attached to the records after comparison with the originals; dismissal shall be without prejudice, and the court may proceed with the counterclaim.

(c) To ascertain from the parties the undisputed facts and admissions on the genuineness and If the defendant fails to appear at the pre-trial, the court shall receive evidence ex parte.
due execution of the documents marked as exhibits;
Section 8. Minutes of pre-trial. - The minutes of each pre-trial conference shall contain
(d) To require the parties to submit the depositions taken under Rule 23 of the Rules of Court, matters taken up therein, more particularly admissions of facts and exhibits, and shall be
the answers to written interrogatories under Rule 25, and the answers to request for signed by the parties and their counsel.
admissions by the adverse party under Rule 26;
Section 9. Pre-trial order. - Within ten (10) days after the termination of the pre-trial, the
(e) To require the production of documents or things requested by a party under Rule 27 and court shall issue a pre-trial order setting forth the actions taken during the pre-trial
the results of the physical and mental examination of persons under Rule 28; conference, the facts stipulated, the admissions made, the evidence marked, the number of
witnesses to be presented and the schedule of trial. Said order shall bind the parties, limit the
(f) To consider such other matters as may aid in its prompt disposition; trial to matters not disposed of and control the course of action during the trial.

(g) To record the proceedings in the "Minutes of Preliminary Conference" to be signed by both Section 10. Efforts to settle. - The court shall endeavor to make the parties agree to
parties or their counsels; compromise or settle in accordance with law at any stage of the proceedings before rendition
of judgment.
(h) To mark the affidavits of witnesses which shall be in question and answer form and shall
constitute the direct examination of the witnesses; and RULE 4
TRIAL
(i) To attach the minutes together with the marked exhibits before the pre-trial proper.
Section 1. Continuous trial. - The judge shall conduct continuous trial which shall not
The parties or their counsel must submit to the branch clerk of court the names, addresses exceed two (2) months from the date of the issuance of the pre-trial order.
and contact numbers of the affiants.
Before the expiration of the two-month period, the judge may ask the Supreme Court for the
During the preliminary conference, the branch clerk of court shall also require the parties to extension of the trial period for justifiable cause.
submit the depositions taken under Rule 23 of the Rules of Court, the answers to written
interrogatories under Rule 25 and the answers to request for admissions by the adverse party Section 2. Affidavits in lieu of direct examination. - In lieu of direct examination,
under Rule 26. The branch clerk of court may also require the production of documents or affidavits marked during the pre-trial shall be presented as direct examination of affiants
things requested by a party under Rule 27 and the results of the physical and mental subject to cross- examination by the adverse party.
examination of persons under Rule 28.
Section 3. One-day examination of witness rule. - The court shall strictly adhere to the
Section 5. Pre-trial conference; consent decree. - The judge shall put the parties and rule that a witness has to be fully examined in one (1) day, subject to the court’s discretion of
their counsels under oath, and they shall remain under oath in all pre-trial conferences. extending the examination for justifiable reason. After the presentation of the last witness,
only oral offer of evidence shall be allowed, and the opposing party shall immediately
The judge shall exert best efforts to persuade the parties to arrive at a settlement of the interpose his objections. The judge shall forthwith rule on the offer of evidence in open court.
dispute. The judge may issue a consent decree approving the agreement between the parties in
accordance with law, morals, public order and public policy to protect the right of the people to Section 4. Submission of case for decision; filing of memoranda. - After the last party
a balanced and healthful ecology. has rested its case, the court shall issue an order submitting the case for decision.

Evidence not presented during the pre-trial, except newly-discovered evidence, shall be The court may require the parties to submit their respective memoranda, if possible in
deemed waived. electronic form, within a non-extendible period of thirty (30) days from the date the case is
submitted for decision.
Section 6. Failure to settle. - If there is no full settlement, the judge shall:
The court shall have a period of sixty (60) days to decide the case from the date the case is
(a) Adopt the minutes of the preliminary conference as part of the pre-trial proceedings and submitted for decision.
confirm the markings of exhibits or substituted photocopies and admissions on the
genuineness and due execution of documents; Section 5. Period to try and decide. - The court shall have a period of one (1) year from
the filing of the complaint to try and decide the case. Before the expiration of the one-year
(b) Determine if there are cases arising out of the same facts pending before other courts and period, the court may petition the Supreme Court for the extension of the period for justifiable
order its consolidation if warranted; cause.

(c) Determine if the pleadings are in order and if not, order the amendments if necessary; The court shall prioritize the adjudication of environmental cases.

(d) Determine if interlocutory issues are involved and resolve the same; RULE 5
JUDGMENT AND EXECUTION
(e) Consider the adding or dropping of parties;
Section 1. Reliefs in a citizen suit. - If warranted, the court may grant to the plaintiff
(f) Scrutinize every single allegation of the complaint, answer and other pleadings and proper reliefs which shall include the protection, preservation or rehabilitation of the
attachments thereto, and the contents of documents and all other evidence identified and pre- environment and the payment of attorney’s fees, costs of suit and other litigation expenses. It
marked during pre-trial in determining further admissions; may also require the violator to submit a program of rehabilitation or restoration of the
environment, the costs of which shall be borne by the violator, or to contribute to a special
(g) Obtain admissions based on the affidavits of witnesses and evidence attached to the trust fund for that purpose subject to the control of the court.
pleadings or submitted during pre-trial;
Section 2. Judgment not stayed by appeal. - Any judgment directing the performance of
(h) Define and simplify the factual and legal issues arising from the pleadings and evidence. acts for the protection, preservation or rehabilitation of the environment shall be executory
Uncontroverted issues and frivolous claims or defenses should be eliminated; pending appeal unless restrained by the appellate court.

(i) Discuss the propriety of rendering a summary judgment or a judgment based on the Section 3. Permanent EPO; writ of continuing mandamus. - In the judgment, the court
pleadings, evidence and admissions made during pre-trial; may convert the TEPO to a permanent EPO or issue a writ of continuing mandamus directing
the performance of acts which shall be effective until the judgment is fully satisfied.
(j) Observe the Most Important Witness Rule in limiting the number of witnesses, determining
the facts to be proved by each witness and fixing the approximate number of hours per The court may, by itself or through the appropriate government agency, monitor the execution
witness; of the judgment and require the party concerned to submit written reports on a quarterly basis
or sooner as may be necessary, detailing the progress of the execution and satisfaction of the
(k) Encourage referral of the case to a trial by commissioner under Rule 32 of the Rules of judgment. The other party may, at its option, submit its comments or observations on the
Court or to a mediator or arbitrator under any of the alternative modes of dispute resolution execution of the judgment.
governed by the Special Rules of Court on Alternative Dispute Resolution;
Section 4. Monitoring of compliance with judgment and orders of the court by a
(l) Determine the necessity of engaging the services of a qualified expert as a friend of the commissioner. - The court may motu proprio, or upon motion of the prevailing party, order
court (amicus curiae); and that the enforcement of the judgment or order be referred to a commissioner to be appointed
by the court. The commissioner shall file with the court written progress reports on a quarterly
(m) Ask parties to agree on the specific trial dates for continuous trial, comply with the one- basis or more frequently when necessary.
day examination of witness rule, adhere to the case flow chart determined by the court which
shall contain the different stages of the proceedings up to the promulgation of the decision and Section 5. Return of writ of execution. - The process of execution shall terminate upon a
use the time frame for each stage in setting the trial dates. sufficient showing that the decision or order has been implemented to the satisfaction of the
court in accordance with Section 14, Rule 39 of the Rules of Court.

188
RECTO, GAYLE ANGELI M.
2011-0008 | AUSL
Personal Notes on Remedial Law 2 Review (based on the syllabus of Prof. Henedino M. Brondial)

RULE 6 Section 6. How the writ is served. - The writ shall be served upon the respondent by a
STRATEGIC LAWSUIT AGAINST PUBLIC PARTICIPATION court officer or any person deputized by the court, who shall retain a copy on which to make a
return of service. In case the writ cannot be served personally, the rule on substituted service
Section 1. Strategic lawsuit against public participation (SLAPP). - A legal action filed shall apply.
to harass, vex, exert undue pressure or stifle any legal recourse that any person, institution or
the government has taken or may take in the enforcement of environmental laws, protection of Section 7. Penalty for refusing to issue or serve the writ. - A clerk of court who unduly
the environment or assertion of environmental rights shall be treated as a SLAPP and shall be delays or refuses to issue the writ after its allowance or a court officer or deputized person
governed by these Rules. who unduly delays or refuses to serve the same shall be punished by the court for contempt
without prejudice to other civil, criminal or administrative actions.
Section 2. SLAPP as a defense; how alleged. - In a SLAPP filed against a person involved
in the enforcement of environmental laws, protection of the environment, or assertion of Section 8. Return of respondent; contents. - Within a non-extendible period of ten (10)
environmental rights, the defendant may file an answer interposing as a defense that the case days after service of the writ, the respondent shall file a verified return which shall contain all
is a SLAPP and shall be supported by documents, affidavits, papers and other evidence; and, by defenses to show that respondent did not violate or threaten to violate, or allow the violation
way of counterclaim, pray for damages, attorney’s fees and costs of suit. of any environmental law, rule or regulation or commit any act resulting to environmental
damage of such magnitude as to prejudice the life, health or property of inhabitants in two or
The court shall direct the plaintiff or adverse party to file an opposition showing the suit is not a more cities or provinces.
SLAPP, attaching evidence in support thereof, within a non-extendible period of five (5) days
from receipt of notice that an answer has been filed. All defenses not raised in the return shall be deemed waived.

The defense of a SLAPP shall be set for hearing by the court after issuance of the order to file The return shall include affidavits of witnesses, documentary evidence, scientific or other
an opposition within fifteen (15) days from filing of the comment or the lapse of the period. expert studies, and if possible, object evidence, in support of the defense of the respondent.

Section 3. Summary hearing. - The hearing on the defense of a SLAPP shall be summary in A general denial of allegations in the petition shall be considered as an admission thereof.
nature. The parties must submit all available evidence in support of their respective positions.
The party seeking the dismissal of the case must prove by substantial evidence that his act for Section 9. Prohibited pleadings and motions. - The following pleadings and motions are
the enforcement of environmental law is a legitimate action for the protection, preservation prohibited:
and rehabilitation of the environment. The party filing the action assailed as a SLAPP shall
prove by preponderance of evidence that the action is not a SLAPP and is a valid claim. (a) Motion to dismiss;

Section 4. Resolution of the defense of a SLAPP. - The affirmative defense of a SLAPP (b) Motion for extension of time to file return;
shall be resolved within thirty (30) days after the summary hearing. If the court dismisses the
action, the court may award damages, attorney’s fees and costs of suit under a counterclaim if (c) Motion for postponement;
such has been filed. The dismissal shall be with prejudice.
(d) Motion for a bill of particulars;
If the court rejects the defense of a SLAPP, the evidence adduced during the summary hearing
shall be treated as evidence of the parties on the merits of the case. The action shall proceed in (e) Counterclaim or cross-claim;
accordance with the Rules of Court.
(f) Third-party complaint;
PART III
SPECIAL CIVIL ACTIONS (g) Reply; and

RULE 7 (h) Motion to declare respondent in default.


WRIT OF KALIKASAN
Section 10. Effect of failure to file return. - In case the respondent fails to file a return,
Section 1. Nature of the writ. - The writ is a remedy available to a natural or juridical the court shall proceed to hear the petition ex parte.
person, entity authorized by law, people’s organization, non-governmental organization, or any
public interest group accredited by or registered with any government agency, on behalf of Section 11. Hearing. - Upon receipt of the return of the respondent, the court may call a
persons whose constitutional right to a balanced and healthful ecology is violated, preliminary conference to simplify the issues, determine the possibility of obtaining stipulations
or threatened with violation by an unlawful act or omission of a public official or or admissions from the parties, and set the petition for hearing.
employee, or private individual or entity, involving environmental damage of such
magnitude as to prejudice the life, health or property of inhabitants in two or more cities or The hearing including the preliminary conference shall not extend beyond sixty (60) days and
provinces. shall be given the same priority as petitions for the writs of habeas corpus, amparo and
habeas data.
Section 2. Contents of the petition. - The verified petition shall contain the following:
Section 12. Discovery Measures. - A party may file a verified motion for the following
(a) The personal circumstances of the petitioner; reliefs:

(b) The name and personal circumstances of the respondent or if the name and personal (a) Ocular Inspection; order — The motion must show that an ocular inspection order is
circumstances are unknown and uncertain, the respondent may be described by an assumed necessary to establish the magnitude of the violation or the threat as to prejudice the life,
appellation; health or property of inhabitants in two or more cities or provinces. It shall state in detail the
place or places to be inspected. It shall be supported by affidavits of witnesses having
(c) The environmental law, rule or regulation violated or threatened to be violated, the act or personal knowledge of the violation or threatened violation of environmental law.
omission complained of, and the environmental damage of such magnitude as to prejudice the
life, health or property of inhabitants in two or more cities or provinces. After hearing, the court may order any person in possession or control of a designated land or
other property to permit entry for the purpose of inspecting or
(d) All relevant and material evidence consisting of the affidavits of witnesses, documentary
evidence, scientific or other expert studies, and if possible, object evidence; photographing the property or any relevant object or operation thereon.

(e) The certification of petitioner under oath that: (1) petitioner has not commenced any The order shall specify the person or persons authorized to make the inspection and the date,
action or filed any claim involving the same issues in any court, tribunal or quasi-judicial time, place and manner of making the inspection and may prescribe other conditions to
agency, and no such other action or claim is pending therein; (2) if there is such other protect the constitutional rights of all parties.
pending action or claim, a complete statement of its present status; (3) if petitioner should
learn that the same or similar action or claim has been filed or is pending, petitioner shall (b) Production or inspection of documents or things; order - The motion must show that a
report to the court that fact within five (5) days therefrom; and production order is necessary to establish the magnitude of the violation or the threat as to
prejudice the life, health or property of inhabitants in two or more cities or provinces.
(f) The reliefs prayed for which may include a prayer for the issuance of a TEPO.
After hearing, the court may order any person in possession, custody or control of any
Section 3. Where to file. - The petition shall be filed with the Supreme Court or with any of designated documents, papers, books, accounts, letters, photographs, objects or tangible
the stations of the Court of Appeals. things, or objects in digitized or electronic form, which constitute or contain evidence relevant
to the petition or the return, to produce and permit their inspection, copying or photographing
Section 4. No docket fees. - The petitioner shall be exempt from the payment of docket by or on behalf of the movant.
fees.
The production order shall specify the person or persons authorized to make the production
Section 5. Issuance of the writ. - Within three (3) days from the date of filing of the and the date, time, place and manner of making the inspection or production and may
petition, if the petition is sufficient in form and substance, the court shall give an order: (a) prescribe other conditions to protect the constitutional rights of all parties.
issuing the writ; and (b) requiring the respondent to file a verified return as provided in
Section 8 of this Rule. The clerk of court shall forthwith issue the writ under the seal of the Section 13. Contempt. - The court may after hearing punish the respondent who refuses or
court including the issuance of a cease and desist order and other temporary reliefs effective unduly delays the filing of a return, or who makes a false return, or any person who disobeys
until further order. or resists a lawful process or order of the court for indirect contempt under Rule 71 of the
Rules of Court.
189
RECTO, GAYLE ANGELI M.
2011-0008 | AUSL
Personal Notes on Remedial Law 2 Review (based on the syllabus of Prof. Henedino M. Brondial)

Section 14. Submission of case for decision; filing of memoranda. - After hearing, the PART IV
court shall issue an order submitting the case for decision. The court may require the filing of CRIMINAL PROCEDURE
memoranda and if possible, in its electronic form, within a non-extendible period of thirty (30)
days from the date the petition is submitted for decision. RULE 9
PROSECUTION OF OFFENSES
Section 15. Judgment. - Within sixty (60) days from the time the petition is submitted for
decision, the court shall render judgment granting or denying the privilege of the writ of Section 1. Who may file. - Any offended party, peace officer or any public officer charged
kalikasan. with the enforcement of an environmental law may file a complaint before the proper officer in
accordance with the Rules of Court.
The reliefs that may be granted under the writ are the following:
Section 2. Filing of the information. - An information, charging a person with a violation
(a) Directing respondent to permanently cease and desist from committing acts or neglecting of an environmental law and subscribed by the prosecutor, shall be filed with the court.
the performance of a duty in violation of environmental laws resulting in environmental
destruction or damage; Section 3. Special prosecutor. - In criminal cases, where there is no private offended party,
a counsel whose services are offered by any person or organization may be allowed by the
(b) Directing the respondent public official, government agency, private person or entity to court as special prosecutor, with the consent of and subject to the control and supervision of
protect, preserve, rehabilitate or restore the environment; the public prosecutor.

(c) Directing the respondent public official, government agency, private person or entity to RULE 10
monitor strict compliance with the decision and orders of the court; PROSECUTION OF CIVIL ACTIONS

(d) Directing the respondent public official, government agency, or private person or entity to Section 1. Institution of criminal and civil actions. - When a criminal action is instituted,
make periodic reports on the execution of the final judgment; and the civil action for the recovery of civil liability arising from the offense charged, shall be
deemed instituted with the criminal action unless the complainant waives the civil action,
(e) Such other reliefs which relate to the right of the people to a balanced and healthful reserves the right to institute it separately or institutes the civil action prior to the criminal
ecology or to the protection, preservation, rehabilitation or restoration of the environment, action.
except the award of damages to individual petitioners.
Unless the civil action has been instituted prior to the criminal action, the reservation of the
Section 16. Appeal. - Within fifteen (15) days from the date of notice of the adverse right to institute separately the civil action shall be made during arraignment.
judgment or denial of motion for reconsideration, any party may appeal to the Supreme Court
under Rule 45 of the Rules of Court. The appeal may raise questions of fact. In case civil liability is imposed or damages are awarded, the filing and other legal fees shall
be imposed on said award in accordance with Rule 141 of the Rules of Court, and the fees
Section 17. Institution of separate actions. - The filing of a petition for the issuance of shall constitute a first lien on the judgment award. The damages awarded in cases where
the writ of kalikasan shall not preclude the filing of separate civil, criminal or administrative there is no private offended party, less the filing fees, shall accrue to the funds of the agency
actions. charged with the implementation of the environmental law violated. The award shall be used
for the restoration and rehabilitation of the environment adversely affected.
RULE 8
WRIT OF CONTINUING MANDAMUS RULE 11
ARREST
Section 1. Petition for continuing mandamus. - When any agency or instrumentality of
the government or officer thereof unlawfully neglects the performance of an act which the law Section 1. Arrest without warrant; when lawful. - A peace officer or an individual
specifically enjoins as a duty resulting from an office, trust or station in connection with the deputized by the proper government agency may, without a warrant, arrest a person:
enforcement or violation of an environmental law rule or regulation or a right therein, or
unlawfully excludes another from the use or enjoyment of such right and there is no other (a) When, in his presence, the person to be arrested has committed, is actually committing or
plain, speedy and adequate remedy in the ordinary course of law, the person aggrieved is attempting to commit an offense; or
thereby may file a verified petition in the proper court, alleging the facts with certainty,
attaching thereto supporting evidence, specifying that the petition concerns an environmental (b) When an offense has just been committed, and he has probable cause to believe based on
law, rule or regulation, and praying that judgment be rendered commanding the respondent to personal knowledge of facts or circumstances that the person to be arrested has committed it.
do an act or series of acts until the judgment is fully satisfied, and to pay damages sustained Individuals deputized by the proper government agency who are enforcing environmental laws
by the petitioner by reason of the malicious neglect to perform the duties of the respondent, shall enjoy the presumption of regularity under Section 3(m), Rule 131 of the Rules of Court
under the law, rules or regulations. The petition shall also contain a sworn certification of non- when effecting arrests for violations of environmental laws.
forum shopping.
Section 2. Warrant of arrest. - All warrants of arrest issued by the court shall be
Section 2. Where to file the petition. - The petition shall be filed with the Regional Trial accompanied by a certified true copy of the information filed with the issuing court.
Court exercising jurisdiction over the territory where the actionable neglect or omission
occurred or with the Court of Appeals or the Supreme Court. RULE 12
CUSTODY AND DISPOSITION OF SEIZED ITEMS, EQUIPMENT,
Section 3. No docket fees. - The petitioner shall be exempt from the payment of docket PARAPHERNALIA, CONVEYANCES AND INSTRUMENTS
fees.
Section 1. Custody and disposition of seized items. - The custody and disposition of
Section 4. Order to comment. - If the petition is sufficient in form and substance, the court seized items shall be in accordance with the applicable laws or rules promulgated by the
shall issue the writ and require the respondent to comment on the petition within ten (10) concerned government agency.
days from receipt of a copy thereof. Such order shall be served on the respondents in such
manner as the court may direct, together with a copy of the petition and any annexes thereto. Section 2. Procedure. - In the absence of applicable laws or rules promulgated by the
concerned government agency, the following procedure shall be observed:
Section 5. Expediting proceedings; TEPO. - The court in which the petition is filed may
issue such orders to expedite the proceedings, and it may also grant a TEPO for the (a) The apprehending officer having initial custody and control of the seized items, equipment,
preservation of the rights of the parties pending such proceedings. paraphernalia, conveyances and instruments shall physically inventory and whenever
practicable, photograph the same in the presence of the person from whom such items were
Section 6. Proceedings after comment is filed. - After the comment is filed or the time seized.
for the filing thereof has expired, the court may hear the case which shall be summary in
nature or require the parties to submit memoranda. The petition shall be resolved without (b) Thereafter, the apprehending officer shall submit to the issuing court the return of the
delay within sixty (60) days from the date of the submission of the petition for resolution. search warrant within five (5) days from date of seizure or in case of warrantless arrest,
submit within five (5) days from date of seizure, the inventory report, compliance report,
Section 7. Judgment. - If warranted, the court shall grant the privilege of the writ of photographs, representative samples and other pertinent documents to the public prosecutor
continuing mandamus requiring respondent to perform an act or series of acts until the for appropriate action.
judgment is fully satisfied and to grant such other reliefs as may be warranted resulting from
the wrongful or illegal acts of the respondent. The court shall require the respondent to submit (c) Upon motion by any interested party, the court may direct the auction sale of seized items,
periodic reports detailing the progress and execution of the judgment, and the court may, by equipment, paraphernalia, tools or instruments of the crime. The court shall, after hearing, fix
itself or through a commissioner or the appropriate government agency, evaluate and monitor the minimum bid price based on the recommendation of the concerned government agency.
compliance. The petitioner may submit its comments or observations on the execution of the The sheriff shall conduct the auction.
judgment.
(d) The auction sale shall be with notice to the accused, the person from whom the items
Section 8. Return of the writ. - The periodic reports submitted by the respondent detailing were seized, or the owner thereof and the concerned government agency.
compliance with the judgment shall be contained in partial returns of the writ.
(e) The notice of auction shall be posted in three conspicuous places in the city or municipality
Upon full satisfaction of the judgment, a final return of the writ shall be made to the court by where the items, equipment, paraphernalia, tools or instruments of the crime were seized.
the respondent. If the court finds that the judgment has been fully implemented, the
satisfaction of judgment shall be entered in the court docket.
190
RECTO, GAYLE ANGELI M.
2011-0008 | AUSL
Personal Notes on Remedial Law 2 Review (based on the syllabus of Prof. Henedino M. Brondial)

(f) The proceeds shall be held in trust and deposited with the government depository bank for
disposition according to the judgment. (a) Place the parties and their counsels under oath;

RULE 13 (b) Adopt the minutes of the preliminary conference as part of the pre-trial proceedings,
PROVISIONAL REMEDIES confirm markings of exhibits or substituted photocopies and admissions on the genuineness
and due execution of documents, and list object and testimonial evidence;
Section 1. Attachment in environmental cases. - The provisional remedy of attachment
under Rule 127 of the Rules of Court may be availed of in environmental cases. (c) Scrutinize the information and the statements in the affidavits and other documents which
form part of the record of the preliminary investigation together with other documents
Section 2. Environmental Protection Order (EPO); Temporary Environmental identified and marked as exhibits to determine further admissions of facts as to:
Protection Order (TEPO) in criminal cases. - The procedure for and issuance of EPO and
TEPO shall be governed by Rule 2 of these Rules. i. The court’s territorial jurisdiction relative to the offense(s) charged;

RULE 14 ii. Qualification of expert witnesses; and


BAIL
iii. Amount of damages;
Section 1. Bail, where filed. - Bail in the amount fixed may be filed with the court where
the case is pending, or in the absence or unavailability of the judge thereof, with any regional (d) Define factual and legal issues;
trial judge, metropolitan trial judge, municipal trial judge or municipal circuit trial judge in the
province, city or municipality. If the accused is arrested in a province, city or municipality other (e) Ask parties to agree on the specific trial dates and adhere to the flow chart determined by
than where the case is pending, bail may also be filed with any Regional Trial Court of said the court which shall contain the time frames for the different stages of the proceeding up to
place, or if no judge thereof is available, with any metropolitan trial judge, municipal trial promulgation of decision;
judge or municipal circuit trial judge therein. If the court grants bail, the court may issue a
hold-departure order in appropriate cases. (f) Require the parties to submit to the branch clerk of court the names, addresses and
contact numbers of witnesses that need to be summoned by subpoena; and
Section 2. Duties of the court. - Before granting the application for bail, the judge must
read the information in a language known to and understood by the accused and require the (g) Consider modification of order of trial if the accused admits the charge but interposes a
accused to sign a written undertaking, as follows: lawful defense.

(a) To appear before the court that issued the warrant of arrest for arraignment purposes on Section 4. Manner of questioning. - All questions or statements must be directed to the
the date scheduled, and if the accused fails to appear without justification on the date of court.
arraignment, accused waives the reading of the information and authorizes the court to enter
a plea of not guilty on behalf of the accused and to set the case for trial; Section 5. Agreements or admissions. - All agreements or admissions made or entered
during the pre-trial conference shall be reduced in writing and signed by the accused and
(b) To appear whenever required by the court where the case is pending; and counsel; otherwise, they cannot be used against the accused. The agreements covering the
matters referred to in Section 1, Rule 118 of the Rules of Court shall be approved by the court.
(c) To waive the right of the accused to be present at the trial, and upon failure of the accused
to appear without justification and despite due notice, the trial may proceed in absentia. Section 6. Record of proceedings. - All proceedings during the pre-trial shall be recorded,
the transcripts prepared and the minutes signed by the parties or their counsels.
RULE 15
ARRAIGNMENT AND PLEA Section 7. Pre-trial order. - The court shall issue a pre-trial order within ten (10) days after
the termination of the pre-trial, setting forth the actions taken during the pre-trial conference,
Section 1. Arraignment. - The court shall set the arraignment of the accused within fifteen the facts stipulated, the admissions made, evidence marked, the number of witnesses to be
(15) days from the time it acquires jurisdiction over the accused, with notice to the public presented and the schedule of trial. The order shall bind the parties and control the course of
prosecutor and offended party or concerned government agency that it will entertain plea- action during the trial.
bargaining on the date of the arraignment.
RULE 17
Section 2. Plea-bargaining. - On the scheduled date of arraignment, the court shall TRIAL
consider plea-bargaining arrangements. Where the prosecution and offended party or
concerned government agency agree to the plea offered by the accused, the court shall: Section 1. Continuous trial. - The court shall endeavor to conduct continuous trial which
shall not exceed three (3) months from the date of the issuance of the pre-trial order.
(a) Issue an order which contains the plea-bargaining arrived at;
Section 2. Affidavit in lieu of direct examination. - Affidavit in lieu of direct examination
(b) Proceed to receive evidence on the civil aspect of the case, if any; and shall be used, subject to cross-examination and the right to object to inadmissible portions of
the affidavit.
(c) Render and promulgate judgment of conviction, including the civil liability for damages.
Section 3. Submission of memoranda. - The court may require the parties to submit their
RULE 16 respective memoranda and if possible, in electronic form, within a non-extendible period of
PRE-TRIAL thirty (30) days from the date the case is submitted for decision.

Section 1. Setting of pre-trial conference. - After the arraignment, the court shall set the With or without any memoranda filed, the court shall have a period of sixty (60) days to
pre-trial conference within thirty (30) days. It may refer the case to the branch clerk of court, decide the case counted from the last day of the 30-day period to file the memoranda.
if warranted, for a preliminary conference to be set at least three (3) days prior to the pre-
trial. Section 4. Disposition period. - The court shall dispose the case within a period of ten (10)
months from the date of arraignment.
Section 2. Preliminary conference. - The preliminary conference shall be for the following
purposes: Section 5. Pro bono lawyers. - If the accused cannot afford the services of counsel or there
is no available public attorney, the court shall require the Integrated Bar of the Philippines to
(a) To assist the parties in reaching a settlement of the civil aspect of the case; provide pro bono lawyers for the accused.

(b) To mark the documents to be presented as exhibits; RULE 18


SUBSIDIARY LIABILITY
(c) To attach copies thereof to the records after comparison with the originals;
Section 1. Subsidiary liability. - In case of conviction of the accused and subsidiary liability
(d) To ascertain from the parties the undisputed facts and admissions on the genuineness and is allowed by law, the court may, by motion of the person entitled to recover under judgment,
due execution of documents marked as exhibits; enforce such subsidiary liability against a person or corporation subsidiary liable under Article
102 and Article 103 of the Revised Penal Code.
(e) To consider such other matters as may aid in the prompt disposition of the case;
RULE 19
(f) To record the proceedings during the preliminary conference in the Minutes of Preliminary STRATEGIC LAWSUIT AGAINST PUBLIC PARTICIPATION IN CRIMINAL CASES
Conference to be signed by the parties and counsel;
Section 1. Motion to dismiss. - Upon the filing of an information in court and before
(g) To mark the affidavits of witnesses which shall be in question and answer form and shall arraignment, the accused may file a motion to dismiss on the ground that the criminal action is
constitute the direct examination of the witnesses; and a SLAPP.

(h) To attach the Minutes and marked exhibits to the case record before the pre-trial proper. Section 2. Summary hearing. - The hearing on the defense of a SLAPP shall be summary in
The parties or their counsel must submit to the branch clerk of court the names, addresses nature. The parties must submit all the available evidence in support of their respective
and contact numbers of the affiants. positions. The party seeking the dismissal of the case must prove by substantial evidence that
his acts for the enforcement of environmental law is a legitimate action for the protection,
Section 3. Pre-trial duty of the judge. - During the pre-trial, the court shall:
191
RECTO, GAYLE ANGELI M.
2011-0008 | AUSL
Personal Notes on Remedial Law 2 Review (based on the syllabus of Prof. Henedino M. Brondial)

preservation and rehabilitation of the environment. The party filing the action assailed as a o 3rd party complaint
SLAPP shall prove by preponderance of evidence that the action is not a SLAPP. x Procedure:
o File complaint
Section 3. Resolution. - The court shall grant the motion if the accused establishes in the
summary hearing that the criminal case has been filed with intent to harass, vex, exert undue o Summons
pressure or stifle any legal recourse that any person, institution or the government has taken
o File answer Æ 15 days
or may take in the enforcement of environmental laws, protection of the environment or ƒ If no answer Æ court shall declare (motu
assertion of environmental rights. proprio) the defendant in default, receive
evidence ex parte, and render judgment
If the court denies the motion, the court shall immediately proceed with the arraignment of
based on the pleadings
the accused.
o Mediation Æ within 30 days from receipt of notice of
PART V referral to mediation
EVIDENCE ƒ Mediation report Æ within 10 days from the
expiration of the 30-day period
RULE 20 o Preliminary conference Æ conducted by the BC when
PRECAUTIONARY PRINCIPLE
mediation fails
Section 1. Applicability. - When there is a lack of full scientific certainty in establishing a o Pre-trial
causal link between human activity and environmental effect, the court shall apply the ƒ Consent Decree Æ a judicially-approved
precautionary principle in resolving the case before it. settlement between the parties based on public
interest and public policy to protect and
The constitutional right of the people to a balanced and healthful ecology shall be given the preserve the environment
benefit of the doubt.
ƒ Evidence NOT presented during pre-trial are
Section 2. Standards for application. - In applying the precautionary principle, the waived
following factors, among others, may be considered: (1) threats to human life or health; (2) o Trial proper
inequity to present or future generations; or (3) prejudice to the environment without legal ƒ Environmental Protection Order (EPO) Æ
consideration of the environmental rights of those affected. an order issued by the court directing or
enjoining any person of government agency
RULE 21
to perform or desist from performing an act in
DOCUMENTARY EVIDENCE order to protect, preserve or rehabilitate the
environment
Section 1. Photographic, video and similar evidence. - Photographs, videos and similar ƒ Temporary Environmental Protection
evidence of events, acts, transactions of wildlife, wildlife by-products or derivatives, forest Order (TEPO) Æ where the issuance of the
products or mineral resources subject of a case shall be admissible when authenticated by the EPO is a matter of extreme urgency and the
person who took the same, by some other person present when said evidence was taken, or by
applicant will suffer grave injustice and
any other person competent to testify on the accuracy thereof.
irreparable injury
Section 2. Entries in official records. - Entries in official records made in the performance x Author’s note: THUS, this is akin
of his duty by a public officer of the Philippines, or by a person in performance of a duty to the preparatory writ of TRO
specially enjoined by law, are prima facie evidence of the facts therein stated. x Issued by the EJ in a multiple-
sala court or the PJ in a single-sala
RULE 22
court
FINAL PROVISIONS
x Valid only for 72 hours from
Section 1. Effectivity. - These Rules shall take effect within fifteen (15) days following receipt of the person enjoined
publication once in a newspaper of general circulation. x After the issuance of a TEPO, a
hearing must be conducted for the
Section 2. Application of the Rules of Court. - The Rules of Court shall apply in a resolution of the issuance of an
suppletory manner, except as otherwise provided herein.
EPO
x Bond is NOT necessary
ƒ Strategic Lawsuit Against Public
NOTES (Based on the Annotations of Riguera and Albano; includes
Author’s own comments and research) Participation (SLAPP) Æ a legal action filed
to harass vex, exert undue pressure or stifle
x Applicability of Rules of Procedure for Environmental Cases (RPEC) Æ any legal recourse that any person,
civil, criminal, and special civil actions pending before MTC and RTC institution, or the government has taken or
involving enforcement or violations of environmental and other may take in the enforcement of environment
related laws, rules and regulations laws, protection of the environment or
x Venue AND jurisdiction (for issuance of writ of kalikasan) Æ where assertion of environmental rights
the principal case is pending x This may be used as a defense by
x Plaintiff (in any civil action) Æ any real party in interest the defendant
o May also be in the form of a class suit for the protection o Supported by
of the minors and unborn generation (Oposa vs. documents, affidavits,
Factoran) papers, and other
x SC Æ the only court which may issue a TRO or WPI against lawful evidence
actions of the government enforcing environmental laws o May be coupled with a
x Allowed pleadings under RPEC: counterclaim for atty’s
o Complaint fees etc
x If SLAPP defense is invoked in the
o Answer (which may include compulsory counterclaims)
o Cross claim Answer, the court shall:
o Motion for intervention o Require the plaintiff to
o Motion for discovery file his Comment/
o MR of judgment x Opposition to the
Prohibited pleadings: SLAPP defense within
o MD (any ground) 5 days from receipt of
o Motion for bill of particulars such order
o Motion for extension of time for filing of pleadings o Set the hearing re
ƒ EXCEPT filing of an ANSWER Æ which SLAPP defense within
extension shall not exceed 15 days 15 days from the filing
o Motion to declare defendant in default of the Comment/
o Reply Opposition
o Rejoinder
192
RECTO, GAYLE ANGELI M.
2011-0008 | AUSL
Personal Notes on Remedial Law 2 Review (based on the syllabus of Prof. Henedino M. Brondial)

o Render a decision administrative appeals followed by appeal to CA via Rule


thereon within 30 days 43 then appeal to SC via Rule 45. By way of exceptions,
from the summary the same may be challenged in such petition when (1) the
hearing (this dismissal environmental damage is of such magnitude as that
is with prejudice) described in the Rules on Writ of Kalikasan and (2) when
x NATURE of the HEARING when the doctrine of exhaustion of administrative remedies is
there is a SLAPP defense Æ not violated. (Paje vs. Casino)
summary x Writ of Continuing Mandamus Æ a remedy resorted to when an
x Quantum of evidence on the part agency or government instrumentality unlawfully neglects to perform
of the defendant (to prove that a duty enjoined by law in connection with the enforcement or
the action is a SLAPP) Æ violation of an environmental law, rule, or right therein OR when
substantial evidence such agency or government instrumentality unlawfully excludes
x Quantum of evidence on the part another from the use or enjoyment of such right where there is no
of the plaintiff (to prove that the other PAS remedy
action is NOT a SLAPP) Æ o Venue Æ where the actionable neglect or omission was
preponderance of evidence committed
o Jurisdiction Æ RTC, CA, SC (concurrent)
o Judgment o When it shall issue Æ within 10 days from filing of the
ƒ Reliefs that may be granted in a citizen suit: petition if the same is sufficient in form and substance
x Protection, rehabilitation, o Hearing Æ summary OR court may require submission of
preservation of environment memoranda
x Payment of atty’s fees
x Costs of suit and other litgation CASE
expensesCompel the violator to
submit a program re restoration of MARICRIS D. DOLOT, CHAIRMAN OF THE BAGONG ALYANSANG MAKABAYAN-SORSOGON,
PETITIONER vs. HON. RAMON PAJE, IN HIS CAPACITY AS THE SECRETARY OF THE
the environment OR to contribute
DEPARTMENT OF ENVIRONMENT AND NATURAL RESOURCES, REYNULFO A. JUAN, REGIONAL
to a special trust fund for that DIRECTOR, MINES AND GEOSCIENCES BUREAU, DENR, HON. RAUL R. LEE, GOVERNOR,
purpose PROVINCE OF SORSOGON, ANTONIO C. OCAMPO, JR., VICTORIA A. AJERO, ALFREDO M.
ƒ The judgment for the protection, AGUILAR, AND JUAN M. AGUILAR, ANTONES ENTERPRISES, GLOBAL SUMMIT MINES DEV'T
rehabilitation, preservation of environment is CORP., AND TR ORE, RESPONDENTS.
IMMEDIATELY EXECUTORY G.R. No. 199199 August 27, 2013
EN BANC
x Unless restrained by the appellate
court (via WPI) FACTS:
ƒ The judgment may CONVERT the TEPO into x Maricris Dolot et al had protested sometime in 2009 the iron ore mining
an EPO or issue a writ of continuing operations being conducted by Antones Enterprises, Global Summit Mines
mandamus Development Corporation and TR Ore in Barangays Balocawe and Bon-ot Daco,
o Execution of Judgment located in the Municipality of Matnog, which was located in the southern tip of
Luzon
ƒ Means to Monitor Execution o The protest was futile
x Court may require (by itself or x Dolot, together with the parish priest of the Holy Infant Jesus Parish and the
another government agency) the officers of Alyansa Laban sa Mina sa Matnog Æ filed a petition for continuing
violator to submit written reports mandamus, damages and attorney’s fees before RTC Sorsogon
on a quarterly basis or sooner re o there is a need to protect, preserve and maintain the geological
progress of execution foundation of the municipality
o Matnog is susceptible to flooding and landslides, and confronted
x Winning party may submit
with the environmental dangers of flood hazard, liquefaction,
comments/ observations on the ground settlement, ground subsidence and landslide hazard
execution of judgment o the mining operators did not have the required permit to operate
x Writ of Kalikasan Æ remedy available to any person/ entity/ any o Sorsogon Governor Raul Lee and his predecessor Sally Lee did not
public interest group which is accredited involving environmental have the authority to issue a small-scale mining permit in favor of
damage of such magnitude as to prejudice the life, health or property the operators
o the representatives of the Presidential Management Staff and the
of inhabitants in 2 or more cities or provinces Department of Environment and Natural Resources (DENR), despite
o Verified petition knowledge, did not do anything to protect the interest of the
o DOES NOT ipso facto restrain the respondent people of Matnog
ƒ UNLESS a TEPO is issued o the respondents violated Republic Act (R.A.) No. 7076 or the
ƒ The latter is required to file a verified return People’s Small-Scale Mining Act of 1991, R.A. No. 7942 or the
within 10 days from the receipt of the writ Philippine Mining Act of 1995, and the Local Government Code.6
Thus, they prayed for the following reliefs
x This is in the nature of a
x The prayers in the petition are as follows:
responsive pleading o (1) the issuance of a writ commanding the respondents to
x Effect of general denial Æ deemed immediately stop the mining operations in the Municipality of
as an admission Matnog;
o (2) the issuance of a temporary environment protection order or
o When it shall issue Æ within 3 days from filing of the
petition if the same is sufficient in form and substance TEPO;
o (3) the creation of an inter-agency group to undertake the
o Interim Reliefs: rehabilitation of the mining site;
ƒ Ocular Inspection Order Æ by (litigated) o (4) award of damages; and
motion; for the purpose of inspecting the o (5) return of the iron ore, among others
magnitude of the environmental damage x The case was referred to the EJ of RTC Sorsogon, his court being that which was
ƒ Production or Inspection of Documents designated as environmental court
of Things Order Æ by (litigated) motion; for x RTC Æ dismissed for lack of jurisdiction
x Dolot et al Æ filed an MR
the purpose of establishing the magnitude of x RTC Æ denied the MR
the environmental damage o (1) there was no final court decree, order or decision yet that the
o Judgment Æ within 60 days from the filing of the public officials allegedly failed to act on, which is a condition for the
petition issuance of the writ of continuing mandamus;
ƒ REMEDY of the losing party Æ MR or appeal o (2) the case was prematurely filed as the petitioners therein failed
via Rule 45 within 15 days from receipt of the to exhaust their administrative remedies; and
o (3) they also failed to attach judicial affidavits and furnish a copy of
judgment the complaint to the government or appropriate agency, as
o An Environmental Compliance Certificate (ECC) may not required by the rules
be questioned in a petition for writ of kalikasan because x Dolot et al Æ filed a Rule 45 before the SC
the proper remedy is the DENR appeal process, viz,
193
RECTO, GAYLE ANGELI M.
2011-0008 | AUSL
Personal Notes on Remedial Law 2 Review (based on the syllabus of Prof. Henedino M. Brondial)

ISSUE # 1: Whether the petition was correctly filed before the RTC Sorsogon. decree, order or decision yet that the public officials allegedly failed to act on;
(2) the case was prematurely filed for failure to exhaust administrative remedies;
HELD # 1: YES. (As to JURISDICTION - BP 129 provides that RTC has original jurisdiction over and (3) there was failure to attach judicial affidavits and furnish a copy of the
mandamus petitions. As to VENUE - while venue in this case might have been incorrect under complaint to the government or appropriate agency.29 The respondents, and
AM 09-6-8, venue may be waived) even the Office of the Solicitor General, in behalf of the public respondents, all
x In dismissing the petition for lack of jurisdiction, the RTC, in its Order dated concur with the view of the RTC.
September 16, 2011, apparently relied on SC Administrative Order (A.O.) No. 7 x The concept of continuing mandamus was first introduced in Metropolitan Manila
defining the territorial areas of the Regional Trial Courts in Regions 1 to 12, and Development Authority v. Concerned Residents of Manila Bay.30 Now cast in
Administrative Circular (Admin. Circular) No. 23-2008,13 designating the stone under Rule 8 of the Rules, the writ of continuing mandamus enjoys a
environmental courts "to try and decide violations of environmental laws x x x distinct procedure than that of ordinary civil actions for the
committed within their respective territorial jurisdictions."14 Thus, it ruled that its enforcement/violation of environmental laws, which are covered by
territorial jurisdiction was limited within the boundaries of Sorsogon City and the Part II (Civil Procedure). Similar to the procedure under Rule 65 of the Rules
neighboring municipalities of Donsol, Pilar, Castilla, Casiguran and Juban and that of Court for special civil actions for certiorari, prohibition and mandamus, Section
it was "bereft of jurisdiction to entertain, hear and decide [the] case, as such 4, Rule 8 of the Rules requires that the petition filed should be
authority rests before another co-equal court."15 sufficient in form and substance before a court may take further
x Such reasoning is plainly erroneous. The RTC cannot solely rely on SC A.O. action; otherwise, the court may dismiss the petition outright. Courts must be
No. 7 and Admin. Circular No. 23-2008 and confine itself within its four cautioned, however, that the determination to give due course to the petition or
corners in determining whether it had jurisdiction over the action filed dismiss it outright is an exercise of discretion that must be applied in a
by the petitioners. reasonable manner in consonance with the spirit of the law and always with the
x None is more well-settled than the rule that jurisdiction, which is the power view in mind of seeing to it that justice is served.31
and authority of the court to hear, try and decide a case, is conferred x Sufficiency in form and substance refers to the contents of the petition filed
by law.16 It may either be over the nature of the action, over the subject under Rule 8, Section 1:
matter, over the person of the defendants or over the issues framed in o When any agency or instrumentality of the government or officer
the pleadings.17 By virtue of Batas Pambansa (B.P.) Blg. 129 or the Judiciary thereof unlawfully neglects the performance of an act which the
Reorganization Act of 1980, jurisdiction over special civil actions for certiorari, law specifically enjoins as a duty resulting from an office, trust or
prohibition and mandamus is vested in the RTC. Particularly, Section 21(1) station in connection with the enforcement or violation of an
thereof provides that the RTCs shall exercise original jurisdiction - environmental law rule or regulation or a right therein, or
o in the issuance of writs of certiorari, prohibition, unlawfully excludes another from the use or enjoyment of such
mandamus, quo warranto, habeas corpus and injunction right and there is no other plain, speedy and adequate remedy in
which may be enforced in any part of their respective the ordinary course of law, the person aggrieved thereby may file a
regions. (Emphasis ours) verified petition in the proper court, alleging the facts with
x A.O. No. 7 and Admin. Circular No. 23-2008 was issued pursuant to Section 18 of certainty, attaching thereto supporting evidence, specifying that the
B.P. Blg. 129, which gave the Court authority to define the territory over which a petition concerns an environmental law, rule or regulation, and
branch of the RTC shall exercise its authority. These administrative orders and praying that judgment be rendered commanding the respondent to
circulars issued by the Court merely provide for the venue where an action may do an act or series of acts until the judgment is fully satisfied, and
be filed. The Court does not have the power to confer jurisdiction on any to pay damages sustained by the petitioner by reason of the
court or tribunal as the allocation of jurisdiction is lodged solely in malicious neglect to perform the duties of the respondent, under
Congress.18 It also cannot be delegated to another office or agency of the the law, rules or regulations. The petition shall also contain a sworn
Government.19 Section 18 of B.P. Blg. 129, in fact, explicitly states that the certification of non-forum shopping.1âwphi1
territory thus defined shall be deemed to be the territorial area of the branch x On matters of form, the petition must be verified and must contain
concerned for purposes of determining the venue of all suits, proceedings or supporting evidence as well as a sworn certification of non-forum
actions. It was also clarified in Office of the Court Administrator v. Judge shopping. It is also necessary that the petitioner must be one who is
Matas20 that - aggrieved by an act or omission of the government agency,
o Administrative Order No. 3 [defining the territorial jurisdiction of instrumentality or its officer concerned.
the Regional Trial Courts in the National Capital Judicial Region] x Sufficiency of substance, on the other hand, necessitates that the petition must
and, in like manner, Circular Nos. 13 and 19, did not per se confer contain substantive allegations specifically constituting an actionable neglect or
jurisdiction on the covered regional trial courts or its branches, omission and must establish, at the very least, a prima facie basis for the
such that non-observance thereof would nullify their judicial acts. issuance of the writ, viz:
The administrative order merely defines the limits of the o (1) an agency or instrumentality of government or its officer
administrative area within which a branch of the court may exercise unlawfully neglects the performance of an act or unlawfully
its authority pursuant to the jurisdiction conferred by Batas excludes another from the use or enjoyment of a right;
Pambansa Blg. 129.21 o (2) the act to be performed by the government agency,
x The RTC need not be reminded that venue relates only to the place of trial or the instrumentality or its officer is specifically enjoined by law as a
geographical location in which an action or proceeding should be brought and duty;
does not equate to the jurisdiction of the court. It is intended to accord o (3) such duty results from an office, trust or station in connection
convenience to the parties, as it relates to the place of trial, and does not restrict with the enforcement or violation of an environmental law, rule or
their access to the courts.22 Consequently, the RTC’s motu proprio dismissal of regulation or a right therein; and
Civil Case No. 2011-8338 on the ground of lack of jurisdiction is patently o (4) there is no other plain, speedy and adequate remedy in the
incorrect. course of law.32
x At most, the error committed by the petitioners in filing the case with x The writ of continuing mandamus is a special civil action that may be availed of
the RTC of Sorsogon was that of improper venue. A.M. No. 09-6-8-SC "to compel the performance of an act specifically enjoined by law."33
or the Rules of Procedure for Environmental Cases (Rules) specifically The petition should mainly involve an environmental and other related
states that a special civil action for continuing mandamus shall be filed law, rule or regulation or a right therein. The RTC’s mistaken notion on the
with the "[RTC] exercising jurisdiction over the territory where the need for a final judgment, decree or order is apparently based on the definition
actionable neglect or omission occurred x x x."23 In this case, it appears of the writ of continuing mandamus under Section 4, Rule 1 of the Rules, to wit:
that the alleged actionable neglect or omission occurred in the Municipality of o (c) Continuing mandamus is a writ issued by a court in an
Matnog and as such, the petition should have been filed in the RTC of Irosin.24 environmental case directing any agency or instrumentality of the
But even then, it does not warrant the outright dismissal of the petition government or officer thereof to perform an act or series of acts
by the RTC as venue may be waived.25 Moreover, the action filed by the decreed by final judgment which shall remain effective until
petitioners is not criminal in nature where venue is an essential element of judgment is fully satisfied. (Emphasis ours)
jurisdiction.26 In Gomez-Castillo v. Commission on Elections,27 the Court even x The final court decree, order or decision erroneously alluded to by the
expressed that what the RTC should have done under the circumstances RTC actually pertains to the judgment or decree that a court would
was to transfer the case (an election protest) to the proper branch. eventually render in an environmental case for continuing mandamus
Similarly, it would serve the higher interest of justice28 if the Court orders the and which judgment or decree shall subsequently become final.
transfer of Civil Case No. 2011 8338 to the RTC of Irosin for proper and speedy x Under the Rules, after the court has rendered a judgment in conformity with
resolution, with the RTC applying the Rules in its disposition of the case. Rule 8, Section 7 and such judgment has become final, the issuing court still
x At this juncture, the Court affirms the continuing applicability of Admin. Circular retains jurisdiction over the case to ensure that the government agency
No. 23-2008 constituting the different "green courts" in the country and setting concerned is performing its tasks as mandated by law and to monitor the
the administrative guidelines in the raffle and disposition of environmental cases. effective performance of said tasks. It is only upon full satisfaction of the final
While the designation and guidelines were made in 2008, the same judgment, order or decision that a final return of the writ shall be made to the
should operate in conjunction with the Rules. court and if the court finds that the judgment has been fully implemented, the
satisfaction of judgment shall be entered in the court docket.34 A writ of
ISSUE # 2: Whether the resort to the remedy of continuing mandamus was proper, continuing mandamus is, in essence, a command of continuing
considering the position of the RTC that a final decision or order is a condition precedent compliance with a final judgment as it "permits the court to retain
thereto. jurisdiction after judgment in order to ensure the successful
implementation of the reliefs mandated under the court’s decision."35
HELD # 2: YES.
x In its Resolution dated October 18, 2011, which resolved the petitioners’ motion ISSUE # 3: Whether there is a need to file the present case before the Panel of Arbitrators, in
for reconsideration of the order of dismissal, the RTC further ruled that the compliance with the doctrine of exhaustion of remedies.
petition was dismissible on the following grounds: (1) there is no final court
194
RECTO, GAYLE ANGELI M.
2011-0008 | AUSL
Personal Notes on Remedial Law 2 Review (based on the syllabus of Prof. Henedino M. Brondial)

HELD # 3: NO. x Tubbataha, declared as National Marine Park by virtue of Proclamation No. 306
x The Court, likewise, cannot sustain the argument that the petitioners should issued by President Corazon C. Aquino on August 11, 1988, is ocated in the
have first filed a case with the Panel of Arbitrators (Panel), which has jurisdiction middle of Central Sulu Sea, 150 kilometers southeast of Puerto Princesa City and
over mining disputes under R.A. No. 7942. lies at the heart of the Coral Triangle, the global center of marine biodiversity.
x Indeed, as pointed out by the respondents, the Panel has jurisdiction over o Republic Act (R.A.) No. 10067,3 otherwise known as the
mining disputes.36 But the petition filed below does not involve a mining "Tubbataha Reefs Natural Park (TRNP) Act of 2009" Æ passed on
dispute. What was being protested are the alleged negative environmental April 6, 2010
impact of the small-scale mining operation being conducted by Antones ƒ "to ensure the protection and conservation of the
Enterprises, Global Summit Mines Development Corporation and TR Ore in the globally significant economic, biological, sociocultural,
Municipality of Matnog; the authority of the Governor of Sorsogon to issue educational and scientific values of the Tubbataha
mining permits in favor of these entities; and the perceived indifference of the Reefs into perpetuity for the enjoyment of present
DENR and local government officials over the issue. and future generations."
x Resolution of these matters does not entail the technical knowledge ƒ Under the "no-take" policy, entry into the waters of
and expertise of the members of the Panel but requires an exercise of TRNP is strictly regulated and many human activities
judicial function. Thus, in Olympic Mines and Development Corp. v. Platinum are prohibited and penalized or fined, including
Group Metals Corporation,37 the Court stated - fishing, gathering, destroying and disturbing the
o Arbitration before the Panel of Arbitrators is proper only when there resources within the TRNP.
is a disagreement between the parties as to some provisions of the ƒ The law likewise created the Tubbataha Protected
contract between them, which needs the interpretation and the Area Management Board (TPAMB) which shall be the
application of that particular knowledge and expertise possessed by sole policy-making and permit-granting body of the
members of that Panel. It is not proper when one of the parties TRNP.
repudiates the existence or validity of such contract or agreement on x US Embassy in the Philippines Æ requested diplomatic clearance for the vessel
the ground of fraud or oppression as in this case. The validity of the called USS Guardian, an Avenger-class mine countermeasures ship of the US
contract cannot be subject of arbitration proceedings. Navy, "to enter and exit the territorial waters of the Philippines and to arrive at
Allegations of fraud and duress in the execution of a contract are the port of Subic Bay for the purpose of routine ship replenishment,
matters within the jurisdiction of the ordinary courts of law. These maintenance, and crew liberty."
questions are legal in nature and require the application and o It had a brief stop for fuel in Okinawa, Japan.
interpretation of laws and jurisprudence which is necessarily a o Then arrived in Subic Bay
judicial function.38 (Emphasis supplied in the former and ours in o Departed for its next port of call in Makassar, Indonesia
the latter) x While transiting the Sulu Sea, the ship ran aground on the northwest side of
x Consequently, resort to the Panel would be completely useless and unnecessary. South Shoal of the Tubbataha Reefs, about 80 miles east-southeast of Palawan.
No cine was injured in the incident, and there have been no reports of leaking
ISSUE # 4: Whether the Judicial Affidavit Rule is applicable in petitions for continuing fuel or oil
mandamus. x US’ actions:
o U.S. 7th Fleet Commander, Vice Admiral Scott Swift Æ expressed
HELD # 4: NO. regret for the incident in a press statement.
x The Court also finds that the RTC erred in ruling that the petition is infirm for o US Ambassador to the Philippines Harry K. Thomas, Jr., in a
failure to attach judicial affidavits. As previously stated, Rule 8 requires that meeting at the Department of Foreign Affairs (DFA) Æ "reiterated
the petition should be verified, contain supporting evidence and must his regrets over the grounding incident and assured Foreign Affairs
be accompanied by a sworn certification of non-forum shopping. There Secretazy Albert F. del Rosario that the United States will provide
is nothing in Rule 8 that compels the inclusion of judicial affidavits, appropriate compensation for damage to the reef caused by the
albeit not prohibited. It is only if the evidence of the petitioner would consist ship.
of testimony of witnesses that it would be the time that judicial affidavits o US Navy-led salvage team Æ finished removing the last piece of
(affidavits of witnesses in the question and answer form) must be attached to the grounded ship from the coral reef.
the petition/complaint.39 x Herein petitioners (on their behalf and in representation of their respective
sector/organization and others, including minors or generations yet unborn) Æ
ISSUE # 5: Whether the failure of Dolot et al to furnish the respondents a copy of their filed the present petition before the SC for issuance of a Writ of Kalikasan with
petition is fatal. prayer for the issuance of a Temporary Environmental Protection Order (TEPO)
under Rule 7 of A.M. No. 09-6-8-SC, otherwise known as the Rules of Procedure
HELD # 5: NO. for Environmental Cases (Rules), involving violations of environmental laws and
x Finally, failure to furnish a copy of the petition to the respondents is not a fatal regulations in relation to the grounding of the US military ship USS Guardian over
defect such that the case should be dismissed. The RTC could have just the Tubbataha Reefs against herein respondents
required the petitioners to furnish a copy of the petition to the o the grounding, salvaging and post-salvaging operations of the USS
respondents. It should be remembered that "courts are not enslaved by Guardian cause and continue to cause environmental damage of
technicalities, and they have the prerogative to relax compliance with procedural such magnitude as to affect the provinces of Palawan, Antique,
rules of even the most mandatory character, mindful of the duty to reconcile both Aklan, Guimaras, Iloilo, Negros Occidental, Negros Oriental,
the need to speedily put an end to litigation and the parties’ right to an Zamboanga del Norte, Basilan, Sulu, and Tawi-Tawi, which events
opportunity to be heard." violate their constitutional rights to a balanced and healthful
ecology
o also seek a directive from this Court for the institution of civil,
(GAR’s note: only issue numbers 5 and 6 relate to Remedial Law re criminal and civil aspects administrative and criminal suits for acts committed in violation of
of a petition for issuance of a writ of kalikasan; the rest are Political Law topics re State environmental laws and regulations in connection with the
immunity, the UNCLOS, VFA, and the political question doctrine) grounding incident.
x Only the PH respondents filed their Comment (and NOT the US respondents)
MOST REV. PEDRO D. ARIGO, Vicar Apostolic of Puerto Princesa D.D.; MOST REV. o In their consolidated comment with opposition to the application for
DEOGRACIAS S. INIGUEZ, JR., Bishop-Emeritus of Caloocan, FRANCES Q. QUIMPO, a TEPO and ocular inspection and production orders, respondents
CLEMENTE G. BAUTISTA, JR., Kalikasan-PNE, MARIA CAROLINA P. ARAULLO, RENATO M. assert that:
REYES, JR., Bagong Alyansang Makabayan, HON. NERI JAVIER COLMENARES, Bayan Muna ƒ ( 1) the grounds relied upon for the issuance of a
Partylist, ROLAND G. SIMBULAN, PH.D., Junk VF A Movement, TERESITA R. PEREZ, PH.D., TEPO or writ of Kalikasan have become fait accompli
HON. RAYMOND V. PALATINO, Kabataan Party-list, PETER SJ. GONZALES, Pamalakaya, as the salvage operations on the USS Guardian were
GIOVANNI A. TAPANG, PH. D., Agham, ELMER C. LABOG, Kilusang Mayo Uno, JOAN MAY E. already completed;
SALVADOR, Gabriela, JOSE ENRIQUE A. AFRICA, THERESA A. CONCEPCION, MARY JOAN A. ƒ (2) the petition is defective in form and substance;
GUAN, NESTOR T. BAGUINON, PH.D., A. EDSEL F. TUPAZ, Petitioners, vs. SCOTT H. SWIFT ƒ (3) the petition improperly raises issues involving the
in his capacity as Commander of the US. 7th Fleet, MARK A. RICE in his capacity as VFA between the Republic of the Philippines and the
Commanding Officer of the USS Guardian, PRESIDENT BENIGNO S. AQUINO III in his capacity United States of America; and
as Commander-in-Chief of the Armed Forces of the Philippines, HON. ALBERT F. DEL ƒ ( 4) the determination of the extent of responsibility
ROSARIO, Secretary, pepartment of Foreign Affair.s, HON. PAQUITO OCHOA, JR., of the US Government as regards the damage to the
Executiv~.:Secretary, Office of the President, . HON. VOLTAIRE T. GAZMIN, Secretary, Tubbataha Reefs rests exdusively with the executive
Department of National Defense, HON. RAMON JESUS P. P AJE, Secretary, Department of branch.
Environment and Natural Resoz!rces, VICE ADMIRAL JOSE LUIS M. ALANO, Philippine Navy Flag x Petitioners Æ filed a motion for early resolution and motion to proceed ex parte
Officer in Command, Armed Forces of the Philippines, ADMIRAL RODOLFO D. ISO RENA, against the US respondents.
Commandant, Philippine Coast Guard, COMMODORE ENRICO EFREN EVANGELISTA, Philippine
Coast Guard Palawan, MAJOR GEN. VIRGILIO 0. DOMINGO, Commandant of Armed Forces of ISSUE # 1: Whether herein petitioners have legal standing to institute the present petition.
the Philippines Command and LT. GEN. TERRY G. ROBLING, US Marine Corps Forces. Pacific
and Balikatan 2013 Exercise Co-Director, Respondents. HELD # 1: YES.
G.R. No. 206510 September 16, 2014 x Locus standi is "a right of appearance in a court of justice on a given
EN BANC question."10 Specifically, it is "a party's personal and substantial interest in a
case where he has sustained or will sustain direct injury as a result" of the act
FACTS: being challenged, and "calls for more than just a generalized grievance."11
However, the rule on standing is a procedural matter which this Court has
195
RECTO, GAYLE ANGELI M.
2011-0008 | AUSL
Personal Notes on Remedial Law 2 Review (based on the syllabus of Prof. Henedino M. Brondial)

relaxed for non-traditional plaintiffs like ordinary citizens, taxpayers and legislators x Under the American Constitution, the doctrine is expressed in the Eleventh
when the public interest so requires, such as when the subject matter of the Amendment which reads:
controversy is of transcendental importance, of overreaching significance to o The Judicial power of the United States shall not be construed to
society, or of paramount public interest.12 extend to any suit in law or equity, commenced or prosecuted
x In the landmark case of Oposa v. Factoran, Jr.,13 we recognized the "public against one of the United States by Citizens of another State, or by
right" of citizens to "a balanced and healthful ecology which, for the Citizens or Subjects of any Foreign State.
first time in our constitutional history, is solemnly incorporated in the x In the case of Minucher v. Court of Appeals,20 we further expounded on the
fundamental law." We declared that the right to a balanced and immunity of foreign states from the jurisdiction of local courts, as follows:
healthful ecology need not be written in the Constitution for it is o The precept that a State cannot be sued in the courts of a foreign
assumed, like other civil and polittcal rights guaranteed in the Bill of state is a long-standing rule of customary international law then
Rights, to exist from the inception of mankind and it is an issue of closely identified with the personal immunity of a foreign sovereign
transcendental importance with intergenerational implications.1âwphi1 from suit and, with the emergence of democratic states, made to
Such right carries with it the correlative duty to refrain from impairing the attach not just to the person of the head of state, or his
environment.14 representative, but also distinctly to the state itself in its sovereign
x On the novel element in the class suit filed by the petitioners minors in Oposa, capacity. If the acts giving rise to a suit arc those of a foreign
this Court ruled that not only do ordinary citizens have legal standing to sue for government done by its foreign agent, although not necessarily a
the enforcement of environmental rights, they can do so in representation of diplomatic personage, but acting in his official capacity, the
their own and future generations. Thus: complaint could be barred by the immunity of the foreign sovereign
o Petitioners minors assert that they represent their generation as from suit without its consent. Suing a representative of a state is
well as generations yet unborn. We find no difficulty in ruling that believed to be, in effect, suing the state itself. The proscription is
they can, for themselves, for others of their generation and for the not accorded for the benefit of an individual but for the State, in
succeeding generations, file a class suit. Their personality to sue in whose service he is, under the maxim -par in parem, non habet
behalf of the succeeding generations can only be based on the imperium -that all states are soverr~ign equals and cannot assert
concept of intergenerational responsibility insofar as the right to a jurisdiction over one another. The implication, in broad terms, is
balanced and healthful ecology is concerned. Such a right, as that if the judgment against an official would rec 1uire the state
hereinafter expounded, considers the "rhythm and harmony of itself to perform an affirmative act to satisfy the award, such as the
nature." Nature means the created world in its entirety. Such appropriation of the amount needed to pay the damages decreed
rhythm and harmony indispensably include, inter alia, the judicious against him, the suit must be regarded as being against the state
disposition, utilization, management, renewal and conservation of itself, although it has not been formally impleaded.21 (Emphasis
the country's forest, mineral, land, waters, fisheries, wildlife, off- supplied.)
shore areas and other natural resources to the end that their x In the same case we also mentioned that in the case of diplomatic immunity, the
exploration, development and utilization be equitably accessible to privilege is not an immunity from the observance of the law of the territorial
the present a:: well as future generations. Needless to say, every sovereign or from ensuing legal liability; it is, rather, an immunity from the
generation has a responsibility to the next to preserve that rhythm exercise of territorial jurisdiction.22
and harmony for the full 1:njoyment of a balanced and healthful x In United States of America v. Judge Guinto,23 one of the consolidated cases
ecology. Put a little differently, the minors' assertion of their right therein involved a Filipino employed at Clark Air Base who was arrested following a
to a sound environment constitutes, at the same time, the buy-bust operation conducted by two officers of the US Air Force, and was
performance of their obligation to ensure the protection of that eventually dismissed from his employment when he was charged in court for
right for the generations to come.15 (Emphasis supplied.) violation of R.A. No. 6425. In a complaint for damages filed by the said employee
x The liberalization of standing first enunciated in Oposa, insofar as it against the military officers, the latter moved to dismiss the case on the ground
refers to minors and generations yet unborn, is now enshrined in the that the suit was against the US Government which had not given its consent.
Rules which allows the filing of a citizen suit in environmental cases. The RTC denied the motion but on a petition for certiorari and prohibition filed
The provision on citizen suits in the Rules "collapses the traditional rule on before this Court, we reversed the RTC and dismissed the complaint. We held
personal and direct interest, on the principle that humans are stewards of that petitioners US military officers were acting in the exercise of their official
nature."1 functions when they conducted the buy-bust operation against the complainant
and thereafter testified against him at his trial. It follows that for discharging
ISSUE # 2: Whether SC acquired jurisdiction over the persons of the US respondents their duties as agents of the United States, they cannot be directly impleaded for
considering that they did not submit any pleading or manifestation. acts imputable to their principal, which has not given its consent to be sued.
x This traditional rule of State immunity which exempts a State from being sued in
HELD # 2: NO. (pursuant to the doctrine of State Immunity which extends to those persons the courts of another State without the former's consent or waiver has evolved
acting as agents of their State) into a restrictive doctrine which distinguishes sovereign and governmental acts
x The immunity of the State from suit, known also as the doctrine of sovereign (Jure imperil") from private, commercial and proprietary acts (Jure gestionis).
immunity or non-suability of the State,17 is expressly provided in Article XVI of Under the restrictive rule of State immunity, State immunity extends only to acts
the 1987 Constitution which states: Jure imperii. The restrictive application of State immunity is proper only when
o Section 3. The State may not be sued without its consent. the proceedings arise out of commercial transactions of the foreign sovereign, its
x In United States of America v. Judge Guinto,18 we discussed the principle of commercial activities or economic affairs.24
state immunity from suit, as follows: x In Shauf v. Court of Appeals,25 we discussed the limitations of the State
o The rule that a state may not be sued without its consent, immunity principle, thus:
now · expressed in Article XVI, Section 3, of the 1987 o It is a different matter where the public official is made to account
Constitution, is one of the generally accepted principles of in his capacity as such for acts contrary to law and injurious to the
international law that we have adopted as part of the law rights of plaintiff. As was clearly set forth by JustiGe Zaldivar in
of our land under Article II, Section 2. x x x. Director of the Bureau of Telecommunications, et al. vs. Aligaen,
x Even without such affirmation, we would still be bound by the generally etc., et al. : "Inasmuch as the State authorizes only legal acts by its
accepted principles of international law under the doctrine of officers, unauthorized acts of government officials or officers are
incorporation. Under this doctrine, as accepted by the majority of states, such not acts of the State, and an action against the officials or officers
principles are deemed incorporated in the law of every civilized state as a by one whose rights have been invaded or violated by such acts,
condition and consequence of its membership in the society of nations. Upon its for the protection of his rights, is not a suit against the State within
admission to such society, the state is automatically obligated to comply with the rule of immunity of the State from suit. In the same tenor, it
these principles in its relations with other states. has been said that an action at law or suit in equity against a State
x As applied to the local state, the doctrine of state immunity is based on the officer or the director of a State department on the ground that,
justification given by Justice Holmes that ''there can be no legal right while claiming to act for the State, he violates or invades the
against the authority which makes the law on which the right personal and property rights of the plaintiff, under an
depends." [Kawanakoa v. Polybank, 205 U.S. 349] There are other practical unconstitutional act or under an assumption of authority which he
reasons for the enforcement of the doctrine. In the case of the foreign state does not have, is not a suit against the State within the
sought to be impleaded in the local jurisdiction, the added inhibition is expressed constitutional provision that the State may not be sued without its
in the maxim par in parem, non habet imperium. All states are sovereign equals consent." The rationale for this ruling is that the doctrine of state
and cannot assert jurisdiction over one another. A contrary disposition would, in immunity cannot be used as an instrument for perpetrating an
the language of a celebrated case, "unduly vex the peace of nations." [De Haber injustice.
v. Queen of Portugal, 17 Q. B. 171] o The aforecited authorities are clear on the matter. They state that
x While the doctrine appears to prohibit only suits against the state without its the doctrine of immunity from suit will not apply and may not be
consent, it is also applicable to complaints filed against officials of the invoked where the public official is being sued in his private and
state for acts allegedly performed by them in the discharge of their personal capacity as an ordinary citizen. The cloak of protection
duties. The rule is that if the judgment against such officials will require the afforded the officers and agents of the government is removed the
state itself to perform an affirmative act to satisfy the same,. such as the moment they are sued in their individual capacity. This situation
appropriation of the amount needed to pay the damages awarded against them, usually arises where the public official acts without authority or in
the suit must be regarded as against the state itself although it has not been excess of the powers vested in him. It is a well-settled principle of
formally impleaded. [Garcia v. Chief of Staff, 16 SCRA 120] In such a situation, law that a public official may be liable in his personal private
the state may move to dismiss the comp.taint on the ground that it has been capacity for whatever damage he may have caused by his act done
filed without its consent.19 (Emphasis supplied.) with malice and in bad faith, or beyond the scope of his authority or
jurisdiction.26 (Emphasis supplied.) In this case, the US
196
RECTO, GAYLE ANGELI M.
2011-0008 | AUSL
Personal Notes on Remedial Law 2 Review (based on the syllabus of Prof. Henedino M. Brondial)

respondents were sued in their official capacity as commanding contained in Part XI. In a remarkable, multilateral effort to induce U.S.
officers of the US Navy who had control and supervision over the membership, the bulk of UNCLOS member states cooperated over the
USS Guardian and its crew. The alleged act or omission resulting in succeeding decade to revise the objection.able provisions. The revisions satisfied
the unfortunate grounding of the USS Guardian on the TRNP was the Clinton administration, which signed the revised Part XI implementing
committed while they we:re performing official military duties. agreement in 1994. In the fall of 1994, President Clinton transmitted UNCLOS and
Considering that the satisfaction of a judgment against said officials the Part XI implementing agreement to the Senate requesting its advice and
will require remedial actions and appropriation of funds by the US consent. Despite consistent support from President Clinton, each of his
government, the suit is deemed to be one against the US itself. The successors, and an ideologically diverse array of stakeholders, the Senate has
principle of State immunity therefore bars the exercise of since withheld the consent required for the President to internationally bind the
jurisdiction by this Court over the persons of respondents Swift, United States to UNCLOS.
Rice and Robling.
x During the deliberations, Senior Associate Justice Antonio T. Carpio took the x While UNCLOS cleared the Senate Foreign Relations Committee (SFRC) during
position that the conduct of the US in this case, when its warship entered a the 108th and 110th Congresses, its progress continues to be hamstrung by
restricted area in violation of R.A. No. 10067 and caused damage to the TRNP significant pockets of political ambivalence over U.S. participation in international
reef system, brings the matter within the ambit of Article 31 of the United institutions. Most recently, 111 th Congress SFRC Chairman Senator John Kerry
Nations Convention on the Law of the Sea (UNCLOS). He explained that while included "voting out" UNCLOS for full Senate consideration among his highest
historically, warships enjoy sovereign immunity from suit as extensions of their priorities. This did not occur, and no Senate action has been taken on UNCLOS
flag State, Art. 31 of the UNCLOS creates an exception to this rule in cases by the 112th Congress.34
where they fail to comply with the rules and regulations of the coastal State x Justice Carpio invited our attention to the policy statement given by President
regarding passage through the latter's internal waters and the territorial sea. Reagan on March 10, 1983 that the US will "recognize the rights of the other ,
states in the waters off their coasts, as reflected in the convention [UNCLOS], so
ISSUE # 3: Whether the US is bound to make reparations to PH, considering the former’s non- long as the rights and freedom of the United States and others under
ratification of the UNCLOS. international law are recognized by such coastal states", and President Clinton's
reiteration of the US policy "to act in a manner consistent with its [UNCLOS]
HELD # 3: YES. provisions relating to traditional uses of the oceans and to encourage other
x According to Justice Carpio, although the US to date has not ratified the countries to do likewise." Since Article 31 relates to the "traditional uses of the
UNCLOS, as a matter of long-standing policy the US considers itself bound by oceans," and "if under its policy, the US 'recognize[s] the rights of the other
customary international rules on the "traditional uses of the oceans" as codified states in the waters off their coasts,"' Justice Carpio postulates that "there is
in UNCLOS, as can be gleaned from previous declarations by former Presidents more reason to expect it to recognize the rights of other states in their internal
Reagan and Clinton, and the US judiciary in the case of United States v. Royal waters, such as the Sulu Sea in this case."
Caribbean Cruise Lines, Ltd.27 x As to the non-ratification by the US, Justice Carpio emphasizes that
x The international law of the sea is generally defined as "a body of treaty rules "the US' refusal to join the UN CLOS was centered on its disagreement
arid customary norms governing the uses of the sea, the exploitation of its with UN CLOS' regime of deep seabed mining (Part XI) which considers
resources, and the exercise of jurisdiction over maritime regimes. It is a branch of the oceans and deep seabed commonly owned by mankind," pointing out
public international law, regulating the relations of states with respect to the uses that such "has nothing to do with its [the US'] acceptance of
of the oceans."28 The UNCLOS is a multilateral treaty which was opened for customary international rules on navigation."
signature on December 10, 1982 at Montego Bay, Jamaica. It was ratified by the x It may be mentioned that even the US Navy Judge Advocate General's Corps
Philippines in 1984 but came into force on November 16, 1994 upon the publicly endorses the ratification of the UNCLOS, as shown by the following
submission of the 60th ratification. statement posted on its official website:
x The UNCLOS is a product of international negotiation that seeks to balance State o The Convention is in the national interest of the United States
sovereignty (mare clausum) and the principle of freedom of the high seas (mare because it establishes stable maritime zones, including a maximum
liberum).29 The freedom to use the world's marine waters is one of the oldest outer limit for territorial seas; codifies innocent passage, transit
customary principles of international law.30 The UNCLOS gives to the coastal passage, and archipelagic sea lanes passage rights; works against
State sovereign rights in varying degrees over the different zones of the sea "jurisdictiomtl creep" by preventing coastal nations from expanding
which are: 1) internal waters, 2) territorial sea, 3) contiguous zone, 4) exclusive their own maritime zones; and reaffirms sovereign immunity of
economic zone, and 5) the high seas. It also gives coastal States more or less warships, auxiliaries anJ government aircraft.
jurisdiction over foreign vessels depending on where the vessel is located.31 o Economically, accession to the Convention would support our
x Insofar as the internal waters and territorial sea is concerned, the Coastal State national interests by enhancing the ability of the US to assert its
exercises sovereignty, subject to the UNCLOS and other rules of international sovereign rights over the resources of one of the largest
law. Such sovereignty extends to the air space over the territorial sea as well as continental shelves in the world. Further, it is the Law of the Sea
to its bed and subsoil.32 Convention that first established the concept of a maritime
x In the case of warships,33 as pointed out by Justice Carpio, they continue to Exclusive Economic Zone out to 200 nautical miles, and recognized
enjoy sovereign immunity subject to the following exceptions: the rights of coastal states to conserve and manage the natural
o Article 30 resources in this Zone.35
ƒ Non-compliance by warships with the laws and x We fully concur with Justice Carpio's view that non-membership in the
regulations of the coastal State UNCLOS does not mean that the US will disregard the rights of the
ƒ If any warship does not comply with the laws and Philippines as a Coastal State over its internal waters and territorial
regulations of the coastal State concerning passage sea. We thus expect the US to bear "international responsibility" under
through the territorial sea and disregards any request Art. 31 in connection with the USS Guardian grounding which adversely affected
for compliance therewith which is made to it, the the Tubbataha reefs. Indeed, it is difficult to imagine that our long-time ally and
coastal State may require it to leave the territorial sea trading partner, which has been actively supporting the country's efforts to
immediately. preserve our vital marine resources, would shirk from its obligation to
o Article 31 compensate the damage caused by its warship while transiting our internal
ƒ Responsibility of the flag State for damage caused by waters. Much less can we comprehend a Government exercising leadership in
a warship or other government ship operated for non- international affairs, unwilling to comply with the UNCLOS directive for all nations
commercial purposes to cooperate in the global task to protect and preserve the marine environment as
ƒ The flag State shall bear international responsibility provided in Article 197, viz:
for any loss or damage to the coastal State resulting o Article 197
from the non-compliance by a warship or other ƒ Cooperation on a global or regional basis
government ship operated for non-commercial ƒ States shall cooperate on a global basis and, as
purposes with the laws and regulations of the coastal appropriate, on a regional basis, directly or through
State concerning passage through the territorial sea or competent international organizations, in formulating
with the provisions of this Convention or other rules of and elaborating international rules, standards and
international law. recommended practices and procedures consistent
o Article 32 with this Convention, for the protection and
ƒ Immunities of warships and other government ships preservation of the marine environment, taking into
operated for non-commercial purposes account characteristic regional features.
x With such exceptions as are contained in subsection A and in articles 30 and 31, x In fine, the relevance of UNCLOS provisions to the present controversy is beyond
nothing in this Convention affects the immunities of warships and other dispute. Although the said treaty upholds the immunity of warships from the
government ships operated for non-commercial purposes. (Emphasis supplied.) jurisdiction of Coastal States while navigating the.latter's territorial sea, the flag
A foreign warship's unauthorized entry into our internal waters with States shall be required to leave the territorial '::;ea immediately if they flout the
resulting damage to marine resources is one situation in which the laws and regulations of the Coastal State, and they will be liable for damages
above provisions may apply. But what if the offending warship is a caused by their warships or any other government vessel operated for non-
non-party to the UNCLOS, as in this case, the US? commercial purposes under Article 31.
x An overwhelming majority - over 80% -- of nation states are now members of
UNCLOS, but despite this the US, the world's leading maritime power, has not ISSUE # 4: Whether the US effectively waived its immunity from suit under the provisions in
ratified it. the VFA.
x While the Reagan administration was instrumental in UNCLOS' negotiation and
drafting, the U.S. delegation ultimately voted against and refrained from signing HELD # 4: NO.
it due to concerns over deep seabed mining technology transfer provisions
197
RECTO, GAYLE ANGELI M.
2011-0008 | AUSL
Personal Notes on Remedial Law 2 Review (based on the syllabus of Prof. Henedino M. Brondial)

x Petitioners argue that there is a waiver of immunity from suit found in the VFA. Guardian from the coral reef. However, we are mindful of the fact that
Likewise, they invoke federal statutes in the US under which agencies of the US the US and Philippine governments both expressed readiness to
have statutorily waived their immunity to any action. Even under the common negotiate and discuss the matter of compensation for the damage
law tort claims, petitioners asseverate that the US respondents are liable for caused by the USS Guardian. The US Embassy has also declared it is
negligence, trespass and nuisance. closely coordinating with local scientists and experts in assessing the
o We are not persuaded. extent of the damage and appropriate methods of rehabilitation.
x The VFA is an agreement which defines the treatment of United States troops x Exploring avenues for settlement of environmental cases is not
and personnel visiting the Philippines to promote "common security interests" proscribed by the Rules. As can be gleaned from the following provisions,
between the US and the Philippines in the region. It provides for the guidelines mediation and settlement are available for the consideration of the parties, and
to govern such visits of military personnel, and further defines the rights of the which dispute resolution methods are encouraged by the court, to wit:
United States and the Philippine government in the matter of criminal o RULE3
jurisdiction, movement of vessel and aircraft, importation and exportation of ƒ SEC. 3. Referral to mediation.-At the start of the
equipment, materials and supplies.36 The invocation of US federal tort laws and pre-trial conference, the court shall inquire from the
even common law is thus improper considering that it is the VF A which governs parties if they have settled the dispute; otherwise, the
disputes involving US military ships and crew navigating Philippine waters in court shall immediately refer the parties or their
pursuance of the objectives of the agreement. counsel, if authorized by their clients, to the Philippine
x As it is, the waiver of State immunity under the VF A pertains only to Mediation Center (PMC) unit for purposes of
criminal jurisdiction and not to special civil actions such as the present mediation. If not available, the court shall refer the
petition for issuance of a writ of Kalikasan. In fact, it can be inferred from case to the clerk of court or legal researcher for
Section 17, Rule 7 of the Rules that a criminal case against a person mediation.
charged with a violation of an environmental law is to be filed ƒ Mediation must be conducted within a non-extendible
separately: period of thirty (30) days from receipt of notice of
o SEC. 17. Institution of separate actions.-The filing of a petition for referral to mediation.
the issuance of the writ of kalikasan shall not preclude the filing of ƒ The mediation report must be submitted within ten
separate civil, criminal or administrative actions. (10) days from the expiration of the 30-day period.
ƒ SEC. 4. Preliminary conference.-If mediation fails,
ISSUE # 5: Whether the SC may properly rule on the application or non-application of the VFA the court will schedule the continuance of the pre-
provisions on criminal jurisdiction as regards the criminal liability of the US respondents. trial. Before the scheduled date of continuance, the
court may refer the case to the branch clerk of court
HELD # 5: NO. for a preliminary conference for the following
purposes:
ISSUE # 6: Whether the SC may award damages in favor of herein petitioners x (a) To assist the parties in reaching a
settlement;
HELD # 6: NO. ƒ SEC. 5. Pre-trial conference; consent decree.-
x In any case, it is our considered view that a ruling on the application or The judge shall put the parties and their counsels
non-application of criminal jurisdiction provisions of the VF A to US under oath, and they shall remain under oath in all
personnel who may be found responsible for the grounding of the USS pre-trial conferences.
Guardian, would be premature and beyond the province of a petition ƒ The judge shall exert best efforts to persuade the
for a writ of Kalikasan. We also find it unnecessary at this point to determine parties to arrive at a settlement of the dispute. The
whether such waiver of State immunity is indeed absolute. In the same vein, judge may issue a consent decree approving the
we cannot grant damages which have resulted from the violation of agreement between the parties in accordance with
environmental laws. The Rules allows the recovery of damages, law, morals, public order and public policy to protect
including the collection of administrative fines under R.A. No. 10067, in the right of the people to a balanced and healthful
a separate civil suit or that deemed instituted with the criminal action ecology.
charging the same violation of an environmental law.37 ƒ SEC. 10. Efforts to settle.- The court shall
o Section 15, Rule 7 enumerates the reliefs which may be granted in endeavor to make the parties to agree to compromise
a petition for issuance of a writ of Kalikasan, to wit: or settle in accordance with law at any stage of the
ƒ SEC. 15. Judgment.-Within sixty (60) days from the proceedings before rendition of judgment.
time the petition is submitted for decision, the court (Underscoring supplied.)
shall render judgment granting or denying the x The Court takes judicial notice of a similar incident in 2009 when a guided-
privilege of the writ of kalikasan. missile cruiser, the USS Port Royal, ran aground about half a mile off the
ƒ The reliefs that may be granted under the writ are Honolulu Airport Reef Runway and remained stuck for four days. After spending
the following: $6.5 million restoring the coral reef, the US government was reported to have
x (a) Directing respondent to permanently paid the State of Hawaii $8.5 million in settlement over coral reef damage
cease and desist from committing acts caused by the grounding.38
or neglecting the performance of a duty in x To underscore that the US government is prepared to pay appropriate
violation of environmental laws compensation for the damage caused by the USS Guardian grounding,
resulting in environmental destruction or the US Embassy in the Philippines has announced the formation of a US
damage; interdisciplinary scientific team which will "initiate discussions with
x (b) Directing the respondent public the Government of the Philippines to review coral reef rehabilitation
official, govemment agency, private options in Tubbataha, based on assessments by Philippine-based
person or entity to protect, preserve, marine scientists." The US team intends to "help assess damage and
rehabilitate or restore the remediation options, in coordination with the Tubbataha Management Office,
environment; appropriate Philippine government entities, non-governmental organizations, and
x (c) Directing the respondent public scientific experts from Philippine universities."39
official, government agency, private x A rehabilitation or restoration program to be implemented at the cost
person or entity to monitor strict of the violator is also a major relief that may be obtained under a
compliance with the decision and judgment rendered in a citizens' suit under the Rules, viz:
orders of the court; o RULES
x (d) Directing the respondent public o SECTION 1. Reliefs in a citizen suit.-If warranted, the court
official, government agency, or private may grant to the plaintiff proper reliefs which shall include the
person or entity to make periodic protection, preservation or rehabilitation of the environment and
reports on the execution of the the payment of attorney's fees, costs of suit and other litigation
final judgment; and expenses. It may also require the violator to submit a program of
x (e) Such other reliefs which relate rehabilitation or restoration of the environment, the costs of which
to the right of the people to a shall be borne by the violator, or to contribute to a special trust
balanced and healthful ecology or fund for that purpose subject to the control of the court.1âwphi1
to the protection, preservation,
rehabilitation or restoration of the
environment, except the award of ISSUE # 7: Whether the SC may properly declare the amount of compensation that should be
damages to individual petitioners. imposed upon the US.
(Emphasis supplied.)
x We agree with respondents (Philippine officials) in asserting that this petition HELD # 7: NO. (this is a political question that should be addressed by the Executive branch)
has become moot in the sense that the salvage operation sought to be x In the light of the foregoing, the Court defers to the Executive Branch
enjoined or restrained had already been accomplished when on the matter of compensation and rehabilitation measures through
petitioners sought recourse from this Court. But insofar as the diplomatic channels. Resolution of these issues impinges on our
directives to Philippine respondents to protect and rehabilitate the relations with another State in the context of common security
coral reef stn icture and marine habitat adversely affected by the interests under the VFA. It is settled that "[t]he conduct of the foreign
grounding incident are concerned, petitioners are entitled to these relations of our government is committed by the Constitution to the executive
reliefs notwithstanding the completion of the removal of the USS and legislative-"the political" --departments of the government, and the propriety
198
RECTO, GAYLE ANGELI M.
2011-0008 | AUSL
Personal Notes on Remedial Law 2 Review (based on the syllabus of Prof. Henedino M. Brondial)

of what may be done in the exercise of this political power is not subject to o HOWEVER, in criminal cases, if the alteration of rules of
judicial inquiry or decision."40 evidence would permit in the reception of a lesser
x On the other hand, we cannot grant the additional reliefs prayed for in
quantum of evidence than what the law required at the
the petition to order a review of the VFA and to nullify certain
immunity provisions thereof. time of the commission of crime, the retroactive
application of the new rule of evidence will be
_________________________________________ unconstitutional for being ex post facto
¾ GR: rules of evidence are found in the RoC
o XPNs:
EVIDENCE ƒ RA 4200 Æ rules on evidence obtained
through wire-tapping
A. PRELIMINARY CONSIDERATION [RULE 128] ƒ Code of Commerce Æ rules on evidentiary
weight of conflicting entries in merchants’
1. Definition/ Meaning and Scope of Evidence books
ƒ CC and RPC Æ both provide for certain
Section 1. Evidence defined. — Evidence is the means, sanctioned by these rules, of
presumptions of law and fact
ascertaining in a judicial proceeding the truth respecting a matter of fact.
ƒ Consti Æ fruit of the poisonous tree doctrine
Section 2. Scope. — The rules of evidence shall be the same in all courts and in all trials ¾ GR: rules of evidence are applicable only in judicial proceeding
and hearings, except as otherwise provided by law or these rules. o XPN: if a particular law specifically on a particular judicial
proceedings adopts the rules of evidence under the RoC
¾ Evidence Æ mode and manner of proving competent facts in judicial ¾ APPLICABILITY Æ Judicial proceedings only
proceedings (Bustos vs. Lucero) o Rule 1 Sec 4 RoC
¾ Not every circumstance which affords an inference as to the truth or ƒ Section 4. In what case not applicable. — These Rules
falsity of a matter alleged is considered as evidence. shall not apply to election cases, land registration, cadastral,
naturalization and insolvency proceedings, and other cases not
o To be considered as such Æ it must be sanctioned by the herein provided for, except by analogy or in a suppletory
Rules character and whenever practicable and convenient.
o INAPPLICABLE IN:
¾ Hence, the ff DO NOT fall within the definition in Sec 1 Rule 128:
o Hearsay evidence ƒ Administrative proceedings
x Administrative bodies are not
o Coerced extrajudicial confession of an accused
o Evidence obtained in violation of the constitutional rights bound by the technical niceties of
even if ultimately shown to correspond to the truth law and procedure and the rules
¾ Proof Æ the result or effect of evidence obtaining in courts of law.
(Samalio vs CA)
Evidence Proof ¾ Application of the Rules on Electronic Evidence:
o A.M. No. 01-7-01-SC (2001)
the probative effect of
medium or means by evidence; the conviction or ƒ Section 2. Cases covered. - These Rules shall
which a fact is proved persuasion of the mind apply to all civil actions and proceedings, as
or disproved resulting from a consideration well as quasi-judicial and administrative cases.
of evidence o A.M. No. 01-7-01-SC (2003 amendment)
ƒ Included criminal proceedings in said
¾ Evidence signifies a relationship between the factum probandum and scope)
the facum probans o But see ruling in the case of Rustan Ang vs. CA (2010):
ƒ Besides, the rules he cites do not apply to the
present criminal action. The Rules on Electronic
Factum Probandum Factum probans
Evidence applies only to civil actions, quasi-
the evidentiary fact/s judicial proceedings, and administrative
the ultimate fact or the by which the factum proceedings. (Author’s note: relate this ruling to the
fact sought to be
established
probandum is to be 2003 amendment on The Rules on Electronic
established Evidence, which added “criminal proceedings” to the
cases where said Rules are to be made applicable.)
refers to the materials
refers to the preposition which establish that o MMC vs. Ssangyong (2007) Æ The ruling of the Appellate
preposition Court is incorrect. R.A. No. 8792, otherwise known as the Electronic
Commerce Act of 2000, considers an electronic data message or an
electronic document as the functional equivalent of a written
¾ Evidence Æ means of ascertaining the truth specifically in judicial document for evidentiary purposes. The Rules on Electronic
proceeding Evidence regards an electronic document as admissible in evidence
o PURPOSE of evidence: to know the truth if it complies with the rules on admissibility prescribed by the Rules
¾ REASON why evidence is required: because of the presumption of Court and related laws, and is authenticated in the manner
prescribed by the said Rules. An electronic document is also the
that the court is not aware of the veracity of the facts involved in the equivalent of an original document under the Best Evidence Rule, if
case it is a printout or output readable by sight or other means, shown
¾ WHEN NOT REQUIRED: to reflect the data accurately. A facsimile is not a genuine and
o When there is no factual issue in the case Æ there is authentic pleading. It is, at best, an exact copy preserving
no need to present evidence because the case presents a all the marks of an original. Without the original, there is
no way of determining on its face whether the facsimile
question of law, hence, may be resolved by the mere
pleading is genuine and authentic and was originally
application of the relevant rules or statutes since there is signed by the party and his counsel. It may, in fact, be a sham
a mandatory judicial notice of the official acts of the pleading
legislature
o When the pleadings in a civil case do not tender an 2. Kinds and Classifications of Evidence
issue of fact Æ there is no trial needed, hence no
presentation of evidence required, and the case is ripe of
adjudication by judgment on the pleadings ¾ Kinds of evidence according to form:
o When there is an agreement between the parties to o Object/ real evidence Æ directly addressed to the
dispense with certain evidence Æ must be in writing senses of the court and consists of tangible things
o When the question involves matters of judicial notice exhibited or demonstrated in open court, in an ocular
and those judicially admitted inspection, or at a place designated by the court for its
¾ Rules on evidence Æ procedural law view and observation of an exhibition, experiment or
o Hence, new rules on evidence are applicable to pending demonstration.
cases at the time of the change in the rule ƒ Ascertainment of controverted fact is made
ƒ Because parties to an action have no vested through the use of different senses of
right in the rules of evidence the presiding magistrate
199
RECTO, GAYLE ANGELI M.
2011-0008 | AUSL
Personal Notes on Remedial Law 2 Review (based on the syllabus of Prof. Henedino M. Brondial)

ƒ Evidence by “autoptic preference” Æ by


proferring or presenting in open court the
evidentiary articles for the observation or Positive Negative
inspection of the tribunal when the witness
o Documentary evidence Æ supplied by written when the witness
states that he did not
Definition affirms that a fact did
instruments or derived from conventional symbols, such or did not occur
see or know the
as letters, by which ideas are represented on material occurrence of a fact
substances
ƒ Sec 2, Rule 130: entitled to lesser
x Section 2. Documentary evidence. weight because there
entitled to greater
— Documents as evidence consist of is a total disclaimer of
weight because the
writing or any material containing personal knowledge,
witness represents of
letters, words, numbers, figures, Weight hence, without any
his personal
symbols or other modes of written given
knowledge of the
representation or
expression offered as proof of their presence or absence
disavowal that the
contents. (n) of a fact
fact in question could
o Testimonial evidence Æ that which is submitted to the or could not have
existed or happened
court through the testimony or deposition of a witness.
¾ Other Classifications of Evidence:

Relevant Material Competent RIANO


evidence directed ¾ GR: Sec 2 Rule 128
evidence having any
value in reason as
to prove a fact in one that is not o XPN: Evidence in Civil Cases vs Evidence in Criminal
Definition tending to prove any
issue as excluded by the Cases
determined by the Rules, a statute or
matter provable in
rules of substantive the Constitution
action
law and pleadings Civil Cases Criminal Cases
Evidence of Guilt
whether the fact Quantum of Preponderance of
that the evidence Evidence beyond Reasonable
intends to prove is Evidence
logical relation of the Doubt
material or not; as
evidentiary fact to the generally, an offer of
to whether a fact is
fact in issue; whether compromise may be
in issue or not -- it
the former tends to offer of compromise is received as evidence
Test is determined by
Effect of not an admission of as an implied
establish the
probability or the substantive law, liability and is not admission of guilt,
Compromise admissible in evidence except in criminal
improbability of the the pleadings, the
latter against the offeror cases involving
pre-trial order and
by the admissions criminal negligence/
or confessions on quasi-offenses
file
does not apply
(generally, there is no
presumption for or
Direct Circumstantial against any party;
Applicability
except when provided accused enjoys the
of
the proof of a fact for by law; example - constitutional right of
Presumption
or fact from which, common carrier is presumption of
of
taken either singly presumed to be at fault innocence
Innocence
or collectively, the when a passenger is
proves the fact in injured in the course of
existence of the
dispute without the transportation by said
particular fact in
aid in interference or carrier
dispute may be
presumption
inferred as a
necessary or
probable
consequence
¾ Doctrine of falsus in uno, falsus in omnibus
o False in one thing, false in everything
o If the testimony of a witness on a material issue is
Cumulative Corroborative
wilfully false and given with an intention to deceive,
the jury may disregard all the witness’ testimony
additional evidence (Hargrave vs. Stockloss)
evidence of the same
of a different
kind and to the same
character to the o Applied when the testimony of a witness may be
state of facts
same point considered unworthy of belief as to all the rest of
his evidence if he is shown to have testified falsely
Prima Facie
in one detail
Conclusive
o NOT an absolute rule
that which, standing o Rarely applied in modern jurisprudence
alone, unexplained class of evidence o Modern trend in jurisprudence favors more flexibility
or uncontradicted, is which the law does
sufficient to maintain not allow to be when the testimony of a witness may be partly
the proposition contradicted believed and partly disbelieved depending on the
affirmed corroborative evidence presented at the trial.
(People vs Negosa)
Primary Secondary
o It is not a positive rule of law or of universal
application. It should not be applied to portions of
that which is inferior the testimony corroborated by other evidence,
that which the law
to the primary particularly where the false portions could be
evidence and is
Definition
regards as affording
permitted by law
innocent mistakes. Moreover, the rule is not
the greatest certainty
only when the best mandatory but merely sanctions a disregard of the
of the fact in question
evidence is not testimony of a witness if the circumstances so
available warrant. To completely disregard all the testimony
of a witness on this ground, his testimony must
Term used Substitutionary
in the Rules
Best Evidence
Evidence have been false as to a material point, and the
witness must have a conscious and deliberate
200
RECTO, GAYLE ANGELI M.
2011-0008 | AUSL
Personal Notes on Remedial Law 2 Review (based on the syllabus of Prof. Henedino M. Brondial)

intention to falsify a material point. (People vs offered at the time


Pacapac) when it is presented for
its view or evaluation
offered at the time offered immediately
OR when the party
When offered when the witness is by proponent before
rests his case and real
called to the stand he rests his case
evidence consists of
objects exhibited in
court

objection to the
Alibi Frame-Up Self-Defense qualification of the
witness -> must be
made at the time he
inherently weak; is called to the stand;
crumbles in the light
must be made either at if witness is qualified,
of positive When the time it is presented other objections ->
identification by a must be made at
objection in an ocular inspection must be raised when
truthful witness; the time it is
should be or demonstration OR the objectionable
negative evidence in formally offered
made when it is formally question is asked or
nature; for this inherently weak offered after the answer is
defense to prosper, it because it can easily
commonly used as given if the
must be be fabricated; objectionable
defense in violations
established that (1) elements are (1)
of the Dangerous features became
accused is in unlawful aggression,
Drugs Act; evidence apparent by reason
another place at the (2) reasonable
must be shown by of such answer
time of the necessity of the
the defense to
commission of the means employed to
overcome the
offense and (2) prevent or repel it,
presumption of
there is physical and (3) lack of
regularity of
impossibility for him sufficient provocation
performance of
to be at the scene of
official duties;
on the part of the a. Relevancy
the crime at the time person defending
of its commission; himself
but this defense is Section 4. Relevancy; collateral matters. — Evidence must have such a relation to the
NOT ALWAYS false fact in issue as to induce belief in its existence or non-existence. Evidence on collateral matters
and without merit shall not be allowed, except when it tends in any reasonable degree to establish the
(reason: no person probability or improbability of the fact in issue
can be in 2 places at
the same time) b. Competency
c. Authenticity
d. Offer
¾ Effect of Delay/ Initial Reluctance in Reporting a Crime
by the witness on their testimony 4. Kinds of Admissibility
o Does NOT render the testimony of such witness
false or incredible a. Conditional
ƒ REASONS: b. Multiple
x People have a natural c. Curative
reticence and abhorrence of
getting involved in a crime x Doctrines/ Rules of Admissibility
x Their fear of reprisal from o Conditional Admissibility
the accused ƒ when the evidence appears to be immaterial
or irrelevant at the time of its offer, such
3. Axioms of Admissibility evidence MAY be received if:
x Other facts may be proved
Section 3. Admissibility of evidence. — Evidence is admissible when it is relevant to the thereafter AND
issue and is not excluded by the law of these rules. o Otherwise, the
evidence given may
¾ 2 requisites for the admissibility of evidence under already be stricken out
Section 3: x There is no bad faith on the part
o Relevancy Æ under Section 4, evidence is relevant of the proponent
if it has a relation to the fact in issue as to induce o Multiple Admissibility
belief in its existence or non-existence ƒ When the evidence presented is relevant AND
o Competency Æ determined by the prevailing competent for 2 or more purposes, then it
exclusionary rules of evidence should be admitted for any or all purposes for
¾ 2 axioms of admissibility according to Prof. Wigmore: which it was offered provided it satisfies all
o None but facts having rational probative value are requirements of law for its admissibility
admissible therefor
ƒ Hence, relevant evidence is any class of ƒ E.g. Æ when, immediately after a fight and
evidence which has rational probative conscious of his impending death, the
value to establish the issue in declarant said that it was he who provoked
controversy the other and seriously wounded his
o All facts having rational probative value are opponent, such declaration may be admitted
admissible UNLESSS some specific rule forbids their as part of his res gestae (dying declaration)
admission and declaration against interest
¾ Admissibility of evidence Æ determined at the time it is offered o Curative Admissibility
to the court (Section 35, Rule 132) ƒ This doctrine treats upon the right of a party
o Objection thereto Æ must be made at the time the to introduce incompetent evidence in his
evidence is offered OR as soon thereafter as the behalf where the court has admitted the same
objection to its admissibility shall have become kind of evidence adduced by the adverse
apparent (otherwise, waived) party
ƒ 3 theories/rules obtaining:
x American Rule Æ the admission of
Object/ Real Testimonial Documentary such incompetent evidence,
Evidence Evidence
Evidence without the objection by the
201
RECTO, GAYLE ANGELI M.
2011-0008 | AUSL
Personal Notes on Remedial Law 2 Review (based on the syllabus of Prof. Henedino M. Brondial)

opponent, does NOT justify such xxx Under Section 2, Rule 11 of the Rules on
opponent in rebutting by similar Electronic Evidence, "Ephemeral electronic
incompetent evidence communications shall be proven by the
x English Rule Æ if a party has testimony of a person who was a party to the
presented inadmissible evidence, same or who has personal knowledge thereof
the adverse party may result to . . . ." xxx In this case, complainant who was
similar inadmissible evidence the recipient of said messages and therefore
x Massachusetts Rule Æthe adverse had personal knowledge thereof testified on
party may be permitted to their contents and import. Respondent
introduce similar incompetent herself admitted that the cellphone number
evidence in order to avoid a plain reflected in complainant’s cellphone from
and unfair prejudice caused by the which the messages originated was hers.
admission of the other party’s ƒ MMC vs. Ssangyong (2007) Æ See page 1
evidence
ƒ Test in determining the application of CASES
curative admissibility:
x Whether the incompetent ONG CHIA, petitioner, vs. REPUBLIC OF THE PHILIPPINES and THE COURT OF APPEALS,
evidence was seasonably objected respondents.
G.R. No. 127240 March 27, 2000
to AND SECOND DIVISION
x Whether regardless of the
objections vel non [or not], the FACTS:
admission of such evidence will x Ong Chia was born in China, arrived in PH when he was 9 years old, stayed in
cause a plain and unfair prejudice PH, married and started his own business herein.
to the party against whom it is x Ong Chia (On July 4, 1989, at the age of 66) Æ filed a verified petition to be
admitted admitted as a Filipino citizen under the Revised Naturalization Law before RTC
o Evidence illegally obtained is not admissible (Stonehill vs. Koronadal, South Cotabato
o He then testified as to his qualifications and presented three
Diokno) witnesses to corroborate his testimony
ƒ How objected: x RTC Æ granted Petitioner’s application for naturalization
x Timely motion OR x SolGen Æ appealed to CA
x Action to suppress o on appeal, contended that petition must fail because (1) petitioner
o Collateral Matters Æ matters other than the facts in issue failed to state all the names by which he is or had been known, (2)
failed to support his petition with the appropriate documentary
and offered as a basis for inference as to the existence or
evidence, among others
non-existence of the facts in issue o in addition to his name of "Ong Chia," he had likewise been known
ƒ GR: Collateral matters are inadmissible since childhood as "Loreto Chia Ong."
ƒ XPN: Collateral matters which are relevant to o annexed income tax returns allegedly filed by petitioner from 1973
the fact in issue, or those which tend in any to 1977 to show that his net income could hardly support himself
reasonable degree to establish the probability and his family
or improbability of the fact in issue, are x CA Æ reversed
o It ruled that due to the importance naturalization cases, the State
admissible is not precluded from raising questions not presented in the lower
x Hence, only irrelevant collateral court and brought up for the first time on appeal.
matters are inadmissible x Ong Chia Æ filed a Rule 45 before the SC
o Circumstantial evidence Æ the evidence of collateral facts o appellate court erred in considering the documents which had
or circumstances from which an inference may be drawn merely been annexed by the State to its appellant's brief and, on
as to the probability or improbability of the facts in the basis of which, justified the reversal of the trial court's decision.
o Not having been presented and formally offered as evidence, they
dispute are mere "scrap(s) of paper devoid of any evidentiary value," so it
ƒ In other words, circumstantial evidence is was argued, because under Rule 132, §34 of the Revised Rules on
evidence of relevant collateral facts Evidence, the court shall consider no evidence which has not been
ƒ Can sustain a judgment if sufficient formally offered.
o In sum:
ƒ Admissibility Æ LOGIC + LAW, and ISSUE: Whether the rules of evidence are applicable in a petition for naturalization in that the
evidence presented by the SolGen before the CA should no longer be admissible, the same
determined by: being presented for the first time on appeal.
x Relevance
x Competence HELD: NO.
ƒ Weight Æ determined by: x The rule on formal offer of evidence (Rule 132, §34) now being invoked
x Judicial evaluation within the by petitioner is clearly not applicable to the present case involving a
petition for naturalization. The only instance when said rules may be applied
guidelines under Rule 133 and
by analogy or suppletorily in such cases is when it is "practicable and
x Decisional rules set forth by the convenient." That is not the case here, since reliance upon the documents
SC presented by the State for the first time on appeal, in fact, appears to be the
o Part IV of the Rules of Court Æ provides for rules of more practical and convenient course of action considering that decisions in
evidence, viz: naturalization proceedings are not covered by the rule on res judicata.
ƒ Rule 128 Æ general provisions Consequently, a final favorable judgment does not preclude the State
ƒ Rule 130 Æ admissibility of evidence from later on moving for a revocation of the grant of naturalization on
the basis of the same documents.
ƒ Rules 131 and 129 Æ burden of proof and x Petitioner claims that as a result of the failure of the State to present and
what need not be proved formally offer its documentary evidence before the trial court, he was denied the
ƒ Rule 132 Æ presentation of evidence right to object against their authenticity, effectively depriving him of his
ƒ Rule 133 Æ weight and sufficiency of fundamental right to procedural due process. We are not persuaded. Indeed, the
evidence reason for the rule prohibiting the admission of evidence which has not been
formally offered is to afford the opposite party the chance to object to their
o Electronic Evidence
admissibility. Petitioner cannot claim that he was deprived of the right to
ƒ Nuez vs Apao (2005) Æ The text messages object to the authenticity of the documents submitted to the appellate
were properly admitted by the Committee court by the State
since the same are now covered by Section x Furthermore, the Court notes that these documents — namely, the petition in
1(k), Rule 2 of the Rules on Electronic SCN Case No. 031767, petitioner's marriage contract, the joint affidavit executed
Evidence which provides: "Ephemeral by him and his wife, and petitioner's income tax returns — are all public
documents. As such, they have been executed under oath. They are thus
electronic communication" refers to telephone
reliable. Since petitioner failed to make a satisfactory showing of any flaw or
conversations, text messages . . . and other irregularity that may cast doubt on the authenticity of these documents, it is our
electronic forms of communication the conclusion that the appellate court did not err in relying upon them.
evidence of which is not recorded or retained. x CA decision was affirmed.
202
RECTO, GAYLE ANGELI M.
2011-0008 | AUSL
Personal Notes on Remedial Law 2 Review (based on the syllabus of Prof. Henedino M. Brondial)

o Anita Wania and fifteen year old Beverly Deneng (prosecution


CECILIA ZULUETA, petitioner, vs. COURT OF APPEALS and ALFREDO MARTIN, witnesses) Æ that at 10AM, they stopped by the house of Isabel,
respondents. the grandmother of the victim; that they saw the appellant at the
G.R. No. 107383 February 20, 1996 back of the said house; that, upon inquiry, appellant told them that
SECOND DIVISION he was there to get lumber to bring to the house of his mother
o Isabel Dawang (prosecution witness) Æ that she arrived home in
FACTS: the same evening and saw the lifeless body of Kathylyn at the
x Cecilia, wife of PR Alfredo Martin, forcibly took from the latter’s office certain second floor, with multiple stab wounds
documents in the presence of his mother, driver and secretary o Semen was later on found inside the genitals of Kathylyn and that,
o Said documents were allegedly communications between PR and through Deoxyribonucleic acid (DNA) test, the same was identical
his paramours which Petitioner may use as evidence in a case for the semen to be that of appellants gene type.
legal separation and for disqualification from the practice of x An automatic review by the SC pursuant to Article 47 of the Revised Penal Code,
medicine which she had filed against PR as amended, ensued
x Alfredo Æ then instituted an action against Cecilia for specific performance o Yatar maintained used the defense of denial and alibi.
before RTC Manila o Yatar contended, among others, that the witnesses’ credibility was
o seeking to recover the said documents and to make the latter pay questionable and that the taking of his semen for the purpose of
damages subjecting the same to DNA test and presenting the result before
x RTC Æ in favor of Alfredo the court was violative of his constitutional right against self-
o declared that the documents were properties of the PR and ordered incrimination and that the resort thereto is tantamount to an ex
Petitioner to return the same and to pay damages post facto aw.
x Cecilia Æ appealed to CA
x CA Æ affirmed ISSUE # 1: Whether the credibility of the witnesses may be ruled upon by the Supreme Court.
x Cecilia Æ filed a Rule 45 before the SC
o Cited Alfredo Martin v. Alfonso Felix, Jr. where the Court held that HELD # 1: NO.
the documents and papers (which were also forcibly taken from the x The issue regarding the credibility of the prosecution witnesses should be
owner) were admitted in evidence, hence, the use of the same did resolved against appellant. This Court will not interfere with the judgment of the
not constitute malpractice or gross misconduct on the part of the trial court in determining the credibility of witnesses unless there appears in the
lawyer who used the same record some fact or circumstance of weight and influence which has been
overlooked or the significance of which has been misinterpreted. Well-
ISSUE: Whether the subject documents, which were forcibly taken from their owner, herein entrenched is the rule that the findings of the trial court on credibility of
PR, may be admitted in evidence against the latter in a case for legal separation and witnesses are entitled to great weight on appeal unless cogent reasons
disqualification from the practice of medicine filed by his wife. are presented necessitating a reexamination if not the disturbance of
the same; the reason being that the former is in a better and unique position of
HELD: NO. hearing first hand the witnesses and observing their deportment, conduct and
x Petitioner's contention has no merit. The case against Atty. Felix, Jr. was for attitude. Absent any showing that the trial judge overlooked, misunderstood, or
disbarment. Among other things, private respondent, Dr. Alfredo Martin, as misapplied some facts or circumstances of weight which would affect the result
complainant in that case, charged that in using the documents in evidence, Atty. of the case, the trial judges assessment of credibility deserves the appellate
Felix, Jr. committed malpractice or gross misconduct because of the injunctive courts highest respect. Where there is nothing to show that the witnesses for the
order of the trial court. prosecution were actuated by improper motive, their testimonies are entitled to
x Thus, the acquittal of Atty. Felix, Jr. in the administrative case amounts to no full faith and credit.
more than a declaration that his use of the documents and papers for the
purpose of securing Dr. Martin's admission as to their genuiness and authenticity ISSUE # 2: Whether the circumstance of the DNA test result has probative value as to the
did not constitute a violation of the injunctive order of the trial court. By no guilt of the appellant.
means does the decision in that case establish the admissibility of the documents
and papers in question. HELD # 2: YES.
x Indeed the documents and papers in question are inadmissible in x DNA is a molecule that encodes the genetic information in all living organisms. A
evidence. The constitutional injunction declaring "the privacy of persons DNA is the same in each cell and it does not change throughout a
communication and correspondence [to be] inviolable" is no less person’s lifetime; the DNA in a person’s blood is the same as the DNA found in his
applicable simply because it is the wife (who thinks herself aggrieved saliva, sweat, bone, the root and shaft of hair, earwax, mucus, urine, skin tissue,
by her husband's infidelity) who is the party against whom the and vaginal and rectal cells. Most importantly, because of polymorphisms in
constitutional provision is to be enforced. The only exception to the human genetic structure, no two individuals have the same DNA, with the notable
prohibition in the Constitution is if there is a "lawful order [from a] court or when exception of identical twins.
public safety or order requires otherwise, as prescribed by law." Any violation of x DNA evidence collected from a crime scene can link a suspect to a crime or
this provision renders the evidence obtained inadmissible "for any purpose in any eliminate one from suspicion in the same principle as fingerprints are used.
proceeding." Incidents involving sexual assault would leave biological evidence such as hair,
x The intimacies between husband and wife do not justify any one of them in skin tissue, semen, blood, or saliva which can be left on the victim’s body or at
breaking the drawers and cabinets of the other and in ransacking them for any the crime scene. Hair and fiber from clothing, carpets, bedding, or furniture
telltale evidence of marital infidelity. A person, by contracting marriage, does not could also be transferred to the victim’s body during the assault. Forensic DNA
shed his/her integrity or his right to privacy as an individual and the evidence is helpful in proving that there was physical contact between an
constitutional protection is ever available to him or to her. assailant and a victim. If properly collected from the victim, crime scene or
x The law insures absolute freedom of communication between the spouses by assailant, DNA can be compared with known samples to place the suspect at the
making it privileged. Neither husband nor wife may testify for or against the scene of the crime.
other without the consent of the affected spouse while the marriage x In assessing the probative value of DNA evidence, courts should consider,
subsists. Neither may be examined without the consent of the other as to any inter alia, the following factors: how the samples were collected, how
communication received in confidence by one from the other during the they were handled, the possibility of contamination of the samples, the
marriage, save for specified exceptions. But one thing is freedom of procedure followed in analyzing the samples, whether the proper
communication; quite another is a compulsion for each one to share what one standards and procedures were followed in conducting the tests, and
knows with the other. And this has nothing to do with the duty of fidelity that the qualification of the analyst who conducted the tests.
each owes to the other. x In the case at bar, Dr. Maria Corazon Abogado de Ungria was duly qualified by
x Petition was denied. the prosecution as an expert witness on DNA print or identification techniques.
Based on Dr. de Ungrias testimony, it was determined that the gene type and
PEOPLE OF THE PHILIPPINES, Appellee, v. JOEL YATAR alias "KAWIT", Appellant. DNA profile of appellant are identical to that of the extracts subject of
G.R. NO. 150224 May 19, 2004 examination. The blood sample taken from the appellant showed that he was of
EN BANC the following gene types: vWA 15/19, TH01 7/8, DHFRP2 9/10 and CSF1PO
10/11, which are identical with semen taken from the victims vaginal canal. Verily,
FACTS: a DNA match exists between the semen found in the victim and the blood sample
x Yatar was convicted by RTC Bulanao, Tabuk, Kalinga, Branch 25 with rape with given by the appellant in open court during the course of the trial.
homicide and sentenced him to death x Admittedly, we are just beginning to integrate these advances in science and
x RTC’s findings: technology in the Philippine criminal justice system, so we must be cautious as
o Judilyn Pas-a (prosecution witness) Æ that at 8:30AM on the day we traverse these relatively uncharted waters. Fortunately, we can benefit from
when the incident occurred she was with the victim, her cousin the wealth of persuasive jurisprudence that has developed in other jurisdictions.
seventeen year old Kathylyn Uba at the house of their Specifically, the prevailing doctrine in the U.S. has proven instructive.
grandmother, Isabel; that at 9AM, she left the said house; Kathylyn x In Daubert v. Merrell Dow, it was ruled that pertinent evidence based on
told her that she may be going to Tuguegarao or staying at home scientifically valid principles could be used as long as it was relevant
and washing the clothes instead; that at 12:30PM, she saw the and reliable. Judges, under Daubert, were allowed greater discretion over
appellant, wearing a white shirt with collar and black pants, which testimony they would allow at trial, including the introduction of new kinds
descending from the 2nd floor to the back of the house of Isabel; of scientific techniques. DNA typing is one such novel procedure.
that at 1:30PM, she saw the appellant again, wearing a black shirt x Under Philippine law, evidence is relevant when it relates directly to a fact
without collar and blue pants, near her house and told her that he in issue as to induce belief in its existence or non-existence. Applying the
will no longer be getting the lumber that he had stacked Daubert test to the case at bar, the DNA evidence obtained through PCR
203
RECTO, GAYLE ANGELI M.
2011-0008 | AUSL
Personal Notes on Remedial Law 2 Review (based on the syllabus of Prof. Henedino M. Brondial)

testing and utilizing STR analysis, and which was appreciated by the x RTC decision was affirmed (with modification as to civil liability)
court a quo is relevant and reliable since it is reasonably based on
scientifically valid principles of human genetics and molecular biology.
x In an attempt to exclude the DNA evidence, the appellant contends that the NENA LAZALITA* TATING, Petitioner, vs. FELICIDAD TATING MARCELLA, represented
blood sample taken from him as well as the DNA tests were conducted in violation by SALVADOR MARCELLA, CARLOS TATING, and the COURT OF APPEALS, Respondents.
of his right to remain silent as well as his right against self-incrimination under G.R. No. 155208 March 27, 2007
Secs. 12 and 17 of Art. III of the Constitution. THIRD DIVISION
o This contention is untenable. The kernel of the right is not
against all compulsion, but against testimonial compulsion. FACTS:
The right against self- incrimination is simply against the legal x A parcel of land denominated as Lot 56 of Subdivision plan Psd-31182, located at
process of extracting from the lips of the accused an admission of Abelarde St., Cadiz City, Negros Occidental, was owned by one Daniela Solano
guilt. It does not apply where the evidence sought to be excluded Vda. de Tating (Daniela)
is not an incrimination but as part of object evidence. o Daniela sold the property to granddaughter, Nena
x Appellant further argues that the DNA tests conducted by the prosecution ƒ Registered under the name of Nena and declared by
against him are unconstitutional on the ground that resort thereto is tantamount her for tax purposes
to the application of an ex-post facto law. o However, the land remained in possession of Daniela.
o This argument is specious. No ex-post facto law is involved in the o Daniela then executed a sworn statement claiming that she had
case at bar. The science of DNA typing involves the admissibility, actually no intention of selling the property; the true agreement
relevance and reliability of the evidence obtained under the Rules between her and Nena was simply to transfer title over the subject
of Court. Whereas an ex-post facto law refers primarily to a property in favor of the latter to enable her to obtain a loan by
question of law, DNA profiling requires a factual determination of mortgaging the subject property for the purpose of helping her
the probative weight of the evidence presented. defray her business expenses; she later discovered that Nena did
not secure any loan nor mortgage the property; she wants the title
ISSUE # 3: Whether appellant’s defense of denial and alibi may be sustained. in the name of Nena cancelled and the subject property reconveyed
to her
HELD # 3: NO. x Daniela died
x Appellants twin defense of denial and alibi cannot be sustained. The forensic o leaving her children as her heirs, namely:
DNA evidence and bloodied shirt, notwithstanding the eyewitness accounts of his ƒ Ricardo,
presence at Isabel Dawangs house during the time when the crime was ƒ Felicidad,
committed, undeniably link him to the June 30, 1998 incident. Appellant did not ƒ Julio,
demonstrate with clear and convincing evidence an impossibility to be in two ƒ Carlos and
places at the same time, especially in this case where the two places are located ƒ Cirilo who predeceased Daniela and was represented
in the same barangay.40 He lives within a one hundred (100) meter radius from by Nena (Cirilo’s daughter)
the scene of the crime, and requires a mere five minute walk to reach one house x Carlos Æ demanded the return of the land (on the basis of the executed sworn
from the other. This fact severely weakens his alibi statement of Daniela)
x Generally, courts should only consider and rely upon duly established evidence x Carlos and Felicidad (represented by her son Salvador) Æ filed a complaint with
and never on mere conjectures or suppositions. The legal relevancy of evidence the RTC of Cadiz City, Negros Occidental against Nena
denotes "something more than a minimum of probative value," suggesting that o praying for the nullification of the Deed of Absolute Sale executed
such evidentiary relevance must contain a "plus value." This may be necessary to by Daniela in her favor, cancellation of the TCT issued in the name
preclude the trial court from being satisfied by matters of slight value, capable of of Nena, and issuance of a new title and tax declaration in favor of
being exaggerated by prejudice and hasty conclusions. Evidence without "plus the heirs of Daniela
value" may be logically relevant but not legally sufficient to convict. It is x Nena Æ filed an Answer
incumbent upon the trial court to balance the probative value of such evidence o denied that any fraud or misrepresentation attended the execution
against the likely harm that would result from its admission. of the subject Deed of Absolute Sale. She also denied having
x The judgment in a criminal case can be upheld only when there is received the letter of her uncle, Carlos. She prayed for the
relevant evidence from which the court can properly find or infer that dismissal of the complaint, and in her counterclaim, she asked the
the accused is guilty beyond reasonable doubt. Proof beyond reasonable trial court for the award of actual, exemplary and moral damages
doubt requires moral certainty of guilt in order to sustain a conviction. as well as attorney’s fees and litigation expenses
Moral certainty is that degree of certainty that convinces and directs x RTC Æ in favor of Carlos and Felicidad
the understanding and satisfies the reason and judgment of those who o Declared that the deed of sale between Daniela and Nena was
are bound to act conscientiously upon it. It is certainty beyond reasonable NULL and VOID
doubt. This requires that the circumstances, taken together, should be of a x Nena Æ appealed to CA
conclusive nature and tendency; leading, on the whole, to a satisfactory x CA Æ affirmed in toto
conclusion that the accused, and no one else, committed the offense charged. In x Nena Æ filed an MR
view of the totality of evidence appreciated thus far, we rule that the present case x CA Æ denied
passes the test of moral certainty. x Nena Æ filed a Rule 65 before the SC
x However, as a matter of procedure, and for the purpose of meeting the o the CA "has decided the instant case without due regard to and in
requirement of proof beyond reasonable doubt, motive is essential for violation of the applicable laws and Decisions of this Honorable
conviction when there is doubt as to the identity of the culprit Court and also because the Decision of the Regional Trial Court,
x Judilyn Pas-a Æ testified that a few days before the victim was raped and killed, which it has affirmed, is not supported by and is even against the
the latter revealed to her that "Joel Yatar attempted to rape her after she came evidence on record."
from the school."; that her auntie Luz Dawang Yatar, wife of appellant,
separated from her husband, "this Joel Yatar threatened to kill our family."; that ISSUE # 1: Whether Rule 65 was correctly resorted to.
the exact words uttered by appellant to his wife in the Ilocano dialect was, "If
you leave me, I will kill all your family and your relatives x x x."; that these HELD # 1: NO.
statements were not contradicted by appellant x At the outset, it must be stated that the filing of the instant petition for certiorari
x Thus, appellant’s motive to sexually assault and kill the victim was evident in the under Rule 65 of the Rules of Court is inappropriate. Considering that the
instant case. It is a rule in criminal law that motive, being a state of assailed Decision and Resolution of the CA finally disposed of the case, the
mind, is established by the testimony of witnesses on the acts or proper remedy is a petition for review under Rule 45 of the Rules of Court.
statements of the accused before or immediately after the commission x The Court notes that while the instant petition is denominated as a Petition for
of the offense, deeds or words that may express it or from which his Certiorari under Rule 65 of the Rules of Court, there is no allegation that the CA
motive or reason for committing it may be inferred. committed grave abuse of discretion. On the other hand, the petition actually
x The following are the elements constitutive of rape with homicide: (1) the avers errors of judgment, rather than of jurisdiction, which are the proper
appellant had carnal knowledge of a woman; (2) carnal knowledge of a woman subjects of a petition for review on certiorari. Hence, in accordance with the
was achieved by means of force, threat or intimidation; and (3) by reason or on liberal spirit pervading the Rules of Court and in the interest of justice, the Court
the occasion of such carnal knowledge by means of force, threat or intimidation, decided to treat the present petition for certiorari as having been filed under Rule
appellant killed the woman. However, in rape committed by close kin, such 45, especially considering that it was filed within the reglementary period for
as the victim’s father, step-father, uncle, or the common-law spouse of filing the same.
her mother, it is not necessary that actual force or intimidation be
employed. Moral influence or ascendancy takes the place of violence ISSUE # 2: Whether the subject sworn statement made by Daniela deserves probative weight
and intimidation. The fact that the victim’s hymen is intact does not in that it sufficiently proved that the prior deed of sale was void.
negate a finding that rape was committed as mere entry by the penis
into the lips of the female genital organ, even without rupture or HELD # 2: NO.
laceration of the hymen, suffices for conviction of rape. The strength x The CA and the trial court ruled that the contract of sale between petitioner and
and dilatability of the hymen are invariable; it may be so elastic as to Daniela is simulated. A contract is simulated if the parties do not intend to be
stretch without laceration during intercourse. Absence of hymenal bound at all (absolutely simulated) or if the parties conceal their true agreement
lacerations does not disprove sexual abuse especially when the victim is of (relatively simulated).19 The primary consideration in determining the true
tender age. nature of a contract is the intention of the parties.20 Such intention is
x Being a relative by affinity within the third civil degree, appellant is deemed in determined from the express terms of their agreement as well as from their
legal contemplation to have moral ascendancy over the victim. contemporaneous and subsequent acts.21
204
RECTO, GAYLE ANGELI M.
2011-0008 | AUSL
Personal Notes on Remedial Law 2 Review (based on the syllabus of Prof. Henedino M. Brondial)

x In the present case, the main evidence presented by private respondents in x However, even if Daniela’s affidavit of June 9, 1983 is disregarded, the fact
proving their allegation that the subject deed of sale did not reflect the true remains that private respondents failed to prove by clear, strong and convincing
intention of the parties thereto is the sworn statement of Daniela dated evidence beyond mere preponderance of evidence37 that the contract of sale
December 28, 1977. The trial court admitted the said sworn statement as part of between Daniela and petitioner was simulated. The legal presumption is in favor
private respondents’ evidence and gave credence to it. The CA also accorded of the validity of contracts and the party who impugns its regularity has the
great probative weight to this document. burden of proving its simulation.38 Since private respondents failed to discharge
x There is no issue in the admissibility of the subject sworn statement. the burden of proving their allegation that the contract of sale between petitioner
However, the admissibility of evidence should not be equated with and Daniela was simulated, the presumption of regularity and validity of the
weight of evidence.22 The admissibility of evidence depends on its October 14, 1969 Deed of Absolute Sale stands.
relevance and competence while the weight of evidence pertains to x Considering that the Court finds the subject contract of sale between petitioner
evidence already admitted and its tendency to convince and and Daniela to be valid and not fictitious or simulated, there is no more necessity
persuade.23 Thus, a particular item of evidence may be admissible, but its to discuss the issue as to whether or not a trust relationship was created
evidentiary weight depends on judicial evaluation within the guidelines provided between them.
by the rules of evidence.24 It is settled that affidavits are classified as
hearsay evidence since they are not generally prepared by the affiant
but by another who uses his own language in writing the affiant’s
statements, which may thus be either omitted or misunderstood by the
one writing them.25 Moreover, the adverse party is deprived of the
opportunity to cross-examine the affiant.26 For this reason, affidavits are
generally rejected for being hearsay, unless the affiants themselves are placed
on the witness stand to testify thereon.27 The Court finds that both the trial
court and the CA committed error in giving the sworn statement probative
weight. Since Daniela is no longer available to take the witness stand as she is PEOPLE OF THE PHILIPPINES, Plaintiff-Appellee, vs. RODRIGO SALAFRANCA y BELLO,
already dead, the RTC and the CA should not have given probative value on Accused-Appellant.
Daniela’s sworn statement for purposes of proving that the contract of sale G.R. No. 173476 February 22, 2012
between her and petitioner was simulated and that, as a consequence, a trust FIRST DIVISION
relationship was created between them.
x Private respondents should have presented other evidence to FACTS:
sufficiently prove their allegation that Daniela, in fact, had no intention x Salafranca was convicted of murder for the fatal stabbing of Johnny Bolanon by
of disposing of her property when she executed the subject deed of the RTC Manila
sale in favor of petitioner. As in all civil cases, the burden is on the plaintiff to x RTC’s findings:
prove the material allegations of his complaint and he must rely on the strength o The evidence is clear that it was Rodrigo Salafranca who delivered
of his evidence and not on the weakness of the evidence of the defendant.28 two (2) stabbing blows to the victim while holding Johnny Bolanon
Aside from Daniela’s sworn statement, private respondents failed to present any with his left arm encircled around Bolanon’s neck stabbing the
other documentary evidence to prove their claim. Even the testimonies of their latter with the use of his right hand at the right sub costal area
witnesses failed to establish that Daniela had a different intention when she which caused Bolanon’s death. Not only because it was testified to
entered into a contract of sale with petitioner. by Augusto Mendoza but corroborated by Rodolfo Estaño, the
x In Suntay v. Court of Appeals,29 the Court ruled that the most protuberant index victim’s uncle who brought Bolanon to the hospital and who relayed
of simulation is the complete absence, on the part of the vendee, of any attempt to the court that when he aided Bolanon and even on their way to
in any manner to assert his rights of ownership over the disputed property.30 In the hospital while the latter was suffering from hard breathing,
the present case, however, the evidence clearly shows that petitioner declared victim Bolanon was able to say that it was Rodrigo Salafranca who
the property for taxation and paid realty taxes on it in her name. Petitioner has stabbed him.
shown that from 1972 to 1988 she religiously paid the real estate taxes due on o Appreciated treachery based on the testimony of Prosecution
the said lot and that it was only in 1974 and 1987 that she failed to pay the witness Mendoza on how Salafranca had effected his attack against
taxes thereon. While tax receipts and declarations and receipts and declarations Bolanon, observing that by "encircling his (accused) left arm, while
of ownership for taxation purposes are not, in themselves, incontrovertible behind the victim on the latter’s neck and stabbing the victim with
evidence of ownership, they constitute at least proof that the holder has a claim the use of his right hand," Salafranca did not give Bolanon "any
of title over the property.31 The voluntary declaration of a piece of property for opportunity to defend himself."4
taxation purposes manifests not only one’s sincere and honest desire to obtain o The RTC noted inconsistencies in Salafranca’s and his witness’
title to the property and announces his adverse claim against the State and all testimonies, as well as the fact that he had fled from his residence
other interested parties, but also the intention to contribute needed revenues to the day after the incident and had stayed away in Bataan for eight
the Government.32 Such an act strengthens one’s bona fide claim of acquisition years until his arrest.
of ownership.33 On the other hand, private respondents failed to present even a o The RTC opined that had he not been hiding, there would be no
single tax receipt or declaration showing that Daniela paid taxes due on the reason for him to immediately leave his residence, especially
disputed lot as proof that she claims ownership thereof. The only Tax Declaration because he was also working near the area.
in the name of Daniela, which private respondents presented in evidence, refers x Salafranca Æ appealed to CA
only to the house standing on the lot in controversy.34 Even the said Tax x CA Æ affirmed
Declaration contains a notation that herein petitioner owns the lot (Lot 56) upon o citing the dying declaration made to his uncle pointing to
which said house was built. Salafranca as his assailant,8 and Salafranca’s positive identification
x Moreover, the Court agrees with petitioner that if the subject Deed of Absolute as the culprit by Mendoza.9 It stressed that Salafranca’s denial and
Sale did not really reflect the real intention of Daniela, why is it that she his alibi of being in his home during the incident did not overcome
remained silent until her death; she never told any of her relatives regarding her the positive identification, especially as his unexplained flight after
actual purpose in executing the subject deed; she simply chose to make known the stabbing, leaving his home and employment, constituted a
her true intentions through the sworn statement she executed on December 28, circumstance highly indicative of his guilt
1977, the existence of which she kept secret from her relatives; and despite her x Salafranca Æ appealed to SC
declaration therein that she is appealing for help in order to get back the subject
lot, she never took any concrete step to recover the subject property from ISSUE # 1: Whether Salafranca’s denial and alibi should be sustained over the testimony of
petitioner until her death more than ten years later. the prosecution witnesses, Mendoza and Estaño.
x It is true that Daniela retained physical possession of the property even after she
executed the subject Absolute Deed of Sale and even after title to the property HELD # 1: NO.
was transferred in petitioner’s favor. In fact, Daniela continued to occupy the x Discrediting Mendoza and Estaño as witnesses against Salafranca would be
property in dispute until her death in 1988 while, in the meantime, petitioner unwarranted. The RTC and the CA correctly concluded that Mendoza and Estaño
continued to reside in Manila. However, it is well-established that ownership and were credible and reliable. The determination of the competence and
possession are two entirely different legal concepts.35 Just as possession is not a credibility of witnesses at trial rested primarily with the RTC as the trial
definite proof of ownership, neither is non-possession inconsistent with court due to its unique and unequalled position of observing their
ownership. The first paragraph of Article 1498 of the Civil Code states that when deportment during testimony, and of assessing their credibility and
the sale is made through a public instrument, the execution thereof shall be appreciating their truthfulness, honesty and candor. Absent a
equivalent to the delivery of the thing which is the object of the contract, if from substantial reason to justify the reversal of the assessment made and
the deed the contrary does not appear or cannot clearly be inferred. Possession, conclusions reached by the RTC, the CA as the reviewing court was
along with ownership, is transferred to the vendee by virtue of the notarized bound by such assessment and conclusions,11 considering that the CA as
deed of conveyance.36 Thus, in light of the circumstances of the present case, it the appellate court could neither substitute its assessment nor draw different
is of no legal consequence that petitioner did not take actual possession or conclusions without a persuasive showing that the RTC misappreciated the
occupation of the disputed property after the execution of the deed of sale in her circumstances or omitted significant evidentiary matters that would alter the
favor because she was already able to perfect and complete her ownership of result.12 Salafranca did not persuasively show a misappreciation or omission by
and title over the subject property. the RTC. Hence, the Court, in this appeal, is in no position to undo or to
x As to Daniela’s affidavit dated June 9, 1983, submitted by petitioner, which contradict the findings of the RTC and the CA, which were entitled to great
confirmed the validity of the sale of the disputed lot in her favor, the same has weight and respect.13
no probative value, as the sworn statement earlier adverted to, for being x Salafranca’s denial and alibi were worthless in the face of his positive
hearsay. Naturally, private respondents were not able to cross-examine the identification by Mendoza as the assailant of Bolanon. The lower courts
deceased-affiant on her declarations contained in the said affidavit. properly accorded full faith to such incrimination by Mendoza considering that
205
RECTO, GAYLE ANGELI M.
2011-0008 | AUSL
Personal Notes on Remedial Law 2 Review (based on the syllabus of Prof. Henedino M. Brondial)

Salafranca did not even project any ill motive that could have impelled Mendoza o (d) that the declaration is offered in a criminal case for
to testify against him unless it was upon the truth.14 homicide, murder, or parricide, in which the declarant is a
x Based on Mendoza’s account, Salafranca had attacked Bolanon from behind and victim.19
had "encircled his left arm over the neck (of Bolanon) and delivered the stabbing x All the requisites were met herein. Bolanon communicated his ante-mortem
blow using the right(hand) and coming from wnnt (sic) up right sideways and statement to Estaño, identifying Salafranca as the person who had stabbed him.
another one encircling the blow towards below the left nipple."15 Relying on At the time of his statement, Bolanon was conscious of his impending death,
Mendoza’s recollection of how Salafranca had attacked Bolanon, the RTC found having sustained a stab wound in the chest and, according to Estaño, was then
treachery to be attendant in the killing. This finding the CA concurred with. We experiencing great difficulty in breathing. Bolanon succumbed in the hospital
join the CA’s concurrence because Mendoza’s eyewitness account of the manner emergency room a few minutes from admission, which occurred under three
of attack remained uncontested by Salafranca who merely insisted on his alibi. hours after the stabbing. There is ample authority for the view that the
The method and means Salafranca employed constituted a surprise deadly attack declarant’s belief in the imminence of his death can be shown by the declarant’s
against Bolanon from behind and included an aggressive physical control of the own statements or from circumstantial evidence, such as the nature of his
latter’s movements that ensured the success of the attack without any retaliation wounds, statements made in his presence, or by the opinion of his physician.20
or defense on the part of Bolanon. According to the Revised Penal Code,16 Bolanon would have been competent to testify on the subject of the declaration
treachery is present when the offender commits any of the crimes against the had he survived. Lastly, the dying declaration was offered in this criminal
person, employing means, methods or forms in the execution thereof which tend prosecution for murder in which Bolanon was the victim.
directly and specially to insure its execution, without risk to himself arising from x A declaration or an utterance is deemed as part of the res gestae and
the defense which the offended party might make. thus admissible in evidence as an exception to the hearsay rule when
the following requisites concur, to wit:
o (a) the principal act, the res gestae, is a startling occurrence;
o (b) the statements are made before the declarant had time to
contrive or devise; and
o (c) the statements must concern the occurrence in question
and its immediately attending circumstances.21
ISSUE # 2: Whether Bolanon’s statements in identifying Salafranca may be given credence by x The requisites for admissibility of a declaration as part of the res gestae concur
the court. herein. Surely, when he gave the identity of the assailant to Estaño, Bolanon was
referring to a startling occurrence, i.e., his stabbing by Salafranca. Bolanon was
HELD # 2: YES. then on board the taxicab that would bring him to the hospital, and thus had no
x The Court further notes Estaño’s testimony on the utterance by Bolanon of time to contrive his identification of Salafranca as the assailant. His utterance
statements identifying Salafranca as his assailant right after the stabbing about Salafranca having stabbed him was made in spontaneity and only in
incident. The testimony follows: reaction to the startling occurrence. The statement was relevant because it
o Q Can you tell what happened on the said date? identified Salafranca as the perpetrator.
o A My nephew arrived in our house with a stab wound on his left x The term res gestae has been defined as "those circumstances which are
chest. the undesigned incidents of a particular litigated act and which are
o Q What time was that? admissible when illustrative of such act."22 In a general way, res gestae
o A 12:50 a.m. refers to the circumstances, facts, and declarations that grow out of
o Q When you saw your nephew with a stab wound, what did he the main fact and serve to illustrate its character and are so
say? spontaneous and contemporaneous with the main fact as to exclude
o A "Tito dalhin mo ako sa Hospital sinaksak ako." the idea of deliberation and fabrication.23 The rule on res gestae
o Q What did you do? encompasses the exclamations and statements made by either the
o A I immediately dressed up and brought him to PGH. participants, victims, or spectators to a crime immediately before,
o Q On the way to the PGH what transpired? during, or immediately after the commission of the crime when the
o A While traveling toward PGH I asked my nephew who circumstances are such that the statements were made as a
stabbed him?, and he answered, Rod Salafranca. spontaneous reaction or utterance inspired by the excitement of the
o Q Do you know this Rod Salafranca? occasion and there was no opportunity for the declarant to deliberate
o A Yes, Sir. and to fabricate a false statement.24 The test of admissibility of
o Q How long have you known him? evidence as a part of the res gestae is, therefore, whether the act,
o A "Matagal na ho kasi mag-neighbor kami." declaration, or exclamation is so intimately interwoven or connected
o Q If you see him inside the courtroom will you be able to identify with the principal fact or event that it characterizes as to be regarded
him? as a part of the transaction itself, and also whether it clearly negatives
o A Yes, Sir. any premeditation or purpose to manufacture testimony.25
o Q Will you look around and point him to us? x We modify the limiting of civil damages by the CA and the RTC to only the death
o A (Witness pointing to a man who answered by the name of Rod indemnity of P50,000.00. We declare that the surviving heirs of Bolanon were
Salafranca.) entitled by law to more than such indemnity, because the damages to be
o COURT When he told you the name of his assailant what was his awarded when death occurs due to a crime may include: (a) civil indemnity ex
condition? delicto for the death of the victim (which was granted herein); (b) actual or
o A He was suffering from hard breathing so I told him not to compensatory damages; (c) moral damages; (d) exemplary damages; and (e)
talk anymore because he will just suffer more. temperate damages.26
o Q What happened when you told him that? x We hold that the CA and the RTC should have further granted moral damages
o A He kept silent. which were different from the death indemnity.27 The death indemnity
o Q What time did you arrive at the PGH? compensated the loss of life due to crime, but appropriate and reasonable moral
o A I cannot remember the time because I was already confused at damages would justly assuage the mental anguish and emotional sufferings of
that time. the surviving family of the victim.28 Although mental anguish and emotional
o Q When you arrived at the PGH what happened? sufferings of the surviving heirs were not quantifiable with mathematical
o A He was brought to Emergency Room. precision, the Court must nonetheless strive to set an amount that would restore
o Q When he was brought to the emergency room what happened? the heirs of Bolanon to their moral status quo ante. Given the circumstances, the
o A He was pronounced dead.17 amount of P50,000.00 is reasonable as moral damages, which, pursuant to
x It appears from the foregoing testimony that Bolanon had gone to the residence prevailing jurisprudence,29 we are bound to award despite the absence of any
of Estaño, his uncle, to seek help right after being stabbed by Salafranca; that allegation and proof of the heirs’ mental anguish and emotional suffering. The
Estaño had hurriedly dressed up to bring his nephew to the Philippine General rationale for doing so rested on human nature and experience having shown
Hospital by taxicab; that on the way to the hospital, Estaño had asked Bolanon that:
who had stabbed him, and the latter had told Estaño that his assailant had been o xxx a violent death invariably and necessarily brings about
Salafranca; that at the time of the utterance Bolanon had seemed to be having a emotional pain and anguish on the part of the victim’s
hard time breathing, causing Estaño to advise him not to talk anymore; and that family.1âwphi1 It is inherently human to suffer sorrow, torment,
about ten minutes after his admission at the emergency ward of the hospital, pain and anger when a loved one becomes the victim of a violent
Bolanon had expired and had been pronounced dead. Such circumstances or brutal killing. Such violent death or brutal killing not only steals
qualified the utterance of Bolanon as both a dying declaration and as part of the from the family of the deceased his precious life, deprives them
res gestae, considering that the Court has recognized that the statement of the forever of his love, affection and support, but often leaves them
victim an hour before his death and right after the hacking incident bore all the with the gnawing feeling that an injustice has been done to
earmarks either of a dying declaration or part of the res gestae either of which them.30
was an exception to the hearsay rule.18 x The CA and the RTC committed another omission consisting in their non-
x A dying declaration, although generally inadmissible as evidence due to recognition of the right of the heirs of Bolanon to temperate damages. It is already
its hearsay character, may nonetheless be admitted when the following settled that when actual damages for burial and related expenses are not
requisites concur, namely: substantiated by receipts, temperate damages of at least P25,000.00 are
o (a) that the declaration must concern the cause and warranted, for it would certainly be unfair to the surviving heirs of the victim to
surrounding circumstances of the declarant’s death; deny them compensation by way of actual damages.31
o (b) that at the time the declaration is made, the declarant is x Moreover, the Civil Code provides that exemplary damages may be imposed in
under a consciousness of an impending death; criminal cases as part of the civil liability "when the crime was committed with
o one or more aggravating circumstances."32 The Civil Code permits such
(c) that the declarant is competent as a witness; and damages to be awarded "by way of example or correction for the public good, in
206
RECTO, GAYLE ANGELI M.
2011-0008 | AUSL
Personal Notes on Remedial Law 2 Review (based on the syllabus of Prof. Henedino M. Brondial)

addition to the moral, temperate, liquidated or compensatory damages."33 Sections 363 and 48,4 Rule 130 of the Rules of Court and it was manifest error
Conformably with such legal provisions, the CA and the RTC should have for the Court of Appeals to have ruled otherwise. In addition, SCC points out that
recognized the entitlement of the heirs of the victim to exemplary damages the sole witness of SIHI did not profess to have seen the document presented in
because of the attendance of treachery. It was of no moment that treachery was evidence executed or written by SCC. Thus, no proof of its genuineness was
an attendant circumstance in murder, and, as such, inseparable and absorbed in adduced. SIHI thus ran afoul of Section 2,5 Rule 132 of the Rules of Court,
murder. The Court explained so in People v. Catubig:34 which requires proof of due execution and authenticity of private documents
o The term "aggravating circumstances" used by the Civil Code, the before the same can be received as evidence. Petitioner likewise submits that
law not having specified otherwise, is to be understood in its broad none of the signatures affixed in the documentary evidence presented by SIHI
or generic sense. The commission of an offense has a two-pronged were offered in evidence. It vehemently argues that such was in violation of the
effect, one on the public as it breaches the social order and the requirement of Section 34,6 Rule 132 of the Rules of Court. It was thus an error
other upon the private victim as it causes personal sufferings, each of law on the part of the appellate court to consider the same. Finally, petitioner
of which is addressed by, respectively, the prescription of heavier posits that the non-production of the originals of the documents presented in
punishment for the accused and by an award of additional evidence allows the presumption of suppression of evidence provided for in
damages to the victim. The increase of the penalty or a shift to a Section 3 (e),7 Rule 131 of the Rules of Court, to come into play.
graver felony underscores the exacerbation of the offense by the o Petitioner's arguments lack merit; they fail to persuade us
attendance of aggravating circumstances, whether ordinary or x We note that the Court of Appeals found that SCC failed to appear several times
qualifying, in its commission. Unlike the criminal liability which is on scheduled hearing dates despite due notice to it and counsel. On all those
basically a State concern, the award of damages, however, is scheduled hearing dates, petitioner was supposed to cross-examine
likewise, if not primarily, intended for the offended party who the lone witness offered by SIHI to prove its case. Petitioner now
suffers thereby. It would make little sense for an award of charges the appellate court with committing an error of law when it
exemplary damages to be due the private offended party when the failed to disallow the admission in evidence of said testimony pursuant
aggravating circumstance is ordinary but to be withheld when it is to the "hearsay rule" contained in Section 36, Rule 130 of the Rules of
qualifying. Withal, the ordinary or qualifying nature of an Court.
aggravating circumstance is a distinction that should only be of x Rule 130, Section 36 reads:
consequence to the criminal, rather than to the civil, liability of the o SEC. 36. Testimony generally confined to personal knowledge;
offender. In fine, relative to the civil aspect of the case, an hearsay excluded. - A witness can testify only to those facts which
aggravating circumstance, whether ordinary or qualifying, should he knows of his personal knowledge; that is, which are derived
entitle the offended party to an award of exemplary damages from his own perception, except as otherwise provided in these
within the unbridled meaning of Article 2230 of the Civil Code. rules.
SCC CHEMICALS CORPORATION, petitioner, vs. THE HONORABLE COURT OF x Petitioner's reliance on Section 36, Rule 130 of the Rules of Court is misplaced.
APPEALS, STATE INVESTMENT HOUSE, INC., DANILO ARRIETA and LEOPOLDO HALILI, As a rule, hearsay evidence is excluded and carries no probative value.8
respondent. However, the rule does admit of an exception. Where a party failed to object
G.R. No. 128538 February 28, 2001 to hearsay evidence, then the same is admissible.9 The rationale for this
SECOND DIVISION exception is to be found in the right of a litigant to cross-examine. It is settled
that it is the opportunity to cross-examine which negates the claim
FACTS: that the matters testified to by a witness are hearsay.10 However, the
x SCC Chemicals Corporation (SCC for brevity) through its chairman, private right to cross-examine may be waived. The repeated failure of a party
respondent Danilo Arrieta and vice president, Pablo (Pablito) Bermundo, obtained to cross-examine the witness is an implied waiver of such right.
a loan from State Investment House Inc., (hereinafter SIHI) in the amount of Petitioner was afforded several opportunities by the trial court to
P129,824.48 cross-examine the other party's witness. Petitioner repeatedly failed to take
o The loan carried an annual interest rate of 30% plus penalty advantage of these opportunities. No error was thus committed by the
charges of 2% per month on the remaining balance of the principal respondent court when it sustained the trial court's finding that petitioner had
upon non-payment on the due date-January 12, 1984 waived its right to cross-examine the opposing party's witness. It is now too late
o To secure the payment of the loan, Danilo Arrieta and private for petitioner to be raising this matter of hearsay evidence.
respondent Leopoldo Halili executed a Comprehensive Surety
Agreement binding themselves jointly and severally to pay the ISSUE # 2: Whether SIHI’s other pieces of evidence were correctly admitted.
obligation on the maturity date.
x SCC failed to pay the loan when it matured. HELD # 2: YES.
x SIHI Æ then sent demand letters to SCC, Arrieta and Halili, but notwithstanding x Nor was the assailed testimony hearsay. The Court of Appeals correctly found
receipt thereof, no payment was made. that the witness of SIHI was a competent witness as he testified to facts, which
x SIHI Æ filed Civil Case No. 84-25881 for a sum of money with a prayer for he knew of his personal knowledge. Thus, the requirements of Section 36, Rule
preliminary attachment against SCC, Arrieta, and Halili before RTC Manila 130 of the Rules of Court as to the admissibility of his testimony were satisfied.
x SCC Æ filed its answer x Respecting petitioner's other submissions, the same are moot and academic. As
o Affirmative defense: lack of COA correctly found by the Court of Appeals, petitioner's admission as to
ƒ SIHI anchored its cause of action was null, void, and the execution of the promissory note by it through private respondent
of no binding effect for lack or failure of Arrieta and Bermundo at pre-trial sufficed to settle the question of the
consideration. genuineness of signatures. The admission having been made in a
x During trial: stipulation of facts at pre-trial by the parties, it must be treated as a
o SIHI presented one witness to prove its claim. The cross- judicial admission. Under Section, 411 Rule 129 of the Rules of Court, a
examination of said witness was postponed several times due to judicial admission requires no proof.
one reason or another at the instance of either party. The case was x Nor will petitioner's reliance on the "best evidence rule"12 advance its cause.
calendared several times for hearing but each time, SCC or its Respondent SIHI had no need to present the original of the documents as there
counsel failed to appear despite notice. SCC was finally declared by was already a judicial admission by petitioner at pre-trial of the execution of the
the trial court to have waived its right to cross-examine the witness promissory note and receipt of the demand letter. It is now too late for petitioner
of SIHI and the case was deemed submitted for decision. to be questioning their authenticity. Its admission of the existence of these
x RTC Æ in favor of SIHI documents was sufficient to establish its obligation. Petitioner failed to submit
x SCC Æ appealed to CA any evidence to the contrary or proof of payment or other forms of
o On appeal, SCC contended that SIHI had failed to show, by a extinguishment of said obligation.
preponderance of evidence, that the latter had a case against it. x No reversible error was thus committed by the appellate court when it held
SCC argued that the lone witness presented by SIHI to prove its petitioner liable on its obligation, pursuant to Article 1159 of the Civil Code which
claim was insufficient as the competency of the witness was not reads:
established and there was no showing that he had personal o ART. 1159. Obligations arising from contracts have the force of law
knowledge of the transaction. SCC further maintained that no proof between the contracting parties and should be complied with in
was shown of the genuineness of the signatures in the good faith.
documentary exhibits presented as evidence and that these x On the second issue, petitioner charges the Court of Appeals with reversible
signatures were neither marked nor offered in evidence by SIHI. error for having sustained the trial court'' award of attorney'' fees. Petitioner
Finally, SCC pointed out that the original copies of the documents relies on Radio Communications of the Philippines v. Rodriguez, 182 SCRA 899,
were not presented in court. 909 (1990), where we held that when attorney's fees are awarded, the reason
x CA Æ affirmed RTC; denied ensuing MR for the award of attorney's fees must be stated in the text of the court's decision.
x SCC Æ filed a Rule 45 before SC Petitioner submits that since the trial court did not state any reason for awarding
the same, the award of attorney's fees should have been disallowed by the
ISSUE # 1: Whether the subject documents presented by SIHI were correctly admitted by the appellate court.1âwphi1.nêt
RTC, considering that SCC was unable to cross-examine SIHI’s witness relative to the said o We find for petitioner in this regard.
documents. x It is settled that the award of attorney's fees is the exception rather than the
rule, hence it is necessary for the trial court to make findings of fact and law,
HELD # 1: YES. which would bring the case within the exception and justify the grant of the
x Anent the first issue, petitioner contends that SIHI introduced documentary award.13 Otherwise stated, given the failure by the trial court to explicitly state
evidence through the testimony of a witness whose competence was not the rationale for the award of attorney's fees, the same shall be disallowed. In
established and whose personal knowledge of the truthfulness of the facts the present case, a perusal of the records shows that the trial court failed to
testified to was not demonstrated. It argues that the same was in violation of
207
RECTO, GAYLE ANGELI M.
2011-0008 | AUSL
Personal Notes on Remedial Law 2 Review (based on the syllabus of Prof. Henedino M. Brondial)

explain the award of attorney's fees. We hold that the same should thereby be ƒ
Factual matters in controversy
deleted. ƒ
Rental value of the premises in question
(Herrera vs. Bollos)
B. WHAT NEED NOT BE PROVED [RULE 129] ƒ Valuation of the property subject of
expropriation proceedings (Landbank vs.
1. Judicial Notice Wycoco)
¾ Judicial Notice vs. Knowledge of a Judge
a. Mandatory and Discretionary
Judicial Notice Knowledge of a Judge
b. When to take Judicial Notice?
may be taken of a fact which
judges ought to know because mere personal knowledge of
¾ Judicial notice Æ based on the maxim “what is known need not be of their judicial functions the judge
proved.”
o Hence, when the rule is invoked, the court may dispense
with the presentation of evidence on judicially cognizable Hence, judicial notice is NOT judicial knowledge. The mere
personal knowledge of a judge is NOT judicial knowledge of the
facts court. The judge is NOT authorized to make his individual
o To say that the curt will take judicial notice of a fact Æ knowledge of the fact as basis of his action, if such fact is NOT
merely saying that the usual orm of evidence will be generally or professionally known
dispensed with if the knowledge of the fact can be
otherwise acquired
¾ Functions of Judicial Notice: Corollarily, judicial notice is NOT limited to judicial knowledge,
o To abbreviate litigation by the admission of matters that such that a judge is required to take judicial notice of certain facts,
need evidence because judicial notice is a substitute for even though such facts are NOT within his personal knowledge
formal proof of a matter by evidence
o To take the place of proof
o To displace evidence and fulfill the purpose for which the ¾ Stage when Judicial Notice may be taken:
evidence is designed to fulfill o During trial Æ the court may announce its intention to
¾ The ff are NOT functions of Judicial Notice: take judicial notice of any matter
o To be used to fill in the gaps in the party’s evidence o After trial and before judgment
o To deprive an adverse party of the opportunity to prove a o On appeal
disputed fact ƒ Either motu proprio OR on the request of an
¾ Mandatory Judicial Notice (Section 1, Rule 129) Æ no motion or party
hearing necessary: ƒ Hearing necessary Æ only for the purpose of
o Includes the ff matters (exhaustive enumeration): determining the propriety of taking judicial
ƒ existence and territorial extent of states, notice of a certain matter and NOT for the
ƒ their political history, forms of government purpose of proving issues in the case
and symbols of nationality, ¾ Doctrine of Processual Presumption
ƒ the law of nations, o GR: Our courts cannot take judicial notice of foreign laws
ƒ the admiralty and maritime courts of the ƒ Foreign laws must be alleged and proved as
world and their seals, facts
ƒ the political constitution and history of the ƒ In the absence of proof, the foreign law will
Philippines, be presumed to be the same as the laws of
ƒ the official acts of legislative, executive and the jurisdiction hearing the case under the
judicial departments of the Philippines, doctrine of processual presumption
ƒ the laws of nature, ƒ Hence, PH laws should be applied under this
ƒ the measure of time, and doctrine
ƒ the geographical divisions. o XPNs: the court may take judicial notice of a foreign law
o No need for any party to raise or allege any amendment in the ff instance:
of any law or administrative order, among others ƒ When the foreign law is within the actual
o As to affidavits attached to pleadings Æ See Candido vs. knowledge of the court, such as when the law
CA at page 15. is generally well-known
o As to declaration of the President informing the President ƒ When it had ruled upon in previous cases
of another country re project between the 2 countries Æ before it and none of the parties claim
Under the rules, it is mandatory and the Court has no otherwise
alternative but to take judicial notice of the official acts of ƒ When the foreign law is part of a published
the President of the Philippines, who heads the executive treatise, periodical or pamphlet and the writer
branch of our government. It is further provided in the is recognized in his profession or calling as
above-quoted rule that the court shall take judicial notice expert in the subject
of the foregoing facts without introduction of evidence. ƒ When foreign law refers to the law of nations
Since we consider the act of cancellation by President (judicial notice is mandatory Æ PH adopts the
Macapagal-Arroyo of the proposed ZTE-NBN Project generally accepted principles of international
during the meeting of October 2, 2007 with the Chinese law as part of the law of the land)
President in China as an official act of the executive ¾ Judicial notice of certain matters
department, the Court must take judicial notice of such o Of Municipal Ordinances
official act without need of evidence. (Suplico vs. ƒ MTCs should take judicial notice of municipal
NEDA) ordinances in the municipality where they sit
¾ Discretionary Judicial Notice (Section 2, Rule 129) ƒ RTCs should also take judicial notice of
o Requisites: municipal ordinances in the municipality
ƒ The matter must be one of common where they sit BUT ONLY when so required by
knowledge law and should take judicial notice of the
ƒ The matter must be settled beyond municipal ordinance on appeal to it from an
reasonable doubt inferior court which the latter took judicial
ƒ The knowledge must exist within the notice of
jurisdiction of the court x E.g. Æ The Charter of Manila
o Hence, judicial notice is limited to expressly requires that all courts
ƒ facts evidenced by public records and sitting therein take judicial notice
ƒ facts of general notoriety
o The ff may NOT be taken judicial notice of:
208
RECTO, GAYLE ANGELI M.
2011-0008 | AUSL
Personal Notes on Remedial Law 2 Review (based on the syllabus of Prof. Henedino M. Brondial)

of the city ordinances passed by ƒ Hence, such MTC and MCTC judges, upon
its city council taking judicial notice of such fact, may
o Of court’s own acts and records perform any act within the competency of a
ƒ AR: Courts may take judicial notice of its own regular notary public, subject to provisions of
acts and records in the SAME case the Notarial Law (Lapena Jr. vs. Marcos)
ƒ GR: BUT courts are NOT authorized to take o Of the age of a victim in a criminal case
judicial notice of the contents of the records ƒ May NOT be subject of judicial notice
of other cases, even when such cases have ƒ The court must require competent evidence to
been tried or are pending in the same court prove such age (People vs. Mitin)
x XPNs: o That business transactions may be made through
o When in the absence teleconferencing
of any objection, with ƒ May be subject of judicial notice
the knowledge of the ƒ BUT although judicial notice may be taken of
opposing party, the teleconferencing as a means of making
contents of said other business transactions, there is no judicial
case are clearly notice that one was conducted in a particular
referred to by title and case (Expertravel and Tours vs. CA)
number in a pending o That the scene of rape is NOT always secluded or
action and adopted or isolated and of a Filipina’s inbred modesty and
read in the record of shyness and her antipathy in publicly airing acts
the latter OR which blemish her honor and virtue
o When the original ƒ May be subject of judicial notice
record of the other ƒ Rape may be and has been committed in
case or any part of it is places where people congregate (People vs.
actually withdrawn Tundang)
from the archives at o That a certain barangay in an urban municipality is
the court’s discretion likewise an urban area
upon the request, or ƒ May be subject of judicial notice
with the consent, of ƒ A municipal jurisdiction, whether designated
the parties, and as a chartered city or provincial capital, is
admitted as part of the considered as urban in its entirety if it has a
record of the pending population of at least 1,000 persons per
case square kilometer. Hence, when a municipality
o Of Post Office practices is urban, all its barangays are considered
ƒ Not a proper subject of judicial notice urban. (Chiongbian-Oliva vs. Republic)
ƒ Not covered by any of the instances under the o That the judiciary is beset with the task of
Rules unclogging dockets
ƒ Not of unquestionable demonstration ƒ May be subject of judicial notice
o Of Banking practices ƒ Hence, judges are required to resolve cases
ƒ Banks’ practice of conducting background within the prescribed period mandated by law
checks on borrowers and sureties may be (GSIS vs. Vallar)
subject of discretionary judicial notice o Of the fact that testimonies during trial are much
ƒ This may fall under matters which are “of more exact and elaborate than those stated in
public knowledge” or “ought to be known to sworn statements
judges because of their judicial functions” ƒ May be subject of judicial notice
o Of financial condition of the government ƒ Sworn statements are usually incomplete and
ƒ The fact that the government is and has been inaccurate (Estioca vs. People)
for many years been financially strapped, to ƒ Such sworn statements are generally inferior
the point that even the most essential to the testimony of a witness in open court
services have suffered serious curtailment, (People vs. Sorilla)
may be taken judicial notice of (La Bugal- o That drug abuse can damage the mental faculties
B’laan Tribal Association vs. Ramos) of a user
o Of Presidential powers under the law ƒ May be subject of judicial notice
ƒ Should be taken judicial notice of, as it falls ƒ Hence, it its beyond question that an
within the contemplation of the phrase employee under the influence of drugs cannot
“official acts of the legislative, executive and possibly continue doing his duties without
judicial departments” posing a serious threat to the lives and
o Of the general increase in rentals in real estate property of his co-workers and employer
ƒ May be subject of judicial notice (Bughaw vs. Treasure Island Industrial)
ƒ BUT the reasonable amount of rent may NOT o That persons may kill and have killed for no reason
be determined by judicial notice BUT by at all
supporting evidence such as: ƒ May be subject of judicial notice (People vs.
x Realty assessment of the land Zeta)
x Increase in realty taxes o That the counsel has moved to another address
x Prevailing rate of rentals in the ƒ May NOT be subject of judicial notice
vicinity ƒ Thus, the said counsel must inform the court
o Of an administrative regulation or statute that is of such change, otherwise such omission or
NOT yet effective neglect is inexcusable and will not stay the
ƒ May NOT be subject of judicial notice finality of the decision (Karen and Khristy
ƒ REASON: a law which is inexistent cannot be Fishing vs. CA)
of common knowledge capable of ready and o Of a person’s home address or office
unquestionable demonstration (State ƒ May NOT be subject of judicial notice
Prosecutors vs. Judge Muro see digest at ƒ This may not be taken judicial notice of,
page 16) notwithstanding the standing of such person
o That there are municipalities that neither have in the business community (Garrucho vs.
lawyers nor notaries public CA)
ƒ May be subject of judicial notice
209
RECTO, GAYLE ANGELI M.
2011-0008 | AUSL
Personal Notes on Remedial Law 2 Review (based on the syllabus of Prof. Henedino M. Brondial)

o Of matters which are within the locality where the genuineness and due execution of
court sits the same)
ƒ May be subject of judicial notice x BUT such party is NOT precluded
ƒ Thus, a court may take judicial notice of the from arguing against the
fact that a person is the congressman or document by evidence of fraud,
representative of the district where the court mistake, compromise, payment,
sits (Saludo Jr vs. American Express) SoL, estoppel, and want of
consideration
2. Judicial Admissions ƒ XPNs: failure to deny the genuineness and
due execution of an actionable document is
a. When is there a Judicial Admission? NOT a judicial admission in the ff cases:
x When the party who fails to deny
¾ Elements of a judicial admission under Section 4, Rule 129: is NOT a party to the said
o Made by a party to the case document OR
o Made in the course of the proceedings in the same case x When compliance with an order
Æ hence, when an admission is made in another case, it for an inspection of the original
is to be considered as an extrajudicial admission for instrument is refused
purposes of the other proceedings where such admission o Stipulations of facts in pre-trial briefs in civil cases are
is being offered judicial admissions
o No requirement as to the form of admission Æ form is ƒ REASON: pre-trial in civil cases is mandatory,
immaterial because the rule allows either a verbal or a hence, the submission of pre-trial briefs is
written admission part of the judicial proceedings (Heirs of
¾ When/where/how judicial admissions may be made Conahap vs. Heirs of Ragana
o During pre-trial through the stipulation of facts (Cuenco o Stipulations of facts made by the accused during the pre-
vs. Talisay Tourist Sports Complex) trial in a criminal case may only considered judicial
o In the pleadings or admissions if (1) reduced into writing AND (2) signed by
o During trial, either by verbal or written manifestations or both the accused and his counsel (Section 2, Rule 118)
stipulations or ƒ BUT the requirements under Section 2 Rule
o Other stages of the judicial proceeding (Spouses 118 DO NOT apply to stipulations of facts
Binarao vs. Plus Builders) made by during the trial itself because such
¾ Rules governing admissions, set forth by jurisprudence: stipulations of facts are automatically reduced
o An admission made in a drafted document for the in writing and are contained in the TSN
purpose of filing as a pleading but was NEVER filed is o Admissions obtained through depositions, written
NOT a judicial admission. When it is signed by the party, interrogatories or requests for admission are also
it may be considered as an extrajudicial admission. If considered as judicial admissions (Programme Inc vs.
signed by the counsel, it NOT an admission because the Province of Bataan)
authority of the counsel to make statements for the client ƒ BUT a request for admission is considered as
only extends to statements made in open court or in for the purpose of the pending action ONLY.
pleadings filed with the court. (Jackson vs. Schine Hence, since under the Rules, the person to
Lexington [US Jurisprudence]) whom the request is directed must specifically
o GR: Admissions made in pleadings and motions are deny by sworn statement the matters of
judicial admissions and are binding upon the person which an admission is requested, failure to do
making them so will be tantamount to a judicial admission
ƒ XPN: When there is a showing that such but only as regards the pending action which
admission was made through palpable the admission is being requested.
mistake (Martinez vs. CA, Herrera-Felix o Admissions in an amended pleading are judicial
vs. CA) admissions
o GR: An admission may likewise be inferred from failure to ƒ HOWEVER, the amended/superseded pleading
specifically deny the material allegations in the other may STILL be used against the pleader BUT
party’s pleadings only as an extrajudicial admission Æ hence,
ƒ XPNs: There is NO inferred admission in cases must be formally offered and proven (Torres
where the ff matters are not specifically vs. CA, Ching vs. CA)
denied: o Admissions in dismissed pleadings may likewise be
x Immaterial allegations considered only as extrajudicial admissions
x Conclusions (Servicewide Specialists Inc vs. CA)
x Non-ultimate facts in the pleading o A motion to dismiss is considered as a judicial admission
x Amount of unliquidated damages BUT ONLY insofar as material allegations which are
o In other words, when sufficiently pleaded are concerned (Magno vs. CA, De
a party fails to Dios vs. Bristol Laboratories, Dalandan vs. Julio)
specifically deny the ƒ REASON: a motion to dismiss partakes the
foregoing, such failure nature of a demurrer which hypothetically
would NOT be admits the truth of the factual allegations
tantamount to a made in the complaint
judicial admission o GR: Admissions made by a counsel is conclusive upon the
o There is likewise an implied admission of allegations of client, hence, constitute judicial admission
usury for failure to specifically deny under oath ƒ XPNs:
o As to implied admissions of actionable documents: x Where the reckless or gross
ƒ GR: Failure to deny the genuineness and due negligence of the counsel deprives
execution of an actionable document at to set the client of due process of law or
forth the facts so claimed is an implied x When its application will result in
admission of such actionable document (PNB outright deprivation of the client’s
vs. Refrigeration Industries) liberty or property or
x Thus, the party who fails to deny x When the interests of justice so
such is PRECLUDED from arguing require (Salazar vs. CA)
that the subject document is a
forgery (because he has already ¾ Consequences of judicial admissions:
admitted impliedly the
210
RECTO, GAYLE ANGELI M.
2011-0008 | AUSL
Personal Notes on Remedial Law 2 Review (based on the syllabus of Prof. Henedino M. Brondial)

o The matter judicially admitted CANNOT be later on


challenged as a fact THE PEOPLE OF THE PHILIPPINES, plaintiff-appellee, vs. JAILON KULAIS, CARLOS
ƒ REASON: judicial admissions are a waiver of FALCASANTOS @ "Commander Falcasantos," AWALON KAMLON HASSAN @ "Commander
Kamlon," MAJID SAMSON @ "Commander Bungi," JUMATIYA AMLANI DE FALCASANTOS,
proof, hence, production of evidence is NORMA SAHIDDAN DE KULAIS, SALVADOR MAMARIL y MENDOZA, HADJIRUL PLASIN y ALIH,
dispensed with JAINUDDIN HASSAN y AHMAD, IMAM TARUK ALAH y SALIH, JALINA HASSAN DE KAMMING,
ƒ GR: Hence, in sum, the ff are EFFECTS of FREDDIE MANUEL @ "Ajid" and several JOHN and JANE DOES, accused, JAILON KULAIS,
judicial admissions: appellant.
x They do not require proof G.R. No. 100901 July 16, 1998
FIRST DIVISION
x They cannot be contradicted
because they are conclusive as to FACTS:
the party making it x RTC convicted Appellant Kulais with 5 counts of kidnapping for ransom, 1 count
ƒ XPNs: The foregoing DO NOT apply in the ff of kidnapping a woman and public officer, and 2 counts of slight illegal detention
cases: for the kidnapping of Monico Saavedra and Calixto Francisco
x Upon showing that the admission o RTC likewise found Jainuddin Hassan, Jailon Kulais, Salvador
Mamaril and Hadjirul Plasin guilty as conspirators in the 8 cases of
was made through palpable
kidnapping
mistake x During trial:
o Palpable mistake Æ o Prosecution (which presented 15 witnesses including some of the
one that is clear to the vctims themselves) Æthat a group of public officials, which were
mind or plain to see organized as a monitoring team to inspect government projects in
x When it is shown that no such Zamboanga City, was heaed to Lincomo Elementary School to
check its classrooms; that the said groupd was composed as
admission was made Æ this may Virginia Gara, as the head, Armando Bacarro, representing COA,
be invoked when a statement of a Felix del Rosario, representing an NGO, Edilberto Perez,
party is taken out of context or representing the City Assessor’s Office, Jessica Calunod and Allan
that his statement was made not Basa, representing the City Budget Office, and Monico Saavedra,
in the sense it is made to appear the driver from the City Engineer's Office; that the said group was
by the other party not able to reach the said school because they were stopped by 9
men who pointed guns at them; that the armed men took their
belongings; that their leader introduced himself as Commander
CASES Falcasantos; that while they were walking towards the mountains,
they encountered government troops; that said encounter cause
LANDBANK OF THE PHILIPPINES, petitioner, vs. SPOUSES VICENTE BANAL and the group to be divided into 2, the others joining a certain
LEONIDAS ARENAS-BANAL, respondents. Commander Kamlon; that they were held in captivity for 54 days in
G.R. No. 143276 July 20, 2004 the forests; that they were released after a ransom amounting to
THIRD DIVISION P122,000 was paid
o Defense Æ that accused-appellant Jumatiya Amlani was picked up
FACTS: by soldiers while she was weeding their farm; that when she was
x Spouses Banal were the owners of an agricultural land, a portion thereof having picked up by the military, she has just escaped captivity of Carlos
been appropriated by the government pursuant to the Comprehensive Agrarian Falcasantos and company, where she was made to sleep with
Reform Law Falcasantos and aide his wives; that accused-appellant Jaliha
x Landbank Æ provided a valuation of Php 173,918.55 Hussin was likewise a kidnap victim who escaped the captivity of
x Spouses Banal rejected the said valuation Daing Kamming; that accused appellant Norma Sahiddan was
x A summary administrative proceeding was conducted before the Provincial likewise arrested by soldiers without telling her the reason behind
Agrarian Reform Adjudicator (PARAD) to determine the valuation of the land her arrest
x PARAD Æ affirmed valuation by Landbank x RTC Æ acquitted Freddie Manuel alias "Ajid", and Imam Taruk Alaas they were
x Spouses Banal Æ filed a petition for determination of just compensation before not positively identified and convicted the rest who were positively identified
the RTC x Kulais et al Æ appealed to SC
x RTC Æ computed the just compensation for the coconut land at P657,137.00 o As earlier noted, Jumatiya Amlani, Jaliha Hussin and Norma
and for the riceland at P46,000.00, or a total of P703,137.00, which is beyond Sahiddan had withdrawn their appeal, and as such, the third and
respondents' valuation of P623,000.00. The court further awarded compounded fourth assigned errors, which pertain to them only, will no longer
interest at P79,732.00 in cash be dealt with. Only the following issues pertaining to Appellant
x CA Æ affirmed in toto Jailon Kulais will be discussed:
x Landbank Æ contended that the RTC, in determining the just compensation ƒ (1) judicial notice of other pending cases,
merely took judicial notice of the average production figures in the Rodriguez ƒ (2) sufficiency of the prosecution evidence, and
case pending before it and applied the same to this case without conducting a ƒ (3) denial as a defense.
hearing and worse, without the knowledge or consent of the parties
ISSUE # 1: Whether the taking of judicial notice by the trial court judge of a material
ISSUE: Whether the RTC judge was correct in taking judicial notice of the figures in the testimony of Lt. Melquiades Feliciano in another case resulted in the denial of the appellants’
Rodriguez case, which was pending before it, for the purpose of determining the just constitutional right to cross-examine the witnesses against him.
compensation applicable in the instant case.
HELD # 1: NO.
HELD: NO. x Appellant Kulais argues that he was denied due process when the trial court took
x Well-settled is the rule that courts are not authorized to take judicial
judicial notice of the testimony given in another case by one Lt. Melquiades
notice of the contents of the records of other cases even when said Feliciano, who was the team leader of the government troops that captured him
cases have been tried or are pending in the same court or before the and his purported cohorts. Because he was allegedly deprived of his right to
same judge. They may only do so "in the absence of objection" and cross-examine a material witness in the person of Lieutenant Feliciano, he
"with the knowledge of the opposing party," which are not obtaining contends that the latter's testimony should not be used against him.
here. x True, as a general rule, courts should not take judicial notice of the
x Furthermore, as earlier stated, the Rules of Court shall apply to all proceedings evidence presented in other proceedings, even if these have been tried
before the Special Agrarian Courts. In this regard, Section 3, Rule 129 of the or are pending in the same court, or have been heard and are actually
Revised Rules on Evidence is explicit on the necessity of a hearing before a court pending before the same judge. This is especially true in criminal cases,
takes judicial notice of a certain matter, thus: where the accused has the constitutional right to confront and cross-examine the
o "SEC. 3. Judicial notice, when hearing necessary. - During the trial, witnesses against him.
the court, on its own initiative, or on request of a party, may x Having said that, we note, however, that even if the court a quo did take
announce its intention to take judicial notice of any matter judicial notice of the testimony of Lieutenant Feliciano, it did not use
and allow the parties to be heard thereon. such testimony in deciding the cases against the appellant. Hence,
o "After the trial, and before judgment or on appeal, the proper Appellant Kulais was not denied due process. His conviction was based
court, on its own initiative or on request of a party, may take mainly on the positive identification made by some of the kidnap victims, namely,
judicial notice of any matter and allow the parties to be heard Jessica Calunod, Armando Bacarro and Edilberto Perez. These witnesses were
thereon if such matter is decisive of a material issue in the case." subjected to meticulous cross-examinations conducted by appellant's counsel. At
x The RTC failed to observe the above provisions. best, then, the trial court's mention of Lieutenant Feliciano's testimony is a
x While the determination of just compensation involves the exercise of judicial decisional surplusage which neither affected the outcome of the case nor
discretion, however, such discretion must be discharged within the bounds of the substantially prejudiced Appellant Kulais.
law. Here, the RTC wantonly disregarded R.A. 6657, as amended, and its x RTC decision was affirmed (with modification as to the penalty)
implementing rules and regulations. (DAR Administrative Order No. 6, as
amended by DAR Administrative Order No.11).
x Petition was granted.
211
RECTO, GAYLE ANGELI M.
2011-0008 | AUSL
Personal Notes on Remedial Law 2 Review (based on the syllabus of Prof. Henedino M. Brondial)

MENANDRO B. LAUREANO, petitioner, vs. COURT OF APPEALS AND SINGAPORE again on January 8, 1987 or more than four (4) years after the effective date of
AIRLINES LIMITED, respondents. his dismissal on November 1, 1982 has already prescribed.
G.R. No. 114776 February 2, 2000 x We base our conclusion not on Article 1144 of the Civil Code but on which sets
SECOND DIVISION the prescription period at three (3) years and which governs under this
jurisdiction.
FACTS: x Petitioner claims that the running of the prescriptive period was tolled when he
x Menandro was hired by private respondent Singapore Airlines Limited (SAL) as filed his complaint for illegal dismissal before the Labor Arbiter of the National
an expatriate B-707 captain for an original period of 2 years, which appointment Labor Relations Commission. However, this claim deserves scant consideration; it
was affirmed after he passed the probation period of 6 months has no legal leg to stand on. In Olympia International, Inc., vs., Court of
x The contract was then extended to 5 years (Jan 21, 1979 - Jan 20 1984) Appeals, we held that "although the commencement of a civil action stops the
x Menandro, while in the employ of PR committed the ff infractions: running of the statute of prescription or limitations, its dismissal or voluntary
o August 24, 1980 Æ noise violation offense; apologized abandonment by the plaintiff leaves in exactly the same position as though no
o Sometime in 1980 Æ tail scraping incident wherein the tail of the action had been commenced at all."
aircraft scraped or touched the runway during landing; suspended x Petition was dismissed; CA decision was affirmed.
for a few days until investigation; reprimanded
x After attending a training, Menandro was confirmed for his solo duty as captain
of the Airbus A-300 and subsequently appointed as captain of the A-300 fleet CASAN MACODE MACQUILING, PETITIONER, vs. COMMISSION ON ELECTIONS,
x Recession came and SAL had to resort to cost-cutting measures ROMMEL ARNADO Y CAGOCO, AND LINOG G. BALUA. RESPONDENTS.
x Menandro was advised by SAL to take advanced leaves but, thinking that G.R. No. 195649 July 2, 2013
recession would only be for a short time, did not immediately terminate its A-300 EN BANC
pilots
x SAL then reviewed the qualifications of the expatriate pilots and subsequently FACTS:
promoted 12 out of the 17 to the B-747 fleet, where Menandro was not included x This an MR of the decision of the SC dated April 16, 2013:
x On October 5, 1982, defendant informed plaintiff of his termination effective o Respondent ROMMEL ARNADO y CAGOCO is disqualified from
November 1, 1982 and that he will be paid three (3) months salary in lieu of running for any local elective position. CASAN MACODE MAQUILING
three-month notice is hereby DECLARED the duly elected Mayor of Kauswagan, Lanao del
o Menandro requested for the 3-month notice instead Norte in the 10 May 2010 elections.
o SAL granted him a 2-month notice and a 1-month salary x Arnado has already successfully ended his term
x Menandro then instituted a case against PR before the LA but subsequently o While the relief sought can no longer be granted, ruling on the
withdrew the same motion for reconsideration is important as it will either affirm the
x Menandro Æ thereafter filed a case for damages due to illegal termination of validity of Arnado’s election or affirm that Arnado never qualified to
contract of services before the RTC Manila run for public office.
x SAL Æ raised the issue of jurisdiction, among others x Arnado Æ raises the ff points in this MR:
x RTC Æ applying PH law, ruled in favor of Menandro o Respondent failed to advance any argument to support his plea for
x SAL Æ appealed to CA the reversal of this Court’s Decision dated April 16, 2013. Instead,
x CA Æ reversed and set aside RTC’s decision he presented his accomplishments as the Mayor of Kauswagan,
o the action for damages due to illegal termination was filed by Lanao del Norte and reiterated that he has taken the Oath of
plaintiff-appellee only on January 8, 1987 or more than four (4) Allegiance not only twice but six times
years after the effectivity date of his dismissal on November 1, o cites Section 349 of the Immigration and Naturalization Act of the
1982, hence, has prescribed United States as having the effect of expatriation when he
x Menandro Æ filed a Rule 45 before SC executed his Affidavit of Renunciation of American Citizenship on
o Petitioner now raises the issue of whether his action is one based April 3, 2009 and thus claims that he was divested of his American
on Article 1144 or on Article 1146 of the Civil Code. According to citizenship.
him, his termination of employment effective November 1, 1982,
was based on an employment contract which is under Article 1144, ISSUE: Whether SC may take judicial notice of Section 349 of the Immigration and
so his action should prescribe in 10 years as provided for in said Naturalization Act of the United States thereby warranting a decision in favor of Arnado.
article.
o Thus he claims the ruling of the appellate court based on Article HELD: NO.
1146 where prescription is only four (4) years, is an error. The x If indeed, respondent was divested of all the rights of an American citizen, the
appellate court concluded that the action for illegal dismissal fact that he was still able to use his US passport after executing his Affidavit of
originally filed before the Labor Arbiter on June 29, 1983, but which Renunciation repudiates this claim.
was withdrawn, then filed again in 1987 before the Regional Trial x The Court cannot take judicial notice of foreign laws,1 which must be presented
Court, had already prescribed. as public documents2 of a foreign country and must be "evidenced by an
official publication thereof."3 Mere reference to a foreign law in a
ISSUE: Whether the trial court is correct in applying PH laws, notwithstanding that the other pleading does not suffice for it to be considered in deciding a case.
party to the case is a foreign entity. x American law does not govern in this jurisdiction. Instead, Section 40(d) of the
Local Government Code calls for application in the case before us, given the fact
HELD: YES. that at the time Arnado filed his certificate of candidacy, he was not only a
x At the outset, we find it necessary to state our concurrence on the assumption of Filipino citizen but, by his own declaration, also an American citizen. It is the
jurisdiction by the Regional Trial Court of Manila, Branch 9. The trial court rightly application of this law and not of any foreign law that serves as the basis for
ruled on the application of Philippine law, thus: Arnado’s disqualification to run for any local elective position.
o Neither can the Court determine whether the termination x With all due respect to the dissent, the declared policy of Republic Act No. (RA)
of the plaintiff is legal under the Singapore Laws because 9225 is that "all Philippine citizens who become citizens of another country shall
of the defendant's failure to show which specific laws of be deemed not to have lost their Philippine citizenship under the conditions of
Singapore Laws apply to this case. As substantially discussed this Act."5 This policy pertains to the reacquisition of Philippine citizenship.
in the preceding paragraphs, the Philippine Courts do not take Section 5(2)6 requires those who have re-acquired Philippine citizenship and who
judicial notice of the laws of Singapore. The defendant that seek elective public office, to renounce any and all foreign citizenship.
claims the applicability of the Singapore Laws to this case x This requirement of renunciation of any and all foreign citizenship, when read
has the burden of proof. The defendant has failed to do so. together with Section 40(d) of the Local Government Code7 which disqualifies
Therefore, the Philippine law should be applied. those with dual citizenship from running for any elective local position, indicates a
x However, the CA was correct in ruling that the action has already prescribed policy that anyone who seeks to run for public office must be solely and
x In illegal dismissal, it is settled, that the ten-year prescriptive period fixed in exclusively a Filipino citizen. To allow a former Filipino who reacquires Philippine
Article 1144 of the Civil Code may not be invoked by petitioners, for the Civil citizenship to continue using a foreign passport - which indicates the recognition
Code is a law of general application, while the prescriptive period fixed in Article of a foreign state of the individual as its national - even after the Filipino has
292 of the Labor Code [now Article 291] is a SPECIAL LAW applicable to claims renounced his foreign citizenship, is to allow a complete disregard of this policy.
arising from employee-employer relations. x Further, we respectfully disagree that the majority decision rules on a situation of
x De Guzman vs. Court of Appeals: doubt.
o The language of Art. 291 of the Labor Code does not limit its x Indeed, there is no doubt that Section 40(d) of the Local Government Code
application only to "money claims specifically recoverable under disqualifies those with dual citizenship from running for local elective positions.
said Code" but covers all money claims arising from an employee- x There is likewise no doubt that the use of a passport is a positive declaration that
employer relations" one is a citizen of the country which issued the passport, or that a passport
o It should be noted further that Article 291 of the Labor Code is a proves that the country which issued it recognizes the person named therein as
special law applicable to money claims arising from employer- its national.
employee relations; thus, it necessarily prevails over Article 1144 of x It is unquestioned that Arnado is a natural born Filipino citizen, or that he
the Civil Code, a general law. Basic is the rule in statutory acquired American citizenship by naturalization. There is no doubt that he
construction that "where two statutes are of equal theoretical reacquired his Filipino citizenship by taking his Oath of Allegiance to the
application to a particular case, the one designed therefore should Philippines and that he renounced his American citizenship. It is also indubitable
prevail." that after renouncing his American citizenship, Arnado used his U.S. passport at
x In the light of Article 291, aforecited, we agree with the appellate court's least six times.
conclusion that petitioner's action for damages due to illegal termination filed
212
RECTO, GAYLE ANGELI M.
2011-0008 | AUSL
Personal Notes on Remedial Law 2 Review (based on the syllabus of Prof. Henedino M. Brondial)

x If there is any remaining doubt, it is regarding the efficacy of Arnado’s o As stipulated during pretrial, accused Trinidad gave ABS-CBN News
renunciation of his American citizenship when he subsequently used his U.S. Network an exclusive interview some time after the incident,
passport. The renunciation of foreign citizenship must be complete and confessing his participation in the Valentine’s Day bombing
unequivocal. The requirement that the renunciation must be made through an incident.
oath emphasizes the solemn duty of the one making the oath of renunciation to o In another exclusive interview on the network, accused Baharan
remain true to what he has sworn to. Allowing the subsequent use of a foreign likewise admitted his role in the bombing incident.
passport because it is convenient for the person to do so is rendering the oath a o Finally, accused Asali gave a television interview, confessing that he
hollow act. It devalues the act of taking of an oath, reducing it to a mere had supplied the explosive devices for the 14 February 2005
ceremonial formality. bombing.
x The dissent states that the Court has effectively left Arnado "a man without a x RTC Æ asked whether accused Baharan and Trinidad were amenable to
country".1âwphi1 On the contrary, this Court has, in fact, found Arnado to have changing their "not guilty" pleas to the charge of multiple frustrated murder,
more than one. Nowhere in the decision does it say that Arnado is not a Filipino considering that they pled "guilty" to the heavier charge of multiple murder,
citizen. What the decision merely points out is that he also possessed another creating an apparent inconsistency in their pleas.
citizenship at the time he filed his certificate of candidacy. o Defense counsel Æ conferred with the two accused and explained
x Well-settled is the rule that findings of fact of administrative bodies will not be to them the consequences of the pleas.
interfered with by the courts in the absence of grave abuse of discretion on the o Baharan and Trinidad Æ acknowledged the inconsistencies and
part of said agencies, or unless the aforementioned findings are not supported manifested their readiness for re-arraignment.
by substantial evidence.8 They are accorded not only great respect but even ƒ THEN pled guilty to the charge of multiple frustrated
finality, and are binding upon this Court, unless it is shown that the murder
administrative body had arbitrarily disregarded or misapprehended evidence x Asali was then discharged as state witness
before it to such an extent as to compel a contrary conclusion had such evidence x RTC Makati Æ CONVICTED and imposed DEATH upon Gamal B. Baharan a.k.a.
been properly appreciated. Tapay, Angelo Trinidad a.k.a. Abu Khalil, and Rohmat Abdurrohim a.k.a. Abu
Jackie or Zaky
o For the complex crime of multiple murder and multiple frustrated
PEOPLE OF THE PHILIPPINES, Plaintiff-Appellee, vs. KHADDAFY JANJALANI, GAMAL B. murder
BAHARAN a.k.a. Tapay, ANGELO TRINIDAD a.k.a. Abu Khalil, GAPPAL BANNAH ASALI a.k.a. x Automatic review before the SC ensued
Maidan or Negro, JAINAL SALI a.k.a. Abu Solaiman, ROHMAT ABDURROHIM a.k.a. Jackie or
Zaky, and other JOHN and JANE DOES, Accused, ISSUE: Whether the herein appellants were correctly convicted by the RTC, considering the
GAMAL B. BAHARAN a.k.a. Tapay, ANGELO TRINIDAD a.k.a. Abu Khalil, and ROHMAT testimony of the state witness and the confessions made by said appellants.
ABDURROHIM a.k.a. Abu Jackie or Zaky, Accused-Appellants.
G.R. No. 188314 January 10, 2011 HELD: YES.
THIRD DIVISION x In People v. Oden, the Court declared that even if the requirement of conducting
a searching inquiry was not complied with, "[t]he manner by which the plea of
FACTS: guilt is made … loses much of great significance where the conviction can be
x Upon filing of the information: based on independent evidence proving the commission by the person accused
o Members of the Abu Sayyaf Group - namely Khaddafy Janjalani, of the offense charged."13 Thus, in People v. Nadera, the Court stated:
Gamal B. Baharan, Angelo Trinidad, Gappal Bannah Asali, Jainal o Convictions based on an improvident plea of guilt are set aside only
Asali, Rohmat Abdurrohim a.k.a. Abu Jackie or Zaky, and other if such plea is the sole basis of the judgment. If the trial court
"John" and "Jane Does" - were then charged with multiple murder relied on sufficient and credible evidence to convict the accused,
and multiple frustrated murder. the conviction must be sustained, because then it is predicated not
o Only Baharan, Trinidad, Asali, and Rohmat were arrested, while the merely on the guilty plea of the accused but on evidence proving
other accused remain at-large. his commission of the offense charged.14 (Emphasis supplied.)
x During arraignment: x In their second assignment of error, accused-appellants assert that guilt was not
o On their arraignment for the multiple murder charge (Crim. Case proven beyond reasonable doubt. They pointed out that the testimony of the
No. 05-476), Baharan, Trinidad, and Asali all entered a plea of conductor was merely circumstantial, while that of Asali as to the conspiracy was
guilty. insufficient.
o On the other hand, upon arraignment for the multiple frustrated x Insofar as accused-appellants Baharan and Trinidad are concerned, the evidence
murder charge (Crim. Case No. 05-477), accused Asali pled guilty. for the prosecution, in addition to that which can be drawn from the stipulation
o Accused Trinidad and Baharan pled not guilty. of facts, primarily consisted of the testimonies of the bus conductor, Elmer
o Rohmat pled not guilty to both charges. Andales, and of the accused-turned-state-witness, Asali. Andales positively
x During pre-trial: identified accused Baharan and Trinidad as the two men who had acted
o 3.) All the same three accused likewise admitted that a bomb suspiciously while inside the bus; who had insisted on getting off the bus in
exploded in the RRCG bus while the bus was plying the EDSA route violation of a Makati ordinance; and who had scampered away from the bus
fronting the MRT terminal which is in front of the Makati moments before the bomb exploded. On the other hand, Asali testified that he
Commercial Center. had given accused Baharan and Trinidad the TNT used in the bombing incident
o 4.) Accused Asali admitted knowing the other accused alias Rohmat in Makati City. The guilt of the accused Baharan and Trinidad was
whom he claims taught him how to make explosive devices. sufficiently established by these corroborating testimonies, coupled
o 5.) The accused Trinidad also admitted knowing Rohmat before the with their respective judicial admissions (pretrial stipulations) and
February 14 bombing incident. extrajudicial confessions (exclusive television interviews, as they both
o 6.) The accused Baharan, Trinidad, and Asali all admitted to stipulated during pretrial) that they were indeed the perpetrators of
causing the bomb explosion inside the RRCG bus which left four the Valentine’s Day bombing.15 Accordingly, the Court upholds the
people dead and more or less forty persons injured. findings of guilt made by the trial court as affirmed by the Court of
o 7.) Both Baharan and Trinidad agreed to stipulate that within the Appeals.
period March 20-24 each gave separate interviews to the ABS-CBN x Anent accused Rohmat, the evidence for the prosecution consisted of
news network admitting their participation in the commission of the the testimony of accused-turned-state-witness Asali. Below is a
said crimes, subject of these cases. reproduction of the transcript of stenographic notes on the state prosecutor’s
o 8.) Accused Trinidad and Baharan also admitted to pleading guilty direct examination of state-witness Asali during the 26 May 2005 trial:
to these crimes, because they were guilt-stricken after seeing a o Q : You stated that Zaky trained you and Trinidad. Under what
man carrying a child in the first bus that they had entered. circumstances did he train you, Mr. Witness, to assemble those
o 9.) Accused Asali likewise admitted that in the middle of March explosives, you and Trinidad?
2005 he gave a television news interview in which he admitted that o A : Abu Zaky, Abu Solaiman, Khadaffy Janjalani, the three of them,
he supplied the explosive devices which resulted in this explosion that Angelo Trinidad and myself be the one to be trained to make
inside the RRCG bus and which resulted in the filing of these an explosive, sir.
charges. o Q : Mr. witness, how long that training, or how long did it take that
o 10.) Finally, accused Baharan, Trinidad, and Asali admitted that training?
they are members of the Abu Sayyaf. o A : If I am not mistaken, we were thought to make bomb about
x During trial: one month and two weeks.
o The herein appellants were among the men who boarded a bus o Q : Now, speaking of that mission, Mr. witness, while you were still
along EDSA in training at Mr. Cararao, is there any mission that you undertook, if
o They alighted near Ayala Avenue any, with respect to that mission?
o Moments after, Andales (the bus conductor) felt an explosion. He o A : Our first mission was to plant a bomb in the malls, LRT, and
then saw fire quickly engulfing the bus. He ran out of the bus other parts of Metro Manila, sir.16
towards a nearby mall. After a while, he went back to where the o The witness then testified that he kept eight kilos of TNT for
bus was. He saw their bus passengers either lying on the ground or accused Baharan and Trinidad.
looking traumatized. A few hours after, he made a statement o Q : Now, going back to the bomb. Mr. witness, did you know what
before the Makati Police Station narrating the whole incident. happened to the 2 kilos of bomb that Trinidad and Tapay took from
o The spokesperson of the Abu Sayyaf Group - Abu Solaiman - you sometime in November 2004?
announced over radio station DZBB that the group had a o A : That was the explosive that he planted in the G-liner, which did
Valentine’s Day "gift" for former President Gloria Macapagal-Arroyo not explode.
x Alleged confessions: o Q : How did you know, Mr. witness?
213
RECTO, GAYLE ANGELI M.
2011-0008 | AUSL
Personal Notes on Remedial Law 2 Review (based on the syllabus of Prof. Henedino M. Brondial)

o A : He was the one who told me, Mr. Angelo Trinidad, sir. after the bomb exploded, the Abu Sayyaf Group declared that there would be
o Q : What happened next, Mr. witness, when the bomb did not more bombings in the future. Asali then received a call from Rohmat, praising
explode, as told to you by Trinidad? the former: "Sa wakas nag success din yung tinuro ko sayo."20
o A : On December 29, Angelo Trinidad got 2 more kilos of TNT x In the light of the foregoing evidence, the Court upholds the finding of guilt
bombs. against Rohmat. Article 17 of the Revised Penal Code reads:
o Q : Did Trinidad tell you why he needed another amount of o Art. 17. Principals. — The following are considered principals:
explosive on that date, December 29, 2004? Will you kindly tell us ƒ 1. Those who take a direct part in the execution of
the reason why? the act
o A : He told me that Abu Solaiman instructed me to get the TNT so ƒ 2. Those who directly force or induce others to
that he could detonate a bomb commit it
o Q : Were there any other person, besides Abu Solaiman, who called ƒ 3. Those who cooperate in the commission of the
you up, with respect to the taking of the explosives from you? offense by another act without which it would not
o A : There is, sir… Abu Zaky, sir, called up also. have been accomplished
o Q : What did Abu Zaky tell you when he called you up? x Accused Rohmat is criminally responsible under the second paragraph, or the
o A : He told me that "this is your first mission." provision on "principal by inducement." The instructions and training he had
o Q : Please enlighten the Honorable Court. What is that mission you given Asali on how to make bombs - coupled with their careful planning and
are referring to? persistent attempts to bomb different areas in Metro Manila and Rohmat’s
o A : That is the first mission where we can show our anger towards confirmation that Trinidad would be getting TNT from Asali as part of their
the Christians. mission - prove the finding that Rohmat’s co-inducement was the determining
o Q : The second time that he got a bomb from you, Mr. witness, do cause of the commission of the crime.21 Such "command or advice [was] of such
you know if the bomb explode? nature that, without it, the crime would not have materialized."22lawphi1
o A : I did not know what happened to the next 2 kilos taken by x Further, the inducement was "so influential in producing the criminal act that
Angelo Trinidad from me until after I was caught, because I was told without it, the act would not have been performed."23 In People v. Sanchez, et
by the policeman that interviewed me after I was arrested that the 2 al., the Court ruled that, notwithstanding the fact that Mayor Sanchez was not at
kilos were planted in a bus, which also did not explode. the crime scene, evidence proved that he was the mastermind of the criminal act
o Q : So besides these two incidents, were there any other incidents or the principal by inducement. Thus, because Mayor Sanchez was a co-principal
that Angelo Trinidad and Tapay get an explosive for you, Mr. and co-conspirator, and because the act of one conspirator is the act of all, the
witness? mayor was rendered liable for all the resulting crimes.24 The same finding must
o A : If I am not mistaken, sir, on February 13, 2005 at 6:30 p.m. be applied to the case at bar.
o Q : Who got from you the explosive Mr. witness?
o A : It’s Angelo Trinidad and Tapay, sir. ISSUE # 2: Whether conspiracy was sufficiently established, considering that Asali’s testimony
o Q : How many explosives did they get from you, Mr. witness, at implicating the herein appellants were made AFTER the termination of their conspiracy.
that time?
o A : They got 2 kilos TNT bomb, sir. HELD # 2: YES.
o Q : Did they tell you, Mr. witness, where are they going to use that x The Court also affirms the finding of the existence of conspiracy
explosive? involving accused Baharan, Trinidad, and Rohmat. Conspiracy was
o A : No, sir. clearly established from the "collective acts of the accused-appellants
o Q : Do you know, Mr. witness, what happened to the third batch of before, during and after the commission of the crime." As correctly
explosives, which were taken from you by Trinidad and Tapay? declared by the trial court in its Omnibus Decision:
o A : That is the bomb that exploded in Makati, sir. o Asali’s clear and categorical testimony, which remains unrebutted
o Q : Why did you know, Mr. witness? on its major points, coupled with the judicial admissions freely and
o A : Because I was called in the evening of February 14 by Abu voluntarily given by the two other accused, are sufficient to prove
Solaiman. He told me not to leave the house because the explosive the existence of a conspiracy hatched between and among the four
that were taken by Tapay and Angelo Trinidad exploded. accused, all members of the terrorist group Abu Sayyaf, to wreak
o Q : Was there any other call during that time, Mr. Witness? chaos and mayhem in the metropolis by indiscriminately killing and
o A : I was told by Angelo Trinidad not to leave the house because injuring civilian victims by utilizing bombs and other similar
the explosive that he took exploded already, sir. destructive explosive devices.
o Q : How sure were you, Mr. witness, at that time, that indeed, the x While said conspiracy involving the four malefactors has not been expressly
bomb exploded at Makati, beside the call of Abu Solaiman and admitted by accused Baharan, Angelo Trinidad, and Rohmat, more specifically
Trinidad? with respect to the latter’s participation in the commission of the crimes,
o A : It was told by Abu Solaiman that the bombing in Makati should nonetheless it has been established by virtue of the aforementioned evidence,
coincide with the bombing in General Santos. which established the existence of the conspiracy itself and the indispensable
o A : He told it to me, sir… I cannot remember the date anymore, but participation of accused Rohmat in seeing to it that the conspirators’ criminal
I know it was sometime in February 2005. design would be realized.
o Q : Any other call, Mr. witness, from Abu Solaiman and Trinidad x It is well-established that conspiracy may be inferred from the acts of the
after the bombing exploded in Makati, any other call? accused, which clearly manifests a concurrence of wills, a common intent or
o A : There is, sir… The call came from Abu Zaky. design to commit a crime (People v. Lenantud, 352 SCRA 544). Hence, where
o Q : What did Abu Zaky tell you, Mr. witness? acts of the accused collectively and individually demonstrate the existence of a
o A : He just greeted us congratulations, because we have a common design towards the accomplishment of the same unlawful purpose,
successful mission. conspiracy is evident and all the perpetrators will be held liable as principals
o A : He told me that "sa wakas, nag success din yung tinuro ko (People v. Ellado, 353 SCRA 643).25
sayo." x In People v. Geronimo, the Court pronounced that it would be justified in
o Q : By the way, Mr. witness, I would just like to clarify this. You concluding that the defendants therein were engaged in a conspiracy "when the
stated that Abu Zaky called you up the following day, that was defendants by their acts aimed at the same object, one performing one part and
February 15, and congratulating you for the success of the mission. the other performing another part so as to complete it, with a view to the
My question to you, Mr. witness, if you know what is the relation of attainment of the same object; and their acts, though apparently independent,
that mission, wherein you were congratulated by Abu Zaky, to the were in fact concerted and cooperative, indicating closeness of personal
mission, which have been indoctrinated to you, while you were in association, concerted action and concurrence of sentiments."26
Mt. Cararao, Mr. witness? x Accused contend that the testimony of Asali is inadmissible pursuant to Sec. 30,
o A : They are connected, sir. Rule 130 of the Rules of Court. It is true that under the rule, statements
o Q : Connected in what sense, Mr. witness? made by a conspirator against a co-conspirator are admissible only
o A : Because when we were undergoing training, we were told that when made during the existence of the conspiracy. However, as the Court
the Abu Sayyaf should not wage war to the forest, but also wage ruled in People v. Buntag, if the declarant repeats the statement in court,
our battles in the city. his extrajudicial confession becomes a judicial admission, making the
o Q : Wage the battle against who, Mr. witness? testimony admissible as to both conspirators.27 Thus, in People v. Palijon,
o A : The government, sir.17 the Court held the following:
x What can be culled from the testimony of Asali is that the Abu Sayyaf Group was o … [W]e must make a distinction between extrajudicial and judicial
determined to sow terror in Metro Manila, so that they could show their "anger confessions. An extrajudicial confession may be given in
towards the Christians."18 It can also be seen that Rohmat, together with evidence against the confessant but not against his co-
Janjalani and Abu Solaiman, had carefully planned the Valentine’s Day bombing accused as they are deprived of the opportunity to cross-
incident, months before it happened. Rohmat had trained Asali and Trinidad to examine him. A judicial confession is admissible against
make bombs and explosives. While in training, Asali and others were told that the declarant’s co-accused since the latter are afforded
their mission was to plant bombs in malls, the LRT, and other parts of Metro opportunity to cross-examine the former. Section 30, Rule
Manila. According to Asali, Rohmat called him on 29 December 2004 to confirm 130 of the Rules of Court applies only to extrajudicial acts
that Trinidad would get two kilos of TNT from Asali, as they were "about to or admissions and not to testimony at trial where the party
commence" their "first mission."19 They made two separate attempts to bomb a adversely affected has the opportunity to cross-examine
bus in Metro Manila, but to no avail. The day before the Valentine’s Day the declarant. Mercene’s admission implicating his co-accused
bombing, Trinidad got another two kilos of TNT from Asali. On Valentine’s Day, was given on the witness stand. It is admissible in evidence against
the Abu Sayyaf Group announced that they had a gift for the former President, appellant Palijon. Moreover, where several accused are tried
Gloria Macapagal-Arroyo. On their third try, their plan finally succeeded. Right
214
RECTO, GAYLE ANGELI M.
2011-0008 | AUSL
Personal Notes on Remedial Law 2 Review (based on the syllabus of Prof. Henedino M. Brondial)

together for the same offense, the testimony of a co-accused Sandiganbayan should have taken judicial notice of the Bane deposition as part
implicating his co-accused is competent evidence against the latter. of its evidence.
x Judicial notice is the cognizance of certain facts that judges may
REPUBLIC OF THE PHILIPPINES, Petitioner, vs. SANDIGANBAYAN (FOURTH properly take and act on without proof because these facts are already
DIVISION), JOSE L. AFRICA (substituted by his heirs), MANUEL H. NIETO, JR., FERDINAND known to them.152 Put differently, it is the assumption by a court of a
E. MARCOS (substituted by his heirs), IMELDA R. MARCOS, FERDINAND R. MARCOS, JR., fact without need of further traditional evidentiary support. The
JUAN PONCE ENRILE, and POTENCIANO ILUSORIO (substituted by his heirs), Respondents. principle is based on convenience and expediency in securing and
G.R. No. 152375 December 16, 2011 introducing evidence on matters which are not ordinarily capable of
EN BANC dispute and are not bona fide disputed.153
x The foundation for judicial notice may be traced to the civil and canon law
FACTS: maxim, manifesta (or notoria) non indigent probatione.154 The taking of judicial
x Republic (through PCGG) Æ filed complaint (docketed as Civil Case No. 0009) notice means that the court will dispense with the traditional form of
against Jose L. Africa, Manuel H. Nieto, Jr., Ferdinand E. Marcos, Imelda R. presentation of evidence. In so doing, the court assumes that the matter is so
Marcos, Ferdinand R. Marcos, Jr., Juan Ponce Enrile, and Potenciano Ilusorio notorious that it would not be disputed.
(collectively, the respondents) for reconveyance, reversion, accounting, x The concept of judicial notice is embodied in Rule 129 of the Revised Rules on
restitution, and damages before the Sandiganbayan Evidence. Rule 129 either requires the court to take judicial notice, inter alia, of
o alleged, inter alia, that the respondents illegally manipulated the "the official acts of the x x x judicial departments of the
purchase of the major shareholdings of Cable and Wireless Limited Philippines,"155 or gives the court the discretion to take judicial notice of
in Eastern Telecommunications Philippines, Inc. (ETPI), which matters "ought to be known to judges because of their judicial functions."156 On
shareholdings respondents Jose Africa and Manuel Nieto, Jr. held the other hand, a party-litigant may ask the court to take judicial notice of any
for themselves and, through their holdings and the corporations matter and the court may allow the parties to be heard on the propriety of taking
they organized, beneficially for respondents Ferdinand E. Marcos judicial notice of the matter involved.157 In the present case, after the petitioner
and Imelda R. Marcos filed its Urgent Motion and/or Request for Judicial Notice, the respondents were
x Civil Case No. 0009 spawned numerous incidental cases,6 among them, Civil also heard through their corresponding oppositions.
Case No. 0130 x In adjudicating a case on trial, generally, courts are not authorized to take
x Civil Case No. 0130 judicial notice of the contents of the records of other cases, even when
o During the pendency of PCGG’s petition (G.R. No. 107789), the such cases have been tried or are pending in the same court, and
PCGG filed with this Court a "Very Urgent Petition for Authority to notwithstanding that both cases may have been tried or are actually
Hold Special Stockholders’ Meeting for [the] Sole Purpose of pending before the same judge.158 This rule though admits of exceptions.
Increasing [ETPI’s] Authorized Capital Stock" (Urgent Petition). In o As a matter of convenience to all the parties, a court may properly
our May 7, 1996 Resolution, we referred this Urgent Petition to the treat all or any part of the original record of a case filed in its
Sandiganbayan for reception of evidence and immediate archives as read into the record of a case pending before it, when,
resolution.16 The Sandiganbayan included the Urgent Petition in with the knowledge of, and absent an objection from, the adverse
Civil Case No. 0130. party, reference is made to it for that purpose, by name and
o In the proceedings to resolve the Urgent Petition, the testimony of number or in some other manner by which it is sufficiently
Mr. Maurice V. Bane (former director and treasurer-in-trust of ETPI) designated; or when the original record of the former case or any
was taken- at the petitioner’s instance and after serving notice of the part of it, is actually withdrawn from the archives at the court's
deposition-taking on the respondents18 - on October 23 and 24, direction, at the request or with the consent of the parties, and
1996 by way of deposition upon oral examination (Bane admitted as a part of the record of the case then pending.159
deposition) before Consul General Ernesto Castro of the Philippine x Courts must also take judicial notice of the records of another case or cases,
Embassy in London, England. where sufficient basis exists in the records of the case before it, warranting the
o Invoking Section 1, Rule 24 (of the old Rules of Court), purportedly dismissal of the latter case.160
allowing the petitioner to depose Bane without leave of court, i.e., x The issue before us does not involve the applicability of the rule on mandatory
as a matter of right after the defendants have filed their answer, taking of judicial notice; neither is the applicability of the rule on discretionary
the notice stated that "[t]he purpose of the deposition is for [Bane] taking of judicial notice seriously pursued. Rather, the petitioner approaches
to identify and testify on the facts set forth in his affidavit19 x x x the concept of judicial notice from a genealogical perspective of
so as to prove the ownership issue in favor of [the petitioner] treating whatever evidence offered in any of the "children" cases -
and/or establish the prima facie factual foundation for Civil Case 0130 - as evidence in the "parent" case - Civil Case 0009 - or
sequestration of [ETPI’s] Class A stock in support of the [Urgent "of the whole family of cases."161 To the petitioner, the supposed
Petition]."20 The notice also states that the petitioner shall use the relationship of these cases warrants the taking of judicial notice.
Bane deposition "in evidence… in the main case of Civil Case No. o We strongly disagree.
0009."21 On the scheduled deposition date, only Africa was present ƒ First, the supporting cases162 the petitioner cited
and he cross-examined Bane. are inapplicable either because these cases
x Republic Æ filed a motion to admit Bane’s deposition in Civil Case No. 0009 involve only a single proceeding or an
o [The petitioner] wishes to adopt in [Civil Case No. 0009] their exception to the rule, which proscribes the
testimonies and the documentary exhibits presented and identified courts from taking judicial notice of the
by them, since their testimonies and the said documentary exhibits contents of the records of other cases.163 ƒ
are very relevant to prove the case of the [petitioner] in [Civil Case Second, the petitioner’s proposition is obviously
No. 0009]. obnoxious to a system of orderly procedure.
x SB Æ promulgated the 1998 resolution DENYING the motion to admit The petitioner itself admits that the present case has
x Republic Æ filed an Urgent Motion and/or Request for Judicial Notice generated a lot of cases, which, in all likelihood,
o SB Æ denied involve issues of varying complexity. If we follow the
ƒ Judicial notice is found under Rule 129 which is titled logic of the petitioner’s argument, we would be
"What Need Not Be Proved." Apparently, this espousing judicial confusion by indiscriminately
provision refers to the Court’s duty to consider allowing the admission of evidence in one case, which
admissions made by the parties in the pleadings, or in was presumably found competent and relevant in
the course of the trial or other proceedings in another case, simply based on the supposed lineage
resolving cases before it. The duty of the Court is of the cases. It is the duty of the petitioner, as a
mandatory and in those cases where it is party-litigant, to properly lay before the court the
discretionary, the initiative is upon the Court. Such evidence it relies upon in support of the relief it
being the case, the Court finds the Urgent Motion seeks, instead of imposing that same duty on the
and/or Request for Judicial Notice as something which court.
need not be acted upon as the same is considered x We invite the petitioner’s attention to our prefatory pronouncement in Lopez v.
redundant. Sandiganbayan:164
ƒ On the matter of the [Bane deposition], [its] o Down the oft-trodden path in our judicial system, by common
admission is done through the ordinary formal offer of sense, tradition and the law, the Judge in trying a case sees only
exhibits wherein the defendant is given ample with judicial eyes as he ought to know nothing about the facts of
opportunity to raise objection on grounds provided by the case, except those which have been adduced judicially in
law. Definitely, it is not under Article (sic) 129 on evidence. Thus, when the case is up for trial, the judicial head is
judicial notice. [Emphasis ours] empty as to facts involved and it is incumbent upon the litigants to
the action to establish by evidence the facts upon which they rely.
(emphasis ours)
ISSUE # 1: Whether the Bane deposition, which had already been previously introduced and x We therefore refuse, in the strongest terms, to entertain the petitioner’s
admitted in Civil Case No. 0130, should have been taken judicial notice of by the
argument that we should take judicial notice of the Bane deposition.
Sandiganbayan in Civil Case No. 0009.
____________________________________
HELD # 1: NO.
x The petitioner also claims that since the Bane deposition had already been
RULES OF ADMISSIBILITY [RULE 130]
previously introduced and admitted in Civil Case No. 0130, then the

215
RECTO, GAYLE ANGELI M.
2011-0008 | AUSL
Personal Notes on Remedial Law 2 Review (based on the syllabus of Prof. Henedino M. Brondial)

1. Object/ Real Evidence consistently rule that the physical evidence should prevail.
(BPI vs. Reyes)
Section 1. Object as evidence. — Objects as evidence are those addressed to the senses o An object evidence when offered in accordance with the
of the court. When an object is relevant to the fact in issue, it may be exhibited to, examined requisites of its admissibility becomes evidence of the
or viewed by the court. (1a)
highest order and speaks more eloquently than witnesses
put together (People vs. Larranaga)
ANNOTATION
¾ Requisites for Admissibility
o Relevant
¾ Object evidence Æ evidence that is addressed to the senses of the
o Authenticated to be considered as competent
court
o Passed the test of authentication by a competent witness
o Refers to the actual thing itself
ƒ Both object and documentary evidence cannot
o NOT the perception of a witness of what the thing is
stand alone
o NOT a replica of such thing
ƒ There must be someone who should identify
o Has persuasive effect on the part of the court
the object to be the actual thing involved in
¾ When an object is relevant to a fact in issue, the court may acquire
the litigation
knowledge thereof by:
ƒ This is done to comply with the element of
o Actually viewing the object Æ in this caser, it becomes an
competence as an essential ingredient of
object evidence
admissibility
o Receiving testimonial thereon Æ the testimony of a
o Formally offered in evidence
person DOES NOT preclude the offering of an object as
ƒ A vital act after authentication because the
object evidence OR the inspection thereof through an
court shall consider no evidence which has
ocular
not been formally offered
ƒ Ocular inspection Æ lies within the discretion
ƒ Laying the foundation Æ common problem in
of the court; conducted by a judge; with
offering an object in evidence, particularly in
notice OR in the presence of the parties to the
convincing the court that the object being
case
presented is the actual thing involved in the
¾ When the court may refuse the introduction of object evidence and
litigation and not a mere substitute or
rely on testimonial evidence alone:
representation thereof
o When the exhibition of the object is contrary to public
¾ Competent Witness Æ one who:
policy, morals or decency
o Has the capacity to identify the object
ƒ XPN: but when the viewing is necessary in the
o Has actual and personal knowledge of the exhibit he is
interest of justice, the object may be viewed,
presenting for admission
even if repulsive, to the exclusion of the
¾ AFTER authentication Æ the object NEEDS to be formally offered in
public
evidence at the appropriate time (that is, when the offeror has rested
o When to require its being viewed would result in delays,
his case)
inconvenience, or unnecessary expenses out of
¾ Laying the Foundation Æ common problem in authenticating an
proportion to the evidentiary value of such object
object evidence
o When such object evidence would be confusing or
o DEFINITION: proving that the object being presented is
misleading Æ example: when the purpose is to prove the
the real thing and not merely a substitute or
former condition of the object and there is no preliminary
representation thereof
showing that there has been no substantial change in the
¾ Once all requisites of admissibility are present, an object evidence
said condition
becomes evidence of HIGHEST ORDER and speaks more eloquently
o When the testimonial or documentary evidence already
than all witnesses put together
presented renders the viewing of the object unnecessary
o BUT it must be remembered that object evidence
¾ Real evidence is NOT limited to the viewing of an object
CANNOT STAND ALONE and requires testimonial evidence
o It also includes objects which may be known by other
for authentication
senses, viz:
¾ Object Evidence vis-à-vis Right Against Self-Incrimination
ƒ Hearing Æ auditory
o Right against self-incrimination cannot be invoked against
ƒ Taste Æ gustatory
object evidence
ƒ Touch Æ tactile
o “If, in other words (the rule) created inviolability not only
ƒ Smell Æ olfactory
for his [physical control of his] own vocal utterances, but
¾ Documents may also be considered as object documents, when
also for his physical control in whatever form exercise,
what is sought to be proven is NOT the contents of the same. Hence,
then, it would be possible for a guilty person to shut
when any of the ff are sought to be proven, a document is only
himself up in his house, with all the tools and indicia of
considered as an object evidence:
his crime, and defy the authority of the law to employ in
o Existence or condition of the document
evidence anything that might be obtained by forcibly
o Nature of the handwritings thereon
overthrowing his possession and compelling the surrender
o Age of the paper used
of the evidential articles — a clear reduction ad
o Blemishes or alterations thereon, as when falsification is
absurdum. In other words, it is not merely compulsion
alleged that is the kernel of the privilege, . . . but testimonial
¾ Persons may also be considered as object documents, as when
compulsion.” (Justice Holmes in Holt vs. US, cited in
any of the ff are sought to be proven:
People vs. Malimit)
o Nature, extent or location of his injuries
¾ Demonstrative Evidence Æ representation or demonstration of a
o His physique thing
o His facial features to determine his resemblance and
o NOT strictly an object evidence
possible relationship to another or his racial origin or
o TEST OF ADMISSIBILITY: Whether the evidence
probable age
sufficiently or accurately represents the object it seeks to
o In case of a woman, to determine the fact that she is demonstrate or represent.
pregnant
o Under the Electronic Rules on Evidence, a photographic
¾ Testimonial vs. Object evidence of events is admissible when:
o Physical evidence is a mute but eloquent manifestation of ƒ It is presented, displayed and shown to the
truth, and it ranks high in our hierarchy of trustworthy court AND
evidence. We have, on many occasions, relied principally
ƒ It is identified, explained or authenticated by
upon physical evidence in ascertaining the truth. Where
EITHER:
the physical evidence on record runs counter to the
x The person who made the
testimonial evidence of the prosecution witnesses, we
recording OR

216
RECTO, GAYLE ANGELI M.
2011-0008 | AUSL
Personal Notes on Remedial Law 2 Review (based on the syllabus of Prof. Henedino M. Brondial)

x Other person who is competent to to the next link in the chain.


testify to the accuracy thereof These witnesses would then
ƒ Same rule applies in motion pictures and describe the precautions taken to
recordings ensure that there had been no
¾ View Æ the process of going out of the courtroom to observe places change in the condition of the
and objects item and no opportunity for
o Authorized under Section 1 Rule 130 BUT, even without someone not in the chain to have
this provision, the court is recognized to have the possession of the same. (Lopez
inherent power to order a view when there is a need to vs. People)
do so, pursuant to Section 5 of Rule 135 x The investigator need not testify
o Must be made in the presence of the parties OR at least that the process of sealing the
with previous notice to them evidence and the submission to
¾ Categories of Object Evidence for purposes of authentication of the chemist were done in the
the object and of laying the foundation of the exhibit: presence of the accused or his
o Unique Objects Æ those that have readily identifiable representative, pursuant to the
marks doctrine of presumption of
ƒ If a thing has a unique characteristic, like a regularity
serial number, it is readily identifiable. ƒ BUT non-compliance with the procedure in
ƒ When the witness testifies that such object the chain of custody of drugs DOES NOT
has a unique characteristic, that he saw such automatically result in the acquittal of the
object on the relevant date, that he accused if the prosecution is able to show
remembers its characteristics, and that is in that:
the same or substantially the same condition x Such non-compliance is justified
when he saw it on the relevant date, the AND
authentication requirement is satisfied x The integrity of the evidence is
o Objects made unique Æ those that are made readily preserved
available ¾ DNA Evidence Æ totality of the DNA profiles, results, other genetic
ƒ If a thing does not have a unique information directly generated from the DNA testing of biological
characteristic, like a kitchen knife, it is has no samples
identifying marks o FACTORS to be considered in assessing probative value of
ƒ Authentication requirement is satisfied when DNA:
the witness testifies that he made the thing ƒ How the samples were collected
acquire a unique characteristic like placing ƒ How they were handled
identifying marks on it and that the object ƒ Possibility of contamination
being presented has the characteristics he ƒ Procedure followed in analyzing
made on the object ƒ Whether proper standards and procedure
o Non-unique Objects Æ those with no identifying marks were followed in conducting the tests
and cannot be marked ƒ Qualifications of the analyst
ƒ Those which are not readily identifiable, were o The ff must be shown as CONDITIONS prior to the
not made identifiable or cannot be made issuance of an order for DNA testing:
identifiable like drops of blood, oil, drugs, ƒ A biological sample exists that has relevance
fiber, grains of sand and similar objects to the case
ƒ In this situation, the proponent of the ƒ Such biological sample
evidence must establish a chain of custody x Was not previously subjected to
x PURPOSE: to guarantee the DNA testing OR
integrity of the physical evidence x If previously subjected, results
and to prevent the authentication require confirmation for good
of evidence which is not authentic reasons
x Links to the chain Æ people who ƒ DNA testing uses a scientifically valid
actually handled or had custody of technique
the object ƒ Has scientific potential to produce new
x As long as one of the chains information that is relevant to the proper
testifies and his testimony negates resolution of the case
the possibility of tampering and ƒ Existence of other factors which may
that the integrity of the evidence potentially affect the accuracy of DNA testing
is preserved, his testimony alone o BUT DNA testing may be done WITHOUT court order if
is adequate to prove the chain of conducted PRIOR to a suit or proceeding
custody ¾ Paraffin Test Æ inconclusive because unreliable in use
x As a method of authenticating o Can only establish the presence or absence of nitrites or
evidence, the chain of custody rule nitrates on the hand
requires that the admission of an ƒ These substances are also found in tobacco
exhibit be preceded by evidence o Considered as merely a corroborative evidence
sufficient to support a finding that ¾ Polygraph Test/ Lie Detector Test Æ operates on the principle
the matter in question is what the that stress causes physiological changes in the body which can be
proponent claims it to be. It would measured to indicate whether a person is telling the truth
include testimony about every link o Uniformly rejected when offered for the purpose of
in the chain, from the moment the establishing guilt or innocence
item was picked up to the time it
is offered into evidence, in such a CASES
way that every person who
touched the exhibit would describe JUNIE MALILLIN Y. LOPEZ, petitioner, vs. PEOPLE OF THE PHILIPPINES, respondent.
G.R. No. 172953 April 30, 2008
how and from whom it was
SECOND DIVISION
received, where it was and what
happened to it while in the FACTS:
witness’ possession, the condition x RTC of Sorsogon City, Branch 52 Æ issued a warrant of search and seizure
in which it was received and the x 5 police officers then raided the house of Malillin in his presence, and in the
condition in which it was delivered presence of his wife, and barangay kagawad Delfin Licup

217
RECTO, GAYLE ANGELI M.
2011-0008 | AUSL
Personal Notes on Remedial Law 2 Review (based on the syllabus of Prof. Henedino M. Brondial)

o The team was headed by P/Insp. Catalino Bolanos (Bolanos), with x Mallillin Æ appealed to CA
PO3 Roberto Esternon (Esternon), SPO1 Pedro Docot, SPO1 Danilo x CA Æ affirmed RTC; denied ensuing MR
Lasala and SPO2 Romeo Gallinera (Gallinera) as members. x Mallillin Æ filed a Rule 45 before the SC
x The raid yielded two (2) plastic sachets of shabu and five (5) empty plastic x OSG Æ filed its Comment
sachets containing residual morsels of the said substance o OSG bids to establish that the raiding team had regularly
x Mallillin was then charged with violation of Section 11,7 Article II of Republic Act performed its duties in the conduct of the search.31
No. 9165, otherwise known as The Comprehensive Dangerous Drugs Act of 2002 o It points to petitioner's incredulous claim that he was framed up by
x Mallillin Æ pleaded not guilty Esternon on the ground that the discovery of the two filled sachets
x Prosecution evidence: was made in his and Licup's presence.
o Taking the witness stand, Bolanos, the leader of the raiding team, o It likewise notes that petitioner's bare denial cannot defeat the
testified on the circumstances surrounding the search as follows: positive assertions of the prosecution and that the same does not
that he and his men were allowed entry into the house by suffice to overcome the prima facie existence of animus possidendi.
petitioner after the latter was shown the search warrant; that upon
entering the premises, he ordered Esternon and barangay kagawad ISSUE: Whether it was sufficiently shown that the sachets of shabu presented before the RTC
Licup, whose assistance had previously been requested in were the same ones obtained from Mallillin.
executing the warrant, to conduct the search; that the rest of the
police team positioned themselves outside the house to make sure HELD: NO.
that nobody flees; that he was observing the conduct of the search x Prefatorily, although the trial court's findings of fact are entitled to great weight
from about a meter away; that the search conducted inside the and will not be disturbed on appeal, this rule does not apply where facts of weight
bedroom of petitioner yielded five empty plastic sachets with and substance have been overlooked, misapprehended or misapplied in a case
suspected shabu residue contained in a denim bag and kept in one under appeal.32 In the case at bar, several circumstances obtain which, if
of the cabinets, and two plastic sachets containing shabu which fell properly appreciated, would warrant a conclusion different from that arrived at by
off from one of the pillows searched by Esternon—a discovery that the trial court and the Court of Appeals.
was made in the presence of petitioner.10 On cross examination, x Prosecutions for illegal possession of prohibited drugs necessitates that the
Bolanos admitted that during the search, he was explaining its elemental act of possession of a prohibited substance be established with moral
progress to petitioner's mother, Norma, but that at the same time certainty, together with the fact that the same is not authorized by law. The
his eyes were fixed on the search being conducted by Esternon.11 dangerous drug itself constitutes the very corpus delicti of the offense and the
o Esternon testified that the denim bag containing the empty plastic fact of its existence is vital to a judgment of conviction.33 Essential therefore in
sachets was found "behind" the door of the bedroom and not inside these cases is that the identity of the prohibited drug be established beyond
the cabinet; that he then found the two filled sachets under a doubt.34 Be that as it may, the mere fact of unauthorized possession will not
pillow on the bed and forthwith called on Gallinera to have the suffice to create in a reasonable mind the moral certainty required to sustain a
items recorded and marked.12 On cross, he admitted that it was he finding of guilt. More than just the fact of possession, the fact that the substance
alone who conducted the search because Bolanos was standing illegally possessed in the first place is the same substance offered in court as
behind him in the living room portion of the house and that exhibit must also be established with the same unwavering exactitude as that
petitioner handed to him the things to be searched, which included requisite to make a finding of guilt. The chain of custody requirement performs
the pillow in which the two sachets of shabu were kept;13 that he this function in that it ensures that unnecessary doubts concerning the identity of
brought the seized items to the Balogo Police Station for a "true the evidence are removed.35
inventory," then to the trial court14 and thereafter to the x As a method of authenticating evidence, the chain of custody rule requires
laboratory.15 that the admission of an exhibit be preceded by evidence sufficient to
o Supt. Lorlie Arroyo (Arroyo), the forensic chemist who administered support a finding that the matter in question is what the proponent
the examination on the seized items, was presented as an expert claims it to be.36 It would include testimony about every link in the
witness to identify the items submitted to the laboratory. She chain, from the moment the item was picked up to the time it is offered
revealed that the two filled sachets were positive of shabu and that into evidence, in such a way that every person who touched the exhibit
of the five empty sachets, four were positive of containing residue of would describe how and from whom it was received, where it was and
the same substance.16 She further admitted that all seven what happened to it while in the witness' possession, the condition in
sachets were delivered to the laboratory by Esternon in the which it was received and the condition in which it was delivered to the
afternoon of the same day that the warrant was executed except next link in the chain. These witnesses would then describe the
that it was not she but rather a certain Mrs. Ofelia Garcia who precautions taken to ensure that there had been no change in the
received the items from Esternon at the laboratory. condition of the item and no opportunity for someone not in the chain
x Defense evidence: to have possession of the same.37
o The evidence for the defense focused on the irregularity of the x While testimony about a perfect chain is not always the standard
search and seizure conducted by the police operatives. Petitioner because it is almost always impossible to obtain, an unbroken chain of
testified that Esternon began the search of the bedroom with Licup custody becomes indispensable and essential when the item of real
and petitioner himself inside. However, it was momentarily evidence is not distinctive and is not readily identifiable, or when its
interrupted when one of the police officers declared to Bolanos that condition at the time of testing or trial is critical, or when a witness has
petitioner's wife, Sheila, was tucking something inside her failed to observe its uniqueness.38 The same standard likewise obtains in case
underwear. Forthwith, a lady officer arrived to conduct the search the evidence is susceptible to alteration, tampering, contamination39 and even
of Sheila's body inside the same bedroom. At that point, everyone substitution and exchange.40 In other words, the exhibit's level of
except Esternon was asked to step out of the room. So, it was in susceptibility to fungibility, alteration or tampering—without regard to whether the
his presence that Sheila was searched by the lady officer. Petitioner same is advertent or otherwise not—dictates the level of strictness in the
was then asked by a police officer to buy cigarettes at a nearby application of the chain of custody rule.
store and when he returned from the errand, he was told that x Indeed, the likelihood of tampering, loss or mistake with respect to an exhibit is
nothing was found on Sheila's body.18 Sheila was ordered to greatest when the exhibit is small and is one that has physical characteristics
transfer to the other bedroom together with her children.19 fungible in nature and similar in form to substances familiar to people in their
o Petitioner asserted that on his return from the errand, he was daily lives.41 Graham vs. State42 positively acknowledged this danger. In that
summoned by Esternon to the bedroom and once inside, the officer case where a substance later analyzed as heroin—was handled by two police
closed the door and asked him to lift the mattress on the bed. And officers prior to examination who however did not testify in court on the
as he was doing as told, Esternon stopped him and ordered him to condition and whereabouts of the exhibit at the time it was in their possession—
lift the portion of the headboard. In that instant, Esternon showed was excluded from the prosecution evidence, the court pointing out that the
him "sachet of shabu" which according to him came from a pillow white powder seized could have been indeed heroin or it could have been sugar
on the bed.20 Petitioner's account in its entirety was corroborated or baking powder. It ruled that unless the state can show by records or
in its material respects by Norma, barangay kagawad Licup and testimony, the continuous whereabouts of the exhibit at least between the time it
Sheila in their testimonies. Norma and Sheila positively declared came into the possession of police officers until it was tested in the laboratory to
that petitioner was not in the house for the entire duration of the determine its composition, testimony of the state as to the laboratory's
search because at one point he was sent by Esternon to the store findings is inadmissible.43
to buy cigarettes while Sheila was being searched by the lady x A unique characteristic of narcotic substances is that they are not readily
officer.21 Licup for his part testified on the circumstances identifiable as in fact they are subject to scientific analysis to determine their
surrounding the discovery of the plastic sachets. He recounted that composition and nature. The Court cannot reluctantly close its eyes to the
after the five empty sachets were found, he went out of the likelihood, or at least the possibility, that at any of the links in the chain of
bedroom and into the living room and after about three minutes, custody over the same there could have been tampering, alteration or
Esternon, who was left inside the bedroom, exclaimed that he had substitution of substances from other cases—by accident or otherwise—in which
just found two filled sachets similar evidence was seized or in which similar evidence was submitted for
x RTC Æ found Mallillin guilty laboratory testing. Hence, in authenticating the same, a standard more stringent
o The trial court reasoned that the fact that shabu was found in the than that applied to cases involving objects which are readily identifiable must be
house of petitioner was prima facie evidence of petitioner's animus applied, a more exacting standard that entails a chain of custody of the item with
possidendi sufficient to convict him of the charge inasmuch as sufficient completeness if only to render it improbable that the original item has
things which a person possesses or over which he exercises acts of either been exchanged with another or been contaminated or tampered with.
ownership are presumptively owned by him. It also noted x A mere fleeting glance at the records readily raises significant doubts as to the
petitioner's failure to ascribe ill motives to the police officers to identity of the sachets of shabu allegedly seized from petitioner. Of the people
fabricate charges against him who came into direct contact with the seized objects, only Esternon and Arroyo
218
RECTO, GAYLE ANGELI M.
2011-0008 | AUSL
Personal Notes on Remedial Law 2 Review (based on the syllabus of Prof. Henedino M. Brondial)

testified for the specific purpose of establishing the identity of the evidence. seizure custody of the evidence, the blind reliance by the trial court and the
Gallinera, to whom Esternon supposedly handed over the confiscated sachets for Court of Appeals on the presumption of regularity in the conduct of police duty is
recording and marking, as well as Garcia, the person to whom Esternon directly manifestly misplaced. The presumption of regularity is merely just that—a mere
handed over the seized items for chemical analysis at the crime laboratory, were presumption disputable by contrary proof and which when challenged by the
not presented in court to establish the circumstances under which they handled evidence cannot be regarded as binding truth.52 Suffice it to say that this
the subject items. Any reasonable mind might then ask the question: Are the presumption cannot preponderate over the presumption of innocence that
sachets of shabu allegedly seized from petitioner the very same objects prevails if not overthrown by proof beyond reasonable doubt.53 In the present
laboratory tested and offered in court as evidence? case the lack of conclusive identification of the illegal drugs allegedly seized from
x The prosecution's evidence is incomplete to provide an affirmative petitioner, coupled with the irregularity in the manner by which the same were
answer. Considering that it was Gallinera who recorded and marked placed under police custody before offered in court, strongly militates a finding of
the seized items, his testimony in court is crucial to affirm whether the guilt.
exhibits were the same items handed over to him by Esternon at the x In our constitutional system, basic and elementary is the presupposition that the
place of seizure and acknowledge the initials marked thereon as his burden of proving the guilt of an accused lies on the prosecution which must rely
own. The same is true of Garcia who could have, but nevertheless on the strength of its own evidence and not on the weakness of the defense.
failed, to testify on the circumstances under which she received the The rule is invariable whatever may be the reputation of the accused, for the law
items from Esternon, what she did with them during the time they presumes his innocence unless and until the contrary is shown.54 In dubio pro
were in her possession until before she delivered the same to Arroyo reo. When moral certainty as to culpability hangs in the balance, acquittal on
for analysis. reasonable doubt inevitably becomes a matter of right.
x The prosecution was thus unsuccessful in discharging its burden of establishing
the identity of the seized items because it failed to offer not only the testimony PEOPLE OF THE PHILIPPINES, Appellee, vs. FELIMON PAGADUAN y TAMAYO,
of Gallinera and Garcia but also any sufficient explanation for such failure. In Appellant.
effect, there is no reasonable guaranty as to the integrity of the exhibits G.R. No. 179029 August 12, 2010
inasmuch as it failed to rule out the possibility of substitution of the exhibits, THIRD DIVISION
which cannot but inure to its own detriment. This holds true not only with
respect to the two filled sachets but also to the five sachets allegedly containing FACTS:
morsels of shabu. x Capt De Vera received information that Pagaduan was selling drugs
x Also, contrary to what has been consistently claimed by the prosecution that the x Captain Jaime de Vera called PO3 Peter Almarez and SPO1 Domingo Balido -
search and seizure was conducted in a regular manner and must be presumed to who were both in Santiago City and informed them re planned buy-bust
be so, the records disclose a series of irregularities committed by the police operation
officers from the commencement of the search of petitioner's house until the x Captain de Vera conducted a briefing and designated PO3 Almarez as the poseur
submission of the seized items to the laboratory for analysis. The Court takes buyer
note of the unrebutted testimony of petitioner, corroborated by that of his wife, x Buy bust ensued
that prior to the discovery of the two filled sachets petitioner was sent out of his o The informant approached Pagaduan and introduced Almarez to
house to buy cigarettes at a nearby store. Equally telling is the testimony of him
Bolanos that he posted some of the members of the raiding team at the door of o Almarez informed Pagaduan that he is interested in buying shabu
petitioner's house in order to forestall the likelihood of petitioner fleeing the o Pagaduan then handed one heat-sealed transparent plastic sachet
scene. By no stretch of logic can it be conclusively explained why petitioner was containing white crystalline substance to PO3 Almarez.
sent out of his house on an errand when in the first place the police officers were o PO3 Almarez, in turn, gave the two pre-marked P100 bills to the
in fact apprehensive that he would flee to evade arrest. This fact assumes prime appellant
importance because the two filled sachets were allegedly discovered by Esternon x Information was then filed against Pagaduan before the Regional Trial Court
immediately after petitioner returned to his house from the errand, such that he (RTC), Branch 27, Bayombong, Nueva Vizcaya for violation of Section 5, Article II
was not able to witness the conduct of the search during the brief but crucial of Republic Act (R.A.) No. 9165 or the Comprehensive Dangerous Drugs Act of
interlude that he was away. 2002.
x It is also strange that, as claimed by Esternon, it was petitioner himself who x RTC Æ convicted Pagaduan
handed to him the items to be searched including the pillow from which the two x Pagaduan Æ appealed to CA
filled sachets allegedly fell. Indeed, it is contrary to ordinary human behavior that o Raised instigation as defense
petitioner would hand over the said pillow to Esternon knowing fully well that x CA Æ affirmed RTC
illegal drugs are concealed therein. In the same breath, the manner by which the o The CA found unmeritorious the appellant’s defense of instigation,
search of Sheila's body was brought up by a member of the raiding team also and held that the appellant was apprehended as a result of a
raises serious doubts as to the necessity thereof. The declaration of one of the legitimate entrapment operation. It explained that in inducement or
police officers that he saw Sheila tuck something in her underwear certainly instigation, an innocent person is lured by a public officer or private
diverted the attention of the members of petitioner's household away from the detective to commit a crime. In the case at bar, the buy-bust
search being conducted by Esternon prior to the discovery of the two filled operation was planned only after the police had received
sachets. Lest it be omitted, the Court likewise takes note of Esternon's suspicious information that the appellant was selling shabu.
presence in the bedroom while Sheila was being searched by a lady officer. The o The CA also held that the failure of the police to conduct a prior
confluence of these circumstances by any objective standard of behavior surveillance on the appellant was not fatal to the prosecution’s
contradicts the prosecution's claim of regularity in the exercise of duty. case. It reasoned out that the police are given wide discretion to
x Moreover, Section 2144 of the Implementing Rules and Regulations of select effective means to apprehend drug dealers. A prior
R.A. No. 9165 clearly outlines the post-seizure procedure in taking surveillance is, therefore, not necessary, especially when the police
custody of seized drugs. In a language too plain to require a different are already accompanied by their informant.
construction, it mandates that the officer acquiring initial custody of o The CA further ruled that the prosecution was able to sufficiently
drugs under a search warrant must conduct the photographing and the prove an unbroken chain of custody of the shabu. It explained that
physical inventory of the item at the place where the warrant has been PO3 Almarez sealed the plastic sachet seized from the appellant,
served. Esternon deviated from this procedure. It was elicited from him that at marked it with his initials, and transmitted it to the PNP Crime
the close of the search of petitioner's house, he brought the seized items Laboratory for examination. PSI Quintero conducted a qualitative
immediately to the police station for the alleged purpose of making a "true examination and found the specimen positive for the presence of
inventory" thereof, but there appears to be no reason why a true inventory could shabu. According to the CA, the prosecution was able to prove that
not be made in petitioner's house when in fact the apprehending team was able the substance seized was the same specimen submitted to the
to record and mark the seized items and there and then prepare a seizure receipt laboratory and presented in court, notwithstanding that this
therefor. Lest it be forgotten, the raiding team has had enough opportunity to specimen was turned over to the crime laboratory only after two
cause the issuance of the warrant which means that it has had as much time to days.
prepare for its implementation. While the final proviso in Section 21 of the rules x Pagaduan Æ filed a Rule 45 before the SC
would appear to excuse non-compliance therewith, the same cannot benefit the o appellant claims that the lower courts erred in convicting him of the
prosecution as it failed to offer any acceptable justification for Esternon's course crime charged despite the prosecution’s failure to prove his guilt
of action. beyond reasonable doubt.
x Likewise, Esternon's failure to deliver the seized items to the court demonstrates o He harps on the fact that the police did not conduct a prior
a departure from the directive in the search warrant that the items seized be surveillance on him before conducting the buy-bust operation.
immediately delivered to the trial court with a true and verified inventory of the x OSG Æ filed its comment
same,45 as required by Rule 126, Section 1246 of the Rules of Court. People v. o counters with the argument that the chain of custody of the shabu
Go47 characterized this requirement as mandatory in order to preclude the was sufficiently established.
substitution of or tampering with said items by interested parties.48 Thus, as a o It explained that the shabu was turned over by the police officers
reasonable safeguard, People vs. Del Castillo49 declared that the approval by the to the PNP Crime Laboratory, where it was found by the forensic
court which issued the search warrant is necessary before police officers can chemist to be positive for the presence of shabu.
retain the property seized and without it, they would have no authority to retain o The OSG likewise claimed that the appellant failed to rebut the
possession thereof and more so to deliver the same to another agency.50 Mere presumption of regularity in the performance of official duties by
tolerance by the trial court of a contrary practice does not make the practice the police.
right because it is violative of the mandatory requirements of the law and it o The OSG further added that a prior surveillance is not indispensable
thereby defeats the very purpose for the enactment.51 to a prosecution for illegal sale of drugs.
x Given the foregoing deviations of police officer Esternon from the standard and
normal procedure in the implementation of the warrant and in taking post-
219
RECTO, GAYLE ANGELI M.
2011-0008 | AUSL
Personal Notes on Remedial Law 2 Review (based on the syllabus of Prof. Henedino M. Brondial)

drugs. We have repeatedly declared that the deviation from the


standard procedure dismally compromises the integrity of the
ISSUE: Whether it was sufficiently shown that the sachets of shabu presented before the RTC evidence. In People v. Morales,31 we acquitted the accused for failure of the
were the same ones obtained from Pagaduan. buy-bust team to photograph and inventory the seized items, without giving any
justifiable ground for the non-observance of the required procedures. People v.
HELD: NO. Garcia32 likewise resulted in an acquittal because no physical inventory was ever
x In a prosecution for illegal sale of a prohibited drug under Section 5 of R.A. No. made, and no photograph of the seized items was taken under the
9165, the prosecution must prove the following elements: (1) the identity of the circumstances required by R.A. No. 9165 and its implementing rules. In Bondad,
buyer and the seller, the object, and the consideration; and (2) the delivery of Jr. v. People,33 we also acquitted the accused for the failure of the police to
the thing sold and the payment therefor. All these require evidence that the sale conduct an inventory and to photograph the seized items, without justifiable
transaction transpired, coupled with the presentation in court of the corpus grounds.
delicti, i.e., the body or substance of the crime that establishes that a crime has x We had the same rulings in People v. Gutierrez,34 People v. Denoman,35 People
actually been committed, as shown by presenting the object of the illegal v. Partoza,36 People v. Robles,37 and People v. dela Cruz,38 where we
transaction.26 To remove any doubt or uncertainty on the identity and integrity emphasized the importance of complying with the required mandatory
of the seized drug, evidence must definitely show that the illegal drug presented procedures under Section 21 of R.A. No. 9165.
in court is the same illegal drug actually recovered from the appellant; otherwise, x We recognize that the strict compliance with the requirements of
the prosecution for possession or for drug pushing under R.A. No. 9165 fails.27 Section 21 of R.A. No. 9165 may not always be possible under field
x The required procedure on the seizure and custody of drugs is embodied in conditions; the police operates under varied conditions, and cannot at all times
Section 21, paragraph 1, Article II of R.A. No. 9165, which states: attend to all the niceties of the procedures in the handling of confiscated
o (1) The apprehending team having initial custody and evidence. For this reason, the last sentence of the implementing rules
control of the drugs shall, immediately after seizure and provides that "non-compliance with these requirements under
confiscation, physically inventory and photograph the same justifiable grounds, as long as the integrity and the evidentiary value of
in the presence of the accused or the person/s from whom the seized items are properly preserved by the apprehending
such items were confiscated and/or seized, or his/her officer/team, shall not render void and invalid such seizures of and
representative or counsel, a representative from the media custody over said items[.]" Thus, noncompliance with the strict
and the Department of Justice (DOJ), and any elected directive of Section 21 of R.A. No. 9165 is not necessarily fatal to the
public official who shall be required to sign the copies of the prosecution’s case; police procedures in the handling of confiscated evidence
inventory and be given a copy thereof[.] may still have some lapses, as in the present case. These lapses, however, must
x This is implemented by Section 21(a), Article II of the Implementing Rules and be recognized and explained in terms of their justifiable grounds, and the
Regulations of R.A. No. 9165, which reads: integrity and evidentiary value of the evidence seized must be shown to have
o (a) The apprehending officer/team having initial custody and been preserved.39
control of the drugs shall, immediately after seizure and x In the present case, the prosecution did not bother to offer any
confiscation, physically inventory and photograph the same in the explanation to justify the failure of the police to conduct the required
presence of the accused or the person/s from whom such items physical inventory and photograph of the seized drugs. The apprehending
were confiscated and/or seized, or his/her representative or team failed to show why an inventory and photograph of the seized evidence
counsel, a representative from the media and the Department of had not been made either in the place of seizure and arrest or at the nearest
Justice (DOJ), and any elected public official who shall be required police station (as required by the Implementing Rules in case of warrantless
to sign the copies of the inventory and be given a copy thereof: arrests). We emphasize that for the saving clause to apply, it is important that
Provided, that the physical inventory and photograph shall be the prosecution explain the reasons behind the procedural lapses, and that the
conducted at the place where the search warrant is served; or at integrity and value of the seized evidence had been preserved.40 In other words,
the nearest police station or at the nearest office of the the justifiable ground for noncompliance must be proven as a fact. The court
apprehending officer/team, whichever is practicable, in case of cannot presume what these grounds are or that they even exist.41
warrantless seizures; Provided, further, that non-compliance with x The "Chain of Custody" Requirement
these requirements under justifiable grounds, as long as the x Proof beyond reasonable doubt demands that unwavering exactitude be
integrity and the evidentiary value of the seized items are properly observed in establishing the corpus delicti - the body of the crime whose core is
preserved by the apprehending officer/team, shall not render void the confiscated illicit drug. Thus, every fact necessary to constitute the crime
and invalid such seizures of and custody over said items[.] must be established. The chain of custody requirement performs this function in
x Strict compliance with the prescribed procedure is required because of buy-bust operations as it ensures that doubts concerning the identity of the
the illegal drug's unique characteristic rendering it indistinct, not evidence are removed.42
readily identifiable, and easily open to tampering, alteration or x Black’s Law Dictionary explains chain of custody in this wise:
substitution either by accident or otherwise.28 The records of the o In evidence, the one who offers real evidence, such as the
present case are bereft of evidence showing that the buy-bust team narcotics in a trial of drug case, must account for the custody of the
followed the outlined procedure despite its mandatory terms. The evidence from the moment in which it reaches his custody until the
deficiency is patent from the following exchanges at the trial: moment in which it is offered in evidence, and such evidence goes to
o PROSECUTOR [EMERSON TURINGAN]: weight not to admissibility of evidence. Com. V. White, 353 Mass.
o Q: After you handed this buy-bust money to the accused, what 409, 232 N.E.2d 335.
happened next? x Likewise, Section 1(b) of Dangerous Drugs Board Regulation No. 1, Series of
o [PO3 ALMAREZ:] 2002 which implements R.A. No. 9165 defines "chain of custody" as follows:
o A: When the shabu was already with me and I gave him the o "Chain of Custody" means the duly recorded authorized
money[,] I signaled the two, Captain Jaime de Vera and SPO1 movements and custody of seized drugs or controlled
Balido, sir. chemicals or plant sources of dangerous drugs or
o Q: After you gave that signal, what happened? laboratory equipment of each stage, from the time of
o A: Then they approached us and helped me in arresting Felimon seizure/confiscation to receipt in the forensic laboratory to
Pagaduan, sir. safekeeping to presentation in court for destruction. Such
o Q: After Pagaduan was arrested, what happened next? record of movements and custody of seized item shall include the
o A: After arresting Pagaduan[,] we brought him directly in Diadi identity and signature of the person who held temporary custody of
Police Station, sir. the seized item, the date and time when such transfer of custody
o Q: What happened when you brought the accused to the Police were made in the course of safekeeping and use in court as
Station in Diadi? evidence, and the final disposition[.]
o A: When we were already in Diadi Police Station, we first put him in x In Malillin v. People,43 the Court explained that the chain of custody rule
jail in the Municipal Jail of Diadi, Nueva Vizcaya, sir. requires that there be testimony about every link in the chain, from the moment
o Q: What did you do with the shabu? the object seized was picked up to the time it is offered in evidence, in such a
o A: The request for laboratory examination was prepared and was way that every person who touched it would describe how and from whom it
brought to the Crime Lab. of Solano, Nueva Vizcaya, sir. was received, where it was and what happened to it while in the witness’
o Q: After making the request, what did you do next[,] if any[,] Mr. possession, the condition in which it was received and the condition in which it
Witness? was delivered to the next link in the chain.
o A: After submission of the request to the Crime Lab.[,] we prepared x In the present case, the prosecution’s evidence failed to establish the chain that
our joint affidavit for submission of the case to the Court, sir.29 would have shown that the shabu presented in court was the very same
x From the foregoing exchanges during trial, it is evident that the apprehending specimen seized from the appellant.
team, upon confiscation of the drug, immediately brought the appellant and the x The first link in the chain of custody starts with the seizure of the heat-
seized items to the police station, and, once there, made the request for sealed plastic sachet from the appellant. PO3 Almarez mentioned on cross-
laboratory examination. No physical inventory and photograph of the seized examination that he placed his initials on the confiscated sachet "after
items were taken in the presence of the accused or his counsel, a representative apprehending" the appellant. Notably, this testimony constituted the totality of
from the media and the Department of Justice, and an elective official. PO3 the prosecution’s evidence on the marking of the seized evidence. PO3 Almarez’s
Almarez, on cross-examination, was unsure and could not give a categorical testimony, however, lacked specifics on how he marked the sachet and who
answer when asked whether he issued a receipt for the shabu confiscated from witnessed the marking. In People v. Sanchez, we ruled that the "marking" of the
the appellant.30 At any rate, no such receipt or certificate of inventory appears seized items - to truly ensure that they are the same items that enter the chain
in the records. and are eventually the ones offered in evidence - should be done (1) in the
x In several cases, we have emphasized the importance of compliance with presence of the apprehended violator (2) immediately upon confiscation. In the
the prescribed procedure in the custody and disposition of the seized present case, nothing in the records gives us an insight on the manner and
220
RECTO, GAYLE ANGELI M.
2011-0008 | AUSL
Personal Notes on Remedial Law 2 Review (based on the syllabus of Prof. Henedino M. Brondial)

circumstances that attended the marking of the confiscated sachet. Whether the a female real estate agent named Felicisima de Guzman.
marking had been done in the presence of the appellant is not at all clear from Respondent had seduced a senior police officer in San Isidro and
the evidence that merely mentioned that the evidence had been marked after the her charge of sexual abuse against said police officer was later
appellant’s apprehension. withdrawn in exchange for the quashing of drug charges against
x The second link in the chain of custody is its turnover from the respondent’s brother-in-law who was then detained at the
apprehending team to the police station. PO3 Almarez testified that the municipal jail. It was at that time respondent introduced herself to
appellant was brought to the Diadi Police Station after his arrest. However, he petitioner whom she pleaded for charity as she was pregnant with
failed to identify the person who had control and possession of the seized drug another child. Petitioner denied paternity of the child Christian
at the time of its transportation to the police station. In the absence of clear Paulo; he was motivated by no other reason except genuine
evidence, we cannot presume that PO3 Almarez, as the poseur buyer, handled altruism when he agreed to shoulder the expenses for the delivery
the seized sachet - to the exclusion of others - during its transfer from the place of said child, unaware of respondent’s chicanery and deceit
of arrest and confiscation to the police station. The prosecution likewise failed to designed to “scandalize” him in exchange for financial favor.
present evidence pertaining to the identity of the duty desk officer who received x Matusalem’s evidence:
the plastic sachet containing shabu from the buy-bust team. This is particularly o Respondent testified that she first met petitioner at the house of
significant since the seized specimen was turned over to the PNP Crime his “kumadre” Felicisima de Guzman at Bgy. Malapit, San Isidro,
Laboratory only after two days. It was not, therefore, clear who had temporary Nueva Ecija. During their subsequent meeting, petitioner told her
custody of the seized items during this significant intervening period of time. he is already a widower and he has no more companion in life
Although the records show that the request for laboratory examination of the because his children are all grown-up. She also learned that
seized plastic sachet was prepared by Captain de Vera, the evidence does not petitioner owns a rice mill, a construction business and a housing
show that he was the official who received the marked plastic sachet from the subdivision (petitioner offered her a job at their family-owned Ma.
buy-bust team. Cristina Village). Petitioner at the time already knows that she is a
x As for the subsequent links in the chain of custody, the records show that single mother as she had a child by her former boyfriend in Italy.
the seized specimen was forwarded by PO3 Almarez to the PNP Crime He then brought her to a motel, promising that he will take care of
Laboratory on December 29, 2003, where it was received by PO2 her and marry her. She believed him and yielded to his advances,
Dulnuan, and later examined by PSI Quintero. However, the person from with the thought that she and her child will have a better life.
whom PO3 Almarez received the seized illegal drug for transfer to the Thereafter, they saw each other weekly and petitioner gave her
crime laboratory was not identified. As earlier discussed, the money for her child. When she became pregnant with petitioner’s
identity of the duty desk officer who received the shabu, as well as the child, it was only then she learned that he is in fact not a widower.
person who had temporary custody of the seized items for two days, She wanted to abort the baby but petitioner opposed it because he
had not been established. wanted to have another child.5
x The procedural lapses mentioned above show the glaring gaps in the o On the fourth month of her pregnancy, petitioner rented an
chain of custody, creating a reasonable doubt whether the drugs apartment where she stayed with a housemaid; he also provided
confiscated from the appellant were the same drugs that were brought for all their expenses. She gave birth to their child on December 28,
to the crime laboratory for chemical analysis, and eventually offered in 1994 at the Good Samaritan Hospital in Cabanatuan City. Before
court as evidence. In the absence of concrete evidence on the illegal drugs delivery, petitioner even walked her at the hospital room and
bought and sold, the body of the crime - the corpus delicti - has not been massaged her stomach, saying he had not done this to his wife.
adequately proven.44 In effect, the prosecution failed to fully prove the elements She filled out the form for the child’s birth certificate and wrote all
of the crime charged, creating reasonable doubt on the appellant’s criminal the information supplied by petitioner himself. It was also petitioner
liability. who paid the hospital bills and drove her baby home. He was
x Presumption of Regularity in the Performance of Official Duties excited and happy to have a son at his advanced age who is his
x In sustaining the appellant’s conviction, the CA relied on the evidentiary “look-alike,” and this was witnessed by other boarders, visitors and
presumption that official duties have been regularly performed. This Grace Murillo, the owner of the apartment unit petitioner rented.
presumption, it must be emphasized, is not conclusive.45 It cannot, by itself, However, on the 18th day after the baby’s birth, petitioner went to
overcome the constitutional presumption of innocence. Any taint of irregularity Baguio City for a medical check-up. He confessed to her daughter
affects the whole performance and should make the presumption unavailable. In and eventually his wife was also informed about his having sired an
the present case, the failure of the apprehending team to comply with paragraph illegitimate child. His family then decided to adopt the baby and
1, Section 21, Article II of R.A. No. 9165, and with the chain of custody just give respondent money so she can go abroad. When she
requirement of this Act effectively negates this presumption. As we explained in refused this offer, petitioner stopped seeing her and sending
Malillin v. People:46 money to her. She and her baby survived through the help of
x The presumption of regularity is merely just that - a mere presumption relatives and friends. Depressed, she tried to commit suicide by
disputable by contrary proof and which when challenged by the evidence cannot drug overdose and was brought to the hospital by Murillo who paid
be regarded as binding truth. Suffice it to say that this presumption cannot the bill. Murillo sought the help of the Cabanatuan City Police
preponderate over the presumption of innocence that prevails if not overthrown Station which set their meeting with petitioner. However, it was
by proof beyond reasonable doubt. In the present case the lack of conclusive only petitioner’s wife who showed up and she was very mad,
identification of the illegal drugs allegedly seized from petitioner, coupled with uttering unsavory words against respondent.6
the irregularity in the manner by which the same were placed under police o Matusalem’s witness, Grace Murillo, corroborated respondent’s
custody before offered in court, strongly militates a finding of guilt. testimony as to the payment by petitioner of apartment rental, his
x We are not unmindful of the pernicious effects of drugs in our society; they are weekly visits to respondent and financial support to her, his
lingering maladies that destroy families and relationships, and engender crimes. presence during and after delivery of respondent’s baby,
The Court is one with all the agencies concerned in pursuing an intensive and respondent’s attempted suicide through sleeping pills overdose and
unrelenting campaign against this social dilemma. Regardless of how much we hospitalization for which she paid the bill, her complaint before the
want to curb this menace, we cannot disregard the protection provided by the police authorities and meeting with petitioner’s wife at the
Constitution, most particularly the presumption of innocence bestowed on the headquarters.
appellant. Proof beyond reasonable doubt, or that quantum of proof sufficient to x Salas Æ waived presentation of evidence
produce moral certainty that would convince and satisfy the conscience of those x RTC Æ in favor of MATUSALEM
who act in judgment, is indispensable to overcome this constitutional x Salas Æ appealed to CA
presumption. If the prosecution has not proved, in the first place, all the o (1) the trial court decided the case without affording him the right
elements of the crime charged, which in this case is the corpus delicti, then the to introduce evidence on his defense; and
appellant deserves no less than an acquittal. o (2) the trial court erred in finding that petitioner is the putative
father of Christian Paulo and ordering him to give monthly support.
NARCISO SALAS, Petitioners, v. ANNABELLE MATUSALEM, Respondent. x CA Æ affirmed RTC
G.R. No. 180284, September 11, 2013 o Christian Paulo, in instant case, does not enjoy the benefit of a
FIRST DIVISION record of birth in the civil registry which bears acknowledgment
signed by Narciso Salas. He cannot claim open and continuous
FACTS: possession of the status of an illegitimate child.
x Matusalem claims that Salas is the father of her child o It had been established by plaintiff’s evidence, however, that
o That he was 56 and she was 24 and he made her believe that he during her pregnancy, Annabelle was provided by Narciso Salas
was a widower with an apartment at a rental of P1,500.00 which he paid for (TSN,
o That Salas once rented an apartment where she stayed October 6, 1995, p. 18). Narciso provided her with a household
o That Salas likewise paid for all the hospital bills when she gave help with a salary of P1,500.00 a month (TSN, October 6, 1995,
birth ibid). He also provided her a monthly food allowance of P1,500.00
o BUT when Matusalem refused Salas’ offer that the latter’s family (Ibid, p. 18). Narciso was with Annabelle at the hospital while the
will take care of the child, Salas left her and refused to give support latter was in labor, “walking” her around and massaging her belly
x Matusalem Æ filed a petition for support with prayer for SPL before RTC (Ibid, p. 11). Narciso brought home Christian Paulo to the rented
Cabanatuan City against Salas apartment after Annabelle’s discharge from the hospital. People
x Salas Æ filed his answer living in the same apartment units were witnesses to Narciso’s
o He described respondent as a woman of loose morals, having delight to father a son at his age which was his “look alike”. It was
borne her first child also out of wedlock when she went to work in only after the 18th day when Annabelle refused to give him
Italy. Jobless upon her return to the country, respondent spent Christian Paulo that Narciso withdrew his support to him and his
time riding on petitioner’s jeepney which was then being utilized by mother.
221
RECTO, GAYLE ANGELI M.
2011-0008 | AUSL
Personal Notes on Remedial Law 2 Review (based on the syllabus of Prof. Henedino M. Brondial)

o Said testimony of Annabelle aside from having been corroborated inexcusable negligence. It should be borne in mind that a client is
by Grace Murillo, the owner of the apartment which Narciso rented, bound by his counsel’s conduct, negligence and mistakes in
was never rebutted on record. Narciso did not present any handling the case.22
evidence, verbal or documentary, to repudiate plaintiff’s evidence. x With our finding that there was no abuse of discretion in the trial court’s denial
o In the cases of Lim vs. CA (270 SCRA 1) and Rodriguez vs. CA (245 of the motion for postponement filed by petitioner’s counsel, petitioner’s
SCRA 150), the Supreme Court made it clear that Article 172 of the contention that he was deprived of his day in court must likewise fail. The
Family Code is an adaptation of Article 283 of the Civil Code. Said essence of due process is that a party is given a reasonable opportunity to be
legal provision provides that the father is obliged to recognize the heard and submit any evidence one may have in support of one’s defense.
child as his natural child x x “3) when the child has in his favor any Where a party was afforded an opportunity to participate in the proceedings but
evidence or proof that the defendant is his father”. failed to do so, he cannot complain of deprivation of due process. If the
x Salas Æ filed a Rule 45 before the SC opportunity is not availed of, it is deemed waived or forfeited without violating
the constitutional guarantee.

ISSUE # 1: Whether the RTC correctly ruled that Salas has waived his right to present ISSUE # 2: Whether Matusalem was able to sufficiently prove that Salas is indeed the father
evidence. of her child.

HELD # 1: YES. HELD # 2: NO.


x As to the denial of the motion for postponement filed by his counsel for the x We now proceed to the main issue of whether the trial and appellate courts
resetting of the initial presentation of defense evidence on April 17, 1998, we erred in ruling that respondent’s evidence sufficiently proved that her son
find that it was not the first time petitioner’s motion for postponement was Christian Paulo is the illegitimate child of petitioner.
denied by the trial court. x Under Article 175 of the Family Code of the Philippines, illegitimate
x Records disclosed that after the termination of the testimony of respondent’s last filiation may be established in the same way and on the same evidence
witness on November 29, 1996, the trial court as prayed for by the parties, set as legitimate children.
the continuation of hearing for the reception of evidence for the defendant x Article 172 of the Family Code of the Philippines states:
(petitioner) on January 27, February 3, and February 10, 1997. In the Order o The filiation of legitimate children is established by any of the
dated December 17, 1996, petitioner was advised to be ready with his evidence following:
at those hearing dates earlier scheduled. At the hearing on January 27, 1997, ƒ (1) The record of birth appearing in the civil register
petitioner’s former counsel, Atty. Rolando S. Bala, requested for the cancellation or a final judgment; or
of the February 3 and 10, 1997 hearings in order to give him time to prepare for ƒ (2) An admission of legitimate filiation in a public
his defense, which request was granted by the trial court which thus reset the document or a private handwritten instrument and
hearing dates to March 3, 14 and 17, 1997. On March 3, 1997, upon oral signed by the parent concerned.
manifestation by Atty. Bala and without objection from respondent’s counsel, o In the absence of the foregoing evidence, the legitimate filiation
Atty. Feliciano Wycoco, the trial court again reset the hearing to March 14 and shall be proved by:
17, 1997. With the non-appearance of both petitioner and Atty. Bala on March ƒ (1) The open and continuous possession of the status
14, 1997, the trial court upon oral manifestation by Atty. Wycoco declared their of a legitimate child; or
absence as a waiver of their right to present evidence and accordingly deemed ƒ (2) Any other means allowed by the Rules of Court
the case submitted for decision.16 and special laws. (Underscoring supplied.)
x On July 4, 1997, Atty. Bala withdrew as counsel for petitioner and Atty. Rafael E. x Respondent presented the Certificate of Live Birth24 (Exhibit “A-1”) of
Villarosa filed his appearance as his new counsel on July 21, 1997. On the same Christian Paulo Salas in which the name of petitioner appears as his father but
date he filed entry of appearance, Atty. Villarosa filed a motion for which is not signed by him. Admittedly, it was only respondent who filled
reconsideration of the March 14, 1997 Order pleading for liberality and up the entries and signed the said document though she claims it was
magnanimity of the trial court, without offering any explanation for Atty. Bala’s petitioner who supplied the information she wrote therein.
failure to appear for the initial presentation of their evidence. The trial court x We have held that a certificate of live birth purportedly identifying the putative
thereupon reconsidered its March 14, 1997 Order, finding it better to give father is not competent evidence of paternity when there is no showing
petitioner a chance to present his evidence. On August 26, 1997, Atty. Villarosa that the putative father had a hand in the preparation of the
received a notice of hearing for the presentation of their evidence scheduled on certificate.25 Thus, if the father did not sign in the birth certificate, the
September 22, 1997. On August 29, 1997, the trial court received his motion placing of his name by the mother, doctor, registrar, or other person is
requesting that the said hearing be re-set to October 10, 1997 for the reason incompetent evidence of paternity.26 Neither can such birth certificate
that he had requested the postponement of a hearing in another case which was be taken as a recognition in a public instrument27 and it has no
incidentally scheduled on September 22, 23 and 24, 1997. As prayed for, the trial probative value to establish filiation to the alleged father.28 x As to
court reset the hearing to October 10, 1997. On said date, however, the hearing the Baptismal Certificate29 (Exhibit “B”) of Christian Paulo Salas also
was again moved to December 15, 1997. On February 16, 1998, the trial court indicating petitioner as the father, we have ruled that while baptismal certificates
itself reset the hearing to April 17, 1998 since it was unclear whether Atty. may be considered public documents, they can only serve as evidence of the
Wycoco received a copy of the motion.17 administration of the sacraments on the dates so specified. They are not
x On April 17, 1998, petitioner and his counsel failed to appear but the trial court necessarily competent evidence of the veracity of entries therein with respect to
received on April 16, 1998 an urgent motion to cancel hearing filed by Atty. the child’s paternity.30
Villarosa. The reason given by the latter was the scheduled hearing on the x The rest of respondent’s documentary evidence consists of handwritten
issuance of writ of preliminary injunction in another case under the April 8, 1998 notes and letters, hospital bill and photographs taken of petitioner and
Order issued by the RTC of Gapan, Nueva Ecija, Branch 36 in Civil Case No. respondent inside their rented apartment unit.
1946. But as clearly stated in the said order, it was the plaintiffs therein who o Pictures taken of the mother and her child together with
requested the postponement of the hearing and it behoved Atty. Villarosa to the alleged father are inconclusive evidence to prove
inform the RTC of Gapan that he had a previous commitment considering that paternity.31
the April 17, 1998 hearing was scheduled as early as February 16, 1998. Acting o Exhibits “E” and “F”32 showing petitioner and respondent inside
on the motion for postponement, the trial court denied for the second time the rented apartment unit thus have scant evidentiary value.
petitioner’s motion for postponement. Even at the hearing of their motion for o The Statement of Account33 (Exhibit “C”) from the Good
reconsideration of the April 17, 1998 Order on September 21, 1998, Atty. Samaritan General Hospital where respondent herself was
Villarosa failed to appear and instead filed another motion for postponement. The indicated as the payee is likewise incompetent to prove that
trial court thus ordered that the case be submitted for decision stressing that the petitioner is the father of her child notwithstanding petitioner’s
case had long been pending and that petitioner and his counsel have been given admission in his answer that he shouldered the expenses in the
opportunities to present their evidence. It likewise denied a second motion for delivery of respondent’s child as an act of charity.
reconsideration filed by Atty. Villarosa, who arrived late during the hearing thereof o As to the handwritten notes34 (Exhibits “D” to “D-13”) of
on December 4, 1998.18 petitioner and respondent showing their exchange of
x A motion for continuance or postponement is not a matter of right, but affectionate words and romantic trysts, these, too, are not
a request addressed to the sound discretion of the court. Parties asking sufficient to establish Christian Paulo’s filiation to
for postponement have absolutely no right to assume that their petitioner as they were not signed by petitioner and
motions would be granted. Thus, they must be prepared on the day of contained no statement of admission by petitioner that he
the hearing.19 Indeed, an order declaring a party to have waived the is the father of said child. Thus, even if these notes were
right to present evidence for performing dilatory actions upholds the authentic, they do not qualify under Article 172 (2) vis-à- vis Article
trial court’s duty to ensure that trial proceeds despite the deliberate 175 of the Family Code which admits as competent evidence of
delay and refusal to proceed on the part of one party.20 illegitimate filiation an admission of filiation in a private handwritten
x Atty. Villarosa’s plea for liberality was correctly rejected by the trial court in view instrument signed by the parent concerned.35
of his own negligence in failing to ensure there will be no conflict in his trial x Petitioner’s reliance on our ruling in Lim v. Court of Appeals36 is misplaced. In
schedules. As we held in Tiomico v. Court of the said case, the handwritten letters of petitioner contained a clear
Appeals21:chanRoblesvirtualLawlibrary admission that he is the father of private respondent’s daughter and
o Motions for postponement are generally frowned upon by were signed by him. The Court therein considered the totality of
Courts if there is evidence of bad faith, malice or evidence which established beyond reasonable doubt that petitioner
inexcusable negligence on the part of the movant. The was indeed the father of private respondent’s daughter. On the other
inadvertence of the defense counsel in failing to take note of the hand, in Ilano v. Court of Appeals,37 the Court sustained the appellate court’s
trial dates and in belatedly informing the trial court of any conflict finding that private respondent’s evidence to establish her filiation with and
in his schedules of trial or court appearances, constitutes paternity of petitioner was overwhelming, particularly the latter’s public
222
RECTO, GAYLE ANGELI M.
2011-0008 | AUSL
Personal Notes on Remedial Law 2 Review (based on the syllabus of Prof. Henedino M. Brondial)

acknowledgment of his amorous relationship with private respondent’s mother, also held that the death of the putative father is not a bar to the action
and private respondent as his own child through acts and words, her testimonial commenced during his lifetime by one claiming to be his illegitimate child.43 The
evidence to that effect was fully supported by documentary evidence. The Court rule on substitution of parties provided in Section 16, Rule 3 of the 1997 Rules of
thus ruled that respondent had adduced sufficient proof of continuous possession Civil Procedure, thus applies.
of status of a spurious child. o SEC. 16. Death of party; duty of counsel. - Whenever a party to a
x Here, while the CA held that Christian Paulo Salas could not claim open and pending action dies, and the claim is not thereby extinguished, it
continuous possession of status of an illegitimate child, it nevertheless shall be the duty of his counsel to inform the court within thirty
considered the testimonial evidence sufficient proof to establish his filiation to (30) days after such death of the fact thereof, and to give the
petitioner. name and address of his legal representative or representatives.
x An illegitimate child is now also allowed to establish his claimed filiation by “any Failure of counsel to comply with his duty shall be a ground for
other means allowed by the Rules of Court and special laws,” like his baptismal disciplinary action.
certificate, a judicial admission, a family Bible in which his name has been o The action must be brought within the same period specified in
entered, common reputation respecting his pedigree, admission by silence, the Article 173, except when the action is based on the second
testimonies of witnesses, and other kinds of proof admissible under Rule 130 of paragraph of Article 172, in which case the action may be brought
the Rules of Court.38 Reviewing the records, we find the totality of respondent’s during the lifetime of the alleged parent.
evidence insufficient to establish that petitioner is the father of Christian Paulo. o The heirs of the deceased may be allowed to be substituted for the
x The testimonies of respondent and Murillo as to the circumstances of the birth of deceased, without requiring the appointment of an executor or
Christian Paulo, petitioner’s financial support while respondent lived in Murillo’s administrator and the court may appoint a guardian ad litem for
apartment and his regular visits to her at the said apartment, though replete the minor heirs.
with details, do not approximate the “overwhelming evidence, documentary and o The court shall forthwith order said legal representative or
testimonial” presented in Ilano. In that case, we sustained the appellate court’s representatives to appear and be substituted within a period of
ruling anchored on the following factual findings by the appellate court which was thirty (30) days from notice.
quoted at length in the ponencia:chanRoblesvirtualLawlibrary o If no legal representative is named by the counsel for the deceased
o It was Artemio who made arrangement for the delivery of party, or if the one so named shall fail to appear within the
Merceditas (sic) at the Manila Sanitarium and Hospital. Prior to the specified period, the court may order the opposing party, within a
delivery, Leoncia underwent prenatal examination accompanied by specified time to procure the appointment of an executor or
Artemio (TSN, p. 33, 5/17/74). After delivery, they went home to administrator for the estate of the deceased and the latter shall
their residence at EDSA in a car owned and driven by Artemio immediately appear for and on behalf of the deceased. The court
himself (id. p. 36). charges in procuring such appointment, if defrayed by the opposing
o Merceditas (sic) bore the surname of “Ilano” since birth without any party, may be recovered as costs.
objection on the part of Artemio, the fact that since Merceditas
(sic) had her discernment she had always known and called PEOPLE OF THE PHILIPPINES, Plaintiff-Appellee, vs. RUPER POSING y ALAYON,
Artemio as her “Daddy” (TSN, pp. 28-29, 10/18/74); the fact that Accused-Appellant.
each time Artemio was at home, he would play with Merceditas G.R. No. 196973 July 31, 2013
(sic), take her for a ride or restaurants to eat, and sometimes SECOND DIVISION
sleeping with Merceditas (sic) (id. p. 34) and does all what a father
should do for his child — bringing home goodies, candies, toys and FACTS:
whatever he can bring her which a child enjoys which Artemio x Posing was charged before the RTC for violation of Sections 53 and 114
gives to Merceditas (sic) (TSN, pp. 38-39, 5/17/74) are positive respectively, Article II of Republic Act (R.A.) No. 9165 or the Comprehensive
evidence that Merceditas (sic) is the child of Artemio and Dangerous Drugs Act of 2002
recognized by Artemio as such. Special attention is called to Exh. x Prosecution evidence:
“E-7” where Artemio was telling Leoncia the need for a “frog test” o Chief P/Inspector Arturo Caballes (Chief Caballes) formed a team to
to know the status of Leoncia. conduct a buy bust operation after being informed of the illegal
o Plaintiff pointed out that the support by Artemio for Leoncia and selling of drugs of Posing
Merceditas (sic) was sometimes in the form of cash personally o SPO1 Angeles together with PO1 Jesus Cortez (PO1 Cortez),
delivered to her by Artemio, thru Melencio, thru Elynia (Exhs. “E-2” PO1Ralph Nicart (PO1 Nicart) and the informant were dispatched to
and “E-3”, and “D-6”), or thru Merceditas (sic) herself (TSN, p. 40, Makabayan St., Brgy. Obrero, Kamuning, Quezon City, and upon
5/17/74) and sometimes in the form of a check as the Manila arrival, the informant and SPO1 Angeles proceeded to the
Banking Corporation Check No. 81532 (Exh. “G”) and the signature squatter’s area
appearing therein which was identified by Leoncia as that of o SPO1 Angeles met Posing beside the basketball court, where he
Artemio because Artemio often gives her checks and Artemio would was introduced by the informant as a buyer of shabu. The former
write the check at home and saw Artemio sign the check (TSN, p. asked if he could buy one hundred peso (P100.00) worth of shabu
49, 7/18/73). Both Artemio and Nilda admitted that the check and for personal use.
signature were those of Artemio (TSN, p. 53, 10/17/77; TSN, p. 19, o Posing then pulled out one (1) transparent plastic sachet from his
10/9/78). pocket and gave it to SPO1 Angeles in exchange for the buy-bust
o During the time that Artemio and Leoncia were living as husband money.
and wife, Artemio has shown concern as the father of Merceditas o Afterwards, SPO1 Angeles took out his cap to alert his companions
(sic). When Merceditas (sic) was in Grade 1 at the St. Joseph that the deal was already concluded.
Parochial School, Artemio signed the Report Card of Merceditas o PO1 Cortez and PO1 Nicart rushed to the scene and introduced
(sic) (Exh. “H”) for the fourth and fifth grading period(s) (Exh. “H- themselves as police officers. Posing was frisked, and the buy-bust
1” and “H-2”) as the parent of Merceditas (sic). Those signatures of money and another transparent plastic sachet were recovered from
Artemio [were] both identified by Leoncia and Merceditas (sic) him.
because Artemio signed Exh. “H-1” and “H-2” at their residence in o Prior to the turnover of the evidence to the desk officer, SPO1
the presence of Leoncia, Merceditas (sic) and of Elynia (TSN, p. 57, Angeles placed his marking on the two (2) small heat sealed
7/18/73; TSN, p. 28, 10/1/73). x x x. transparent plastic sachets.9 The same were then turned over to
o When Artemio run as a candidate in the Provincial Board of PO2 John Sales (PO2 Sales), who prepared a request for laboratory
Cavite[,] Artemio gave Leoncia his picture with the following examination.10
dedication: “To Nene, with best regards, Temiong”. (Exh. “I”). (pp. o On the same day, the specimens were delivered by PO1 Nicart to
19-20, Appellant’s Brief) the Philippine National Police (PNP) Crime Laboratory for
o The mere denial by defendant of his signature is not sufficient to quantitative and qualitative examination, wherein each sachet was
offset the totality of the evidence indubitably showing that the found to contain 0.03 gram and tested positive for
signature thereon belongs to him. The entry in the Certificate of methylamphetamine hydrochloride or shabu, a dangerous drug
Live Birth that Leoncia and Artemio was falsely stated therein as x Defense evidence:
married does not mean that Leoncia is not appellee’s daughter. o On the contrary, Posing testified that on 13 August 2004, between
This particular entry was caused to be made by Artemio himself in 4:00 to 5:00 o’clock in the afternoon, he was walking along a
order to avoid embarrassment.39 basketball court at Makabayan St., Kamuning, Quezon City, when
o In sum, we hold that the testimonies of respondent and Murillo, by he was arrested by PO1 Cortez and PO1 Nicart, who he came to
themselves are not competent proof of paternity and the totality of know based on their name plates.14 When he asked the officers
respondent’s evidence failed to establish Christian Paulo’s filiation what his violation was, they replied: "Nag-mamaang-maangan ka
to petitioner. pa."15 He was then led to their vehicle and was brought to Station
x Time and again, this Court has ruled that a high standard of proof is 10 wherein he was asked to point to a certain "Nene" whom he did
required to establish paternity and filiation. An order for recognition not know. He refused, which was why he was detained and
and support may create an unwholesome situation or may be an charged with violation of R.A. No. 9165
irritant to the family or the lives of the parties so that it must be issued x RTC Æ convicted Posing
only if paternity or filiation is established by clear and convincing x Posing Æ appealed to CA
evidence.40 o contended that the trial court gravely erred when it failed to
x Finally, we note the Manifestation and Motion41 filed by petitioner’s counsel consider the police officers’ failure to comply with the proper
informing this Court that petitioner had died on May 6, 2010. procedure in the handling and custody of the seized drugs, as
x The action for support having been filed in the trial court when petitioner was provided under Section 21 of R. A. No. 9165, which ultimately
still alive, it is not barred under Article 175 (2)42 of the Family Code. We have affected the chain of custody of the confiscated drugs.21
223
RECTO, GAYLE ANGELI M.
2011-0008 | AUSL
Personal Notes on Remedial Law 2 Review (based on the syllabus of Prof. Henedino M. Brondial)

o Further, it was posited that there was no prior surveillance Request for Laboratory Examination, and on the same day, the specimens were
conducted to verify the informant’s tip and that there was no delivered by PO1 Nicart to the PNP Crime Laboratory for quantitative and
coordination made with the Philippine Drug Enforcement Agency qualitative examination, conducted by Engr. Jabonillo.37
(PDEA).22 Furthermore, the accused-appellant invoked his right to
be presumed innocent until proven guilty beyond reasonable doubt x The same was corroborated by PO1 Sales and Engr. Jabonillo, whose testimonies
x OSG Æ filed its brief were dispensed with, and formed part of the stipulations of facts agreed upon by
o countered that although the requirements under Section 21 of R. A. both the prosecution and defense.38
No. 9165 has been held to be mandatory, non-compliance with the x The defense kept on harping on alleged lapses in the procedure observed by the
same, does not necessarily warrant an acquittal.24 apprehending officers, like SPO1 Angeles’ failure to recall the duty officer to whom
o In addition, it was averred that the police officers are entitled to he turned over the specimens, and the officer who brought the specimens to the
the presumption of regularity in the performance of official duties. crime laboratory. Also, they questioned the absence of an inventory report of the
Finally, the accused-appellant did not interpose any evidence in confiscated drugs and that there were no photographs taken in the presence of
support of his defense aside from his bare denial the accused-appellant and that of a representative from the media or the
x CA Æ affirmed RTC Department of Justice or any elected public officer.
o ruled that the requisites laid down under Section 21 of R. A. No. x Section 21, paragraph 1, Article II of Republic Act No. 9165 provides:
9165 were complied with, more particularly, through the o Section 21. Custody and Disposition of Confiscated, Seized, and/or
testimonies of the police officers which sufficiently established that Surrendered Dangerous Drugs, Plant Sources of Dangerous Drugs,
the integrity and the evidentiary value of the seized items were Controlled Precursors and Essential Chemicals,
preserved.27 Instruments/Paraphernalia and/or Laboratory Equipment. - The
o As to the alleged non-coordination with the PDEA, it was held that PDEA shall take charge and have custody of all dangerous drugs,
although the PDEA is the lead agency, it is not to be considered as plant sources of dangerous drugs, controlled precursors and
the exclusive agency, in enforcing drug-related matters. essential chemicals, as well as instruments/paraphernalia and/or
o Lastly, the evidence presented by the prosecution clearly showed laboratory equipment so confiscated, seized and/or surrendered,
that the elements of illegal sale and possession of dangerous drugs for proper disposition in the following manner:
were proven by competent evidence, as compared to the bare ƒ (1) The apprehending team having initial custody and
denial interposed by the accused-appellant control of the drugs shall, immediately after seizure
x Posing Æ filed a Rule 45 before the SC and confiscation, physically inventory and photograph
the same in the presence of the accused or the
ISSUE: Whether it was sufficiently shown that the sachets of shabu presented before the RTC person/s from whom such items were confiscated
were the same ones obtained from Posing. and/or seized, or his/her representative or counsel, a
representative from the media and the Department of
HELD: YES. Justice (DOJ), and any elected public official who shall
x In cases involving violations of Dangerous Drugs Act, credence should be given be required to sign the copies of the inventory and be
to the narration of the incident by the prosecution witnesses especially when given a copy thereof; x x x
they are police officers who are presumed to have performed their duties in a x But time and again, jurisprudence is consistent in stating that less than
regular manner, unless there is evidence to the contrary. In this regard, the strict compliance with the procedural aspect of the chain of custody
defense failed to show any ill motive or odious intent on the part of the police rule does not necessarily render the seized drug items inadmissible.39
operatives to impute such a serious crime that would put in jeopardy the life and x As held in People v. Llanita40 as cited in People v. Ara:41
liberty of an innocent person, such as in the case of appellant. Incidentally, if o RA 9165 and its subsequent Implementing Rules and Regulations
these were simply trumped-up charges against him, it remains a question why no (IRR) do not require strict compliance as to the chain of custody
administrative charges were brought against the police operatives. Moreover, in rule. x x x We have emphasized that what is essential is "the
weighing the testimonies of the prosecution witnesses vis-à-vis those of the preservation of the integrity and the evidentiary value of the seized
defense, it is a well-settled rule that in the absence of palpable error or grave items, as the same would be utilized in the determination of the
abuse of discretion on the part of the trial judge, the trial court’s evaluation of the guilt or innocence of the accused." Briefly stated, non-compliance
credibility of witnesses will not be disturbed on appeal.34 with the procedural requirements under RA 9165 and its IRR
x With the illegal sale of dangerous drugs established beyond reasonable doubt, relative to the custody, photographing, and drugtesting of the
the handling of the evidence, or the observance of the proper chain of custody, apprehended persons, is not a serious flaw that can render void the
which is also an indispensable factor in prosecution for illegal sale of dangerous seizures and custody of drugs in a buy-bust operation.
drugs, is the next matter to be resolved. x As to the charge of illegal possession of dangerous drugs, the prosecution must
x The accused-appellant, argued that the following instances would constitute a establish the following elements:
break in the chain of custody of the seized plastic sachets of shabu: o (1) the accused is in possession of an item or object, which is
o (1) SPO1 Angeles failed to identify the duty officer to whom identified to be a prohibited or regulated drug;
he turned over the alleged confiscated shabu; o (2) such possession is not authorized by law; and
o (2) SPO1 Angeles was not able to recall who brought the drug o (3) the accused freely and consciously possessed the drug.42
specimens to the crime laboratory; x In the case at hand, the prosecution was able to prove that the accused-
o (3) SPO1 Angeles failed to mark the confiscated sachets at appellant was in possession of one (1) plastic sachet of shabu, when he was
the crime scene immediately after the accused-appellant was frisked on the occasion of his arrest. There was also no showing that he had the
arrested; and authority to possess the drugs that was in his person.
o (4) the police officers failed to prepare an inventory report of x This Court held in a catena of cases that mere possession of a regulated drug
the confiscated drugs, no photographs of the same were per se constitutes prima facie evidence of knowledge or animus possidendi
taken in the presence of the accused-appellant and that of a sufficient to convict an accused absent a satisfactory explanation of such
representative from the media or the Department of Justice or any possession - the onus probandi is shifted to the accused, to explain the
elected public official.35 absence·ofknowledge or animus possidendi.43
x Section 1(b) of Dangerous Drugs Board Regulation No. 1, Series of 2002 which x In fine, considering the pieces of evidence presented by the prosecution, the
implements R.A. No. 9165 defines "Chain of Custody" as follows: denial of the accused-appellant fails.1âwphi1 Courts generally view the defense of
o "Chain of Custody" means the duly recorded authorized movements denial with disfavor due to the facility with which an accused can concoct it to suit
and custody of seized drugs or controlled chemicals or plant his or her defense. As evidence that is both negative and self-serving, this defense
sources of dangerous drugs or laboratory equipment of each stage, cannot attain more credibility than the testimonies of the prosecution witnesses
from the time of seizure/confiscation to receipt in the forensic who testify clearly, providing thereby positive evidence on the various aspects of
laboratory to safekeeping to presentation in court for destruction. the crime committed.44
Such record of movements and custody of seized item shall include x Also, it is a well-entrenched principle that findings of fact of the trial court as to
the identity and signature of the person who held temporary the credibility of witnesses are accorded great weight and respect when no
custody of the seized item, the date and time when such transfer of glaring errors, gross misapprehension of facts, and speculative, arbitrary and
custody were made in the course of safekeeping and use in court unsupported conclusions can be gathered from such findings. The rationale
as evidence, and the final disposition. behind this rule is that the trial court is in a better position to decide the
x In Malillin v. People,36 we laid down the chain of custody requirements that credibility of witnesses, having heard their testimonies and observed their
must be met in proving that the seized drugs are the same ones presented in deportment and manner of testifying during trial. This rule finds an even more
court: stringent application where said findings are sustained by the Court of
o (1) testimony about every link in the chain, from the moment Appeals.45 This Court does not find any convincing reason to depart from the
the item was picked up to the time it is offered into evidence; and ruling of the trial court, which was affirmed by the appellate court.
o (2) witnesses should describe the precautions taken to
ensure that there had been no change in the condition of PEOPLE OF THE PHILIPPINES, Plaintiff-Appellee, vs. ASIR GANI y ALIH and NORMINA
the item and no opportunity for someone not in the chain to have GANI y GALOS, Accused-Appellants.
possession of the item. G.R. No. 198318 November 27, 2013
x In this case, the prosecution was able to prove, through the testimonies of its FIRST DIVISION
witnesses that the integrity of the seized item was preserved every step of the
process. After the sale of shabu and another sachet was discovered in the person FACTS:
of accused-appellant, SPO1 Angeles, who was the poseur-buyer in the buy-bust x Asir and Normina were charged before the RTC Pasig with conspiracy for illegal
operation, marked the drug specimens, and then turned over the same to the sale of dangerous drugs defined and penalized under Article II, Section 5 of
desk officer, who in turn handed it to PO1 Sales. The latter then prepared a
224
RECTO, GAYLE ANGELI M.
2011-0008 | AUSL
Personal Notes on Remedial Law 2 Review (based on the syllabus of Prof. Henedino M. Brondial)

Republic Act No. 9165 otherwise known as the Dangerous Drugs Act of 2002, in x In the prosecution for the crime of illegal sale of prohibited drugs, the following
relation to Paragraph 2, Article 62 of the Revised Penal Code. elements must concur:
x Prosecution evidence: o (1) the identities of the buyer and seller, object, and consideration;
o The prosecution presented the testimonies of Special Investigator and
(SI) Elson Saul (Saul),6 SI Joel Otic (Otic),7 SI Salvador Arteche, o (2) the delivery of the thing sold and the payment thereof.
Jr. (Arteche),8 SI Melvin Escurel (Escurel),9 and Atty. Ross x What is material to the prosecution for illegal sale of dangerous drugs
Jonathan Galicia (Galicia),10 all of the National Bureau of is the proof that the transaction or sale actually occurred, coupled with
Investigation (NBI) assigned to the Special Enforcement Services of the presentation in court of the substance seized as evidence.27
the Philippine Drug Enforcement Agency (PDEA). The prosecution
dispensed with the presentation of the testimony of NBI Forensic x It has been clearly established herein that a buy-bust operation took place on
Chemist II Rommel G. Patingo, who conducted the chemical May 6, 2004 conducted by a team of NBI agents. SI Saul, as the poseur-buyer,
analysis of the specimens submitted for his examination, since the and accused-appellants, as the sellers, agreed on the price of One Hundred Fifty
subject matter of his testimony had already been stipulated during Thousand Pesos (P150,000.00) for One Hundred (100) grams of shabu. After SI
the pre-trial conference. Saul handed over the buy-bust money to accused-appellants, the latter gave
o The documentary evidence for the prosecution consisted of the him, in exchange, two plastic sachets containing white crystalline substance.
NBI-PDEA Pre-Operation Report11 dated May 6, 2004; Coordination Thereafter, accused-appellants were immediately arrested by the buy-bust team.
Letter12 dated May 6, 2004 of NBI-PDEA to the Chief of Police of During the search incidental to accused-appellants’ arrest, a .45 caliber handgun,
Taguig, City; Joint Affidavit of Arrest13 dated May 7, 2004 signed the buy-bust money, and two more sachets of suspected shabu were recovered
by several members of the buy-bust team, namely, SI Saul, SI Otic, from their possession. Chemical examination confirmed that the contents of the
SI Arteche, Atty. Galicia, SI Antonio Erum, SI Garry I. Meñez, SI two plastic sachets sold to SI Saul were indeed shabu. These two sachets of
Bertrand Gamaliel A. Mendoza, and SI Junnel Malaluan; Booking shabu, marked "ES-1 05-06-04" and "ES-2 05-06-04" and with a total weight of
Sheet and Arrest Report14 of each accused-appellant; Inventory of 98.7249 grams, together with two other sachets, were duly presented as
Seized Properties15 signed by SI Saul and two witnesses; buy-bust evidence by the prosecution before the RTC.
money consisting of two P1,000.00 bills and several pieces of x Contrary to accused-appellants’ averment, prosecution witness, SI Saul, was
P20.00 bills;16 request dated May 7, 2004 for the laboratory able to explain why there were a total of four sachets of shabu
examination of "two (2) transparent heat-sealed plastic sachets presented during trial, when SI Saul only bought two sachets during the
containing undetermined amount of white crystalline substance" buy- bust operation. SI Saul testified that in addition to the two plastic
recovered from accused-appellants and marked "ES-1 05-06-04" sachets of shabu sold to him by accused-appellants, there were two
and "ES-2 05-06-04";17 Dangerous Drugs Report No. DD-04-16118 more sachets of shabu recovered from accused-appellants’
dated May 13, 2004 prepared by NBI Forensic Chemist II Patingo possession by the buy-bust team during the body search conducted
and Forensic Chemist III Aida R. Viloria-Magsipoc (Viloria- incidental to accused-appellants’ lawful arrest.28
Magsipoc); a brown envelope and four plastic sachets of shabu, x The Court further finds that the arresting officers had substantially
including the two sachets marked "ES-1 05-06-04" and "ES-2 05- complied with the rule on the chain of custody of the dangerous drugs
06-04";19 and the Toxicology Report Nos. TDD-04-1788 and TDD- as provided under Section 21 of Republic Act No. 9165. Jurisprudence
04-178920 prepared by NBI Forensic Chemist II Patingo and has decreed that, in dangerous drugs cases, the failure of the police
Forensic Chemist III Viloria-Magsipoc. officers to make a physical inventory and to photograph the sachets of
x Defense evidence: shabu, as well as to mark the sachets at the place of arrest, do not
o The evidence for the defense consisted of accused-appellants’ render the seized drugs inadmissible in evidence or automatically
testimonies.21 Both denied the crime charged against them and impair the integrity of the chain of custody of the said drugs.29 What is
claimed that they were the victims of extortion. They were charged of utmost importance is the preservation of the integrity and the
only because they failed to produce the money demanded from evidentiary value of the seized items, as these would be utilized in the
them. determination of the guilt or innocence of the accused.30
x RTC Æ convicted both Asir and Normina x In this case, testimonial and documentary evidence for the prosecution proved
x Asir and Normina Æ appealed to CA that immediately after accused-appellants’ arrest, they were brought to the FTI
x CA Æ affirmed Barangay Hall. It was there, in the presence of two barangay officials, that SI
o The appellate court accorded weight to the assessment by the RTC Saul conducted an inventory of the two plastic sachets of shabu subject of the
of the veracity of the witnesses’ testimonies. The prosecution buy-bust operation, plus the other items seized from accused-appellants’
witnesses gave a clear and candid narration of the buy-bust possession during the search conducted incidental to accused-appellants’ arrest.
operation against accused-appellants; while accused-appellants’ It was also at the barangay hall where SI Saul marked the two plastic sachets of
denial and alibi fail in the absence of clear and convincing evidence shabu sold to him by accused-appellants as "ES-1 05-06-04" and "ES-2 05-06-
of ill motive or bad faith on the part of the buy-bust team. The 04," representing SI Saul’s initials and the date of the buy-bust operation.
appellate court also declared that there was substantial compliance Thereafter, the buy-bust team, with accused-appellants, proceeded to the NBI
with the rule on the chain of custody of the seized drugs, thus, Headquarters. At the NBI Headquarters, SI Saul made a request for examination
preserving the integrity and evidentiary value of the same of the two plastic sachets of shabu, marked "ES-1 05-06-04" and "ES-2 05-06-
x Asir and Normina Æ filed Rule 45 before the SC 04," and personally handed the same to NBI Forensic Chemist II Patingo. NBI
Forensic Chemist II Patingo, together with NBI Forensic Chemist III Viloria-
ISSUE: Whether it was sufficiently shown that the sachets of shabu presented before the RTC Magsipoc, conducted the laboratory examination of the contents of the two
were the same ones obtained from Asir and Normina. sachets marked "ES-1 05-06-04" and "ES-2 05-06-04" and kept said sachets in
his custody until the same were submitted to the RTC as evidence during trial.
HELD: YES. x Thus, the Court of Appeals was correct in its observation that the failure
x In their Brief, accused-appellants assert that the prosecution failed to comply of the buy-bust team to take pictures of the seized drugs immediately
with the rules on the custody of seized drugs provided under Section 21 of upon seizure and at the site of accused-appellants’ apprehension, and
Republic Act No. 9165. According to accused-appellants, there is no showing that to mark and make an inventory of the same in the presence of all the
the inventory and picture-taking of the shabu were conducted in their presence, persons named in Section 21 of Republic Act No. 9165, are not fatal
as well as in the presence of a representative from the media, the Department of and did not render the seized drugs inadmissible in evidence given that
Justice (DOJ), and any elected public official, immediately after accused- the prosecution was able to trace and establish each and every link in
appellants’ arrest and seizure of the shabu purportedly sold by them. When the chain of custody of the seized drugs and, hence, the identity and
accused-appellants were brought by the buy-bust team to the barangay hall integrity of the said drugs had been duly preserved. For the same
following their arrest, there was already a typewritten inventory report for reasons, it was not imperative for the prosecution to present as witnesses before
signature by the barangay officials, which, accused-appellants surmise, was the RTC the two barangay officials who witnessed the conduct of the inventory.
already prepared at the NBI Office. It is likewise not clearly established where At best, the testimonies of these two barangay officials will only be
and when the markings on the plastic sachets of shabu were made. Accused- corroborative, and would have no significant impact on the identity and integrity
appellants reason that the suspicions regarding the actual conduct of an of the seized drugs.
inventory of the shabu allegedly sold by them could have been avoided had the x Moreover, accused-appellants’ uncorroborated defenses of denial and frame-up
prosecution presented the testimonies of the barangay officials who signed the cannot prevail over the prosecution witnesses’ positive testimonies, coupled with
inventory report. the presentation in court by the prosecution of the corpus delicti. Prosecutions
x Accused-appellants further point out that the prosecution’s evidence conflicted as involving illegal drugs depend largely on the credibility of the police officers who
to the number of sachets of shabu seized from them. It was stipulated during conducted the buy-bust operation. Oft-repeated is the rule that in cases involving
the pre-trial that there were four plastic sachets of shabu but prosecution violations of Republic Act No. 9165, credence is given to prosecution witnesses
witness SI Saul testified that as poseur-buyer, he bought and received only two who are police officers (or in this case, NBI agents) for they are presumed to
sachets of shabu from accused-appellants. No details were provided about the have performed their duties in a regular manner, unless there is evidence to the
seizure of the other two sachets of shabu. contrary. Absent any indication that the NBI agents herein were ill motivated in
x Plaintiff-appellee, in its Brief, maintains that the rule on the chain of custody of testifying against accused-appellants, their testimonies deserve full credence.31
the seized shabu had been substantially complied with and the issues raised by In contrast, the defenses of denial and frame-up have been invariably viewed by
accused-appellants are trivial and unfounded. this Court with disfavor for it can easily be concocted and is a common and
o The Court finds the appeal bereft of merit. standard defense ploy in prosecutions for violation of Republic Act No. 9165. In
x The combined testimonial, documentary, and object evidence of the order to prosper, the defenses of denial and frame-up must be proved with
prosecution produced a detailed account of the buy-bust operation strong and convincing evidence.32 Accused-appellants presented no such
against accused-appellants and proved all the essential elements of the evidence in this case.
crime charged against them.
225
RECTO, GAYLE ANGELI M.
2011-0008 | AUSL
Personal Notes on Remedial Law 2 Review (based on the syllabus of Prof. Henedino M. Brondial)

x The penalty for illegal sale of shabu, regardless of the quantity and purity
involved, under Article II, Section 5 of Republic Act No. 9165, shall be life (a) When the original has been lost or destroyed, or cannot be produced in court,
imprisonment to death and a fine ranging from Five Hundred Thousand Pesos without bad faith on the part of the offeror;
(P500,000.00) to Ten Million Pesos (P10,000,000.00). Hence, the imposition of
the penalty of life imprisonment upon accused-appellants and an order for each (b) When the original is in the custody or under the control of the party against
of them to pay a fine of Five Hundred Thousand Pesos PS00,000.00) are correct. whom the evidence is offered, and the latter fails to produce it after reasonable notice;

(c) When the original consists of numerous accounts or other documents which
2. Documentary Evidence cannot be examined in court without great loss of time and the fact sought to be established
from them is only the general result of the whole; and
Section 2. Documentary evidence. — Documents as evidence consist of writing or any
material containing letters, words, numbers, figures, symbols or other modes of written (d) When the original is a public record in the custody of a public officer or is
expression offered as proof of their contents. (n) recorded in a public office. (2a)

Section 4. Original of document. —


ANNOTATION
(a) The original of the document is one the contents of which are the subject of
x Document Æ a deed, instrument or other duly authorized paper by inquiry.
which something is proved, evidenced or set forth
o Hence, “document” do NOT exclusively refer to writings (b) When a document is in two or more copies executed at or about the same time,
with identical contents, all such copies are equally regarded as originals.
x Documentary evidence Æ that which is furnished by written
instruments, inscriptions and documents of all kinds (c) When an entry is repeated in the regular course of business, one being copied
o To be considered as DOCUMENTARY EVIDENCE, a from another at or near the time of the transaction, all the entries are likewise equally
writing/material must be offered as proof of its regarded as originals. (3a)
contents
ƒ Otherwise Æ the same may be considered as Secondary Evidence
object evidence
Section 5. When original document is unavailable. — When the original document has
x 2 categories of documents: been lost or destroyed, or cannot be produced in court, the offeror, upon proof of its execution
o Writings or existence and the cause of its unavailability without bad faith on his part, may prove its
ƒ Those instantly recognizable as documents contents by a copy, or by a recital of its contents in some authentic document, or by the
ƒ Example Æ contract and wills testimony of witnesses in the order stated. (4a)
o Any other material containing modes of written
Section 6. When original document is in adverse party's custody or control. — If
expressions the document is in the custody or under the control of adverse party, he must have reasonable
ƒ Those not traditionally considered as writings notice to produce it. If after such notice and after satisfactory proof of its existence, he fails to
BUT are actually objects which contain modes produce the document, secondary evidence may be presented as in the case of its loss. (5a)
of written expression
x Electronic Evidence Æ information or the representation of Section 7. Evidence admissible when original document is a public record. — When
information, data, figures, symbols or other modes of written the original of document is in the custody of public officer or is recorded in a public office, its
contents may be proved by a certified copy issued by the public officer in custody thereof. (2a)
expression, described or however represented, by which a right is
established or an obligation extinguished, or by which a fact may be Section 8. Party who calls for document not bound to offer it. — A party who calls
prove and affirmed, which is receive, recorded, transmitted, stored, for the production of a document and inspects the same is not obliged to offer it as evidence.
processed, retrieved or produced electronically. (6a)
o Also referred to as electronic data message
o DOES NOT only refer to information itself ANNOTATION
o Functional equivalents of paper-based documents
o May be used for any of the ff purposes: x Best Evidence Rule Æ applies as rule of exclusion
ƒ To establish a right o “Best” Æ has NOTHING to do with the hierarchy of
ƒ To extinguish an obligation OR evidence nor with the probative value in relation to other
ƒ To prove or affirm a fact types of evidentiary rules
o Burden of proving Æ the one seeking to introduce it o RATIONALE:
o Manner of authenticating: ƒ Hazard of inaccuracy in the human process of
ƒ by evidence that it had been digitally signed making a copy by handwriting or typewriting
by the person purported to have signed the ƒ Special risk of error as regards oral testimony
same; o PURPOSE:
ƒ by evidence that other appropriate security ƒ BASIC: the need to present to the court the
procedures or devices as may be authorized exact words of a writing where a slight
by the Supreme Court or by law for variation may mean great difference in rights
authentication of electronic documents were ƒ ANCILLARY: to prevent fraud or mistake in
applied to the document; the proof of the contents of a writing
ƒ by other evidence showing its integrity and o Requisites for Applicability:
reliability to the satisfaction of the judge. ƒ Subject matter must involve a document
o The manner above ONLY applies where the electronic ƒ Subject of inquiry is the contents of such
document being offered is private and is offered as an document
authentic document x GR: documents, when presented as evidence, should be in their
o When such electronic document is NOTARIZED, there is original
no need to authenticate it as prescribed above o XPNs: Section 3, viz:
x When a document is offered as a documentary evidence Æ the ƒ Lost, destroyed or cannot be produced Æ See
hearsay rule, parol evidence rule and best evidence rule may come Section 5
into play ƒ In the custody or under the control of the
x Requisites for Admissibility of Documentary Evidence: adverse party who fails to produce it Æ See
o Relevant Section 6 and 8
o Authenticated ƒ Records are voluminous and the factum
o By a competent witness probandum is the general result of the whole
o Formally offered in evidence and not the detailed contents of the results
x REQUISITES:
a. The Best Evidence Rule o Voluminous character
must be established
Section 3. Original document must be produced; exceptions. — When the subject of o Such records must be
inquiry is the contents of a document, no evidence shall be admissible other than the original accessible to the other
document itself, except in the following cases: party so that their
226
RECTO, GAYLE ANGELI M.
2011-0008 | AUSL
Personal Notes on Remedial Law 2 Review (based on the syllabus of Prof. Henedino M. Brondial)

correctness may be o Copy


tested on cross o Recital of the contents in some authentic document
examination o Testimony of witness
ƒ In the custody of a public officer or recorded x GR: The foregoing order shall be followed in presenting secondary
in public office Æ See Section 7 evidence
x APPLICABILITY of the Best Evidence Rule: o XPN: When the law specifically provides for the class and
o When the contents of the document is the subject of quantum of secondary evidence, viz:
inquiry ƒ Notarial will Ætestimony of at least 2 credible
o In criminal cases when the issue is not only with respect witnesses who can clearly and distinctly
to the contents of the document but also as to whether establish its contents
such document actually existed with the participation ƒ Holographic will Æ no testimonial evidence
imputed to the accused allowed BUT MAY BE proven by photostatic
x NON-APPLICABILITY of the Best Evidence Rule: copies thereof
o When the document is presented to prove its existence or
condition x Sections 6 and 8 Æ govern cases where the original is in the custody
ƒ In such cases, parol evidence is admissible or under the control of the adverse party who fails to produce it
o When what is sought to be proven is a collateral or x CONDITION PRECEDENT: notice to the party having custody or
concominant fact that took place while the document was control of the original
being prepared o No form required
o When the document is merely collateral in issue o May be an oral demand in open court
(meaning, it is being presented NOT for the purpose of o XPN: Notice is NOT required where the nature of the
proving its contents but for some other purpose) action is in itself a notice
x Depositions and affidavits Æ NOT the best evidence ƒ Example Æ action for recovery or annulment
x Section 4 Æ documents which are considered as originals , viz: of documents wrongfully obtained or withheld
o Regular entries in journals and ledgers by the other party
o Signed carbon copy or duplicate of the document x JUSTIFIED refusal to produce original DOES NOT give rise to the
executed at the same time as the original (duplicate presumption of suppression of evidence
original) o It only authorizes the introduction of secondary evidence x
o Carbon copy if the same includes the signature of the Secondary evidence Æ NOT automatically admissible UNTIL the
party sought to be charged thereby and which signature requirements for admissibility are present
being thus reproduced by the same stroke of pen x Section 7 Æ governs cases where the original document is a public
o Telegrams and cables Æ best evidence depends on what record
the issue is, viz: x When a document is a public record, there is no need to present the
ƒ If the issue is the contents of the telegram original thereof
received by the addressee Æ original dispatch o REASON: Under Rule 132, such document may be
received evidenced:
ƒ If the issue is the telegram sent by the sender ƒ by an official publication thereof OR
Æ message delivered for transmission ƒ ƒ by a copy attested by the officer having legal
Inaccuracy of transmission Æboth telegrams custody of the record OR
as sent and received ƒ in the case of a private writing, may be
x Section 5 Æ governs cases where the original is lost, destroyed or proved by a copy thereof attested by the legal
cannot be produced keeper of the record
x For secondary evidence to be admissible, the ff must be proven:
o Due execution of the original CASES
ƒ Proven through the testimony of any of the
ff: CITIBANK, N.A. MASTERCARD, petitioner, vs. EFREN S. TEODORO, respondent.
x Person/s who executed it G.R. No. 150905. September 23, 2003
THIRD DIVISION
x Person before whom the execution
was acknowledged FACTS:
x Any person who was present and x Teodoro was one of the credit card holders of Citibank
saw it executed and delivered and o Under the terms and conditions governing the use of the Citibank
who thereafter saw it and credit card, the cardholder undertakes to pay all the purchases
recognized the signatures made using the card within the period indicated on the statement
of account or within thirty (30) days from the date or dates of its
x Any person to whom the parties use. Charges that remain unpaid within the period fixed in the
thereto had previously confessed monthly statement of account shall earn interest at the rate of 3.5
the execution thereof percent per month plus a penalty fee equivalent to 5 percent of the
o Lost, destruction or unavailability of such original/s amount due for every month or even a fraction of a months delay.
ƒ Proven by the testimony of any of the ff: x Teodoro made certain purchases for which he tendered various payments.
x Any person who knew of such fact x Citi Æ claims that as of January 20, 1995, the obligations of respondent stood at
P191,693.25, inclusive of interest and service charges
x Anyone who, in the judgment of
x Citi claimed to have made several demands from Teodoro
the court, had made a sufficient o Teodoro was still unable to pay
examination in the places where x Citi Æ filed an action for sum of money against Teodoro before RTC Makati
the documents or papers of similar x RTC Makati Æ dismissed for lack of jurisdiction
character are usually kept by the x Case was transferred to MeTC Makati
person in whose custody the x Citi’s evidence:
o During the trial, petitioner presented several sales invoices or
document was and has been
charge slips, which added up to only P24,388.36. Although mere
unable to find it photocopies of the originals, the invoices were marked in evidence
x Anyone who has made any other as Exhibits F to F-4. Because all these copies appeared to bear the
investigation which is sufficient to signatures of respondent, the trial court deemed them sufficient
satisfy the court that the proof of his purchases with the use of the credit card.
x MeTC Æ in favor of CITI
document is indeed lost o Ordered Teodoro to pay Citi the amount of P24,388.36 plus
ƒ This also includes when the document is
interest and penalty fee
OUTSIDE the JURISDICTION of the court, as x Teodoro Æ appealed to RTC
when it is in a foreign country x RTC Æ affirmed MeTC in toto
o Reasonable diligence and good faith in the search for or x Teodoro Æ appealed to CA
attempt to produce the original/s x CA Æ reversed RTC
x Order on which the contents of the document shall be proven:
227
RECTO, GAYLE ANGELI M.
2011-0008 | AUSL
Personal Notes on Remedial Law 2 Review (based on the syllabus of Prof. Henedino M. Brondial)

o The focal issue of the case according to the CA was whether the its contents by a copy, or by a recital of its contents in some
photocopies of the sales invoices or charge slips, marked as authentic document, or by the testimony of witnesses in the order
Exhibits F to F-4, were competent proofs of the obligations of stated.
respondent. These were the only evidence presented by petitioner x Applying the above Rule to the present case, before a party is allowed to adduce
that could prove the actual amount of obligation he had incurred in secondary evidence to prove the contents of the original sales invoices, the
favor of the former. In reversing the trial courts, the CA ruled that offeror must prove the following:
this evidence was insufficient to prove any liability on respondents o (1) the existence or due execution of the original;
part. o (2) the loss and destruction of the original or the reason for its
o According to Sections 3 and 5 of Rule 130 of the Rules of Court, nonproduction in court; and
whenever the subject of inquiry is the content of a document, its o (3) on the part of the offeror, the absence of bad faith to which
original must be produced, as it is the best evidence to prove such the unavailability of the original can be attributed.[14]
content. Secondary evidence, like the subject photocopies, is x The correct order of proof is as follows: existence, execution, loss, and
inadmissible. It will be admissible only if the offeror proves (a) any contents. At the sound discretion of the court, this order may be changed if
of the exceptions enumerated in Section 3 and (b) the conditions necessary.[15]
for its admissibility set forth in Section 5 of Rule 130. For secondary x In the present case, the existence of the original sales invoices was established
evidence to be admissible, there must be satisfactory proof of (1) by the photocopies and the testimony of Hernandez. Petitioner, however,
the due execution of the original; (2) the originals loss, destruction failed to prove that the originals had been lost or could not be
or unavailability that is not due to the offerors bad faith; and (3) produced in court after reasonable diligence and good faith in
reasonable diligence and good faith in the search for or attempt to searching for them.
produce the original. x Indeed, the loss of the originals and reasonable diligence in the search for them
o Although petitioner was able to prove the existence of the original were conditions that were not met, because the sales invoices might have been
sales invoices, it failed to prove their due execution or to account found by Equitable. Hernandez, testifying that he had requested the originals
for their loss or unavailability. from Equitable, failed to show that he had subsequently followed up the
x Citi Æ filed a Rule 45 before the SC request.[16]
o Petitioner contends that the testimony[10] of its principal witness - x Finally, when more than one original copy exists, it must appear that all
Mark Hernando, assistant manager of Citibank, N.A. Mastercard -- of them have been lost, destroyed, or cannot be produced in court
proves the following: before secondary evidence can be given of any one. A photocopy may
ƒ a) the existence or due execution of the original sales not be used without accounting for the other originals.[17]
invoices which sufficiently proved respondents liability x In Santos v. Santos[18] the Court upheld the pronouncement of the CA that
of P24,388.36; before the appellees therein could be allowed to adduce secondary evidence to
ƒ b) the loss or unavailability of the original sales prove the contents of the original, they had to prove -- with the requisite
invoices; and quantum of evidence -- the loss, the destruction or the unavailability of all
ƒ c) petitioners reasonable diligence and good faith in original copies of the document.
the search for or attempt to produce the originals. x In the present case, triplicates were produced, although the cardholder signed
x It further argues that Hernando the sales invoice only once.[19] During the trial, Hernandez explained that an
competently identified the signatures of original copy had gone to respondent, another to the merchant, and still another
respondent on the sales invoices, having to petitioner.[20]
recognized them as identical to the x Each of these three copies is regarded as an original in accordance with Section
signature on the latters credit card 4 (b) of Rule 130 of the Rules of Court.[21] Petitioner failed to show that all three
application form. original copies were unavailable, and that due diligence had been exercised in the
x Teodoro Æ filed his comment search for them.
o On the other hand, respondent maintains that petitioner failed to
prove the due execution of the sales invoices. According to him,
Hernando was not privy to such execution and could not have
properly or competently declared that the signatures on the
invoices and on the application form belonged to the former. The WILGEN LOON, JERRY ARCILLA, ALBERTPEREYE, ARNOLD PEREYE, EDGARDO OBOSE,
latter was not the person before whom the application form was ARNEL MALARAS, PATROCINO TOETIN, EVELYN LEONARDO, ELMER GLOCENDA, RUFO
signed, executed or acknowledged; he was not even present then. CUNAMAY, ROLANDOSAJOL, ROLANDO ABUCAYON, JENNIFER NATIVIDAD, MARITESS
As to the sales invoices and respondents alleged signatures TORION, ARMANDO LONZAGA, RIZAL GELLIDO, EVIRDE HAQUE,1 MYRNA VINAS, RODELITO
thereon, he saw them only after the Complaint had been filed in AYALA, WINELITO OJEL, RENATO RODREGO, NENA ABINA, EMALYN OLIVEROS, LOUIE
court or long after those invoices had been executed. He was ILAGAN, JOEL ENTIG, ARNEL ARANETA, BENJAMIN COSE, WELITO LOON and WILLIAM
therefore not competent to identify the signatures. ALIPAO, Petitioners, vs. POWER MASTER, INC., TRI-C GENERAL SERVICES, and SPOUSES
o Because Hernandez had not actually witnessed the execution of the HOMER and CARINA ALUMISIN, Respondents.
sales invoices and the application form, respondent concludes that G.R. No. 189404 December 11, 2013
petitioner failed to observe Section 5 of Rule 130 of the Rules of SECOND DIVISION
Court, which provides that the contents of the original may be
proven by the testimony of witnesses. FACTS:
o Finally, respondent contends that the alleged loss or unavailability x Herein petitioners were employed by Power Master (PM) as janitors and
of the original sales invoices was not sufficiently established. leadsmen in various Philippine Long Distance Telephone Company (PLDT) offices
Allegedly, Hernandez had requested the originals from Equitable in Metro Manila area
Credit Card Network, Inc., but failed to show in court that he had x Petitioners Æ filed before the LA a complaint for money claims against Power
followed up his request as advised by another witness, Zen Master, Inc., Tri-C General Services and their officers, the spouses Homer and
Hipolito. Therefore, the requirement of reasonable diligence and Carina Alumisin (collectively, the respondents)
good faith in the search for or attempt to produce the originals was o alleged in their complaint that they were not paid minimum wages,
not satisfied, because he had shown no proof of having followed up overtime, holiday, premium, service incentive leave, and thirteenth
the request. month pays
o averred that the respondents made them sign blank payroll sheets.
ISSUE: Whether the photocopies of the subject invoices were sufficient in proving the liabilities x Petitioners Æ then amended their complaint and included illegal dismissal in their
of Teodoro. COA
o They claimed that the respondents relieved them from service in
HELD: NO retaliation for the filing of their original complaint.
x The burden of proof rests upon petitioner, as plaintiff, to establish its case based x PM et al Æ did NOT file any position paper nor proffered pieces of evidence in
on a preponderance of evidence. It is well-settled that in civil cases, the party that their defense despite their knowledge of the pendency of the case.
alleges a fact has the burden of proving it.[11] Petitioner failed to prove that x LA Æ ruled PARTIALLY in favor of petitioners
respondent had an obligation in the principal amount of P24,388.36, because the o awarded the petitioners salary differential, service incentive leave,
photocopies of the original sales invoices it had presented in court were and thirteenth month pays. In awarding these claims, the LA stated
inadmissible in evidence. Moreover, had they been admissible, they would still that the burden of proving the payment of these money claims
have had little probative value.[12] rests with the employer. The LA also awarded attorney’s fees in
x The original copies of the sales invoices are the best evidence to prove favor of the petitioners, pursuant to Article 111 of the Labor Code.
the alleged obligation. Photocopies thereof are mere secondary o denied the petitioners’ claims for backwages, overtime, holiday,
evidence. As such, they are inadmissible because petitioner, as the and premium pays. The LA observed that the petitioners failed to
offeror, failed to prove any of the exceptions provided under Section show that they rendered overtime work and worked on holidays
3[13] of Rule 130 of the Rules of Court, as well s the conditions of their and rest days without compensation. The LA further concluded that
admissibility. Because of the inadmissibility of the photocopies in the the petitioners cannot be declared to have been dismissed from
absence of the originals, respondents obligation was not established. employment because they did not show any notice of termination
x Section 5 of Rule 130 of the Rules of Court states: of employment. They were also not barred from entering the
o SEC. 5. When original document is unavailable. When the original respondents’ premises.
document has been lost or destroyed, or cannot be produced in x BOTH parties Æ appealed to NLRC
court, the offeror, upon proof of its execution or existence and the o PM et al Æ filed an unverified supplemental appeal.
cause of its unavailability without bad faith on his part, may prove
228
RECTO, GAYLE ANGELI M.
2011-0008 | AUSL
Personal Notes on Remedial Law 2 Review (based on the syllabus of Prof. Henedino M. Brondial)

ƒ They attached photocopied and computerized copies probative value to the respondents’ evidence despite their failure to
of list of employees with automated teller machine adequately explain their delay in the submission of evidence. Notably,
(ATM) cards to the supplemental appeal. This list also the respondents’ delay was anchored on their assertion that they were
showed the amounts allegedly deposited in the oblivious of the proceedings before the LA. However, the respondents did
employees’ ATM cards.11 They also attached not dispute the LA’s finding that Mr. Romulo Pacia, Jr. appeared on their behalf
documentary evidence showing that the petitioners on April 19, 2001 and May 21, 2001.32 The respondents also failed to contest
were dismissed for cause and had been accorded due the petitioners’ assertion that the respondents’ counsel appeared in a preliminary
process. mandatory conference on July 5, 2001.33
o Petitioners Æ filed an Urgent Manifestation and Motion12 where x Indeed, the NLRC capriciously and whimsically admitted and gave weight to the
they asked for the deletion of the supplemental appeal from the respondents’ evidence despite its finding that they voluntarily appeared in the
records because it allegedly suffered from infirmities. compulsory arbitration proceedings. The NLRC blatantly disregarded the fact that
ƒ First, the supplemental appeal was not verified. the respondents voluntarily opted not to participate, to adduce evidence in their
ƒ Second, it was belatedly filed six months from the defense and to file a position paper despite their knowledge of the pendency of
filing of the respondents’ notice of appeal with the proceedings before the LA. The respondents were also grossly negligent in
memorandum on appeal. The petitioners pointed out not informing the LA of the specific building unit where the respondents were
that they only agreed to the respondents’ filing of a conducting their business and their counsel’s address despite their knowledge of
responsive pleading until December 18, 2002.13 their non-receipt of the processes.34
ƒ Third¸ the attached documentary evidence on the
supplemental appeal bore the petitioners’ forged ISSUE # 2: Whether the photocopies of the subject documents deserve credence.
signatures.
x NLRC Æ ruled PARTIALLY in favor of Power Master HELD # 2: NO.
o The NLRC affirmed the LA’s awards of holiday pay and attorney’s x Furthermore, the respondents failed to sufficiently prove the allegations sought
fees. It also maintained that the LA acquired jurisdiction over the to be proven. Why the respondents’ photocopied and computerized
persons of the respondents through their voluntary appearance. copies of documentary evidence were not presented at the earliest
o However, it allowed the respondents to submit pieces of evidence opportunity is a serious question that lends credence to the petitioners’
for the first time on appeal on the ground that they had been claim that the respondents fabricated the evidence for purposes of
deprived of due process. It found that the respondents did not appeal. While we generally admit in evidence and give probative value to
actually receive the LA’s processes. It also admitted the photocopied documents in administrative proceedings, allegations of
respondents’ unverified supplemental appeal on the ground that forgery and fabrication should prompt the adverse party to present the
technicalities may be disregarded to serve the greater interest of original documents for inspection.35 It was incumbent upon the
substantial due process. Furthermore, the Rules of Court do not respondents to present the originals, especially in this case where the
require the verification of a supplemental pleading. petitioners had submitted their specimen signatures. Instead, the
o NLRC also vacated the LA’s awards of salary differential, thirteenth respondents effectively deprived the petitioners of the opportunity to
month and service incentive leave pays. In so ruling, it gave weight examine and controvert the alleged spurious evidence by not adducing
to the pieces of evidence attached to the memorandum on appeal the originals. This Court is thus left with no option but to rule that the
and the supplemental appeal. It maintained that the absence of the respondents’ failure to present the originals raises the presumption
petitioners’ signatures in the payrolls was not an indispensable that evidence willfully suppressed would be adverse if produced.36 x It
factor for their authenticity. It pointed out that the payment of was also gross error for the CA to affirm the NLRC’s proposition that "[i]t is of
money claims was further evidenced by the list of employees with common knowledge that there are many people who use at least two or more
ATM cards. It also found that the petitioners’ signatures were not different signatures."37 The NLRC cannot take judicial notice that many people
forged. It took judicial notice that many people use at least two or use at least two signatures, especially in this case where the petitioners
more different signatures. themselves disown the signatures in the respondents’ assailed documentary
o The NLRC further ruled that the petitioners were lawfully dismissed evidence.38 The NLRC’s position is unwarranted and is patently unsupported by
on grounds of serious misconduct and willful disobedience. It found the law and jurisprudence.
that the petitioners failed to comply with various memoranda x Viewed in these lights, the scales of justice must tilt in favor of the employees.
directing them to transfer to other workplaces and to attend This conclusion is consistent with the rule that the employer’s cause can only
training seminars for the intended reorganization and reshuffling. succeed on the strength of its own evidence and not on the weakness of the
x Petitioners Æ filed an MR employee’s evidence.
o NLRC Æ denied
x Petitioners Æ filed a Rule 65 before the CA
x CA Æ affirmed NLRC THERESITA, JUAN, ASUNCION, PATROCINIA, RICARDO, and GLORIA, all surnamed
o The CA held that the petitioners were afforded substantive and DIMAGUILA, Petitioners, vs. JOSE and SONIA A. MONTEIRO, Respondents.
procedural due process. Accordingly, the petitioners deliberately did G.R. No. 201011 January 27, 2014
not explain their side. Instead, they continuously resisted their THIRD DIVISION
transfer to other PLDT offices and violated company rules and
regulations. It also upheld the NLRC’s findings on the petitioners’ FACTS:
monetary claims. x Monteiros Æ filed a Complaint for Partition and Damages before the RTC Sta.
x Petitioners Æ filed a Rule 45 before the SC Cruz, Laguna against the Dumaguilas
o They were co-owners and prayed for the partition of a residential
ISSUE # 1: Whether NLRC correctly admitted the pieces of evidence which Power Master house and lot located at Gat. Tayaw St., Liliw, Laguna, with an area
presented for the first time on appeal. of 489 square meters, and covered by Tax Declaration No. 1453.
o Spouses Monteiro anchored their claim on a deed of sale executed
HELD # 1: NO. in their favor by the heirs of Pedro Dimaguila (Pedro).
x A party may only adduce evidence for the first time on appeal if he adequately x Dumaguilas Æ filed their answer
explains his delay in the submission of evidence and he sufficiently proves the o there was no co-ownership to speak of in the first place.
allegations sought to be proven o They alleged that the subject property, then owned by Maria
x In labor cases, strict adherence to the technical rules of procedure is not Ignacio Buenaseda, had long been partitioned equally between her
required. Time and again, we have allowed evidence to be submitted for the first two sons, Perfecto and Vitaliano Dimaguila, through a Deed of
time on appeal with the NLRC in the interest of substantial justice.28 Thus, we Extrajudicial Partition, with its southern-half portion assigned to
have consistently supported the rule that labor officials should use all reasonable Perfecto and the northern-half portion to Vitaliano.
means to ascertain the facts in each case speedily and objectively, without o They claimed that they were the heirs of Vitaliano and that Spouses
regard to technicalities of law or procedure, in the interest of due process.29 Monteiro had nothing to do with the property as they were not
x However, this liberal policy should still be subject to rules of reason and fairplay. heirs of either Perfecto or Vitaliano.
The liberality of procedural rules is qualified by two requirements: x Monteiros Æ filed a motion for leave to amend complaint
o (1) a party should adequately explain any delay in the submission o The amended complaint abandoned the original claim for partition
of evidence; and and instead sought the recovery of possession of a portion of the
o (2) a party should sufficiently prove the allegations sought to be subject property occupied by the Dimagui as and other defendants,
proven.30 specifically, the potiion sold to the couple by the heirs of Pedro.
x The reason for these requirements is that the liberal application of the rules Furthermore, only Spouses Monteiro were retained as plaintiffs and
before quasi-judicial agencies cannot be used to perpetuate injustice and hamper the Dimaguilas as defendants.
the just resolution of the case. Neither is the rule on liberal construction a license x RTC Æ granted the motion
to disregard the rules of procedure.31 x Dimaguilas Æ filed an answer
x Guided by these principles, the CA grossly erred in ruling that the NLRC did not o admitted that the subject property was inherited by, and divided
commit grave abuse of discretion in arbitrarily admitting and giving weight to the equally between Perfecto and Vitaliano, but denied the admission in
respondents’ pieces of evidence for the first time on appeal. their original answer that it had been actually divided into southern
x A. The respondents failed to adequately explain their delay in the submission of and nmihern portions. Instead, they argued that the Extrajudicial
evidence Partition mentioned only the division of the subject property "into two
x We cannot accept the respondents’ cavalier attitude in blatantly and share and share alike." In effect, they argued the
disregarding the NLRC Rules of Procedure. The CA gravely erred when existence of a co-owenrship, contrary to their original position. The
it overlooked that the NLRC blindly admitted and arbitrarily gave Dimaguilas further argued that the Bilihan did not specify the
229
RECTO, GAYLE ANGELI M.
2011-0008 | AUSL
Personal Notes on Remedial Law 2 Review (based on the syllabus of Prof. Henedino M. Brondial)

metes and bounds of the property sold, in violation of Article 1458 appears that the subject property had already been partitioned into definite
of the Civil Code. Even assuming that such had been specified, they portions more than 20 years prior to the original complaint for partition filed in
averred that the sale of a definite portion of a property owned in 1993, and that such division had been observed by the brothers' heirs. As earlier
common was void since a co-owner could only sell his undivided pointed out, the petitioners themselves admitted to this very fact in their original
share in the property. answer, to wit:
x Monteiros’ evidence: o (b) On September 5, 1945 the brothers PERFECTO and VITALIANO
o Spouses Monteiro presented Pedrito Adrieta, brother of Sonia DIMAGUILA executed a deed of EXTRA JUDICIAL PARTITION of
Monteiro (Sonia), who testified that Perfecto was his grandfather and the aforedescribed property dividing the same into two (2) equal
that at the time of Perfecto's death, he had two properties, one of parts as indicated in the aforesaid deed as follows, to wit:
which was the subject property in Liliw, Laguna, which went to his ƒ (c) As a result of the foregoing partition and as
children, Esperanza, Leonardo and Pedro. Pedro was survived by his known by all the parties in this case from the
children Pedrito, Theresita, Francisco, and Luis, who, in turn, sold beginning or as soon as they reached the age of
their rights over the subject property to Sonia. discernment PERFECTO DIMAGUILA became the sole
o Sonia testified that she was approached by Pedro's son, Francisco, and exclusive owner of the southern half of the
and was asked if she was interested in purchasing Pedro's 1/3 aforedescribed property and VITALIANO DIMAGUILA
share of the southern portion of the Bahay na Sato, and that he became the sole owner of the northern half of the
showed her a deed of extrajudicial partition executed by and same property; the house that was built thereon and
between Perfecto and Vitaliano, as well as the tax declaration of still existing up to this time was likewise equally
the property to prove that the property had already been divided between the two (2) DIMAGUILA brothers in
partitioned between the two brothers. accordance with the extrajudicial partition of half
o Engineer Baltazar F. Mesina testified that he was the geodetic equal shares;
engineer hired by Spouses Monteiro to survey the property in Liliw, o 2. In other words, the share of VITALIANO DIMAGUILA in the
and recounted that he checked the boundary of the subject above described property has already been long segregated and
property, subdivided the lot into two and came up with a survey had passed on to his heirs as is very well known by all the parties
plan. in this case;17
o Crisostomo Arves, an employee from the Office of the Municipal x Section 4 of Rule 129 of the Rules of Court provides that an admission
Assessor, presented a certified true copy of the cadastral map of made by a party in the course of the proceedings in the same case does
Liliw and a list of claimants/owners. not require proof, and may be contradicted only by showing that it was
o Dominga Tolentino, a record officer of the Department of made through palpable mistake. The petitioners argue that such admission
Environment and Natural Resources (DENR), testified that as part was the palpable mistake of their former counsel in his rush to file the answer, a
of her duties, she certifies and safekeeps the records of surveyed copy of which was not provided to them. Petitioner Asuncion testified:
land, including cadastral maps from the region. o Q So, why was that allegations (sic) made in the Answer?
x Dimaguilas’ evidence: o A May be, (sic) in his rush to file the Answer, Atty. Paredes filed the
o One of the Dimaguilas, Asuncion, was the sole witness for the same without giving us a copy ...19
defendants. She testified that their first counsel made a mistake x This contention is unacceptable.
when he alleged in their original answer that the property had x It is a purely self-serving claim unsupported by any iota of evidence. Bare
already been partitioned into n01ihern and southern portions allegations, unsubstantiated by evidence, are not equivalent to proof.20
between the two brothers, as the original answer had been rushed Furthermore, the Court notes that this position was adopted by the petitioners
and they were never given a copy of it. only almost eight (8) years after their original answer was filed, in response to
o She claimed that the mistake was only pointed out to her by their the amended complaint of the respondent spouses. In their original answer
new counsel after their former counsel withdrew due to cancer. to the complaint for partition, their claim that there was already a
o She further testified that there was no intention to partition the partition into northern-half and southern-half portions, was the very
"bahay na bato" which stood on the subject property, in order to essence of their defense. It was precisely this admission which moved
preserve its historical and sentimental value. the respondent spouses to amend their complaint. The petitioners
x RTC Æ in favor of the MONTEIROS; ordered the Dimaguilas to turn over the cannot now insist that the very foundation of their original defense
possession of the subject 1/3 portion of the southern-half of the property was a palpable mistake.
x Dimaguilas Æ appealed to CA x Article 1431 of the Civil Code provides that through estoppel, an admission is
o The Dimaguilas also insisted on their argument, which was raised rendered conclusive upon the person making it, and cannot be denied or
before the RTC, but not addressed, that the Bilihan should not have disproved as against the person relying thereon. The respondent spouses had
been admitted as evidence for lack of a documentary stamp tax, in clearly relied on the petitioners' admission and so amended their
accordance with Section 201 of the National Internal Revenue Code original complaint for partition to one for recovery of possession of a
(N!RC). Citing Gabucan v. Manta10 and Del Rosario v. Hamoy,11 portion of the subject property. Thus, the petitioners are now estopped from
the CA, however, ruled that if a document which did not bear the denying or attempting to prove that there was no partition of the property.
required documentary stamp was presented in evidence, the court
should require the proponent to affix the requisite stamp. x Considering that an admission does not require proof, the admission of the
x CA Æ affirmed RTC petitioners would actually be sufficient to prove the partition even without the
o The CA noted that the RTC had failed to direct Spouses Monteiro to documents presented by the respondent spouses. If anything, the additional
affix the stamp and merely reminded the presiding judge to be evidence they presented only served to corroborate the petitioners' admission.
more vigilant on similar situations in the future. Nonetheless, it held
that the petitioners did not possess the necessary personality to ISSUE # 2: Whether the copies of the subject cadastral map and the list of claimants
assail the sale between Spouses Monteiro and the heirs of Pedro presented by the Monteiros are admissible in evidence.
because it pe1iained to the southern-half of the property to which
they had no claim. HELD # 2: YES.
x Dimaguilas Æ filed a Rule 45 before the SC x The petitioners argue that they timely objected to the cadastral map and the list
of claimants presented by the respondent spouses, on the ground that they
ISSUE # 1: Whether the Dimagauilas have judicially admitted the partition of the land so as to violated the rule on hearsay and the best evidence rule.
estop them from denying the same in their appeal. x Anent the best evidence rule, Section 3( d) of Rule 130 of the Rules of Court
provides that when the subject of inquiry is the contents of a document,
HELD # 1: YES. no evidence shall be admissible other than the original document itself,
x Spouses Monteiro, as plaintiffs in the original case, had the burden of proof to except when the original is a public record in the custody of a public
establish their case by a preponderance of evidence, which is the weight, credit, officer or is recorded in a public office.22 Section 7 of the same Rule
and value of the aggregate evidence on either side, synonymous with the term provides that when the original of a document is in the custody of a
"greater weight of the evidence." Preponderance of evidence is evidence which is public officer or is recorded in a public office, its contents may be
more convincing to the court as worthy of belief than that which is offered in proved by a certified copy issued by the public officer in custody
opposition thereto.14 thereof.23 Section 24 of Rule 132 provides that the record of public
x To prove their claim of partition, the respondent spouses presented the documents may be evidenced by a copy attested by the officer having
following: the legal custody or the record.24
o (1) the Deed of Extrajudicial Partition, dated October 5, 1945, x Certified true copies of the cadastral map of Liliw and the corresponding list of
executed by and between the brothers Perfecto and Vitaliano; claimants of the area covered by the map were presented by two public officers.
o (2) the cadastral map of Liliw Cadm-484,15 dated August 6, 1976, The first was Crisostomo Arves, Clerk III of the Municipal Assessor's Office, a
showing that the subject property had been divided into southern repository of such documents. The second was Dominga Tolentino, a DENR
and northern portions, registered as Lot Nos. 876 and 877; and employee, who, as a record officer, certifies and safekeeps records of surveyed
o (3) the Municipal Assessor's records16 showing that the said lots land involving cadastral maps. The cadastral maps and the list of
were respectively claimed by Buenaventura and Perfecto. claimants, as certified true copies of original public records, fall under
x It is undisputed that the Deed of Extrajudicial Partition stated that Perfecto and the exception to the best evidence rule.
Vitaliano agreed "to divide between them into two and share and share alike" the x As to the hearsay rule, Section 44 of Rule 130 of the Rules of Court similarly
subject property, including the house situated thereon. It appears, however, that provides that entries in official records are an exception to the rule.25 The rule
the property was actually partitioned into definite portions, namely, southern and provides that entries in official records made in the performance of the duty of a
northern halves, as reflected in the cadastral map of Liliw, which were public officer of the Philippines, or by a person in the performance of a duty
respectively claimed by an heir of Vitaliano and Perfecto himself. It, thus, specially enjoined by law, are prima facie evidence of the facts therein stated.
230
RECTO, GAYLE ANGELI M.
2011-0008 | AUSL
Personal Notes on Remedial Law 2 Review (based on the syllabus of Prof. Henedino M. Brondial)

The necessity of this rule consists in the inconvenience and difficulty of requiring (d) The existence of other terms agreed to by the parties or their successors in
the official's attendance as a witness to testify to the innumerable transactions in interest after the execution of the written agreement.
the course of his duty. The document's trustworthiness consists in the
presumption of regularity of performance of official duty.26 The term "agreement" includes wills. (7a)
x Cadastral maps are the output of cadastral surveys. The DENR is the
department tasked to execute, supervise and manage the conduct of Interpretation of Documents
cadastral surveys.27 It is, therefore, clear that the cadastral map and the
corresponding list of claimants qualify as entries in official records as they were Section 10. Interpretation of a writing according to its legal meaning. — The language of a
prepared by the DENR, as mandated by law. As such, they are exceptions to the writing is to be interpreted according to the legal meaning it bears in the place of its
hearsay rule and are primafacie evidence of the facts stated therein. execution, unless the parties intended otherwise. (8)
x Even granting that the petitioners had not admitted the partition, they
presented no evidence to contradict the evidence of the respondent Section 11. Instrument construed so as to give effect to all provisions. — In the
spouses. Thus, even without the admission of the petitioners, the construction of an instrument, where there are several provisions or particulars, such a
respondent spouses proved by a preponderance of evidence that there construction is, if possible, to be adopted as will give effect to all. (9)
had indeed been a partition of the subject property.
Section 12. Interpretation according to intention; general and particular
ISSUE # 3: Whether the RTC correctly admitted the subject Bilihan ng Lahat Naming provisions. — In the construction of an instrument, the intention of the parties is to be
Karapatan, considering the contention of the Dimaguilas that the DST of the same has not pursued; and when a general and a particular provision are inconsistent, the latter is
been paid. paramount to the former. So a particular intent will control a general one that is inconsistent
with it. (10)
HELD # 3: YES.
x To prove that 1/3 of the southern-half portion of the subject property was sold Section 13. Interpretation according to circumstances. — For the proper construction of
to them, Spouses Monteiro presented a deed of sale entitled Bilihan ng Lahat an instrument, the circumstances under which it was made, including the situation of the
Naming Karapatan,28 dated September 29, 1992, wherein Pedro's share was subject thereof and of the parties to it, may be shown, so that the judge may be placed in the
sold by his heirs to them, with the acquiescence of the heirs of Esperanza and position of those who language he is to interpret. (11)
Leandro in an Affidavit of Conformity and Waiver.29 The petitioners argue that
the Bilihan should not have been admitted into evidence because it lacked the Section 14. Peculiar signification of terms. — The terms of a writing are presumed to
documentary stamp tax required by Section 201 of the NIRC. have been used in their primary and general acceptation, but evidence is admissible to show
x On August 29, 1994, the petitioners filed a motion for the production and/or that they have a local, technical, or otherwise peculiar signification, and were so used and
inspection of documents,30 praying that Spouses Monteiro be ordered to understood in the particular instance, in which case the agreement must be construed
produce the deed of sale, which they cited as the source of their rights as co- accordingly. (12)
owners. On November 20, 1995, Spouses Monteiro submitted their
compliance,31 furnishing the RTC and the petitioners with a copy32 of the Section 15. Written words control printed. — When an instrument consists partly of
Bilihan. On January 3, 1996, the petitioners filed a notice of consignation,33 written words and partly of a printed form, and the two are inconsistent, the former controls
manifesting that they had attempted to exercise their right of redemption as co- the latter. (13)
owners of the 1/3 portion of the southern half of the property under Article
162334 of the Civil Code by sending and tendering payment of redemption to Section 16. Experts and interpreters to be used in explaining certain writings. —
Spouses Monteiro, which was, however, returned. When the characters in which an instrument is written are difficult to be deciphered, or the
x By filing the notice of consignation and tendering their payment for the language is not understood by the court, the evidence of persons skilled in deciphering the
redemption of the 1/3 portion of the southern-half of the property, the characters, or who understand the language, is admissible to declare the characters or the
petitioners, in effect, admitted the existence, due execution and validity of the meaning of the language. (14)
Bilihan. Consequently, they are now estopped from questioning its admissiblity in
evidence for relying on such for their right of redemption. Additionally, the Section 17. Of Two constructions, which preferred. — When the terms of an agreement
Court notes that the copy35 of the Bilihan which was originally have been intended in a different sense by the different parties to it, that sense is to prevail
submitted by Spouses Monteiro with its compliance filed on November against either party in which he supposed the other understood it, and when different
20, 1995, does in fact bear a documentary stamp tax. It could only constructions of a provision are otherwise equally proper, that is to be taken which is the most
mean that the documentary stamp tax on the sale was properly paid. favorable to the party in whose favor the provision was made. (15)
The Bilihan was, therefore, properly admitted into evidence and
considered by the RTC. Section 18. Construction in favor of natural right. — When an instrument is equally
x In any case, as correctly held by the lower cou1is, the petitioners, as heirs of susceptible of two interpretations, one in favor of natural right and the other against it, the
Vitaliano, who inherited the northern-half po1iion of the subject property, do not former is to be adopted. (16)
possess the necessary personality to assail the sale of the southern-half portion
between Spouses Monteiro and the heirs of Pedro.1âwphi1 They are not real Section 19. Interpretation according to usage. — An instrument may be construed
parties-in-interest who stand to be benefited or injured by the sale of the 1/3 according to usage, in order to determine its true character. (17)
portion of the southern-half over which they have absolutely no right. As
correctly ruled by the courts below, only fellow co-owners have the personality to
assail the sale, namely, the heirs of Pedro's siblings, Esperanza and Leandro.
They have, however, expressly aquiesced to the sale and waived their right to
the property in the affidavit presented by Spouses Monteiro.36 As such, the
petitioners have no right to their counterclaims of demolition of improvements
and payment of damages.
x With Spouses Monteiro having sufficiently proved their claim over the subject I ANNOTATION
/3 portion of the southern-half of the prope1iy through the Bilihan, the lower
courts did not err in awarding possession, rentals, attorney's fees, and litigation
expenses to them. x ONLY rule that has direct application to written contracts
x The Court, however, finds that the award of rentals should be reckoned from
o No distinction between a public and private document x
January 2, 2001, the date the Spouses Monteiro filed their Amended Complaint
seeking recovery of the subject portion. Interest at the rate of 6% per annum GR: parol evidence rule only applies to written contracts
shall also be imposed on the total amount of rent due from finality of this o XPN: it is likewise applicable to wills, though the latter is
Decision until fully paid. not a contract
x Contract Æ meeting of the minds between 2 or more persons
b. Parol Evidence o It is the “meeting of the minds” that constitutes a
contract and NOT the document evidencing it
Section 9. Evidence of written agreements. — When the terms of an agreement have o Such document is merely the instrument, that is, the
been reduced to writing, it is considered as containing all the terms agreed upon and there tangible evidence of the contract
can be, between the parties and their successors in interest, no evidence of such terms other
than the contents of the written agreement.
x As a rule, a contract need not be in any form
o Hence, an oral contract is generally VALID
However, a party may present evidence to modify, explain or add to the terms of written o However, for legal convenience, most persons choose to
agreement if he puts in issue in his pleading: reduce into writing their agreement
x Once such parties reduce such agreement into writing, the parol
(a) An intrinsic ambiguity, mistake or imperfection in the written agreement; evidence rule comes into play
(b) The failure of the written agreement to express the true intent and agreement of
o “When the terms of an agreement have been reduced to
the parties thereto; writing” Æ it is considered as containing all the terms
agreed upon and there can be, between the parties and
(c) The validity of the written agreement; or their successors in interest

231
RECTO, GAYLE ANGELI M.
2011-0008 | AUSL
Personal Notes on Remedial Law 2 Review (based on the syllabus of Prof. Henedino M. Brondial)

ƒ EFFECT: Generally, no evidence of such terms incompleteness in the writing, or the presence
other than the contents of the written of inconsistent provisions therein
agreement. o Failure of the written agreement to express the true
ƒ Hence, generally, no extraneous evidence intent and agreement of the parties thereto
may be presented for any of the ff purposes: ƒ Aside from mistake and imperfection, the ff
x Modify contract may be causes of failure to express the true
x Explain it intent and agreement of the parties to the
x Add to it contract (Arts 1359 and 1364 CC):
ƒ TOTAL INTEGRATION of the agreement Æ in x Fraud
PH, the American jurisprudence on partial and x Inequitable conduct
total integration of agreements is irrelevant x Accident
since Section 9 provides that once the x Ignorance
agreement is reduced into writing, it is x Lack of skill
considered as containing ALL terms agreed x Negligence
upon by the parties x Bad faith
o Whatever is NOT found in the writing is understood to ƒ Action for reformation of the instrument Æ
have been waived and abandoned proper remedy of the aggrieved party where
x Parol Æ literally means oral or verbal parol evidence may be presented by the latter
o BUT if used with reference to contracts Æ it means to prove the real intention of the parties
extraneous or evidence aliunde x In such action, it is the
x PURPOSE of the parol evidence rule: to give certainty to written INSTRUMENT and NOT the
transactions; to preserve the reliability thereof; to protect the sanctity contract that is being sought to be
of written agreements reformed
x DOES NOT apply to oral agreements x Hence, an action for the
x Applies only to the parties to the written contract AND their reformation of the instrument
successors-in-interest PRESUPPOSES that there is a
o Hence, although parol evidence is generally inadmissible, contract, that is, that there is a
this rule does not apply to a stranger to the contract or meeting of the minds
one who is not privy of the terms thereof x In other words, when it is shown
x In case of WILLS: that there was NO meeting of the
o General, a will may not be probated except upon minds between the parties
evidence of the contents thereof by presenting before the because consent was vitiated by
court the actual will itself fraud etc., the proper remedy is
ƒ XPNs: NOT an action for reformation
x Notarial will Æ may be BUT one for the annulment of the
reconstituted by parol evidence contract
when lost or destroyed o Validity of the written agreement
x Holographic will Æ may be proven o Existence of other terms agreed to by the parties or their
by a mere photostatic copy successors in interest after the execution of the written
x HOW to introduce parol evidence Æ put in issue any of the agreement
circumstances set forth in Section 9 and prove the same
o “Putting in issue” Æ to allege in the pleadings Best Evidence Rule Parol Evidence Rule
o Hence, the mere existence of a circumstance that may Province/ establishes preference for the not concerned with the primacy of
justify the presentation of parol evidence DOES NOT ipso Office of the original document over a evidence but presupposes that the
facto result in the allowance of the same rule secondary evidence thereof original is available
x It is NOT the province of the court to amend a contract by precludes the admission of other
construction evidence to prove the terms of a
What is precludes the admission of
o Contract is considered as a law between the parties document other than the contents
precluded by secondary evidence if the original
thereto the rule document is available
of the document itself for the
x As to PRIOR, CONTEMPORANEOUS and SUBSEQUENT agreements Æ purpose of varying the terms of the
writing
no distinction under the new Rules, hence, should still be put in issue
so that parol evidence may be admitted any litigant to an action whether
Who may only by the parties to the document
x XPNs to the parol evidence rule under Section 9: invoke the rule
said litigant is a party to the
and their successors-in-interest
o An intrinsic ambiguity, mistake or imperfection in document involved
the written agreement applies to all forms of writing
applies to written agreements and
Application where the contents thereof are
wills
Intrinsic/ Latent Extrinsic/ Patent sought to be proven
one which is not apparent on
the face of the document but
one which appears on the o Parol evidence rule may be WAIVED Æ by failure to nvoke the
which lies in the person or benefits of the said rule
very face of the instrument
thing; when the language is
Definition
clear and intelligible and
and arises from the defective, o The fact that the court has admitted parol evidence does not ipso
suggests a single meaning but
obscure or insensible facto result in great probative weight which is a matter of judicial
language used
some matters to the writing discretion
creates ambiguity
parol evidence shall never be CASES
When parol when such intrinsic ambiguity is admitted because to do so
evidence may put in issue, parol evidence would be tantamount to the RAFAEL S. ORTANEZ, petitioner, vs. THE COURT OF APPEALS, OSCAR INOCENTES, AND
be admitted may be admitted creation of a new contract by ASUNCION LLANES INOCENTES, respondents.
the court G.R. No. 107372. January 23, 1997
THIRD DIVISION

ƒ Mistake and imperfection Æ may result in FACTS:


failure of the instrument to capture the true x Inocentes Æ sold 2 parcels of land in QC to Ortanez for a consideration of
intent and agreement of the parties P35,000.00 and P20,000.00, respectively
ƒ Mistake Æ refers to a mistake of fact and one x Inocentes received the sums of money BUT failed to deliver the Ortanez the TCT
x Inocentes refused
which is mutual to the parties
o on the ground that the title of the first lot is in the possession of
ƒ Imperfection Æ refers to an inaccurate
another person,[4] and petitioner's acquisition of the title of the
statement in the agreement, or other lot is subject to certain conditions.
232
RECTO, GAYLE ANGELI M.
2011-0008 | AUSL
Personal Notes on Remedial Law 2 Review (based on the syllabus of Prof. Henedino M. Brondial)

x Ortanez Æ filed an action for specific performance before the RTC intention[21] of the buyer (petitioner) and the seller (private
x Inocentes’ evidence: respondents). Such issue must be "squarely presented."[22] Private
o private respondent Oscar Inocentes, a former judge, orally testified respondents merely alleged that the sale was subject to four (4)
that the sale was subject to the above conditions,[7] although such conditions which they tried to prove during trial by parol
conditions were not incorporated in the deeds of sale. evidence.[23] Obviously, this cannot be done, because they
x Ortanez Æ timely objected to the oral testimony of Oscar did not plead any of the exceptions mentioned in the parol
x RTC Æ overruled the objection evidence rule.[24] Their case is covered by the general rule that
x RTC Æ dismissed Ortanez’s complaint the contents of the writing are the only repository of the terms of
o Despite petitioner's timely objections on the ground that the the agreement. Considering that private respondent Oscar
introduction of said oral conditions was barred by the parol Inocentes is a lawyer (and former judge) he was "supposed to be
evidence rule, the lower court nonetheless, admitted them and steeped in legal knowledge and practices" and was "expected to
eventually dismissed the complaint as well as the counterclaim know the consequences"[25] of his signing a deed of absolute sale.
x Ortanez Æ appealed to CA Had he given an iota's attention to scrutinize the deeds, he would
x CA Æ affirmed RTC have incorporated important stipulations that the transfer of title to
x Ortanez Æ filed a Rule 45 before the SC said lots were conditional.[26
x One last thing, assuming arguendo that the parol evidence is admissible,
ISSUE: Whether the subject parol evidence was correctly admitted to establish the alleged oral it should nonetheless be disbelieved as no other evidence appears from
conditions-precedent to a contract of sale, considering that the deeds of sale are silent on the record to sustain the existence of the alleged conditions. Not even
such conditions the other seller, Asuncion Inocentes, was presented to testify on such conditions.

HELD: NO.
x The parol evidence herein introduced is inadmissible. LAPULAPU FOUNDATION, INC. and ELIAS Q. TAN, Petitioners, vs. COURT OF APPEALS
o First, private respondents' oral testimony on the alleged conditions, (Seventeenth Division) and ALLIED BANKING CORP., Respondents.
coming from a party who has an interest in the outcome of the G.R. No. 126006 January 29, 2004
case, depending exclusively on human memory, is not as reliable as SECOND DIVISION
written or documentary evidence.[8] Spoken words could be
notoriously unreliable unlike a written contract which FACTS:
speaks of a uniform language.[9] Thus, under the general x Elias Q. Tan (then President of the co-petitioner Lapulapu Foundation, Inc.) Æ
rule in Section 9 of Rule 130[10] of the Rules of Court, when obtained four loans from the respondent Allied Banking Corporation covered by
the terms of an agreement were reduced to writing, as in four promissory notes in the amounts of P100,000 each.
this case, it is deemed to contain all the terms agreed upon x Allied Bank Æ demanded payment
and no evidence of such terms can be admitted other than x Lapu Lapu and Tan failed to pay
the contents thereof.[11] Considering that the written deeds of x Allied Bank Æ filed an action for sum of money before RTC Cebu
sale were the only repository of the truth, whatever is not found in o seeking payment by the petitioners, jointly and solidarily, of the
said instruments must have been waived and abandoned by the sum of P493,566.61 representing their loan obligation, exclusive of
parties.[12] Examining the deeds of sale, we cannot even make an interests, penalty charges, attorney’s fees and costs.
inference that the sale was subject to any condition. As a contract, x Lapu lapu Æ filed answer
it is the law between the parties.[13] o denied incurring indebtedness from the respondent Bank alleging
o Secondly, to buttress their argument, private respondents rely on that the loans were obtained by petitioner Tan in his personal
the case of Land Settlement Development, Co. vs. Garcia capacity, for his own use and benefit and on the strength of the
Plantation[14] where the Court ruled that a condition precedent to personal information he furnished the respondent Bank.
a contract may be established by parol evidence. However, the o The petitioner Foundation maintained that it never authorized
material facts of that case are different from this case. In the petitioner Tan to co-sign in his capacity as its President any
former, the contract sought to be enforced[15] expressly promissory note and that the respondent Bank fully knew that the
stated that it is subject to an agreement containing the loans contracted were made in petitioner Tan’s personal capacity
conditions-precedent which were proven through parol and for his own use and that the petitioner Foundation never
evidence. Whereas, the deeds of sale in this case, made no benefited, directly or indirectly, therefrom.
reference to any pre- conditions or other agreement. In o The petitioner Foundation then interposed a cross-claim against
fact, the sale is denominated as absolute in its own terms. petitioner Tan alleging that he, having exceeded his authority,
o Third, the parol evidence herein sought to be introduced should be solely liable for said loans, and a counterclaim against
would vary, contradict or defeat the operation of a valid the respondent Bank for damages and attorney’s fees.
instrument,[16] hence, contrary to the rule that: x Tan Æ filed his answer
ƒ The parol evidence rule forbids any addition to o admitted that he contracted the loans from the respondent Bank in
x x x the terms of a written instrument by his personal capacity.
testimony purporting to show that, at or before o The parties, however, agreed that the loans were to be paid from
the signing of the document, other or different the proceeds of petitioner Tan’s shares of common stocks in the
terms were orally agreed upon by the Lapulapu Industries Corporation, a real estate firm.
parties.[17] o The loans were covered by promissory notes which were
ƒ Although parol evidence is admissible to explain the automatically renewable ("rolled-over") every year at an amount
meaning of a contract, "it cannot serve the purpose of including unpaid interests, until such time as petitioner Tan was
incorporating into the contract additional able to pay the same from the proceeds of his aforesaid shares.
contemporaneous conditions which are not mentioned o According to petitioner Tan, the respondent Bank’s employee
at all in the writing unless there has been fraud or required him to affix two signatures on every promissory note,
mistake." [18] No such fraud or mistake exists in this assuring him that the loan documents would be filled out in
case. accordance with their agreement.
o Fourth, we disagree with private respondents' argument that their o However, after he signed and delivered the loan documents to the
parol evidence is admissible under the exceptions provided by the respondent Bank, these were filled out in a manner not in accord
Rules, specifically, the alleged failure of the agreement to express with their agreement, such that the petitioner Foundation was
the true intent of the parties. Such exception obtains only in the included as party thereto.
following instance: o Further, prior to its filing of the complaint, the respondent Bank
ƒ "[W]here the written contract is so ambiguous made no demand on him.
or obscure in terms that the contractual x RTC Æ in favor of ALLIED BANK; ordered both Lapu Lapu and Tan to pay
intention of the parties cannot be understood x Lapu Lapu and Tan Æ appealed to CA
from a mere reading of the instrument. In such x CA Æ affirmed RTC
a case, extrinsic evidence of the subject matter of the o disbelieved petitioner Tan’s claim that the loans were his personal
contract, of the relations of the parties to each other, loans as the promissory notes evidencing them showed upon their
and of the facts and circumstances surrounding them faces that these were obligations of the petitioner Foundation, as
when they entered into the contract may be received contracted by petitioner Tan himself in his "official and personal
to enable the court to make a proper interpretation of character." Applying the parol evidence rule, the CA likewise
the instrument." [19] rejected petitioner Tan’s assertion that there was an unwritten
x In this case, the deeds of sale are agreement between him and the respondent Bank that he would
clear, without any ambiguity, pay the loans from the proceeds of his shares of stocks in the
mistake or imperfection, much less Lapulapu Industries Corp.
obscurity or doubt in the terms x Lapu Lapu and Tan Æ filed a Rule 45 before the SC
thereof.
o Fifth, we are not persuaded by private respondents contention that ISSUE: Whether Tan’s assertion as regards the unwritten agreement between him and the
they "put in issue by the pleadings" the failure of the written respondent Bank that he would pay the loans from the proceeds of his shares of stocks in the
agreement to express the true intent of the parties. Record Lapulapu Industries Corp. may be given credence by the court.
shows[20] that private respondents did not expressly plead that the
deeds of sale were incomplete or that it did not reflect the HELD: NO.
233
RECTO, GAYLE ANGELI M.
2011-0008 | AUSL
Personal Notes on Remedial Law 2 Review (based on the syllabus of Prof. Henedino M. Brondial)

x The Court particularly finds as incredulous petitioner Tan’s allegation that he was of 4,024 square meters, which he
made to sign blank loan documents and that the phrase "IN MY subdivided into two portions and caused
OFFICIAL/PERSONAL CAPACITY" was superimposed by the respondent Bank’s to be covered by the two TCTs in
employee despite petitioner Tan’s protestation. The Court is hard pressed to question.
believe that a businessman of petitioner Tan’s stature could have been so o The petitioner claimed that in signing the Agreement, he was led to
careless as to sign blank loan documents. believe, based on the parties’ rough estimation, that the area he
x In contrast, as found by the CA, the promissory notes11 clearly showed upon actually possessed is only 3,020 square meters contrary to the
their faces that they are the obligation of the petitioner Foundation, as parties’ real intention - i.e., the extent of their ownership would be
contracted by petitioner Tan "in his official and personal capacity."12 Moreover, based on their actual possession
the application for credit accommodation,13 the signature cards of the two o further claimed that the respondent voluntarily participated in
accounts in the name of petitioner Foundation,14 as well as New Current executing the Affidavit, which corrected the mistake in the
Account Record,15 all accompanying the promissory notes, were signed by previously executed Agreement26 and confirmed the petitioner’s
petitioner Tan for and in the name of the petitioner Foundation.16 These ownership over the disputed property. The petitioner asked for the
documentary evidence unequivocally and categorically establish that the loans dismissal of the complaint and for a declaration that he is the lawful
were solidarily contracted by the petitioner Foundation and petitioner Tan. owner of the parcels of land covered by his titles.
x As a corollary, the parol evidence rule likewise constrains this Court to x RTC Æ DISMISSED the action
reject petitioner Tan’s claim regarding the purported unwritten o Valdez failed to preponderantly prove that the Benigna Deed and
agreement between him and the respondent Bank on the payment of the Affidavit are fabricated and, consequently, no ground exists to
the obligation. Section 9, Rule 130 of the of the Revised Rules of Court nullify the petitioner’s titles.
provides that "[w]hen the terms of an agreement have been reduced to writing, o The court observed that the respondent did not even compare his
it is to be considered as containing all the terms agreed upon and there can be, genuine signature with the signatures appearing in these
between the parties and their successors-in-interest, no evidence of such terms documents.
other than the contents of the written agreement."17 x Valdez Æ appealed to CA
x In this case, the promissory notes are the law between the petitioners and the x CA Æ reversed RTC
respondent Bank. These promissory notes contained maturity dates as follows: o ruling against the authenticity of the Benigna Deed and the
o February 5, 1978, March 28, 1978, April 11, 1978 and May 5, 1978, Affidavit.
respectively. That these notes were to be paid on these dates is o The CA gave weight to Benigna’s death certificate which shows the
clear and explicit. Nowhere was it stated therein that they would be impossibility of Benigna’s execution of the deed in 1969.
renewed on a year-to-year basis or "rolled-over" annually until paid o The CA also noted the discrepancy between the respondent’s
from the proceeds of petitioner Tan’s shares in the Lapulapu signatures as appearing in the Affidavit, on one hand, and the
Industries Corp. Accordingly, this purported unwritten agreement documents on record, on the other.27
could not be made to vary or contradict the terms and conditions in o The CA added that the respondent’s failure to compare his genuine
the promissory notes. signature from his purported signatures appearing in the
x Evidence of a prior or contemporaneous verbal agreement is generally petitioner’s documents is not fatal, since Section 22, Rule 132 of
not admissible to vary, contradict or defeat the operation of a valid the Rules of Court allows the court to make its own comparison.
contract.18 While parol evidence is admissible to explain the meaning x Leoveras Æ filed a Rule 45 before the SC
of written contracts, it cannot serve the purpose of incorporating into o he claims that since the Agreement does not reflect the true
the contract additional contemporaneous conditions which are not intention of the parties, the Affidavit was subsequently executed in
mentioned at all in writing, unless there has been fraud or mistake.19 order to reflect the parties’ true intention
No such allegation had been made by the petitioners in this case.
ISSUE: Whether the subject affidavit was sufficiently proved that the prior agreement did not
reflect the true intention of the parties thereto.
MODESTO LEOVERAS, Petitioner, vs. CASIMERO VALDEZ, Respondent.
G.R. No. 169985 June 15, 2011 HELD: NO.
THIRD DIVISION x The petitioner’s argument calls to fore the application of the parol evidence
rule,35 i.e., when the terms of an agreement are reduced to writing, the written
FACTS: agreement is deemed to contain all the terms agreed upon and no evidence of
x Maria Sta. Maria and Dominga Manangan were the registered owners - three- these terms can be admitted other than what is contained in the written
fourths (¾) and one-fourth (¼) pro-indiviso, respectively - of a parcel of land agreement.36 Whatever is not found in the writing is understood to have been
located in Pangasinan waived and abandoned.37
x Sta. Maria sold her three-fourths (¾) share to Benigna Llamas x To avoid the operation of the parol evidence rule, the Rules of Court
o When Benigna died in 1944,7 she willed her three-fourths (¾) allows a party to present evidence modifying, explaining or adding to
share equally to her sisters Alejandra Llamas and Josefa Llamas the terms of the written agreement if he puts in issue in his pleading, as
o THUS, Alejandra and Josefa each owned one-half (½) of Benigna’s in this case, the failure of the written agreement to express the true
three-fourths (¾) share. intent and agreement of the parties. The failure of the written
x Alejandra’s heirs sold their predecessor’s one-half (½) share (roughly equivalent agreement to express the true intention of the parties is either by
to 10,564 square meters) to Valdez, as evidenced by a Deed of Absolute Sale reason of mistake, fraud, inequitable conduct or accident, which
x Josefa sold her own one-half (½) share (subject property) to Valdez and the nevertheless did not prevent a meeting of the minds of the parties.38 x
petitioner, as evidenced by another Deed of Absolute Sale At the trial, the petitioner attempted to prove, by parol evidence, the
x Valdez then asked the Register of Deeds of Lingayen, Pangasinan on the alleged true intention of the parties by presenting the Affidavit, which
requirements for the transfer of title over the portion allotted to him on the allegedly corrected the mistake in the previously executed Agreement
subject property and confirmed his ownership of the parcels of land covered by his
o He learned that Leoveras had already obtained in his name two titles. It was the petitioner’s staunch assertion that the respondent co-executed
transfer certificates of title (TCTs): one, TCT No. 195812 - covering this Affidavit supposedly to reflect the parties’ true intention.
an area of 3,020 square meters; and two, TCT No. 195813 - x In the present petition, however, the petitioner made a damaging
covering an area of 1,004 square meters (or a total of 4,024 square admission that the Benigna Deed is fabricated, thereby completely
meters). bolstering the respondent’s cause of action for reconveyance of the
x Valdez Æ filed an action for Annulment of Title, Reconveyance and Damages disputed property on the ground of fraudulent registration of title. Since
against Leoveras the Affidavit merely reflects what is embodied in the Benigna Deed, the
o seeking the reconveyance of the 1,004-square meter portion petitioner’s admission, coupled with the respondent’s denial of his purported
(disputed property) covered by TCT No. 195813, on the ground that signature in the Affidavit, placed in serious doubt the reliability of this document,
Leoveras is entitled only to the 3,020 square meters identified in the supposedly the bedrock of the petitioner’s defense.
parties’ Agreement. x Curiously, if the parties truly intended to include in the petitioner’s share
o The respondent sought the nullification of the petitioner’s titles by the disputed property, the petitioner obviously need not go at length of
contesting the authenticity of the petitioner’s documents. fabricating a deed of sale to support his application for the transfer of
Particularly, the respondent assailed the Benigna Deed by title of his rightful portion of the subject property. Notably, there is
presenting Benigna’s death certificate. The respondent argued that nothing in the Affidavit (that supposedly corrected the mistake in the
Benigna could not have executed a deed, which purports to convey earlier Agreement) that supports the petitioner’s claim that the
4,024 square meters to the petitioner, in 1969 because Benigna partition of the subject property is based on the parties’ actual
already died in 1944. The respondent added that neither could Sta. possession.
Maria have sold to the parties her three-fourths (¾) share in 1969 x Note that the RTC dismissed the complaint based on the respondent’s alleged
because she had already sold her share to Benigna in 1932. failure to prove the spuriousness of the documents submitted by the petitioner to
x Leoveras Æ filed his answer the Register of Deeds. However, by admitting the presentation of a false deed in
o claimed that the parties already had securing his title, the petitioner rendered moot the issue of authenticity of the
ƒ (i) delineated their respective portions of the subject Benigna Deed and relieved the respondent of the burden of proving its falsity as a
property even before they acquired it in 1969 and ground to nullify the petitioner’s titles.
ƒ (ii) agreed that upon acquisition, each would own the x By fraudulently causing the transfer of the registration of title over the
portion as delineated; disputed property in his name, the petitioner holds the title to this
x that the area he actually possessed and disputed property in trust for the benefit of the respondent as the true
subsequently acquired has a total area owner;39 registration does not vest title but merely confirms or records
234
RECTO, GAYLE ANGELI M.
2011-0008 | AUSL
Personal Notes on Remedial Law 2 Review (based on the syllabus of Prof. Henedino M. Brondial)

title already existing and vested. The Torrens system of registration


cannot be used to protect a usurper from the true owner, nor can it be
used as a shield for the commission of fraud, or to permit one to enrich c. Electronic Evidence
oneself at the expense of others.40 Hence, the CA correctly ordered the
reconveyance of the disputed property, covered by TCT No. 195813, to
Rules on Electronic Evidence
the respondent.
x The parties’ Agreement effectively partitioned the subject property A.M. No. 01-7-01-SC July 17, 2001
x The petitioner also relies on his alleged actual possession of the disputed RULES ON ELECTRONIC EVIDENCE
property to support his claim of ownership. Notably, both parties make
conflicting assertions of possession of the disputed property.41 The petitioner Acting on the Memorandum dated 18 June 2001 of the Committee on the Revision of the
testified on his possession as follows: Rules of Court to Draft the Rules on E-Commerce Law [R.A. No. 8792] submitting the Rules on
o Q: How many square meters did you get from the land and how Electronic Evidence for this Court's consideration and approval, the Court Resolved to
many square meters was the share of [respondent]? APPROVED the same.
o A: 4[0]20 square meters and my brother-in-law 6,000 plus square
meters. The Rules on Electronic Evidence shall apply to cases pending after their effectivity. These
o Q: Was there a boundary between the 4,020 square meters and Rules shall take effect on the first day of August 2001 following thier publication before the
the rest of the property which (sic) designated by your brother-in- 20th of July in two newspapers of general circulation in the Philippines
law?
o A: There is sir, and the boundary is the fence. 17th July 2001.
o Q: When did you put up that fence which is the boundary?
o A: After the deed of sale was made. RULES ON ELECTRONIC EVIDENCE
o Q: And that boundary fence which you put according to you since
the execution of the Deed of Absolute Sale in 1969 up to the Rule 1
present does it still exist? COVERAGE
o A: Yes, sir.
o Q: Since the time you purchased the property according to you you Section 1. Scope. - Unless otherwise provided herein, these Rules shall apply whenever an
already divided the property, is that correct? electronic document or electronic data message, as defined in Rule 2 hereof, is offered or used
o A: Yes, sir. in evidence.
o Q: And that as of today who is in possession of that 4,020 square
meters? Section 2. Cases covered. - These Rules shall apply to all civil actions and proceedings, as
o A: I, sir.42 well as quasi-judicial and administrative cases.
x The petitioner and the respondent were originally co-owners of the subject
property when they jointly bought it from the same vendor in 1969. However, Section 3. Application of other rules on evidence. - In all matters not specifically
the parties immediately terminated this state of indivision by executing an covered by these Rules, the Rules of Court and pertinent provisions of statutes containing
Agreement, which is in the nature of a partition agreement. rules on evidence shall apply.
x The Civil Code of the Philippines defines partition as the separation, division and
assignment of a thing held in common among those to whom it may belong.43 Rule 2
Partition is the division between two or more persons of real or personal DEFINITION OF TERMS AND CONSTRUCTION
property, owned in common, by setting apart their respective interests so that
they may enjoy and possess these in severalty,44 resulting in the partial or total Section 1. Definition of terms. - For purposes of these Rules, the following terms are
extinguishment of co-ownership.45 defined, as follows:
x In the present case, the parties agreed to divide the subject property by giving
the petitioner the 3,020 square meters "residential portion on the northern part (a) "Asymmetric or public cryptosystem" means a system capable of generating a secure key
near the Municipal road."46 There is no dispute that this 3,020- square meter pair, consisting of a private key for creating a digital signature, and a public key for verifying
portion is the same parcel of land identified as Lot No. 2 (which is not the the digital signature.
subject of the respondent’s action for reconveyance) in the Affidavit and the
Subdivision Plan presented by the petitioner before the Register of Deeds. The (b) "Business records" include records of any business, institution, association, profession,
fact that the Agreement lacks technical description of the parties’ respective occupation, and calling of every kind, whether or not conducted for profit, or for legitimate or
portions or that the subject property was then still embraced by a single illegitimate purposes.
certificate of title could not legally prevent a partition, where the different
portions allotted to each were determined and became separately identifiable, as (c) "Certificate" means an electronic document issued to support a digital signature which
in this case.47 purports to confirm the identity or other significant characteristics of the person who holds a
x What is strikingly significant is that even the petitioner’s own testimony merely particular key pair.
attempted to confirm his actual possession of the disputed property, without,
however, supporting his claim - contrary to the written Agreement - that the (d) "Computer" refers to any single or interconnected device or apparatus, which, by electronic,
parties’ ownership of the subject property would be co-extensive with their electro-mechanical or magnetic impulse, or by other means with the same function, can receive,
possession. This is the core of the petitioner’s defense. At any rate, just as non- record, transmit, store, process, correlate, analyze, project, retrieve and/or produce
possession does not negate ownership, neither does possession automatically information, data, text, graphics, figures, voice, video, symbols or other modes of expression or
prove ownership,48 especially in the face of an unambiguous document executed perform any one or more of these functions.
by the parties themselves.1avvphi1
x Contrary to the petitioner’s claim that his actual possession determines the (e) "Digital signature" refers to an electronic signature consisting of a transformation of an
extent of his ownership, it is the parties’ Agreement that defines the extent of electronic document or an electronic data message using an asymmetric or public
their ownership in the subject property. One of the legal effects of partition, cryptosystem such that a person having the initial untransformed electronic document and the
whether by agreement among the co-owners or by judicial proceeding, is to signer's public key can accurately determine:
terminate the co-ownership and, consequently, to make the previous co-owners
the absolute and exclusive owner of the share allotted to him.49 i. whether the transformation was created using the private key that corresponds to the
x Parenthetically, the respondent declared for taxation purposes the portion he signer's public key; and
claims in December 1987.50 The total area (7,544 square meters) of the
properties declared is equivalent to the area allotted to the respondent under the ii. whether the initial electronic document had been altered after the transformation was
Agreement. On the other hand, the petitioner declared the 1,004-square meter made.
portion only in September 1994, under Tax Declaration No. 9393,51 despite his
claim of exclusive and adverse possession since 1969. (f) "Digitally signed" refers to an electronic document or electronic data message bearing a
x Nullification of the petitioner’s title over the 3,020 square meter portion digital signature verified by the public key listed in a certificate.
x While the petitioner admitted using a spurious document in securing his titles,
nonetheless, he questions the CA’s nullification of TCT No. 195812 on the ground (g) "Electronic data message" refers to information generated, sent, received or stored by
that, per the respondent’s own admission and the parties’ Agreement, he is the electronic, optical or similar means.
rightful owner of the land covered by this title.
o We disagree. (h) "Electronic document" refers to information or the representation of information, data,
x The petitioner’s argument confuses registration of title with ownership.52 While figures, symbols or other modes of written expression, described or however represented, by
the petitioner’s ownership over the land covered by TCT No. 195812 is which a right is established or an obligation extinguished, or by which a fact may be proved
undisputed, his ownership only gave him the right to apply for the proper and affirmed, which is received, recorded, transmitted, stored, processed, retrieved or
transfer of title to the property in his name. Obviously, the petitioner, even as a produced electronically. It includes digitally signed documents and any print-out or output,
rightful owner, must comply with the statutory provisions on the transfer of readable by sight or other means, which accurately reflects the electronic data message or
electronic document. For purposes of these Rules, the term "electronic document" may be
registered title to lands.53 Section 53 of Presidential Decree No. 1529 provides
used interchangeably with "electronic data message".
that the subsequent registration of title procured by the presentation of a forged
deed or other instrument is null and void. Thus, the subsequent issuance of TCT
No. 195812 gave the petitioner no better right than the tainted registration (i) "Electronic key" refers to a secret code which secures and defends sensitive information
which was the basis for the issuance of the same title. The Court simply cannot that crosses over public channels into a form decipherable only with a matching electronic key.
allow the petitioner’s attempt to get around the proper procedure for registering
the transfer of title in his name by using spurious documents.
235
RECTO, GAYLE ANGELI M.
2011-0008 | AUSL
Personal Notes on Remedial Law 2 Review (based on the syllabus of Prof. Henedino M. Brondial)

(j) "Electronic signature" refers to any distinctive mark, characteristic and/or sound in
electronic form, representing the identity of a person and attached to or logically associated Rule 6
with the electronic data message or electronic document or any methodology or procedure ELECTRONIC SIGNATURES
employed or adopted by a person and executed or adopted by such person with the intention
of authenticating, signing or approving an electronic data message or electronic document. For Section 1. Electronic signature. - An electronic signature or a digital signature
purposes of these Rules, an electronic signature includes digital signatures. authenticated in the manner prescribed hereunder is admissible in evidence as the functional
equivalent of the signature of a person on a written document.
(k) "Ephemeral electronic communication" refers to telephone conversations, text messages,
chatroom sessions, streaming audio, streaming video, and other electronic forms of Section 2. Authentication of electronic signatures. - An electronic signature may be
communication the evidence of which is not recorded or retained. authenticated in any of the following manner:

(l) "Information and communication system" refers to a system for generating, sending, (a) By evidence that a method or process was utilized to establish a digital signature and verify
receiving, storing or otherwise processing electronic data messages or electronic documents the same;
and includes the computer system or other similar devices by or in which data are recorded or
stored and any procedure related to the recording or storage of electronic data messages or (b) By any other means provided by law; or
electronic documents.
(c) By any other means satisfactory to the judge as establishing the genuineness of the
(m) "Key pair" in an asymmetric cryptosystem refers to the private key and its mathematically electronic signature.
related public key such that the latter can verify the digital signature that the former creates.
Section 3. Disputable presumptions relating to electronic signatures. - Upon the
(n) "Private key" refers to the key of a key pair used to create a digital signature. authentication of an electronic signature, it shall be presumed that:

(o) "Public key" refers to the key of a key pair used to verify a digital signature. (a) The electronic signature is that of the person to whom it correlates;

Section 2. Construction. - These Rules shall be liberally construed to assist the parties in (b) The electronic signature was affixed by that person with the intention of authenticating or
obtaining a just, expeditious, and inexpensive determination of cases. approving the electronic document to which it is related or to indicate such person's consent to
the transaction embodied therein; and
The interpretation of these Rules shall also take into consideration the international origin of
Republic Act No. 8792, otherwise known as the Electronic Commerce Act. (c) The methods or processes utilized to affix or verify the electronic signature operated
without error or fault.
Rule 3
ELECTRONIC DOCUMENTS Section 4. Disputable presumptions relating to digital signatures. - Upon the
authentication of a digital signature, it shall be presumed, in addition to those mentioned in
Section 1. Electronic documents as functional equivalent of paper-based documents. - the immediately preceding section, that:
Whenever a rule of evidence refers to the term writing, document, record, instrument,
memorandum or any other form of writing, such term shall be deemed to include an electronic (a) The information contained in a certificate is correct;
document as defined in these Rules.
(b) The digital signature was created during the operational period of a certificate;
Section 2. Admissibility. - An electronic document is admissible in evidence if it complies
with the rules on admissibility prescribed by the Rules of Court and related laws and is (c) No cause exists to render a certificate invalid or revocable;
authenticated in the manner prescribed by these Rules.
(d) The message associated with a digital signature has not been altered from the time it was
Section 3. Privileged communication. - The confidential character of a privileged signed; and,
communication is not lost solely on the ground that it is in the form of an electronic document.
(e) A certificate had been issued by the certification authority indicated therein.
Rule 4
BEST EVIDENCE RULE Rule 7
EVIDENTIARY WEIGHT OF ELECTRONIC DOCUMENTS
Section 1. Original of an electronic document. - An electronic document shall be
regarded as the equivalent of an original document under the Best Evidence Rule if it is a Section 1. Factors for assessing evidentiary weight. - In assessing the evidentiary
printout or output readable by sight or other means, shown to reflect the data accurately. weight of an electronic document, the following factors may be considered:

Section 2. Copies as equivalent of the originals. - When a document is in two or more (a) The reliability of the manner or method in which it was generated, stored or
copies executed at or about the same time with identical contents, or is a counterpart communicated, including but not limited to input and output procedures, controls, tests and
produced by the same impression as the original, or from the same matrix, or by mechanical checks for accuracy and reliability of the electronic data message or document, in the light of
or electronic re-recording, or by chemical reproduction, or by other equivalent techniques all the circumstances as well as any relevant agreement;
which accurately reproduces the original, such copies or duplicates shall be regarded as the
equivalent of the original. (b) The reliability of the manner in which its originator was identified;

Notwithstanding the foregoing, copies or duplicates shall not be admissible to the same extent (c) The integrity of the information and communication system in which it is recorded or
as the original if: stored, including but not limited to the hardware and computer programs or software used as
well as programming errors;
(a) a genuine question is raised as to the authenticity of the original; or
(d) The familiarity of the witness or the person who made the entry with the communication
(b) in the circumstances it would be unjust or inequitable to admit the copy in lieu of the and information system;
original.
(e) The nature and quality of the information which went into the communication and
Rule 5 information system upon which the electronic data message or electronic document was
AUTHENTICATION OF ELECTRONIC DOCUMENTS based; or

Section 1. Burden of proving authenticity. - The person seeking to introduce an (f) Other factors which the court may consider as affecting the accuracy or integrity of the
electronic document in any legal proceeding has the burden of proving its authenticity in the electronic document or electronic data message.
manner provided in this Rule.
Section 2. Integrity of an information and communication system. - In any dispute
Section 2. Manner of authentication. - Before any private electronic document offered as involving the integrity of the information and communication system in which an electronic
authentic is received in evidence, its authenticity must be proved by any of the following document or electronic data message is recorded or stored, the court may consider, among
means: others, the following factors:

(a) by evidence that it had been digitally signed by the person purported to have signed the (a) Whether the information and communication system or other similar device was operated in
same; a manner that did not affect the integrity of the electronic document, and there are no other
reasonable grounds to doubt the integrity of the information and communication system;
(b) by evidence that other appropriate security procedures or devices as may be authorized by
the Supreme Court or by law for authentication of electronic documents were applied to the (b) Whether the electronic document was recorded or stored by a party to the proceedings
document; or with interest adverse to that of the party using it; or

(c) by other evidence showing its integrity and reliability to the satisfaction of the judge. (c) Whether the electronic document was recorded or stored in the usual and ordinary course
of business by a person who is not a party to the proceedings and who did not act under the
Section 3. Proof of electronically notarized document. - A document electronically notarized in control of the party using it.
accordance with the rules promulgated by the Supreme Court shall be considered as a public
document and proved as a notarial document under the Rules of Court. Rule 8
236
RECTO, GAYLE ANGELI M.
2011-0008 | AUSL
Personal Notes on Remedial Law 2 Review (based on the syllabus of Prof. Henedino M. Brondial)

BUSINESS RECORDS AS EXCEPTION TO THE HEARSAY RULE Section 2. Declaration of Policy - The State recognizes the vital role of information and
communications technology (ICT) in nation-building; the need to create an information-
Section 1. Inapplicability of the hearsay rule. - A memorandum, report, record or data friendly environment which supports and ensures the availability, diversity and affordability of
compilation of acts, events, conditions, opinions, or diagnoses, made by electronic, optical or ICT products and services; the primary responsibility of the private sector in contributing
other similar means at or near the time of or from transmission or supply of information by a investments and services in telecommunications and information technology; the need to
person with knowledge thereof, and kept in the regular course or conduct of a business develop, with appropriate training programs and institutional policy changes, human resources
activity, and such was the regular practice to make the memorandum, report, record, or data for the information technology age, a labor force skilled in the use of ICT and a population
compilation by electronic, optical or similar means, all of which are shown by the testimony of capable of operating and utilizing electronic appliances and computers; its obligation to
the custodian or other qualified witnesses, is excepted from the rule on hearsay evidence. facilitate the transfer and promotion of technology; to ensure network security, connectivity
and neutrality of technology for the national benefit; and the need to marshal, organize and
Section 2. Overcoming the presumption. - The presumption provided for in Section 1 of deploy national information infrastructures, comprising in both telecommunications network
this Rule may be overcome by evidence of the untrustworthiness of the source of information and strategic information services, including their interconnection to the global information
or the method or circumstances of the preparation, transmission or storage thereof. networks, with the necessary and appropriate legal, financial, diplomatic and technical
framework, systems and facilities.
Rule 9
METHOD OF PROOF PART II
ELECTRONIC COMMERCE IN GENERAL
Section 1. Affidavit evidence. - All matters relating to the admissibility and evidentiary
weight of an electronic document may be established by an affidavit stating facts of direct CHAPTER I
personal knowledge of the affiant or based on authentic records. The affidavit must GENERAL PROVISIONS
affirmatively show the competence of the affiant to testify on the matters contained therein.
Section 3. Objective - This Act aims to facilitate domestic and international dealings,
Section 2. Cross-examination of deponent. - The affiant shall be made to affirm the transactions, arrangements agreements, contracts and exchanges and storage of information
contents of the affidavit in open court and may be cross-examined as a matter of right by the through the utilization of electronic, optical and similar medium, mode, instrumentality and
adverse party. technology to recognize the authenticity and reliability of electronic documents related to such
activities and to promote the universal use of electronic transaction in the government and
Rule 10 general public.
EXAMINATION OF WITNESSES
Section 4. Sphere of Application - This Act shall apply to any kind of data message and
Section 1. Electronic testimony. - After summarily hearing the parties pursuant to Rule 9 of electronic document used in the context of commercial and non-commercial activities to
these Rules, the court may authorize the presentation of testimonial evidence by electronic include domestic and international dealings, transactions, arrangements, agreements contracts
means. Before so authorizing, the court shall determine the necessity for such presentation and exchanges and storage of information.
and prescribe terms and conditions as may be necessary under the circumstances, including
the protection of the rights of the parties and witnesses concerned. Section 5. Definition of Terms - For the purposes of this Act, the following terms are
defined, as follows:
Section 2. Transcript of electronic testimony. - When examination of a witness is done
electronically, the entire proceedings, including the questions and answers, shall be (a) "Addressee" refers to a person who is intended by the originator to receive the electronic
transcribed by a stenographer, stenotypist or other recorder authorized for the purpose, who data message or electronic document. The term does not include a person acting as an
shall certify as correct the transcript done by him. The transcript should reflect the fact that intermediary with respect to that electronic data message or electronic data document.
the proceedings, either in whole or in part, had been electronically recorded.
(b) "Computer" refers to any device or apparatus which, by electronic, electro-mechanical, or
Section 3. Storage of electronic evidence. - The electronic evidence and recording magnetic impulse, or by other means, is capable of receiving, recording, transmitting, storing,
thereof as well as the stenographic notes shall form part of the record of the case. Such processing, retrieving, or producing information, data, figures, symbols or other modes of
transcript and recording shall be deemed prima facie evidence of such proceedings. written expression according to mathematical and logical rules or of performing any one or
more of these functions.
Rule 11
AUDIO, PHOTOGRAPHIC, VIDEO, AND EPHEMERAL EVIDENCE (c) "Electronic Data Message" refers to information generated, sent, received or stored by
electronic, optical or similar means.
Section 1. Audio, video and similar evidence. - Audio, photographic and video evidence
of events, acts or transactions shall be admissible provided it shall be shown, presented or (d) "Information and Communications System" refers to a system intended for and capable of
displayed to the court and shall be identified, explained or authenticated by the person who generating, sending, receiving, storing, or otherwise processing electronic data messages or
made the recording or by some other person competent to testify on the accuracy thereof. electronic documents and includes the computer system or other similar device by or in which
data is recorded or stored and any procedures related to the recording or storage of electronic
Section 2. Ephemeral electronic communications. - Ephemeral electronic data message or electronic document.
communications shall be proven by the testimony of a person who was a party to the same or
has personal knowledge thereof. In the absence or unavailability of such witnesses, other (e) "Electronic Signature" refers to any distinctive mark, characteristic and/or sound in
competent evidence may be admitted. electronic form, representing the identity of a person and attached to or logically associated
with the electronic data message or electronic document or any methodology or procedures
A recording of the telephone conversation or ephemeral electronic communication shall be employed or adopted by a person and executed or adopted by such person with the intention
covered by the immediately preceding section. of authenticating or approving an electronic data message or electronic document.

If the foregoing communications are recorded or embodied in an electronic document, then (f) "Electronic Document" refers to information or the representation of information, data,
the provisions of Rule 5 shall apply. figures, symbols or other modes of written expression, described or however represented, by
which a right is established or an obligation extinguished, or by which a fact may be prove and
Rule 12 affirmed, which is receive, recorded, transmitted, stored, processed, retrieved or produced
EFFECTIVITY electronically.

Section 1. Applicability to pending cases. - These Rules shall apply to cases pending (g) "Electronic Key" refers to a secret code which secures and defends sensitive information
after their effectivity. that cross over public channels into a form decipherable only with a matching electronic key.

Section 2. Effectivity. - These Rules shall take effect on the first day of August 2001 (h) "Intermediary" refers to a person who in behalf of another person and with respect to a
following their publication before the 20th of July 2001 in two newspapers of general particular electronic document sends, receives and/or stores provides other services in respect
circulation in the Philippines. of that electronic data message or electronic document.

E-COMMERCE LAW (i) "Originator" refers to a person by whom, or on whose behalf, the electronic document
purports to have been created, generated and/or sent. The term does not include a person
REPUBLIC ACT NO. 8792 June 14, 2000 acting as an intermediary with respect to that electronic document.

AN ACT PROVIDING FOR THE RECOGNITION AND USE OF ELECTRONIC (j) "Service provider" refers to a provider of -
COMMERCIAL AND NON-COMMERCIAL TRANSACTIONS AND DOCUMENTS,
PENALTIES FOR UNLAWFUL USE THEREOF, AND FOR OTHER PURPOSES i. On-line services or network access or the operator of facilities therefor, including entities
offering the transmission, routing, or providing of connections for online communications,
Be it enacted by the Senate and House of Representatives of the Philippines in Congress digital or otherwise, between or among points specified by a user, of electronic documents of
assembled: the user's choosing; or

PART I ii. The necessary technical means by which electronic documents of an originator may be
stored and made accessible to designated or undesignated third party.
SHORT TITLE AND DECLARATION OF POLICY
Such service providers shall have no authority to modify or alter the content of the electronic
Section 1. Short Title - This Act shall be known as the "Electronic Commerce Act of 2000."
data message or electronic document received or to make any entry therein on behalf of the
237
RECTO, GAYLE ANGELI M.
2011-0008 | AUSL
Personal Notes on Remedial Law 2 Review (based on the syllabus of Prof. Henedino M. Brondial)

originator, addressee or any third party unless specifically authorized to do so, and who shall (2) Paragraph (1) applies whether the requirement therein is in the form of an obligation or
retain the electronic document in accordance with the specific request or as necessary for the whether the law simply provides consequences for the information not being presented or
purpose of performing the services it was engaged to perform. retained in its original form.

CHAPTER II (3) For the purpose of subparagraph (a) of paragraph (1):


LEGAL RECOGNITION OF ELECTRONIC WRITING
OR DOCUMENT AND DATA MESSAGES (a) the criteria for assessing integrity shall be whether the information has remained complete
and unaltered, apart from the addition of any endorsement and any change which arises in the
Section 6. Legal Recognition of Electronic Data Messages - Information shall not be normal course of communication, storage and display ; and
denied legal effect, validity or enforceability solely on the grounds that it is in the data
message purporting to give rise to such legal effect, or that it is merely referred to in that (b) the standard of reliability required shall be assessed in the light of purposed for which the
electronic data message. information was generated and in the light of all the relevant circumstances.

Section 7. Legal Recognition of Electronic Documents - Electronic documents shall have Section 11. Authentication of Electronic Data Messages and Electronic Documents.
the legal effect, validity or enforceability as any other document or legal writing, and - - Until the Supreme Court by appropriate rules shall have so provided, electronic documents,
electronic data messages and electronic signatures, shall be authenticated by demonstrating,
(a) Where the law requires a document to be in writing, that requirement is met by an substantiating and validating a claimed identity of a user, device, or another entity is an
electronic document if the said electronic document maintains its integrity and reliability and information or communication system, among other ways, as follows;
can be authenticated so as to be usable for subsequent reference, in that -
(a) The electronic signature shall be authenticated by proof than a letter , character, number
i. The electronic document has remained complete and unaltered, apart from the addition of any or other symbol in electronic form representing the persons named in and attached to or
endorsement and any authorized change, or any change which arises in the normal course of logically associated with an electronic data message, electronic document, or that the
communication, storage and display; and appropriate methodology or security procedures, when applicable, were employed or adopted
by such person, with the intention of authenticating or approving in an electronic data
ii. The electronic document is reliable in the light of the purpose for which it was generated message or electronic document;
and in the light of all relevant circumstances.
(b) The electronic data message or electronic document shall be authenticated by proof that
(b) Paragraph (a) applies whether the requirement therein is in the form of an obligation or an appropriate security procedure, when applicable was adopted and employed for the
whether the law simply provides consequences for the document not being presented or purpose of verifying the originator of an electronic data message and/or electronic document,
retained in its original from. or detecting error or alteration in the communication, content or storage of an electronic
document or electronic data message from a specific point, which, using algorithm or codes,
(c) Where the law requires that a document be presented or retained in its original form, that identifying words or numbers, encryptions, answers back or acknowledgement procedures, or
requirement is met by an electronic document if - similar security devices.

i. There exists a reliable assurance as to the integrity of the document from the time when it The supreme court may adopt such other authentication procedures, including the use of
was first generated in its final form; and electronic notarization systems as necessary and advisable, as well as the certificate of
authentication on printed or hard copies of the electronic document or electronic data
ii. That document is capable of being displayed to the person to whom it is to be presented: messages by electronic notaries, service providers and other duly recognized or appointed
Provided, That no provision of this Act shall apply to vary any and all requirements of existing certification authorities.
laws on formalities required in the execution of documents for their validity.
The person seeking to introduce an electronic data message or electronic document in any
For evidentiary purposes, an electronic document shall be the functional equivalent of a legal proceeding has the burden of proving its authenticity by evidence capable of supporting
written document under existing laws. a finding that the electronic data message or electronic document is what the person claims it
be.
This Act does not modify any statutory rule relating to admissibility of electronic data
massages or electronic documents, except the rules relating to authentication and best In the absence of evidence to the contrary, the integrity of the information and communication
evidence. system in which an electronic data message or electronic document is recorded or stored may
be established in any legal proceeding -
Section 8. Legal Recognition of Electronic Signatures. - An electronic signature on the
electronic document shall be equivalent to the signature of a person on a written document if a.) By evidence that at all material times the information and communication system or other
that signature is proved by showing that a prescribed procedure, not alterable by the parties similar device was operating in a manner that did not affect the integrity of the electronic data
interested in the electronic document, existed under which - message and/or electronic document, and there are no other reasonable grounds to doubt the
integrity of the information and communication system,
(a) A method is used to identify the party sought to be bound and to indicate said party's
access to the electronic document necessary for his consent or approval through the electronic b.) By showing that the electronic data message and/or electronic document was recorded or
signature; stored by a party to the proceedings who is adverse in interest to the party using it; or

(b) Said method is reliable and appropriate for the purpose for which the electronic document c.) By showing that the electronic data message and/or electronic document was recorded or
was generated or communicated, in the light of all circumstances, including any relevant stored in the usual and ordinary course of business by a person who is not a party to the
agreement; proceedings and who did not act under the control of the party using the record.

(c) It is necessary for the party sought to be bound, in or order to proceed further with the Section 12. Admissibility and Evidential Weight of Electronic Data Message or
transaction, to have executed or provided the electronic signature; and Electronic Document. - In any legal proceedings, nothing in the application of the rules on
evidence shall deny the admissibility of an electronic data message or electronic document in
(d) The other party is authorized and enabled to verify the electronic signature and to make evidence -
the decision to proceed with the transaction authenticated by the same.
(a) On the sole ground that it is in electronic form; or
Section 9. Presumption Relating to Electronic Signatures - In any proceedings
involving an electronic signature, it shall be presumed that - (b) On the ground that it is not in the standard written form, and the electronic data message
or electronic document meeting, and complying with the requirements under Sections 6 or 7
(a) The electronic signature is the signature of the person to whom it correlates; and hereof shall be the best evidence of the agreement and transaction contained therein.

(b) The electronic signature was affixed by that person with the intention of signing or In assessing the evidential weight of an electronic data message or electronic document, the
approving the electronic document unless the person relying on the electronically signed reliability of the manner in which it was generated, stored or communicated, the reliability of
electronic document knows or has noticed of defects in or unreliability of the signature or the manner in which its originator was identified, and other relevant factors shall be given due
reliance on the electronic signature is not reasonable under the circumstances. regard.

Section 10. Original Documents. - Section 13. Retention of Electronic Data Message or Electronic Document. -
Notwithstanding any provision of law, rule or regulation to the contrary -
(1) Where the law requires information to be presented or retained in its original form, that
requirement is met by an electronic data message or electronic document if; (a) The requirement in any provision of law that certain documents be retained in their original
form is satisfied by retaining them in the form of an electronic data message or electronic
(a) the integrity of the information from the time when it was first generated in its final form, document which -
as an electronic data message or electronic document is shown by evidence aliunde or
otherwise; and (i) Remains accessible so as to be usable for subsequent reference;

(b) where it is required that information be resented, that the information is capable of being (ii) Is retained in the format in which it was generated, sent or received, or in a format which
displayed to the person to whom it is to be presented. can be demonstrated to accurately represent the electronic data message or electronic
document generated, sent or received;

238
RECTO, GAYLE ANGELI M.
2011-0008 | AUSL
Personal Notes on Remedial Law 2 Review (based on the syllabus of Prof. Henedino M. Brondial)

(iii) Enables the identification of its originator and addressee, as well as the determination of knew or should have known, had it exercised treasonable care or used any agreed procedure,
the date and the time it was sent or received. that the transmission resulted in any error in the electronic data message or electronic
document as received.
(b) The requirement referred to in paragraph (a) is satisfied by using the services of a third
party, provided that the conditions set fourth in subparagraph s (i), (ii) and (iii) of paragraph (6) The addressee is entitled to regard each electronic data message or electronic document
(a) are met. received as a separate electronic data message or electronic document and to act on that
assumption, except to the extent that it duplicates another electronic data message or
Section 14. Proof by Affidavit. - The matters referred to in Section 12, on admissibility and electronic document and the addressee knew or should have known, had it exercised
Section 9, on the presumption of integrity, may be presumed to have been established by an reasonable care or used any agreed procedure, that the electronic data message or electronic
affidavit given to the best of the deponent's knowledge subject to the rights of parties in document was a duplicate.
interest as defined in the following section.
Section 19. Error on Electronic Data Message or Electronic Document. - The
Section 15. Cross - Examination. addressee is entitled to regard the electronic data message or electronic document received as
that which the originator intended to send, and to act on that assumption, unless the
(1) A deponent of an affidavit referred to in Section 14 that has been introduced in evidence addressee knew or should have known, had the addressee exercised reasonable care or used
may be cross-examined as of right by a party to the proceedings who is adverse in interest to the appropriate procedure -
the party who has introduced the affidavit or has caused the affidavit to be introduced.
(a) That the transmission resulted in any error therein or in the electronic document when the
(2) Any party to the proceedings has the right to cross-examine a person referred to in section electronic data message or electronic document enters the designated information system, or
11, paragraph 4, sub paragraph c.
(b) That electronic data message or electronic document is sent to an information system
CHAPTER III. which is not so designated by the addressee for the purposes.
COMMUNICATION OF ELECTRONIC DATA MESSAGES OR ELECTRONIC DOCUMENTS
Section 20. Agreement on Acknowledgement of Receipt of Electronic Data
Section 16. Formation of Validity of Electronic Contracts. Messages or Electronic Documents. - The following rules shall apply where, on or before
sending an electronic data message or electronic document, the originator and the addressee
(1) Except as otherwise agreed by the parties, an offer, the acceptance of an offer and such have agreed, or in that electronic document or electronic data message, the originator has
other elements required under existing laws for the formation of contracts may be expressed requested, that receipt of the electronic document or electronic data message be
in, demonstrated and proved by means of electronic data messages or electronic documents acknowledged:
and no contract shall be denied validity or enforceability on the sole ground that it is in the
form of an electronic data message or electronic document, or that any or all of the elements a.) Where the originator has not agreed with the addressee that the acknowledgement be given
required under existing laws for the formation of contracts is expressed, demonstrated and in a particular form or by a particular method, an acknowledgement may be given by or through
proved by means of electronic data messages or electronic documents. any communication by the addressee, automated or otherwise, or any conduct of the
addressee, sufficient to indicate to the originator that the electronic data message or electronic
(2) Electronic transactions made through networking among banks, or linkages thereof with document has been received.
other entities or networks, and vice versa, shall be deemed consummated upon the actual
dispensing of cash or the debit of one account and the corresponding credit to another, b.) Where the originator has stated that the effect or significance of the electronic data
whether such transaction is initiated by the depositor or by an authorized collecting party: message or electronic document is conditional on receipt of the acknowledgement thereof, the
Provided, that the obligation of one bank, entity, or person similarly situated to another arising electronic data message or electronic document is treated as though it has never been sent,
therefrom shall be considered absolute and shall not be subjected to the process of preference until the acknowledgement is received.
of credits.
c.) Where the originator has not stated that the effect or significance of the electronic data
Section 17. Recognition by Parties of Electronic Data Message or Electronic message or electronic document is conditional on receipt of the acknowledgement, and the
Document. - As between the originator and the addressee of an electronic data message or acknowledgement has not been received by the originator within the time specified or agreed
electronic document, a declaration of will or other statement shall not be denied legal effect, or, if no time has been specified or agreed, within the reasonable time, the originator may give
validity or enforceability solely on the ground that it is in the form of an electronic data notice to the addressee stating that no acknowledgement has been received and specifying a
message. reasonable time by which the acknowledgement must be received; and if the
acknowledgement is not received within the time specified in subparagraph (c), the originator
Section 18. Attribution of Electronic Data Message. - may, upon notice to the addressee, treat the electronic document or electronic data as though
it had never been sent, or exercise any other rights it may have.
(1) An electronic data message or electronic document is that of the originator if it was sent by
the originator himself. Section 21. Time of Dispatch of Electronic Data Messages or Electronic Documents.
- Unless otherwise agreed between the originator and the addressee, the dispatch of an
(2) As between the originator and the addressee, an electronic data message or electronic electronic data message or electronic document occurs when it enters an information system
document is deemed to be that of the originator if it was sent: outside the control of the originator or of the person who sent the electronic data message or
electronic document on behalf of the originator.
(a) by a person who had the authority to act on behalf of the originator with respect to that
electronic data message or electronic document; or Section 22. Time of Receipt of Electronic Data Messages or Electronic Documents. -
Unless otherwise agreed between the originator and the addressee, the time of receipt of an
(b) by an information system programmed by, or on behalf of the originator to operate electronic data message or electronic document is as follows:
automatically.
a.) If the addressee has designated an information system for the purpose of receiving
(3) As between the originator and the addressee, an addressee is entitled to regard an electronic data message or electronic document, receipt occurs at the time when the electronic
electronic data message or electronic document as being that of the originator, and to act on data message or electronic document enters the designated information system: Provide,
that assumption, if: however, that if the originator and the addressee are both participants in the designated
information system, receipt occurs at the time when the electronic data message or electronic
(a) in order to ascertain whether the electronic data message or electronic document was that document is retrieved by the addressee;
of the originator, the addressee properly applied a procedure previously agreed to by the
originator for that purpose; or b.) If the electronic data message or electronic document is sent to an information system of
the addressee that is not the designated information system, receipt occurs at the time when
(b) the electronic data message or electronic document as received by the addressee resulted the electronic data message or electronic document is retrieved by the addressee;
from the actions of a person whose relationship with the originator or with any agent of the
originator enabled that person to gain access to a method used by the originator to identify c.) If the addressee has not designated an information system, receipt occurs when the
electronic data messages as his own. electronic data message or electronic document enters an information system of the
addressee.
(4) Paragraph (3) does not apply:
These rules apply notwithstanding that the place where the information system is located may
(a) as of the time when the addressee has both received notice from the originator that the be different from the place where the electronic data message or electronic document is
electronic data message or electronic document is not that of the originator, and has deemed to be received.
reasonable time to act accordingly; or
Section 23. Place of Dispatch and Receipt of Electronic Data Messages or Electronic
(b) in a case within paragraph (3) sub-paragraph (b), at any time when the addressee knew or Documents. - Unless otherwise agreed between the originator and the addressee, an
should have known, had it exercised reasonable care of used any agreed procedure, that the electronic data message or electronic document is deemed to be dispatched at the place
electronic data message or electronic document was not that of the originator. where the originator has its place of business and received at the place where the addressee
has its place of business. This rule shall apply even if the originator or addressee had used a
(5) Where an electronic data message or electronic document is that of the originator or is laptop other portable device to transmit or received his electronic data message or electronic
deemed to be that of the originator, or the addressee is entitled to act on that assumption, document. This rule shall also apply to determine the tax situs of such transaction.
then, as between the originator and the addressee, the addressee is entitled to regard the
electronic data message or electronic document as received as being what the originator For the purpose hereof -
intended to send, and to act on that assumption. The addressee is not so entitled when it
239
RECTO, GAYLE ANGELI M.
2011-0008 | AUSL
Personal Notes on Remedial Law 2 Review (based on the syllabus of Prof. Henedino M. Brondial)

a. If the originator or addressee has more than one place of business, the place of business is the government, as well as all government-owned and -controlled corporations, that pursuant
that which has the closest relationship to the underlying transaction or, where there is no to law require or accept the filling of documents, require that documents be created, or
underlying transaction, the principal place of business. retained and/or submitted, issue permits, licenses or certificates of registration or approval, or
provide for the method and manner of payment or settlement of fees and other obligations to
b. If the originator or the addressee does not have a place of business, reference is to be the government, shall -
made to its habitual residence; or
(a) accept the creation, filing or retention of such documents in the form of electronic data
c. The "usual place of residence" in relation to a body corporate, means the place where it is messages or electronic documents;
incorporated or otherwise legally constituted.
(b) issue permits, licenses, or approval in the form of electronic data messages or electronic
Section 24. Choice of Security Methods. - Subject to applicable laws and /or rules and documents;
guidelines promulgated by the Department of Trade and Industry with other appropriate
government agencies, parties to any electronic transaction shall be free to determine the type of (c) require and/or accept payments, and issue receipts acknowledging such payments,
level of electronic data message and electronic document security needed, and to select and use through systems using electronic data messages or electronic documents; or
or implement appropriate technological methods that suit their need.
(d) transact the government business and/or perform governmental functions using electronic
PART III data messages or electronic documents, and for the purpose, are authorized to adopt and
ELECTRONIC COMMERCE IN SPECIFIC AREAS promulgate, after appropriate public hearing and with due publication in newspapers of
general circulation, the appropriate rules, regulations, or guidelines, to, among others, specify
CHAPTER I. -
CARRIAGE OF GOODS
1) the manner and format in which such electronic data messages or electronic documents
Section 25. Actions Related to Contracts of Carriage of Goods. - Without derogating shall be filed, created, retained or issued;
from the provisions of part two of this law, this chapter applies to any action in connection
with, or in pursuance of, a contract of carriage of goods, including but not limited to: 2) where and when such electronic data messages or electronic documents have to signed,
the use of an electronic signature, the type of electronic signature required;
(a) (i) furnishing the marks, number, quantity or weight of goods;
3) the format of an electronic data message or electronic document and the manner the
(ii) stating or declaring the nature or value of goods; electronic signature shall be affixed to the electronic data message or electronic document;

(iii) issuing a receipt for goods; 4) the control processes and procedures as appropriate to ensure adequate integrity, security
and confidentiality of electronic data messages or electronic documents or records of
(iv) confirming that goods have been loaded; payments;

(b) (i) notifying a person of terms and conditions of the contract; 5) other attributes required to electronic data messages or electronic documents or payments;
and
(ii) giving instructions to a carrier;
6) the full or limited use of the documents and papers for compliance with the government
(c) (i) claiming delivery of goods; requirements: Provided, that this Act shall be itself mandate any department of the
government, organ of state or statutory corporation to accept or issue any document in the
(ii) authorizing release of goods; form of electronic data messages or electronic documents upon the adoption, promulgation
and publication of the appropriate rules, regulations or guidelines.
(iii) giving notice of loss of, or damage to goods;
Section 28. RPWEB To Promote the Use of Electronic Documents or Electronic Data
(d) giving any other notice or statement in connection with the performance of the contract; Messages In Government and to the General Public. - Within two (2) years from the
effectivity of this Act, there shall be installed an electronic online network in accordance with
(e) undertaking to deliver goods to a named person or a person authorized to claim delivery; Administrative Order 332 and House of Representatives Resolution 890, otherwise known as
RPWEB, to implement Part IV of this Act to facilitate the open, speedy and efficient electronic
(f) granting, acquiring, renouncing, surrendering, transferring or negotiating rights in goods; online transmission, conveyance and use of electronic data messages or electronic documents
amongst all government departments, agencies, bureaus, offices down to the division level
(g) acquiring or transferring rights and obligations under the contract. and to the regional and provincial offices as practicable as possible, government owned and
controlled corporations, local government units, other public instrumentalities, universities,
Section 26. Transport Documents. - (1) Where the law requires that any action referred to colleges and other schools, and universal access to the general public.
contract of carriage of goods be carried out in writing or by using a paper document, that
requirement is met if the action is carried out by using one or more data messages or The RPWEB network shall serve as initial platform of the government information
electronic documents. infrastructure (GII) to facilitate the electronic online transmission and conveyance of
government services to evolve and improve by better technologies or kinds and electronic
(2) Paragraph (1) applies whether the requirement there in is in the form of an obligation or online wide area networks utilizing, but not limited to, fiber optic, satellite, wireless and other
whether the law simply provides consequences for failing either to carry out the action in broadband telecommunication mediums or modes.
writing or to use a paper document.
To facilitate the rapid development of the GII, the Department of Transportation and
(3) If a right is to be granted to, or an obligation is to be acquired by, one person and no Communications, National Telecommunications Commission and the National Computer Center
person, and if the law requires that, in order to effect this, the right or obligation must be are hereby directed to aggressively promote and implement a policy environment and
conveyed to that person by the transfer, or use of, a paper document, that requirement is met regulatory framework that shall lead to the substantial reduction of costs of including, but not
if the right or obligation is conveyed by using one or more electronic data messages or limited to, lease lines, land, satellite and dial-up telephone access, cheap broadband and
electronic documents unique; wireless accessibility by government departments, agencies, bureaus, offices, government
owned and controlled corporations, local government units, other public instrumentalities and
(4) For the purposes of paragraph (3), the standard of reliability required shall be assessed in the general public, to include the establishment of a government website portal and a
the light of the purpose for which the right or obligation was conveyed and in the light of all domestic internal exchange system to facilitate strategic access to government and amongst
the circumstances, including any relevant agreement. agencies thereof and the general public and for the speedier flow of locally generated internal
traffic within the Philippines.
(5) Where one or more data messages are used to effect any action in subparagraphs (f) and
(g) of Section 25, no paper document used to effect any such action is valid unless the use of The physical infrastructure of cable and wireless system for cable TV and broadcast excluding
electronic data message or electronic document has been terminated and replaced by the used programming content and the management thereof shall be considered as within the activity of
of paper documents. A paper document issued in these circumstances shall contain a telecommunications for the purpose of electronic commerce and to maximize the
statement of such termination. The replacement of the electronic data messages or electronic convergence of ICT in the installation of the GII.
documents by paper documents shall not affect the rights or obligation of the parties involved.
Section 29. Authority of the Department of Trade and Industry and Participating
(6) If a rule of laws is compulsorily applicable to a contract of carriage of goods which is in, or Entities. - The Department of Trade and Industry (DTI) shall direct supervise the promotion
is evidenced by, a paper document, that rule shall not be inapplicable to such a contract of and development of electronic commerce in the country with relevant government agencies,
carriage of goods which is evidenced by one or more electronic data messages or electronic without prejudice to the provisions of Republic Act 7653 (Charter of Bangko Sentral ng
documents by reason of the fact that the contract is evidenced by such electronic data Pilipinas) and Republic Act No. 337, (General Banking Act) as amended.
messages or electronic documents instead of by a paper document.
Among others, the DTI is empowered to promulgate rules and regulations, as well as provide
PART IV quality standards or issue certifications, as the case may be, and perform such other functions
ELECTRONIC TRANSACTIONS IN GOVERNMENT as may be necessary for the implementation of this Act in the area of electronic commerce to
include, but shall not limited to, the installation of an online public information and quality and
Section 27. Government Use of Electronic Data Messages, Electronic Documents price monitoring system for goods and services aimed in protecting the interests of the
and Electronic Signatures. - Notwithstanding any law to the contrary, within two (2) years consuming public availing of the advantages of this Act.
from the date of the effectivity of this Act, all departments, bureaus, offices and agencies of
240
RECTO, GAYLE ANGELI M.
2011-0008 | AUSL
Personal Notes on Remedial Law 2 Review (based on the syllabus of Prof. Henedino M. Brondial)

Commission, National Computer Center, National Information Technology Council, Commission


on Audit, other concerned agencies and the private sector, to implement this Act within sixty
(60) days after its approval.
PART V
FINAL PROVISIONS Failure to Issue rules and regulations shall not in any manner affect the executory nature of
the provisions of this Act.
Section 30. Extent of Liability of a Service Provider. - Except as otherwise provided in this
Section, no person or party shall be subject to any civil or criminal liability in respect of the Section 35. Oversight Committee. - There shall be Congressional Oversight Committee
electronic data message or electronic document for which the person or party acting as a composed of the Committees and Trade and Industry/Commerce, Science and Technology,
service provider as defined in Section 5 merely provides access if such liability is founded on - Finance and Appropriations of both the Senate and House of Representatives, which shall
meet at least every quarter of the first two years and every semester for the third year after
(a) The obligations and liabilities of the parties under the electronic data message or electronic the approval of this Act to oversee its implementation. The DTI, DBM, Bangko Sentral ng
document; Pilipinas, and other government agencies as may be determined by the Congressional
Committee shall provide a quarterly performance report of their actions taking in the
(b) The making, publication, dissemination or distribution of such material or any statement implementation of this Act for the first three (3) years.
made in such material, including possible infringement of any right subsisting in or in relation
to such material. Provided, That: Section 36. Appropriations. - The amount necessary to carry out the provisions of Sections
27 and 28 of this Act shall be charged against any available funds and/or savings under the
i. The service provider does not have actual knowledge, or is not aware of the facts or General Appropriations Act of 2000 in the first year of effectivity of this Act. Thereafter, the
circumstances from which it is apparent, that the making, publication, dissemination or funds needed for the continued implementation shall be included in the annual General
distribution of such material is unlawful or infringes any rights subsisting in or in relation to Appropriations Act.
such material;
Section 37. Statutory Interpretation. - Unless otherwise expressly provided for, the
ii The service provider does not knowingly receive a financial benefit directly attributable to the interpretation of this Act shall give due regard to its international origin and the need to
unlawful or infringing activity; and promote uniformity in its application and the observance of good faith in international trade
relations. The generally accepted principles of international law and convention on electronic
iii. The service provider does not directly commit any infringement or other unlawful act and commerce shall likewise be considered.
does not induce or cause another person or party to commit any infringement or other
unlawful act and/or does not benefit financially from the infringing activity or unlawful act or Section 38. Variation by Agreement. - As between parties involved in generating, sending,
another person or party; Provider, further, That nothing in this Section shall affect - receiving, storing or otherwise processing electronic data message or electronic document, any
provision of this Act may be varied by agreement between and among them.
(a) Any obligation founded on contract;
Section 39. Reciprocity. - All benefits, privileges, advantages or statutory rules established
(b) The obligation of a service provider as such under a licensing or other regulatory regime under this Act, including those involving practice of profession, shall be enjoyed only by parties
established under written law; or whose country origin grants the same benefits and privileges or advantages to Filipino citizens.

(c) Any obligation imposed under any written law; Section 40. Separability Clause. - The provisions of this Act are hereby declared separable
and in the event of any such provision is declared unconstitutional, the other provisions, which
(d) The civil liability of any party to the extent that such liability forms the basis for injunctive are not affected, shall remain in force and effect.
relief issued by a court under any law requiring that the service provider take or refrain from
actions necessary to remove, block or deny access to any material, or to preserve evidence of Section 41. Repealing Clause. - All other laws, decrees, rules and regulations or parts
a violation of law. thereof which are inconsistent with the provisions of this Act are hereby repealed, amended or
modified accordingly.
Section 31. Lawful Access. - Access to an electronic file, or an electronic signature of an
electronic data message or electronic document shall only be authorized and enforced in favor Section 42. Effectivity. - This Act shall take effect immediately after its publication in the
of the individual or entity having a legal right to the possession or the use of plaintext, Official Gazette or in at least two (2) national newspapers of general circulation.
electronic signature or file or solely for the authorized purposes. The electronic key for identity
or integrity shall not be made available to any person or party without the consent of the Approved:
individual or entity in lawful possession of that electronic key;
June 14, 2000
Section 32. Obligation of Confidentiality. - Except for the purposes authorized under this CASES
Act, any person who obtained access to any electronic key, electronic data message or
electronic document, book, register, correspondence, information, or other material pursuant
HEIRS OF LOURDES SAEZ SABANPAN: BERNARDO S. SABANPAN, RENE S. SABANPAN,
to any powers conferred under this Act, shall not convey to or share the same with any other
DANILO S. SABANPAN and THELMA S. CHU; HEIRS OF ADOLFO SAEZ: MA. LUISA SAEZ
person.
TAPIZ, MA. VICTORIA SAEZ LAPITAN, MA. BELEN SAEZ and EMMANUEL SAEZ; and HEIRS OF
CRISTINA SAEZ GUTIERREZ: ROY SAEZ GUTIERREZ and LUIS SAEZ JR., petitioners, vs.
Section 33. Penalties. - The following Acts, shall be penalized by fine and/or imprisonment,
ALBERTO C. COMORPOSA, HERDIN C. COMORPOSA, OFELIA C. ARIEGO,1 REMEDIOS
as follows:
COMORPOSA, VIRGILIO A. LARIEGO,1a BELINDA M. COMORPOSA and ISABELITA H.
COMORPOSA, respondents.
(a) Hacking or crackling with refers to unauthorized access into or interference in a computer
G.R. No. 152807 August 12, 2003
system/server or information and communication system; or any access in order to corrupt,
THIRD DIVISION
alter, steal, or destroy using a computer or other similar information and communication
devices, without the knowledge and consent of the owner of the computer or information and
FACTS:
communications system, including the introduction of computer viruses and the like, resulting in
x Sabanpans Æ filed an unlawful detainer case against the Comoporsas before the
the corruption, destruction, alteration, theft or loss of electronic data messages or electronic
MTC Davao Del Sur
documents shall be punished by a minimum fine of One Hundred Thousand pesos (P
o alleged that Marcos Saez was the lawful and actual possessor of
100,000.00) and a maximum commensurate to the damage incurred and a mandatory
Lot No. 845, Land 275 located at Darong, Sta. Cruz, Davao del Sur
imprisonment of six (6) months to three (3) years;
with an area of 1.2 hectares. In 1960, he died leaving all his heirs,
his children and grandchildren.
(b) Piracy or the unauthorized copying, reproduction, dissemination, or distribution,
o Francisco Comorposa who was working in the land of Oboza was
importation, use, removal, alteration, substitution, modification, storage, uploading,
terminated from his job. The termination of his employment caused a
downloading, communication, making available to the public, or broadcasting of protected
problem in relocating his house. Being a close family friend of
material, electronic signature or copyrighted works including legally protected sound
[Marcos] Saez, Francisco Comorposa approached the late Marcos
recordings or phonograms or information material on protected works, through the use of
Saez's son, [Adolfo] Saez, the husband of Gloria Leano Saez, about
telecommunication networks, such as, but not limited to, the internet, in a manner that
his problem. Out of pity and for humanitarian consideration, Adolfo
infringes intellectual property rights shall be punished by a minimum fine of one hundred
allowed Francisco Comorposa to occupy the land of Marcos Saez.
thousand pesos (P 100,000.00) and a maximum commensurate to the damage incurred and a
Hence, his nipa hut was carried by his neighbors and transferred to
mandatory imprisonment of six (6) months to three (3) years;
a portion of the land subject matter of this case. Such transfer was
witnessed by several people, among them, Gloria Leano and Noel
(c) Violations of the Consumer Act of Republic Act No. 7394 and other relevant to pertinent
Oboza. Francisco Comorposa occupied a portion of Marcos Saez'
laws through transaction covered by or using electronic data messages or electronic
property without paying any rental.
documents, shall be penalized with the same penalties as provided in those laws;
o Francisco Comorposa left for Hawaii, U.S.A. He was succeeded in
his possession by the respondents who likewise did not pay any
(d) Other violations of the provisions of this Act, shall be penalized with a maximum penalty of
rental and are occupying the premises through petitioners'
one million pesos (P 1,000,000.00) or six (6) years imprisonment.
tolerance.
o a formal demand was made upon the respondents to vacate the
Section 34. Implementing Rules and Regulations. - The DTI, Department of Budget and
premises but the latter refused to vacate the same and claimed
Management and the Bangko Sentral ng Pilipinas are hereby empowered to enforced the
that they [were] the legitimate claimants and the actual and lawful
provisions of this Act and issue implementing rules and regulations necessary, in coordination
possessor[s] of the premises. A [C]omplaint was filed with the
with the Department of Transportation and Communications, National Telecommunications
241
RECTO, GAYLE ANGELI M.
2011-0008 | AUSL
Personal Notes on Remedial Law 2 Review (based on the syllabus of Prof. Henedino M. Brondial)

barangay office of Sta. Cruz[,] Davao del Sur, but the parties failed FACTS:
to arrive at an amicable settlement. Thus, the corresponding x Torres was a Slot Machine Operations Supervisor (SMOS) of respondent
Certificate to File Action was issued by the said barangay and an Philippine Amusement and Gaming Corporation (PAGCOR)
action for unlawful detainer was filed by petitioners against x Torres was among those subjected to an investigation by PAGCOR after an
respondents. alleged intelligence report of padding of the Credit Meter Readings (CMR) of the
x Comorposas Æ filed their answer slot machines at PAGCOR-Hyatt Manila, then Casino Filipino-Hyatt (CF Hyatt)
o denied the material allegations of the [C]omplaint and alleged that x PAGCOR's Corporate Investigation Unit (CIU) [after conducting the said
they entered and occupied the premises in their own right as true, investigation] Æ served Torres a Memorandum of Charges7 for dishonesty,
valid and lawful claimants, possessors and owners of the said lot serious misconduct, fraud and violation of office rules and regulations which
way back in 1960 and up to the present time; that they have were considered grave offenses where the penalty imposable is dismissal.
acquired just and valid ownership and possession of the premises x Torres Æ wrote Manager Bangsil a letter explanation/refutation10 of the charges
by ordinary or extraordinary prescription, and that the Regional against him.
Director of the DENR, Region XI has already upheld their o He denied any involvement or participation in any fraudulent
possession over the land in question when it ruled that they [were] manipulation of the CMR or padding of the slot machine receipts,
the rightful claimants and possessors and [were], therefore, and he asked for a formal investigation of the accusations against
entitled to the issuance of a title. him.
x MTC Æ in favor of the SABANPANS; ordered the Comorposas to vacate x Torres Æ then received a letter from Atty. Lizette F. Mortel, Managing Head of
x Comorposas Æ appealed to RTC PAGCOR's Human Resource and Development Department, dismissing him from
x RTC Æ reversed MTC the service
x Sabanpans Æ appealed to CA x Torres Æ filed a complaint before the CSC for illegal dismissal, non-payment of
x CA Æ affirmed RTC backwages and other benefits
o the CA upheld the right of respondents as claimants and o (1) that he denied all the charges against him;
possessors. o (2) that he did ask for a formal investigation of the accusations
o The appellate court held that -- although not yet final -- the Order against him and for PAGCOR to produce evidence and proofs to
issued by the regional executive director of the Department of substantiate the charges, but respondent PAGCOR did not call for
Environment and Natural Resources (DENR) remained in full force any formal administrative hearing;
and effect, unless declared null and void. o (3) that he tried to persuade respondent PAGCOR to review and
o The CA added that the Certification issued by the DENR's reverse its decision in a letter of reconsideration dated August 13,
community environment and natural resources (CENR) officer was 2007 addressed to the Chairman, the members of the Board of
proof that when the cadastral survey was conducted, the land was Directors and the Merit Systems Protection Board; and
still alienable and was not yet allocated to any person. o (4) that no resolution was issued on his letter reconsideration, thus,
o According to the CA, respondents had the better right to possess the filing of the complaint.
alienable and disposable land of the public domain, because they ƒ Petitioner claimed that as a result of his unlawful,
have sufficiently proven their actual, physical, open, notorious, unjustified and illegal termination/dismissal, he was
exclusive, continuous and uninterrupted possession thereof since compelled to hire the services of a counsel in order to
1960. The appellate court deemed as self-serving, and therefore protect his rights.
incredible, the Affidavits executed by Gloria Leano Saez, Noel x PAGCOR Æ filed its comment
Oboza and Paulina Paran. o petitioner failed to perfect an appeal within the period and manner
x Sabanpans Æ filed a Rule 45 before the SC provided by the Uniform Rules on Administrative Cases in the Civil
o Petitioners contend that the CENR Certification dated July 22, 1997 Service Law.
is a sham document, because the signature of the CENR officer is a x CSC Æ treated the complaint as an APPEAL of PAGCOR’s decision in dismissing
mere facsimile. In support of their argument, they cite Garvida v. Torres; RULED AGAINST TORRES
Sales Jr.17 and argue that the Certification is a new matter being o CSC found that the issue for resolution was whether petitioner's
raised by respondents for the first time on appeal. appeal had already prescribed which the former answered in the
positive.
ISSUE: Whether the subject CENR Certification was correctly admitted by the RTC, considering o The CSC did not give credit to petitioner's claim that he sent a
that it was a mere facsimile. facsimile transmission of his letter reconsideration within the period
prescribed by the Uniform Rules on Administrative Cases in the Civil
HELD: YES. Service.
x In Garvida, the Court held: o It found PAGCOR's denial of having received petitioner's letter more
o "A facsimile or fax transmission is a process involving the credible as it was supported by certifications issued by its
transmission and reproduction of printed and graphic employees.
matter by scanning an original copy, one elemental area at o It found that a verification of one of the telephone numbers where
a time, and representing the shade or tone of each area by a petitioner allegedly sent his letter reconsideration disclosed that such
specified amount of electric current. x x x"18 number did not belong to the PAGCOR's Office of the Board of
x Pleadings filed via fax machines are not considered originals and are at best Directors; and that petitioner should have mentioned about the
exact copies. As such, they are not admissible in evidence, as there is no way of alleged facsimile transmission at the first instance when he filed his
determining whether they are genuine or authentic.19 complaint and not only when respondent PAGCOR raised the issue of
x The Certification, on the other hand, is being contested for bearing a facsimile of prescription in its Comment.
the signature of CENR Officer Jose F. Tagorda. The facsimile referred to is not x Torres Æ filed an MR
the same as that which is alluded to in Garvida. The one mentioned here o CSC Æ denied
refers to a facsimile signature, which is defined as a signature x Torres Æ filed a Rule 43 before the CA
produced by mechanical means but recognized as valid in banking, x CA Æ affirmed CSC
financial, and business transactions.20 o CA found that petitioner failed to adduce clear and convincing
x Note that the CENR officer has not disclaimed the Certification. In fact, evidence that he had filed a motion for reconsideration.
the DENR regional director has acknowledged and used it as reference in his o It found insufficient to merit consideration petitioner's claim that he
Order dated April 2, 1998: had sent through a facsimile transmission a letter/reconsideration
o "x x x. CENR Officer Jose F. Tagorda, in a 'CERTIFICATION' dated dated August 13, 2007 addressed to PAGCOR's Chairman, members
22 July 1997, certified among others, that: x x x per records of the Board of Directors and the Merit Systems Protection Board;
available in his Office, x x x the controverted lot x x x was not that assuming arguendo that a letter reconsideration was indeed
allocated to any person x x x."21 sent through a facsimile transmission, such facsimile transmission is
x If the Certification were a sham as petitioner claims, then the regional director inadmissible as electronic evidence under the Electronic Commerce
would not have used it as reference in his Order. Instead, he would have either Act of 2000; and that a review of the CSC assailed resolution
verified it or directed the CENR officer to take the appropriate action, as the revealed that the telephone numbers where petitioner claimed to
latter was under the former's direct control and supervision. be the recipient of the faxed document sent was not that of
x Petitioners' claim that the Certification was raised for the first time on appeal is PAGCOR's Office of Board of Directors.
incorrect. As early as the pretrial conference at the Municipal Trial Court (MTC), o The CA found baseless and conjectural petitioner's claim that
the CENR Certification had already been marked as evidence for respondents as PAGCOR can easily deny having received the letter by giving orders
stated in the Pre-trial Order.22 The Certification was not formally offered, to their employees to execute an affidavit of denial under pain and
however, because respondents had not been able to file their position paper. threat of administrative sanction or termination from service.
x Neither the rules of procedure23 nor jurisprudence24 would sanction the x Torres Æ filed an MR
admission of evidence that has not been formally offered during the trial. But this o CA Æ denied
evidentiary rule is applicable only to ordinary trials, not to cases covered by the x Torres Æ filed a Rule 45 before the SC
rule on summary procedure -- cases in which no full-blown trial is held. o Petitioner contends that he filed his letter reconsideration of his
dismissal16 on August 13, 2007, which was within the 15-day
ELLERY MARCH G. TORRES, Petitioner, vs. PHILIPPINE AMUSEMENT and GAMING period for filing the same; and that he did so by means of a
CORPORATION, represented by ATTY. CARLOS R. BAUTISTA, JR., Respondent. facsimile transmission sent to the PAGCOR's Office of the Board of
G.R. No. 193531 December 14, 2011 Directors. He claims that the sending of documents thru electronic
EN BANC data message, which includes facsimile, is sanctioned under
Republic Act No. 8792, the Electronic Commerce Act of 2000.
242
RECTO, GAYLE ANGELI M.
2011-0008 | AUSL
Personal Notes on Remedial Law 2 Review (based on the syllabus of Prof. Henedino M. Brondial)

Petitioner further contends that since his letter reconsideration was complaint against PAGCOR and its Chairman for illegal dismissal, non-payment of
not acted upon by PAGCOR, he then filed his complaint before the backwages and other benefits on September 14, 2007. The CSC treated the
CSC. complaint as an appeal from the PAGCOR's dismissal of petitioner. Under Section
43 which we earlier quoted, petitioner had 15 days from receipt of the letter of
ISSUE: Whether the letter of reconsideration allegedly sent by Torres through facsimile dismissal to file his appeal. However, at the time petitioner filed his complaint
transmission, assuming that such was in fact sent, may be considered as a pleading which with the CSC, which was considered as petitioner's appeal, 41 days had already
necessarily tolls the period to appeal to the CSC. elapsed from the time he received his letter of dismissal on August 4, 2007;
hence, the CSC correctly found that it has no jurisdiction to entertain the appeal
HELD: NO. since petitioner's dismissal had already attained finality. Petitioner's dismissal
x Sections 37, 38, 39, and 43 of the Revised Uniform Rules on Administrative from the service became final and executory after he failed to file his motion for
Cases in the Civil Service, which are applicable to this case, respectively provide, reconsideration or appeal in the manner and within the period provided for under
to wit: the Revised Uniform Rules on Administrative Cases in the Civil Service.
o Section 37. Finality of Decisions - A decision rendered by heads of x In Peña v. Government Service and Insurance System,21 We said:
agencies whereby a penalty of suspension for not more than thirty o Noteworthy is that the right to appeal is neither a natural right nor
days or a fine in an amount not exceeding thirty (30) days' salary is a part of due process, except where it is granted by statute in
imposed, shall be final and executory. However, if the penalty which case it should be exercised in the manner and in accordance
imposed is suspension exceeding thirty days, or fine in an amount with the provisions of law. In other words, appeal is a right of
exceeding thirty days’ salary, the same shall be final and executory statutory and not of constitutional origin. The perfection of an
after the lapse of the reglementary period for filing a motion for appeal in the manner and within the period prescribed by law is not
reconsideration or an appeal and no such pleading has been filed. only mandatory but also jurisdictional and the failure of a party to
o Section 38. Filing of motion for reconsideration. - The party conform to the rules regarding appeal will render the judgment
adversely affected by the decision may file a motion for final and executory and, hence, unappealable, for it is more
reconsideration with the disciplining authority who rendered the important that a case be settled than it be settled right.
same within fifteen days from receipt thereof. Furthermore, it is axiomatic that final and executory judgments can
o Section 39. When deemed filed. - A motion for reconsideration sent no longer be attacked by any of the parties or be modified, directly
by mail shall be deemed filed on the date shown by the postmark or indirectly, even by the highest court of the land. Just as the
on the envelope which shall be attached to the records of the case losing party has the right to file an appeal within the prescribed
and in case of personal delivery, the date stamped thereon by the period, so also the winning party has the correlative right to enjoy
proper office. the finality of the resolution of the case.
o Section 43. Filing of Appeals. - Decisions of heads of departments,
agencies, provinces, cities, municipalities and other
instrumentalities imposing a penalty exceeding thirty (30) days RUSTAN ANG y PASCUA, Petitioner vs. THE HONORABLE COURT OF APPEALS and
suspension or fine in an amount exceeding thirty (30) days’ salary, IRISH SAGUD, Respondents
maybe appealed to the Commission Proper within a period of fifteen G.R. No. 182835; April 20, 2010
(15) days from receipt thereof. SECOND DIVISION
x Clearly, a motion for reconsideration may either be filed by mail or personal
delivery. When a motion for reconsideration was sent by mail, the same shall be FACTS:
deemed filed on the date shown by the postmark on the envelope which shall be x Sagud, former girlfriend of herein Ang, broke up with the latter after finding out
attached to the records of the case. On the other hand, in case of personal that petitioner took in a live-in partner, who then became his wife and
delivery, the motion is deemed filed on the date stamped thereon by the proper impregnated the same.
office. And the movant has 15 days from receipt of the decision within which to x After several requests from Ang through SMS, using 2 different cellphone
file a motion for reconsideration or an appeal therefrom. numbers, Sagud refused to reconcile with him.
x Petitioner received a copy of the letter/notice of dismissal on August 4, 2007; x Ang then allegedly sent to Sagud, using one of the cellphone numbers he used in
thus, the motion for reconsideration should have been submitted either by mail sending SMS to the latter, a pornographic picture, in the form of MMS, of a
or by personal delivery on or before August 19, 2007. However, records do not naked woman with spread legs, with respondent’s face superimposed on the
show that petitioner had filed his motion for reconsideration. In fact, the CSC same, making it appear that it was respondent on the said picture.
found that the non-receipt of petitioner's letter reconsideration was duly x Ang, through SMS, further threatened Sagud that he will be posting said picture
supported by certifications issued by PAGCOR employees. online.
x Even assuming arguendo that petitioner indeed submitted a letter x Sagud asked Ang if he could meet her, to which petitioner agreed.
reconsideration which he claims was sent through a facsimile transmission, such x Police officers then arrested Angand seized his cellphone and SIM cards.
letter reconsideration did not toll the period to appeal. The mode used by x Ang was then charged with violation of the Anti-Violence Against Women and
petitioner in filing his reconsideration is not sanctioned by the Uniform Their Children Act or Republic Act (R.A.) 9262 before the RTC Baler, Aurora
Rules on Administrative Cases in the Civil Service. As we stated earlier, x Ang’s evidence:
the motion for reconsideration may be filed only in two ways, either by mail or o As his defense, petitioner contended that he met with respondent
personal delivery. to help the latter in identifying a text message prankster and,
x In Garvida v. Sales, Jr.,17 we found inadmissible in evidence the filing of pretending to be the respondent and using her cellphone number,
pleadings through fax machines and ruled that: contacted the said prankster.
o A facsimile or fax transmission is a process involving the o Petitioner maintained that he received obscene messages and the
transmission and reproduction of printed and graphic said pornographic picture from said prankster, which he forwarded
matter by scanning an original copy, one elemental area at to the respondent.
a time, and representing the shade or tone of each area by o This, according to petitioner, explains why such messages and
a specified amount of electric current. The current is picture appeared to be coming from his cellphone number.
transmitted as a signal over regular telephone lines or via x RTC Æ CONVICTED Ang
microwave relay and is used by the receiver to reproduce x Ang Æ appealed to CA
an image of the elemental area in the proper position and x CA Æ affirmed
the correct shade. The receiver is equipped with a stylus or
other device that produces a printed record on paper ISSUE # 1: Whether the RTC was correct in admitting in evidence the pornographic picture for
referred to as a facsimile. the purpose of proving petitioner’s guilt in violating RA 9262, “Anti-Violence Against Women and
o x x x A facsimile is not a genuine and authentic pleading. It Their Children Act”.
is, at best, an exact copy preserving all the marks of an
original. Without the original, there is no way of determining on HELD # 1: YES.
its face whether the facsimile pleading is genuine and authentic x Rustan argues that, since he was arrested and certain items were seized from
and was originally signed by the party and his counsel. It may, in him without any warrant, the evidence presented against him should be deemed
fact, be a sham pleading. x x x181avvphi1 inadmissible. But the fact is that the prosecution did not present in evidence
x Moreover, a facsimile transmission is not considered as an electronic either the cellphone or the SIM cards that the police officers seized from him at
evidence under the Electronic Commerce Act. In MCC Industrial Sales the time of his arrest. The prosecution did not need such items to prove its case.
Corporation v. Ssangyong Corporation,19 We determined the question of Exhibit C for the prosecution was but a photograph depicting the Sony Ericsson
whether the original facsimile transmissions are "electronic data messages" or P900 cellphone that was used, which cellphone Rustan admitted owning during
"electronic documents" within the context of the Electronic Commerce Act, and the pre-trial conference.
We said: o Actually, though, the bulk of the evidence against him consisted in
o We, therefore, conclude that the terms "electronic data Irish’s testimony that she received the obscene picture and
message" and "electronic document," as defined under the malicious text messages that the sender’s cellphone numbers
Electronic Commerce Act of 2000, do not include a facsimile belonged to Rustan with whom she had been previously in
transmission. Accordingly, a facsimile transmission cannot communication. Indeed, to prove that the cellphone numbers
be considered as electronic evidence. It is not the belonged to Rustan, Irish and the police used such
functional equivalent of an original under the Best numbers to summon him to come to Lorentess Resort and
Evidence Rule and is not admissible as electronic evidence. he did.Consequently, the prosecution did not have to
(Italics ours.)20 present the confiscated cellphone and SIM cards to prove
x We, therefore, found no reversible error committed by the CA when it affirmed that Rustan sent those messages.
the CSC in dismissing petitioner's appeal. Petitioner filed with the CSC a
243
RECTO, GAYLE ANGELI M.
2011-0008 | AUSL
Personal Notes on Remedial Law 2 Review (based on the syllabus of Prof. Henedino M. Brondial)

o Moreover, Rustan admitted having sent the malicious text an entrapment operation that resulted in the arrest of accused
messages to Irish. His defense was that he himself received those Santos and Jalandoni. Subsequently, the police were also able to
messages from an unidentified person who was harassing Irish and capture accused Enojas and Gomez. The prosecution presented the
he merely forwarded the same to her, using his cellphone. But transcripts of the mobile phone text messages between Enojas and
Rustan never presented the cellphone number of the unidentified some of his co-accused.5
person who sent the messages to him to authenticate the same. o The victim’s father, Ricardo Pangilinan, testified that his son was at
The RTC did not give credence to such version and neither will this the time of his death 28 years old, unmarried, and was receiving
Court. Besides, it was most unlikely for Irish to pin the things on police pay of P8,000.00 to P10,000.00 per month. Ricardo spent
Rustan if he had merely tried to help her identify the sender. P99,999 for burial expense, P16,000.00 for the interment services,
and P50,000.00 for purchase of the cemetery lot
x Defense evidence:
ISSUE # 2: Whether there is a need to authenticate the subject text messages, the same o Manifesting in open court that they did not want to adduce any
constituting electronic documents, by means of an electronic signature. evidence or testify in the case,7 the accused opted to instead file a
trial memorandum on March 10, 2008 for their defense.
HELD: NO. o They pointed out that they were entitled to an acquittal since they
x Rustan claims that the obscene picture sent to Irish through a text message were all illegally arrested and since the evidence of the text
constitutes an electronic document. Thus, it should be authenticated by means of messages were inadmissible, not having been properly identified.
an electronic signature, as provided under Section 1, Rule 5 of the Rules on x RTC Æ CONVICTED the ENOJAS ET AL
Electronic Evidence (A.M. 01-7-01-SC). x Enojas et al Æ appealed to CA
o But, firstly, Rustan is raising this objection to the x CA Æ affirmed RTC
admissibility of the obscene picture, Exhibit A, for the first x Enojas et al Æ filed a Rue 45 before the SC
time before this Court. The objection is too late since he should
have objected to the admission of the picture on such ground at the ISSUE: Whether the RTC correctly admitted the text messages in evidence.
time it was offered in evidence. He should be deemed to have
already waived such ground for objection. HELD: YES.
o Besides, the rules he cites do not apply to the present x As to the admissibility of the text messages, the RTC admitted them in
criminal action. The Rules on Electronic Evidence applies conformity with the Court’s earlier Resolution applying the Rules on
only to civil actions, quasi-judicial proceedings, and Electronic Evidence to criminal actions.15 Text messages are to be
administrative proceedings. (Author’s note: relate this ruling to proved by the testimony of a person who was a party to the same or
the 2003 amendment on The Rules on Electronic Evidence, which has personal knowledge of them.16 Here, PO3 Cambi, posing as the accused
added “criminal proceedings” to the cases where said Rules are to Enojas, exchanged text messages with the other accused in order to identify and
be made applicable, AND the recent Enojas case which finally entrap them. As the recipient of those messages sent from and to the mobile
abandoned this obiter dictum in Ang) phone in his possession, PO3 Cambi had personal knowledge of such messages
and was competent to testify on them.
x The accused lament that they were arrested without a valid warrant of
PEOPLE OF THE PHILIPPINES, Appellee, vs. NOEL ENOJAS y HINGPIT, ARNOLD GOMEZ arrest.1âwphi1 But, assuming that this was so, it cannot be a ground for
y FABREGAS, FERNANDO SANTOS y DELANTAR, and ROGER JALANDONI y ARI, Appellants. acquitting them of the crime charged but for rejecting any evidence that may have
G.R. No. 204894 March 10, 2014 been taken from them after an unauthorized search as an incident of an unlawful
THIRD DIVISION arrest, a point that is not in issue here. At any rate, a crime had been committed—
the killing of PO2 Pangilinan—and the investigating police officers had personal
FACTS: knowledge of facts indicating that the persons they were to arrest had committed
x Noel Enojas y Hingpit (Enojas), Arnold Gomez y Fabregas (Gomez), Fernando it.17 The text messages to and from the mobile phone left at the scene
Santos y Delantar (Santos), and Roger Jalandoni y Ari (Jalandoni) were charged by accused Enojas provided strong leads on the
with MURDER before the RTC Las Pinas participation and identities of the accused. Indeed, the police caught
x Prosecution evidence: them in an entrapment using this knowledge.
o PO2 Eduardo Gregorio, Jr. (P02 Gregorio) testified that at around
10:30 in the evening of August 29, 2006, he and P02 Francisco 3. Testimonial Evidence
Pangilinan (PO2 Pangilinan) were patrolling the vicinity of Toyota
Alabang and SM Southmall when they spotted a taxi that was
suspiciously parked in front of the Aguila Auto Glass shop near the a. Qualifications: “one who can perceive and perceiving can
intersection of BF Almanza and Alabang-Zapote Roads. The officers make known his perception
approached the taxi and asked the driver, later identified as
accused Enojas, for his documents. The latter complied but, having i. ability to observe/ perceive
entertained doubts regarding the veracity of documents shown ii. ability to recall/ remember
them, they asked him to come with them to the police station in
iii. ability to relate/ communicate
their mobile car for further questioning.2
o Accused Enojas voluntarily went with the police officers and left his
taxi behind. On reaching the 7-11 convenience store on the Section 20. Witnesses; their qualifications. — Except as provided in the next succeeding
Zapote-Alabang Road, however, they stopped and PO2 Pangilinan section, all persons who can perceive, and perceiving, can make their known perception to
went down to relieve himself there. As he approached the store’s others, may be witnesses.
door, however, he came upon two suspected robbers and shot it
out with them. PO2 Pangilinan shot one suspect dead and hit the Religious or political belief, interest in the outcome of the case, or conviction of a crime unless
other who still managed to escape. But someone fired at PO2 otherwise provided by law, shall not be ground for disqualification. (18a)
Pangilinan causing his death.
o On hearing the shots, PO2 Gregorio came around and fired at an b. Disqualifications
armed man whom he saw running towards Pilar Village. He saw
another man, who came from the Jollibbee outlet, run towards CASE
Alabang-Zapote Road while firing his gun at PO2 Gregorio. The
latter returned fire but the men were able to take a taxi and
LUISA NAVARRO MARCOS*, Petitioner, vs. THE HEIRS OFTHE LATE DR. ANDRES
escape. PO2 Gregorio radioed for help and for an ambulance. On NAVARRO, JR., namely NONITA NAVARRO, FRANCISCA NAVARRO MALAPITAN, SOLEDAD
returning to his mobile car, he realized that accused Enojas, the NAVARRO BROCHLER, NONITA BARRUN NAVARRO, JR., IMELDA NAVARRO, ANDRES
taxi driver they had with them had fled.
NAVARRO III, MILAGROS NAVARRO YAP, PILAR NAVARRO, TERESA NAVARRO-TABITA, and
o P/Insp. Ferjen Torred (Torred), the Chief of Investigation Division LOURDES BARRUN-REJUSO, Respondents.
of the Las Piñas Police, testified that he and PO2 Teoson Rosarito G.R. No. 198240 July 3, 2013
(PO2 Rosarito) immediately responded to PO2 Gregorio’s urgent FIRST DIVISION
call. Suspecting that accused Enojas, the taxi driver who fled, was
involved in the attempted robbery, they searched the abandoned FACTS:
taxi and found a mobile phone that Enojas apparently left behind. x Spouses Andres Navarro, Sr. and Concepcion Medina-Navarro died in 1958 and
P/Ins. Torred instructed PO3 Joel Cambi (PO3 Cambi) to monitor its 1993, respectively and left behind several parcels of land including a 108.3997-
incoming messages.3 hectare lot (subject lot) located in Cayabon, Milagros, Masbate
o The police later ascertained that the suspect whom PO2 Pangilinan x Surviving heirs:
had killed was someone named Reynaldo Mendoza who was armed o Luisa Æ petitioner
with a .38 caliber revolver. The police found spent 9 mm and M-16
o Lydia Navarro Grageda
rifle shells at the crime scene. Follow-up operations at nearby o Heirs of their only son Andres Navarro, Jr. Æ respondents
provinces resulted in finding the dead body of one of the suspects, x Heirs Æ claimed ownership over the subject parcel of land
Alex Angeles, at the Metro South Medical Center along Molino,
o based their claim on the Affidavit of Transfer of Real Property dated
Bacoor, Cavite.4 May 19, 1954 where Andres, Sr. donated the subject lot to Andres,
o PO3 Cambi and PO2 Rosarito testified that they monitored the Jr.
messages in accused Enojas’ mobile phone and, posing as Enojas,
communicated with the other accused. The police then conducted
244
RECTO, GAYLE ANGELI M.
2011-0008 | AUSL
Personal Notes on Remedial Law 2 Review (based on the syllabus of Prof. Henedino M. Brondial)

x Marcos (and her sister Lydia) believing that the affidavit was a forgery Æ Section 21 provides for disqualification based on privileged
requested a handwriting examination of the affidavit through Assistant Fiscal communications. Section 15 of Rule 132 may not be a rule on
Andres Marcos disqualification of witnesses but it states the grounds when a
o The PNP handwriting expert PO2 Mary Grace Alvarez found that witness may be impeached by the party against whom he was
Andres, Sr.’s signature on the affidavit and the submitted standard called.
signatures of Andres, Sr. were not written by one and the same o There is no provision of the Rules disqualifying parties
person declared in default from taking the witness stand for non-
x Marcos (and her sister Lydia) Æ filed an action for annulment of deed of disqualified parties. The law does not provide default as an
donation before RTC Masbate exception. The specific enumeration of disqualified
x Heirs Æ moved to disqualify PO2 Alvarez as a witness witnesses excludes the operation of causes of disability
o argued that the RTC did not authorize the handwriting examination other than those mentioned in the Rules. It is a maxim of
of the affidavit. They added that presenting PO2 Alvarez as a witness recognized utility and merit in the construction of statutes that an
will violate their constitutional right to due process since no notice express exception, exemption, or saving clause excludes other
was given to them before the examination was conducted.8 Thus, exceptions. x x x As a general rule, where there are express
PO2 Alvarez’s report is a worthless piece of paper and her testimony exceptions these comprise the only limitations on the operation of a
would be useless and irrelevant. statute and no other exception will be implied. x x x The Rules should
x RTC Æ granted the motion to disqualify not be interpreted to include an exception not embodied therein.
o ruled that PO2 Alvarez’s supposed testimony would be hearsay as (Emphasis supplied; citations omitted.)
she has no personal knowledge of the alleged handwriting of x As a handwriting expert of the PNP, PO2 Alvarez can surely perceive and make
Andres, Sr. Also, there is no need for PO2 Alvarez to be presented, known her perception to others.1âwphi1 We have no doubt that she is qualified
if she is to be presented as an expert witness, because her as a witness. She cannot be disqualified as a witness since she possesses none of
testimony is not yet needed. the disqualifications specified under the Rules. Respondents’ motion to
x Marcos (and her sister Lydia) Æ filed an MR disqualify her should have been denied by the RTC for it was not based on any of
o RTC Æ denied these grounds for disqualification. The RTC rather confused the qualification of the
x Marcos (and her sister Lydia) Æ filed a Rule 65 before the CA witness with the credibility and weight of her testimony.
o CA Æ dismissed the petition (at this time, the RTC has already x Moreover, Section 49, Rule 130 of the Rules of Evidence is clear that the opinion
dismissed the annulment case) of an expert witness may be received in evidence, to wit:
ƒ on the ground that the dismissal of Civil Case No. o SEC. 49. Opinion of expert witness.-The opinion of a witness on a
5215 has mooted the issue of PO2 Alvarez’s matter requiring special knowledge, skill, experience or training
disqualification as a witness. which he is shown to possess, may be received in evidence.
x Marcos Æ filed a Rule 45 before the SC x For instance, in Tamani v. Salvador,22 we were inclined to believe that Tamani’s
o Petitioner adds that the CA erred in not ruling that the RTC signature was forged after considering the testimony of the PNP document
committed grave abuse of discretion in disqualifying PO2 Alvarez as examiner that the case involved simulated or copied forgery, such that the
a witness.12 They stress that PO2 Alvarez will be presented as an similarities will be superficial. We said that the value of the opinion of a
expert witness to render an opinion on whether the disputed handwriting expert depends not upon his mere statements of whether a writing
handwriting was indeed made by Andres, Sr. or whether it is a is genuine or false, but upon the assistance he may afford in pointing out
forgery. distinguishing marks, characteristics and discrepancies in and between genuine
and false specimens of writing which would ordinarily escape notice or detection
ISSUE: Whether PO2 Alvarez is disqualified to be a witness. from an unpracticed observer.
x Thus, we disagree with the RTC that PO2 Alvarez’s testimony would be hearsay.
HELD: NO. Under Section 49, Rule 130 of the Rules on Evidence, PO2 Alvarez is
x The CA ruling that the dismissal of Civil Case No. 5215 has mooted the issue of allowed to render an expert opinion, as the PNP document examiner
PO2 Alvarez’s disqualification as a witness can no longer be justified. Hence, we was allowed in Tamani. But the RTC already ruled at the outset that
reverse the CA ruling. While we agree with the CA in considering the RTC’s PO2 Alvarez’s testimony is hearsay even before her testimony is
Orders15 which dismissed Civil Case No. 5215, we are unable to agree with its offered and she is called to the witness stand. Under the
refusal to take judicial notice of the Decision16 of another CA Division which circumstances, the CA should have issued a corrective writ of certiorari
reinstated Civil Case No. 5215. Subsequent proceedings were even held in the and annulled the RTC ruling.
reinstated Civil Case No. 5215 per Orders17 issued by the RTC which were x True, the use of the word "may" in Section 49, Rule 130 of the Rules on
already submitted to the CA. That Civil Case No. 5215 was reinstated is a fact Evidence signifies that the use of opinion of an expert witness is
that cannot be ignored. permissive and not mandatory on the part of the courts.23 Jurisprudence
x We also agree with petitioner that the RTC committed grave abuse of discretion is also replete with instances wherein this Court dispensed with the testimony of
in disqualifying PO2 Alvarez as a witness. Grave abuse of discretion defies exact expert witnesses to prove forgeries.24 However, we have also recognized that
definition, but it generally refers to capricious or whimsical exercise of judgment handwriting experts are often offered as expert witnesses considering the
as is equivalent to lack of jurisdiction. The abuse of discretion must be patent technical nature of the procedure in examining forged documents.25 More
and gross as to amount to an evasion of a positive duty or a virtual refusal to important, analysis of the questioned signature in the deed of donation executed
perform a duty enjoined by law, or to act at all in contemplation of law, as where by the late Andres Navarro, Sr. in crucial to the resolution of the case.
the power is exercised in an arbitrary and despotic manner by reason of passion x In sum, the RTC should not have disqualified P02 Alvarez as a witness. She has
and hostility.18 Grave abuse of discretion arises when a lower court or tribunal the qualifications of witness and possess none of the disqualifications under the
violates the Constitution or grossly disregards the law or existing Rules. The Rules allow the opinion of an expert witness to be received as
jurisprudence.19 evidence. In Tamani, we used the opinion of an expert witness. The value of P02
x In Armed Forces of the Philippines Retirement and Separation Benefits System v. Alvarez's expert opinion cannot be determined if P02 Alvarez is not even allowed
Republic of the Philippines,20 we said that a witness must only possess all the to testify on the handwriting examination she conducted.
qualifications and none of the disqualifications provided in the Rules of Court.
Section 20, Rule 130 of the Rules on Evidence provides: i. Mental Capacity of Immaturity
o SEC. 20. Witnesses; their qualifications.-Except as provided in the
next succeeding section, all persons who can perceive, and
Section 21. Disqualification by reason of mental incapacity or immaturity. — The
perceiving, can make known their perception to others, may be
following persons cannot be witnesses:
witnesses.
o Religious or political belief, interest in the outcome of the case, or
(a) Those whose mental condition, at the time of their production for examination, is
conviction of a crime unless otherwise provided by law, shall not be
such that they are incapable of intelligently making known their perception to others;
a ground for disqualification.
x Specific rules of witness disqualification are provided under Sections
(b) Children whose mental maturity is such as to render them incapable of
21 to 24, Rule 130 of the Rules on Evidence. Section 21 disqualifies a
perceiving the facts respecting which they are examined and of relating them truthfully. (19a)
witness by reason of mental incapacity or immaturity. Section 22
disqualifies a witness by reason of marriage. Section 23 disqualifies a
witness by reason of death or insanity of the adverse party. Section 24
ANNOTATIONS
disqualifies a witness by reason of privileged communication.
x In Cavili v. Judge Florendo,21 we have held that the specific enumeration of x Qualifications and disqualifications Æ determined at the time the
disqualified witnesses excludes the operation of causes of disability other witness is produced for examination in court or at the taking of their
than those mentioned in the Rules. The Rules should not be interpreted to depositions
include an exception not embodied therein. We said: x GR: The interest of a witness to a case DOES NOT disqualify him
o The generosity with which the Rule allows people to testify is
apparent. Interest in the outcome of a case, conviction of a crime from testifying
unless otherwise provided by law, and religious belief are not o REASON: such interest only affects his credibility and NOT
grounds for disqualification. his competency
o Sections 19 and 20 of Rule 130 provide for specific o XPN: Dead Man Statute/ Survivorship Disqualification
disqualifications. Section 19 disqualifies those who are mentally Rule under Section 23 of Rule 130
incapacitated and children whose tender age or immaturity renders x Other rules re competency of witnesses:
them incapable of being witnesses. Section 20 provides for
disqualification based on conflicts of interest or on relationship.
245
RECTO, GAYLE ANGELI M.
2011-0008 | AUSL
Personal Notes on Remedial Law 2 Review (based on the syllabus of Prof. Henedino M. Brondial)

o A defendant declared in default is NOT disqualified from x That Evelyn is a mental retardate does not disqualify her as a witness nor render
testifying for his non-defaulting co-defendant her testimony bereft of truth.
x Sections 20 and 21 of Rule 130 of the Revised Rules of Court provide:
o GR: a person convicted of a crime is NOT disqualified
o SEC. 20. Witnesses; their qualifications. - Except as provided in the
from being a witness next succeeding section, all persons who can perceive, and
ƒ PROVISO: provided that he answers to the perceiving, can make known their perception to others, may be
fact of a previous final conviction witnesses.
ƒ XPN: When the law expressly provides that he o SEC. 21. Disqualification by reason of mental incapacity or
is so disqualified immaturity. - The following persons cannot be witnesses:
ƒ (a) Those whose mental condition, at the time of their
x Example Æ under the Civil Code, a
production for examination, is such that they are
person who has been convicted of incapable of intelligently making known their
falsification of a document, perjury perception to others;
or false testimony is disqualified ƒ (b) Children whose mental maturity is such as to
from being a witness to the render them incapable of perceiving the facts
execution of a will respecting which they are examined and of relating
them truthfully.
o Witness of “unsound mind Æ includes any mental
x In People v. Trelles,24 where the trial court relied heavily on the therein mentally
aberration, whether organic or functional, or induced by retarded private complainant’s testimony irregardless of her "monosyllabic
drugs or hypnosis responses and vacillations between lucidity and ambiguity," this Court held:
ƒ Affects only his credibility o A mental retardate or a feebleminded person is not, per se,
ƒ HOWEVER, if the witness can still convey disqualified from being a witness, her mental condition not
ideas by words or signs and give sufficient being a vitiation of her credibility. It is now universally
accepted that intellectual weakness, no matter what form
intelligent answers, he may still be a
it assumes, is not a valid objection to the competency of a
competent witness even if feeble-minded witness so long as the latter can still give a fairly intelligent
o Deaf mute witness Æ competent if he can understand and reasonable narrative of the matter testified to.25
and appreciate the sanctity of oath and comprehend facts x It can not then be gainsaid that a mental retardate can be a witness,
o Child witness Æ to be able to determine competency, depending on his or her ability to relate what he or she knows.26 If his
capacity must be determined through the ff indications: or her testimony is coherent, the same is admissible in court.27
x To be sure, modern rules on evidence have downgraded mental incapacity as a
ƒ Observation Æ to comprehend the obligation
ground to disqualify a witness. As observed by McCormick, the remedy of
of an oath excluding such a witness who may be the only person available who
ƒ Recollection Æ the time the fact to be knows the facts, seems inept and primitive. Our rules follow the modern
testified to occurred, such that he could trend of evidence.28
receive correct impressions thereof x Thus, in a long line of cases,29 this Court has upheld the conviction of the
ƒ Communication Æ to relate such facts truly at accused based mainly on statements given in court by the victim who was a
mental retardate.
the time he is offered as a witness x From a meticulous scrutiny of the records of this case, there is no reason to
doubt Evelyn’s credibility. To be sure, her testimony is not without discrepancies,
CASE given of course her feeblemindedness.
x By the account of Dr. Chona Cuyos-Belmonte, Medical Specialist II at the
PEOPLE OF THE PHILIPPINES, appellee, vs. SALVADOR GOLIMLIM @ "BADONG", Psychiatric Department of the Bicol Medical Center, who examined Evelyn,
appellants. although Evelyn was suffering from moderate mental retardation with an IQ of
G.R. No. 145225 April 2, 2004 46,30 she is capable of perceiving and relating events which happened to her.
THIRD DIVISION Thus the doctor testified:
o Q: So do you try to impress that although she answers in general
FACTS: terms it does not necessarily mean that she might be inventing
x Golimlim was charged with the rape of one Evelyn Canchela before the RTC answers - only that she could not go to the specific details because
Sorsogon of dullness?
o Canchela was left to the care and custody of Golimlim’s wife when o A: I don’t think she was inventing her answer because I conducted
Canchela’s mother left for SG mental status examination for three (3) times and I tried to see the
x Canchela was a mental retardate consistency in the narration but very poor (sic) in giving details.
x Canchela’s testimony: o Q: May we know what she related to you?
o 1) Despite her weak and dull mental state the victim was consistent o A: She related to me that she was raped by her uncle ‘Tatay
in her claim that her Papay Badong (accused Salvador Golimlim) Badong’. What she mentioned was that, and I quote: ‘hinila ang
had carnal knowledge of her and was the author of her pregnancy, panty ko, pinasok ang pisot at bayag niya sa pipi ko’. She would
and nobody else (See: For comparison her Sworn Statement on p. laugh inappropriately after telling me that particular incident. I also
3/Record; her narration in the Psychiatric Report on pp. 47 & tried to ask her regarding the dates, the time of the incident, but
48/Record; the TSNs of her testimony in open court); she could not really…. I tried to elicit those important things, but the
o 2) She remains consistent that her Papay Badong raped her only patient had a hard time remembering those dates.
once; o Q: But considering that you have evaluated her mentally, gave her
o 3) That the contradictory statements she made in open court I.Q. test, in your honest opinion, do you believe that this narration
relative to the details of how she was raped, although would seem by the patient to you about the rape is reliable?
derogatory to her credibility and reliability as a witness under o A: Yes, sir.
normal conditions, were amply explained by the psychiatrist who o Q: Why do you consider that reliable?
examined her and supported by her findings (See: Exhibits F to F- o A: Being a (sic) moderately retarded, I have noticed the
2); spontaneity of her answers during the time of the testing. She was
o 4) Despite her claim that several persons laid on top of her (which not even hesitating when she told me she was raped once at home
is still subject to question considering that the victim could not by her Tatay Badong; and she was laughing when she told me
elaborate on its meaning), the lucid fact remains that she never about how it was done on (sic) her. So, although she may be
pointed to anybody else as the author of her pregnancy, but her inappropriate but (sic) she was spontaneous, she was consistent.
Papay Badong. Which only shows that the trauma that was created o Q: Now, I would like to relate to you an incident that happened in
in her mind by the incident has remained printed in her memory this Court for you to give us your expert opinion. I tried to present
despite her weak mental state. Furthermore, granting for the sake the victim in this case to testify. While she testified that she was
of argument that other men also laid on top of her, this does not raped by her uncle Badong, when asked about the details, thereof,
deviate from the fact that her Papay Badong (the accused) had she would not make (sic) the detail. She only answered ‘wala’ (no).
sexual intercourse with her I ask this question because somehow this seems related to your
x RTC Æ convicted Golimlim previous evaluation that while she gave an answer, she gave no
o trial judge’s assessment of the credibility of witnesses’ testimonies detail. Now, I was thinking because I am a man and I was the one
is, as has repeatedly been held by this Court, accorded great asking and the Judge is a man also. And while the mother would say
respect on appeal in the absence of grave abuse of discretion on its that she would relate to her and she related to you, can you explain
part, it having had the advantage of actually examining both real to us why when she was presented in court that
and testimonial evidence including the demeanor of the witnesses occurrence, that event happened?
x GolimlimÆ appealed directly to SC o A: There are a lot of possible answers to that question; one, is the
court’s atmosphere itself. This may have brought a little anxiety on
ISSUE: Whether Canchela was disqualified as a witness, considering that she was a mental the part of the patient and this inhibits her from relating some of
retardate. the details relative to the incident-in-question. When I conducted
my interview with the patient, there were only two (2) of us in the
HELD: NO. room. I normally do not ask this question during the first session
246
RECTO, GAYLE ANGELI M.
2011-0008 | AUSL
Personal Notes on Remedial Law 2 Review (based on the syllabus of Prof. Henedino M. Brondial)

with the patient because these are emotionally leading questions, o Q: Did it enter your vagina?
and I do not expect the patient to be very trusting. So, I usually ask o A: Yes, Your Honor.
this type of questions during the later part of my examination to o Q: Madam Witness, is it true that your Papay Badong inserted his
make her relax during my evaluation. So in this way, she will be penis into your vagina or sexual organ during that time that he was
more cooperative with me. I don’t think that this kind of on top of you?
atmosphere within the courtroom with some people around, this o A: (The witness nods, yes.)33 (Underscoring supplied)
could have inhibited the patient from answering questions. x Appellant’s bare denial is not only an inherently weak defense. It is not
o Q: What if the victim is being coached or led by someone else, will supported by clear and convincing evidence. It cannot thus prevail over the
she be able to answer the questions? positive declaration of Evelyn who convincingly identified him as her rapist.
o A: Yes, she may be able to answer the questions, but you would
notice the inconsistency of the answers because what we normally
do is that we present the questions in different ways, and we ii. Marital Disqualifications
expect the same answer. This is how we try to evaluate the
patient. If the person, especially a retarded, is being coached by
Section 22. Disqualification by reason of marriage. — During their marriage, neither the
somebody, the answers will no longer be consistent.
husband nor the wife may testify for or against the other without the consent of the affected
o Q: You also mentioned a while ago that the answers given by the
spouse, except in a civil case by one against the other, or in a criminal case for a crime
patient, taken all in all, were consistent?
committed by one against the other or the latter's direct descendants or ascendants. (20a)
o A: Yes, sir.31 (Underscoring supplied)
x As noted in the above-quoted testimony of Dr. Belmonte, Evelyn could give
spontaneous and consistent answers to the same but differently framed ANNOTATIONS
questions under conditions which do not inhibit her from answering. It could
have been in this light that Evelyn was able to relate in court, upon examination x Section 22 Æ refers to Spousal Immunity
by a female government prosecutor and the exclusion of the public from the o Different from Marital Privilege under Section 24(a) x
proceedings, on Dr. Belmonte’s suggestion,32 how, as quoted below, she was Rationale behind the Spousal Immunity Rule:
raped and that it was appellant who did it:
o The identity of interest between the spouses
o Q: Lorna Hachero testified before this Court that you gave birth to o The consequent danger of perjury where one spouse
a baby girl named Johanna, is this true? testifies against the other
o A: (The witness nods, yes.) o The legal policy on guarding marital confidences and
o Q: Who is the father of Johanna? preventing domestic disunion
o A: Papay Badong o The danger of punishing one spouse through the hostile
o Q: Who is this Papay Badong that you are referring to?
o A: The husband of Mamay Bita.
testimony of the other
o Q: Is he here in court? x GR: Husband and wife may not testify for or against each other
o A: He is here. without the consent of the affected spouse during their marriage
o Q: Please look around and point him to us. o XPNs: such spousal immunity will in the ff cases:
o A: (The witness pointing to the lone man sitting in the first row of ƒ In a civil case by one against the other
the gallery wearing a regular prison orange t-shirt who gave his [Section 22]
name as Salvador Golimlim when asked.)
o Q: Why were you able to say that it is Papay Badong who is the
ƒ In a criminal case for a crime committed by
father of your child Johanna? one against the other or the latter’s direct
o A: Because then I was left at Mamay Bita’s house, although I am descendants or ascendants [Section 22]
not there now. ƒ When marital relations are so strained, as
o Q: And that house where you were left is also the house of your when the spouses are separated de facto
Papay Badong? [jurisprudence]
o A: Yes ma’am.
x RATIONALE behind the inclusion
o Q: What did Salvador Golimlim or your Papay Badong do to you
that’s why you were able to say that he is the father of your child? of this XPN: in such case, the
o A: I was undressed by him. preservation of marriage between
o Q: What did you do after you were undressed? the spouses is no longer an
o A: I was scolded by the wife, Mamay Bita. interest which the State aims to
o Q: I am referring to that very moment when you were undressed. protect
Immediately after your Papay Badong undressed you, what did you
ƒ When the offense committed by one spouse
do?
o A: He laid on top of me. directly attacks, or directly and vitally impairs,
o Q: What was your position when he laid on top of you? the conjugal relation [jurisprudence]
o A: I was lying down. x Requisites for application:
o Q: Then after he went on top of you, what did he do there? o Valid marriage existing at the time of the offer of
o A: He made (sic) sexual intercourse with me. testimony
o Q: When you said he had a (sic) sexual intercourse with you, what
did he do exactly? o The other spouse is a party to the action
o A: He kissed me.
o Q: Where? CASES
o A: On the cheeks (witness motioning indicating her cheeks).
o Q: What else did he do? Please describe before this Honorable MAXIMO ALVAREZ, Petitioner, vs. SUSAN RAMIREZ, Respondent.
Court the sexual intercourse which you are referring to which the G.R. No. 143439 October 14, 2005
accused did to you. THIRD DIVISION
o A: ‘Initoy’ and he slept after that.
o (to Court) Nevertheless, may we request that the local term for FACTS:
sexual intercourse, the word ‘Initoy’ which was used by the witness x Susan is the private complainant in a criminal charge for arson against Maximo
be put on the record, and we request judicial notice of the fact that o Maximo is the husband of the Susan’s sister, Esparanza
‘initoy’ is the local term for sexual intercourse. x During trial of the said criminal case, Esperanza was called by the prosecutor to
o Q: What did you feel when your Papay Badong had sexual the witness stand:
intercourse with you? o "ATTY. ALCANTARA:
o A: I felt a knife; it was like a knife. o We are calling Mrs. Esperanza Alvarez, the wife of the accused,
o Q: Where did you feel that knife? Your Honor.
o A: I forgot. o COURT:
o Q: Why did you allow your Papay Badong to have sexual o Swear in the witness.
intercourse with you? o ATTY. MESIAH: (sic)
o A: I will not consent to it. o Your Honor, we are offering the testimony of this witness for the
o Q: Did you like what he did to you? purpose of proving that the accused Maximo Alvarez committed all
o A: I do not want it. the elements of the crime being charged particularly that accused
o Q: But why did it happen? Maximo Alvarez pour on May 29, 1998 gasoline in the house
o A: I was forced to. located at Blk. 5, Lot 9, Phase 1-C, Dagat-dagatan, Navotas, Metro
o Q: Did you feel anything when he inserted into your vagina when Manila, the house owned by his sister-in-law Susan Ramirez; that
your Papay Badong laid on top of you? accused Maximo Alvarez after pouring the gasoline on the door of
o A: His sexual organ/penis. the house of Susan Ramirez ignited and set it on fire; that the
o Q: How did you know that it was the penis of your Papay Badong accused at the time he successfully set the house on fire (sic) of
that was entered into your vagina? Susan Ramirez knew that it was occupied by Susan Ramirez, the
o A: It was put on top of me. members of the family as well as Esperanza Alvarez, the estranged
247
RECTO, GAYLE ANGELI M.
2011-0008 | AUSL
Personal Notes on Remedial Law 2 Review (based on the syllabus of Prof. Henedino M. Brondial)

wife of the accused; that as a consequence of the accused in become so strained that there is no more harmony, peace or
successfully setting the fire to the house of Susan Ramirez, the tranquility to be preserved. The Supreme Court has held that in
door of said house was burned and together with several articles of such a case, identity is non-existent. In such a situation, the
the house, including shoes, chairs and others. security and confidences of private life which the law aims to
o COURT: protect are nothing but ideals which through their absence, merely
o You may proceed. leave a void in the unhappy home. (People v. Castañeda, 271 SCRA
o DIRECT EXAMINATION 504). Thus, there is no longer any reason to apply the Marital
o ATTY. ALCANTARA: Disqualification Rule."
o Q: When you were able to find the source, incidentally what was x It should be stressed that as shown by the records, prior to the commission of
the source of that scent? the offense, the relationship between petitioner and his wife was already
o A: When I stand by the window, sir, I saw a man pouring the strained. In fact, they were separated de facto almost six months before the
gasoline in the house of my sister (and witness pointing to the incident. Indeed, the evidence and facts presented reveal that the preservation
person of the accused inside the court room). of the marriage between petitioner and Esperanza is no longer an interest the
o Q: For the record, Mrs. Witness, can you state the name of that State aims to protect.
person, if you know? x At this point, it bears emphasis that the State, being interested in laying the truth
o A: He is my husband, sir, Maximo Alvarez. before the courts so that the guilty may be punished and the innocent
o Q: If that Maximo Alvarez you were able to see, can you identify exonerated, must have the right to offer the direct testimony of Esperanza, even
him? against the objection of the accused, because (as stated by this Court in
o A: Yes, sir. Francisco14), "it was the latter himself who gave rise to its necessity."
o Q: If you can see him inside the Court room, can you please point
him? PEOPLE OF THE PHILIPPINES, petitioner, vs. HON. MARIANO C. CASTAÑEDA, JR., as
o A: Witness pointing to a person and when asked to stand and Judge of the Court of First Instance of Pampanga, Branch III, and BENJAMIN F. MANALOTO,
asked his name, he gave his name as Maximo Alvarez. respondents.
x Maximo Æ moved that Esperanza be DQed pursuant to Rule 130 of the Revised G.R. No. L-46306 February 27, 1979
Rules of Court on marital disqualification SECOND DIVISION
x RTC Æ granted the motion; denied Susan’s ensuing MR
x Susan Æ filed a Rule 65 before the CA FACTS:
o CA Æ granted the petition; nullified RTC’s order x Victoria M. Manaloto was the private complainant in a criminal case for
x Maximo Æ filed a Rule 45 to SC falsification of public document against her husband, Benjamin, filed before CRI
Pampanga
ISSUE: Whether Esperanza should be disqualified as witness under the marital privilege rule. o did then and there willfully, unlawfully and feloniously counterfeit,
imitate and forge the signature of his spouse Victoria M. Manaloto
HELD: NO. in a deed of sale executed by said accused wherein he sold a house
x Section 22, Rule 130 of the Revised Rules of Court provides: and lot belonging to the conjugal partnership of said spouse in
o "Sec. 22. Disqualification by reason of marriage. - During their favor of Ponciano Lacsamana
marriage, neither the husband nor the wife may testify for or x Victoria was called to the witness stand by the prosecution
against the other without the consent of the affected spouse, x Benjamin Æ moved to DQ Victoria under the marital privilege rule
except in a civil case by one against the other, or in a criminal case x Prosecution Æ opposed
for a crime committed by one against the other or the latter’s direct x CFI Æ granted the motion; DQed Victoria
descendants or ascendants." x Prosecution Æ filed a Rule 65 before the SC
x The reasons given for the rule are: o seeking set aside the aforesaid order of the respondent Judge and
o 1. There is identity of interests between husband and wife; praying that a preliminary injunction or a ternporary restraining
o 2. If one were to testify for or against the other, there is order be issued by this Court enjoining said judge from further
consequent danger of perjury; proceeding with the trial of aforesaid Criminal Case No. 1011
o 3. The policy of the law is to guard the security and confidences of
private life, even at the risk of an occasional failure of justice, and ISSUE: Whether Victoria should be disqualified under the marital privilege rule.
to prevent domestic disunion and unhappiness; and
o 4. Where there is want of domestic tranquility there is danger of HELD: NO.
punishing one spouse through the hostile testimony of the other.11 x From the foregoing factual and procedural antecedents emerges the sole issues
x But like all other general rules, the marital disqualification rule has its determinative of the instant petition, to wit: Whether or not the criminal case for
own exceptions, both in civil actions between the spouses and in Falsification of Public Document filed against herein private respondent Benjamin
criminal cases for offenses committed by one against the other. Like the F. Manaloto — who allegedly forged the signature of his wife, Victoria M.
rule itself, the exceptions are backed by sound reasons which, in the Manaloto, in a deed of sale, thereby making it appear that the latter gave her
excepted cases, outweigh those in support of the general rule. For marital consent to the sale of a house and lot belonging to their conjugal
instance, where the marital and domestic relations are so strained that partnership when in fact and in truth she did not — may be considered as a
there is no more harmony to be preserved nor peace and tranquility criminal case for a crime committed by a husband against his wife and,
which may be disturbed, the reason based upon such harmony and therefore, an exception to the rule on marital disqualification.
tranquility fails. In such a case, identity of interests disappears and the x We sustain petitioner's stand that the case is an exception to the
consequent danger of perjury based on that identity is non-existent. Likewise, in marital disqualification rule, as a criminal case for a crime committed
such a situation, the security and confidences of private life, which the law aims by the accused-husband against the witness-wife.
at protecting, will be nothing but ideals, which through their absence, merely x 1. The act complained of as constituting the crime of Falsification of
leave a void in the unhappy home.12 Public Document is the forgery by the accused of his wife's signature in a deed
x In Ordoño vs. Daquigan,13 this Court held: of sale, thereby making it appear therein that said wife consented to the sale of
o "We think that the correct rule, which may be adopted in this a house and lot belonging to their conjugal partnership when in fact and in truth
jurisdiction, is that laid down in Cargil vs. State, 35 ALR 133, 220 she did not. It must be noted that had the sale of the said house and lot, and the
Pac. 64, 25 Okl. 314, wherein the court said: signing of the wife's name by her husband in the deed of sale, been made with
ƒ ‘The rule that the injury must amount to a physical the consent of the wife, no crime could have been charged against said husband
wrong upon the person is too narrow; and the rule Clearly, therefore, it is the husband's breach of his wife's confidence which gave
that any offense remotely or indirectly affecting rise to the offense charged. And it is this same breach of trust which prompted
domestic harmony comes within the exception is too the wife to make the necessary complaint with the Office of the Provincial Fiscal
broad. The better rule is that, when an offense which, accordingly, filed the aforesaid criminal case with the Court of First
directly attacks, or directly and vitally impairs, the Instance of Pampanga. To rule, therefore, that such criminal case is not one for a
conjugal relation, it comes within the exception to the crime committed by one spouse against the other is to advance a conclusion
statute that one shall not be a witness against the which completely disregards the factual antecedents of the instant case.
other except in a criminal prosecution for a crime x 2. This is not the first time that the issue of whether a specific offense
committee (by) one against the other.’" may be classified as a crime committed by one spouse against the other is
x Obviously, the offense of arson attributed to petitioner, directly impairs presented to this Court for resolution. Thus, in the case of Ordoño v. Daquigan,
the conjugal relation between him and his wife Esperanza. His act, as 8 this Court, through Mr. Justice Ramon C. Aquino, set up the criterion to be
embodied in the Information for arson filed against him, eradicates all followed in resolving the issue, stating that:
the major aspects of marital life such as trust, confidence, respect and o We think that the correct rule, which may be adopted in this
love by which virtues the conjugal relationship survives and flourishes. jurisdiction, is that laid down in Cargill v. State, 35 ALR, 133, 220,
x As correctly observed by the Court of Appeals: Pac 64,26 OkL 314, wherein the court said:
o "The act of private respondent in setting fire to the house of his o The rule that the injury must amount to a physical wrong upon the
sister-in-law Susan Ramirez, knowing fully well that his wife was is too narrow; and the rule that any offense remotely or indirectly
there, and in fact with the alleged intent of injuring the latter, is an affecting domestic within the exception is too broad. The better
act totally alien to the harmony and confidences of marital relation rule is that, WHEN AN OFFENSE DIRECTLY ATTACKS, OR
which the disqualification primarily seeks to protect. The criminal DIRECTLY AND VITALLY IMPAIRS, THE CONJUGAL
act complained of had the effect of directly and vitally impairing the RELATION, IT COMES WITHIN THE EXCEPTION to the
conjugal relation. It underscored the fact that the marital and statute that one shall not be a witness against the other
domestic relations between her and the accused-husband have
248
RECTO, GAYLE ANGELI M.
2011-0008 | AUSL
Personal Notes on Remedial Law 2 Review (based on the syllabus of Prof. Henedino M. Brondial)

except in a criminal prosecution for a crime committed (by) ƒ This rule applies REGARDLESS of whether the
one against the other. deceased died before or after the sit against
x Applying the foregoing criterion in said case of Ordoño v. Daquigan this Court
him is filed, PROVIDED that he is already
held that the rape committed by the husband of the witness-wife against their
daughter was a crime committed by the husband against his wife. Although the dead at the time the testimony is sought to be
victim of the crime committed by the accused in that can was not his wife but given
their daughter, this Court, nevertheless, applied the exception for the reason that o The case is upon a claim or demand against the
said criminal act "Positively undermine(d) the connubial relationship. 9 estate of such person who is deceased or of unsound
x With more reason must the exception apply to the instant case where the victim
of the crime and the person who stands to be directly prejudiced by the
mind
ƒ Hence, this rule DOES NOT apply where it is
falsification is not a third person but the wife herself. And it is undeniable
that the act comp of had the effect of directly and vitally impairing the conjugal the administrator who brings an action to
relation. This is apparent not only in the act Of the wife in personally lodging her recover property allegedly belonging to the
complaint with the Office of the Provincial Fiscal, but also in her insistent efforts estate
10 in connection with the instant petition, which seeks to set aside the order ƒ Also DOES NOT apply where the action is by
disqualified her from testifying against her husband. Taken collectively, the the heirs of a deceased plaintiff who were
actuations of the witness-wife underacore the fact that the martial and domestic
substituted for the latter
relations between her and the accused-husband have become so strained that
there is no more harmony to be preserved said nor peace and tranquility which o The testimony to be given is on a matter of fact
may be disturbed. In such a case, as We have occasion to point out in previous occurring before the death of such deceased person
decisions, "identity of interests disappears and the consequent danger of perjury or before such person became of unsound mind
based on that Identity is nonexistent. Likewise, in such a situation, the security ƒ Includes any matter of fact which bears upon
and confidence of private life which the law aims at protecting will be nothing a transaction or communication between the
but Ideals which, through their absence, merely leave a void in the unhappy
witness and the decedent even though
home. 11 Thus, there is no reason to apply the martial disqualification
rule. without the presence or participation of the
x 3. Finally, overriding considerations of public policy demand that the latter
wife should not be disqualified from testifying against her husband in the instant ƒ NOT covered:
case. For, as aptly observed by the Solicitor General," (t)o espouse the x Negative testimony
contrary view would spawn the dangerous precedent of a husband x Testimony on the existence and
committing as many falsifications against his wife as he could conjure,
possession of a written instrument
seeking shelter in the anti-marital privilege as a license to injure and
prejudice her in secret — all with unabashed and complete impunity. signed by the deceased Æ such
fact exists even after the death of
iii. Death or Insanity (Dead Man’s Statute) the deceased
x PURPOSE of this rule: to discourage perjury and protect the estate
Section 23. Disqualification by reason of death or insanity of adverse party. — from fictitious claims
Parties or assignor of parties to a case, or persons in whose behalf a case is prosecuted, o Hence, the rule DOES NOT apply, even if all 4 requisites
against an executor or administrator or other representative of a deceased person, or against a are present when:
person of unsound mind, upon a claim or demand against the estate of such deceased
ƒ Where the testimony is offered to prove a
person or against such person of unsound mind, cannot testify as to any matter of fact
occurring before the death of such deceased person or before such person became of unsound claim less than what is established under a
mind. (20a) written document
ƒ Where it is intended to prove a fraudulent
ANNOTATIONS transaction of the deceased
x WAIVED: when the defendant does not timely object to the
x Section 23 Æ Survivorship Disqualification Rule/ Dead Man Statute admission of such evidence or testifies on the prohibited matters or
x Only a partial disqualification cross-examines thereon

CASES
Survivorship Disqualification Spousal Immunity/ Marital
Rule/ Dead Man Statute Disqualification ENRIQUE RAZON, petitioner, vs. INTERMEDIATE APPELLATE COURT and VICENTE B.
CHUIDIAN, in his capacity as Administrator of the Estate of the Deceased JUAN T. CHUIDIAN,
respondents.
Section 23 Section 22
G.R. No. 74306 March 16, 1992
only a PARTIAL disqualification
because the witness is not VICENTE B. CHUIDIAN, petitioner, vs. INTERMEDIATE APPELLATE COURT, ENRIQUE RAZ0N,
completely disqualified but is only complete and absolute and E. RAZON, INC., respondents.
prohibited from testifying on disqualification G.R. No. 74315 March 16, 1992
matters specified therein THIRD DIVISION

applies only to a civil case or applies to a civil or criminal case, FACTS:


special proceeding over the estate subject to the 2 exceptions x Vicente is the administrator of the estate of the deceased Juan
of a deceased or insane person provided therein x Vicente Æ filed an action for specific performance before CFI Manila against
Razon et al
x Requisites for the application of Section 22: o Praying that the latter be ordered to deliver certificates of stocks
representing the shareholdings of the deceased Juan T. Chuidian in
o Witness offered for examination is a party plaintiff OR
the E. Razon, Inc. with a prayer for an order to restrain the
the assignor of the said party OR a person in whose defendants from disposing of the said shares of stock, for a writ of
behalf a case is prosecuted preliminary attachment v. properties of defendants having
ƒ Party plaintiff Æ such plaintiff must be the possession of shares of stock and for receivership of the properties
real party in interest of defendant corporation
ƒ DOES NOT apply where there is a x Razon et al Æ filed their answer
o alleged that all the shares of stock in the name of stockholders of
counterclaim interposed by the defendant and
record of the corporation were fully paid for by defendant, Razon;
the latter would be testifying as a plaintiff o that said shares are subject to the agreement between defendants
o The case is against the executor or administrator or and incorporators;
other representative of a deceased person or one who o that the shares of stock were actually owned and remained in the
is of unsound mind possession of Razon.
ƒ Necessary that the defendant is being sued in o Appellees also alleged . . . that neither the late Juan T. Chuidian
nor the appellant had paid any amount whatsoever for the 1,500
such representative capacity
shares of stock in question
ƒ EVEN if the property involved has been x Vicente’s evidence:
adjudicated to the heirs, they are still o Sometime in 1962, Enrique Razon organized the E. Razon, Inc. for
protected under this rule the purpose of bidding for the arrastre services in South Harbor,
x REASON: they are considered as Manila. The incorporators consisted of Enrique Razon, Enrique
representatives of the deceased Valles, Luisa M. de Razon, Jose Tuason, Jr., Victor Lim, Jose F.
Castro and Salvador Perez de Tagle.

249
RECTO, GAYLE ANGELI M.
2011-0008 | AUSL
Personal Notes on Remedial Law 2 Review (based on the syllabus of Prof. Henedino M. Brondial)

o On April 23, 1966, stock certificate No. 003 for 1,500 shares of o The reason for the rule is that if persons having a claim
stock of defendant corporation was issued in the name of Juan T. against the estate of the deceased or his properties were
Chuidian. allowed to testify as to the supposed statements made by
o On the basis of the 1,500 shares of stock, the late Juan T. Chuidian him (deceased person), many would be tempted to falsely
and after him, the plaintiff-appellant, were elected as directors of E. impute statements to deceased persons as the latter can
Razon, Inc. Both of them actually served and were paid no longer deny or refute them, thus unjustly subjecting
compensation as directors of E. Razon, Inc. their properties or rights to false or unscrupulous claims or
o From the time the certificate of stock was issued on April 1966 up demands. The purpose of the law is to "guard against the
to April 1971, Enrique Razon had not questioned the ownership by temptation to give false testimony in regard to the
Juan T. Chuidian of the shares of stock in question and had not transaction in question on the part of the surviving party."
brought any action to have the certificate of stock over the said (Tongco v. Vianzon, 50 Phil. 698; Go Chi Gun, et al. v. Co Cho, et
shares cancelled. al., 622 [1955])
o The certificate of stock was in the possession of defendant Razon x The rule, however, delimits the prohibition it contemplates in that it is
who refused to deliver said shares to the plaintiff, until the same was applicable to a case against the administrator or its representative of
surrendered by defendant Razon and deposited in a safety box in an estate upon a claim against the estate of the deceased person. (See
Philippine Bank of Commerce. Tongco v. Vianzon, 50 Phil. 698 [1927])
x Razon et al’s evidence: x In the instant case, the testimony excluded by the appellate court is that of the
o Defendants allege that after organizing the E. Razon, Inc., Enrique defendant (petitioner herein) to the affect that the late Juan Chuidian, (the
Razon distributed shares of stock previously placed in the names of father of private respondent Vicente Chuidian, the administrator of the estate of
the withdrawing nominal incorporators to some friends including Juan Chuidian) and the defendant agreed in the lifetime of Juan Chuidian that the
Juan T. Chuidian 1,500 shares of stock in E. Razon, Inc. are actually owned by the defendant unless
o Stock Certificate No. 003 covering 1,500 shares of stock upon the deceased Juan Chuidian opted to pay the same which never
instruction of the late Chuidian on April 23, 1986 was personally happened. The case was filed by the administrator of the estate of the
delivered by Chuidian on July 1, 1966 to the Corporate Secretary of late Juan Chuidian to recover shares of stock in E. Razon, Inc. allegedly
Attorney Silverio B. de Leon who was himself an associate of the owned by the late Juan T. Chuidian.
Chuidian Law Office (Exhs. C & 11). Since then, Enrique Razon was x It is clear, therefore, that the testimony of the petitioner is not within
in possession of said stock certificate even during the lifetime of the prohibition of the rule. The case was not filed against the
the late Chuidian, from the time the late Chuidian delivered the administrator of the estate, nor was it filed upon claims against the
said stock certificate to defendant Razon until the time (sic) of estate.
defendant Razon. By agreement of the parties (sic) delivered it for x Furthermore, the records show that the private respondent never objected
deposit with the bank under the joint custody of the parties as to the testimony of the petitioner as regards the true nature of his
confirmed by the trial court in its order of August 7, 1971. transaction with the late elder Chuidian. The petitioner's testimony was
o Thus, the 1,500 shares of stook under Stock Certificate No. 003 subject to cross-examination by the private respondent's counsel. Hence,
were delivered by the late Chuidian to Enrique because it was the granting that the petitioner's testimony is within the prohibition of Section 20(a),
latter who paid for all the subscription on the shares of stock in the Rule 130 of the Rules of Court, the private respondent is deemed to have waived
defendant corporation and the understanding was that he the rule. We ruled in the case of Cruz v. Court of Appeals (192 SCRA 209
(defendant Razon) was the owner of the said shares of stock and [1990]):
was to have possession thereof until such time as he was paid o It is also settled that the court cannot disregard evidence which
therefor by the other nominal incorporators/stockholders would ordinarily be incompetent under the rules but has been
x RTC Æ ruled in favor of RAZON ET AL rendered admissible by the failure of a party to object thereto.
o declared that Enrique Razon, the petitioner in G.R. No. 74306 is the Thus:
owner of the said shares of stock ƒ . . . The acceptance of an incompetent witness
x Vicente Æ appealed to CA to testify in a civil suit, as well as the allowance
x CA Æ reversed RTC of improper questions that may be put to him
o ruled that Juan T. Chuidian, the deceased father of petitioner while on the stand is a matter resting in the
Vicente B. Chuidian in G.R. No. 74315 is the owner of the shares of discretion of the litigant. He may assert his
stock right by timely objection or he may waive it,
x BOTH parties Æ filed their respective MRs expressly or by silence. In any case the option
o Razon Æ sought reversal of CA’s decision rests with him. Once admitted, the testimony is in
o Vicente Æ asked that all cash and stock dividends and all the pre- the case for what it is worth and the judge has no
emptive rights accruing to the 1,500 shares of stock be ordered power to disregard it for the sole reason that it could
delivered to him have been excluded, if it had been objected to, nor to
x CA Æ denied both MRs strike it out on its own motion (Emphasis supplied).
x BOTH parties Æ filed their Rule 45 before the SC (Marella v. Reyes, 12 Phil. 1.)
o Razon’s petition:
ƒ he appellate court's decision on its alleged LILIBETH SUNGA-CHAN and CECILIA SUNGA, petitioners, vs. LAMBERTO T. CHUA,
misapplication of the dead man's statute rule under respondent.
Section 20(a) Rule 130 of the Rules of Court. G.R. No. 143340 August 15, 2001
According to him, the "dead man's statute" rule is not THIRD DIVISION
applicable to the instant case.
ƒ Moreover, the private respondent, as plaintiff in the FACTS:
case did not object to his oral testimony regarding the x Chua and Jacinto Sunga formed a partnership to engage in the marketing of
oral agreement between him and the deceased Juan liquefied petroleum gas
T. Chuidian that the ownership of the shares of stock x It was called Shellite Gas Appliance Center (Shellite) and was registered as a sole
was actually vested in the petitioner unless the proprietorship in the name of Jacinto, albeit the partnership arrangement called
deceased opted to pay the same; and that the for equal sharing of the net profit
petitioner was subjected to a rigid cross examination x Jacinto died
regarding such testimony. x Wife Cecilia and daughter Lilibeth Æ continued the business WITHOUT the
x The petitions were then consolidated consent of Chua
x Chua demanded accounting and liquidation
ISSUE: Whether the dead man’s statute was correctly applied by the CA, considering that it o These demands were unheeded
was the administrator Vicente who filed the original action. x Chua Æ filed a complaint for Winding Up of a Partnership Affairs, Accounting,
Appraisal and Recovery of Shares and Damages with Writ of Preliminary
HELD: NO. Attachment before the RTC Zamboanga Del Norte
x Section 20(a) Rule 130 of the Rules of Court (Section 23 of the Revised Rules on x Chua’s allegations:
Evidence) States: o Respondent alleged that in 1977, he verbally entered into a
o Sec. 20. Disqualification by reason of interest or relationship — partnership with Jacinto in the distribution of Shellane Liquefied
The following persons cannot testify as to matters in which they Petroleum Gas (LPG) in Manila. For business convenience,
are interested directly or indirectly, as herein enumerated. respondent and Jacinto allegedly agreed to register the business
ƒ (a) Parties or assignors of parties to a case, name of their partnership, SHELLITE GAS APPLIANCE CENTER
or persons in whose behalf a case is prosecuted, (hereafter Shellite), under the name of Jacinto as a sole
against an executor or administrator or other proprietorship. Respondent allegedly delivered his initial capital
representative of a deceased person, or against a contribution of P100,000.00 to Jacinto while the latter in turn
person of unsound mind, upon a claim or demand produced P100,000.00 as his counterpart contribution, with the
against the estate of such deceased person or against intention that the profits would be equally divided between them.
such person of unsound mind, cannot testify as to The partnership allegedly had Jacinto as manager, assisted by
any matter of fact accruing before the death of such Josephine Sy (hereafter Josephine), a sister of the wife respondent,
deceased person or before such person became of Erlinda Sy. As compensation, Jacinto would receive a manager's fee
unsound mind." (Emphasis supplied) or remuneration of 10% of the gross profit and Josephine would
x The purpose of the rule has been explained by this Court in this wise:
250
RECTO, GAYLE ANGELI M.
2011-0008 | AUSL
Personal Notes on Remedial Law 2 Review (based on the syllabus of Prof. Henedino M. Brondial)

receive 10% of the net profits, in addition to her wages and other o First, petitioners filed a compulsory counterclaim11 against
remuneration from the business. respondents in their answer before the trial court, and with the
o Allegedly, from the time that Shellite opened for business on July 8, filing of their counterclaim, petitioners themselves effectively
1977, its business operation went quite and was profitable. removed this case from the ambit of the "Dead Man's
Respondent claimed that he could attest to success of their Statute".12 Well entrenched is the rule that when it is the
business because of the volume of orders and deliveries of filled executor or administrator or representatives of the estates
Shellane cylinder tanks supplied by Pilipinas Shell Petroleum that sets up the counterclaim, the plaintiff, herein
Corporation. While Jacinto furnished respondent with the respondent, may testify to occurrences before the death of
merchandise inventories, balance sheets and net worth of Shellite the deceased to defeat the counterclaim.13 Moreover, as
from 1977 to 1989, respondent however suspected that the defendant in the counterclaim, respondent is not disqualified from
amount indicated in these documents were understated and testifying as to matters of facts occurring before the death of the
undervalued by Jacinto and Josephine for their own selfish reasons deceased, said action not having been brought against but by the
and for tax avoidance. estate or representatives of the deceased.14
x Cecilia and Lilibeth Æ filed an MD on the ground of lack of jurisdiction over the o Second, the testimony of Josephine is not covered by the
SM "Dead Man's Statute" for the simple reason that she is not "a
o the Securities and Exchange Commission (SEC) in Manila, not the party or assignor of a party to a case or persons in whose
Regional Trial Court in Zamboanga del Norte had jurisdiction over behalf a case is prosecuted." Records show that respondent
the action offered the testimony of Josephine to establish the existence of the
x RTC Æ denied the MD partnership between respondent and Jacinto. Petitioners' insistence
x Cecilia and Lilibeth Æ filed their answer that Josephine is the alter ego of respondent does not make her an
o contending that they are not liable for partnership shares, assignor because the term "assignor" of a party means "assignor of
unreceived income/profits, interests, damages and attorney's fees, a cause of action which has arisen, and not the assignor of a right
that respondent does not have a cause of action against them, and assigned before any cause of action has arisen."15 Plainly then,
that the trial court has no jurisdiction over the nature of the action, Josephine is merely a witness of respondent, the latter being the
the SEC being the agency that has original and exclusive party plaintiff.
jurisdiction over the case. x We are not convinced by petitioners' allegation that Josephine's testimony lacks
o As counterclaim, petitioner sought attorney's fees and expenses of probative value because she was allegedly coerced coerced by respondent, her
litigation. brother-in-law, to testify in his favor, Josephine merely declared in court that she
x Cecilia and Lilibeth Æ filed another MD was requested by respondent to testify and that if she were not requested to do
o on the ground that the claim for winding up of partnership affairs, so she would not have testified. We fail to see how we can conclude from this
accounting and recovery of shares in partnership affairs, accounting candid admission that Josephine's testimony is involuntary when she did not in
and recovery of shares in partnership assets/properties should be any way categorically say that she was forced to be a witness of respondent.
dismissed and prosecuted agains x Also, the fact that Josephine is the sister of the wife of respondent does not
x RTC Æ denied diminish the value of her testimony since relationship per se, without more, does
x Cecilia and Lilibeth Æ filed a Rule 65 before the CA not affect the credibility of witnesses.16
x CA Æ dismissed the petition x Petitioners' reliance alone on the "Dead Man's Statute" to defeat
x Trial before the RTC ensued respondent's claim cannot prevail over the factual findings of the trial
x RTC Æ ruled in favor of CHUA court and the Court of Appeals that a partnership was established
x Cecilia and Lilibeth Æ appealed to CA between respondent and Jacinto. Based not only on the testimonial evidence,
o CA Æ dismissed appeal; denied ensuing MR but the documentary evidence as well, the trial court and the Court of Appeals
x Cecilia and Lilibeth Æ filed a Rule 45 before the SC considered the evidence for respondent as sufficient to prove the formation of
o Petitioners question the correctness of the finding of the trial court partnership, albeit an informal one.
and the Court of Appeals that a partnership existed between x Notably, petitioners did not present any evidence in their favor during trial. By
respondent and Jacinto from 1977 until Jacinto's death. the weight of judicial precedents, a factual matter like the finding of the
o In the absence of any written document to show such partnership existence of a partnership between respondent and Jacinto cannot be inquired
between respondent and Jacinto, petitioners argues that these into by this Court on review.17 This Court can no longer be tasked to go over the
courts were proscribes from hearing the testimonies of respondent proofs presented by the parties and analyze, assess and weigh them to ascertain
and his witness, Josephine, to prove the alleged partnership three if the trial court and the appellate court were correct in according superior credit
years after Jacinto's death to this or that piece of evidence of one party or the other.18 It must be also
o Petitioners thus implore this Court to rule that the testimonies of pointed out that petitioners failed to attend the presentation of evidence of
respondent and his alter ego, Josephine, should not have been respondent. Petitioners cannot now turn to this Court to question the
admitted to prove certain claims against a deceased person admissibility and authenticity of the documentary evidence of respondent when
(Jacinto), now represented by petitioners. petitioners failed to object to the admissibility of the evidence at the time that
such evidence was offered.19
ISSUE: Whether the testimonies of Chua and his witness Josephine were correctly admitted by x With regard to petitioners' insistence that laches and/or prescription should have
the RTC for the purpose of proving the existence of the partnership, considering that it was extinguished respondent's claim, we agree with the trial court and the Court of
given 3 years after the death of the supposed partner, Jacinto. Appeals that the action for accounting filed by respondents three (3) years after
Jacinto's death was well within the prescribed period. The Civil Code provides
HELD: YES. that an action to enforce an oral contract prescribes in six (6) years20 while the
x A partnership may be constituted in any form, except where immovable property right to demand an accounting for a partner's interest as against the person
of real rights are contributed thereto, in which case a public instrument shall continuing the business accrues at the date of dissolution, in the absence of any
necessary.6 Hence, based on the intention of the parties, as gathered from the contrary agreement.21 Considering that the death of a partner results in the
facts and ascertained from their language and conduct, a verbal contract of dissolution of the partnership22, in this case, it was Jacinto's death that
partnership may arise.7 The essential profits that must be proven to that a respondent as the surviving partner had the right to an account of his interest as
partnership was agreed upon are (1) mutual contribution to a common stock, against petitioners. It bears stressing that while Jacinto's death dissolved the
and (2) a joint interest in the profits.8 Understandably so, in view of the absence partnership, the dissolution did not immediately terminate the partnership. The
of the written contract of partnership between respondent and Jacinto, Civil Code23 expressly provides that upon dissolution, the partnership continues
respondent resorted to the introduction of documentary and testimonial evidence and its legal personality is retained until the complete winding up of its business,
to prove said partnership. The crucial issue to settle then is to whether or not the culminating in its termination.24
"Dead Man's Statute" applies to this case so as to render inadmissible x In a desperate bid to cast doubt on the validity of the oral partnership between
respondent's testimony and that of his witness, Josephine. respondent and Jacinto, petitioners maintain that said partnership that had initial
x The "Dead Man's Statute" provides that if one party to the alleged capital of P200,000.00 should have been registered with the Securities and
transaction is precluded from testifying by death, insanity, or other Exchange Commission (SEC) since registration is mandated by the Civil Code,
mental disabilities, the surviving party is not entitled to the undue True, Article 1772 of the Civil Code requires that partnerships with a capital of
advantage of giving his own uncontradicted and unexplained account of P3,000.00 or more must register with the SEC, however, this registration
the transaction.9 But before this rule can be successfully invoked to bar the requirement is not mandatory. Article 1768 of the Civil Code25 explicitly provides
introduction of testimonial evidence, it is necessary that: that the partnership retains its juridical personality even if it fails to register. The
o "1. The witness is a party or assignor of a party to case or persons failure to register the contract of partnership does not invalidate the same as
in whose behalf a case in prosecuted. among the partners, so long as the contract has the essential requisites, because
o 2. The action is against an executor or administrator or other the main purpose of registration is to give notice to third parties, and it can be
representative of a deceased person or a person of unsound mind; assumed that the members themselves knew of the contents of their contract.26
o 3. The subject-matter of the action is a claim or demand against In the case at bar, non-compliance with this directory provision of the law will
the estate of such deceased person or against person of unsound not invalidate the partnership considering that the totality of the evidence proves
mind; that respondent and Jacinto indeed forged the partnership in question.
o 4. His testimony refers to any matter of fact of which occurred
before the death of such deceased person or before such person TERESITA P. BORDALBA, petitioner, vs. COURT OF APPEALS, HEIRS OF NICANOR
became of unsound mind."10 JAYME, namely, CANDIDA FLORES, EMANNUEL JAYME, DINA JAYME DEJORAS, EVELIA
x Two reasons forestall the application of the "Dead Man's Statute" to JAYME, and GESILA JAYME; AND HEIRS OF ASUNCION JAYME-BACLAY, namely, ANGELO
this case. JAYME-BACLAY, CARMEN JAYME-DACLAN and ELNORA JAYME BACLAY, respondents.
G.R. No. 112443 January 25, 2002
251
RECTO, GAYLE ANGELI M.
2011-0008 | AUSL
Personal Notes on Remedial Law 2 Review (based on the syllabus of Prof. Henedino M. Brondial)

FIRST DIVISION x The Court sees no reason to deviate from the findings of the trial court that
petitioner resorted to fraud and misrepresentation in obtaining a free patent and
FACTS: title over the lot under scrutiny. The Court of Appeals correctly pointed out that
x The land located in Mandaue City, Cebu was originally owned by the late spouses misrepresentation tainted petitioner’s application, insofar as her declaration that
Carmeno Jayme and Margarita Espina de Jayme the land applied for was not occupied or claimed by any other person. Her
x Upon their death, an EJ partition was executed: declaration is belied by the extra-judicial partition which she acknowledged, her
o 1) 1/3 in favor of - (a) their grandchild Nicanor Jayme, the mother’s aborted attempt to have the lot registered, private respondents’
deceased spouse of private respondent Candida Flores and the predecessors-in-interest’s opposition thereto, and by the occupancy of a portion
father of private respondents Emmanuel, Dina, Evelia and Gesila, of the said lot by Nicanor Jayme and his family since 1945.
all surnamed Jayme; and (b) their grandchild Asuncion Jayme- x It is a settled rule that the Land Registration Act protects only holders of title in
Baclay, whose heirs are private respondents Agelio Baclay, Elnora good faith, and does not permit its provision to be used as a shield for the
Baclay and Carmen Jayme-Daclan; commission of fraud, or as a means to enrich oneself at the expense of others.17
o 2) 1/3 to their daughter Elena Jayme Vda. de Perez, mother of x As to the alleged violation of the dead man’s statute,18 suffice it to state that
petitioner Teresita P. Bordalba; and said rule finds no application in the present case. The dead man’s statute
o 3) 1/3 to an unidentified party. does not operate to close the mouth of a witness as to any matter of
x Elena Jayme Vda. de Perez Æ filed with the RTC Cebu amended application for fact coming to his knowledge in any other way than through personal
the registration of the land dealings with the deceased person, or communication made by the
o alleged that the lot sought to be registered was originally a part of deceased to the witness.19
a land owned by her late parents, the spouses Carmeno Jayme and x Since the claim of private respondents and the testimony of their witnesses in
Margarita Espina de Jayme; and that 1/3 of said land was the present case is based, inter alia, on the 1947 Deed of Extra-judicial Partition
adjudicated to her in an extra-judicial partition. and other documents, and not on dealings and communications with the
o She further stated that a portion of the lot for which title is applied deceased, the questioned testimonies were properly admitted by the trial court.
for is occupied by Nicanor Jayme with her permission.
x Nicanor Jayme and Asuncion Jayme-Baclay Æ filed their opposition iv. Privileged Communication
o contending that said application included the 1/3 portion inherited
by them in the 1947 extra-judicial partition. Section 24. Disqualification by reason of privileged communication. — The following
x RTC Æ dismissed the action for lack of interest of the parties persons cannot testify as to matters learned in confidence in the following cases:
x Teresita (daughter of Elena) Æ filed with the Bureau of Lands of Cebu City an
application seeking the issuance of a Free Patent over the same lot subject of the (a) The husband or the wife, during or after the marriage, cannot be examined
aborted application of her mother, Elena Jayme, now known as Lot No. 1242 without the consent of the other as to any communication received in confidence by one from
(799-C) the other during the marriage except in a civil case by one against the other, or in a criminal
x Bureau Æ granted the application case for a crime committed by one against the other or the latter's direct descendants or
o Free Patent No. (VII-I) 11421 and Original Certificate of Title No. 0-
ascendants;
571 (FP) over said lot
x Teresita caused the subdivision of the land (b) An attorney cannot, without the consent of his client, be examined as to any
x Heirs of Nicanor Æ then filed before the RTC Mandaue an action against Teresita communication made by the client to him, or his advice given thereon in the course of, or with
Bordalba, spouses Genaro U. Cabahug, and Rita Capala, Rural Bank of Mandaue a view to, professional employment, nor can an attorney's secretary, stenographer, or clerk be
and the Director of the Bureau of Lands. examined, without the consent of the client and his employer, concerning any fact the
o prayed that Free Patent No. (VII-I) 11421 and OCT No. 0-571 (FP), knowledge of which has been acquired in such capacity;
as well as TCT Nos. 22771-22776 be declared void and ordered
cancelled. (c) A person authorized to practice medicine, surgery or obstetrics cannot in a civil
o Private respondents also prayed that they be adjudged owners of case, without the consent of the patient, be examined as to any advice or treatment given by
Lot No. 1242 (799-C), and that spouses Genaro V. Cabahug and him or any information which he may have acquired in attending such patient in a professional
Rita Capala as well as the Rural Bank of Mandaue be declared capacity, which information was necessary to enable him to act in capacity, and which would
buyers and mortgagee in bad faith, respectively. blacken the reputation of the patient;
o In addition, they asked the court to award them actual,
compensatory, and moral damages plus attorney’s fees in the (d) A minister or priest cannot, without the consent of the person making the
amount of P20,000.00. confession, be examined as to any confession made to or any advice given by him in his
x Teresita Æ filed her answer
professional character in the course of discipline enjoined by the church to which the minister
o Lot No. 1242 (799-C) was acquired by her through purchase from or priest belongs;
her mother,11 who was in possession of the lot in the concept of
an owner since 1947. (e) A public officer cannot be examined during his term of office or afterwards, as to
o In her answer, petitioner traced her mother’s ownership of the lot
communications made to him in official confidence, when the court finds that the public
partly from the 1947 deed of extra-judicial partition presented by interest would suffer by the disclosure. (21a)
private respondents,12 and claimed that Nicanor Jayme, and
Candida Flores occupied a portion of Lot No. 1242 (799-C) by mere ANNOTATIONS
tolerance of her mother.
o On cross-examination, petitioner admitted that the properties of
the late Carmeno Jayme and Margarita Espina de Jayme were x Objections under Section 24 may only be invoked by the persons
partitioned by their heirs in 1947, but claimed that she was not mentioned herein
aware of the existence of said Deed of Extra-judicial Partition. She, x Section 24(a) Æ Marital Privilege
however, identified one of the signatures in the said Deed to be the o Requisites:
signature of her mother ƒ There was a valid marital relation
x RTC Æ in favor of the HEIRS
ƒ Privilege is invoked with respect to a
o declared said patent and title void and ordered its cancellation.
However, it declared that spouses Genaro U. Cabahug and Rita confidential communication between the
Capala as well as the Rural Bank of Mandaue are purchasers and spouses during the marriage
mortgagee in good faith, respectively; and consequently upheld as ƒ The spouse against whom such evidence is
valid the sale of Lot No. 1242-A covered by Transfer Certificate of being offered has not given his or her consent
Title No. 22771 (FP) to spouses Genaro U. Cabahug and Rita to such testimony
Capala, and the mortgage of Lot No. 1242-B covered by TCT No.
o Term of privilege: during and after the marriage
22772 in favor of the Rural Bank of Mandaue
x BOTH Teresita AND the HeirsÆ appealed to CA o XPNs to the rule:
x CA Æ affirmed with modification the decision of RTC ƒ Civil case by one against the other
o It ruled that since private respondents are entitled only to 1/3 ƒ Criminal case for a crime committed by one
portion of Lot No. 1242 (799-C), petitioner should be ordered to against the other or the latter's direct
reconvey 1/3 of Lot No. 1242 (799-C) to private respondents descendants or ascendants;
x Teresita Æ filed a Rule 45 before the SC
o Privilege may NOT be invoked in the ff instances:
o Petitioner contends that the testimonies given by the witnesses for
ƒ Where it was not intended to be kept in
private respondents which touched on matters occurring prior to the
death of her mother should not have been admitted by the trial confidence by the spouse who received the
court, as the same violated the dead man’s statute. Likewise, same
petitioner questions the right of private respondents to inherit from ƒ If the communication is heard or comes into
the late Nicanor Jayme and Asuncion Jayme-Baclay, as well as the the hands of 3rd persons, whether legally or
identity between the disputed lot and the parcel of land adjudicated
not
in the Deed of Extra-judicial Partition.
x REASON: while the spouse is
ISSUE: Whether the testimonies of the witnesses of the Heirs violated the Dead Man’s Statute. covered by the prohibition, a 3rd
person is not and the latter can
HELD: NO. testify thereon
252
RECTO, GAYLE ANGELI M.
2011-0008 | AUSL
Personal Notes on Remedial Law 2 Review (based on the syllabus of Prof. Henedino M. Brondial)

ƒ Communication was not given in confidence


Spousal Immunity/ Marital ƒ Communication is irrelevant to the
Marital Privilege
Disqualification professional employment
Section 22 Section 24(a) ƒ Communication was made for an unlawful
this privilege may be claimed purpose
may only be invoked when one
whether or not the spouse is a ƒ The information was intended to be made
spouse is a party to the action
party to the action public
applies only when the marriage is ƒ There was a waiver of the privilege either by
may be claimed even after the
existing at the time the testimony is
marriage has been dissolved
a contract or law
being offered ƒ In cases falling under Rule 28 where the
constitutes a prohibition against applies only to confidential results of the physical and mental examination
any testimony for or against the communication between the of a person are intended to be made public by
spouse of the witness spouses an order by the court to be divulged in that
proceeding
x Section 24(b) Æ Attorney-Client Privilege ƒ Results of autopsies or postmortem
o Requisites: examinations which are generally intended to
ƒ FIRST PROHIBITION: be divulged in court
x There is attorney-client relation x Section 24(d) Æ Minister/Priest-Penitent Privilege
o Attorney must have o Requisites:
been consulted in his ƒ Communication was made pursuant to a
professional capacity religious duty enjoined in the course of
o This includes discipline of the sect or denomination to which
preliminary they belong
communication made ƒ Must be confidential and penitential in
for the purpose of character
creating an attorney- x Section 24(e) Æ Privilege of Public Officers
client relationship o Requisites:
x The privilege is invoked with ƒ Communication was made to the public officer
respect to a confidential in official confidence
communication between them in ƒ Public interest would suffer by the disclosure
the course of professional of such communication
employment x Special laws which provide for privileges of a similar nature:
x The client has not given his o RA 53 as amended by RA 1477 Æ publisher, editor or
consent duly accredited reporters of any newspaper, magazine or
ƒ SECOND PROHIBITION: periodical cannot be compelled to reveal their sources
x The attorney’s secretary, o Art 223 of Labor Code Æ information and statements
stenographer or clerk is sought to made during conciliation proceedings shall be treated as
be examined privileged and conciliators and similar official shall not
x With respect to any fact the testify in any court or tribunal with respect to any matter
knowledge of which has been taken up during such conciliation proceedings
acquired in such capacity
x The client and the attorney have
not given their consent
o CANNOT be invoked in cases where communications are: CASES
ƒ Intended to be made public
ƒ Intended to be communicated to others JOSIELENE LARA CHAN, Petitioner, vs. JOHNNY T. CHAN, Respondent.
ƒ Intended for unlawful purpose G.R. No. 179786 July 24, 2013
THIRD DIVISION
ƒ Received from 3rd persons not acting in behalf
or as agents of the client FACTS:
ƒ Made in the presence of 3rd parties who are x Josielene Æ filed a petition for declaration of nullity of marriage against Johnny
strangers to the attorney-client relationship before RTC Makati
o Term of privilege: when the privileged communication o claimed that Johnny failed to care for and support his family and
was made by the client to the attorney and even after the that a psychiatrist diagnosed him as mentally deficient due to
incessant drinking and excessive use of prohibited drugs. Indeed,
client-attorney relation ceases to exist she had convinced him to undergo hospital confinement for
o Privilege may NOT be invoked in the ff instances:
detoxification and rehabilitation.
ƒ When the communication is for an unlawful x Johnny Æ filed his answer
purpose o claiming that it was Josielene who failed in her wifely duties. To
ƒ When the attorney himself was a conspirator save their marriage, he agreed to marriage counseling but when he
in the commission of the crime and Josielene got to the hospital, two men forcibly held him by
both arms while another gave him an injection. The marriage
o Includes verbal statements and documents or papers relations got worse when the police temporarily detained Josielene
entrusted to the attorney for an unrelated crime and released her only after the case against
x Section 24(c) Æ Physician-Patient Privilege her ended. By then, their marriage relationship could no longer be
o Requisites: repaired.
ƒ The physician is authorized to practice x Josielene Æ filed a request for the issuance of a subpoena duces tecum
medicine, surgery or obstetrics addressed to Medical City, covering Johnny’s medical records when he was there
confined.
ƒ The information was acquired or the advice or o The request was accompanied by a motion to "be allowed to
treatment was given by him in his submit in evidence" the records sought by subpoena duces tecum
professional capacity for the purpose of x Johnny Æ filed his opposition
treating and curing the patient o arguing that the medical records were covered by physician-patient
ƒ The information, advice, or treatment, if privilege
revealed, would blacken the reputation of the x RTC Æ denied the motion; denied ensuing MR
x Josielene Æ filed a Rule 65 before the CA
patient x CA Æ dismissed the petition
ƒ The privilege is invoked in a civil case, o It ruled that, if courts were to allow the production of medical
whether the patient is a party thereto or not records, then patients would be left with no assurance that
o Not necessary that the physical-patient relation was whatever relevant disclosures they may have made to their
created by the voluntary act of the patient physicians would be kept confidential. The prohibition covers not
o Privilege may NOT be invoked in the ff instances: only testimonies, but also affidavits, certificates, and pertinent

253
RECTO, GAYLE ANGELI M.
2011-0008 | AUSL
Personal Notes on Remedial Law 2 Review (based on the syllabus of Prof. Henedino M. Brondial)

hospital records. The CA added that, although Johnny can waive x To allow, however, the disclosure during discovery procedure of the
the privilege, he did not do so in this case. He attached the hospital records—the results of tests that the physician ordered, the
Philhealth form to his answer for the limited purpose of showing his diagnosis of the patient’s illness, and the advice or treatment he gave
alleged forcible confinement. him—would be to allow access to evidence that is inadmissible without
x Josielene Æ filed a Rule 45 before the SC the patient’s consent. Physician memorializes all these information in the
patient’s records. Disclosing them would be the equivalent of compelling the
ISSUE: Whether the RTC and CA correctly denied the request for the release of Johnny’s physician to testify on privileged matters he gained while dealing with the
medical records. patient, without the latter’s prior consent.
x 3. Josielene argues that since Johnny admitted in his answer to the petition
HELD: YES. before the RTC that he had been confined in a hospital against his will and in
x Josielene requested the issuance of a subpoena duces tecum covering the fact attached to his answer a Philhealth claim form covering that confinement, he
hospital records of Johnny’s confinement, which records she wanted to present in should be deemed to have waived the privileged character of its records.
court as evidence in support of her action to have their marriage declared a Josielene invokes Section 17, Rule 132 of the Rules of Evidence that provides:
nullity. Respondent Johnny resisted her request for subpoena, however, invoking o SEC. 17. When part of transaction, writing or record given in
the privileged character of those records. He cites Section 24(c), Rule 130 of the evidence, the remainder admissible.— When part of an act,
Rules of Evidence which reads: declaration, conversation, writing or record is given in evidence by
o SEC. 24. Disqualification by reason of privileged communication.— one party, the whole of the same subject may be inquired into by
The following persons cannot testify as to matters learned in the other, and when a detached act, declaration, conversation,
confidence in the following cases: writing or record is given in evidence, any other act, declaration,
ƒ (c) A person authorized to practice medicine, surgery conversation, writing or record necessary to its understanding may
or obstetrics cannot in a civil case, without the also be given in evidence.1âwphi1
consent of the patient, be examined as to any advice x But, trial in the case had not yet begun. Consequently, it cannot be said that
or treatment given by him or any information which Johnny had already presented the Philhealth claim form in evidence, the act
he may have acquired in attending such patient in a contemplated above which would justify Josielene into requesting an inquiry into
professional capacity, which information was the details of his hospital confinement. Johnny was not yet bound to adduce
necessary to enable him to act in that capacity, and evidence in the case when he filed his answer. Any request for disclosure of his
which would blacken the reputation of the patient. hospital records would again be premature.
x The physician-patient privileged communication rule essentially means x For all of the above reasons, the CA and the RTC were justified in denying
that a physician who gets information while professionally attending a Josielene her request for the production in court of Johnny’s hospital records.
patient cannot in a civil case be examined without the patient’s
consent as to any facts which would blacken the latter’s reputation. JUDGE UBALDINO A. LACUROM, Presiding Judge, Regional Trial Court, Cabanatuan City,
This rule is intended to encourage the patient to open up to the Branch 29 and Pairing Judge, Branch 30, Complainant, vs. ATTY. ELLIS F. JACOBA and
physician, relate to him the history of his ailment, and give him access ATTY. OLIVIA VELASCO-JACOBA, Respondents.
to his body, enabling the physician to make a correct diagnosis of that A.C. No. 5921 March 10, 2006
ailment and provide the appropriate cure. Any fear that a physician could THIRD DIVISION
be compelled in the future to come to court and narrate all that had transpired
between him and the patient might prompt the latter to clam up, thus putting his FACTS:
own health at great risk.4 x There was an unlawful detainer case pending before the MTC Cabanatuan
x 1. The case presents a procedural issue, given that the time to object to the entitled Veneracion vs. Barrientos
admission of evidence, such as the hospital records, would be at the time they x Jacoba-Velasco-Jacoba Law Firm is counsel for Veneraction
are offered. The offer could be made part of the physician’s testimony or as x MTC Æ ruled in favor of Veneracion
independent evidence that he had made entries in those records that concern x Barrientos Æ appealed to RTC
the patient’s health problems. x RTC (presided by Judge Lacurom) Æ reversed MTC
x Section 36, Rule 132, states that objections to evidence must be made after the x Veneracion (through counsel Atty. Olivia Jacoba) Æ filed an MR (with Request
offer of such evidence for admission in court. Thus: for Inhibition)
o SEC. 36. Objection.— Objection to evidence offered orally must be o This RESOLUTION of REVERSAL is an ABHORRENT NULLITY as it is
made immediately after the offer is made. entirely DEVOID of factual and legal basis. It is a Legal
o Objection to a question propounded in the course of the oral MONSTROSITY in the sense that the Honorable REGIONAL TRIAL
examination of a witness shall be made as soon as the grounds COURT acted as if it were the DARAB (Dept. of Agrarian Reform
therefor shall become reasonably apparent. ADJUDICATION BOARD)! x x x HOW HORRIBLE and TERRIBLE!
o An offer of evidence in writing shall be objected to within three (3) The mistakes are very patent and glaring! x x x
days after notice of the offer unless a different period is allowed by o The Resolution] assumes FACTS that have not been established
the court. and presumes FACTS not part of the records of the case, all
o In any case, the grounds for the objections must be specified. "loaded" in favor of the alleged "TENANT." Clearly, the
x Since the offer of evidence is made at the trial, Josielene’s request for RESOLUTION is an INSULT to the Judiciary and an ANACHRONISM
subpoena duces tecum is premature. She will have to wait for trial to in the Judicial Process. Need we say more?
begin before making a request for the issuance of a subpoena duces o Another HORRIBLE ERROR! Even an average Law Student knows
tecum covering Johnny’s hospital records. It is when those records are that JURISDICTION is determined by the averments of the
produced for examination at the trial, that Johnny may opt to object, COMPLAINT and not by the averments in the answer! This is
not just to their admission in evidence, but more so to their disclosure. backed up by a Litany of Cases!
Section 24(c), Rule 130 of the Rules of Evidence quoted above is about non- x Judge Lacurom Æ ordered Atty. Olivia to appear before his sala and show cause
disclosure of privileged matters. why she should not be held liable for contempt
x 2. It is of course possible to treat Josielene’s motion for the issuance of a x Atty Olivia Æ filed her comment
subpoena duces tecum covering the hospital records as a motion for production o Averred that she only signed the subject MR but DID NOT actually
of documents, a discovery procedure available to a litigant prior to trial. Section take part in writing it
1, Rule 27 of the Rules of Civil Procedure provides: o It was her husband, Atty. Ellis who wrote the MR
o SEC. 1. Motion for production or inspection; order.— Upon motion o She recounted that on her way out of the house for an afternoon
of any party showing good cause therefor, the court in which an hearing, Atty. Ellis Jacoba ("Jacoba") stopped her and said "O,
action is pending may (a) order any party to produce and permit pirmahan mo na ito kasi last day na, baka mahuli." (Sign this as it
the inspection and copying or photographing, by or on behalf of the is due today, or it might not be filed on time.) She signed the
moving party, of any designated documents, papers, books, pleading handed to her without reading it, in "trusting blind faith"
accounts, letters, photographs, objects or tangible things, not on her husband of 35 years with whom she "entrusted her whole
privileged, which constitute or contain evidence material to any life and future."17
matter involved in the action and which are in his possession, o This pleading turned out to be the 30 July 2001 motion which
custody or control; or (b) order any party to permit entry upon Jacoba drafted but could not sign because of his then suspension
designated land or other property in his possession or control for from the practice of law.
the purpose of inspecting, measuring, surveying, or photographing x Judge Lacurom Æ held Atty. Olivia guilty of contempt
the property or any designated relevant object or operation x Atty. Olivia Æ filed an MR
thereon. The order shall specify the time, place and manner of x Judge Lacurom Æ issued an order directing Atty. Ellis to show cause why he
making the inspection and taking copies and photographs, and may should not be indicted for contempt
prescribe such terms and conditions as are just. (Emphasis x Atty Ellis Æ filed his comment
supplied) o he denied that he typed or prepared the 30 July 2001 motion.
x But the above right to compel the production of documents has a o Against Velasco-Jacoba’s statements implicating him, Jacoba
limitation: the documents to be disclosed are "not privileged." x invoked the marital privilege rule in evidence
Josielene of course claims that the hospital records subject of this case are not x Judge Lacurom Æ filed an administrative case against the 2 before the IBP
privileged since it is the "testimonial" evidence of the physician that may be x IBP Æ issued its resolution
regarded as privileged. Section 24(c) of Rule 130 states that the physician o recommended the suspension of respondents from the practice of
"cannot in a civil case, without the consent of the patient, be examined" law for six months.29 IBP Commissioner Navarro found that
regarding their professional conversation. The privilege, says Josielene, does not "respondents were prone to us[ing] offensive and derogatory
cover the hospital records, but only the examination of the physician at the trial. remarks and phrases which amounted to discourtesy and disrespect
254
RECTO, GAYLE ANGELI M.
2011-0008 | AUSL
Personal Notes on Remedial Law 2 Review (based on the syllabus of Prof. Henedino M. Brondial)

for authority."30 Although the remarks were not directed at Judge applies to Jacoba who impliedly admitted authorship of the 30 July
Lacurom personally, they were aimed at "his position as a judge, 2001 motion.
which is a smack on the judiciary system as a whole x The Code of Professional Responsibility provides:
x Atty Oliva Æ filed an MR o Rule 11.03.—A lawyer shall abstain from scandalous, offensive or
o the present complaint of Judge Lacurom is sub judice; the same menacing language or behavior before the Courts.
issues involved in this case are raised before the Honorable Court o Rule 11.04.—A lawyer shall not attribute to a Judge motives not
of Appeals presently pending in CA-G.R. SP No. 66973 for Certiorari supported by the record or have no materiality to the case.
and Mandatory Inhibition with TRO and Preliminary Injunction x x x No doubt, the language contained in the 30 July 2001 motion greatly exceeded
x; the vigor required of Jacoba to defend ably his client’s cause. We recall his use of
the following words and phrases: abhorrent nullity, legal monstrosity, horrendous
ISSUE: Whether Atty. Ellis may invoke marital privilege against the statements of his wife mistake, horrible error, boner, and an insult to the judiciary and an anachronism
Atty. Oliva in the present case. in the judicial process. Even Velasco-Jacoba acknowledged that the words
created "a cacophonic picture of total and utter disrespect."44
HELD: NO. x Respondents nonetheless try to exculpate themselves by saying that every
x On a preliminary note, we reject Velasco-Jacoba’s contention that the present remark in the 30 July 2001 motion was warranted. We disagree.
complaint should be considered sub judice in view of the petition for certiorari x Well-recognized is the right of a lawyer, both as an officer of the court and as a
and mandatory inhibition with preliminary injunction ("petition for certiorari")35 citizen, to criticize in properly respectful terms and through legitimate channels
filed before the Court of Appeals. the acts of courts and judges.45 However, even the most hardened judge would
x The petition for certiorari, instituted by Veneracion and Velasco-Jacoba on 4 be scarred by the scurrilous attack made by the 30 July 2001 motion on Judge
October 2001, seeks to nullify the following orders issued by Judge Lacurom in Lacurom’s Resolution. On its face, the Resolution presented the facts correctly
Civil Case No. 2836: (1) the Orders dated 26 September 2001 and 9 November and decided the case according to supporting law and jurisprudence. Though a
2001 denying respondents’ respective motions for inhibition; and (2) the 13 lawyer’s language may be forceful and emphatic, it should always be dignified
September 2001 Order which found Velasco-Jacoba guilty of contempt. The and respectful, befitting the dignity of the legal profession.46 The use of
petitioners allege that Judge Lacurom acted "with grave abuse of discretion unnecessary language is proscribed if we are to promote high esteem in the
[amounting] to lack of jurisdiction, in violation of express provisions of the law courts and trust in judicial administration.47
and applicable decisions of the Supreme Court."36 x In maintaining the respect due to the courts, a lawyer is not merely enjoined to
x Plainly, the issue before us is respondents’ liability under the Code of Professional use dignified language but also to pursue the client’s cause through fair and
Responsibility. The outcome of this case has no bearing on the resolution of the honest means, thus:
petition for certiorari, as there is neither identity of issues nor causes of action. o Rule 19.01.—A lawyer shall employ only fair and honest means to
x Neither should the Court’s dismissal of the administrative complaint against attain the lawful objectives of his client and shall not present,
Judge Lacurom for being premature impel us to dismiss this complaint. Judge participate in presenting or threaten to present unfounded criminal
Lacurom’s orders in Civil Case No. 2836 could not be the subject of an charges to obtain an improper advantage in any case or
administrative complaint against him while a petition for certiorari assailing the proceeding.
same orders is pending with an appellate court. Administrative remedies are x Shortly after the filing of the 30 July 2001 motion but before its resolution,
neither alternative nor cumulative to judicial review where such review is Jacoba assisted his client in instituting two administrative cases against Judge
available to the aggrieved parties and the same has not been resolved with Lacurom. As we have earlier noted, Civil Case No. 2836 was then pending before
finality. Until there is a final declaration that the challenged order or judgment is Judge Lacurom’s sala. The Court’s attention is drawn to the fact that the timing of
manifestly erroneous, there will be no basis to conclude whether the judge is the filing of these administrative cases could very well raise the suspicion that the
administratively liable.37 cases were intended as leverage against Judge Lacurom.
x The respondents are situated differently within the factual setting of this case. x Respondent spouses have both been the subject of administrative cases before
The corresponding implications of their actions also give rise to different this Court. In Administrative Case No. 2594, we suspended Jacoba from the
liabilities. We first examine the charge against Velasco-Jacoba. practice of law for a period of six months because of "his failure to file an action
x There is no dispute that the genuine signature of Velasco-Jacoba appears on the for the recovery of possession of property despite the lapse of two and a half
30 July 2001 motion. Velasco-Jacoba’s responsibility as counsel is governed by years from receipt by him of P550 which his client gave him as filing and sheriff’s
Section 3, Rule 7 of the Rules of Court: fees."48 In Administrative Case No. 5505, Jacoba was once again found remiss
o SEC. 3.Signature and address.—Every pleading must be signed by in his duties when he failed to file the appellant’s brief, resulting in the dismissal
the party or counsel representing him x x x. of his client’s appeal. We imposed the penalty of one year suspension.49
o The signature of counsel constitutes a certificate by him that he x As for Velasco-Jacoba, only recently this Court fined her P5,000 for appearing in
has read the pleading, that to the best of his knowledge, barangay conciliation proceedings on behalf of a party, knowing fully well the
information, and belief there is good ground to support it, and that prohibition contained in Section 415 of the Local Government Code.50
it is not interposed for delay. x In these cases, the Court sternly warned respondents that a repetition of similar
o x x x Counsel who x x x signs a pleading in violation of this Rule, or acts would merit a stiffer penalty. Yet, here again we are faced with the question
alleges scandalous or indecent matter therein x x x shall be subject of whether respondents have conducted themselves with the courtesy and
to appropriate disciplinary action. (Emphasis supplied) candor required of them as members of the bar and officers of the court. We
x By signing the 30 July 2001 motion, Velasco-Jacoba in effect certified that she find respondents to have fallen short of the mark.
had read it, she knew it to be meritorious, and it was not for the purpose of
delaying the case. Her signature supplied the motion with legal effect and
elevated its status from a mere scrap of paper to that of a court document.
x Velasco-Jacoba insists, however, that she signed the 30 July 2001 motion only CLARITA J. SAMALA, Complainant, vs. ATTY. LUCIANO D. VALENCIA, Respondent.
because of her husband’s request but she did not know its contents beforehand. A.C. No. 5439 January 22, 2007
Apparently, this practice of signing each other’s pleadings is a long-standing EN BANC
arrangement between the spouses. According to Velasco-Jacoba, "[s]o implicit is
[their] trust for each other that this happens all the time. Through the years, FACTS:
[she] already lost count of the number of pleadings prepared by one that is x Samala filed a disbarment case against Atty. Valencia on the ff grounds:
signed by the other."38 By Velasco-Jacoba’s own admission, therefore, she o (a) serving on two separate occasions as counsel for contending
violated Section 3 of Rule 7. This violation is an act of falsehood before the parties;
courts, which in itself is a ground for subjecting her to disciplinary action, ƒ Records further reveal that at the hearing of
independent of any other ground arising from the contents of the 30 July 2001 November 14, 2003, respondent admitted that in Civil
motion.39 Case No. 95-105-MK, he was the lawyer for Lagmay
x We now consider the evidence as regards Jacoba. His name does not appear in (one of the tenants) but not for Bustamante and
the 30 July 2001 motion. He asserts the inadmissibility of Velasco-Jacoba’s Bayuga 10 albeit he filed the Explanation and
statement pointing to him as the author of the motion. Compliance for and in behalf of the tenants. 11
x The Court cannot easily let Jacoba off the hook. Firstly, his Answer with Second Respondent also admitted that he represented Valdez
Motion for Inhibition did not contain a denial of his wife’s account. Instead, in Civil Case No. 98-6804 and SCA Case No. 99-341-
Jacoba impliedly admitted authorship of the motion by stating that he "trained MK against Bustamante and her husband but denied
his guns and fired at the errors which he perceived and believed to be gigantic being the counsel for Alba although the case is
and monumental."40 entitled "Valdez and Alba v. Bustamante and her
x Secondly, we find Velasco-Jacoba’s version of the facts more plausible, for two husband," because Valdez told him to include Alba as
reasons: (1) her reaction to the events was immediate and spontaneous, unlike the two were the owners of the property 12 and it
Jacoba’s defense which was raised only after a considerable time had elapsed was only Valdez who signed the complaint for
from the eruption of the controversy; and (2) Jacoba had been counsel of record ejectment. 13 But, while claiming that respondent did
for Veneracion in Civil Case No. 2836, supporting Velasco-Jacoba’s assertion that not represent Alba, respondent, however, avers that
she had not "actually participate[d]" in the prosecution of the case. he already severed his representation for Alba when
x Moreover, Jacoba filed a Manifestation in Civil Case No. 2836, praying that Judge the latter charged respondent with estafa. 14 Thus,
Lacurom await the outcome of the petition for certiorari before deciding the the filing of Civil Case No. 2000-657-MK against Alba.
contempt charge against him.41 This petition for certiorari anchors some of its o (b) knowingly misleading the court by submitting false
arguments on the premise that the motion was, in fact, Jacoba’s handiwork.42 documentary evidence;
x The marital privilege rule, being a rule of evidence, may be waived by o (c) initiating numerous cases in exchange for nonpayment of rental
failure of the claimant to object timely to its presentation or by any fees; and
conduct that may be construed as implied consent.43 This waiver
255
RECTO, GAYLE ANGELI M.
2011-0008 | AUSL
Personal Notes on Remedial Law 2 Review (based on the syllabus of Prof. Henedino M. Brondial)

o (d) having a reputation of being immoral by siring illegitimate circumstances indicating conflict of interests, and to behave at all times with
children. circumspection and dedication befitting a member of the Bar, especially
x Commissioner Raval Æ recommended the penalty of suspension for six months. observing candor, fairness and loyalty in all transactions with his clients.
x IBP Board of Governors Æ approved
COMMISSIONER JOSE T. ALMONTE, VILLAMOR C. PEREZ, NERIO ROGADO, and ELISA
ISSUE: Whether there was a violation of privileged communication. RIVERA, petitioners, vs. HONORABLE CONRADO M. VASQUEZ and CONCERNED CITIZENS,
respondents.
HELD: YES. G.R. No. 95367 May 23, 1995
x Rule 15.03, Canon 15 of the Code of Professional Responsibility provides that a EN BANC
lawyer shall not represent conflicting interests except by written consent of all
concerned given after a full disclosure of the facts. FACTS:
x A lawyer may not, without being guilty of professional misconduct, act as counsel x Jose T. Almonte was formerly Commissioner of the Economic Intelligence and
for a person whose interest conflicts with that of his present or former client. 15 Investigation Bureau (EIIB), while Villamor C. Perez is Chief of the EIIB's Budget
He may not also undertake to discharge conflicting duties any more than he may and Fiscal Management Division
represent antagonistic interests. This stern rule is founded on the principles of x An anonymous letter alleging that funds representing savings from unfilled
public policy and good taste. 16 It springs from the relation of attorney and positions in the EIIB had been illegally disbursed was received by Ombudsman
client which is one of trust and confidence. Lawyers are expected not only to o The letter, purporting to have been written by an employee of the
keep inviolate the client's confidence, but also to avoid the appearance of EIIB and a concerned citizen, was addressed to the Secretary of
treachery and double-dealing for only then can litigants be encouraged to entrust Finance, with copies furnished several government offices,
their secrets to their lawyers, which is of paramount importance in the including the Office of the Ombudsman
administration of justice. 17 x Ombudsman Æ issued an SDT against Almonte and Perez
x One of the tests of inconsistency of interests is whether the acceptance x Almonte and Perez Æ moved to quash the SDT
of a new relation would prevent the full discharge of the lawyer's duty x Ombudsman Æ DENIED the MQ; denied ensuing MR
of undivided fidelity and loyalty to the client or invite suspicion of o He ruled that petitioners were not being forced to produce
unfaithfulness or double-dealing in the performance of that duty. 18 x evidence against themselves, since the subpoena duces tecum was
The stern rule against representation of conflicting interests is founded directed to the Chief Accountant, petitioner Nerio Rogado. In
on principles of public policy and good taste. It springs from the addition the Ombudsman ordered the Chief of the Records a
attorney's duty to represent his client with undivided fidelity and to Section of the EIIB, petitioner Elisa Rivera, to produce before the
maintain inviolate the client's confidence as well as from the injunction investigator "all documents relating to Personnel Service Funds, for
forbidding the examination of an attorney as to any of the privileged the year 1988, and all documents, salary vouchers for the whole
communications of his client. 19 plantilla of the EIIB for 1988, within ten (10) days from receipt
x An attorney owes loyalty to his client not only in the case in which he has hereof."
represented him but also after the relation of attorney and client has terminated. x Almonte and Perez Æ filed a Rule 45 before the SC
20 The bare attorney-client relationship with a client precludes an attorney from
accepting professional employment from the client's adversary either in the same ISSUE: Whether the subject SDT may issue against Almonte and Perez, considering that they
case 21 or in a different but related action. 22 A lawyer is forbidden from are public officials and that the subject documents sought are allegedly of confidential nature.
representing a subsequent client against a former client when the subject matter
of the present controversy is related, directly or indirectly, to the subject matter of HELD: YES.
the previous litigation in which he appeared for the former client. 23 x At common law a governmental privilege against disclosure is recognized with
x We held in Nombrado v. Hernandez 24 that the termination of the relation of respect to state secrets bearing on military, diplomatic and similar matters. This
attorney and client provides no justification for a lawyer to represent an interest privilege is based upon public interest of such paramount importance
adverse to or in conflict with that of the former client. The reason for the rule is as in and of itself transcending the individual interests of a private
that the client's confidence once reposed cannot be divested by the expiration of citizen, even though, as a consequence thereof, the plaintiff cannot
the professional employment. 25 Consequently, a lawyer should not, even after enforce his legal rights. 10
the severance of the relation with his client, do anything which will injuriously x In addition, in the litigation over the Watergate tape subpoena in 1973, the U.S.
affect his former client in any matter in which he previously represented him nor Supreme Court recognized the right of the President to the confidentiality of his
should he disclose or use any of the client's confidences acquired in the previous conversations and correspondence, which it likened to "the claim of
relation. 26 confidentiality of judicial deliberations." Said the Court in United States v. Nixon:
x In this case, respondent's averment that his relationship with Alba has long been o The expectation of a President to the confidentiality of his
severed by the act of the latter of not turning over the proceeds collected in Civil conversations and correspondence, like the claim of confidentiality
Case No. 98-6804, in connivance with the complainant, is unavailing. of judicial deliberations, for example, has all the values to which we
Termination of the attorney-client relationship precludes an attorney from accord deference for the privacy of all citizens and, added to those
representing a new client whose interest is adverse to his former client. Alba may values, is the necessity for protection of the public interest in
not be his original client but the fact that he filed a case entitled "Valdez and Alba candid, objective, and even blunt or harsh opinions in
v. Bustamante and her husband," is a clear indication that respondent is protecting Presidential decision-making. A President and those who assist him
the interests of both Valdez and Alba in the said case. Respondent cannot just must be free to explore alternatives in the process of shaping
claim that the lawyer-client relationship between him and Alba has long been policies and making decisions and to do so in a way many would
severed without observing Section 26, Rule 138 of the Rules of Court wherein the be unwilling to express except privately. These are the
written consent of his client is required. considerations justifying a presumptive privilege for Presidential
x In Gonzales v. Cabucana, Jr., 27 citing the case of Quiambao v. Bamba, 28 we communications. The privilege is fundamental to the operation of
held that: the government and inextricably rooted in the separation of
o The proscription against representation of conflicting interests powers under the Constitution. . .
applies to a situation where the opposing parties are present clients o Thus, the Court for the first time gave executive privilege a
in the same action or in an unrelated action. It is of no moment constitutional status and a new name, although not necessarily a
that the lawyer would not be called upon to contend for one client new birth. 12
that which the lawyer has to oppose for the other client, or that x "The confidentiality of judicial deliberations" mentioned in the opinion of
there would be no occasion to use the confidential information the Court referred to the fact that Justices of the U.S. Supreme Court and
acquired from one to the disadvantage of the other as the two judges of lower federal courts have traditionally treated their working papers
actions are wholly unrelated. It is enough that the opposing parties and judicial notes as private property. A 1977 proposal in the U.S. Congress that
in one case, one of whom would lose the suit, are present clients Justices and judges of lower federal courts "should be encouraged to make such
and the nature or conditions of the lawyer's respective retainers with arrangements as will assure the preservation and eventual availability of their
each of them would affect the performance of the duty of undivided personal papers, especially the deposit of their papers in the same depository
fidelity to both clients. 29 they select for [their] Public Papers" 13 was rebuffed by the Justices who, in a
x Respondent is bound to comply with Canon 21 of the Code of Professional letter to the Chairman of the Subcommittee on Regulation and Government
Responsibility which states that "a lawyer shall preserve the confidences and Information of the U.S. Senate, referred to "difficult concerns respecting the
secrets of his client even after the attorney-client relation is terminated." appropriate separation that must be maintained between the legislative branch
x The reason for the prohibition is found in the relation of attorney and and this Court." 14
client, which is one of trust and confidence of the highest degree. A x There are, in addition to such privileges, statutorily-created ones such as the
lawyer becomes familiar with all the facts connected with his client's case. He Government's privilege to withhold the identity of persons who furnish
learns from his client the weak points of the action as well as the strong ones. information of violations of laws. 15
Such knowledge must be considered sacred and guarded with care. 30 x With respect to the privilege based on state secret, the rule was stated by
x From the foregoing, it is evident that respondent's representation of Valdez and the U.S. Supreme Court as follows:
Alba against Bustamante and her husband, in one case, and Valdez against Alba, o Judicial control over the evidence in a case cannot be abdicated to
in another case, is a clear case of conflict of interests which merits a the caprice of executive officers. Yet we will not go so far as to say
corresponding sanction from this Court. Respondent may have withdrawn his that the court may automatically require a complete disclosure to
representation in Civil Case No. 95-105-MK upon being warned by the court, 31 the judge before the claim of privilege will be accepted in any
but the same will not exculpate him from the charge of representing conflicting case. It may be possible to satisfy the court, from all the
interests in his representation in Civil Case No. 2000-657-MK. circumstances of the case, that there is a reasonable danger that
x Respondent is reminded to be more cautious in accepting professional compulsion of the evidence will expose military matters which, in
employments, to refrain from all appearances and acts of impropriety including the interest of national security, should not be divulged. When this
256
RECTO, GAYLE ANGELI M.
2011-0008 | AUSL
Personal Notes on Remedial Law 2 Review (based on the syllabus of Prof. Henedino M. Brondial)

is the case, the occasion for the privilege is appropriate, and the but only to the extent that it will not reveal covert activities of the agency.
court should not jeopardize the security which the privilege is Above all, there must be a scrupulous protection of the documents delivered.
meant to protect by insisting upon an examination of the evidence, x With these safeguards outlined, it is believed that a satisfactory resolution of the
even by the judge alone, in chambers ..... In each case, the conflicting claims of the parties is achieved. It is not amiss to state that even
showing of necessity which is made will determine how far the matters of national security have been inquired into in appropriate in camera
court should probe in satisfying itself that the occasion for invoking proceedings by the courts. In Lansang v. Garcia 23 this Court held closed door
the privilege is appropriate. Where there is a strong showing of sessions, with only the immediate parties and their counsel present, to
necessity, the claim of privilege should not be lightly accepted, but determine claims that because of subversion there was imminent danger to
even the most compelling necessity cannot overcome the claim of public safety warranting the suspension of the writ of habeas corpus in 1971.
privilege if the court is ultimately satisfied that military secrets are Again in Marcos v. Manglapus 24 the Court met behind closed doors to receive
at stake. A fortiori, where necessity is dubious, a formal claim of military briefings on the threat posed to national security by the return to the
privilege, made under the circumstances of this case, will have to country of the former President and his family. In the United States, a similar
prevail. 16 inquiry into the danger to national security as a result of the publication of
x On the other hand, where the claim of confidentiality does not rest on classified documents on the Vietnam war was upheld by the U.S. Supreme
the need to protect military, diplomatic or other national security Court. 25 We see no reason why similar safeguards cannot be made to enable
secrets but on a general public interest in the confidentiality of his an agency of the Government, like the Office of the Ombudsman, to carry out
conversations, courts have declined to find in the Constitution an its constitutional duty to protect public interests 26 while insuring the
absolute privilege of the President against a subpoena considered confidentiality of classified documents.
essential to the enforcement of criminal laws. 17 x Petitioners contend that under Art. XI, § 13(4) the Ombudsman can act only "in
x In the case at bar, there is no claim that military or diplomatic secrets any appropriate case, and subject to such limitations as may be provided by
will be disclosed by the production of records pertaining to the law" and that because the complaint in this case is unsigned and unverified, the
personnel of the EIIB. Indeed, EIIB's function is the gathering and case is not an appropriate one. This contention lacks merit. As already stated,
evaluation of intelligence reports and information regarding "illegal the Constitution expressly enjoins the Ombudsman to act on any complaint filed
activities affecting the national economy, such as, but not limited to, "in any form or manner" concerning official acts or omissions. Thus, Art. XI, §
economic sabotage, smuggling, tax evasion, dollar salting." 18 12 provides:
Consequently, while in cases which involve state secrets it may be o The Ombudsman and his Deputies, as protectors of the people,
sufficient to determine from the circumstances of the case that there shall act promptly on complaints filed in any form or manner
is reasonable danger that compulsion of the evidence will expose against public officials or employees of the Government, or any
military matters without compelling production, 19 no similar excuse subdivision, agency, or instrumentality thereof, including
can be made for a privilege resting on other considerations. government-owned or controlled corporations and shall in
x Nor has our attention been called to any law or regulation which considers appropriate cases, notify the complainants of the action taken and
personnel records of the EIIB as classified information. To the contrary, COA the result thereof. (Emphasis added)
Circular No. 88-293, which petitioners invoke to support their contention that x Similarly, the Ombudsman Act of 1989 (Rep. Act No. 6770) provides in § 26(2):
there is adequate safeguard against misuse of public funds, provides that the o The Office of the Ombudsman shall receive complaints from any
"only item of expenditure which should be treated strictly confidential" is that source in whatever form concerning an official act or omission. It
which refers to the "purchase of information and payment of rewards." Thus, shall act on the complaint immediately and if it finds the same
part V, No. 7 of the Circular reads: entirely baseless, it shall dismiss the same and inform the
o The only item of expenditure which should be treated as strictly complainant of such dismissal citing the reasons therefor. If it
confidential because it falls under the category of classified finds a reasonable ground to investigate further, it shall first
information is that relating to purchase of information and furnish the respondent public officer or employee with a summary
payment of rewards. However, reasonable records should be of the complaint and require him to submit a written answer within
maintained and kept for inspection of the Chairman, Commission seventy-two hours from receipt thereof. If the answer is found
on Audit or his duly authorized representative. All other satisfactory, it shall dismiss the case. (Emphasis added)
expenditures are to be considered unclassified supported by x Accordingly, in Diaz v. Sandiganbayan 27 the Court held that testimony given at
invoices, receipts and other documents, and, therefore, subject to a fact-finding investigation and charges made in a pleading in a case in court
reasonable inquiry by the Chairman or his duly authorized constituted a sufficient basis for the Ombudsman to commence investigation,
representative. 20 because a formal complaint was really not necessary.
x It should be noted that the regulation requires that "reasonable x Rather than referring to the form of complaints, therefore, the phrase "in an
records" be kept justifying the confidential or privileged character of appropriate case" in Art. XI, § 12 means any case concerning official act or
the information relating to informers. There are no such reasonable omission which is alleged to be "illegal, unjust, improper, or inefficient." 28 The
records in this case to substitute for the records claimed to be phrase "subject to such limitations as may be provided by law" refers to such
confidential. limitations as may be provided by Congress or, in the absence thereof, to such
x The other statutes and regulations 21 invoked by petitioners in support of their limitations as may be imposed by the courts. Such limitations may well include a
contention that the documents sought in the subpoena duces tecum of the requirement that the investigation be concluded in camera, with the public
Ombudsman are classified merely indicate the confidential nature of the EIIB's excluded, as exception to the general nature of the proceedings in the Office of
functions, but they do not exempt the EIIB from the duty to account for its the Ombudsman. 29 A reconciliation is thereby made between the demands of
funds to the proper authorities. Indeed by denying that there were savings national security and the requirement of accountability enshrined in the
made from certain items in the agency and alleging that the DBM had released Constitution. 30
to the EIIB only the allocations needed for the 947 personnel retained after its x What has been said above disposes of petitioners' contention that the
reorganization, petitioners in effect invited inquiry into the veracity of their anonymous letter-complaint against them is nothing but a vexatious
claim. If, as petitioners claim, the subpoenaed records have been examined by prosecution. It only remains to say that the general investigation in the
the COA and found by it to be regular in all respects, there is no reason why Ombudsman' s office is precisely for the purpose of protecting those against
they cannot be shown to another agency of the government which by whom a complaint is filed against hasty, malicious, and oppressive prosecution
constitutional mandate is required to look into any complaint concerning public as much as securing the State from useless and expensive trials. There may also
office. be benefit resulting from such limited in camera inspection in terms of increased
x On the other hand, the Ombudsman is investigating a complaint that several public confidence that the privilege is not being abused and increased likelihood
items in the EIIB were filled by fictitious persons and that the allotments for that no abuse is in fact occurring.
these items in 1988 were used for illegal purposes. The plantilla and other
personnel records are relevant to his investigation. He and his Deputies are
designated by the Constitution "protectors of the people" and as such they are c. Testimonial Privilege
required by it "to act promptly on complaints in any form or manner against
public officials or employees of the Government, or any subdivision, agency or Section 25. Parental and filial privilege. — No person may be compelled to testify against
instrumentality thereof, including government-owned or controlled corporation." his parents, other direct ascendants, children or other direct descendants. (20a)
x His need for the documents thus outweighs the claim of confidentiality
of petitioners. What is more, while there might have been compelling reasons ANNOTATIONS
for the claim of privilege in 1988 when it was asserted by petitioners, now,
seven years later, these reasons may have been attenuated, if they have not in
x Section 25 Æ Filial Privilege
fact ceased. The agents whose identities could not then be revealed may have
ceased from the service of the EIIB, while the covert missions to which they x Granted to any person
might have been deployed might either have been accomplished or abandoned. x Art 215 of the FC provides an exception:
On the other hand, the Ombudsman's duty to investigate the complaint that o No descendant shall be compelled, in a criminal case, to
there were in 1988 unfilled positions in the EIIB for which continued funding testify against his parents and grandparents, except when
was received by its officials and put to illegal use, remains.
x Above all, even if the subpoenaed documents are treated as presumptively
such testimony is indispensable in a crime against the
privileged, this decision would only justify ordering their inspection in camera descendant or by one parent against the other.
but not their nonproduction. However, as concession to the nature of the
functions of the EIIB and just to be sure no information of a confidential
character is disclosed, the examination of records in this case should be made in d. Admissions
strict confidence by the Ombudsman himself. Reference may be made to the
documents in any decision or order which the Ombudsman may render or issue
257
RECTO, GAYLE ANGELI M.
2011-0008 | AUSL
Personal Notes on Remedial Law 2 Review (based on the syllabus of Prof. Henedino M. Brondial)

Section 26. Admission of a party. — The act, declaration or omission of a party as to a o May be given as evidence as against the person admitting
relevant fact may be given in evidence against him. (22) ƒ RATIONALE: no man would make any
declaration against himself unless it is true
ANNOTATIONS
o NOT admissible in favor of the one admitting Æ
otherwise, it would be self-serving
x Admission Æ any statement of fact made by a party against his
x Effects of an extrajudicial confession
interest or unfavorable to the conclusion for which he contends or is o Not sufficient for conviction
inconsistent with the facts alleged by him
o For conviction to be sustained, it must be corroborated
x Requisites for admissibility:
with the corpus delicti
o Involves matters of fact and not of law
ƒ Corpus delicti Æ the body of the crime; the
o Categorical and definite
offense itself; the actual commission of the
o Knowingly and voluntarily made
crime and someone criminally responsible
o Adverse to the admitter’s interest therefor
ƒ Otherwise, it would be self-serving and
ƒ 2 elements of corpus delicti:
inadmissible x Proof of occurrence of a certain
x Types of admission:
event
o As to form: x Some person’s criminal
ƒ Verbal responsibility for the act
ƒ Written ƒ EXAMPLE: elements (corpus delicti) of illegal
o As to means:
possession of drugs:
ƒ Express x Accused is in possession of an
ƒ Tacit
item or object which is identified
o As to nature:
as a prohibited drug
ƒ Judicial
x Possession is authorized by law
ƒ Extrajudicial
x Accused freely and consciously
possessed said drug
Admission Confession
ƒ EXAMPLE: elements (corpus delicti) of theft:
Statement of fact which does not An acknowledgment of guilt or x Property was lost by the owner
involve an acknowledgment of guilt
or liability
liability x Lost by felonious taking
ƒ EXAMPLE: elements (corpus delicti) of illegal
May be express or tacit Must be express
possession of firearms:
May be made by 3rd persons and, Can be made only by the party x Accused’s lack of license or permit
in certain cases, are admissible himself and, in some instances, are
against a party admissible against his co-accused
to possess or carry the item
o Must comply with constitutional and statutory
admission is broader, hence, includes confessions requirements before it becomes admissible in evidence,
viz:
ƒ Must be done in the presence of his counsel
Admission Confession ƒ If not, there must be a written waiver to that
Statement of fact which does not effect signed by both accused and counsel
involve an acknowledgment of guilt An acknowledgment of guilt or
liability AND extrajudicial confession must be done
or liability
before any of the accused’s parents, older
May be express or tacit Must be express brothers or sisters, spouse, municipal mayor,
May be made by 3rd persons and, Can be made only by the party municipal judge, district school supervisor,
in certain cases, are admissible himself and, in some instances, are priest, or minister
against a party admissible against his co-accused
x Admission by SILENCE
May be adoptive (this admission
May not be adoptive as a o Section 32, Rule 130 of the RoC:
occurs when a person manifests
confession is an acknowledgment ƒ Section 32. Admission by silence. — An
his assent to the statements of
another person)
of guilt act or declaration made in the presence and
within the hearing or observation of a party
who does or says nothing when the act or
ADOPTIVE ADMISSIONS may
declaration is such as naturally to call for
occur when: (1) a party expressly
agrees to or concurs in an oral action or comment if not true, and when
statement made by another; (2) he proper and possible for him to do so, may be
hears a statement and later on given in evidence against him. (23a)
essentially repeats it; (3) he utters o ADMISSIBLE
an acceptance or builds upon the
assertion of another; (4) replies by o Involves a statement by a person in the presence of a
way of rebuttal to some specific party to the action, criminal or civil, which contains
points raised by another but assertions against such party which, if untrue, would be
ignores further points which s/he sufficient cause for the party to deny. Failure to speak
has heard the other make; (5)
against such statement is ADMISSIBLE as an ADMISSION
reads and subsequently signs a
written statement made by another o BUT not every silence is an implied admission
ƒ EXAMPLE: the silence of a person under
investigation for the commission of an offense
May be judicial or extrajudicial
admission is broader, hence, includes confessions should not be construed as an admission by
silence because of constitutional reasons
x Self-serving declaration Æ one which has been made extrajudicially o REQUISITES for silence to be deemed an admission:
ƒ The person admitting by silence heard and
by the party to favor his interests
understood the statement
o INADMISSIBLE
ƒ He was at liberty to make a denial
x Self-serving testimony Æ refers to an extrajudicial statement of a
ƒ The statement was about a matter affecting
party which is being urged for admission in court
his rights or in which he was interested and
o Does not include testimony as a witness in court
which naturals calls for a response
o No application to court declaration
ƒ The facts were within his knowledge
o Where the statement was not made in anticipation of a
ƒ The fact admitted from his silence is material
future litigation Æ it cannot be considered as self-serving
to the issue
x Effects of admission:

258
RECTO, GAYLE ANGELI M.
2011-0008 | AUSL
Personal Notes on Remedial Law 2 Review (based on the syllabus of Prof. Henedino M. Brondial)

x Res inter alios acta alteri nocere non debet Æ things done x REASON: An offer of compromise
between strangers ought not to injure those who are not parties to in criminal cases is an implied
them admission of guilt
o 2 branches: ƒ XPNs: An offer of compromise is
ƒ The rule that the rights of a party cannot be INADMISSIBLE in the ff cases:
prejudiced by an act, declaration or omission x Quasi-offenses
of another x Those which the law allows to be
x In other words, a man’s actions compromised
and declarations should affect him x If the accused is allowed by the
alone and should not affect others court to prove that the offer of
x This rule ONLY has application to compromise was not made under
extrajudicial admissions consciousness of guilt but merely
x XPNs to this 1st branch of the to avoid the risks of criminal
rule: action against him and he proves
o Admission by a co- the same [US vs. Maqui]
partner or agent o Although an offer of
o Admission by a marriage (and
conspirator subsequent actual
o Admission by privies marriage) generally
ƒ The rule that evidence of previous conduct or extinguishes criminal
similar acts at one time is not admissible to liability in rape, such
prove that one did or did not do the same act offer is still an
at another time admission of guilt
o Section 28 Rule 130 of the RoC: o TAX CASES:
ƒ Section 28. Admission by third party. — ƒ GR: INADMISSIBLE
The rights of a party cannot be prejudiced by x REASON: Tax Code provides that
an act, declaration, or omission of another, the payment of internal revenue
except as hereinafter provided. (25a) taxes may be compromised and all
o Section 34 Rule 130 of the RoC: criminal violations may likewise be
ƒ Section 34. Similar acts as evidence. — compromised
Evidence that one did or did not do a certain ƒ XPN: an offer of compromise may be
thing at one time is not admissible to prove admitted in tax cases already filed in court
that he did or did not do the same or similar AND those involving fraud
thing at another time; but it may be received x OTHER RULES:
to prove a specific intent or knowledge; o The ff are INADMISSIBLE to prove guilt or liability, as the
identity, plan, system, scheme, habit, custom case may be:
or usage, and the like. (48a) ƒ Plea of guilty later withdrawn [Section 2 Rule
116 of the RoC]
Judicial Admissions Extrajudicial Admissions x Accused may plea to a lesser
May be given in evidence against offense which is necessarily
Admissible against the declarant's
co-accused since the latter is/are
the confessant but NOT against his included in the crime charged
afforded opportunity to cross-
co-accused as the latter is/are during arraignment
deprived of the opportunity to
examine the former x Such plea of guilty is
cross-examine the former
INADMISSIBLE against such
accused
ƒ An unaccepted offer of a plea of guilty to
lesser offense [RoC]
Section 27. Offer of compromise not admissible. — In civil cases, an offer of ƒ An offer to pay or the payment of medical,
compromise is not an admission of any liability, and is not admissible in evidence against the
offeror. hospital or other expenses occasioned by an
injury [RoC]
In criminal cases, except those involving quasi-offenses (criminal negligence) or those allowed REASON: Good Samaritan Rule Æ such humanitarian acts or charitable
by law to be compromised, an offer of compromised by the accused may be received in responses should be encouraged and rewarded, instead of being discouraged or
evidence as an implied admission of guilt.
penalized by being considered as admissions of liability
A plea of guilty later withdrawn, or an unaccepted offer of a plea of guilty to lesser offense, is
not admissible in evidence against the accused who made the plea or offer. - by third parties

An offer to pay or the payment of medical, hospital or other expenses occasioned by an injury Section 28. Admission by third party. — The rights of a party cannot be prejudiced by an
is not admissible in evidence as proof of civil or criminal liability for the injury. (24a) act, declaration, or omission of another, except as hereinafter provided. (25a)

ANNOTATIONS ANNOTATIONS

x RULES in the admissibility of an offer of compromise: x Branches of res inter alios acta alteri nocere non debet
o CIVIL CASES: o Section 28
ƒ GR: INADMISSIBLE to prove the liability of o Section 34
the offeror
x REASON: An offer of compromise
in civil cases is NOT an admission - by partners
of liability
Section 29. Admission by co-partner or agent. — The act or declaration of a partner or
ƒ XPN: If such offer is not only to “buy peace”
agent of the party within the scope of his authority and during the existence of the partnership
but amounts to an admission of liability, the or agency, may be given in evidence against such party after the partnership or agency is
offered compromise directed only to the shown by evidence other than such act or declaration. The same rule applies to the act or
amount to be paid [El Varadero de Manila vs. declaration of a joint owner, joint debtor, or other person jointly interested with the party.
Insular Lumber] (26a)
o CRIMINAL CASES:
ƒ GR: ADMISSIBLE to prove the guilt of the ANNOTATIONS
offeror

259
RECTO, GAYLE ANGELI M.
2011-0008 | AUSL
Personal Notes on Remedial Law 2 Review (based on the syllabus of Prof. Henedino M. Brondial)

x Section 29 Æ one of the XPNs to the rule of res inter alios acta alteri affected has the right to cross-examine the
nocere non debet (1st branch Æ Section 28) x declarant
Requisites for the application of Section 29: x Requisites for the application of Section 30:
o Partnership, agency or joint interest is established by o Such conspiracy is shown by evidence aliunde
evidence other than the act or declaration ƒ The existence of the conspiracy may be
o That such act or declarations is within the scope of the inferred from the acts or confessions of the
partnership, agency or joint interest accused
o Such act or declaration must have been made during the ƒ If the conspiracy is not shown by evidence
existence of the partnership aliuende, the extrajudicial admission of
x Related Substantive provisions under the Civil Code: conspiracy CANNOT be used against the
o Article 1803. When the manner of management has not accused by virtue of the res inter alios acta
been agreed upon, the following rules shall be observed: alteri nocere non debet rule
ƒ (1) All the partners shall be considered agents o The admission was made during the existence of the
and whatever any one of them may do alone conspiracy
shall bind the partnership, without prejudice o The admission relates to the conspiracy itself
to the provisions of article 1801.
x GR: res inter alios acta alteri nocere non debet (1st branch Æ Section
ƒ (2) None of the partners may, without the
28)
consent of the others, make any important
alteration in the immovable property of the o XPN: Section 30 Æ acts or declarations made during the
partnership, even if it may be useful to the existence of the conspiracy
ƒ XPNs to the XPN: statements made by a
partnership. But if the refusal of consent by
the other partners is manifestly prejudicial to conspirator even AFTER the existence of the
the interest of the partnership, the court's conspiracy may be admitted against the co-
intervention may be sought. conspirator in the ff cases:
o Article 1910. The principal must comply with all the x If made in the presence of the co-
obligations which the agent may have contracted within conspirator who expressly or
the scope of his authority. impliedly agreed therein, as in the
ƒ As for any obligation wherein the agent has latter case, it would be a tacit
exceeded his power, the principal is not admission under Section 32
x Where the facts in said admission
bound except when he ratifies it expressly or
tacitly. are confirmed in the individual
o Article 487. Any one of the co-owners may bring an extrajudicial made by the con-
action in ejectment. conspirators after their
o Article 1222. A solidary debtor may, in actions filed by apprehension
the creditor, avail himself of all defenses which are x As a circumstance to determine
derived from the nature of the obligation and of those the credibility of a witness
x As a circumstantial evidence to
which are personal to him, or pertain to his own share.
With respect to those which personally belong to the show the probability of the co-
others, he may avail himself thereof only as regards that conspirator’s participation in the
part of the debt for which the latter are responsible. offense
x GR: res inter alios acta alteri nocere non debet x Requisites in order that an extrajudicial statement of a co-accused
o XPN: Section 29 Æ acts or declarations made during the may be used in judging the testimony of a witness:
existence of the P/A/JI o Statements are made by several accused
ƒ XPN to the XPN: statements made, even o The same are all in respects identical
AFTER the existence of said P/A/JI, are o There could have been no collusion among said accused
admissible if the same are made in connection in making such statements
with the winding up of the partnership affairs
x REASON: said partner making the - by privies
act or declaration is acting as an
agent of his co-partners in the Section 31. Admission by privies. — Where one derives title to property from another, the
act, declaration, or omission of the latter, while holding the title, in relation to the property, is
winding up evidence against the former. (28)
x GR: admissions by a counsel are admissible against the client
o REASON: the counsel acts in representation and as an ANNOTATIONS
agent of the client
o LIMITATIONS: such admission should NOT amount to a:
ƒ Compromise or x Section 31 Æ another XPN to the res inter alios acta alteri nocere non
ƒ Confession of judgment debet (1st branch Æ Section 28)
x Joint debtor Æ does not refer to mere community of interest but x Requisites for the application of Section 31:
should be understood according to its meaning in the common law o There must be a relation of privity between the party and
system from which the provision was taken, that is, in solidum, and the declarant
NOT mancomunanda ƒ Such privity may have arisen through:
x Succession
- by conspirators x Acts mortis causa
x Acts inter vivos
Section 30. Admission by conspirator. — The act or declaration of a conspirator relating
to the conspiracy and during its existence, may be given in evidence against the co- o The admission was made by the declarant, as
conspirator after the conspiracy is shown by evidence other than such act of declaration. (27) predecessor-in-interest, while holding the property
o The admission was in relation to the said property
ANNOTATIONS
- by silence

x Section 30 Æ one of the XPNs to the rule of res inter alios acta alteri Section 32. Admission by silence. — An act or declaration made in the presence and
nocere non debet within the hearing or observation of a party who does or says nothing when the act or
declaration is such as naturally to call for action or comment if not true, and when proper and
o Applies only to EXTRAJUDICIAL acts and statements
possible for him to do so, may be given in evidence against him. (23a)
ƒ REASON: does not apply to a testimony made
before the court because the party adversely ANNOTATIONS

260
RECTO, GAYLE ANGELI M.
2011-0008 | AUSL
Personal Notes on Remedial Law 2 Review (based on the syllabus of Prof. Henedino M. Brondial)

x REQUISITES for silence to be deemed an admission: v. Ong, 367 SCRA 559). Parties who are equally guilty cannot
o The person admitting by silence heard or observed the complain against each other
x HEIRS Æ appealed to CA
statement
o Questioning the in pari delicto ruling of RTC
o He was at liberty to make a denial x CA Æ affirmed RTC with MODIFICATION
o He must have understood it o The 192 square meters lot which was adjudicated in the
o The statement was about a matter affecting his rights or "Extrajudicial Settlement with Waiver" dated 5 December 1968
in which he was interested and which naturals calls for a among the heirs of Pedro Jr. namely Angelo, Maria, Arcadio and
response Mercedes is a property belonging to Pedro Jr. although there is a
typographical error in that the name of Pedro Jr. was inadvertently
o The facts were within his knowledge
typed only as Pedro Constantino. It is clear from the reading of the
o The fact admitted from his silence is material to the issue document that a typographical error was committed because the
x Applicability of rule on admission by silence: four (4) children of PedroJr. by Felipa dela Cruz were specifically
o Where a person is surprised in the act identified. Further, during the presentation of evidence of the
o Even if he is already in custody of the police plaintiffs-appellants, it was rebutted that Pedro Sr. had six (6)
o Voluntary participation in a reenactment of a crime Æ legitimate children namely: Pedro Jr., Antonia, Clara, Santiago,
Bruno and Eduardo19 and Pedro Jr. had four (4).20
considered as tacit admission of complicity
o Thus, the CA went on to state that the respondents, heirs of Pedro
ƒ HOWEVER, for said reenactment to be given Jr., did not adjudicate the 192 sq m lot unto themselves to the
evidentiary weight, the validity and efficacy of exclusion of all the other heirs of Pedro Sr. Rather, the adjudication
the confession must be shown in the document entitled "Extrajudicial Settlement with Waiver dated
o Applies to adverse statements in writing if the party was 5 December 1968 pertains to a different property and is valid absent
carrying on a mutual correspondence with the declarant any evidence to the contrary. Hence, it is erroneous for the trial court
to declare the parties in pari delicto.
ƒ HOWEVER, if there was no such mutual
correspondence Æ the rule is relaxed on the ISSUE: Whether the Heirs may still contradict their admissions during the pre-trial
theory that while the party would have
immediately reacted by a denial if the HELD: NO.
statements were orally made in his presence, x Weak as the reasoning is, the CA actually contradicted the admissions made no
such prompt response can generally not be less by the respondents during the pre-trial conference where they stipulated
that the land covered by Tax Declaration No. 9534 consisting of 192 sq. m
expected if the party has to resort to a written
belongs to Pedro Sr.35
reply x A portion of the admission and stipulations made by both parties during the pre-
x Non-applicability of rule on admission by silence: trial is hereunder quoted, thus:
o If the statements adverse to the party were made in the o Respondents’ admissions:
course of an official investigation ƒ "1. That the land covered by Tax Declaration No.
ƒ EXAMPLE: where he was pointed out in the 9534 previously owned by Pedro Constantino, Sr. was
transferred to Maria Constantino under Tax
course of a custodial investigation and was
Declaration No. 9535; (highlighting ours)
neither asked to reply nor comment on such ƒ 1. The existence of Extrajudicial Settlement with
imputations Waiver per Doc. No.319, Page No. 44, Book No. 11,
o Where the party had justifiable reason to remain silent Series of 1968 by Notary Public Romerico Flores, Jr."
ƒ EXAMPLE: where he was acting on advice of a x Clearly, the above stipulation is an admission against respondents’ interest of the
counsel fact of ownership by Pedro Sr. of the 192 sq m lot covered by Tax Declaration
No. 9534, which was transferred to respondents’ mother, the daughter of Pedro
Jr. Such that, in one of the issues submitted to be resolved by the trial court, this
CASES was included: "Whether or not the "Deed of Extrajudicial Settlement with
Waiver" is enforceable against the plaintiffs, thus curing the legal infirmities, if
OSCAR CONSTANTINO, MAXIMA CONSTANTINO and CASIMIRA MATURINGAN, Petitioners, any, of the "Pagmamana sa Labas ng Hukuman"36 - an issue earlier mentioned.
vs. HEIRS OF PEDRO CONSTANTINO, JR., represented by ASUNCION LAQUINDANUM, x Judicial admissions are legally binding on the party making the
Respondents. admissions. Pre-trial admission in civil cases is one of the instances of
G.R. No. 181508 October 2, 2013 judicial admissions explicitly provided for under Section 7, Rule 18 of
SECOND DIVISION the Rules of Court, which mandates that the contents of the pre-trial
order shall control the subsequent course of the action, thereby,
FACTS: defining and limiting the issues to be tried. In Bayas, et. al. v.
x Pedro Constantino, Sr., (Pedro Sr.) ancestors of the petitioners and respondents, Sandiganbayan, et. al.,37 this Court emphasized that:
owned several parcels of land, one of which is an unregistered parcel of land o Once the stipulations are reduced into writing and signed by the
declared for taxation purposes located in Bulacan parties and their counsels, they become binding on the parties who
x Upon his death, was survived by his six (6) children, namely: made them. They become judicial admissions of the fact or facts
o 1) PEDRO CONSTANTINO, JR. (Pedro Jr.), the grandfather of the stipulated.38 Even if placed at a disadvantageous position, a party
respondents; may not be allowed to rescind them unilaterally, it must assume
o 2) ANTONIA CONSTANTINO, who later died without issue; the consequences of the disadvantage.39 (Highlighting ours)
o 3) CLARA CONSTANTINO, who also later died without issue; x Moreover, in Alfelor v. Halasan,40 this Court declared that:
o 4) BRUNOCONSTANTINO, who was survived by his 6 children o A party who judicially admits a fact cannot later challenge
including petitioner Casimira Constantino-Maturingan; the fact as judicial admissions are a waiver of proof;
o 5) EDUARDO CONSTANTINO, who is survived by his daughter
production of evidence is dispensed with. A judicial
Maura; and admission also removes an admitted fact from the field of
o 6) SANTIAGO CONSTANTINO, who was survived by his five (5) controversy. Consequently, an admission made in the pleadings
children which includes petitioner Oscar Constantino. cannot be controverted by the party making such admission and
x Asuncion Laquindanum (Asuncion)and Josefina Cailipan (Josefina), great are conclusive as to such party, and all proofs to the contrary or
grandchildren of Pedro Sr., in representation of Pedro, Jr.(HEIRS) Æ filed a inconsistent therewith should be ignored, whether objection is
complaint against petitioners Oscar Constantino, Maxima Constantino and interposed by the party or not. The allegations, statements or
Casimira Maturingan, grandchildren of Pedro Sr., for the nullification of a admissions contained in a pleading are conclusive as against the
document denominated as "Pagmamana sa Labas ng Hukuman" dated 10 August pleader. A party cannot subsequently take a position contrary of or
1992 before the RTC Malolos inconsistent with what was pleaded.41 (Citations omitted)
x Pre-trial conference15 was conducted wherein the parties entered into x We are aware that the last paragraph of Section 7, Rule 18 of the Rules of Court
stipulations and admissions as well as identification of the issues to be litigated serves as a caveat for the rule of conclusiveness of judicial admissions - for, in the
x RTC Æ ruled in favor of HEIRS interest of justice, issues that may arise in the course of the proceedings but
o As a result of execution of "Extrajudicial Settlement with Waiver" which may not have been taken up in the pre-trial can still be taken up.
dated December 5, 1968 (Exh. "2") executed by the heirs of Pedro x Section 7, Rule 18 of the Rules of Court reads:
Constantino, Jr., a son of Pedro Constantino, Sr. and the o Section 7. Record of pre-trial. - The proceedings in the pre-trial
subsequent execution of another deed denominated as shall be recorded.1awp++i1 Upon the termination thereof, the
"Pagmamana sa Labas ng Hukuman" dated August 10, 1992 (Exh. court shall issue an order which shall recite in detail the matters
"E") executed by the heirs of Santiago and Bruno Constantino, also taken up in the conference, the action taken thereon, the
other sons of Pedro Constantino, Sr., to the exclusion of the other amendments allowed to the pleadings, and the agreements or
heirs, namely, those of ANTONIA, CLARA, and EDUARDO admissions made by the parties as to any of the matters
CONSTANTINO, both plaintiffs and defendants acted equally at considered. Should the action proceed to trial, the order shall,
fault. They are in pari delicto, whereby the law leaves them as they explicitly define and limit the issues to be tried. The contents of the
are and denies recovery by either one of them. (See:Yu Bun Guan order shall control the subsequent course of the action, unless
modified before trial to prevent injustice.
261
RECTO, GAYLE ANGELI M.
2011-0008 | AUSL
Personal Notes on Remedial Law 2 Review (based on the syllabus of Prof. Henedino M. Brondial)

x In addition, Section 4 of Rule 129 of the Rules of Court, provides that: x The prosecution presented the victim’s mother, Amalia Loyola, as its primary
o An admission, verbal or written, made by a party in the course of witness: testified that she left her two children Remelyn (3 1/2 years old) and
the proceedings in the same case, does not require proof. The Kimberly (1 year old) at their house to gather pigs’ food at Bulatukan; when she
admission may be contradicted only by showing that it was made returned home and she found Remelyn crying, naked, with fresh and dried blood
through palpable mistake or that no such admission was made. on her body; a neighbor, came and informed Amalia that he had seen the
x As contemplated in the aforementioned provision of the Rules of Court, the appellant pass by her house and take Remelyn; she brought Remelyn to a medical
general rule regarding conclusiveness of judicial admission upon the party center for genital examination which confirmed that Remelyn was raped; she
making it and the dispensation of proof admits of two exceptions: 1) when it is received an offer of compromise from the parents of the appellant and her
shown that the admission was made through palpable mistake, and 2) when it is husband likewise receive an offer from the appellant himself
shown that no such admission was in fact made. The latter exception allows one x The prosecution also presented Tulon Mik, Remelyn’s neighbor who saw
to contradict an admission by denying that he made such an admission.42 appellant carrying a small girl in his arms
x However, respondents failed to refute the earlier admission/stipulation before x The appellant interposed the defense of alibi corroborated by another witness
and during the trial. While denying ownership by Pedro Sr. of the 192 sq m lot, Alex Loyola who said that at the time the incident happened, they were at the
respondent Asuncion Laquindanum, when placed on the stand, offered a vague COMELEC office registering as voters in the barangay
explanation as to how such parcel of land was acquired by Pedro Jr. A portion of x RTC Æ convicted Gaudia
her testimony43 is hereto reproduced as follows: o found that there was sufficient circumstantial evidence to convict
o "ATTY. DOMINGO: appellant for the crime of qualified rape (victim was below 7 years
o Q: Do you know if as part of the estate of the late Pedro old)
Constantino, Sr. is another parcel of land also situated at Sta.
Maria, Hagonoy, Bulacan with an area of 192 square meters? ISSUE: Whether the offer of compromise herein should be taken as evidence.
o A: It is not owned by Pedro Constantino, Sr., sir. It is our property
owned by Pedro Constantino, Jr. that was inherited by my mother HELD: NO.
Maria Constantino. x Appellant’s charge that the offers of compromise allegedly made by the parents
o Q: And do you know how Pedro Constantino, Jr. acquired that of the appellant to Amalia, and by the appellant himself to Amalia’s husband
parcel of land, the one that you mentioned a while ago? should not have been taken against him because the offer of
o A: Kinagisnan ko na po yong lupang yon pagkabata pa na yon e compromise allegedly made is hearsay evidence, and of no probative
amin." (Highlighting ours) value (It was only Amalia who testified as to the alleged offer, and she
x The above assertion of denial is simply a self-serving declaration unsupported by was not a party to the conversation)
evidence. This renders conclusive the stipulations made during the pre-trial o A witness can only testify on facts which are based on his
conference. Consequently, respondents are bound by the infirmities of the personal knowledge or perception.
contract on which they based their right over the property subject matter x The offer of compromise allegedly made by the appellant’s parents to
thereof. Considering that the infirmities in the two deeds relate to exclusion of Amalia may have been the subject of testimony of Amalia. However,
heirs, a circumvention of an heir’s right to his or her legitime, it is apt to reiterate following the principle of res inter alios acta alteri nocere non debet,
our ruling in Neri v. Heirs of Hadji Yusop Uy,44 disposing that: the actions of his parents cannot prejudice the appellant, since he was
o Hence, in the execution of the Extra-Judicial Settlement of the not a party to the said conversation, nor was it shown that he was
Estate with Absolute Deed of Sale in favour of spouses Uy, all the privy to the offer of compromise made by them to the mother of the
heirs of Annunciation should have participated. Considering that victim. They cannot be considered as evidence against appellant but the court
Eutropia and Victoria were admittedly excluded and that then reiterates that these errors are not enough to reverse the conviction of the
minors Rosa and Douglas were not properly represented therein, appellant
the settlement was not valid and binding upon them and x Under Rule 133, Section 4 of the Revised Rules of Court, conviction may be
consequently, a total nullity. (Highlighting ours) based on circumstantial evidence provided three requisites concur: (a) there is
x Further highlighting the effect of excluding the heirs in the settlement of estate, more than one circumstance; (b) the facts from which the inferences are derived
the case of Segura v. Segura,45 elucidated thus: are proven; and (c) the combination of all the circumstances is such as to
o It is clear that Section 1 of Rule 74 does not apply to the partition produce a conviction beyond reasonable doubt. → all circumstances must be
in question which was null and void as far as the plaintiffs were consistent with each other, consistent with the hypothesis that the accused is
concerned. The rule covers only partition. The partition in the guilty, and at the same time inconsistent with the hypothesis that he is innocent
present case was invalid because it excluded six of the nine heirs and with every other rational hypothesis except that of guilt.
who were entitled to equal shares in the partitioned property. x The testimonies of Tulon Mik, Amalia and Remelyn have proven the culpability of
Under the rule "no extrajudicial settlement shall be binding upon the appellant with moral certainty. All the aforementioned circumstances have
any person who has not participated therein or had no notice been indubitably proven, both by the testimonial and documentary evidence
thereof." As the partition was a total nullity and did not affect the presented by the prosecution, and by the inability of the appellant to discredit
excluded heirs, it was not correct for the trial court to hold that their veracity.
their right to challenge the partition had prescribed after two years x Appellant contends that Tulon Mik’s testimony is weak, on the ground that Mik is
from its execution x x x. a relative of the husband of Amalia: mere relationship to one of the parties,
x In light of the foregoing, while both parties acted in violation of the without a showing of any other improper motive, is not sufficient basis to impair
law on legitimes, the pari delicto rule, expressed in the maxims "Ex the credibility of the witness
dolo malo non oritur action" and "in pari delicto potior est condition
defendentis," which refuses remedy to either party to an illegal CONRADO C. DOLDOL, Petitioners, vs. PEOPLE OF THE PHILIPPINES and THE
agreement and leaves them where they are, does not apply in this HONORABLE COURT OF APPEALS, Respondent.
case. (Underline supplied)46 As held in De Leon v. CA:47 G.R. No. 164481 September 20, 2005
o In the ultimate analysis, therefore, both acted in violation of laws.
SECOND DIVISION
However, the pari delicto rule expressed in the maxims "Ex dolo
malo non oritur action" and "In pari delicto potior est condition FACTS:
defendentis," which refuses remedy to either party to an illegal x A team of State Auditors led by State Auditor Emilie S. Ritua, with State Auditors
agreement and leaves them where they are does not apply in this Lydia Naoe and Beverly T. Cruz as members, conducted an audit of the cash and
case. cash account of Conrado C. Doldol, the Municipal Treasurer of Urbiztondo,
o Since the Letter-Agreement was repudiated before the purpose has Pangasinan
been accomplished and to adhere to the pari delicto rule in this case o discovered that Doldol had a shortage of P801,933.26.
is to put a premium to the circumvention or the laws, positive relief o They also noted that on June 5, 1995, he made cash withdrawals
should be granted to Macaria. Justice would be served by allowing from the municipality’s deposit account with the Land Bank of the
her to be placed in the position in which she was before the Philippines (LBP) amounting to P360,000.59.
transaction was entered into. o The withdrawal, purportedly for salaries, wages, allowances and
x Accordingly, in order not to put a premium to the circumvention or the laws as
mid-year bonuses of municipal officers and employees, had not
contemplated by the parties in the instant case, we must declare both contracts been recorded in the General Fund Cashbook as of June 8, 1995.
as void. Indeed, any circumvention of the law cannot be48 countenanced. o The State Auditors also noted that Doldol made adjustments in the
said cashbook on June 8, 1995, increasing his P801,933.26
shortage to P1,134,421.54.
x State Auditors Æ demanded from Doldol the refund of the said amounts
x Doldol failed to respond
x Doldol was then relieved of his duties and was directed to transfer the account to
Assistant Municipal Treasurer Loida Cancino
x State Auditors Æ conducted a second audit
PEOPLE OF THE PHILIPPINES, appellee vs. ROLENDO GAUDIA @ "LENDOY" or "DODO", o discovered that Doldol incurred an added cash shortage of
appellant. P149,905.92
G.R. No. 146111 February 23, 2004 x State Auditors Æ demanded from Doldol the refund of the said amounts
EN BANC x Doldol AGAIN failed to respond
x State Auditors Æ submitted their findings to the Provincial Auditor
FACTS: x Doldol Æ wrote the Provincial Treasurer requesting that a re-audit be conducted
x Rolendo Gaudia was accused of the crime of rape
on his cash and cash account, taking exception to the findings of the State
Auditors
262
RECTO, GAYLE ANGELI M.
2011-0008 | AUSL
Personal Notes on Remedial Law 2 Review (based on the syllabus of Prof. Henedino M. Brondial)

x BUT instead of pursuing his request, Doldol PARTIALLY PAID the said amounts ƒ Otherwise stated, the accused must have
x 2 Informations for malversation were then filed against Doldol before RTC San realized the importance or legal significance of
Carlos City
his act
x Defense evidence:
o Doldol testified that the funds which the State Auditors found o There must have been no violation of Section 12 Art III of
missing were, in fact, cash advances availed of by the municipal the 1987 Constitution
employees. He insisted that not a single centavo was used for his ƒ Section 12.
personal benefit. He averred that the charges lodged against him x Any person under investigation for the
were premature because the same were based on an incomplete commission of an offense shall have the
audit. right to be informed of his right to
x RTC Æ convicted Doldol remain silent and to have competent
x Doldol Æ appealed to CA and independent counsel preferably of
x CA Æ affirmed his own choice. If the person cannot
x Doldol Æ filed a Rule 45 before the SC afford the services of counsel, he must
be provided with one. These rights
ISSUE: What is the effect of Doldol’s partial payment of the missing amounts? cannot be waived except in writing and
in the presence of counsel.
HELD: Implied Admission x No torture, force, violence, threat,
x Except for his bare testimony, the petitioner offered no competent and credible intimidation, or any other means which
evidence to prove that the missing funds were actually cash advances of vitiate the free will shall be used against
employees in the municipality. The petitioner could have offered in evidence the him. Secret detention places, solitary,
documents evidencing the names of the recipients and amounts of the cash incommunicado, or other similar forms of
advances, but failed to do so. Moreover, the petitioner wrote the Provincial detention are prohibited.
x Any confession or admission obtained in
Auditor and offered to refund the missing funds as follows: P200,000.00 on
September 15, 1995, P200,000.00 on or before October 31, 1995, and violation of this or Section 17 hereof
P884,139.66 on November 30, 1995. He was able to pay only P200,000.00 shall be inadmissible in evidence against
on September 15, 1995, and failed to remit the balance of his shortage. him.
x The law shall provide for penal and civil
Such partial restitution of the petitioners of the cash shortage is an
implied admission of misappropriation of the missing funds. The ruling sanctions for violations of this Section as
of the CA on this matter is correct: well as compensation to the
o As We have already stated hereinabove, on September 15, 1995, rehabilitation of victims of torture or
not too long after the shortages in the municipal funds were similar practices, and their families.
discovered, appellant made a partial payment/settlement in the x PRESUMPTION: Confessions are made voluntarily
amount of 200,187.80 pesos as evidenced by Official Receipt No. o Onus probandi: rests on the defense to prove that it was
436756 (Exhibit "8," Record, Volume III, p. 6). With respect to the involuntary for having been obtained by violence,
balance of the missing funds, appellant promised to pay the same
intimation, threat or promise of reward or leniency
in installment basis. Appellant, though, failed to comply with his
undertaking (Record, Volume I, p. 457; TSN, Amando T. Sison, July x Circumstances which may be indicia of voluntariness:
27, 1998, pp. 32-33). Said payment is of no moment and could not o Confession contains details which the police could not
have legally brought acquittal for the appellant. On the contrary, as have supplied or invented
guided by Section 27, Rule 130 of the Rules on Evidence, We hold o Confession contains details which could have been known
that said payment, particularly when taken in conjunction with only to the accused
appellant’s commitment to gradually pay the remainder of the
missing funds, is a clear offer of compromise which must be
o Confession contains statements which are exculpatory in
treated as an implied admission of appellant’s guilt that he nature
embezzled or converted the missing funds to his personal use o Confession contains corrections made by the accused in
his handwriting or with his initials and which corrected
e. Confessions facts are best known to the accused
o Accused is sufficiently educated and aware of the
Judicial vs. Extra-Judicial consequences of his act
o Made in the presence of impartial witnesses with the
Section 33. Confession. — The declaration of an accused acknowledging his guilt of the accused acting normally on that occasion
offense charged, or of any offense necessarily included therein, may be given in evidence o Lack of motive on the part of the investigators to extract
against him. (29a)
a confession, with improbabilities and inconsistencies in
ANNOTATIONS the attempt of the accused to repudiate his confession
o Accused questioned the voluntariness of the confession
x Confession Æ categorical acknowledgement of guilt by an accused only for the first time at the trial of the case
in a criminal case, without any exculpatory statement or explanation o Contents of the confession were affirmed by the accused
o May also be a confession of judgment in a civil case Æ in in his voluntary reenactment of the crime
such case, the party express admits his liability o Facts contained in the confession were confirmed by
o If accused admits having committed an offense BUT other subsequent facts.
provides for a justification therefor Æ this is merely an o There were no signs of maltreatment after physical
admission and NOT a confession examination nor did the accuse complain of the same
o May be ORAL or IN WRITING x RATIONALE behind the INADMISSIBILITY of involuntary confessions:
ƒ If in writing Æ need not be under oath o Unreliable
o May be JUDICIAL or EXTRAJUDICIAL o Humanitarian considerations
ƒ Judicial confession Æ one made before a o Legal considerations of their being violative of the
court in which the case is pending and in the constitutional right against self-incrimination
course of legal proceedings therein and, by x Stonehill vs. Diokno (1967) Æ abandoned previous ruling which
itself, can sustain a conviction even in capital upheld the admissibility of confessions involuntarily given by an
offenses accused
ƒ Extrajudicial confession Æ one made in any o This was AGAIN abandoned in People vs. Chaw Yaw
other place or occasion and cannot sustain a Shun in 1968 Æ an involuntary confession is not
conviction unless corroborated by evidence of admissible in evidence UNLESS found to be true
the corpus delicti o BUT the decisions in People vs Alto (1968) and People
x Section 33 refers to EXTRAJUDICIAL CONFESSIONS vs. Urro (1972) reverted to that enunciated in Stonehill
x Requisites for admissibility of extrajudicial confessions under x 1973 Constitution provision:
Section 33: o Section 20 Article IV:
ƒ Section 20. No person shall be compelled to be a
o Confession must involve an express and categorical witness against himself. Any person under
acknowledgement of guilt investigation for the commission of an offense shall
o Facts admitted must be constitutive of a criminal offense have the right to remain silent and to counsel, and to
o Confession must have been voluntarily given be informed of such right. No force, violence, threat,
o Confession must have been intelligently made intimidation, or any other means which vitiates the
free will shall be used against him. Any confession
263
RECTO, GAYLE ANGELI M.
2011-0008 | AUSL
Personal Notes on Remedial Law 2 Review (based on the syllabus of Prof. Henedino M. Brondial)

obtained in violation of this section shall be


inadmissible in evidence. ADOPTIVE ADMISSIONS may
o EFFECT: if the extrajudicial was obtained PRIOR to the occur when: (1) a party expressly
effectivity of the 1973 Constitution (January 17, 1973) Æ agrees to or concurs in an oral
the same is ADMISSIBLE in evidence EVEN if the accused statement made by another; (2) he
hears a statement and later on
was not informed of his constitutional right because this essentially repeats it; (3) he utters
constitutional mandate was not given retroactive effect an acceptance or builds upon the
o SUFFICIENT COMPLIANCE of the new 1973 Consti assertion of another; (4) replies by
provision Æ where the accused was asked if he was way of rebuttal to some specific
points raised by another but
familiar of Section 20 Art IV of the 197s Consti and he ignores further points which s/he
answered in the affirmative and he signed a statement has heard the other make; (5)
thereafter that he was apprised of his constitutional rights reads and subsequently signs a
with the warning that anything he would say might be written statement made by another
used against him in court, such extrajudicial admission is
ADMISSIBLE May be judicial or extrajudicial
x 1987 Constitution admission is broader, hence, includes confessions
o See Section 12 Art III on page 1
o Extrajudicial confession must be done with the assistance
of a counsel CASES
o Waiver of the right to counsel during custodial
investigation Æ must be in writing and must be signed by JOSUE R. LADIANA, petitioner, vs. PEOPLE OF THE PHILIPPINES, respondent.
G.R. No. 144293 December 4, 2002
both counsel and accused
o Any form of coercion renders the extrajudicial confession THIRD DIVISION
INADMISSIBLE FACTS:
ƒ But this rule DOES NOT apply in cases where x The prosecution presented 5 witnesses: Caridad San Juan, PO2 Leopoldo
the coercion came from a private person Cacalda, Dr. Rogelio Javan, SPO2 Percival Gabinete, and Maria Cortez
x GR: the extrajudicial confession of an accused is binding only upon x CARIDAD, the wife of Francisco, the victim in the case at bar testified that an
himself and is NOT admissible against his co-accused unidentified woman came and told her that her husband was killed by Ladiana yet
o XPNs: she testified that, at the time of the incidenct, she was in her house and that she
did not hear any gunshot
ƒ The co-accused impliedly acquiesced in or x PO2 CACALDA recounted that after somebody reported to him about an existing
adopted said confession by not questioning its trouble, he responded by going to the scene and upon arrival therein saw the
truthfulness lifeless body of Francisco. He did not examine the body thereof yet he had
ƒ Co-accused made interlocking confessions Æ gathered from the people milling around the body of Francisco that it was
those identical confessions made voluntarily accused who shot and killed Francisco. Consequently, Cacalda executed a written
and independently without conclusion, which statement as the responding officer.
x DR. JAVAN recounted that he was the one who performed the necropsy on the
are corroborated by other evidence and cadaver of Francisco and that he had prepared the corresponding reports and/or
without contradiction by the co-accused who documents relating thereto.
was present x SPO2 GABINETE: his testimony was subsequently dispensed with, upon the
ƒ Co-accused admitted the facts stated by the admission of the defense that he was part of the group of policemen who
confessant after being apprised of such proceeded to the place of the subject incident and that he found the body of
confession Francisco lying along the road.
x MARIO CORTEZ who emphasized that he was not the one who conducted the
ƒ If they are charged as co-conspirators of the preliminary investigation of the complaint
crime which was confessed by one of the x After the presentation of Cortez, the prosecution filed its formal offer of evidence
accused and said confession is only used as a and rested its case
corroborating evidence x Ladiana filed a Motion for Leave of Court to File Demurrer to Evidence:
ƒ Where the confession is used as o (i) a review of the documentary and testimonial evidence adduced
circumstantial evidence to show the by the prosecution allegedly failed to show that the accused is
guilty of the offense charged;
probability of participation by the con-
o (ii) at best, the evidence submitted by the prosecution are allegedly
conspirator hearsay in character, considering that the supposed eyewitness in
ƒ When confessant testified for his co- the person of Rodolfo Cabrera was never presented in court; and
defendant o (iii) the prosecution was allegedly merely able to prove the fact of
ƒ When co-conspirator’s extrajudicial confession death of the victim, but not the identity of the person who caused
is corroborated by other evidence of record said death.
x SB Æ CONVICTED Ladiana
x Section 33 Æ extrajudicial confession is admissible not only as to the
o held that his Counter-Affidavit, in which he had admitted to having
crime charged BUT ALSO as to crimes necessarily included therein fired the fatal shots that caused the victim’s death, may be used as
x ILLEGAL extrajudicial confessions are NOT admissible against the evidence against him thus the document had sufficiently
accused BUT are ADMISSIBLE AGAINST the person/s obtaining the established his responsibility for the death of the victim.
such illegal confession or admission x Ladiana Æ filed a Rule 4 before the SC
x Fruit of the poisonous tree doctrine Æ refers to the object which
ISSUE: Whether the counter-affidavit executed by Ladiana during the preliminary investigation
was seized in the course of an illegal search and seizure
of this case is admissible proof showing his complicity in the crime.
o DOES NOT refer to the testimony or confession obtained
through illegal arrest HELD: YES.
x The admissions made by petitioner in his Counter-Affidavit are not violative of his
Admission Confession constitutional rights.
x The rights enumerated in the constitutional provision "exist only in custodial
Statement of fact which does not
An acknowledgment of guilt or interrogations, or in-custody interrogation of accused persons"
involve an acknowledgment of guilt o Custodial interrogation is the questioning initiated by law
liability
or liability enforcement officers after a person has been taken into custody or
May be express or tacit Must be express otherwise deprived of his freedom of action in any significant way.
x In the present case, petitioner admits that the questioned statements
May be made by 3rd persons and, Can be made only by the party
in certain cases, are admissible himself and, in some instances, are were made during the preliminary investigation, not during the
against a party admissible against his co-accused custodial investigation. The right to competent and independent
counsel does not apply during preliminary investigations.
May be adoptive (this admission x Evidently, a person undergoing preliminary investigation before the public
May not be adoptive as a
occurs when a person manifests prosecutor cannot be considered as being under custodial investigation.
confession is an acknowledgment
his assent to the statements of
of guilt x However, SC does not agree with SB’s characterization of petitioner’s
another person) Counter-Affidavit as an extrajudicial confession. It is only an
admission. Sections 26 and 33 of Rule 130 of the Revised Rules on
Evidence distinguish one from the other as follows

264
RECTO, GAYLE ANGELI M.
2011-0008 | AUSL
Personal Notes on Remedial Law 2 Review (based on the syllabus of Prof. Henedino M. Brondial)

o "SEC. 26. Admissions of a party. - The act, declaration or omission evidence to determine whether the accused committed the crimes
of a party as to a relevant fact may be given in evidence against charged and the precise degree of his criminal culpability therefor, he
him may still be convicted if there is ample proof on record, not contingent
o "SEC. 33. Confession. - The declaration of an accused on the plea of guilty, on which to predicate conviction.33
acknowledging his guilt of the offense charged, or of any offense x In this case, the prosecution had already rested its case when the appellant
necessarily included therein, may be given in evidence against decided to change his plea. In fact, the trial court granted the prosecution’s
him." motion that the evidence it had presented be considered proof of the degree of
x In a confession, there is an acknowledgment of guilt; in an admission, culpability of the appellant. It is, thus, incumbent upon this Court to determine
there is merely a statement of fact not directly involving an whether the evidence adduced by the prosecution in Criminal Case No. 97-385 is
acknowledgment of guilt or of the criminal intent to commit the sufficient to establish beyond reasonable doubt the appellant’s guilt for qualified
offense with which one is charged. Thus, in the case at bar, a rape.
statement by the accused admitting the commission of the act charged x In determining the guilt of the accused in rape cases, the Court is guided by the
against him but denying that it was done with criminal intent is an following considerations: (a) that an accusation of rape can be made with
admission, not a confession. facility; it is difficult to prove, but more difficult for the person accused, though
x In general, admissions may be rebutted by confessing their untruth or by innocent, to disprove; (b) that in view of the intrinsic nature of the crime which
showing they were made by mistake. The party may also establish that the usually involves two persons, the testimony of the complainant must be
response that formed the admission was made in a jocular, not a serious, scrutinized with extreme caution; and (c) that the evidence for the prosecution
manner; or that the admission was made in ignorance of the true state of facts. must stand or fall on its own merits and cannot be allowed to draw strength from
Yet, petitioner never offered any rationalization why such admissions had been the weakness of the evidence of the defense.34 It, likewise, bears stressing that in
made, thus, leaving them unrebutted. all criminal prosecutions, without regard to the nature of the defense which the
accused may raise, the burden of proof remains at all times upon the prosecution
THE PEOPLE OF THE PHILIPPINES, appellee vs. FELICIANO ULIT y TAMPOY, appellant. to establish his guilt beyond reasonable doubt.
G.R. Nos. 131799-801 February 23, 2004
EN BANC PEOPLE OF THE PHILIPPINES, appellee, vs. BENJAMIN SAYABOC y SEGUBA, PATRICIO
ESCORPISO y VALDEZ, MARLON BUENVIAJE y PINEDA, and MIGUEL BUENVIAJE y FLORES,
FACTS: appellants.
x Ulit was charged with qualified rape before RTC Makati G.R. No. 147201 January 15, 2004
o For the rape of his niece LUCELLE SERRANO y ULIT EN BANC
x Ulit changed his plea from not guilty to guilty
x RTC Æ convicted Ulit FACTS:
x Automatic review ensued x Benjamin Sayaboc, Patricio Escorpiso, Marlon Buenviaje, and Miguel Buenviaje
were charged with murder before RTC Bayombong, Nueva Vizcaya
ISSUE: Whether Ulit’s conviction should be set aside, considering how his plea of guilty was o For the killing of Joseph Galam y Antonio
taken by the RTC x Prosecution evidence:
o At about 9:00 a.m. of 13 August 1994, while prosecution witness
HELD: NO. Abel Ramos was at a vulcanizing shop in Barangay Quezon, Solano,
x First. The trial court did not ask the appellant his reasons for changing Nueva Vizcaya, he heard one Tessie Pawid screaming from across
his plea, from not guilty to that of guilty, and the cogent circumstances the road: "Enough, enough, enough!" In front of her were Marlon
that led him to decide to do so. Buenviaje and Joseph Galam, who were engaged in a fisticuff. By
x Second. It appears in the Informations filed by the Public Prosecutor that the the time Pawid was able to subdue the two men by standing
appellant opted not to avail himself of his right to a regular preliminary between them and embracing Galam, Buenviaje’s face was already
investigation and refused to execute a waiver under Article 125 of the bloodied and Galam’s shirt collar torn. As Buenviaje was leaving, he
Revised Penal Code. The records also show that the appellant executed a turned to face Galam and, with his right index finger making a
Sinumpaang Salaysay while detained at the barangay hall where he confessed to slicing motion across his throat, shouted: "Putang-ina mo Joseph,
having raped the victim in February 1997 and March 2, 1997. However, the trial may araw ka rin, papatayin kita." Galam retorted, "Gago, traydor,
court did not ask the appellant whether he was assisted by counsel when he was gold digger, halika." Buenviaje did not respond anymore and left on
brought to the Office of the Public Prosecutor for inquest investigation. Neither did a tricycle
the court a quo inquire about the circumstances and the appellant’s reasons for o More than three months thereafter, or on 2 December 1994, Galam
refusing to execute the said waiver. was shot to death at the Rooftop Disco and Lodging House
o The records show that when the prosecution offered the appellant’s (Rooftop, for short) owned by him, which was located at Barangay
Sinumpaang Salaysay in evidence to prove that he confessed to Quezon, Solano, Nueva Vizcaya.
having raped the victim in February 1997 and March 2, 1997, the o On 8 March 1995, Pilar (care taker ) and Jaramillo (waitress)
appellant objected thereto on the ground that he was not assisted identified Benjamin Sayaboc at the PNP Provincial Headquarters in
by counsel and that he was coerced into signing the same. Bayombong as the gunman who shot Joseph Galam to death.10
x Third. The trial court also failed to ascertain from the appellant whether o On the afternoon of that day, SPO4 Cagungao was called to the
he was assisted by counsel when he executed his Sinumpaang Provincial Command Headquarters in Bayombong, Nueva Vizcaya,
Salaysay while detained at the barangay hall; and, if he was not so to take the statement of Sayaboc. When he arrived at the
assisted by counsel, whether he had waived his right thereto, before and when headquarters he saw Sayaboc being interviewed by reporters inside
he signed his Sinumpaang Salaysay. the investigation room. He then brought Sayaboc to the inner part
x Fourth. The trial court failed to ask the appellant why he was pleading of the room. Before taking the statement of Sayaboc, he advised
guilty to a rape committed in November 1996, when in his Sinumpaang the latter of his constitutional rights. Then Sayaboc told him that he
Salaysay,31 he confessed to having raped the victim only in February wanted to have a counsel of his own choice. But since Sayaboc
1997 and March 2, 1997. The appellant did not admit having raped her in could not name one, Cagungao asked the police officers to get a
November 1996 as alleged in the Information in Criminal Case No. 97-385. The lawyer. Half an hour later, the police officers brought Atty. Rodolfo
trial court did not even inquire from the appellant who prepared and typed his Cornejo of the PAO, who then conferred with Sayaboc for a while.
Sinumpaang Salaysay and if the contents of his statement were explained to him After Cagungao heard Sayaboc say, "okay," he continued the
before he signed the same. investigation, during which Atty. Cornejo remained silent the entire
x Fifth. The trial court did not explain the following to the appellant, in time. However, Cagungao would stop questioning Sayaboc
plain and simple terms so as to be understood by him: (a) the elements whenever Atty. Cornejo would leave to go to the comfort room.11
of the crime of qualified rape; (b) the circumstances of relationship and That night Sayaboc executed an extrajudicial confession12 in
the minority of the victim; and (c) that his plea of guilty to qualified Ilocano dialect. He therein confessed to killing Joseph Galam at the
rape would not mitigate the penalty for the crime in light of Article 63 of the behest of Marlon Buenviaje for the sum of P100,000. He likewise
Revised Penal Code. implicated Miguel Buenviaje and Patricio Escorpiso. The confession
x Sixth. It was not explained to the appellant that if convicted of qualified was also signed by Atty. Cornejo and attested to by one Fiscal
rape, he would be civilly liable to the victim in the amount of P50,000 as Melvin Tiongson.
moral damages and P75,000 as civil indemnity ex delicto. x Defense evidence:
x Seventh. Neither did the trial court inquire from the appellant’s counsel whether o Sayaboc denied having committed the crime and proffered the
the meaning and the consequences of a guilty plea were explained to defense of alibi. He also flatly denied having met Atty. Cornejo or
the appellant in a language or dialect known to and understood by him. having been informed of his rights. He testified to having been
x Eight. The trial court failed to delve into and ascertain from the appellant his beaten by six or seven police officers in the investigating room,
age, educational attainment and socio-economic status. who then coerced him to confess to having killed Galam.17 Apart
x Ninth. The trial court failed to ask the appellant to narrate the facts and from his testimony, he submitted a handwritten statement dated 20
circumstances surrounding the incident of qualified rape as charged in March 199518 and an affidavit dated 10 April 199519 to support his
Criminal Case No. 97-385. claim of police brutality and retraction of his confession.
x Tenth. The appellant was not asked if he desired to adduce evidence in x RTC Æ CONVICTED:
Criminal Case No. 97-385 in spite of his plea of guilty. o Benjamin Sayaboc guilty of the crime of murder, with treachery as
x As a rule, this Court has set aside convictions based on pleas of guilty the qualifying circumstance and craft and price or reward as
in capital offenses because of the improvidence thereof, and when such aggravating circumstances
plea is the sole basis of the condemnatory judgment.32 However, o As for Marlon Buenviaje, Miguel Buenviaje, and Patricio Escorpiso,
where the trial court receives, independently of his plea of guilty, the court held that the treachery employed by Sayaboc could not
265
RECTO, GAYLE ANGELI M.
2011-0008 | AUSL
Personal Notes on Remedial Law 2 Review (based on the syllabus of Prof. Henedino M. Brondial)

be taken against them and, therefore, declared them guilty of the drama, albeit tragic - for others. He would do what he wanted to
crime of homicide only, with the first as principal and the two do regardless of the advice of others. Hence, Atty. Cornejo could
others as accomplices. only advise him of his constitutional rights, which was apparently
x Sayaboc et al Æ filed a Rule 45 before the SC done. The said counsel could not stop him from making his
confession even if he did try.28
x We find this explanation unacceptable. That Sayaboc was a "garrulous" man
ISSUE: Whether Sayaboc’s alleged extrajudicial confession was correctly admitted by the RTC. who would "do what he wanted to do regardless of the advice of
others" is immaterial. The waiver of a right is within the rights of a
HELD: NO. suspect. What is lacking is a showing, to the satisfaction of this Court, of a
x Beginning with the admissibility of Sayaboc’s extrajudicial confession, we hold faithful attempt at each stage of the investigation to make Sayaboc aware of the
that such cannot be used in evidence in this case. consequences of his actions. If anything, it appears that Sayaboc’s counsel was
x Section 12 of Article III of the 1987 Constitution provides: ineffectual for having been cowed by his client’s enthusiasm to speak, or, worse,
o Sec. 12. (1) Any person under investigation for the commission of was indifferent to it.
an offense shall have the right to be informed of his right to remain x The right to a competent and independent counsel means that the
silent and to have competent and independent counsel preferably of counsel should satisfy himself, during the conduct of the investigation,
his own choice. If the person cannot afford the services of that the suspect understands the import and consequences of
counsel, he must be provided with one. These rights cannot be answering the questions propounded. In People v. Deniega,29 we said:
waived except in writing and in the presence of counsel. o The desired role of counsel in the process of custodial investigation
o (3) Any confession or admission obtained in violation of this or the is rendered meaningless if the lawyer merely gives perfunctory
preceding section shall be inadmissible in evidence against him. advice as opposed to a meaningful advocacy of the rights of the
x Jurisprudence provides that extrajudicial confessions are presumed to person undergoing questioning. If the advice given is so cursory as
be voluntary.22 The condition for this presumption, however, is that the to be useless, voluntariness is impaired.
prosecution is able to show that the constitutional requirements x This is not to say that a counsel should try to prevent an accused from making a
safeguarding an accused’s rights during custodial investigation have confession. Indeed, as an officer of the court, it is an attorney’s duty to, first and
been strictly complied with, especially when the extrajudicial foremost, seek the truth. However, counsel should be able, throughout the
confession has been denounced. The rationale for this requirement is to allay investigation, to explain the nature of the questions by conferring with his client
any fear that the person being investigated would succumb to coercion while in and halting the investigation should the need arise. The duty of a lawyer includes
the unfamiliar or intimidating environment that is inherent in custodial ensuring that the suspect under custodial investigation is aware that the right of
investigations. Therefore, even if the confession may appear to have been given an accused to remain silent may be invoked at any time.
voluntarily since the confessant did not file charges against his alleged x We understand the difficulty and frustration of police investigators in obtaining
intimidators for maltreatment,23 the failure to properly inform a suspect of his evidence to bring criminals to justice. But even the hardest of criminals have
rights during a custodial investigation renders the confession valueless and rights that cannot be interfered with. Those tasked with the enforcement of the
inadmissible.24 law and who accuse those who violate it carry the burden of ensuring that all
x In this case, contrary to SPO4 Cagungao’s claim that he conferred with Sayaboc evidence obtained by them in the course of the performance of their duties are
for half an hour informing him about his constitutional rights, the extrajudicial untainted with constitutional infirmity. The purpose of the stringent requirements
confession provides only the following: of the law is to protect all persons, especially the innocent and the weak, against
o PRELIMINARY: I would like to inform you Mr. Sayaboc that possible indiscriminate use of the powers of the government. Any deviation
questions will be asked to you regarding an incident last December cannot be tolerated, and any fruit of such deviation shall be excluded from
2, 1994 at the Rooftop, Brgy. Quezon, Solano, Nueva Vizcaya, in evidence.
connection with the shooting of Joseph Galam, owner of the said x For these reasons, the extrajudicial confession of Sayaboc cannot be
Disco House as a result of his death. Before questions will be asked used in evidence against him. We hold, however, that the prosecution has
[of] you I would like to inform you about your ri[g]hts under the discharged its burden of proving his guilt for the crime of homicide.
new Constitution of the Philippines, as follows: That you have the x From the records of the case, there can be no doubt that Sayaboc shot and killed
right to remain silent or refuse to answer the questions which you Galam in the early evening of 2 December 1994. He was seen waiting at the
think will incriminate you; That you have the right to seek the Rooftop from 3:00 to 6:00 p.m. of that day, shooting Galam shortly after the
services of a counsel of your own choice or if not, this office will latter’s arrival, and fleeing from the scene of the crime to a waiting tricycle.
provide you a lawyer if you wish. Credible witnesses described Sayaboc’s appearance to the police soon after the
o QUESTIONS: After informing you all your constitutional rights, are shooting incident and prepared affidavits about the incident. They identified
you willing to give your true statement regarding the death of Sayaboc at the police station while he was in custody, during the preliminary
Joseph Galam? investigation, and, again, in open court. Such positive identification constitutes
o ANSWER: Yes, sir. more than sufficient direct evidence to uphold the finding that Sayaboc was
o QUESTIONS: Do you want to get a lawyer to assist in this Galam’s killer. It cannot just be rebutted by Sayaboc’s bare denial and weak alibi.
investigation?
o ANSWER: Yes, sir. I want to seek the assistance of Atty. Rodolfo CARLOS L. TANENGGEE, Petitioner, vs. PEOPLE OF THE PHILIPPINES, Respondent.
Cornejo. G.R. No. 179448 June 26, 2013
o QUESTIONS: Atty. Rodolfo Cornejo is here now, do you want him SECOND DIVISION
to assist you in this investigation?
o ANSWER: Yes, sir. 25 FACTS:
x Apart from the absence of an express waiver of his rights, the x Tanenggee was charged with 5 counts of estafa through falsification of
confession contains the passing of information of the kind held to be in commercial documents before RTC Manila
violation of the right to be informed under Section 12, Article III of the o being then the Manager of the COMMERCIO BRANCH OF
Constitution. In People v. Jara,26 the Court explained: METROBANK located at the New Divisoria Market Bldg., Divisoria,
o The stereotyped "advice" appearing in practically all extrajudicial Manila, and taking advantage of his position as such, prepared and
confessions which are later repudiated has assumed the nature of a filled up or caused to be prepared and filled up METROBANK
"legal form" or model. Police investigators either automatically type Promissory Note Form No. 366857 with letters and figures reading
it together with the curt "Opo" as the answer or ask the accused to "BD#083/97" after the letters reading "PN", with figures reading
sign it or even copy it in their handwriting. Its tired, punctilious, "07.24.97" after the word "DATE", with the amount of
fixed, and artificially stately style does not create an impression of P16,000,000.00 in words and in figures
voluntariness or even understanding on the part of the accused. x Prosecution evidence:
The showing of a spontaneous, free, and unconstrained giving up o Valentino Elevado, a member of the Internal Affairs Department of
of a right is missing. Metrobank, testified that he conducted and interviewed the
o The right to be informed requires "the transmission of appellant in January 1998; that in said interview, appellant
meaningful information rather than just the ceremonial admitted having committed the allegations in the Informations,
and perfunctory recitation of an abstract constitutional specifically forging the promissory notes; that the proceeds of the
principle."27 It should allow the suspect to consider the effects loan were secured or personally received by the appellant although it
and consequences of any waiver he might make of these rights. should be the client of the bank who should receive the same; and
More so when the suspect is one like Sayaboc, who has an that all the answers of the appellant were contained in a
educational attainment of Grade IV, was a stranger in Nueva typewritten document voluntarily executed, thumbmarked, and
Vizcaya, and had already been under the control of the police signed by him (Exhibit "N").
officers for two days previous to the investigation, albeit for o Rosemarie Tan Apostol, assistant branch manager, testified that
another offense. the signatures appearing on the promissory notes were not the
x We likewise rule that Sayaboc was not afforded his constitutional right to signatures of Romeo Tan; that the promissory notes did not bear
a competent counsel. While we are unable to rule on the unsubstantiated her signature although it is required, due to the fact that Romeo
claim that Atty. Cornejo was partial to the police, still, the facts show through the Tan is a valued client and her manager accommodated valued
testimonies of Sayaboc and prosecution witness SPO4 Cagungao that Atty. clients; that she signed the corresponding checks upon instruction
Cornejo remained silent throughout the duration of the custodial investigation. The of appellant; and that after signing the checks, appellant took the
trial court attributed the silence of Atty. Cornejo to the garrulous nature and same which remained in his custody.
intelligence of Sayaboc, thus: o Eliodoro M. Constantino, NBI Supervisor and a handwriting expert,
o As already stated, Sayaboc was a garrulous man and intelligent. It
testified that the signatures appearing on the promissory notes and
was in his character for him to want to be a central figure in a
266
RECTO, GAYLE ANGELI M.
2011-0008 | AUSL
Personal Notes on Remedial Law 2 Review (based on the syllabus of Prof. Henedino M. Brondial)

specimen signatures on the signature card of Romeo Tan were not x Indeed, a person under custodial investigation is guaranteed certain rights which
written by one and the same person. attach upon the commencement thereof, viz:
o Maria Dolores Miranda, a Loans Clerk at Metrobank Commercio o (1) to remain silent,
Branch, testified that several cashier’s checks were issued in favor o (2) to have competent and independent counsel preferably of his
of Romeo Tan; that appellant instructed her to encash the same; own choice, and
and that it was appellant who received the proceeds of the loan. o (3) to be informed of the two other rights above.19
x Defense evidence: x In the present case, while it is undisputed that petitioner gave an uncounselled
o For his defense, appellant Carlos Lo Tanenggee testified that he is written statement regarding an anomaly discovered in the branch he managed,
a holder of a Masters degree from the Asian Institute of the following are clear:
Management, and was the Branch Manager of Metrobank o (1) the questioning was not initiated by a law enforcement
Commercio Branch from 1994 until he was charged in 1998 [with] authority but merely by an internal affairs manager of the bank;
the above-named offense. He was with Metrobank for nine (9) and,
years starting as assistant manager of Metrobank Dasmariñas o (2) petitioner was neither arrested nor restrained of his liberty in
Branch, Binondo, Manila. As manager, he oversaw the day to day any significant manner during the questioning.
operations of the branch, solicited accounts and processed loans, x Clearly, petitioner cannot be said to be under custodial investigation
among others. and to have been deprived of the constitutional prerogative during the
o Appellant claimed that he was able to solicit Romeo Tan as a client- taking of his written statement.
depositor when he was the branch manager of Metrobank x Moreover, in Remolona v. Civil Service Commission,20 we declared that the right
Commercio. As a valued client, Romeo Tan was granted a credit to counsel "applies only to admissions made in a criminal investigation but not to
line for forty million pesos (P40,000,000.00) by Metrobank. Tan those made in an administrative investigation." Amplifying further on the matter,
was also allowed to open a fictitious account for his personal use the Court made clear in the recent case of Carbonel v. Civil Service
and was assisted personally by appellant in his dealings with the Commission:21
bank. In the middle of 1997, Tan allegedly opened a fictitious o However, it must be remembered that the right to counsel under
account and used the name Jose Tan. Such practice for valued Section 12 of the Bill of Rights is meant to protect a suspect during
clients was allowed by and known to the bank to hide their finances custodial investigation. Thus, the exclusionary rule under paragraph
due to rampantkidnappings or from the Bureau of Internal Revenue (2), Section 12 of the Bill of Rights applies only to admissions made
(BIR) or from their spouses. in a criminal investigation but not to those made in an
o According to appellant, Tan availed of his standing credit line administrative investigation.22
(through promissory notes) for five (5) times on the following x Here, petitioner’s written statement was given during an
dates: 1) 24 July 1997 for sixteen million pesos (P16,000,000.00), administrative inquiry conducted by his employer in connection with an
2) 27 October 1997 for six million pesos (P6,000,000.00), 3) 12 anomaly/irregularity he allegedly committed in the course of his
November 1997 for three million pesos (P3,000,000.00), 4) 21 employment. No error can therefore be attributed to the courts below in
November 1997 for sixteen million pesos (P16,000,000,00), 5) 22 admitting in evidence and in giving due consideration to petitioner’s
December 1997 for two million pesos (P2,000,000.00). On all these written statement as there is no constitutional impediment to its
occasions except the loan on 24 July 1997 when Tan personally admissibility.
went to the bank, Tan allegedly gave his instructions regarding the x Petitioner’s written statement was given voluntarily, knowingly and intelligently.
loan through the telephone. Upon receiving the instructions, x Petitioner attempts to convince us that he signed, under duress and intimidation,
appellant would order the Loans clerk to prepare the promissory an already prepared typewritten statement. However, his claim lacks sustainable
note and send the same through the bank’s messenger to Tan’s basis and his supposition is just an afterthought for there is nothing in the
office, which was located across the street. The latter would then records that would support his claim of duress and intimidation.
return to the bank, through his own messenger, the promissory x Moreover, "it is settled that a confession or admission is presumed
notes already signed by him. Upon receipt of the promissory note, voluntary until the contrary is proved and the confessant bears the
appellant would order the preparation of the corresponding burden of proving the contrary."23 Petitioner failed to overcome this
cashier’s check representing the proceeds of the particular loan, presumption. On the contrary, his written statement was found to have been
send the same through the bank’s messenger to the office of Tan, executed freely and consciously. The pertinent details he narrated in his
and the latter would return the same through his own messenger statement were of such nature and quality that only a perpetrator of the crime
already endorsed together with a deposit slip under Current could furnish. The details contained therein attest to its voluntariness. As
Account No. 258-250133-7 of Jose Tan. Only Cashier’s Check dated correctly pointed out by the CA:
21 November 1997 for sixteen million pesos (P16,000,000.00) was o As the trial court noted, the written statement (Exhibit N) of
not endorsed and deposited for, allegedly, it was used to pay the appellant is replete with details which could only be supplied by
loan obtained on 24 July 1997. Appellant claimed that all the appellant. The statement reflects spontaneity and coherence which
signatures of Tan appearing on the promissory notes and the cannot be associated with a mind to which intimidation has been
cashier’s checks were the genuine signatures of Tan although he applied. Appellant’s answers to questions 14 and 24 were even
never saw the latter affix them thereon. initialed by him to indicate his conformity to the corrections made
o In the middle of January 1998, two (2) Metrobank auditors therein. The response to every question was fully informative, even
conducted an audit of the Commercio Branch for more than a beyond the required answers, which only indicates the mind to be
week. Thereafter or on 26 January 1998, appellant was asked by free from extraneous restraints.24
Elvira Ong-Chan, senior vice president of Metrobank, to report to x In People v. Muit,25 it was held that "one of the indicia of voluntariness in the
the Head Office on the following day. When appellant arrived at the execution of petitioner’s extrajudicial statement is that it contains many details
said office, he was surprised that there were seven (7) other and facts which the investigating officers could not have known and could not
people present: two (2) senior branch officers, two (2) bank have supplied without the knowledge and information given by him."
lawyers, two (2) policemen (one in uniform and the other in plain x Also, the fact that petitioner did not raise a whimper of protest and file any
clothes), and a representative of the Internal Affairs unit of the charges, criminal or administrative, against the investigator and the two
bank, Valentino Elevado. policemen present who allegedly intimidated him and forced him to sign negate his
o Appellant claimed that Elevado asked him to sign a paper (Exhibit bare assertions of compulsion and intimidation. It is a settled rule that where the
"N") in connection with the audit investigation; that he inquired defendant did not present evidence of compulsion, where he did not institute any
what he was made to sign but was not offered any explanation; criminal or administrative action against his supposed intimidators, where no
that he was intimidated to sign and was threatened by the police physical evidence of violence was presented, his extrajudicial statement shall be
that he will be brought to the precinct if he will not sign; that he considered as having been voluntarily executed.26
was not able to consult a lawyer since he was not apprised of the x Neither will petitioner’s assertion that he did not read the contents of his
purpose of the meeting; and that "just to get it over with" he statement before affixing his signature thereon "just to get it over with" prop up
signed the paper which turned out to be a confession. After the the instant Petition. To recall, petitioner has a masteral degree from a reputable
said meeting, appellant went to see Tan at his office but was educational institution and had been a bank manager for quite a number of
unable to find the latter. He also tried to phone him but to no avail. years. He is thus expected to fully understand and comprehend the significance
x RTC Æ CONVICTED Tanenggee of signing an instrument. It is just unfortunate that he did not exercise due
x Tanenggee Æ appealed to CA diligence in the conduct of his own affairs. He can therefore expect no
x CA Æ affirmed RTC consideration for it.
x Tanenggee Æ filed a Rule 45 before the SC

ISSUE: Whether Tangengee’s written statement is admissible in evidence and may be f. Conduct and Character
regarded as a confession.
Section 34. Similar acts as evidence. — Evidence that one did or did not do a certain thing
HELD: YES.
at one time is not admissible to prove that he did or did not do the same or similar thing at
x The constitutional proscription against the admissibility of admission or
another time; but it may be received to prove a specific intent or knowledge; identity, plan,
confession of guilt obtained in violation of Section 12, Article III of the
system, scheme, habit, custom or usage, and the like. (48a)
Constitution, as correctly observed by the CA and the OSG, is applicable only in
custodial interrogation.
Section 35. Unaccepted offer. — An offer in writing to pay a particular sum of money or to
x Custodial interrogation means any questioning initiated by law enforcement
deliver a written instrument or specific personal property is, if rejected without valid cause,
authorities after a person is taken into custody or otherwise deprived of his
equivalent to the actual production and tender of the money, instrument, or property. (49a)
freedom of action in any significant manner.
267
RECTO, GAYLE ANGELI M.
2011-0008 | AUSL
Personal Notes on Remedial Law 2 Review (based on the syllabus of Prof. Henedino M. Brondial)

ANNOTATIONS of such sworn statment; the trial court sustained the objection and
rejected the evidence for the purpose it was initially offered.
x Section 34 Æ 2nd branch of the res inter alios acta rule
ISSUE: Whether the RTC was correct in admitting Exhibit “EE”, considering Santos’ contention
x Applies to both civil and criminal cases that it was admitted in violation of Section 34 of Rule 130.
x Strictly enforced whenever applicable, just like the 1st branch of the
rule HELD: YES.
x XPNs to this branch of the res inter alios acta rule: x The trial court admitted the same as falling within one or more of the exceptions
o Where the evidence or similar acts may prove any of the set out in Section 34, Rule 130 of the Rules of Court, which reads:
o Sec. 34. Similar Acts as Evifence. — Evidence that one did or
ff:
didnot do a certain thing at one time is not admissible to prove that
ƒ Specific intent or knowledge he did or did not do the same or a similar thing at another time;
ƒ Identity but it may be received to prove a specific intent or knowledge,
ƒ Plan, System, or Scheme identity, plan, system, shceme, habit, custom or usage and the like.
ƒ Established Habit, Custom or usage, and the (Empahsis supplied).
like x Appellant Santos now complains that the affidavit of Ronaldo Guerrero was
hearsay evidence, considering that the prosecution did not present Ronaldo
x Hence, evidence of another crime (crime # 1) is ADMISSIBLE in a
Guerrero as a witness during the trial. We consider that the trial court did
prosecution for a certain crime (crime # 2) where it has a tendency not commit reversible error in admitting the Guerrero affidavit for the
to identify the accused or show his presence at the scene of the limited purpose for provign knowledge or plan or shceme, and more
crime specifically, that appellant knew that the particular corner of two (2)
o BUT it shall NOT be admitted to prove that the accused particular streets in Manila was a good place to ambus a vehicle and its
committed another crime wholly independent of that for passenters. Appellant also had waived the hearsay character of this
evidence by failure seasonably to ojbect to the admission of the
which he is on trial
affidavit; it is too late in that day to raise the hearsay rule in the
x Previous acts of negligence Æ may be admitted in evidence on a trial appellant's memorandum after prosecution and defense had presented
for a crime for the purpose of showing knowledge or intent their respective cases and had made their repsective offers of
x Section 35 Æ evidentiary complement to the rule on tender of evidence. 21 Finally, and in any cae, as pointed out by the Solicitor General, the
payment under the Civil Code, viz: exclusion of the Guerrero affidavit would not result in any change in the result
o Article 1256. If the creditor to whom tender of payment has been reache by the trial court. For that result is esentially and adequately based upon
made refuses without just cause to accept it, the debtor shall be the positive identification of appellant Santos as one of the gunmen by Baustisa
released from responsibility by the consignation of the thing or sum and Bohol.
x That it took the police authorities five (5) months to locate and apprehend
due.
Consignation alone shall produce the same effect in the following appellant Santos who, it turned out, resided close by the very locale of the
cases: ambush-slaying, did not in any way weaken the evidence of the prosecution of
ƒ (1) When the creditor is absent or unknown, or does detract from the conclusions reached by the trial court. The length of that period
not appear at the place of payment; of time shows only that police procedures are not always as efficient as they
ƒ (2) When he is incapacitated to receive the payment could be and that witnesses are frequently reluctant to voluntee information to the
at the time it is due; police authorities in criminal cases, a point noted so frequently as to have become
ƒ (3) When, without just cause, he refuses to give a a matter of judicial notice
receipt;
ƒ (4) When two or more persons claim the same right
to collect; PEOPLE OF THE PHILIPPINES, plaintiff-appellee, vs. ALFREDO NARDO Y ROSALES,
ƒ (5) When the title of the obligation has been lost. accused-appellant.
G.R. No. 133888 March 1, 2001
(1176a)
x Such offer of payment must be made in writing EN BANC
x REQUIREMENT so that said offer of payment shall produce the FACTS:
effects of a valid payment Æ consignation of the amount in x Nardo was charged with rape before the RTC Albay
court o Accused of raping his eldest daughter, Lorelyn
x RTC Æ convicted Nardo
x Automatic review ensued
PEOPLE OF THE PHILIPPINES, plaintiff-appellee, vs. RAUL SANTOS Y NARCISO, MARIO o Nardo relies on 4 letters allegedly written by Lorelyn addressed to
MORALES Y BACANI, PETER DOE and RICHARD DOE, Accused, RAUL SANTOS y NARCISO, the PAO lawyer of Nardo requesting that the latter be released
accused-appellant. from jail.
G.R. Nos. 100225-26 May 11, 1993
THIRD DIVISION ISSUE: Whether the allegations relative to Lorelyn’s lying behavior which were raised during
trial may be used as basis for the acquittal of her father, Nardo, from the crime of rape.
FACTS:
x Santos and 3 other persons were charged with murder and frustrated murder HELD: NO.
before RTC x Accused-appellant relies on these letters to obtain a reversal of the trial court's
x RTC Æ CONVICTED Santos judgment of his conviction. However, the said letters were not subscribed and
x Santos Æ appealed to SC sworn to by Lorielyn.
o In respect of the first assigned error, appellant Santos contendes x Be that as it may, recantations are frowned upon by the courts. A recantation of
that the testimonies of the principal prosecution witnesses do not a testimony is exceedingly unreliable, for there is always the probability that such
coform with the "knowledge and common experience of mankind." recantation may later on be itself repudiated. Courts look with disfavor upon
o Appellant argues that the two (2) prosecution witnesses, the victim retractions, because they can easily be obtained from witnesses through
Bautista and Police Aide Bohol, testified that they saw the accused intimidation or for monetary consideration. A retraction does not necessarily
for the first time in their lives when the crime was committed and yet negate an earlier declaration.40 Especially, recantations made after the
identified him as one of the gunmen five (5) months later in the conviction of the accused deserve only scant consideration.41
Police headquarters in Navotas. x Moreover, any recantation or affidavit of desistance, by itself, even when
o The ambuscade and the slaying of Glicierio Cupcupin happened on construed as a pardon in the so-called "private crimes," is not a ground for the
26 May 1989; appellant Santos was identified at the police station dismissal of the criminal case once the action has been instituted.42 The pardon to
on 25 October 1989. justify the dismissal of the complaint should be made prior to the institution of the
o Appellant argues that this lapse of time was unreasonable, which, criminal action.43 Parenthetically, the crime in the case at bar was
when coupled with the brief, limited and obstructed view which the committed in 1996, i.e., prior to the passage of the R.A. 8353, The Anti-Rape
prosecution witnesses had of the gunmen at the time of the Law of 1997, which reclassified rape as a crime against persons.
shooting, casts serious doubt on the accuracy and reliabitlity of the x Even if it were sworn, Lorielyn's recantation could hardly suffice to overturn the
identification by the witnesses. finding of guilt by the trial court which was based on her own clear and
o In his second assignment of error, appellant Santos in effect convincing testimony, given during a full-blown trial. An affidavit of recantation,
questions the trial court for admitting a sworn statement by one being usually taken ex parte, would be considered inferior to the testimony given
Ronaldo Guerrero (Exhibit "EE"), a witnesses in another criminal in open court. It would be a dangerous rule to reject the testimony taken before
case (Criminal Case No. 8117) where appellant Santos was also a court of justice simply because the witness who gave it later on changed
charged with the murder of one Daniel Nuguera which had taken his/her mind for one reason or another. Such a rule would make a solemn trial a
place in the very same site where Bautista and Cupcupin were mockery, and place the proceedings at the mercy of unscrupulous witnesses.44
ambushed, i.e., at the corner of Yangco Street and Estrella Street, x As stated, the trial court arrived at its finding of guilt after a careful assessment
Malabon, Metro Manila. When the prosecution first presented the of the evidence presented, foremost of which was the testimony of the victim in
sworn statement of Guerrero in order to show criminal propensity open court, where the trial judge was able to personally evaluate her manner of
on the part of appellant Santos, the defesne objected to admission testifying, and from there reach a studied opinion as to her credibility. As a rule,

268
RECTO, GAYLE ANGELI M.
2011-0008 | AUSL
Personal Notes on Remedial Law 2 Review (based on the syllabus of Prof. Henedino M. Brondial)

we do not disturb the findings by the trial court on the credibility of witnesses, impossible for him to be at the scene of the crime when it was committed.
for the trial court is in a better position to pass upon the same.45 Accused-appellant's defense of alibi must, therefore, necessarily fail.
x "The trial judge is in a better position to decide the question of credibility, since x Carefully sifting through the entire body of evidence presented in this case, we
he personally heard the witnesses and observed their deportment and manner of find nothing which would destroy the moral certainty of accused- appellant's
testifying. He had before him the essential aids to determine whether a witness guilt. While there may be some inconsistencies in the testimony of Lorielyn,
was telling the truth or lying. Truth does not always stalk boldly forth naked; she these to our mind are minor inconsistencies which serve to strengthen her
often hides in nooks and crannies visible only to the mind's eye of the judge who credibility as they are badges of truth rather than indicia of falsehood.55 Minor
tried the case. To him appears the furtive glance, the blush of conscious shame, inconsistencies do not affect the credibility of witnesses, as they may even tend
the hesitation, the sincere or flippant or sneering tone, the heat, the calmness, to strengthen rather than weaken their credibility. Inconsistencies in the
the yawn, the sigh, the candor or lack of it, the scant or full realization of the testimony of prosecution witnesses with respect to minor details and collateral
solemnity of an oath, the carriage and mien."46 matters do not affect either the substance of their declaration, their veracity, or
x We find nothing in the records which would indicate that the findings of fact of the weight of their testimony. Such minor flaws may even enhance the worth of
the trial court are not supported by the evidence or were arrived at in manifest a testimony, for they guard against memorized falsities.56 Besides, a rape victim
or palpable error, such as to warrant a departure from the foregoing rule. The can not be expected to recall vividly all the sordid details of the violation
trial court was correct in lending credibility to the testimony of Lorielyn. The sole committed against her virtue.
testimony of Lorielyn was sufficient to establish the guilt of accused-appellant. It x Article 335 of the Revised Penal Code, as amended by Republic Act No. 7659,
is settled that a person accused of rape can be convicted solely on the testimony provides:
of the victim if the trial court finds said testimony to be credible, natural, o The death penalty shall also be imposed if the crime of rape is
convincing, and consistent with human nature and the course of things.47 committed with any of the following attendant circumstances
x Indeed, a daughter, especially one in her minority, would not accuse her own ƒ 1. when the victim is under eighteen (18)
father of such an unspeakable crime as incestuous rape had she really not been years of age and the offender is a parent, ascendant,
aggrieved.48 More importantly, Lorielyn withstood all the rigors of the case, stepparent, guardian, relative by consanguinity or
starting from the initial police interrogation, the medical examination, the formal affinity within the third civil degree, or the common-
charge, the public trial, to the cross-examination. She went through the court law-spouse of the parent of the victim ....... 57
hearings, where she came face to face with her father. If it was true that she x The concurrence of the two special qualifying circumstances, namely the victim's
merely made up the charge, she should have been bothered by her conscience minority and the relationship between the victim and the culprit, increases the
at the sight of her father in prison garb and upon the realization of his sorry penalty of rape to one (1) degree, thus resulting in the imposition of the death
state while in detention. The fact that she maintained her story during her penalty. In order to be appreciated as qualifying circumstances, however, these
testimony-in-chief all the way up to her rebuttal testimony only serves to must be properly pleaded in the indictment.58 In addition, the qualifying
substantiate the veracity of her claim. circumstances should be duly proved during the trial.59
x Well settled is the rule that no woman would concoct a story of defloration, allow x These requirements are met in this case. The Information sufficiently alleges that
an examination of her private parts and submit herself to public humiliation and accused-appellant is the father of the victim, and that the latter was fourteen
scrutiny via an open trial, if her sordid tale was not true and her sole motivation (14) years old at the time of commission of the rape. These elements,
was not to have the culprit apprehended and punished.49 A young girl's furthermore, were categorically affirmed by Elizabeth Nardo, the victim's mother
revelation that she has been raped, coupled with her voluntary submission to and the most competent witness. She testified that accused-appellant is
medical examination and her willingness to undergo public trial where she could Lorielyn's father, and that Lorielyn was born on September 11, 1981,60 thus
be compelled to give out the details of an assault on her dignity by, as in this placing her age at the time of the rape at fourteen (14) years. Moreover, the
case, her own father, cannot be so easily dismissed as a mere concoction. 50 Lorielyn's birth date and her relationship to accused-appellant are shown by her
Courts usually give credence to the testimony of a girl who is a victim of sexual Certificate of Baptism.61 This was presented by her mother, Elizabeth, in lieu of
assault, particularly if it constitutes incestuous rape because, normally, no person her Certificate of Live Birth, which was destroyed by fire.62 The baptismal
would be willing to undergo the humiliation of a public trial and to testify on the certificate, coupled by her mother's testimony, is sufficient to establish Lorielyn's
details of her ordeal were it not to condemn an injustice. Needless to say, it is age.63
settled jurisprudence that testimonies of child-victims are given full weight and o We therefore affirm the trial court's imposition of the death
credit, since when a woman, more so if she is a minor, says that she has been penalty.
raped, she says in effect all that is necessary to show that rape was committed. x Four justices of the Court have continued to maintain the unconstitutionality of
Youth and immaturity are generally badges of truth and sincerity.51 Republic Act No. 7659 insofar as it prescribes the death penalty; nevertheless
x During the trial, the defense endeavored to portray Lorielyn as an they submit to the ruling of the majority to the effect that this law is
incorrigible liar. Occasions were cited wherein Lorielyn supposedly lied constitutional and that the death penalty can be lawfully imposed in the case at
in order to obtain money or her parents' permission to leave the house. bar.
However, Rule 130, Section 34, of the Rules of Court provides that:
"Evidence that one did or did not do a certain thing at one time is not REPUBLIC OF THE PHILIPPINES, represented by the Department of Environment and
admissible to prove that he did nor did not do the same or a similar Natural Resources, petitioner, vs. HEIRS OF FELIPE ALEJAGA SR., represented by
thing at another time; but it may be received to prove a specific intent ROQUETA ALEJAGA, FELIPE ALEJAGA JR., MARIA DULLA ALEJAGA, FELIPE ALEJAGA III,
or knowledge, identity, plan, system, scheme, habit, custom or usage, ROQUETA ALEJAGA, JENNIFER ALEJAGA, EVERETTE CAPUNDAN, AND LYNETTE ALEJAGA;
and the like." While lying may constitute a habit, we believe that the THE PHILIPPINE NATIONAL BANK and THE REGISTER OF DEEDS OF ROXAS CITY,
falsehoods committed by Lorielyn, assuming them for the moment to respondents
be true, are petty and inconsequential. They are not as serious as G.R. No. 146030 December 3, 2002
charging one's own father of the sordid crime of rape, with all of its THIRD DIVISION
serious repercussions.
x Accused-appellant argues that the trial court should have given credence to his FACTS:
witness, Atty. Santer G. Gonzales, because he is a member of the bar. Atty. x The District Land Officer of Roxas City approved the application and the issuance
Gonzales, however, took the witness stand not as a lawyer but as an ordinary of Free Patent to Felipe Alejaga.
person. He testified in his capacity as accused-appellant's employer. As such, no x The government through the Solicitor General Æ then instituted an action for
special privilege should be accorded him by the trial court by reason only of his Annulment/Cancellation of Patent and Title and Reversion against Alejaga, on the
being a member of the bar. He did not appear in that case as an officer of the ground that the approval of the same was acquired through fraud, hence null
court but as a mere witness, and hence should be treated as one. and void ab initio.
x Likewise, accused-appellant insists that Lorielyn's conduct after the rape, during x RTC Æ ruled in favor of Alejaga.
which she continued to perform her tasks and lived with her father in their x SolGen Æ appealed to CA
house, negates the commission of rape. Accused-appellant's proposition is x CA Æreversed
derived from Lorielyn's perfunctory yes-or-no answers to the leading questions o Petitioner failed to prove its allegation that respondents had
propounded to her on cross-examination. Rather than sustain this argument, we obtained the free patent and the Certificate of Title through fraud
rely instead on the observations of the Social Welfare Officer, whom we find to be and misrepresentation
an impartial witness, in this wise: o Also, CA brushed aside as hearsay Special Investigator Isagani
x Per observation, Lorielyn is a shy and silent type person. She talked in a very Cartagena’s testimony that Land Inspector Efren L. Recio had not
small voice and during the interview she only talks when being asked. She also conducted an investigation on the free patent application of
appears to be very sad and have been staring blankly (sic).52 Alejaga.
x Accused-appellant assigns as error the trial court's failure to give the reasons for
recommending the commutation of his sentence from death to reclusion ISSUE: Whether CA was correct in considering Cartagena’s testimony as hearsay.
perpetua. As correctly observed by the Solicitor General, the trial court was
impelled by humanitarian reason.53 Moreover, the commutation of sentence is a HELD: NO.
prerogative of the Chief Executive. x The report of Special Investigator Cartagena has not been successfully rebutted.
x As against the positive and categorical testimony of Lorielyn, accused-appellant o In that report, Recio supposedly admitted that he had not actually
can only proffer the defense of alibi. However, in order to overcome the evidence conducted an investigation and ocular inspection of the parcel of
of the prosecution with the defense of alibi, he must establish not only that he was land.
somewhere else when the crime was committed but also that it was o Cartagena’s statement on Recio’s alleged admission may be
physically impossible for him to have been at the scene of the crime at the time considered as “independently relevant.” A witness may testify
it was committed.54 In the instant case, the testimonies for the defense sought as to the state of mind of another person -- the latter’s
to establish that accused-appellant was 400 to 500 meters, or 15 minutes, away knowledge, belief, or good or bad faith -- and the former’s
from the scene of the crime. This hardly qualifies as proof that it was physically statements may then be regarded as independently
relevant without violating the hearsay rule.
269
RECTO, GAYLE ANGELI M.
2011-0008 | AUSL
Personal Notes on Remedial Law 2 Review (based on the syllabus of Prof. Henedino M. Brondial)

x Thus, because Cartagena took the witness stand and opened himself to cross- x Showing the lack of credibility of
examination, the Investigation Report he had submitted to the director of the the witness
Bureau of Lands constitutes part of his testimony. Those portions of the report
x Newspaper clippings Æ hearsay
that consisted of his personal knowledge, perceptions and conclusions are not
hearsay. On the other hand, the part referring to the statement made by Recio o Have NO evidentiary value unless substantiated by
may be considered as independently relevant. persons with personal knowledge of said facts
x The doctrine on independently relevant statements holds that
conversations communicated to a witness by a third person may be CASE
admitted as proof that, regardless of their truth or falsity, they were
actually made. ANNA LERIMA PATULA, Petitioner, vs. PEOPLE OF THE PHILIPPINES, Respondent.
o Evidence as to the making of such statements is not secondary but G.R. No. 164457 April 11, 2012
primary, for in itself it: FIRST DIVISION
ƒ constitutes a fact in issue or
ƒ is circumstantially relevant to the existence of such FACTS:
fact x Patula was charged with estafa before RTC Dumaguete
x Since Cartagena’s testimony was based on the report of the investigation he had o employee of Footlucker’s, starting as a saleslady in 1996 until she
conducted, his testimony was not hearsay and was, hence, properly admitted by became a sales representative; that as a sales representative she
the trial court. was authorized to take orders from wholesale customers coming
from different towns (like Bacong, Zamboanguita, Valencia,
Lumbangan and Mabinay in Negros Oriental, and Siquijor), and to
collect payments from them; that she could issue and sign official
receipts of Footlucker’s for the payments, which she would then
remit; that she would then submit the receipts for the payments for
tallying and reconciliation
g. Hearsay Evidence Rule
x Prosecution evidence:
o The only other witness for the Prosecution was Karen Guivencan,
Section 36. Testimony generally confined to personal knowledge; hearsay whomFootlucker’s employed as its store auditor since November
excluded. — A witness can testify only to those facts which he knows of his personal 16, 1995 until her resignation on March 31, 2001. She declared
knowledge; that is, which are derived from his own perception, except as otherwise provided that Go had requested her to audit petitioner after some customers
in these rules. (30a) had told him that they had already paid their accounts but the
office ledger had still reflected outstandingbalances for them; that
ANNOTATIONS she first conducted her audit by going to the customers in places
from Mabinay to Zamboanguitain Negros Oriental, and then in
x Section 36 Æ hearsay evidence rule Siquijor; thatshe discovered in the course of her audit that the
x SCOPE: any form of evidence amounts appearing on the original copies of receipts in the
possession of around 50 customers varied from the amounts
o NOT based on personal knowledge of the witness x
written on the duplicate copies of the receipts petitioner submitted
REASON behind exclusion: the party against whom it is presented is to the office; that upon completing her audit, she submittedto Go a
deprived of his right and opportunity to cross-examine the persons to written report denominated as "List of Customers Covered by
whom the statements or writings are attributed Saleswoman LERIMA PATULA w/ Differences in Records as per
x If NOT timely objected to Æ WAIVED Audit Duly Verified March 16-20, 1997" marked as Exhibit A; and
o Implied waiver Æ repeated failure of a party to cross- that based on the report, petitioner had misappropriated the total
amount ofP131,286.92.3
examine a witness
o During Guivencan’s stint as a witness, the Prosecution marked the
ƒ EFFECT: the testimony of the witness shall ledgers of petitioner’s various customers allegedly with
NOT be excluded discrepancies as Exhibits B to YYand their derivatives, inclusive.
o EFFECT if waived: hearsay evidence shall be given Each of the ledgers had a first column that contained the dates of
importance as if it were not hearsay the entries, a second that identified the invoices by the number, a
x Section 28 of AM 00-4-07-SC (Rule on Examination of a Child third that statedthe debit, a fourth that noted the credit (or the
amounts paid), and a fifth that summed the balances (debit minus
Witness)
credit).Only 49 of theledgerswere formally offered and admitted by
o Hearsay evidence of a child regarding sexual abuses is the RTC because the 50thledger could no longer be found.
now ADMISSIBLE subject to the right to cross- x RTC Æ convicted Patula
examination and other prerequisites x Patula Æ filed Rule 45 before the SC
x Sections 37-47 of the RoC Æ provides for the 11 XPNs to the o Insisting that the RTC’s judgment "grossly violated [her]
hearsay rule Constitutional and statutory right to be informed of the nature and
cause of the accusation against her because, while the charge
o Section 47 (re testimony or deposition at a former
against her is estafa under Art. 315, par. 1 (b) of the Revised Penal
proceeding) Æ technically NOT an XPN to the hearsay Code, the evidence presented against her and upon which her
rule because it requires that the party had either cross- conviction was based, was falsification, an offense not alleged or
examined OR had the opportunity of cross-examining the included in the Information under which she was arraigned and
witness pleaded not guilty," and that said judgment likewise "blatantly
x Hearsay rule CANNOT be invoked in the ff cases: ignored and manifestly disregarded the rules on admission of
evidence in that the documentary evidence admitted by the trial
o Where the statements/writings attributed to a person
court were all private documents, the due execution and
who is not in the witness stand are being presented ONLY authenticity of which were not proved in accordance with Sec. 20
for to prove that those statements were actually made or of Rule 132 of the Revised Rules on Evidence,"
that those writings were actually executed and NOT for
the purpose of proving the truth of the facts therein ISSUE: Whether RTC correctly admitted the subject testimonial and documentary pieces of
ƒ Examples: evidence.
x Witness may testify that he heard HELD: NO.
or saw the execution of a x Nonetheless, in all criminal prosecutions, the Prosecution bears the burden to
document establish the guilt of the accused beyond reasonable doubt. In discharging this
o Doctrine of independently relevant statements Æ burden, the Prosecution’s duty is to prove each and every element of the crime
independent of whether the facts stated are true or not, charged in the information to warrant a finding of guilt for that crime or for any
they are relevant since they are the facts in issue or are other crime necessarily included therein.14 The Prosecution must further prove
the participation of the accused in the commission of the offense.15In doing all
circumstantial evidence of the facts in issue these, the Prosecution must rely on the strength of its own evidence, and not
ƒ Examples: anchor its success upon the weakness of the evidence of the accused. The
x Showing his state of mind burden of proof placed on the Prosecution arises from the presumption of
x Which show his physical condition innocence in favor of the accused that no less than the Constitution has
x From which an inference may be guaranteed.16Conversely, as to his innocence, the accused has no burden of
made as to the state of the mind proof,17that he must then be acquitted and set free should the Prosecution not
overcome the presumption of innocence in his favor.In other words, the
of another weakness of the defense put up by the accused is inconsequential in the
x Which may identify the date, place proceedings for as long as the Prosecution has not discharged its burden of proof
and person in question in establishing the commission of the crime charged and in identifying the accused
as the malefactor responsible for it.

270
RECTO, GAYLE ANGELI M.
2011-0008 | AUSL
Personal Notes on Remedial Law 2 Review (based on the syllabus of Prof. Henedino M. Brondial)

x Did the Prosecution adduce evidence that proved beyond reasonable doubt the x The secondsolution is to require that all witnesses besubject to the cross-
guilt of petitioner for the estafa charged in the information? examination by the adverse party. Section 6, Rule 132 of the Rules of
x To establish the elements of estafaearlier mentioned, the Prosecution presented Courtensuresthis solutionthusly:
the testimonies of Go and Guivencan, and various documentsconsisting of: (a) o Section 6. Cross-examination; its purpose and extent. - Upon the
the receipts allegedly issued by petitioner to each of her customers upon their termination of the direct examination, the witness may be cross-
payment, (b) the ledgers listing the accounts pertaining to each customer with examined by the adverse party as to any matters stated in the
the corresponding notations of the receipt numbers for each of the payments, direct examination, or connected therewith, with sufficient fullness
and (c) the confirmation sheets accomplished by Guivencan herself.18The and freedom to test his accuracy and truthfulness and freedom
ledgers and receipts were marked and formally offered as Exhibits B to YY, and from interest or bias, or the reverse, and to elicit all important facts
their derivatives, inclusive. bearing upon the issue. (8a)
x On his part, Go essentially described for the trial court the various duties of x Although the second solution traces its existence to a Constitutional precept
petitioner as Footlucker’s sales representative. On her part, Guivencan conceded relevant to criminal cases, i.e., Section 14, (2), Article III, of the 1987
having no personal knowledge of the amounts actually received by petitioner Constitution,which guarantees that: "In all criminal prosecutions, the accused
from the customersor remitted by petitioner to Footlucker’s.This means that shall xxx enjoy the right xxx to meet the witnesses face to face xxx," the rule
persons other than Guivencan prepared Exhibits B to YY and their derivatives, requiring the cross-examination by the adverse party equally applies to non-
inclusive,and that Guivencan based her testimony on the entries found in the criminal proceedings.
receipts supposedly issued by petitioner and in the ledgers held by Footlucker’s x We thus stress that the rule excluding hearsay as evidence is based upon serious
corresponding to each customer, as well as on the unsworn statements of some concerns about the trustworthiness and reliability of hearsay evidence due to its
of the customers. Accordingly, her being the only witness who testified on the not being given under oath or solemn affirmation and due to its not being
entries effectively deprived the RTC of the reasonable opportunity to validate and subjected to cross-examination by the opposing counsel to test the perception,
test the veracity and reliability of the entries as evidence of petitioner’s memory, veracity and articulateness of the out-of-court declarant or actor upon
misappropriation or conversion through cross-examination by petitioner. The whose reliability the worth of the out-of-court statement depends.27
denial of that opportunity rendered theentire proof of misappropriation or x Based on the foregoing considerations, Guivencan’s testimony as well as Exhibits
conversion hearsay, and thus unreliable and untrustworthy for purposes of B to YY, and their derivatives, inclusive, must be entirely rejected as proof of
determining the guilt or innocence of the accused. petitioner’s misappropriation or conversion.
x To elucidate why the Prosecution’s hearsay evidence was unreliable and x Lack of their proper authentication rendered Exhibits B to YY and their
untrustworthy, and thus devoid of probative value, reference is made toSection derivatives inadmissible as judicial evidence
36 of Rule 130, Rules of Court, a rule that states that a witness can testify x Petitioner also contends that the RTC grossly erred in admitting as evidence
only to those facts that she knows of her personal knowledge; that is, Exhibits B to YY, and their derivatives, inclusive, despite their being private
which are derived from her own perception, except as otherwise documents that were not duly authenticated as required by Section 20, Rule 132
provided in the Rules of Court. The personal knowledge of a witness is of the Rules of Court.
a substantive prerequisite for accepting testimonial evidence that x Section 19, Rule 132 of the Rules of Courtdistinguishes between a public
establishes the truth of a disputed fact. A witness bereft ofpersonal document and a private document for the purpose of their presentation in
knowledge of the disputed fact cannot be called upon for that purpose evidence, viz:
because her testimony derives its value not from the credit accorded to o Section 19. Classes of documents. - For the purpose of their
her as a witness presently testifying but from the veracity and presentation in evidence, documents are either public or private.
competency of the extrajudicial source of her information. o Public documents are:
x In case a witness is permitted to testify based on what she has heard another ƒ (a) The written official acts, or records of the official
person say about the facts in dispute, the person from whom the witness derived acts of the sovereign authority, official bodies and
the information on the facts in dispute is not in court and under oath to be tribunals, and public officers, whether of the
examined and cross-examined. The weight of such testimony thendepends Philippines, or of a foreign country;
not upon theveracity of the witness but upon the veracity of the other ƒ (b) Documents acknowledged before a notary public
person giving the information to the witness without oath. The except last wills and testaments, and
information cannot be tested because the declarant is not standing in ƒ (c) Public records, kept in the Philippines, of private
court as a witness andcannot, therefore, be cross-examined. documents required by law to be entered therein.
x It is apparent, too, that a person who relates a hearsay is not obliged to enter o All other writings are private.
into any particular, to answer any question, to solve any difficulties, to reconcile x The nature of documents as either public or private determines how the
any contradictions, to explain any obscurities, to remove any ambiguities; and documents may be presented as evidence in court. A public document, by virtue
that she entrenches herself in the simple assertion that she was told so, and of its official or sovereign character, or because it has been acknowledged before
leaves the burden entirely upon the dead or absent author.19 Thus, the rule a notary public (except a notarial will) or a competent public official with the
against hearsay testimony rests mainly on the ground that there was no formalities required by law, or because it is a public record of a private writing
opportunity to cross-examine the declarant.20 The testimony may have been authorized by law, is self-authenticating and requires no further authentication in
given under oath and before a court of justice, but if it is offered against a party order to be presented as evidence in court.In contrast, a private document is any
who is afforded no opportunity to cross-examine the witness, it is hearsay just other writing, deed, or instrument executed by a private person without the
the same.21 intervention of a notary or other person legally authorized by which some
x Moreover, the theory of the hearsay rule is that when a human utterance is disposition or agreement is proved or set forth. Lacking the official or sovereign
offered as evidence of the truth of the fact asserted, the credit of the assertor character of a public document, or the solemnities prescribed by law, a private
becomes the basis of inference, and, therefore, the assertion can be received as document requires authentication in the manner allowed by law or the Rules of
evidence only when made on the witness stand, subject to the test of cross- Court before its acceptance as evidence in court.
examination. However, if an extrajudicial utterance is offered, not as an assertion x The requirement of authentication of a private document is excused only in four
to prove the matter asserted but without reference to the truth of the matter instances, specifically:
asserted, the hearsay rule does not apply. For example, in a slander case, if a o (a) when the document is an ancient one within the context of
prosecution witness testifies that he heard the accused say that the complainant Section 21,28 Rule 132 of the Rules of Court;
was a thief, this testimony is admissible not to prove that the complainant was o (b) when the genuineness and authenticity of an actionable
really a thief, but merely to show that the accused uttered those words.22 This document have not been specifically denied under oath by the
kind of utterance ishearsay in character but is not legal hearsay.23The distinction adverse party;29
is, therefore, between (a) the fact that the statement was made, to which the o (c) when thegenuineness and authenticity of the document have
hearsay rule does not apply, and (b) the truth of the facts asserted in the been admitted;30 or
statement, to which the hearsay rule applies.24 o (d) when the document is not being offered as genuine.31
x Section 36, Rule 130 of the Rules of Court is understandably not the x There is no question that Exhibits B to YY and their derivatives were private
only rule that explains why testimony that is hearsay should be documents because private individuals executed or generated them for private or
excluded from consideration. Excluding hearsay also aims to preserve business purposes or uses. Considering that none of the exhibits came under any
the right of the opposing party to cross-examine the originaldeclarant of the four exceptions, they could not be presented and admitted as evidence
claiming to have a direct knowledge of the transaction or against petitioner without the Prosecution dutifully seeing to their authentication
occurrence.25If hearsay is allowed, the right stands to be denied because the in the manner provided in Section20 of Rule 132 of the Rules of Court,viz:
declarant is not in court.26It is then to be stressed that the right to cross- o Section 20. Proof of private documents. - Before any private
examine the adverse party’s witness, being the only means of testing the document offered as authentic is received in evidence, its due
credibility of witnesses and their testimonies, is essential to the administration of execution and authenticity must be proved either:
justice. ƒ (a) By anyone who saw the document executed or
x To address the problem of controlling inadmissible hearsay as evidence to written; or
establish the truth in a dispute while also safeguardinga party’s right to cross- ƒ (b) By evidence of the genuineness of the signature
examine her adversary’s witness,the Rules of Court offers two solutions. The or handwriting of the maker.
firstsolution is to require that allthe witnesses in a judicial trial or hearing be o Any other private document need only be identified as that which it
examined only in courtunder oath or affirmation. Section 1, Rule 132 of the Rules is claimed to be.
of Court formalizes this solution,viz: x The mystery shrouding the RTC’s soft treatment of the Prosecution’s flawed
o Section 1. Examination to be done in open court. - The examination presentation was avoidable simply by the RTC adhering to the instructions of the
of witnesses presented in a trial or hearing shall be done in open rules earlier quoted, as well as withSection 22 of Rule 132 of the Rules of
court, and under oath or affirmation. Unless the witness is Court,which contains instructions on how to prove the genuineness of a
incapacitated to speak, or the question calls for a different mode of handwriting in a judicial proceeding, as follows:
answer, the answers of the witness shall be given orally. (1a
271
RECTO, GAYLE ANGELI M.
2011-0008 | AUSL
Personal Notes on Remedial Law 2 Review (based on the syllabus of Prof. Henedino M. Brondial)

o Section 22. How genuineness of handwriting proved. - The overruling of petitioner’s objections imbue the flawed evidence with any virtue
handwriting of a person may be proved by any witness who and value.
believes it to be the handwriting of such person because he has x Curiously, the RTC excepted the entries in the ledgers from the application of the
seen the person write, or has seen writing purporting to be his hearsay rule by also terselystating that the ledgers "were prepared in the regular
upon which the witness has acted or been charged, and has thus course of business."40Seemingly, the RTC applied Section 43, Rule 130 of the
acquired knowledge of the handwriting of such person. Evidence Rules of Court, to wit:
respecting the handwriting may also be given by a comparison, o Section 43. Entries in the course of business. - Entries made at, or
made by the witness or the court, with writings admitted or treated near the time of the transactions to which they refer, by a person
as genuine by the party against whom the evidence is offered, or deceased, or unable to testify, who was in a position to know the
proved to be genuine to the satisfaction of the judge. (Emphases facts therein stated, may be received as prima facie evidence, if
supplied) such person made the entries in his professional capacity or in the
x If it is already clear that Go and Guivencan had not themselves seen the performance of duty and in the ordinary or regular course of
execution or signing of the documents,the Prosecution surely did not business or duty.
authenticate Exhibits B to YY and their derivatives conformably with the x This was another grave error of the RTC.The terse yet sweeping mannerof
aforequoted rules. Hence, Exhibits B to YY, and their derivatives, inclusive, were justifying the application of Section 43 was unacceptable due to the need to
inescapably bereft of probative value as evidence. That was the onlyfair and just show the concurrence of the several requisites before entries in the course of
result, as the Court held in Malayan Insurance Co., Inc. v. Philippine Nails and business could be excepted from the hearsay rule. The requisites are as follows:
Wires Corporation:38 o (a) The person who made the entry must be dead or unable to
o On the first issue, petitioner Malayan Insurance Co., Inc., contends testify;
that Jeanne King’s testimony was hearsay because she had no o (b) The entries were made at or near the time of the transactions
personal knowledge of the execution of the documents supporting to which they refer;
respondent’s cause of action, such as the sales contract, invoice, o (c) The entrant was in a position to know the facts stated in the
packing list, bill of lading, SGS Report, and the Marine Cargo Policy. entries;
Petitioner avers that even though King was personally assigned to o (d) The entries were made in his professional capacity or in the
handle and monitor the importation of Philippine Nails and Wires performance of a duty, whether legal, contractual, moral, or
Corporation, herein respondent, this cannot be equated with religious;
personal knowledge of the facts which gave rise to respondent’s o (e) The entries were made in the ordinary or regular course of
cause of action. Further, petitioner asserts, even though she business or duty.41
personally prepared the summary of weight of steel billets received x The Court has to acquit petitioner for failure of the State to establish her guilt
by respondent, she did not have personal knowledge of the weight beyond reasonable doubt. The Court reiterates that in the trial of every criminal
of steel billets actually shipped and delivered. case, a judge must rigidly test the State’s evidence of guilt in order to ensure
o At the outset, we must stress that respondent’s cause of action is that such evidence adhered to the basic rules of admissibility before pronouncing
founded on breach of insurance contract covering cargo consisting an accused guilty of the crime charged upon such evidence. The failure of the
of imported steel billets. To hold petitioner liable, respondent has to judge to do so herein nullified the guarantee of due of process of law in favor of
prove, first, its importation of 10,053.400 metric tons of steel billets the accused, who had no obligation to prove her innocence. Heracquittal should
valued at P67,156,300.00, and second, the actual steel billets follow.
delivered to and received by the importer, namely the respondent.
Witness Jeanne King, who was assigned to handle respondent’s ______________________________________
importations, including their insurance coverage, has personal
knowledge of the volume of steel billets being imported, and i. Dying Declaration
therefore competent to testify thereon. Her testimony is not
hearsay, as this doctrine is defined in Section 36, Rule 130 of the
Section 37. Dying declaration. — The declaration of a dying person, made under the
Rules of Court.However, she is not qualified to testify on the
consciousness of an impending death, may be received in any case wherein his death is the
shortage in the delivery of the imported steel billets. She did not
subject of inquiry, as evidence of the cause and surrounding circumstances of such death.
have personal knowledge of the actual steel billets received. Even
(31a)
though she prepared the summary of the received steel billets, she
based the summary only on the receipts prepared by other
persons. Her testimony on steel billets received was hearsay. It has ANNOTATIONS
no probative value even if not objected to at the trial.
o On the second issue, petitioner avers that King failed to properly x Other terms for dying declaration:
authenticate respondent’s documentary evidence. Under Section o Ante mortem statement
20, Rule 132, Rules of Court, before a private document is admitted o Statement in articulo mortis
in evidence, it must be authenticated either by the person who x Requisites:
executed it, the person before whom its execution was
o Death is imminent and declarant is conscious of such fact
acknowledged, any person who was present and saw it executed,
or who after its execution, saw it and recognized the signatures, or ƒ Things to consider to determine
the person to whom the parties to the instruments had previously consciousness of declarant of imminence of
confessed execution thereof. In this case, respondent admits that death:
King was none of the aforementioned persons. She merely made x Words or statements of the
the summary of the weight of steel billets based on the declarant on the same occasion
unauthenticated bill of lading and the SGS report. Thus, the
summary of steel billets actually received had no proven real basis,
x Conduct at the time the
and King’s testimony on this point could not be taken at face value. declaration was made
o xxx Under the rules on evidence, documents are either public or x Serious nature of wounds as
private. Private documents are those that do not fall under any of would necessary engender a belief
the enumerations in Section 19, Rule 132 of the Rules of on his part that he would not
Court.Section 20of the same law, in turn, provides that before any survive therefrom
private document is received in evidence, its due execution and
authenticity must be proved either by anyone who saw the
ƒ GR: Intervening time Æ IMMATERIAL
document executed or written, or by evidence of the genuineness x XPN: Intervening time may be
of the signature or handwriting of the maker. Here, respondent’s taken into account where the
documentary exhibits are private documents. They are not among declaration is ambiguous as to
those enumerated in Section 19, thus, their due execution and whether the declarant believed
authenticity need to be proved before they can be admitted in that his death was imminent when
evidence.With the exception concerning the summary of the weight
of the steel billets imported, respondent presented no supporting
he made such declaration, viz:
evidence concerning their authenticity. Consequently, they cannot be o When declarant was
utilized to prove less of the insured cargo and/or the short delivery of unsure of imminence
the imported steel billets. In sum, we find no sufficient competent of death and he made
evidence to prove petitioner’s liability. a declaration and died
x That the Prosecution’s evidence was left uncontested because petitioner decided
a day after Æ such
not to subject Guivencan to cross-examination, and did not tender her contrary
evidencewas inconsequential. Although the trial court had overruled the may be considered as
seasonable objections to Guivencan’s testimony bypetitioner’s counsel due to the a dying declaration
hearsay character, it could not be denied thathearsay evidence, whether o When declarant was
objected to or not, had no probative value.39Verily, the flaws of the unsure of imminence
Prosecution’s evidence were fundamental and substantive, not merely technical of death and he made
and procedural, and were defects that the adverse party’s waiver of her cross-
a declaration and died
examination or failure to rebutcould not set right or cure. Nor did the trial court’s
several days after Æ

272
RECTO, GAYLE ANGELI M.
2011-0008 | AUSL
Personal Notes on Remedial Law 2 Review (based on the syllabus of Prof. Henedino M. Brondial)

such may not be ANNOTATIONS


considered as a dying
declaration BUT may
be classified as a res vi. Res Gestae
gestae IF made
immediately after the Section 42. Part of res gestae. — Statements made by a person while a starting
incident occurrence is taking place or immediately prior or subsequent thereto with respect to the
circumstances thereof, may be given in evidence as part of res gestae. So, also, statements
o Declaration refers to the cause and surrounding
accompanying an equivocal act material to the issue, and giving it a legal significance, may be
circumstances of such death received as part of the res gestae. (36a)
o Declaration relates to facts which the victim is competent
to testify to ANNOTATIONS
o Declaration is offered in a case wherein the declarant’s
death is the subject of inquiry CASES
x Once admitted Æ dying declaration TREATED similarly as testimonial
evidence vii. Entries in the Course of Business
x ADMISSIBILITY: only insofar as the facts regarding the cause and
surrounding the circumstances of the death are concerned Section 43. Entries in the course of business. — Entries made at, or near the time of
x SCOPE: any case transactions to which they refer, by a person deceased, or unable to testify, who was in a
x FORM: oral or written or made by signs which may be interpreted position to know the facts therein stated, may be received as prima facie evidence, if such
o If oral Æ witness who heard it must be produced without person made the entries in his professional capacity or in the performance of duty and in the
ordinary or regular course of business or duty. (37a)
being required to say the exact words
x HOW TO ATTACK: lacks requisites for admissibility ANNOTATIONS
CASE
CASE
PEOPLE OF THE PHILIPPINES, appellee, vs. CESARIO MONTAEZ and DANIEL SUMAYLO,
accused. viii. Entries in Official Records
CESARIO MONTAEZ, appellant.
G.R. No. 148257. March 17, 2004 Section 44. Entries in official records. — Entries in official records made in the
SECOND DIVISION performance of his duty by a public officer of the Philippines, or by a person in the
performance of a duty specially enjoined by law, are prima facie evidence of the facts therein
FACTS: stated. (38)
x Montaez and Sumaylo were charged with murder before RTC Tangub
x ANNOTATIONS
ii. Declaration against Interest CASES
Section 38. Declaration against interest. — The declaration made by a person deceased,
or unable to testify, against the interest of the declarant, if the fact is asserted in the
declaration was at the time it was made so far contrary to declarant's own interest, that a ix. Commercial Lists
reasonable man in his position would not have made the declaration unless he believed it to
be true, may be received in evidence against himself or his successors in interest and against Section 45. Commercial lists and the like. — Evidence of statements of matters of
third persons. (32a) interest to persons engaged in an occupation contained in a list, register, periodical, or other
published compilation is admissible as tending to prove the truth of any relevant matter so
ANNOTATIONS stated if that compilation is published for use by persons engaged in that occupation and is
generally used and relied upon by them therein. (39)
CASE
ANNOTATIONS
iii. Declaration about Pedigree
CASE
Section 39. Act or declaration about pedigree. — The act or declaration of a person
deceased, or unable to testify, in respect to the pedigree of another person related to him by x. Learned Treatises
birth or marriage, may be received in evidence where it occurred before the controversy, and
the relationship between the two persons is shown by evidence other than such act or Section 46. Learned treatises. — A published treatise, periodical or pamphlet on a subject
declaration. The word "pedigree" includes relationship, family genealogy, birth, marriage, of history, law, science, or art is admissible as tending to prove the truth of a matter stated
death, the dates when and the places where these fast occurred, and the names of the therein if the court takes judicial notice, or a witness expert in the subject testifies, that the
relatives. It embraces also facts of family history intimately connected with pedigree. (33a) writer of the statement in the treatise, periodical or pamphlet is recognized in his profession or
calling as expert in the subject. (40a)
ANNOTATIONS
ANNOTATIONS
CASE
xi. Testimony or Deposition at a Former Proceeding
iv. Family Reputation
Section 47. Testimony or deposition at a former proceeding. — The testimony or
Section 40. Family reputation or tradition regarding pedigree. — The reputation or deposition of a witness deceased or unable to testify, given in a former case or proceeding,
tradition existing in a family previous to the controversy, in respect to the pedigree of any one judicial or administrative, involving the same parties and subject matter, may be given in
of its members, may be received in evidence if the witness testifying thereon be also a evidence against the adverse party who had the opportunity to cross-examine him. (41a)
member of the family, either by consanguinity or affinity. Entries in family bibles or other
family books or charts, engravings on rings, family portraits and the like, may be received as ANNOTATIONS
evidence of pedigree. (34a)
CASE
ANNOTATIONS
xii. Child Witness Rule
CASE
CASE
v. Common Reputation

Section 41. Common reputation. — Common reputation existing previous to the


controversy, respecting facts of public or general interest more than thirty years old, or h. Opinion Rule
respecting marriage or moral character, may be given in evidence. Monuments and
inscriptions in public places may be received as evidence of common reputation. (35) Section 48. General rule. — The opinion of witness is not admissible, except as indicated in
the following sections. (42)

273
RECTO, GAYLE ANGELI M.
2011-0008 | AUSL
Personal Notes on Remedial Law 2 Review (based on the syllabus of Prof. Henedino M. Brondial)

is the case of People v. Bulimlit (not sure), they spoke of the ability to perceive,
ANNOTATIONS then ability of communication. And they added the other one, the ability to know
the difference between what is right from wrong. In other words, some cases
i. Expert Witness use it as the ability to understand the nature of an oath.

Section 49. Opinion of expert witness. — The opinion of a witness on a matter requiring In another case, the SC said that the child was not qualified to testify because he
special knowledge, skill, experience or training which he shown to posses, may be received in cannot perceive and cannot make known his perception. The child was 2 1/2
evidence. (43a)
years old.
ANNOTATIONS
Can a retardate testify?
She was the only witness in a rape case where she was a victim, the defendant's
CASES
counsel was able to destroy the testimony to the point of even getting an answer
that she likes the act of rape. But the SC convicted the accused nonetheless on
ii. Ordinary Witness the sole testimony of the retardate. Rationale: the retardate is qualified because
she can perceive and perceiving can make known his perception.
Section 50. Opinion of ordinary witnesses. — The opinion of a witness for which proper
basis is given, may be received in evidence regarding —
Disqualification?
(a) the identity of a person about whom he has adequate knowledge; Basic exception to the general rule (one can perceive and perceiving can make
known his perception) is if the law disqualifies him. So if there is a law
(b) A handwriting with which he has sufficient familiarity; and disqualifying a person, even if he can perceive and perceiving can make known
his perception, he is disqualified. You get that from other laws, substantive laws.
(c) The mental sanity of a person with whom he is sufficiently acquainted.
We have studied in Rule 119 regarding a state witness. One of the qualifications
The witness may also testify on his impressions of the emotion, behavior, condition or of a state witness is that he must not have been convicted of a crime involving
appearance of a person. (44a) moral turpitude. If you are convicted of a crime of moral turpitude, you cannot
testify as a state witness because the law disqualifies you to testify.
ANNOTATIONS If you have been convicted of perjury, defamation or misrepresentation, or
forgery, you cannot be a witness to a will under your wills and successions law.
CASES And remember, before you authenticate a will, you must present the three
instrumental witnesses. If one witness is convicted of perjury etc. then he cannot
i. Character Evidence testify as a witness.

Section 51. Character evidence not generally admissible; exceptions: — Other disqualifications:
Mental incapacity or Immaturity.
(a) In Criminal Cases: When you speak of maturity, it does not go with age. You may be of age but still
(1) The accused may prove his good moral character which is pertinent to the moral
immature. You may not be of age but already mature. One who cannot decipher
trait involved in the offense charged. what is good and right. One who does not appreciate the sanctity of an oath.
These are signs of immaturity. In other words, you are irresponsible. Who is
(2) Unless in rebuttal, the prosecution may not prove his bad moral character which irresponsible? One who cannot live up to situations. Who is responsible? If he
is pertinent to the moral trait involved in the offense charged. has the ability to respond. So you are irresponsible if the incident calls for a
correct response and you did not.
(3) The good or bad moral character of the offended party may be proved if it tends
to establish in any reasonable degree the probability or improbability of the offense charged.
But if you are pronounced to be insane or psychotic, you are disqualified.
(b) In Civil Cases: Children, under the Rule on Section 21, because of their maturity but remember
that the children may even be more mature than the adults. So this is a case-to-
Evidence of the moral character of a party in civil case is admissible only when pertinent to the case basis. And because of the Child Witness Rule it has demasculated or
issue of character involved in the case. efeminated, wherein children cannot testify. Because under the Child Witness
(c) In the case provided for in Rule 132, Section 14, (46a, 47a)
Rule, there are a lot of exceptions where a child can testify. There are a lot of
what you call testimonial aids. If a child is testifying in a crime of rape, you can
ANNOTATIONS give her a doll, then she would testify. What are your parts in the body which are
similar to the parts of this doll. Or she can ask to be held by the hand of her
CASE grandmother, mother, brother, sister, etc. In fact, two years ago in the bar
exams, there was a question about fiddling testimony, this is under the Child
OLD LECTURE Witness Rule.

TESTIMONIAL EVIDENCE
Marital disqualification rule.
The weakest of all kinds of evidences precisely because it emanates from man
who can always try to controvert matters. You often confuse that with marital privilege rule. A very important guide here is
that the marital privilege rule, the statements in the testimony which are not
What are the qualifications? allowed are those of communications which are confidential. But this is not so in
You better memorize the qualifications. It's very simple because almost all marital disqualification rule. And in the marital disqualification rule, the time
problems of the qualification of a witness can be answered through these basic frame is within or during marriage. But this is not in the marital privilege rule.
characteristics of a witness. These are the things. but what are the waivers here? The waiver is upon
consent. Another waiver is one against another. Example is for annulment of
A witness is one who can perceive and perceiving and can make known his marriage, the witness in chief is the party involved.
perception. So anybody.
The Dead Man's Statute
Can a deaf-mute testify? Can a blind testify? So if A files a case against the estate of B, or the property belonging to B being
If he can perceive and perceiving and can make known his perception, he can insane, then A cannot testify on matters ante litem motam (before the
testify. Even if you don't get a perfect score, you will not get a zero for that controversy). The rationale behind is that when law closes the lips of someone,
because that is a correct answer. the adverse party's lips must likewise be closed. But you might be given certain
problems, remember that the action here is limited to actions against the estate
Can a child of tender age testify? in case of a deceased and the property of the insane in case the defendant is
insane. So if it is the estate, it must be an action against Rule 87 (action by or
If he can perceive and perceiving can make known his perception, he can testify.
against the executor or administrator), which are recovery of real or personal
property, recovery of decedent's interest or lien thereon or recovery for damages
In one and recent case wherein a child was asked to testify, the SC allowed the
arising from an offense or action. So it will not be a contractual money claim,
child to testify because they find out that what is the criteria for ability to
perceive is not just perceiving but making known your perception to others. This
274
RECTO, GAYLE ANGELI M.
2011-0008 | AUSL
Personal Notes on Remedial Law 2 Review (based on the syllabus of Prof. Henedino M. Brondial)

because it is a claim against the estate. This one is against the executor or as against the other 2 who did not confess, pursuant to the doctrine of
administrator. interlocking confession.
Exceptions: If there is a waiver, if one consents to the other. But an
implied waiver here is if the defendant executor or administrator interposes a NOTE: When you speak of confession, personal yan. It refers only to the
counterclaim. The dead man's statute does not apply because who will establish confessant. But when you speak of admission, ordinarily it is admissible as
the counterclaim. How will you oppose the counterclaim. And remember that the against the admitter. That is not absolute as it can be admissible as against other
subject matter are those before the controversy, ante litem motam. persons. So Section 26 says act, declaration or omission of a person is admissible
as to him. So even in the negative it is admissible as to him.
Privilege Rule
Basic characteristic here in these instances is the confidential nature of the The act referred to there refers to a physical act. Declaration refers to a
communication between one party and the other. So you start with husband and statement. Omission is the failure to do something which the law calls you to do
wife. It is not stated there but this husband and wife relationship can only be or provides that you do it but you did not do it. So that is always admissible as to
invoked by those who are legally married. Although it is not stated in the Rules. the declarant, actor, or omitter.
Because if you give that privilege to those who are not legally married, you are
giving a premium to illegality. Therefore, if you are only a common law wife, this Don't confuse this with the res inter alios acta rule. That is provided for in
rule will not apply to you. Section 28. These following sections must be read together. Section 26, 28 and
34 (other side of the res inter alios acta rule).
Lawyer and Client relationship
It is not the client who is privileged here but rather the lawyer who cannot be Section 26, the act, declaration, or omission of a party is admissible as to the
compelled to testify on matters which he receives from the client or regarding actor, declarant or omitter. Self-explanatory. If you do it, then you are liable. But
matters he gave as advices to the client. How about the client testifying? Of remember that the admissibility of the declaration must be against their interest.
course, the client can testify, but not the lawyer. But, together with the lawyer Because under the Rules of Evidence, a declaration which is self-serving is
and the alter ego of the lawyer which is the secretary or his stenographer. So inadmissible. So if i declare that I did not steal the car, it is a self-serving
that would even be sui generis, that would even include his clerk in the office. declaration. It is in fact a denial. And a denial, although stronger than affirmative
Because, practically, the secretary or his stenographer knows everything. But this statement cannot always be taken in your favor. So the act, declaration, or
has something to do in the course of the practice of the profession. For example: omission must all be positive. The declaration must be against one's own
a client goes to you and communicate matters regarding her amorous affairs and interest. So when you declare, for example, that I was with Mr. A when he
starts insinuating something, this has nothing to do with the case. And of course if robbed the bank that is a declaration against interest. That can be taken against
there is consent, this is another waiver. you. But when you speak of the res inter alios acta rule, it's different. The rights
of a party cannot be prejudiced by the act, declaration or omission of another.
Doctor and Patient relationship So if Pedro acts, declares or omits, that act, declaration, or omission cannot be
Look, there is a peculiarity there. The disqualification is only in civil cases and not taken against Juan. Yung kay Pedro kay Pedro. Yung kay Juan kay Juan. So
in criminal cases. Because in many instances, the doctor is even required to don't confuse Section 26 with Section 28. The act, declaration or omission of a
testify in criminal cases. The term doctor expands also to his alter ego, or similar, party cannot be taken against the other.
sui generis also. But how about the quack doctor? No. It is not within the
privilege because that would again be giving premium to an illegal practice of What is the general rule?
medicine. And also the matters privileged are those within the communication If you do something, you are responsible for it. If you do something, another is
within the confidential nature of the communication in relation to the medical not responsible for it. Ganun lang yan. Although, what you have to look into are
practice of the person. the exceptions in the res inter alios acta rule. So when you speak of an
exception, the keyword there is privity. Pag may privity of relationships, then
Priest and Penitent relationship that is an exception. So when A does something, that act, declaration, or
At least in the catholic church, this has never been violated. A lot of priests have omission is not admissible as against B.
fallen and got out of their priestly ministry but had never broken this confession. Exceptions:
If there is privity in their relationship. What is that relationship? It can be a
Public Officer relation of partnership, agency, co-ownership, co-debtorship. So yan ang
But here what you have to look into is the public interest. If in disclosing exception. B can be liable for the act of A, if B and A are partners, agents of
matters, it would be detrimental or prejudicial to public interest, then you cannot each other, co-owners, or co-debtors. But in establishing the exception, you have
force. Example is military secrets. But not the testimony of oakland mutineers. to establish the existence of the agency, partnership, co-ownership by evidence
other than the act, declaration or omission of a party. So that is where the
Parental and Filial Privilege difficulty lies because you have to get other evidence other than the act,
Parental is with regard to the Parent who cannot be compelled to testify against declaration or omission. So for example, A says, I entered into a contract with X
their children. Filial is with regard to the child who cannot be compelled to testify together with B who is my partner, the contract or the act of contracting cannot
against their parents. This is not a disqualification per se. That's why it is establish partnership. You have to establish partnership by other evidence other
privilege. It is more of legal incompetency. Parental privilege and Filial privilege. than the act of contracting. And that holds true with agency, co-ownership, and
Compulsion. But if parents would like to testify against children, then go ahead. co-debtorship.
And if the children would like to testify against their parents, there is no Conspiracy. In criminal law, the act of one is the act of all. you have to establish
prohibition. You cannot force them if they do not want to. This is where the conspiracy by evidence other than the act, declaration or omission of the party.
prohibition lies. There are many jurisprudence where the accused is the father in Admission by privies. So these relationships that we have exemplified are
the crime of rape. The daughter can definitely testify against the father. Even the actually legal relationships. But when you speak of privies, they are other forms
mother can testify against the father. This is not a marital disqualification. of relationship. For example, the relationship between the successor-in-interest
and predecessor-in-interest, father and son relationship, by succession. That is
ADMISSIONS AND CONFESSIONS also privity in relationship.
When we speak of admission, this is a statement of a fact. There is no admission Admission by silence. If A, B, C, D, and E, were the accused of robbery and they
of liability here, necessarily. But when you speak of a confession, it is always an were put in jail. They were confronted by the private complainant. Private
acknowledgment of guilt. It is an admission of liability. complainant pointed them as the accused and A, B, and C admitted their guilt
reasoning poverty, D and E kept silent. That is an admission by silence. Because
So an admission, as a general rule, is admissible as against the admitter but not they could have reacted. Silence means consent. Exceptions to admissions by
against other people. While in confession, it must definitely be against the silence, if you are supposed to react and you did not react, your silence is
confessant and never against other people. admissible against you. If you are under advisement by your counsel or if you
invoke your right to remain silent. If your answer would be self-incriminatory.
What is the exception to that?
Inter-locking confession. The other side of res inter alios acta rule is found in Section 34. This is the flip
side of res inter alios acta rule doctrine. And you will note that Section 34 says,
What is the doctrine of inter-locking confession? that declaration cannot be admissible as to others except that if Pedro does
If 6 are accused and 4 of them executed an affidavit detailing how the crime was something or does not do anything on this particular occasion, it does not follow
committed and pointing to the other 2, that extrajudicial confession is admissible that he did or did not do the same thing in another occasion. So, you emphasize

275
RECTO, GAYLE ANGELI M.
2011-0008 | AUSL
Personal Notes on Remedial Law 2 Review (based on the syllabus of Prof. Henedino M. Brondial)

again there the exceptions. It is an evidence, nonetheless, of intent, knowledge, qualification, the one who testifies on that must be knowledgeable on the subject
scheme, plot, habit, custom. Yun ang ginagamit ng mga police investigators. matter. Suppose there is a book by Gerardo Sicat. He is well-known Economist,
then the one who is testifying about his book is Bernardo Villegas, another well-
EXAMPLE: There is a complainant who goes to the police precinct and said known economist. Definitely, even if that is hearsay as far as the witness is
ninakaw ang kanyang cellphone. Saang parte ka ninakawan? Dun ho sa kanto ng concerned but this is considered as an exception because it is learned treatises.
extramadura at espanya. Kukuha ng file yung pulis and points out to the person. Another example is a doctoral thesis. So you tie this up with expert witnesses.
Parang Ocean Eleven.
Entries or commercial lists
HEARSAY RULE Just go over the requirements of each.
General Rule: You can only testify on what you know. Kaya nga, in connection
with the general guideline i gave you regarding qualification, one who can Testimony at a former proceeding or deposition
perceive and perceiving can make known his perception, that is the general rule. We have extensively studied deposition.
Someone must personally perceive and not on what others perceive.
OPINION RULE
What is the rationale behind the prohibition? Going back to the general formulation, one who can perceive and perceiving can
There is no opportunity on the part of the party to cross-examine the witness. make known his perception. So what are you suppose to testify on? What you
So, this is the general rule. You only testify on what you personally know. perceive and not what you think about what you perceive because that is already
a matter of opinion. Opinions are not allowed in testimonies.
What are the exceptions? Memorize. Exceptions:
Hindi na lang 11 ito. Kundi 12 na. And what is the 12th exception? Expert Witness
Child Witness Rule. Ordinary Witness
Dying Declaration; identity of a person about whom he has adequate knowledge
Declaration against interest; handwriting with which he has sufficient familiarity
Act or declaration about pedigree; mental sanity of a person with whom he is sufficiently acquainted
Family reputation or tradition regarding pedigree;
Common reputation; An expert witness must testify on his experties. Example is in a case of
Res Gestae; annulment of marriage, mandatorily, the law says that an expert witness must
Entries in the ordinary course of business; testify when the ground is Psychological incapacity.
Entries in official records; An ordinary witness may testify on his opinion but limited only to 3 instances.
Commercial lists;
Learned treatises; Doctrine of independently relevant statement
Testimony or deposition at a former proceeding; and You are not trying to establish the content whether it is true or not but you are
Child Witness Rule only trying to establish the fact that someone (C) told you that X went with A. As
to the existence of the fact and not as to the truth or veracity of the issue.
Dying Declaration
Bugbog na bugbog na yang dying declarations, in relation to the res gestae.. Just ______________________________________________
look at the qualifications.
Remaining cases on the syllabus under Evidence
In res gestae, the declarant does not have to die. But in dying declaration, kaya
nga dying, namatay. Kung buhay pa yan, hindi dying declaration yan. Remember Hearsay Evidence
that these exceptions to the hearsay rule, there is a basic requirement of
unavailability of the witness. Because if the witness is available, let him testify
orally. The declarant must be conscious upon pending death. >Entries in the course of business

Requirements of dying declarations: FIRST DIVISION


That death is imminent and the declarant is conscious of that fact;
That the declaration refers to the cause and the surrounding circumstances of G.R. No. 92740 March 23, 1992
such death;
That the declaration relates to the facts which the victim is competent to testify;
PHILIPPINE AIRLINES, INC., petitioner,
That the declaration is offered in a case wherein the declarant's death is subject
of the inquiry (the victim necessarily must have died);
That the statement is complete in itself. vs. JAIME M. RAMOS, NILDA RAMOS, ERLINDA ILANO, MILAGROS ILANO, DANIEL
ILANO AND FELIPA JAVALERA, respondents.
Declaration against interest. We discussed that partly.
MEDIALDEA, J.:

Pedigree
Pedigree is a kind of relationship. If one testifies about the relationship of
Facts
another because it is known by him, then that is not hearsay. If I testify, I know
Mr. X to be the illegitimate child of Mr. Y. That is not hearsay. But when I say
* Plaintiffs Jaime Ramos, Nilda Ramos, Erlinda Ilano, Milagros Ilano, Daniel Ilano and Felipe
that it is well known in the community that Mr. X is not legally married to Y. It
Javalera, are officers of the Negros Telephone Company who held confirmed tickets for PAL
can be about pedigree of marriage. Common reputation about pedigree.
Flight No. 264 from Naga City to Manila on September 24, 1985, scheduled to depart for
Manila at 4:25 p.m. The tickets were brought sometime in August 1985. Among the conditions
Res Gestae
included in plaintiffs tickets is the following:
Two kinds. It can either be statements during an occurrence or spontaneous
articulation in the course of an event. If you are on top of a building, and you
are looking over on the road and you see both men coming at each other armed 1. CHECK-IN TIME — Please check in at the Airport Passenger check-in counter at
with a bolo. These are all part of the res gestae yung reactions. Multiple least one hour before PUBLISHED departure time of your flight. We will
admissibility, a witness or a testimony can be admitted because of a res gestae consider your accommodation forfeited in favor of waitlisted passenger if you fail to
or a dying declaration. check-in at least 30 minutes before PUBLISHED departure time.

Treatises * Plaintiffs claim in their Complaint that they went tot he check-in counter of the defendant's
It refers to publications and the witness who testifies is not the author. Because if Naga branch at least one (1) hour before the published departure time but no one was at the
the author testifies on his own writing, then it is not hearsay. It is someone else counter until 30 minutes before departure, but upon checking -in and presentation of their
who testified on a treaty or a publication by a well-known author or write in tickets to the employee/clerk who showed up, their tickets were cancelled and the seats
specialized fields, this is considered hearsay but admissible. But another
276
RECTO, GAYLE ANGELI M.
2011-0008 | AUSL
Personal Notes on Remedial Law 2 Review (based on the syllabus of Prof. Henedino M. Brondial)

awarded to chance passengers; plaintiffs had to go to Manila by bus, and seek actual, moral It is an admitted fact that the private respondents knew of the required check-in time for
and exemplary damages, and attorney's fees for breach of contract of carriage. passengers. The time requirement is prominently printed as one of the conditions of carriage
on their tickets, i.e., that the airport passenger should check-in at least one hour before
* Defendant disclaim any liability, claiming that the non-accommodation of plaintiff on the said published departure time of his flight and PAL shall consider his accommodation forfeited in
flight was due to their having check-in (sic) late for their flight. It is averred even if defendant is favor of waistlisted passengers if he fails to check-in at least 30 minutes.
found liable to the plaintiffs such liability is confined to, and limited by, the CAB Economic
Regulations No. 7 in conjunction with P.D. 589.

* The trial court rendered judgment finding defendant guilty of breach of contract of carriage We note that while the aforequoted condition has always been applied strictly and without
in bumping-off the plaintiffs from its F264 flight of September 25, 1985, and ordered exception (TSN, December 16, 1987, p. 11), the station manager, however, may exercise his
defendant to pay: discretion to allow passengers who checked-in late to board provided the flight is not fully
booked and seats are available (ibid, pp. 17-18). On September 24, 1985, flight 264 from
1) P1,250.20 — the total value of the tickets: Naga to Manila was fully booked owing to the Peñafrancia Festival (TSN, January 25, 1988,
p.5). In addition, PAL morning flights 261 and 262 were canceled resulting in a big number o f
waitlisted passengers. (TSN, November 23,
2) P22.50 — the total value of airport security fees and terminal fees;

1987, p. 6).
3) P20,000.00 — for each of the plaintiffs for moral and temperate damages; and

4) P5,000.00 — for attorney's fees and expenses of litigation. (Rollo, pp. 35-36)

The private respondents claim that they were on time in checking-in for their flight; that no
* PAL appealed to the Court of Appeals. On March 15, 990, the appellate court rendered a
PAL personnel attended to them until much later which accounted for their late check-in; that
decision, the dispositive portion of which, reads:
PAL advanced the check-in time and the departure of their flight resulting in their non-
accommodation; and that they suffered physical difficulties, anxieties and business losses.
WHEREFORE, the decision appealed from is AFFIRMED in toto, with costs against
appellant.

SO ORDERED.
The evidence on record does not support the above contentions. We note that there were two
other confirmed passengers who came ahead of the private respondents but were refused
* This petition for review on certiorari seeks to reverse the decision of the Court of Appeals
accommodation because they were late. Edmundo Araquel, then the check-in-clerk, testified
dated March 15, 1990 affirming in toto the decision of the Regional Trial Court of Imus, Cavite,
on this point, as follows:
Branch 21, directing the Philippine Airlines, Inc. (PAL, for short) to pay the private respondents
the amounts specified therein as actual, moral and temperate damages as well as attorney's
fees and expenses of litigation.

Atty. Marcelino C. Calica, counsel for PAL

Q Before the plaintiffs arrive (sic) at the check-in counter, do you recall if there
Issues
were other passengers who arrived at the counter and they were advised that
they were late?
1. Can the Honorable Court of Appeals validity promulgate the questioned decision by the
simple expedient of adopting in toto the trial court's finding that defendant-appellant is liable
A Yes, sir.
for damages on the sole issue of credibility of witnesses without considering the material
admissions made by the plaintiffs and other evidence on record that substantiate the defense
of defendant-appellant.

2. Can the Honorable Court award legally moral and temperate damages plus attorney's fees Q Who were those persons?
of P5,000.00 contrary to the evidence and established jurisprudence. (Rollo, p. 9)
A My former classmates at Ateneo, sir, Rose Capati and Go, Merly.

Ruling
Q Were these two passengers also confirmed passengers on this flight?
Under Section 1, Rule 131 of the Rules of Court, each party in a case is required to prove his
affirmative allegations. In civil cases, the degree of evidence required of a party in order to A Yes, sir.
support his claim is preponderance of evidence or that evidence adduced by one party which is
more conclusive and credible than that of the other party (Stronghold Insurance Company,
Inc. vs. Court of Appeals, et al., G.R. No. 83376, May 29, 1989, 173 SCRA 619, 625).

Q I show to you a document which is entitled "Passenger Manifest of flight 264,


September 24, 1985," which we request to be marked as Exh. "5" you said
earlier that aside from the plaintiffs here there were two other passengers who
The case at bar presents a simple question of fact: Whether or not the private respondents also checked in but they were also late and you mentioned the names of these
were late in checking-in for their flight from Naga City to Manila on September 24. 1985. It is passengers as Capati and Go, please point to us that entry which will show the
immediately apparent from the records of this case that the claims of the parties on this names of Go and Capati?
question are dramatically opposed. As a rule, the determination of a question of fact depends
largely on the credibility of witnesses unless some documentary evidence is available which A Here, sir, numbers 13 and 14 of the Manifest.
clearly substantiates the issue and whose genuineness and probative value is not disputed
(Legarda v. Miaile, 88 Phil. 637, 642). The exception to the rule now runs true in this case.

ATTY. CALICA: We request that passengers 13 and 14 be marked in evidence, Go


for 13 and Capati for 14 as Exh. "5-A."
We reverse. This case once more illustrates Our power to re-weigh the findings of lower courts
when the same are not supported by the record or not based on substantial evidence (see
Cruz v. Villarin, G.R. No. 75679, January 12, 1990, 181 SCRA 53, 61).

277
RECTO, GAYLE ANGELI M.
2011-0008 | AUSL
Personal Notes on Remedial Law 2 Review (based on the syllabus of Prof. Henedino M. Brondial)

Q You said that these two passengers you mentioned were also similarly denied Q On the tickets being tendered for check-in and noting that they were late, you
accommodations because they checked in late, did they check in before or after mean to say you immediately made annotations?
the plaintiffs?
A Yes, sir. That is an S.O.P. of the office.
A Before, sir.

Q So on what time did you base that 4:02?


Q What time did they appear at the counter?
A At the check-in counter clock, sir.
A 4:01 p.m., sir.

Q At the time you placed the time, what was the time reflected at the counter
Q What happened when they checked in at 4:01? clock?

A I told them also that they were late so they cannot be accommodated and they A 4:02, sir. (ibid, pp. 8-11)
tried to protest, but they decided later on just to refund the ticket. (TSN of
November 23, 1987, pp. 11-12)

The private respondents submitted no controverting evidence. As clearly manifested above,


the intervening time between Capati and Go and the private respondents took only a mere
Shortly after, the private respondents followed the aforesaid two passengers at second. If indeed, the private respondents were at the check-in counter at 3:30 p.m., they
the counter. At this juncture, Araquel declared, thus: could have been the first ones to be attended to by Araquel than Capati and Go. They cold
have also protested if they were the earliest passengers at the counter but were ignored by
Araquel in favor of Go and Capati. They did not.

Q Now, you said that you met the plaintiffs in this case because they were
passengers of Flight 264 on September 24, 1985 and they were not
accommodated because they checked in late, what time did these plaintiffs check It is likewise improbable that not a single PAL personnel was in attendance at the counter
in? when the check-in counter was supposed to be opened at 3:25 p.m. It mist be remembered
that the morning flight to Manila was canceled and hence, it is not farfetched for Us to believe
A Around 4:02 p.m., sir. that the PAL personnel then have their hands full in dealing with the passengers of the
morning flight who became waitlisted passengers. Moreover, the emphatic assertions of
private respondent Daniel Javalora Ilano regarding the absence of a PAL personnel lost its
impact during the cross examination:

Q Who was the clerk at the check in counter who attended to them?

A I was the one, sir.


ATTY. CALICA —

Q So, you maintain therefore that for all the time that you waited for there for the
whole twenty (20) minutes the check-in counter and other PAL Offices there —
xxx xxx xxx
the whole counter was completely unmanned? I am referring to the whole area
there where it is enclosed by a counter.

Q You said when you were presented the tickets of the plaintiffs in this case and
noting that they were late for checking in, immediately after advising them that
I will describe to you, for the benefit of the court.
they were late, you said you made annotation on the tickets?

A Yes, sir.

When you approach the counter at Naga Airport, the counter is enclosed, I
mean, you cannot just go inside the PAL office, right? there is some sort of
counter where you deal with the PAL personnel and you approximate this counter
Q I am showing to you Exhs. "A," "B," "C," and "D," which are the tickets of Mr. to be five (5) to six (6) meters. Now, this space after the counter, did you
& Mrs. Jaime Ramos for Exh. "A," Exh. "B" ticket of Mr. & Mrs. Daniel Ilano, "C" observe what fixtures or enclosures are contained there inside the enclosed
ticket of Felipa Javalera and "D" ticket of Erlinda Ilano, will you please go over space?
the same and point to us the notations you said you made on these tickets?

A This particular time, sir. (Witness pointing to the notation "Late" and the time
"4:02" appearing at the upper righthand of the tickets Exhs. "A," "B," "C," and
A I am not sure whether there are offices or enclosures there.
"D.")

Q You have been traveling and had opportunity to check-in your tickets so may
Q How long did it take after the tickets were tendered to you for checking in and
times. Everytime that you check-in, how many personnel are manning the check-
before you made this notation?
in counter?

A It was just seconds, sir.


A There are about three (3) or four (4), sir.

278
RECTO, GAYLE ANGELI M.
2011-0008 | AUSL
Personal Notes on Remedial Law 2 Review (based on the syllabus of Prof. Henedino M. Brondial)

Q Everytime, there are three (3) or four (4)?

A Everytime but not that time. Q Who was assigned as check-in clerk that particular time?

A I was the one with Mr. Valencia, sir.

Q I am referring to your previous trips, I am not referring to this incident.

Q What was Mr. Valencia doing?

On previous occasions when you took the flight with Pili Airport and you see A He assisted me, sir.
three (3) or four (4) personnel everytime, are all these three (3) or four (4)
personnel at the counter or some are standing at the counter or others are
seated on the table doing something or what? Will you describe to us?
Q How?

A If a group of passengers simultaneously check in, we divided the work


A Some are handling the baggages and some are checking-in the tickets. between us. (TSN, November 23, 1987, p. 7)

Q So, on most occasions when you check-in and say, there were at least three xxx xxx xxx
(3) of four (4) people at the check-in counter, one would attend to the tickets,
another to the check in baggage, if any. Now, do you notice if somebody evade
when you check-in your ticket. This other person would receive the flight coupon
which is detached from your ticket and record it on what we call passenger
Q When the plaintiffs testified in this case particularly plaintiff Daniel Ilano and
manifest?
Felipa Javalera at the previous hearings said plaintiffs stated that they arrived at
the check-in counter at about 3:25 or 3:30 and there was nobody in the counter,
A That's true.
what can you say to that?

A We cannot leave the counter, sir. That was always manned from 3:25 up to
the last minute. We were there assigned to handle the checking in of the
Q Now, it is clear one would attend to the baggage, another person would passengers.
receive the ticket, detach the coupon and one would record it on the passenger
manifest. What about the fourth, what was he doing, if you recall?

A I think, putting the identification tags on the baggages (sic). (TSN, November
Q You mentioned earlier that aside from you there were other personnel
17, 1986, p. 38)
assigned to the check-in counter and you even mentioned about a certain
Valencia assisting you, do you have any evidence to show said assignment of
personnel at the airport?

Ilano's declaration becomes even more patently unreliable in the face of the Daily A Yes, sir.
Station Report of PAL dated September 24, 1985 which contained the working
hours of its personnel from 0600 to 1700 and their respective assignments, as
follows:

Q I show to you a daily station report from 24 September 1985 covering working
hours 0600 to 1700, will you please go over the same and thereafter tell us from
the personnel listed in this Daily Station Report what were the name (sic) of the
ATTY. CALICA personnel assigned to man a check-in counter at that time?

Q Normally upon opening of the check-in counter, how many PAL personnel are A There (sic) persons assigned were Mr. Oropesa, Mr. Espiritu, Mr. Medevilla,
assigned to man the counter? myself and Mr. Valencia.

EDMUNDO ARAQUEL Q You mentioned about Mr. Espiritu, what was his specific task at that time?

A A total of four personnel with the assistance of others. A He was handling the ticketing, sir.

Q Who are these personnel are assigned to the counter and what specific duties Q What about Mr. Medevilla?
they performed?
A He was taking care of the ramp handling.
A Mr. Oropesa handled the cargo, Mr. Espiritu handled the ticketing, Mr. Valencia
and me handled the checking in of passengers.

Q And Mr. Oropesa?

Q Are you referring to this particular flight 264 on September 24, 1985?
A He was handling the incoming cargo.

A Yes, sir.

279
RECTO, GAYLE ANGELI M.
2011-0008 | AUSL
Personal Notes on Remedial Law 2 Review (based on the syllabus of Prof. Henedino M. Brondial)

ATTY. CALICA: We request that this Daily Station Report be marked Exh. "6" and as apart of the res gestae (32 C.J.S., S. 411, 30-31). Based on these circumstances, We
the portion of the Report which shows the deployment of personnel of PAL Naga are inclined to believe the version of PAL. When the private respondents purchased their
Station on September 24, 1985 as "6-A." tickets, they were instantaneously bound by the conditions of the contract of carriage
particularly the check-in time requirement. The terms of the contract are clear. Their failure to
come on time for check-in should not militate against PAL. Their non-accommodation on that
flight was the result of their own action or inaction and the ensuing cancellation of their tickets
by PAL is only proper.
Q Plaintiffs in this case testified that when they checked in there was nobody
manning the counter and they had to wait for twenty minutes before someone
came in to the counter, what can you say to that?

A It is not true because all the time we were there from the start, an hour before Furthermore, We do not find anything suspicious in the fact that PAL flight 264 departed at
the flight we were there because we were assigned there. 4:13 p.m. instead of 4:25 p.m. Apart from their verbal assertions, the private respondents did
not show any evidence of irregularity. It being clear that all the passengers have already
boarded, there was no sense in keeping them waiting for the scheduled time of departure
before the plane could take flight.

Q Plaintiff Daniel Ilano testified that he went to the counter twice, first at 3:25
and it was only at 4:00 p.m. that somebody went to the counter and attended to
him and while he expected his boarding pass he was told instead that plaintiffs
could not be accommodated because they were late, what can you say to that? ACCORDINGLY, the petition is GRANTED. The questioned decision of the Court of Appeals
dated March 15, 1990 is hereby ANNULLED and SET ASIDE. No costs.
A The truth is we were always there and we never left the counter from the start
of the check-in time of 3:25 we were all there, we never left the counter.

SO ORDERED.

Q Until what time did you remain at the check-in counter?

A At around 4:15 p.m., sir. Narvasa, C.J., Cruz and Griño-Aquino, JJ., concur.

Bellosillo, J., is on leave.

Q You said that the check-in counter was closed at 3:55, for what purpose were
you still manning the check-in counter?

Entries in Official Records


A To attend to the passenger who are late in checking in because they also need
assistance in explaining to them the situation.
SECOND DIVISION

G.R. No. 140023 August 14, 2003


Q So it was for that purpose you were there?
RUDY LAO, Petitioner,
A Yes, sir. (ibid., pp. 16-18)
vs. STANDARD INSURANCE CO., INC., Respondent.

QUISUMBING, J.:
It is significant to note that there were no other passenger who checked-in late after the
private respondents (TSN, November 23, 1987, p. 13). In the absence of any
Facts
controverting evidence, the documentary evidence presented to corroborate the
testimonies of PAL's witnesses are prima facie evidence of the truth of their
* Petitioner Rudy Lao is the owner of a Fuso truck with Plate No. FCG-538. The truck was
allegations. The plane tickets of the private respondents, exhs. "1," "2," "3," "4," (with
insured with respondent Standard Insurance Co., Inc. under Policy No. CV-210743 for the
emphasis on the printed condition of the contract of carriage regarding check-in time as well
maximum amount of P200,000 and an additional sum of P50,000 to cover any damages that
as on the notation "late 4:02" stamped on the flight coupon by the check-in clerk immediately
might be caused to his goods.
upon the check-in of private respondents) and the passenger Manifest of Flight PR 264,
exh. "5," (which showed the non-accommodation of Capati and Go an the private
* While the policy was in effect, an accident occurred. At around 8:00 p.m. of April 24, 1985,
respondents) are entries made in the regular course of business which the private
in Barangay Buhang, Jaro, Iloilo City, the insured truck bumped another truck, with Plate No.
respondents failed to overcome with substantial and convincing evidence other
FBS-917, also owned by petitioner Lao. The latter truck was running ahead of the insured
than their testimonies. Consequently, they carry more weight and credence. A
truck and was bumped from the rear. The insured truck sustained damages estimated to be
writing or document made contemporaneously with a transaction in which are
around P110,692, while the damage to the other truck and to properties in the vicinity of the
evidenced facts pertinent to an issue, when admitted as proof of those facts, is
accident, were placed at P35,000 more or less.
ordinarily regarded as more reliable proof and of greater probative force than the
oral testimony of a witness as to such facts based upon memory and recollection
(20 Am Jur S 1179, 1029 cited in Francisco, Revised Rules of Court in the Philippines * Petitioner filed a claim with the insurance company for the proceeds from his policy.
Annotated, 1973 Edition, Volume VII, Part II, p. 654). Spoken words could be notoriously However, the claim was denied by the insurance company on the ground that when its
unreliable as against a written document that speaks a uniform language (Spouses adjuster went to investigate the matter, it was found that the driver of the insured truck,
Vicente and Salome de Leon v. CA., et al., G.R. No. 95511, January 30, 1992). This dictum is Leonardo Anit, did not possess a proper driver’s license at the time of the accident. The
amply demonstrated by the diverse allegations of the private respondents in their restriction4 in Leonardo Anit’s driver’s license provided that he can only drive four -wheeled
complaint (where they claimed that no one was at the counter until thirty (30) minutes vehicles weighing not more than 4,500 kgs. Since the insured truck he was driving weighed
before the published departure time and that the employee who finally attended to them more than 4,500 kgs., he therefore violated the "authorized driver" clause5 of the insurance
marked them late, Records, p. 2) and in their testimonies (where they contended that policy. In addition, respondent cited the following excerpts from the police blotter of the Iloilo
there were two different PAL personnel who attended to them at the check-in counter. TSNs INP, to wit:
of November 17, 1986, pp. 41-45 and of May 18, 1987, pp. 5-6). Private respondents' only
objection to these documents is that they are self-serving cannot be sustained. The
hearsay rule will not apply in this case as statements, acts or conduct
accompanying or so nearly connected with the main transaction as to form a part C-UN-85 DAMAGE TO PROPERTY W/ PHY INJURIES - R/ IMPRUDENCE
of it, and which illustrate, elucidate, qualify or characterize the act, are admissible
280
RECTO, GAYLE ANGELI M.
2011-0008 | AUSL
Personal Notes on Remedial Law 2 Review (based on the syllabus of Prof. Henedino M. Brondial)

11:30 PM - Sgt. A. Bernas informed this office that a collision took place at Brgy. D. THE HONORABLE COURT OF APPEALS MISAPPLIED ARTICLES 2232 AND 2208 OF THE
Buhang, Jaro, IC. Investigation conducted by Pat. Villahermosa, assisted by Lt. P. NEW CIVIL CODE IN GRANTING EXEMPLARY DAMAGES AND ATTORNEY’S FEES TO
Baclaron (OD), disclosed that at about 8:00 PM this date at the aforementioned RESPONDENT. UNDER ARTICLES 2229 AND 2234 OF THE NEW CIVIL CODE, EXEMPLARY
place, a collision took place between a truck (Hino) with Plate Nr FB[S] 917 DAMAGES CANNOT BE AWARDED IN THE ABSENCE OF AN AWARD FOR MORAL, TEMPERATE,
owned by Rudy Lao and driven by BOY GIDDIE Y COYEL, 38 yrs, a res. of LIQUIDATED OR COMPENSATORY DAMAGES;13
Balasan, Iloilo, with License Nr DLR 1108142 and another truck with Plate Nr.
FCG-538 owned by Rudy Lao and driver (sic) by LEONARDO ANIT Y PANES, 33 E. TESTIMONIES OF THE WITNESSES OF RESPONDENT NAMELY, SGT. BERNAS, THE DESK
yrs, a res. of Brgy Laya, Balasan, Iloilo with License Nr 1836482.… 6 OFFICER AND ROMEO GUIERGEN, INSURANCE ADJUSTER, WERE INCONSISTENT AND
UNRELIABLE;14 and

F. THE HONORABLE COURT OF APPEALS UPHELD THE DECISION OF THE LOWER COURT
* Petitioner claims that at the time of the accident, it was in fact another driver named Giddie DESPITE GLARING MISAPPLICATION OF THE LAW AND JURISPRUDENCE ESTABLISHED BY
Boy Y Coyel who was driving the insured truck. Giddie Boy possessed a driver’s license THIS HONORABLE SUPREME COURT AS WELL AS CLEAR MISAPPREHENSION OF THE FACTS
authorizing him to drive vehicles such as the truck which weighed more than 4,500 kgs. As IN THIS CASE.15
evidence, petitioner presented the Motor Vehicle Accident Report7 wherein the Investigating
Officer, Pat. Felipe D. Villahermosa, stated that it was Giddie Boy driving the insured truck and
not Leonardo Anit. The said report was made three days after the accident or on April 27,
1985. However, respondent insurance company was firm in its denial of the claim.
Ruling

* The original action was lodged before the Regional Trial Court of Iloilo City, Branch 25, as
Three issues must be resolved: (1) The admissibility and probative value of the police blotter
Civil Case No. 17045 for breach of contract and damages, as a result of the insurance
as evidence; (2) The assessment of the credibility of witnesses; and (3) The propriety and
company’s refusal of petitioner’s claim on the insurance policy of his truck which figured in an
basis of the awards for exemplary damages and attorney’s fees. Also pertinent here is the
accident during the effectivity of the policy.
factual issue of whether or not Leonardo Anit, an unauthorized driver, was driving the insured
truck at the time of the accident.
* After trial, the court disposed of the case as follows:

Petitioner assails the admissibility and evidentiary weight given to the police blotter, as a basis
WHEREFORE, premises considered, the Court finds that plaintiff lacks sufficient for the factual finding of the RTC and the CA. He contends that the same entry was belied by
cause of action against the defendant and hence ordered his case dismissed and the Motor Vehicle Accident Report and testimony of the investigating policeman himself,
further orderes (sic) him to pay the defendant the following: attesting that it was Giddie Boy Coyel, not Leonardo Anit, who was driving the insured
vehicle.16
1) P20,000.00 as attorney’s fees plus P500.00 for appearance fee;
and

2) P50,000.00 as exemplary damages. Respondent avers that the same police report and testimony were of dubious nature. Both trial
and appellate courts noted that the report was made three days after the accident and did not
SO ORDERED.8 form part of the official police records.17

* On appeal with the Court of Appeals, the RTC decision was affirmed. The petition was
dismissed and the motion for reconsideration was denied. The CA stated:
The police blotter was admitted under Rule 130, Section 44 of the Rules of
IN VIEW OF THE FOREGOING, the decision appealed from is hereby AFFIRMED. Court.18 Under the said rule, the following are the requisites for its admissibility:
Consequently, the complaint is DISMISSED for lack of merit.

SO ORDERED.9
(a) that the entry was made by a public officer, or by another person, specially
* The instant petition seeks the reversal of the Court of Appeals’ Decision1 dated February 4, enjoined by law to do so;
1999, as well as its Resolution,2 dated September 7, 1999, in CA-G.R. CV No. 47227. The
assailed decision dismissed petitioner’s appeal and the resolution denied petitioner’s motion for (b) that it was made by the public officer in the performance of his duties, or by
reconsideration. such other person in the performance of a duty specially enjoined by law;

(c) that the public officer or other person had sufficient knowledge of the facts by
him stated, which must have been acquired by him personally or through official
Issues information.19

A. THE HONORABLE COURT OF APPEALS AND THE LOWER COURT RELIED MAINLY ON We agree with the trial and appellate courts in finding that the police blotter was properly
SECTION 44, RULE 130 OF THE RULES OF COURT IN UPHOLDING THE ENTRY IN THE POLICE admitted as they form part of official records.20 Entries in police records made by a
BLOTTER WHICH STATED THAT THE DRIVER OF THE INSURED VEHICLE WAS LEONARDO police officer in the performance of the duty especially enjoined by law are prima
ANIT Y PANES, WHO WAS NOT AN AUTHORIZED DRIVER. UNDER THE SAID SECTION 44, facie evidence of the fact therein stated, and their probative value may be either
RULE 130 ITSELF HOWEVER, THE POLICE BLOTTER IS MERELY A PRIMA FACIE EVIDENCE OF substantiated or nullified by other competent evidence.21 Although police blotters
THE FACTS STATED THEREIN WHICH MAY BE NULLIFIED BY OTHER EVIDENCE;10 are of little probative value, they are nevertheless admitted and considered in the
absence of competent evidence to refute the facts stated therein.
B. PERCEPTION OF THE HONORABLE COURT OF APPEALS’ ON THE "DIMINISHED"
CREDIBILITY OF PAT. FELIPE VILLAHERMOSA, THE TRAFFIC POLICE INVESTIGATOR, IS
MISPLACED AND UNFOUNDED;11
In this case, the entries in the police blotter reflected the information subject of the
C. THE DRIVER OF THE INSURED TRUCK WITH PLATE NR. FCG-538 WAS GIDDIE BOY Y controversy. Stated therein was the fact that Leonardo Anit was driving the insured truck with
COYEL, AN AUTHORIZED DRIVER OF THE SAID TRUCK. THE DRIVER OF THE OTHER TRUCK plate number FCG-538. This is unlike People v. Mejia,22 where we said that "entries in the
INVOLVED IN THE ACCIDENT WITH PLATE NR. FBS-917 WAS LEONARDO ANIT Y PANES;12 police blotters should not be given undue significance or probative value," since the Court
there found that "the entries in question are sadly wanting in material particulars".

281
RECTO, GAYLE ANGELI M.
2011-0008 | AUSL
Personal Notes on Remedial Law 2 Review (based on the syllabus of Prof. Henedino M. Brondial)

SERENO, J.:

Furthermore, in this case the police blotter was identified and formally offered as evidence.
The person who made the entries was likewise presented in court; he identified and certified
as correct the entries he made on the blotter. The information was supplied to the entrant by
Facts
the investigating officer who did not protest about any inaccuracy when the blotter was
presented to him. No explanation was likewise given by the investigating officer for the alleged
interchange of names. * During the 2007 elections, petitioner ran for the position of Representative of the 4th District
of Batangas, but lost. The 4th District of Batangas includes Lipa City.2 However, it is
undisputed that when petitioner filed his COC during the 2007 elections, he and his family
were then staying at his ancestral home in Barangay (Brgy.) Sico, San Juan, Batangas.

Petitioner also assails the credence given by the trial court to the version of the respondents
* When petitioner filed his COC1 for mayor of Lipa City for the 2010 elections, he stated
vis-à-vis the testimonies of the witnesses. Time and again we have reiterated the settled
therein that he had been a resident of the city for two (2) years and eight (8) months. Prior to
doctrine that great weight, and even finality, is given to the factual conclusions of the Court of
the 2010 elections, he had been twice elected (in 1995 and in 1998) as Provincial Board
Appeals which affirm those of the trial courts.23 We find on this score no reason to overturn
Member representing the 4th District of Batangas.
such conclusions.

* Private respondent Florencio Librea (private respondent) filed a "Petition to Deny Due
Course and to Cancel Certificate of Candidacy and to Disqualify a Candidate for Possessing
Some Grounds for Disqualification"3 against him before the COMELEC, docketed as SPA No.
On the issue of damages, we agree with petitioner that the award of exemplary damages was 09-047 (DC). Citing Section 78 in relation to Section 74 of the Omnibus Election Code,4 private
improper. In Tiongco v. Atty. Deguma24 we held that the entitlement to the recovery of respondent alleged that petitioner made material misrepresentations of fact in the latter’s COC
exemplary damages must be shown. In the case at bar, respondent have not shown sufficient and likewise failed to comply with the one-year residency requirement under Section 39 of the
evidence that petitioner indeed schemed to procure the dubious documents and lied through Local Government Code. 5 Allegedly, petitioner falsely declared under oath in his COC that he
his teeth to establish his version of the facts. What was found was that the document he had already been a resident of Lipa City for two years and eight months prior to the scheduled
presented was inadmissible, and its contents were dubious. However, no proof was adduced to 10 May 2010 local elections.
sufficiently establish that it came to his hands through his employment of underhanded
means. In Tiongco, we further stated:
* In support of his allegation, private respondent presented the following:

1. Petitioner’s COC for the 2010 elections filed on 1 December 20096

Although exemplary damages cannot be recovered as a matter of right, they also need not be
2. 2009 Tax Declarations for a house and lot (TCT Nos. 173355, 173356 and
proved. But a complainant must still show that he is entitled to moral, temperate or
buildings thereon) in Pinagtong-ulan, Lipa City registered under the name of
compensatory damages before the court may consider the question of whether or not
Bernadette Palomares, petitioner’s common-law wife7
exemplary damages should be awarded.25

3. Lipa City Assessor Certification of Property Holdings of properties under the


name of Bernadette Palomares8

Thus, it was error for the courts below to award exemplary damages in the absence of any
4. Affidavit executed by private respondent Florencio Librea9
award for moral, temperate or compensatory damages.

5. Sinumpaang Salaysay executed by Eladio de Torres10

6. Voter Certification on petitioner issued by COMELEC Election Officer Juan D.


The award of attorney’s fees must also be deleted. Such award was given in its extraordinary
Aguila, Jr.11
concept as indemnity for damages to be paid by the losing party to the prevailing party.26 But it
was not sufficiently shown that petitioner acted maliciously in instituting the claim for
damages. Perforce, the award of attorney’s fees was improper. 7. 1997 Voter Registration Record of petitioner12

8. National Statistics Office (NSO) Advisory on Marriages regarding petitioner13

WHEREFORE, the assailed Decision and Resolution of the Court of Appeals are AFFIRMED, 9. Lipa City Assessor Certificate of No Improvement on Block 2, Lot 3, Brgy.
with the MODIFICATION that the award of exemplary damages and attorney’s fees is hereby Lood, Lipa City registered in the name of petitioner14
DELETED. No pronouncement as to costs.
10. NSO Certificate of No Marriage of Bernadette Palomares15

11. Lipa City Assessor Certificate of No Improvement on Block 2, Lot 5, Brgy.


SO ORDERED. Lood, Lipa City registered in the name of petitioner16

12. Lipa City Permits and Licensing Office Certification that petitioner has no
business therein17

Bellosillo, (Chairman), Austria-Martinez, and Tinga, JJ., concur.


13. Apparent printout of a Facebook webpage of petitioner’s daughter, Mey
Bernadette Sabili18
Callejo, Sr., J., on leave.

14. Department of Education (DepEd) Lipa City Division Certification that the
names Bernadette Palomares, Mey Bernadette Sabili and Francis Meynard Sabili
(petitioner’s son) do not appear on its list of graduates19
EN BANC
15. Certification from the Office of the Election Officer of Lipa City that
G.R. No. 193261 April 24, 2012 Bernadette Palomares, Mey Bernadette Sabili and Francis Meynard Sabili do not
appear in its list of voters20
MEYNARDO SABILI, Petitioner,
16. Affidavit executed by Violeta Fernandez21
vs. COMMISSION ON ELECTIONS and FLORENCIO LIBREA, Respondents.
17. Affidavit executed by Rodrigo Macasaet22

282
RECTO, GAYLE ANGELI M.
2011-0008 | AUSL
Personal Notes on Remedial Law 2 Review (based on the syllabus of Prof. Henedino M. Brondial)

18. Affidavit Executed by Pablo Lorzano23 SEC. 6. Promulgation. - The promulgation of a Decision or Resolution of the
Commission or a Division shall be made on a date previously fixed, notice of
19. Petitioner’s 2007 COC for Member of House of Representative24 which shall be served in advance upon the parties or their attorneys personally,
or by registered mail, telegram, fax, or thru the fastest means of communication.
For ease of later discussion, private respondent’s evidence shall be grouped as follows: (1)
Certificates regarding ownership of real property; (2) petitioner’s Voter Registration and * Hence, petitioner filed with this Court a Petition (Petition for Certiorari with Extremely Urgent
Certification (common exhibits of the parties); (3) petitioner’s COCs in previous elections; (3) Application for the Issuance of a Status Quo Order and for the Conduct of a Special Raffle of
Certifications regarding petitioner’s family members; and (4) Affidavits of Lipa City residents. this Case) under Rule 64 in relation to Rule 65 of the Rules of Court, seeking the annulment of
the 26 January 2010 and 17 August 2010 Resolutions of the COMELEC. Petitioner attached to
his Petition a Certificate of Canvass of Votes and proclamation of Winning Candidates for Lipa
* On the other hand, petitioner presented the following evidence to establish the fact of his
City Mayor and Vice-Mayor issued by the City/Municipal Board of Canvassers,44 as well as a
residence in Lipa City:
copy of his Oath of Office.45 He also attached to his Petition another Certification of
Residency46 issued by Pinagtong-ulan Barangay Captain Dominador Honrade and sworn to
1. Affidavit executed by Bernadette Palomares25
before a notary public.

2. Birth Certificate of Francis Meynard Sabili26


* On 7 September 2010, this Court issued a Status Quo Ante Order47 requiring the parties to
observe the status quo prevailing before the issuance of the assailed COMELEC Resolutions.
3. Affidavit of Leonila Suarez (Suarez)27 Thereafter, the parties filed their responsive pleadings.

4. Certification of Residency issued by Pinagtong-ulan Barangay Captain,


Dominador Honrade28
Issues
5. Affidavit executed by Rosalinda Macasaet29
1. Whether the COMELEC acted with grave abuse of discretion when it failed to promulgate its
6. Certificate of Appreciation issued to petitioner by the parish of Sto. Nino of Resolution dated 17 August 2010 in accordance with its own Rules of Procedure; and
Pinagtong-ulan30
2. Whether the COMELEC committed grave abuse of discretion in holding that Sabili failed to
7. Designation of petitioner in the Advisory Body (AB) of Pinagtong-ulan, San prove compliance with the one-year residency requirement for local elective officials.
Jose/Lipa City Chapter of Guardians Brotherhood, Inc.31

8. COMELEC Voter Certification on petitioner issued by Election Officer Juan


Aguila, Jr.32
Ruling

9. COMELEC Application for Transfer/Transfer with Reactivation dated 6 June


2009 signed by Election Officer Juan Aguila, Jr.33

1. On whether the COMELEC acted with grave abuse of discretion when it failed to promulgate
10. Petitioner’s Income Tax Return for 200734
its Resolution dated 17 August 2010 in accordance with its own Rules of Procedure

11. Official Receipt for petitioner’s income tax payment for 200735

12. Petitioner’s Income Tax Return for 200836


Petitioner argues that the assailed 17 August 2010 COMELEC Resolution, which denied
petitioner’s Motion for Reconsideration, is null and void. The Resolution was allegedly not
13. Official Receipt for petitioner’s income tax payment for 200837 promulgated in accordance with the COMELEC’s own Rules of Procedure and, hence, violated
petitioner’s right to due process of law.
14. Birth Certificate of Mey Bernadette Sabili38

15. Affidavit executed by Jacinto Cornejo, Sr.39


The rules governing the Petition for Cancellation of COC in this case is COMELEC Resolution
16. Joint Affidavit of twenty-one (21) Pinagtong-ulan residents, including past No. 8696 (Rules on Disqualification of Cases Filed in Connection with the May 10, 2010
and incumbent Pinagtong-ulan officials.40 Automated National and Local Elections), which was promulgated on 11 November 2009.
Sections 6 and 7 thereof provide as follows:
For ease of later discussion, petitioner’s evidence shall be grouped as follows: (1) his Income
Tax Returns and corresponding Official Receipts for the years 2007 and 2008; (2) Certification
from the barangay captain of Pinagtong-ulan; (3) Affidavit of his common-law wife, Bernadette
Palomares; and (4) Affidavits from a previous property owner, neighbors, Certificate of SEC. 6. Promulgation. - The promulgation of a Decision or Resolution of the
Appreciation from the barangay parish and Memorandum from the local chapter of Guardians Commission or a Division shall be made on a date previously fixed, notice of
Brotherhood, Inc. which shall be served in advance upon the parties or their attorneys personally,
or by registered mail, telegram, fax or thru the fastest means of communication.
* In its Resolution dated 26 January 2010,41 the COMELEC Second Division granted the
Petition of private respondent, declared petitioner as disqualified from seeking the mayoralty
post in Lipa City, and canceled his Certificate of Candidacy for his not being a resident of Lipa
City and for his failure to meet the statutory one-year residency requirement under the law. SEC. 7. Motion for Reconsideration. - A motion to reconsider a Decision,
Resolution, Order or Ruling of a Division shall be filed within three (3) days from
* Petitioner moved for reconsideration of the 26 January 2010 Resolution of the COMELEC, the promulgation thereof. Such motion, if not pro-forma, suspends the execution
during the pendency of which the 10 May 2010 local elections were held. The next day, he for implementation of the Decision, Resolution, Order or Ruling.
was proclaimed the duly elected mayor of Lipa City after garnering the highest number of
votes cast for the said position. He accordingly filed a Manifestation42 with the COMELEC en
banc to reflect this fact.

Within twenty-four (24) hours from the filing thereof, the Clerk of the
* In its Resolution dated 17 August 2010,43 the COMELEC en banc denied the Motion for
Commission shall notify the Presiding Commissioner. The latter shall within two
Reconsideration of petitioner. Although he was able to receive his copy of the Resolution, no
(2) days thereafter certify the case to the Commission en banc.
prior notice setting the date of promulgation of the said Resolution was received by him.
Meanwhile, Section 6 of COMELEC Resolution No. 8696 (Rules on Disqualification Cases Filed
in Connection with the May 10, 2012 Automated National and Local Elections) requires the
parties to be notified in advance of the date of the promulgation of the Resolution.
283
RECTO, GAYLE ANGELI M.
2011-0008 | AUSL
Personal Notes on Remedial Law 2 Review (based on the syllabus of Prof. Henedino M. Brondial)

The Clerk of the Commission shall calendar the Motion for Reconsideration for This contention is untenable. Promulgation is the process by which a decision is published,
the resolution of the Commission en banc within three (3) days from the officially announced, made known to the public or delivered to the clerk of court for filing,
certification thereof. coupled with notice to the parties or their counsel (Neria v. Commissioner of Immigration, L-
24800, May 27, 1968, 23 SCRA 812). It is the delivery of a court decision to the clerk of court
for filing and publication (Araneta v. Dinglasan, 84 Phil. 433). It is the filing of the signed
decision with the clerk of court (Sumbing v. Davide, G.R. Nos. 86850-51, July 20, 1989, En
Banc Minute Resolution). The additional requirement imposed by the COMELEC rules of notice in
However, the COMELEC Order dated 4 May 201048 suspended Section 6 of COMELEC
advance of promulgation is not part of the process of promulgation. Hence, We do not
Resolution No. 8696 by ordering that "all resolutions be delivered to the Clerk of the
agree with petitioner’s contention that there was no promulgation of the trial court's decision.
Commission for immediate promulgation" in view of "the proximity of the Automated National
The trial court did not deny that it had officially made the decision public. From the recital of
and Local Elections and lack of material time." The Order states:
facts of both parties, copies of the decision were sent to petitioner's counsel of record and
petitioner’s (sic) himself. Another copy was sent to private respondent.

ORDER

What was wanting and what the petitioner apparently objected to was not the promulgation of
the decision but the failure of the trial court to serve notice in advance of the promulgation of
its decision as required by the COMELEC rules. The failure to serve such notice in advance of
Considering the proximity of the Automated National and Local Elections and lack the promulgation may be considered a procedural lapse on the part of the trial court which did
of material time, the Commission hereby suspends Sec. 6 of Resolution No. 8696 not prejudice the rights of the parties and did not vitiate the validity of the decision of the trial
promulgated on November 11, 2009, which reads: court nor (sic) of the promulgation of said decision.

Sec. 6. Promulgation. - The promulgation of a Decision or Resolution of the Moreover, quoting Pimping v. COMELEC,50 citing Macabingkil v. Yatco,51 we further held in
Commission or a Division shall be made on a date previously fixed, notice of the same case that failure to receive advance notice of the promulgation of a decision is not
which shall be served upon the parties or their attorneys personally, or by sufficient to set aside the COMELEC’s judgment, as long as the parties have been afforded an
registered mail, telegram, fax or thru the fastest means of communication." opportunity to be heard before judgment is rendered, viz:

Let all resolutions be delivered to the Clerk of the Commission for immediate The fact that petitioners were not served notice in advance of the promulgation of the decision
promulgation. in the election protest cases, in Our view, does not constitute reversible error or a reason
sufficient enough to compel and warrant the setting aside of the judgment rendered by the
Comelec. Petitioners anchor their argument on an alleged denial to them (sic) due process to
the deviation by the Comelec from its own made rules. However, the essence of due process is
SO ORDERED. that, the parties in the case were afforded an opportunity to be heard.

Petitioner claims that he did not receive notice of the said suspension of Section 6 of COMELEC In the present case, we read from the COMELEC Order that the exigencies attendant to the
Resolution No. 8696. Thus, his right to due process was still violated. On the other hand, the holding of the country’s first automated national elections had necessitated that the COMELEC
COMELEC claims that it has the power to suspend its own rules of procedure and invokes suspend the rule on notice prior to promulgation, and that it instead direct the delivery of all
Section 6, Article IX-A of the Constitution, which gives it the power "to promulgate its own resolutions to the Clerk of the Commission for immediate promulgation. Notably, we see no
rules concerning pleadings and practice before it or before any of its offices." prejudice to the parties caused thereby. The COMELEC’s Order did not affect the right of the
parties to due process. They were still furnished a copy of the COMELEC Decision and were
able to reckon the period for perfecting an appeal. In fact, petitioner was able to timely lodge
a Petition with this Court.

We agree with the COMELEC on this issue.

Clearly, the COMELEC validly exercised its constitutionally granted power to make its own rules
of procedure when it issued the 4 May 2010 Order suspending Section 6 of COMELEC
In Lindo v. Commission on Elections,49 petitioner claimed that there was no valid Resolution No. 8696. Consequently, the second assailed Resolution of the COMELEC cannot be
promulgation of a Decision in an election protest case when a copy thereof was merely set aside on the ground of COMELEC’s failure to issue to petitioner a notice setting the date of
furnished the parties, instead of first notifying the parties of a set date for the promulgation the promulgation thereof.
thereof, in accordance with Section 20 of Rule 35 of the COMELEC’s own Rules of Procedure,
as follows:

2. On whether the COMELEC committed grave abuse of discretion in holding that Sabili failed
to prove compliance with the one-year residency requirement for local elective officials
Sec. 20. Promulgation and Finality of Decision. — The decision of the court shall
be promulgated on a date set by it of which due notice must be given the
parties. It shall become final five (5) days after promulgation. No motion for
reconsideration shall be entertained.
As a general rule, the Court does not ordinarily review the COMELEC’s appreciation and
evaluation of evidence. However, exceptions thereto have been established, including when
the COMELEC's appreciation and evaluation of evidence become so grossly unreasonable as to
turn into an error of jurisdiction. In these instances, the Court is compelled by its bounden
Rejecting petitioner’s argument, we held therein that the additional rule requiring notice to the constitutional duty to intervene and correct the COMELEC's error.52
parties prior to promulgation of a decision is not part of the process of promulgation. Since
lack of such notice does not prejudice the rights of the parties, noncompliance with this rule is a
procedural lapse that does not vitiate the validity of the decision. Thus:

In Mitra v. Commission on Elections, (G.R. No. 191938, 2 July 2010), we explained that the
COMELEC’s use of wrong or irrelevant considerations in deciding an issue is sufficient to taint
its action with grave abuse of discretion -
284
RECTO, GAYLE ANGELI M.
2011-0008 | AUSL
Personal Notes on Remedial Law 2 Review (based on the syllabus of Prof. Henedino M. Brondial)

As a concept, "grave abuse of discretion" defies exact definition; generally, it refers to The ruling on private respondent’s evidence
"capricious or whimsical exercise of judgment as is equivalent to lack of jurisdiction;" the
abuse of discretion must be patent and gross as to amount to an evasion of a positive duty or a
virtual refusal to perform a duty enjoined by law, or to act at all in contemplation of law, as
where the power is exercised in an arbitrary and despotic manner by reason of passion and
We begin with an evaluation of the COMELEC’s appreciation of private respondent’s evidence.
hostility. Mere abuse of discretion is not enough; it must be grave. We have held, too, that the
use of wrong or irrelevant considerations in deciding an issue is sufficient to taint a decision-
maker's action with grave abuse of discretion.

a) Petitioner’s Voter Certification, Registration and COCs in previous elections

Closely related with the limited focus of the present petition is the condition, under Section 5,
Rule 64 of the Rules of Court, that findings of fact of the COMELEC, supported by substantial
evidence, shall be final and non-reviewable. Substantial evidence is that degree of evidence Petitioner’s Voter Certification is a common exhibit of the parties. It states, among others, that
that a reasonable mind might accept to support a conclusion. petitioner is a resident of Pinagtong-ulan, Lipa City, Batangas; that he had been a resident of
Lipa City for two (2) years and three (3) months; and that he was so registered on 31 October
2009. The information therein was "certified correct" by COMELEC Election Officer Juan B.
Aguila, Jr.

In light of our limited authority to review findings of fact, we do not ordinarily review in a
certiorari case the COMELEC's appreciation and evaluation of evidence. Any misstep by the
COMELEC in this regard generally involves an error of judgment, not of jurisdiction.
Private respondent presented this document as proof that petitioner misrepresented that he is
a resident of Lipa City. On the other hand, the latter presented this document as proof of his
residency.

In exceptional cases, however, when the COMELEC's action on the appreciation and evaluation
of evidence oversteps the limits of its discretion to the point of being grossly unreasonable, the
Court is not only obliged, but has the constitutional duty to intervene. When grave abuse of
discretion is present, resulting errors arising from the grave abuse mutate from error of The COMELEC correctly ruled that the Voter Certification issued by the COMELEC Election
judgment to one of jurisdiction. Officer, Atty. Juan B. Aguila, Jr., was not conclusive proof that petitioner had been a resident
of Lipa City since April 2007. It noted that Aguila is not the competent public officer to certify
the veracity of this claim, particularly because petitioner’s COMELEC registration was approved
only in October 2009.

Before us, petitioner has alleged and shown the COMELEC’s use of wrong or irrelevant
considerations in deciding the issue of whether petitioner made a material misrepresentation
of his residency qualification in his COC as to order its cancellation. Among others, petitioner
pointed to the COMELEC’s inordinate emphasis on the issue of property ownership of The Voter Registration Record of petitioner accomplished on 21 June 1997 showing that he
petitioner’s declared residence in Lipa City, its inconsistent stance regarding Palomares’s was a resident of Sico, San Juan, Batangas, as well as his various COCs dated 21 June 1997
relationship to the Pinagtong-ulan property, and its failure to consider in the first instance the and March 2007 indicating the same thing, were no longer discussed by the COMELEC - and
certification of residence issued by the barangay captain of Pinagtong-ulan. Petitioner bewails rightly so. These pieces of evidence showing that he was a resident of Sico, San Juan,
that the COMELEC required "more" evidence to show the change in his residence, Batangas on the said dates are irrelevant as, prior to April 2007, petitioner was admittedly a
notwithstanding the various pieces of evidence he presented and the fact that under the law, resident of Sico, San Juan Batangas. Rather, the relevant time period for consideration is that
the quantum of evidence required in these cases is merely substantial evidence and not clear from April 2007 onwards, after petitioner’s alleged change of domicile.
and convincing evidence. Petitioner further ascribes grave abuse of discretion in the
COMELEC’s brushing aside of the fact that he has been filing his ITR in Lipa City (where he
indicates that he is a resident of Pinagtong-ulan) on the mere expedient that the law allows
the filing of the ITR not only in the place of legal residence but, alternately, in his place of b) Certificates regarding ownership of real property
business. Petitioner notes that private respondent’s own evidence shows that petitioner has no
business in Lipa City, leaving only his residence therein as basis for filing his ITR therein.

The various certificates and tax declarations adduced by private respondent showed that the
Lipa property was solely registered in the name of petitioner’s common-law wife, Bernadette
Hence, in resolving the issue of whether the COMELEC gravely abused its discretion in ruling Palomares. In discussing the import of this document, the COMELEC reasoned that, being a
that petitioner had not sufficiently shown that he had resided in Lipa City for at least one year "seasoned politician," he should have registered the Lipa property (which he claimed to have
prior to the May 2010 elections, we examine the evidence adduced by the parties and the purchased with his personal funds) in his own name. Such action "would have offered positive
COMELEC’s appreciation thereof. proof of intent to change actual residence" from San Juan, Batangas to Lipa City, considering
that he had previously declared his ancestral home in San Juan, Batangas as his domicile.
Since Palomares and petitioner are common-law spouses not capacitated to marry each other,
the property relation between them is governed by Article 148 of the Family Code,55 where
In the present case, the parties are in agreement that the domicile of origin of Sabili was Brgy. only the parties’ actual contributions are recognized. Hence, petitioner cannot prove ownership
Sico, San Juan, Batangas. He claims that he abandoned his domicile of origin and established of a property and residence in Lipa City through the registered ownership of the common-law
his domicile of choice in Brgy. Pinagtong-ulan, Lipa City, thereby making him qualified to run wife of the property in Lipa City.
for Lipa City mayor. On the other hand, respondent COMELEC held that no such change in
domicile or residence took place and, hence, the entry in his Certificate of Candidacy showing
that he was a resident of Brgy. Pinagtong-ulan, Lipa City constituted a misrepresentation that
disqualified him from running for Lipa City mayor. On the other hand, petitioner bewails the inordinate emphasis that the COMELEC bestowed
upon the question of whether the Lipa property could be considered as his residence, for the
reason that it was not registered in his name. He stresses that the issue should be residence,
not property ownership.
To establish a new domicile of choice, personal presence in the place must be coupled with
conduct indicative of the intention to make it one's fixed and permanent place of abode.53 As
in all administrative cases, the quantum of proof necessary in election cases is substantial
evidence, or such relevant evidence as a reasonable mind will accept as adequate to support a It is true that property ownership is not among the qualifications required of candidates for
conclusion.54 local election.56 Rather, it is a candidate’s residence in a locality through actual residence in
285
RECTO, GAYLE ANGELI M.
2011-0008 | AUSL
Personal Notes on Remedial Law 2 Review (based on the syllabus of Prof. Henedino M. Brondial)

whatever capacity. Indeed, we sustained the COMELEC when it considered as evidence other than in the place where one is elected, does not constitute loss of residence."62 In fact,
tending to establish a candidate’s domicile of choice the mere lease (rather than ownership) of Section 117 of the Omnibus Election Code provides that transfer of residence to any other
an apartment by a candidate in the same province where he ran for the position of place by reason of one's "occupation; profession; employment in private and public service;
governor.57 In the more recent case of Mitra v. Commission on Elections,58 we reversed the educational activities; work in military or naval reservations; service in the army, navy or air
COMELEC ruling that a candidate’s sparsely furnished, leased room on the mezzanine of a force, the constabulary or national police force; or confinement or detention in government
feedmill could not be considered as his residence for the purpose of complying with the institutions in accordance with law" is not deemed as loss of residence.
residency requirement of Section 78 of the Omnibus Election Code.59

As to the Dissent’s second assertion, petitioner apparently does not maintain a business in Lipa
The Dissent claims that the registration of the property in Palomares’s name does not prove City. However, apart from the Pinagtong-ulan property which both Suarez (the previous
petitioner’s residence as it merely showed "donative intent" without the necessary formalities property owner) and Palomares swear was purchased with petitioner’s own funds, the records
or payment of taxes. also indicate that there are two other lots in Lipa City, particularly in Barangay Lodlod, Lipa
City63 which are registered jointly in the name of petitioner and Palomares. In fact, it was
private respondent who presented the Lipa City Assessor’s Certificate to this effect. Even
assuming that this Court were to disregard the two Lodlod lots, it is well-established that
property ownership (and similarly, business interest) in the locality where one intends to run
However, whatever the nature of the transaction might be, this point is immaterial for the
for local elective post is not requirement of the Constitution.64
purpose of ascertaining petitioner’s residence. We have long held that it is not required that a
candidate should have his own house in order to establish his residence or domicile in a place.
It is enough that he should live in the locality, even in a rented house or that of a friend or
relative.60 What is of central concern then is that petitioner identified and established a place
in Lipa City where he intended to live in and return to for an indefinite period of time. More importantly, we have gone so far as to rule that there is nothing "wrong in an individual
changing residences so he could run for an elective post, for as long as he is able to prove
with reasonable certainty that he has

Hence, while the COMELEC correctly ruled that, of itself, Palomares’ ownership of the Lipa
property does not prove that she or - and in view of their common-law relations, petitioner -
resides in Lipa City, nevertheless, the existence of a house and lot apparently owned by effected a change of residence for election law purposes for the period required by law."65
petitioner’s common-law wife, with whom he has been living for over two decades, makes
plausible petitioner’s allegation of bodily presence and intent to reside in the area.

d) Affidavits of Lipa City residents

c) Certifications regarding the family members of petitioner

Private respondent also presented the affidavits of Violeta Fernandez66 and Rodrigo
Macasaet,67 who were also residents of Pinagtong-ulan. Both stated that petitioner did not
Private respondent presented a Certification from the DepEd, Lipa City Division, indicating that reside in Pinagtong-ulan, as they had "rarely seen" him in the area. Meanwhile, Pablo
the names Bernadette Palomares, Mey Bernadette Sabili (petitioner’s daughter) and Francis Lorzano,68 in his Affidavit, attested that although the Lipa property was sometimes used for
Meynard Sabili (petitioner’s son) do not appear on the list of graduates of Lipa City. Private gatherings, he did "not recall having seen" petitioner in their barangay. On the other hand,
respondent also presented a Certification from the Office of the Election Officer of Lipa City private respondent69 and Eladio de Torres,70 both residents of Brgy. Calamias, reasoned that
that the names of these family members of petitioner do not appear in its list of voters. petitioner was not a resident of Lipa City because he has no work or family there.

As the issue at hand is petitioner’s residence, and not the educational or voting record of his The COMELEC did not discuss these Affidavits in its assailed Resolution. It was correct in doing
family, the COMELEC properly did not consider these pieces of evidence in arriving at its so, particularly considering that these Affidavits were duly controverted by those presented by
Resolution. petitioner.

The Dissent nevertheless asserts that because his children do not attend educational Moreover, even assuming the truth of the allegation in the Affidavits that petitioner was "rarely
institutions in Lipa and are not registered voters therein, and because petitioner does not seen" in the area, this does not preclude the possibility of his residence therein. In Fernandez
maintain a business therein nor has property v. House of Representatives Electoral Tribunal,71 we held that the averments of certain
barangay health workers - that they failed to see a particular candidate whenever they made
rounds of the locality of which he was supposed to be a resident - is of no moment. It is
possible that the candidate was out of the house to attend to his own business at the time.
The law does not require a person to be in his home twenty-four (24) hours a day, seven (7)
in his name, petitioner is unable to show the existence of real and substantial reason for his
days a week, to fulfill the residency requirement.
stay in Lipa City.

The ruling on petitioner’s evidence


As to the Dissent’s first assertion, it must be stressed that the children, like the wife, do not
dictate the family domicile. Even in the context of marriage, the family domicile is jointly
decided by both husband and wife.61 In addition, we note that the transfer to Lipa City
occurred in 2007, when petitioner’s children were already well into college and could very well
have chosen to study elsewhere than in Lipa City. We now evaluate how the COMELEC appreciated petitioner’s evidence:

Also, it is petitioner’s domicile which is at issue, and not that of his children. But even a) Petitioner’s Income Tax Returns for 2007 and 2008
assuming that it was petitioner himself (rather than his children) who attended educational
institutions or who registered as a voter in a place other than Lipa City, we have held that
"absence from residence to pursue studies or practice a profession or registration as a voter

286
RECTO, GAYLE ANGELI M.
2011-0008 | AUSL
Personal Notes on Remedial Law 2 Review (based on the syllabus of Prof. Henedino M. Brondial)

The Income Tax Returns of petitioner presented below showed that petitioner had been In Country Bankers Insurance Corporation v. Lianga Bay and Community Multi-
paying his Income Tax (2007 and 2008) to the Revenue District Office of Lipa City. In waving purpose Cooperative, Inc.,75 we explained that the following three (3) requisites
aside his Income Tax Returns, the COMELEC held that these were not indications of residence must concur for entries in official records to be admissible in evidence:
since Section 51(B) of the National Internal Revenue Code does not only state that it shall be
filed in a person’s legal residence, but that it may alternatively be filed in a person’s principal
place of business.
(a) The entry was made by a public officer, or by another person specially enjoined
by law to do so;

In particular, Section 51(B) of the National Internal Revenue Code72 provides that the Income
Tax Return shall be filed either in the place where a person resides or where his principal place
of business is located. However, private respondent’s own evidence - a Certification from the
(b) It was made by the public officer in the performance of his duties, or by such
City Permits and Licensing Office of Lipa City- showed that there was no business registered in
other person in the performance of a duty specially enjoined by law; and
the City under petitioner’s name.

(c) The public officer or other person had sufficient knowledge of the facts stated
Thus, COMELEC failed to appreciate that precisely because an individual income tax return
by him, which facts must have been acquired by him personally or through official
may only be filed either in the legal residence OR the principal place of business, as prescribed
information.
under the law, the fact that Sabili was filing his Income Tax Returns in Lipa City
notwithstanding that he had no business therein showed that he had actively elected to
establish his residence in that city.

As to the first requisite, the Barangay Secretary is required by the Local


Government Code to "keep an updated record of all inhabitants of the
barangay."76 Regarding the second requisite, we have explicitly recognized in
The Dissent claims that since the jurisdiction of RDO Lipa City includes both San Juan and Lipa
Mitra v. Commission on Elections,77 that "it is the business of a punong barangay
City, petitioner’s filing of his ITR therein can also support an intent to remain in San Juan,
to know who the residents are in his own barangay." Anent the third requisite, the
Batangas - petitioner’s domicile of origin.
Barangay Captain’s exercise of powers and duties78 concomitant to his position
requires him to be privy to these records kept by the Barangay Secretary.

However, a simple perusal of the Income Tax Returns and Revenue Official Receipts for 2007
and 2008 shows that petitioner invariably declares his residence to be Pinagtong-ulan, Lipa
Accordingly, there is basis in faulting the COMELEC for its failure to consider
City, rather than San Juan, Batangas.73 Hence, while petitioner may be submitting his income
Honrade’s Certification on the sole ground that it was initially not notarized.
tax return in the same RDO, the declaration therein is unmistakable. Petitioner considers Lipa
City to be his domicile.

Meanwhile, the Dissent opines that the sworn affidavit of the barangay chair of Pinagtong-ulan
that petitioner is a resident of Lipa City does not help petitioner’s case because it was not
b) Certification from the Barangay Captain of Pinagtong-ulan
shown that the term "resident" as used therein carries the same meaning as domicile, that is,
not merely bodily presence but also, animus manendi or intent to return. This Court has ruled
otherwise.

The COMELEC did not consider in the first instance the Certification issued by Pinagtong-ulan
Barangay Captain Dominador Honrade74 (Honrade) that petitioner had been residing in Brgy
Pinagtong-ulan since 2007. When this oversight was raised as an issue in petitioner’s Motion
In Mitra v. Commission on Elections,79 the declaration of Aborlan’s punong barangay that
for Reconsideration, the COMELEC brushed it aside on the ground that the said Certification
petitioner resides in his barangay was taken to have the same meaning as domicile, inasmuch
was not sworn to before a notary public and, hence, "cannot be relied on." Subsequently,
as the said declaration was made in the face of the Court’s recognition that Mitra "might not
petitioner presented another, substantially identical, Certification from the said Pinagtong-ulan
have stayed in Aborlan nor in Palawan for most of 2008 and 2009 because his office and
Barangay Captain, save for the fact that it had now been sworn to before a notary public.
activities as a Representative were in Manila."

We disagree with the COMELEC’s treatment of the Barangay Captain’s Certification and find
Assuming that the barangay captain’s certification only pertains to petitioner’s bodily presence in
the same tainted with grave abuse of discretion.
Pinagtong-ulan, still, the COMELEC cannot deny the strength of this evidence in establishing
petitioner’s bodily presence in Pinagtong-ulan since 2007.

Even without being sworn to before a notary public, Honrade’s Certification would
not only be admissible in evidence, but would also be entitled to due consideration.
c) Affidavit of petitioner’s common law wife

Rule 130, Section 44 of the Rules of Court provides:


To substantiate his claim of change of domicile, petitioner also presented the affidavit of
Palomares, wherein the latter swore that she and petitioner began residing in Lipa City in
2007, and that the funds used to purchase the Lipa property were petitioner’s personal funds.
The COMELEC ruled that the Affidavit was self-serving for having been executed by petitioner’s
SEC. 44. Entries in official records.—Entries in official records made in common-law wife. Also, despite the presentation by petitioner of other Affidavits stating that
the performance of his duty by a public officer of the Philippines, or by a he and Palomares had lived in Brgy. Pinagtong-ulan since 2007, the latter’s Affidavit was
person in the performance of a duty specially enjoined by law, are rejected by the COMELEC for having no independent collaboration.
prima facie evidence of the facts therein stated.

287
RECTO, GAYLE ANGELI M.
2011-0008 | AUSL
Personal Notes on Remedial Law 2 Review (based on the syllabus of Prof. Henedino M. Brondial)

Petitioner faults the COMELEC’s stand, which it claims to be inconsistent. He argues that since Before further discussing this, it is pertinent to point out that these were not the reasons
the property regime between him and Palomares is governed by Article 148 of the Family adduced by the COMELEC in the assailed Resolutions. Assuming that the above reasons were
Code (based on the parties’ actual contribution) as the COMELEC stressed, then Palomares’s the unuttered considerations of the COMELEC in coming up with its conclusions, such
Affidavit expressly stating that petitioner’s money alone had been used to purchase the Lipa reasoning still exhibits grave abuse of discretion.
property (notwithstanding that it was registered in her name) was not self-serving, but was in
fact, a declaration against interest.

As to the Dissent’s first argument, it must be remembered that a transfer of


domicile/residence need not be completed in one single instance. Thus, in Mitra v. Commission
Petitioner’s argument that Palomares’s affidavit was a "declaration against interest" is, strictly on Elections,85 where the evidence showed that in 2008, petitioner Mitra had leased a small
speaking, inaccurate and irrelevant. A declaration against interest, under the Rules of Civil room at Maligaya Feedmills located in Aborlan and, in 2009 purchased in the same locality a
Procedure, refers to a "declaration made by a person deceased, or unable to testify against lot where he began constructing his house, we recognized that petitioner "transferred by
the interest of a declarant, if the fact asserted in the declaration was at the time it was made incremental process to Aborlan beginning 2008 and concluded his transfer in early 2009" and
so far contrary to declarant’s own interest, that a reasonable man in his position would not thus, he transferred his residence from Puerto Princesa City to Aborlan within the period
have made the declaration unless he believed it to be true."80 A declaration against interest is required by law. We cannot treat the transfer to the Pinagtong-ulan house any less than we
an exception to the hearsay rule.81 As such, it pertains only to the admissibility of, not the did Mitra’s transfer to the Maligaya Feedmills room.
weight accorded to, testimonial evidence.82

Moreover, the Joint Affidavit of twenty-one (21) Pinagtong-ulan residents, including former
Nevertheless, we see the logic in petitioner’s claim that the COMELEC had committed grave and incumbent barangay officials, attests that petitioner had begun living in the Pinagtong-
abuse of discretion in being inconsistent in its stand regarding Palomares, particularly ulan house and lot before the May 2007 elections such that it was where his coordinators for
regarding her assertion that the Lipa property had been purchased solely with petitioner’s the May 2007 elections went to meet him.86 Jacinto Cornejo Sr., the contractor who
money. If the COMELEC accepts the registration of the Lipa property in her name to be renovated the Pinagtong-ulan house when it was bought by petitioner, also swore that
accurate, her affidavit disavowing ownership thereof in favor of petitioner was far from self - petitioner and his family began living therein even while it was being renovated.87 Another
serving as it ran counter to her (and her children’s) property interest. Affidavit petitioner adduced was that of Rosalinda Macasaet, a resident of Brgy. Pinagtong-
ulan,88 who stated that she also sold a lot she owned in favor of petitioner and Palomares.
The latter bought her lot since it was adjacent to the Lipa house and lot they had earlier
acquired. Macasaet also swore that the couple had actually resided in the house located in
Pinagtong-ulan since April 2007, and that she knew this because her own house was very near
The Dissent states that it was not unreasonable for the COMELEC to believe that Palomares
the couple’s own. Macasaet’s Affidavit is a positive assertion of petitioner’s actual physical
may have committed misrepresentations in her affidavit considering that she had perjured
presence in Brgy. Pinagtong-ulan, Lipa City.
herself as an informant on the birth certificates of her children with respect to the supposed
date and place of her marriage to petitioner. However, this was not the reason propounded by
the COMELEC when it rejected Palomares’ affidavit.

While private respondent had adduced affidavits of two Pinagtong-ulan residents (that of
Violeta Fernandez89 and Rodrigo Macasaet)90 attesting that petitioner could not be a resident
of Pinagtong-ulan as he was "rarely seen" in the area, these affidavits were controverted by
Moreover, it is notable that Palomares’ assertion in her affidavit that she and petitioner have
the Joint affidavit of twenty-one (21) Pinagtong-ulan residents who plainly accused the two of
been living in the Pinagtong-ulan property since April 2007 is corroborated by other evidence,
lying. Meanwhile, the affidavits of private respondent91 and Eladio de Torres92 stating that
including the affidavits of Pinagtong-ulan barangay officials and neighbors.
petitioner is not a resident of Lipa City because he has no work or family there is hardly
worthy of credence since both are residents of Barangay Calamias, which is, and private
respondent does not contest this, about 15 kilometers from Pinagtong-ulan.

d) Affidavits from a previous property owner, neighbors, certificate from parish and
designation from socio-civic organization

As to the Dissent’s second argument, the fact that the notarization of the deed of absolute
sale of the property was made months after April 2007 does not negate petitioner’s claim that
he started residing therein in April 2007. It is clear from the Affidavit of the property’s seller,
The Affidavit issued by Leonila Suarez83 (erstwhile owner of the Lipa house and lot) states Leonila Suarez, that it was not yet fully paid in April 2007, so it was understandable that a
that in April 2007, after she received the down payment for the Lipa property and signed an deed of absolute sale was not executed at the time. Thus:
agreement that petitioner would settle her bank obligations in connection with the said
transaction, he and Palomares actually started residing at Pinagtong-ulan. The COMELEC
brushed this Affidavit aside as one that "merely narrates the circumstances surrounding the
sale of the property and mentions in passing that Sabili and Palomares lived in Pinagtong-ulan
That initially, the contract to sell was entered into by and between Mr. & Mrs. Meynardo Asa
since April 2007 up to the present."84
Sabili and Bernadette Palomares and myself, but eventually the spouses changed their mind,
and after the couple settled all my loan obligations to the bank, they requested me to put the
name of Ms. Bernadette P. Palomares instead of Mr. & Mrs. Meynardo Asa Sabili and
Bernadette Palomares in the absolute deed of sale;
We disagree with the COMELEC’s appreciation of the Suarez Affidavit. Since she was its owner,
transactions for the purchase of the Lipa property was within her personal knowledge.
Ordinarily, this includes the arrangement regarding who shall pay for the property and when, if
ever, it shall be occupied by the buyers. We thus consider that her statements impact
That it was Mr. Meynardo Asa Sabili who came to my former residence at Barangay Pinagtong-
positively on petitioner’s claim of residence.
ulan sometime in the month of April 2007. At that time, Mr. Meynardo Asa Sabili was still
running for Representative (Congressman) in the 4th District of Batangas;

The Dissent on the other hand argues that the claim that petitioner started living in the Lipa
house and lot in April 2007 is made dubious by the fact that (1) there might not be enough
That after payment of the down payment and signing of an agreement that Mr. Meynardo Asa
time to effect an actual and physical change in residence a month before the May 2007
Sabili will be the one to settle my bank obligations, Mr. & Mrs. Meynardo A. Sabili and
elections when petitioner ran for representative of the 4th District of Batangas; and (2) the
Bernadette Palomares had an actual transfer of their residence at Barangay Pinagtong-ulan,
Deed of Absolute Sale was notarized, and the subsequent transfer of ownership in the tax
Lipa City;
declaration was made, only in August 2008.

288
RECTO, GAYLE ANGELI M.
2011-0008 | AUSL
Personal Notes on Remedial Law 2 Review (based on the syllabus of Prof. Henedino M. Brondial)

That they started living and residing in Pinagtong-ulan in the month of April, 2007 up to this In this regard, we reiterate our ruling in Frivaldo v. Commission on Elections97 that "(t)o
point in time; xxx93 successfully challenge a winning candidate's qualifications, the petitioner must clearly
demonstrate that the ineligibility is so patently antagonistic to constitutional and legal
principles that overriding such ineligibility and thereby giving effect to the apparent will of the
people, would ultimately create greater prejudice to the very democratic institutions and
juristic traditions that our Constitution and laws so zealously protect and promote."
As to the rest of the documents presented by petitioner, the COMELEC held that the
Memorandum issued by the Guardians Brotherhood Inc. San Jose/Lipa City Chapter merely
declares the designation of petitioner in the organization, without any showing that residence
in the locality was a requirement for that designation. Meanwhile, the Certificate of
Appreciation was nothing more than an acknowledgment of petitioner’s material and financial Similarly, in Japzon v. Commission on Elections,98 we concluded that "when the evidence of
support, and not an indication of residence. the alleged lack of residence qualification of a candidate for an elective position is weak or
inconclusive and it clearly appears that the purpose of the law would not be thwarted by
upholding the victor's right to the office, the will of the electorate should be respected. For the
purpose of election laws is to give effect to, rather than frustrate, the will of the voters."
We agree that considered separately, the Guardians Brotherhood Memorandum and the
Pinagtong-ulan Parish Certificate of Appreciation do not establish petitioner’s residence in
Pinagtong-ulan, Lipa City. Nevertheless, coupled with the fact that petitioner had twice been
elected as Provincial Board Member representing the Fourth District of Batangas, which In sum, we grant the Petition not only because petitioner sufficiently established his
encompasses Lipa City, petitioner’s involvement in the religious life of the community, as compliance with the one-year residency requirement for local elective officials under the law.
attested to by the certificate of appreciation issued to him by the Pinagtong-ulan parish for his We also recognize that "(a)bove and beyond all, the determination of the true will of the
"material and financial support" as President of the Barangay Fiesta Committee in 2009, as electorate should be paramount. It is their voice, not ours or of anyone else, that must prevail.
well as his assumption of a leadership role in the socio-civic sphere of the locality as a member This, in essence, is the democracy we continue to hold sacred."99
of the advisory body of the Pinagtong-ulan, San Jose/Lipa City Chapter of the Guardians
Brotherhood Inc. , manifests a significant level of knowledge of and sensitivity to the needs of
the said community. Such, after all, is the rationale for the residency requirement in our
elections laws, to wit:
WHEREFORE, premises considered, the Petition is GRANTED. The assailed COMELEC
Resolutions dated 26 January 2010 and 17 August 2010 in Florencio Librea v. Meynardo A.
Sabili [SPA No. 09-047(DC)] are ANNULLED. Private respondent’s Petition to cancel the
Certificate of Candidacy of Meynardo A. Sabili is DENIED. The Status Quo Ante Order issued by
The Constitution and the law requires residence as a qualification for seeking and holding this Court on 7 September 2010 is MADE PERMANENT.
elective public office, in order to give candidates the opportunity to be familiar with the needs,
difficulties, aspirations, potentials for growth and all matters vital to the welfare of their
constituencies; likewise, it enables the electorate to evaluate the office seekers’ qualifications
and fitness for the job they aspire for xxx. 94
SO ORDERED.

Considering all of the foregoing discussion, it is clear that while separately, each evidence
Corona, CJ., Carpio, Velasco Jr., Leonardo-De Castro, Brion, Peralta, Bersamin, Del Castillo,
presented by petitioner might fail to convincingly show the fact of his residence at Pinagtong-
Abad, Villarama Jr., Perez, Mendoza, Reyes, Perlas-Bernabe. JJ. Concur.
ulan since 2007, collectively, these pieces of evidence tend to sufficiently establish the said
fact.

Commercial lists
Petitioner’s actual physical presence in Lipa City is established not only by the presence of a
place (Pinagtong-ulan house and lot) he can actually live in, but also the affidavits of various SPECIAL FIRST DIVISION
persons in Pinagtong-ulan, and the Certification of its barangay captain. Petitioner’s substantial
and real interest in establishing his domicile of choice in Lipa City is also sufficiently shown not
G.R. No. 127598 February 22, 2000
only by the acquisition of additional property in the area and the transfer of his voter
registration, but also his participation in the community’s socio-civic and religious life, as well
as his declaration in his ITR that he is a resident thereof. MANILA ELECTRIC COMPANY, petitioner,

vs. Hon. SECRETARY OF LABOR LEONARDO QUISUMBING and MERALCO


EMPLOYEES and WORKERS ASSOCIATION (MEWA), respondent.

We therefore rule that petitioner has been able to adduce substantial evidence to demonstrate
compliance with the one-year residency requirement for local elective officials under the law. RESOLUTION

YNARES-SANTIAGO, J.:

In view of this Court’s finding that petitioner has not misrepresented his residence at
Pinagtong-ulan and the duration thereof, there is no need to further discuss whether there
was material and deliberate misrepresentation of the residency qualification in his COC. Facts

* In the Decision promulgated on January 27, 1999, the Court disposed of the case as follows:

As a final note, we do not lose sight of the fact that Lipa City voters manifested their own WHEREFORE, the petition is granted and the orders of public respondent
judgment regarding the qualifications of petitioner when they voted for him, notwithstanding Secretary of Labor dated August 19, 1996 and December 28, 1996 are set aside
that the issue of his residency qualification had been raised prior to the elections. Petitioner to the extent set forth above. The parties are directed to execute a Collective
has garnered the highest number of votes (55,268 votes as opposed to the 48,825 votes in Bargaining Agreement incorporating the terms and conditions contained in the
favor of his opponent, Oscar Gozos)95 legally cast for the position of Mayor of Lipa City and unaffected portions of the Secretary of Labor's orders of August 19, 1996 and
has consequently been proclaimed duly elected municipal Mayor of Lipa City during the last December 28, 1996, and the modifications set forth above. The retirement fund
May 2010 elections96 issue is remanded to the Secretary of Labor for reception of evidence and
determination of the legal personality of the MERALCO retirement fund.1

* The modifications of the public respondent's resolutions include the following:


289
RECTO, GAYLE ANGELI M.
2011-0008 | AUSL
Personal Notes on Remedial Law 2 Review (based on the syllabus of Prof. Henedino M. Brondial)

January 27, 1999 decision Secretary's resolution Commercial lists and the like. — Evidence of statements of matters of
interest to persons engaged in an occupation contained in a list,
Wages - P1,900.00 for 1995-96 P2,200.00 register, periodical, or other published compilation is admissible as
tending to prove the truth of any relevant matter so stated if that
compilation is published for use by persons engaged in that occupation
X'mas bonus - modified to one month 2 months
and is generally used and relied upon by them therein.

Retirees - remanded to the Secretary granted

Loan to coops- denied granted


Under the afore-quoted rule, statement of matters contained in a periodical, may
be admitted only "if that compilation is published for use by persons engaged in
GHSIP, HMP and
that occupation and is generally used and relied upon by them therein." As
correctly held in our Decision dated January 27, 1999, the cited report is a mere
Housing loans- granted up to P60,000.00 granted newspaper account and not even a commercial list. At most, it is but an analysis or
opinion which carries no persuasive weight for purposes of this case as no
Signing bonus- denied granted sufficient figures to support it were presented. Neither did anybody testify to its
accuracy. It cannot be said that businessmen generally rely on news items such as
Union leave - 40 days (typo error) 30 days this in their occupation. Besides, no evidence was presented that the publication
was regularly prepared by a person in touch with the market and that it is
generally regarded as trustworthy and reliable. Absent extrinsic proof of their
High voltage/pole- not apply to those who are members of a team
accuracy, these reports are not admissible.6 In the same manner, newspapers
containing stock quotations are not admissible in evidence when the source of the
not exposed to the risk reports is available.7 With more reason, mere analyses or projections of such
reports cannot be admitted. In particular, the source of the report in this case can
Collectors - no need for cash bond, no be easily made available considering that the same is necessary for compliance
with certain governmental requirements.
need to reduce quota and MAPL

CBU - exclude confidential employees include


Nonetheless, by petitioner's own allegations, its actual total net income for 1996 was P5.1
Union security- maintenance of membership closed shop billion.8 An estimate by the All Asia financial analyst stated that petitioner's net operating
income for the same year was about P5.7 billion, a figure which the Union relies on to support
Contracting out- no need to consult union consult first its claim. Assuming without admitting the truth thereof, the figure is higher than the P4.171
billion allegedly suggested by petitioner as its projected net operating income. The P5.7 billion
which was the Secretary's basis for granting the P2,200.00 is higher than the actual net
All benefits - existing terms and conditions all terms
income of P5.1 billion admitted by petitioner. It would be proper then to increase this Court's
award of P1,900.00 to P2,000.00 for the two years of the CBA award. For 1992, the agreed
Retroactivity - Dec. 28, 1996-Dec. 27, 199(9) from Dec. 1, 1995 CBA wage increase for rank-and-file was P1,400.00 and was reduced to P1,350.00; for 1993;
further reduced to P1,150.00 for 1994. For supervisory employees, the agreed wage increase
* Dissatisfied with the Decision, some alleged members of private respondent union (Union for for the years 1992-1994 are P1,742.50, P1,682.50 and P1,442.50, respectively. Based on the
brevity) filed a motion for intervention and a motion for reconsideration of the said Decision. A foregoing figures, the P2,000.00 increase for the two-year period awarded to the rank-and-file
separate intervention was likewise made by the supervisor's union (FLAMES2) of petitioner is much higher than the highest increase granted to supervisory employees.9 As mentioned in
corporation alleging that it has bona fide legal interest in the outcome of the case.3 The Court the January 27, 1999 Decision, the Court does "not seek to enumerate in this decision the
required the "proper parties" to file a comment to the three motions for reconsideration but factors that should affect wage determination" because collective bargaining disputes
the Solicitor-General asked that he be excused from filing the comment because the "petition particularly those affecting the national interest and public service "requires due consideration
filed in the instant case was granted" by the Court.4 Consequently, petitioner filed its own and proper balancing of the interests of the parties to the dispute and of those who might be
consolidated comment. An "Appeal Seeking Immediate Reconsideration" was also filed by the affected by the dispute."10 The Court takes judicial notice that the new amounts granted
alleged newly elected president of the Union.5 Other subsequent pleadings were filed by the herein are significantly higher than the weighted average salary currently enjoyed by other
parties and intervenors. rank-and-file employees within the community. It should be noted that the relations between
labor and capital is impressed with public interest which must yield to the common good.11
Neither party should act oppressively against the other or impair the interest or convenience of
the public.12 Besides, matters of salary increases are part of management prerogative.13
Ruling

The issues raised in the motions for reconsideration had already been passed upon by the
Court in the January 27, 1999 decision. No new arguments were presented for consideration On the retroactivity of the CBA arbitral award, it is well to recall that this petition had its origin in
of the Court. Nonetheless, certain matters will be considered herein, particularly those the renegotiation of the parties' 1992-1997 CBA insofar as the last two-year period thereof is
involving the amount of wages and the retroactivity of the Collective Bargaining Agreement concerned. When the Secretary of Labor assumed jurisdiction and granted the arbitral
(CBA) arbitral awards. awards, there was no question that these arbitral awards were to be given retroactive effect.
However, the parties dispute the reckoning period when retroaction shall commence.
Petitioner claims that the award should retroact only from such time that the Secretary of
Labor rendered the award, invoking the 1995 decision in Pier 8 case14 where the Court, citing
Union of Filipino Employees v. NLRC,15 said:
Petitioner warns that if the wage increase of P2,200.00 per month as ordered by the Secretary
is allowed, it would simply pass the cost covering such increase to the consumers through an
increase in the rate of electricity. This is a non sequitur. The Court cannot be threatened with
such a misleading argument. An increase in the prices of electric current needs the approval of
the appropriate regulatory government agency and does not automatically result from a mere The assailed resolution which incorporated the CBA to be signed by the parties was
increase in the wages of petitioner's employees. Besides, this argument presupposes that promulgated on June 5, 1989, the expiry date of the past CBA. Based on the provision of
petitioner is capable of meeting a wage increase. The All Asia Capital report upon which the Section 253-A, its retroactivity should be agreed upon by the parties. But since no agreement
Union relies to support its position regarding the wage issue cannot be an accurate basis and to that effect was made, public respondent did not abuse its discretion in giving the said CBA a
conclusive determinant of the rate of wage increase. Section 45 of Rule 130 Rules of prospective effect. The action of the public respondent is within the ambit of its authority
Evidence provides: vested by existing law.

290
RECTO, GAYLE ANGELI M.
2011-0008 | AUSL
Personal Notes on Remedial Law 2 Review (based on the syllabus of Prof. Henedino M. Brondial)

On the other hand, the Union argues that the award should retroact to such time granted by in determining the duration of retroactivity. In this connection, the Court considers the letter of
the Secretary, citing the 1993 decision of St. Luke's.16 petitioner's Chairman of the Board and its President addressed to their stockholders, which
states that the CBA "for the rank-and-file employees covering the period December 1, 1995 to
November 30, 1997 is still with the Supreme Court,"20 as indicative of petitioner's recognition
that the CBA award covers the said period. Earlier, petitioner's negotiating panel transmitted to
the Union a copy of its proposed CBA covering the same period inclusive.21 In addition,
Finally, the effectivity of the Order of January 28, 1991, must retroact to the date of the
petitioner does not dispute the allegation that in the past CBA arbitral awards, the Secretary
expiration of the previous CBA, contrary to the position of petitioner. Under the circumstances
granted retroactivity commencing from the period immediately following the last day of the
of the case, Article 253-A cannot be properly applied to herein case. As correctly stated by
expired CBA. Thus, by petitioner's own actions, the Court sees no reason to retroact the
public respondent in his assailed Order of April 12, 1991 dismissing petitioner's Motion for
subject CBA awards to a different date. The period is herein set at two (2) years from
Reconsideration —
December 1, 1995 to November 30, 1997.

Anent the alleged lack of basis for the retroactivity provisions awarded; we would stress that
On the allegation concerning the grant of loan to a cooperative, there is no merit in the union's
the provision of law invoked by the Hospital, Article 253-A of the Labor Code, speaks of
claim that it is no different from housing loans granted by the employer. The award of loans
agreements by and between the parties, and not arbitral awards . . .
for housing is justified because it pertains to a basic necessity of life. It is part of a privilege
recognized by the employer and allowed by law. In contrast, providing seed money for the
establishment of the employee's cooperative is a matter in which the employer has no
business interest or legal obligation. Courts should not be utilized as a tool to compel any
Therefore, in the absence of a specific provision of law prohibiting retroactivity of the person to grant loans to another nor to force parties to undertake an obligation without
effectivity of arbitral awards issued by the Secretary of Labor pursuant to Article 263(g) of the justification. On the contrary, it is the government that has the obligation to render financial
Labor Code, such as herein involved, public respondent is deemed vested with plenary and assistance to cooperatives and the Cooperative Code does not make it an obligation of the
discretionary powers to determine the effectivity thereof. employer or any private individual.22

In the 1997 case of Mindanao Terminal,17 the Court applied the St. Luke's doctrine and ruled Anent the 40-day union leave, the Court finds that the same is a typographical error. In order
that: to avoid any confusion, it is herein declared that the union leave is only thirty (30) days as
granted by the Secretary of Labor and affirmed in the Decision of this Court.

In St. Luke's Medical Center v. Torres, a deadlock also developed during the CBA negotiations
between management and the union. The Secretary of Labor assumed jurisdiction and ordered The added requirement of consultation imposed by the Secretary in cases of contracting out
the retroaction of the CBA to the date of expiration of the previous CBA. As in this case, it was for six (6) months or more has been rejected by the Court. Suffice it to say that the employer
alleged that the Secretary of Labor gravely abused its discretion in making his award is allowed to contract out services for six months or more. However, a line must be drawn
retroactive. In dismissing this contention this Court held: between management prerogatives regarding business operations per se and those which
affect the rights of employees, and in treating the latter, the employer should see to it that its
employees are at least properly informed of its decision or modes of action in order to attain a
harmonious labor-management relationship and enlighten the workers concerning their
Therefore, in the absence of a specific provision of law prohibiting retroactive of the effectivity rights.23 Hiring of workers is within the employer's inherent freedom to regulate and is a valid
of arbitral awards issued by the Secretary of Labor pursuant to Article 263(g) of the Labor exercise of its management prerogative subject only to special laws and agreements on the
Code, such as herein involved, public respondent is deemed vested with plenary and matter and the fair standards of justice.24 The management cannot be denied the faculty of
discretionary powers to determine the effectivity thereof. promoting efficiency and attaining economy by a study of what units are essential for its
operation. It has the ultimate determination of whether services should be performed by its
personnel or contracted to outside agencies. While there should be mutual consultation,
eventually deference is to be paid to what management decides.25 Contracting out of services
is an exercise of business judgment or management prerogative.26 Absent proof that
The Court in the January 27, 1999 Decision, stated that the CBA shall be "effective for a period management acted in a malicious or arbitrary manner, the Court will not interfere with the
of 2 years counted from December 28, 1996 up to December 27, 1999." Parenthetically, this exercise of judgment by an employer.27 As mentioned in the January 27, 1999 Decision, the
actually covers a three-year period. Labor laws are silent as to when an arbitral award in a law already sufficiently regulates this matter.28 Jurisprudence also provides adequate
labor dispute where the Secretary had assumed jurisdiction by virtue of Article 263 (g) of the limitations, such that the employer must be motivated by good faith and the contracting out
Labor Code shall retroact. In general, a CBA negotiated within six months after the expiration should not be resorted to circumvent the law or must not have been the result of malicious or
of the existing CBA retroacts to the day immediately following such date and if agreed arbitrary actions.29 These are matters that may be categorically determined only when an
thereafter, the effectivity depends on the agreement of the parties.18 On the other hand, the actual suit on the matter arises.
law is silent as to the retroactivity of a CBA arbitral award or that granted not by virtue of the
mutual agreement of the parties but by intervention of the government. Despite the silence of
the law, the Court rules herein that CBA arbitral awards granted after six months from the
expiration of the last CBA shall retroact to such time agreed upon by both employer and the
employees or their union. Absent such an agreement as to retroactivity, the award shall WHEREFORE, the motion for reconsideration is PARTIALLY GRANTED and the assailed
retroact to the first day after the six-month period following the expiration of the last day of Decision is MODIFIED as follows: (1) the arbitral award shall retroact from December 1, 1995
the CBA should there be one. In the absence of a CBA, the Secretary's determination of the to November 30, 1997; and (2) the award of wage is increased from the original amount of
date of retroactivity as part of his discretionary powers over arbitral awards shall control. One Thousand Nine Hundred Pesos (P1,900.00) to Two Thousand Pesos (P2,000.00) for the
years 1995 and 1996. This Resolution is subject to the monetary advances granted by
petitioner to its rank-and-file employees during the pendency of this case assuming such
advances had actually been distributed to them. The assailed Decision is AFFIRMED in all other
respects..nêt
It is true that an arbitral award cannot per se be categorized as an agreement voluntarily
entered into by the parties because it requires the interference and imposing power of the
State thru the Secretary of Labor when he assumes jurisdiction. However, the arbitral award
can be considered as an approximation of a collective bargaining agreement which would
otherwise have been entered into by the parties.19 The terms or periods set forth in Article SO ORDERED.
253-A pertains explicitly to a CBA. But there is nothing that would prevent its application by
analogy to an arbitral award by the Secretary considering the absence of an applicable law. Davide, Jr., C.J., Melo, Kapunan and Pardo, JJ., concur.
Under Article 253-A: "(I)f any such agreement is entered into beyond six months, the parties
shall agree on the duration of retroactivity thereof." In other words, the law contemplates
retroactivity whether the agreement be entered into before or after the said six-month period.
The agreement of the parties need not be categorically stated for their acts may be considered
291
RECTO, GAYLE ANGELI M.
2011-0008 | AUSL
Personal Notes on Remedial Law 2 Review (based on the syllabus of Prof. Henedino M. Brondial)

Testimony or deposition at a former proceeding her name. 8 Appellant issued Marasigan a photocopy of her plane ticket, 9 the
original of which was promised to be given to her before her departure. 10

SECOND DIVISION
* Marasigan was never issued a visa. 11 Neither was she given the promised
plane ticket. Unable to depart for Taiwan, she went to the travel agency which
G.R. Nos. 115338-39 September 16, 1997 issued the ticket and was informed that not only was she not booked by
appellant for the alleged flight, but that the staff in the agency did not even
PEOPLE OF THE PHILIPPINES, plaintiff-appellee, know appellant.

vs. LANIE ORTIZ-MIYAKE, accused-appellant. * Later, Marasigan proceeded to the supposed residence of appellant and was
informed that appellant did not live there. 12 Upon verification with the Philippine
REGALADO, J.: Overseas Employment Administration (POEA), it was revealed that appellant was
not authorized to recruit workers for overseas employment. 13 Marasigan wanted
to recover her money but, by then, appellant could no longer be located.

* The prosecution sought to prove that Generillo and Del Rosario, the two other complainants in
Facts the illegal recruitment case, were also victimized by appellant. In lieu of their testimonies, the
prosecution presented as witnesses Lilia Generillo, the mother of Imelda Generillo, and
* Accused-appellant Lanie Ortiz-Miyake was charged with illegal recruitment in large scale in Victoria Amin, the sister of Del Rosario.
the Regional Trial Court of Makati on a complaint initiated by Elenita Marasigan, Imelda
Generillo and Rosamar del Rosario. In addition, she was indicted for estafa by means of false * Lilia Generillo claimed that she gave her daughter P8,000.00 to cover her
pretenses in the same court, the offended party being Elenita Marasigan alone. application for placement abroad which was made through appellant. 14 Twice,
she accompanied her daughter to the residence of appellant so that she could
* The information in the charge of illegal recruitment in large scale in Criminal Case No. 92- meet her; however, she was not involved in the transactions between her
6153 reads as follows: daughter and appellant. 15 Neither was she around when payments were made
to appellant. Imelda Generillo was unable to leave for abroad and Lilia Generillo
That in or about the period comprised from June 1992 to August 1992, in the concluded that she had become a victim of illegal recruitment.
Municipality of Parañaque, Metro Manila, Philippines and within the jurisdiction of
this Honorable Court, the above-named accused, falsely representing herself to * The prosecution presented Victoria Amin, the sister of Rosamar Del Rosario, to
have the capacity and power to contract, enlist and recruit workers for show that the latter was also a victim of illegal recruitment. Victoria Amin
employment abroad did then and there willfully, unlawfully, and feloniously testified that appellant was supposed to provide her sister a job abroad. She
collect for a fee, recruit and promise employment/job placement abroad to the claimed that she gave her sister a total of P10,000.00 which was intended to
following persons, to wit: 1) Rosamar del Rosario; 2) Elenita Marasigan; 3) cover the latter's processing fee. 16
Imelda Generillo, without first securing the required license or authority from the
Department of Labor and Employment, thus amounting to illegal recruitment in * Victoria Amin never met appellant and was not around when her sister made
large scale, in violation of the aforecited law. 1 payments. She assumed that the money was paid to appellant based on receipts,
allegedly issued by appellant, which her sister showed her. 17 Del Rosario was
* The information in the charge for estafa in Criminal Case No. 92-6154 alleges: unable to leave for abroad despite the representations of appellant. Victoria Amin
claimed that her sister, like Marasigan and Generillo, was a victim of illegal
That in or about or sometime in the month of August, 1992, in the Municipality of recruitment.
Parañaque, Metro Manila, Philippines and within the jurisdiction of this Honorable
Court, the above-named accused, by means of false pretenses executed prior to * The final witness for the prosecution was Riza Balberte, 18 a representative of
or simultaneously with the commission of the fraud, falsely pretending to have the POEA, who testified that appellant was neither licensed nor authorized to
the capacity and power to send complainant Elenita Marasigan to work abroad, recruit workers for overseas employment, POEA certificate certification. 19
succeeded in inducing the latter to give and deliver to her the total sum of
P23,000.00, the accused knowing fully well that the said manifestations and * Upon the foregoing evidence, the prosecution sought to prove that although
representation are false and fraudulent and calculated only to deceive the said two of the three complainants in the illegal recruitment case were unable to
complainant to part with her money, and, once in possession thereof, the said testify, appellant was guilty of committing the offense against all three
accused did then and there willfully, unlawfully and feloniously appropriate, apply complainants and, therefore, should be convicted as charged.
and convert the same to her own personal use and benefit, to the damage and
prejudice of the said Elenita Marasigan, in the aforementioned amount of
* On the other hand, appellant, who was the sole witness for the defense, denied that she
P23,000.00. 2
recruited the complainants for overseas employment and claimed that the payments made to
her were solely for purchasing plane tickets at a discounted rate as she had connections with a
* Upon arraignment, appellant pleaded not guilty to the charges and the cases were tried travel agency. 20
jointly in Branch 145 of the Regional Trial Court of Makati.

* She denied that she was paid by Marasigan the amount of P23,000.00,
* Of the three complainants in the case for illegal recruitment in large scale, Marasigan was claiming that she was paid only P8,000.00, as shown by a receipt. She further
the only one who testified at the trial. The two other complainants, Generillo and Del Rosario, insisted that, through the travel agency, 21 she was able to purchase discounted
were unable to testify as they were then abroad. plane tickets for the complainants upon partial payment of the ticket prices, the
balance of which she guaranteed. According to her, the complainants were
* Marasigan testified that she was a 32 year-old unmarried sales representative in supposed to pay her the balance but because they failed to do so, she was
1992 when she was introduced to appellant by her co-complainants. 3 obliged to pay the entire cost of each ticket.
Appellant promised Marasigan a job as a factory worker in Taiwan for a
P5,000.00 fee. At that time, Marasigan had a pending application for overseas * The evidence presented by the parties were thus contradictory but the trial court found the
employment pending in a recruitment agency. Realizing that the fee charged by prosecution's evidence more credible. On December 17, 1993, judgment was rendered by said
appellant was much lower than that of the agency, Marasigan withdrew her court convicting appellant of both crimes as charged. 22
money from the agency and gave it to appellant. 4

* In convicting appellant of illegal recruitment in large scale, the lower court adopted a
( Marasigan paid appellant P5,000.00, but she was later required to make previous decision of Branch 78 of the Metropolitan Trial Court of Parañaque as a basis for the
additional payments. By the middle of the year, she had paid a total of judgment. Said previous decision was a conviction for estafa promulgated on July 26, 1993, 23
P23,000.00 on installment basis. 5 Save for two receipts, 6 Marasigan was not rendered in Criminal Cases Nos. 74852-53, involving the same circumstances in the instant
issued receipts for the foregoing payments despite her persistence in requesting case, wherein complainants Generillo and Del Rosario charged appellant with two counts of
for the same. estafa. This decision was not appealed and had become final and executory.

* Marasigan was assured by appellant that obtaining a Taiwanese visa would not * In thus convicting appellant in the illegal recruitment case, the decision therein
be a problem. 7 She was also shown a plane ticket to Taiwan, allegedly issued in of the Regional Trial Court stated that the facts in the foregoing estafa cases
292
RECTO, GAYLE ANGELI M.
2011-0008 | AUSL
Personal Notes on Remedial Law 2 Review (based on the syllabus of Prof. Henedino M. Brondial)

were the same as those in the illegal recruitment case before it. It, therefore,
adopted the facts and conclusions established in the earlier decision as its own
findings of facts and as its retionale for the conviction in the case before it. 24 . . . Illegal recruitment is deemed committed in large scale if committed against three (3) or
more persons individually or as a group.
* In Criminal Case No. 92-6153, the Makati court sentenced appellant to serve
the penalty of life imprisonment for illegal recruitment in large scale, as well as to
pay a fine of P100,000.00. Appellant was also ordered to reimburse the
complainants the following payments made to her, viz.: (a) Marasigan,
Art. 39. Penalties. —
P23,000.00; (b) Generillo, P2,500.00; and (c) Del Rosario, P2,500.00.

* In the same judgment and for the estafa charged in Criminal Case No. 92-
6154, the Makati court sentenced appellant to suffer imprisonment of four (4)
(a) The penalty of life imprisonment and a fine of One Hundred Thousand Pesos
years and two (2) months of prision correccional, as minimum, to eight (8) years of
(P100,000.00) shall be imposed if Illegal Recruitment constitutes economic sabotage as
prision mayor, as maximum, and to pay the costs.
defined herein;

* In the instant petition, appellant seeks the reversal of the foregoing judgment of the
Regional Trial Court of Makati convicting her of illegal recruitment in large scale and estafa.
Specifically, she insists that the trial court erred in convicting her of illegal recruitment in large
scale as the evidence presented was insufficient. xxx xxx xxx

* Moreover, appellant claims that she is not guilty of acts constituting illegal
recruitment, in large scale or otherwise, because contrary to the findings of the
trial court, she did not recruit the complainants but merely purchased plane (c) Any person who is neither a licensee nor a holder of authority under this Title found
tickets for them. Finally, she contends that in convicting her of estafa, the lower violating any provision thereof or its implementing rules and regulations shall, upon conviction
court erred as she did not misappropriate the money paid to her by Marasigan, thereof, suffer the penalty of imprisonment of not less than four (4) years nor more than eight
hence there was no damage to the complainants which would substantiate the (8) years or a fine of not less than P20,000.00 nor more than P100,000.00, or both such
conviction. imprisonment and fine, at the discretion of the court ...... 26

Ruling During the pendency of this case, Republic Act No. 8042, otherwise known as the "Migrant
Workers and Overseas Filipinos Act of 1995," was passed increasing the penalty for illegal
We uphold the finding that appellant is guilty but we are, compelled to modify the judgment recruitment. This new law, however, does not apply to the instant case because the offense
for the offenses she should be convicted of and the corresponding penalties therefor. charged herein was committed in 1992, before the effectivity of said Republic Act No. 8042.
Hence, what are applicable are the aforecited Labor Code provisions.

Appellant maintains that her conviction for illegal recruitment in large scale is erroneous. It is
her view that in the prosecution of a case for such offense, at least three complainants are It is evident that in illegal recruitment cases, the number of persons victimized is
required to appear as witnesses in the trial and, since Marasigan was the only complainant determinative. Where illegal recruitment is committed against a lone victim, the accused may
presented as a witness, the conviction was groundless. be convicted of simple illegal recruitment which is punishable with a lower penalty under
Article 39(c) of the Labor Code. Corollarily, where the offense is committed against three or
more persons, it is qualified to illegal recruitment in large scale which provides a higher
penalty under Article 39(a) of the same Code.

The Solicitor General also advocates the conviction of appellant for simple illegal recruitment
which provides a lower penalty. The Court finds the arguments of the Solicitor General
meritorious and adopts his position.
The position of the Solicitor General is that the conviction of appellant should be merely for the
lesser offense of simple illegal recruitment. He submits that the Regional Trial Court of Makati
erred in convicting appellant of illegal recruitment in large scale because the conviction was
based on an earlier decision of the Metropolitan Trial Court of Parañaque where appellant was
The Labor Code defines recruitment and placement as ". . . any act of canvassing, enlisting,
found guilty of estafa committed against Generillo and Del Rosario.
contracting transporting, utilizing, hiring or procuring workers and includes referrals, contract
services, promising or advertising for employment, locally or abroad, whether for profit or not .
. . ." 25

It is argued that the Makati court could not validly adopt the facts embodied in the decision of
the Parañaque court to show that illegal recruitment was committed against Generillo and Del
Rosario as well. Illegal recruitment was allegedly proven to have been committed against only
Illegal recruitment is likewise defined and made punishable under the Labor Code, thus:
one person, particularly, Elenita Marasigan. Appellant, therefore, may only be held guilty of
simple illegal recruitment and not of such offense in large scale.

Art. 38. Illegal Recruitment. —

He further submits that the adoption by the Makati court of the facts in the
decision of the Parañaque court for estafa to constitute the basis of the subsequent
conviction for illegal recruitment is erroneous as it is a violation of the right of
(a) Any recruitment activities, including the prohibited practices enumerated under Article 34 appellant to confront the witnesses, that is, complainants Generillo and Del
of this Code, to be undertaken by non-licensees or non-holders of authority shall be deemed Rosario, during trial before it. He cites the pertinent provision of Rule 115 of the
illegal and punishable under Article 39 of this Code ...... Rules of Court, to wit:

(b) Illegal recruitment when committed by a syndicate or in large scale shall be considered an Sec. 1. Rights of accused at the trial. In all criminal prosecutions, the
offense involving economic sabotage and shall be penalized in accordance with Article 39 accused shall be entitled:
hereof.

293
RECTO, GAYLE ANGELI M.
2011-0008 | AUSL
Personal Notes on Remedial Law 2 Review (based on the syllabus of Prof. Henedino M. Brondial)

xxx xxx xxx As earlier stated, the Makati court's utilization of and reliance on the previous
decision of the Parañaque court must be rejected. Every conviction must be based
on the findings of fact made by a trial court according to its appreciation of the
evidence before it. A conviction may not be based merely on the findings of fact of
another court, especially where what is presented is only its decision sans the
(f) To confront and cross-examine the witnesses against him at the
transcript of the testimony of the witnesses who testified therein and upon which
trial. Either party may utilize as part of its evidence the testimony of a
the decision is based.
witness who is deceased, out of or cannot, with due diligence be found
in the Philippines, unavailable or otherwise unable testify, given in
another case or proceeding, judicial or administrative, involving the
same parties and subject matter, the adverse party having had the
opportunity to cross-examine him. Furthermore, this is not the only reason why appellant may not be held liable for
illegal recruitment in large scale. An evaluation of the evidence presented before
the trial court shows us that, apart from the adopted decision in the previous
estafa case, there was no other basis for said trial court's conclusion that illegal
recruitment in large scale was committed against all three complainants.
xxx xxx xxx

The distinction between simple illegal recruitment and illegal recruitment in large scale are
It will be noted that the principle embodied in the foregoing rule is likewise found
emphasized by jurisprudence. Simple illegal recruitment is committed where a person: (a)
in the following provision of Rule 130:
undertakes any recruitment activity defined under Article 13(b) or any prohibited practice
enumerated under Articles 34 and 38 of the Labor Code; and (b) does not have a license or
authority to lawfully engage in the recruitment and placement of workers. 31 On the other
hand, illegal recruitment in large scale further requires a third element, that is, the offense is
Sec. 47. Testimony or deposition at a former proceeding. — The
committed against three or more persons, individually or as a group. 32
testimony or deposition of a witness deceased or unable to testify,
given in a former case or proceeding, judicial or administrative,
involving the same parties and subject matter, may be given in
evidence against the adverse party who had the opportunity to cross-
In illegal recruitment in large scale, while the law does not require that at least three victims
examine him.
testify at the trial, it is necessary that there is sufficient evidence proving that the offense was
committed against three or more persons. This Court agrees with the trial court that the
evidence presented sufficiently proves that illegal recruitment was committed by appellant
against Marasigan, but the same conclusion cannot be made as regards Generillo and Del
Under the aforecited rules, the accused in a criminal case is guaranteed the right of Rosario as well.
confrontation. Such right has two purposes: first, to secure the opportunity of
cross-examination; and, second, to allow the judge to observe the deportment and
appearance of the witness while testifying. 27

The testimonies of Generillo's mother, Lilia Generillo, and Del Rosario's sister, Victoria Amin,
reveal that these witnesses had no personal knowledge of the actual circumstances
surrounding the charges filed by Generillo and Del Rosario for illegal recruitment in large scale.
This right, however, is not absolute as it is recognized that it is sometimes Neither of these witnesses was privy to the transactions between appellant and each of the
impossible to recall or produce a witness who has already testified in a previous two complainants. The witnesses claimed that appellant illegally recruited Generillo and Del
proceeding, in which event his previous testimony is made admissible as a distinct Rosario. Nonetheless, we find their averments to be unfounded as they were not even present
piece of evidence, by way of exception to the hearsay rule. 28 The previous when Generillo and Del Rosario negotiated with and made payments to appellant.
testimony is made admissible because it makes the administration of justice
orderly and expeditious. 29

For insufficiency of evidence and in the absence of the third element of illegal recruitment in
large scale, particularly, that "the offense is committed against three or more persons," we
Under these rules, the adoption by the Makati trial court of the facts stated in the cannot affirm the conviction for illegal recruitment in large scale. Nonetheless, we agree with
decision of the Parañaque trial court does not fall under the exception to the right the finding of the trial court that appellant illegally recruited Marasigan, for which she must be
of confrontation as the exception contemplated by law covers only the utilization held liable for the lesser offense of simple illegal recruitment.
of testimonies of absent witnesses made in previous proceedings, and does not
include utilization of previous decisions or judgments.

Appellant's defense that she did not recruit Marasigan but merely purchased a plane ticket for
her is belied by the evidence as it is undeniable that she represented to Marasigan that she
In the instant case, the prosecution did not offer the testimonies made by had the ability to send people to work as factory workers in Taiwan. Her pretext that the fees
complainants Generillo and Del Rosario in the previous estafa case. Instead, what paid to her were merely payments for a plane ticket is a desperate attempt to exonerate
was offered, admitted in evidence, and utilized as a basis for the conviction in the herself from the charges and cannot be sustained.
case for illegal recruitment in large scale was the previous decision in the estafa
case.

Furthermore, no improper motive may be attributed to Marasigan in charging appellant. The


fact that Marasigan was poor does not make her so heartless as to contrive a criminal charge
A previous decision or judgment, while admissible in evidence, may only prove that against appellant. She was a simple woman with big dreams and it was appellant's duplicity
an accused was previously convicted of a crime. 30 It may not be used to prove which reduced those dreams to naught. Marasigan had no motive to testify falsely against
that the accused is guilty of a crime charged in a subsequent case, in lieu of the appellant except to tell the truth. 33
requisite evidence proving the commission of the crime, as said previous decision
is hearsay. To sanction its being used as a basis for conviction in a subsequent case
would constitute a violation of the right of the accused to confront the witnesses
against him.
Besides, if there was anyone whose testimony needed corroboration, it was appellant as there
was nothing in her testimony except the bare denial of the accusations. 34 If appellant really
294
RECTO, GAYLE ANGELI M.
2011-0008 | AUSL
Personal Notes on Remedial Law 2 Review (based on the syllabus of Prof. Henedino M. Brondial)

intended to purchase a plane ticket and not to recruit Marasigan, she should have presented WHEREFORE, the judgment of the court a quo finding accused-appellant Lanie Ortiz-Miyake
evidence to support this claim. Also, in her testimony, appellant named an employee in the guilty beyond reasonable doubt of the crimes of illegal recruitment in large scale (Criminal
travel agency who was allegedly her contact person for the purchase of the ticket. She could Case No. 92-6153) and estafa (Criminal Case No. 92-6154) is hereby MODIFIED, as follows.
have presented that person, or some other employee of the agency, to show that the
transaction was merely for buying a ticket. Her failure to do the foregoing acts belies her
pretensions.
1) Accused-appellant is declared guilty beyond reasonable doubt of simple illegal recruitment,
as defined in Article 38(a) of the Labor Code, as amended. She is hereby ordered to serve an
indeterminate sentence of four (4) years, as minimum, to eight (8) years, as maximum, and to
The Court likewise affirms the conviction of appellant for estafa which was committed against pay a fine of P100,000.00.
Marasigan. Conviction under the Labor Code for illegal recruitment does not preclude
punishment under the Revised Penal Code for the felony of estafa. 35 This Court is convinced
that the prosecution proved beyond reasonable doubt that appellant violated Article 315(2) (a)
of the Revised Penal Code which provides that estafa is committed:
2) In Criminal Case No. 92-6154 for estafa, herein accused-appellant is ordered to serve an
indeterminate sentence of four (4) years and two (2) months of prision correccional, as
minimum, to eight (8) years of prision mayor, as maximum, and to reimburse Elenita
Marasigan the sum of P23,000.00.
2. By means of any of the following false pretenses or fraudulent acts executed prior to or
simultaneously with the commission of the fraud:

In all other respects, the aforestated judgment is AFFIRMED, with costs against accused-
appellant in both instances.
(a) By using fictitious name or falsely pretending to possess power, influence, qualifications,
property, credit, agency, business or imaginary transactions, or by means of other similar
deceits.
SO ORDERED.

The evidence is clear that in falsely pretending to possess power to deploy persons for
overseas placement, appellant deceived the complainant into believing that she would provide
Puno, Mendoza and Torres, Jr., JJ., concur.
her a job in Taiwan. Her assurances made Marasigan exhaust whatever resources she had to
pay the placement fee required in exchange for the promised job. The elements of deceit and
damage for this form of estafa are indisputably present, hence the conviction for estafa in
Criminal Case No. 92-6154 should be affirmed.
THIRD DIVISION

G.R. No. 185527 July 18, 2012


Under the Revised Penal Code, an accused found guilty of estafa shall be sentenced to:
HARRY L. GO, TONNY NGO, JERRY NGO AND JANE GO, Petitioners,

vs. THE PEOPLE OF THE PHILIPPINES and HIGHDONE COMPANY, LTD., ET AL.,
. . . The penalty of prision correccional in its maximum period to prision mayor in its minimum Respondents.
period, if the amount of the fraud is over 12,000 but does not exceed 22,000 pesos, and if
such amount exceeds the latter sum, the penalty provided in this paragraph shall be imposed PERLAS-BERNABE, J.:
in its maximum period, adding one year for each additional 10,000 pesos ..... 36

Facts
The amount involved in the estafa case is P23,000.00. Applying the Indeterminate Sentence
Law, the maximum penalty shall be taken from the maximum period of the foregoing basic
* Petitioners Harry Go, Tonny Ngo, Jerry Ngo and Jane Go were charged before the
penalty, specifically, within the range of imprisonment from six (6) years, eight (8) months and
Metropolitan Trial Court (MeTC) of Manila for Other Deceits under Article 318 of the Revised
twenty-one (21) days to eight (8) years.
Penal Code (RPC) docketed as Criminal Case No. 396447. The Information4 dated September
24, 2003, later amended5 on September 14, 2004, reads:

"That sometime in August 1996, in the City of Manila, Philippines, the said
On the other hand, the minimum penalty of the indeterminate sentence shall be within the accused, conspiring, confederating together and helping one another, did then
range of the penalty next lower in degree to that provided by law, without considering the and there willfully, unlawfully and feloniously defraud Highdone Company Ltd.
incremental penalty for the amount in excess of P22,000.00. 37 That penalty immediately Represented by Li Luen Ping, in the following manner, to wit: all said accused, by
lower in degree is prison correccional in its minimum and medium periods, with a duration of means of false manifestations and fraudulent representations which they made
six (6) months and one (1) day to four (4) years and two (2) months. On these considerations, to said Li Luen Ping to the effect that they have chattels such as machinery,
the trial court correctly fixed the minimum and maximum terms of the indeterminate sentence spare parts, equipment and raw materials installed and fixed in the premises of
in the estafa case. BGB Industrial Textile Mills Factory located in the Bataan Export Processing Zone
(BEPZ) in Mariveles, Bataan, executed a Deed of Mortgage for a consideration of
the amount of $464,266.90 or its peso equivalent at P20,892,010.50 more or less
in favor of ML Resources and Highdone Company Ltd. Representing that the said
While we must be vigilant and should punish, to the fullest extent of the law, those who prey deed is a FIRST MORTGAGE when in truth and in fact the accused well knew that
upon the desperate with empty promises of better lives, only to feed on their aspirations, we the same had been previously encumbered, mortgaged and foreclosed by CHINA
must not be heedless of the basic rule that a conviction may be sustained only where it is for BANK CORPORATION as early as September 1994 thereby causing damage and
the correct offense and the burden of proof of the guilt of the accused has been met by the prejudice to said HIGHDONE COMPANY LTD., in the said amount of $464,266.90
prosecution. or its peso equivalent at P20,892,010.50 more or less."

* Upon arraignment, petitioners pleaded not guilty to the charge.

295
RECTO, GAYLE ANGELI M.
2011-0008 | AUSL
Personal Notes on Remedial Law 2 Review (based on the syllabus of Prof. Henedino M. Brondial)

* The prosecution's complaining witness, Li Luen Ping, a frail old businessman from Laos,
Cambodia, traveled from his home country back to the Philippines in order to attend the
hearing held on September 9, 2004. However, trial dates were subsequently postponed due to The examination of witnesses must be done orally before a judge in open court.13
his unavailability. This is true especially in criminal cases where the Constitution secures to the
accused his right to a public trial and to meet the witnessess against him face to
* On October 13, 2005, the private prosecutor filed with the MeTC a Motion to Take Oral face. The requirement is the "safest and most satisfactory method of investigating
Deposition6 of Li Luen Ping, alleging that he was being treated for lung infection at the facts" as it enables the judge to test the witness' credibility through his manner
Cambodia Charity Hospital in Laos, Cambodia and that, upon doctor's advice, he could not and deportment while testifying.14 It is not without exceptions, however, as the
make the long travel to the Philippines by reason of ill health. Rules of Court recognizes the conditional examination of witnesses and the use of
their depositions as testimonial evidence in lieu of direct court testimony.
* Notwithstanding petitioners' Opposition,7 the MeTC granted8 the motion after the
prosecution complied with the directive to submit a Medical Certificate of Li Luen Ping.
Petitioners sought its reconsideration which the MeTC denied,9 prompting petitioners to file a
Petition for Certiorari10 before the RTC. Even in criminal proceedings, there is no doubt as to the availability of conditional
examination of witnesses - both for the benefit of the defense, as well as the
* On September 12, 2006, the RTC granted the petition and declared the MeTC Orders null prosecution. The Court's ruling in the case of Vda. de Manguerra v. Risos15
and void.11 The RTC held that Section 17, Rule 23 on the taking of depositions of witnesses in explicitly states that -
civil cases cannot apply suppletorily to the case since there is a specific provision in the Rules
of Court with respect to the taking of depositions of prosecution witnesses in criminal cases,
which is primarily intended to safeguard the constitutional rights of t he accused to meet the
witness against him face to face.
"x x x As exceptions, Rule 23 to 28 of the Rules of Court provide for the
different modes of discovery that may be resorted to by a party to an
* Upon denial by the RTC of their motion for reconsideration through an Order dated March 5, action. These rules are adopted either to perpetuate the testimonies of
2006,12 the prosecution elevated the case to the CA. witnesses or as modes of discovery. In criminal proceedings, Sections
12, 13 and 15, Rule 119 of the Revised Rules of Criminal Procedure,
* On February 19, 2008, the CA promulgated the assailed Decision which held that no grave which took effect on December 1, 2000, allow the conditional
abuse of discretion can be imputed upon the MeTC for allowing the deposition-taking of the examination of both the defense and prosecution witnesses."
complaining witness Li Luen Ping because no rule of procedure expressly disallows the taking (Underscoring supplied)16
of depositions in criminal cases and that, in any case, petitioners would still have every
opportunity to cross-examine the complaining witness and make timely objections during the
taking of the oral deposition either through counsel or through the consular officer who would
be taking the deposition of the witness.
The procedure under Rule 23 to 28 of the Rules of Court allows the taking of
depositions in civil cases, either upon oral examination or written interrogatories,
* On November 28, 2008, the CA denied petitioners' motion for reconsideration. before any judge, notary public or person authorized to administer oaths at any
time or place within the Philippines; or before any Philippine consular official,
* In this Petition for Review on Certiorari under Rule 45 of the Revised Rules of Court, commissioned officer or person authorized to administer oaths in a foreign state or
petitioners seek to nullify and set aside the February 19, 2008 Decision1 and November 28, country, with no additional requirement except reasonable notice in writing to the
2008 Resolution2 of the Court of Appeals (CA) in CA-G.R. SP No. 99383, which reversed the other party.17
September 12, 2006 Order3 issued by the Regional Trial Court (RTC) of Manila, Branch 27 in
Civil Case No. 06-114844 and upheld the grant of the prosecution’s motion to take the
testimony of a witness by oral depositions in Laos, Cambodia.
But for purposes of taking the deposition in criminal cases, more particularly of a
prosecution witness who would forseeably be unavailable for trial, the testimonial
examination should be made before the court, or at least before the judge, where
Issues the case is pending as required by the clear mandate of Section 15, Rule 119 of the
Revised Rules of Criminal Procedure. The pertinent provision reads thus:
I.THE COURT OF APPEALS ERRED IN NOT FINDING THAT THE METROPOLITAN TRIAL COURT
INFRINGED THE CONSTITUTIONAL RIGHT OF THE PETITIONERS TO A PUBLIC TRIAL IN
ALLOWING THE TAKING OF THE DEPOSITION OF THE COMPLAINING WITNESS IN LAOS,
CAMBODIA. SEC. 15. Examination of witness for the prosecution. - When it
satisfactorily appears that a witness for the prosecution is too sick or
II.THE COURT OF APPEALS ERRED IN NOT FINDING THAT THE DEPOSITION TAKING OF THE infirm to appear at the trial as directed by the court, or has to leave the
COMPLAINING WITNESS IN LAOS, CAMBODIA IS AN INFRINGEMENT OF THE Philippines with no definite date of returning, he may forthwith be
CONSTITUTIONAL RIGHT OF THE PETITIONERS TO CONFRONT THE SAID WITNESS FACE TO conditionally examined before the court where the case is pending.
FACE. Such examination, in the presence of the accused, or in his absence
after reasonable notice to attend the examination has been served on
III.THE COURT OF APPEALS ERRED IN SUSTAINING THE JUDICIAL LEGISLATION him shall be conducted in the same manner as an examination at the
COMMITTED BY THE METROPOLITAN TRIAL COURT IN APPLYING THE RULES ON trial. Failure or refusal of the accused to attend the examination after
DEPOSITION-TAKING IN CIVIL CASES TO CRIMINAL CASES. notice shall be considered a waiver. The statement taken may be
admitted in behalf of or against the accused.
IV.THE COURT OF APPEALS ERRED IN LIMITING THE TRADITIONAL DEFINITION OF GRAVE
ABUSE OF DISCRETION, OVERLOOKING THE ESTABLISHED RULE THAT VIOLATION OF THE
CONSTITUTION, THE LAW OR JURISPRUDENCE SIMILARLY COMES WITHIN THE PURVIEW
OF GRAVE ABUSE OF DISCRETION. Since the conditional examination of a prosecution witness must take place at no
other place than the court where the case is pending, the RTC properly nullified the
MeTC's orders granting the motion to take the deposition of Li Luen Ping before the
Philippine consular official in Laos, Cambodia. We quote with approval the RTC's
ratiocination in this wise:
Ruling

We rule in favor of petitioners.

The condition of the private complainant being sick and of advanced age falls
within the provision of Section 15 Rule 119 of the Rules of Court. However, said
rule substantially provides that he should be conditionally examined before the
The Procedure for Testimonial Examination of an Unavailable Prosecution Witness is Covered court where the case is pending. Thus, this Court concludes that the language of
Under Section 15, Rule 119.
296
RECTO, GAYLE ANGELI M.
2011-0008 | AUSL
Personal Notes on Remedial Law 2 Review (based on the syllabus of Prof. Henedino M. Brondial)

Section 15 Rule 119 must be interpreted to require the parties to present


testimony at the hearing through live witnesses, whose demeanor and credibility
can be evaluated by the judge presiding at the hearing, rather than by means of (2) In all criminal prosecutions, the accused shall be presumed innocent until the contrary is
deposition. No where in the said rule permits the taking of deposition outside the proved, and shall enjoy the right to be heard by himself and counsel, to be informed of the
Philippines whether the deponent is sick or not.18 (Underscoring supplied) nature and cause of the accusation against him, to have a speedy, impartial and public trial, to
meet the witnesses face to face, and to have compulsory process to secure the attendance of
witnesses and the production of evidence in his behalf. However, after arraignment, trial may
proceed notwithstanding the absence of the accused provided that he has been duly notified
Certainly, to take the deposition of the prosecution witness elsewhere and not and his failure to appear is unjustifiable. (Underscoring supplied)
before the very same court where the case is pending would not only deprive a
detained accused of his right to attend the proceedings but also deprive the trial
judge of the opportunity to observe the prosecution witness' deportment and
properly assess his credibility, which is especially intolerable when the witness' In dismissing petitioners' apprehensions concerning the deprivation of their constitutional
testimony is crucial to the prosecution's case against the accused. This is the rights to a public trial and confrontation, the CA opined that petitioners would still be accorded
import of the Court's ruling in Vda. de Manguerra19 where we further declared that the right to cross-examine the deponent witness and raise their objections during the
- deposition-taking in the same manner as in a regular court trial.

While we recognize the prosecution's right to preserve the testimony of We disagree. There is a great deal of difference between the face-to- face confrontation in a
its witness in order to prove its case, we cannot disregard the rules public criminal trial in the presence of the presiding judge and the cross-examination of a
which are designed mainly for the protection of the accused's witness in a foreign place outside the courtroom in the absence of a trial judge. In the aptly
constitutional rights. The giving of testimony during trial is the general cited case of People v. Estenzo,21 the Court noted the uniqueness and significance of a
rule. The conditional examination of a witness outside of the trial is witness testifying in open court, thus:
only an exception, and as such, calls for a strict construction of the
rules.20 (Underscoring supplied)

"The main and essential purpose of requiring a witness to appear and testify orally at a trial is
to secure for the adverse party the opportunity of cross-examination. "The opponent",
It is argued that since the Rules of Civil Procedure is made explicitly applicable in according to an eminent authority, "demands confrontation, not for the idle purpose of gazing
all cases, both civil and criminal as well as special proceedings, the deposition- upon the witness, or of being gazed upon by him, but for the purpose of cross examination
taking before a Philippine consular official under Rule 23 should be deemed
which cannot be had except by the direct and personal putting of questions and obtaining
allowable also under the circumstances. immediate answers." There is also the advantage of the witness before the judge, and it is this
- it enables the judge as trier of facts "to obtain the elusive and incommunicable evidence of a
witness' deportment while testifying, and a certain subjective moral effect is produced upon
the witness. It is only when the witness testifies orally that the judge may have a true idea of
However, the suggested suppletory application of Rule 23 in the testimonial his countenance, manner and expression, which may confirm or detract from the weight of his
examination of an unavailable prosecution witness has been categorically ruled out testimony. Certainly, the physical condition of the witness will reveal his capacity for accurate
by the Court in the same case of Vda. de Manguerra, as follows: observation and memory, and his deportment and physiognomy will reveal clues to his
character. These can only be observed by the judge if the witness testifies orally in court. x x
x"22 (Underscoring supplied)

It is true that Section 3, Rule 1 of the Rules of Court provides that the
rules of civil procedure apply to all actions, civil or criminal, and special
proceedings. In effect, it says that the rules of civil procedure have The right of confrontation, on the other hand, is held to apply specifically to criminal
suppletory application to criminal cases. However, it is likewise true proceedings and to have a twofold purpose: (1) to afford the accused an opportunity to test
that criminal proceedings are primarily governed by the Revised Rules the testimony of witnesses by cross-examination, and (2) to allow the judge to observe the
of Criminal Procedure. deportment of witnesses.23 The Court explained in People v. Seneris24 that the constitutional
requirement "insures that the witness will give his testimony under oath, thus deterring lying
by the threat of perjury charge; it forces the witness to submit to cross-examination, a
valuable instrument in exposing falsehood and bringing out the truth; and it enables the court to
observe the demeanor of the witness and assess his credibility."25
Considering that Rule 119 adequately and squarely covers the situation
in the instant case, we find no cogent reason to apply Rule 23
suppletorily or otherwise." (Underscoring supplied)

As the right of confrontation is intended "to secure the accused in the right to be tried as far
as facts provable by witnesses as meet him face to face at the trial who give their testimony in
his presence, and give to the accused an opportunity of cross-examination,"26 it is properly
The Conditional Examination of a Prosecution Witness Cannot Defeat the Rights of the
viewed as a guarantee against the use of unreliable testimony in criminal trials. In the
Accused to Public Trial and Confrontation of Witnesses
American case of Crawford v. Washington,27 the US Supreme Court had expounded on the
procedural intent of the confrontation requirement, thus:

The CA took a simplistic view on the use of depositions in criminal cases and overlooked
fundamental considerations no less than the Constitution secures to the accused, i.e., the right
Where testimonial statements are involved, we do not think the Framers meant to leave the
to a public trial and the right to confrontation of witnesses. Section 14(2), Article III of the
Sixth Amendment's right to confront witness face to face protection to the vagaries of the
rules of evidence, much less to amorphous notions of "reliability". Certainly, none of the
authorities discussed above acknowledges any general reliability exception to the common-law
rule.
Constitution provides as follows:

Admitting statements deemed reliable by a judge is fundamentally at odds with the right of
Section 14. (1) x x x confrontation. To be sure, the Clause's ultimate goal is to ensure reliability of evidence, but it
is a procedural rather than a substantive guarantee. It commands, not that evidence be

297
RECTO, GAYLE ANGELI M.
2011-0008 | AUSL
Personal Notes on Remedial Law 2 Review (based on the syllabus of Prof. Henedino M. Brondial)

reliable, but that reliability be assessed in a particular manner: by testing in the crucible of ASIDE. Accordingly, the Decision of the Regional Trial Court which disallowed the deposition-
cross-examination. The Clause thus reflects a judgment, not only about the desirability of taking in Laos, Cambodia is REINSTATED.
reliable evidence (a point on which there could be little dissent), but about how reliability can
best be determined." (Underscoring supplied)

Velasco Jr., Peralta, Abad, JJ, concur.

The Webb Ruling is Not on All Fours with the Instant Case

Child witness Rule


The CA found the frail and infirm condition of the prosecution witness as sufficient and
compelling reason to uphold the MeTC Orders granting the deposition-taking, following the SECOND DIVISION
ruling in the case of People v. Webb28 that the taking of an unavailable witness' deposition is
in the nature of a discovery procedure the use of which is within the trial court's sound G.R. No. 197813 September 25, 2013
discretion which needs only to be exercised in a reasonable manner and in consonance with
the spirit of the law.29 PEOPLE OF THE PHILIPPINES, Plaintiff-Appellee,

vs. EDWIN IBANEZ y ALBANTE and ALFREDO (FREDDIE) NULLA y IBANEZ,


Accused-appellants.
But the ruling in the cited case is not instantly applicable herein as the factual settings are not
similar.The accused in the Webb case had sought to take the oral deposition of five defense PEREZ, J.:
witnesses before a Philippine consular agent in lieu of presenting them as live witnesses,
alleging that they were all residents of the United States who could not be compelled by
subpoena to testify in court. The trial court denied the motion of the accused but the CA
differed and ordered the deposition taken. When the matter was raised before this Court, we
sustained the trial court's disallowance of the deposition-taking on the limited ground that Facts
there was no necessity for the procedure as the matter sought to be proved by way of
deposition was considered merely corroborative of the evidence for the defense.30 * Appellants Edwin and Alfredo, with Jesus Monsillo y Taniares (Jesus), were all charged in an
Information for Murder under Article 248 of the Revised Penal Code, which reads:

The undersigned Asst. Provincial Prosecutor accuses Jesus Montisillo y Taniares @


In this case, where it is the prosecution that seeks to depose the complaining witness against Dodong, Edwin Ibañez y Albante and Alfredo(Freddie) Nulla y Ibañez of the crime
the accused, the stringent procedure under Section 15, Rule 119 cannot be ignored without of murder, penalized under the provisions of Article 248 of the Revised Penal
violating the constitutional rights of the accused to due process. Code, committed as follows:

That on or about the 29th day of August, 2004, in the municipality of Bocaue,
province of Bulacan, Philippines, and within the jurisdiction of this Honorable
Finally, the Court takes note that prosecution witness Li Luen Ping had managed to attend the Court, the above-named accused, armed with a soil digger (bareta) and with
initial trial proceedings before the MeTC of Manila on September 9, 2004. At that time, Li Luen intent to kill one Wilfredo Atendido y Dohenog, conspiring, confederating and
Ping's old age and fragile constitution should have been unmistakably apparent and yet the helping one another did then and there willfully, unlawfully and feloniously, with
prosecution failed to act with zeal and foresight in having his deposition or testimony taken evident premeditation, abuse of superior strength and treachery, attack, assault
before the MeTC pursuant to Section 15, Rule 119 of the Revised Rules of Court. In fact, it and hit with the said soildigger (bareta) the said Wilfredo Atendido y Dohenog,
should have been imperative for the prosecution to have moved for the preservation of Li Luen hitting the latter on his head, thereby inflicting upon him serious physical injuries
Ping's testimony at that first instance given the fact that the witness is a non-resident alien which directly caused his death.3
who can leave the Philippines anytime without any definite date of return. Obviously, the
prosecution allowed its main witness to leave the court's jurisdiction without availing of the * During arraignment, Edwin and Alfredo pleaded not guilty. Jesus, on the other hand,
court procedure intended to preserve the testimony of such witness. The loss of its cause is remained at large; the case against him was archived. Thereafter, trial ensued.
attributable to no other party.
* The prosecution’s version was testified to by the victim’s wife and daughter, in succession.

* On that fateful day, Wilfredo Atendido y Dohenog (Wilfredo) was invited by


Still, even after failing to secure Li Luen Ping's conditional examination before the MeTC prior Alfredo to a drinking session with Jesus and Edwin making them a party of four.
to said witness' becoming sick and unavailable, the prosecution would capitalize upon its own Rachel, Wilfredo’s daughter, an adolescent at the time, was underneath the
failure by pleading for a liberal application of the rules on depositions. It must be emphasized house (silong in the vernacular) of a neighbor, three (3)meters away from the
that while the prosecution must provide the accused every opportunity to take the deposition place where Wilfredo and his companions were ostensibly in merrymaking.
of witnesses that are material to his defense in order to avoid charges of violating the right of
the accused to compulsory process, the State itself must resort to deposition-taking sparingly * Rachel saw her father step away from the group to urinate. While Wilfredo
if it is to guard against accusations of violating the right of the accused to meet the witnesses relieved himself, Edwin snatched a t-shirt from a nearby clothesline, and hooded
against him face to face. Great care must be observed in the taking and use of depositions of the t-shirt over the head and face of Wilfredo. Robbed of vision as his head was
prosecution witnesses to the end that no conviction of an accused will rely on ex parte fully covered, Wilfredo was wrestled and pinned down by Edwin, while Alfredo
affidavits and deposition.31 boxed the left side of Wilfredo’s chest. Jesus, armed with a long iron bar, swung
at and hit Wilfredo in the head. Terrified, Rachel stood immobilized as she
watched the attack on father. Thereafter, she saw her mother running out of
their house and crying for help.
Thus, the CA ignored the procedure under the Revised Rules of Criminal Procedure for taking
the deposition of an unavailable prosecution witness when it upheld the trial court's order * On that same auspicious date, 29 August 2004, Rowena, Wilfredo’s wife and
allowing the deposition of prosecution witness Li Luen Ping to take place in a venue other than Rachel’s mother, was inside their house taking care of their youngest daughter.
the court where the case is pending. This was certainly grave abuse of discretion. She heard a commotion coming from the neighboring house, about eight (8)
steps away, so she rushed in that direction. Once outside their house, she saw
Wilfredo prostrate on the ground covered with blood on his face and forehead.
Upon reaching Wilfredo, Rowena saw accused Jesus, standing one meter away
from Wilfredo, holding an iron bar. Edwin and Alfredo stood beside Jesus; Edwin
WHEREFORE, the petition is hereby GRANTED. The assailed Decision dated February 19, 2008
held a white shirt. Forthwith, Jesus and Alfredo ran away while Edwin went
and the Resolution dated November 28, 2008 of the Court of Appeals are REVERSED and SET
home. Rowena asked for help to bring Wilfredo to the hospital. However,
Wilfredo did not reach the hospital alive and was pronounced dead on arrival.
298
RECTO, GAYLE ANGELI M.
2011-0008 | AUSL
Personal Notes on Remedial Law 2 Review (based on the syllabus of Prof. Henedino M. Brondial)

* Expectedly, the defense mainly of Edwin and Alfredo, proffered an altogether different Regional Trial Court (RTC), Branch18, Malolos, Bulacan which convicted accused-appellants
version of the events. Edwin Ibañez y Albante (Edwin) and Alfredo Nulla y Ibañez (Alfredo) of Murder in Criminal
Case No. 3517-M-2004.
* The two accused-appellants pointed to Jesus as the sole culprit, proclaimed
their innocence and professed to being at the scene of the crime only because of
their curiosity for what had occurred.
Issues
* Allegedly, on that day, the two buddies were having their regular drinking
session at Edwin’s house when they heard a commotion outside. Curious about I. THE LOWER COURTS GRAVELY ERRED IN GIVING FULLWEIGHT AND CREDENCE TO THE
the ruckus, they approached and saw Wilfredo prostrate on the ground; Jesus, TESTIMONY OF THEALLEGED PROSECUTION EYEWITNESS.
held an iron bar and was being held back by his sister who was shouting, "Tama
na! Tama na!." Edwin then called for a tricycle so Wilfredo could be brought to a
II. THE LOWER COURTS GRAVELY ERRED IN NOT GIVINGWEIGHT AND CREDENCE TO THE
hospital and given medical attention. Alfredo stood by and merely watched as
DEFENSE‘S EVIDENCE.
events transpired.

III. THE LOWER COURTS GRAVELY ERRED IN CONVICTING THEACCUSED-APPELLANTS


* To corroborate their claim of innocence, the defense called Aniceta Dosil
WHEN THEIR GUILT WAS NOT PROVENBEYOND REASONABLE DOUBT.5
(Aniceta) to the witness stand who testified as follows:

(1) She sold doormats for a living which she peddled on the road;

Ruling
(2) On 29 August 2004, Rachel helped her in selling the doormats;

In sum, the issue is whether the accused are guilty of murder.


(3) On that day, they finished at around 6:00 p.m. and headed to their respective
residences along the railroad track;

(4) Upon arriving at their vicinity, Aniceta witnessed the immediate aftermath of
Edwin and Alfredo maintain their innocence and point to Jesus as the sole perpetrator of the
the purported fight between Jesus and Wilfredo;
crime. They insist that they were at the scene of the crime only because they wanted to know
what the commotion was all about. They claim that, in fact, Edwin called for a tricycle so
(5) At that juncture, Jesus was being embraced by his sister, Marilou, and the
Wilfredo could be brought to a hospital. To discredit the eyewitness testimony of Rachel, they
two were two meters away from the body of Wilfredo;
presented Aniceta who testified that she and Rachel were out on that day selling doormats
and only returned at 6:00 p.m. Thus, Rachel could not have witnessed the murder of Wilfredo.
(6) Marilou recounted to Aniceta that Jesus had hit Wilfredo with an iron bar, a
preemptive move because Wilfredo was about to stab Jesus;

(7) While Aniceta and Marilou discussed the incident, Rachel stood and listened
Both lower courts, however, found the testimony of Rachel credible:
to them;

(8) At that time, only the four of them, Jesus, Marilou, Aniceta and Rachel, were
at the place of the incident;
This Court finds the testimony of Rachel clear and convincing. The testimony flows from a
person who was present in the place where the killing occurred. They are replete with details
(9) After learning the entirety of what had transpired, Aniceta, who was afraid to
sufficient to shift the burden of evidence to appellants. We have no reason to doubt Rachel’s
get involved, and Rachel, ran to their respective houses;
credibility. Her candid account of the incident, standing alone, clearly established the
components of the crime of murder. Appellants’ defense of denial, not sufficiently proven,
(10) For the duration of the day, Aniceta did not step out of her house, neither cannot overcome the conclusions drawn from said evidence. We find no cogent reason to
did she volunteer information to the police when the case was investigated in the deviate from the findings and conclusions of the trial court. Rachel’s testimony was delivered
following days; and in a firm, candid, and straightforward manner. There is no showing that Rachel wavered from
the basic facts of her testimony, even when she was subjected to a rigorous examination.
(11) Aniceta only came forward to testify at the request of Adela Ibañez, wife of
Edwin.

* As previously adverted to, the trial court convicted Edwin and Alfredo of Murder. It disposed of Rachel was only ten (10) years old when she witnessed the murder of the victim. She testified in
the case, to wit: open court two (2) years later. Thus, she cannot be expected to give an error-free narration of
the events that happened two years earlier. The alleged inconsistencies between her sworn
WHEREFORE, accused Edwin Ibañez y Albante and Alfredo (Freddie) Nulla y statement and testimony referred to by appellants do not affect her credibility. What is
Ibañez are hereby found GUILTY beyond reasonable doubt of the crime of important is that in all her narrations she consistently and clearly identified appellants as the
murder and are hereby sentenced to suffer imprisonment of reclusion perpetua perpetrators of the crime. Inconsistencies between the sworn statement and the testimony in
and to indemnify the heirs of Wilfredo D. Atendido in the amount of: court do not militate against witness’ credibility since sworn statements are generally
considered inferior to the testimony in open court.6
a) Fifty Thousand Pesos (P50,000.00) as civil indemnity;

b) Twenty-Five Thousand Pesos (P25,000.00) as temperate damages;


We find no error in the lower courts’ disposal of the issue.
c) Fifty Thousand Pesos (P50,000.00) as moral damages;

d) Twenty-Five Thousand Pesos (P25,000.00) as exemplary damages; and


Well-entrenched in jurisprudence is that the trial court's evaluation of the testimony of a
e) One Million Nine Hundred Forty-Six Thousand and One Hundred Eighty Pesos witness is accorded the highest respect because of its direct opportunity to observe the
(P1,946,180.00) for the unearned income of Wilfredo Atendido.4 witnesses on the stand and to determine if they are telling the truth or not.7 This opportunity
enables the trial judge to detect better that thin line between fact and prevarication that will
determine the guilt or innocence of the accused. That line may not be discernible from a mere
* On appeal, Edwin and Alfredo found no reprieve. The Court of Appeals did not deviate from
reading of the impersonal record by the reviewing court. Thus, the trial judge's evaluation of
the RTC’s ruling and affirmed in toto its finding of guilt.
the competence and credibility of a witness will not be disturbed on review, unless it is clear
from the records that his judgment is erroneous.8
* Before us is an appeal via a Notice of Appeal from the Decision of the Court of Appeals in
CA-G.R. CR-H.C. No. 04051.1 The appellate court affirmed in toto the Decision2 of the
299
RECTO, GAYLE ANGELI M.
2011-0008 | AUSL
Personal Notes on Remedial Law 2 Review (based on the syllabus of Prof. Henedino M. Brondial)

A: Yes, ma’am.

We have scrutinized the testimony of lone eyewitness, Rachel. Throughout her testimony, in
her direct, cross and re-direct and re-cross examinations, she candidly recounted the events
surrounding the killing of her father as follows: Q: Also inside the house?

A: Yes, ma’am.

PROS. LAGROSA:

Your Honor please, may we invoke the right of the child the provisions (sic) under the child Q: When your father woke up, what did he do?
witness wherein we can ask leading questions and in Tagalog.
A: All of us ate rice, ma’am. ("Kumain po kaming lahat ng kanin.")

COURT:
Q: Can you tell us if that is already dark or still daytime?
Anyway, the questions can be interpreted.
A: It was still daytime, ma’am.

PROS. LAGROSA:
xxxx
Only the leading questions, your Honor.

Q: After eating rice, will you tell us what happened, if you still remember?
Q: You said that your father came from sleeping in your house, did you know what time of the
day your father went to sleep?
A: My father was called by his compadre, ma’am.

A: I do not know because I do not know how to read time.

Q: And who was that compadre who called your father?

xxxx
A: Freddie, ma’am.

Q: But do you know whether or when your father went to sleep? It was morning, noon or
Q: Do you know the full name of this Freddie?
afternoon or nighttime or daytime?

A: Freddie Nulla, ma’am.


A: "Hapon po." (In the afternoon.)

Q: Why do you know Freddie Nulla?


Q: Early afternoon, late afternoon or mid-afternoon?

A: He is a compadre of my father, ma’am.


A: Late in the afternoon, Your Honor. ("bandang hapon-hapon po.")

Q: Did you often see him in your place?


Q: Was it already dark?

A: Yes, ma’am.
A: Not yet, your Honor.

Q: Is Freddie Nulla now here in court?


PROS. LAGROSA:

A: Yes, ma’am.
Q: According to you, your father went to sleep, where were you when your father went to
sleep?

A: I was in the house, ma’am.


Q: Will you look around and point to him?

xxxx
INTERPRETER:

Witness pointed to a detention prisoner (sic) when asked to identify himself answered
FREDDIE NULLA.Q: Now, you said that Freddie Nulla, the compadre, called your father, do you
Q: And when your father woke up, were you still in the house?
still remember how he was called?

300
RECTO, GAYLE ANGELI M.
2011-0008 | AUSL
Personal Notes on Remedial Law 2 Review (based on the syllabus of Prof. Henedino M. Brondial)

A: Just near, ma’am, 3 to 4 meters.9

A: Yes, ma’am.

xxxx

Q: How?

A: "Pare. Pare." Q: Rachel, last time you testified that your father followed Freddie Nulla at the back of the
house of Kuya Unyo and at that time you were under the house of Kuya Unyo, do you
remember having stated that last time?

Q: And when your father was called, what did your father do? A: Yes, ma’am.

A: My father followed Freddie at the back of the house of Kuya Edwin.

Q: While you were at the house of Kuya Unyo, do you remember anything unusual that
happened at that time?
Q: At the time your father followed Freddie at the back of the house of your Kuya Edwin,
where were you? A: When my father was being killed, ma’am.

A: I was under the house of Kuya Unyo, ma’am.

Q: You said that your father was being killed or "pinapatay na po si papa ko," who killed your
father?
Q: Now, you mentioned that your father followed Freddie at the back of the house of Kuya
Edwin, who is this Kuya Edwin? A: Kuya Edwin, Kuya Freddie and Kuya Dodong, ma’am.

INTERPRETER: Q: You said that Kuya Freddie, Kuya Edwin and Kuya Dodong were killing your father, how did
Kuya Edwin, how was he killing your father as you said?
Witness pointing to a detention prisoner who identified himself as EDWIN IBAÑEZ.PROS.
LAGROSA: A: "Pinuluputan po sa mukha ng damit ni Kuya Edwin." (Kuya Edwin put around a piece of
cloth).

Q: You said that at that time you were under the house of Kuya Unyo, what is the full name of
this Kuya Unyo, if you know? Q: You said that Kuya Edwin put around a piece of cloth on your papa, in what part of your
father’s body (sic) that cloth being put around by Kuya Edwin?
A: I do not know, ma’am.
A: He put it around all over the face and the head, ma’am.

Q: What were you doing under the house of Kuya Unyo?


PROS. LAGROSA:
A: I was throwing stones, ma’am.
The witness was demonstrating by making a circling movement or motion of her hand all over
the head and the face.

Q: And this house of Kuya Unyo, is that near or far from your house?

Q: And then what happened when Kuya Edwin put around that piece of cloth all over the head
A: Just near our house, ma’am.
and face of your papa?

A: "Itinumba po siya."

Q: Can you point a place here where you are now sitted (sic) up to this courtroom to show the
distance between your house and the house of Kuya Unyo?

Q: You said "itinumba po siya," who caused your father to tumble down?

A: After Kuya Edwin had put around the piece of cloth on my father, he tumbled him down.
PROS. LAGROSA

The witness pointed up to the wall.

Q: And when your father tumbled down, what else happened?

A: Kuya Freddie boxed him, ma’am.


ATTY. MALLILLIN:

Can we estimate, your Honor.

Q: Did you see in what part of your father’s body was he boxed by Kuya Freddie?
301
RECTO, GAYLE ANGELI M.
2011-0008 | AUSL
Personal Notes on Remedial Law 2 Review (based on the syllabus of Prof. Henedino M. Brondial)

A: Yes, ma’am. and tell time carry no weight and cannot overcome the clear and convincing
testimony of Rachel as to who killed her father.

Q: What part of his body was boxed?


We likewise note that the line of questioning of the defense during cross-examination on the
A: On the left portion of the shoulder blade, ma’am. competency of Rachel to read and tell time did not distract her in recollecting how her father
was attacked by accused-appellants. From her position underneath the house of her "Kuya
Unyo," she saw her father, Wilfredo, attacked by accused-appellants. Although she was
astonished as the happening unfolded, her ability to perceive, remember, and make known her
perception was not diminished.
Q: And how about Kuya Dodong when Kuya Edwin put around a piece of cloth and when Kuya
Freddie boxed your father, where was Kuya Dodong at that time?

A: He was also there, ma’am.


As regards Aniceta’s version of the events that Jesus was the sole perpetrator of the crime
who attacked Wilfredo only in self-defense, we easily see the fatal flaw: Aniceta arrived after
the supposed fight between Wilfredo and Jesus, and what transpired was merely relayed to
her by Jesus’ sister, Marilou.
Q: And what was he doing, if he was doing anything at that time?

A: "Binareta na po ‘yong papa ko sa ulo."


Quite apparent from Aniceta’s narration of events is that she has no personal knowledge of
Wilfredo’s killing. Aniceta’s testimony is mainly hearsay, specially on the purported fight
between Wilfredo and Jesus that ended in Wilfredo’s death. Aniceta’s testimony as such carries
COURT: no probative weight. At best, Aniceta’s testimony is an independent relevant statement:
offered only as to the fact of its declaration and the substance of what had been relayed to
Aniceta by Marilou, not as to the truth thereof.13

Q: What did he use noong" binareta"?

A: It is a long iron bar used in digging soil? Section 36 of Rule 130 of the Rules of Court explicitly provides:

PROS. LAGROSA: SEC. 36. Testimony generally confined to personal knowledge; hearsay excluded. - A witness
can testify only to those facts which he knows of his personal knowledge; that is, which are
derived from his own perception, except as otherwise provided in these rules.

Q: Now, what happened after Kuya Dodong " binareta" (sic) your father on the head?

We detect a clever, albeit transparent ploy, to pin Jesus who had already fled and is
A: "Nandoon pa po ako sa silong nila Kuya Unyo nakita ko nalang ponandoon na po ang nanay
temporarily out of reach of the law. Thus, with Jesus temporarily shielded from punishment,
ko pati po mga kapatid ko tsaka na poako lumabas."10
accused-appellants freely accuse and point to him as the sole perpetrator of the crime. This
cannot trump the solid testimony of Rachel on accused-appellants’ direct participation in killing
Wilfredo.

As the lower courts have done, we accord full faith and credence to Rachel’s testimony. She
was young and unschooled, but her narration of the incident was categorical, without
wavering. It has no markings of a concocted story, impressed upon her by other people.
We likewise affirm the lower courts’ appreciation of the aggravating circumstance of treachery:

The defense, accused-appellants herein, tried to further discredit Rachel’s testimony by


The essence of treachery is the sudden and unexpected attack by an aggressor without the
arguing that Rachel was a mere child who had studied only until the first grade of elementary
slightest provocation on the part of the victim, depriving the latter of any real chance to
school and could barely read, and did not know how to tell time.
defend himself, thereby ensuring its commission without risk to the aggressor. Treachery
attended the killing of the victim because he was unarmed and the attack on him was swift
and sudden. He had not means and there was no time for him to defend himself. Indeed,
nothing can be more sudden and unexpected than when petitioners Edwin and Alfredo
We cannot take Rachel’s testimony lightly simply because she was a mere child when she attacked the victim. The latter did not have the slightest idea that he was going to be attacked
witnessed the incident and when she gave her testimony in court. There is no showing that because he was urinating and his back was turned from his assailants. The prosecution was
her mental maturity rendered her incapable of testifying and of relating the incident truthfully. able to establish that petitioners’ attack on the victim was without any slightest provocation on
the latter’s part and that it was sudden and unexpected. This is a clear case of treachery.14

With exceptions provided in the Rules of Court,11 all persons who can perceive,
and perceiving, can make known their perception to others, may be witnesses. Finally, we affirm the lower court’s award of damages consistent with jurisprudence:15 (1)
That is even buttressed by the Rule on Examination of a Child Witness which P50,000.00 as civil indemnity; (2) P25,000.00 as temperate damages; and (3) P50,000.00 as
specifies that every child is presumed qualified to be a witness. To rebut this moral damages. Consistent with current jurisprudence, we increase the award of exemplary
presumption, the burden of proof lies on the party challenging the child's damages from P25,000.00 to P30,000.00.16 However, we delete the award of P1,946,180.00
competence. Only when substantial doubt exists regarding the ability of the child representing the unearned income of Wilfredo.
to perceive, remember, communicate, distinguish truth from falsehood, or
appreciate the duty to tell the truth in court will the court, motu proprio or on
motion of a party, conduct a competency examination of a child.12 Thus,
petitioners’ flimsy objections on Rachel’s lack of education and inability to read

302
RECTO, GAYLE ANGELI M.
2011-0008 | AUSL
Personal Notes on Remedial Law 2 Review (based on the syllabus of Prof. Henedino M. Brondial)

To obviate confusion on the award of loss of earning capacity, we reiterate herein that * Meanwhile, the "Jiffy" followed. It stopped beside the fallen figure and a tall, thin man
compensation for lost income is in the nature of damages and as such requires due proof of alighted. The man fired several shots at the prostrate figure. He boarded the "Jiffy" which
the damages suffered; there must be unbiased proof of the deceased’s average income.17 In sped away towards Leon Kilat Street. Romeo Sta. Cruz, Jr., moved his jeep and focused its
this case, we only had he testimony of Wilfredo’s spouse, Rowena, who claimed that Wilfredo headlights on the victim.
earned P400.00 to P500.00 daily as a doormat vendor.
* In the meantime, PO3 Alexander Rustela was at a vulcanizing shop near the intersection of
Bacalso Avenue and Leon Kilat Street, when he heard gunshots coming from the north. He ran
towards where the gunshots came and saw people scampering. All of a sudden, the "Jiffy"
On more than one occasion, we have held that the bare testimony of a deceased’s mother or with three persons on board sped past him and made an abrupt left turn at Leon Kilat Street.
spouse as to the income or earning capacity of the deceased must be supported by competent Rustela immediately radioed for assistance. Minutes later, patrol car No. 201 with PO2 Herbert
evidence like income tax returns or receipts.18 Ramos on board arrived. Rustela boarded the car and they followed the "Jiffy," while
broadcasting an alarm to police headquarters and other mobile patrol cars.

* On nearby Colon Street, SPO1 Eleazar Abrigana and PO2 Romeo Abellana were cruising
aboard patrol car No. 208, when they heard a radio message that the suspects in the shooting
In People v. Caraig,19 we have drawn two exceptions to the rule that "documentary evidence
incident were aboard a "Jiffy." As they turned left at Leon Kilat Street, they saw the "Jiffy"
should be presented to substantiate the claim for damages for loss of earning capacity," and
heading towards Carbon Market. They pursued the "Jiffy" which stopped in front of t he Don
have thus awarded damages where there is testimony that the victim was either (1) self-
Bosco Building near BBRC, when police car No. 205, with PO Eugenio Badrinas and PO2 Gerald
employed earning less than the minimum wage under current labor laws, and judicial notice
Cue aboard, blocked the "Jiffy's" path. Cue fired a warning shot and three persons alighted.
may be taken of the fact that in the victim's line of work no documentary evidence is available;
The driver was appellant Astellero, whom Cue had recognized and seen before at the BBRC.
or (2) employed as a daily-wage worker earning less than the minimum wage under current
Abrigana and Cue approached the trio who stood a meter away from the "Jiffy." SPO1
labor laws."
Abrigana frisked Abriol and seized from his waist a .38 caliber revolver with serial number
PO8485 with six (6) empty shells in its cylinder.11 Under Abriol's seat, the police also found a
.45 caliber pistol bearing serial number PGO 13506 with nine (9) live rounds in its magazine
and another .45 caliber pistol with serial number 52469 loaded with five (5) unfired bullets.
Although Wilfredo’s occupation as a doormat vendor may fall under the first exception, the
minimum wage for Region III, which includes the province of Bulacan, is below P400.00 as per
* While the patrol cars were chasing the "Jiffy," another police team proceeded to the crime
the National Wages and Productivity Commission Regional Daily Minimum Wage Rates as of
scene in response to the alarm. This team from Police Station No. 3 in San Nicolas, Cebu City
August 2013.20 Regrettably, except for the bare assertion of Rowena, Wilfredo's spouse, we
rushed the victim to the Cebu City Medical Center, where he was pronounced dead on arrival.
have nothing to anchor the award for loss of earning capacity. Thus, we delete the award for
Meanwhile, PO3 Celso Seville, Jr., a homicide investigator of Police Station No. 3 found four
loss of earning capacity in the amount of P1,946,180.00.
(4) .45 caliber shells some four (4) feet away from the victim's body, and two (2) deformed
slugs where the victim had lain, and submitted them to the Region 7 PNP Crime Laboratory for
ballistics testing.

WHEREFORE, the appeal is DISMISSED. The Decisions of the Court of Appeals in CA-G.R. H.C. * Dr. Ladislao Diola, Jr., Chief of the PNP Region 7 Crime Laboratory autopsied the victim's
No. 04051 and the Regional Trial Court, Branch 18, Malolos, Bulacan in Criminal Case No. body. He found that the cause of the victim's death was "cardiorespiratory arrest due to shock
3517-M-2004 are AFFIRMED with MODIFICATION. The award of exemplary damages is and hemorrhage secondary to multiple gunshot wounds to the trunk and head.''14 Dr. Diola
increased from P25,000.00 to P30,000.00 and we delete the award for loss of earning capacity recovered a .38 caliber slug from the corpse, which he later submitted for ballistics
in the amount of P1,946, 180.00. examination.

* SPO4 Lemuel Caser, ballistician of the PNP Crime Laboratory, reported the following:

Carpio, Del Castillo, Perlas-Bernabe, JJ., concur. 1. Fired cartridge cases marked "JA-1" to "JA-3" possesses similar individual
characteristics markings with the test cartridge cases fired from cal .45 with SN:
PGO13506;

2. Fired cartridge cases marked "JA-4" and "E-69-6" possesses similar individual
Opinion rule characteristics markings with the test cartridge cases fired from cal .45 pistol
with SN: 52469;
> Expert witness
3. Fired bullet metal jacket marked "JA-5" possesses similar individual
characteristics markings with test bullets fired from cal .45 pistol with SN:
SECOND DIVISION
PGO13506;

G.R. No. 123137 October 17, 2001


4. Fired cartridge cases marked "E-45-1 " to "E-45-6" possesses similar individual
characteristics markings with the test cartridge cases fired from cal .38 Rev. SN:
PEOPLE OF THE PHILIPPINES, plaintiff-appellee, P8445;

vs. PO2 ALBERT ABRIOL, MACARIO ASTELLERO, and JANUARIO DOSDOS, accused- 5. Fired bullets marked as "JA-6" and "LD" possesses similar individual
appellants. characteristic markings with the test bullets fired from cal .38 Rev. SN: P8445.

QUISUMBING, J.: * The following day, appellants underwent a paraffin test. The hands of appellants were found
positive for gunpowder residues. A chemistry test on the firearms showed that the three
handguns were also positive. Inspector Myrna Areola, Chief of the Chemistry Section of the
PNP Region 7 Crime Laboratory, stated in her testimony that the firearms had been fired, and
Facts that appellants had fired the guns within a period of seventy-two (72) hours prior to the
examination.

* At around 11:50 P.M., June 5, 1993, Romeo Sta. Cruz, Jr., a radio news reporter then
aboard his jeep, had just reached the ABS-CBN compound in P. del Rosario Street, Cebu City, * PO2 Albert Abriol, a policeman previously detailed as a jailguard at the Bagong Buhay
Rehabilitation Center (BBRC) in Cebu City, was himself a detention prisoner in BBRC. He was
when he heard a couple of gunshots. He looked around and saw a man running unsteadily
charged with murder, a non-bailable offense, in Criminal Case No. CBU-28843 before the RTC
towards the intersection of P. del Rosario Street and Jones Avenue (Osmeña Boulevard). The
of Cebu City, Branch 14. [On the other hand,] Macario Astellero was a former prisoner at
man was shouting "Tabang, tabang!" ("Help! Help!"). Sta. Cruz, Jr., saw a red "Jiffy" make a
U-turn near the gate of the city central school that nearly ran over the man shouting for help. BBRC, who had served time for grave threats. The warden then, Chief Inspector Navales,
The man turned back and staggered towards the direction of Bacalso Avenue and Urgello employed him as his personal driver and general factotum. Navales was found guilty of grave
Private Road, but after a few meters on wobbly legs, he stopped and collapsed. misconduct in Administrative Case No. 01-93 for allowing Abriol and Dosdos out of BBRC on
the day of the murder and was summarily dismissed from the police force.[Lastly,] Januario
303
RECTO, GAYLE ANGELI M.
2011-0008 | AUSL
Personal Notes on Remedial Law 2 Review (based on the syllabus of Prof. Henedino M. Brondial)

Dosdos had been convicted by the RTC of Cebu City, Branch 10, of highway robbery in said that on the day of the incident he was, as a BBRC jailguard, authorized to carry his
Criminal Case No. CBU-18152 but Navales failed to act on the mittimus ordering Dosdos' service firearm.20 He presented a Memorandum Receipt21 authorizing him to carry the
transfer to the national penitentiary, and he remained in BBRC. Abriol and Dosdos enjoyed government-issued .38 revolver.22
special privileges at BBRC as the warden's errand boys or "trustees." The victim, Alejandro
Flores alias "Alex," was a former policeman. He was dismissed from the PNP in August 1992 * On the witness stand, Astellero and Dosdos narrated a similar version of the incident as did
after testing positive for prohibited drugs. Abriol. Both vehemently denied having any knowledge of the two .45 caliber pistols found by
PO3 Cue in the "Jiffy."23
* Amended Information dated September 6, 1993, docketed as Criminal Case No. CBU-30350,
wherein appellants PO2 Albert Abriol of the Philippine National Police (PNP), Macario Astellero, * The defense also presented Dr. Jesus P. Cerna, medico-legal officer of the Cebu City PNP
Januario Dosdos, and PNP P/Chief Inspector Gaudioso Navales were charged with murder Command, to testify on the caliber of the firearms which might have caused the gunshot
allegedly committed as follows: wounds of the victim. Relying on the Necropsy Report prepared by Dr. Diola, Dr. Cerna
declared that wound nos. 1 and 2, which each measured 0.6 cm. by 0.6 cm., may have been
That on or about the 5th day of June, 1993, at about 11:50 P.M., in the City of caused by a .38 caliber firearm. As to wound nos. 3 and 4, which each measured 0.5 cm. by
Cebu, Philippines and within the jurisdiction of this Honorable Court, the said 0.5 cm., it was possible that a .38 handgun was used, or one with a smaller bore. Dr. Cerna
accused, armed with handguns, conniving and confederating together and opined that a .45 pistol could not have inflicted all the foregoing wounds, as the entry points
mutually helping one another, with treachery and evident premeditation, with were too small for a .45 caliber bullet. With respect to the grazing wounds found on the
deliberate intent, with intent to kill, did then and there shot one Alejandro Flores victim's body, Dr. Cerna testified that it was impossible to determine the caliber of the firearm
alias Alex with the said handguns, hitting him on the different parts of his body, used.24
thereby inflicting upon him the following physical injuries:
* The trial court found appellants' version of the incident neither convincing and credible and,
CARDIO RESPIRATORY ARREST DUE TO SHOCK AND as earlier stated, it believed the prosecution's version. Petitioners' were convicted of the
HEMORRHAGE SECONDARY TO MULTIPLE GUNSHOT WOUNDS TO offenses charged.
THE TRUNK AND THE HEAD
* Its decretal portion reads:
as a consequence of which the said Alejandro Flores alias Alex died later.
WHEREFORE, judgment is hereby rendered:
* Abriol, Astellero, and Dosdos were also indicted for illegal possession of firearms in Criminal
Case No. CBU-33664. The charge sheet reads: In Criminal Case No. CBU-30350 for Murder, the Court finds accused Albert
Abriol, Macario Astellero and Januario Dosdos, GUILTY of murder beyond
That on or about the 5th day of June 1993 at about 11:48 P.M. in the City of reasonable doubt and each is hereby sentenced to reclusion perpetua, with the
Cebu, Philippines, and within the jurisdiction of this Honorable Court, the said accessory penalties provided by law; to indemnify the heirs of deceased
accused, conniving and confederating together and mutually helping one Alejandro Flores the sum of P50,000.00; actual damages of P30,000.00,
another, with deliberate intent, did then and there keep under their control and representing a reasonable amount for the embalming, vigil, wake, and burial
possession the following: expenses; P30,000.00 for attorney's fees; and to pay the costs.

1. one (1) .38 cal. revolver (Armscor) with SN P08445 with six empty shells; For insufficiency of evidence, accused Gaudioso Navales is hereby ACQUITTED
with costs de officio.
2. one (1) .45 cal. pistol (Colt) with SN P6013506 with 9 live ammunitions (sic);
In Criminal Case No. CBU-33664 for Illegal Possession of Firearms, accused
3. one (1) .45 cal. Pistol (Colt) with SN 52469 with five live ammunition. Albert Abriol, Macario Astellero and Januario Dosdos, are hereby sentenced to
suffer an indeterminate penalty of 14 years, 8 months and 1 day to 17 years and
4 months and to pay the costs.
without first obtaining a permit or license therefor from competent authority.

The .38 caliber revolver, SN P08445 and the two .45 caliber pistols with SN PGO
* When arraigned, all the accused pleaded not guilty to both charges. Since the indictments
13506 and SN 52469, are hereby confiscated and forfeited in favor of the
arose from the same incident, the cases were jointly tried.
Government and accordingly, the Clerk of Court of this Branch is directed to turn
over the said firearms to the Chief of Police, Cebu City, or to the Firearms and
* The widow and relatives of the victim testified on the possible motive behind the killing.
Explosives Office (FEO) of the PNP Region 7, upon proper receipt.
They claimed the victim, a confessed drug user, may have been "rubbed out" on the orders of
Navales for failure to remit P31,000 as proceeds from pushing prohibited drugs. After failing to
The Cebu City Chief of Police is directed to release immediately upon receipt
deliver the drug money to Navales, for whom he was repeatedly pushing drugs, the victim
hereof, the person of Gaudioso Navales, unless there be any other valid reason
went into hiding, but later returned to Cebu City because he missed his family.17
for his continued detention.

* Appellants deny the accusations. Abriol averred that he and Dosdos were among the several
SO ORDERED.1
"trustees" at BBRC assigned to work in the kitchen. Appellant Astellero, who was the warden's
driver, was also in charge of marketing for the prisoners' food. On the day of the incident,
Astellero realized that there was no money for the next day's marketing so he asked Abriol to * Hence, this appeal.
accompany him to the house of Navales, but since he was not in, they returned to BBRC and
saw Navales an hour later. After they received the money from Navales' niece on their way
back to BBRC, Dosdos heard gunshots. Abriol ordered Astellero, who was driving, to turn back.
Then Abriol claimed he saw a tall, slim man alight from a "Jiffy" and shoot at a prone figure on Issues
the ground. Seconds later, the gunman returned to the "Jiffy," which sped off. Abriol said he
ordered Astellero to chase that "Jiffy" but it had too much of a headstart and they lost sight of
I. THE LOWER COURT ERRED IN CONVICTING THE ACCUSED-APPELLANTS OF THE CRIMES
it. Abriol ordered Astellero to proceed to BBRC. At Colon Street, they heard gunshots behind
OF MURDER AND ILLEGAL POSSESSION OF FIREARMS DESPITE THE FLIMSY AND
them and the blaring siren of a police car. They explained that since they were detention
UNRELIABLE EVIDENCE PRESENTED BY THE PROSECUTION.
prisoners, they had to evade meeting the police. They heard more gun shots. Upon reaching
BBRC, the gates were closed, so they drove to the old airport. On their way back to BBRC
II. THE LOWER COURT ERRED IN FINDING THE GUILT OF THE ACCUSED-APPELLANTS OF
several police cars blocked them and arrested them. SPO4 Eleazar Abrigana frisked him and
THE CRIME OF MURDER AND ILLEGAL POSSESSION OF FIREARMS BEYOND REASONABLE
took the .38 service revolver from his waist.18
DOUBT.

* Abriol also testified that he surrendered his service firearm to the BBRC Administrative
Officer when he was served a warrant of arrest for murder in Criminal Case No. CBU -28843.
However, the handgun was defective and it was returned to him for repair by Armscor, and
upon repair he handed it over to the BBRC armory. The armorer returned it to him since there Ruling
was no place to keep it. He said that although he was a detention prisoner, he had yet to be
discharged from the service. He was assigned guard and escort duties by the warden.19 Abriol

304
RECTO, GAYLE ANGELI M.
2011-0008 | AUSL
Personal Notes on Remedial Law 2 Review (based on the syllabus of Prof. Henedino M. Brondial)

At issue is whether the prosecution's evidence, which is mainly circumstantial, suffices to xxx xxx xxx
convict appellants for murder and violation of Presidential Decree No. 1866, beyond
reasonable doubt.

Q: How many persons fired a shot at the fallen man?

A. Criminal Case No. CBU-30350 A: I only saw that man Your Honor who alighted from the Jiffy.

On their conviction for murder, appellants argue that the prosecution's circumstantial evidence Q: Did you see his physical features?
against them is weak, ambiguous, and inconclusive. Specifically, appellants contend that they
should be acquitted because:
A: Only (t)his, I can only tell his height, he was tall and his body build is thin. Tall and
thin. (Emphasis supplied)25

First, eyewitness Romeo Sta. Cruz, Jr., did not personally identify them as the culprits. At no
point in his testimony did eyewitness Sta. Cruz, Jr., positively identify any of the appellants or
Since the sole eyewitness could not identify the gunman and his companions, the prosecution
appellant Abriol as the gunman. Sta. Cruz, Jr. only gave a general description of the assailants,
relied on circumstantial evidence from which the trial court could draw its findings and
despite attempts to make him give a categorical identification. He admitted he found out the
conclusion of culpability.26 Circumstantial evidence may be relied upon, as in this case, when
name of Abriol from television and news reports and could not identify Abriol as the one whom
to insist on direct testimony would result in setting felons free.
he saw shot the victim. The transcript of his testimony is revealing.

Second, appellants assert that the paraffin tests are judicially recognized as unreliable and
Q: Then after the Jiffy stopped in front of the fallen victim, what happened next?
inconclusive. A paraffin test could establish the presence or absence of nitrates on the hand.
However, it cannot establish that the source of the nitrates was the discharge of firearms.
A: I saw that there was a man who disembarked from the Jiffy. He was a tall, thin fellow Nitrates are also found in substances other than gunpowder. A person who tests positive may
who disembarked from the Jiffy and at the same time, he shot the fallen victim. have handled one or more substances with the same positive reaction for nitrates such as
explosives, fireworks, fertilizers, pharmaceuticals, tobacco, and leguminous plants. Hence, the
presence of nitrates should only be taken as an indication of a possibility that a person has
fired a gun.27 However, it must be borne in mind that appellants were not convicted on the
Q: How many times did he shoot the victim? sole basis of the paraffin test.

A: I cannot count attorney but I saw him shooting the victim.

Third, appellants claim that the autopsy report of prosecution witness Dr. Ladislao
Diola revealed serious ambiguities.28 Dr. Jesus P. Cerna, using the same autopsy
report, said that the gunshot wounds measuring 0.6 x 0.6 centimeters could not
Q: In your affidavit, you said that the person who disembarked from the Jiffy, whose
have been caused by a .45 caliber pistol because an entrance wound of that size
name you know later on as PO2 Albert Abriol, PNP, shot the victim in the different parts of his
was too small for a .45 caliber bullet.29 Dr. Cerna claimed that a wound inflicted by
body. If Albert Abriol is now in the courtroom, will you please point to him?
a .45 pistol would have an entry point of anywhere from 1.1 to 1.3 centimeters. He
declared that it was with more reason that an entrance wound measuring .5 x .5
A: I will know him attorney because of the TV shows and newspapers. centimeters could not be caused by a caliber .45 bullet.30 Since no firearm smaller
than a .38 caliber pistol was seized from appellants, they claim the observation of
Dr. Cerna only shows that they could not have shot the victim.

COURT: (TO WITNESS)

Q: You are referring to the name of that man who disembarked from the Jiffy and fired We note, however, that during cross-examination, Dr. Diola carefully explained
several shots at the fallen victim? that a firearm's caliber is not the only basis for determining the cause of the
gunshot wound. He said:
A: Yes, I know his name Your Honor on (sic) the news cast.

ATTY. REMOTIQUE:
COURT: (TO WITNESS)
Q: So, normally the size of .5 cm x .5 cm which is the point of entry of gunshot
Q: Alright, forget the news. The man you saw when he alighted from the Jiffy and wound No. 3 this may have been caused by a firearm of lesser caliber than caliber
poured (sic) several bullets on the fallen man, look around if he is in the courtroom? .38?

A: I cannot identify Your Honor. A: Not necessarily. There is a very small difference in the size and this does
not preclude that gunshot wound No. 3 may have also been caused by the same
firearm which caused gunshot wounds Nos. 1 and 2. There are factors which often
affect the size of the wounds at the time of the examination, perhaps a recission
(sic) of the skin in the area where gunshot Wound No. 3 was inflicted so that
COURT:
gunshot wound becomes smaller.

Q: You cannot?

A: But [because] what I saw is a man who is tall and thin because it was dark.
Q: Did you not say that normally the point of entry of the gunshot wounds
vary with the caliber of the firearm which caused it, so that the point of entry
caused by one firearm of a particular caliber may be bigger than the point of entry
of a gunshot wound caused by another firearm of lesser caliber?

305
RECTO, GAYLE ANGELI M.
2011-0008 | AUSL
Personal Notes on Remedial Law 2 Review (based on the syllabus of Prof. Henedino M. Brondial)

A: I told you of other factors that often affect the size of the entry of the bullet
although the caliber is one basis of the size of the wounds.
We agree with the trial court that P/Inspector Caser qualifies as a ballistics expert. He is a
licensed criminologist, trained at the Ballistics Command and Laboratory Center in Fort
Bonifacio, in the PNP Crime Laboratory in Camp Crame, and in the National Bureau of
xxx xxx xxx Investigation. He had previously testified as an expert witness in at least twenty-seven (27)
murder and homicide cases all over the country.42 An expert witness need not present
comparative microphotographs of test bullets and cartridges to support his findings.43
Examination under a comparison microscope showing that the test bullet and the evidence
bullet both came from the same gun is sufficient.44 Moreover, the ballistician conclusively
Q: Will you explain further on that because my understanding is that .5 cm
found similar characteristic markings in the evidence, test cartridges and slugs.
wound must perforce be caused by a firearm of lesser caliber than that which
caused the .6 cm wound?

A: As I said there are ranges in the size of the wounds. The variance in the size
Fifth, appellants aver that the prosecution failed to show any plausible motive for appellants to
of the wound when it is minimal does not exclude the possibility that a wound with
kill the victim. The prosecution tried to prove that their co-accused Navales instigated them to
a .5 cm size and .6 cm size could have been caused by the same caliber. (Emphasis
kill the victim because Navales had a grudge against him. However, as Navales was acquitted,
supplied).31
appellants insist that Navales' acquittal should redound to their benefit since no motive was
imputed on their part.

The Office of the Solicitor General points out that Dr. Diola's testimony is supported
by Dr. Pedro P. Solis, a medical expert, in his book entitled Legal Medicine. The
Motive is not an essential element of a crime,45 particularly of murder.46 It becomes relevant
factors which could make the wound of entrance bigger than the caliber include:
only where there is no positive evidence of an accused's direct participation in the commission
(1) shooting in contact or near fire; (2) deformity of the bullet which entered; (3) a
of a crime.47 Stated otherwise, proof of motive becomes essential to a conviction only where
bullet which might have entered the skin sidewise; and (4) an acute angular
the evidence of an accused's participation in an offense is circumstantial.48 A careful perusal
approach of the bullet. However, where the wound of entrance is smaller than the
of the State's evidence reveals that the prosecution had established sufficient motive why
firearm's caliber, the same may be attributed to the fragmentation of the bullet
appellants killed the victim, independent of any grudge which Navales may have had against
before entering the skin or to a contraction of the elastic tissues of the skin (stress
the latter. At the time of the incident, appellants Abriol and Dosdos were both BBRC detention
supplied).32 Dr. Diola testified that a .45 caliber pistol could have caused the
prisoners during Navales' term as warden. Abriol and Dosdos were treated as highly favored
grazing wounds on the victim's head and extremities.33 Dr. Cerna corroborated Dr.
"trustees" of Navales and were never locked up. Abriol and Dosdos were even allowed to go
Diola's findings in this regard.34 Such expert opinions disprove appellants' theory
out of BBRC to do the marketing for the prison's kitchen. Appellant Astellero, a former
that the .45 caliber handguns confiscated from them could not have been used in
detention prisoner, was also a recipient of Navales' favors. Navales hired Astellero as his
killing the victim.
personal driver after the latter served his sentence. Navales and the victim, a former BBRC
jailguard, were associates in dealing with prohibited drugs, until they had a falling out
allegedly after the victim failed to remit to Navales proceeds from the sale of illegal drugs
amounting to P31,000. Appellants apparently killed the victim to return the "special favors"
Fourth, appellants allege that the testimony of P/Inspector Lemuel Caser, the prosecution's Navales had showered them. Lack of a motive does not necessarily preclude conviction.
ballistics expert, clearly shows that: (1) He is ignorant about such ballistics instruments such Persons have been killed or assaulted for no reason at all, and friendship or even relationship
as the micrometer, goniometer, and pressure barrel.35 (2) He is not conversant with "the is no deterrent to the commission of a crime.49
required references concerning ballistics," particularly books on the subject by foreign
authorities.36 (3) He could not "scientifically determine the caliber of a bullet."37 Since
P/Inspector Caser lacked adequate training and expertise in ballistics, they claim that his
opinion that the test bullets and cartridges matched the slugs and cartridges recovered from
Sixth, in the present case, appellants contend that the PNP cannot be presumed to have done
the scene of the crime was not reliable. Appellants also assail Caser's failure to take the
their work since it committed errors and blunders in transferring possession and custody of the
necessary photographs to support his findings.
physical evidence. They allege there was a possibility that the evidence was tainted, planted,
or manufactured. Besides, appellants point out that the presumption of regularity cannot
prevail over the constitutional presumption of innocence of the accused.

An expert witness is "one who belongs to the profession or calling to which the subject matter
of the inquiry relates and who possesses special knowledge on questions on which he
proposes to express an opinion."38 There is no definite standard of determining the degree of
The record shows that the police officers did not issue acknowledgment receipts in some
skill or knowledge that a witness must possess in order to testify as an expert. It is sufficient
instances. However, minor lapses do not mean that the State had failed to show an unbroken
that the following factors be present: (1) training and education; (2) particular, first-hand
chain of custody of the subject firearms and ammunition, nor that said firearms and
familiarity with the facts of the case; and (3) presentation of authorities or standards upon
ammunition were tampered. The slugs and spent shells recovered from the scene of the crime
which his opinion is based.39 The question of whether a witness is properly qualified to give
and the victim's corpse were plainly identified in open court by the PNP investigators. The
an expert opinion on ballistics rests with the discretion of the trial court.40
ballistician testified that the bullets and cartridges recovered from the crime scene had been
fired from the subject handguns. Under these circumstances, we must respect the
presumption of the regularity in the performance of duties.

In giving credence to Caser's expert testimony, the trial court explained:

Seventh, appellants insist that the prosecution failed to show that the red "Jiffy" used by them
and seized by the police officers was the same vehicle used by the gunmen who killed
The defense downgraded the capability of Caser in forensics ballistics and identifying firearms. Alejandro Flores. Appellants point out that PO3 Rustela, who was aboard police car No. 201,
Much stress is given to the absence of photographs of his examination. Nonetheless, the Court testified that they lost sight of the red "Jiffy" while chasing it along Leon Kilat Street.
is satisfied (with) Caser's examination, findings and conclusions with the use of a microscope. Appellants argue that the "Jiffy" which was chased by patrol car No. 208 until it was cornered
Caser's conclusion based on his examination deserves credit. He found the impressions on the near BBRC by the other pursuing patrol cars was not the same vehicle originally sighted and
primer of the fired cartridges that were test-fired to have the same characteristics with those tailed by patrol car No. 201.
recovered at the scene of the crime. Whenever a triggerman pumps a bullet (into) the body of
his victim, he releases a chunk of concrete evidence that binds him inseparably to his act.
Every gun barrel deeply imprints on every bullet its characteristic marking peculiar to that gun
and that gun alone. These marking might be microscopic but they are terribly vocal in
In rejecting this theory, the trial court stated that:
announcing their origin. And they are as infallible for purposes of identification, as the print
left by the human finger.41

306
RECTO, GAYLE ANGELI M.
2011-0008 | AUSL
Personal Notes on Remedial Law 2 Review (based on the syllabus of Prof. Henedino M. Brondial)

. . . PO3 Rustela who was nearby, immediately ran to the scene of the crime and met the red
jiffy with three persons on board, that speedily passed by him proceeding towards Leon Kilat
Street. Car 208 readily picked up the trail and pursued the red jiffy from Leon Kilat, then These unbroken chain of events prove not only appellants' identities but also their participation
making abrupt turns on downtown streets until other patrol cars joined the chase and and collective responsibility in the murder of Alejandro Flores. They reveal a unity of purpose
captured them in Lahug, near the BBRC. The identity of the red jiffy was never interrupted. and concerted action evidencing their conspiracy to kill him. Against this matrix of facts and
Members of the Mobile Patrol Cars identified in court without batting an eyelash, the red jiffy circumstances, appellants' bare denials cannot stand. Their story of chasing a red "Jiffy" is
which was the object of the shooting alarm. There was no interruption, no let-up in the chase, merely a disingenuous diversion of no evidentiary value for the defense.
right after Alejandro Flores was shot and there was no other red jiffy that the crews of the
(pursuing) patrol cars noticed.

Finally, the information for murder alleged treachery and evident premeditation. We note,
though, that the trial court did not state which circumstance qualified the killing into murder.
The Court rejects their claim of innocence, for their very acts belied the same.

A review of the record would reveal that there was no evident premeditation. There is evident
Astellero could have stopped the jeep upon noticing that patrol cars were already running premeditation when the following are shown: (a) the time when the accused determined to
after them with sirens, blinkers and warning shots fired. From Leon Kilat Street to Lahug commit the crime; (b) an act or acts manifestly indicating that the accused has clung to his
airport, there were several police stations that they could have sought shelter and police determination; and (c) a lapse of time between the determination to commit the crime and the
assistance. Guilt has many ways of surfacing. Instead of stopping, Abriol ordered Astellero to execution thereof sufficient to allow him to reflect upon the consequences of his act.54
accelerate their speed. Their obvious purpose was to elude the patrol cars. Flight is indicative Evident premeditation indicates deliberate planning and preparation. Nowhere in the record is
of guilt.50 it shown when and how appellants planned and prepared to kill the victim.

But, in this case, is the totality of the circumstantial evidence relied upon by the trial court Concerning treachery, however, it was shown that: (1) the means of execution employed gave
sufficient to support a conviction? the person attacked no opportunity to defend himself or retaliate; and (2) the means of
execution was deliberately or consciously adopted.55 These twin requisites were adequately
proved.

Circumstantial evidence is that which indirectly proves a fact in issue. For circumstantial
evidence to be sufficient to support a conviction, all the circumstances must be consistent with
each other, consistent with the theory that the accused is guilty of the offense charged, and at Appellants had superiority in numbers and weapons. The victim was without any means to
the same time inconsistent with the hypothesis that he is innocent and with every other defend himself as no weapon was found or even intimated to be in his possession. The victim
possible, rational hypothesis, except that of guilt.51 An accused can be convicted on the basis was running away from the "Jiffy" prior to the killing. That he was warned or threatened
of circumstantial evidence where all the circumstances constitute an unbroken chain leading to earlier is of no moment. Even when the victim is warned of danger to his person, if the
one fair and reasonable conclusion pointing to the accused, to the exclusion of all others, as execution of the attack made it impossible for the victim to defend himself or to retaliate,
the culprit.52 treachery can still be appreciated.56 The victim was lying prostrate on the ground when he
was deliberately and mercilessly riddled with bullets. The weapons used, the number of
assailants, the swift and planned manner of the attack, and the multiple number of wounds
inflicted upon the victim all demonstrate a determined assault with intent to kill the victim. No
In our assessment, the prosecution's evidence constitutes an unbroken chain of events leading doubt there was treachery.
to the inevitable conclusion of guilt on the part of appellants. First, the fatal shooting of
Alejandro Flores occurred at around 11:50 P.M. of June 5, 1993 in front of the ABS-CBN
compound in Cebu City. The gunman, who was tall and thin, alighted from a red "Jiffy,"
pumped several bullets into the prone victim, and got back aboard the "Jiffy" which then sped B. Criminal Case No. CBU-33664
towards Leon Kilat Street. Second, eyewitness Romeo Sta. Cruz, Jr.'s description of the
gunman as "tall and thin" perfectly matches the physique of appellant Abriol. Third, PO3
Alexander Rustela, who was close to the crime scene, heard the gunshots and ran towards the
place where the sound of gunshots emanated. A red "Jiffy" with three persons aboard whizzed
On their conviction for illegal possession of firearms, appellants contend that the handguns
by him and abruptly turned at Leon Kilat Street. After Sta. Cruz, Jr. informed him that the
and ammunition allegedly taken from them by the police officers were illegally seized. They
gunmen were aboard a red "Jiffy," Rustela boarded patrol car No. 201, radioed an alarm, and
assert that the police had no warrant to effect a search and seizure, such that these illegally
commenced a pursuit of the fleeing vehicle. Police car no. 208 received the alarm, and on
seized firearms were inadmissible as evidence, and it was error for the trial court to admit
turning into Leon Kilat Street, encountered the speeding red "Jiffy." They immediately chased
them.
the "Jiffy" but failed to catch it. Police cars Nos. 208 and 205 cornered the vehicle in front of
the Don Bosco building near BBRC. PO2 Gerald Cue, on patrol car no. 205 fired a warning shot
at the vehicle and directed all those aboard to disembark. Three men got out, with their hands
raised. SPO1 Abrigana, on patrol car no. 208 and PO2 Cue approached the trio. Abrigana
frisked the man who was seated in the front passenger seat, who turned out to be appellant There are eight (8) instances where a warrantless search and seizure is valid. They are: (1)
Abriol, and recovered from his waist a .38 caliber revolver with six empty shells. Cue searched consented searches;57 (2) as an incident to a lawful arrest;58 (3) searches of vessels and
the red "Jiffy" and found two loaded .45 caliber pistols under the front seat where Abriol had aircraft for violation of immigration, customs, and drug laws;59 (4) searches of moving
sat. Other police officers immediately went to the crime scene where they found the victim vehicles;60 (5) searches of automobiles at borders or constructive borders; (6) where the
barely alive. PO3 Seville retrieved four .45 caliber slugs and two deformed slugs at the spot prohibited articles are in "plain view;"61 (7) searches of buildings and premises to enforce fire,
where the victim was shot. The autopsy of the victim's remains showed that he died of cardio sanitary, and building regulations; and (8) "stop and frisk" operations.62
respiratory arrest due to shock and hemorrhage secondary to gunshot wounds. A deformed
metal jacket of a .38 caliber slug was recovered from the corpse. Ballistics tests showed that
the bullets and cartridges had identical individual characteristics with those of the test bullets
and cartridges. Paraffin tests conducted on each of the appellants, one day after the incident,
In this case, the warrantless search and seizure of the subject handguns and ammunition is
revealed that all were positive for gunpowder residues. The subject firearms were also
valid for two reasons. It was a search incidental to a lawful arrest. It was made after a fatal
chemically examined and found positive for gunpowder residue. Before the shooting incident,
shooting, and pursuit of a fast-moving vehicle seeking to elude pursuing police officers, and a
appellants were seen at Navales' house until around 7:30 P.M., when they left aboard Navales'
more than reasonable belief on the part of the police officers that the fleeing suspects aboard
red "Jiffy" with Astellero driving, Abriol in the front passenger seat, and Dosdos in the back
said vehicle had just engaged in criminal activity. The urgent need of the police to take
seat.53 Appellants' seating arrangements were exactly the same, several hours later, after
immediate action in the light of the foregoing exigencies clearly satisfies the requirements for
they were pursued and cornered by police cars near BBRC. Appellants admitted that they
warrantless arrests under the Rules of Court.63 Moreover, when caught in flagrante delicto
dropped by the Navales residence at around 7:00 P.M. and 11:00 P.M.
with firearms and ammunition which they were not authorized to carry, appellants were
actually violating P.D. No. 1866, another ground for valid arrest under the Rules.64
307
RECTO, GAYLE ANGELI M.
2011-0008 | AUSL
Personal Notes on Remedial Law 2 Review (based on the syllabus of Prof. Henedino M. Brondial)

8294, which amended said decree, should be applied to appellants retroactively, citing People
v. Molina, 292 SCRA 742, 779 (1998) interpreting R.A. No. 8294.
Appellants further contend that the trial court erred in convicting appellants Astellero and
Dosdos of illegal possession of firearms. They point out that the .38 caliber revolver was
recovered from appellant Abriol, who as a policeman was authorized to carry and possess said
firearm, as evidenced by his Memorandum Receipt (MR), which had "not been recalled, We agree. We ruled in Molina that with the passage of R.A. No. 8294 on June 6, 1997, the use
cancelled or revoked until the time of the trial of these cases." Appellants claim that the two of an unlicensed firearm in murder or homicide is not a separate crime, but merely a special
.45 caliber pistols could have been left in the vehicle by PNP personnel assigned at BBRC, aggravating circumstance. This was recently reiterated in People v. Castillo, G.R. Nos. 131592-
considering that the red "Jiffy" was generally used as a service vehicle by BBRC personnel. 93, February 15, 2000.72 Appellants are thus guilty only of murder with the special
They also argue that the prosecution failed to prove appellants' ownership, control, and aggravating circumstance of use of unlicensed firearms. The imposition of the penalty of
possession of the .45 caliber pistols, considering that appellants were six meters away from reclusion perpetua cannot however be modified since the murder took place before the
the "Jiffy" when said handguns were allegedly found. effectivity of R.A. No. 7659.

To sustain a conviction for violation of P.D. No. 1866, the prosecution must prove two A final word on the damages. In addition to the award of P50,000 as indemnity ex delicto, the
elements of the offense: (1) the existence of the subject firearm; (2) the fact that the accused trial court awarded P30,000 in actual damages, "representing a reasonable amount for the
who owned or possessed the firearm does not have the corresponding license or permit to embalming, vigil, wake and burial expenses," and P30,000 as attorney's fees. To be entitled to
possess it.65 These the prosecution did. It presented a .38 caliber revolver with serial number actual damages, it is necessary to prove the actual amount of loss with a reasonable degree of
PO8445, a .45 caliber pistol with serial number PGO 13506 Para Ordinance, and a .45 caliber certainty, premised upon competent proof, and on the best evidence obtainable by the injured
pistol with serial number 52469. The .38 caliber handgun was recovered from appellant Abriol, party.73 No such evidence was offered. The award of actual damages must, therefore, be
while the two .45 caliber automatics were found and seized from under the front passenger deleted. However, temperate damages may be awarded since the family of the victim has
seat of appellants' vehicle. SPO4 Aquilles Famoso of the Cebu City PNP Metropolitan District demonstrably spent for the wake, funeral and burial arrangements. The amount of P20,000
Command's Firearms and Explosive Unit testified that appellants were not listed as licensed should suffice as temperate damages. In addition, we find an award of exemplary damages in
firearm owners in Cebu City.66 The prosecution also presented a certification from P/Senior order, pursuant to Article 2230 of the Civil Code.74 The killing was attended by the special
Inspector Edwin Roque of the Firearms and Explosives Division of PNP Headquarters at Camp aggravating circumstance of use of unlicensed firearms. Moreover, the public good demands
Crame, Quezon City that appellant Abriol is not licensed to hold any firearm; that the .45 that detained prisoners should not abuse their status as "trustees." Had the police been
caliber pistols were unlicensed; and that a certification from the PNP Firearms and Explosives unsuccessful in their pursuit of appellants, the latter would have used the BBRC as shelter and
Office attesting that a person is not a licensee of any firearm, proves beyond reasonable doubt as an alibi that they could not have committed the crime since they were then in detention.
the second element of illegal possession of firearm.67 Thus, we find an award of P10,000 as exemplary damages in order. Accordingly, the award of
attorney's fees is sustained.75

Abriol insists that he had a valid MR authorizing him to carry the .38 revolver. We agree with
the observation of the trial court that: WHEREFORE, the assailed Decision of the Regional Trial Court of Cebu City, Branch 10, in
Criminal Cases Nos. CBU-30350 and CBU-33664 is hereby MODIFIED. Appellants Albert Abriol,
Macario Astellero, and Januario Dosdos are hereby found GUILTY of murder, qualified by
treachery, with the special aggravating circumstance of use of unlicensed firearms and are
The claim of Abriol that .38 caliber was issued to him, as evidenced by the corresponding hereby sentenced to suffer the penalty of reclusion perpetua with the accessory penalties
receipt (MR), is of no moment. While an MR is an authority of Abriol to possess the provided for by law. Appellants Abriol, Astellero, and Dosdos are also ordered to pay, jointly
government firearm that was issued to him, when he was charged and detained at BBRC for and severally, the heirs of Alejandro Flores the sum of P50,000 as death indemnity, P20,000
an earlier case of murder, other than the case at bar, he was already then at that moment a as temperate damages, P10,000 as exemplary damages, and P30,000 as attorney's fees, as
detained prisoner and therefore, (un)authorized to carry a firearm. A military man or a well as the costs.
member of the PNP who commits a crime, is immediately disarmed upon his arrest and
stripped of all the rights and privileges that go with the function of his office, and this includes,
in the case of Abriol, his MR. Thus, when he shot Alejandro Flores with his .38 caliber revolver,
this firearm was already unauthorized and its use and possession illegal.68 SO ORDERED.

Even if Abriol's MR was valid, said authorization was limited only to the .38 caliber revolver Bellosillo, Mendoza, Buena, and De Leon, Jr., JJ., concur.
and not the two .45 caliber automatic pistols found under the front passenger seat of the
"Jiffy." Appellants were still in the unlawful possession of the .45 caliber pistols. Under P.D.
No. 1866, possession is not limited to actual possession.69 In this case, appellants had control
over the pistols. They were all liable since conspiracy was established and the act of one is the
act of all.70 FIRST DIVISION

G.R. No. 158015 August 11, 2004

Appellants claim that they were six meters away from the "Jiffy" when it was searched and the LAURA and ERIBERTO BAUTISTA, petitioner,
two .45 caliber pistols were seized. They suggest that the policemen who searched the vehicle
could have planted said firearms. The trial court found that they were in fact only one meter vs. HON. COURT OF APPEALS and FERNANDO MORELOS, respondents.
away from the vehicle. Findings of fact of the trial court, when supported by the evidence on
record, are binding and conclusive upon appellate courts.71 YNARES-SANTIAGO, J.:

All told, on the charge of illegal possession of firearms, no reversible error was committed by
Facts
the trial court when it found appellants guilty beyond reasonable doubt.

* The dispute involves a parcel of land situated along Maceda (formerly Washington) Street,
Sampaloc, Manila, containing an area of approximately 105 square meters. This parcel of land
was previously owned and registered in the name of the late Cesar Morelos under Transfer
The Office of the Solicitor General recommends that although appellants were charged with Certificate of Title No. 27604. Cesar is the uncle of petitioner Laura Morelos Bautista, being the
and convicted of two separate offenses of murder and violation of P.D. No. 1866, R.A. No. brother of her mother, Rosario Morelos.3

308
RECTO, GAYLE ANGELI M.
2011-0008 | AUSL
Personal Notes on Remedial Law 2 Review (based on the syllabus of Prof. Henedino M. Brondial)

* Cesar, who was married to Rosario Duran, did not have any children. Rosario died in 1972. Another witness, Major Braulio Monge, Chief of the Fingerprint Division of the PC-INP, testified
Cesar died of cardiac arrest on April 15, 1982. During his lifetime, Cesar sold and conveyed the that the thumbmark of Cesar Morelos appearing on the residence certificate indicated in the
above-mentioned parcel of land in favor of petitioner Laura Morelos Bautista, as evidenced by Deed of Absolute Sale, when compared to those affixed on previous residence certificates
a "Deed of Absolute Sale" notarized by Luis M. de Guzman. Accordingly, Transfer Certificate of issued to the decedent, did not match and appears to be the thumbmark of another person.
Title No. 254843 was issued in the name of petitioner Laura Bautista.4

* Respondent Fernando Morelos, claiming to be the illegitimate child of Cesar Morelos with
Angelina Lim-Gue, instituted a complaint for the declaration of nullity of sale and title with Under Rule 132, Section 22 of the Rules of Court, the genuineness of handwriting may be
damages, docketed as Civil Case No. 83-17900, before the Regional Trial Court of Manila, proved in the following manner: (1) by any witness who believes it to be the handwriting of
Branch VII. At the trial, he presented testimonies of expert witnesses who claimed that the such person because he has seen the person write; or he has seen writing purporting to be his
signature of Cesar Morelos on the Deed of Absolute Sale and the fingerprint appearing on his upon which the witness has acted or been charged; (2) by a comparison, made by the witness
Residence Certificate were not his.5 or the court, with writings admitted or treated as genuine by the party, against whom the
evidence is offered, or proved to be genuine to the satisfaction of the judge.
* Petitioners countered that the Deed of Absolute Sale was valid. The witness to the Deed,
Carmelita Marcelino, testified that she saw Cesar Morelos and petitioner Laura Bautista sign
the same.6
It is well-settled that a duly notarized contract enjoys the prima facie presumption of
* After hearing, the court a quo rendered judgment declaring the Deed of Sale dated April 5, authenticity and due execution as well as the full faith and credence attached to a public
1982 executed between the late Cesar Morelos in favor of Laura Bautista valid, and dismissed instrument.11 To overturn this legal presumption, evidence must be clear, convincing and
for insufficient evidence the claims and counterclaims for damages of the parties.7 more than merely preponderant to establish that there was forgery that gave rise to a
spurious contract.
* Respondent appealed to the Court of Appeals, which reversed and set aside the judgment of
the trial court. The dispositive portion of the CA decision reads:

WHEREFORE, premises considered, the appealed decision is hereby REVERSED As a general rule, forgery cannot be presumed and must be proved by clear,
AND SET ASIDE. In lieu thereof, another one is entered declaring AS NULL AND positive and convincing evidence. The burden of proof lies on the party alleging
VOID the Deed of Absolute Sale, dated April 5, 1982, executed between the late forgery. In Heirs of Severa P. Gregorio v. Court of Appeals,12 we held that due to
Cesar G. Morelos and defendant-appellee Laura R. Bautista. The Register of the technicality of the procedure involved in the examination of the forged
Deeds of Manila is DIRECTED to cause the cancellation of Transfer Certificate of documents, the expertise of questioned document examiners is usually helpful;
Title No. 154043 in the name of defendant-appellee LAURA R. BAUTISTA and to however, resort to questioned document examiners is not mandatory and while
issue another one in the name of the ESTATE OF CESAR G. MORELOS. probably useful, they are not indispensable in examining or comparing
Defendants-appellees are also directed to surrender possession of the disputed handwriting.
property to plaintiff-appellant.

SO ORDERED.8
Hence, a finding of forgery does not depend entirely on the testimony of
* Petitioners' motion for reconsideration was denied. handwriting experts. Although such testimony may be useful, the judge still
exercises independent judgment on the issue of authenticity of the signatures
* Hence, this petition for review on certiorari. under scrutiny; he cannot rely on the mere testimony of the handwriting expert.13

Issues The authenticity of signatures is not a highly technical issue in the same sense that
questions concerning, e.g., quantum physics or topology or molecular biology,
I. WHETHER OR NOT THE TESTIMONIES OF EXPERT WITNESSES ARE CONCLUSIVE TO BE A would constitute matters of a highly technical nature. The opinion of a handwriting
STRONG BASIS TO NULLIFY A DULY EXECUTED AND NOTARIZED DEED OF ABSOLUTE SALE. expert on the genuineness of a questioned signature is certainly much less
compelling upon a judge than an opinion rendered by a specialist on a highly
technical issue.14
II. WHETHER OR NOT THE DEED OF ABSOLUTE SALE (ANNEX "3") IS VALID.

III. WHETHER OR NOT PRIVATE RESPONDENT HAS THE LEGAL PERSONALITY TO SEEK THE
ANNULMENT OF THE DEED OF ABSOLUTE SALE.9
In the case at bar, the presumption of validity and regularity prevails over
allegations of forgery and fraud. As against direct evidence consisting of the
testimony of a witness who was physically present at the signing of the contract
and who had personal knowledge thereof, the testimony of an expert witness
Ruling constitutes indirect or circumstantial evidence at best. Carmelita Marcelino, the
witness to the Deed of Absolute Sale, confirmed the genuineness, authenticity and
Petitioner asserts the validity of the Deed of Absolute Sale and invoke the testimony of due execution thereof.15 Having been physically present to see the decedent Cesar
Carmelita Marcelino, the instrumental witness to the signing of the document, who confirmed Morelos and petitioner Laura Bautista affix their signatures on the document, the
that it was the decedent Cesar Morelos who affixed his signature to the document. weight of evidence preponderates in favor of petitioners.

On the other hand, respondent contends that the decedent's signature on the Deed was Witness Francisco Cruz, Jr. failed to establish the fact that the signature on the
forged. He presented the testimony of Francisco Cruz, Jr., Chief Examiner of the PC-INP Crime Deed of Absolute Sale was not that of Cesar Morelos. He merely concluded that the
Laboratory Service, that the signature of decedent on the questioned instrument, when document was a forgery without citing any factual basis for arriving at that
compared to other documents bearing the authentic signature of Cesar Morelos, did not match conclusion. Cruz did not point out distinguishing marks, characteristics and
and appeared to have been authored by a different person. Cruz, Jr. declared that the latest discrepancies in and between genuine and false specimens of writing, which would
document bearing the genuine signature of the decedent is dated March 31, 1982, while the ordinarily escape detection by an ordinary lay person.16
alleged forged signature was made on April 5, 1982, or a mere lapse of five days. According to
him, it is not possible to have significant variation between the two signatures, considering the
proximity of time when the signatures where affixed.10

When the trial court and the appellate court arrived at divergent factual
assessments in their respective decisions and the bases therefor refer to
documents made available to the scrutiny of both courts, the well-settled rule that
309
RECTO, GAYLE ANGELI M.
2011-0008 | AUSL
Personal Notes on Remedial Law 2 Review (based on the syllabus of Prof. Henedino M. Brondial)

factual findings of trial courts deserve respect and even finality will not apply.17 In * Around 2:00 o’clock in the afternoon of September 2000, Renato Sosas y Verzosa, an
the case at bar, the varying factual assessments pertained to the authenticity of employee of appellant Bobby Avelino y Bulawan in his wood business, was directed by
the signature of the late Cesar Morelos on the questioned Deed of Absolute Sale appellant to summon Toto Mata, Nato, Domeng Bakukang, Bobot Tuwad, Boy Muslim, Angkol,
conveying the 105-square meter property in favor of his niece, Laura Bautista. Charlie, Sonny Muslim and Mon (TSN, January 29, 2002, pp. 5-6). An hour later, the group
called by Renato Sosas met at appellant’s warehouse in Tagaytay, Baseco Compound, Tondo,
Manila. Renato Sosas, who was just a step away from the group, was astounded when he
heard appellant utter "Papatayin si Chairman." Bobot Tuwad reacted by asking appellant "Sino
pong chairman?", to which appellant Avelino replied "Sino pa, Ninong Chairman Gener."
In Jimenez v. Commission on Ecumenical Mission and Relations of the United
Terrified, Sosas kept mum about what he discovered (TSN, January 29, 2002, pp.10-12).
Presbyterian Church in the United States of America,18 we held that the
authenticity of a questioned signature cannot be determined solely upon its
general characteristics, similarities or dissimilarities with the genuine signature. * On October 5, 2000, around 9:00 o’clock in the evening, Alfredo Manalangsang was riding
Dissimilarities as regards spontaneity, rhythm, pressure of the pen, loops in the on a tricycle going to Baseco Compound, Tondo, Manila. Since Manalangsang was the last
strokes, signs of stops, shades, that may be found between the questioned passenger to board the tricycle, he sat behind the driver. Upon reaching a certain point
signature and the genuine one are not decisive on the question of the former's between Muelle Del Rio and 2nd Street, Port Area, Manila, the tricycle which Manalangsang
authenticity. The result of examinations of questioned handwriting, even with the was riding on passed at the left lane instead of the right lane of the road to give way to the
benefit of aid of experts and scientific instruments, is, at best, inconclusive. There owner-type (sic) jeep owned by the barangay and driven by its Chairman, Generoso Hispano,
are other factors that must be taken into consideration, such as the position of the herein victim (TSN, September 26, 2001, pp. 11-17, Exhs. "R-1" and "4").
writer, the condition of the surface on which the paper where the questioned
signature is written, his state of mind, feelings and nerves, and the kind of pen and * While Chairman Hispano was entering the nearest route near the center island, a man
paper used. These play an important role on the general appearance of the suddenly emerged and blocked Chairman Hispano’s vehicle. Instantaneously, Manalangsang
signature. Unless, therefore, there is, in a given case, absolute absence, or heard bursts of gunshot which prompted him to jump from the tricycle. Manalangsang
manifest dearth, of direct or circumstantial competent evidence on the character of instinctively hid behind the center island of the road (TSN, September 26, 2001, pp. 17-21).
a questioned handwriting, much weight should not be given to characteristic
similarities, or dissimilarities, between a questioned handwriting and an authentic * At this juncture, Manalangsang peeped at the direction of Chairman Hispano’s jeep and saw
one. three (3) men wearing bonnets, two of whom were strategically blocking the jeep of Chairman
Hispano. The third man, who was wearing a green jacket and positioned himself near the
gutter, fired successive shots at Chairman Hispano and thereafter approached the jeep of
Chairman Hispano. He pulled down from the jeep the almost lifeless body of Chairman
Besides, a notarial document is evidence of the facts in the clear unequivocal manner therein Hispano. Since Manalangsang was situated near the third assailant, he failed to identify the
expressed and has in its favor the presumption of regularity.19 The authenticity and due other two assailants. However, Manalangsang positively identified the third assailant as
execution of the Deed of Absolute Sale must therefore be upheld. appellant Bobby "Abel" Avelino, whom he saw stooping down at the Chairman’s body and
pulling the opening of his bonnet down to his chin to ascertain if the Chairman was still alive.
Sensing that it was safe for him to leave the scene, Manalangsang boarded a tricycle again
and went home (TSN, September 26, 2001, pp. 22-26).

As to the alleged insufficient consideration of the sale of the property, the mere inadequacy of
* Thereafter, appellant and the other assailants drove away using the owner-type jeep of
the price does not affect its validity when both parties are in a position to form an independent
Chairman Hispano. However, on their way towards Divisoria, the jeep was incidentally blocked
judgment concerning the transaction,20 unless fraud, mistake or undue influence indicative of a
by a tricycle and a white car which prompted the companion of appellant to shout "tabi-tabi."
defect in consent is present.21 A contract may consequently be annulled on the ground of
At that moment, Mary Ann Ca[ñ]ada saw appellant, who was wearing a green jacket and a
vitiated consent and not due to the inadequacy of the price. In the case at bar, however, no
bonnet rolled up to his forehead, driving the owner-type (sic) jeep of Chairman Hispano.
evidence to prove fraud, mistake or undue influence indicative of vitiated consent was
Ca[ñ]ada readily recognized appellant as she was familiar with the face of appellant having
presented other than the respondent's self-serving allegations.
seen him driving the jeep of the Chairman on several occasions before (TSN, November 19,
2001, pp. 17-28).

* When the police arrived at the crime scene, Chairman Hispano was already dead. The
WHEREFORE, in view of the foregoing, the Petition is GRANTED. The Decision of the Court of
owner-type (sic) jeep of Chairman Hispano was recovered in front of house No. 440,
Appeals in CA-G.R. CV No. 45549 is REVERSED and SET ASIDE. The judgment of the Regional
Orbiztondo Street, Binondo, Manila, with several pieces of empty shells of 9 mm caliber gun
Trial Court of Makati, Branch VII in Civil Case No. 83-17900, declaring the Deed of Absolute
scattered on its floor (TSN, May 7, 2003, pp. 6-7) (Rollo, pages 120-123).6
Sale between petitioner Laura Morelos Bautista and Cesar Morelos over the subject parcel of
land covered by Transfer Certificate of Title No. 2760 as valid is REINSTATED. No costs.
* Petitioner Bobby "Abel" Avelino y Bulawan, together with Ricardo Tolentino, Alias Sonny
Muslim, Farouk Musa a.k.a. Boy Muslim, Alias Bubut Tuwad, Alias Angkol, Alias Mon, Renato
Meneses a.k.a. Nato, Benjamin Elbona a.k.a. Toto Mata, and Dominic Apan a.k.a. Domeng
Bakukang, was charged with murder4 before the Regional Trial Court (RTC) of Manila with the
SO ORDERED. qualifying circumstances of treachery and evident premeditation.

* Upon arraignment, petitioner and his co-accused Renato Meneses, Benjamin Elbona, and
Farouk Musa entered a plea of not guilty. The other accused remain at-large.
Davide, Jr., C.J., (Chairman), Quisumbing, Carpio, and Azcuna, JJ., concur.
* At the trial, the prosecution presented eight witnesses: Delia Hispano, the wife of
the victim; Diana Espinosa; Alfredo Manalangsang (Manalangsang); Mary Ann
Cañada (Cañada); Renato Sosas; Dr. Romeo T. Salen; P/Insp. Mario Prado; and
National Bureau of Investigation (NBI) agent Rizaldi Jaymalin.5
FIRST DIVISION

* Denying the accusation, the defense presented as evidence the testimonies of


G.R. No. 181444 July 17, 2013
petitioner, PO2 Anthony P. Galang, Adonis T. Bantiling and Scene of the Crime
Operative (SOCO) PSI Lito D. Cabamongan (Cabamongan).7
BOBBY "ABEL" AVELINO y BULAWAN, Petitioner,

* Petitioner advanced the defense of denial and alibi. He testified that on October
vs. PEOPLE OF THE PHILIPPINES, Respondent. 5, 2000, he and his wife went to the Land Transportation Office in Pasay City to
renew his license as they planned to go to Baguio that day. But as he was issued
VILLARAMA, JR., J.: a temporary license late in the afternoon, instead of going home, he and his wife
checked in at the Pharaoh Hotel in Sta. Cruz, Manila to spend the night. He
parked his car along Dasmariñas Bridge and slept. Later, he woke up to transfer
his car but his car was gone. Thus, he and his wife went to the police station in
Sta. Cruz, Manila then to the AntiCarnapping Unit along U.N. Avenue to report
Facts
the incident. At the latter location, they learned from a certain Tata Randy, an

310
RECTO, GAYLE ANGELI M.
2011-0008 | AUSL
Personal Notes on Remedial Law 2 Review (based on the syllabus of Prof. Henedino M. Brondial)

acquaintance and former police officer, that the victim had been gunned down. Manalangsang unequivocally identified the petitioner as the gunman. Manalangsang was able
Around 1:00 a.m., he and his wife returned to the hotel. On October 23, 2000, he to identify the petitioner because the latter revealed his face when he pulled down the bonnet
was arrested by agents of the NBI.8 he was wearing, thereby exposing his eyes, nose, mouth, and chin.16 Moreover, the certainty
of Manalangsang in identifying the petitioner as the one who shot Hispano is bolstered by the
* After trial, the RTC, on April 28, 2006, found petitioner guilty beyond reasonable doubt of fact that he and petitioner were neighbors for five years in Baseco.17 The RTC cites the
the crime of murder qualified by treachery, and imposed upon him the penalty of reclusion following statement by Manalangsang as an added indication of his certainty - "Si Avelino,
perpetua. The RTC likewise ordered him to indemnify the heirs of the victim Generoso Hispano kahit ubod ng layo, kahit naglalakad lang, kilala ko na. Dahil unang-una, matagal ko na siyang
(Hispano) the sum of P50,000 and to pay them an additional sum of P50,000 as moral kilala, dahil ako hindi niya ako gaanong kilala, pero sila kilala ko, kahit nakatagilid, kilala ko
damages, the sum of P158,471.75 as actual damages, and costs.9 siya."18 It cannot be denied that once a person gains familiarity of another, identification
becomes quite an easy task even from a considerable distance.19
* For failure of the prosecution to prove their guilt beyond reasonable doubt, accused Farouk
Musa, Benjamin Elbona, and Renato Meneses were acquitted of the crime charged.10

* As aforesaid, the CA, in its assailed decision, denied petitioner’s appeal and upheld the RTC Even the theory of the defense that identification of the petitioner by Manalangsang and
decision with modification by increasing the award of actual damages to P171,128.75.11 Cañada is unlikely due allegedly to the lack of sufficient illumination at the scene of the crime,
Petitioner’s motion for reconsideration was likewise denied by the appellate court on January has been overcome by the fact that there are lampposts and signboards in the subject area
25, 2008.12 which can provide illumination despite the black of night. Indeed, even assuming arguendo
that the lampposts were not functioning at the time, the headlights of passing vehicles
provided sufficient illumination at the crime scene.20 "The Court has previously held that the
* Aggrieved, petitioner now seeks to reverse his conviction, arguing that the CA erred in
light from the stars or the moon, an oven, or a wick lamp or gasera can give ample
relying on the testimonies of the prosecution witnesses Manalangsang and Cañada and
illumination to enable a person to identify or recognize another."21 Similarly, the headlights of
disregarding the inconsistencies between the statements of Manalangsang and the findings of
vehicles are sufficient to enable eyewitnesses to identify individuals at a distance of four to ten
the medico-legal and SOCO PSI Cabamongan as to the position of the gunman. He also
meters,22 and it should be noted that the distance between Manalangsang and the jeep
reiterated his defense of denial and alibi.
where Hispano was felled was only 31 feet23 or a little over nine meters.24

Ruling
The identification made by Manalangsang was likewise sufficiently corroborated by the
testimony of Cañada, that she saw the petitioner, with whom she was familiar, drive away in
We have carefully studied the records of this case and find no cogent reason to overturn the
Hispano’s owner-type jeep, wearing a green jacket and black bonnet rolled up to his
ruling of the CA which is in accord with law and jurisprudence.
forehead.25

As for the defense of the petitioner which is grounded, firstly, upon denial and alibi, basic is
Further, as can be gleaned from the excerpt below, the petitioner’s defense that
the rule that the defense of denial and alibi cannot prevail over the witness’ positive
Manalangsang’s testimony contradicts with the medical findings, and should then be
identification of the accused-appellants.13 Moreover, as oft-repeated in jurisprudence
disregarded, must fail. Petitioner claims that Manalangsang’s statements that Hispano was
shot in a downward direction conflict with the findings of the medico-legal that the trajectory
of the bullets is in an upward direction. The testimony of Dr. Salen is pertinent and
enlightening:
For alibi to prosper, it is not enough to prove that appellant was somewhere else when the
crime was committed; he must also demonstrate that it was physically impossible for him to
have been at the scene of the crime at the time of its commission. Unless substantiated by
clear and convincing proof, such defense is negative, self-serving, and undeserving of any
Q: The trajectory of the bullet is upward?
weight in law. Denial, like alibi, as an exonerating justification, is inh erently weak and if
uncorroborated regresses to blatant impotence. Like alibi, it also constitutes self-serving
A: Yes, sir.
negative evidence which cannot be accorded greater evidentiary weight than the declaration of
credible witnesses who testify on affirmative matters.14

Q: So the gunman must be at a lower level from the decease[d]?


In this case, the defense failed to establish that it was physically impossible for the petitioner
to have been at the scene of the crime at the time of its commission. Pharaoh Hotel, where A: We can not [sic] say that, sir.
petitioner claims to have stayed with his wife at the time of the commission of the crime, is in
Sta. Cruz, Manila.15 The said hotel is not so far from the scene of the crime, which is in
Baseco Compound in Tondo, Manila, so as not to afford the petitioner an opportunity to easily
go to the place of the shooting at the time Hispano was killed. Indeed, for the defense of alibi to Q: But the trajectory of the bullet is upward?
prosper, the accused must prove (a) that he was present at another place at the time of the
perpetration of the crime, and (b) that it was physically impossible for him to be at the scene
A: It depends on the matter of the position of the head when the head was hit. It could be
of the crime. These, the defense failed to do.
when the trajectory is upward it [sic] could be lying down with his back and the gunman and
the barrel of the gun is here and if we will put the normal position of the body it is still upward
but the normal position is like that so…

The defense of the petitioner is based, secondly, on his allegations that prosecution witnesses
Manalangsang and Cañada failed to positively identify him as the gunman who mortally
wounded Hispano, and that Manalangsang’s testimony as to the locations and number of
COURT: Make of record that the witness is demonstrating a slightly incline position of the head
gunshot wounds, as well as the position of the gunman, is inconsistent with the physical
and the body.
evidence as provided by the medico-legal officer and the testimony of SOCO PSI Cabamongan.

WITNESS:
These allegations cannot exculpate the petitioner from criminal liability.

A: So we can not [sic] determine the position of the gunman when it was related [sic] the
gunshot wound of the entry and the victim it will depend on the position of the gunman but
likewise the position of the victim during the infliction [sic] of the gun.

311
RECTO, GAYLE ANGELI M.
2011-0008 | AUSL
Personal Notes on Remedial Law 2 Review (based on the syllabus of Prof. Henedino M. Brondial)

means, method and manner of execution were deliberately and consciously adopted by the
offender.30 The two elements are present in this case.
ATTY. VARGAS:

Q: Mr. Witness, if the gunman is standing on an elevated floor of about three feet do you think
that the trajectory of a bullet is upward? These elements are established by the testimony of Manalangsang showing the unexpected
attack by the petitioner on the unsuspecting Hispano whose vehicle was suddenly blocked by
A: It is possible also.26 three men, at least one of whom was armed with a firearm.31 The victim was then unarmed
and had no opportunity to defend himself.

Clearly, the fact that the trajectory of the bullets is in an upward direction does not negate the
veracity of Manalangsang’s statement that Hispano was shot by the gunman from an elevated Thus, considering all the above-mentioned facts, we uphold the conviction of the petitioner for
plane. the crime of murder.

The CA was also correct in not giving credence to the opinion of SOCO PSI Regarding the award of damages, we affirm the trial court and CA in ordering the petitioner to
Cabamongan as regards the position of the gunman when the latter shot Hispano. pay the heirs of Generoso Hispano the amount of P50,000 as moral damages. In cases of
Cabamongan asserted that the gunman was on board the owner-type jeep when murder and homicide, the award of moral damages is mandatory, without need of allegation
Hispano was shot, which is opposed to Manalangsang’s testimony. However, case and proof other than the death of the victim.32 Similarly, the CA correctly awarded his heirs
records reveal that Cabamongan was presented as an ordinary witness. Hence, his the amount of P171,128.75 as actual damages, as said amount which was spent for funeral
opinion regarding the location of the gunman in relation to the place where the and burial expenses was duly supported by receipts. However, as regards the award of civil
empty shells were found is immaterial. indemnity, the same should be increased to P75,000 to conform with recent jurisprudence.33
Also, the heirs of the victim are entitled to exemplary damages which recent jurisprudence
pegs at P30,00034 considering the presence of the aggravating circumstance of treachery.
Lastly, we impose on all the monetary awards for damages interest at the legal rate of 6% per
annum from date of finality of this Decision until fully paid, consistent with current policy.
Expert evidence is admissible only if: (a) the matter to be testified to is one that
requires expertise, and (b) the witness has been qualified as an expert.27 In this
case, counsel for the petitioner failed to make the necessary qualification upon
presenting Cabamongan during trial.
WHEREFORE, the petition is DENIED. The October 22, 2007 Decision of the Court of Appeals
in CA-G.R. CR-H.C. No. 02297 is AFFIRMED. Petitioner BOBBY "ABEL" AVELINO y BULAWAN is
found GUILTY beyond reasonable doubt of MURDER and is sentenced to suffer the penalty of
reclusion perpetua. He is further ordered to pay the heirs of Generoso Hispano the amounts of
Jurisprudence further provides that minor inconsistencies in immaterial details do not destroy
P171,128.75 as actual damages, P75,000.00 as civil indemnity, P50,000.00 as moral damages,
the probative value of the testimony of a witness regarding the very act of the accused. The
and P30,000.00 as exemplary damages. All monetary awards for damages shall earn interest
case of Madali v. People28 elucidates thus:
at the legal rate of 6% per annum from date of finality of this Decision until fully paid.

Given the natural frailties of the human mind and its incapacity to assimilate all material details
With costs against the petitioner.
of a given incident, slight inconsistencies and variances in the declarations of a witness hardly
weaken their probative value. It is well settled that immaterial and insignificant details do not
discredit a testimony on the very material and significant point bearing on the very act of
accused-appellants. As long as the testimonies of the witnesses corroborate one another on
material points, minor inconsistencies therein cannot destroy their credibility. Inconsistencies Sereno, CJ., Leonardo-De Castro, Bersamin, Reyes, JJ., concur.
on minor details do not undermine the integrity of a prosecution witness. (Emphasis and
underscoring supplied.)

> Ordinary Witness


Thus, the positive identification of the petitioner as the gunman by Manalangsang, as
corroborated by Cañada, must stand. Indeed, it has been consistently held by this Court that SECOND DIVISION
in criminal cases the evaluation of the credibility of witnesses is addressed to the sound
G.R. No. 134074-75 January 16, 2001
discretion of the trial judge, whose conclusion thereon deserves much weight and respect
because the judge has the direct opportunity to observe said witnesses on the stand and
ascertain if they are telling the truth or not. Absent any showing that the lower courts PEOPLE OF THE PHILIPPINES, plaintiff-appellee,
overlooked substantial facts and circumstances, which if considered, would change the result
of the case, this Court gives deference to the trial court’s appreciation of the facts and of the vs. EMILIANO DURANAN, a.k.a "Kalbo,", accused-appellant.
credibility of witnesses, especially since Manalangsang and Cañada’s testimony meets the test
of credibility.29 The Court also notes that other than his claim of denial, petitioner failed to
MENDOZA, J.:
show how the prosecution failed to overcome the presumption of innocence.

The qualifying circumstance of treachery or alevosia was additionally properly appreciated in * The information in Criminal Case No. Q-94-55711 alleged ---
this case.
That on or about the 8th of March 1994, in Quezon City, Philippines, said
accused with lewd designs and by means of force and intimidation, to wit, by
then and there, willfully, unlawfully and feloniously taking advantage of
undersigned complainant, Maria Nympha Lozada y de Lara's feeblemindedness
The two elements that must be proven to establish treachery are: (a) the employment of
and thereafter have carnal knowledge with (sic) the undersigned complainant
means of execution which would ensure the safety of the offender from defensive and
against her will and without her consent.
retaliatory acts of the victim, giving the victim no opportunity to defend himself; and (b) the

Contrary to law2.
312
RECTO, GAYLE ANGELI M.
2011-0008 | AUSL
Personal Notes on Remedial Law 2 Review (based on the syllabus of Prof. Henedino M. Brondial)

* The information in Criminal Case No. Q-94-55712 averred --- There is an abundant growth of public hair. Labia minora are full
convex and gaping with the pinkish brown and congested labia
That on or about the 7th of March 1994, in Quezon City, Philippines, the said minor presenting in between. On separating, the same is disclosed
accused with lewd designs and by means of force and intimidation, to wit, did an abraded posterior fourchette and an elastic, fleshy type hymen
with shallow healing laceration at 5 o'clock position. External
then and there, willfully, unlawfully and feloniously taking (sic) advantage of the
undersigned (sic) feeblemindedness, and thereafter have carnal knowledge with vaginal orifice offers strong resistance to the introduction of the
(sic) the undersigned complainant against her will and without her consent. examining index finger and the virgin-sized vaginal speculum.
Vaginal canal is narrow with prominent rugosities.
Contrary to law3.

* Upon arraignment, accused-appellant pleaded not guilty to each charge of rape against him,
whereupon he was tried. CONCLUSION

* The prosecution presented three witnesses, namely, complainant Nympha Lozada y de Lara, Cervix is normal in size, color and consistency.
complainant's mother Virginia de Lara Lozada, and the attending medico-legal officer at Camp
Crame, Dr. Rosalina O. Cosidon.

* Complainant Nympha Lozada, who was 25 years old at the time of the Finding are compatible with recent loss of virginity.
incidents in question, is considered to be retarded and finished up to the sixth
grade only. She is unemployed and simply does household chores for her family.
Accused-appellant lived with the complainant's family in the same apartment in
K-6 No. 28 Kamuning St., Quezon City where he rented a room that he shared
Baring unforeseen complications, it is estimated that the above
with several other people.
injury will resolve in 7 to 9 days.

& The first rape took place in the afternoon of March 7, 1994.4 Nympha was
standing by the door of her grandfather's house when accused-appellant
suddenly placed his arm on her neck and dragged her inside the common
REMARKS
bathroom.5 Complainant said that accused-appellant kissed her and then
removed her shorts and underwear as he held her hands with his other hand.
She did not cry for help because accused-appellant threatened her that he would Vaginal and pre-urethral smears are negative for gram-negative
get angry if she did.6 she claimed that accused-appellant was able to rape her diplococci and for spermatozoa.15
while standing up despite her resistance.7 After the incident, complainant was
sent out of the bathroom and went directly home8. * Dr. Cosidon explained that the term "congested" used in reference to the labia
minora meant that there was some inflammation that could have been caused by
* The second incident occurred in the early morning of March 8, 1994,9 friction due to intercourse.16 Taken together with the presence of a shallow
according to complainant. She said she was cleaning the premises of her family hymenal laceration, this finding indicates the possibility of intercourse that
residence when accused-appellant pulled her from her house and took her to his caused complainant's loss of virginity within the last five days. 17
room. According to complainant, accused-appellant asked his brother, who was
then cooking, to leave the room. As soon as his brother had left, accused- * Accused-appellant filed a demurrer to the evidence, but the trial court denied it in its
appellant laid her on the floor and raped her.10 Complainant said she was forced November 17, 1995 order.18 The defense thereafter presented its witnesses, namely accused-
to submit to accused-appellant's lust because of his threats.11 After the incident, appellant Emiliano Duranan, accused-appellant's alleged roommates, Rico Bariquit and Carlito
accused-appellant sent her letters professing love her and telling her how Catubig, and his wife Carlita Duranan.
beautiful she was. Complainant said she tore up the letters after reading them.12
* With respect to the first incident of rape, which allegedly took place in the
* In another incident, on March 12, 1994, accused-appellant asked complainant afternoon of March 7, 1994, it is contended that accused-appellant could not
to let him use their bathroom. However, after being given permission, he have committed such, because his daily schedule was such that he was not at
grabbed complainant by the hand, pulled her inside the bathroom, and started home at that time. He said that because of his work, he used to leave the house
kissing her on the lips and neck after closing the door behind them. He only at 3 a.m. , arrive home at 1 p.m., and leave for work again at 3 p.m. and arrive
stopped molesting her when he heard somebody coming.13 home at 6:30 p.m.19 He also alleged that on March 7, 1994 he left and was with
Rico Bariquit throughout the day.<20 as to the second incident of rape, accused-
* Virginia Lozada testified that she saw her daughter leave the bathroom, quickly appellant contends that it was impossible for him to commit rape in his room
followed by accused-appellant. Virginia noticed that her daughter's lower lip was because there were at least six other people there at the time(i.e., morning of
bruised. When she confronted her daughter about it, the latter revealed for the March 8, 1994) of the alleged rape.21 He charged that the complaints were filed
first time what had happened to her. Virginia went to camp Karingal, together against him because complainant's family wanted to evict and his housemates
with complainant and her other children, Teresa and Fernando, where they filed from their house.22
affidavits and two informations. They then took complainant to Camp Crame for
examination.14 Dr. Rosalina O. Cosidon, who examined complainant, submitted a * Rico Bariquit and Carlito Catubig confirmed accused-appellant's schedule.23
report which contained the following findings: Bariquit claimed that he was always with accused-appellant and knew where he
was all the time. Both witnesses said rape could not have been committed in
GENERAL AND EXTRAGENITAL: aroom where at least five other people were sleeping.24

Fairly developed, fairly nourished and coherent female subject. * Virginia Lozada and complainant denied seeing Rico Bariquit and Carlito
Breasts are hemispherical with pale brown areola and nipples from Catubig before the two testified in court.25
which no secretions could be pressed out. Abdomen is flat and soft.
There is injury noted at the head; * Based on the evidence of the parties, the trial court rendered a decision pn April 22, 1998,
finding the accused-appellant guilty of two counts of rape. The dispositive portion of its
decision reads:26

Conusion, mucosa of the lower lip, measuring 1.2 x 0.3 cm, and 1 WHEREFORE, in view of all the foregoing, the Court finds the accused guilty
cm left of the anterior midline. beyond reasonable doubt as principal two (2) counts of rape punishable under
Article 335 of the Revised Penal code, as amended by Section 11 of R. A. 7659,
and sentences him suffer the penalty of imprisonment of two (2) counts of
reclusion perpetua with all its accessory penalties and to indemnify the private
complainant the amount of FIFTY THOUSAND PESOS(P50,000.00).
GENITAL

SO ORDERD.
313
RECTO, GAYLE ANGELI M.
2011-0008 | AUSL
Personal Notes on Remedial Law 2 Review (based on the syllabus of Prof. Henedino M. Brondial)

* Hence this appeal. had with such person, or upon his appearance, or upon any fact bearing upon his
mental condition, with the witness' own knowledge and observation, he having
first testified to such conversations, dealings, appearance or other observed facts,
as the basis for his opinion.28
Issues

I. THE TRIAL COURT GRAVELY ERRED IN HOLDING THAT THE PRIVATE OFFENDED PARTY IS
"DEPRIVED OF REASON" DESPITE THE ABSENCE OF TESTIMONY BY A COMPETENT MEDICAL In the case at bar, Virginia Lozada testified on the mental condition of her daughter, thus:
EXPERT TO THAT EFFECT AND DESPITE STRONG EVIDENCE ON THE RECORD TO THE
CONTRARY.

II. THE TRIAL COURT GRAVELY ERRED IN ALTERNATIVELY HODING THAT THE ACCUSED IS Q: How would you describe your daughter? (sic)
GUILTY OG RAPING THE PRIVATE OFFENDED WOMAN THROUGH "FORCE AND
INTIMIFATION". A: When she was still a child while walking she accidentally bumped her head and then on
she acted quite not normal from then on we noticed changes because she acted like a child.

Ruling
Q: How old is (sic) Nympha Lozada when this happened?
First. Accused-appellant contends that he cannot be convicted of rape since the victim's
mental age was not proven. He argues that under Art. 335 (2) of the Revised Penal Code, an A: 3 to 4 years old.
essential element for the prosecution for rape of a mental retardate is a psychiatric evaluation
of the complainant's mental age to determine if her mental age is under twelve.27 He further
claims that only in cases where the retardation is apparent due to the presence of physical
deformities symptomatic of mental retardation can the mental evaluation be waived.
Q: At the age of 25, how would you described? (sic)

A: She still thinks like a child but from her narration or statement we can see that her
declaration are (sic) true or believable.
The contention has no merit.

Q: You mean to say that she could be intelligent.


Rule 130, $50 of the Revised Rules on Evidence provides:

A: Yes, ma'am. She finished her elementary and I can say she is quite intelligent.

Opinion of Ordinary witnesses. -- The opinion of a witness for which


proper basis is given may be received in evidence regarding ---
Q: So she can somewhat understand what is happening around us?

A: Yes, she can understand things around as along as she would be provided some basis
and some reference inorder (sic) to establish time, places and incident (sic)
a. the identify of a person about whom he has adequate knowledge;

b. a handwriting with which he has sufficient familiarity; and

Q: At your house do you still assigned (sic) household chores(?)


c. the mental sanity of a person with whom he is sufficiently
acquainted.
A: Yes, ma'am.

Discussing this provision of the Rule on Evidence, Sen. Vicente J. Francisco writes
Q: Could she relied (sic) upon madam witness?
in his treatise:

A: Yes ma'am.29

The mother of an offended party in case of rate, though not a psychiatrist, if she
knows the physical and mental condition of the party, how she was born, what she
is suffering from, and what her attainments are, is competent to testify on the To rebut this, accused-appellant points to the mother's statement that complainant is "quite
matter. intelligent." The statement that complainant is "quite intelligent" must be read in the context
of Virginia Lozada's previous statement that complainant "thinks like a child but from her
narration or statement we can see that her declaration are (sic) true or believable." Thus,
what complainant's mother meant was that complainant, although she thought like a child,
nevertheless could tell others what happened to her. Indeed, even the trial court admonished
….
the defense counsel not to use inculpatory questions because complainant might give
inculpatory answer.30 At another stage of the trial court reminded counsel. "The witness
[complainant] is not very intelligent. I think the witness cannot even distinguish fates."31

It is competent for the ordinary witness to give his opinion as to the sanity or
mental condition of a person, provided the witness has had sufficient opportunity
to observe the speech, manner, habits, and conduct of the person in question.
Thus, the trial court itself found in dealing with complainant that she was mentally deficient.
Generally, it is required that the witness details the factors and reasons upon
The rule that findings of fact of the trial court should not be disturbed since the trial court is in
which he bases his opinion before he can testify as to what it is. As the Supreme
the best position to determine the findings of facts32 cannot be more apt than in this case.
Court of Vermont said: "A non-expert witness may give his opinion as to the sanity
or insanity of another, when based upon conversations or dealings which he has

314
RECTO, GAYLE ANGELI M.
2011-0008 | AUSL
Personal Notes on Remedial Law 2 Review (based on the syllabus of Prof. Henedino M. Brondial)

Accused-appellant cites the medico-legal report which describes complainant as A: Yes, ma'am.
"coherent" and contends that this is an evaluation of the mental state of
complainant. This contention is totally without basis. The medico-legal report
categorically states that the purpose of the medical examination is limited to
determining whether the complainant had been sexually abused.33 In order
Q: Aside from kissing you, what did Emiliano do, if any?
words, the purpose of the examination was to determine her physical, not her
mental, state.
A: Aside from kissing me on my lips and my neck, he removed my underwear, my panty
and he inserted his sex organ into my sex organ.

Second. On the alternative, accused-appellant argues that indeed, complainant


could not be a competent witness if she is a retardate. Under Rule 130, $20, any
Q: When "Kalbo" inserted his sex organ in your sex organ, what position were you then?
person who can perceive and make known his/her perception is qualified to be a
witness. In this case, although complainant is a retardate, she was nevertheless
A: We were standing.
able to tell the court what accused-appellant had done to her and to answer the
questions of both the prosecutor and the defense counsel. This is clear from her
testimony, thus:

Q: Inside the bathroom?

Q: Now, you said that you were raped by Emiliano Duranan. A: yes, ma'am.

A: In the bathroom of my Tiya Ineng.

Q: Nympha, do you know how to tell the days of the week?

Q: Where is this bathroom of your Tiay Ineng? A: No, ma'am.

A: This bathroom is located at an alley, a "pasillo" towards our house.

Q: How about the dates?

Q: Where is your house, Miss witness? A: No, ma'am.

A: Our house is located at No. 28, K-6, Kamuning, Quezon City.

Q: Do you know what day is today?

Q: You said that you were raped inside the bathroom of your Tiya Ineng. How were you A: Wednesday.
able to get inside that bathroom of your Tiya Ineng?

A: I was able to get inside the bathroom of Tiya Ineng because Emiliano Duranan pulled
me inside. Q: How about yesterday, what date was that?

A: Tuesday.

Q: How did Emiliano Duranna pulled you? (sic)

A: He pulled me inside the bathroom by holding his arm against my neck, pulling me Q: Do you know what date is today?
towards the bathroom.

COURT:

She knows that today is Wednesday but she doesn't know the exact date.34

Q: When Emiliano Duranan pulled you inside the bathroom, what happened after that?

….
A: He kissed me

Q: You also testified before that you were rape (sic) by Kalbo twice, is that correct?
(Witness is gesturing his (sic) hands towards her neck)
A: Yes, ma'am.

And he had my panty removed.


Q: When was the second time?

A: The second time at their house that was Tuesday.


Q: Now, you said that when you were inside the bathroom of your Tiya Ineng, Emiliano
Duranan kissed you in (sic) your lips?
315
RECTO, GAYLE ANGELI M.
2011-0008 | AUSL
Personal Notes on Remedial Law 2 Review (based on the syllabus of Prof. Henedino M. Brondial)

However, the award of P50,000.00 as civil indemnity should be doubled because there are two
counts of rape. In addition, complainant should also be awarded P50,000.00 as moral
Q: Is that next day? After the incident in the bathroom? damages for each count of rape, or a total of P100,000.00 in accordance with our rulings.47

A: Yes, Ma'am.

WHEREFORE, the decision of the Regional Trial Court, Branch 220, Quezon City, finding
accused-appellant guilty beyond reasonable doubt of the crime of rape is AFFIRMED, with the
modification that the award of P50,000.00 as civil indemnity is increased to P100,000.00 and,
….
in addition, accused-appellant is ordered to pay complaint Nympha Lozada y de Lara the
further sum of P100,000.00 as moral damages..nêt

Q: So you said that you were laid down by Kalbo on the floor and then Kalbo kissed you,
where did Kalbo kiss you?
SO ORDERED.

A: From (sic) my lips.

Bellosillo, Quisumbing, Buena, De Leon, Jr. JJ., concur.

Q: After that what did Kalbo do if any?

A: Proceeded to removed (sic) my panty and inserted his organ to mine.35


Character evidence

At all events, any objection to the competency of complainant to testify should have been
raised by the defense at the outset. It cannot be raised for the first in this appeal. It has been
held:

A party may waive his objections to the competency of witness and permit him to testify…[I]f, Burden of Proof
after such incompetency appears, there is failure to make timely objection, by a party having
knowledge of the incompetency, the objection will be deemed waived, whether it is on the FIRST DIVISION
ground of want of mental capacity or for some other reason. If the objection could have been
taken during the trial, a new trial will be refused and the objection will not be available on writ
G.R. No. 170598 October 9, 2013
of error.36

FAR EAST BANK TRUST COMPANY, Petitioner,

vs. ROBERTO MAR CHANTE, a.k.a. ROBERT MAR G. CHAN, Respondents.


Third. Accused-appellant contends that the absence of injury sustained by complainant
negates the presence of any force and intimidation. This contention is likewise without merit.
The presence or absence of injuries is not essential in proving rape. What is essential is proof BERSAMIN, J.:
that sexual intercourse with woman was accomplished without her consent. In this case, the
absence of consent is shown by the fact that complainant is a mental retardate vulnerable to
intimidation by accused-appellant.

Facts

* Robert Mar Chante, also known as Robert Mar G. Chan (Chan), was a current account
Indeed, the degree of force or intimidation required for the act to constitute rape is relative, depositor of petitioner Far East Bank & Trust Co. (FEBTC) at its Ongpin Branch (Current
and must be viewed in the light of the complainant's perception and judgement at the time of Account No. 5012-00340-3). FEBTC issued to him Far East Card No. 05-01120-5-0 with July
the commission of the offense.37 What is vital is that such force or intimidation be sufficient to 1993 as the expiry date. The card, known as a "Do-It-All" card to handle credit card and ATM
consummate the purpose the accused-appellant had in mind.38 In this case, due to the transactions, was tagged in his current account. As a security feature, a personal identification
complainant's mental retardation, the force or intimidation required is not very great since it number (PIN), known only to Chan as the depositor, was required in order to gain access to
does not take much to force a child into submission. Indeed, complainant said she submitted the account. Upon the card’s issuance, FEBTC required him as the depositor to key in the six-
to accused-appellant's demands because she was afraid he would get angry at her if she digit PIN. Thus, with the use of his card and the PIN, he could then deposit and withdraw
refused them. In People v. Rosare,39 it was held that, in the instances where the victim is so funds from his current account from any FEBTC ATM facility, including the MEGALINK facilities of
weak in intellect that she is incapable of rational consent, the applied may be constructive. other member banks that included the Philippine National Bank (PNB).

* Civil Case No. 92-61706 sprang from the complaint brought by petitioner Far East Bank &
Trust Co. (FEBTC) on July 1, 1992 in the RTC,3 to recover from Chan the principal sum of
In sum, the mental retardation of the complainant is proven by the testimony of her P770,488.30 representing the unpaid balance of the amount fraudulently withdrawn from
mother,40 the trial court's observations during the trial of her demeanor, behavior, and her Chan’s Current Account No. 5012-00340-3 with the use of Far East Card No. 05-01120-5-0.
intelligence,41 while the fact of sexual intercourse is proven by the medico-legal certificate.42
In addition, the prosecution proved the presence of force and intimidation, and the court * FEBTC alleged that between 8:52 p.m. of May 4, 1992 and 4:06 a.m. of May 5,
appreciated such.43 The intimidation, in this case, is constituted by the threats that accused- 1992, Chan had used Far East Card No. 05-01120-5-0 to withdraw funds totaling
appellant made to the complainant,44 not to mention the force employed by accused- P967,000.00 from the PNB-MEGALINK ATM facility at the Manila Pavilion Hotel in
appellant in placing his arm on the complainant's neck45 and holding her hands while Manila; that the withdrawals were done in a series of 242 transactions with the
undressing her.46 use of the same machine, at P4,000.00/withdrawal, except for transaction No.
108 at 3:51 a.m. of May 5, 1992, when the machine dispensed only P3,000.00;
that MEGALINK’S journal tapes showed that Far East Card No. 05-01120-5-0 had
been used in all the 242 transactions; and that the transactions were processed
and recorded by the respective computer systems of PNB and MEGALINK despite
the following circumstances, namely: (a) the offline status of the branch of
316
RECTO, GAYLE ANGELI M.
2011-0008 | AUSL
Personal Notes on Remedial Law 2 Review (based on the syllabus of Prof. Henedino M. Brondial)

account (FEBTC Ongpin Branch); (b) Chan’s account balance being only hoping that the machine nearby may likewise dispense so much amount without
P198,511.70 at the time, as shown in the bank statement; (c) the maximum being detected. He will not definitely go back to the U.N. branch as he may think
withdrawal limit of the ATM facility being P50,000.00/day; and (d) his withdrawal that it is being watched and so he went to a nearby branch. Unfortunately, luck
transactions not being reflected in his account, and no debits or deductions from was not with him this time and his card was taken by the bank. The fact that he
his current account with the FEBTC Ongpin Branch being recorded. hastily withdrew the balance of his account after his card was retained by the
bank only showed his knowledge that the bank may debit his account. It also
* FEBTC added that at the time of the ATM withdrawal transactions, there was showed his intent to do something further other than first inquire why his card
an error in its computer system known as "system bug" whose nature had was considered a hot card if he is really innocent. When he went to the Ermita
allowed Chan to successfully withdraw funds in excess of his current credit branch to withdraw from the ATM booth he was intending to withdraw not more
balance of P198,511.70; and that Chan had taken advantage of the system bug than P50,000.00 as it is the bank’s limit for the day and if ever he needed a
to do the withdrawal transactions. bigger amount than P50,000.00 immediately he should have gone to the branch
for an over the counter transaction but he did not do so and instead issued a
check for P190,000.00 dated May 7, 1992 and another check for P5,000.00 dated
* On his part, Chan denied liability. Although admitting his physical possession of
May 13, 1992. To the mind of the Court, to take advantage of a computer error,
Far East Card No. 05-01120-5-0 on May 4 and May 5, 1992, he denied making
to gain sudden and undeserved amount of money should be condemned in the
the ATM withdrawals totaling P967,000.00, and instead insisted that he had been
strongest terms.
actually home at the time of the withdrawals. He alluded to a possible "inside
job" as the cause of the supposed withdrawals, citing a newspaper report to the
effect that an employee of FEBTC’s had admitted having debited accounts of its There are no available precedents in this case regarding computer errors, but the
depositors by using his knowledge of computers as well as information available Court feels that defendant should be held liable for the mistaken amount he was
to him. Chan claimed that it would be physically impossible for any human being able to get from the machine based on the following provisions of the law.
like him to stand long hours in front of the ATM facility just to withdraw funds.
He contested the debiting of his account, stating that the debiting had affected Articles 19, 21, 22 and 23 of the Civil Code x x x.
his business and had caused him to suffer great humiliation after the dishonor of
his sufficiently-funded checks by FEBTC. xxxx

* The records show that FEBTC discovered the system bug only after its routine There is likewise one point that the Court would like to discuss about the
reconciliation of the ATM-MEGALINK transactions on May 7, 1992; that it
allegation of the defendant that it was impossible for him to withdraw the money
immediately adopted remedial and corrective measures to protect its interest in in such long period and almost minute after minute. This Court believes that
order to avoid incurring further damage as well as to prevent a recurrence of the money is the least of all, a person may give priority in life. There are many who
incident; that one of the measures it adopted pursuant to its ATM Service would sacrifice a lot just to have lots of it, so it would not be impossible for one
Agreement with Chan was to program its computer system to repossess his ATM to take time, stand for several hours and just enter some items in the computer
card; that his ATM card was repossessed at the Ermita Branch of FEBTC when he if the return would be something like a million or close to a million. In fact, the
again attempted to withdraw at the ATM facility there; that the ATM facility effort exerted was just peanuts compared to other legitimate ways of earning a
retained his ATM card until its recovery by the bank; and that FEBTC conducted living as the only capital or means used to obtain it was the defendant’s loss of
an in-depth investigation and a time-and-motion study of the withdrawals in sleep and the time spent in withdrawing the same. Moreover, though the cause
question. of action in this case may be the erroneous dispensation of money due to
computer bug which is not of defendant’s wrong doing, the Court sees that what
* On May 14, 1992, FEBTC debited his current account in the amount of was wrong was the failure to return the amount in excess of what was legally
P192,517.20 pursuant to Chan’s ATM Service Agreement. It debited the further his. There is such a thing as JUSTICE. Justice means rendering to others their
sum of P3,000.00 on May 18, 1992, leaving the unrecovered portion of the funds due. A person is just when he is careful about respecting the rights of others,
allegedly withdrawn by him at P770,488.30. Thus, on May 14 and May 18, 1992, and who knows too, how to claim what he rightfully deserves as a consequence
FEBTC sent to Chan letters demanding the reimbursement of the unrecovered of fulfilling his duties.
balance of P770,488.30, but he turned a deaf ear to the demands, impelling it to
bring this case on July 1, 1992.4 From the foregoing, the conclusion is manifest that plaintiff is within its right in
initiating the instant suit, as defendant’s refusal to pay the claim constitutes the
* As reflected in the pre-trial order of October 19, 1992, the issues to be resolved were, firstly, cause of action for sum of money.
whether or not Chan had himself withdrawn the total sum of P967,000.00 with the use of his
Far East Card No. 05-01120-5-0 at the PNB-MEGALINK ATM facility; and, secondly, if the xxxx
answer to the first issue was that he did, whether or not he was liable to reimburse to FEBTC
the amount of P770,488.30 as actual damages, plus interest.5
WHEREFORE, judgment is hereby rendered in favor of the plaintiff Far East Bank
and Trust Company and against the defendant Robert Mar Chante a.k.a. Robert
* On May 14, 1998, the RTC rendered judgment in favor of FEBTC, pertinently holding and Mar G. Chan ordering the latter to pay the former the following:
ruling as follows:6

1. the amount of P770,488.30 as actual damages representing the


In the instant case, what happened was that the defendant who was at the U.N. unrecovered balance of the amounts withdrawn by defendant;
Branch of the PNB used his card. He entered his PIN to have access to a
withdrawal transaction from his account in Far East Bank, Ongpin Branch.
2. interest of 24% per annum on the actual damages from July 1,
However, after recognizing the card and went to the path of his account it could
1992, the date of the filing of the complaint until fully paid;
not get a signal to proceed with the transaction so it proceeded to the other path
who gave the signal to go on and dispense money. But there was a computer
error as it did not only dispense the money limit for the day buty it continued to 3. the amount of P100,000.00 as exemplary damages;
dispense a lot more until it reached the amount of P967,000.00 which took the
defendant till the hours of the morning to obtain. But defendant says he did not 4. the sum of P30,000.00 as and for attorney’s fees; and
use his card. He alleges that it could be an inside job just like what happened to
the said bank which was published in the newspaper wherein the bank employee 5. the costs of the suit. Defendant’s counterclaim is hereby
admitted having done the theft through his knowledge of the computer. Could dismissed for lack of merit.
this be true?
SO ORDERED.
The Court opines that it is not far-fetched. However why did this Court state that
plaintiff’s cause of action will survive? The action of the defendant after the * Chan appealed,7 assigning the following errors to the RTC, to wit:
incident gave him away. Merely two days after the heavy withdrawal, the
defendant returned not at the exact scene of the incident but at a nearby branch
1. THE TRIAL COURT ERRED IN HOLDING DEFENDANT-APPELLANT LIABLE FOR
which is also in Ermita and tried again to withdraw. But at this time the bank
THE ALLEGED WITHDRAWAL OF THE AMOUNT OF P967,000.00 WITH INTEREST
already knew what happened so it blocked the card and retained it being a hot
AT THE RATE OF 24% PER ANNUM BASED MERELY ON CONJECTURES AND
card. The defendant was not successful this time so what he did was to issue a
SUSPICIONS NOT ESTABLISHED BY SOLID EVIDENCE;
check almost for the whole amount of his balance in his account leaving only a
minimal amount. This incident puzzles the Court. Maybe the defendant was
317
RECTO, GAYLE ANGELI M.
2011-0008 | AUSL
Personal Notes on Remedial Law 2 Review (based on the syllabus of Prof. Henedino M. Brondial)

2. THE TRIAL COURT ERRED IN AWARDING IN FAVOR OF APPELLEE WHEREFORE , premises considered, we hereby GRANT the appeal and
EXEMPLARY DAMAGES IN THE AMOUNT OF P100,000.00 AND ATTORNEY’S FEES accordingly REVERSE and SET ASIDE the Decision dated May 14, 1998 of the
IN THE AMOUNT OF P30,000.00; Regional Trial Court of Manila, Branch 51, in Civil Case No. 92-61706. We
accordingly ORDER plaintiff-appellee Far East Bank and Trust Company (FEBTC)
3. THE TRIAL COURT ERRED IN NOT ORDERING THE RESTITUTION OF THE to return to Chan the amount of Php196,571.30 plus 12% interest per annum
AMOUNT OF P196,521.30 ILLEGALLY DEBITED BY APPELLEE FROM APPELLANT’S computed from August 7, 1992 - the time Chan filed his counterclaim - until the
ACCOUNT. obligation is satisfied. Costs against the plaintiff-appellee FEBTC.

* On August 1, 2005, the CA promulgated the assailed decision, reversing the RTC’s judgment, SO ORDERED.8
to wit:
* FEBTC moved for reconsideration, but the CA denied its motion on November 24, 2005.9
x x x. The issues really before us are issues of contract application and issues of
fact that would require an examination and appreciation of the evidence * Hence, FEBTC has appealed, urging the reversal of the CA’s adverse decision, and praying
presented. The first order therefore in our review of the trial court’s decision is to that Chan be held liable for the withdrawals made from his account on May 4 and May 5,
take stock of the established and undisputed facts, and of the evidence the 1992; and that it should not be held liable to return to Chan the sum of P196,571.30 debited
parties have presented. We say this at the outset as we believe that it was in this from his account.
respect that the lower court failed in its consideration and appreciation of the
case.

xxxx Ruling

An evidentiary dilemma we face in this case is the fact that there is no direct
evidence on the issue of who made the actual withdrawals. Chan correctly claims
that the bank failed to present any witness testifying that he (Chan) made the
The appeal lacks merit.
actual withdrawals. At the same time, Chan can only rely on his own
uncorroborated testimony that he was at home on the night that withdrawals
were made. We recognize that the bank can claim that no other evidence of
actual withdrawal is necessary because the PIN unique to Chan is already
evidence that only Chan or his authorized representative - and none other - FEBTC would want us to hold that Chan had authored the May 4 and May 5, 1992 ATM
could have accessed his account. But at the same time, we cannot close our eyes withdrawals based on the following attendant factors, namely: (a) ATM transactions were
to the fact that computers and the ATM system is not perfect as shown by an processed and identified by the PIN, among others; (b) the PIN was exclusive and known only
incident cited by Chan involving the FEBTC itself. Aside from the vulnerability to to the account holder; (c) the ATM was tagged in the cardholder’s account where the ATM
transactions were debited or credited; (d) the account number tagged in the ATM card
inside staff members, we take judicial notice that no less than our own Central
Bank has publicly warned banks about other nefarious schemes involving ATM identified the cardholder; (e) the ATM withdrawals were documented transactions; and (f) the
machines. In a March 7, 2003 letter, the Central Bank stated: transactions were strictly monitored and recorded not only by FEBTC as the bank of account
but also by the ATM machine and MEGALINK. In other words, the ATM transactions in
question would not be processed unless the PIN, which was known only to Chan as the
cardholder, had been correctly entered, an indication both that it was his ATM card that had
been used, and that all the transactions had been processed successfully by the PNB-
March 7, 2003
MEGALINK ATM facility at the Manila Pavilion Hotel with the use of the correct PIN.

BSP CIRCULAR LETTER

TO : All Banks
We disagree with FEBTC.

SUBJECT : Technology Fraud on ATM Systems

Although there was no question that Chan had the physical possession of Far East Card No.
05-01120-5-0 at the time of the withdrawals, the exclusive possession of the card alone did
Please be advised that there were incidents in other countries regarding not suffice to preponderantly establish that he had himself made the withdrawals, or that he
technology fraud in ATM systems perpetrated by unscrupulous individuals and/ or had caused the withdrawals to be made. In his answer, he denied using the card to withdraw
syndicates. funds from his account on the dates in question, and averred that the withdrawals had been
an "inside job." His denial effectively traversed FEBTC’s claim of his direct and personal liability
for the withdrawals, that it would lose the case unless it competently and sufficiently
established that he had personally made the withdrawals himself, or that he had caused the
These acts are carried out by: withdrawals. In other words, it carried the burden of proof.

1. A specialized scanner attached to the ATM card slot, and;

2. A pinhole camera Burden of proof is a term that refers to two separate and quite different concepts,
namely: (a) the risk of non-persuasion, or the burden of persuasion, or simply
persuasion burden; and (b) the duty of producing evidence, or the burden of going
forward with the evidence, or simply the production burden or the burden of
evidence.10 In its first concept, it is the duty to establish the truth of a given
xxxx
proposition or issue by such a quantum of evidence as the law demands in the case
at which the issue arises.11 In its other concept, it is the duty of producing
evidence at the beginning or at any subsequent stage of trial in order to make or
meet a prima facie case. Generally speaking, burden of proof in its second concept
In light of the absence of conclusive direct evidence of actual withdrawal that we passes from party to party as the case progresses, while in its first concept it rests
can rely upon, we have to depend on evidence "other than direct" to reach throughout upon the party asserting the affirmative of the issue.12
verdict in this case.

The party who alleges an affirmative fact has the burden of proving it because
xxxx mere allegation of the fact is not evidence of it.13 Verily, the party who asserts,
not he who denies, must prove.14
318
RECTO, GAYLE ANGELI M.
2011-0008 | AUSL
Personal Notes on Remedial Law 2 Review (based on the syllabus of Prof. Henedino M. Brondial)

the amount withdrawn - and at the same time declared that these tapes are authentic and
genuine. These tapes, however, are not as reliable as FEBTC represented them to be as they
In civil cases, the burden of proof is on the party who would be defeated if no are not even internally consistent. A disturbing internal discrepancy we note relates to the
evidence is given on either side.15 This is because our system frees the trier of amounts reflected as "ledger balance" and "available balance". We find it strange that for
facts from the responsibility of investigating and presenting the facts and every 4,000.00 pesos allegedly withdrawn by Chan, the available balance increased rather
arguments, placing that responsibility entirely upon the respective parties.16 The than diminished. Worse, the amount of available balance as reflected in the tapes was way
burden of proof, which may either be on the plaintiff or the defendant, is on the above the actual available balance of less than Php200,000.00 that Chan’s current account had
plaintiff if the defendant denies the factual allegations of the complaint in the at that time. These discrepancies must inevitably reflect on the integrity of the journal tapes;
manner required by the Rules of Court; or on the defendant if he admits expressly the proven inconsistencies in some aspects of these tapes leave the other aspects suspect and
or impliedly the essential allegations but raises an affirmative defense or defenses, uncertain.
that, if proved, would exculpate him from liability.17

But more than this, we are not convinced that the tapes lead us to the inevitable conclusion
Section 1, Rule 133 of the Rules of Court sets the quantum of evidence for civil that Chan’s card, rather than a replacement card containing Chan’s PIN and card number or
actions, and delineates how preponderance of evidence is determined, viz : some other equivalent scheme, was used. To our mind, we cannot discount this possibility
given the available technology making computer fraud a possibility, the cited instances of
computer security breaches, the admitted system bug, and - most notably - the fact that the
withdrawals were made under circumstances that took advantage of the system bug. System
errors of this kind, when taken advantage of to the extent that had happened in this case, are
Section 1. In civil cases, the party having the burden of proof must establish his
planned for. Indeed, prior preparation must take place to avoid suspicion and attention where
case by a preponderance of evidence. In determining where the preponderance or
the withdrawal was made for seven (7) long hours in a place frequented by hundreds of
superior weight of evidence on the issues involved lies, the court may consider all
guests, over 242 transactions where the physical volume of the money withdrawn was not
the facts and circumstances of the case, the witnesses’ manner of testifying, their
insignificant. To say that this was done by the owner of the account based solely on the
intelligence, their means and opportunity of knowing the facts to which they are
records of the transactions, is a convenient but not a convincing explanation.20
testifying, the nature of the facts to which they testify, the probability or
improbability of their testimony, their interest or want of interest, and also their
personal credibility so far as the same may legitimately appear upon the trial. The
court may also consider the number of witnesses, though the preponderance is not
necessarily with the greater number. (Emphasis supplied) In our view, the CA’s ruling was correct.

As the rule indicates, preponderant evidence refers to evidence that is of greater To start with, Edgar Munarriz, FEBTC’s very own Systems Analyst, admitted that the bug
weight, or more convincing, than the evidence offered in opposition to it.18 It is infecting the bank’s computer system had facilitated the fraudulent withdrawals.21 This
proof that leads the trier of facts to find that the existence of the contested fact is admission impelled the CA to thoroughly dissect the situation in order to determine the
more probable than its nonexistence.19 consequences of the intervention of the system bug in FEBTC’s computer system. It ultimately
determined thusly:

Being the plaintiff, FEBTC must rely on the strength of its own evidence instead of
upon the weakness of Chan’s evidence. Its burden of proof thus required it to Significantly, FEBTC made the admission that there was a program bug in its computer
preponderantly demonstrate that his ATM card had been used to make the system. To digress, computers are run based on specific pre-arranged instructions or
withdrawals, and that he had used the ATM card and PIN by himself or by another "programs" that act on data or information that computer users input. Computers can only
person to make the fraudulent withdrawals. Otherwise, it could not recover from process these inputted data or information according to the installed programs. Thus,
him any funds supposedly improperly withdrawn from the ATM account. We remind computers are as efficient, as accurate and as convenient to use as the instructions in their
that as a banking institution, FEBTC had the duty and responsibility to ensure the installed programs. They can count, sort, compute and arrive at decisions but they do so only
safety of the funds it held in trust for its depositors. It could not avoid the duty or and strictly in accordance with the programs that make them work. To cite an easy example, a
evade the responsibility because it alone should bear the price for the fraud computer can be programmed to sort a stack of cards prepared by male and female clients,
resulting from the system bug on account of its exclusive control of its computer into male and female stacks, respectively. To do this, the computer will first scan a card and
system. look at the place ("a field") where the male/female information can be found. This information
may be in an appropriate box which the bank client checks or shades to indicate if he/she is
male or female. The computer will check if the box beside the word "Female" is shaded. If it
is, it will send the card to the "Female" bin. If the box beside the "male" is shaded, it will send
the card to the "Male" bin. If both the squares are shaded or none is shaded or the card
Did FEBTC discharge its burden of proof?
cannot be read, it will send the card to the "Unknown" bin. This way, the female cards and the
male cards can be sorted efficiently. However, the program instructions can be written in such
a way that the computer can only make two decisions, that is, if the Female box is shaded,
then the card goes to the "Female" bin; otherwise, the card goes to the "Male" bin. In this
The CA ruled that FEBTC did not because - program, all the Female cards will be sorted correctly but the Male bin will contain all the other
cards, that is, the Male cards, the cards with no shading at all, and all the other cards that
cannot be classified.

After a review of the records of this case, we find the totality of evidence submitted by FEBTC
insufficient to establish the crucial facts that would justify a judgment in its favor.
The imperfect results arose from the imperfect program instructions or from a program "bug".
Something very close to this example happened in the present case.

To our mind, the fact that Chan’s account number and ATM card number were the ones used
for the withdrawals, by itself, is not sufficient to support the conclusion that he should be
deemed to have made the withdrawals. According to the testimony of the FEBTC’s systems analyst, there were two computer
programs that were involved in the transactions: CAPDROTH and SCPUP 900. CAPDROTH is
the program that validates if the account exists in the FEBTC files, if the transaction is valid,
and if the branch where the account is maintained is ON-LINE (i.e. continuously sending data).
When the Chan transaction entered the system, it was validated by CAPDROTH which, on
FEBTC offers in this regard the PNB ATM’s journal tapes to prove the withdrawals and their
seeing that the FEBTC-Ongpin branch was off-line, returned a decision code passing on the
details - the time of the transactions; the account number used; the ATM card number; and
decision to authorize the transaction to the SCPUP 900, another module. However, SCPUP 900
319
RECTO, GAYLE ANGELI M.
2011-0008 | AUSL
Personal Notes on Remedial Law 2 Review (based on the syllabus of Prof. Henedino M. Brondial)

was not expecting this type of response or decision code. As the SCPUP 900 program was ATM facility at the Manila Pavilion Hotel had exactly carried at the time of the withdrawals,
originally written, it will send back an error message and abort a requested transaction if it particularly the amounts immediately preceding and immediately following the series of
receives an error message from any other module; otherwise, it will send a message withdrawals. The omission left a yawning gap in the evidence against Chan.
authorizing the transaction. In other words, SCPUP 900 had only two decisions to make: check if
the message is an error message, if not then, authorize. Since what it received in the
disputed transactions were not error messages and were not also authorizations, it sent back
authorization messages allowing the cash withdrawals. It kept on sending authorization
And lastly, Chan’s allegation of an "inside job" accounting for the anomalous withdrawals
messages for the 242 cash withdrawal transactions made from Chan’s account between the
should not be quickly dismissed as unworthy of credence or weight. FEBTC employee Manuel
evening of May 4 and early morning of May 5, 1992. This program bug was the reason the
Del Castillo, another witness for FEBTC, revealed that FEBTC had previously encountered
242 cash withdrawals were allowed by the PNB ATM-Megalink machine.
problems of bank accounts being debited despite the absence of any withdrawal transactions
by their owners. He attributed the problems to the erroneous tagging of the affected accounts
as somebody else’s account, allowing the latter to withdraw from the affected accounts with
the use of the latter’s own ATM card, and to the former’s account being debited.27 The
The program bug occurred because of the simultaneous presence of three conditions that revelation of Del Castillo tended to support Chan’s denial of liability, as it showed the
allowed it to happen: (1) the withdrawal transactions involved a current account; (2) the possibility of withdrawals being made by another person despite the PIN being an exclusive
current account was with a branch that at that time was off-line; and (3) the transaction access number known only to the cardholder.28
originated from MEGALINK (i.e., through MEGALINK through a member bank other than
FEBTC). Because of the bug, Chan’s account was not accessed at the time of the transactions
so that withdrawals in excess of what the account contained were allowed. Additionally,
FEBTC’s rule that only a maximum withdrawable amount per day (in the present case
It is true that Del Castillo also declared that FEBTC did not store the PINs of its clients’ ATM
P50,000.00 per day) can be made from an ATM account, was by-passed. Thus, 242
cards. However, he mentioned that FEBTC had stored the opposite numbers corresponding to
withdrawals were made over an eight hour period, in the total amount of P967,000.00.22
the PINs, which meant that the PINs did not remain entirely irretrievable at all times and in all
cases by any of its officers or employees with access to the bank’s computer system.
Accordingly, Del Castillo’s assertion that the PINs were rendered useless upon being entered in
the bank’s computer system did not entirely disclose how the information on the PINs of the
Secondly, the RTC’s deductions on the cause of the withdrawals were faulty. In holding depositors was stored or discarded as to become useless for any purpose.
against Chan, the RTC chiefly relied on inferences drawn from his acts subsequent to the
series of withdrawals, specifically his attempt to withdraw funds from his account at an FEBTC
ATM facility in Ermita, Manila barely two days after the questioned withdrawals; his issuance of
a check for P190,000.00 immediately after the capture of his ATM card by the ATM facility; his
In view of the foregoing, FEBTC did not present preponderant evidence proving Chan’s liability
failure to immediately report the capture of his ATM card to FEBTC; and his going to FEBTC only
for the supposedly fraudulent withdrawals. It thus failed in discharging its burden of
after the dishonor of the check he had issued following the freezing of his account. The
persuasion.
inferences were not warranted, however, because the subsequent acts would not persuasively
establish his actual participation in the withdrawals due to their being actually susceptible of
other interpretations consistent with his innocence.

WHEREFORE, the Court AFFIRMS the decision of the Court of Appeals; and DIRECTS the
petitioner to pay the costs of suit.

We join the CA’s observation that Chan’s subsequent acts "could have been impelled by so
many reasons and motivations, and cannot simply be given the meaning that the lower court
attributed to them," and, instead, were even consistent with the purpose and nature of his
maintaining the current account deposit with FEBTC, rendering the acts "not unusual nor … SO ORDERED.
illegal."23 Although he was expected to forthwith bring his card’s capture to FEBTC’s attention,
that he did not do so could have other plausible explanations consistent with good faith,
among them his being constantly occupied as a businessman to attend to the multifarious
activities of his business. He might have also honestly believed that he still had the sufficient Sereno, CJ., Leonardo-De Castro, Reyes, Leonen *, JJ., concur.
funds in his current account, as borne out by his issuance of a check instead after the capture
of the card so as not for him to undermine any financial obligation then becoming due. Nor
should his opting to withdraw funds from his account at the ATM facility in Ermita in less than
two days after the questioned withdrawals manifest responsibility on his part, for he could also
be properly presumed to be then still unaware of the situation involving his account. We note Presumptions
that his letters24 written in response to FEBTC’s written demands to him disclosed honest
intentions rather than malice.
> Conclusive presumptions

SECOND DIVISION
Thirdly, the RTC ignored the likelihood that somebody other than Chan familiar with the bug
infection of FEBTC’s computer system at the time of the withdrawals and adept with the G.R. No. 123817 December 17, 1999
workings of the computer system had committed the fraud. This likelihood was not far-fetched
considering that FEBTC had immediately adopted corrective measures upon its discovery of IBAAN RURAL BANK INC., petitioner,
the system bug, by which FEBTC admitted its negligence in ensuring an error -free computer
system; and that the system bug had affected only the account of Chan.25 Truly, the trial
vs. THE COURT OF APPEALS and MR. and MRS. RAMON TARNATE, respondents.
court misapprehended the extent to which the system bug had made the computer system of
FEBTC stumble in serious error.
QUISUMBING, J.:

Fourthly, and perhaps the most damaging lapse, was that FEBTC failed to establish that the
PNB-MEGALINK’s ATM facility at the Manila Pavilion Hotel had actually dispensed cash in the Facts
very significantly large amount alleged during the series of questioned withdrawals. For sure,
FEBTC should have proved the actual dispensing of funds from the ATM facility as the factual * Spouses Cesar and Leonila Reyes were the owners of three (3) lots covered by Transfer
basis for its claim against Chan. It did require PNB to furnish a validated showing of the exact Certificate of Title (TCT) Nos. 33206, 33207 and 33208 of the Register of Deeds of Lipa City.
level of cash then carried by the latter’s ATM facility in the Manila Pavilion Hotel on May 4, On March 21, 1976, the spouses mortgaged these lots to Ibaan Rural Bank, Inc. [herein
1992.26 Yet, when PNB employee Erwin Arellano stood as a witness for FEBTC, he confirmed petitioner]. On June 11, 1976, with the knowledge and consent of the petitioner, the spouses
the authenticity of the journal tapes that had recorded Chan’s May 4 and May 5, 1992 as sellers, and Mr. and Mrs. Ramon Tarnate [herein private respondents] as buyers, entered
supposed ATM transactions but did not categorically state how much funds PNB-MEGALINK’s into a Deed of Absolute Sale with Assumption of Mortgage of the lots in question. Private

320
RECTO, GAYLE ANGELI M.
2011-0008 | AUSL
Personal Notes on Remedial Law 2 Review (based on the syllabus of Prof. Henedino M. Brondial)

respondents failed to pay the loan and the bank extra-judicially foreclosed on the mortgaged Issues
lots. The Provincial Sheriff conducted a public auction of the lots and awarded the lots to the
bank, the sole bidder. On December 13, 1978, the Provincial Sheriff issued a Certificate of Sale 1. THE RESPONDENT COURT ERRED AND, ACCORDINGLY, THE PETITIONER IS ENTITLED TO
which was registered on October 16, 1979. The certificate stated that the redemption period A REVIEW OF ITS DECISION, WHEN IT SUSTAINED AVAILABILITY OF REDEMPTION DESPITE
expires two (2) years from the registration of the sale. No notice of the extrajudicial
THE LAPSE OF ONE YEAR FROM DATE OF REGISTRATION OF THE CERTIFICATE OF SALE.
foreclosure was given to the private respondents. On September 23, 1981, private
respondents offered to redeem the foreclosed lots and tendered the redemption amount of
2. THE RESPONDENT COURT ERRED AND, ACCORDINGLY, THE PETITIONER IS ENTITLED TO
P77,737.45. However, petitioner Bank refused the redemption on the ground that it had
A REVIEW OF ITS DECISION, WHEN THE RESPONDENT COURT ALLOWED RECOVERY OF
consolidated its titles over the lots. The Provincial Sheriff also denied the redemption on the
ATTORNEY'S FEES SIMPLY BECAUSE THE PETITIONER DID NOT ALLOW THE PRIVATE
ground that private respondents did not appear on the title to be the owners of the lots.
RESPONDENTS TO EXERCISE BELATEDLY REDEMPTION OF THE FORECLOSED PROPERTY. 4

* Private respondents filed a complaint to compel the bank to allow their redemption of the
foreclosed lots. They alleged that the extra-judicial foreclosure was null and void for lack of
valid notice and demand upon them. They further argued that they were entitled to redeem
Ruling
the foreclosed lots because they offered to redeem and tendered the redemption price before
October 16, 1981, the deadline of the 2-year redemption period.
Essentially, two issues are raised for resolution. What was the period of redemption: two years
* The bank opposed the redemption, contending that the private respondents as unilaterally fixed by the sheriff in the contract, or one year as fixed by law? May respondent
had no right to redeem the lots because they were not the real parties in court properly award attorney's fees solely on the basis of the refusal of the bank to allow
interest; that at the time they offered to redeem on September 23, 1981, the redemption?
right to redeem had prescribed, as more than one year had elapsed from the
registration of the Certificate of Sale on October 16, 1979; that there was no
need of personal notice to them because under Section 3 of Act 3135, only the
posting of notice of sale at three public places of the municipality where the We now resolve these issues.
properties are located was required. 1

* After trial on the merits, the lower court ruled in favor of herein private respondents and
against the petitioner, thus: When petitioner received a copy of the Certificate of Sale registered in the Office of
the Register of Deeds of Lipa City, it had actual and constructive knowledge of the
WHEREFORE, in view of the foregoing, the Court renders judgment in favor of certificate and its contents. 5 For two years, it did not object to the two-year
the plaintiffs and against the defendants, to wit: redemption period provided in the certificate. Thus, it could be said that petitioner
consented to the two-year redemption period specially since it had time to object
(a) Ordering the defendant Ibaan Rural Bank Inc., and Provincial Sheriff of and did not. When circumstances imply a duty to speak on the part of the person
Batangas for the redemption of the foreclosed properties covered by Transfer for whom an obligation is proposed, his silence can be construed as consent. 6 By
Certificate of Title Nos. T-33206, T-33207 and T-33208 of the Registry of Deeds, its silence and inaction, petitioner misled private respondents to believe that they
Lipa City by the plaintiffs by paying the mortgaged obligation. had two years within which to redeem the mortgage. After the lapse of two years,
petitioner is estopped from asserting that the period for redemption was only one
(b) Ordering the Provincial Sheriff of Batangas to cancel the Transfer Certificate year and that the period had already lapsed. Estoppel in pais arises when one, by
of Titles issued to defendant Ibaan Rural Bank, Inc. and its successors-in-interest his acts, representations or admissions, or by his own silence when he ought to
and to issue the corresponding Transfer of Certificate of Titles to plaintiffs upon speak out, intentionally or through culpable negligence, induces another to believe
payment of the required legal fees. certain facts to exist and such other rightfully relies and acts on such belief, so that
he will be prejudiced if the former is permitted to deny the existence of such facts.
7
(c) Ordering the defendant Ibaan Rural Bank, Inc., to pay plaintiffs moral
damages in the amount of P200,000.00, and attorney's fees in the sum of
P20,000.00.

In affirming the decision of the trial court, the Court of Appeals relied on Lazo vs. Republic
All other claims not having been duly proved are ordered DISMISSED.
Surety and Insurance Co., Inc., 8 where the court held that the one year period of redemption
provided in Act No. 3135 is only directory and can be extended by agreement of the parties.
Without pronouncement as to costs.
True, but it bears noting that in Lazo the parties voluntarily agreed to extend the redemption
period. Thus, the concept of legal redemption was converted by the parties in Lazo into
SO ORDERED. 2 conventional redemption. This is not so in the instant case. There was no voluntary
agreement. In fact, the sheriff unilaterally and arbitrarily extended the period of redemption to
* On appeal, the Court of Appeals affirmed with modification the decision of the lower court. two (2) years in the Certificate of Sale. The parties were not even privy to the extension made
The dispositive portion of the CA decision reads: by the sheriff. Nonetheless, as above discussed, the bank can not after the lapse of two years
insist that the redemption period was one year only.
WHEREFORE, the decision appealed from is hereby AFFIRMED with the following
modifications:

1. The register of Deeds of Lipa City is hereby ordered to cancel the Certificate of Additionally, the rule on redemption is liberally interpreted in favor of the original owner of a
Titles issued to defendant Ibaan Rural Bank, Inc. and its successor-in-interest property. The fact alone that he is allowed the right to redeem clearly demonstrates the
and to issue the corresponding Transfer Certificate of Title to plaintiffs-appellees solicitousness of the law in giving him another opportunity, should his fortune improve, to
upon proper redemption of the properties and payment of the required legal recover his lost property. 9
fees.

2. Defendant Ibaan Rural bank, is hereby ordered to pay to plaintiffs the amount of
P15,000.00 as attorney's fees. Lastly, petitioner is a banking institution on whom the public expects diligence, meticulousness
and mastery of its transactions. Had petitioner diligently reviewed the Certificate of Sale it
3. The moral damages awarded in favor of plaintiffs is hereby ordered deleted. could have easily discovered that the period was extended one year beyond the usual period
for redemption. Banks, being greatly affected with public interest, are expected to exercise a
SO ORDERED. 3 degree of diligence in the handling of its affairs higher than that expected of an ordinary
business firm. 10
* A timely Motion for Reconsideration was filed by the petitioner but the same was denied in a
Resolution dated February 14, 1996. Hence, this petition.

321
RECTO, GAYLE ANGELI M.
2011-0008 | AUSL
Personal Notes on Remedial Law 2 Review (based on the syllabus of Prof. Henedino M. Brondial)

On the second issue, the award of attorney's fees must be disallowed for lack of legal basis. WHEREFORE, Judgment is entered by way of preponderance of evidence in favor
The fact that private respondents were compelled to litigate and incur expenses to protect and of plaintiffs and against the defendants, Ordering the latter to vacate the
enforce their claim does not justify the award of attorney's fees. The general rule is that premises immediately, including all those who are occupying the subject house in
attorney's fees cannot be recovered as part of damages because of the public policy that no relation to them; They are also jointly ordered to pay the sum of P48,000
premium should be placed on the right to litigate. 11 The award of attorney's fees must be representing rental payment in arrears from November, 1993 up to October,
deleted where the award of moral and exemplary damages are eliminated. 12 1994 and to update monthly payment of P4,000 thereafter until their vacation
therefrom; They are saddled to pay attorney’s fees in the sum of P5,000 and
litigation costs in the amount of P1,000.

WHEREFORE, the decision of the Court of Appeals in CA-G.R. CV No. 32984 is AFFIRMED, with SO ORDERED.5
the MODIFICATION that the award of attorney's fees is deleted. No pronouncement as to
costs. * In ruling in favor of the respondents, the MTC held that the petitioner spouses clearly
violated the contract of lease due to non-payment of rent. They failed to show that the subject
house belonged to Virgilio alone. On the other hand, the respondents proved that, after the
death of Virgilia, they registered said house in the name of their trustees, co-respondents
Hermes Tangga-an and his wife. Furthermore, considering that Virgilio’s claim of ownership
SO ORDERED.
over the lot was the subject of a pending litigation for annulment of deed of sale and
reconveyance of property involving the Tangga-ans, the MTC ruled that it "cannot usurp to
pass judgment on the issues, as well as the conflicting claims of the parties therein."6

Bellosillo, Mendoza, Buena and De Leon, Jr., JJ., concur.


* On appeal, the RTC affirmed the decision of the MTC, and held that:

xxx [D]efendants failed to present any documentary evidence modifying or


amending the contract of lease (Annex "C", complaint) to justify the transfer of
THIRD DIVISION payment of the monthly rental to Virgilio Tanga-an who claims only as the
registered owner of the lot on which the leased house is located. It appears that
G.R. No. 128568 April 9, 2003 Virgilio Tanga-an does not possess any proof of ownership of the rented house.
Clearly, defendants had violated the lease agreement executed between them
and the deceased lessor Virginia R. Tangga-an (sic) the predecessor in interest of
SPOUSES REYNALDO ALCARAZ and ESMERALDA ALCARAZ, petitioners,
Hermes Tangaa-an and his wife as shown in the Tax Declaration of the said
spouses (Annex "A", complaint) whose name appears under the space for
vs. PEDRO M. TANGGA-AN, MENAS R. TANGGA-AN, VIRGINIA III YVETTE R. previous owner by stopping payment of rental to the present owner despite the
TANGGA-AN, CECIL T. VILLAFLOR, HERMES R. TANGGA-AN, VENUS R. TANGGA-AN, existence of the contract of lease which expires on November 22, 1996. The law
JUPITER R. TANGGA-AN, YVONNE T. FRI, VIVIEN R. TANGGA-AN and HON. JUDGE on contracts basically states:
P. BURGOS and THE COURT OF APPEALS, respondents.

CORONA, J.:

"Obligations arising fro contracts have the force of law between the contracting
parties and should be complied with in good faith." (Article 1159, New Civil Code of
the Philippines).
Facts

* On October 4, 1994, respondents Pedro Tangga-an, Menas Tangga-an, Virginia III Yvette
Tangga-an, Cecil Villaflor, Hermes Tangga-an, Venus Tangga-an, Jupiter Tangga-an, Yvonne xxx xxx xxx7
Fri and Vivien Tangga-an filed a complaint for unlawful detainer, with damages, docketed as
Civil Case No. R-33928, against petitioner spouses Reynaldo Alcaraz and Esmeralda Alcaraz. * In denying the petition for review and affirming the judgments of the courts a quo, the
Court of Appeals ruled that:
* The complaint alleged that the late Virginia Tangga-an (the spouse of
respondent Pedro Tangaa-an and mother of the rest of the respondents) leased We also concur with the holding of both courts that as heirs of Virginia Tangga-
a residential building (house) located at Premier Street, Hipodromo, Cebu City to an, private respondents have the right to institute the action for ejectment, in
the petitioner spouses. The lease contract was limited to the use and occupancy accordance with Article 487 of the Civil Code; and that the claim of petitioner -
of the said residential building and did not include the lot on which it was that Virgilio Tangga-an owns the lot where the leased residential building stands
constructed because the said lot was then owned by the National Housing and occupied by petitioners - is still the subject of a civil action for annulment of
Authority (NHA). Under the contract, the petitioner spouses bound themselves the sale of the lot before the Regional Trial Court of Cebu. It does not follow as a
for five years to pay Virginia a monthly rental of P4,000 beginning November 22, matter of course that whoever owns the lot owns the building in question.
1991. However, since November 1993, they failed to pay rent. Thus, as of Ownership of the lot cannot change the nature and ownership of the building,
October, 1994, they were in arrears in the amount of P48,000. Despite repeated which belongs to the plaintiffs as heirs of the late Virginia Tangga-an through
demands by respondents to pay the rentals in arrears and to surrender the Ernest Tangga-an and his wife. Respondent court correctly reasoned out that
possession of the residential building, the petitioner spouses refused to vacate "xxx defendants cannot hide over the cloak of Virgilio Tangga-an, his claim of
the same. Respondents sought to repossess the property for their own use and ownership over the lot as far as the Court is concerned being irrelevant to this
benefit. case xxx." Most importantly, the action involving the question of ownership of the
lot is not a lawful ground to suspend/abate the ejectment proceeding. The
* On the other hand, the petitioner spouses alleged that, on July 23, 1993, the rationale of the rule being that an ejecment suit involves only the issue of
ownership of the lot on which the house stood was transferred by the NHA to material possession or possession de facto (San Pedro vs. Court of Appeals, 235
Virgilio and Angelita D. Tangga-an. Virgilio Tangga-an is the son of the late SCRA 145, 150, and cases cited).8
Virgilia Tangga-an and respondent Pedro Tangga-an, and the brother of the
other respondents. Transfer Certificate of Title No. 125657 was consequently * Hence, this petition on the following assignments of error:
issued in the name of Virgilio Tangga-an. According to the petitioner spouses, the
subsequent change in ownership of the lot and the house resulted in the
cancellation of the contract of lease between respondents and petitioner
spouses. Thereafter, they paid the rent to the new owners of the lot (Virgilio and
Issues
Angelita) and not to respondents since the latter supposedly no longer had the
legal right to collect rentals.
I. THE LEASE CONTRACT EXECUTED BY PETITIONERS WITH VIRGINIA TANGGA-AN,
PLAINTIFFS’ PREDECESSOR-IN-INTEREST, COVERED NOT ONLY THE LAND, BUT ALSO THE
* On January 5, 1995, the MTC rendered a decision, the dispositive portion of which read:
IMPROVEMENT THEREON, INCLUDING THE BUILDING.

322
RECTO, GAYLE ANGELI M.
2011-0008 | AUSL
Personal Notes on Remedial Law 2 Review (based on the syllabus of Prof. Henedino M. Brondial)

II. VIRGILIO TANGGA-AN, AS ONE OF THE HEIRS OF VIRGINIA, HAD THE SAME RIGHTS The courts a quo were unanimous in holding that the petitioner spouses failed to substantiate
OVER THE PROPERTY AS THOSE OF THE OTHER HEIRS, THE PLAINTIFFS. HENCE, VIRGILIO their factual averment that Virgilio not only acquired the lot but also the house. After
MAY NOT BE EXCLUDED UNILATERALLY BY THE OTHER HEIRS IN HIS ENJOYMENT OF HIS examining the records, we found nothing to disprove the facts determined by the lower courts.
HEREDITARY RIGHTS. All the petitioner spouses presented was Virgilio’s uncertified xerox copy of the certificate of
title over the lot. No document was ever shown evidencing cession of the subject house in
III. THE REGISTRATION OF THE LAND, INCLUDING THE IMPROVEMENTS THEREON, IN THE Virgilio’s favor. Virgilio’s title could not be used to prove ownership over the house built on said
NAME OF VIRGILIO TANGGA-AN UNDER THE TORRENS SYSTEM IS INDEFEASIBLE AND MAY lot as it carried no reference at all to the house. A building by itself is a real or immovable
NOT BE ATTACKED COLLATERALLY IN THE PRESENT ILLEGAL DETAINER CASE.9 property distinct from the land on which it is constructed12 and therefore can be a separate
subject of contracts.

Ruling
On the other hand, the respondents proved that, as compulsory heirs of Virginia, they were
the rightful owners of the subject house. They presented a tax declaration in the name of their
We rule in favor of the respondents.
trustees, co-respondent Hermes Tangga-an and his wife, which tax declaration sufficiently
evidences their co-ownership and acquisition of title following the death of the decedent
Virginia. We have ruled that:

Section 16 of the 1997 Revised Rules of Civil Procedure provides that:

Although tax declarations or realty tax payment of property are not conclusive evidence of
ownership, nevertheless, they are good indicia of possession in the concept of owner for no
SEC. 16. Resolving defense of ownership. - When the defendant raises the defense of
one in his right mind would be paying taxes for a property that is not in his actual or at least
ownership in his pleadings and the question of possession cannot be resolved without deciding constructive possession. They constitute at least proof that the holder has a claim of title over
the issue of ownership, the issue of ownership shall be resolved only to determine the issue of the property. The voluntary declaration of a piece of property for taxation purposes manifests
possession. not only one’s sincere and honest desire to obtain title to the property and announces his
adverse claim against the State and all other interested parties, but also the intention to
contribute needed revenues to the Government. Such an act strengthens one’s bona fide claim
of acquisition of ownership.13
The issue of ownership is precisely what the petitioner spouses raised to justify their non-
payment of rent and to resist eviction from the house they leased from respondents. Being
indispensable to the resolution of the issue of possession, we herein render a provisional ruling
on ownership. One of the factual issues raised by the petitioner spouses concerns the alleged waiver and
cession of Virginia’s rights over the house and lot to Virgilio. But the petitioner spouses did not
mention any consideration received by Virginia for the waiver of the house, in effect making
said waiver a donation thereof to Virgilio. However, in order for a donation of real property like
Petitioner spouses seek a dismissal of the case for lack of jurisdiction claiming that the only a house to be valid, a public instrument duly signed by the donor and accepted by the donee
issue to be resolved is ownership over the house which is improper in an ejectment case. We (which acceptance must be known to the donor while alive) must be executed.14 Moreover,
disagree. The issue in the case at bar is whether the petitioner spouses, as lessees, were said donation must not impair the legitime of the forced heirs of the donor in order for the
excused from paying the rent because of the change in the ownership of the land on which same not to be inofficious.15 In the case at bar, no such public instrument was presented.
the rented house was built. The main question therefore is still the lawful possession of the Neither was it explained why said waiver did not impair the rights of the other compulsory
subject premises by the petitioner spouses. To resolve it, a discussion of the ownership issue heirs of Virginia.
is necessary.

To support their argument that the house necessarily became Virgilio’s property as a result of
The petitioner spouses insist that the courts a quo erred in not finding that Virgilio Tangga-an the acquisition of the lot on which the same was built, the petitioner spouses invoke the
became the new owner not only of the lot but also of the residential house. They claim that, principle that the accessory follows the principal. Being an accessory, the house is necessarily
before she died, Virginia, the original owner of the subject house, waived and ceded her rights owned by the owner of the lot on which it is built.
over the land in favor of Virgilio. The said transfer allegedly included the subject house
because, pursuant to Article 440 of the Civil Code, "the ownership of the property gives the
right of accession to everything which is produced thereby, or which is incorporated or
attached thereto, either naturally or artificially." They also maintain that the NHA executed a There is no need, however, to disturb and analyze the applicability of this well-entrenched
deed of sale of both the house and the lot in favor of Virgilio. According to the petitioner principle because the petitioner spouses are estopped from raising the same. Both parties
spouses, the tax declaration over the house in the name of respondent Hermes Tangga-an, as knew that their contract pertained only to the lease of the house, without including the land.
trustee of the other respondents, was self-serving and had no probative value compared to the The contract states: "1. That the lessor is the owner of a building of mixed materials situated
certificate of title over the lot in the name of Virgilio Tangga-an. at Premier St., Mabolo, Hipodromo, Cebu City."16 At the time of the perfection of the
contract, the petitioner spouses, as lessees, were aware that the NHA, and not
Virginia, the lessor, owned the land on which the rented house stood yet they
signed the same, obliged themselves to comply with the terms thereof for five
We find no merit in petitioners’ arguments. years and performed their obligations as lessees for two years.

Pursuant to Section 1, Rule 45 of the 1997 Revised Rules of Civil Procedure, a petition for Now they assume a completely different legal position. They claim that the lease
review before this Court should only raise questions of law. In the absence of showing that the contract ceased to be effective because Virgilio’s assumption of ownership of the
case falls under one of the exceptions,10 factual findings of the Court of Appeals are land stripped the respondents of ownership of the building. They argue that, under
conclusive on the parties and not reviewable by this Court. And they carry even more weight Article 440 of the Civil Code, Virgilio’s title over the lot necessarily included the
when the Court of Appeals affirms the factual findings of the trial court. As such, this Court is house on the said lot, thus automatically canceling the contract.
not duty-bound to analyze and weigh all over again the evidence already considered in the
proceedings below.11

Section 2, Rule 131 of the Rules of Court provides as a conclusive presumption


that:

323
RECTO, GAYLE ANGELI M.
2011-0008 | AUSL
Personal Notes on Remedial Law 2 Review (based on the syllabus of Prof. Henedino M. Brondial)

G.R. No. 194846 June 19, 2013

Sec. 2. Conclusive presumptions. - The following are instances of conclusive *HOSPICIO D. ROSAROSO, ANTONIO D. ROSAROSO, MANUEL D.
presumptions: ROSAROSO,
ALGERICA D. ROSAROSO, and CLEOFE R. LABINDAO, Petitioners,

vs. LUCILA LABORTE SORIA, SPOUSES HAM SOLUTAN and **LAILA SOLUTAN,
and
MERIDIAN REALTY CORPORATION, Respondents.
(a) Whenever a party has, by his own declaration, act, or omission, intentionally
and deliberately led another to believe a particular thing true, and to act upon such MENDOZA, J.:
belief, he cannot, in any litigation arising out of such declaration, act or omission,
be permitted to falsify it;

Facts

xxx xxx xxx


* Spouses Luis Rosaroso (Luis) and Honorata Duazo (Honorata) acquired several real
properties in Daan Bantayan, Cebu City, including the subject properties. The couple had nine
(9) children namely: Hospicio, Arturo, Florita, Lucila, Eduardo, Manuel, Cleofe, Antonio, and
Angelica. On April 25, 1952, Honorata died. Later on, Luis married Lourdes Pastor Rosaroso
After recognizing the validity of the lease contract for two years, the petitioner (Lourdes).
spouses are barred from alleging the automatic cancellation of the contract on the
ground that the respondents lost ownership of the house after Virgilio acquired * On January 16, 1995, a complaint for Declaration of Nullity of Documents with Damages was
title over the lot. filed by Luis, as one of the plaintiffs, against his daughter, Lucila R. Soria (Lucila); Lucila’s
daughter, Laila S. Solutan (Laila); and Meridian Realty Corporation (Meridian). Due to Luis’
untimely death, however, an amended complaint was filed on January 6, 1996, with the
spouse of Laila, Ham Solutan (Ham); and Luis’ second wife, Lourdes, included as defendants.3
We also note that the petitioner spouses rescinded the contract of lease without judicial
approval. Due to the change in ownership of the land, the petitioner spouses decided to * In the Amended Complaint, it was alleged by petitioners Hospicio D. Rosaroso, Antonio D.
unilaterally cancel the contract because Virgilio supposedly became the new owner of the Rosaroso (Antonio), Angelica D. Rosaroso (Angelica), and Cleofe R. Labindao (petitioners) that
house after acquiring title to the lot. They alleged that there was no reason anymore to on November 4, 1991, Luis, with the full knowledge and consent of his second wife, Lourdes,
perform their obligations as lessees because the lessor had ceased to be the owner of the executed the Deed of Absolute Sale4 (First Sale) covering the properties with Transfer
house. But there is nothing in their lease contract that allows the parties to extrajudicially Certificate of Title (TCT) No. 31852 (Lot No. 8); TCT. No. 11155 (Lot 19); TCT No. 10885 (Lot
rescind the same in case of violation of the terms thereof. Extrajudicial rescission of a contract No. 22); TCT No. 10886 (Lot No. 23); and Lot Nos. 5665 and 7967, all located at
is not possible without an express stipulation to that effect.17 What the petitioner spouses Daanbantayan, Cebu, in their favor.5
should have done was to file a special civil action for interpleader for the claimants to litigate
their claims and to deposit the rentals in court. * They also alleged that, despite the fact that the said properties had already
been sold to them, respondent Laila, in conspiracy with her mother, Lucila,
obtained the Special Power of Attorney (SPA),6 dated April 3, 1993, from Luis
(First SPA); that Luis was then sick, infirm, blind, and of unsound mind; that
The petitioner spouses aver that their payments to Virgilio beginning November, 1993 were Lucila and Laila accomplished this by affixing Luis’ thumb mark on the SPA which
payments made in good faith to a person in possession of the credit, in consonance with purportedly authorized Laila to sell and convey, among others, Lot Nos. 8, 22
Article 1242 of the Civil Code.18 This therefore released them from their obligation. They claim and 23, which had already been sold to them; and that on the strength of
that Virgilio collected the rentals in his capacity as a co-owner. Being a son of Virginia, he was another SPA7 by Luis, dated July 21, 1993 (Second SPA), respondents Laila and
also entitled to the rent of the subject house. We disagree. Virgilio collected the rentals not as Ham mortgaged Lot No. 19 to Vital Lending Investors, Inc. for and in
a co-owner but as the alleged sole owner of the subject house. The petitioner spouses consideration of the amount of P150,000.00 with the concurrence of Lourdes.8
themselves admitted that Virgilio claimed sole ownership of the house and lot. It would be
incongruous for them to now assert payment in good faith to a person they believed was * Petitioners further averred that a second sale took place on August 23, 1994,
collecting in behalf of his co-heirs after admitting that they paid rent to Virgilio as the sole when the respondents made Luis sign the Deed of Absolute Sale9 conveying to
owner thereof. Meridian three (3) parcels of residential land for P960,500.00 (Second Sale); that
Meridian was in bad faith when it did not make any inquiry as to who were the
occupants and owners of said lots; and that if Meridian had only investigated, it
would have been informed as to the true status of the subject properties and
Hence, for violating of the terms of the lease contract, i.e., payment of rent, respondents can would have desisted in pursuing their acquisition.
legally demand the ejectment of the petitioner spouses.
* Petitioners, thus, prayed that they be awarded moral damages, exemplary
damages, attorney’s fees, actual damages, and litigation expenses and that the
two SPAs and the deed of sale in favor of Meridian be declared null and void ab
initio.10
WHEREFORE, the decision dated January 10, 1997 of the Court of Appeals is hereby
AFFIRMED. With costs against the petitioners.
* On their part, respondents Lucila and Laila contested the First Sale in favor of petitioners.
They submitted that even assuming that it was valid, petitioners were estopped from
questioning the Second Sale in favor of Meridian because they failed not only in effecting the
necessary transfer of the title, but also in annotating their interests on the titles of the
SO ORDERED. questioned properties. With respect to the assailed SPAs and the deed of absolute sale
executed by Luis, they claimed that the documents were valid because he was conscious and
of sound mind and body when he executed them. In fact, it was Luis together with his wife
who received the check payment issued by Meridian where a big part of it was used to foot his
Puno, Panganiban, Sandoval-Gutierrez, and Carpio-Morales, JJ., concur. hospital and medical expenses.11

* Respondent Meridian, in its Answer with Compulsory Counterclaim, averred


that Luis was fully aware of the conveyances he made. In fact, Sophia Sanchez
(Sanchez), Vice-President of the corporation, personally witnessed Luis affix his
Disputable presumptions thumb mark on the deed of sale in its favor. As to petitioners’ contention that
Meridian acted in bad faith when it did not endeavor to make some inquiries as
THIRD DIVISION to the status of the properties in question, it countered that before purchasing
the properties, it checked the titles of the said lots with the Register of Deeds of

324
RECTO, GAYLE ANGELI M.
2011-0008 | AUSL
Personal Notes on Remedial Law 2 Review (based on the syllabus of Prof. Henedino M. Brondial)

Cebu and discovered therein that the First Sale purportedly executed in favor of * With regard to petitioners’ assertion that the First SPA was revoked by Luis
the plaintiffs was not registered with the said Register of Deeds. Finally, it argued when he executed the affidavit, dated November 24, 1994, the CA ruled that the
that the suit against it was filed in bad faith.12 Second Sale remained valid. The Second Sale was transacted on August 23,
1994, before the First SPA was revoked. In other words, when the Second Sale
* On her part, Lourdes posited that her signature as well as that of Luis was consummated, the First SPA was still valid and subsisting. Thus, "Meridian
appearing on the deed of sale in favor of petitioners, was obtained through had all the reasons to rely on the said SPA during the time of its validity until the
fraud, deceit and trickery. She explained that they signed the prepared deed out time of its actual filing with the Register of Deeds considering that constructive
of pity because petitioners told them that it was necessary for a loan application. notice of the revocation of the SPA only came into effect upon the filing of the
In fact, there was no consideration involved in the First Sale. With respect to the Adverse Claim and the aforementioned Letters addressed to the Register of
Second Sale, she never encouraged the same and neither did she participate in Deeds on 17 December 1994 and 25 November 1994, respectively, informing the
it. It was purely her husband’s own volition that the Second Sale materialized. Register of Deeds of the revocation of the first SPA."18 Moreover, the CA
She, however, affirmed that she received Meridian’s payment on behalf of her observed that the affidavit revoking the first SPA was also revoked by Luis on
husband who was then bedridden.13 December 12, 1994.19

* After the case was submitted for decision, the RTC ruled in favor of petitioners. It held that * Furthermore, although Luis revoked the First SPA, he did not revoke the
when Luis executed the second deed of sale in favor of Meridian, he was no longer the owner Second SPA which authorized respondent Laila to sell, convey and mortgage,
of Lot Nos. 19, 22 and 23 as he had already sold them to his children by his first marriage. In among others, the property covered by TCT T-11155 (Lot No. 19). The CA
fact, the subject properties had already been delivered to the vendees who had been living opined that had it been the intention of Luis to discredit the Second Sale, he
there since birth and so had been in actual possession of the said properties. The trial court should have revoked not only the First SPA but also the Second SPA. The latter
stated that although the deed of sale was not registered, this fact was not prejudicial to their being valid, all transactions emanating from it, particularly the mortgage of Lot
interest. It was of the view that the actual registration of the deed of sale was not necessary 19, its subsequent redemption and its second sale, were valid.20 Thus, the CA
to render a contract valid and effective because where the vendor delivered the possession of disposed in this wise:
the parcel of land to the vendee and no superior rights of third persons had intervened, the
efficacy of said deed was not destroyed. In other words, Luis lost his right to dispose of the WHEREFORE, the appeal is hereby GRANTED. The Decision dated 30 July 2004 is
said properties to Meridian from the time he executed the first deed of sale in favor of hereby REVERSED AND SET ASIDE, and in its stead a new decision is hereby
petitioners. The same held true with his alleged sale of Lot 8 to Lucila Soria.14 Specifically, the rendered:
dispositive portion of the RTC decision reads:
1. DECLARING the Special Power of Attorney, dated 21 July 1993, as valid;
IN VIEW OF THE FOREGOING, the Court finds that a preponderance of evidence
exists in favor of the plaintiffs and against the defendants. Judgment is hereby 2. DECLARING the Special Power of Attorney, dated 03 April 1993, as valid up to
rendered: the time of its revocation on 24 November 1994;

a. Declaring that the Special Power of Attorney, Exhibit "K," for the plaintiffs and 3. DECLARING the Deed of Absolute sale, dated 04 November 1991, as
Exhibit "3" for the defendants null and void including all transactions subsequent ineffective and without any force and effect;
thereto and all proceedings arising therefrom;
4. DECLARING the Deed of Absolute Sale of Three (3) Parcels of Residential
b. Declaring the Deed of Sale marked as Exhibit "E" valid and binding; Land, dated 23 August 1994, valid and binding from the very beginning;

c. Declaring the Deed of Absolute Sale of Three (3) Parcels of Residential Land 5. DECLARING the Deed of Absolute Sale, dated 27 September 1994, also valid
marked as Exhibit "F" null and void from the beginning; and binding from the very beginning;

d. Declaring the Deed of Sale, Exhibit "16" (Solutan) or Exhibit "FF," null and void 6. ORDERING the substituted plaintiffs to pay jointly and severally the defendant-
from the beginning; appellant Meridian Realty Corporation the sum of Php100,000.00 as moral
damages, Php100,000.00 as attorney’s fee and Php100,000.00 as litigation
e. Declaring the vendees named in the Deed of Sale marked as Exhibit "E" to be expenses; and
the lawful, exclusive and absolute owners and possessors of Lots Nos. 8, 19, 22,
and 23; 7. ORDERING the substituted plaintiffs to pay jointly and severally the defendant-
appellants Leila Solutan et al., the sum of Php50,000.00 as moral damages.
f. Ordering the defendants to pay jointly and severally each plaintiff P50,000.00
as moral damages; and SO ORDERED.21

g. Ordering the defendants to pay plaintiffs P50,000.00 as attorney’s fees; and * Petitioners filed a motion for reconsideration, but it was denied in the CA Resolution,22
P20,000.00 as litigation expenses. dated November 18, 2010. Consequently, they filed the present petition.

The crossclaim made by defendant Meridian Realty Corporation against


defendants Soria and Solutan is ordered dismissed for lack of sufficient
evidentiary basis.
Issues

SO ORDERED."15
I. THE HONORABLE COURT OF APPEALS (19TH DIVISION) GRAVELY ERRED WHEN IT
DECLARED AS VOID THE FIRST SALE EXECUTED BY THE LATE LUIS ROSAROSO IN FAVOR OF
* On appeal, the CA reversed and set aside the RTC decision. The CA ruled that the first deed HIS CHILDREN OF HIS FIRST MARRIAGE.
of sale in favor of petitioners was void because they failed to prove that they indeed tendered
a consideration for the four (4) parcels of land. It relied on the testimony of Lourdes that
II. THE HONORABLE COURT OF APPEALS GRAVELY ERRED IN NOT SUSTAINING AND
petitioners did not pay her husband. The price or consideration for the sale was simulated to
AFFIRMING THE RULING OF THE TRIAL COURT DECLARING THE MERIDIAN REALTY
make it appear that payment had been tendered when in fact no payment was made at all.16
CORPORATION A BUYER IN BAD FAITH, DESPITE THE TRIAL COURT’S FINDINGS THAT THE
DEED OF SALE (First Sale), IS GENUINE AND HAD FULLY COMPLIED WITH ALL THE LEGAL
* With respect to the validity of the Second Sale, the CA stated that it was valid FORMALITIES.
because the documents were notarized and, as such, they enjoyed the
presumption of regularity. Although petitioners alleged that Luis was manipulated
III. THE HONORABLE COURT OF APPEALS FURTHER ERRED IN NOT HOLDING THE SALE
into signing the SPAs, the CA opined that evidence was wanting in this regard.
(DATED 27 SEPTEMBER 1994), NULL AND VOID FROM THE VERY BEGINNING SINCE LUIS
Dr. Arlene Letigio Pesquira, the attending physician of Luis, testified that while
ROSAROSO ON NOVEMBER 4, 1991 WAS NO LONGER THE OWNER OF LOTS 8, 19, 22 AND 23
the latter was physically infirmed, he was of sound mind when he executed the
AS HE HAD EARLIER DISPOSED SAID LOTS IN FAVOR OF THE CHILDREN OF HIS (LUIS
first SPA.17
ROSAROSO) FIRST MARRIAGE.23

325
RECTO, GAYLE ANGELI M.
2011-0008 | AUSL
Personal Notes on Remedial Law 2 Review (based on the syllabus of Prof. Henedino M. Brondial)

Ruling presented and offered, will prevail. The burden of proof remains where it is but, by
the presumption, the one who has that burden is relieved for the time being from
Petitioners argue that the second deed of sale was null and void because Luis could not have introducing evidence in support of the averment, because the presumption stands
validly transferred the ownership of the subject properties to Meridian, he being no longer the in the place of evidence unless rebutted.30
owner after selling them to his children. No less than Atty. William Boco, the lawyer who
notarized the first deed of sale, appeared and testified in court that the said deed was the one
he notarized and that Luis and his second wife, Lourdes, signed the same before him. He also
identified the signatures of the subscribing witnesses.24 Thus, they invoke the finding of the In this case, the respondents failed to trounce the said presumption. Aside from
RTC which wrote: their bare allegation that the sale was made without a consideration, they failed to
supply clear and convincing evidence to back up this claim. It is elementary in
procedural law that bare allegations, unsubstantiated by evidence, are not
equivalent to proof under the Rules of Court.31
In the case of Heirs of Joaquin Teves, Ricardo Teves versus Court of Appeals, et al., G.R. No.
109963, October 13, 1999, the Supreme Court held that a public document executed [with] all
the legal formalities is entitled to a presumption of truth as to the recitals contained therein. In
order to overthrow a certificate of a notary public to the effect that a grantor executed a The CA decision ran counter to this established rule regarding disputable
certain document and acknowledged the fact of its execution before him, mere preponderance presumption. It relied heavily on the account of Lourdes who testified that the
of evidence will not suffice. Rather, the evidence must (be) so clear, strong and convincing as to children of Luis approached him and convinced him to sign the deed of sale,
exclude all reasonable dispute as to the falsity of the certificate. When the evidence is explaining that it was necessary for a loan application, but they did not pay the
conflicting, the certificate will be upheld x x x . purchase price for the subject properties.32 This testimony, however, is self-
serving and would not amount to a clear and convincing evidence required by law
to dispute the said presumption. As such, the presumption that there was sufficient
consideration will not be disturbed.
A notarial document is by law entitled to full faith and credit upon its face. (Ramirez vs. Ner,
21 SCRA 207). As such it … must be sustained in full force and effect so long as he who
impugns it shall not have presented strong, complete and conclusive proof of its falsity or
nullity on account of some flaw or defect provided against by law (Robinson vs. Villafuerte, 18 Granting that there was no delivery of the consideration, the seller would have no right to sell
Phil. 171, 189-190).25 again what he no longer owned. His remedy would be to rescind the sale for failure on the
part of the buyer to perform his part of their obligation pursuant to Article 1191 of the New
Civil Code. In the case of Clara M. Balatbat v. Court Of Appeals and Spouses Jose Repuyan
and Aurora Repuyan,33 it was written:
Furthermore, petitioners aver that it was erroneous for the CA to say that the records of the
case were bereft of evidence that they paid the price of the lots sold to them. In fact, a
perusal of the records would reveal that during the cross-examination of Antonio Rosaroso,
when asked if there was a monetary consideration, he testified that they indeed paid their The failure of the buyer to make good the price does not, in law, cause the ownership to
father and their payment helped him sustain his daily needs.26 revest to the seller unless the bilateral contract of sale is first rescinded or resolved pursuant
to Article 1191 of the New Civil Code. Non-payment only creates a right to demand the
fulfillment of the obligation or to rescind the contract. [Emphases supplied]

Petitioners also assert that Meridian was a buyer in bad faith because when its representative
visited the site, she did not make the necessary inquiries. The fact that there were already
houses on the said lots should have put Meridian on its guard and, for said reason, should Meridian is Not a
have made inquiries as to who owned those houses and what their rights were over the
same.27
Buyer in Good Faith

Meridian’s assertion that the Second Sale was registered in the Register of Deeds was a falsity.
Respondents Meridian and Lucila argue that, granting that the First Sale was valid, the
The subject titles, namely: TCT No. 11155 for Lot 19, TCT No. 10885 for Lot 22, and TCT No.
properties belong to them as they acquired these in good faith and had them first recorded in
10886 for Lot 23 were free from any annotation of the alleged sale.28
the Registry of Property, as they were unaware of the First Sale.34

After an assiduous assessment of the records, the Court finds for the petitioners.
Again, the Court is not persuaded.

The First Deed Of Sale Was Valid


The fact that Meridian had them first registered will not help its cause. In case of double sale,
Article 1544 of the Civil Code provides:

The fact that the first deed of sale was executed, conveying the subject properties
in favor of petitioners, was never contested by the respondents. What they
ART. 1544. If the same thing should have been sold to different vendees, the ownership shall
vehemently insist, though, is that the said sale was simulated because the
be transferred to the person who may have first possession thereof in good faith, if it should
purported sale was made without a valid consideration.
be movable property.

Under Section 3, Rule 131 of the Rules of Court, the following are disputable
Should it be immovable property, the ownership shall belong to the person acquiring it who in
presumptions: (1) private transactions have been fair and regular; (2) the ordinary
good faith first recorded it in the Registry of Property.
course of business has been followed; and (3) there was sufficient consideration
for a contract.29 These presumptions operate against an adversary who has not
introduced proof to rebut them. They create the necessity of presenting evidence
to rebut the prima facie case they created, and which, if no proof to the contrary is

326
RECTO, GAYLE ANGELI M.
2011-0008 | AUSL
Personal Notes on Remedial Law 2 Review (based on the syllabus of Prof. Henedino M. Brondial)

Should there be no inscription, the ownership shall pertain to the person who in good faith Prescinding from the foregoing, the fact that private respondent RRC did not investigate the
was first in possession; and, in the absence thereof; to the person who presents the oldest Sarmiento spouses' claim over the subject land despite its knowledge that Pedro Ogsiner, as
title, provided there is good faith. their overseer, was in actual possession thereof means that it was not an innocent purchaser
for value upon said land. Article 524 of the Civil Code directs that possession may be exercised
in one's name or in that of another. In herein case, Pedro Ogsiner had informed RRC that he
was occupying the subject land on behalf of the Sarmiento spouses. Being a corporation
engaged in the business of buying and selling real estate, it was gross negligence on its part
Otherwise stated, ownership of an immovable property which is the subject of a double sale
to merely rely on Mr. Puzon's assurance that the occupants of the property were mere
shall be transferred: (1) to the person acquiring it who in good faith first recorded it in the
squatters considering the invaluable information it acquired from Pedro Ogsiner and
Registry of Property; (2) in default thereof, to the person who in good faith was first in
considering further that it had the means and the opportunity to investigate for itself the
possession; and (3) in default thereof, to the person who presents the oldest title, provided
accuracy of such information. [Emphases supplied]
there is good faith. The requirement of the law then is two-fold: acquisition in good faith and
registration in good faith. Good faith must concur with the registration. If it would be shown
that a buyer was in bad faith, the alleged registration they have made amounted to no
registration at all.
In another case, it was held that if a vendee in a double sale registers the sale after he has
acquired knowledge of a previous sale, the registration constitutes a registration in bad faith
and does not confer upon him any right. If the registration is done in bad faith, it is as if there is
no registration at all, and the buyer who has first taken possession of the property in good
The principle of primus tempore, potior jure (first in time, stronger in right) gains greater
faith shall be preferred.38
significance in case of a double sale of immovable property. When the thing sold twice is an
immovable, the one who acquires it and first records it in the Registry of Property, both made in
good faith, shall be deemed the owner. Verily, the act of registration must be coupled with
good faith— that is, the registrant must have no knowledge of the defect or lack of title of his
vendor or must not have been aware of facts which should have put him upon such inquiry In the case at bench, the fact that the subject properties were already in the possession of
and investigation as might be necessary to acquaint him with the defects in the title of his persons other than Luis was never disputed. Sanchez, representative and witness for Meridian,
vendor.)35 [Emphases and underlining supplied] even testified as follows:

When a piece of land is in the actual possession of persons other than the seller, the buyer x x x; that she together with the two agents, defendant Laila Solutan and Corazon Lua, the
must be wary and should investigate the rights of those in possession. Without making such president of Meridian Realty Corporation, went immediately to site of the lots; that the agents
inquiry, one cannot claim that he is a buyer in good faith. When a man proposes to buy or brought with them the three titles of the lots and Laila Solutan brought with her a special
deal with realty, his duty is to read the public manuscript, that is, to look and see who is there power of attorney executed by Luis B. Rosaroso in her favor but she went instead directly to
upon it and what his rights are. A want of caution and diligence, which an honest man of Luis Rosaroso to be sure; that the lots were pointed to them and she saw that there were
ordinary prudence is accustomed to exercise in making purchases, is in contemplation of law, houses on it but she did not have any interest of the houses because her interest was on the
a want of good faith. The buyer who has failed to know or discover that the land sold to him is lots; that Luis Rosaroso said that the houses belonged to him; that he owns the property and
in adverse possession of another is a buyer in bad faith.36 In the case of Spouses Sarmiento that he will sell the same because he is very sickly and he wanted to buy medicines; that she
v. Court of Appeals,37 it was written: requested someone to check the records of the lots in the Register of Deeds; that one of the
titles was mortgaged and she told them to redeem the mortgage because the corporation will
buy the property; that the registered owner of the lots was Luis Rosaroso; that in more or less
three months, the encumbrance was cancelled and she told the prospective sellers to prepare
the deed of sale; that there were no encumbrances or liens in the title; that when the deed of
Verily, every person dealing with registered land may safely rely on the correctness of the
absolute sale was prepared it was signed by the vendor Luis Rosaroso in their house in Opra x
certificate of title issued therefor and the law will in no way oblige him to go behind the
x x.39 (Underscoring supplied)
certificate to determine the condition of the property. Thus, the general rule is that a
purchaser may be considered a purchaser in good faith when he has examined the latest
certificate of title. An exception to this rule is when there exist important facts that would
create suspicion in an otherwise reasonable man to go beyond the present title and to
investigate those that preceded it. Thus, it has been said that a person who deliberately From the above testimony, it is clear that Meridian, through its agent, knew that the subject
ignores a significant fact which would create suspicion in an otherwise reasonable man is not properties were in possession of persons other than the seller. Instead of investigating the
an innocent purchaser for value. A purchaser cannot close his eyes to facts which should put a rights and interests of the persons occupying the said lots, however, it chose to just believe
reasonable man upon his guard, and then claim that he acted in good faith under the belief that Luis still owned them. Simply, Meridian Realty failed to exercise the due diligence required
that there was no defect in the title of the vendor. As we have held: by law of purchasers in acquiring a piece of land in the possession of person or persons other
than the seller.

The failure of appellees to take the ordinary precautions which a prudent man would have
taken under the circumstances, specially in buying a piece of land in the actual, visible and In this regard, great weight is accorded to the findings of fact of the RTC. Basic is the rule that
public possession of another person, other than the vendor, constitutes gross negligence the trial court is in a better position to examine real evidence as well as to observe the
amounting to bad faith. demeanor of witnesses who testify in the case.40

In this connection, it has been held that where, as in this case, the land sold is in the WHEREFORE, the petition is GRANTED. The December 4, 2009 Decision and the November 18,
possession of a person other than the vendor, the purchaser is required to go beyond the 201 0 Resolution of the Court of Appeals, in CA-G.R. CV No. 00351, are REVERSED and SET
certificate of title to make inquiries concerning the rights of the actual possessor. Failure to do ASIDE. The July 30, 2004 Decision of the Regional Trial Court, Branch 8, 7th Judicial Region,
so would make him a purchaser in bad faith. (Citations omitted). Cebu City, in Civil Case No. CEB-16957, is hereby REINSTATED.

One who purchases real property which is in the actual possession of another should, at least SO ORDERED.
make some inquiry concerning the right of those in possession. The actual possession by other
than the vendor should, at least put the purchaser upon inquiry. He can scarely, in the
absence of such inquiry, be regarded as a bona fide purchaser as against such possessors.
(Emphases supplied)
Velasco Jr., Peralta, Abad, Leonen, JJ., concur.

327
RECTO, GAYLE ANGELI M.
2011-0008 | AUSL
Personal Notes on Remedial Law 2 Review (based on the syllabus of Prof. Henedino M. Brondial)

FIRST DIVISION ownership, but could not present any.24 Thus, from 1960 onwards, petitioners
enjoyed the fruits of the property.25 Later, respondents were able to find a copy of
G.R. No. 175874 December 11, 2013 the assailed deed in the National Archives, thereby enabling them to cause the
issuance of Tax Declaration No. 23959.26
HEIRS OF CIPRIANO TRAZONA, Namely: FRANCISCA T. MATBAGON, NATIVIDAD T.
ABADIANO, CARLITO C. TRAZONA; and Heirs of EDELBERTO C. TRAZONA * In the presentation of their rebuttal evidence, petitioners presented a Deed of
represented by his daughter DOMICINA T. ARANAS, ELADIA T. ALICAMEN (Now Absolute Sale dated 11 April 1953,27 executed by Pilar Diaz in favor of Dionisio.
Deceased) Substituted by DOMINGO ALICAMEN, LUPECIO ALICAMEN, REBECCA This prior sale involved the exact same portion allegedly sold to him by Cipriano
ALICAMEN-BALBUTIN, ELSEI ALICAMEN, GLENN ALICAMEN, LENNEI ALICAMEN- - except that in the date of approval of the subdivision plan by the Director of
GEONZON, DANILO ALICAMEN, JOVELYN ALICAMEN-VILLETA, JIMBIE ALICAMEN Lands, two figures were interchanged. Whereas the assailed deed showed the
and HERMOGENES C. TRAZONA (Now Deceased) Substituted by LILYBETH date as "September 5, 1953," the Deed of Absolute Sale dated 11 April 1953
TRAZONA-MANGILA, GEMMA TRAZONA, ELIZALDE TRAZONA, BOBBY TRAZONA, showed the date as "September 5, 1935."
and PALABIANA B. TRAZONA, Petitioners,
* In its Decision dated 6 April 2004, the RTC annulled the assailed deed and ordered the
vs. HEIRS OF DIONISIO CANADA, Namely: ROSITA C. GERSALINA, CONCEPTION C. cancellation of Tax Declaration No. 23959, as well as the reinstatement of Tax Declaration No.
GEONZON, DANIEL CANADA, GORGONIO CANADA, LEOPOLDO CANADA, SUSANA C. 07764.28 Respondents were also ordered to demolish their residential house on Lot No. 5053-
DUNGOG, LUZVIMINDA C. TABUADA, AND CEFERINA CANADA; PROVINCIAL H and to pay petitioners attorney’s fees and litigation expenses.29
ASSESSOR of Cebu and MUNICIPAL ASSESSOR of Minglanilla, Cebu, Respondents.
* The RTC found that respondents’ failure to present the deed for 40 years from
SERENO, CJ.: its alleged execution had not been satisfactorily and convincingly explained.30 It
also found that the assailed deed was indeed a forgery for the following reasons:

1. It would have been pointless for Dionisio to buy the same property twice from
different owners.
Facts

2. Cipriano’s residence certificate, whose number was indicated in the assailed


* Petitioners are heirs of Cipriano Trazona (Cipriano), who owned an untitled parcel of land deed, as well as in the notarial register where the deed was recorded, was
referred to as Lot No. 5053-H. The property, located in Minglanilla, Cebu, is covered by Tax allegedly issued in Minglanilla, Cebu. The other persons’ residence certificates,
Declaration No. 07764 and has an area of 9,515 square meters.4 The land was purchased whose numbers were indicated on the same page of the notarial register, appear
from the government in 1940.5 Since then, Cipriano had taken possession of the land, to have come from the same booklet as the residence certificate of Cipriano,
cultivated it and diligently paid taxes thereon.6 judging from their numerical sequence. However, the residence certificates of
these other persons had been issued in Sogod, Cebu.
* In 1949, Dionisio bought the adjacent parcel of land from Pilar Diaz.7 It was later found
that he had encroached on a small portion of Lot No. 5053-H. He was then summoned by 3. There was indeed a glaring difference between the alleged signature of
Cipriano for a confrontation before the barangay captain in 1952.8 Dionisio offered to buy the Cipriano in the assailed deed and in his standard signatures in 10 other
encroached portion, but Cipriano refused the offer.9 In 1956, the latter gave Dionisio documents submitted by plaintiffs.
permission to temporarily build a house on said portion, where it still stands.10 No action for
ejectment was filed against Dionisio during the lifetime of Cipriano,11 who eventually died on
* Respondents filed a Notice of Appeal dated 30 April 2004.
18 May 1982.12 The latter’s son Hermogenes, one of the petitioners herein who had cultivated
the lot since 1972, took over.13 On 24 March 1992, Dionisio died.14
* On 25 May 2006, the CA issued a Decision reversing that of the RTC. The appellate court
ruled that petitioners had failed to prove by requisite evidence their allegation that the assailed
* The present controversy arose in 1997. Petitioners went to the Office of the Municipal
deed was a forgery.31 The deed, being a notarized document, enjoyed the presumption of
Assessor to secure a copy of Tax Declaration No. 07764, as they intended to sell Lot No. 5053-
authenticity and due execution. Also, the fact that it was an ancient document that "remained
H to an interested buyer.15 To their surprise, they were informed that Tax Declaration No.
unaltered after so many years, bodes well for its authenticity."32
07764 had been cancelled and, in lieu thereof, Tax Declaration No. 23959 was issued on 24
June 1996 in the name of Dionisio.16 Apparently, respondents had caused the issuance of Tax
Declaration No. 23959 by submitting a Deed of Absolute Sale dated 27 June 1956 supposedly * The CA also concluded that the document examiner was not able to determine
executed by Cipriano in favor of Dionisio.17 That sale involved a portion of Lot No. 5053-H the forgery with certainty. What he had examined was a mere machine copy of
described as follows: the assailed deed.33 Furthermore, even he admitted that the standard signatures
of Cipriano had shown variations among themselves.

x x x that portion of land of Lot No. FIVE THOUSAND FIFTY THREE-H (5053-H)
under subdivision plan FLR-133 approved by the Director of Lands Jose P. Dans * Finally, the CA ruled that respondents were the actual possessors of Lot No.
on September 5, 1953, covered by monuments No. 7, 8, 9, 10, 11, of said Lot 5053-H, since it was their house that was standing on the property.34 Thus, the
No. 5053 bounded on the North by Lot No. 5954 & portion of Lot 5053-H; East CA granted the appeal and consequently dismissed the Complaint of petitioners.
by portion of Lot 5053-H; South by Lot no. 5053-J of Domingo Ababon; West by
Lot no. 9479; x x x.18

* Petitioners summoned respondents before the Lupon Tagapamayapa, but the conciliation Issues
was not successful.19 On 28 July 1997, petitioners filed a Complaint20 against respondents for
quieting of title, annulment of deed of sale, cancellation of Tax Declaration No. 23959, 1. Ruling that petitioners were not able to overturn the presumption of regularity of the
recovery of possession and ownership, damages, and payment of attorney’s fees. Petitioners assailed deed;
alleged therein that the Deed of Absolute Sale dated 27 June 1956 was a forgery.
Respondents, in their Answer,21 alleged that the assailed deed was a genuine document and
2. Finding that the document examiner was not able to establish the forgery with certainty;
asked for the payment of moral and exemplary damages, and attorney’s fees, as
counterclaims.
3. Finding that respondents were in actual possession of Lot No. 5053-H;

* During trial, among the witnesses presented by petitioners was Romeo O. Varona, document
4. Ruling that there was no merit in petitioners’ prayer for the award of attorney’s fees and
examiner of the Philippine National Police Crime Laboratory, Region VII. He testified that
litigation expenses.
according to his comparative analysis of Cipriano’s signature on the assailed deed and
standard signatures on other documents, Cipriano’s signature on the deed in question was a
forgery.22

* For their part, respondents presented Dionisio’s son Gorgonio, who testified Ruling
that he was present when the assailed deed was executed.23 He also stated that
they had enjoyed the fruits of the lot in question from 1956 until 1960, when Petitioners presented clear and convincing
they were confronted by petitioners. Respondents were asked to show proof of

328
RECTO, GAYLE ANGELI M.
2011-0008 | AUSL
Personal Notes on Remedial Law 2 Review (based on the syllabus of Prof. Henedino M. Brondial)

evidence that the assailed deed is a forgery.

The fact that the document examiner himself admitted that even the standard signatures of
Cipriano showed variations among themselves does not make the former’s determination any
Well-settled is the rule that petitions for review on certiorari under Rule 45 before this Court less convincing. He explained that while every signature of the same person varies, the
should involve only questions of law.36 A reading of the issues raised by petitioners readily individual handwriting characteristics of the person remain the same.42 In Cesar v.
show that they are questions of fact, which are generally not within the purview of this Court. Sandiganbayan,43 we recognized that there is bound to be some variation in the different
When a question involves facts, the findings of the CA, including the probative weight samples of genuine signatures of the same person.
accorded to certain pieces of evidence, are binding on this Court. Also well-settled, however,
are exceptions to this rule,37 such as when the findings of fact of the CA are contrary to those
of the RTC, as in this case.
Second, the RTC did not just rely on expert testimony in ruling that the signature was forged.
It likewise supported its finding that the signature was forged through independent
observation:
We sustain the findings of the RTC.

Finally, a scrutiny of the signature on the questioned deed of sale compared to the eleven (11)
At the outset, it is worth pointing out that the sale of a mere portion of Lot No. 5053-H was signatures on the ten (10) standard documents there exists a glaring difference in the letter
what brought about the cancellation of Tax Declaration No. 07764 and the consequent formation of capital letters "C" in Cipriano and "T" in Trazona. The capital C in questioned
issuance of Tax Declaration No. 23959, each of which covered the entire lot. The fact that the signature, the initial stroke stopped at the upper curve of the letter C while in the standard
assailed deed covers only a portion of Lot No. 5053-H becomes clearer still when one signatures, it overlaps from the upper curve. In the word Trazona, the capital T in the
considers that it was bounded on the north and the east by portions of Lot No. 5053-H itself. questioned signature is disconnected from the T bar to the body of the questioned signature
whereas, in the standard signatures, the capital T is connected. These discrepancies can easily
be noticed by mere physical appearance that the letters C and T were written.44

As will be shown below, the assailed deed is a forgery. Assuming it were genuine, petitioners
have a right to the rest of the property not covered by the purported sale. If the procedure for
the issuance of tax declarations was followed - if care had been observed to make sure that all Third, the existence of the Deed of Absolute Sale dated 11 April 1953 brings into question the
papers were in order and understood - this irregularity would not have taken place. regularity of the assailed deed. This deed was never disputed by respondents at any stage of
the proceedings, and was in fact admitted by them in their Comments to Plaintiffs’ Additional
Formal Offer of Exhibits.45 Indeed, the RTC was correct in its observation that no one in
complete possession of one’s mental faculties would buy the same property twice from
different owners. Respondents never provided any explanation for this anomalous situation. In
It is true that notarized documents are accorded evidentiary weight as regards
any case, it has been established that Lot No. 5053-H is in the name of Cipriano, who bought
their due execution.38 Nevertheless, while notarized documents enjoy the
it from the government in 1940. Thus, only Cipriano had the right to dispose of the property,
presumption of regularity, this presumption is disputable. They can be contradicted
or portions thereof.
by evidence that is clear, convincing, and more than merely preponderant.39 Here,
contrary to the conclusion of the CA, we find clear and convincing evidence that is
enough to overturn the presumption of regularity of the assailed deed.

Fourth, Cipriano had cultivated the property and paid taxes thereon since the time he acquired
it from the government, and even after its purported sale to Dionisio, until his death.46
Petitioners continued paying the taxes thereon even after Cipriano had died.47 Respondents
First, the document examiner determined that the signature of Cipriano in the assailed deed
started paying taxes on the property only after Tax Declaration No. 23959 was issued in
had been forged. No issue has been raised about his expertise. The finding of the CA that he
Dionisio’s name in 1997.48 It would be absurd for petitioners to pay taxes on a property they
had examined a mere machine copy of the assailed deed was erroneous. The pertinent portion
do not own.
of his testimony clearly shows otherwise, to wit:

Fifth, as admitted by Gorgonio himself, petitioners were the ones enjoying the fruits of the
ATTY. DURANO:
property from 1960 until the present controversy.49 Again, it is incongruous for petitioners to
enjoy the fruits if respondents owned the property.
Q: Now you made mention of the standard documents, could you kindly tell the Honorable
Court what is [the] questioned document stated in your report?

Sixth, as the RTC noted, there was an irregularity regarding the place of issuance of Cipriano’s
residence certificate indicated in the assailed deed, as compared with the residence certificates
[ROMEO O. VARONA] of the other persons indicated on the same page of the notarial register.

[A]: The questioned document is the Deed of Absolute Sale dated June 27, 1956.

Finally, when the record management analyst from the Bureau of Archives presented the
assailed deed, the paper was noted to be white, while its supposed contemporaries in the
Q: Do you have a copy of that Deed of Sale as examined by you? bunch from where it was taken had turned yellow with age. 50 Further, when the analyst was
asked the question of when- the assailed deed was received by the Bureau of Archives, she
A: Well, I have a machine copy. I have examined the original copy at the archive’s office, answered that it was forwarded to them only on 28 September 1987 by RTC Region 7,
Mandaue City.40 (Emphasis supplied) Notarial Division.51

In concluding that the signature of Cipriano in the assailed deed was a forgery, the document Clearly, the evidence adduced fully supports the position of petitioners that the assailed deed
examiner found that there were "significant differences in letter formation, construction and of sale is forged and that they are the owners of the property. Having been forced to litigate in
other individual handwriting characteristics" between the assailed and the standard signatures order to protect their interest therein, the award of attorney's fees and litigation expenses to
of Cipriano.41 them is in order.

329
RECTO, GAYLE ANGELI M.
2011-0008 | AUSL
Personal Notes on Remedial Law 2 Review (based on the syllabus of Prof. Henedino M. Brondial)

IE - hymenal tear, recent 6", 9" don't bleed on manipulation, but complained of
tenderness upon insertion of 1 finger, copious vaginal discharge.3
The actual possession of Lot No. 5053-H by petitioners has been properly ruled on by the RTC.
Much has been made by the CA of the fact that respondents' house was standing on the * According to Dr. Claveria, there is a possibility that the fluids found inside
property. However, petitioners have explained that the house was erected only after Cipriano Rowena's vagina may be semen. She added that it was possible for Rowena to
permitted it. have only two hymenal tears even if four men had sexual intercourse with her.

* Dr. Chona C. Belmonte, a psychiatrist of Cadlan Mental Hospital in Pili, Camarines Sur,
testified that while she interviewed Rowena, the latter was crying, incoherent and had
Dionisio was then well aware that this temporary arrangement may be terminated at any time. shouting episodes. She was confined at the Cadlan Mental Hospital for further treatment. Upon
Respondents cannot now refuse to vacate the property or eventually demand reimbursement further medical consultation, Dr. Belmonte observed thus:
of necessary and useful expenses under Articles 448 and 546 of the New Civil Code, because
the provisions apply only to a possessor in good faith, i.e., one who builds on land with the Rowena was in a depressed mood and at the same time overactive. She was
belief that he is the owner thereof. 52 Persons who occupy land by virtue of tolerance of the combative, violent, and was experiencing auditory hallucination, meaning, she
owners are not possessors in good faith. 53 Thus, the directive of the RTC for respondents to heard things that only she could hear. She was also grandiously deluded, falsely
demolish their residential house on Lot No. 5053-H was also proper. believing that she could do things others could not do. By that time, according to
Dr. Belmonte, Rowena had already lost touch with reality.4

* Dr. Belmonte diagnosed her illness as "Acute Psychotic Depressive Condition."5


WHEREFORE, the Decision and Resolution of the Court of Appeals Cebu City in CA-G.R. CV No. She found that her mental disorder was not hereditary because before the
00099 are REVERSED and SET ASIDE. The Decision of the Regional Trial Court of Cebu City, incident took place, she did not exhibit any unusual behavior. She concluded that
Branch 57, in Civil Case No. CEB-20620 is REINSTATED in all respects. her mental illness was strongly related to a traumatic experience. She noted that
at one point in the treatment, Rowena confided to her that "she was raped."6

* Roberto Padrigone a.k.a. Roberto San Miguel, Michael San Antonio, Jocel Ibaneta and
Abelardo Triumpante were charged with rape in an amended information which reads:
SO ORDERED.

That on or about the 3rd day of January, 1995, in Salvacion, Buhi, Camarines
Sur, Philippines and within the jurisdiction of this Honorable Court, the above-
named accused, conspiring, confederating together and mutually helping each
Leonardo-De Castro, Bersamin, Villarama Jr., Reyes, JJ. Concur.
other and by means of force and intimidation, did then and there willfully,
unlawfully and feloniously have carnal knowledge with (sic) Rowena Contridas
against her will, to her damage and prejudice in the amount that may be proven
in court.
Suppression of testimony
Acts contrary to law.1

FIRST DIVISION
* All the accused pleaded "not guilty." Trial on the merits thereafter ensued.

G.R. No. 137664 May 9, 2002


* All the accused, including appellant Roberto Padrigone, interposed the defense of denial and
alibi. Appellant claimed that in the evening of January 2, 1995, he and his companions, Jocel
PEOPLE OF THE PHILIPPINES, plaintiff-appellee,
Ibanita and Michael San Antonio, visited Rowena at her house. According to him, Rowena was
crying when they arrived. When appellant asked her what was wrong, she told him that she
vs. ROBERTO PADRIGONE a.k.a. "ROBERTO SAN MIGUEL", accused-appellant. wanted to elope with him. He replied that he was not ready as he was still studying. Rowena
snapped, "it's up to him but he might regret it."7 While appellant and Rowena were talking,
YNARES-SANTIAGO, J.: Jocel Ibanita and Michael San Antonio were in the kitchen cooking noodles. Later, a certain
Ismeraldo Quirante, in the presence of several barangay watchmen patrolling the area, passed
by the Contridas' house and advised the accused to go home because it was getting late. They
heeded the advice and left the Contridas' house at around 11:30 p.m.

Facts
* The trial court gave credence to the prosecution evidence and rendered a decision, the
dispositive portion of which reads:
* It appears that at 3:00 in the morning of January 3, 1995, appellant Roberto Padrigone and
the other accused broke into the house of Rowena Contridas, then 16 years old, situated in
WHEREFORE, in view of the foregoing considerations, this Court finds the
San Benito, Salvacion, Buhi, Camarines Sur. Appellant Roberto Padrigone and accused Jocel
accused, ROBERTO PADRIGONE a.k.a. ROBERTO SAN MIGUEL, GUILTY of the
Ibaneta poked a knife at Rowena and her fourteen year-old sister, Nimfa,2 and threatened to
crime of Rape, under Article 335 of the Revised Penal Code (as amended by
kill them if they reported the incident to others. They gagged Rowena with a handkerchief and
Section 11, R.A. 7659) and hereby sentences him to suffer imprisonment of
Nimfa with a handtowel. Then, appellant undressed Rowena, forced her to lie down and
RECLUSION PERPETUA, considering the mitigating circumstance of voluntary
sexually violated her while his co-accused watched with glee. Accused Jocel Ibanita tried to
surrender. He is likewise directed to indemnify the offended party, Rowena
rape Nimfa but failed because she was able to elude him.
Contridas, the amount of Fifty thousand Pesos (P50,000.00) as moral damages
and to pay the costs of this suit. Accused JOCEL IBANITA, MICHAEL SAN
* After appellant satisfied his lust on Rowena, the other accused took their turns. Every one of ANTONIO and ABELARDO TRIUMPANTE are ACQUITTED for insufficiency of
the accused raped Rowena. Before they left, they warned the sisters not to report the incident evidence. It being shown that the three accused are presently detained at the
or else they will kill them. Municipal Jail at PNP, Buhi, Camarines Sur, their immediate release is hereby
ordered.
* Despite the threats, Rowena and Nimfa reported the incident to the police and identified
appellant and his co-accused as the perpetrators. However, based on the police blotter, SO ORDERED.8
Rowena stated that it was only appellant who raped her.

* Appellant interposed the instant appeal.


* Dr. Damiana Claveria, Municipal Health Officer, conducted a medical examination on Rowena
and found the following:

patient - very talkative, incoherent as to questions asked.


Issues

PE - no signs of external injury

330
RECTO, GAYLE ANGELI M.
2011-0008 | AUSL
Personal Notes on Remedial Law 2 Review (based on the syllabus of Prof. Henedino M. Brondial)

I. THE TRIAL COURT GRAVELY ERRED IN CONVICTING ACCUSED-APPELLANT OF THE CRIME We find that Nimfa's credibility has not been impaired despite rigorous cross-examination. In
OF RAPE INSPITE OF THE INHERENT WEAKNESSES AND INSUFFICIENCY OF PROSECUTION'S fact, defense counsel was not able to point to any inconsistency in Nimfa's testimony. A
EVIDENCE. perusal of the transcripts of stenographic notes reveals that she was steadfast in narrating the
circumstances of the rape and in pointing to appellant as one of the perpetrators.
II. THE TRIAL COURT GRAVELY ERRED IN DECIDING THE INSTANT CASE NOT IN
ACCORDANCE WITH THE ESTABLISHED PRINCIPLE IN CRIMINAL LAW THAT THE
PROSECUTION MUST RELY ON THE STRENGTH OF ITS EVIDENCE AND NOT ON THE
WEAKNESS OF THAT OF THE DEFENSE. Appellant likewise alleges that it was error for the trial court to have dismissed his
"sweetheart" defense by the mere absence of love notes, mementos or pictures.

Ruling
In People v. Corea,15 we held that:
Appellant contends that the prosecution evidence was insufficient to prove his guilt beyond
reasonable doubt.

x x x Moreover, even if such averment is true, it does not necessarily follow that no rape can
be committed against one's sweetheart. Such a relationship provides no license to explore and
Appellant argues that according to the prosecution witness, Nimfa, he and his co-accused invade that which every virtuous woman holds so dearly and trample upon her honor and
Michael San Antonio, Abelardo Triumpante and Jocel Ibanita, took turns in raping Rowena dignity. That relationship is held sacred by many x x x. A sweetheart cannot be forced to
while Jocel Ibanita also attempted to rape her. However, after preliminary investigation, the engage in sexual intercourse against her will. As a matter of fact, proof even of a prior history of
Municipal Trial Court of Buhi, Camarines Sur, dismissed Nimfa's complaint for attempted rape a common-law marital relationship will not prevail over clear and positive evidence of
against Jocel Ibanita because of its findings that the latter committed only acts of copulation by the use of force or intimidation.
lasciviousness, considering his voluntary and spontaneous desistance from continuing to
perform the acts leading to carnal knowledge. Furthermore, the investigating Judge
entertained doubts about the truth of her story, which was uncorroborated.9
Regardless, the most telling indication that would belie appellant's "sweetheart theory" was
the fact that he had carnal knowledge of Rowena in the presence of Nimfa and his co-accused.
It is most unnatural for lovers to engage in the ultimate expression of their love for each other
We agree with the following observation of the Solicitor General: in the presence of other people.

[T]he dismissal of the complaint for attempted rape filed by Nimfa against one of the accused, Appellant assails the procedural irregularities committed by the prosecution and by the trial
Jocel Ibanita, during the preliminary investigation stage should not detract from the credibility of court. He claims that the prosecution suppressed evidence by not presenting Rowena, the
her testimony. Even if the prosecution wanted to, the merits of the dismissal of Nimfa's victim, when the latter should have had her sane moments. As a consequence, the trial court
complaint for attempted rape could not be properly challenged in the criminal proceedings deprived appellant of the opportunity to cross-examine her when she allegedly declared before
below since the said proceedings involved only the culpability of the four accused for the crime the Chief of Police of Buhi that it was only appellant who raped her which declaration became
of rape committed against Rowena, the sister of Nimfa.10 the basis for the latter's conviction.

Appellant further claims that Nimfa's lack of credibility was underscored when the trial court Appellant's contention is misplaced if not misleading. The basis of his conviction was not
acquitted appellant's co-accused. Appellant's claim is not well taken. Evidence shows that the Rowena's declaration before the Chief of Police but rather Nimfa's testimony before the trial
trial court acquitted appellant's co-accused because of doubt engendered on the extent of court that it was him who raped Rowena, among others.16 In fact, the trial court found, thus:
their participation in the sexual assault committed against Rowena in light of Rowena's own
statement as recorded in the police blotter.11

x x x The evidence adduced by the parties in this case disclosed that accused Roberto
Padrigone, a.k.a. Roberto San Miguel, Jocel Ibanita, Michael San Antonio and Abel Triumpante
Appellant alleges that Nimfa's reactions after the rape of her sister are "unnatural, unexpected entered the dwelling of the Contridas sisters at 3:00 a.m. of January 3, 1995, and at knifepoint
and mind-boggling,"12 specifically when she resumed her sleep after having been raped and successively raped Rowena Contridas, a 16 year old lass. The victim became insane after the
even reported for work the following day. The contention deserves scant consideration. It is an incident and was not able to testify in Court. Nimfa Contridas, her fourteen year old sister,
accepted maxim that different people react differently to a given situation or type of situation who was also present that time narrated the incident when her elder sister's innocence was
and there is no standard form of behavioral response when one is confronted with a strange forcibly violated. Accused interposed the defense of denial and alibi. x x x
or startling experience.13

The prosecution has established beyond reasonable doubt that accused Roberto Padrigone
Further, appellant argues that Nimfa admitted before the police that she did not recognize the ravished Rowena Contridas against her will and consent, and with the use of a bladed
rapists of Rowena. In this connection, we quote with approval the observation of the Solicitor weapon.17
General, to wit:

Besides, the non-presentation of Rowena on the witness stand cannot be considered as


Anent the portion of Nimfa's testimony wherein she admitted to the defense counsel that she suppression of evidence. Under Rule 131, Section 3(e) of the Rules of Court, the
told the Chief of Police that she was not able to recognize the persons who raped her sister rule that "evidence willfully suppressed would be adverse if produced" does not
Rowena, the same is capable of explanation. Accused-appellant Roberto Padrigone was apply if (a) the evidence is at the disposal of both parties; (b) the suppression was
present when Nimfa uttered the statement. Hence, she was afraid to tell the truth because of not willful; (c) it is merely corroborative or cumulative; and (d) the suppression is
the earlier threat to her and sister Rowena's lives by accused-appellant Padrigone.14 an exercise of a privilege.18

331
RECTO, GAYLE ANGELI M.
2011-0008 | AUSL
Personal Notes on Remedial Law 2 Review (based on the syllabus of Prof. Henedino M. Brondial)

Plainly, there was no suppression of evidence in this case. First, the defense had G.R. No. 122899 June 8, 2000
the opportunity to subpoena Rowena even if the prosecution did not present her as
a witness. Instead, the defense failed to call her to the witness stand. Second, METROPOLITAN BANK & TRUST COMPANY, petitioner,
Rowena was certified to be suffering from "Acute Psychotic Depressive Condition"
and thus "cannot stand judicial proceedings yet."19 The non-presentation,
vs. COURT OF APPEALS and G.T.P. DEVELOPMENT CORPORATION, respondents.
therefore, of Rowena was not willful. Third, in any case, while Rowena was the
victim, Nimfa was also present and in fact witnessed the violation committed on
BUENA, J.
her sister.

Appellant cannot claim that the trial court erred in convicting him on the basis of Rowena's Facts
statement as recorded in the police blotter. His conviction was based on the trial court's
findings of facts and assessment of the witnesses' credibility. Well-settled is the rule that the * The subject property is a parcel of land in Diliman, Quezon City consisting of six hundred
findings of facts and assessment of credibility of witnesses is a matter best left to the trial ninety (690) square meters originally owned by businessman Tomas Chia under Transfer
court because of its unique position of having observed that elusive and incommunicable Certificate of Title No. RT-16753 (106901) of the Registry of Deeds for Quezon City. Saddled
evidence of the witnesses' deportment on the stand while testifying, which opportunity is with debts and business reverses, Mr. Chia offered the subject property for sale to private
denied to the appellate courts. Only the trial judge can observe the "furtive glance, blush of respondent G.T.P. Development Corporation (hereafter, GTP), with assumption of the
conscious shame, hesitation, flippant or sneering tone, calmness, sigh, or the scant or full mortgage indebtedness in favor of petitioner METROBANK secured by the subject property.
realization of an oath," all of which are useful aids for an accurate determination of a witness'
honesty and sincerity. The trial court's findings are accorded finality, unless there appears in * Pending negotiations for the proposed sale, Atty. Bernardo Atienza, acting in behalf of
the record some fact or circumstance of weight which the lower court may have overlooked, respondent GTP, went to the METROBANK branch in Quiapo, Manila sometime in the last week
misunderstood or misappreciated and which, if properly considered, would alter the results of of August 1980 to inquire on Mr. Chia's remaining balance on the real estate mortgage.
the case.20 METROBANK obliged with a statement of account of Mr. Chia amounting to about P115,000.00
as of August, 1980.

* The deed of sale 2 and the memorandum of agreement 3 between Mr. Chia and respondent
Besides, in rape cases where the offended parties are young and immature girls from the ages GTP were eventually executed and signed on 04 September 1980 in the office of Atty. Atienza.
of twelve to sixteen, we have consistently held that the victim's version of what transpired Twelve (12) days later, or on 16 September 1980, Atty. Atienza went to METROBANK Quiapo
deserves credence, considering not only their relative vulnerability but also the shame and Branch and paid one hundred sixteen thousand four hundred sixteen pesos and seventy-one
embarrassment to which such a grueling experience as a court trial, where they are called centavos (P116,416.71), 4 for which METROBANK issued an official receipt acknowledging
upon to lay bare what perhaps should be shrouded in secrecy, exposed them to. This is not to payment.
say that an uncritical acceptance should be the rule. It is only to emphasize that skepticism
should be kept under control.21 * This notwithstanding, petitioner METROBANK refused to release the real estate mortgage on
the subject property despite repeated requests from Atty. Atienza, thus prompting respondent
GTP to file on October 17, 1980 an action for specific performance against petitioner
METROBANK and Mr. Chia.
Nonetheless, no young and decent Filipina would publicly admit that she was ravished and her
honor tainted unless the same were true, for it would be instinctive on her part to protect her * In answer to the complaint, Mr. Chia denied having executed any deed of sale in favor of
honor and obtain justice for the wicked acts committed upon her.22 Not to be overlooked is respondent GTP involving the subject property. Petitioner for its part justified its non-release
the complainant's willingness to face police investigators and to submit to a physical of the real estate mortgage (1) upon the advise of Mr. Chia that he never executed any sales
examination which are eloquent and sufficient affirmations of the truth of her charge.23 agreement with respondent GTP, and (2) by the fact that there are other loans incurred by Mr.
Chia which are also secured by the subject property.

* After trial, judgment was rendered by the regional trial court on 11 December 1990 granting
As regards the matter of damages, the trial court ordered accused-appellant "to indemnify the the reliefs prayed for by respondent GTP as plaintiff, viz:
offended party, Rowena Contridas, the amount of Fifty Thousand Pesos (P50,000.00) as moral
damages."24 In People v. Belga,25 it was held that civil indemnity is mandatory upon the WHEREFORE, after a careful and thorough study of the record, this Court holds that in view of
finding of the fact of rape; it is distinct from and should not be denominated as moral the facts contained in the records, judgment is hereby rendered in favor of plaintiff and
damages which are based on different jural foundations and assessed by the court in the against defendants, ordering —
exercise of sound discretion. Thus, consistently with present case law which treats the
imposition of civil indemnity as mandatory upon a finding of rape, accused-appellant is ordered
1. Defendant Metropolitan Bank & Trust Co. to execute the release or
to pay the additional amount of fifty thousand (P50,000.00) pesos as civil indemnity ex
cancellation of the real estate mortgages executed by the deceased defendant
delicto.26
Tomas Chia and his wife, defendant Vicenta Chia, over the property described in
TCT No. 106901 of the registry of deeds for Quezon City;

2. Defendants to surrender or deliver the owner's duplicate copy of said TCT No.
WHEREFORE, based on the foregoing, the assailed Decision, finding accused-appellant 106991; and,
Roberto Padrigone a.k.a. Roberto San Miguel guilty beyond reasonable doubt of the crime of
rape and sentencing him to suffer the penalty of reclusion perpetua, is AFFIRMED with the
3. Defendants to pay, jointly and severally, the sum of P10,000.00 as and for
MODIFICATION that he is ordered to pay Rowena Contridas civil indemnity in the amount of
attorney's fees, plus costs of suit.
P50,000.00 in addition to moral damages in the amount of P50,000.00. Costs de oficio.

The counterclaims set up by both defendants are dismissed.

IT IS SO ORDERED. 5
SO ORDERED.

* On appeal, respondent Court of Appeals rendered a Decision dated 24 October 1994 6


reversing the trial court's 11 December 1990 judgment, ruling in the main that the one
hundred sixteen thousand four hundred sixteen pesos and seventy-one centavos
Davide, Jr., C.J., (Chairman), Puno, Kapunan, and Austria-Martinez, JJ., concur. (P116,416.71) paid by respondent GTP to petitioner METROBANK did not extinguish the real
estate mortgage inasmuch as there are other unliquidated past due loans secured by the
subject property.

SECOND DIVISION * With this unfavorable turn of events, respondent GTP, on 07 November 1994, 7 filed before
respondent Court of Appeals a "motion for reconsideration with alternative prayer to require

332
RECTO, GAYLE ANGELI M.
2011-0008 | AUSL
Personal Notes on Remedial Law 2 Review (based on the syllabus of Prof. Henedino M. Brondial)

METROBANK to furnish appellee (GTP) of the alleged unpaid balance of Mr. Chia." At the re- 2. the representation must have been with knowledge of the facts;
scheduled date of oral arguments on 08 March 1995 where METROBANK was supposed to
bring before the respondent Court the current statement of the mortgage debt of Mr. Chia
secured by the deeds of mortgage sought to be released, METROBANK's counsel did not
appear; only the lawyers of respondent GTP and Mr. Chia appeared. Thus, the Court required
3. the party to whom it was made must have been ignorant of the truth of the matter; and
GTP's counsel to file a memorandum in lieu of oral arguments in support of its motion for
reconsideration. 8 GTP filed its memorandum on March 17, 1995 9 to which a reply
memorandum was filed by METROBANK on April 10, 1995. 10

* On 03 July 1995, 11 the now assailed amended decision was rendered reconsidering the 4. it must have been with the intention that the other party would act upon it.
original 24 October 1994 Decision and thus affirming the 11 December 1990 judgment of the
regional trial court. Respondent Court of Appeals took a second hard look at the evidence on
hand and seriously considered METROBANK's refusal to specify any unpaid debt secured by
the subject property, in concluding anew that "the present case for specific performance is Respondent GTP, thru Atty. Atienza, requested from METROBANK that he be furnished a copy
well-grounded, absent indubitable showing that the aforesaid amount of P116,416.71 paid by of the full indebtedness secured by the real estate mortgage. 15 In response thereto,
appellee on September 16, 1980 did not suffice to pay in full the mortgage debt assumed petitioner METROBANK issued a statement of account as of September 15, 1980 16 which
under the Deed of Absolute Sale, with assumption of mortgage, it inked with the late Tomas amount was immediately settled and paid the next day amounting to P116,416.71. Petitioner
Chia. There is therefore merit in its motion for reconsideration at bench." Petitioner METROBANK is thus barred from taking a stand inconsistent with its representation upon
METROBANK is now before us after its motion for reconsideration of the 03 July 1995 which respondent GTP, as an innocent third person to the real mortgage agreement, placed
amended decision was denied by respondent Court of Appeals per Resolution of 04 December exclusive reliance. Respondent GTP had the reasonable right to rely upon such representations
1995. 12 as true, considering that it had no participation whatsoever in the mortgage agreement and
the preparation of the statement of account, coupled with the expectation that a reputable
banking institution such as petitioner METROBANK do conduct their business concerns in the
highest standards of efficiency and professionalism. For an admission or representation is
rendered conclusive upon the person making it, and cannot be denied or disproved as against
Ruling
a person relying thereon. A party may not go back on his own acts and representations to the
prejudice of the other party who relied upon them. In the law of evidence, whenever a party
We find no compelling reasons to disturb the assailed decision.
has, by his own declaration, act or omission, intentionally and deliberately led another to
believe a particular thing true, and to act upon such belief, he cannot, in any litigation arising
out of such declaration, act, or omission, be permitted to falsify it. 17

We quote with favor the following pronouncements of respondent Court of Appeals in the
Amended Decision, thus:
Just as decisive is petitioner METROBANK's failure to bring before respondent
Court of Appeals the current statement evidencing what it claims as "other
unliquidated past due loans" at the scheduled hearing of 8 March 1995. It was a
. . . . In the case under scrutiny, we are convinced that We erred in reversing the appealed golden opportunity, so to speak, lost for petitioner METROBANK to defend its non-
judgment despite the finding that subject property covered by TCT 106901 — Quezon City had release of the real estate mortgage. Thus, the following pronouncements of this
been sold, in a manner absolute and irrevocable, by the spouses, Tomas Chia and Vicenta Court in Manila Bay Club Corporation vs. Court of Appeals et. al, 18 speaking thru
Chan, to plaintiff-appellee, and on September 16, 1980, the latter complied with its contractual Mr. Justice Ricardo Francisco, 19 find rightful application, viz. —
obligation thereunder by paying the total mortgage debt it assumed, amounting according to
Metrobank itself, to P116,416.71, as of September 16, 1980.

It is a well-settled rule that when the evidence tends to prove a material fact which
imposes a liability on a party, and he has it in his power to produce evidence which
All things studiedly viewed in proper perspective, we are of the opinion, and so rule, that from its very nature must overthrow the case made against him if it is not founded
whatever debts or loans mortgagor Chia contracted with Metrobank after September 4, 1980, on fact, and he refuses to produce such evidence, the presumption arises that the
without the conformity of plaintiff-appellee, could not be adjudged as part of the mortgage evidence, if produced would operate to his prejudice, and support the case of his
debt the latter so assumed. We are persuaded that the contrary ruling on this point in Our adversary ......
October 24, 1994 decision would be unfair and unjust to plaintiff-appellee because, before
buying subject property and assuming the mortgage debt thereon, the latter inquired from
Metrobank about the exact amount of the mortgage debt involved.
No rule of law is better settled than that a party having it in his power to prove a
fact, if it exists, which, if proved, would benefit him, his failure to prove it must be
taken as conclusive that the fact does not exist.
The stipulation in subject Deeds of Mortgage that mortgagors' debts subsequently obtained
would be covered by the same security became inapplicable, when mortgagor sold to appellee
the mortgaged property with the knowledge of the mortgagee bank. Thus, since September 4,
1980, it was obvious that whatever additional loan mortgagor got from Metrobank, the same
xxx xxx xxx
was not chargeable to and collectible from plaintiff-appellee. It is then decisively clear that
Metrobank is without any valid cause or ground not to release the Deeds of Mortgage in
question, despite full payment of the mortgage debt assumed by appellee. 13

Where facts are in evidence affording legitimate inferences going to establish the
ultimate fact that the evidence is designed to prove, and the party to be affected
by the proof, with an opportunity to do so, fails to deny or explain them, they may
Petitioner METROBANK is estopped from refusing the discharge of the real estate mortgage on
well be taken as admitted with all the effect of the inferences afforded .......
the claim that the subject property still secures "other unliquidated past due loans." In
Maneclang vs. Baun, 14 this Court enumerated the requisites for estoppel by conduct to
operate, to wit:

The ordinary rule is that one who has knowledge peculiarly within his own control,
and refuses to divulge it, cannot complain if the court puts the most unfavorable
construction upon his silence, and infers that a disclosure would have shown the
1. there must have been a representation or concealment of material facts;
fact to be as claimed by the opposing party.

333
RECTO, GAYLE ANGELI M.
2011-0008 | AUSL
Personal Notes on Remedial Law 2 Review (based on the syllabus of Prof. Henedino M. Brondial)

Verily, petitioner METROBANK's omission to present its evidence only created an AOM No. 2004-21B x x x involving issuance of vitamins worth
adverse inference against its cause. Therefore, it cannot now be heard to complain P138,964.80 mostly to the staff of VSMMC and TNT Office covering
since respondent Court extended a reasonable opportunity to petitioner the period January to April 2004; and
METROBANK that it did not avail.
AOM No. 2004-21C x x x covering fictitious patients and falsified
prescriptions for other drugs and medicines worth P552,853.85 and
unpaid falsified prescriptions and referral letters for drugs and
WHEREFORE, the petition is DENIED. The amended decision of respondent Court of Appeals medicines costing P602,063.50.15
dated 3 July 1995 as well as its resolution of 4 December 1995 is AFFIRMED, with costs
against petitioner. * Meanwhile, the fact-finding committee created by Delos Santos submitted its
Report16 dated January 18, 2005 essentially affirming the “unseen and
unnoticeable” irregularities attendant to the availment of the TNT Program but
pointing out, however, that: (a) VSMMC was made an “unwilling tool to
perpetuate a scandal involving government funds”;17 (b) the VSMMC
SO ORDERED.
management was completely “blinded” as its participation involved merely “a
routinary ministerial duty” in issuing the checks upon receipt of the referral slips,
prescriptions, and delivery receipts that appeared on their faces to be regular
and complete;18 and (c) the detection of the falsification and forgeries “could not
Bellosillo, Mendoza, Quisumbing and De Leon, Jr., JJ., concur. be attained even in the exercise of the highest degree or form of diligence”19 as
the VSMMC personnel were not handwriting experts.

* In the initial investigation conducted by the CoA, the results of which were
Official duty reflected in AOM No. 2005-00120 dated October 26, 2005, it was found that: (a)
133 prescriptions for vaccines, drugs and medicines for anti-rabies allegedly
dispensed by Dell Pharmacy costing P3,407,108.40, and already paid by VSMMC
EN BANC from the PDAF of Cuenco appeared to be falsified;21 (b) 46 prescriptions for
other drugs and medicines allegedly dispensed by Dell Pharmacy costing
G.R. No. 198457, August 13, 2013 P705,750.50, and already paid by VSMMC from the PDAF of Cuenco likewise
appeared to be falsified;22 and (c) 25 prescriptions for drugs and medicines
FILOMENA G. DELOS SANTOS, JOSEFA A. BACALTOS, NELANIE A. ANTONI, AND allegedly issued by Dell Pharmacy costing P602,063.50 were also ascertained to
MAUREEN A. BIEN, Petitioners, be falsified and have not been paid by VSMMC.23

v. COMMISSION ON AUDIT, REPRESENTED BY ITS COMMISSIONERS, Respondent. * In her Comment/Reply24 to the aforementioned AOM No. 2005-001 addressed
to Leonor D. Boado (Boado), Director of the CoA Regional Office VII in Cebu City,
Delos Santos explained that during the initial stage of the implementation of the
PERLAS-BERNABE, J.:
MOA (i.e., from 2000 to 2002) the hospital screened, interviewed, and
determined the qualifications of the patients-beneficiaries through the hospital’s
social worker.25 However, sometime in 2002, Cuenco put up the TNT Office in
VSMMC, which was run by his own staff who took all pro forma referral slips
Facts bearing the names of the social worker and the Medical Center Chief, as well as
the logbook.26 From then on, the hospital had no more participation in the said
program and was relegated to a mere “bag keeper.”27 Since the benefactor of
* Sometime in October 2001, then Congressman Antonio V. Cuenco (Cuenco) of the Second
the funds chose Dell Pharmacy as the sole supplier, anti-rabies medicines were
District of Cebu City entered into a Memorandum of Agreement5 (MOA) with the Vicente Sotto
purchased from the said pharmacy and, by practice, no public bidding was
Memorial Medical Center (VSMMC or hospital), represented by Dr. Eusebio M. Alquizalas (Dr.
anymore required.28
Alquizalas), Medical Center Chief, appropriating to the hospital the amount of P1,500,000.00
from his Priority Development Assistance Fund (PDAF) to cover the medical assistance of
indigent patients under the Tony N' Tommy (TNT) Health Program (TNT Program).6 It was * Consequently, a special audit team (SAT), led by Team Leader Atty. Federico E.
agreed, inter alia, that: (a) Cuenco shall identify and recommend the indigent patients who Dinapo, Jr., State Auditor V, was formed pursuant to Legal and Adjudication
may avail of the benefits of the TNT Program for an amount not exceeding P5,000.00 per Office (LAO) Order Nos. 2005-019-A dated August 17, 2005 and 2005-019-B
patient, except those with major illnesses for whom a separate limit may be specified; (b) an dated March 10, 2006 to conduct a special audit investigation with respect to the
indigent patient who has been a beneficiary will be subsequently disqualified from seeking findings of Booc and her team.29 Due to time constraints, however, AOM No.
further medical assistance; and (c) the hospital shall purchase medicines intended for the 2005-001 was no longer included in the SAT focus.30 On October 15, 2007, the
indigent patients from outside sources if the same are not available in its pharmacy, subject to SAT reported31 the following findings and observations:
reimbursement when such expenses are supported by official receipts and other documents.7
In line with this, Ma. Isabel Cuenco, Project Director of the TNT Program, wrote8 petitioner The provision of National Budget Circular No. 476 dated September
Nelanie Antoni (Antoni), Pharmacist V of VSMMC, requesting the latter to purchase needed 20, 2001 prescribing the guidelines on the release of funds for the
medicines not available at the hospital pharmacy from Sacred Heart Pharmacy or Dell PDAF authorized under Republic Act (R.A.) No. 8760, as Reenacted
Pharmacy which were supposedly accredited suppliers of the Department of Health. The said (GAA for CY 2001) were not followed;32
request was approved.9
Existing auditing law, rules and regulations governing procurement
* Several years after the enforcement of the MOA, allegations of forgery and falsification of of medicines were not followed in the [program's]
prescriptions and referrals for the availment of medicines under the TNT Program surfaced. On implementation;33
December 14, 2004, petitioner Filomena G. Delos Santos (Delos Santos), who succeeded10 Dr.
Alquizalas, created, through Hospital Order No. 1112,11 a fact-finding committee to The [program's] implementation did not follow the provisions of the
investigate the matter. MOA by and between [Congressman Cuenco] and the Hospital;34
and
* Within the same month, Beatriz M. Booc (Booc), State Auditor IV, who was
assigned to audit the hospital, came up with her own review of the account for Acts committed in the implementation of the project were as
drugs and medicines charged to the PDAF of Cuenco. She furnished Delos Santos follows:
the results of her review as contained in Audit Observation Memoranda (AOM)
Nos. 2004-21,12 2004-21B,13 and 2004-21C,14 all dated December 29, 2004,
There were [one hundred thirty-three (133)] falsified
recommending the investigation of the following irregularities:
prescriptions for anti-rabies vaccines, drugs and
medicines [costing] P3,345,515.75 [allegedly]
AOM No. 2004-21 x x x involving fictitious patients and falsified dispensed by Dell Pharmacy [were] paid by VSMMC
prescriptions for anti-rabies and drugs costing P3,290,083.29; from the [PDAF of Congressman Cuenco];

334
RECTO, GAYLE ANGELI M.
2011-0008 | AUSL
Personal Notes on Remedial Law 2 Review (based on the syllabus of Prof. Henedino M. Brondial)

[Forty-six (46) falsified prescriptions] for other drugs Pharmacy Unit for stamping and submission to the Dell Pharmacy.
and medicines costing P695,410.10 [were likewise She had no opportunity then to see the patients personally.
reportedly] dispensed by Dell Pharmacy and paid by
VSMMC from the [said PDAF] x x x; and Mr. Louies James S. Yrastorza has stated under oath the falsity of
the claims for payment. He stated that he was ordered to submit to
[Twenty-five (25) prescriptions worth] P602,063.50 the Pharmacy Unit falsified prescriptions accompanied by referral
[were also claimed to have been] served by Dell slips signed by Mr. James Cuenco for non-existing patients.
Pharmacy but still unpaid x x x.35 Subsequently, sometime in September 2007 Mr. Yrastorza
“clarified” his statements effectively recanting his first oath.
* Examination by the SAT of the records and interviews with the personnel
involved showed that the purported patients-beneficiaries of the TNT Program The Office of the Provincial Election Supervisor certified that out of
were mostly non-existent and there was no actual procedure followed except for the 30 names of the patients randomly selected, only 15 were
the mere preparation of payment documents which were found to be falsified as found listed in the registered voters' database.
evidenced by the following:
Prescriptions were stamped “VSMMC” signed/initialed by the
Thirteen (13) hospital surgeons disowned the signatures on the Pharmacist who is off duty as shown by the attendance record, e.g.
prescriptions supporting the claims. Surgeons do not prescribe anti- Mesdames Arly Capuyan, Norma Chiong, Corazon Quiao, Rowena
rabies vaccines; they operate on patients. Rabillas, and Riza Sei[s]mundo.36

Almost all of the patients named in the prescriptions were not * Subsequently, or on September 8, 2008, the SAT Team Supervisor, Boado,
treated or admitted at the Hospital or in its Out-patient Department. issued ND No. 2008-09-01,37 disallowing the amount of P3,386,697.10 for the
Those whose names appeared on Hospital records were treated at payment of drugs and medicines for anti-rabies with falsified prescription and
different dates than those appearing on the prescriptions: documents, and holding petitioners, together with other VSMMC officials,
solidarily liable therefor.38 Petitioners’ respective participations were detailed as
PATIENT TREATED BILL DATE OF follows: (a) for Delos Santos, in her capacity as Medical Center Chief, for signing
PRESCRIPTION and approving the disbursement vouchers and checks; (b) for petitioner Dr.
Josefa A. Bacaltos, in her capacity as Chief Administrative Officer, for certifying in
Box A that the expenses were lawful, necessary and incurred in her direct
Leah Clamon Nov. 12, 2003 11/11/03 11/03/03
supervision; (c) for Antoni, in her capacity as Chief of the Pharmacy Unit, for
approving the supporting documents when the imputed delivery of the medicines
Jean Cañacao Nov. 30, 2003 11/25/03 11/18/03 had already been consummated; (d) for petitioner Maureen A. Bien, in her
capacity as Hospital Accountant, for certifying in Box B of the disbursement
Felipe Sumalinog Dec. 17, 2004 12/10/03 12/08/03
voucher that the supporting documents for the payment to Dell Pharmacy were
complete and proper.39
Vicente Perez Mar. 12, 2004 11/26/03 11/17/03
* Aggrieved, petitioners filed their respective appeals40 before the CoA which were denied
Vincent Rabaya Sept. 8, 2003 12/12/03 11/28/03 through Decision No. 2010-05141 dated April 8, 2010, maintaining their solidary liability, to
wit:
Rodulfo Cañete July 24, 2004 01/16/04 01/12/04
WHEREFORE, premises considered, the appeal[s] of Dr. Filomena [G]. Delos
Full dosages of anti-rabies vaccines were allegedly given to the Santos, Dr. Josefa A. Bacaltos, Ms. Nelanie A. Antoni and Ms. Maureen A. Bien
patients although it is gross error to do so for these medicines are [are] hereby DENIED for lack of merit. However, the appeal of Ms. Corazon
highly perishable. These should be refrigerated and injected Quiao, Ms. Norma Chiong, Ms. Rowena Rabillas and Ms. Riza Seismundo is
immediately and periodically. For instance: hereby given due course. Likewise, Ms. Arly Capuyan who is similarly situated is
excluded although she did not file her appeal. ND No. 2008-09-01 (SAT) dated
September 8, 2008 involving the amount of P3,386,697.10 is hereby affirmed
Mr. Vicente Perez received the full dosage on November 26, 2003
with the modification by excluding therein the names [of[ Ms. Corazon Quiao,
and again on November 27, 2003. (Hospital records showed that Mr.
Ms. Norma Chiong, Ms. Rowena Rabillas, Ms. Riza Seismundo, and Ms. Arly
Perez was admitted in March 2003 for surgery.)
Capuyan as persons liable. The other persons named liable therein, i.e., Ma.
Isabel Cuenco and Mr. James R. Cuenco, TNT Health Program Directors, and Mr.
Mr. Maximo Buaya received the full dosage on January 25 and on Sisinio Villacin, Jr., proprietor of Dell Pharmacy, and herein appellants Delos
February 29, 2004. Santos, Bacaltos, Antoni and Bien remain solidarily liable for the disallowance.42
(Emphasis supplied)
Mr. Gregorio Rabago received his full dosage on December 6, 2003.
* The Motion for Reconsideration43 of the foregoing decision was further denied in Decision
The dates of 80 prescriptions for anti-rabies and 45 for other drugs No. 2011-04544 dated August 8, 2011. Hence, the instant petition.
and medicines are earlier than the dates of the corresponding
delivery receipts. The gaps in the dates ranged from 1 to 47 days.
On the other hand, 33 prescriptions for anti-rabies had later dates
than the dates of the delivery receipts. The difference in the dates
Issue
ranged from 1 to 22 days.

The essential issue in this case is whether or not the CoA committed grave abuse of discretion in
The Pharmacy Unit still prepared Purchase Request [PR] for the
holding petitioners solidarily liable for the disallowed amount of P3,386,697.10.
claims Dell [Pharmacy] submitted to that office when the PR is no
longer necessary as the medicines have already been taken by the
patients.

Of the three South District residents personally interviewed by the Ruling


Team, two denied having sought or received help from the [TNT]
Program or being hospitalized at VSMMC for dog bite. At the outset, it must be emphasized that the CoA is endowed with enough latitude to
determine, prevent, and disallow irregular, unnecessary, excessive, extravagant or
The hospital social worker, Ms. Mergin Acido, declared that she was unconscionable expenditures of government funds. It is tasked to be vigilant and conscientious
bypassed in the evaluation of the alleged patients for the TNT in safeguarding the proper use of the government's, and ultimately the people's, property. The
Office has clerks who “evaluate” the eligibility of the patients. The exercise of its general audit power is among the constitutional mechanisms that gives life to
prescriptions and referral slips were directly forwarded to the the check and balance system inherent in our form of government.45

335
RECTO, GAYLE ANGELI M.
2011-0008 | AUSL
Personal Notes on Remedial Law 2 Review (based on the syllabus of Prof. Henedino M. Brondial)

Lending Operations.61 In similar regard, the Court, in Casal v. CoA62 (Casal),


sustained the liability of certain officers of the National Museum who again,
Corollary thereto, it is the general policy of the Court to sustain the decisions of administrative notwithstanding their good faith participated in approving and authorizing the
authorities, especially one which is constitutionally-created, such as the CoA, not only on the incentive award granted to its officials and employees in violation of
basis of the doctrine of separation of powers but also for their presumed expertise in the laws Administrative Order Nos. 268 and 29 which prohibit the grant of productivity
they are entrusted to enforce. Findings of administrative agencies are accorded not only incentive benefits or other allowances of similar nature unless authorized by the
respect but also finality when the decision and order are not tainted with unfairness or Office of the President.63 In Casal, it was held that, even if the grant of the
arbitrariness that would amount to grave abuse of discretion. It is only when the CoA has incentive award was not for a dishonest purpose, the patent disregard of the
acted without or in excess of jurisdiction, or with grave abuse of discretion amounting to lack issuances of the President and the directives of the CoA amounts to gross
or excess of jurisdiction, that this Court entertains a petition questioning its rulings. There is negligence, making the [“approving officers”] liable for the refund [of the
grave abuse of discretion when there is an evasion of a positive duty or a virtual refusal to disallowed incentive award].64
perform a duty enjoined by law or to act in contemplation of law as when the judgment
rendered is not based on law and evidence but on caprice, whim, and despotism.46 In this
case, the Court finds no grave abuse of discretion on the part of the CoA in issuing the
assailed Decisions as will be discussed below. Just as the foregoing public officers in Reyna and Casal were not able to dispute
their respective violations of the applicable rules in those cases, the Court finds
that the petitioners herein have equally failed to make a case justifying their non-
observance of existing auditing rules and regulations, and of their duties under the
The CoA correctly pointed out that VSMMC, through its officials, should have been deeply MOA. Evidently, petitioners’ neglect to properly monitor the disbursement of
involved in the implementation of the TNT Program as the hospital is a party to the MOA and, Cuenco's PDAF facilitated the validation and eventual payment of 133 falsified
as such, has acted as custodian and disbursing agency of Cuenco’s PDAF.47 Further, under prescriptions and fictitious claims for anti-rabies vaccines supplied by both the
the MOA executed between VSMMC and Cuenco, the hospital represented itself as “willing to VSMMC and Dell Pharmacy, despite the patent irregularities borne out by the
cooperate/coordinate and monitor the implementation of a Medical Indigent Support referral slips and prescriptions related thereto.65 Had there been an internal
Program.”48 More importantly, it undertook to ascertain that “[a]ll payments and releases control system installed by petitioners, the irregularities would have been exposed,
under [the] program x x x shall be made in accordance with existing government accounting and the hospital would have been prevented from processing falsified claims and
and auditing rules and regulations.”49 It is a standing rule that public officers who are unlawfully disbursing funds from the said PDAF. Verily, petitioners cannot escape
custodians of government funds shall be liable for their failure to ensure that such funds are liability for failing to monitor the procedures implemented by the TNT Office on the
safely guarded against loss or damage, and that they are expended, utilized, disposed of or ground that Cuenco always reminded them that it was his money.66 Neither may
transferred in accordance with the law and existing regulations, and on the basis of prescribed deviations, from the usual procedure at the hospital, such as the admitted
documents and necessary records.50 However, as pointed out by the SAT, provisions of the bypassing of the VSMMC social worker in the qualification of the indigent-
National Budget Circular No. (NBC) 47651 dated September 20, 2001 prescribing the beneficiaries,67 be justified as “a welcome relief to the already overworked and
guidelines on the release of funds for a congressman’s PDAF authorized under Republic Act undermanned section of the hospital.”68
No. 876052 were not followed in the implementation of the TNT Program, as well as other
existing auditing laws, rules and regulations governing the procurement of medicines.

In this relation, it bears stating that Delos Santos’ argument that the practices of the TNT
Office were already pre-existing when she assumed her post and that she found no reason to
In particular, the TNT Program was not implemented by the appropriate implementing agency, change the same69 remains highly untenable. Records clearly reveal that she, in fact,
i.e., the Department of Health, but by the office set up by Cuenco. Further, the medicines admitted that when she was installed as the new Medical Center Chief of VSMMC sometime “in
purchased from Dell Pharmacy did not go through the required public bidding in violation of the late 2003,” Antoni disclosed to her the irregularities occurring in the hospital specifically on
the applicable procurement laws and rules.53 Similarly, specific provisions of the MOA itself pre-signed and forged prescriptions.70 Hence, having known this significant information, she
setting standards for the implementation of the same program were not observed. For and Antoni should have probed into the matter further, and, likewise, have taken more
instance, only seven of the 133 prescriptions served and paid were within the maximum limit stringent measures to correct the situation. Instead, Delos Santos contented herself with
of P5,000.00 that an indigent patient can avail of from Cuenco’s PDAF. Also, several indigent giving oral instructions to resident doctors, training officers, and Chiefs of Clinics not to leave
patients availed of the benefits more than once, again in violation of the provisions of the pre-signed prescriptions pads, which Antoni allegedly followed during the orientations for new
MOA.54 Clearly, by allowing the TNT Office and the staff of Cuenco to take over the entire doctors.71 But, just the same, the falsification and forgeries continued, and it was only a year
process of availing of the benefits of the TNT Program without proper monitoring and after, or in December 2004, that Delos Santos ordered a formal investigation of the attendant
observance of internal control safeguards, the hospital and its accountable officers reneged on irregularities. By then, too much damage had already been done.
their undertaking under the MOA to “cooperate/coordinate and monitor” the implementation of
the said health program. They likewise violated paragraph 555 of NBC 476 which requires a
“regular monitoring activity” of all programs and projects funded by the PDAF, as well as
Sections 12356 and 12457 of Presidential Decree No. 1445,58 otherwise known as the All told, petitioners’ acts and/or omissions as detailed in the assailed CoA issuances72 and as
“Government Auditing Code of the Philippines” (Auditing Code), which mandates the
aforedescribed reasonably figure into the finding that they failed to faithfully discharge their
installation, implementation, and monitoring of a “sound system of internal control” to respective duties and to exercise the required diligence which resulted to the irregular
safeguard assets and check the accuracy and reliability of the accounting data. disbursements from Cuenco’s PDAF. In this light, their liability pursuant to Sections 10473 and
10574 of the Auditing Code, as well as Section 16 of the 2009 Rules and Regulations on
Settlement of Accounts,75 as prescribed in CoA Circular No. 2009-006, must perforce be
upheld. Truly, the degree of their neglect in handling Cuenco’s PDAF and the resulting
By way of defense, petitioners nonetheless argue that VSMMC was merely a passive entity in detriment to the public cannot pass unsanctioned, else the standard of public accountability be
the disbursement of funds under the TNT Program and, thus, invoke good faith in the loosely protected and even rendered illusory.
performance of their respective duties, capitalizing on the failure of the assailed Decisions of
the CoA to show that their lapses in the implementation of the TNT Program were attended by
malice or bad faith.
WHEREFORE, the petition is hereby DISMISSED.

The Court is not persuaded.


SO ORDERED.

Jurisprudence holds that, absent any showing of bad faith and malice, there is a
presumption of regularity in the performance of official duties. However, this Sereno, C.J., Carpio, Velasco, Jr., Leonardo-De Castro, Brion, Peralta, Bersamin, Del Castillo,
presumption must fail in the presence of an explicit rule that was violated. 59 For Abad, Villarama, Jr., Perez, Mendoza, Reyes, and Leonen, JJ., concur.
instance, in Reyna v. CoA60 (Reyna), the Court affirmed the liability of the public
officers therein, notwithstanding their proffered claims of good faith, since their
actions violated an explicit rule in the Landbank of the Philippines’ Manual on

336
RECTO, GAYLE ANGELI M.
2011-0008 | AUSL
Personal Notes on Remedial Law 2 Review (based on the syllabus of Prof. Henedino M. Brondial)

1. General Fairly developed and nourished, patient was still under sedation
during the exam Approximately 5'4" in Ht.; wt., 118 lbs.

Cohabitation 2. Head & Face contusion left temporal area 2x2 cm. dia. Multiple superficial
abrasions on the left forehead, right and left side of the face.

EN BANC
3. Abdomen Linear abrasion, post lumbar, 3 inches length, longitudinal.

G.R. No. 119072 April 11, 1997


4. Breast slightly globular, dark brown areola and nipple, presence of multiple
contusion just below the areola on both breast.
PEOPLE OF THE PHILIPPINES, plaintiff-appelee,

5. Upper extremities: presence of multiple linear abrasions on both arm and


vs. JESUS EDUALINO, accused-appellant. forearm.

PADILLA, J.: 6. External genitalia: numerous pubic hair, labia majora and minora both gaping,
presence of numerous dry leaves (grass) noted on both buttocks.

7. I.E. hymen fimbriated in shape, no laceration noted, easily admits 2 fingers


Facts vaginal wall lax, less prominent rugae, uterus enlarges to 2-3 mos. gestation.

* On 12 May 1994, the complainant and her mother Leonora Caabay were in Mambalot,
Brooke's Point, Palawan to attend a dance. At about ten (10) o'clock in the evening of that day
Rowena saw her cousin Antero Bacosa at the dance and she asked him to drink beer with her. Note: no sperm cell exam. done no available microscope.

* Antero got drunk and fell asleep. It was at this time that accused Jesus Edualino approached
her and offered her a glass of beer. Rowena noticed that Edualino was drunk so she accepted
the glass. She then felt dizzy after drinking the beer. Conclusion: hymenal opening admits easily 2 fingers, it can admit an average
size penis in erection without laceration, uterus enlarges to 2-3 months gestation.
* Edualino then dragged her toward a grassy area where no people were present. The
accused then forced himself on top of her and succeeded in raping her while she was in a
semi-unconscious state.

(SGD.) Rogelio C. Divinagracia, M.D.


* Rowena further stated that she was continuously resisting the assault upon her but Edualino
was stronger and he even boxed her in the stomach. She stated that she passed out after the
Medical Officer 2
rape was consummated.

* Prosecution witness Aileen Yayen testified that she saw the accused in the act of raping
Rowena in the grassy area near the store of a certain Sgt. Edep and the house of a certain
Mrs. Adier. * Accused Jesus Edualino was charged with rape in an information dated 5 July 1993 reading
as follows:

* Aileen stated that she was looking for her cousin Rowena Nantiza in the early morning of 12
May 1994 at Brooke's Point where a dance was being held. She saw Rowena with the accused That on or about the 12th day of May, 1994, at Bgy. Mambalot, Municipality of
on top of her in a dark grassy area near the site of the dance. Both the accused and Rowena Brooke's Point, Province of Palawan, Philippines, and within the jurisdiction of this
were naked. She was able to identify the accused by pointing her flashlight from a distance of Honorable Court, the said accused with lewd design, did then and there wilfully,
less than two (2) meters away. unlawfully and feloniously have carnal knowledge with one ROWENA C.
NANTIZA, a pregnant woman, against her will and consent to her damage and
prejudice.
* She then called her aunt Erlinda de la Cruz, the victim's mother, but when they returned, the
accused immediately left when he noticed their presence.
CONTRARY TO LAW. 1

* Dr. Rogelio Divinagracia, a physician at the Brooke's Point District Hospital, testified that on
13 May 1994, he examined Rowena Nantiza who alleged that she was sexually abused. * The defense had a different version of the incident.

* The medical certificate issued by Dr. Divinagracia reads as follows: * Accused Jesus Edualino, while admitting that he was at the dance at Brook's
Point on 11 May 1994, denied that he raped complainant Rowena Nantiza.

* Edualino testified that after leaving the dance, he and a certain Calixto Flora
went to the store or Sgt Edep to drink beer. After he and Flora had finished a big
Medical Certificate
bottle of beer, complainant Nantiza and a certain Antero Bacosa arrived. They
noticed that Nantiza and Bacosa were already drunk. Accused Edualino testified
that complainant Nantiza then began teasing him to kiss her. He (Edualino) stood
up to get away from her but the latter followed him Flora then held on to
This is to certify that Mrs. Rowena Nantiza, 22 years old, married, of Bgy. Nantiza's arm to prevent the latter from following him Edualino testified that he
Mambalot, Bks Point, with a 2-1/2 yrs old child, was examined of this date She and Flora then went to his house where the they stayed until the morning of 12
alleged to have been sexually forcefully assaulted by a known person, last May May 1994.
12, 1994 She was accompanied by her mother.
* Edualino also testified that Bacosa and Nantiza may have been under the
influence of marijuana since he heard the two (2) talking about having taken
drugs.
The patient upon admission was found to be combative, with emotional outburst,
shouting and crying. She was then put to sleep. * Calixto Flora corroborated the accused-appellant's version of the incident.

* Felix Alberto, a resident of Brooke's Point, testified that in the evening of 11


May 1994 while they were walking towards the place where the dance was being
Findings held, he and his sister Rose saw complainant Rowena Nantiza sitting by the
roadside with her hands cupped covering her mouth. Upon seeing them, Nantiza
337
RECTO, GAYLE ANGELI M.
2011-0008 | AUSL
Personal Notes on Remedial Law 2 Review (based on the syllabus of Prof. Henedino M. Brondial)

allegedly called out "Rose, halika, tikman mo, masarap ito." (Rose, come and try of partiality and bias. However, this per se does not warrant nullification of the entire
this. It's tasty). When they approached Nantiza, Alberto testified that he saw her proceeding in the case.
holding what appeared to be dried marijuana leaves Alberto then testified that he
even scolded Nantiza saying. "Why are you doing that? You have already two
children and you know that is bad" Alberto then took his sister and left.
In People v. Hatton, 6 this Court citing People v. Ibasan 7 held thus:
* Rodolfo Caabay, then barangay captain of Mambalot, Brooke's Point, Palawan
testified that in the early morning of 12 May 1994, an unusual incident was
reported to him Leonora Caabay complained that her daughter Rowena Caabay
Nantiza was found lying on the ground about eight (8) meters from the store
. . . It is not denied that the court had at certain points conducted its own questioning during
owned by a certain Sgt. Edep. He found Rowena was very hysterical and he
the proceedings. The records, however, show that the court's questions did not amount to
observed that she had too much to drink. He turned over Rowena to the police.
interference as to make the case for the prosecution and deprive the accused of their defense.
He later learned that accused-appellant was picked up for questioning regarding
The question of the judge addressed to the witnesses and the accused were merely to clarify
his alleged rape of Rowena Nantiza.
certain points and confirm certain statements. The number of times that a judge intervenes is
not necessarily an indication of bias. It cannot be taken against a judge if the clarificatory
* Epifania Caabay, Rodolfo's wife, testified that she accompanied Rowena and
questions he propounds happen to reveal certain truths which tend to destroy the theory of
her mother on board the police vehicle which took them to Brooke's Point District
one party.
Hospital. She stated that Rowena was hysterical and kept on shouting in the
vernacular, "I want water!" Epifania further stated that Rowena's mother slapped
her and hit her on different parts of the body to quiet her down. Epifania agreed
with the other defense witnesses that Rowena was quite drunk at the time.
As held in the case of Ventura v. Yatco (105 Phil. 287) "Judges are not mere referees like
those of a boxing bout, only to watch and decide the results of a game; they should have as
* On 23 December 1994, the trial court rendered a decision, the dispositive part of which
much interest as counsel in the orderly and expeditious presentation of evidence calling
reads:
attention of counsel to points at issue that are overlooked, directing them to ask questions
that would elicit the facts on the issues involved, clarifying ambiguous remarks by witnesses,
WHEREFORE, in view of all the foregoing facts and considerations, the Court
etc.
hereby finds the herein accused, JESUS EDUALINO guilty beyond reasonable
doubt of the crime of RAPE charged in the above-entitled case as defined and
penalized under Article 335 of the Revised Penal Code in relation to and as
amended by Republic Act No. 7659 and accordingly, he is hereby sentenced to
suffer the penalty of DEATH in the gas chamber or in the electric cha ir and A judge may properly intervene in the trial of a case to promote expedition and avoid
ordered to indemnify the raped victim, ROWENA NANTIZA moral and exemplary unnecessary waste of time or to clear up some obscurity (People v. Catindihan, 97 SCRA 196;
damages amounting to P60,000.00, and to pay the costs. Par. 14 Canons of Judicial Ethics; Administrative Order No. 162 dated August 1, 1946, 42 O.G.
1803). In this respect, the record shows no irregularity in the conduct of the trial judge.
With this conviction and imposition of the death penalty to the accused, he is
hereby ordered immediately shipped to the national penitentiary. Muntinlupa,
Metro Manila, under maximum security, to await the execution of this sentence
there and the review of this decision by the Honorable Supreme Court, Manila, Moreover, it is of note that counsel for accused-appellant did not object, during the trial, to the
Philippines. manner of questioning of the trial judge nor was his inhibition sought by the defense for
alleged bias and technicality for the prosecution.
SO ORDERED. 3

* The conviction of accused-appellant is now before this Court on automatic review.


The Court will now proceed to determine if the guilt of accused-appellant has been proven
beyond reasonable doubt.

Issues

1. The trial court acted with grave abuse of discretion and demonstrated bias and partiality in The elements of the crime of rape, as allegedly committed by accused-appellant, are:
favor of the prosecution during the entire proceedings of the case.

2. The trial court erred in giving credence to the false and incredible testimony of the
complainant and other witnesses for the prosecution and in not giving due credence to the 1. That the accused-appellant had carnal knowledge of the complainant.
evidence for the defense;

3. The trial court erred in making findings of facts not supported by the evidence and in
making conclusions based on mere surmises, conjectures and speculation; and 2. That the act was done against the complainant's will;

4. The trial court erred in convicting the appellant of the heinous crime of rape instead of
upholding his innocence based on the evidence and the law. 4
3. That force and/or intimidation was used in the commission of the act.

Ruling

In the present case, the prosecution's evidence consists mainly of the testimonies of the
Accused-appellant contends that the trial judge actively and "enthusiastically" assisted the complainant Rowena Nantiza, Aileen Yayen and Dr. Rogelio Divinagracia.
prosecution, both in the direct and cross-examination of the witnesses. It is argued that "the
undue interest and bias of (the trial judge) is revealed by his active participation in the entire
proceeding, consistently taking the cudgels for the prosecution, instead of conducting the trial
with the cold neutrality of an impartial judge." 5
On the other hand, accused-appellant relies on alternative defenses of alibi and consent on the
part of complainant. While accused-appellant's defense before the trial court alleges that he
had left the scene of the incident together with defense witness Calixto Flora, he alternatively
raises before this Court the contention that the elements of the crime of rape have not been
A close and careful scrutiny of the transcripts of the proceedings before the trial court shows established. 8
that the trial court judge may have exhibited a degree of zeal which could lead to impressions
338
RECTO, GAYLE ANGELI M.
2011-0008 | AUSL
Personal Notes on Remedial Law 2 Review (based on the syllabus of Prof. Henedino M. Brondial)

Accused-appellant posits the following arguments: At the outset of this discussion, it should be pointed out that the moral character of
a rape victim is immaterial in the prosecution and conviction of the accused. The
Court has ruled that prostitutes can be the victims of rape. 11

1) No carnal knowledge occurred

In the present case, even if accused-appellant's allegations that the victim was
drunk and under the influence of drugs and that she (the victim) cannot be
considered a decent and responsible married woman, were true, said
It is argued that since Dr. Rogelio Divinagracia did not examine specimens from the
circumstances will not per se preclude a finding that she was raped.
complainant's private parts for the presence of spermatozoa, then complainant's testimony to
the effect she, although in a state of semi-unconsciousness, felt accused-appellant on top of
her consummating the sexual act, deserves no credence.

Accused-appellant cannot successfully argue that no rape occurred because no medical


examination was conducted to confirm the presence of spermatozoa in her private parts.
2) No force or intimidation was employed

The Court has repeatedly held that a medical examination of the victim is not a prerequisite in
prosecutions for rape. 12
It is argued that the force allegedly employed to consummate the rape was merely implied by
the trial court from complainant's testimony that she did nor enjoy the sexual act. Accused-
appellant contends that even assuming that the sexual act was consummated, the same could
only have been successfully done with the consent of the complainant, "for if she ever
attempted to resist or evade the thrust of the penis of appellant, the latter could not have A person accused of rape can be convicted solely on the testimony of the victim provided the
successfully hit the mark and penetrate the vagina." 9 testimony is credible, natural, convincing and otherwise consistent with human nature and the
course of things. 13

Accused-appellant likewise argues that the medical examination conducted on complainant


fails to support the latter's testimony that accused-appellant boxed her in the stomach. After a careful and thorough study of the records of the case, the Court is convinced that the
constitutional presumption of accused-appellant's innocence has been overcome by proof of
guilt beyond reasonable doubt.

3) The identity of the assailant has not been established

On accused-appellant's contention that the presence of force and intimidation was not proven,
the Court has consistently ruled that force and intimidation should be viewed in the light of the
victim's perception and judgment at the time of the commission of the offense. 14
Accused-appellant assails the finding that the complainant and prosecution witness Aileen
Yayen had adequately established that it was accused-appellant who committed the rape.

Indeed, there can be no hard and fast rule on the matter specially in a situation like the
present case where the victim testified to being in a state of semi-consciousness after drinking a
It is argued that complainant, who admitted being only semi-conscious, could not have seen
glass of beer given to her by accused-appellant.
who raped her and Aileen Yayen who, in a written statement before trial, stated that she only
saw accused-appellant in shorts beside the complainant, at the time and place of the alleged
rape, contradicted herself when she testified at the trial that she saw accused-appellant on top
of the complainant in a grassy area behind the store of Sgt. Edep.
Besides, the testimony of the victim is supported by the findings in the aforequoted medical
certificate which shows that the injuries suffered by the victim are consistent with the charges of
rape and contrary to the theory of the defense that the injuries were inflicted by the victim's
mother when she was trying to quiet her daughter who was hysterical.
4) The offense of rape has not been established

The allegation that accused-appellant's identity has not been established deserves scant
Accused-appellant contends that the testimony of the complainant tends to show "that there
consideration. It is to be noted that accused-appellant was known to the victim and
was foreplay before the alleged rape whereby the accused allegedly kissed her, caressed her
prosecution witness Aileen Yayen long before the incident. Both witnesses positively identified
breast and bit her nipple; that the accused was on top of her and inserted his penis in her
the accused as the perpetrator of the rape. There is nothing to show that these two (2)
vagina and did the push and pull movement, that she cannot remember how long it lasted but
witnesses would or did falsely implicate accused-appellant.
she knew [accused] had an orgasm after which the accused stood up and left, all this bear the
earmarks of a voluntary and mutual coition, a consensual intercourse. There was no rape." 10

On whether the acts of accused-appellant constitute rape, the victim Rowena Nantiza's
testimony was sufficiently clear to show that the carnal knowledge was without her consent
Finally, accused-appellant raises the issue of the character of complainant Rowena
and with force and intimidation. There is no doubt that the crime committed by accused-
Nantiza. It is argued that a responsible and decent married woman, who was then
appellant is rape.
three (3) months pregnant, would not be out at two (2) o'clock in the morning
getting drunk much less would a decent Filipina ask a man to accompany her to
drink beer. It is contended that complainant merely concocted the charge of rape
to save her marriage since her husband had found out that she was using drugs
and drinking alcohol and even made a spectacle of herself when she tried to seduce Accused-appellant in a final attempt to absolve himself argues that the charge of
accused-appellant on 11 May 1994 while she was under the influence of drug and rape was concocted by the victim to save her marriage.
alcohol.
339
RECTO, GAYLE ANGELI M.
2011-0008 | AUSL
Personal Notes on Remedial Law 2 Review (based on the syllabus of Prof. Henedino M. Brondial)

7. When by reason or on the occasion of the rape, the victim has suffered permanent physical
mutilation. (As amended by Sec. II, R.A. 7659.) 15
The Court cannot believe that a married woman would invent a story that she was
raped in an attempt to conceal addiction to drugs or alcohol, in order to save her
marriage. We fail to understand how a false rape story can save a marriage under
the circumstances averred by accused-appellant. In the present case, the prosecution has not proved any circumstance which would justify or
call for the imposition of the supreme penalty of death.

The other arguments adduced by accused-appellant pertaining to credibility of the two (2)
prosecution witnesses are basically issues that cannot be reviewed by the Court absent Finally, with regard to the award of P60,000.00 as moral and exemplary damages, it is noted
attendant circumstances that do not exist in this case. that there is no basis for said award. Consequently the award of moral and exemplary
damages is deleted. However, the accused-appellant is liable to indemnify the victim the
amount of Fifty Thousand Pesos (P50,000.00) consistent with prevailing jurisprudence.

The alleged inconsistencies in the testimonies of the prosecution witnesses pertain to minor
matters and are even badges that the witnesses were unrehearsed and honest.
WHEREFORE, based on the foregoing, the judgment of the trial court finding accused-
appellant Jesus Edualino guilty of the crime of rape is AFFIRMED with the following
modifications:
Besides, in reviewing the entire records of this case, we find no reversible error in the
judgment of conviction except as to the penalty of death imposed by the trial court.

1) Accused-appellant is hereby sentenced to suffer the penalty of reclusion perpetua, and

The Solicitor General correctly points out that absent the attending circumstances provided for
under Article 335 of the Revised Penal Code as amended by Republic Act No. 7659 wherein
the penalty for rape is death, the correct penalty is reclusion perpetua. 2) He is ordered to indemnify the victim the amount of Fifty Thousand Pesos (P50,000.00) in
lieu of the award of moral and exemplary damages.

Under Article 335 of the Revised Penal Code, as amended by Section II, R. A. No. 7659.
SO ORDERED.

xxx xxx xxx


Narvasa, C.J., Regalado, Davide, Jr., Romero, Bellosillo, Melo, Puno, Vitug, Kapunan, Mendoza,
Francisco, Hermosisima, Jr., Panganiban and Torres, Jr., JJ., concur.

The death penalty shall be imposed if the crime of rape is committed with any of the following
circumstances:

Presentation of evidence

1. When the victim is under eighteen (18) years of age and the offender is a parent, > Order of presentation of evidence
ascendant, step-parent, guardian, relative by consanguinity or affinity within the third civil
degree, or the common-law spouse of the parent of the victim.
EN BANC

G.R. No. 146697 July 23, 2002

2. When the victim is under the custody of the police or military authorities.
PEOPLE OF THE PHILIPPINES, plaintiff-appellee,

vs. LEONARDO FABRE y VICENTE, accused-appellant.

3. When the rape is committed in full view of the husband, parent, any of the children or other
VITUG, J.:
relatives within the third degree of consanguinity.

4. When the victim is a religious or a child below seven (7) years old. Facts

* Fabre was indicted in an Information that read:1

5. When the offender knows that he is afflicted with Acquired Immune Deficiency Syndrome "That on or about 4:00 o'clock in the afternoon of April 26, 1995 in the house of
(AIDS) disease. the accused located at Manat, Trento, Agusan del Sur, Philippines and within the
jurisdiction of this Honorable Court, the above-named accused by force, threats
and intimidation, with lewd design, did then and there willfully, unlawfully and
feloniously succeed in having sexual intercourse with his own daughter MARILOU
FABRE, a girl thirteen (13) years of age, of good reputation, against her will and
6. When committed by any member of the Armed Forces of the Philippines or the Philippine consent to the damage and prejudice of the said victim consisting of moral,
National Police or any law enforcement agency. actual and compensatory damages."

340
RECTO, GAYLE ANGELI M.
2011-0008 | AUSL
Personal Notes on Remedial Law 2 Review (based on the syllabus of Prof. Henedino M. Brondial)

* Accused pleaded not guilty to the crime charged. At the trial, the prosecution presented the On 26 April 1995, around four o'clock in the afternoon, Marilou Fabre was alone in their house
testimony of Marilou, that of Adela Fabre, her mother and the wife of the accused, and that of in Barangay Manat, Trento, Agusan del Sur. Adela Fabre, her mother, had gone to Purok 4 to
Dr. Reinerio Jalalon, the doctor who examined Marilou, along with the medico-legal certificate buy fish while her siblings were out strolling. After cleaning their yard, Marilou went to the
issued by Dr. Jalalon, the sworn statement of Adela, and the criminal complaint signed by both adjacent palm plantation, about fourteen to fifteen meters away from their house, to gather
Marilou and Adela. The defense, during its turn in the presentation of evidence, countered palm oil. Marilou had been gathering palm oil for about a minute when her father, appellant
with the testimony of the accused himself. It also called Adela Fabre back to the witness Leonardo Fabre, arrived. He suddenly gripped Marilou's hands and forcibly dragged her
stand. towards the house. He closed the door and removed his daughter's underwear. He took off his
pants and asked Marilou to hold his sex organ. In tears, Marilou obeyed her father. He then
* The trial court gave credence to the evidence given by the prosecution, particularly to the began touching the girl's breasts and vagina. He forced her to lie down, mounted her and
narration of the young complainant, expressing a quote from an observation once made by sought to insert his penis into her organ. Marilou cried in pain. When after some time he still
this Tribunal in one of its decision that "even when consumed with revenge, it (would) take a could not insert his penis into Marilou's vagina, he applied coconut oil to lubricate his and his
certain amount of psychological depravity for a young woman to concoct a story which (could) daughter's sexual organs. He was finally able to penetrate her. Once inside her, appellant
put her own father for the rest of his remaining life in jail and drag herself and the rest of her made push and pull movements until he was through with her. Appellant threatened to kill her
family to a lifetime of shame."2 Convinced that the accused committed the crime of rape on if she would tell anybody about the sexual encounter. The young girl's mother, Adela Fabre,
his own daughter, the trial judge disposed of the case thusly: arrived home about five o'clock that afternoon but, remembering her father's threats, she kept
mum about her ordeal.
"WHEREFORE, the Court finds accused LEONARDO FABRE y VICENTE alias
Nardo, GUILTY beyond reasonable doubt as principal of the crime of RAPE as
defined and penalized under Article 335 of the Revised Penal Code as amended
by R.A. No. 7659 Section 11 thereof and hereby imposes upon the accused The credibility of Marilou would not be all that difficult to discern from her narration that, as so
Leonardo Fabre y Vicente alias Nardo the penalty of DEATH; to pay the victim described by the prosecution, "was full of graphic details which a young provincial girl could
Marilou Fabre civil indemnity in the amount of FIFTY THOUSAND (P50,000.00) not possibly have concocted and which could only have come from someone who must have
PESOS and the costs."3 personally experienced a brutal rape assault." She testified:

* [Hence,] this automatic review.

"PROS. ENRIQUEZ:

Issues "Q Now, Miss Marilou, can you recall where were you on April 26, 1995 at about 4:00
o'clock in the afternoon?
"I. "THE TRIAL COURT GRAVELY ERRED IN NOT GIVING CREDENCE TO ACCUSED-
APPELLANT'S DEFENSE OF ALIBI AND DENIAL. "A Yes, sir.

"II. "ASSUMING IN ARGUENDO THAT ACCUSED-APPELLANT IS GUILTY, THE TRIAL COURT


GRAVELY ERRED IN IMPOSING THE DEATH SENTENCE UPON ACCUSED-APPELLANT DESPITE
THE FAILURE OF THE PROSECUTION TO ESTABLISH THE ACTUAL AGE OF MARILOU FABRE "Q Where were you that time?
AT THE TIME OF THE COMMISSION OF THE ALLEGED RAPE."4
"A In our house, sir.

Rulings
"Q What were you doing in your house?
The defense argues, rather desperately, that the testimony of appellant should
acquire added strength for the failure of the prosecution to conduct cross-
"A I was cleaning our yard, sir.
examination on him and to present any rebuttal evidence. The cross-examination
of a witness is a prerogative of the party against whom the witness is called.5 The
purpose of cross-examination is to test the truth or accuracy of the statements of a
witness made on direct examination.6 The party against whom the witness
testifies may deem any further examination unnecessary and instead rely on any "Q How far is your yard where you were doing some works from your house?
other evidence theretofore adduced or thereafter to be adduced or on what would
be believed is the perception of the court thereon. Certainly, the trial court is not "A (Witness pointing a distance of around 2 to 3 meters.)
bound to give full weight to the testimony of a witness on direct examination
merely because he is not cross-examined by the other party.

"Q Now, while you were doing your work in your yard, can you recall if there was an
incident that occurred?
The alibi of appellant itself would not appear to be deserving of serious consideration. His
account that at the time of the alleged rape he was working at a coconut plantation, just "A Yes, sir.
about one kilometer away from the place of the crime, hardly would amount to much. Nor
would the testimony of Adela Fabre, his wife, merit any better regard. At first, she testified
that on the day of the rape incident, she had left their house at four o'clock in the afternoon.
Later, however, she changed her story by saying that she had left the house in the morning
and returned only at ten o'clock that same morning, staying home the whole day thereafter. In "Q What was that incident that occurred?
any event, in order that alibi might prosper, it would not be enough for an accused to prove
"A While I was gathering a palm oil my father arrived and suddenly dragged me to our
that he was somewhere else when the crime was committed; he would have to demonstrate
likewise that he could not have been physically present at the place of the crime or in its house, sir.
immediate vicinity at the time of its commission.7 Clearly, in the instant case, it was not at all
impossible nor even improbable for appellant to have been at the crime scene.

"COURT:

Upon the other hand, the evidently candid and straightforward testimony of Marilou should be "Q Where is your house located?
more than enough to rebut the claim of innocence made by appellant.8
"A At Purok 4, Manat, Trento, Agusan del Sur, Your Honor.

341
RECTO, GAYLE ANGELI M.
2011-0008 | AUSL
Personal Notes on Remedial Law 2 Review (based on the syllabus of Prof. Henedino M. Brondial)

"Q And did he use anything to facilitate the insertion of his penis to your vagina?

"PROS. ENRIQUEZ: "A Yes, sir.

"Q What did you do when your father dragged you to your house?

"A Because I was dragged by my father to our house I just went with him, sir. "Q What was that?

"A He used coconut oil in his penis and also in my vagina so that his penis can easily
insert my vagina, sir.
"Q While you were in your house after having been dragged by your father, what
happened if any?

"A He closed our house and he removed my panty, sir. "Q Now, while his penis was in your vagina, can you tell this Honorable Court if he did
anything also on top of you and while his penis was inside your vagina?

"A None, sir.


"Q And after removing your panty, what did your father do next?

"A He removed his pants and he let me hold his penis, sir.
"Q Did he make any movement?

"A Yes, sir.


"Q And what did you do next after holding his penis?

"A I was crying, sir.


"Q What was that movement?

"A He made a push and pull movement on my body, sir.


"Q While you were crying what did your father do?

"A He was touching my breast and my vagina, sir.


"Q Now, while your father was doing it to you where was your mother that time?

"A She was in Purok 4, Manat, Trento, Agusan del Sur, sir.
"Q After that what did he do next?

"A He let me lie down, sir.


"Q And did you report this incident to your mother?

"A Not yet sir because he told me not to tell anybody.


"Q And while lying down, what did your father do?

"A He mounted me and he inserted his penis, to my vagina, sir.


"Q So when did you had a chance to tell your mother about this incident?

"A On May 1, 1995, sir.


"Q And what did you feel while your father was inserting his penis to your vagina?

"A Very painful, sir.


"Q And what did your mother do after you reported to her this incident?

"A She reported [the matter] to the Kagawad, sir."9


"Q And what did you do while your father was inserting his penis to your vagina?

"A I was crying, sir.


It has been stressed quite often enough that the testimony of a rape victim, who is young and
still immature, deserves faith and credence10 for it simply would be unnatural for a young and
innocent girl to invent a story of defloration, allow an examination of her private parts and
thereafter subject herself and her family to the trauma of a public trial unless she indeed has
"Q And while you were crying what did your father do if any?
spoken the truth.11 Most especially, a daughter would not accuse her own father of such a
serious offense or allow herself to be perverted if she were not truly motivated by a desire to
"A He told me not to tell anybody because if I will do it he will kill me, sir.
seek a just retribution for a violation brazenly committed against her.12

"Q Now, did your father find it easy to insert his penis to your vagina? Confirming Marilou's story was the medical report and testimony of Dr. Reinerio Jalalon, the
government physician stationed at the Bunawan District Hospital in Agusan del Sur, who
"A It [took] a long time, sir.
examined Marilou. Dr. Jalalon made these findings; viz:

342
RECTO, GAYLE ANGELI M.
2011-0008 | AUSL
Personal Notes on Remedial Law 2 Review (based on the syllabus of Prof. Henedino M. Brondial)

"Abrasion at (L) labia minora at 3:00 o'clock position.

"Vaginal smear (-) negative for spermatozoa."13 Davide, Jr.*, Bellosillo, Puno, Kapunan, Mendoza, Panganiban, Quisumbing, Ynares-Santiago,
Sandoval-Gutierrez, Carpio, Austria-Martinez, and Corona, JJ., concur.

The doctor concluded that it was possible that genital penetration on the victim did occur and
that a penis could have caused the abrasion on the victim's labia minora. > Leading and misleading questions

EN BANC

There is merit, however, in the plea of the defense, seconded by the prosecution, that the
G.R. No. 142556 February 5, 2003
penalty of death imposed by the trial court should be reduced to the penalty of reclusion
perpetua. Article 335 of the Revised Penal Code, as amended by Section 11 of Republic Act
No. 7659, provides: PEOPLE OF THE PHILIPPINES, plaintiff-appellee,

vs. JESUS PEREZ y SEBUNGA, accused-appellant.

"The death penalty shall also be imposed if the crime of rape is committed with any of the PER CURIAM:
following attendant circumstances:

Facts
1. when the victim is under eighteen (18) years of age and the offender is a parent,
ascendant, stepparent, guardian, relative by consanguinity or affinity within the third civil * On January 22, 1997, the Second Assistant Provincial Prosecutor2 of Zambales filed an
degree, or the common-law-spouse of the parent of the victim." Information3 charging appellant with the crime of rape "penalized under Article 335 of the
Revised Penal Code in relation to Section 5 (b), Article III of Republic Act No. 7610,"
committed as follows:

While the father-daughter relationship between appellant and private complainant has been "That on or about the 17th day of January, 1997 at 12:00 noon at Sitio Baco,
sufficiently established, the fact of minority of the victim, although specifically averred in the Brgy. Macarang, in the Municipality of Palauig, Province of Zambales, Philippines,
information, has not been equally shown in evidence. These qualifying circumstances of and within the jurisdiction of this Honorable Court, the said accused, with lewd
relationship and minority are twin requirements that should be both alleged in the information design and by means of coercion, inducement and other consideration, did then
and established beyond reasonable doubt during trial in order to sustain an imposition of the and there, wilfully (sic), unlawfully and feloniously have sexual intercourse with
death penalty.14 Neither an obvious minority of the victim nor the failure of the defense to one Mayia P. Ponseca, a minor of 6 years old, without her consent and against
contest her real age always excuse the prosecution from the desired proof required by law.15 her will, to the damage and prejudice of the latter."
Judicial notice of the issue of age without the requisite hearing conducted under Section 3,
Rule 129, of the Rules of Court, would not be considered sufficient compliance with the law. * Upon arraignment, appellant, assisted by counsel de officio Atty. Genaro N. Montefalcon,
The birth certificate of the victim or, in lieu thereof, any other documentary evidence, like a pleaded not guilty to the offense charged.4 Subsequently, the trial court allowed the
baptismal certificate, school records and documents of similar nature, or credible testimonial withdrawal of Atty. Montefalcon as counsel for health reasons. The trial court appointed Atty.
evidence, that can help establish the age of the victim should be presented.16 While the Roberto Blanco as appellant’s counsel de oficio.5
declaration of a victim as to her age, being an exception to the hearsay proscription, would be
admissible under the rule on pedigree, the question on the relative weight that may be * At the pre-trial, the prosecution and defense stipulated on the following facts:
accorded to it is a totally different matter.17

"1. The identity of the accused;

2. The accused was at the time of the incident in the vicinity thereof;
In the case at bar, the complainant claimed that she was 13 years old at the time of the
incident.18 Her mother stated, however, that she was 14.19 The birth certificate of the victim,
3. The victim in this case, Mayia P. Ponseca, was born on 23 May 1990 as
at least already in her teens, was not presented to ascertain her true age on the bare
evidenced by her birth certificate;
allegation that the document was lost when their house burned down.20 No other document
that could somehow help establish the real age of the victim was submitted.
4. That after the incident, the child was subjected to a medico-legal examination
to which a medico-legal certificate was issued by Dr. Editha Divino.

* The prosecution marked in evidence the birth certificate of the victim Mayia O.
The Court, in sum, upholds the decision of the trial court convicting Leonardo Fabre of the
Ponseca as Exhibit ‘A’, and the medico-legal certificate issued by Dr. Editha
crime of rape but must reduce, on account of insufficiency of proof on the qualifying
Divino as Exhibit ‘B’."6
circumstance of minority of the victim, the penalty of death to reclusion perpetua. With
respect to the civil liability, the Court sustains the award of P50,000.00 civil indemnity but, in
keeping with prevailing jurisprudence, must additionally order the payment of P50,000.00 * Thereafter, trial ensued. The prosecution presented the following witnesses: the victim,
moral damages21 and P20,000.00 exemplary damages.22 Mayia Ponseca; the victim’s mother, Hermie Ponseca; the victim’s father, Osias Ponseca;
Virginia Espejo Giron; and Dr. Editha dela Cruz Divino. On the other hand, the defense
presented appellant and his employer, Bartolome Tolentino.

* The Office of the Solicitor General ("OSG" for brevity) summarized the prosecution’s version
WHEREFORE, the judgment of the court a quo finding LEONARDO FABRE guilty of rape is
of the incident in the appellee’s brief, to wit:
AFFIRMED but the sentence of death therein imposed should be, as it is hereby so, reduced to
reclusion perpetua. The award of P50,000.00 civil liability in favor of victim, Marilou Fabre, is
sustained; however, appellant is further ordered to pay to the victim the amounts of * "On January 17, 1997, about noontime, in Sitio Baco, Barangay Macarang,
P50,000.00 moral damages and P20,000.00 exemplary damages. Palauig, Zambales, six-year old Mayia Ponseca was walking along Sulok on her
way to her house in Sitio Camiling when appellant Jesus Sebunga Perez
approached her (pp. 7-8, TSN, December 15, 1998). Appellant introduced himself
as "Johnny" and immediately afterwards, strangled her neck and boxed her
abdomen (p. 10, TSN, December 15, 1998). Still in shock, Mayia fell down (id.).
SO ORDERED. At that point, a dog arrived and barked at them.

343
RECTO, GAYLE ANGELI M.
2011-0008 | AUSL
Personal Notes on Remedial Law 2 Review (based on the syllabus of Prof. Henedino M. Brondial)

* Appellant then proceeded to lower his black denim pants while simultaneously of the offense, in relation to Section 5 (b), Article III, Republic Act 7610, and is
removing Mayia’s panty. He then inserted his penis inside Mayia’s vagina (p. 11, sentenced to suffer the penalty of DEATH. Jesus Perez is directed to pay to the
id.). Mayia felt excruciating pain in her private parts (sic) but was not able to private complainant the amount of Seventy-Five Thousand Pesos (P75,000.00) as
repel her aggressor whose strength and weight totally engulfed her. Her only and by way of civil indemnity and Fifty Thousand (P50,000.00) as and by way of
recourse was to cry while her young body was being ravished (p. 13, id.). moral damages."

* After satisfying his beastly desires, appellant raised his pants and ran away (p. * Hence, this automatic review.
14, id.). Notwithstanding that her vagina was bleeding profusely and her dress
now covered with her own blood, Mayia managed to stand up and seek help. She
ran to the house of Virginia Giron, which was only fifty (50) meters away from
the scene of the crime. In fact, Giron was outside when she heard her dog
Issue
barking (apparently, it was the same dog barking at appellant while he was
consummating his lust on Mayia, pp. 2-3, TSN, January 12, 1999; p. 11, TSN,
"THE COURT A QUO GRAVELY ERRED IN FINDING THAT THE GUILT OF THE APPELLANT HAS
December 15, 1998). Looking at the direction of the noise, she saw a confused
BEEN PROVEN BEYOND REASONABLE DOUBT."
Mayia approaching her with blood dripping from her private parts and thighs.
When Giron asked Mayia what happened, the latter shouted "ni-rape ako, ni-rape
ako" (p. 4, TSN, January 4, 1999). Giron then summoned her husband and other
companions to look for Mayia’s attacker but was unable to find him. Giron then
proceeded to Hermie Ponseca and Osias Ponseca, Mayia’s parents, to inform Ruling
them of what happened (p. 5, TSN, January 5, 1999; p. 2, TSN, January 19,
1999). Appellant contends that his identification in open court by Mayia was highly
irregular.l^vvphi1.net Appellant points out that the prosecutor had already identified him as
* When her parents asked Mayia if she knew her assailant, the latter answered the man wearing an orange t-shirt when the prosecutor asked Mayia to identify her alleged
the name "Johnny." (id.) The couple brought their daughter to the President rapist. Appellant stresses that when Mayia identified him in open court, she referred to him as
Ramon Magsaysay Memorial Hospital for medical examination (p. 2, TSN, a man named "Johnny" and did not give any description or any identifying mark. Moreover,
February 24, 1999). She was examined by Dra. Editha Dela Cruz Divino, who appellant claims he was alone in the cell when Mayia identified him after the police arrested
issued a medico-legal certificate dated January 23, 1997 stating the following: him. Appellant bewails that the identification was not done with the usual police line-up.

a. Bleeding of genitalia coming from median laceration at the


vaginal floor around four (4) centimeters in size. Possible cause, a
fall and then hitting a sharp object and also an alleged sexual Appellant’s contention is untenable.
assault (p. 4, TSN, February 24, 1999).

b. Genitalia had hymenal lacerations at 3, 6, 9 and 12 o’clock


positions.
As a rule, leading questions are not allowed. However, the rules provide for
exceptions when the witness is a child of tender years13 as it is usually difficult for
(pp. 4-6 id.) such child to state facts without prompting or suggestion.14 Leading questions are
necessary to coax the truth out of their reluctant lips.15 In the case at bar, the trial
* Because of the extent of the damage on her genitals, Mayia undertook an IV court was justified in allowing leading questions to Mayia as she was evidently
sedation operation to repair her lacerations (p. 6, id.) During her confinement at young and unlettered, making the recall of events difficult, if not uncertain.16 As
the hospital, the Ponseca couple reported the incident to the Palauig PNP Police explained in People v. Rodito Dagamos:17
Station and recounted their daughter’s narration including the name of the culprit
as "Johnny" who, according to their neighbors, was a worker at the fishpond of
Bartolome Tolentino (pp. 11-12, TSN, January 5, 1999). Police operatives then
proceeded to the said fishpond and arrested appellant. After her discharge from
"The trend in procedural law is to give wide latitude to the courts in exercising
the hospital, Mayia learned that appellant was already apprehended (pp. 3-8,
control over the questioning of a child witness. The reasons are spelled out in our
TSN, January 5, 1999). In the police station, she was able to positively identify
Rule on Examination of a Child Witness, which took effect on December 15, 2000,
the appellant as the person who sexually assaulted her (p. 18, TSN, December
namely, (1) to facilitate the ascertainment of the truth, (2) to ensure that
15, 1998)."7
questions are stated in a form appropriate to the developmental level of the child,
(3) to protect children from harassment or undue embarrassment, and (4) avoid
* Appellant denied raping Mayia. Appellant testified that on the date of the alleged rape waste of time. Leading questions in all stages of examination of a child are allowed if
incident, he was working at a fishpond at Macarang, Zambales. He heard of the rape of a the same will further the interests of justice."
young girl from his manager, Bartolome Tolentino ("Tolentino" for brevity).8 Appellant further
testified that on January 25, 1997, policemen went to the fishpond where he worked. The
policemen arrested appellant and brought him to the police station at Palauig. Later, the
policemen took him to the municipal jail of Palauig.
The Court has repeatedly stated that it is highly inconceivable for a child of tender age,
inexperienced in the ways of the world, to fabricate a charge of defloration, undergo a medical
* On cross-examination, appellant testified that his nickname is not "Johnny" but
examination of her private part, subject herself to public trial, and tarnish her family’s honor
"Jessie."9 He testified that on January 17, 1997, at around 12 o’clock noon, he
and reputation, unless she was motivated by a strong desire to seek justice for the wrong
left the fishpond and walked home to Barangay Alwa which was about thirty
committed against her.18
meters from the fishpond.10

* The defense formally offered the testimony of witness Tolentino to prove that
appellant was employed as caretaker of Tolentino’s fishpond for almost two years
before the alleged rape incident. Appellant was purportedly of good moral Mayia recounted her harrowing experience, thus:
character while employed as a fishpond caretaker. The prosecution admitted the
offer of testimony. Hence, the trial court dispensed with the testimony of
Tolentino in open court.11
"Q What time was this when Johnny introduced himself to you?
* After trial, the court a quo rendered judgment12 on October 26, 1999, the dispositive
portion of which reads: A I do not recall, ma’m.

"WHEREFORE, foregoing considered, accused Jesus Perez y Sabung (SIC) is


found GUILTY beyond reasonable doubt of the crime of Statutore Rape, defined
and penalized under Article 335 of the Revised Penal Code with the qualifying Q Was it in the morning, noontime or in the afternoon or in the evening?
circumstance that the victim was only 6 years old at the time of the commission

344
RECTO, GAYLE ANGELI M.
2011-0008 | AUSL
Personal Notes on Remedial Law 2 Review (based on the syllabus of Prof. Henedino M. Brondial)

A Noontime, ma’m. Q When he used his penis in entering your private part, did he remove his pants?

A No, ma’m.

Q So, when Johnny said, ‘Ako si Johnny,’ what did you do?

A None, ma’m. Q What did he do with his pants?

A He brought out his penis, ma’m.

Q After that when Johnny said, ‘Ako si Johnny’, what happened?

A He strangled (sinakal) me. Q You mean to say Mayia, he lowered his pants?

A Yes, ma’m.

Q Were there persons around in the place when Johnny strangled you?

A None, ma’m. Q What about you, were you wearing any panty?

A Yes, ma’m.

Q So, what did he do then after he strangled you?

A He boxed me on my stomach, ma’m. Q What was your clothes at that time?

A A dress, ma’m.

Q When he boxed you on your stomach, what happened to you?

A I was shocked, ma’m. Q When his penis entered your vagina Mayia, did he remove your panty?

A Yes, ma’m."19

Q Did you fall down?

A Before that, I was already lying down, so when he boxed me, I was shocked. The identity of appellant as the rapist has been established by the clear, convincing and
straightforward testimony of Mayia. During the trial, she testified as follows:

Q You said that you were already lying down. Who made you lie down?
"Q Mayia, there is a man sitting wearing orange t-shirt, do you know this man?
A The person, ma’m.
A Yes, ma’m.

Q Why were you shocked, Mayia?


Q Do you know his name?
A Because he strangled me and boxed me.
A Yes, ma’m.

Q After he boxed you on your abdomen, what happened? What else did he do to you?
Q What is his name?
A There was a dog that arrived in the place and it barked at us. Then Johnny moved in a hurry
by penetrating my private part and after he dressing (SIC) me, he ran away. A Johnny, ma’m.

Q You said that Johnny penetrated your private part. With what instrument did he use in Q Why do you know him?
penetrating your private part?
A Because he introduced himself to me.
A His penis, ma’m.

Q Where did he introduced himself to you?


Q What was he wearing at that time?
A At Sulok, ma’m.
A A black denim, ma’m.

345
RECTO, GAYLE ANGELI M.
2011-0008 | AUSL
Personal Notes on Remedial Law 2 Review (based on the syllabus of Prof. Henedino M. Brondial)

Q Sulok is a place? Mayia’s identification in open court of appellant as her rapist dispels any doubt as to the
proper identification of appellant. Mayia positively identified and pointed to appellant as her
A Yes, ma’m. rapist. We are satisfied that her testimony, by itself, is sufficient identification of her rapist. As
held in People v. Marquez:27

Q Do you have any companion when this man introduced himself to you?
"xxx. Indeed, the revelation of an innocent child whose chastity was abused deserves full
credit, as the willingness of complainant to face police investigation and to undergo the trouble
A None, ma’m.
and humiliation of a public trial is eloquent testimony of the truth of her complaint. Stated
differently, it is most improbable for a five-year old girl of tender years, so innocent and so
guileless as the herein offended party, to brazenly impute a crime so serious as rape to any
man if it were not true."
Q How did he introduce himself to you?

A The man introduced himself to me by saying, ‘Kilala mo ba ako? Hindi po. Ako si Johnny.’"20
In his Reply Brief, appellant contends that even assuming that the guilt of appellant has been
proven beyond reasonable doubt, the trial court erred in imposing the death penalty. Appellant
maintains that the death penalty cannot be imposed on him for failure of the prosecution to
The trial court further asked Mayia: prove Mayia’s age by independent evidence. Appellant points out that while Mayia’s birth
certificate was duly marked during the pre-trial, it was not presented and identified during the
trial. Appellant asserts that Mayia’s minority must not only be specifically alleged in the
Information but must also be established beyond reasonable doubt during the
trial.1awphi1.nét
"Q You were talking of a certain Johnny. s this Johnny in court now?

A Yes, sir.

Appellant’s argument deserves scant consideration.

Q Can you point to him?

At the pre-trial, the parties mutually worked out a satisfactory disposition of the criminal case.
A Yes, sir.
Appellant, assisted by counsel, signed a Pre-Trial Agreement28 which, as incorporated in the
Pre-Trial Order, stated that:

Q Point to him.

"x x x.
A (Witness pointing to the person sitting at the accused bench and when asked of his name
answered Jesus Perez)

3. The victim in this case, Mayia P. Ponseca was born on 23 May 1990 as evidenced by her
birth certificate;
Q Is this Johnny whom you point to the person whom you saw in that ‘Sulok?’

A Yes, sir." 21

x x x." (Emphasis supplied)

Mayia’s simple, positive and straightforward recounting on the witness stand of her harrowing
experience lends credence to her accusation. Her tender age belies any allegation that her
During the pre-trial, the prosecution marked in evidence Mayia’s birth certificate as Exhibit
accusation was a mere invention impelled by some ill-motive. As the Court has stressed in
"A".29 The prosecution submitted its Offer of Evidence30 which included Exhibit "A", a
numerous cases, when a woman or a child victim says that she has been raped, she in effect
certified true copy of Mayia’s birth certificate. The trial court admitted Exhibit "A"31 without
says all that is necessary to show that rape was indeed committed.22
any objection from the defense.

Mayia had a clear sight of appellant’s face since the rape occurred at "noontime."23 Her
The purpose of pre-trial is to consider the following: (a) plea bargaining; (b) stipulation of
proximity to appellant during the sexual assault leaves no doubt as to the correctness of her
facts; (c) marking for identification of evidence of the parties; (d) waiver of objections to
identification for a man and woman cannot be physically closer to each other than during the
admissibility of evidence; (e) modification of the order of trial if the accused admits the charge
sexual act.24 Thus, even if Mayia did not give the identifying marks of appellant, her positive
but interposes lawful defenses; and (f) such matters as will promote a fair and expeditious trial
identification of appellant sufficed to establish clearly the identity of her sexual assailant.
of the criminal and civil aspects of the case.32 Facts stipulated and evidence admitted during
pre-trial bind the parties. Section 4, Rule 118 of the Revised Rules of Criminal Procedure33
provides:

Appellant’s claim that the police improperly suggested to Mayia to identify appellant is without
basis. True, Mayia did not identify appellant in a police line-up when Mayia identified appellant
in his cell. However, appellant, in his testimony admitted that he had two other companions in
"SEC. 4. Pre-trial order. - After the pre-trial conference, the court shall issue an order reciting
his cell.25 Moreover, the Court has held that there is no law requiring a police line-up as
the actions taken, the facts stipulated, and evidence marked. Such order shall bind the parties,
essential to a proper identification. Even without a police line-up, there could still be a proper
limit the trial to matters not disposed of, and control the course of the action during the trial,
identification as long as the police did not suggest such identification to the witnesses.26 The
unless modified by the court to prevent manifest injustice." (Emphasis supplied)
records are bereft of any indication that the police suggested to Mayia to identify appellant as
the rapist.

346
RECTO, GAYLE ANGELI M.
2011-0008 | AUSL
Personal Notes on Remedial Law 2 Review (based on the syllabus of Prof. Henedino M. Brondial)

Moreover, Mayia herself testified in open court as to her age. During the trial on December 15, * Diosdado Volante, who eked out a living as a farmer, his wife Luz,2 and their four children
1998, which was about twenty-three (23) months after the rape incident occurred on January lived in their farmland located in the outskirt of Sitio Danawan, Barangay Sagrada, Bula,
17, 1997, Mayia testified on cross-examination that she was "8 years old last May 23."34 Thus, Camarines Sur.
by deduction, since Mayia was born on May 23, 1990 as shown in her birth certificate, she was
about six (6) years and seven (7) months old on January 17, 1997, the day the crime took * About 200 meters away from Diosdado’s farmland was the farmhouse of Jaime Castillano,
place. We rule that the prosecution has indisputably proven that Mayia was below seven years Sr.3 He tasked his son, Jaime Castillano, Jr., to take care of the farmhouse and allowed him to
old at the time appellant raped her.1a\^/phi1.net reside there.4 Jaime, Sr., his wife Concepcion, their son Ronald (Nono) Castillano and other
children lived at their family residence in Sagrada, Bula, Camarines Sur, approximately three
kilometers away from their farmhouse in Sitio Danawan.5

Finally, the trial court was correct in imposing the death penalty on appellant. Under Article * Sometime in the early part of June 1996,6 Jaime, Sr. fired his gun indiscriminately. Afraid
33535 of the Revised Penal Code, as amended by Section 11 of Republic Act No. 7659,36 the that a stray bullet might hit any member of his family, Diosdado accosted Jaime, Sr. and asked
death penalty shall be imposed if the crime of rape is committed against a child below seven him to desist from firing his gun indiscriminately. Jaime, Sr. resented the intrusion. He
(7) years old. Mayia was six (6) years and seven (7) months old when appellant raped her. remonstrated that neighbors did not even complain about him firing his gun. A heated
altercation ensued. Jaime, Sr. then fired his gun towards the house of Diosdado. The incident
germinated deep animosity between the two and their respective families.7 Jaime, Sr. always
carried a bolo whenever he passed by the house of Diosdado.
If rape is qualified by any of the circumstances37 warranting the imposition of the death
penalty, the civil indemnity for actual or compensatory damages is mandatory.38 Following * On July 8, 1996, between 5:00 p.m. to 6:00 p.m., Levy Avila, a teacher, was in his house
prevailing jurisprudence, the civil indemnity is fixed at P75,000.00. In addition, moral damages doing some repairs. He noticed Jaime, Jr. and Ronald talking by the roadside near the gate of
of P50,000.00 should also be awarded to the rape victim without need for pleading or proving his (Levy’s) house. Levy overheard the two planning to go to Diosdado’s house. Jaime, Jr. and
it.39 Ronald even told Levy: "Ayaw namin kasing inaasar." Suspecting that the two were intending
to harm Diosdado, Levy urged them to amicably settle their differences with Diosdado.

* At around 8:00 p.m., Luz and Diosdado were about to retire for the night. Their children
were already fast asleep. Diosdado was tired after a day’s work of spraying chemicals at the
WHEREFORE, the Decision dated October 26, 1999 of the Regional Trial Court of Iba,
rice field. He reclined on a bamboo bench near the main door of their house. A kerosene lamp
Zambales, Branch 69, in Criminal Case No. RTC-2116-I, finding appellant Jesus S. Perez guilty
lighted the house. Suddenly, Luz heard voices near their house. She saw Jaime, Sr. holding a
beyond reasonable doubt of the crime of qualified rape, sentencing him to suffer the death
flashlight and his two sons, Jaime Jr. and Ronald, on their way to the house. Luz immediately
penalty,40 and ordering him to pay the victim Mayia P. Ponseca the amount of P75,000.00 as
alerted her husband and told him that the Castillanos were in their yard. However, Diosdado
civil indemnity and P50,000.00 as moral damages, is AFFIRMED in toto.
was nonchalant and simply told Luz not to mind them. All of a sudden, Jaime, Sr. fired his gun
at Diosdado’s house. Terrified, Luz hastily carried her baby daughter Mary Jane, sought cover
and hid near the rear door. She was about five meters away from her husband when the
Castillanos barged inside their house and ganged up on Diosdado. Jaime, Jr. and Ronald,
In accordance with Article 83 of the Revised Penal Code, as amended by Section 25 of the armed with bladed weapons, took turns in stabbing Diosdado. Ronald stabbed Diosdado on
Republic Act No. 7659, upon the finality of this Decision, let the records of this case be the right side of his breast, right thigh and on the back. He also struck him with a one-meter
forthwith forwarded to the Office of the President of the Philippines for possible exercise of the long pipe. Not satisfied, Jaime, Sr. fired his gun hitting the right thigh of Diosdado. Luz was so
pardoning power. shocked by the sudden turn of events. To silence her one year old baby, she breastfed her. As
soon as she could, Luz fled to the rice paddies where she hid for a time. The Castillanos fled
on board a jeep parked in the NIA road about 200 meters from the house of Diosdado. When
Luz returned to their house, she saw her husband sprawled on the ground in a pool of his own
SO ORDERED. blood. Diosdado, at the point of death, asked her for help. Not knowing what to do, Luz lost
no time and ran to the house of their neighbor Celedonio Espiritu for help. Celedonio rushed to
the Bula Police Station and reported the incident.

* A team composed of SPO4 Jaime Javier, SPO3 Jaime Bellano and SPO3 Nilo Fornillos,8 the
Davide, Jr., C.J., Bellosillo, Puno, Vitug, Mendoza, Panganiban, Quisumbing, Sandoval-
duty investigator,9 went to the crime scene10 to conduct an on-the-spot investigation.
Gutierrez, Carpio, Austria-Martinez, Corona, Carpio-Morales, Callejo, Sr., and Azcuna, JJ.,
Photographs were taken of the cadaver.11 SPO3 Fornillo drew rough sketch12 of the scene.
concur.
The policemen saw a bolo at the place where Diosdado was sprawled near the door of their
house. A scabbard of a bolo was found a meter away from the house of Diosdado.13 The
Ynares-Santiago, J., on leave. policemen also found a bullet hole on the wall of the house.14 Thereafter, the cadaver was
placed on a hamak [hammock] brought to the police station. The police investigators turned
over the scabbard and bolo to the desk officer of the police station.15

> Impeachment * From the police station, SPO4 Javier, SPO3 Bellano and Sgt. Rogelio Palacio boarded their
mobile police car and set out a manhunt for the malefactors. They proceeded towards the
boundary in Sto. Domingo where they put up a checkpoint. The police officers inspected every
SECOND DIVISION vehicle that passed by. At around 12:45 a.m., SPO4 Javier halted a passenger jeepney. On
board were Jaime, Sr. and his two sons, Jaime Jr. and Ronald, each of whom carried a bag
G.R. No. 139412 April 2, 2003 containing their clothes. The policemen brought the Castillanos to the police station.16 The
bags of Jaime, Jr. and Ronald were turned over to the police investigators. The three were
THE PEOPLE OF THE PHILIPPINES, appellee, placed under arrest for the killing of Diosdado. The policemen submitted their investigation
report.17
vs. JAIME CASTILLANO, SR. alias "Talino," RONALD CASTILLANO alias "Nono" and
JAIME CASTILLANO, JR. alias "Junjun," accused, * In the meantime, at 7:00 a.m., Dr. Evangeline Consolacion, the Municipal Health Officer of
Bula, conducted an autopsy on the cadaver of Diosdado. Her autopsy report revealed the
RONALD CASTILLANO alias "Nono" and JAIME CASTILLANO, JR. alias "Junjun," following findings:
appellants.
External Findings
CALLEJO, SR., J.:
1. Incise Wound 3 cm Superior pinna R ear

2. Incise woud (sic) 10 cm. from nasal bridge extending to mandible R

Facts
3. Stab wound 2 cm.x 5 cm. Epigastrium R

347
RECTO, GAYLE ANGELI M.
2011-0008 | AUSL
Personal Notes on Remedial Law 2 Review (based on the syllabus of Prof. Henedino M. Brondial)

4. Stab wound 2 cm.x 4 cm. Epigastrium L * Ronald admitted when he testified that he killed Diosdado but insisted that he did so in self-
defense and in defense of his brother Jaime, Jr. He asserted that his father Jaime, Sr. and
5. Stab wound 2.5 cm. Middle third Arm R brother Jaime, Jr. had nothing to do with Diosdado’s death. Ronald alleged that on September
8, 1996, at about 7:30 p.m., he was driving a passenger jeepney on his way to the poblacion
of Bula. Jaime, Jr. flagged down the jeepney. He boarded the jeepney and told Ronald that he
6. Stab wound 2cm x 5 cm. posterior Back.
was instructed by their mother to go to the house of Jose del Socorro to ask the latter to
accompany them to their farmhouse in order to fetch Gilda Albes. Ronald was armed with a
7. Amputating middle third finger L
.38 paltik gun, while Jaime, Jr. was armed with a bolo sheathed in a scabbard. They fetched
Jose and then Ronald parked the jeepney at the NIA road. Jaime, Jr., who was holding a
8. Hacked wound posterior ankle L flashlight, walked along the footpath on top of a pilapil (a narrow earthen barrier between two
rice fields). Ronald and Jose walked behind Jaime, Jr. As they passed by the house of
9. Gunshot wound POE 2 x 2cm. with contusion collar medial aspect middle third Diosdado, a man suddenly shouted: "you shit, I have await (sic) for you for a while, why just
R thigh now." Surprised, Jaime, Jr. forthwith focused his flashlight towards the man who shouted. He
was aghast when he saw Diosdado armed with a bolo running towards them and about to
attack them with his bolo. Ronald shoved Jaime, Jr. who fell on the muddy rice paddies below
the pilapil. Ronald forthwith shot Diosdado. Diosdado took a step but fell on a kneeling
position. Diosdado brandished his bolo. Ronald shot Diosdado once more but his gun misfired.
No point of exit noted
To defend himself, Ronald took Jaime, Jr.’s bolo and hacked Diosdado to death.32 Ronald then
fled from the scene and ran to the jeepney at the NIA road. Jaime, Jr. and Jose boarded the
jeep and left the scene. Ronald threw the bolo along the way. He threw his gun into a rice
farm in Danawan.
Internal Findings:
* Jaime, Jr. corroborated the testimony of his brother. He, however, testified that
Fracture femur with Foreign body bullet lodge in middle third femur with he did not see his brother hack and kill Diosdado. He claimed that when Ronald
hematoma about about 100 cc R thigh got hold of his bolo, he ran away and proceeded to their jeepney which was then
parked at the roadside. Minutes later, Ronald followed. They then hastily went
home to Sagrada and told their father Jaime, Sr. of the incident.33

Cause of Death; Hypovolemia secondary to Multiple Stab Wound18 * Jose Del Socorro corroborated the testimony of Ronald. He testified that on
July 8, 1996, at about 5:00 p.m. he was on his way home when he met Diosdado
* The doctor recovered a slug from the right thigh of Diosdado. She later signed the victim’s whom he noticed to be inebriated and unruly Diosdado was throwing dried mud
post-mortem certificate of death.19 Senior Inspector Edgardo B. Sambo, Chief of Police of Bula at the farmhouse of the Castillanos and challenging the occupants of the
Police Station, filed with the Municipal Trial Court of Bula, Camarines Sur, a criminal farmhouse to a fight. He advised Diosdado to stop what he was doing and
complaint20 for murder against the Castillano brothers.21 Judge Francisco O. Tolentino warned him that he was only inviting trouble. Diosdado told him to mind his own
conducted the preliminary examination and thereafter issued an order of arrest against the business and not to intervene. Jose thereafter left Diosdado and went, home.34
Castillanos.22 No bail was recommended for their provisional release. On July 9, 1996, Luz When Jose arrived home, Dominador Briña was waiting for him. He and
gave a sworn statement to the police investigators.23 Dominador talked business for a while and subsequently had dinner. After some
time, Jaime, Jr. and Ronald arrived at Jose’s house.

* On July 10, 1996, the accused were transferred to the Tinangis Penal Farm. Senior Inspector
Sambo requested the PNP-CLRU5 Provincial Unit to conduct a paraffin test on the * Concepcion Castillano testified that on July 8, 1996 at around 5:00 a.m., her
Castillanos.24 son Jaime, Jr. arrived home and told her that Diosdado threw stones at their
farmhouse and challenged everybody to a fight. She felt nervous and reported
the incident to the police and caused the same to be entered in the police
* On July 12, 1997, Major Lorlie Arroyo, the Head Forensic Chemist of PNP-Region 5,
blotter.35 Thereafter, she went home and told her sons Jaime, Jr. and Ronald to
conducted the paraffin test on the Castillanos. Ronald was found positive for gunpowder
immediately fetch Gilda. She, likewise, instructed her sons to first drop by the
residue.25 Jaime, Sr. and Jaime, Jr. were found negative for gunpowder residue.
house of Jose so that the latter could accompany them to the farmhouse.

* The MTC issued a subpoena requiring the accused to submit their counter-affidavits from
* Jaime, Sr. vehemently denied any participation in the killing of Diosdado. He
notice thereof. However, the accused failed to submit any counter-affidavit.26
claimed that at the time of the alleged incident, he was at their house in
Sagrada, bedridden due to his debilitating diabetes. He narrated to the trial court
* On August 2, 1996, an Information for murder was filed against Jaime, Sr., Ronald and his medical history and his confinement at the Mandaluyong Medical Center
Jaime Jr. with the Regional Trial Court of Pili, Camarines Sur, Branch 31. The accusatory sometime in 1994.36 He presented documents and receipts showing that he had
portion of the Information reads: been and is still under medication.37 He declared that upon learning from his son
Ronald that the latter killed Diosdado, he advised his sons to look for a lawyer for
That on or about the 8th day of July 1996 at about 8:00 o’clock in the evening at legal representation. He told the trial court that at around 11:30 p.m., he and his
Barangay Sagrada, Municipality of Bula, Province of Camarines Sur, Philippines, two sons had decided to go to Andangnan in order to meet a cousin of his who
and within the jurisdiction of this Honorable Court, the above-named accused knew of a lawyer named Atty. Rotor. As they traversed the road to Andangan,
conspiring, confederating and mutually helping one another with intent to kill they were stopped by some policemen at a checkpoint and were invited to the
with treachery and evident premeditation armed with a handgun, bladed weapon police station where they were investigated and eventually incarcerated.38
and piece of wood did then and there wilfully, unlawfully and feloniously attack,
assault and shot and stab one DIOSDADO VOLANTE y LOZANO inflicting upon * Gilda Abes, the last witness for the defense, affirmed that she was the
the latter several mortal wounds on the different parts of his body which caused girlfriend of Jaime, Jr. She told the trial court that on July 8, 1996 she was at the
his instantaneous death, to the damage and prejudice of the heirs of said farmhouse of the Castillanos. She corroborated the testimony of Jose that
Diosdado Volante the amount of which to be proven in Court. Diosdado was combative and drunk. According to Gilda, Jaime, Jr. left the
farmhouse before sundown to go to his parent’s place at Sagrada. Jaime, Jr.
ACTS CONTRARY TO LAW.27 never returned to the farmhouse that night. Gilda learned of the incident the
next morning when she went home.39
* Upon their arraignment28 on August 29, 1996, accused Jaime Sr., Jaime, Jr and Ronald,
duly assisted by counsel de parte, Atty. Avelino Sales Jr., pleaded not guilty to the offense * On December 22, 1998, the trial court rendered a decision convicting Jaime, Jr. and Ronald
charged. Thereafter, trial on the merits ensued. of murder qualified by evident premeditation and treachery. The trial court exonerated Jaime,
Sr. of the crime on reasonable doubt. The trial court gave no credence to Ronald’s claim that
* Luz testified that when Diosdado was still alive, he had an annual income of he acted in self-defense. The decretal portion of the decision reads:
over P65,000. She spent P18,000 for the funeral services,29 P300 for religious
services,30 P9,111 for food and other expenses31 during her husband’s wake WHEREFORE, in view of all the foregoing, judgment is hereby rendered, finding
and funeral. She suffered sleepless nights and mental anguish for his sudden the two (2) accused RONALD CASTILLANO and JAIME CASTILLANO, JR. guilty
death. beyond reasonable doubt of the offense of MURDER and they are hereby
sentenced to suffer the penalty of imprisonment of RECLUSION PERPETUA with
348
RECTO, GAYLE ANGELI M.
2011-0008 | AUSL
Personal Notes on Remedial Law 2 Review (based on the syllabus of Prof. Henedino M. Brondial)

all the accessory penalties imposed thereby. Further, as civil liability, the said two Q Danawan, is that a lake?
(2) accused are hereby ordered to pay the legal heirs of the late Diosdado L.
Volante, through his widow Luz R. Volante, the total sum of ONE HUNDRED A No, Your Honor, it is a ricefarm.
SEVENTY-SEVEN THOUSAND FOUR HUNDRED TWENTY ONE PESOS
(P177,421.00) Philippine Currency as actual and moral damages including death
indemnity, with costs against both accused.

Q What kind of gun is this?


The accused Jaime Castillano, Sr. is hereby acquitted on the ground of
reasonable doubt.
A Paltik .38, Your Honor.45

SO ORDERED.40

* The accused, now appellants, interposed their appeal from the decision of the trial court.
ATTY. BALLEBAR:

Issues
Q By the way, where is that bolo that you used in hacking and stabbing Diosdado
Volante?
(a) in rejecting appellant Ronald’s plea of self-defense; and (b) in not acquitting appellant
Jaime, Jr. of the crime charged for failure of the prosecution to prove his guilt beyond
reasonable doubt. A I do not know anymore because I was able to throw it away also when I ran away.

Q Where is that place where you throw it?


Ruling

A It was by the NIA road.


Anent the first issue, appellant Ronald posits that he adduced proof that he acted in self-
defense when he stabbed the victim.

Q You mentioned also a while ago that this gun that you said is a "paltik" and you throw it
away also, is it not?
The Court disagrees with appellant Ronald. The Court has consistently held that like alibi, self-
defense is inherently weak because it is easy to fabricate.41 In a case where self-defense and
A Yes, Ma’am.
defense of relatives is invoked by the accused, the burden of evidence is shifted to him to
prove with clear and convincing evidence the essential requisites of self-defense, namely (a)
unlawful aggression on the part of the victim; (b) reasonable necessity of the means employed
to repel or prevent it; and (c) lack of sufficient provocation on the part of the person
defending himself. There can be no complete or incomplete self-defense or defense of Q And that gun had been in your possession the whole day that you are driving up to the
relatives unless the accused proves unlawful aggression on the part of the victim.42 The time you shot the victim, Diosdado Volante?
accused must rely on the strength of his evidence and not on the weakness of the evidence of
the prosecution for by pleading self-defense, the accused thereby admits having killed the
A Yes, Ma’am.
victim and he can no longer be exonerated of the crime charged if he fails to prove the
confluence of the essential requisites for self-defense and defense of a relative.43

Q Do you have license to possess that firearm?

Appellant Ronald failed to discharge his burden.


A None, Ma’am.46

First. After shooting and stabbing Diosdado, appellant Ronald fled from the situs criminis.
Flight from the situs of the crime is a veritable badge of guilt and negates his plea of self- The failure of appellant Ronald to surrender the bolo and his gun to the police authorities
defense.44 belies his claim of self-defense.

Second. Appellant Ronald threw away his paltik .38 gun and the bolo he used in hacking Third. Appellant Ronald failed to report the incident to the police authorities even when they
Diosdado as he fled from the scene of the crime instead of surrendering the same to the police arrested him. Curiously, he failed to inform the police officers who arrested him that he acted
authorities. Appellant Ronald admitted that he had no license for the gun: in self-defense when he shot and stabbed the victim The resounding silence of the appellant is
another indicium of the incredibility of his defense.47 Moreover, the records show that the
municipal trial court issued a subpoena on July 9, 1996 requiring appellant Ronald to submit
his counter-affidavit but he refused and/or failed to submit the same despite service on him of
the subpoena. It was only during the trial that appellant Ronald, for the first time, invoked
Q Where is that gun now that you use?
self-defense and defense of a relative.

A I do not know, Your Honor, I think I was able to throw it away.

Fourth. The cadaver of the victim was found inside his house when the police investigators
arrived.48 This belies appellant Ronald’s claim that he shot the victim in the rice paddies, near
Q Where? his house and that he (appellant Ronald) took the bolo of appellant Jaime, Jr. and used it to
stab the victim. Appellant Ronald failed to prove his claim that when the police investigators
A At Danawan, Your Honor. arrived in the victim’s house, they carried his (the victim’s) body from the rice paddies to the
house. The only evidence adduced by appellant Ronald was his testimony which is hearsay,
and besides being hearsay, it is speculative and mere conjecture.

349
RECTO, GAYLE ANGELI M.
2011-0008 | AUSL
Personal Notes on Remedial Law 2 Review (based on the syllabus of Prof. Henedino M. Brondial)

Fifth. Appellant Ronald hacked the victim no less than five times. Two of the stab wounds Surrounding house, dark, total darkness (Tsn pp. 12-13, 4/1/97).
sustained by the victim were at his back and posterior portion of his left ankle. The number
and nature of the wounds of the victim negate the appellant’s claim that he shot the victim in
self-defense. On the contrary, they prove that appellant Ronald was determined to kill the
victim.49
8. Scene Photography by Jaime Jornales (Tsn, p. 21, 2/17/97).

Appellant Jaime, Jr. avers that the prosecution failed to prove his guilt beyond reasonable
-do- by Mr. Lozano (Tsn., p. 12, 3/7/97).
doubt of the crime charged. He asserts that the testimony of Luz Volante, the widow of
Diosdado, was inconsistent with her testimony during the preliminary examination in the
municipal trial court and her sworn statement before the police investigators as well as the
testimonies of SPO1 Fornillos and SPO4 Jaime Favier, and the physical evidence on record.
9. SPO1 Nilo Pornillos learned of the incident at 8:00 o’clock of July 8, 1996 (page 5 of
The appellant catalogued said inconsistencies, thus:
Complainant’s Memorandum.

1. He was lying on the bench inside just upon entering. (Tsn p. 9, 2/17/97).
SPO4 Jaime Javier received report at 9:00 o’clock P.M. of July 8, 1996 of Complainant’s
Memorandum.

- I was lying down with my husband inside our house but we were still awake (9th
Answer, Prel. Exam. MTC, 7/9/96).
SPO4 Jaime Javier received report at 8:00 P.M. (page 7 of Complainant’s
Memorandum).50

2. JCS fired towards our house hitting the wall (Tsn p. 11, 2/17/96).

On the other hand, the Office of the Solicitor General asserts that the credibility of
the testimony of Luz, the prosecution’s principal witness, cannot be impeached via
her testimony during the preliminary examination before the municipal trial court
JCS fired twice (16th answer, Prel. Exam. MTC, 7/9/96).
nor by her sworn statement given to the police investigators for the reason that
the transcripts and sworn statement were neither marked and offered in evidence
by the appellants nor admitted in evidence by the trial court. Moreover, the
appellants did not confront Luz with her testimony during the preliminary
JCS kept on firing the gun pointing towards the body of my husband (9th Answer, examination and her sworn statement to the police investigators. Luz was not,
Sworn Statement, PNP, 7/9/96). therefore, accorded a chance to explain the purported inconsistencies, as
mandated by Section 13, Rule 132 of the Revised Rules of Evidence which reads:

JCS shot my husband three (3) times (Tsn p. 16, 2/17/97)


How witness is impeached by evidence of inconsistent statement. - Before a
witness can be impeached by evidence that he has made at other times statements
inconsistent with his present testimony, the statements must be related to him,
with the circumstances of the times and places and the persons present, and he
3. My husband was shot and hit on the right thigh (Tsn p. 14, 2/17/97). He was hit on the
must be asked whether he made such statements, and if so, allowed to explain
left lap (23rd Answer, Prel. Exam. MTC, 7/9/96). He was hit on his side (Tsn p. 43, 2/17/97).
them. If the statements be in writing they must be shown to the witness before
any question is put to him concerning them.

4. RC struck my husband with a 1-meter long Pipe (Tsn p. 13, 2/17/97). RC & JCJ smashed
my husband with a hard object (5th Answer, Sworn Statement, 7/9/96).
The Court agrees with the Office of the Solicitor General. Before the credibility of a
witness and the truthfulness of his testimony can be impeached by evidence
consisting of his prior statements which are inconsistent with his present
testimony, the cross-examiner must lay the predicate or the foundation for
RC smashed my husband (22nd Answer, Prel. Exam. MTC, 7/9/96). impeachment and thereby prevent an injustice to the witness being cross-
examined. The witness must be given a chance to recollect and to explain the
apparent inconsistency between his two statements and state the circumstances
under which they were made.51 This Court held in People v. Escosura52 that the
5. He was not able to fight back (Tsn p. 43, 2/17/97). He was standing and was trying to statements of a witness prior to her present testimony cannot serve as basis for
parry the attack of the accused (26th Answer, Prel. Exam. 7/9/96). impeaching her credibility unless her attention was directed to the inconsistencies
or discrepancies and she was given an opportunity to explain said inconsistencies.
In a case where the cross-examiner tries to impeach the credibility and
truthfulness of a witness via her testimony during a preliminary examination, this
Court outlined the procedure in United States vs. Baluyot,53 thus:
6. When I went back to the house, he was still alive (Tsn p. 19, 2/17/97). - LV "Yes," the
victim could have died instantly (Tsn p. 35, 2/3/97) With wounds sustained, he could have
died instantly (p. 8, Complainant’s Memorandum).

...For instance, if the attorney for the accused had information that a certain
witness, say Pedro Gonzales, had made and signed a sworn statement before the
fiscal materially different from that given in his testimony before the court, it was
7. It was bright inside our house with a kerosene and a bottle lamp (Tsn pp. 33-34, incumbent upon the attorney when cross-examining said witness to direct his
2/17/97). Only one kerosene lamp - bottle of gin with wick and light (Tsn p. 10, 4/1/97 - SPO1 attention to the discrepancy and to ask him if he did not make such and such
Pornillos
350
RECTO, GAYLE ANGELI M.
2011-0008 | AUSL
Personal Notes on Remedial Law 2 Review (based on the syllabus of Prof. Henedino M. Brondial)

statement before the fiscal or if he did not there make a statement different from The Court fully agrees with the foregoing ruminations of the Office of the Solicitor General.
that delivered in court. If the witness admits the making of such contradictory The inconsistencies adverted to by the appellants pertained only to minor and collateral
statement, the accused has the benefit of the admission, while the witness has the matters and not to the elements of the crime charged; hence, they do not dilute the probative
opportunity to explain the discrepancy, if he can. On the other hand, if the witness weight of the testimony. It bears stressing that even the most truthful witness can make
denies making any such contradictory statement, the accused has the right to mistakes but such innocent lapses do not necessarily affect his credibility. The testimonies of
prove that the witness did make such statement; and if the fiscal should refuse witnesses must be considered and calibrated in their entirety and not by their truncated
upon due notice to produce the document, secondary evidence of the contents portions or isolated passages.56 And then again, minor contradictions among several
thereof would be admissible. This process of cross-examining a witness upon the witnesses of a particular incident and aspect thereof which do not relate to the gravamen of
point of prior contradictory statements is called in the practice of the American the crime charged are to be expected in view of their differences in impressions, memory,
courts "laying a predicate" for the introduction of contradictory statements. It is vantage points and other related factors.57
almost universally accepted that unless a ground is thus laid upon cross-
examination, evidence of contradictory statements are not admissible to impeach a
witness; though undoubtedly the matter is to a large extent in the discretion of the
court.
Contrary to appellant Jaime, Jr.’s claim, the prosecution adduced proof that he and appellant
Ronald conspired to kill and did kill Diosdado by their simultaneous acts of stabbing the victim.
As narrated by Luz:

In this case, the appellants never confronted Luz with her testimony during the
preliminary examination and her sworn statement. She was not afforded any
chance to explain any discrepancies between her present testimony and her
ATTY. BALLEBAR:
testimony during the preliminary examination and her sworn statement. The
appellants did not even mark and offer in evidence the said transcript and sworn
statement for the specific purpose of impeaching her credibility and her present Q Now after Jaime Castillano Sr. fired at your house, what happened next if any?
testimony. Unless so marked and offered in evidence and accepted by the trial
A They entered our house.
court, said transcript and sworn statement cannot be considered by the court.54

Q Now, when you say they to whom are you referring to?
On the purported inconsistencies or discrepancies catalogued by the appellants relating to the
testimony of Luz during the preliminary examination and her sworn statement, the Office of
the Solicitor General posits that: A Jaime Castillano Sr., Jaime Castillano, Jr., and Ronald Castillano.

… Q Now, where did they enter?

A In the other door.

Sixth, Volante indeed testified that when she returned to their house from the ricefield, after
the three accused had left the premises, her husband was still alive (TSN, February 17, 1997,
p. 19) as he was still able to ask for her assistance (Ibid, p. 20). But it is not inconsistent with Q Now at the time they entered your house was the door of your house closed or
the expert opinion of Dr. Consolacion that by the nature of the wounds sustained by the opened?
victim, the latter could have died thereof instantaneously (TSN, February 3, 1997, p. 35). It is
clear that the said physician was merely stating a possibility and not what happened in the
A It was closed.
instant case because in the first place, she was not present at the scene right after the
incident.

Q Now, after the accused entered your house what happened next, if any?

Seventh, Volante was insistent in her testimony that at the time of the commission of the
subject crime, it was bright inside their house because they had a "kerosene lamp" and a A Jaime Castillano Jr. stabbed my husband and also Ronal Castillano stabbed my
"bottle lamp" both lighted up, one placed on the wall and the other on the ceiling (Ibid, pp. husband.
33, 52-53). While it may appear contradictory to SPO1 Pornillos’ testimony that there was only
a kerosene lamp at the time, he could not have been expected to notice all the things found
inside the house, including the "bottle lamp", because he might not have been familiar with its
interiors. Or, he could have focused his attention primarily on the body of the fallen victim and Q Now, was your husband hit by the stabbing of Ronald Castillano, Jr. (sic)?
the objects that may be used later as evidence against the perpetrators of the crime.
A Yes, sir.

Eight, it is admitted that the testimonies of Volante and SPO1 Pornillos as to who took pictures
of the crime scene including the lifeless body of the victim are contradictory. But again, such Q Will you tell us on what part of his body was he hit?
contradiction, being only minor and irrelevant, does not affect the credibility of their
testimonies.
A My husband was still struck by Ronald Castillano hitting him on his right side of his
body including on his right thigh and also on his back..

And ninth, the apparently inconsistent statements of the prosecution witnesses (SPO1 Pornillos
and SPO4 Javier) as to the exact time the subject incident was reported to the police
ATTY. BALLEBAR:
authorities are similarly irrelevant to the matters in issue. Of consequence here is the fact that
on the night the crime was committed, it was reported to the authorities who later effected
the arrest of the perpetrators thereof.55 Q Now, you said Ronald Castillano struck your husband, now with what instrument did
he use in strucking (sic) your husband?

351
RECTO, GAYLE ANGELI M.
2011-0008 | AUSL
Personal Notes on Remedial Law 2 Review (based on the syllabus of Prof. Henedino M. Brondial)

ATTY. BERNALES:

We object, misleading, your Honor. ATTY. BALLEBAR:

Q By the way, will you tell us how many times did Ronald Castillano stab your husband?

COURT: A I cannot determine how many times he even stabbed my husband on his left eye.

Witness may answer.

Q How about Jaime Castillano Jr., how many times did he stab your husband?

WITNESS: A I cannot determine exactly how many times but he repeatedly stabbed my husband.59

A A pipe.

The mere denial appellant Jaime, Jr. of the crime charged is but a negative self-serving which
cannot prevail over the positive and straightforward testimony of Luz and the physical
ATTY. BALLEBAR: evidence on record.60

Q Now, will you tell us more or less how long was that pipe that was used by Ronald
Castillano?
The Crime Committed by Appellants
A About one (1) meter, Maam.58

The trial court correctly convicted the appellants of murder, qualified by treachery, under
Luz was merely five meters away from where Diosado was attacked and stabbed by the Article 248 of the Revised Penal Code. The Court, however, does not agree with the trial
appellants. Appellant Jaime, Jr. even tried to cut the ankle of the victim: court’s finding that evident premeditation attended the commission of the crime.

ATTY. BALLEBAR: Case law has it that the prosecution has the burden to prove beyond reasonable doubt
qualifying circumstances in the commission of the crime. For evident premeditation to qualify a
crime, the prosecution must prove the confluence of the essential requites thereof: (a) the
Q Now during this incident, how far were you from the accused and your husband?
time when the offender has determined to commit the crime; (b) an act manifestly indicating
that the offender has clung to his determination; (c) an interval of time between the
A From where I am sitting up to that window which is about five (5) meters. determination and the execution of the crime enough to allow him to reflect upon the
consequences of his act.61 There must be proof beyond cavil when and how the offender
planned to kill the victim and that sufficient time had elapsed between the time he had
decided to kill the victim and the actual killing of the victim, and that in the interim, the
Q Now after the accused strucked (sic) and shot your husband, what else happened if offender performed overt acts positively and conclusively showing his determination to commit
any? the said crime.62 In this case, the only evidence adduced by the prosecution to prove evident
premeditation is the testimony of Levy Avila that between 5:00 p.m. and 6:00 p.m. on July 8,
A Jaime Castillano Jr. stabbed my husband on his breast (Witness is pointing to her 1996, he heard the appellants planning to go to the house of Diosdado and that he heard
breast). them say: "Ayaw namin kasing inaasar," and that at 8:00 p.m., the appellants arrived in the
house of the victim and stabbed him to death. There is no evidence of any overt acts of the
appellants when they decided to kill Diosdado and how they would consummate the crime.
There is no evidence of any overt acts perpetrated by the appellants between 5:00 and 8:00
p.m. that they clung to their determination to kill Diosdado.
ATTY. BERNALES:

We will move that the answer be striken off from the records because it is not responsive to
the question. The question is after your husband has been stabbed strucked (sic) and shot.
There is treachery in the commission of a crime when (a) at the time of attack, the victim was
not in a position to defend himself; (b) the offender consciously and deliberately adopted the
particular means, methods and forms of attack employed by him.63 Even a frontal attack may
be treacherous when unexpected on an unarmed victim who would not be in a position to
COURT: repel the attack or avoid it.64 In this case, the victim was unarmed and was supinely resting
before sleeping after a hard day’s work. Although Luz warned the victim that the appellants
Q Your are being asked what happened after the accused was already stabbed, strucked were already approaching their house, however, the victim remained unperturbed when the
(sic) and shot, what happened next? appellants barged into the victim’s house. They stabbed him repeatedly with diverse deadly
weapons. The victim had nary a chance to defend himself and avoid the fatal thrusts of the
appellants.

WITNESS:

The crime was committed in the house of the victim. There was no provocation on the part of
Q Jaime Castillano Junior still stabbed my husband and try to cut his ankle, Your Honor.
the victim. Dwelling thus aggravated the crime. However, dwelling was not alleged in the
information, as mandated by Section 8, Rule 110 of the Revised Rules of Criminal Procedure:

COURT:

Sec. 8. Designation of the offense. - The complaint or information shall state the designation
Strike our (sic) the previous answer of the witness.
of the offense given by the statute, aver the acts or omissions constituting the offense, and

352
RECTO, GAYLE ANGELI M.
2011-0008 | AUSL
Personal Notes on Remedial Law 2 Review (based on the syllabus of Prof. Henedino M. Brondial)

specify its qualifying and aggravating circumstances. If there is no designation of the offense,
reference shall be made to the section or subsection of the statute punishing it.
Bellosillo, (Chairman), Mendoza, Quisumbing, and Austria-Martinez, JJ., concur.

The use by appellant Ronald of an unlicensed firearm to shoot Diosdado on the thigh is not an
aggravating circumstance because (1) there is no allegation in the information that said
> Reference to memorandum
appellant had no license to possess the firearm. That appellant lacked the license to possess
the firearm is an essential element of the crime and must be alleged in the information.65
Although the crime was committed before the new rule took effect on December 1, 2002, the THIRD DIVISION
rule should, however, be applied retroactively as it is favorable to the appellants.66
G.R. No. 90198 November 7, 1995

PEOPLE OF THE PHILIPPINES, plaintiff-appellee,


The appellants are not entitled to the mitigating circumstance of voluntary surrender. The
evidence shows that the appellants were arrested when the police officers manning the vs. ANTONIO PLASENCIA y DESAMPARADO alias "Tonying," ROBERTO DESCARTIN
checkpoint stopped the passenger jeepney driven by appellant Ronald and arrested the y PASICARAN alias "Ruby" and JOELITO (JULITO), DESCARTIN y PASICARAN,
appellants. The fact that the appellants did not resist but went peacefully with the peace accused-appellants.
officers does not mean that they surrendered voluntarily.67

VITUG, J.:

* Antonio Plasencia, Roberto Descartin and Joelito (Julito) Descartin were accused of robbery
There being no mitigating and aggravating circumstances in the commission of the crime, the with homicide in an information, dated 20 December 1984, that read:
appellants should be meted the penalty of reclusion perpetua conformably with Article 63 of
the Revised Penal Code.
That on or about the 29th day of November, 1984 at around 3:00 o'clock in the
afternoon, more or less, in sitio San Juan, Barangay Patao, Municipality of
Bantayan, Province of Cebu, Philippines, and within the jurisdiction of this
Honorable Court, the said accused conspiring and confederating together and
The Civil Liabilities of the Appellants mutually helping one another, did then and there wilfully, unlawfully and
feloniously, and with treachery, evident premeditation and taking advantage of
their superior number and strength and with intent to kill, treacherously attack,
assault and use personal violence upon Herminio Mansueto, thereby inflicting
upon him the following physical injuries:
The trial court awarded the total amount of P177,421 as civil indemnity, actual and moral
damages in favor of the heirs of the victim Diosdado. The Court has to modify the awards.
1. Stab wounds which was approximately two inches in length,
parallel to the ribs and is located 1 1/2 inches below the right nipple
on the right anterior axillary line and on the fifth intercostal space.
On probing the wound was penetrating immediately up to the left
Appellants Ronald and Jaime, Jr. are obliged to pay jointly and severally the amount of
parasternal border approximately hitting the heart;
P50,000 as civil indemnity; P50,000 as moral damages; P25,000 as exemplary damages in
view of the aggravating circumstance of dwelling;68 and the amount of P18,300 for funeral
2. Hacking wound 9 inches in length extending from the coracoid
and religious services. The heirs of the victim failed to adduce in evidence any receipts or
process of the left clavicle passing between the left anterior and the
documentary evidence to prove their claim for food and other expenses during the wake.
left mid axillary line up to the left 4th intercostal space including all
However, they are entitled to temperate damages in the amount of P5,000, conformably with
muscle underlying the skin exposing the ribs.
the ruling of the Court in People v. dela Tongga.69 His wife Luz’s testimony that the victim had
an annual income of more than P65,000 is not sufficient as basis for an award for unearned
income for being self-serving. There was no proof of the average expense of the victim and his Cause of death: Internal hemorrhage due to stab wound.
family and his net income. In People v. Ereño,70 this Court held that:
after which the body was placed inside a plastic bag and brought to an open sea
by the pump boat owned by Roberto Descartin y Pasicaran and operated by
Joelito Descartin y Pasicaran and dumped to the water by herein accused, and as
a result of which said Herminio Mansueto died, herein accused, in pursuance of
… It bears stress that compensation for lost income is in the nature of damages and as such
their conspiracy, wilfully, unlawfully and feloniously and with intent to gain, took
requires due proof of the damage suffered; there must be unbiased proof of the deceased’s
and carried away the personal property belonging to Herminio Mansueto,
average income. In the instant case, the victim’s mother, Lita Honrubia, gave only a self-
namely: one (1) Seiko 5 "Stop Watch" valued at P3,000.00; one (1) Bicycle
serving hence unreliable statement of her deceased daughter’s income. Moreover, the award
(standard size) valued at P1,000.00; and cash in the amount of P10,000.00, all in
for lost income refers to the net income of the deceased, that is, her total income less her
the total amount of FOUR-TEEN THOUSAND PESOS (P14,000.00), Philippine
average expenses. No proof of the victim’s average expenses was presented. Hence, there can
Currency, to the damage and prejudice of said oner (sic) in the said total sum.
be no reliable estimate of the deceased’s lost income.

All contrary to law, and with the qualifying circumstance of alevosia, and the
generic aggravating circumstance of known premeditation.

IN LIGHT OF ALL THE FOREGOING, the Decision of the Regional Trial Court of Camarines Sur,
CONTRARY TO LAW. 1
Branch 31 in Criminal Case No. P-2542 is AFFIRMED with MODIFICATION. Appellants Ronald
Castillano alias "Nono" and Jaime Castillano, Jr. alias "Junjun" are found guilty beyond
reasonable doubt of murder, qualified by treachery, punishable by reclusion perpetua to death, * When arraigned, all the accused entered a plea of "not guilty" to the charge; whereupon,
under Article 248 of the Revised Penal Code. There being no modifying circumstances in the trial commenced.
commission of the crime, the appellants are sentenced to suffer the penalty of reclusion
perpetua, conformably with Article 63 of the Revised Penal Code. They are, likewise, ordered * The prosecution sought to establish, as follows:
to pay jointly and severally to the heirs of the victim, Diosdado Volante, the amounts of
P50,000 as civil indemnity; P50,000 as moral damages; P18,300 as actual damages; P25,000 * At around ten o'clock in the morning of 29 November 1984, Herminio
as exemplary damages; and P5,000 as temperate damages. Costs against the appellants. Mansueto, wearing a blue and white striped t-shirt, maong pants, Seiko 5 stop
watch and a pandan hat, left on his bicycle for Barangay Patao, Bantayan, Cebu.
He had with him P10,000.00 cash which he would use to purchase hogs from a
certain "Ruby."
SO ORDERED.

353
RECTO, GAYLE ANGELI M.
2011-0008 | AUSL
Personal Notes on Remedial Law 2 Review (based on the syllabus of Prof. Henedino M. Brondial)

* In Patao, Francisca Espina, also known in the locality as Pansing and whose about an hour's walk from his residence, at the house of his fiancee. He returned
house was just across the street from the respective residences of the three to his house, he said, only the day after. Roberto ("Ruby"), Joelito's uncle,
accused, saw at the roadside Herminio Mansueto and Roberto Descartin alias testified that on that fateful day, he was in Samoco Purok 2, Iligan City, and then
"Ruby" engaged in conversation. Pansing approached them and asked Mansueto left for Cebu on 06 December 1984 only after receiving a telegraph that Joelito
if he would be interested in buying two of her pigs for P1,400.00. Mansueto said was implicated in the crime.
"yes" and promised that he would be right back.
* The Regional Trial Court 14 did not give credence to the defense of alibi. It convicted the
* Mansueto and Ruby meantime proceeded to the latter's piggery. Joelito three accused of murder (punishable under Article 248 of the Revised Penal Code), instead of
Descartin and his brother-in-law Rene were also seen going to the place. After robbery with homicide, explaining that the term "homicide" was used in the information in its
some time, Pansing noticed Joelito take Mansueto's bicycle. Believing that generic sense. 15 Finding conspiracy, the trial court ruled that the killing was qualified by both
Mansueto was already preparing to leave and in her desire to catch up with him, treachery and abuse of superior strength with the latter, however, being absorbed by the
Pansing promptly walked towards the piggery which was around 100 meters former. No other aggravating or mitigating circumstances being attendant in the commission
away from her house. She could see Mansueto leaning on the pigsty with Ruby of the crime, the trial court said, the penalty that could be imposed upon each of the accused
on his right side and Antonio Plasencia alias "Tonying" on his left; behind was was reclusion perpetua with a joint and several civil liability for indemnification to the heirs of
Joelito. 2 Midway, she was halted on her tracks; she suddenly saw Antonio stab Herminio Mansueto in the amount of P30,000.00.
Mansueto. The latter staggered towards Ruby who himself then delivered
another stab blow. Mansueto fell on his back. Joelito started hitting Mansueto on * The instant appeal was interposed by the three convicted appellants.
the forehead while Rene held Mansueto's legs. 3 Except for a coconut tree and
some ipil-ipil trees around the area, nothing obstructed Pansing's line of vision.
Pansing rushed back home. The image of Antonio waving the weapon and the
thought that she might herself be killed kept her from revealing to anyone what
she saw. 4 Ruling

* The following day, in Kodia, Madridejos, Cebu, where Mansueto resided, his Appellant Antonio Plasencia attacks the credibility of the prosecution's lone eyewitness,
daughter Rosalinda reported to Francisca Tayo, the barangay captain, that her Francisca Espina, alleging that she is a pejured witness who has an axe to grind against him
father had not returned home. Tayo proceeded to Putian, which was in because his dog had once bitten Francisca's child. 16 He bewails the fact that it has taken
Mansueto's itinerary, and then to Ruby's piggery in Patao, where a youngster, Francisca until 29 December 1984 to reveal what she supposedly has seen to the police
who turned out to be Ruby's son, innocently informed her that Mansueto's authorities. Contending that treachery has not been duly proven as "no wound was inflicted at
bicycle was taken by Joelito. 5 the back and as a matter of fact only one wound was fatal," 17 appellant argues that even if
conspiracy were to be considered to have attended the commission of the crime, he could be
held liable with the others, if at all, only for homicide.
* The day after, Francisca Tayo, accompanied by police officers of Madridejos,
Cebu, and some relatives of Mansueto, went back to Ruby's place. On a railing of
the pigpen, she saw blood stains. When she asked Ruby's father about it, he said
that the stains had come from chicken blood. Going around the piggery, she also
saw blood stains on a bamboo pole, which Ruby's father once again so identified Appellant Roberto Descartin, likewise challenging Francisca Espina's credibility because of her
as chicken blood. At the back of the piggery, Francisca noticed a digging which alleged inconsistencies, faults the trial court for allowing the witness to glance at the notes
looked like an empty grave. The digging was measured and photos were taken. written on her palm while testifying. He also argues that his alibi, being corroborated, should
The police found a hat at the back of a hut beside the piggery, which was later have been given weight.
recognized to be that which belonged to Mansueto. 6

* In the morning of 30 November 1984, Patrolman Elpidio Desquitado of the


Bantayan police went back to the piggery. This time, the police learned from Appellant Joelito Descartin, in assailing the credibility of Francisca, has noted her "jittery
Pansing herself that Joelito took Mansueto's bicycle. 7 Joelito was invited to the actuation" while giving her testimony. He also questions the findings of the ponente for not
police headquarters to shed light on the case. Later, Joelito, waiving his right to being the presiding judge during the examination of Francisca on the witness stand.
counsel, executed a "confession." 8

* Joelito narrated that, upon Ruby's instruction, he brought the bicycle to the
piggery. Unexpectedly, he said, Tonying Plasencia stabbed Mansueto. Stunned, The focus of this appeal is clearly one of credibility. The initial assessment on the testimony of
Joelito tried to run away but Tonying stopped him. Tonying then dragged the a witness is done by the trial court, and its findings still deserve due regard notwithstanding
victim to a nearby house. Threatened by Tonying, Joelito agreed to later return that the presiding judge who pens the decision is not the one who personally may have heard
to where the victim's body was dragged. At around eleven o'clock that evening, the testimony. 18 The reliance on the transcript of stenographic notes should not, for that
tonying and Joelito placed the body in a sack. Tonying asked Ruby to allow the reason alone, render the judgment subject to challenge. 19 The continuity of the court and the
use of the latter's pumpboat to ferry the body. Tonying paddled the pumpboat to efficacy of its decision are not affected by the cessation from the service of the judge presiding
the island of Po-Po'o where he picked up some pieces of stones. Then, again it 20 or by the fact that its writer merely took over from a colleague who presided at the trial.
paddling the pumpboat farther away from the island, he ordered Joelito to start 21
the engine of the boat. They headed for the islet of Gilotongin (Hilotongan). On
the way, Tonying filled the sack with stones and, using a rope, tied to it the body
of the victim. Tonying then unloaded their cargo into the sea.

It is asserted that the testimony of Francisca Espina should not be given worth
* Guided by Joelito, members of the Bantayan police force headed for the islet of
since, while testifying, she would at times be seen reading some notes written on
Hilotongan on two pumpboats 9 in the area pinpointed to be the place where the
her left palm. Thus —
body was dumped. On the second day of the search, the group was informed
that the body had already surfaced near the vicinity of the search and delivered
to the municipal building. 10

* The municipal health officer of Bantayan, Dr. Oscar Quirante, examined the Q. May I see your left hand, may I see what is written there?
body and concluded that the victim died of internal hemorrhage due to stab
wounds. 11 The bloated body was in a late stage of decomposition and its skin A. Witness showing to the court her left palm and the following words have been written in
had sloughed off. 12 He found the victim's face to be "beyond recognition." her palm in ball pen handwritten words and number of the pumpboat No. 56 and there is
There were "some rope signs in the body particularly in the waistline and in the another word "petsa" and there are words which cannot be deciphered and all found in the
knees." 13 palm of the left hand.

* The main defense interposed is one of alibi.

* Antonio stated that on the whole day of 29 November 1984, he was out at sea ATTY. MONTECLAR:
fishing with his son. Joelito, on his part, asserted that he was in Barrio Baod,

354
RECTO, GAYLE ANGELI M.
2011-0008 | AUSL
Personal Notes on Remedial Law 2 Review (based on the syllabus of Prof. Henedino M. Brondial)

That is all. A Yes, sir. 22

ATTY. GONZALES: RE-CROSS The use of memory aids during an examination of a witness is not altogether
proscribed. Section 16, Rule 132, of the Rules of Court states:
Q Mrs. witness, you cannot deny of what these physical evidences or writings on the palm of
your left hand. I want you to be honest, the law will not allow you to lie, you are subject to
punishment and penalty. My question is, who wrote this on the palm of your left hand?
Sec. 16. When witness may refer to memorandum. — A witness may be allowed to
A I was the one who wrote this. refresh his memory respecting a fact, by anything written or recorded by himself or
under his direction at the time when the fact occurred, or immediately thereafter,
or at any other time when the fact was fresh in his memory and he knew that the
same was correctly written or recorded; but in such case the writing or record
must be produced and may be inspected by the adverse party, who may, if he
Q Why did you write that down?
chooses, cross-examine the witness upon it and may read it in evidence. So, also, a
witness may testify from such a writing or record, though he retain no recollection
A I was the one who wrote this.
of the particular facts, if he is able to swear that the writing or record correctly
stated the transaction when made; but such evidence must be received with
caution. (Emphasis supplied.)

Q Why, what was your purpose of writing that in your palm?

A I wrote this in my palm because I wanted to be sure of what time the incident happened, Allowing a witness to refer to her notes rests on the sound discretion of the trial
was the same as that I wrote in my palm. court. 23 In this case, the exercise of that discretion has not been abused; the
witness herself has explained that she merely wanted to be accurate on dates and
like details.

Q And who furnished you the data in which you wrote in the palm of your hand?

A I was the one who made that. Appellants see inadvertency on Francisca's appearing to be "jittery" on the witness stand.
Nervousness and anxiety of a witness is a natural reaction particularly in the case of those
who are called to testify for the first time. The real concern, in fact, should be when they show
no such emotions.

ATTY. GONZALES:

Q You don't understand my question. You wrote that writing but where did you get that data?
Francisca did fail in immediately reporting the killing to the police authorities. Delay or
vacillation, however, in making a criminal accusation does not necessarily adulterate the
A. This is just of what I know.
credibility of the witness. 24 Francisca, in her case, has expressed fears for her life considering
that the assailants, being her neighbors, could easily exact retribution on her. 25 Also, the
hesitancy in reporting the occurrence of a crime in rural areas is not unknown. 26

Q Since you claim to have all this knowledge of your mind, why did you find it necessary to
write that in the palm of your hand and I notice during the trial that you used to look in your
palm, why, is that necessary in your believe to testify here to wh at you knew about the
Francisca's inability to respond to the summons for another appearance in court for further
incident.
questioning was satisfactorily explained by the prosecution. Francisca at the time just had a
miscarriage and was found to be too weak to travel. The recall of the witness was, after all, at
A Because of the fact that I have an headache. the sound discretion of the trial court. 27

Q When did this headache occur? The claim of appellant Roberto Descartin that Francisca and her husband, a tuba-gatherer,
owed him P300.00, and the assertion made by appellant Antonio Plasencia on the dog-biting
A After I left my house because my sick child. story involving Francisca's son truly were too petty to consider. It would be absurd to think
that Francisca, for such trivial reasons was actually impelled to falsely implicate appellants for
so grave an offense as murder.

Q Now, knowing that you have an headache, did you not bring this to the attention of the
Fiscal?
Appellants questioned Francisca's ability to recognize them from a distance. Francisca knew
A No, I did not tell the Fiscal. appellants well; they all were her neighbors while Antonio Plasencia himself was her cousin. 28
The crime occurred at around three o'clock in the afternoon only about fifty (50) meters away
from her. With an unobstructed view, Francisca's positive identification of the culprits should
be a foregone matter. 29

Q Do you know of your own that doing this is unfair and is not allowable while testifying in
open court, do you know that is illegal act?

The alleged inconsistencies in Francisca's testimony and in her sworn statement of 18


A No, I did not, know.
December 1984, cover matters of little significance. Minor inconsistencies in the testimonies of
witnesses do not detract from their credibility; 30 on the contrary, they serve to strengthen
their credibility and are taken as badges of truth rather than as indicia of falsehood 31 even as
they also erase suspicion of rehearsed testimony. 32
Q And you did all of this claiming that you do not know about the incident for the purpose of
giving here testimony against the accused?
355
RECTO, GAYLE ANGELI M.
2011-0008 | AUSL
Personal Notes on Remedial Law 2 Review (based on the syllabus of Prof. Henedino M. Brondial)

All considered, the case against the appellants has been proven beyond reasonable doubt This petition for review on certiorari seeks a reversal of the decision 1 of the Court of Appeals
even with the retracted extra-judicial admission of Joelito Descartin. 33 The testimony of a affirming the judgment 2 of the Regional Trial Court of Cebu City ordering petitioner —
single witness, if found to be credible, is adequate for conviction, 34 The defense of alibi
hardly can overcome the positive identification of an unprejudiced eyewitness. 35

. . . to pay [private respondent] the principal sum of Two Hundred Ninety Nine Thousand
Seven Hundred Seventeen Pesos and Seventy Five Centavos (P299,717.75) plus interest
Like the trial court, we are not persuaded that robbery has been proven to be the principal thereon at 12% per annum from September 22, 1986, the date of the filing of the complaint
motive for the crime that can warrant the conviction of appellants for the complex crime of until fully paid; to pay [private respondent] the further sum of Ten Thousand Pesos
robbery with homicide. 36 Appellants could only thus be held responsible for the killing of (P10,000.00) for reasonable attorney's fees; to pay the sum of Five Hundred Fifty Two Pesos
Mansueto. Conspiracy among the appellants has been established beyond doubt by the sum of and Eighty Six Centavos (P552.86) for filing fees and to pay the costs of suit. Since [private
their deeds pointing to a joint purpose and design. 37 respondent] withdrew its prayer for an alias writ of preliminary attachment vis-a-vis the
[petitioner's] counterbound, the incident on the alias writ of preliminary attachment has
become moot and academic.

Three aggravating circumstances were alleged in the information, i.e., treachery, evident
premeditation and abuse of superior strength. The trial court disregarded the circumstance of
evident premeditation and concluded that the attack upon Mansueto was committed with Facts
treachery and abuse of superior strength. On its finding that the assault was unexpectedly
perpetrated upon the unarmed victim to ensure its execution without risk to themselves from * Petitioner Rosella D. Canque is a contractor doing business under the name and style RDC
the defense that the victim might make, the trial court appreciated treachery, which it deemed Construction. At the time material to this case, she had contracts with the government for (a)
as having so absorbed abuse of superior strength. the restoration of Cebu-Toledo wharf road; (b) the asphalting of Lutopan access road; and (c)
the asphalting of Babag road in Lapulapu City. 3 In connection with these projects, petitioner
entered into two contracts with private respondent Socor Construction Corporation. The first
contract (Exh. A), 4 dated April 26, 1985, provided:
The trial court was correct when it concluded that the crime committed was murder, a crime
technically lower than robbery with homicide, 38 not, however, because of the attendance of The Sub-Contractor (SOCOR Corporation) and the Contractor (RDC Construction)
treachery but of abuse of superior strength. Treachery, in our view, was not satisfactorily for the consideration hereinafter named, hereby agree as follows:
proven by the prosecution. Francisca Espina simply testified that appellant Plasencia stabbed
Mansueto while the latter and the appellants were in a huddle. There was nothing adduced on 1. SCOPE OF WORK:
whether or not the victim gave provocation, an indispensable issue in the proper appreciation
of treachery. 39 The presence, nonetheless, of the aggravating circumstance of abuse of
a. The Sub-Contractor agrees to perform and execute the Supply, Lay and
superior strength qualified the killing to murder. 40 The three appellants utilized superiority in
Compact Item 310 and Item 302;
numbers and employed deadly weapons in assaulting the unarmed Mansueto.

b. That Contractor shall provide the labor and materials needed to complete the
project;

There being no other aggravating or mitigating circumstances to consider, the trial court aptly
c. That the Contractor agrees to pay the Sub-Contractor the price of One
imposed the penalty of reclusion perpetua, the medium period 41 of the penalty of reclusion
Thousand Pesos only (P1,000.00) per Metric Ton of Item 310 and Eight
temporal maximum to death prescribed by Article 248 of the Revised Penal Code. In
Thousand Only (P8,000.00) per Metric Ton of Item 302.
conformity with prevailing jurisprudential law, the heirs of the victim should be indemnified in
the amount of P50,000.00. 42
d. That the Contractor shall pay the Sub-Contractor the volume of the supplied
Item based on the actual weight in Metric Tons delivered, laid and compacted
and accepted by the MPWH;

WHEREFORE, the decision of the trial court convicting appellants Antonio Plasencia, Roberto
e. The construction will commence upon the acceptance of the offer.
Descartin and Joelito (Julito) Descartin of the crime of murder and imposing on each of them
the penalty of reclusion perpetua is hereby AFFIRMED with the modification that the indemnity
to the heirs of the victim, Herminio Mansueto, is raised to P50,000.00. Costs against * The second contract (Exh. B), 5 dated July 23, 1985, stated:
appellants.
The Supplier (SOCOR Construction) and the Contractor (RDC Construction) for
the consideration hereinafter named, hereby agree as follows:

SO ORDERED. 1. SCOPE OF WORK:

a. The Supplier agrees to perform and execute the delivery of Item 310 and Item
302 to the jobsite for the Asphalting of DAS Access Road and the Front Gate of
ACMDC, Toledo City;
Feliciano, Romero, Melo and Panganiban, JJ., concur.

b. That the Contractor should inform or give notice to the Supplier two (2) days
before the delivery of such items;

SECOND DIVISION c. That the Contractor shall pay the Supplier the volume of the supplied items on
the actual weight in metric tons delivered and accepted by the MPWH fifteen (15)
G.R. No. 96202 April 13, 1999 days after the submission of the bill;

ROSELLA D. CANQUE, petitioner, d. The delivery will commence upon the acceptance of the offer.

vs. THE COURT OF APPEALS and SOCOR CONSTRUCTION CORPORATION, * On May 28, 1986, private respondent sent petitioner a bill (Exh. C), containing a revised
respondents. computation, 6 for P299,717.75, plus interest at rate of 3% a month, representing the balance
of petitioner's total account of P2,098,400.25 for materials delivered and services rendered by
MENDOZA, J private respondent under the two contracts. However, petitioner refused to pay the amount,

356
RECTO, GAYLE ANGELI M.
2011-0008 | AUSL
Personal Notes on Remedial Law 2 Review (based on the syllabus of Prof. Henedino M. Brondial)

claiming that private respondent failed to submit the delivery receipts showing the actual We agree with the appellate court that the stipulation in the two contracts requiring the
weight in metric tons of the items delivered and the acceptance thereof by the government. 7 submission of delivery receipts does not preclude proof of delivery of materials by private
respondent in some other way. The question is whether the entries in the Book of Collectible
* Hence, on September 22, 1986, private respondent brought suit in the Regional Trial Court Accounts (Exh. K) constitute competent evidence to show such delivery. Private respondent
of Cebu to recover from petitioner the sum of P299,717.75, plus interest at the rate of 3% a cites Rule 130, §37 of the Rules of Court and argues that the entries in question constitute
month. "entries in the course of business" sufficient to prove deliveries made for the government
projects. This provision reads:
* In her answer, petitioner admitted the existence of the contracts with private respondent as
well as receipt of the billing (Exh. C), dated May 28, 1986. However, she disputed the
correctness of the bill —
Entries in the course of business. — Entries made at, or near the time of the transactions to
. . . considering that the deliveries of [private respondent] were not signed and which they refer, by a person deceased, outside of the Philippines or unable to testify, who
acknowledged by the checkers of [petitioner], the bituminous tack coat it was in a position to know the facts therein stated, may be received as prima facie evidence, if
delivered to [petitioner] consisted of 60% water, and [petitioner] has already such person made the entries in his professional capacity or in the performance of duty and in
paid [private respondent] about P1,400,000.00 but [private respondent] has not the ordinary or regular course of business or duty. 15
issued any receipt to [petitioner] for said payments and there is no agreement
that [private respondent] will charge 3% per month interest. 8

* Petitioner subsequently amended her answer denying she had entered into sub-contracts The admission in evidence of entries in corporate books requires the satisfaction of the
with private respondent. 9 following conditions:

* During the trial, private respondent, as plaintiff, presented its vice-president, Sofia O.
Sanchez, and Dolores Aday, its bookkeeper.
1. The person who made the entry must be dead, outside the country or unable to testify;
* Petitioner's evidence consisted of her lone testimony. 10

* On June 22, 1988, the trial court rendered its decision ordering petitioner to pay private
respondent the sum of P299,717.75 plus interest at 12% per annum, and costs. It held: 2. The entries were made at or near the time of the transactions to which they refer;

[B]y analyzing the plaintiff's Book of Collectible Accounts particularly page


17 thereof (Exh. "K") this Court is convinced that the entries (both payments and
billings) recorded thereat are credible. Undeniably, the book contains a detailed
3. The entrant was in a position to know the facts stated in the entries;
account of SOCOR's commercial transactions with RDC which were entered
therein in the course of business. We cannot therefore disregard the entries
recorded under Exhibit "K" because the fact of their having been made in the
course of business carries with it some degree of trustworthiness. Besides, no
proof was ever offered to demonstrate the irregularity of the said entries thus, 4. The entries were made in his professional capacity or in the performance of a duty, whether
there is then no cogent reason for us to doubt their authenticity. 11 legal, contractual, moral or religious; and

* The trial court further ruled that in spite of the fact that the contracts did not
have any stipulation on interest, interest may be awarded in the form of
damages under Article 2209 of the Civil Code. 12 5. The entries were made in the ordinary or regular course of business or duty. 16

* On appeal, the Court of Appeals affirmed. It upheld the trial court's' reliance on private
respondent's Book of Collectible Accounts (Exh. K) on the basis of Rule 130, §37 13 of the
Rules of Court. As petitioner points out, the business entries in question (Exh. K) do not meet the first and
third requisites. Dolores Aday, who made the entries, was presented by private respondent to
* Hence, this appeal. testify on the account of RDC Construction. It was in the course of her testimony that the
entries were presented and marked in evidence. There was, therefore, neither justification nor
necessity for the presentation of the entries as the person who made them was available to
testify in court.
Issues

I. THE RESPONDENT COURT ERRED IN ADMITTING IN EVIDENCE AS ENTRIES IN THE


COURSE OF BUSINESS THE ENTRIES IN PRIVATE RESPONDENT'S BOOK OF COLLECTIBLE Necessity is given as a ground for admitting entries, in that they are the best available
ACCOUNTS CONSIDERING THAT THE PERSON WHO MADE SAID ENTRIES ACTUALLY evidence. Said a learned judge: "What a man has actually done and committed to writing
TESTIFIED IN THIS CASE BUT UNFORTUNATELY HAD NO PERSONAL KNOWLEDGE OF SAID when under obligation to do the act, it being in the course of the business he has undertaken,
ENTRIES. and he being dead, there seems to be no danger in submitting to the consideration of the
court." The person who maybe called to court to testify on these entries being dead, there
II. THE DECISION OF THE RESPONDENT COURT SHOULD BE REVERSED AS IT HAS ONLY arises the necessity of their admission without the one who made them being called to court
INADMISSIBLE EVIDENCE TO SUPPORT IT. be sworn and subjected to cross-examination. And this is permissible in order to prevent a
failure of justice. 17

Ruling
Moreover, Aday admitted that she had no personal knowledge of the facts constituting the
entry. She said she made the entries based on the bills given to her. But she has no
First. Petitioner contends that the presentation of the delivery receipts duly accepted by the
knowledge of the truth or falsity of the facts stated in the bills. The deliveries of the materials
then Ministry of Public Works and Highways (MPWH) is required under the contracts (Exhs. A
stated in the bills were supervised by "an engineer for (such) functions." 18 The person,
and B) and is a condition precedent for her payment of the amount claimed by private
therefore, who has personal knowledge of the facts stated in the entries, i.e., that such
respondent. Petitioner argues that the entries in private respondent's Book of Collectible
deliveries were made in the amounts and on the dates stated, was the company's project
Accounts (Exh. K) cannot take the place of the delivery receipts and that such entries are mere
engineer. The entries made by Aday show only that the billings had been submitted to her by
hearsay and, thus, inadmissible in evidence. 14
the engineer and that she faithfully recorded the amounts stared therein in the books of
account. Whether or not the bills given to Aday correctly reflected the deliveries made in the
amounts and on the dates indicated was a fact that could be established by the project

357
RECTO, GAYLE ANGELI M.
2011-0008 | AUSL
Personal Notes on Remedial Law 2 Review (based on the syllabus of Prof. Henedino M. Brondial)

engineer alone who, however, was not presented during trial. The rule is stated by former Be that as it may, considered as a memorandum, Exh. K does not itself constitute
Chief Justice Moran, thus: evidence. As explained in Borromeo v. Court of Appeals: 23

[W]hen the witness had no personal knowledge of the facts entered by him, and the person Under the above provision (Rule 132, §10), the memorandum used to refresh the
who gave him the information is individually known and may testify as to the facts stated in memory of the witness does not constitute evidence, and may not be admitted as
the entry which is not part of a system of entries where scores of employees have intervened, such, for the simple reason that the witness has just the same to testify on the
such entry is not admissible without the testimony of the informer. 19 basis of refreshed memory. In other words, where the witness has testified
independently of or after his testimony has been refreshed by a memorandum of
the events in dispute, such memorandum is not admissible as corroborative
evidence. It is self-evident that a witness may not be corroborated by any written
statement prepared wholly by him. He cannot be more credible just because he
Second. It is nonetheless argued by private respondent that although the entries
supports his open-court declaration with written statements of the same facts
cannot be considered an exception to the hearsay rule, they may be admitted
even if he did prepare them during the occasion in dispute, unless the proper
under Rule 132, §10 20 of the Rules of Court which provides:
predicate of his failing memory is priorly laid down. What is more, even where this
requirement has been satisfied, the express injunction of the rule itself is that such
evidence must be received with caution, if only because it is not very difficult to
conceive and fabricate evidence of this nature. This is doubly true when the
Sec. 10. When witness may refer to memorandum. — A witness may be allowed to witness stands to gain materially or otherwise from the admission of such evidence
refresh his memory respecting a fact, by anything written by himself or under his . . . . 24
direction at the time when the fact occurred, or immediately thereafter, or at any
other time when the fact was fresh in his memory and he knew that the same was
correctly stated in the writing; but in such case the writing must be produced and
may be inspected by the adverse party, who may, if he chooses, cross-examine the
As the entries in question (Exh. K) were not made based on personal knowledge,
witness upon it, and may read it in evidence. So, also, a witness may testify from
they could only corroborate Dolores Aday's testimony that she made the entries as
such a writing, though he retain no recollection of the particular facts, if he is able
she received the bills.
to swear that the writing correctly stated the transaction when made; but such
evidence must be received with caution.

Third. Does this, therefore, mean there is no competent evidence of private respondent's claim
as petitioner argues? 25 The answer is in the negative. Aside from Exh. K, private respondent
On the other hand, petitioner contends that evidence which is inadmissible for the purpose for
presented the following documents:
which it was offered cannot be admitted for another purpose. She cites the following from
Chief Justice Moran's commentaries:

1) Exhibits A — Contract Agreement dated 26 April 1985 which contract covers both the
Toledo wharf project and the Babag Road project in Lapulapu City.
The purpose for which the evidence is offered must be specified. Where the offer is general,
and the evidence is admissible for one purpose and inadmissible for another, the evidence
should be rejected. Likewise, where the offer is made for two or more purposes and the
evidence is incompetent for one of them, the evidence should be excluded. The reason for the
rule is that "it is the duty of a party to select the competent from the incompetent in offering 2) Exhibit B — Contract Agreement dated 23 July 1985 which covers the DAS Asphalting
testimony, and he cannot impose this duty upon the trial court." Where the evidence is Project.
inadmissible for the purpose stated in the offer, it must be rejected, though the same may be
admissible for another purpose. The rule is stated thus: "If a party . . . opens the particular
view with which he offers any part of his evidence, or states the object to be attained by it, he
precludes himself from insisting on its operation in any other direction, or for any other object;
3) Exhibit C — Revised Computation of Billings submitted on May 28, 1986.
and the reason is, that the opposite party is prevented from objecting to its competency in any
view different from the one proposed. 21

4) Exhibit D — an affidavit executed by [petitioner] to the effect that she has no more pending
or unsettled obligations as far as Toledo Wharf Road is concerned.
It should be noted, however, that Exh. K is not really being presented for another
purpose. Private respondent's counsel offered it for the purpose of showing the
amount of petitioner's indebtedness. He said:

5) Exhibit D-1 — Statement of Work Accomplished on the Road Restoration of Cebu-Toledo


wharf project.

Exhibit "K," your Honor — faithful reproduction of page (17) of the book on Collectible
Accounts of the plaintiff, reflecting the principal indebtedness of defendant in the amount of
Two hundred ninety-nine thousand seven hundred seventeen pesos and seventy-five centavos
(P299,717.75) and reflecting as well the accumulated interest of three percent (3%) monthly 6) Exhibit E — another affidavit executed by [petitioner] attesting that she has completely paid
compounded such that as of December 11, 1987, the amount collectible from the defendant her laborers at the project located at Babag, Lapulapu City
by the plaintiff is Six hundred sixteen thousand four hundred thirty-five pesos and seventy-two
centavos (P616,435.72); 22

7) Exhibits F, G, G-1, G-2, G-3 — Premiums paid by [private respondent] together with the
receipts for filing fees.
This is also the purpose for which its admission is sought as a memorandum to
refresh the memory of Dolores Aday as a witness. In other words, it is the nature of
the evidence that is changed, not the purpose for which it is offered.
8) Exhibits H, I, J — certifications issued by OIC, MPWH, Regional Office; Lapulapu City, City
Engineer; Toledo City Treasurer's Office respectively, proving that RDC construction has no
more collectibles with all the said government offices in connection with its projects.

358
RECTO, GAYLE ANGELI M.
2011-0008 | AUSL
Personal Notes on Remedial Law 2 Review (based on the syllabus of Prof. Henedino M. Brondial)

SO ORDERED.

10) Exhibit L — Bill No. 057 under the account of RDC Construction in the amount of
P153,382.75 dated August 24, 1985.
Bellosillo, Puno, Quisumbing and Buena, JJ., concur.

11) Exhibit M — Bill No. 069 (RDC's account), in the amount of P1,701,795.00 dated
November 20, 1985.
> Classes of Documents

>> Public documents


12) Exhibit N — Bill No. 071 (RDC's account) in the amount of P47,250.00 dated November 22,
1985.
FIRST DIVISION

G.R. No. 204169 September 11, 2013

13) Exhibit O — Bill No. 079 (RDC's account) in the amount of P7,290.00 dated December 6,
YASUO IWASAWA, PETITIONER,
1985.

vs. FELISA CUSTODIO GANGAN1 (A.K.A FELISA GANGAN ARAMBULO, AND FELISA
GANGAN IWASAWA) AND THE LOCAL CIVIL REGISTRAR OF PASAY CITY,
RESPONDENTS.
As the trial court found:
VILLARAMA, JR., J.:

The entries recorded under Exhibit "K" were supported by Exhibits "L", "M", "N", "O" which are
all Socor Billings under the account of RDC Construction. These billings were presented and
duly received by the authorized representatives of defendant. The circumstances obtaining in * Petitioner, a Japanese national, met private respondent sometime in 2002 in one of his visits
the case at bar clearly show that for a long period of time after receipt thereof, RDC never to the Philippines. Private respondent introduced herself as "single" and "has never married
manifested its dissatisfaction or objection to the aforestated billings submitted by plaintiff. before." Since then, the two became close to each other. Later that year, petitioner came back
Neither did defendant immediately protest to plaintiff's alleged incomplete or irregular to the Philippines and married private respondent on November 28, 2002 in Pasay City. After
performance. In view of these facts, we believe Art. 1235 of the New Civil Code is applicable. the wedding, the couple resided in Japan.4

* In July 2009, petitioner noticed his wife become depressed. Suspecting that something
might have happened in the Philippines, he confronted his wife about it. To his shock, pr ivate
respondent confessed to him that she received news that her previous husband passed away.5
Art. 1235. When the obligee accepts the performance, knowing its incompleteness and
irregularity and without expressing any protest or objection, the obligation is deemed complied
with. * Petitioner sought to confirm the truth of his wife’s confession and discovered that indeed,
she was married to one Raymond Maglonzo Arambulo and that their marriage took place on
June 20, 1994.6 This prompted petitioner to file a petition7 for the declaration of his marriage to
private respondent as null and void on the ground that their marriage is a bigamous one,
based on Article 35(4) in relation to Article 41 of the Family Code of the Philippines.
FINALLY, after a conscientious scrutiny of the records, we find Exhibit "D-1" (p. 85 record) to
be a material proof of plaintiff's complete fulfillment of its obligation.
* During trial, aside from his testimony, petitioner also offered the following pieces of
documentary evidence issued by the National Statistics Office (NSO):

(1) Certificate of Marriage8 between petitioner and private respondent marked as


There is no question that plaintiff supplied RDC Construction with Item 302 (Bitunimous Prime Exhibit "A" to prove the fact of marriage between the parties on November 28,
Coat), Item 303 (Bituminous Tack Coat) and Item 310 (Bitunimous Concrete Surface Course) 2002;
in all the three projects of the latter. The Lutopan Access Road project, the Toledo wharf
project and the Babag-Lapulapu Road project.
(2) Certificate of Marriage9 between private respondent and Raymond Maglonzo
Arambulo marked as Exhibit "B" to prove the fact of marriage between the
parties on June 20, 1994;

On the other hand, no proof was ever offered by defendant to show the presence of other (3) Certificate of Death10 of Raymond Maglonzo Arambulo marked as Exhibits
contractors in those projects. We can therefore conclude that it was Socor Construction Corp. "C" and "C-1" to prove the fact of the latter’s death on July 14, 2009; and
ALONE who supplied RDC with Bituminous Prime Coat, Bituminous Tack Coat and Bituminous
Concrete Surface Course for all the aforenamed three projects. 26
(4) Certification11 from the NSO to the effect that there are two entries of
marriage recorded by the office pertaining to private respondent marked as
Exhibit "D" to prove that private respondent in fact contracted two marriages, the
first one was to a Raymond Maglonzo Arambulo on June 20, 1994, and second, to
Indeed, while petitioner had previously paid private respondent about P1,400,000.00 for petitioner on November 28, 2002.
deliveries made in the past, she did not show that she made such payments only after the
delivery receipts had been presented by private respondent. On the other hand, it appears * The prosecutor appearing on behalf of the Office of the Solicitor General (OSG) admitted the
that petitioner was able to collect the full amount of project costs from the government, so authenticity and due execution of the above documentary exhibits during pre-trial.12
that petitioner would be unjustly enriched at the expense of private respondent if she is not
made to pay what is her just obligation the contracts.
* On September 4, 2012, the RTC rendered the assailed decision. It ruled that there was
insufficient evidence to prove private respondent’s prior existing valid marriage to another
man. It held that while petitioner offered the certificate of marriage of private respondent to
Arambulo, it was only petitioner who testified about said marriage. The RTC ruled that
WHEREFORE, the decision of the Court of Appeals is AFFIRMED..nêt petitioner’s testimony is unreliable because he has no personal knowledge of private
respondent’s prior marriage nor of Arambulo’s death which makes him a complete stranger to
the marriage certificate between private respondent and Arambulo and the latter’s death

359
RECTO, GAYLE ANGELI M.
2011-0008 | AUSL
Personal Notes on Remedial Law 2 Review (based on the syllabus of Prof. Henedino M. Brondial)

certificate. It further ruled that petitioner’s testimony about the NSO certification is likewise
unreliable since he is a stranger to the preparation of said document.
This Court has consistently held that a judicial declaration of nullity is required before a valid
* Petitioner filed a motion for reconsideration, but the same was denied by the RTC in an subsequent marriage can be contracted; or else, what transpires is a bigamous marriage,16
Order dated October 16, 2012. which is void from the beginning as provided in Article 35(4) of the Family Code of the
Philippines. And this is what transpired in the instant case.
* Hence this petition raising the sole legal issue of whether the testimony of the NSO records
custodian certifying the authenticity and due execution of the public documents issued by said
office was necessary before they could be accorded evidentiary weight.
As correctly pointed out by the OSG, the documentary exhibits taken together concretely
establish the nullity of the marriage of petitioner to private respondent on the ground that
their marriage is bigamous. The exhibits directly prove the following facts: (1) that private
Ruling respondent married Arambulo on June 20, 1994 in the City of Manila; (2) that private
respondent contracted a second marriage this time with petitioner on November 28, 2002 in
Pasay City; (3) that there was no judicial declaration of nullity of the marriage of private
Petitioner argues that the documentary evidence he presented are public documents which are
respondent with Arambulo at the time she married petitioner; (3) that Arambulo died on July
considered self-authenticating and thus it was unnecessary to call the NSO Records Custodian as
14, 2009 and that it was only on said date that private respondent’s marriage with Arambulo
witness. He cites Article 410 of the Civil Code which provides that books making up the civil
was deemed to have been dissolved; and (4) that the second marriage of private respondent
register and all documents relating thereto shall be considered public documents and shall be
to petitioner is bigamous, hence null and void, since the first marriage was still valid and
prima facie evidence of the facts stated therein. Moreover, the trial prosecutor himself also
subsisting when the second marriage was contracted.
admitted the authenticity of said documents.

WHEREFORE, the petition for review on certiorari is GRANTED. The September 4, 2012
The OSG, in its Comment,13 submits that the findings of the RTC are not in accord with law
Decision and October 16, 2012 Order of the Regional Trial Court of Manila, Branch 43, in Civil
and established jurisprudence. It contends that both Republic Act No. 3753, otherwise known
Case No. 11-126203 are hereby SET ASIDE. The marriage of petitioner Yasuo Iwasawa and
as the Law on Registry of Civil Status, and the Civil Code elaborated on the character of
private respondent Felisa Custodio Gangan is declared NULL and VOID.
documents arising from records and entries made by the civil registrar and categorically
declared them as public documents. Being public documents, said documents are admissible in
evidence even without further proof of their due execution and genuineness and consequently,
there was no need for the court to require petitioner to present the records custodian or
officer from the NSO to testify on them. The OSG further contends that public documents have The Local Civil Registrar of Pasay City and the National Statistics Office are hereby ORDERED
probative value since they are prima facie evidence of the facts stated therein as provided in to make proper entries into the records of the abovementioned parties in accordance with this
the above-quoted provision of the Civil Code. Thus, the OSG submits that the public Decision.
documents presented by petitioner, considered together, completely establish the facts in
issue.

No pronouncement as to costs.

In her letter14 dated March 19, 2013 to this Court, private respondent indicated that she is
not against her husband’s petition to have their marriage declared null and void. She likewise
admitted therein that she contracted marriage with Arambulo on June 20, 1994 and
SO ORDERED.
contracted a second marriage with petitioner on November 28, 2002. She further admitted
that it was due to poverty and joblessness that she married petitioner without telling the latter
that she was previously married. Private respondent also confirmed that it was when she
found out that Arambulo passed away on July 14, 2009 that she had the guts to confess to
petitioner about her previous marriage. Thereafter, she and petitioner have separated. Sereno, CJ., Leonardo-De Castro, Bersamin, Reyes, JJ. concur.

We grant the petition. FIRST DIVISION

G.R. No. 181163 July 24, 2013

There is no question that the documentary evidence submitted by petitioner are all ASIAN TERMINALS, INC., Petitioner,
public documents. As provided in the Civil Code:
vs. PHILAM INSURANCE CO., INC. (now Chartis Philippines Insurance, Inc.),
Respondent.

ART. 410. The books making up the civil register and all documents relating thereto x-----------------------x
shall be considered public documents and shall be prima facie evidence of the facts
therein contained. G.R. No. 181262

PHILAM INSURANCE CO., INC. (now Chartis Philippines Insurance, Inc.), Petitioner,

As public documents, they are admissible in evidence even without further proof of vs. WESTWIND SHIPPING CORPORATION and ASIAN TERMINALS, INC., Respondents.
their due execution and genuineness.15 Thus, the RTC erred when it disregarded
said documents on the sole ground that the petitioner did not present the records
x-----------------------x
custodian of the NSO who issued them to testify on their authenticity and due
execution since proof of authenticity and due execution was not anymore
necessary. Moreover, not only are said documents admissible, they deserve to be G.R. No. 181319
given evidentiary weight because they constitute prima facie evidence of the facts
stated therein. And in the instant case, the facts stated therein remain unrebutted WESTWIND SHIPPING CORPORATION, Petitioner,
since neither the private respondent nor the public prosecutor presented evidence
to the contrary. vs. PHILAM INSURANCE CO., INC. (now Chartis Philippines Insurance, Inc.) and ASIAN
TERMINALS, INC., Respondents.
360
RECTO, GAYLE ANGELI M.
2011-0008 | AUSL
Personal Notes on Remedial Law 2 Review (based on the syllabus of Prof. Henedino M. Brondial)

VILLARAMA, JR., J.: * The appellate court accordingly affirmed Westwind and ATI’s joint and solidary
liability for the damage to only one (1) unit of Frame Axle Sub without Lower
inside Case No. 03-245-42K/1. It also noted that when said cargo sustained
damage, it was not yet in the custody of the consignee or the person who had
the right to receive it. The CA pointed out that Westwind’s duty to observe
Facts extraordinary diligence in the care of the cargoes subsisted during unloading
thereof by ATI’s personnel since the former exercised full control and supervision
* On April 15, 1995, Nichimen Corporation shipped to Universal Motors Corporation (Universal over the discharging operation.
Motors) 219 packages containing 120 units of brand new Nissan Pickup Truck Double Cab 4x2
model, without engine, tires and batteries, on board the vessel S/S "Calayan Iris" from Japan * Similarly, the appellate court held ATI liable for the negligence of its employees
to Manila. The shipment, which had a declared value of US$81,368 or P29,400,000, was who carried out the offloading of cargoes from the ship to the pier. As regards
insured with Philam against all risks under Marine Policy No. 708-8006717-4.4 the extent of ATI’s liability, the CA ruled that ATI cannot limit its liability to
P5,000 per damaged package. It explained that Section 7.0119 of the Contract
* The carrying vessel arrived at the port of Manila on April 20, 1995, and when the shipment for Cargo Handling Services20 does not apply in this case since ATI was not yet
was unloaded by the staff of ATI, it was found that the package marked as 03-245-42K/1 was in custody and control of the cargoes when the Frame Axle Sub without Lower
in bad order.5 The Turn Over Survey of Bad Order Cargoes6 dated April 21, 1995 identified suffered damage.
two packages, labeled 03-245-42K/1 and 03/237/7CK/2, as being dented and broken.
Thereafter, the cargoes were stored for temporary safekeeping inside CFS Warehouse in Pier * Citing Belgian Overseas Chartering and Shipping N.V. v. Philippine First
No. 5. Insurance Co., Inc.,21 the appellate court also held that Philam’s action for
damages had not prescribed notwithstanding the absence of a notice of claim.
* On May 11, 1995, the shipment was withdrawn by R.F. Revilla Customs Brokerage, Inc., the
authorized broker of Universal Motors, and delivered to the latter’s warehouse in Mandaluyong * All the parties moved for reconsideration, but their motions were denied in a Resolution
City. Upon the request7 of Universal Motors, a bad order survey was conducted on the cargoes dated January 11, 2008. Thus, they each filed a petition for review on certiorari which were
and it was found that one Frame Axle Sub without LWR was deeply dented on the buffle plate consolidated together by this Court considering that all three petitions assail the same CA
while six Frame Assembly with Bush were deformed and misaligned.8 Owing to the extent of decision and resolution and involve the same parties.
the damage to said cargoes, Universal Motors declared them a total loss.

* On August 4, 1995, Universal Motors filed a formal claim for damages in the amount of
P643,963.84 against Westwind,9 ATI10 and R.F. Revilla Customs Brokerage, Inc.11 When
Issues
Universal Motors’ demands remained unheeded, it sought reparation from and was
compensated in the sum of P633,957.15 by Philam. Accordingly, Universal Motors issued a
Subrogation Receipt12 dated November 15, 1995 in favor of Philam. Essentially, the issues posed by petitioner ATI in G.R. No. 181163, petitioner Philam in G.R.
No. 181262 and petitioner Westwind in G.R. No. 181319 can be summed up into and resolved
by addressing three questions: (1) Has Philam’s action for damages prescribed? (2) Who
* On January 18, 1996, Philam, as subrogee of Universal Motors, filed a Complaint13 for
between Westwind and ATI should be held liable for the damaged cargoes? and (3) What is
damages against Westwind, ATI and R.F. Revilla Customs Brokerage, Inc. before the RTC of
the extent of their liability?
Makati City, Branch 148.

* On September 24, 1999, the RTC rendered judgment in favor of Philam and ordered
Westwind and ATI to pay Philam, jointly and severally, the sum of P633,957.15 with interest at
the rate of 12% per annum, P158,989.28 by way of attorney’s fees and expenses of litigation. Ruling

* The court a quo ruled that there was sufficient evidence to establish the Petitioners’ Arguments
respective participation of Westwind and ATI in the discharge of and consequent
damage to the shipment. It found that the subject cargoes were compressed
while being hoisted using a cable that was too short and taut.
G.R. No. 181163
* The trial court observed that while the staff of ATI undertook the physical
unloading of the cargoes from the carrying vessel, Westwind’s duty officer Petitioner ATI disowns liability for the damage to the Frame Axle Sub without Lower inside
exercised full supervision and control throughout the process. It held Westwind Case No. 03-245-42K/1. It shifts the blame to Westwind, whom it charges with negligence in
vicariously liable for failing to prove that it exercised extraordinary diligence in the supervision of the stevedores who unloaded the cargoes. ATI admits that the damage
the supervision of the ATI stevedores who unloaded the cargoes from the vessel. could have been averted had Westwind observed extraordinary diligence in handling the
However, the court absolved R.F. Revilla Customs Brokerage, Inc. from liability in goods. Even so, ATI suspects that Case No. 03-245-42K/1 is "weak and defective"22
light of its finding that the cargoes had been damaged before delivery to the considering that it alone sustained damage out of the 219 packages.
consignee.

* The trial court acknowledged the subrogation between Philam and Universal
Motors on the strength of the Subrogation Receipt dated November 15, 1995. It
Notwithstanding, petitioner ATI submits that, at most, it can be held liable to pay only P5,000
likewise upheld Philam’s claim for the value of the alleged damaged vehicle parts
per package pursuant to its Contract for Cargo Handling Services. ATI maintains that it was
contained in Case Nos. 03-245-42K/1 and 03-245-51K or specifically for "7 pieces
not properly notified of the actual value of the cargoes prior to their discharge from the vessel.
of Frame Axle Sub Without Lower and Frame Assembly with Bush."14

* Westwind filed a Motion for Reconsideration15 which was, however, denied in an Order16
dated October 26, 2000.
G.R. No. 181262

* On appeal, the CA affirmed with modification the ruling of the RTC. In a Decision dated
Petitioner Philam supports the CA in holding both Westwind and ATI liable for the deformed
October 15, 2007, the appellate court directed Westwind and ATI to pay Philam, jointly and
and misaligned Frame Axle Sub without Lower inside Case No. 03-245-42K/1. It, however,
severally, the amount of P190,684.48 with interest at the rate of 12% per annum until fully
faults the appellate court for disallowing its claim for the value of six Chassis Frame Assembly
paid, attorney’s fees of P47,671 and litigation expenses.
which were likewise supposedly inside Case Nos. 03-245-51K and 03-245-42K/1. As to the
latter container, Philam anchors its claim on the results of the Inspection/Survey Report23 of
* The CA stressed that Philam may not modify its allegations by claiming in its
Chartered Adjusters, Inc., which the court received without objection from Westwind and ATI.
Appellee’s Brief17 that the six pieces of Frame Assembly with Bush, which were
Petitioner believes that with the offer and consequent admission of evidence to the effect that
purportedly damaged, were also inside Case No. 03-245-42K/1. The CA noted Case No. 03-245-42K/1 contains six pieces of dented Chassis Frame Assembly, Philam’s claim
that in its Complaint, Philam alleged that "one (1) pc. FRAME AXLE SUB W/O thereon should be treated, in all respects, as if it has been raised in the pleadings. Thus,
LWR from Case No. 03-245-42K/1 was completely deformed and misaligned, and Philam insists on the reinstatement of the trial court’s award in its favor for the payment of
six (6) other pcs. of FRAME ASSEMBLY WITH BUSH from Case No. 03-245-51K
P633,957.15 plus legal interest, P158,989.28 as attorney’s fees and costs.
were likewise completely deformed and misaligned."18

361
RECTO, GAYLE ANGELI M.
2011-0008 | AUSL
Personal Notes on Remedial Law 2 Review (based on the syllabus of Prof. Henedino M. Brondial)

nonetheless resolve questions of fact when the case falls under any of the following
exceptions:
G.R. No. 181319

Petitioner Westwind denies joint liability with ATI for the value of the deformed Frame Axle
Sub without Lower in Case No. 03-245-42K/1. Westwind argues that the evidence shows that (1) when the findings are grounded entirely on speculation, surmises, or conjectures; (2)
ATI was already in actual custody of said case when the Frame Axle Sub without Lower inside when the inference made is manifestly mistaken, absurd, or impossible; (3) when there is
it was misaligned from being compressed by the tight cable used to unload it. Accordingly, grave abuse of discretion; (4) when the judgment is based on a misapprehension of facts; (5)
Westwind ceased to have responsibility over the cargoes as provided in paragraph 4 of the Bill when the findings of fact are conflicting; (6) when in making its findings the Court of Appeals
of Lading which provides that the responsibility of the carrier shall cease when the goods are went beyond the issues of the case, or its findings are contrary to the admissions of both the
taken into the custody of the arrastre. appellant and the appellee; (7) when the findings are contrary to those of the trial court; (8)
when the findings are conclusions without citation of specific evidence on which they are
based; (9) when the facts set forth in the petition as well as in the petitioner’s main and reply
briefs are not disputed by the respondent; and (10) when the findings of fact are premised on
the supposed absence of evidence and contradicted by the evidence on record.29
Westwind contends that sole liability for the damage rests on ATI since it was the latter’s
stevedores who operated the ship’s gear to unload the cargoes. Westwind reasons that ATI is
an independent company, over whose employees and operations it does not exercise control.
Moreover, it was ATI’s employees who selected and used the wrong cable to lift the box
containing the cargo which was damaged. In the cases at bar, the fifth and seventh exceptions apply. While the CA affirmed the joint
liability of ATI and Westwind, it held them liable only for the value of one unit of Frame Axle
Sub without Lower inside Case No. 03-245-42K/1. The appellate court disallowed the award of
damages for the six pieces of Frame Assembly with Bush, which petitioner Philam alleged, for
the first time in its Appellee’s Brief, to be likewise inside Case No. 03-245-42K/1. Lastly, the CA
Westwind likewise believes that ATI is bound by its acceptance of the goods in good order
reduced the award of attorney’s fees to P47,671.
despite a finding that Case No. 03-245-42K/1 was partly torn and crumpled on one side.
Westwind also notes that the discovery that a piece of Frame Axle Sub without Lower was
completely deformed and misaligned came only on May 12, 1995 or 22 days after the cargoes
were turned over to ATI and after the same had been hauled by R.F. Revilla Customs
Brokerage, Inc. Foremost, the Court holds that petitioner Philam has adequately established the basis of its
claim against petitioners ATI and Westwind. Philam, as insurer, was subrogated to the rights
of the consignee, Universal Motors Corporation, pursuant to the Subrogation Receipt executed
by the latter in favor of the former. The right of subrogation accrues simply upon payment by
the insurance company of the insurance claim.30 Petitioner Philam’s action finds support in
Westwind further argues that the CA erred in holding it liable considering that Philam’s cause
Article 2207 of the Civil Code, which provides as follows:
of action has prescribed since the latter filed a formal claim with it only on August 17, 1995 or
four months after the cargoes arrived on April 20, 1995. Westwind stresses that according to
the provisions of clause 20, paragraph 224 of the Bill of Lading as well as Article 36625 of the
Code of Commerce, the consignee had until April 20, 1995 within which to make a claim
considering the readily apparent nature of the damage, or until April 27, 1995 at the latest, if it Art. 2207. If the plaintiff’s property has been insured, and he has received indemnity from the
is assumed that the damage is not readily apparent. insurance company for the injury or loss arising out of the wrong or breach of contract
complained of, the insurance company shall be subrogated to the rights of the insured against
the wrongdoer or the person who has violated the contract. x x x.

Lastly, petitioner Westwind contests the imposition of 12% interest on the award of damages
to Philam reckoned from the time of extrajudicial demand. Westwind asserts that, at most, it
can only be charged with 6% interest since the damages claimed by Philam does not In their respective comments31 to Philam’s Formal Offer of Evidence,32 petitioners ATI and
constitute a loan or forbearance of money. Westwind objected to the admission of Marine Certificate No. 708-8006717-4 and the
Subrogation Receipt as documentary exhibits "B" and "P," respectively. Petitioner Westwind
objects to the admission of both documents for being hearsay as they were not authenticated
by the persons who executed them. For the same reason, petitioner ATI assails the
admissibility of the Subrogation Receipt. As regards Marine Certificate No. 708-8006717-4, ATI
The Court’s Ruling
makes issue of the fact that the same was issued only on April 27, 1995 or 12 days after the
shipment was loaded on and transported via S/S "Calayan Iris."

The three consolidated petitions before us call for a determination of who between ATI and
Westwind is liable for the damage suffered by the subject cargo and to what extent. However,
The nature of documents as either public or private determines how the documents
the resolution of the issues raised by the present petitions is predicated on the appreciation of
may be presented as evidence in court. Public documents, as enumerated under
factual issues which is beyond the scope of a petition for review on certiorari under Rule 45 of
Section 19,33 Rule 132 of the Rules of Court, are self-authenticating and require
the 1997 Rules of Civil Procedure, as amended. It is settled that in petitions for review on
no further authentication in order to be presented as evidence in court.34
certiorari, only questions of law may be put in issue. Questions of fact cannot be
entertained.26

In contrast, a private document is any other writing, deed or instrument executed


by a private person without the intervention of a notary or other person legally
There is a question of law if the issue raised is capable of being resolved without need of
authorized by which some disposition or agreement is proved or set forth. Lacking
reviewing the probative value of the evidence. The resolution of the issue must rest solely on
the official or sovereign character of a public document, or the solemnities
what the law provides on the given set of circumstances. Once it is clear that the issue invites a
prescribed by law, a private document requires authentication35 in the manner
review of the evidence presented, the question posed is one of fact. If the query requires a re-
prescribed under Section 20, Rule 132 of the Rules:
evaluation of the credibility of witnesses, or the existence or relevance of surrounding
circumstances and their relation to each other, the issue in that query is factual.27

SEC. 20. Proof of private document. - Before any private document offered as
authentic is received in evidence, its due execution and authenticity must be
In the present petitions, the resolution of the question as to who between Westwind and ATI
proved either:
should be liable for the damages to the cargo and to what extent would have this Court pass
upon the evidence on record. But while it is not our duty to review, examine and evaluate or
weigh all over again the probative value of the evidence presented,28 the Court may
362
RECTO, GAYLE ANGELI M.
2011-0008 | AUSL
Personal Notes on Remedial Law 2 Review (based on the syllabus of Prof. Henedino M. Brondial)

(a) By anyone who saw the document executed or written; or Q Now, I am presenting to you a copy of this marine certificate 708-8006717-4 issued by
Philam Insurance Company, Inc. to Universal Motors Corporation on April 15, 1995. Will you
tell us what relation does it have to that policy risk claim mentioned in that letter?

(b) By evidence of the genuineness of the signature or handwriting of the maker. A This is a photocopy of the said policy issued by the consignee Universal Motors Corporation.

Any other private document need only be identified as that which it is claimed to ATTY. PALACIOS
be.
I see. May I request, if Your Honor please, that this marine risk policy of the plaintiff as
submitted by claimant Universal Motors Corporation be marked as Exhibit B.

The requirement of authentication of a private document is excused only in four


instances, specifically: (a) when the document is an ancient one within the context
of Section 21,36 Rule 132 of the Rules; (b) when the genuineness and authenticity COURT
of the actionable document have not been specifically denied under oath by the
adverse party; (c) when the genuineness and authenticity of the document have Mark it.39
been admitted; or (d) when the document is not being offered as genuine.37

As regards the issuance of Marine Certificate No. 708-8006717-4 after the fact of loss
Indubitably, Marine Certificate No. 708-8006717-4 and the Subrogation Receipt occurred, suffice it to say that said document simply certifies the existence of an open
are private documents which Philam and the consignee, respectively, issue in the insurance policy in favor of the consignee. Hence, the reference to an "Open Policy Number
pursuit of their business. Since none of the exceptions to the requirement of 9595093" in said certificate. The Court finds it completely absurd to suppose that any
authentication of a private document obtains in these cases, said documents may insurance company, of sound business practice, would assume a loss that has already been
not be admitted in evidence for Philam without being properly authenticated. realized, when the profitability of its business rests precisely on the non-happening of the risk
insured against.

Contrary to the contention of petitioners ATI and Westwind, however, Philam presented its
claims officer, Ricardo Ongchangco, Jr. to testify on the execution of the Subrogation Receipt, Yet, even with the exclusion of Marine Certificate No. 708-8006717-4, the Subrogation
as follows: Receipt, on its own, is adequate proof that petitioner Philam paid the consignee’s claim on the
damaged goods. Petitioners ATI and Westwind failed to offer any evidence to controvert the
same. In Malayan Insurance Co., Inc. v. Alberto,40 the Court explained the effect of payment
by the insurer of the insurance claim in this wise:
ATTY. PALACIOS

Q How were you able to get hold of this subrogation receipt?


We have held that payment by the insurer to the insured operates as an equitable assignment
A Because I personally delivered the claim check to consignee and have them receive the said to the insurer of all the remedies that the insured may have against the third party whose
check. negligence or wrongful act caused the loss. The right of subrogation is not dependent upon,
nor does it grow out of, any privity of contract. It accrues simply upon payment by the
insurance company of the insurance claim. The doctrine of subrogation has its roots in equity.
It is designed to promote and accomplish justice; and is the mode that equity adopts to
compel the ultimate payment of a debt by one who, in justice, equity, and good conscience,
Q I see. Therefore, what you are saying is that you personally delivered the claim check of
ought to pay.41
Universal Motors Corporation to that company and you have the subrogation receipt signed by
them personally?

A Yes, sir.
Neither do we find support in petitioner Westwind’s contention that Philam’s right of action has
prescribed.

Q And it was signed in your presence?

The Carriage of Goods by Sea Act (COGSA) or Public Act No. 521 of the 74th US Congress,
A Yes, sir.38
was accepted to be made applicable to all contracts for the carriage of goods by sea to and
from Philippine ports in foreign trade by virtue of Commonwealth Act (C.A.) No. 65.42 Section
1 of C.A. No. 65 states:

Indeed, all that the Rules require to establish the authenticity of a document is the testimony
of a person who saw the document executed or written. Thus, the trial court did not err in
admitting the Subrogation Receipt in evidence despite petitioners ATI and Westwind’s
Section 1. That the provisions of Public Act Numbered Five hundred and twenty-one of the
objections that it was not authenticated by the person who signed it.
Seventy-fourth Congress of the United States, approved on April sixteenth, nineteen hundred
and thirty-six, be accepted, as it is hereby accepted to be made applicable to all contracts for
the carriage of goods by sea to and from Philippine ports in foreign trade: Provided, That
nothing in the Act shall be construed as repealing any existing provision of the Code of
However, the same cannot be said about Marine Certificate No. 708-8006717-4 which Commerce which is now in force, or as limiting its application.
Ongchangcho, Jr. merely identified in court. There is nothing in Ongchangco, Jr.’s testimony
which indicates that he saw Philam’s authorized representative sign said document, thus:

The prescriptive period for filing an action for the loss or damage of the goods under the
COGSA is found in paragraph (6), Section 3, thus:
ATTY. PALACIOS
363
RECTO, GAYLE ANGELI M.
2011-0008 | AUSL
Personal Notes on Remedial Law 2 Review (based on the syllabus of Prof. Henedino M. Brondial)

Complaint for damages on January 18, 1996, just eight months after all the packages were
delivered to its possession on May 17, 1995. Evidently, petitioner Philam’s action against
(6) Unless notice of loss or damage and the general nature of such loss or damage be given in petitioners Westwind and ATI was seasonably filed.
writing to the carrier or his agent at the port of discharge before or at the time of the removal of
the goods into the custody of the person entitled to delivery thereof under the contract of
carriage, such removal shall be prima facie evidence of the delivery by the carrier of the goods
as described in the bill of lading. If the loss or damage is not apparent, the notice must be This brings us to the question that must be resolved in these consolidated petitions. Who
given within three days of the delivery. between Westwind and ATI should be liable for the damage to the cargo?

Said notice of loss or damage maybe endorsed upon the receipt for the goods given by the It is undisputed that Steel Case No. 03-245-42K/1 was partly torn and crumpled on one side
person taking delivery thereof. while it was being unloaded from the carrying vessel. The damage to said container was noted
in the Bad Order Cargo Receipt48 dated April 20, 1995 and Turn Over Survey of Bad Order
Cargoes dated April 21, 1995. The Turn Over Survey of Bad Order Cargoes indicates that said
steel case was not opened at the time of survey and was accepted by the arrastre in good
The notice in writing need not be given if the state of the goods has at the time of their order. Meanwhile, the Bad Order Cargo Receipt bore a notation "B.O. not yet t/over to ATI."
receipt been the subject of joint survey or inspection. On the basis of these documents, petitioner ATI claims that the contents of Steel Case No. 03-
245-42K/1 were damaged while in the custody of petitioner Westwind.

In any event the carrier and the ship shall be discharged from all liability in respect of loss or
damage unless suit is brought within one year after delivery of the goods or the date when the We agree.
goods should have been delivered: Provided, That if a notice of loss or damage, either
apparent or concealed, is not given as provided for in this section, that fact shall not affect or
prejudice the right of the shipper to bring suit within one year after the delivery of the goods
or the date when the goods should have been delivered. Common carriers, from the nature of their business and for reasons of public policy, are bound
to observe extraordinary diligence in the vigilance over the goods transported by them.
Subject to certain exceptions enumerated under Article 173449 of the Civil Code, common
carriers are responsible for the loss, destruction, or deterioration of the goods. The
In the Bill of Lading43 dated April 15, 1995, Rizal Commercial Banking Corporation (RCBC) is extraordinary responsibility of the common carrier lasts from the time the goods are
indicated as the consignee while Universal Motors is listed as the notify party. These unconditionally placed in the possession of, and received by the carrier for transportation until
designations are in line with the subject shipment being covered by Letter of Credit No. the same are delivered, actually or constructively, by the carrier to the consignee, or to the
I501054, which RCBC issued upon the request of Universal Motors. person who has a right to receive them.50

A letter of credit is a financial device developed by merchants as a convenient and relatively The court a quo, however, found both petitioners Westwind and ATI, jointly and severally,
safe mode of dealing with sales of goods to satisfy the seemingly irreconcilable interests of a liable for the damage to the cargo. It observed that while the staff of ATI undertook the
seller, who refuses to part with his goods before he is paid, and a buyer, who wants to have physical unloading of the cargoes from the carrying vessel, Westwind’s duty officer exercised
control of his goods before paying.44 However, letters of credit are employed by the parties full supervision and control over the entire process. The appellate court affirmed the solidary
desiring to enter into commercial transactions, not for the benefit of the issuing bank but liability of Westwind and ATI, but only for the damage to one Frame Axle Sub without Lower.
mainly for the benefit of the parties to the original transaction,45 in these cases, Nichimen
Corporation as the seller and Universal Motors as the buyer. Hence, the latter, as the buyer of
the Nissan CKD parts, should be regarded as the person entitled to delivery of the goods.
Accordingly, for purposes of reckoning when notice of loss or damage should be given to the Upon a careful review of the records, the Court finds no reason to deviate from the finding
carrier or its agent, the date of delivery to Universal Motors is controlling. that petitioners Westwind and ATI are concurrently accountable for the damage to the content
of Steel Case No. 03-245-42K/1.

S/S "Calayan Iris" arrived at the port of Manila on April 20, 1995, and the subject cargoes
were discharged to the custody of ATI the next day. The goods were then withdrawn from the Section 251 of the COGSA provides that under every contract of carriage of goods by the sea,
CFS Warehouse on May 11, 1995 and the last of the packages delivered to Universal Motors the carrier in relation to the loading, handling, stowage, carriage, custody, care and discharge
on May 17, 1995. Prior to this, the latter filed a Request for Bad Order Survey46 on May of such goods, shall be subject to the responsibilities and liab ilities and entitled to the rights
12,1995 following a joint inspection where it was discovered that six pieces of Chassis Frame and immunities set forth in the Act. Section 3 (2)52 thereof then states that among the
Assembly from two bundles were deformed and one Front Axle Sub without Lower from a carrier’s responsibilities are to properly load, handle, stow, carry, keep, care for and discharge
steel case was dented. Yet, it was not until August 4, 1995 that Universal Motors filed a formal the goods carried.53
claim for damages against petitioner Westwind.

At the trial, Westwind’s Operation Assistant, Menandro G. Ramirez, testified on the presence of a
Even so, we have held in Insurance Company of North America v. Asian Terminals, Inc. that a ship officer to supervise the unloading of the subject cargoes.
request for, and the result of a bad order examination, done within the reglementary period
for furnishing notice of loss or damage to the carrier or its agent, serves the purpose of a
claim. A claim is required to be filed within the reglementary period to afford the carrier or
depositary reasonable opportunity and facilities to check the validity of the claims while facts
are still fresh in the minds of the persons who took part in the transaction and documents are ATTY. LLAMAS
still available.47 Here, Universal Motors filed a request for bad order survey on May 12, 1995,
even before all the packages could be unloaded to its warehouse. Q Having been present during the entire discharging operation, do you remember who else
were present at that time?

A Our surveyor and our checker the foreman of ATI.


Moreover, paragraph (6), Section 3 of the COGSA clearly states that failure to comply with the
notice requirement shall not affect or prejudice the right of the shipper to bring suit within one
year after delivery of the goods. Petitioner Philam, as subrogee of Universal Motors, filed the
364
RECTO, GAYLE ANGELI M.
2011-0008 | AUSL
Personal Notes on Remedial Law 2 Review (based on the syllabus of Prof. Henedino M. Brondial)

Q Were there officials of the ship present also? to hoist the packages for transfer to the dock. Moreover, the fact that 218 of the 219 packages
were unloaded with the same sling unharmed is telling of the inadequate care with which ATI’s
A Yes, sir there was an officer of the vessel on duty at that time.54 stevedore handled and discharged Case No. 03-245-42K/1.

xxxx With respect to petitioners ATI and Westwind’s liability, we agree with the CA that the same
should be confined to the value of the one piece Frame Axle Sub without Lower.

Q Who selected the cable slink to be used?


In the Bad Order Inspection Report62 prepared by Universal Motors, the latter referred to
Case No. 03-245-42K/1 as the source of said Frame Axle Sub without Lower which suffered a
A ATI Operation.
deep dent on its buffle plate. Yet, it identified Case No. 03-245-51K as the container which
bore the six pieces Frame Assembly with Bush. Thus, in Philam’s Complaint, it alleged that
"the entire shipment showed one (1) pc. FRAME AXLE SUB W/O LWR from Case No. 03-245-
42K/1 was completely deformed and misaligned, and six (6) other pcs. of FRAME ASSEMBLY
Q Are you aware of how they made that selection? WITH BUSH from Case No. 03-245-51K were likewise completely deformed and misaligned."63
Philam later claimed in its Appellee’s Brief that the six pieces of Frame Assembly with Bush
A Before the vessel arrived we issued a manifesto of the storage plan informing the ATI of were also inside the damaged Case No. 03-245-42K/1.
what type of cargo and equipment will be utilitized in discharging the cargo.55

However, there is nothing in the records to show conclusively that the six Frame Assembly
xxxx with Bush were likewise contained in and damaged inside Case No. 03-245-42K/1. In the
Inspection Survey Report of Chartered Adjusters, Inc., it mentioned six pieces of chassis frame
assembly with deformed body mounting bracket. However, it merely noted the same as
coming from two bundles with no identifying marks.

Q You testified that it was the ATI foremen who select the cable slink to be used in
discharging, is that correct?

A Yes sir, because they are the one who select the slink and they know the kind of cargoes Lastly, we agree with petitioner Westwind that the CA erred in imposing an interest rate of
because they inspected it before the discharge of said cargo. 12% on the award of damages. Under Article 2209 of the Civil Code, when an obligation not
constituting a loan or forbearance of money is breached, an interest on the amount of
damages awarded may be imposed at the discretion of the court at the rate of 6% per
annum.64 In the similar case of Belgian Overseas Chartering and Shipping NV v. Philippine
First Insurance Co., lnc.,65 the Court reduced the rate of interest on the damages awarded to
Q Are you aware that the ship captain is consulted in the selection of the cable sling? the carrier therein to 6% from the time of the filing of the complaint until the finality of the
decision.
A Because the ship captain knows for a fact the equipment being utilized in the discharge of
the cargoes because before the ship leave the port of Japan the crew already utilized the
proper equipment fitted to the cargo.56 (Emphasis supplied.)

WHEREFORE, the Court AFFIRMS with MODIFICATION the Decision dated October 15,2007
and the Resolution dated January 11, 2008 of the Court of Appeals in CA-G.R. CV No. 69284 in
that the interest rate on the award of P190,684.48 is reduced to 6% per annum from the date
It is settled in maritime law jurisprudence that cargoes while being unloaded generally remain of extrajudicial demand, until fully paid.
under the custody of the carrier.57 The Damage Survey Report58 of the survey conducted by
Phil. Navtech Services, Inc. from April 20-21, 1995 reveals that Case No. 03-245-42K/1 was
damaged by ATI stevedores due to overtightening of a cable sling hold during discharge from
the vessel’s hatch to the pier. Since the damage to the cargo was incurred during the
With costs against the petitioners in G.R. No. 181163 and G.R. No. 181319, respectively.
discharge of the shipment and while under the supervision of the carrier, the latter is liable for
the damage caused to the cargo.

SO ORDERED.

This is not to say, however, that petitioner ATI is without liability for the damaged cargo.

Sereno, CJ., Leonardo-De Castro, Peralta, Bersamin, JJ. concur

The functions of an arrastre operator involve the handling of cargo deposited on the wharf or
between the establishment of the consignee or shipper and the ship’s tackle. Being the
custodian of the goods discharged from a vessel, an arrastre operator’s dut y is to take good
care of the goods and to turn them over to the party entitled to their possession.59 >> Offer of evidence

SECOND DIVISION

Handling cargo is mainly the arrastre operator’s principal work so its drivers/operators or
G.R. No. 165285 June 18, 2012
employees should observe the standards and measures necessary to prevent losses and
damage to shipments under its custody.60
LOMISES ALUDOS, deceased, substituted by FLORA ALUDOS, Petitioner,

vs. JOHNNY M. SUERTE,* Respondent.


While it is true that an arrastre operator and a carrier may not be held solidarily liable at all
times,61 the facts of these cases show that apart from ATI’s stevedores being directly in BRION, J.:
charge of the physical unloading of the cargo, its foreman picked the cable sling that was used
365
RECTO, GAYLE ANGELI M.
2011-0008 | AUSL
Personal Notes on Remedial Law 2 Review (based on the syllabus of Prof. Henedino M. Brondial)

P68,000.00

Facts Received from Mr. Lomises Aludos the sum of Sixty-eight thousand
(P68,000.00) Pesos as reimbursement of my money.
* Sometime in January 1969, Lomises acquired from the Baguio City Government the right to
occupy two stalls in the Hangar Market in Baguio City, as evidenced by a permit issued by the Baguio City, October 9, 1985.
City Treasurer.4
[Signature affixed]
* On September 8, 1984, Lomises entered into an agreement with respondent Johnny M.
Suerte for the transfer of all improvements and rights over the two market stalls (Stall Nos. 9 JAIME SUERTE [Signature affixed]
and 10) for the amount of P260,000.00. Johnny gave a down payment of P45,000.00 to
Lomises, who acknowledged receipt of the amount in a document5 executed on the same date
DOMES SUERTE
as the agreement:

Witnesses
RECEIPT

[Illegible signature] [Illegible signature]


P45,000.00 September 8, 1984

* Through a letter dated October 15, 1985, Johnny protested the return of his money, and
Received the Sum of Forty Five Thousand Pesos (P45,000.00) from JOHNNY M.
insisted on the continuation and enforcement of his agreement with Lomises. When Lomises
SUERTE, with postal address at Kamog, Sablan, Benguet Province, Philippine
refused Johnny’s protest, Johnny filed a complaint against Lomises before the Regional Trial
Currency as an advance or partial downpayment of Improvements and Rights
Court (RTC), Branch 7, Baguio City, for specific performance with damages, docketed as Civil
over Stall Nos. 9 and 10, situated at Refreshment Section, Hangar Market
Case No. 720-R. Johnny prayed that, after due proceedings, judgment be rendered ordering
Compound, Baguio City, and the said amount will be deducted from the agreed
Lomises to (1) accept the payment of the balance of P192,000.00; and (2) execute a final
proceeds of the transaction in the amount of Two Hundred Sixty Thousand Pesos
deed of sale and/or transfer the improvements and rights over the two market stalls in his
(P260,000.00), Philippine Currency and payable starting from September 1984 up
favor.
to December 1985, and/or (16) months.

* In a decision dated November 24, 1998,7 the RTC nullified the agreement between Johnny
This receipt will be formalise (sic) later, and the Deed of Absolute Transfer of
and Lomises for failure to secure the consent of the Baguio City Government to the
Improvements and Rights over the said Stall be executed immediately upon full
agreement. The RTC found that Lomises was a mere lessee of the market stalls, and the
payment of the balance stated in the above.
Baguio City Government was the owner-lessor of the stalls. Under Article 1649 of the Civil
Code, "[t]he lessee cannot assign the lease without the consent of the lessor, unless there is a
Right hand thumbmark: stipulation to the contrary." As the permit issued to Lomises did not contain any provision that
the lease of the market stalls could further be assigned, and in the absence of the consent of
[Thumbmark affixed] the Baguio City Government to the agreement, the RTC declared the agreement between
Lomises and Johnny null and void. The nullification of the agreement required the parties to
LOMISES F. ALUDOS return what had been received under the agreement; thus, the RTC ordered Lomises to return
the down payment made by Johnny, with interest of 12% per annum, computed from the time
the complaint was filed until the amount is fully paid. It dismissed the parties’ claims for
(Registered Stall Holder)
damages.

With the Consent of the Wife:


* Lomises appealed the RTC decision to the CA, arguing that the real agreement between the
parties was merely one of loan, and not of sale; he further claimed that the loan had been
[Signature affixed]
extinguished upon the return of the P68,000.00 to Johnny’s mother, Domes.

FLORA MENES * In a decision dated August 29, 2002,8 the CA rejected Lomises’ claim that the true
agreement was one of loan. The CA found that there were two agreements entered into
(Wife)

You might also like